Chemistry Problem JEE PDF

You might also like

Download as pdf or txt
Download as pdf or txt
You are on page 1of 1071

CHEMISTRY

PROBLEM BOOK

for JEE
Note to the Student
Wiley Chemistry Problem Book is specifically designed to meet the needs of engineering (JEE) aspirants and give an edge to their
preparation. The book offers complete coverage of the chemistry curriculum (Physical, Inorganic, Organic) for JEE. It is enriched with
unique elements and features that help students recapitulate the concepts, build problem-solving skills and apply them to solve all
question-types asked in the engineering entrance examinations. The book is a valuable resource for both JEE (Main) and JEE (Advanced)
aspirants. The chapter flow of the book is aligned with JEE Main syllabus and its coverage in the classroom. However, topics specific to
JEE (Advanced) and advanced level questions are also covered both as solved examples and practice exercises.
We will now walk you through the target examinations and some key features of the book that enhance the learning experience.

TARGET EXAMINATION
Admission to Undergraduate Engineering Programs at NITs, IITs and other Center and State (participating) funded Technical Institutions
use the Joint Entrance Examination Main (JEE Main) score as eligibility/merit criteria. The JEE (Main) is also an eligibility test for the Joint
Entrance Examination Advanced [JEE (Advanced)], which is mandatory for the candidate if he/she is aspiring for admission to the under-
graduate program offered by the IITs. The JEE (Advanced) scores are used as an eligibility criteria for admission into IITs.
An effective exam strategy for success in these examinations can be based on the detailed analysis of previous years question papers and
planning your preparation accordingly. The Chemistry Question Paper of these examinations is a judicious mix of easy, moderate and
tough questions. The analysis of question distribution over the three sections of chemistry for these examinations is given below. The
detailed analysis for each topic is offered at the beginning of each chapter.

EXAM ANALYSIS OF PAPERS


Chemistry question paper comes as an amalgamation of easy moderate and tough questions. Broadly, the pattern of distribution of
questions over different sections of chemistry is as follows:

JEE Main

Year
Subject
2007 2008 2009 2010 2011 2012 2013 2014 2015 2016
Physical Chemistry 19 16 13 16 14 14 15 13 11 8
Inorganic Chemistry 13 7 7 4 7 6 6 7 10 13
Organic Chemistry 7 12 10 10 6 10 9 10 9 9

JEE Advanced

Year
Subject
2007 2008 2009 2010 2011 2012 2013 2014 2015 2016
Physical Chemistry 12 16 13 17 19 14 12 15 12 13
Inorganic Chemistry 9 13 9 13 8 10 11 9 12 11
Organic Chemistry 12 16 17 16 13 13 12 12 14 12

FM.indd 3 8/8/2016 6:20:19 PM


iv Note to the Student

FEATURES OF THE BOOK

A. Chapter Opener
6 Thermodynamics
Each chapter starts with exam analysis (question number and concept distribu-
tion) for the topic in previous ten years JEE papers.

Questions Distribution in JEE (Main and Advanced)

3
No. of Questions

JEE (Main)
2
JEE (Adv)

0
2016 2015 2014 2013 2012 2011 2010 2009 2008 2007

Concept Distribution in JEE (Main and Advanced)

Topics Covered
Year
JEE (Main) JEE (Advanced)
2007 First Law of Thermodynamics, Gibb’s Free energy Gibb’s Free energy
2008 Enthalpy, Gibb’s Free energy Second law of thermodynamics, Gibb’s Free energy
2009 Thermochemistry, Gibb’s Free energy Fundamental of thermodynamics, Enthalpy
Fundamental of thermodynamics, Pressure-Volume
2010 Thermochemistry, Gibb’s Free energy
work, Enthalpy
2011 Entropy Enthalpy
2012 Gibb’s Free energy Entropy, Pressure-Volume work
2013 First Law of Thermodynamics Thermochemistry, Pressure-Volume work
2014 Enthalpy, Thermochemistry, Entropy Pressure-Volume work, Entropy
2015 Gibb’s Free energy Entropy, Pressure-Volume work, Gibb’s Free energy
2016 Enthalpy, Gibb’s Free Energy, Pressure Volume Work Entropy

B. Summary
Chapter-6.indd 1 6/11/2016 2:03:51 PM

Summary for recapitulating all concepts as per the JEE curriculum, enriched
with figures, graphs, schematic representations, and, tables. Important points to
remember
2 about
Chapter concepts
11 The highlighted as tip.
p-Block Elements

SUMMARY
1. The elements in which the highest energy electron enters the outermost p orbital are known as p-block elements. There are six
groups (ns2 np1−6) of p-block elements in the periodic table, named as Group13–18 as shown below.
Group 13 14 15 16 17 18
First element Boron (B) Carbon (C) Nitrogen (N) Oxygen (O) Fluorine (F) Helium (He)
Electronic configuration ns2 np1 ns2 np2 ns2 np3 ns2 np4 ns2 np5 ns2 np6
(He: 1s2)
Group oxidation state +3 +4 +5 +6 +7 +8
Other oxidation states +1 +2, −4 +3, −3 +4, +2, −2 +5, +3, +1, −1 +6, +4, +2
2. This occurrence of an oxidation state two units below the Group oxidation state is sometimes attributed to the inert pair effect. The
term refers to the resistance of a pair of s electrons to be lost or to participate in covalent bond formation.
3. General properties of Group13 elements

Ionization enthalpy
(kJ mol−1)
Atomic Electronic Oxidation m.p. b.p.
Elements Occurrence number configuration states 1st 2nd 3rd (çC) (çC)
B Concentrated deposits of borax 5 [He] 3 801 2427 3659 2180 3650
Na2[B4O5(OH)4] ⋅ 8H2O and 2s2 2p1
kernite Na2[B4O5(OH)4] ⋅ 2H2O
Al Aluminosilicate rocks, such as 13 [Ne] 1, 3 577 1816 2744 660 2467
feldspars and micas 3s2 3p1
Ga 31 [Ar] 1, 3 579 1979 2962 30 2403
3d10 4s2 4p1
In Occur as sulphides, in trace 49 [Kr] 1, 3 558 1820 2704 157 2080
amounts 4d10 5s2 5p1
Tl 81 [Xe] 1, 3 589 1971 2877 303 1457
4f14 5d10 6s2 6p1

(a) The metallic radii of the atoms do not increase regularly on descending the Group. The ionic radii for M3+ increase down the Group.
(b) The electronegativity decreases from B to Al due to difference in atomic sizes of the elements.
(c) The ionization enthalpies increase as expected (first ionization enthalpy < second ionization enthalpy < third ionization enthalpy).
The ionization enthalpy values do not decrease smoothly down the Group. The decrease from B to Al is the usual trend on
descending a Group associated with increased size. The poor shielding by d electrons and the resulting d-block contraction affect
the values for the later elements.
(d) The melting points of the Group13 elements do not show a regular trend.
(e) Boron has an unusual crystal structure while Al, In and Tl all have close-packed metal structures. Gallium has an unusual structure
with an incredibly low melting point of gallium of 30°C and also because the liquid expands when it forms the solid, that is, the
solid is less dense than the liquid.
(f) The boiling point of B is unusually high, but the values for Ga, In and Tl decrease on descending the Group as expected.
(g) The densities of elements increase down the Group.
(h) The electropositive or metallic nature of the elements increases from B to Al, but then decreases from Al to Tl.
4. Chemical properties of Group13 elements
FM.indd 4 (a) Oxidation state 8/8/2016 6:20:21 PM
(i) The elements all have three outer electrons. Apart from Tl, they normally use these to form three bonds, giving an oxidation
Tip
Work and heat conventions:
q is (+) Heat is absorbed by the system. q is (−) Heat is released by the system.
w is (+) Work is done on the system. w is (−) Work is done by the system.

3. The first law of thermodynamics


The first law of thermodynamics states that energy can neither be created nor destroyed, that is, the total energy of the system
remains constant, though it may change from one form to another.
Mathematical expressions for the first law of thermodynamics: ∆U = q + w Tools for Problem Solving 7
4. Pressure−volume work
If the process take place in a manner that pressure at each stage decreases by an infinitesimal amount dp and the volume increases
1
(c)byPlaster of paris amount
an infinitesimal (CaSO 4 dV, H2O) the total work done by the gas can be obtained by the relation
2 then
(i) Preparation: By dehydration of gypsum
150 °C
Vf
w = ∫ pext dV
V200 °C 1100 °C
Note to the Student v
CaSO 4 ⋅ 2H2O → CaSO 4 ⋅ 21 H2O i → CaSO 4  → CaO + SO3
Gypsum Plaster ofwparis Vf
Anhydride
Work done by the gas rev = 2.303nRT log
1
Vi
(ii) Properties: When powdered plaster of Paris, CaSO 4 2 H2O , is mixed with correct amount of water it sets into a solid mass
V
Work done of CaSO
on the4⋅2Hgas
2O. wrev = − 2.303nRT log f
(d) Cements Vi
(i) The principal constituents of cement used for constructional purposes are compounds of lime (i.e., calcareous).
Tip (ii) Portland cement: It is made from finely ground limestone and finely divided clay with composition: Lime-60–69%, Silica
(a) (SiO ) - isothermal
For 2an 17–25%, Alumina (Al2OV3)>- V3–8
expansion, %, Iron oxide (Fe2O3) - 2–4%, Magnesium oxide (MgO)- 1–5%, Sulphur trioxide (SO3)-
f i, work done, w, by an ideal gas is positive.
1–3%, Alkali oxides (Na
(b) For a compression, Vf <2Vi, work O + K O)- 0.3–1.5%
2 done, w, by an ideal gas is negative.
(iii) Setting and hardening:
• When cement
(a) Expression of first is mixed
law with water
for different to a plastic mass,
thermodynamic called “cement paste”, hydrating reaction begins, resulting in the for-
processes
mation of gel and crystalline products. The process of solidification comprises of setting, and then hardening.
(i)• For an isochoric process: w = − p ∆V = 0; U = q
Setting is defined as stiffening of the original plastic mass due to initial gel formation.
ext V
(ii)• For
Hardening
reversible is the
anddevelopment
irreversible of strength due
isothermal to crystallization
processes
10. Biological•significance
For irreversible change,
of Na, = −Ca
K, Mg qand w = pext (Vf − Vi )
(a) Biological functions of Na and K are very different. + expelled from cells, whereas K+ ions are not. This ion
Vf Na ions are actively
V
• For reversible change, q = −
transport is sometimes called a sodium pump, w = nRT lnand = 2.303nRT log f + +
Vi it involves bothVithe active expulsion of Na and the active take-up of K .
(b) The different ratio of Na+ to K+ inside and outside cells produces an electrical potential across the cell membrane, which is essen-
(iii) For an adiabatic process: In this process
tial for the functioning of nerve and muscle cells. q = 0, so we get ∆U = w.
(c) K+(iv)
ionsFree
inside the cell are essential for the metabolism of glucose, the synthesis of proteins, and the activation of some enzymes.
expansion
C. Tools For Problem Solving
(d) Mg2+ is concentrated in animal cells and Ca2+ is concentrated in the body fluids outside the cell.
2+ • The expansion of a gas in vacuum is called free expansion and during the process, no work is done on or by the system,
(e) Mg ions form a complex with ATP, and are constituents of enzymes for reactions involving ATP and energy release. They are also
irrespective of whether the process is reversible or irreversible.
Concept maps, important formulas provided for each chapter for last minute
essential for the transmission of impulses along nerve fibers.
(f) Mg2+ is an important constituent of chlorophyll in the green plants.
revision.
(g) Ca2+ is important in bones and teeth as apatite Ca3(PO4)2, and the enamel on teeth as fluoroapatite [3(Ca3(PO4)2) · CaF2].
(h) Ca2+ ions are important in blood clotting, and are required to trigger the contraction of muscles and to maintain the regular
beating of the heart.

TOOLS FOR PROBLEM SOLVING


Chapter-6.indd 4 6/11/2016 2:04:04 PM

Reactions of Alkali Metals


NaNH2

+FeCl3 cat.

−H2

LiOH NaNHR
NaL3Cl(s)
CH3Li Na+(am) + e− (am)
LiNHR
+H2O ∆ Solved Examples 15
+RNH2
+CH3I Na2O2
Li2O −H2 +L = chelate
−RI +NH3 +Cl−
+RNH2
LiR −H2 pV g = constant +O2 +H2O
+O2 −H2O2
where g = Cp,m+RX
/CV,m. −
H2 +e
T2
NaCl(l) Na NaOH
14. Kirchoff’s law
Li +H2
∫ ∫
−LiX for temperature dependence of enthalpies: d( ∆H ) = ∆C p dT
H1 T1 +H2O
LiH +H2 −H2
+e− ( ∆H2 − ∆H1) = ∆C p (T2 − T1)
+ROH +Mg(UO2)3OA
−H2 NaH
q
LiCl(l) change:
15. Entropy ∆S = rev +N2
+ROH
+MCl +C222
−HT2 NaOR −NaCl
Li3N= ∆H
16. Change in Gibb’s free energy: ∆Gsystem system − T ∆Ssystem
+Na[Mg(UO2)3OAc9](s)
LiOR DrG and DGç at equilibrium: ∆rG ° = − RT ln K = −2.303
17. Relationship between K, log
M =RT K
Rb, Cs [NaC222]+Na−

18. Relation between equilibrium constant to enthalpy change and entropy: ∆ rG ° = ∆r H ° − T ∆ r S ° = − RT ln K = −2.303RT log K
19. Free energy change for a multicomponent system: dG = Vdp − SdT + ∑ mi dni
i
 ∂m 
(a) Variation of m as a function of temperature:  i  = − Si
 ∂ T  p ,n
Chapter-10.indd 7 i 6/11/2016 3:05:50 PM

 ∂m 
(b) Variation of m as a function of pressure:  i  = −Vi
 ∂ p  T ,n
i

20. Van’t Hoff equation: The variation in equilibrium constant K with temperature
∆G ° = − RT ln K
K  − ∆H  T2 − T1 
log  2  =
 K1  2.303R  T1T2 
D. Solved Examples
21. Van’t Hoff isochore:
∂ lnK ∆E
=
∂T RT 2  V2   V3 
Topic-wise solved problems suitable for JEE (Main); including
22. Total work done during the one complete Carnot cycle: w = mRT1 ln   − mRT2 ln   .
T V1   V4  solved questions
23. Efficiency of the Carnot’s engine: h = 1− 2
from previous 10 years JEE Main T1 (AIEEE) papers.

SOLVED EXAMPLES
First Law of Thermodynamics 100°C. (Given: Molar enthalpy of vapourization of water at
1 bar and 373 K = 41 k J mol−1 and R = 8.3 J mol−1 K−1) will be
1. (∆H − ∆U) for the formation of carbon monoxide (CO) from its (a) 4.1 k J mol−1
elements at 298 K is (R = 8.314 J K−1 mol−1) (b) 3.7904 k J mol−1
(a) −1238.78 J mol−1 (c) 37.904 k J mol−1
(b) 1238.78 J mol−1 (d) 41.0 k J mol−1
(c) −2477.57 J mol−1 (AIEEE 2007)
(d) 2477.57 J mol−1
(AIEEE 2006) Solution
(c) The reaction involved is H2O(l) → H2O(g), where ∆ng =
Solution
1 mol, ∆vapH = 41 k J mol−1. Now,
(d) H = U + pV ; ∆H = ∆U + ∆(pV) ⇒ ΔH − ΔU = Δ(pV)
Also, ∆(pV) = ∆ngRT for an ideal gas, therefore, ΔH − ΔU ∆U = ∆H − ∆ngRT
= ∆ngRT
= 41 kJ mol−1 − 1× 8.3 × 10 −3 kJ K −1 mol−1 × 373 K
∆ng = 2 for the reaction O2 (g) + 2C(s) → 2CO(g)
= 41− 3.096 kJ mol−1 = 37.904 kJ mol−1

∆H − ∆U = 1 × 8.314 × 298 = 2477.57 J mol −1 3. A piston filled with 0.04 mol of an ideal gas expands reversibly
from 50.0 mL to 375 mL at a constant temperature of 37.0°C.
2. Assuming that water vapour is an ideal gas, the internal energy As it does so, it absorbs 208 J of heat. The value of q and w for
(∆U) when 1 mol of water is vapourized at 1 bar pressure and the process will be (R = 8.314 J mol−1 K−1) (ln 7.5 = 2.01)

Chapter-6.indd 15 6/11/2016 2:05:59 PM

FM.indd 5 8/8/2016 6:20:22 PM


CO(NH2)2(s) + H2O(l) → 2NH3(g) + CO2(g), ∆G° = +6.3 k J obtained from the combustion of 48.0 g of methane is
−(∆G ) = wuseful work
53. At 1500°C, Kc = 5.67 for the reaction
 1 mol CH4   800.8 kJ 
CH4(g) + H2O(g) CO(g) + 3H2(g) kJ = ( 48.0 g CH4 )  = 2.40 × 103 kJ
 16.0 g CH4   1 mol CH4 
Calculate the value of ΔG°1773  for the reaction at that 55. For a reaction at equilibrium,
temperature. (a) ∆G = ∆G° = 0
(a) −25.6 k J (b) −30.2 k J (b) ΔG = 0 but not ΔG°
(c) −22.7 k J (d) −35.6 k J (c) ∆G° = 0 but not ∆G
Solution (d) ∆G = ∆G°≠ 0
° = −RT ln K = −(8.314 J mol−1 K−1)(1773 K)(ln 5.67)
(a) ∆G1773 Solution
= −25,600 J = −25.6 k J (b) For a reaction at equilibrium ∆G = 0 but not ∆G. ∆G = ∆G°
54. What is the maximum amount of useful work that could pos- + RT ln Q at equilibrium, ∆G = 0 and ∆G° = −RT ln Keq
sibly be obtained at 25°C and 1 atm from the combustion of
vi Note to the Student 48.0 g of methane, to give CO2 and H2O
56. Methanol, a potential replacement for gasoline as an auto-
motive fuel, can be made from H2 and CO by the reaction
∆ f G°[CO2(g)] = −394.4 k J mol−1, ∆ f G°[H2O(g)] = −228.6 k J CO(g) + 2H2 (g)  CH3OH(g)
mol−1, ∆ f G° [CH4(g)] = −50.79 k J mol−1 At 500 K, this reaction has  KP  = 6.25 × 10−3. Calculate
ΔG°500 for this reaction in units of kJ.
(a) 2.40 × 103 kJ (b) −3.1 × 10 4 kJ
(a) 25.6 k J (b) 36.9 k J
E. Advanced Level Problems
(c) 4.01 × 10 kJ 3
(d) −5.2 × 10 kJ 3
(c) 85.2 k J (d) 21.1 k J
Solution Solution
These includes
(a) We must firstsolved questions
determine ∆G° for the reaction: from previous
(d) ∆G° 10
= −RTyears
ln K JEE Advanced (IIT-JEE) 500 p
CH (g) + 2O (g) → CO (g) + 2H O(g) Practice
= −(8.314 J K mol )(500 K) ln (6.25 × 10 ) Exercise −1 −1 −3 39
papers. ∆G° = (sum ∆ G° [products]) − (sum ∆ G° [reactants])
4 2

f
2 2
= 2.11 × 10 J = 21.1 k J
f
4

vacant. There is no d-orbital of suitable energy in boron Oxidation state of sulphur in compounds (A), (B), (C) and
ADVANCED LEVEL
atom. So, it PROBLEMS
can accommodate only one additional elec- (D) are as follows:
tron pair in its outermost shell. NaHSO3 = (+1) + (+1) + x + 3(−2) = +4
1. The direct conversion of A to B is difficult; hence it is carried Na2SO3 = 2(+1) + x + 3(−2) = +4
2. For the reaction,
+ B(OH)
H2O shown
out by the following path:
3 OH+2 The oxidation state of S in Na2S2O3 is +6, and −2.
2CO + O2 → 2CO2 ; ∆H = −560 kJ
Base CAcid DB− −2
Two moles of CO and one mole ofSO 2 are taken in a container
HO OH of volume 1 L. They completely O− form
OH S+6 two moles of CO2, the
gases deviated appreciably from− ideal O behaviour. If the pres-
O
− + sure in the vessel changes from 70 to 40 atm, find the magni-
A B 4] + H
[B(OH)
∆U atof +5 kand
The oxidation
tude (absolute value) of state 500S in
K. Na 4O6 =is0.1
(1 L2Satm J) 0.
Given (IIT-JEE 2006)
29. (CH3)2 SiCl2 on hydrolysis produces O O
∆ S
(a) (CH3)2Si(OH)2 (A → C) = 50 eu(b) (CH3)2Si O Solution
+5 0 0 +5
(c) [ O (CH3)2Si∆S(CO→D)]n= 30 eu(d) (CH3)2SiCl(OH) From the definition O−ofSenthalpy,
S S S O−
∆ S = 20 eu ΔH = ΔU + Δ(pV) ⇒ ΔH = ΔU + VΔp + pΔV (at constant
Solution (B→D)
volume ΔV = 0)
O O
(c) e.u.
where It appears at the
is entropy first
unit, sight
then ∆S(A that Me SiCl2 on hydrolysis will 31. ΔU = ΔHof− VΔp
Which = −560 + 1 isomers
the following × 30 × 0.1 of=phosphorus
−557 k J are thermo-
→B) is 2
produce Me2Si(OH)2 which ultimately upon loss of water, dynamically least and most stable?
(a) +100
willeuform Me2Si O. But silicon (b) +60 eu because of its very 3. For the process H2O(L , 1 bar , 373 K) → H2O(g, 1 bar, 373 K),
atom,
the (a) White
correct set (least),
of thermodynamic (b) Yellow
Red (most)parameters is (least), Red (most)
(c) −100 eu size in comparison(d)
large −60 eu is unable to form π
to oxygen, (c) =Red
(a) ∆G S = + veWhite (most) (d) White (least), Black (most)
0, ∆(least),
(IIT-JEE 2006)
bond. Thus, the product of hydrolysis is polymeric in (b) ∆G = 0, ∆S = −ve
Solution Solution
nature. (c) ∆G = + ve , ∆S = 0
(b) ∆SA →B = ∆SA → C + ∆S(C→D) − ∆SB→D (d) Black phosphorous is most layered because it has lay-
SO2 Na2 CO3 Elemental I (d) ∆G = −ve , ∆S = + ve
30. Na2CO3 = ered structure, whereas white is the(IIT-JEE least because it is
50+→ ) 
30 (−A20 =+ 
60 eu→ (B) Sulphur
 /∆
→ (C ) 2 → (D) 2007)
very reactive and catches fire in air (so, it is stored under
Identify the compounds (A), (B), (C), (D), and give oxidation water).
state of sulphur in each compound.
32. A pale blue liquid obtained by equimolar mixture of two
Solution
gases at −30°C is
Chapter-6.indd 23 Na2CO3 + 2SO2 + H2O → 2NaHSO3 + CO2 (a) nitric oxide (b) nitrogen trioxide 6/11/2016 1:00:47 PM
(A) (c) nitrogen tetroxide (d) nitrogen pentoxide
2NaHSO3 + Na2CO3 → 2Na2SO3 + H2O + CO2
F. Practice Exercise (B )
Solution
(b) N2O3 can only be obtained at low temperatures. It can be
Na2SO3 + S → 2Na2S2O3
made by condensing equimolar of NO and NO2 together.
Graded as Level I (JEE Main)(C) and Level II (JEE Advanced).
2Na2S2O3 + I2 → 2Na2S4 O6 + 2NaI
All
This gives a blue question
liquid or solid which types
is unstableas
and per
dis-
sociates into NO and NO2 at −30°C.
JEE (Main and Advanced)(Dcovered.)

NO + NO2 → N2O3

PRACTICE EXERCISE
Level I 5. Salicylic acid reacts with ICl vapour to produce
(a) iodinated product. (b) chlorinated product.
Single Correct Choice Type (c) mixture of both. (d) virtually no reaction.
32 Chapter 6 Thermodynamics
1. Which of the following is not a peroxy acid? 6. Which of the following has the smallest bond angle?
(a) Caro’s acid (b) Marshall’s acid (a) H2S (b) H2O (c) H2Se (d) H2Te
(c) Thiosulphuric acid (d) Sulphurous acid
66. Which of the following sets show the7.intensive The mostproperties?
stable hydrate of noble72. gases
When is 2 mol of NO2 change into equilibrium mixture with
2. Which of the following pairs (a) ofTemperature and pressure
species are non-linear? (a) Kr ⋅ 4H2O (b) NKr
2O 4, then
⋅ 6H 2O ∆G° is

(a) OCN and Br3 − (b) Viscosity and surface
(b) (SCN)2 and I5 − tension (c) Xe ⋅ 4H O
Practice
2
Exercise 31 (d) Xe
(a) ⋅ 6H2O k J
−5.40 (b) +5.40 k J (c) −6.24 k J (d) 6.24 k J
(c) NCN2− and N3− (c) Refractive index
(d) HN3 and (CN)and specific heat
8. Which of the following is an organo silicon
Answer Key
polymer? 33
2 73. When 1 mol of N2O4 changes into equilibrium mixture with
∆H2 3 (d) Volume and heat capacity p (a) Silica (b) Orthosilicic acid Answer Key 33
(a) ∆ f H = − ∆H1 −
3. Xenon difluoride 2
− ∆H3
is
2 (c)
A Silicon
B carbide
NO2, then
(d) Silicic acid
∆G° is
∆H2change
78.(b) The
(a) entropy
linear.
∆fH = ∆H1 + − ∆H367.
3
Which
in an of the
adiabatic
(b) following
process
angular. is process1 atm
take(s)83.
place withthe
Match decrease
process in (a) −5.40 k J (b) 0.84 k J
with the expression. (c) −0.84 k J (d) 6.24
(a)
(c) zero.
trigonal.
2 2
entropy?(d) pyramidal. 9. How many peroxy linkages are present in pyrophosphoric
∆H2 3
(c) ∆ f H = ∆H1 − − ∆H3
(b) always 2positive. 78. The entropy change Column
in acid?
an I
adiabatic process is 74.
ColumnSelect II out the
83.correct
Matchstatement.
the process with the expression.
2 (a) 3CO2(g) → 2O3(g) 0.5 atm C
4. (c)The reagent ∆used
H2 3to distinguish between
always negative. H (a)
O and O is
zero. (a)(a)For
0 spontaneous (b) 1 (c) (a)
2 The conversion(d) 3 of N2O4 into 2NO2 is spontaneous.
(d) ∆ f H = − ∆H1 + + ∆H3 (b) Sugar + Water 2 2 → Solution3 ∑ ∆ bondH (reactants) − ∑ ∆bond
(p) (b)
(d)
(a) sometimes
PbS.
2 2
positive (c) (b) starchnegative.
and sometimes and
(b) iodine.
always 10. A + SbF5 → B V ColumnofI 2NO2 into N2O4 is non-spontaneous.
The conversion Column II
54. Find the maximum work available, in 2SO
kilojoules, from +O
(g)the oxi- → 2SO3positive.
2(g)powder. (g) + + X−(c)
H (products)
(c) KMnO . (d) 2 bleaching (c)→always negative.
20 L
B + 40 L
tert-butane → [tert-butyl] + H2The attainment
(a) For of equilibrium from N2O4 and from NO2
spontaneous
+ (p) ∑ ∆ bondH (reactants) − ∑ ∆bond
4 octane, C H (l), (d) Hat (g) Cl
79.dation of 1 mol
If water in ofan insulated by oxygen
8 18 vessel −10°C
to give 2(g)
COsuddenly
(g) 2HCl(g)
freezes,
(a) the
The entropy change (b) For endothermic
for the overall process is 0 (zero).
and H2O(l) at 25°C and 1 atm. Given that free 2
and sometimes negative. (q) ∆r H =are ∆ rboth equally spontaneous.
2 U
entropy
(∆ f G°) inchange
k J mol−1 isofC8the
H18(g)system
energy of forma-
will be (d) sometimes
(b) ΔH > ΔU forpositive
the overall process.
= +17.3, O 2(g) = 0, CO2(g) = q. = 0 and dq < 0, then for H (products)
−168. In a given process on an (d)
tion (c) ideal gas,wdw
Total work, The attainment of equilibrium from 2 mol of NO2 with
(a) +10andJ K
= −394.359 H2−1O(l)mol
= −237.2. (d) Total work = − 620.77 J.
(c) Bond dissociation (r) ∆G < 0
the gas 79. If water in an insulated vessel at −10°C suddenly freezes, Nthe 2O4 is more spontaneous than the conversion
(b) −10 J K mol k J (c) −5277 k J (d) −3327 k J 60. An adiabatic process involvesenergy
(a) −5307 k J (b)−1−2575−1 (b) For endothermic (q) ∆ r H of
= ∆2NO
rU 2
(c) zero. (a) the volume entropy will increase.change of the system will be into N2O4.
(a) no exchange of heat. (d) For solids and liquids (s) ∑ H (products) > ∑Bond
(c) H (reactants) dissociation (r) ∆G < 0
Level
Chapter-11.indd 39 II
(d) equal to that of surroundings. (b) the temperature (a) +10 will K−1 mol
(b) Jexchange
decrease. −1 with
of heat change in volume.
in ain thermochemical 6/13/2016 1:55:11 PM
(c) work −1 done equal−1 to change internal energy. Paragraph for Questions energy 75 to 77: Enthalpy of neutralization is
Multiple Correct Choice Type (c) the pressure will remain (b) −10 J K mol
constant.
in temperaturereaction
80.When The amelting point ofadiabatically
a (d)
solidthe isagainst
300 K and
(d) increase
its zero.
latent heat of
of the system under reversible
defined as the enthalpy change when 1 mol of H+ in dilute solution
55. real gas expands internal (c)
a finiteenergy will decrease.
compression. (d) For solids and liquids (s) ∑ H (products)
fusion is 600 cal mol−1. The entropy change for
pressure, the fusion of combines with 1 mol of OH− to give rise to undissociated water, > ∑ H (reactants)
(a) its internal energy increases. (d) 61.equal
Whichto that
of the of surroundings.
following statement(s) is/are correct?
84. in a thermochemical
(b)1 its
mol of the
internal energysolid (in −1
cal K ) at
Passage
decreases. Typethe same temperature wouldof constant entropy and constant volume will that is, H (aq) + OH (aq) → H2O(l) ∆H = −57.54 k J mol−1 or −13.7
(a) The system
+ −
(c)be attain the equilibrium in a state of minimum energy. −1 reaction
its temperature always decreases. 80. The melting point Column
of a solid I
is 300 K and its latent kcalColumn
mol
heat . ofIIWhen 1 mol of strong monoprotic acid is mixed with
(b) Paragraph for
(c) Questions 69
(b) toThe71: Figure −1below is a volume vs.
(d)(a)
its entropy entropy of the universe is increasing.
200 always remains 2constant. 0.2 fusion(d) is 60020 process
cal mol . The entropy change 1 mol ofwstrong
of base, log the(pabove neutralization reaction takes place.
(c) (a) Reversible U,V < for
isothermal0, the fusion
(p) = −2.303nRT 2/p1)
The would be spontaneous when (∆S)
temperature plot forkJ;one mole(∆U) ofS,Vgas.
weak84.
56. For the C(s) + H2O(g) → CO(g) + H2 (g) reaction; ΔH o = +131
> 0. −1) at the same temperature
1 mol of the solid (in cal K process When would a acid/base reacts with a strong base/acid, the
Matrix-Match
ΔSo = +134 J K−1 Type (d) The process would be spontaneous when (∆S)U,V > 0,
−1
Mark out the correct statement(s). beA (∆U)BS,V < 0.
(b) Reversible adiabatic (q) w = 0 release of heat is lessColumn than 57.54 I k J mol . It is because of theII fact
Column
20 L
81.(a)Match the
Reaction column correctly:
is spontaneous even at room temperature. (a) 62.
200 (b) 2statementsprocess
Which of the following (c) 0.2
are correct? that20
(d) these acids/bases are not completely ionized in the solution.
(b) Reaction is not spontaneous at room temperature. (a) When ∆G = 0, the system is at equilibrium. (a) Reversible isothermal (p) w = −2.303nRT log (p2/p1)
(c)Column
Reaction isI spontaneous above 705°C. Column II Some heat is consumed in ionizing them.
(b) When ∆G < 0, the process(c)willIrreversible
be spontaneous. adiabatic (r) w = −2.303nRT log (V process
2/V1)
(d) ΔH outweighs the entropy factor TΔS at Matrix-Match
room Type
(c) When ∆G is negative, the process is said to be exergonic.
V

(a)temperature.
Extensive properties 10 (p)
L Density (at constant process is said to be 75. When 1 g equiv.
p, V,is positive, theprocess of NaOH reacts with 0.33 g equiv. of HCl,
and C
(d) When ∆G (b) Reversible adiabatic (q) w = 0
T)
81. Match the column correctly:
endergonic.
(d) mole
Irreversible enthalpy change is −4.6 cal. When 0.66 g equiv. of HCl is fur-
gas isothermal (s) w = nCV , m (T2 − T1) process
57. Choose the incorrect combination(s).
Graph for one
(b) Closed system (q) Bursting 63. When a solid melts there will be
of tire process ther added, enthalpy change would be
ΔH ΔS Temperature (T) Spontaneity Column I (a) an increase in enthalpy. Column II (c) Irreversible = −2.303nRT
(c)+ Path
− function (r) Additive in nature (b) a decrease in free energy. (a) −13.7 cal (b) −9.2 cal 13.7 cal (r)(d)w 9.2
(c) adiabatic cal log (V2/V1)
(a) Any T Non-spontaneous
(a) 200Extensive
K(c) no400changeproperties
K in enthalpy. (p) Density (at constant (t) w = p,−V,popp (V2 − V1) process
(b) − −
(d) Adiabatic Low T Non-spontaneous
(s) U ≠ constant (d) a decrease in internal energy.
and T) 76. In the following reaction involving neutralization of HF
T (d)NaOH
Irreversible
(c) + +
(e) Intensive
Low T Spontaneous
(t) Heat 64. Which statements areIntegercorrect forType neutralization of a strong (a weak acid) with (a strongisothermal
base). (s) w = nCV , m (T2 − T1)
(d) − + Anyproperties
T 69. The process A (b)
Spontaneous → B isClosed strong alkali in aqueous solution(q)
acid by asystem Bursting of tire
at 25°C. process→ NaF(aq) + H O(l)
(a) It is an endothermic process. HF(aq) + NaOH(aq) 2
82.One Match the
an signs of ∆H and (a)
gas (C∆S
isobaric.
with the nature (c) of Path
(b) function
the Itreaction. (b) isochoric.
85. Standard
can be represented as H3O (aq) + OH + Gibbs energy
(r) (aqAdditive
− change
) → 2H2O(l) in nature for a reaction is zero. The value
the heat of neutralization is −68.6 k J mol−1.(t)Thisw value is (V2 − V1)
58. mole of ideal diatomic V = 5 cal) was trans- = − popp
formed from initial 25°C and 1 L to(c) isothermal.
the state when temper- (c) The enthalpy (d) change
adiabatic. ofmole
per equilibrium
of H2O formed constant
is inde- is ___.
ature is 100°C and volume 10 L. Then for this process (R = (d) pendent of the acid or alkali used. (s) U ≠ constant
Adiabatic much higher than the heat of neutralization of strong acid
Column I
cal mol−1 K−1) (take calories 70.
as unitThe
Column II for2 86. A gas expands against a constant external pressure so that −1
with strong base (= −57.3 k J mol ). This is because
pressure at C is
of energy and kelvin (d) Some sort of energy is consumed to make the acid or Integer Type
(a) 3.284 atm (e) Intensive onlyalkali properties work (t) Heat
temperature). Mark out thenegative
correct one(s). (p) Spontaneous completely ionized.
(a) Negative and at
1
low(b) 1.642the atm done is 607.8 J. The work (a) done in litre atmosphere
in aqueous solution, HF is behaves as a strong acid.
(a) ∆H = 525 65. C(s) + O2 → CO(g); ∆H ° = − 26 kcal mol−1
(c) 0.0821 temperature.
atm 2 (d) 0.821___. atm (b) there is85. hydration of F−Gibbs
Standard ion inenergy
aqueous solution,
change for aand pro- is zero. The value
reaction
(b) ∆S = 5 ln
373
+ 2 ln 10 82. Match the signs 1 of ∆H and ∆S with the nature of the reaction.cess when
is exothermic.
(b) Negative 298 and positive (q) Spontaneous only CO(g)at+ high
O2 (g) → CO87.2 (g); The∆amount H ° = − 68 kcalofmol
−1
ice (in g) that remains 52ofgequilibrium
of ice is constant is ___.
(c) ∆U = 525 71. Work done in the process C 2 A is
temperature. → addedColumn to 100 g II (c)
of water at 40°C is ___. non-solubility of HF in water.
(d) ∆G of the process cannot be(a) zero using givenColumn
calculated I is/are correct
Which
(b) statement(s)?
8.21
of COL atm −1
(d) formation 86.ofAhydrogen
gas expands against
bonding amonga constant external pressure so that
HF molecules.
(c)information.
Positive and positive (r) Spontaneous (a) Heat at of formation
all 2 is −68 kcal mol
(c) 16.2 L atm (a) Negative (b) Heat ofand combustion 88.of C(s)
(d)negative
cannot One isbe
−26mole of −1monatomic perfect
predicted
(p) Spontaneous
kcal mol only atgas,
lowinitially at 1.0 theatm
workpres-
done is 607.8 J. The work done in litre atmosphere is
temperature.
59. Mark out the correct one(s) about the pV plot system shown (c) Heat of combustion of CO(g) is −68
sure and at 300 kcal mol −1
K is heated77. Heat to
reversibly of 400
dissociation
K at constant CH3COOH is 0.005 kcal g−1; hence,
of vol-
in below figure. Paragraph for Questions 72 (d) to
Heat74: Consider
of formation of CO(g)theis −26following temperature.
−1
kcal mol reac- ___.
(d) Positive and negative (s) Non-spontaneous at all ume. Calculate the change in internal enthalpyenergy
change andwhen 1 mol of Ca(OH)2 is completely neu-
the work
tion N2O4(g) →2NO (g). The
2(b)
temperature. Gibbsand
Negative energy of the
positivedone.reaction occur-
(q) Spontaneous only at high by CH
tralized 87.3COOH
The amount
is of ice (in g) that remains when 52 g of ice is
ring at 298 K and 1 atm has been plotted against the fraction of
temperature. (a) −27.4 kcal added to 100 g(b) −13.6atkcal
of water 40°C is ___.
N2O4 dissociated as shown in below figure.
(c) Positive and positive (r) Spontaneous at(c) −26.8
all kcal (d) −27.1 kcal
88. One mole of monatomic perfect gas, initially at 1.0 atm pres-
Chapter-6.indd 31
temperature. Paragraph for Questions
6/11/2016 1:01:36 PM
78 toat80: The
K isthermodynamic property
ANSWER KEY 6
that measures
sure and 300 heated reversibly to 400 K at constant vol-
5
(d) Positive and negative (s) Non-spontaneous at all the extent ume.ofCalculate
molecular disorder
the changeis in
called entropy.
internal energy and the work
temperature. The direction of a spontaneous
done. process for which the energy is
∆G° (kJ mol−1)

Level I constant is always the one that increases the molecular disorder.
∆G° = 5.40 kJ

4
1. (c) 2. (a) 3. (a) 4. (c) 5. (b)Entropy change of phase transformation can be calculated using
6. (b)
7. (b) 8. (c) 3 9. (a) 10. (a) 11. (b)Trouton’s formula (∆S = ∆H/T). In the reversible adiabatic pro-
12. (c)
cess, however, ∆S
18. (a) be zero. The rise in temperature in isobaric
will
2 ANSWER KEY
13. (a) 14. (c) 15. (b) 16. (a) 17. (c)
19. (d) 20. (d) 21. (b) 22. (c) 23. (c)and isochoric conditions 24. (b) is found to increase the randomness or
FM.indd 6 25. (b) 26. (a) 1 27. (d) 28. (b)
N O 29. (d)entropy of the system. 30. (a) 8/8/2016 6:20:25 PM
2 4
31. (b) 32. (c) 33. (d) 34.kJ
0.84 (a) 35. (d) 36. (a)
2 1
(a) Extensive properties (p) Density (at constant p, V, process
and T) (d) Irreversible isothermal (s) w = nC (T − T )
V ,m 2 1
(b) Closed system (q) Bursting of tire process
(c) Path function (r) Additive in nature (t) w = − popp (V2 − V1)
(d) Adiabatic (s) U ≠ constant
Integer Type
(e) Intensive properties (t) Heat

82. Match the signs of ∆H and ∆S with the nature of the reaction. 85. Standard Gibbs energy change for a reaction is zero. The value
of equilibrium constant is ___.
Column I Column II 86. A gas expands against a constant external pressure so that
(a) Negative and negative (p) Spontaneous only at low the work done is 607.8 J. The work done in litre atmosphere is
temperature. ___.
(b) Negative and positive (q) Spontaneous only at high
temperature.
87. The amount of ice (in g) that remains when 52 g of ice is
added to 100 g of water at 40°C is ___.
Note to the Student vii
(c) Positive and positive (r) Spontaneous at all 88. One mole of monatomic perfect gas, initially at 1.0 atm pres-
temperature. sure and at 300 K is heated reversibly to 400 K at constant vol-
(d) Positive and negative (s) Non-spontaneous at all ume. Calculate the change in internal energy and the work
G. Answer Key temperature. done.

ANSWER KEY

Level I
1. (c) 2. (a) 3. (a) 4. (c) 5. (b) 6. (b)
7. (b) 8. (c) 9. (a) 10. (a) 11. (b) 12. (c)
13. (a) 14. (c) 15. (b) 16. (a) 17. (c) 18. (a)
19. (d) 20. (d) 21. (b) 22. (c) 23. (c) 24. (b)
25. (b) 26. (a) 27. (d) 28. (b) 29. (d) 30. (a)
31. (b) 32. (c) 33. (d) 34. (a) 35. (d) 36. (a)
37. (c) 38. (b) 39. (c) 40. (d) 41. (c) 42. (a)
43. (c) 44. (a) 45. (b) 46. (a) 47. (b) 48. (d)
49. (a) 50. (d) 51. (b) 52. (d) 53. (a) 54. (a)

Chapter-6.indd 33 6/11/2016 1:01:40 PM

34 Chapter 6 Thermodynamics

Level II
55. (b), (c), (d) 56. (b), (c), (d) 57. (b), (c) 58. (a), (b) 59. (a), (d) 60. (a), (b),(c)
61. (a), (b), (d) 62. (a), (b), (c), (d) 63. (a), (b) 64. (b), (c) 65. (c), (d) 66. (a), (b), (c)

H. Hints and Explanations


67. (a), (c)
73. (c)
68. (b), (d)
74. (d)
69. (b)
75. (b)
70. (b)
76. (b)
71. (c)
77. (c)
72. (c)
78. (a)
79. (c) 80. (b) 81. (a) → r; (b) → t; (c) → s; (d) → q; (e) → p 82. (a) → p, (b) → r, (c) → q, (d) → s
Answers and explanations provided for all practice questions.
83. (a) → r; (b) → s; (c) → p; (d) → q
87. (44) 88. (0)
84. (a) → p, r; (b) → s; (c) → q; (d) → t 85. (1) 86. (6)

HINTS AND EXPLANATIONS


Level I 13. (a) ∆ f H° is the enthalpy of formation; in standard state of
Single Correct Choice Type 1 mol of a substance from its constituent elements in their
1 1
∆H ∆H standard states, for example, H2 (g) + F2 (g) → HF(g) .
1. (c) ∆S = ⇒ 14.4 = ⇒ ∆H = 391 × 14.4 2 2
T (K ) 391 14. (c) ΔH = ΔU + pΔV ⇒
−1 5630
= 5630 cal mol = = 93.8 caal g−1 8.314 JK −1 mol−1
60 100 = ∆U + 1.5 atm (2 − 8 ) L ×
2. (a) Using Kirchhoff’s equation, H2 = H1 + (T2 – T1)ΔCp 0.0821 L atm K −1 mol−1
⇒ ΔU = 1011.4 J
= –3.57 + (373 – 273) × 0
= –3.57 kJ mol–1. 15. (b) In an ideal gas, there are no intermolecular forces of
attraction. Hence, Tf = Ti for both reversible and irreversi-
3. (a) ΔHsub = ΔHfus + ΔHvap ble processes.
ΔHvap = ΔHsub – ΔHfus = (57.3 – 15.5) kJ mol–1 = 41.8 kJ mol–1.
16. (a) The third law of thermodynamics states that the entropy
4. (c) In a reversible process, the driving force and opposing force of a perfectly crystalline solid approaches zero as the
of that process are nearly equal, and hence, the system and temperature approaches absolute zero. However, some
the surrounding always remain in equilibrium with each other. solids do not have zero entropy even at absolute zero, for
5. (b) Conceptual example, CO, NO, H2O, etc.
17. (c)
6. (b) A balanced chemical equation that indicates the
enthalpy change accompanying the reaction is called a 1 112
∆Hcombustion = ( −241.8 − 283) kJ mol−1 × mol = − 1312.0 kJ
thermochemical equation. It gives the physical states of 2 22.4
the reactants and products, and its ΔH° value is true only
18. (a) Density is mass per unit volume and specific volume is
when the coefficients of the reactants and products are
volume per unit mass.
taken to mean moles of the corresponding substances.
19. (d) The required equation is:
This ΔH° value provides the information that the reac-
tion is exothermic or endothermic. It gives information 1
H2 (g) + O2 (g) → H2O(g)
about the allotropic form (if any) of the reactants. Thus, 2
a thermochemical reaction does not tell about whether The equation can be obtained as:
the reaction is spontaneous or not. 1
H2 (g) + O2 (g) → H2O(l); ∆H298K = − 68.32 kcal (1)
7. (b) ΔG = –2.303 RT log K 2
1.546 × 10–3 = −2.303 × 8.314 × 773 × log K H2O(l) → H2O(g); ∆H = 10.52 kcal (2)
K = antilog (−0.105). Adding Eqs. (1) and (2),
1
8. (c) As the system is insulated from the surroundings, no H2 (g) + O2 (g) → H2O(g); ∆H = − 57.80 kcal
heat enters or leaves the system. Hence, ΔS = q/T = 0. 2
20. (d) The heat of formation of HCl is to be calculated on per
9. (a) Melting of ice at 280 K is a non-spontaneous process.
mole of HCl produced
44
10. (a) Work is not a state function; instead it is dependent on Heat of formation of HCl = = − 22 kcal. (as ∆H = −44
the path of the reaction. kcal per 2 mol of HCl) 2

11. (b) The larger the value of bond enthalpy, the stronger the 21. (b) Polymerization reaction nCH2 CH2 → (CH2 CH2)n
bond. 1 mol of C C bond is broken and 2 mol of C C bonds
12. (c) As ΔG = −ve for spontaneous change and ΔG = ΔH − TΔS, are formed per mole of ethylene.
we have ∆H = 590 − 2 × 331 = 590 − 662 (590 is spent, and 2 × 331
∆H 4000 is formed).
T ∆S > ∆H ⇒ T > ⇒T > ⇒ T > 400 K = −72 k J mol−1 of ethylene.
∆S 10

Chapter-6.indd 34 6/11/2016 1:01:44 PM

FM.indd 7 8/8/2016 6:20:27 PM


FM.indd 8 8/8/2016 6:20:27 PM
Contents
Note to the Student iii Advanced Level Problems 78
Practice Exercise 79
Chapter 1 Some Basic Concepts of Answer Key 82
Chemistry 1 Hints and Explanations 83
Summary 2
Tools for Problem Solving 9
Solved Examples 10
Chapter 4 Chemical Bonding 87
Representation of Physical Quantities 10 Summary 88
Concept of Atoms, Molecules and Ions 11 Solved Examples 100
Mole Concept 11 KÖSSEL–LEWIS Approach to Chemical Bonding 100
Percentage Calculation in Compounds 12 Ionic Bond 102
Empirical and Molecular Formula Determination 13 Polar and Non-Polar Covalent Bonds 102
Concentration Terms 14 Bond Characteristics 102
Stoichiometry of Reactions 16 VSEPR Theory 103
Advanced Level Problems 18 Hybridization Theory 105
Molecular Orbital Theory 107
Practice Exercise 20
Hydrogen Bonding 109
Answer Key 23
Advanced Level Problems 109
Hints and Explanations 23
Practice Exercise 112
Solved JEE 2016 Questions 29
Answer Key 116
Hints and Explanations 117
Chapter 2 Atomic Structure 31
Solved JEE 2016 Questions 122
Summary 32
Tools for Problem Solving 40
Solved Examples 41 Chapter 5 States of Matter 125
Nature of Electromagnetic Radiation 41
Summary 126
Planck’s Quantum Theory 42
Solved Examples 130
Photoelectric Effect 42
Bohr’s Atomic Model 43 Gas Laws (Boyle’s Law, Charles’ Law, Gay–Lussac’s Law,
Dual Nature of Matter 45 Avogadro’s Law) 130
Quantum Numbers and Rules for Filling Electrons 46 Ideal Gas Equation 131
Combined Gas Law 132
Advanced Level Problems 48
Mixtures of Gases (Dalton’s Law, Graham’s Law) 132
Practice Exercise 52 Kinetic Theory of Gases (RMS Velocity, Most Probable Velocity,
Answer Key 56 Average Velocity, Most Probable Kinetic Energy) 133
Hints and Explanations 56 Real Gases (van der Waals Equation, Compressibility
Factor) 135
Solved JEE 2016 Questions 61
Advanced Level Problems 136

Chapter 3 Classification of Elements and Practice Exercise 140

Periodicity in Properties 63 Answer Key 146


Hints and Explanations 146
Summary 64
Tools for Problem Solving 70 Solved JEE 2016 Questions 152
Solved Examples 71
Modern Periodic Table 71 Chapter 6 Thermodynamics 155
Periodic Trends in Properties 72 Summary 156
Chemical Reactivity 77 Tools for Problem Solving 168

TOC.indd 9 8/8/2016 7:12:57 PM


x Contents

Solved Examples 169 Chapter 9 Hydrogen 281


First Law of Thermodynamics 169
Pressure Volume Work 170 Summary 282
Enthalpy 171 Tools for Problem Solving 289
Heat Capacities 171 Some Reactions of Hydrogen 289
Thermochemistry 172 Solved Examples 290
Entropy 174 Preparation, Properties and Uses of Hydrogen 290
Gibb’s Free Energy 175 Isotopes 291
Hydrides 292
Advanced Level Problems 177
Physical and Chemical Properties of Water and Heavy Water 293
Practice Exercise 182 Hydrogen Peroxide 293
Answer Key 187 Hydrogen as a Fuel 295
Hints and Explanations 188 Advanced Level Problems 295
Solved JEE 2016 Questions 193 Practice Exercise 296
Answer Key 299
Chapter 7 Equilibrium 195 Hints and Explanations 300
Summary 196 Solved JEE 2016 Questions 306
Tools for Problem Solving 209
Solved Examples 210
Equilibrium in Physical and Chemical Processes 210
Chapter 10 The s-Block Elements 307
Factors Affecting Equilibria and Le Chatelier’s Principle 216 Summary 308
Acid-Base Concept 218 Tools for Problem Solving 313
Acid-Base Equilibria 219 Reactions of Alkali Metals 313
Hydrolysis of Salts and pH Determination of their Aqueous Solved Examples 314
Solutions 220 General Trends in Physical and Chemical Properties of
Buffer Solutions 221 Elements 314
Equilibria for Sparingly Soluble Salts 221 Anomalous Properties of the First Element of Each Group and
Advanced Level Problems 223 Diagonal Relationship 315
Preparation and Properties of Some Important Compounds of
Practice Exercise 229
Sodium 315
Answer Key 238 Compounds of Calcium 316
Hints and Explanations 239 Biological Significance of Na, K, Mg and Ca 317
Solved JEE 2016 Questions 253 Advanced Level Problems 317
Practice Exercise 317
Chapter 8 Redox Reactions 255 Answer Key 319
Summary 256 Hints and Explanations 320
Solved Examples 264 Solved JEE 2016 Questions 322
Modern Approach to Redox Reactions 264
Types of Redox Reactions 265
Balancing of Ionic Redox Reactions 266 Chapter 11 The p-Block Elements 323
Applications of Redox Reactions 267 Summary 324
Stoichiometry of Redox Reactions 267 Solved Examples 348
Redox Reaction Titrations 269 General Properties of Group 13 Elements 348
Advanced Level Problems 269 Compounds of Boron 349
Practice Exercise 272 Compounds of Aluminium 350
General Properties of Group 14 Elements 350
Answer Key 275
Compounds of Silicon 351
Hints and Explanations 275 General Properties of Group 15 Elements 352
Solved JEE 2016 Questions 279 Chemical Properties of Group 15 Elements 353

TOC.indd 10 8/8/2016 7:12:57 PM


Contents xi

Compounds of Phosphorus 353 Solved Examples 472


General Properties of Group 16 Elements 354 Nomenclature 472
Allotrope and Oxygen 354 Structure of Aromatic Compounds 472
Compounds of Sulphur 354 Chemical Properties 473
Chemical Properties of Group 17 Elements 355 Advanced Level Problems 479
Oxoacids of Halogens 355
Practice Exercise 482
General Properties of Group 18 Elements 355
Compounds of Xenon 355 Answer Key 486
Advanced Level Problems 356 Hints and Explanations 487
Practice Exercise 361 Solved JEE 2016 Questions 491
Answer Key 365
Hints and Explanations 365
Chapter 15 Environmental
Solved JEE 2016 Questions 370
Chemistry 493
Summary 494
Solved Examples 499
Chapter 12 Some Basic Principles in
Atmospheric Pollution 499
Organic Chemistry 373 Particulate Pollution 501
Summary 374 Water Pollution 502
Solved Examples 388 Soil Pollution 503
Hybridization, Shapes of Simple Molecules and Practice Exercise 503
Nomenclature 388 Answer Key 506
Isomerism – Structural and Stereoisomerism 390
Hints and Explanations 507
Reaction Intermediates 393
Electronic Displacement in a Covalent Bond 395 Solved JEE 2016 Questions 510
Electrophiles, Nucleophiles and Type of Organic Reactions 396
Advanced Level Problems 397 Chapter 16 Solid State 511
Practice Exercise 401 Summary 512
Answer Key 407 Solved Examples 525
Hints and Explanations 408 Unit Cells 525
Crystal Lattices 526
Solved JEE 2016 Questions 416
Calculations Involving Unit Cell Dimensions 526
Packing Efficiency and Packing Fraction 527
Chapter 13 Aliphatic Hydrocarbons 417 Formation of Voids in Close Packing 528
Summary 418 Simple Ionic Compounds 528
Tools for Problem Solving 433 Bragg’s Law 529
Solved Examples 434 Point Defects in Crystals 529
Alkanes 434 Advanced Level Problems 530
Alkenes 436 Practice Exercise 534
Alkynes 440
Answer Key 537
Advanced Level Problems 441
Hints and Explanations 538
Practice Exercise 445
Solved JEE 2016 Questions 542
Answer Key 450
Hints and Explanations 451
Chapter 17 Solutions 543
Solved JEE 2016 Questions 458
Summary 544
Tools for Problem Solving 550
Chapter 14 Aromatic Hydrocarbons 459 Solved Examples 551
Summary 460 Solubility 551
Tools for Problem Solving 471 Vapour Pressure of Ideal Solutions 551
Electrophilic Aromatic Substitution Reactions 471 Real Solutions 554

TOC.indd 11 8/8/2016 7:12:57 PM


xii Contents

Colligative Properties of Dilute Solutions 554 Properties, Classification and Purification of Colloids 662
Molecular Weight Calculations 556 Protection of Colloids 663
Advanced Level Problems 557 Emulsions 664

Practice Exercise 561 Advanced Level Problems 664

Answer Key 566 Practice Exercise 666

Hints and Explanations 566 Answer Key 669

Solved JEE 2016 Questions 572 Hints and Explanations 670


Solved JEE 2016 Questions 672
Chapter 18 Electrochemistry 575
Summary 576 Chapter 21 Nuclear Chemistry 673
Tools for Problem Solving 591
Solved Examples 592 Summary 674
Electrode Potential 592 Solved Examples 680
Electrochemical Series 594 Modes of Radioactive Decay 680
Nernst Equation 595 Radioactive Decay Rate 680
Law of Electrolysis 596 Energy Changes in Nuclear Reactions 681
Electrical Conductance 598 Advanced Level Problems 681
Variation of Molar Conductivity 598 Practice Exercise 683
Kohlraush’s Law 598
Answer Key 685
Types of Batteries 599
Applications of Electrolytic Process 599 Hints and Explanations 685

Advanced Level Problems 599 Solved JEE 2016 Questions 687

Practise Exercise 606


Answer Key 610 Chapter 22 General Principles and
Hints and Explanations 610 Processes of Isolation of Elements 689
Solved JEE 2016 Questions 616 Summary 690
Tools for Problem Solving 701
Chapter 19 Chemical Kinetics 619 Solved Examples 701
Concentration of Ore 701
Summary 620
Extraction of Crude Metal from Concentrated Ore 702
Tools for Problem Solving 631
Refining of Metals 703
Solved Examples 631
Application of Metallurgy 703
Rate of Reactions 631
Order and Molecularity of Reactions 634 Advanced Level Problems 704
Zero-Order Reactions 634 Practice Exercise 705
First-Order Reactions 635 Answer Key 707
Second-Order Reactions 636
Hints and Explanations 707
Experimental Determination of Rate Law 637
Effect of Temperature on Rate Constant 638 Solved JEE 2016 Questions 710
Advanced Level Problems 640
Practice Exercise 644 Chapter 23 The d- and f-Block
Answer Key 648 Elements 713
Hints and Explanations 648 Summary 714
Solved JEE 2016 Questions 653 Solved Examples 727
General Properties of d-block Elements 727
Chapter 20 Surface Chemistry 655 Some Important Compounds 730
f-Block Elements (The Inner Transition Elements) 730
Summary 656
Advanced Level Problems 732
Solved Examples 661
Adsorption 661 Practice Exercise 734
Adsorption Isotherms 662 Answer Key 737

TOC.indd 12 8/8/2016 7:12:57 PM


Contents xiii

Hints and Explanations 738 Test for Primary, Secondary and Tertiary Alcohols 834
Solved JEE 2016 Questions 741 Physical Properties of Phenols 835
Chemical Properties of Phenols 835
Preparation of Ethers 840
Chapter 24 Coordination Compounds 743 Chemical Properties of Ethers 840
Summary 744 Cleavage of Epoxides 841
Solved Examples 754 Advanced Level Problems 842
Terminology Used 754
Practice Exercise 847
Werner's Theory 755
IUPAC Nomenclature of Mononuclear Coordination Answer Key 852
Compounds 756 Hints and Explanations 852
Isomerism in Coordination Compounds 757 Solved JEE 2016 Questions 858
Structure, Hybridization of Mononuclear Coordination
Compounds 759
Valence Bond Theory 759 Chapter 27 Aldehydes and Ketones 859
Crystal Field Theory 761 Summary 860
Bonding in Metal Carbonyls 763 Tools for Problem Solving 870
Applications of Coordination Compounds 763 Some Synthetic Connections of Aldehydes, Ketones and
Advanced Level Problems 763 Other Functional Groups 870
Practice Exercise 768 Solved Examples 871
Methods of Preparation of Aldehydes and Ketones 871
Answer Key 771
Chemical Reactions of Aldehydes and Ketones 872
Hints and Explanations 771 Chemical Tests to Distinguish Aldehydes and Ketones 877
Solved JEE 2016 Questions 776 Advanced Level Problems 878
Practice Exercise 887
Chapter 25 Organic Compounds
Answer Key 894
Containing Halogens 779
Hints and Explanations 895
Summary 780
Solved JEE 2016 Questions 902
Tools for Problem Solving 792
Reactions of 1° Alkyl Halides 792
Electrophilic Substitution Reaction of Haloarenes 793 Chapter 28 Carboxylic Acids and Its
Solved Examples 793
Derivatives 903
Structure of Alkyl Halides 793
General Methods of Preparation of Alkyl Halides 793 Summary 904
General Methods of Preparation of Aryl Halides 795 Tools for Problem Solving 913
Reactions of Alkyl Halide 795 Some Synthetic Connections of Carboxylic Acids and their
Reactions of Haloarenes 798 Derivatives 913
Advanced Level Problems 800 Solved Examples 913
Structure and Properties 913
Practice Exercise 802
General Methods of Preparation 914
Answer Key 808 Chemical Properties 915
Hints and Explanations 808 Advanced Level Problems 917
Solved JEE 2016 Questions 814 Practice Exercise 921
Answer Key 923
Chapter 26 Alcohols, Phenols and Hints and Explanations 924
Ethers 815 Solved JEE 2016 Questions 926

Summary 816
Tools for Problem Solving 831 Chapter 29 Organic Compounds
Reactions of Phenols and Related Aromatic Compounds 831
Containing Nitrogen 929
Solved Examples 831
Preparation of Alcohols 831 Summary 930
Chemical Properties of Alcohols 832 Tools for Problem Solving 938

TOC.indd 13 8/8/2016 7:12:57 PM


xiv Contents

Reaction Summary for Primary Amines 938 Chapter 32 Chemistry in Everyday


Reaction Summary for Diazonium Salts 939
Solved Examples 939
Life 1007
General Methods of Preparation of Amines 939 Summary 1008
Chemical Properties of Amines 940 Solved Examples 1011
Tests for Amines 942 Medicines 1011
Diazonium Salts 942 Chemicals in Food 1012
Advanced Level Problems 944 Cleansing Agent 1012
Practice Exercise 948 Practice Exercise 1013
Answer Key 953 Answer Key 1015
Hints and Explanations 953 Hints and Explanations 1015
Solved JEE 2016 Questions 959 Solved JEE 2016 Questions 1017

Chapter 30 Polymers 961


Chapter 33 Purification and
Summary 962
Characterization of Organic
Solved Examples 967
Classification of Polymers 967 Compounds 1019
Chain-Growth or Addition Polymerization 968 Summary 1020
Step-Growth or Condensation Polymerization 969 Tools for Problem Solving 1025
Copolymerization 970 Qualitative Analysis of Organic Compounds 1025
Rubbers 970 Quantitative Analysis of Organic Compounds 1026
Polymers of Commercial Importance 971 Solved Examples 1026
Advanced Level Problems 971 Qualitative Analysis 1026
Practice Exercise 973 Quantitative Analysis 1026
Calculations of Empirical Formulae and Molecular
Answer Key 976
Formulae 1027
Hints and Explanations 976
Advanced Level Problems 1028
Solved JEE 2016 Questions 979
Practice Exercise 1028
Chapter 31 Biomolecules 981 Answer Key 1029

Summary 982 Hints and Explanations 1029


Solved Examples 992 Solved JEE 2016 Questions 1031
Carbohydrates 992
Proteins 993
Enzymes 995
Chapter 34 Qualitative Analysis 1033
Hormones 995 Summary 1034
Vitamins 995 Solved Examples 1050
Nucleic Acids 996 Advanced Level Problems 1050
Advanced Level Problems 996 Practice Exercise 1052
Practice Exercise 1000 Answer Key 1055
Answer Key 1002 Hints and Explanations 1055
Hints and Explanations 1002 Solved JEE 2016 Questions 1058
Solved JEE 2016 Questions 1005

TOC.indd 14 8/8/2016 7:12:57 PM


1 Some Basic Concepts
of Chemistry

Question Distribution in JEE (Main and Advanced)

3
No. of Questions

JEE (Main)
2
JEE (Adv)

0
2016 2015 2014 2013 2012 2011 2010 2009 2008 2007

Concept Distribution in JEE (Main and Advanced)

Topics Covered
Year
JEE (Main) JEE (Advanced)
2007 Mole Concept
2009 Concept of Atoms, Molecules and Ions
2010 Representation of Physical Quantities
Concentration Terms, Stoichiometry of
2011 Concentration Terms
Reactions
2012 Concentration Terms Concentration Terms
2013 Concentration Terms
Mole Concept, Empirical and Molecular Formula
2014 Representation of Physical Quantities
Determination, Concentration Terms
2015 Mole Concept, Stoichiometry of Reactions
2016 Stoichiometry of Reactions, Concentration Terms

Chapter-1.indd 1 8/4/2016 9:53:28 AM


2 Chapter 1 Some Basic Concepts of Chemistry

SUMMARY
1. Matter is anything that occupies space and has mass. It exists in three physical states – solid, liquid and gas. Classification of matter
can be summarized as shown in figure below:

Matter

Pure substances Mixtures of two


(homogeneous composition) or more substances

Solutions Heterogeneous
(homogeneous mixtures
Elements Compounds
composition–one (two or more
phase) phases)

2. Properties or the characteristics of substances are classified as:


(a) Physical properties are the in-built characteristics of a substance that can be determined without altering its composition; they
are associated with its physical existence. Common physical properties include colour, taste, odour, states of matter (solid, liquid
or gas), density, melting point and boiling point.
(b) Chemical properties are the ability of a substance to form new substances, either by reaction with other substances or by
decomposition.
3. Dalton’s Atomic Theory
The main postulates of Dalton’s atomic theory:
(a) Matter consists of tiny particles called atoms.
(b) In any sample of a pure element, all the atoms are identical in mass and other properties. The atoms of different elements differ
in mass and other properties.
(c) When atoms of different elements combine to form compounds, new and more complex particles form. However, in a given
compound the constituent atoms are always present in the same fixed numerical ratio.
(d) Atoms are indestructible. In chemical reactions, the atoms rearrange but they do not themselves break apart.
The theory proposed by John Dalton (1766–1844), an English scientist, used the Greek concept of atoms to explain the laws of
conservation of mass and definite proportions.
4. A pure substance is a particular kind of matter with a definite, fixed composition. It can either be an element or a compound.
(a) Elements
(i) Substances that contain only one kind of particles – atoms or molecules. For example, oxygen in the atmosphere, aluminium
in aluminium foil, iron in nails, and so on.
(ii) Elements are the fundamental building blocks from which all other substances are made.
(iii) An atom, is the smallest particle that can be used to identify an element.
(b) Compounds
(i) Substances that contain more than one element combined in fixed proportions. Water, for example, is composed of the
elements hydrogen and oxygen in the ratio of two atoms of hydrogen to one atom of oxygen.
(ii) A molecule is the smallest particle that can be used to identify a compound.
5. Physical quantities and their measurements in chemistry
(a) Mass and weight
(i) The mass of an object as the amount of matter in the object. Mass is measured on an instrument called a balance.
(ii) The weight of an object is a measure of the effect of gravity on the object. Weight is determined by using an instrument
called a scale, which measures force against a spring.
(iii) In the SI, the base unit for mass is the kilogram (kg), although the gram (g) is a more conveniently sized unit for most labo-
ratory measurements. One gram is 1/1000 of a kilogram (1 kilogram = 1000 g, so 1 g must equal 0.001 kg).
(b) Volume
It is a derived unit with dimensions of (length)3. With these dimensions expressed in meters, the derived SI unit for volume is the
cubic meter, m3.
(c) Density
(i) It is defined as the ratio of an object’s mass to its volume.

Chapter-1.indd 2 8/4/2016 9:53:29 AM


Summary 3

(ii) Density (d ) is a physical characteristic of a substance and can be used to identify a substance.
Mass g g
d= = or
Volume mL cm3

Tip Since the volume of a substance (especially liquids and gases) varies with temperature, it is important to state the
temperature along with the density.
(d) Temperature
(i) It is usually measured with a thermometer. Thermometers are graduated in degrees according to any one of the two tem-
perature scales – Celsius and Fahrenheit. Both scales use as reference points the temperature at which water freezes and the
temperature at which it boils.
(ii) On the Celsius scale water freezes at 0°C and boils at 100°C. On the Fahrenheit scale, water freezes at 32°F and boils at 212°F.
(iii) The SI unit of temperature is the kelvin (K), which is the degree unit on the Kelvin temperature scale. The kelvin is exactly
the same size as the Celsius degree. The only difference between these two temperature scales is the zero point.
(iv) The zero point on the Kelvin scale is called absolute zero and corresponds to nature’s coldest temperature. It is 273.15
degree units below the zero point on the Celsius scale, which means that 0°C equals 273.15 K, and 0 K equals −273.15°C.

Tip The two temperature scales (Celsius and Fahrenheit) can be converted using the following equation:
5
°C = (°F − 32)
9

6. Precision and accuracy


(a) Accuracy refers to how close a measurement is to the true or correct value. For measurements to be accurate, the measuring
device must be carefully calibrated (adjusted) so it gives correct values when a standard reference is used with it.
(b) Precision refers to how closely repeated measurements of a quantity come to each other and to the average. For measurements
to be precise, the uncertainty (i.e., the “plus or minus” part of the measurement) should be smaller.

Tip Accuracy is correctness; precision is reproducibility.

7. In measurement, for representing numbers that are very small or very large, scientific notation is used. It is a method of writing a
number as the product of a number between 1 and 10 multiplied by 10 raised to some power. For example, 24989 can be written as
2.4989 × 104 and 0.0325 can be written as 3.25 × 10−2.

Tip (a) Positive exponent (like in the above case) indicates decimal was moved to the left.
(b) Negative exponent indicates decimal was moved to the right.
(c) For multiplying and dividing two scientific notations, the numerals are multiplied or divided, while the exponents
are added or subtracted.
(d) For adding or subtracting two numbers, the exponents must be made the same, so that they can be taken out
as a common factor.

8. Significant figures
(a) Significant figures (or significant digits) are digits in a number that results from a measurement which are known with certainty.

This digit has some This digit has some


uncertainty uncertainty

24.3°C 24.32°C

These two digits These three digits


are known for sure are known for sure

(b) Rules for counting significant figures


Rule 1: All non-zero digits are significant. For example, in two temperature measurements 25.3 and 25.32, the first reading has
three significant figures while the second reading has four significant figures.

Chapter-1.indd 3 8/4/2016 9:53:29 AM


4 Chapter 1 Some Basic Concepts of Chemistry

Rule 2: All zeros between non-zero digits are significant. For example, 204 has three significant figures (2, 0, 4).
Rule 3: All zeros at the end of a number that includes a decimal point are significant. For example, 0.400 has three significant
figures (4, 0, 0).
Rule 4: All zeros before the first non-zero digit are not significant. These zeros are used to locate a decimal point. For example,
0.0035 has two significant figures (3, 5).
Rule 5: Some numbers are called exact numbers and have an infinite number of significant figures. For example, amount of
Rs 25 or 12 inches in a foot, etc.
(c) Mathematical operations and significant figures
(i) For addition and subtraction, the answer should have the same number of decimal places as the quantity with the fewest
number of decimal places.
3.247
41.36
+ 125.2 (This number has only 1 decimal place)
169.8 (The answer has been rounded to 1 decimal place)

(ii) For multiplication and division, the number of significant figures in the answer should not be greater than the number of
significant figures in the least precise measurement.

3 sig. figures 4 sig. figures

3.14 × 2.751
= 13
0.64

2 sig. figures

(d) Rounding off numbers


It is necessary to remove the excess digits in order to express the result by limiting it to proper number of significant figures. This
process of removing digits from a number, the value of the last digit retained is determined by a process known as rounding
off numbers. The rules for rounding off numbers in order to limit the result are as follows:
Rule 1: When the first digit after those you want to retain is 5 or greater, that digit and all others to the right are removed and the
last digit retained is increased by one.
Rule 2: When the first digit after those you want to retain is 4 or less, that digit and all others to its right are removed. The last
digit retained is not changed.
9. S.I. units
(a) The simplification of original metric system of units was adopted by the 11th General Conference on Weights and Measures. It is
called the International System of Units, abbreviated SI units from the French name, Le Système International d’Unités.
(b) The SI has a set of base units for seven measured quantities as shown in below table:

Measurement Unit Symbol


Length (l) meter m
Mass (m) kilogram kg
Time (t) second s
Electric current (I) ampere A
Temperature (T) kelvin K
Amount of substance (n) mole mol
Luminous intensity (Iv) candela cd

10. Dimensional analysis (also known as factor-label method or unit factor method) is a system used in numerical problems to help
perform the correct arithmetic operations. In these methods, we treat a numerical problem as one involving a conversion of units
from one kind to another.
(Given quantity) × (Conversion factor) = (Desired quantity)

Chapter-1.indd 4 8/4/2016 9:53:31 AM


Summary 5

11. Laws of chemical combination


(a) Law of conservation of mass states that no detectable gain or loss of mass occurs in chemical reactions, that is, mass is con-
served. This explanation of the law of conservation of mass works so well that it serves as the basis for balancing the chemical
equations.
(b) Law of definite proportions states that in a given chemical compound, the elements are always combined in the same propor-
tions by mass.
We can apply this law on the water molecule to check its validity. If we decompose samples of water (a compound) into the ele-
ments oxygen and hydrogen, we always find that the ratio of oxygen to hydrogen, by mass, is 8:1. In other words, in any sample
of water, the mass of oxygen obtained is always eight times the mass of hydrogen.

1 g hydrogen 2 g hydrogen
9 g water 18 g water
8 g oxygen 16 g oxygen

(c) Law of multiple proportions was given by Dalton in 1803. It states that whenever two elements form more than one com-
pound, the different masses of one element that combine with the same mass of the other element are in the ratio of small whole
numbers.
Suppose a molecule of sulphur trioxide contains one sulphur and three oxygen atoms, and a molecule of sulphur dioxide con-
tains one sulphur and two oxygen atoms. If we had just one molecule of each, then our samples each would have one sulphur
atom and, therefore, the same mass of sulphur. Now, on comparing the oxygen atoms, we find that they are in a numerical ratio
of 3:2. However, because all the oxygen atoms have the same mass, the mass ratio must also be 3:2.
(d) Law of reciprocal proportion states that if two different elements combine separately with the same mass of a third element,
the ratio of the masses in which they do so are either the same or a simple multiple of the mass ratio in which they combine.
For example, oxygen and sulphur react with copper to give copper oxide and copper sulphide respectively. Suphur and oxygen
also react with each other to give SO2.
In CuS, Cu:S = 63.5:32, in CuO, Cu:O = 63.5:16, therefore, the ratio of  S:O = 32:16 or 2:1. Now in SO2, S:O = 32:32 or 1:1, Therefore,
2 1
the ratio between the two ratios is : = 2 : 1
1 1
(e) Gay-Lussac’s law of gaseous volume states that when measured at the same temperature and pressure, the ratios of the vol-
umes of reacting gases are small whole numbers. For example, H2 and O2 combine to form water vapour in a volume ratio of
2:1. Similarly, H2 and Cl2 react to form HCl in a volume ratio of 1:1; and H2 and N2 react to form NH3 in a volume ratio of 3:1.

2H2(g) + O2(g) 2H2O(g)

H2 H2 + O2 H2O H2O

2 volumes 1 volume 2 volumes

(f) The Avogadro’s law states that equal volumes of different gases at the same temperature and pressure contain the same num-
ber of molecules.
By Avogadro’s law, equal volumes of hydrogen and chlorine at the same temperature and pressure contain the same number of
molecules. On a volume basis, hydrogen and chlorine react as:
Hydrogen + Chlorine →
 Hydrogen chloride
1 volume 1 volume 2 volumes
12. Atomic, molecular and formula masses
(a) Atomic mass
(i) It is impossible to measure the absolute mass of a single atom, however, we can measure the relative masses of different
atoms through experimental means. The standard used to calibrate these measurements is the carbon-12 (12C), which is
an isotope of carbon.
(ii) The unit in which atomic mass measurements are reported is the atomic mass unit (amu).
(iii) This weighted average of all the naturally occurring isotopes of an atom is known as the atomic mass of the element.

Tip One atomic mass unit is equal to 1/12th of the mass of one 12C atom. Nowadays, amu has been replaced by u which
stands for unified mass.

Chapter-1.indd 5 8/4/2016 9:53:33 AM


6 Chapter 1 Some Basic Concepts of Chemistry

(b) Relative atomic mass


The relative atomic mass of an element is an average value for the isotopes of the element. For example, the relative atomic mass
for chlorine is 35.5 because it contains two different isotopes.
(c) Average atomic mass
Average atomic mass is a very similar concept to relative atomic masses. In order to find it for an element, we have to consider the
atomic masses of all isotopes for that element and the relative proportion in which they are present.

Atomic mass of isotope (1) × %abundance


+ Atomic mass of isotope (2) × %abundance + ...
Average atomic mass =
100

(d) Molecular mass


It is a number which indicates how many times one molecule of a substance is heavier in comparison to 1/12th of the mass of
one atom of C-12.
Mass of one molecule of a substance
Molecular mass=
1/ 12th mass of one atom of C-12

(e) Formula mass


It is the sum of the atomic masses of the elements in the formula. Sodium chloride (NaCl) has a formula mass of 58.44 u, which is
determined by adding the atomic masses of sodium and chlorine.
(f) Equivalent mass
Equivalent mass is the number of parts by mass of a substance which combines or displaces (directly or indirectly) 1.008 parts by
mass of hydrogen or 8 parts by mass of oxygen or 35.5 parts by mass of chlorine or 108 parts by mass of silver. Equivalent mass
is related to the atomic mass by the relation:

Atomic mass of element


Equivalent mass =
Valence (n)

The equivalent mass of acids, bases and oxidizing/reducing agents can be obtained as follows:
(i) Equivalent mass of an acid: The ratio of molecular weight of acid to basicity of acid or the ratio of molecular weight of acid
to the number of replaceable hydrogen atoms in one molecule of the acid.
(ii) Equivalent mass of a base: The ratio of molecular weight of base to acidity of acid or the ratio of molecular weight of base
to the number of replaceable OH− atoms in one molecule of base.
(iii) Equivalent weight of an oxidizing or reducing agent: The ratio of molecular weight of the substance to the number of
electrons gained or lost by one molecule in a redox reaction.
13. Mole concept
A mole is the amount of a substance which contains as many entities as there are atoms in 0.012 kg of 12C. It is the SI unit for the
amount of substance. The number of particles in a mole is known as Avogadro’s constant.
1 mol of element X = gram atomic mass of X
1 mol of X = 6.022 × 1023 units of X

14. Molar mass


(a) The molecular mass of a compound is the sum of the atomic masses of the atoms in the formula of the compound.
(b) The term molecular mass is misleading because the molecules may contain isotopes of the same element. Some chemists, there-
fore, recommend that the results of calculations should be expressed as the mass of a mole or the molar mass of a compound.
In fact mass of one mole of substance is its molar mass. It is expressed in units of gmol−1. Molar mass is given for molecular and
ionic substances.
15. The percentage composition of an element in a compound is its percent by mass in the compound.

Mass of the element in the compound


Mass % of an element = ×100
Molar mass of the compound

16. Empirical and molecular formulae


(a) Empirical formula
(i) It is the simplest formula, gives the smallest whole number ratio of atoms present in a compound.
(ii) This formula gives the relative number of atoms of each element in the compound.

Chapter-1.indd 6 8/4/2016 9:53:34 AM


Summary 7

(b) Molecular formula


(i) It is the actual whole number ratio of atoms in a molecule. It is the true formula, representing the total number of atoms of
each element present in one molecule of a compound.
(ii) Often the molecular formula is the same as the empirical formula. If the molecular formula is not the same, it will be an
integral (whole number) multiple of the empirical formula. The molecular formula is the true formula, representing the
total number of atoms of each element present in one molecule of a compound. For example, both acetylene (C2H2) and
benzene (C6H6) contain 92.3% C and 7.7% H. The smallest ratio of C and H corresponding to these percentages is CH (1:1).
Therefore, the empirical formula for both acetylene and benzene is CH, even though the molecular formulas are C2H2 and
C6H6, respectively.
(iii) To find the molecular formula, the integral multiple can be found as
Molecular mass
n=
Empirical formula mass
Now, Molecular formula = Empirical formula × n
17. Chemical equations and stoichiometry
(a) Chemical equation
A chemical equation uses the chemical symbols and formulas of the reactants and products and other symbolic terms to repre-
sent a chemical reaction. For example, 2H2 (g) + O2 (g) → 2H2O(l)
(b) Stoichiometry
(i) It is the quantitative relationships among the reacting materials and the products formed.
(ii) When reactants are mixed in exactly the mass ratio determined from the balanced equation, the mixture is said to be
stoichiometric.
(c) Limiting reagent
In the case of a chemical reaction, if specific amounts of each reactant are mixed, the reactant that produces the least amount
of product is called the limiting reagent.
We can illustrate this with the simple example of the production of water from its elements, hydrogen and oxygen.
2H2 (g) + O2 (g) → 2H2O(l)
The stoichiometry of the reaction tells us that two moles (4.0 g) of hydrogen react with one mole (32.0 g) of oxygen to produce
two moles (36.0 g) of water. Thus, any time hydrogen and oxygen react in a 4:32 mass ratio, all reactants are consumed and only
product appears.
4.0 g H2 + 32.0 g O2 → 36.0 g H2O (stoichiometric )
What if we mix a 6.0 g quantity of H2 with a 32.0 g quantity of O2? Do we produce 38.0 g water? No, we still produce only 36.0 g
of H2O using only 4.0 g of the H2. Thus, H2 is present in excess, and the amount of product is limited by the amount of O2 present.
In this case, O2 becomes the limiting reactant.
6.0 g H2 + 32.0 g O2 → 36.0 g H2O + 2.0 g H2
(Limiting reactant) (In excess)
If we mix a 4.0 g quantity of H2 with a 36.0 g quantity of O2, 36.0 g of H2O is again produced. In this case, the H2 is completely
consumed and limits the amount of water formed. Thus, H2 is now the limiting reactant and O2 is present in excess.
4.0 g H2 + 36.0 g O2 → 36.0 g H2O + 4.0 g O2
(Limiting reactant) (In excess)
18. Concentration terms
(a) Mass percent (w/w)
This quantity expresses the mass of solute per 100 g of solution. Therefore, in 100 g of a solution that is 25% by mass HCl, there
are 25 g of HCl and 75 g of H2O. The expression for percent by mass is
Mass of solute
% by mass (solute) = × 100
Mass of solute + Mass of solvent
Mass of solute
= × 100
Mass of solution
(b) Volume percentage (V/V)
Solutions that are formulated from two liquids are often expressed as volume percent with respect to the solute. The volume
percent is the volume of a liquid in 100 mL of solution. The label on a bottle of ordinary rubbing alcohol reads “isopropyl alco-
hol, 70% by volume”. Such a solution could be made by mixing 70 mL of alcohol with water to make a total volume of 100 mL,
but we cannot use 30 mL of water, because the two volumes are not necessarily additive:
Volume of liquid
% by volume (solute) = ×100
0
Volume of solution

Chapter-1.indd 7 8/4/2016 9:53:37 AM


8 Chapter 1 Some Basic Concepts of Chemistry

(c) Mass by volume (w/V) percentage


This method expresses concentration as grams of solute per 100 mL of solution. With this system, a 10.0% m/V glucose solution
is made by dissolving 10.0 g of glucose in water, diluting to 100 mL and mixing.
Mass of solute
% mass by volume (solute) = ×100
Volume of solution (mL)
(d) Mole fraction
It is the ratio of number of moles of the component (solute) to the total number of moles of the components of the solution
(solute + solvent). The expression for mole fraction is
Moles of solute
Mole fraction of solute =
Moles of solute + Moles of solvent
If a solution is prepared by dissolving nA moles of component A in nB moles of component of B, then the mole fraction of A and
B is given by the relation:
nA
Mole fraction of A (x A ) =
nA + nB
nB
Mole fraction of B (xB ) =
nA + nB
xA + xB = 1
(e) Molarity
The molarity of a solution is defined as the number of moles of solute per litre of solution. Molarity is calculated by dividing the
number of moles of solute in the solution by the volume of the solution in litres. The symbol of molarity is M.
Moles of solute
Molarity =
Volume of solution

Tip (a) Molarity of dilution: The relation between molarity before and after dilution is given by
M1V1 = M2V2
(b) Molarity of mixing: When three solutions (containing same solvent and solute) of molarity M1, M2, M3 are mixed
in volumes V1, V2, V3 respectively, the molarity of mixing can be given as:
M1V1 + M2V2 + M3V3 = MTotal (V1 + V2 + V3 )

(f) Molality
It is defined as the number of moles of solute per mass of solvent measured in kilograms. The symbol of molality is m and it is
expressed as:
Number of moles of solute
Molality =
Mass of solvent in kg
Caution: Molality (m) should not be confused with molarity whose symbol is capital M. Neither the mass nor volume of the sol-
vent is used to define molarity. The combined volume of the two components (solute and solvent), the volume of the solution,
is used for molarity.
(g) Normality
It is defined as the number of gram equivalents of a solute dissolved per litre of solution. Mathematically, it is expressed as
Number of gram equivalents
N= × 1000
Volume in mL
where number of gram equivalents can be found as follows:
Given weight
Number of gram equivalents =
Equivalent weight
Milliequivalents of the solute can be found as
Weight of solute
Milliequivalent of solute = N × V(in mL ) = ×1000
Equivalent weight of solute
(h) Strength of a solution
It is defined as the amount of solute (in grams) dissolved per litre of solution. Mathematically, it is expressed as
Weight of solute (g)
S= = N × Equiv. wt. = M × Mol. wt.
Volume of solution (L)

Chapter-1.indd 8 8/4/2016 9:53:39 AM


Tools for Problem Solving 9

19. Relation between stoichiometric quantities


(a) Relation between mole fraction and molality

nA and xB = nB
xA = nA + nB
nA + nB

x A nA Moles of solute w A × MB
= = =
xB nB Moles of solvent wB × MA

x A × 1000 w A × 1000 x × 1000


= = m⇒ A =m
xB × MB wB × MA (1− x A ) MB
(b) Relationship between molality and molarity
1 d M
= − B
m M 1000
where d = density of solution, m = molality, M = molarity and MB = molar mass of solute.
(c) Relation between molarity and mole fraction
xB × 1000 × r
M=
x A MA + xB MB
where xA and xB are mole fractions of solvent and solute, nA and nB are moles of solvent and solute and MA and MB are molar
masses of solvent and solute, respectively.
(d) Relationship between normality and molarity
Molarity × Molecular mass = Normality × Equivalent mass
Normality Molecular mass
or = =n
Molarity Equivalent mass
Therefore, Normality = n × Molarity

TOOLS FOR PROBLEM SOLVING


Mass g g
1. d = = or
Volume mL cm3
5
2. °C = (°F − 32)
9
Atomic mass of element
3. Equivalent mass =
Valence (n)
Mass of the element in the compound
4. Mass % of an element = ×100
Molar mass of the compound
5. Concentration terms
Mass of solute
(a) % by mass (solute) = × 100
Mass of solute + Mass of solvent
Mass of solute
= × 100
Mass of solution
Volume of solute
(b) % by volume (solute) = ×1000
Volume of solution
Mass of solute
(c) % mass by volume (solute) = ×100
Volume of solution (mL)
Moles of solute
(d) Mole fraction of solute =
Moles of solute + Moles of solvent
Moles of solute
(e) Molarity = ; Molarity of dilution: M1V1 = M2V2 ; Molarity of mixing: M1V1 + M2V2 + M3V3 = MTotal (V1 + V2 + V3 )
Volume of solution

Chapter-1.indd 9 8/4/2016 9:53:43 AM


10 Chapter 1 Some Basic Concepts of Chemistry

Number of moles of solute


(f) Molality =
Mass of solvent in kg
Number of gram equivalents
(g) Normality = ×1000
Volume in mL
Weight of solute (g)
(h) Strength = = N × Equiv. wt. = M × Mol. wt.
Volume of solution (L)
x A × 1000 w A × 1000 x × 1000
6. Relation between mole fraction and molality: = = m⇒ A = m;
xB × MB wB × MA (1− x A ) MB

1 d M
Relationship between molality and molarity: = − B ;
m M 1000
x × 1000 × r
Relation between molarity and mole fraction M = B ;
x A MA + xB MB
Normality Molecular mass
Relationship between normality and molarity: = =n
Molarity Equivalent mass

7. Empirical formula determination


Molecular mass
n=
Empirical formula mass
To calculate the empirical formula of a compound from percent composition, we follow three steps:
(a) Convert percent composition into an actual mass.
(b) Convert mass into moles of each element.
(c) Find the whole number ratio of the moles of different elements.
8. Stoichiometric calculations
The three main steps in stoichiometric calculations for any chemical reaction are:
(a) Determine the number of moles of starting substance: Find the starting material or product of the reaction for which you
know both the mass of the sample and the formula of the substance. Use the molecular mass of this substance to convert the
number of grams in the sample into an equivalent number of moles.
(b) Determine the mole ratio of the desired substance to the starting substance: Use the balanced equation for the reaction to
create a mole ratio that can convert the number of moles of the starting substance into moles of the desired substance
(c) Calculate the desired substance in the units specified in the problem: Use the molecular mass of the desired substance of the
reaction to convert the number of moles involved in the reaction into grams or the units specified
9. Limiting reagent calculation
(a) Calculate the amount of product (moles or grams, as needed) formed from each reactant.
(b) Determine which reactant is limiting. (The reactant that gives the least amount of product is the limiting reagent; the other reac-
tant is in excess.)
(c) Once we have identified the limiting reagent, the amount of product formed can be determined. It is the amount determined by
the limiting reagent.
(d) If we need to know how much of the other reactant remains, we calculate the amount of the other reactant required to react
with the limiting reagent, and then subtract this amount from the starting quantity of the reactant. This gives the amount of that
substance that remains unreacted.

SOLVED EXAMPLES
Representation of Physical Quantities 0.0025 (2 and 5); 208 (2, 0 and 8); 5005 (5, 0, 0 and 5);
126,000 (1, 2 and 6); 500.0 (5, 0, 0 and 0), 2.0034 (2, 0, 0, 3
1. How many significant figures are present in the following: and 4).
0.0025, 208, 5005, 126,000, 500.0, 2.0034?
(a) 1, 2, 4, 5, 3, 2 (b) 2, 3, 4, 3, 4, 5 2. Which one of the following measurements possesses the
(c) 3, 2, 4, 4, 3, 5 (d) 3, 4, 2, 4, 3, 3 greatest accuracy?
(a) 90 ± 1 (b) 900 ± 1
Solution
(c) 1.80 ± 0.02 (d) 9.000 ± 0.001
(b) The digits that are significant figures in a quantity are
those that are known (measured) with certainty plus the Solution
last digit, which contains some uncertainty. (d) The greatest accuracy is in 9.000 ± 0.001.

Chapter-1.indd 10 8/4/2016 9:53:44 AM


Solved Examples 11

3. Five students obtain the following grades on an exam. What is But, one atom of S = 32 u. Now, 3.4 u of sulphur is present in
the average score on this exam for these students: 88, 69, 81, 100 u of insulin.
92, 79? Therefore, 32 u of sulphur will be present in insulin
(a) 81.8 (b) 82 (c) 68 (d) 81.80 100
= × 32 = 941.2 u
Solution 3.4
(b) The average is (88 + 69 + 81 + 92 + 79)/5 = 81.8 ≈ 82 with Mole Concept
two significant digits.
8. How many moles of magnesium phosphate, Mg3(PO4)2 will
4. An electronic balance used in the mailroom displays tenths of a contain 0.25 mol of oxygen atoms?
kilogram from 0 to 140 kg. How many significant figures should (a) 0.02 (b) 3.125 × 10−2
be used to express the mass of any packages whose mass is (c) 1.25 × 10−2 (d) 2.5 × 10−2
between 80.2 and 83.5 kg? (AIEEE 2006)
(a) 3 (b) 5 (c) 4 (d) 2
Solution
Solution
(b) One mole of Mg3(PO4)2 contains 8 oxygen atoms
(a) Since there are 3 significant digits in 80.2 and 83.5 so the n moles contain 8n oxygen atoms
answer should also contain 3 significant digits. Therefore, for 0.25 moles, 0.25 = 8n
Concept of Atoms, Molecules and Ions 0.25
⇒n = = 3.125 × 10 −2
8
5. If we consider that 1/6, in place of 1/12, mass of carbon atom is
taken to be the relative atomic mass unit, the mass of one mole 9. In the reaction,
of substance will 2Al(s) + 6 HCl(aq) → 2Al3+ (aq) + 6Cl− (aq) + 3H2 (g)
(a) decrease twice.
(b) increase two fold. (a) 11.2 L H2(g) at STP is produced for every mole of HCl(aq)
(c) remain unchanged. consumed.
(d) be a function of the molecular mass of the substance. (b) 6 L HCl(aq) is consumed for every 3 L H2(g) produced.
(c) 33.6 L H2(g) produced regardless of temperature and
Solution pressure for every mole Al that reacts.
(a) The relative atomic mass is expressed as (d) 67.2 L H2(g) at STP is produced for every mole of Al that
reacts.
Mass of one atom of the element
Relative atomic mass = (AIEEE 2007)
1/ 12th part of the mass of one
Solution
atom of carbon
(a) According to the given reaction, for each mole of HCl
If 1/12 is replaced with 1/6, then we have reacted, 0.5 mol of H2 gas is formed at STP.
Mass of one atom of the element 1 mol of an ideal gas occupies 22.4 L at STP.
Relative atomic mass = Volume of H2 gas formed at STP per mole of HCl reacted
1/ 6 th part of the mass of one
atom of carbon = 22.4 × 0.5 = 11.2 L
Now, 1 mol of substance = 6.023 × 1023 atoms. If we con- 10. The ratio of masses of oxygen and nitrogen in a particular gas-
sider the mass of 1 mol in kg then it will remain the same, eous mixture is 1:4. The ratio of number of their molecule is
and in relative atomic mass units, it will become half times (a) 1:4 (b) 7:32 (c) 1:8 (d) 3:16
the original. (JEE Main 2014)
6. Naturally occurring zinc is 48.6% 64Zn atoms (63.9291 u), 27.9% Solution
66Zn atoms (65.9260 u), 4.1% 67Zn atoms (66.9721 u), 18.8%
(b) Let the mass of oxygen be x and that of nitrogen be 4x.
68Zn atoms (67.9249 u) and 0.6% 70Zn atoms (69.9253 u).
So, the number of moles of oxygen is x/32 and that of
Calculate the atomic mass of zinc. nitrogen is 4x/28 = x/7
(a) 60.4 u (b) 66.4 u (c) 68.4 u (d) 65.4 u x 7 7
The required ratio is × = .
Solution 32 x 32
(d) The atomic mass of zinc is 0.486(63.9291 u) + 0.279 11. The amount of oxygen in 3.6 moles of water is
(65.9260 u) + 0.041(66.9721 u) + 0.188(67.9249 u) + (a) 115.2 g (b) 57.6 g
0.006(69.9253 u) = 65.4 u. (c) 28.8 g (d) 18.4 g
7. Insulin contains 3.4% sulphur. Calculate the minimum molecu- (JEE Main Online 2014)
lar mass of insulin. Solution
(a) 925 u (b) 941 u (c) 983 u (d) 931 u (b) Weight = n × Molecular weight
O in 1 mol of H2O = 1 mol
Solution
3.6 moles of H2O = 3.6 moles of O atom
(b) Minimum molecular mass of insulin will be the mass con- Weight of 1 mol of O atom = 16 g
taining at least containing one atom of sulphur. Therefore, weight of 3.6 moles of O atom = 16 × 3.6 = 57.6 g

Chapter-1.indd 11 8/4/2016 9:53:46 AM


12 Chapter 1 Some Basic Concepts of Chemistry

12. 3 g of activated charcoal was added to 50 mL of acetic acid 48


solution (0.06 N) in a flask. After an hour it was filtered and Number of atoms in 48 g ozone = × NA × 3 = 3NA
48
the strength of the filtrate was found to be 0.042 N. The
amount of acetic acid adsorbed (per gram of charcoal) is Quick Tip/Alternate solution
(a) 36 mg (b) 42 mg (c) 54 mg (d) 18 mg 16 g of oxygen atoms = 1 mol of atoms
(JEE Main 2015) 16 g CH4 = 5 mol of atoms
Solution 48 g of O3 = 3 mol of atoms
(d) Initial mmoles of CH3COOH = 0.06 × 50
Final mmoles of CH3COOH = 0.042 × 50 16. Arrange the following in order of their increasing masses in
So, the mass of CH3COOH adsorbed per gram of charcoal grams: (I) One atom of silver, (II) one gram-atom of nitrogen,
(III) one mole of calcium, (IV) one mole of oxygen molecules,
(0.06 − 0.042) × 50 × 10 −3 × 60 × 103 (V) 1023 atoms of carbon and (VI) one gram of iron.
= = 18 mg
3 (a) (I) < (II) < (III) < (IV) < (V) < (VI)
13. What is the mass in grams of one molecule of caffeine (b) (VI) < (II) < (V) < (IV) < (III) < (I)
(C8H10N4O2)? (c) (I) < (III) < (II) < (V) < (IV) < (VI)
(a) 3.22 × 10−18 g (b) 3.18 × 10−18 g (d) (I) < (VI) < (V) < (II) < (IV) < (III)
(c) 3.22 × 10−22 g (d) 3.18 × 10−22 g Solution
Solution (d) (I) 1 mol of Ag atoms = 108 g = 6.023 × 1023 atoms.
Molar mass
(c) Mass of onemolecule= Mass of 6.023 × 1023 atoms of Ag = 108 g.
Avogadro’s constant 108
 
194 g Mass of 1 atom of Ag =  = 1.793 × 10 −22 g.
= = 3.22 × 10 −22 g  6.023 × 1023 
6.023 × 1023
(II) Mass of 1 gram atom of N = Atomic mass of N in
14. Vitamin K5 contains 76.27% carbon by mass and has a molec-
grams = 14.0 g.
ular mass 173.21 g mol−1. How many carbon atoms are in one
(III) Mass of 1 mol of Ca = Atomic mass of Ca in grams
K5 molecule?
= 40.0 g.
(a) 6.63 × 1024 (b) 3.83 × 1024
(IV) Mass of 1 mol of oxygen molecules = Molar mass of
(c) 11 (d) 5
oxygen in grams = 32.0 g.
Solution (V) Mass of 1 mol of C atom = 12 g = 6.023 × 1023 atoms.
(c) Mass of one molecule of K5 vitamin = 173.21 u Mass of 6.023 × 1023 atoms of C = 12 g.
= 173.21/NA g  12 
173.21 76.27 132.1 Mass of 1 atom of C =  g.
Therefore, the mass of carbon is × = g  6.023 × 1023 
NA 100 NA
 12 
Therefore, the number of atoms of C in one molecule Mass of 1023 atoms of C =  × 1023 g = 1.992g
= Moles of C × NA  6.023 × 1023 
132.1 132.1 (VI) Mass of iron = 1.0 g.
Moles of C = ⇒ Number of C atoms = × NA = 11
12NA 12NA Percentage Calculation in Compounds

Quick Tip/Alternate solution 17. MnO2 on ignition converts into Mn3O4. A sample of pyro-
lusite having 75% MnO2, 20% inert impurities and the rest
Mass of one molecule of K5 = 173.21 u water is ignited in air to constant weight. What is the percent-
76.27
Therefore, the mass of carbon in it = 173.21× = 132.1 u age of Mn in the ignited sample? [At. mass of Mn = 55]
100
(a) 69.6% (b) 49.9% (c) 55.24% (d) 72.05%
Mass of one atom of carbon is 12 u, so the number of carbon
132.1 Solution
atoms in one molecule = = 11
12 (c) Let the weight of sample be x g. Therefore, weight of
MnO2 is 0.75x g
15. Which of the following contains the maximum number of 0.75 x
atoms? Moles of MnO2 =
87
(a) 16 g of oxygen atoms
(b) 16 g of methane gas 0.75 x
Weight of Mn = × 55 = 0.474 x
(c) 48 g of ozone 87
(d) All contain the same number of atoms The reaction is 3MnO2 → Mn3O 4 + O2 ; Mn3O4 = 229 g
mol−1
Solution 0.75 x 1
16 Weight of Mn3O4 produced = × × 229 = 0.658 x g
(b) Number of atoms in16 g O atoms = × NA = NA 87 3
16 Total weight of residue = 0.658 x + 0.2 x (wt. of impurities)
16 0.474 x
Number of atoms in16 g CH4 = × NA × 5 = 5NA % of Mn in residue = × 100 = 55.24%
16 0.858 x

Chapter-1.indd 12 8/4/2016 9:53:50 AM


Solved Examples 13

18. A sample of mercury (II) sulphide contains 82.6% mercury by Empirical formula = NH2; Molecular formula = n (empirical
mass. The mass of HgS that can be made from 30 g of mercury formula)
is [At. wt. of Hg = 201, S = 32] Molecular weight 2 × V .d 2 × 16
n= = = =2
(a) 34.8 g (b) 36.3 g (c) 4.8 g (d) 24.8 g Empirical formula weight 16 16
Solution Thus, the molecular formula is = 2(NH2) = N2H4
201
(a) The percentage of Hg in pure HgS = × 100 = 86.2% 21. The compound X2O3, contains element X and oxygen com-
233
bined in a ratio of 2.167 g of X to 1.0 g of oxygen. Another
This is more than the given percentage of Hg it implies
compound of X and oxygen gave a different analysis and
that the sample is not pure. So, we cannot use given per-
properties. A 2.5 g sample of this second compound contains
centage to calculate mass of pure HgS produced by 30 g
1.3 g of element X. The formula of the second compound is
Hg.
(a) X13O12 (b) XO3 (c) X6O15 (d) XO6
Consider x g of HgS can be obtained from 30 g of Hg. It
indicates 30 g Hg is the 86.2% of x g HgS. Solution
86.2 (b) In second compound, 1.2 g of oxygen combines with 1.3 g
⇒x× = 30 ⇒ x = 34.8 g of X, so 1 g oxygen would combine with 1.3/1.2 = 1.08 g of X
100
Mass ratio of X and O in first compound = 2.167:1
19. A sample of a liquid with a mass of 8.657 g was decomposed Mass ratio of X and O in second compound = 1.08:1
into its elements and gave 5.217 g of carbon, 0.9620 g of Applying the law of multiple proportions, we get
hydrogen, and 2.478 g of oxygen. What is the percentage Ratio of masses of X in both compounds = 2.167/1.08 ≅ 2:1.
composition of this compound? Therefore, the number of X atoms in the second compound
(a) 99.99 % (b) 99.98% is just half of the first compound.
(c) 99.97 % (b) 100% Therefore, the formula of the second compound is XO3.
Solution
Quick Tip/Alternate solution
(a) The mass of whole sample here is 8.657 g, so we take
each element in turn and perform the calculations. For the first compound,
nX mX / MX 2 M 2.167 M 2 ×1
5.214 g C = ⇒ = O× ⇒ O=
For C : × 100% = 60.26% C nO mO / MO 3 MX 1 MX 3 × 2.167
8.657 g sample For the second compound,
0.9620 g H nX mX MO 1.3 2 1
For H : × 100% = 11.11% H = × = × × = 0.333
8.657 g sample nO mO MX 1.2 3 2.167

2.478 g O Therefore, the ratio of numbers in second compound is 1:3


For O : × 100% = 28.62% O and the formula will be XO3.
8.657 g sample
Sum of percentage: 99.99% 22. A 1.525 g sample of a compound between sulphur and oxy-
gen was prepared by burning 0.7625 g of sulphur in air and
One of the useful things about a percentage composi- collecting the product. What is the empirical formula for the
tion is that it tells us the mass of each of the elements in compound formed?
100 g of the substance. For example, the results in this (a) SO (b) SO2 (c) S2O (d) SO3
problem tell us that in 100.00 g of the liquid there are Solution
60.26 g of carbon, 11.11 g of hydrogen, and 28.62 g of
(b) First, find the number of moles of each element, then
oxygen.
determine the empirical formula by comparing the ratio
Empirical and Molecular Formula Determination of the number of moles of each element.
Start with the number of moles of S:
20. A gaseous compound of nitrogen and hydrogen contains
 1 mol S 
12.5% (by mass) of hydrogen. The density of the compound mol S = 0.7625 g S  = 0.02378 mol S
relative to hydrogen is 16. The molecular formula of the com-  32.066 g S 
pound is Then find the number of moles of O: since there are only
(a) NH2 (b) N3H (c) NH3 (d) N2H4 two elements in the compound, S and O, the remaining
(JEE Main Online 2014) mass is O
g O = 1.525 g compound – 0.7625 g S = 0.7625 g O
Solution
 1 mol O 
(d) % by mass At weight mol O = 0.7625 g O  = 0.04766 mol O
Ratio Simplest Ratio  15.9994 g O 
87.5 6.2566 The empirical formula is S0.02378O0.4766
N = 87.5 14 = 6.256 =1
14 .
6 256 The empirical formula must be in whole numbers, so
12.5 divide by the smaller subscript: S 0.02378 O 0.04766 which
H = 12.5 1 12.5 = 12.5 =2
6.25 becomes SO2. 0.02378 0.02378

Chapter-1.indd 13 8/4/2016 9:53:52 AM


14 Chapter 1 Some Basic Concepts of Chemistry

23. Predict the formula of the compound produced when 1.00 g Since these two mole amounts are the same, the empirical
of chromium metal reacts with 0.923 g of oxygen, O2. formula is N0.0373O0.0931; to have the empirical formula in
(a) CrO (b) CrO2 (c) Cr2O (d) CrO3 whole numbers, first divide by the smaller number of moles:
Solution N 0.0373 O 0.0931 which is NO2.5, now to have whole numbers,
0.0373 0.0373
(d) Based on the given data, we have
multiply the subscripts by 2: N2O5.
Given Atomic Moles of the Simplest
Element 27. After determining the empirical formulas of two different
mass mass element molar ratio
compounds that were CH2Cl and CHCl, a student mixed up
1.00/51.996 0.0192/0.0192 the data for the molecular masses. However, the student knew
Cr 1.00 g 51.996
= 0.0192 =1 that one compound had a molecular mass of 100 g mol−1 and
(0.923×2)/ the other had a molecular mass of 289 g mol−1. What are the
0.0577/0.0192
O 0.923 g 16 31.998 likely molecular formulas of the two compounds?
=3
= 0.0577 (a) C2H4Cl2 and C5H5Cl6 (b) C3H4Cl2 and C6H5Cl6
Therefore, the predicted formula for the compound is CrO3. (c) C3H6Cl2 and C6H6Cl6 (d) C2H4Cl2 and C6H6Cl6
Solution
24. Styrene, the raw material for polystyrene foam plastics, has
an empirical formula of CH. Its molecular mass is 104 g mol−1. (d) To find the molecular formula, divide the molecular
What is its molecular formula? mass by the formula mass of the empirical formula, then
(a) C8H8 (b) C6H12 (c) C8H16 (d) C6H14 multiply the subscripts of the empirical formula by that
value.
Solution Formula mass of CH2Cl: 49.48 g mol−1
(a) For the empirical formula, CH, the formula mass is 12.01 Formula mass of CHCl: 48.47 g mol−1
+ 1.008 = 13.02 100 289
To find how many CH units weighing 13.02 are in a mass For CH2Cl: = 2.02 and = 5.84
49.48 49.48
104 100 289
of 104, we divide = 7.99 For CHCl: = 2.06 and = 5.96
13.02 48.47 48.47
Rounding this to 8, Styrene, therefore, is C8H8.
The CH2Cl rounds better using the molecular mass of
25. If 3.54 g of an oxide of lead contains 3.22 g of lead. Identify 100, therefore multiply the subscripts by 2 and the for-
the formula of this oxide from the following: mula is C2H4Cl2.
(a) PbO3 (b) PbO (c) Pb3O4 (d) PbO2 For CHCl, the molecular mass of 289 gives a multiple of 6,
therefore, the formula is C6H6Cl6.
Solution
(c) From the given data, we have Concentration Terms
Atomic Relative number 28. Density of a 2.05 M solution of acetic acid in water is 1.02 g mL−1.
Symbol % Simple ratio
mass of moles The molality of the solution is
Pb 90.9 207.2 90.9/207.2 1×3=3 (a) 1.14 mol kg−1 (b) 3.28 mol kg−1
= 0.4343 (c) 2.009 mol kg−1 (d) 0.44 mol kg−1
(AIEEE 2006)
O 9.1 16 9.1/16 = 0.56875 1.3 × 3 = 3.9  4
Solution
Thus, the formula is Pb3O4. Number of moles ( m /w ) moles
(c) Molality = =
26. A 2.012 g sample of a compound of nitrogen and oxygen has kg of solvent kg of solvent
0.522 g of nitrogen. Calculate its empirical formula. Now, assume 1 L of solution. We need to convert the
(a) NO2 (b) N2O (c) N2O5 (b) NO5 molarity units from mol L−1 to mol kg−1.
Solution 2.05 mol 1mL 1L 1000 g
Molality = × × × = 2.0098 mol kg−1
(c) We first determine the number of mol of each element 1L 1.02 g 1000 mL 1kg
as follows:
29. A 5.2 molal aqueous solution of methyl alcohol CH3OH is
 1 mol N  supplied. What is the mole fraction of methyl alcohol in the
mol N = (0.522 g N)  = 0.0373 mol N
 14.01 g N  solution?
(a) 0.190 (b) 0.086 (c) 0.050 (d) 0.100
We need to know the number of grams of O. Since there (AIEEE 2011)
is a total of 2.012 g of compound and the only other
element present is N, the mass of O = 2.012 g – 0.522 g Solution
= 1.490 g O. (b) We know that
Mole fraction of methyl alcohol
 1 mol O 
mol O = (1.490 g O )  = 0.0931 mol O Number of moles of methyl alcohol
 16.00 g O  =
Total number of moles

Chapter-1.indd 14 8/4/2016 9:53:55 AM


Solved Examples 15

Therefore, for aqueous solution of methyl alcohol, that Solution


is, both CH3OH + H2O, the mole fraction is Given weight d
(b) Molarity = × × 1000
Molecular weight w
m 5.2
= = 0.086
 1000   1000  120 1.12 120 1.12
m + 5.2 + M= × × 1000 = × × 1000 = 2 M
  
18   18 

60 (1000 + 120 ) 60 1120

30. The density of a solution prepared by dissolving 120 g of 34. The amount of BaSO4 formed upon mixing 100 mL of 20.8%
urea (molar mass = 60 u) in 1000 g of water is 1.15 g mL−1. BaCl2 solution with 50 mL of 9.8% H2SO4 solution will be:
The molarity of the solution is (Ba = 137, Cl = 35.5, S = 32, H = l and O = 16)
(a) 0.50 M (b) 1.78 M (c) 1.02 M (d) 2.05 M (a) 23.3 g (b) 11.65 g (c) 30.6 g (d) 33.2 g
(AIEEE 2012) (JEE Main Online 2014)

Solution Solution
(d) Molarity of solution is defined as (b) BaCl2 + H2SO4 → Ba2SO4 + 2HCl
Number of moles of urea Weight of BaCl2 = 20.8 g; Weight of H2SO4 = 9.8%
M= × 1000 = 4.9 mL
Volume of solution in mL
Given mass of urea 120 20.8 4.9
Now, number of moles of urea = = nBaCl2 = = 0.1; nH2 SO4 = = 0.05 mol
Molar mass of urea 60 208 98
Also, density of solution = Mass of solution/Volume of BaCl2 + H2SO4→ BaSO4 + 2HCl
solution (1) 0.1 0.05 0.05
where mass of solution = mass of urea + mass of water Weight of BaSO4 = 0.05 × 233 = 11.65 g
= 120 + 1000 = 1120 g
35. Two solutions of a substance (non-electrolyte) are mixed
Substituting values in Eq. (1), we get
in the following manner. 480 ml of 1.5 M first solution
1120 1120 + 520 mL of 1.2 M second solution. What is the molarity of
1.15 = ⇒ Volume = = 973.9 mL
Volume 1.15 the final mixture?
120 / 60 (a) 1.20 M (b) 1.50 M (c) 1.344 M (d) 2.70 M
Therefore, M = × 1000 = 2.05 M
973.9 Solution
31. The molarity of a solution obtained by mixing 750 mL of (c) According to molarity equation, we have
0.5 (M) HCl with 250 mL of 2(M) HCl will be M3 (V1 + V2 ) = M1V1 + M2V2
(a) 1.00 M (b) 1.75 M
(c) 0.975 M (d) 0.875 M M3 = ( 480 × 15
. + 520 × 12
. )/1000 = 1.344 M
(JEE Main 2013) 36. 25 mL of a solution of barium hydroxide on titration with
Solution 0.1 molar solution of hydrochloric acid gave a litre value of
35 mL. The molarity of barium hydroxide solution was
(d) M1V1 + M2V2 = MV
(a) 0.07 (b) 0.14 (c) 0.28 (d) 0.35
M1V1 + M2V2 0.5 × 750 + 2 × 250
M= = Solution
V 1000
M = 0.875 M (b) Using the molarity equation, we have
32. How many litres of water must be added to 1 litre of an aque- M1V1 = M2V2 (HCl)
ous solution of HCl with a pH of 1 to create an aqueous solu- 35
M1 = 0.1 × = 0.14
tion with pH of 2? 25
(a) 0.9 L (b) 2.0 L (c) 9.0 L (d) 0.1 L
37. 6.02 × 1020 molecules of urea are present in 100 mL of its
(JEE Main 2013)
solution. The concentration of urea solution is
Solution (a) 0.001 M (b) 0.1 M (c) 0.02 M (d) 0.01 M
(c) Initial pH = 1 ⇒ [H+] = 10−1 = 0.1 M and New, pH = 2 Solution
⇒ [H+] = 10−2 = 0.01 M
(d) 6.02 × 1023 molecule makes 1 mol
In case of dilution, M1V1 = M2V2
So, 6.02 × 1020 molecules make 1/1000 = (10–3) mol
0.1 × 1 = 0.01 × V2 ⇒ V2 = 10 L
Volume of water added = 9.0 L number of moles
concentration =
volume (in litres )
33. Dissolving 120 g of a compound of (mol. wt. 60) in 1000 g of
water gave a solution of density 1.12 g mL−1. The molarity of 10 −3
= = 0.01 M
the solution is 100 / 1000
(a) 1.00 M (b) 2.00 M (c) 2.50 M (d) 4.00 M Now, concentration = number of moles/volume (in litres)
(JEE Main Online 2014) = 10–3/100/1000 = 0.01 M

Chapter-1.indd 15 8/4/2016 9:53:58 AM


16 Chapter 1 Some Basic Concepts of Chemistry

38. To neutralize completely 20 mL of 0.1 M aqueous solution The limiting reagent in the reaction is O2, since it will be
of phosphorous acid (H3PO3), the volume of 0.1 M aqueous consumed completely in the reaction. Therefore,
KOH solution required is 6
Mass of water produced = × 1.5 × 18 = 32.4 g
(a) 10 mL (b) 60 mL (c) 40 mL (d) 20 mL 5
Solution 42. Ammonium nitrate detonates above 300°C according to the
chemical equation given below:
(c) Using the molarity equation, we have
n1 × M1 × V1 (H3PO3 ) = n2 × M2 × V2 (KOH) NH4NO3 (g) → N2 (g) + O2 (g) + H2O(g)
2 × 0.1 × 20 = 1 × 0.1 × V2 Select the correct statement from the following.
2 × 0.1 × 20 (a) The volume change is approximately 3.5 times the initial
V2 = = 40 mL volume at STP.
0.1
(b) The volume change is approximately 1.5 times the initial
Stoichiometry of Reactions volume taken at 323°C.
39. A + 2B + 3C = AB2C3 (c) On explosion of 80 g of ammonium nitrate, 78.4 L of total
Reaction of 6.0 g of A, 6.0 × 1023 atoms of B, and 0.036 mol volume is obtained at STP.
of C yields 4.8 g of compound AB2C3. If the atomic mass of A (d) 100 g of NH4NO3 produces volume of 215.3 L of the total
and C are 60 and 80 amu, respectively, the atomic mass of B gases at 323°C and 1 atm pressure.
is (Avogadro no. = 6 × 1023): Solution
(a) 70 amu (b) 60 amu (c) 50 amu (d) 40 amu
(d) The balanced form of equation is:
(JEE Main Online 2015)
2NH4NO3 (s) → 2N2 (g) + O2 (g) + 4H2O(g)
Solution
Since the reactant is in solid state, so the volume occu-
(c) A + 2B + 3C → AB2C3 pied by it is negligible.
The moles are approximately 3.5 times, but the volume
t=0 0.6 g 6 × 1023 0.036 mol 0
= 0.1 mol atoms = 1 mol is not 3.5 times. At STP, water is liquefied so volume is
reduced and corresponds to the gaseous moles left.
t =t 4.8 g
Taking 80 g of NH4NO3 (one mole) will produces 3.5 mol
But at STP, gaseous moles are 1.5 mol, so volume pro-
C is the limiting reagent, hence, 0.036 mol of C will be duced is approximately 1.5 × 22.4 L = 33.6 L
0.036
completely consumed, therefore = 0.012 mol of 100
3 100 g of NH4NO3 contains = 1.25 mol
AB2C3 80
Mass
Number of moles of AB2C3 = Therefore, gaseous moles appeared at given condition
Molar mass = 3.5 × 1.25 = 4.375 mol
4.8 The volume of gases,
0.012 =
60 + MB × 2 + 80 × 3 nRT 3.5 × 1.25 × 0.082 × 600
V= = ≈ 215.3 L
300 + 2 MB = 400 ⇒ MB = 50 amu p 1
43. Nitrogen monoxide forms as follows.
40. In the reaction 2SO2(g) + O2(g) → 2SO3(g) how many moles
of O2 are needed to produce 6.76 moles of SO3? 4NH3 + 5 O2 → 4NO +6H2O
(a) 2.95 mol (b) 3.08 mol How many grams of nitrogen monoxide can form if a mixture
(c) 2.56 mol (d) 3.38 mol initially contains 30.00 g of NH3 and 40.00 g of O2?
Solution (a) 25.35 g (b) 30.01 g
 1 mol O2  (c) 28.36 g (d) 32.05 g
(d) mol O2 = (6.76 mol SO3 )  = 3.38 mol O2
 2 mol SO3  Solution
41. Under the right conditions, ammonia can be converted (b) First determine the number of grams of O2 that would
to nitrogen monoxide, NO, by the following reaction be required to react completely with the given amount
NH3 + O2 → NO + H2O of ammonia. Amount of O2
How many grams of H2O could be formed from a mixture of  1 mol NH3   5 mol O2 
= (30.00 g NH3 ) 
 17.03 g NH3   4 mol NH3 
51 g of NH3 and 48 g of O2?
(a) 32.4 g (b) 81 g (c) 27 g (d) 54 g
 32.00 g O2 
Solution  1 mol O 
2
(a) First of all we need balanced chemical equation for these
types of calculations. = 70.46 g O2
4NH3 + 5O2 → 4NO + 6H2O
Since this is more than the amount that is available, we
51 48
Moles = 3 mol = 1.5 mol conclude that oxygen is the limiting reactant. The rest
17 32

Chapter-1.indd 16 8/4/2016 9:54:03 AM


Solved Examples 17

of the calculation is therefore based on the available Also, 1 mol H2S 5 mol of H2SO 4 (from stoichiometry of
amount of oxygen: reaction)
Amount of NO = ( 40.00 g O2 ) 1 5
or mol of H2S mol of H2SO 4
2 2
 1 mol O2   4 mol NO   30.01 g NO  2.5
 32.00 g O   5 mol O   1 mol NO  = M × VL ⇒ VL = = 25.0 L
2 2 0.1
= 30.0 g NO 47. H3PO4 is an acid and one of its salt is NaH2PO4. What volume
of 1 M NaOH solution should be added to 12 g of NaH2PO4 to
44. Silver nitrate, AgNO3, reacts with iron(III) chloride, FeCl3, to convert it into Na3PO4?
give silver chloride, AgCl, and iron(III) nitrate, Fe(NO3)3. A (a) 100 mL (b) 200 mL (c) 80 mL (d) 300 mL
solution containing 18.0 g of AgNO3 was mixed with a solu-
tion containing 32.4 g of FeCl3. How many grams of which Solution
reactant remain after the reaction is over? (b) The reaction involved is
(a) 22.9 g (b) 30.1 g (c) 35.9 g (d) 26.7 g
NaH2PO 4 + 2NaOH → Na3PO 4 + 2H2O
Solutions
From the reaction, 1 mol NaH2PO4 reacts with 2 mol
(d) 3AgNO3 + FeCl3 → 3AgCl + Fe(NO3)3 NaOH
The amount of FeCl3 that is required to react completely 12
Number of moles of NaH2PO4 = = 0.1 mol
with all of the available silver nitrate is 120
= (18.0 g AgNO3) Therefore, number moles of NaOH = 2 × 0.1 = 0.2 mol
Given that molarity of NaOH = 1 M, so
 1 mol AgNO 3   1 mol FeCl3   162.21 g FeCl3 
 169.87 g AgNO   3 mol AgNO   1 mol FeCl  0.2
3 3 3
1= × 1000 ⇒ V = 200 mL
V
= 5.73 g FeCl3
Since more than this minimum amount is available, 48. 20 mL of a solution containing 0.2 g of impure sample of
FeCl3 is present in excess, and AgNO3 must be the limit- H2O2 reacts with 0.316 g of KMnO4 in presence of H2SO4 as
ing reactant. per the following reaction.
We know that only 5.73 g FeCl3 will be used. Therefore, KMnO 4 + H2O2 + H2SO 4 → MnSO 4 + O2 + H2O
the amount left unused is:
Find the purity of H2O2 solution.
32.4 g total – 5.73 g used = 26.7 g FeCl3 (a) 85% (b) 82% (c) 86% (d) 83%
45. If 20 g CaCO3 is added to 20 g HCl, select the true statement. Solution
(a) CaCO3 is limiting reagent. (a) The complete balanced reaction is
(b) HCl is limiting reagent.
(c) Both are consumed completely. 2KMnO 4 + 5H2O2 + 3H2SO 4 → 2MnSO 4 + K 2SO 4 + 5O2 + 8H2O
(d) There is no reaction.
From the balanced reaction, we have
Solution 5 mol of H2O2 2 mol of KMnO4
(a) CaCO3 is a limiting reagent. The reaction involved is 0.316
Moles of KMnO4 =
CaCO3 + 2HCl → CaCl2 + CO2 + H2O 158 5 0.316
20 g 20 g So, moles of H2O2 required = ×
20 g 2 158
Moles of CaCO3 = = 0.2 moles of CaCO3 5 0.316
100 Grams of H2O2 = × × 34 = 0.17 g
20 2 158
Moles of HCl required in reacction = 0.17
73.1 Thus, % purity of H2O2 = × 100 = 85%
= 0.27 mol of HCl 0.2
49. Chlorophyll present in leaves of green plants has the formula
We are giving more moles of HCl than CaCO3. HCl is in C55H72MgN4O5. It is synthesized using carbon dioxide and
excess and some of it will remain unreacted when the magnesium present in plant cells. If a plant cell contains 0.0033
reaction is over. So CaCO3 is the limiting reagent and grams of magnesium, the amount of carbon required to com-
controls the amount of product. pletely react with it to form chlorophyll will be
46. What volume of 0.1 M H2SO4 will be required to produce (a) 0.030 g (b) 0.090 g
17.0 g of H2S by the following reaction? (c) 0.060 g (d) 0.0090 g

5H2SO 4 + 10NaI → 4Na2SO 4 + 5I2 + H2S + 4H2O Solution


(b) From the given formula of chlorophyll C55H72MgN4O5,
(a) 30.0 L (b) 50.0 L (c) 25.0 L (d) 5.0 L
we have that
Solution
17 17 1 1 mol of Mg ⇔ 55 mol of C
(c) Moles of H2S = = = mol
MH2 S 34 2 Using atomic weight of magnesium as 24 u, we have

Chapter-1.indd 17 8/4/2016 9:54:08 AM


18 Chapter 1 Some Basic Concepts of Chemistry

24 g of Mg = 1 mol 55
0.0001375 mol of Mg ⇔ × 0.0001375 = 0.0075 mol of carbon
0.0033 g Mg = 0.0001375 mol 1
or
= 0.0075 × 12
One mole of magnesium required 55 moles of carbon,
therefore, = 0.090 g of carbon

ADVANCED LEVEL PROBLEMS


1. Given that the abundances of isotopes 54Fe, 56Fe and 57Fe are 5. Reaction of Br2 with Na2CO3 in aqueous solution gives sodium
5%, 90% and 5%, respectively. The atomic mass of Fe is bromide and sodium bromate with evolution of CO2 gas.
(a) 55.85 (b) 55.95 The number of sodium bromide molecules involved in the bal-
(c) 55.75 (d) 56.75 anced chemical equation is ___.
(IIT-JEE 2009) (IIT-JEE 2011)
Solution Solution
54 × 5 + 56 × 90 + 57 × 5
(b) Atomic mass of iron = = 55.95 u (5) The balanced chemical equation is
100
3Na2CO3 + 3Br2 → 5NaBr + NaBrO3 + 3CO2
2. A student performs a titration with different burettes and
finds titre value of 25.2 mL, 25.25 mL and 25.0 mL. The number 6. The volume (in mL) of 0.1 M AgNO3 required for complete pre-
of significant figures in the average titre value is ___. cipitation of chloride ions present in 30 mL of 0.01 M solution
(IIT-JEE 2010) of [Cr(H2O)5Cl]Cl2, as silver chloride is close to ___.
(IIT-JEE 2011)
Solution
Solution
(3) The average titre value is (25.2 + 25.25 + 25.0)/3 = 25.15 mL
= 25.2 mL (according to addition of significant figures) (6) The reaction involved is
which means there are three significant figures. 2AgNO3 + [Cr(H2O)5Cl]Cl2 → 2AgCl + [Cr(H2O)5Cl](NO3 )2
3. Silver (atomic weight = 108 u) has a density of 10.5 g cm−3. Using molarity equation
The number of silver atoms on a surface of area 10−12 m2 can ( M × n × V )AgNO3 = ( M × n × V )[ Cr(H2 O )5 Cl]Cl2 , we get
be expressed in scientific notation as y × 10x. The value of x is
___. 0.1 × 1 × V = 0.01 × 2 × 30 ⇒ V = 6 mL
(IIT-JEE 2010)
7. 29.2% (w/w) HCl stock solution has density of 1.25 g mL−1. The
Solution
molecular weight of HCl is 36.5 g mol−1. The volume (mL) of
(7) We know that density = mass/volume. Given that density stock solution required to prepare a 200 mL solution of 0.4 M
= 10.5 g cm−3. This means 10.5 g of Ag is present in 1 cm3. HCl is ___.
Thus, number of atoms of Ag present in one cm3 (IIT-JEE 2012)
= 10.5/108 × NA
In 1 cm, number of atoms of Ag = (10.5/108 × NA)1/3 Solution
In 1 cm2, number of atoms of Ag = (10.5/108 × NA)2/3 (8) Given that density of solution = 1.25 g mL−1.
In 10−8 cm2, number of atoms of Ag = (10.5/108 × NA)2/3 29.2% HCl means 29.2 g of HCl in 100 g of the solution
× 10−8 = (1.05 × 6.022 × 1024/108) × 10−8 = 1.5 × 107. Density of the solution = Mass of the solution/Volume of
Hence, x = 7. the solution ⇒ V = 100/1.25 mL.
Now, molarity of the solution
4. Dissolving 120 g of urea (mol. wt. = 60) in 1000 g of water gave
Number of moles of HCl 29.2 / 36.5
a solution of density 1.15 g mL−1. The molarity of the solution = × 1000 = × 1000 = 10 M
is ____. Volume of solution 100 / 1.25
(a) 1.78 M (b) 2.00 M Using molarity equation M1V1 = M2V2, we get
(c) 2.05 M (d) 2.22 M 10 × V = 0.4 × 200 ⇒ V = 8 mL
(IIT-JEE 2011)
8. If the value of Avogadro constant is 6.023 × 1023 mol−1 and
Solution
the value of Boltzmann constant is 1.380 × 10−23 J K−1, then
(c) Since the solution consists of both urea and water, so the the number of significant digits in the calculated value of the
mass of the solution = 1000 + 120 = 1120 g. universal gas constant is
Volume of solution = mass of solution/density of solution (JEE Advanced 2014)
= 1120 g/1.15 g mL−1 = 973.9 mL.
Now, molarity = Number of moles/Volume of solution in Solution
litres, where number of moles = 120/60 = 2 mol (d) R = NA × k = 6.023 × 1023 × 1.380 × 10−23 = 8.312 (four sig-
Thus, molarity = 2/0.974 = 2.05 M. nificant figures)

Chapter-1.indd 18 8/4/2016 9:54:10 AM


Advanced Level Problems 19

9. 100 g of 0.1 M Ca(OH)2 aqueous solution of density 1.5 g cm−3 (b) In a solution of 100 mL 0.5 M acetic acid, one g of active
is mixed with 100 g of 0.1 M H2SO4 of density 1.1 g cm−3. charcoal is added, which absorbs acetic acid. It is found
Which of the following statements is not correct about the that the concentration of acetic acid becomes 0.49 M. if
final mixture? surface area of charcoal is 3.01× 102 m2 , calculate the
(a) Final solution is neutral with [H+] ≅ 10−7 M. area occupied by single acetic acid molecule on surface
(b) The final calcium ion concentration, [Ca2+] ≅ 0.04 M. of charcoal.
(c) The final sulphate ion concentration, [SO24− ] ≅ 0.06 M.
Solution
(d) Ca(OH)2 is the limiting reagent in the reaction.
(a) Given that side of the square = 40 mm and diameter of
Solution marbles = 10 mm. The number of marbles along an edge
(a) Ca(OH)2 + H2SO 4 → CaSO 4 + 2H2O of the square with their centers within the square = 5
Moles of Ca(OH)2 taken = Molarity × Volume 10 mm
Mass
= Molarity ×
density
100
= 0.1× mmol = 6.66 mmol
1.5
100
Similarly, moles of H2SO 4 taken = 0.1 × mmol
1.1
= 9.09 mmol 40 mm
Therefore, Ca(OH)2 is the limiting reagent and it cannot
Maximum no. of marbles per unit area = 5 × 5 = 25.
neutralize the acid completely.
(b) The number of moles of acetic acid in 100 mL (before
The final volume of solution = 66.6 mL + 90.9 mL = 157.5 mL
adding charcoal) = 0.05
mmol of Ca2+ ions 6.66 The number of moles of acetic acid in 100 mL (after add-
[Ca2 + ] = = = 0.04 M
total volume 157.5 ing charcoal) = 0.049
9.09 The number of moles of acetic acid adsorbed on the sur-
[SO24− ] = ≈ 0.06 M face of charcoal = 0.001 (0.05 − 0.049)
157.5
The number of molecules of acetic acid adsorbed on the
2 × (9.09 − 6.66 ) surface of charcoal = 0.001 × 6.02 × 1023 = 6.02 × 1020
[H+ ] = = 0.03 M
157.5 Given that the surface area of charcoal = 3.01 × 102 m2.
Therefore, the final mixture is not neutral as equivalents of Hence, the area occupied by single acetic acid molecule
acid and base are not equal. 3 × 102
on the surface of charcoal is = 5 × 10 −19 m2
10. Manganese trifluoride, MnF3, can be prepared by the follow- 6.02 × 1020
ing reaction 12. Calculate the amount calcium oxide required when it reacts
with 852 g of P4O10.
MnI2 (s) + F2 (g) → MnF3 ( s) + IF5 (l)
Solution
If the percentage yield of MnF3 is always approximately 56%,
how many grams of IF5 can be expected if 10 grams of each The required reaction is: 6CaO + P4 O10 → 2Ca3 (PO 4 )2
reactant is used in an experiment? Moles of P4O10 = 852/284 = 3 and that of CaO = 3 × 6 = 18.
[At. wt: I = 127, Mn = 55, F = 19] According to the reaction, we have
(a) 8.0 g (b) 4.0 g (c) 14.3 g (d) 10.0 g 1 mol of P4O10 reacts with 6 mol of CaO
So, 3 mol of P4O10 reacts with (3 × 6) = 18 mol of CaO.
Solution Thus, the amount of calcium oxide required = 18 × 56
(a) Balanced chemical equation: = 1008 g.
2 MnI2 (s) +13 F2 (g) → 2MnF3 (s) + 4IF5 (l)
13. Find the molarity of water. Given: d = 1000 kg m−3.
10 10
Moles of MnI2 taken = mol, Moles of F2 taken = Solution
mol, 309 38
Molarity is defined as the number of moles of solute present/
In this reaction, MnI2 is the limiting reagent, so, theoreti-
volume of solution in litres.
10 20 Given, 1 L of water = 1 kg = 1000 g (because density
cal yield of IF5 = ×2 = mol
309 309 = 1000 kg m−3). Therefore, the number of moles of solute
20 1000 (given mass)
Thus experimental yield = × 222 g ≈ 8.0 g
309 present = = 55.55 mol of H2O
18 (molecular mass of water)
11. (a) Marbles of diameter 10 mm are to be put in a square So, the molarity is 55.55 mol L−1 = 55.55 M.
area of side 40 mm so that their centers are within this
area. Find the maximum number of marbles per unit 14. At 100 °C and 1 atm, if the density of liquid water is 1.0 g cm−3
area and deduce an expression for calculating it. and that of water vapour is 0.0006 g cm−3, then the volume

Chapter-1.indd 19 8/4/2016 9:54:14 AM


20 Chapter 1 Some Basic Concepts of Chemistry

occupied by water molecules in 1 L of steam at this temper- Solution


ature is (d) 1 mol of electron weighs 9.108 × 10 −31 × 6.023 × 1023 kg.
(a) 6 cm3 (b) 60 cm3 So, the number of moles of electron in 1 kg is
(c) 0.6 cm3 (d) 0.06 cm3
1 1 × 108
Solution −31 23
=
9.108 × 10 × 6.023 × 10 9.108 × 6.023
(c) Mass of 1 L of steam = 1000 × 0.006 = 0.6 g = Mass of
liquid water 16. Which of the following has maximum number of atoms?
Now, density = mass/volume = 1.0 g cm−3. Therefore, (a) 24 g of C (Mol. wt. = 12 g mol−1)
volume = 0.6/1.0 = 0.6 cm3. (b) 23 g of Na (Mol. wt. = 23 g mol−1)
The same volume will be occupied by water molecules (c) 48 g of S (Mol. wt. = 32 g mol−1)
in steam. (d) 108 g of Ag (Mol. wt. = 108 g mol−1)
15. How many moles of electrons weighs 1 kg? Solution
1 (a) 24 g carbon amounts to 24/12 = 2 mol, that is, 2 × 6.023
(a) 6.023 × 1023 (b) × 1031
9.108 × 1023 atoms, whereas (b), (c) and (d) amount to 1 mol,
6.023 1 1.5 mol and 1 mol, respectively.
(c) × 1054 (d) × 108
9.108 9.108 × 6.023

PRACTICE EXERCISE

Level I 7. What mass of silver nitrate will react with 5.85 g of sodium
chloride to produce 14.35 g of silver chloride and 8.5 g of
Single Correct Choice Type sodium nitrate, if the law of conservation of mass is true?
1. How many significant figures, respectively, are there in (a) 15 g (b) 16 g (c) 17 g (d) 18 g
73.000 g, 0.0503 g and 2.001 g? 8. 40% w/V NaCl solution (specific gravity = 1.12) is equivalent to
(a) 3, 3, 4 (b) 3, 4, 5 (c) 2, 5, 4 (d) 5, 3, 4 (a) 3.57 × 105 ppm (b) 3.57 × 106 ppm
(c) 1 × 106 ppm (d) 4 × 105 ppm
2. How many moles of sulphuric acid, H2SO4, are needed to react
with 0.366 mol of NaOH by the following reaction? 9. 2 mol of H2S and 11.2 L of SO2 at STP react according to the
following equation:
2NaOH(aq) + H2SO 4 (aq) → Na2SO 4 (aq) + 2H2O SO2 + 2H2S → 2H2O + 3S
(a) 0.195 mol (b) 0.169 mol What will be the number of moles of sulphur formed in the
(c) 0.183 mol (d) 0.158 mol reaction?
(a) 1.5 (b) 3 (c) 11.2 (d) 6
3. Which one contains the greatest number of oxygen atoms?
(a) 1 g of O 10. Aluminum sulphate is analyzed and it is determined that
(b) 1 g of O2 the sample contains 0.0774 mol of sulphate ions. How many
(c) 1 g of O3 moles of aluminum does the sample contain? 
(d) All have the same number of atoms (a) 0.0421 mol (b) 0.0529 mol
(c) 0.0458 mol (d) 0.0516 mol
4. Which response gives the correct number of significant fig- 11. A compound contains 84.2% boron and 15.7% hydrogen.
ures for all three of the following measurements: 7.103 cm, Its molecular mass was calculated from its vapour density as
0.00005 inch, and 1.3400 × 10−4 dm3? 76.7 g. What is its molecular formula?
(a) 3, 5 and 4 (b) 3, 1 and 3 (a) B6H6 (b) B6H12 (c) BH3 (d) B2H6
(c) 4, 1 and 3 (d) 4, 1 and 5
12. An organic compound weighing 0.6672 g is decomposed
5. Use the data given in the following table to calculate the molar giving 0.3481 g carbon and 0.0870 g hydrogen. What are the
mass of naturally occurring argon isotopes: percentages of hydrogen and carbon in this compound?
(a) 13.04%, 52.2% (b) 10.5%, 35.9%
Isotope Isotopic molar mass Abundance
(c) 18.9%, 60.2% (d) 15.6%, 45.2%
36Ar 35.96755 g mol−1 0.337% 13. The density of 1 M solution of NaCl is 1.0585 g mL−1. The
38Ar 37.96272 g mol−1 0.063% molality of the solution is
(a) 1.0585 (b) 1.00
40Ar 39.9624 g mol−1 99.600% (c) 0.10 (d) 0.0585
(a) 39.35 g mol−1 (b) 40.35 g mol−1 14. H3PO4 is an acid and one of its salts is NaH2PO4. What volume
(c) 37.35 g mol−1 (d) 38.35 g mol−1 of 1 M NaOH solution should be added to 12 g of NaH2PO4 to
convert it into Na3PO4?
6. The number of significant figures in 10.3106 g is (a) 100 mL (b) 200 mL
(a) 2 (b) 3 (c) 6 (d) 4 (c) 80 mL (d) 300 mL

Chapter-1.indd 20 8/4/2016 9:54:15 AM


Practice Exercise 21

15. Weight of oxygen in Fe2O3 and FeO is in the simple ratio 26. The simplest formula of a compound containing 50% of ele-
(a) 3:2 (b) 1:2 (c) 2:1 (d) 3:1 ment X (atomic mass 10) and 50% of element Y (atomic mass
20) is
16. The sterile saline solution used to rinse contact lenses can (a) XY (b) X2Y (c) XY3 (d) X2Y3
be made by dissolving 400 mg of NaCl in sterile water and
diluting to 100 mL. What is the molarity of the solution? 27. Oxygen gas can be produced in the laboratory by decompo-
(a) 0.0685 M (b) 0.0312 M sition of hydrogen peroxide (H2O2):
(c) 0.0212 M (d) 0.0418 M
2H2O2 (aq) → 2H2O + O2 (g)
17. In dilute nitric acid, HNO3, copper metal dissolves according
to the following equation. How many kilograms of O2 can be produced from 1.0 kg of
H2O2?
3Cu(s) + 8HNO3 (aq) → 3Cu(NO3 )2 ( aq) + 2NO(g) + 4H2O
(a) 0.47 kg (b) 0.35 kg (c) 0.54 kg (d) 0.29 kg
How many grams of HNO3 are needed to dissolve 11.45 g of 28. 0.2 M H2SO 4 (1 mL) is diluted to 1000 times of its initial vol-
Cu according to this equation? ume. The final normality of H2SO 4 is
(a) 25.36 g (b) 30.28 g (c) 42.5 g (d) 19.5 g
(a) 2 × 10−4 (b) 2 × 10−3
−4
18. Calculate the number of moles left after removing 1021 mol- (c) 4 × 10 (d) 2 × 10−2
ecules from 200 mg of CO2.
(a) 3.02 × 10−3 mol (b) 1.35 × 10−3 mol 29. A white solid used to whiten paper has the following per-
(c) 2.88 × 10−3 mol (d) 3.25 × 10−3 mol centage composition: Na, 32.4%; S, 22.6%. The unanalyzed
element is oxygen. What is the compound’s empirical
19. An oxide of iodine (I = 27) contains 25.4 g of iodine for 8 g of formula?
oxygen. Its formula could be (a) Na2SO2 (b) Na2SO3
(a) I2O3 (b) I2O (c) I2O5 (d) I2O7 (c) Na2S2O2 (d) Na2SO4

20. The reaction of hydrazine, N2H4, with hydrogen peroxide, 30. 12.78 g of an organic compound containing C, H and O and
H2O2, has been used in rocket engines. One way these com- undergoes combustion to produce 25.56 g of carbon dioxide
pounds react is described by the equation and 10.46 g of water. Empirical formula of the organic com-
pound is
N2H4 + 7H2O2 → 2HNO3 + 8H2O
(a) CHO (b) CH2O
According to this equation, how many grams of H2O2 are (c) CH4O (d) C2H4O
needed to react completely with 852 g of N2H4?
31. How many moles of H2 and N2 can be formed by the decom-
(a) 7052 g (b) 5486 g (c) 6330 g (d) 4256 g
position of 0.145 mol of ammonia, NH3?
21. Chlorine is prepared in the laboratory by treating manga- (a) 0.245 mol, 0.0589 mol (b) 0.325 mol, 0.0639 mol
nese dioxide (MnO2) with aqueous hydrochloric acid accord- (c) 0.169 mol, 0.0685 mol (d) 0.218 mol, 0.0725 mol
ing to the following reaction: 32. Oxygen gas can be produced in the laboratory by decompo-
4HCl(aq) + MnO2(s) → 2H2O(l) + MnCl2(aq) + Cl2(g) sition of potassium chlorate (KClO3)
The grams of HCl that react with 5.0 g of manganese dioxide 2KClO3 (s) → 2KCl(s) + 3 O2 (g)
will be (at. mass of Mn = 55)
(a) 84 g (b) 0.84 g (c) 8.4 g (d) 4.2 g How many kilograms of O2 can be produced from 1.0 kg of
KClO3?
22. If 30 mL of a gaseous hydrocarbon requires 90 mL of O2 for (a) 0.25 kg (b) 0.56 kg (c) 0.78 kg (d) 0.39 kg
complete combustion and 60 mL of CO2 is formed in the pro-
cess, the molecular formula of hydrocarbon will be 33. Copper reacts with dilute nitric acid according to the follow-
(a) C2H2 (b) CH4 (c) C2H4 (d) C2H6 ing equation:
3Cu(s) + 8HNO3(aq) → 3Cu(NO3)2(aq) + 2NO(g) + 4H2O(l)
23. Titanium metal is obtained from the mineral rutile, TiO2. How
many kilograms of rutile are needed to produce 100.0 kg of Ti? If a copper penny weighing 3.045 g is dissolved in a small
(a) 166.8 kg (b) 132.4 kg (c) 11.4 kg (d) 14.3 kg amount of nitric acid and the resultant solution is diluted to
50.0 mL with water, what is the molarity of the Cu(NO3)2?
24. How many mole of lead(II) chloride will be formed from a (a) 0.958 M (b) 0.278 M
reaction between 6.5 g of PbO and 3.2 g of HCl? (c) 0.145 M (d) 0.312 M
(a) 0.0356 mol (b) 0.0119 mol
(c) 0.0563 mol (d) 0.029 mol 34. A 100 mL solution of 0.1 N HCl was titrated with 0.2 N NaOH
solution. The titration was discontinued after adding 30 mL
25. A metal oxide has 40% oxygen. The equivalent weight of the of NaOH solution. The remaining titration was completed by
metal is adding 0.25 N KOH solution. The volume of KOH required for
(a) 12 g equiv. (b) 16 g equiv. completing the titration is
(c) 24 g equiv. (d) 48 g equiv. (a) 70 mL (b) 32 mL (c) 35 mL (d) 16 mL

Chapter-1.indd 21 8/4/2016 9:54:18 AM


22 Chapter 1 Some Basic Concepts of Chemistry

35. An inorganic substance has the following composition: 43. Sulphuric acid reacts with sodium hydroxide as follows:
N = 35%, H = 5%, O = 60%. On being heated, it yielded a H2SO4 + 2NaOH → Na2SO4 + 2H2O
gaseous compound containing N = 63.3% and O = 36.37%.
Which of the following reaction can be suggested based on When 1 L of 0.1 M sulphuric acid solution is allowed to react with
the given data? 1 L of 0.1 M sodium hydroxide solution, the amount of sodium
(a) 2HNO3 → N2O5 + H2O sulphate formed and its molarity in the solution obtained is
(b) NH2OH + HONO → N2O + 2H2O (a) 0.1 mol L−1 (b) 7.10 g
(c) NH4NO3 → N2O + 2H2O (c) 0.025 mol L−1 (d) 3.55 g
(d) 2HNO2 → N2O3 + H2O
44. PF3 reacts with XeF4 to give PF5.
36. Manganese trifluoride can be prepared by the following
2PF3 (g) + XeF4 (g) → 2PF5 (g) + Xe(g)
reaction:
MnI2(s) + F2(g) → MnF3 + IF5 If 100.0 g of PF3 and 50.0 g of XeF4 react, then which of the
following statements is true?
What is minimum number of grams of F2 that must be used
(a) XeF4 is the limiting reagent.
to react with 12.0 g of MnI2 if overall yield of MnF3 is no more
(b) PF3 is the limiting reagent.
than 75%?
(c) 1.137 mol of PF5 are produced.
(a) 12.78 (b) 9.2 (c) 6.1 (d) 7.4
(d) 0.482 mol of PF5 are produced.
37. How much quantity of zinc will have to be reacted with
45. Which of the following solutions have the same
excess of dilute HCl solution to produce sufficient hydrogen
concentration?
gas for completely reacting with the oxygen obtained by
(a) 20 g of NaOH in 200 mL of solution.
decomposing 5.104 g of potassium chlorate?
(b) 0.5 mol of KCl in 200 mL of solution.
(a) 8.124 g (b) 81.24 g
(c) 40 g of NaOH in 100 mL of solution.
(c) 0.08 g (d) 812.4 g
(d) 20 g of KOH in 200 mL of solution.
38. The reaction of powdered aluminum and iron(III) oxide,
46. One gram of Mg is burnt in a closed vessel that contains 0.5 g
2Al + Fe2O3 → Al2O3 + 2 Fe of O2. It means
(a) equivalent of MgO is equal to that of magnesium (1 g/12).
produces so much heat the iron that forms is molten. (b) equivalent of MgO is equal to that of oxygen (0.5 g/8).
Because of this, railroads use the reaction to provide (c) equivalent of oxygen = equivalent of magnesium
molten steel to weld steel rails together when laying track. = equivalent of MgO in the reaction.
Suppose that in one batch of reactants 4.20 mol of Al was (d) moles of oxygen = moles of magnesium = moles of MgO
mixed with 1.75 mol of Fe2O3. Calculate the number of in the reaction.
grams of iron that can be formed from this mixture of
reactants. Passage Type
(a) 195 g (b) 254 g (c) 135 g (d) 201 g Paragraph for Questions 47 to 49: Oleum is a mixture of H2SO4
39. Calculate the number of moles of phosphorus in 15.95 g of and SO3 (i.e., H2S2O7). It is produced by passing SO3 in H2SO4 solu-
tetraphosphorus decaoxide, P4O10. tion. In order to dissolve free SO3 in oleum, dilution of oleum is
(a) 0.02247 mol (b) 2.247 mol done, in which oleum converts into pure H2SO4. It is shown by the
(c) 0.2247 mol (d) 0.0658 mol reaction as under:
H2SO4 + SO3 + H2O → 2H2SO4 (Pure)
Level II
Multiple Correct Choice Type SO3 + H2O → H2SO4 (Pure)

40. One of the statements of Dalton’s atomic theory is given When 100 g sample of oleum is diluted with desired amount
below: “Compounds are formed when atoms of different ele- of H2O (in grams), then the total mass of pure H2SO4 obtained
ments combine in a fixed ratio”. Which of the following laws after dilution is known as percentage labeling in oleum. Through
is not related to this statement? this process, the percentage composition of H2SO4, SO3 (free), and
(a) Law of conservation of mass SO3 (combined) is calculated.
(b) Law of definite proportions If oleum sample is labeled as “109% H2SO4,” that is, 100 g of oleum
(c) Law of multiple proportions on dilution with 9 g of H2O provides 109 g pure H2SO4, in which
(d) Avogadro’s law all free SO3 in 100 g of oleum is dissolved.

41. 1 mol of H2SO4 will exactly neutralize 47. For 109% labeled oleum, if the number of moles of H2SO4
(a) 2 mol of ammonia (b) 1 mol of Ca(OH)2 and free SO3 be x and y, respectively, then what will be the
(c) 0.5 mol of Ba(OH)2 (d) 2 mol NaOH value of x2 + y2?
(a) 0.15 (b) 0.42 (c) 0.62 (d) 0.80
42. Which of the following quantities are dependent on
temperature? 48. The percentage of combined SO3 in the given oleum
(a) Molarity (b) Normality sample is
(c) Molality (d) Mole fraction (a) 20% (b) 30% (c) 48.98% (d) 51%

Chapter-1.indd 22 8/4/2016 9:54:19 AM


Hints and Explanations 23

49. The percentage composition of free SO3 and H2SO4 in the (c) 0.0025 (r) 1
oleum sample, respectively, are (d) 204 (s) 3
(a) 60%, 40% (b) 30%, 70%
54. Match the number of moles with their amount.
(c) 85%, 15% (d) 40%, 60%.
Column I Column II
Paragraph for Questions 50 to 51: 1.00 g of a mixture having (a) 0.1 mol (p) 4480 mL of CO2 at STP
equal number of moles of carbonates of two alkali metals required (b) 0.2 mol (q) 0.1 g atom of iron
44.4 mL of 0.5 N HCl for complete reaction. Atomic weight of one (c) 0.25 mol (r) 1.5 × 1023 molecules of
of the metals is 7.00. oxygen gas
(d) 0.5 mol (s) 9 mL of water
50. The number of moles of each metal carbonate are
(t) 200 mg of hydrogen gas
(a) 0.1 (b) 0.0111 (c) 0.0055 (d) 0.00275
Integer Type
51. The number of equivalents of HCl used is
55. The number of moles and equivalents in 196 g of H3PO4 are ___
(a) 0.222 (b) 2.22 (c) 22.22 (d) 0.0222
(Given n-factor = 3).
Matrix-Match Type 56. In the reaction CO(g) + 2H2(g) → CH3OH(g), the number of
52. Match the concentration terms with the factors affecting the moles of H2 required to react completely with 2 mol of CO is
concentrations. ___.
Column I Column II 57. Two moles of 50% pure Ca(HCO3)2 on heating forms 1 mol of
(a) Molarity (M) (p) Temperature CO2. The percentage yield of CO2 is ____.
(b) Molality (m) (q) Pressure 58. The number of significant figures in Avogadro’s constant
(c) Mole fraction (x) (r) Dilution (6.023 × 1023) is ___.
(d) Normality (s) Volume
59. The number of gram atoms in 24 g of magnesium is ___.
53. Match the numbers with the number of significant figures
60. 4.48 L of ammonia at STP is neutralized using 100 mL of a
they contain.
solution of H2SO4. The molarity of acid is ___.
Column I Column II
(a) 62.01 (p) 2 61. Molecular formula of acetic acid is CH3COOH. The number of
(b) 100 (q) 4 atoms present in its empirical formula is ___.

ANSWER KEY
Level I
1. (d) 2. (c) 3. (d) 4. (d) 5. (a) 6. (c)
7. (c) 8. (a) 9. (a) 10. (d) 11. (b) 12. (a)
13. (b) 14. (b) 15. (d) 16. (a) 17. (b) 18. (c)
19. (c) 20. (c) 21. (c) 22. (c) 23. (a) 24. (d)
25. (a) 26. (b) 27. (a) 28. (c) 29. (d) 30. (d)
31. (d) 32. (d) 33. (a) 34. (d) 35. (c) 36. (a)
37. (a) 38. (a) 39. (c)

Level II
40. (a), (d) 41. (a), (b), (d) 42. (a), (b) 43. (b), (c) 44. (a), (d) 45. (a), (b)
46. (b), (d) 47. (c) 48. (c) 49. (d) 50. (b) 51. (d)
52. (a) → p, r, s; (b) → r; (c) → q, r; (d) → r, s 53. (a) → q; (b) → r; (c) → p; (d) → s
54. (a) → q, t; (b) → p; (c) → r; (d) → s 55. (2), (6) 56. (4) 57. (100) 58. (4)
59. (1) 60. (1) 61. (4)

HINTS AND EXPLANATIONS


Level I 2. (c)
Single Correct Choice Type  1 mol H2SO 4 
Number of moles of H2SO 4 = (0.366 mol NaOH) 
 2 mol NaOH 
1. (d) The number of significant figures in each is as follows:
73.000 g → 5 significant figures = 0.183 mol
0.0503 g → 3 significant figures
2.001 g → 4 significant figures 3. (d) 1 g atom or molecules = N atoms = 6.023 × 1023 atoms

Chapter-1.indd 23 8/4/2016 9:54:20 AM


24 Chapter 1 Some Basic Concepts of Chemistry

4. (d) 7.103 = four significant digits (7, 1, 0, 3) Empirical formula mass = 10.8 × 1+ 2 × 1 = 12.8 g
0.00005 = one significant digit (5)
Molecular mass 76.7
13.400 × 10−4 = five significant digits (1, 3, 4, 0, 0) n= = =6
Empirical formula mass 12.8
5. (a) From the values given in the table, we get Therefore,
Molecular formula = Empirical formula × 6 = BH2 × 6 = B6H12
 0
0..337
337   37.96272 × 0 0..063
063 
Molar
Molar mass
mass of argon =
of argon =  35 96755 ×
35..96755 × 100  + +  37.96272 × 100   0.0870 g H 
 100   100   mass H 
12. (a) % H =  × 100% =  × 100%
 90 .60    total mass   0.6672 g total 
+ 9624 ×
39..9624 90.60  g mol−1 −1
+  39 × 100  g mol  = 13.04%
 100  
−1  mass C   0.3481 g C 
= [ 0 .121 + 0 .024 + 36 .206
= [0.121+ 0.024 + 36.206] g mol−1 ] g mol %C=  × 100% =  × 100%
 total mass   0.6672 g total 
=
= 39
39..351
351 gmol−−11
g mol
= 52.17%
6. (c) 10.3106; there are 6 significant figures. It is likely that the compound contains another element
since the percentages do not add up to 100%.
7. (c) Given that mass of sodium chloride = 5.85 g, mass of
w m
silver chloride = 14.35 g, mass of sodium nitrate = 8.5 g. 13. (b) m = and r = ⇒ m = 1 molal
Let the mass of silver nitrate be m. Now, according to M × W (kg) V
law of conservation of mass,
Total mass of reactants = Total mass of products 14. (b) The reaction involved is
NaH2PO 4 + 2NaOH → Na3PO 4 + 2H2O
Mass of silver nitrate + Mass of sodium chloride = Mass
of silver chloride + Mass of sodium nitrate From the reaction, 1 mol NaH2PO4 reacts with 2 mol
NaOH
m + 5.85 g = 14.35 g +8.5 g 12
Number of moles of NaH2PO4 = = 0.1 mol
m = (14.35 g + 8.5 g) − 5.85 g = 22.85 g − 5.85 g = 17 g 120
Therefore, number moles of NaOH = 2 × 0.1 = 0.2 mol
8. (a) For 40% w/V NaCl solution, 100 mL of solution contains Given that molarity of NaOH = 1 M, so
40 g of NaCl. 0.2
1= × 1000 ⇒ V = 200 mL
Given density (specific gravity) = 1.12 g mL−1 V
100 mL solution (=112 g) contains 40 g NaCl or 112 g of 15. (d) The weight of oxygen is
solution contains 40 g of NaCl.
Fe2O3 : FeO = 48 : 16 = 3 : 1
106 g of solution contains x g of NaCl.
or 3 × 16 : 1× 16
40
x= × 106 = 3.57 × 105 ppm Weight of oxygen is in the ratio = 3:1
112
16. (a) Molecular mass of NaCl is 58.5.
9. (a) The reaction is as follows:
1 molar solution contains 58.5 g in1000 L
SO2 + 2H2S → 2H2O + 3S
11.2 L 2 mol 2 mol 3 mol 1 molar solution contains 5.85 g in100 mL
Now, 1 molar contains 5.85 g
1 mol of SO2 gives 3 mol of sulphur atoms.
1 3 Given that x molar contains 400 × 10 −3 g
mol of SO2 will give mol of sulphur atom.
2 2
400 × 10 −3
10. (d) In aluminum sulphate, Al2(SO4)3, aluminum is Al3+. Therefore, x = = 0.0685 M
5.85
Number of moles of Al3+ = 0.0774 mol
17. (b) The amount of HNO3 required is
 2 mol Al3+ 
SO42–   = 0.0516 mol g HNO3 = (11.45 g Cu) ×
3 mol SO 4 2− 
 1 mol Cu   8 mol HNO3   63.013 g HNO3 
11. (b) Calculation of empirical formula  63.546 g Cu   3 mol Cu   1 mol HNO 
3

At. Relative number Simple ratio of = 30.28 g HNO3


Element % mass of moles moles
18. (c) Given weight of CO2 = 200 mg
B 84.2 10.8 7.78 1 6.023 × 1023 molecules of CO2 = 44 g
H 15.7 1 15.7 2 44 × 1021
1021 molecules of CO2 = = 7.31× 10 −2 g
Thus, empirical formula is BH2. 6.023 × 1023
Calculation of molecular formula: = 73.1 mg

Chapter-1.indd 24 8/4/2016 9:54:25 AM


Hints and Explanations 25

Now, weight of CO2 left = Given weight of CO2 − Weight 6.5


of CO2 removed = 200 − 73.1 = 126.9 mg Moles of PbO = = 0.029 mol and moles of HCl
223
Weight 126.9 × 10 −3 g 3.2
So, the moles left = = = = 0.088 mol
Molecular weight 44 g mol−1 36.5
1 mol of PbO reacts with 2 mol of HCl. Therefore, PbO is
= 2.88 × 10 −3 mol
a limiting reagent.
Hence, 0.029 mol of PbO will produce PbCl2 = 0.029 mol
19. (c) I2O5
25. (a) The 100 g of sample of metal oxide contains 40 g of oxy-
Atomic Number gen and 60 g of metal, M. By the definition of equivalent
Element Mass mass of mol Simplest ratio mass,
25.4/127 60
I 25.4 127 0.2/0.2 = 1 × 2 = I2 × 8 = 12 g equiv.
= 0.2
40
O 8 16 8/16 = 0.5 0.5/0.2 = 2.5 × 2 = O5
26. (b) From the given data, we have
20. (c) The amount of H2O2 required is
Element % (a) Atomic weight (b) a/b Ratio
g H2O2 = (852 g N2H4 ) X 50 10 5 2
 1 mol N2H4   7 mol H2O2   34.02 g H2O2  Y 50 20 2.5 1
 32.05 g N H   1 mol N H   1 mol H O 
2 4 2 4 2 2 So, the simplest formula = X2Y.
= 6330 g H2O2 27. (a) The amount of O2 that can be produced is
kg O2 = 1.0 kg H2O2 ×
21. (c) 4HCl + MnO2 → MnCl2 + Cl2 + 2H2O
 1000 g H2O2   1 mol H2O2   1 mol O2 
We have n =
5
, so  1 kg H O   34.01 g H O   2 mol H O  ×
2 2 2 2 2 2
87
 32.00 g O2   1 kg O2 
5  1 mol O   1000 g O 
Moles of HCl reacted = × 4 = 0.05747 mol 2 2
87
= 0.47 kg O2
Mass of HCl = 0.05747 × 36.5 = 8.4 g
22. (c) a = 2, b = 2 28. (c) N1V1 (before dilution) = N2V2 (after dilution)
 y y Normality of H2SO4 = 0.2 × 2 = 0.4 N
C xH y +  x +  O2 →
 xCO2 + H2O
 4  2 So, 0.4 × 1 = N2 × 1000 ⇒ N2 = 4 × 10 −4 N
30 mL 90 mL 60 mL
1 mL 3 mL 2 mL 29. (d) It is convenient to assume that we have 100 g of
or 1 mol 3 mol 2 mol the sample, so that the % by mass values may be
taken directly to represent masses. Thus there is
Equal volumes of all gases contain equal number of 32.4 g of Na, 22.6 g of S and (100.00 – 32.4 – 22.6) =
molecules at the same conditions of temperature and 45.0 g of O. Now, convert these masses to a number
pressure. of mol:
y
x = 2 and x + = 3  1 mol Na 
4 mol Na = (32.4 g Na)  = 1.40 mol Na
 23.00 g Na 
y
= 3 − 2 = 1⇒ y = 4
4  1 mol S 
mol S = (22.6 g S)  = 0.705 mol S
Thus, the molecular formula is C2H4.  32.06 g S 

23. (a) TiO2 → Ti + O2  1 mol O 


mol O = ( 45.0 g O )  = 2.81 mol O
 16.00 g O 
80 g 48 g 32 g
100.0 kg
80 g TiO2 gives 48 g of Ti and x g TiO2 gives 100 × 103 g
Next, we divide each of these mol amounts by the
of Ti. So,
smallest in order to deduce the simplest whole number
80 × 100 × 103 g ratio:
x= = 166.7 kg
48 For Na: 1.40 mol/0.705 mol = 1.99
For S: 0.705 mol/0.705 mol = 1.00
24. (d) The reaction equation is PbO + 2HCl → PbCl2 + H2O. For O: 2.81 mol/0.705 mol = 3.99
Molar mass of PbO = 223 g mol−1. Now, The empirical formula is Na2SO4.

Chapter-1.indd 25 8/4/2016 9:54:30 AM


26 Chapter 1 Some Basic Concepts of Chemistry

30. (d) CO2 = 25.56 g, H2O = 1046 g So, the remaining 40 mL 0.1 N HCl is now neutralized by
0.25 N KOH
1 mol
Number of moles of C = × 25.56 g (HCl) N1V1 = N2V2 (KOH)
44 g
= 0.581mol (6.97 g) 0.1 × 40 = 0.25 × V2 ⇒ V2 = 16 mL
2 mol 35. (c) From the question, we have
Number of moles of H = × 10.46 g
18 g Simplest
= 1.162 mol (1.16 g) Relative Simplest whole
Percent- no. of ratio of number
Mass of oxygen = 12.78 − 6.97 − 1.16 = 4.65 g Element Symbol age moles moles ratio
= 0.291mol 35 2.5
Nitrogen N 35 = 2.5 = 1× 2 2
14 2.5
Relative number of Simplest whole
Element Moles moles number ratio 60 3.75
Oxygen O 60 = 3.75 = 1.5 × 2 3
0.581 16 2.5
C 0.581 2 2
0.291 5 5
Hydrogen H 5 =5 = 2×2 4
1.162 1 2.5
H 1.162 4 4
0.291 Therefore, empirical formula N2H4O3.
0.291 N2H4O3 is nothing but NH4NO3.
O 0.291 1 1
0.291
36. (a) 2MnI2 + 13F2 → 2MnF3 + 4IF5
Thus, the empirical formula is C2H4O. 2( 55 + 2 ×127 ) g (13 × 38 ) g
= 618 g = 494 g
31. (d) Based on the balanced equation: 12 g ?
2 NH3(g) → N2(g) + 3H2(g) 618 g of MnI2 reacts with 494 g of F2 to form MnF3; 12 g
From this equation the conversion factors can be of MnI2 reacts with x g to form MnF3.
written:
494 × 12
 1 mol N2   3 mol H2  x= = 9.59 g of F2
618
 2 mol NH  and  2 mol NH 
3 3 Given that yield should be 75%. Therefore,
To determine the moles produced, simply convert from
9.59
starting moles to end moles: Actual requirement = × 100 = 12.78 g of F2
75
 1 mol N2 
mole N2 = 0.145 mol NH3  = 0.0725 mol N2
 2 mol NH 3 37. (a) The reactions are
The moles of hydrogen are calculated similarly: 2KClO3 → 2KCl + 3O2
 3 mol H2  245 g 96 g
mole H2 = 0.145 mol NH3  = 0.218 mol H2 5.104 g ? (x) g
 2 mol NH3  96 g
x= × 5.104 g = 2 g of O2
32. (d) kg O2 = 1.0 kg KCIO2 × 245 g
2H2 + O2 → 2H2O
 1000 g KClO3   1 mol KClO3   3 mol O2  4g 32 g
 1 kg KClO   122.6 g KClO   2 mol KClO  × ? 2g
3 3 3
4g
 32.00 g O2   1 kg O2  x= × 2 g = 0.25 g of H2
 1 mol O   1000 g O  32 g
2 2
= 0.39 kg O2 Zn + 2HCl → ZnCl2 + H2 ↑
65 g 2g
? 0.25 g
Molecular weight × Volume required
33. (a) Molarity = 65 g
Weight required ×1000 x= × 0.25 g = 8.125 g
2g
57.8 × 50
= = 0.958 M 38. (a) First determine the amount of Fe2O3 that would be
3.045 × 1000
required to react completely with the given amount of Al
34. (d) Volume of HCl neutralized by NaOH = (Caustic soda) = V1
 1 mol Fe2O3 
mol Fe2O3 = ( 4.20 mol Al)  = 2.10 mol Fe2O3
(HCl) N1V1 = N2V2 (NaOH)  2 mol Al 
0.1 × V1 = 0.2 × 30 ⇒ V1 = 60 mL

Chapter-1.indd 26 8/4/2016 9:54:37 AM


Hints and Explanations 27

Since only 1.75 mol of Fe2O3 are supplied, it is the lim- But only 0.05 mol of H2SO4 is involved in the reaction.
iting reactant. This can be confirmed by calculating Therefore,
the amount of Al that would be required to react com- Number of moles of Na2SO 4 = 0.05 mol = 0.05 × 142 g = 7.10 g
pletely with all of the available Fe2O3
and
 2 mol Al  Number of moles of Na2SO 4 0.05 mol
mol Al = (1.75 mol Fe2O3 )  3.50 mol Al Molarity = =
 1 mol Fe2O3  Liters of solution 2L

Since an excess (4.20 mol – 3.50 mol = 0.70 mol) of Al is = 0.025 mol L−1
present, Fe2O3 must be the limiting reactant, as deter- 44. (a), (d) 2 PF3(g) + XeF4(s) → 2 PF5(g) + Xe(g)
mined above.
1 mol PF3 2 mol PF5
 2 mol Fe   55.847 g Fe  100.0 g PF3 × × = 1.137 mol PF5
g Fe = (1.75 mol Fe2O3 )    87.968 g 2 mol PF3
 1 mol Fe2O3   1 mol Fe 
1 mol XeF4 2 mol PF5
= 195 g Fe 50.0 g XeF4 × × = 0.482 mol PF5
207.28 g 1 mol XeF4
39. (c) XeF4 produces fewer moles of PF5; therefore, it is the limiting
 1 mol P4 O10   4 mol P  reagent and 0.482 mol of PF5 would be produced.
(15.95 g P4 O10 ) ×   ×  1 P O  = 0.2247 mol
 283.886 g P O
4 10 4 10 45. (a), (b)
(a) 20 g of NaOH in 200 mL of solution
Level II Weight required × 1000 20 × 1000
0
M= = M
Molecular weight × Volume required 40 × 200
Multiple Correct Choice Type
= 2.5 M
40. (a), (d)
(a) Law of conservation of mass: During any chemical change, (b) 0.5 mol of KCl in 200 mL of solution
the total mass of the products remains equal to the total Number of moles 0.5
M= = = 2.5 M
mass of the reactants. Volume of solution 0.2 L
(b) Law of definite proportion: A chemical compound always (c) 40 g of NaOH in 100 mL of solution
contains same elements combined together in same propor- Weight required × 1000 40 × 1000
0
tion by mass. M= = = 10 M
Molecular weight × Volume required 40 × 100
(c) Law of multiple proportions: When two elements combine (d) 20 g KOH in 200 mL of solution
with each other to form two or more than two compounds, the Molecular weight of KOH = 56 g
masses of one of the elements that combine with fixed mass of
20 × 1000
the other bear a simple whole number ratio to one another. M= = 1.785 M
56 × 200
(d) Avogadro’s law: Equal volumes of all gases under similar
Hence, options (a) and (b) are the same concentration.
conditions of temperature and pressure contain equal num-
ber of molecules. 46. (b), (d) The reaction involved is
41. (a), (b), (d) 2Mg + O2 →
 2MgO
Number of equivalent of H2SO4 = Moles × n-factor = 1 × 2 = 2 2 × 24 g 32 g 80 g
0.5 × 32 g
Number of equivalent of Ca(OH)2 = 1 × 2 = 2 (neutralized) = 16 g
Number of equivalent of NaOH = 2 × 1 = 2 (neutralized)
(b) 1 g Mg is burnt in 0.5 g of O2. That is, 24 g of Mg is burnt in
Number of equivalent of NH3 = 2 × 1 = 2 (neutralized)
0.5 × 32 g of oxygen.
The required reactions are
24 g of Mg is burnt in 16 g of oxygen.
H2SO4 + Ca(OH)2 → CaSO4 + 2H2O Therefore, equivalent of MgO is equal to that of oxygen
2NaOH + H2SO4 → Na2SO4 + 2H2O = 0.5/8
(d) Moles of oxygen = Moles of magnesium = Moles of MgO
2NH3 + H2SO4 → 2NH+4 + SO2−
4
in the reaction
Therefore, 1 mol = 6.023 × 1023 atoms or molecules.
42. (a), (b) Molarity and normality are dependent on tempera-
ture, since molarity and normality involve the use of volume Passage Type
of solution. Molality and mole fraction do not depend on
47. (c) Therefore, 109% labeled oleum will contain 9 g H2O,
temperature.
40 g free SO3, 60 g H2SO4; as 109% means diluting 100 g
43. (b), (c) The reaction involved is of oleum gives 109 g of H2SO4.
40 g (free) SO3 = Moles of SO3 (free) = 0.5 = y
H2SO 4 + 2NaOH → Na2SO 4 + 2H2O 60 g (H2SO4) = 0.6122 mol H2SO4 = x
1 L, 0.1 M 1 L, 0.1 M
0.1 mol 0.1 mol Therefore, x2 + y2 = 0.25 + 0.37 = 0.62

Chapter-1.indd 27 8/4/2016 9:54:41 AM


28 Chapter 1 Some Basic Concepts of Chemistry

48. (c) 98 g H2SO4 contains 80 g SO3 (c) Zeroes to the left of the first non-zero digit in the number
80 × 60 are not significant. For example: 0.0025 has only 2 significant
60 g H2SO4 contains = = 48.98 g figures
98
% of combined SO3 in oleum = 48.98% 54. (a) ã q, t; (b) ã p; (c) ã r; (d) ã s
(p) 4480 mL of CO2 at STP
49. (d) In 100 g oleum sample,
1 mol
SO3 + H2O → H2SO 4 × 4480 mL = 0.2 mol
0.5 mol
22400 mL
Therefore, SO3 = 0.5 mol = 40 g (q) 0.1 g atom of iron = 0.1 mol
Weight of SO3 = 40 g (r) 1.5 × 1023 molecules of oxygen gas
Weight of H2SO4 = 60 g
1 mol
% of SO3 = 40% × 1.5 × 1023 = 0.25 mol
% of H2SO4 = 60% 6 × 1023
(s) 9 mL H2O
50. (b) 1 mol of alkali metal carbonate would require 2 mol of
HCl. As 1 mol
× 9 mL = 0.5 mol
18 mL H2O
M2CO3 + 2HCl → MCl2 + H2O + CO2
(t) 200 mg hydrogen gas
Hence, number of equivalents of each metal carbonates
= 0.0111 1 mol
× 0.2 g = 0.1 mol
So, number of moles of metal carbonates = 0.0055 2g
as n-factor = 2.
Hence, number of equivalents of HCl required to neu- Integer Type
tralize metal carbonates = 0.0222.
Since alkali metal carbonates are equimolar, the num- 196
55. (2), (6); n = =2
ber of equivalents of HCl required to neutralize each 98
metal carbonates = 0.0222/2 = 0.0111.
Equivalents = Moles × n-Factor = 2 × 3 = 6
51. (d) Number of equivalents of acid is equal to
56. (4) From the reaction, we have 1 mol of CO reacts with 2 mol
44.4
N × V (L) = 0.5 × = 0.0222 of H2; so, 2 mol of CO will react with 2 × 2 = 4 mol of H2.
1000
1 mol
57. (100) Ca(HCO3 )2 
→ CaCO3 + H2O + CO2 ↑
Matrix-Match Type 1 mol 1 mol

52. (a) ã p, r, s; (b) ã r; (c) ã q, r; (d) ã r, s Hence, the yield is 100%


Molality, and mole fraction, and normality are independent
58. (4) 6.023 × 1023 has four significant figures. The exponential
of temperature because all these involve weight, which does
term does not add to the number of significant figures.
not depend on temperature.
On diluting a solution, all concentrations change. w 24 g
59. (1) n = = = 1 mol
Volume affects the concentration of molarity and Molecular weight 24 g mol−1
normality. 60. (1) By equating equivalents, we get
53. (a) ã q; (b) ã r; (c) ã p; (d) ã s 4.48
(a), (d) Zeroes in between non-zero digit are significant. × 1000 = 100 × M × 2 ⇒ M = 1 M
22.4
For example: 6.201 has 4 significant figures. Also, 204 has 3
significant figures. CH3COOH CH O
(b) If a number ends in zero, which is not to the right of a 61. (4) Empirical formula = or 2 4 2 = CH2O
2 2
decimal, the zero may or may not be significant. For example:
100 = 3 significant figures The number of atoms in empirical formula is four.

Chapter-1.indd 28 8/4/2016 9:54:45 AM


Solved JEE 2016 Questions 29

SOLVED JEE 2016 QUESTIONS


JEE Main 2016 2. The volume of 0.1 N dibasic acid sufficient to neutralize 1 g of a
base that furnishes 0.04 mol of OH− in aqueous solution is
1. The amount of arsenic pentasulphide that can be obtained
(a) 400 mL (b) 600 mL (c) 200 mL (d) 800 mL
when 35.5 g arsenic acid is treated with excess H2S in the pres-
ence of conc. HCl (assuming 100% conversion) is (Online)
(a) 0.25 mol (b) 0.50 mol
Solution
(c) 0.333 mol (d) 0.125 mol
(Online) (a) On applying law of equivalence, we have
N1V1 = N2V2
Solution
0.1 × V1 = 0.04 × 1
(d) The reaction is
H2 S / HCl
H3AsO 4  → As2S5 V = 0.4 L or 4 00 mL

According to the principle of conservation of atoms


1× nH3 AsO4 = 2 × nAs2 S5
35.5
= 2 × nAs2 S5
142
nAs2 S5 = 0.125 mol

Chapter-1.indd 29 8/4/2016 9:54:46 AM


Chapter-1.indd 30 8/4/2016 9:54:46 AM
2 Atomic Structure

Question Distribution in JEE (Main and Advanced)

3
No. of Questions

JEE (Main)
2
JEE (Adv)

0
2016 2015 2014 2013 2012 2011 2010 2009 2008 2007

Concept Distribution in JEE (Main and Advanced)


Topics Covered
Year
JEE (Main) JEE (Advanced)
2007 Quantum Numbers and Rules for Filling Electrons
2008 Bohr’s Atomic Model Quantum Mechanical Model of Atom
2009 Dual Nature of Matter
Bohr’s Atomic Model, Quantum
2010 Planck’s Quantum Theory, Bohr’s Atomic Model
Mechanical Model of Atom
Quantum Mechanical Model of Atom,
2011 Nature of Electromagnetic Radiation
Photoelectric Effect
2012 Quantum Numbers and Rules for Filling Electrons Bohr’s Atomic Model
2013 Dual Nature of Matter Dual Nature of Matter
Nature of Electromagnetic Radiation, Photoelectric Effect,
Quantum Numbers and Rules for Filling
2014 Bohr’s Atomic Model, Dual Nature of Matter, Quantum
Electrons
Numbers and Rules for Filling Electrons
Bohr’s Atomic Model, Quantum Numbers and Rules for
2015 Bohr’s Atomic Model
Filling Electrons
Dual Nature of Matter, Quantum Numbers and Rules for
2016 Quantum Mechanical Model of Atom
Filling Electrons

Chapter-2.indd 31 8/4/2016 12:07:02 PM


32 Chapter 2 Atomic Structure

SUMMARY
1. Properties of subatomic particles
Mass
Particle Electric charge (C) Symbol Discoverer Location
Kilograms (kg) Atomic mass units (u)
Electron −1.60 × 10−19 9.109 382 × 10−31 5.485799 × 10−4 e− J. J. Thomson Outside the nucleus
Proton +1.60 × 10−19 1.672 622 × 10−27 1.007276 p Goldstein Inside the nucleus
Neutron 0 1.674 927 × 10−27 1.008665 n Chadwick Inside the nucleus

2. Atomic models
(a) Thomson model of atom
(i) In this model of the atom, the electrons are negatively charged particles embedded in the atomic sphere of approximate
radius 10−10 m.
(ii) The sphere also contains an equal number of positive charges to make atom electrically neutral.
(iii) The positive charge was assumed to be spread throughout the atom, forming a kind of pudding in which the negative elec-
trons were suspended like plums.
Limitation
• The model could not account for the distribution of mass.
(b) Rutherford’s nuclear model of atom
(i) The positive charge and most of the mass of the atom is concentrated in a small region of the atom and vast majority of the
volume of an atom is, therefore, empty space.
(ii) The electrons surround the nucleus and move around the nucleus in circular paths called orbits.
(iii) The electrons and nucleus are held together by the electrostatic forces.
Limitations
• It could not explain the stability of the atom.
• It was not able to explain the line spectra for various elements.
• This model was unable to explain the energies of electrons and their distribution around the nucleus.

3. Representation of atom with electrons and nucleons


(a) The number of protons in the nucleus (which is equal to the total positive charge) is the atom’s atomic number (Z). It is also equal
to the number of electrons in the neutral atom.
(b) The total number of nucleons (protons and neutrons) is called the mass number (A). An atom can be represented as ZA X where
X represents the symbol of the element, and A and Z represent the mass number and atomic number, respectively.
(c) The atoms which have the same mass number but different atomic numbers are called isobars. For example, N–14 and C–14.
(d) The isotopes of an element have the same atomic number but different mass numbers. Carbon, for example, has three naturally
occurring isotopes: 12C, 13C and 14C.
(e) Atoms of different elements that contain same number of neutrons are called isotones. They differ in their atomic number and
mass number. For example,146 C, 157 N, 188 O.
(f) Atoms or ions containing same number of electrons are called isoelectronic species. For example, F−, Na+, Mg2+, Al3+, Ne.

Tip The chemical properties are dependent on the number of electrons which in turn depends on the number of protons. The
number of protons in the nucleus of an atom determines the identity of the atom. So, all isotopes show similar chemical properties.
As a result, all atoms of an element must have the same number of protons. But they do not have to contain the same number of
neutrons.

4. Wave nature of electromagnetic radiation


According to the electromagnetic wave theory, energy is emitted from any source continuously and consists of electric and magnetic
fields oscillating perpendicular to each other and also perpendicular to the direction of propagation.
Characteristics of electromagnetic
Symbol Definition S.I. unit
radiation
The distance between two maxima of either electrical or
Wavelength l magnetic components of the wave (minima to minima is m
also same)
Frequency n The number of waves that pass a fixed point in unit time s−1 or Hz
(Continued)

Chapter-2.indd 32 8/4/2016 12:07:02 PM


Summary 33

(Continued)

Wave number n The wave number n = 1/l is the reciprocal of the wave- meters (m−1)
length
The energy associated with regions of the electromagnetic
spectrum is related to wavelengths and frequency by the
hc
Energy E equation: E = hn = Joules (J)
l
where E = energy of the radiation in joules; h = Planck’s
constant, 6.626 × 10−34 J s; ν = frequency of radiation in Hz
and c = velocity of light, 2.998 × 108 ms−1.

c
Tip Wavelengths, in meters, are related to the frequency, v, in Hertz (cycles per second) by the equation n = .
l

5. Particle nature of electromagnetic radiation


German physicist, Max Planck proposed that electromagnetic radiation can be viewed as a stream of tiny packets or quanta of
energy called photons.
The electromagnetic theory of light was effective in explaining a number of experimental observations such as diffraction and
interference of light. However, the following observations which could not be explained on the basis of electromagnetic theory:
(a) Blackbody radiation
(b) Photoelectric effect
(c) Line emission spectra (for hydrogen atom).
(d) Variation in heat capacity of solids with temperature.

6. Blackbody radiation
A body which can emit or absorb radiation of all frequencies is known as blackbody and the radiation emitted by it is known as the
blackbody radiation.

7. Photoelectric effect
It is the phenomenon in which electrons are emitted from a metal surface when radiation of sufficient energy falls on it.
Einstein’s photoelectric theory
An electron absorbs a quantum of energy (hv) from light and if the energy absorbed is greater than the minimum energy (hv0)
required by an electron to escape from the metal surface (known as the work function W0), then the electron is ejected from the
metal surface with some kinetic energy.
1
hv = K.E. + hv 0 = mv 2 + hv 0
2

K.E. = hv − W0

where K.E. is the kinetic energy of the electron that is emitted, hv is the energy of the photon of frequency v and hv0 is the minimum
energy needed to eject the electron from the metal’s surface.

8. Planck’s quantum theory


(a) This theory states that energy is not continuous but is discrete. The smallest quantity of this energy is called quantum.
(b) The energy can be given as E = hv. This means that energy occurs in “packets” called quanta, of magnitude h/2p, where h is
Planck’s constant and its value is 6.626 × 10−34 J s.
(c) With this theory, Planck could explain the distribution of intensity of radiation emitted by a blackbody as a function of wave-
length or frequency at a given temperature.

9. Dual behaviour of electromagnetic radiation


(a) The phenomenon of diffraction and interference of light could be explained only by assuming the wave nature of light.
(b) Phenomenon such as blackbody radiation and photoelectric effect could only be explained by the particle nature of radiation.
This proved that light possess both the wave nature and the particle nature (observed when it strikes a metal).

10. Atomic Spectra


A spectrum is formed when the light from an object that has been heated to a very high temperature is split by a prism and dis-
played on a screen.

Chapter-2.indd 33 8/4/2016 12:07:05 PM


34 Chapter 2 Atomic Structure

(a) Continuous spectrum


A spectrum (ranging from 7.5 × 1014 Hz for violet to 4 × 1014 Hz for red) containing a continuous unbroken distribution of light
of all colours. The red light with the longest wavelength deviates the least, while the violet light with the shortest wavelength
deviates the most.
(b) The spectrum of radiation emitted by a substance that has adsorbed electromagnetic radiation is known as an emission
spectrum.
(c) When the radiation is passed through a sample, it absorbs certain wavelengths. When the continuous spectrum is observed,
there are a few dark spaces which represent the radiation that is absorbed by the matter. This is called absorption spectrum.
(d) Discontinuous spectrum
When a narrow beam of this light is passed through a prism, we do not see a continuous spectrum. Instead, only a few colours are
observed, displayed as a series of individual lines. This series of lines is called the element’s atomic spectrum or emission spec-
trum. The emission spectra of visible light are continuous, but that of gaseous atoms contain lines with dark spaces between
them. This is called line spectra because the light corresponding to the individual emissions appears as lines on the screen.

Tips
(a) Spectroscopy is a study of either emission or absorption spectra.
(b) Atomic spectroscopy is an important technique for studying the energy and the arrangement of electrons in atoms. Each
element has its own unique atomic spectrum that is its characteristic fingerprint, and can be used to identify the element.

11. Line spectra of hydrogen


The appearance of spectral lines in the emission spectrum of hydrogen, four lines can be seen by eye, but many more are observed
photographically in the ultraviolet region. The lines become increasingly close together as the wavelength (l) decreases, until the
continuum is reached.
 
Balmer formula: n = R  12 − 12 
2 n 
where R is the Rydberg constant and n has the values 3, 4, 5… thus giving a series of lines. The series of lines given by this formula
is called the Balmer series which appear only in the visible region of the electromagnetic spectrum.
 1 1
The formation of lines pertaining to different values of n is given by: n = 109677  2 − 2  cm−1
 n1 n2 
where n1 = 1, 2, … and n2 = n1 + 1, n1 + 2, …. The value of Rydberg constant is 109677 cm−1.
Spectral series found in atomic hydrogen
Series Region of spectrum Equations

1 1
Lyman series Ultraviolet Lyman n = R  2 − 2  n = 2, 3, 4 , 5,…
1 n 

Balmer series Visible Balmer  1 1


n = R  2 − 2  n = 3, 4 , 5, 6 ,…
2 n 

 1 1
Paschen series Infrared Paschen n = R  2 − 2  n = 4 , 5, 6 , 7,…
3 n 

 1 1
Brackett series Infrared Brackett n = R  2 − 2  n = 5, 6 , 7, 8 ,…
4 n 

 1 1
Pfund series Infrared Pfund n = R  2 − 2  n = 6 , 7, 8 , 9 ,…
5 n 

Tip The limiting line of any spectral line in hydrogen spectrum is the line when n2 = ∞. The limiting line will have the shortest
wavelength and the largest wave number.

12. Bohr’s model for hydrogen atom


(a) The electron moves around the nucleus in orbits (arranged concentrically around the nucleus). These orbitals have a fixed
radius and energy. An electron does not radiate energy if it stays in one orbit, and therefore does not slow down.
(b) When an electron moves from one orbit to another it either radiates or absorbs energy. If it moves towards the nucleus, energy
is radiated and if it moved away from the nucleus, energy is absorbed.

Chapter-2.indd 34 8/4/2016 12:07:07 PM


Summary 35

(c) The difference in energy observed when the frequency of radiation is absorbed or emitted is given by the Bohr frequency rule
∆E E2 − E1
as follows: n = = .
h h
nh
(d) When an electron remains in its orbit or stationary state, the angular momentum is given by mevr = .
where n = 1, 2, 3, …. 2p
(e) The integral numbers n = 1, 2, 3, …, are the principal quantum numbers and correspond to the permitted stationary states for
the electron in an atom.
∈ n2h2
(f) Radius of Bohr orbits in hydrogen r = 0 2
p me Z
Bohr radius can be also be expressed as rn = n2a0, where a0 = 52.9 pm.
nh Ze2
(g) Velocity of electron: v = =
2p me r 2 ∈0 nh
− Z 2m e 4
(h) Energy of electron in hydrogen atom: E total = 2 2e 2
8n h ∈0
−13.6  1
(i) Energy of electron: E = 2 eV En = − RH  2  n = 1, 2, 3
n n 
where RH is the Rydberg constant for hydrogen and its value is 2.18 × 10−18 J.
(j) Energy change for an electron undergoes a transition from initial state ni to final state nf is given by
 1 1 
∆E = E2 − E1 = 2.18 × 10 −18  −  J atom
−1
 (nf ) (ni )2 
2

Tip When an electron moves from one orbit to another, it should give a single sharp line in the spectrum, corresponding precisely to
the energy difference between the initial and final orbits. If the hydrogen spectrum is observed with a high resolution spectrometer, it
is found that some of the lines reveal “fine structure”. This means that a line is really composed of several lines close together.

13. Bohr model for hydrogen-like species


Bohr’s theory can also be applied to study one electron systems such as He+, Li2+, Be3+, etc., which are known as hydrogen-like
species.
 − Z 2Rhc   2
−18 Z
(a) The energy of the electron in any one-electron system is given by E =  2  = −2.18 × 10  2  J
 n  n 
52.9(n2 )
(b) The radii of hydrogen-like species is given by the expression rn = pm
Z
(c) Ionization enthalpy is the energy required to remove the electron from the atom in gaseous state and convert it into an ion. It
amounts to a transition of electron from ground state n = 1 to n = ∞ on absorption of energy equal to ionization enthalpy. The
ionization enthalpy of hydrogen and hydrogen-like species is given as:

2.18 × 10 −18
En = − J atom−1 (for hydrogen)
n2
2.18 × 10 −18
En = − Z 2 J atom−1 (for hydrogen-like species)
n2
(d) Limitations of Bohr’s model
(i) It accounts for the line spectra of only one-electron systems such as H, He+ and Li2+.
(ii) It does not explain splitting of spectral lines by magnetic fields (Zeeman effect) and electric fields (Stark effect).
(iii) The energy of the spin of electron is not taken into account in Bohr’s theory.
(iv) Bohr assumes that the angular momentum of an electron in an atom of hydrogen is an integral multiple of h/2p but his
theory does not explain the reason for this assumption.
(v) Using this theory, we are able to find the position of an electron and its velocity in an atom of hydrogen. This contradicts with
Heisenberg’s uncertainty principle.
(vi) It does not take into account the wave nature of the electron.
14. Bohr–Sommerfeld model
(a) Orbits containing electrons revolving around the nucleus to be elliptical instead of being circular with definite energy levels.
(b) It described the atom using two quantum numbers instead of one, that is, n and k where k is called azimuthal quantum number.
It is equal to 1, 2,…n but k ≠ 0.

Chapter-2.indd 35 8/4/2016 12:07:10 PM


36 Chapter 2 Atomic Structure

(c) It stated that the angular momentum of electron in closed elliptical paths is quantized, that is Lk = k(h/2p)
(d) The ratio of n to k is numerically equal to the ratio of the length of major axis to that of minor axis. On increasing k, the path
becomes more eccentric, whereas if n/k = 1, then the path is circular.
Limitations:
• It was restricted to one-electron systems only, and could not explain the spectrum and electron–electron repulsions of pol-
yelectron systems.
• It was unable to explain the de Broglie equation, even though it indicated that momentum in closed elliptical paths is
quantized.
15. Dual nature of matter
(a) de Broglie equation
According to de Broglie, matter also exhibits wave like behaviour. The wavelength of a matter wave, l is given by the equation
h h
l= =
mv p
where h is Planck’s constant, m is the particle’s mass, v is its velocity and p is the momentum.
If the charged particle (electron) is accelerated with a potential of V, then the kinetic energy is given by:
1 2
mv = eV ⇒ m2v 2 = 2eVm ⇒ mv = 2eVm
2
Then from de Broglie’s relation, we have
h h
l= =
mv 2eVm
For any charged particle of charge q,
h
l=
2qVm
de Broglie wavelengths for subatomic particles
Particle Electron Proton Neutron `-particle Gas molecule

1.227 0.0286 0.0286 0.0101 h h


de Broglie wavelength l= nm l= nm l= nm l = nm l= =
V V E V m × vrms 3mkT

(b) Heisenberg’s uncertainty principle


It is impossible to measure precisely both the position and momentum of a particle accurately at the same time, the uncertainty
principle may be expressed mathematically as:
h
∆x ⋅ ∆p x ≥
4p
h h
or ∆x ⋅ ∆( mv x ) ≥ ⇒ ∆x ⋅ ∆v x ≥
4p 4p m
where ∆x is the uncertainty in defining the position and ∆px (or ∆vx) the uncertainty in the momentum/velocity, h = Planck’s
constant = 6.626 × 10−34 J s.
16. Quantum mechanical model of atom
This was proposed to overcome the theoretical limitations in predicting the exact position of an electron at a particular instant.
Postulates of quantum mechanics were developed independently by Heisenberg and Schrödinger.
(a) The fundamental equation of wave mechanics is the Schrödinger wave equation which provides a satisfactory description of an
atom in these terms of duality of wave and particle.
Ĥy = Ey
Here Ĥ is a mathematical operator called Hamiltonian.
(b) Solutions to the wave equation are called wave functions and given by the symboly. The solution gives the probability of
finding an electron at a point in space whose coordinates are x, y and z is y 2(x, y, z) and the energy associated with it.
An electron may move in three directions x, y and z so the Schrödinger wave equation is given as:
∂2y ∂2y ∂2y 4p 2
2
+ 2
+ 2
=− y
∂x ∂y ∂z l2
4p 2
or, ∇2y = − y
l2
(Using the symbol ∇ instead of the three partial differentials.)
h2
(c) Hamiltonian operator is given by Hˆ = ∇2 + V
8p 2 m

Chapter-2.indd 36 8/4/2016 12:07:16 PM


Summary 37

(d) Hydrogen atom and the Schrödinger equation


The solution to the Schrödinger’s wave equation for hydrogen atom gives the possible energy level that the electron may
occupy and the corresponding wave functiony.

8p 2 m
∇2y + (E − V )y = 0
h2
Acceptable solutions to the wave equation, that is, solutions which are physically possible, must have certain properties:
(i) y must be continuous; (ii) y must be finite; (iii) y must be single valued.
+∞
∫−∞ y
2
(e) The probability of finding an electron at a point x, y, z over all space: dx dy dz = 1.

8p 2 m
(f) The solution of the equation ∇2y + (E − V )y = 0 is of the form y = R(r ) ⋅ Θ(q ) ⋅ Φ(f ) which can be rewritten as
y = R(r )nl ⋅ Aml h2

where R(r) the radial function, which depends on the quantum numbers n and l and Aml the total angular wave function, which
depends on the quantum numbers m and l.
(g) Important features of quantum mechanical model
(i) The energy of electrons in an atom is quantized.
(ii) All the information about a particle (electron) in a given energy level is contained in the wave function y.
(iii) The probability of finding an electron in an atom is proportional to |y |2 .
(iv) The wave function and its square |y |2 have values for all locations about the nucleus. The probability of finding the elec-
tron is the maximum near the nucleus and decreases as the distance from nucleus increases. The value becomes extremely
small with increasing distance but at no point, it becomes zero.
(v) In multi-electron atoms, the electrons are filled in various orbitals in increasing order of energy.
(vi) For a given type of atom, there are a number of solutions to the wave equation which are acceptable, and each orbital may
be described uniquely by a set of three quantum numbers, n (principal quantum number), l (azimuthal quantum number)
and ml (magnetic quantum number).
17. Quantum numbers
(a) The quantum numbers specify the shape and direction of the orbital in which the probability of finding the electron is the high-
est. The set of four quantum numbers are principal quantum number (n), angular quantum number (l) and magnetic quantum
number (ml) and spin quantum number (ms).
Information obtained from four quantum numbers

Principal quantum number (n) Azimuthal quantum number (l) Magnetic quantum number (ml) Spin quantum number (ms)

The number of orbitals present in


The main shell in which the The number of subshells present
each subshell or the orientation
electron resides in the nth shell
of the subshells

The energy of the orbital

Approximate distance of the


Shape of the orbitals
electron from the nucleus
For a given value of n, l can have val-
The maximum number of elec- For a particular value of m,
ues from 0 to n − 1. In each subshell, For a given value of l, m = −l to +l
trons present in the shell we have ms = +1/2 or −1/2
there are (2l + 1) types of orbitals
l = 0, s subshell For s subshell: s
Represented by two arrows
Represented by integers 1, 2, l = 1, p subshell For p subshell: px, py , pz
pointing in opposite direc-
3,… or K, L, M, N, etc. l = 2, d subshell For d subshell: dxy, dyz, dxz, d z 2 ,
tions, +1/2 and −1/2
l = 3, f subshell dx2 − y2
Accounts for the main lines in Accounts for the fine lines in the Accounts for the splitting of lines Accounts for magnetic prop-
the atomic spectrum atomic spectra in the atomic spectrum erties of substances

(b) Selection rules governing allowed combinations of quantum numbers


(i) The three quantum numbers (n, l and ml) are all integers.
(ii) The principal quantum number (n) cannot be zero. The allowed values of n are therefore 1, 2, 3, 4, ….
(iii) The angular quantum number (l) can be any integer between 0 and n − 1. If n = 3, for example, l can be either 0, 1 or 2.
(iv) The magnetic quantum number (ml) can be any integer between −l and +l. If l = 2, ml can be −2, −1, 0, 1 or 2.

Chapter-2.indd 37 8/4/2016 12:07:19 PM


38 Chapter 2 Atomic Structure

18. Shapes of atomic orbitals


The radial function R has no physical meaning, but R2 gives the probability of finding the electron in a small volume dV near the
point at which R is measured. For a given value of r, the number of small volumes is 4πr2, so the probability of the electron being at
a distance r from the nucleus is 4πr2R2. This is called the radial distribution function.
Node
8 3 3
4p r 2R 2(r )

4p r 2R 2(r )

4p r 2R 2(r )
2 2
4
1 1
1s 2s 2p
0 0 0
5 10 5 10 5 10

Tip Node is a point where probability of finding an electron is zero.

Number of peaks or maxima Number of spherical nodes Number of angular node


Type of orbital
(n − l) (n − l − 1) or nodal plane (l)
s n n− 1 zero
p n− 1 n− 2 1
d n− 2 n− 3 2
f n− 3 n− 4 3
(a) All s orbitals are spherical but their sizes increase with increasing n. All the s orbitals except the first one (1s) have a shell-like
structure, consisting of concentric layers of electron density.
z

+
x

y
1s
(b) In p orbitals the electron density is equally distributed in two regions on opposite sides of the nucleus. There are thus two
“lobes” of one p orbital. Between the lobes is a nodal plane where electron density of zero. The size of the p orbitals also
increases with increasing n. A p subshell consists of three orbitals of equal energy with each orbital have their maximum elec-
tron densities oriented at 90° to each other, corresponding to the axes of an imaginary xyz coordinate system. The individual p
orbitals they are often labeled as px, py and pz.
z z z

+ +
− + x − x x

y y y
2px 2py 2pz
(c) There are five orbitals in a d subshell.
(i) Four of the five d orbitals have the same shape and consist of four lobes of electron density. These four d orbitals each have
two perpendicular nodal planes that intersect the nucleus. These orbitals differ only in their orientations around the nucleus
(their labels come from the mathematics of wave mechanics).
y z z y

+
− + − + − +

x x x x

+ − + − + −
+

3dxy 3dxz 3dyz 3dx 2− y 2

Chapter-2.indd 38 8/4/2016 12:07:22 PM


Summary 39

(ii) The fifth d orbital, labeled dz 2 , has two lobes that point in opposite directions along the z axis plus a doughnut-shaped ring
of electron density around the center that lies in the xy plane. The two nodes for the dz 2 orbital are conic surfaces whose
peaks meet at the nucleus.
z

−− x

3dz 2

19. Rules for filling of orbitals in atom


(a) Aufbau principle
(i) In the case of atoms, electrons occupy the available orbitals in the subshells of lowest energy. This is known as the Aufbau
principle which determines the assignment of all the electrons in an atom into specific shells and subshells is known as the
element’s electronic configuration.
(ii) The order of energy levels in case of hydrogen is 1s < 2 s = 2 p < 3s = 3 p = 3d < 4 s = 4 p = 4 d = 4 f .
The order of filling of energy levels other than H atom is: 1s, 2s, 2p, 3s, 3p, 4s, 3d, 4p, 5s, 4d, 5p, 6s, 4f, 5d, 6p, 7s, etc.

1s

2s 2p

3s 3p 3d

4s 4p 4d 4f

5s 5p 5d

6s 6p 6d

7s

(iii) The energy of the orbital is given by (n + l) rule. Lower the (n + l) value for an orbital, lower is the energy. If the value of (n + l)
for two orbital is same, then the orbital having higher value of n has higher energy.
(b) Pauli’s exclusion principle
(i) This states that no two electrons in one atom can have all four quantum numbers the same.
(ii) By permutating the quantum numbers, the maximum number of electrons which can be contained in each main energy
level can be calculated.

Subshell Number of orbitals Maximum number of electrons


s 1 2
p 3 6
d 5 10
f 7 14
(c) Hund’s rule of maximum multiplicity
It states that pairing of electrons in orbitals of p, d and f subshells does not take place till each orbital belonging to that subshell
has got one electron each.
20. Electronic configuration of atoms
Electronic configuration of atoms is the distribution of electrons into atomic orbitals. When atoms are in their ground state, the
electrons occupy the lowest possible energy levels.

Chapter-2.indd 39 8/4/2016 12:07:24 PM


40 Chapter 2 Atomic Structure

(a) The elements from scandium to copper (Note: Cr and Cu have only one electron in the 4s orbital.) have two electrons in the 4s
level and an incomplete 3d level, and all behave in a similar manner chemically. Such a series of atoms is known as a transition
series. A second transition series starts after the 5s orbital has been filled, at strontium, because in the next element, yttrium,
the 4d level begins to fill up. A third transition series starts at lanthanum where the electrons start to fill the 5d level after the 6d
level has been filled with two electrons.
(b) A further complication arises here because after lanthanum, which has one electron in the 5d level, the 4f level begins to fill up,
giving the elements from cerium to lutetium with one to 14f electrons. These are sometimes called the inner transition ele-
ments, but are usually known as the lanthanoids or rare earth metals.
(c) The distribution of electrons in the outer shell or the valence shell of the atom (the occupied shell with the largest value of n)
are called valence electrons. These are the ones that are exposed to other atoms when the atoms react. The inner electrons of
an atom, called the core electrons, are buried deep within the atom and normally do not play a role when chemical bonds are
formed.
21. Stability of completely filled and half-filled subshells
Half-filled and completely filled subshells (particularly the latter) have some special stability. The stability of half-filled and com-
pletely filled subshells can be attributed to the following:
(a) The half-filled and completely filled shells have symmetrical distribution of electrons, and therefore, are more stable as symme-
try is associated with greater stability.
(b) When two or more electrons are present in degenerate orbitals of a subshell with the same spin, they tend to exchange their
positions. The energy released during this exchange is known as the exchange energy.
(c) Electronic configurations with half-filled shells permit maximum number of exchanges between electrons of same spin, and
thus maximum exchange energy is released which contributes to the stability of the state.
For example, two important exceptions are for chromium and copper. Following the rules, we would expect the
configurations to be
Cr: [Ar] 3d4 4s2 Cu: [Ar] 3d9 4s2
However, the actual electronic configurations, determined experimentally, are
Cr: [Ar] 3d5 4s1 Cu: [Ar] 3d10 4s2
For chromium, an electron is “borrowed” from the 4s subshell to give to 3d subshell that becomes exactly half-filled. For copper
the 4s electron is borrowed to give a completely filled 3d subshell. A similar thing happens with silver and gold, which have
filled 4d and 5d subshells, respectively.
Ag: [Kr] 4d10 5s2 Au: [Xe] 4f14 5d10 6s2
Similar irregularities occur among the lanthanoid and the actinoid elements.
22. Magnetic properties
(a) The magnetic moment (m) can be measured from spin-only formula, which assumes that the magnetic moment arises entirely
from the unpaired electron spin.
(b) The spin magnetic moment is expressed as ms = n(n + 2)
where n is the number of unpaired electrons and the magnetic moment is expressed in Bohr Magneton.
(c) If the species has unpaired electrons, it is paramagnetic and if all the electrons are paired, it is said to diamagnetic.

TOOLS FOR PROBLEM SOLVING


c
1. Relation between frequency and wavelength: v =
l hc
2. The energy associated with regions of the electromagnetic spectrum is related to wavelengths and frequency: E = hn =
l
3. Kinetic energy of the ejected electron from the metal surface is given by
1
hn = K.E. + hn 0 = mv 2 + hn 0
2

K.E. = hn − W
2 2
∈0 n h
4. Radius of Bohr orbits in hydrogen r =
p me2 Z
Bohr radius can be also be expressed as rn = n2a0, where a0 = 52.9 pm.

nh Ze2
5. Velocity of electron: v = =
2p me r 2 ∈0 nh

Chapter-2.indd 40 8/4/2016 12:07:26 PM


Solved Examples 41

− Z 2 me e 4
6. Energy of electron in hydrogen atom: E total =
8n2h2 ∈20
−13.6  1
7. Energy of electron: E = eV En = − RH  2  n = 1, 2, 3,….
n 2 n 
8. Energy change for an electron undergoes a transition from initial state ni to final state nf ,
 1 1 
∆E = E2 − E1 = 2.18 × 10 −18  −  J atom
−1
 (nf ) (ni )2 
2

 − Z 2R 
9. Energy of the electron in any one-electron system is given by E =  2 
 n 
h h
10. de Broglie equation l = =
mv p h
11. Heisenberg’s uncertainty principle ∆x ⋅ ∆p x ≥
4p 8p 2 m
12. Solution to the Schrödinger’s wave equation for hydrogen atom: ∇2y + (E − V )y = 0
h2
13. Radial and angular wave functions
y = R(r ) ⋅ Θ(q ) ⋅ Φ(f )
y = R(r )nl ⋅ Aml
+∞

∫y
2
14. The probability of finding an electron at a point x, y, z over all space is = dxdydz = 1
−∞
15. Spin magnetic moment is expressed as ms = n(n + 2)
16. Summary of Relationships among the quantum numbers n, l, and ml
Value of n Value of l Value of ml Subshell Number of orbitals
1 0 0 1s 1
2 0 0 2s 1
  1 −1, 0, 1 2p 3
3 0 0 3s 1
  1 −1, 0, 1 3p 3
  2 −2, −1, 0, 1, 2 3d 5
4 0 0 4s 1
  1 −1, 0, 1 4p 3
  2 −2, −1, 0, 1, 2 4d 5
  3 −3, −2, −1, 0, 1, 2, 3 4f 7

SOLVED EXAMPLES
Nature of Electromagnetic Radiation (a) 8.041 × 10−40 J (b) 2.680 × 10−19 J
(c) 1.640 × 10−18 J (d) 6.111 × 10−17 J
1. A gas absorbs a photon of 355 nm and emits at two wave- (JEE Main Online 2014)
lengths. If one of the emissions is at 680 nm, the other is at Solution
(a) 325 nm (b) 743 nm (c) 518 nm (d) 1035 nm
(AIEEE 2011) (c) The energy is given by E = hv = 6.626 × 10−34 × 2.47 × 1015
= 16.366 × 10−19 J ≈ 1.64 × 10−18 J
Solution
(b) The wavelength of absorbed radiation is related to those 3. Ionization energy of gaseous Na atoms is 495.5 kJ mol−1. The
of emitted radiation as lowest possible frequency of light that ionizes a sodium atom
is (h = 6.626 × 10−34 J s, NA = 6.022 × 1023 mol−1)
1 1 1
= + (a) 7.50 × 104 s−1 (b) 4.76 × 1014 s−1
labsorbed l1 l2
(c) 3.15 × 1015 s−1 (d) 1.24 × 1015 s−1
1 1 1 (JEE Main Online 2014)
Therefore, = +
355 680 l2 Solution
Solving, we get l2 = 743 nm. (d) The ionization energy is given by E = NAhv
E = 495.5 × 103 = 6.022 × 1023 × 6.626 × 10−34 × n
2. Excited hydrogen atom emits light in the ultraviolet region at
2.47 × 1015 Hz. With this frequency, the energy of a single photon is: 495.5 × 103
n= = 1.24 × 1015 s −1
(h = 6.626 × 10−34 J s) 6.022 × 6.626 × 10 −11

Chapter-2.indd 41 8/4/2016 12:07:30 PM


42 Chapter 2 Atomic Structure

Planck’s Quantum Theory hc 6.626 × 10 −34 × 3 × 108 m s −1


E = hn = =
4. The energy required to break one mole of Cl Cl bonds in Cl2 l 560 × 10 −9 m
is 242 kJ mol−1. The longest wavelength of light capable of
Light energy required by the eye = 2.5 × 10−7 J
breaking Cl–Cl bonds is (c = 3 × 108 m s−1 and NA = 6.02 × 1023
mol−1) Total energy required
Number of photons =
(a) 494 nm (b) 594 nm (c) 640 nm (d) 700 nm Energy of one photon
(AIEEE 2010)
2.5 × 10 −7 J
Solution =
6.626 × 10 −34 J s × 3 × 108 m s −1
(a) Energy required to break Cl2 molecule = (242 × 103)/NA J
c 560 × 10 −9 m
From E = hn ⇒ E = h
l = 7.04 × 1011
We have
8. A 1-kW radio transmitter operates at a frequency of 880 Hz.
6.626 × 10 −34 × 3 × 108 × 6.022 × 1023
l= = 494.5 nm How many photons per second does it emit?
242 × 103
(a) 1.71 × 1021 (b) 1.71 × 1033
5. Calculate the energy of a single photon of red light with a (c) 6.02 × 1023 (d) 2.85 × 1028
wavelength of 700.0 nm.
Solution
(a) 2.838 × 10 −19 J (b) 3.024 × 10 −18 J
(b) For 1-kW radio transmitter, the energy is given by
(c) 2.325 × 10 −19 J (d) 3.452 × 10 −18 J E = nhv = 1000 J s −1
Solution Given that n = 880 s −1 and h = 6.62 × 10 −34 J s
(a) The red light with a wavelength of 700.0 nm has a fre- Substituting in the above equation, we get the number
quency of 4.283 × 1014 s−1. This can be calculated as of photons emitted per second
c 3.0 × 108 1000
n= = = 4.286 × 1014 s −1 n=
l 700 × 10 −9 (880 s −1) × (6.62 × 10 −34 J s)
Substituting this frequency into the Planck–Einstein 1000
= = 1.717 × 1033
equation gives the following result in units of joules (J). 5.831× 10 −31

E = hn = (6.626 × 10 −34 J s)(4.286 × 1014 s −1)


Photoelectric Effect
= 2.84 × 10 −19 J
9. Photoelectric emission is observed from a surface for fre-
6. A 100 W bulb emits monochromatic light of wavelength 400 nm. quencies n1 and n2 of the incident radiation (n1 > n2). If the
Calculate the number of photons emitted per second by the maximum kinetic energies of the photoelectrons in the two
bulb. cases are in the ratio 1:k, then the threshold frequency n0 is
(a) 2.695 × 10 −20 s −1 (b) 3.201× 10 −20 s −1 given by
(c) 3.358 × 10 −20 J (d) 2.012 × 1020 s −1 n −n kn1 − n 2
(a) 2 1 (b)
k −1 k −1
Solution
kn 2 − n1 n 2 − n1
(d) Given that power of the bulb = 100 W = 100 J s−1. (c) (d)
k −1 k
We know that, energy of one photon Solution
6.626 × 10 −34 Js × 3 × 108 ms −1 (b) We have hv = hv 0 + K.E.
E = nhn = = 4.969 × 10 −19 J
400 × 10 −9 m Therefore, hv1 = hv 0 + (K.E.)1 and hv 2 = hv 0 + (K.E.)2.
In terms of (K.E.)1 and (K.E.)2, we have
Number of photons emitted
(K.E.)1 = h(v1 − v 0 )
100 J s −1
= = 2.012 × 1020 s −1 (K.E.)2 = h(v 2 − v 0 )
4.969 × 10 −19 J
Dividing both equations, we have
7. Calculate the number of photons of green light with wave- (K.E.)1 h(v1 − v 0 ) (v1 − v 0 )
length 560 nm that can be observed by human eye. Note that = =
(K.E.)2 h(v 2 − v 0 ) (v 2 − v 0 )
2.5 × 10−7 J of light energy is required by the interior part of
human eye. (K .E.)1 1
Since, = , so
(a) 7 × 109 (b) 7 × 1013 (K .E.)2 k
(c) 7 × 1011 (d) 7 ×106 1 (v1 − v 0 )
=
k (v 2 − v 0 )
Solution
kv1 − v 2
(c) Energy of one photon, Solving, we get v 0 =
k −1

Chapter-2.indd 42 8/4/2016 12:07:39 PM


Solved Examples 43

10. The work function for a metal is 4 eV. To emit a photo electron 5h 2.5h
Since n = 5, we get mvr = =
of zero velocity from the surface of the metal, the wavelength 2p p
of incident light should be
14. The ionization enthalpy of hydrogen atom is 1.312 × 106 J mol−1.
(a) 2700 Å (b) 1700 Å (c) 5900 Å (d) 3100 Å
The energy required to excite the electron in the atom from
Solution n = 1 to n = 2 is
(d) hv = hv 0 + K.E. (a) 8.51 × 105 J mol−1 (b) 6.56 × 105 J mol−1
For zero velocity K.E. = 0 where hv0 is work function. So, (c) 7.56 × 105 J mol−1 (d) 9.84 × 105 J mol−1
(AIEEE 2008)
hv = hv 0 .
Solution
Given that hv = 4 eV. Therefore,
hc h × c 6.626 × 10 −34 × 3 × 108 (d) The energy for stationary states n = 1 and n = 2 are,
= 4 eV ⇒ l = = = 3100 Å respectively,
l 4 eV 4 × 1.6 × 10 −19
1.312 × 106 1.312 × 106
11. If l0 and l be the threshold wavelength and wavelength of E1 = and E2 = −
(1)2 (2)2
incident light, the velocity of photoelectron ejected from the
metal surface is Therefore,
2h 2hc 1.312 × 106  1.312 × 106 
(a) ( l0 − l ) (b) ( l0 − l ) ∆E = E2 − E1 = − 2
−− 
m m 2  1 

2hc  l0 − l  2h  1 1  = 9.84 × 105 J mol−1


(c) (d) −
m  ll0  m  l0 l  15. Ionization energy of He+ is 19.6 × 10−18 J atom−1. The energy
(JEE Main Online 2014) of the first stationary state (n = 1) of Li2+ is
(a) 8.82 × 10−17 J atom−1 (b) 4.41 × 10−16 J atom−1
Solution
1 (c) −4.41 × 10−17 J atom−1 (d) −2.2 × 10−15 J atom−1
(c) hn = hn0 + K.E. ⇒ hn = hn 0 + mv 2 (AIEEE 2010)
2
Solution
2hc  1 1  2hc  l0 − l  (c) The ionization enthalpy (IE) is expressed as
v2 = − ⇒v=
m  l l0  m  l l0  2 1 1  2
IEHe+ = 13.6 ZHe +
12 − ∞2  = 13.6 ZHe+ ( where ZHe+ = 2)
12. A certain metal was irradiated with light of frequency  
3.2 × 1016 Hz. The photoelectrons emitted had twice the
Given that IEHe+ = 19.6 × 10 −18 J
kinetic energy as did photoelectrons emitted when the same
metal was irradiated with light of frequency 2 × 1016 Hz. So, 2
13.6 × ZHe + = 19.6 × 10
−18
J atom−1 (1)
Calculate the threshold frequency for the metal.
1 1 
(a) 5 × 1016 Hz (b) 6 × 1015 Hz Now, for Li2+, we have (E1)Li2+ = −13.6 ZLi2 2+  − 
 1 ∞2 
2

(c) 8 × 1015 Hz (d) 7 × 1016 Hz  1


= −13.6 ZLi2 2+  2 
Solution 1 
K.E. Multiplying and dividing by ZHe 2
+ , we get
(c) We have the relation K.E. = hv − hv 0 or v − v 0 =
h
Given that (K.E.)2 = 2(K.E.)1. Therefore, we have 2
 Z 2 2+  2 9
Li
(E1)Li2+ = −13.6 ZHe +   = −13.6 ZHe + ×
v 2 − v 0 (K.E.)2 / h 2
 He 
Z 4
= =2 +

v1 − v 0 (K.E.)1 / h
From Eq. (1), we have
or v 2 − v 0 = 2v1 − 2v 0 9
(E1)Li2+ = −19.6 × 10 −18 × = −4.41× 10 −7 J atom−1
Hence, v 0 = 2v1 − v 2 = ( 4 × 1016 ) − (3.2 × 1016 ) = 8 × 1015 Hz 4
 Z2 
Bohr’s Atomic Model 16. Energy of an electron is given by E = −2.178 × 10 −18 J   .
 n2 
13. According to Bohr’s theory, the angular momentum of an Wavelength of light required to excite an electron in an hydro-
electron in 5th orbit is gen atom from level n = 1 to n = 2 will be (h = 6.62 × 10−34 J s
(a) 25h/p (b) 1.0h/p and c = 3.0 × 108 m s−1)
(c) 10h/p (d) 2.5h/p (a) 2.816 × 10−7 m (b) 6.500 × 10−7 m
(AIEEE 2006) −
(c) 8.500 × 10 m7 (d) 1.214 × 10−7 m
Solution (JEE Main 2013)
(d) According to Bohr’s postulate of quantization of angu- Solution
hc 1 1
lar momentum, the angular momentum of an electron (d) E = = 2.178 × 10 −18 × 12  2 − 2  = 1.634 J
nh l 1 2 
should be an integral multiple of h/2p, that is, mvr =
2p ⇒ l = 1.214 × 10 −7 m

Chapter-2.indd 43 8/4/2016 12:07:49 PM


44 Chapter 2 Atomic Structure

17. If m and e are the mass and charge of the revolving electron Solution
in the orbit of radius r for hydrogen atom, the total energy of 1 1 1
(a) Using the formula, = R 2 − 2
the revolving electron will be l  1
n n 2

1 e2 e2 me2 1 e2 Substituting n2 = infinity (given) and n1 = 1, we get,
(a) (b) − (c) (d) −
2 r r r 2 r 1 1
= 1.097 × 107 × [1] or l = = 91 nm
(JEE Main Online 2014) l 1.097 × 107
Solution
22. How far from the nucleus is the electron in a hydrogen atom
(d) Total energy of e− is one half of potential energy
if it has energy of −0.850 eV.
1 Ze2 1 e2 (a) 7342 Å (b) 8464 Å (c) 7535 Å (d) 6590 Å
E=− =− (Since Z = 1 for H atom)
2 r 2 r Solution
18. Based on the equation: Z2
(b) According to Bohr’s theory, En = − × 2.18 × 10 −19
 1 1 n2
∆E = −2.0 × 10 −18 J  2 − 2  where En is the total energy of electron in that shell.
 n2 n1  Substituting −0.850 eV, we get
the wavelength of the light that must be absorbed to excite Z2
hydrogen electron from level n = 1 to level n = 2 will be: − × 2.18 × 10 −19 = −0.85 × 1.6 × 10 −19 ⇒ n = 4
n2
(h = 6.625 × l0−34 J s, c = 3 × 108 m s−1)
(a) 1.325 × 10−7 m (b) 1.325 × 10−10 m Substituting n = 4 for Z = 1, we get
(c) 2.650 × 10−7 m (d) 5.300 × 10−10 m n2 16
(JEE Main Online 2014) r= × 52.9 pm = × 52.9 pm = 8464 Å
Z 1
Solution
23. Calculate the energy of radiation emitted for the elec-
hc  1 1
(a) E= = −2.0 × 10 −18 J  2 − 2  tronic transition from infinity to ground state for hydro-
l  n2 n1  gen atom. Given c = 3 × 108 m s−1, RH = 1.09678 × 107 m−1,
1 −2.0 × 10 −18  1 1 h = 6.6256 × 10−34 J s−1.
= −34 8
× 2 − 2 (a) 2.18 × 10 −18 J (b) 3.25 × 10 −18 J
l 6.625 × 10 × 3 × 10  n2 n1 
(c) 4.05 × 10 −18 J (d) 2.39 × 10 −18 J
1 2 10 −18  1  2 3 1 ⇒ 1.325 × 10 −7
=− × −26  − 1 = × × Solution
l 19.8 10  4  19.8 4 10 −8
(a) According to Rydberg equation, we have
19. Which of the following is the energy of a possible excited 1
1 1
state of hydrogen? v = = RZ 2  2 − 2 
(a) − 6.8 eV (b) −3.4 eV l  n1 n2 
(c) + 6.8 eV (d) +13.6 eV where Z = 1, R = 1.09678 × 107 m−1, n1 = 1 and n2 = ∞.
(JEE Main 2015) 1 1 1 
Solution Therefore, = 1.09678 × 107  − 
Z2 l 1 ∞ 
(b) Energy of hydrogen atom En = −13.6 × 2 eV 1
n or l = = 0.911× 10 −7 = 9.11 × 10 −8 m
where Z = 1 and n = 2. Substituting, we get 1.09678 × 107
1 The energy is given by
E = −13.6 × = −3.4 eV
4 hc 6.6256 × 10 −34 × 3 × 108
20. In Bohr series of lines hydrogen spectrum, the third line E= = = 2.18 × 10 −18 J
l 9.11× 10 −8
from the red end corresponds to which one of the following
inter-orbit jumps of the electron for Bohr orbits in an atom of 24. Calculate the ratio of the radius of Be3+ ion in third energy
hydrogen? level to that of He+ ion in the second energy level.
(a) 3 → 2 (b) 5 → 2 (c) 4 → 1 (d) 2 → 5 (a) 8 : 9 (b) 8 : 7
(c) 9 : 8 (d) 9 : 7
Solution
Solution
(b) Line from the red end belongs to Balmer series for which
n2
n1 = 2 and n2 = 3, 4, 5 for first, second and third lines (c) From the expression r ∝ where n is the principal shell
respectively. Z 2
r n  Z 
and Z is the atomic number, we have 1 =  1   2 
21. The wavelength of the radiation emitted, when in hydrogen r n   Z 
2 2 1
atom electron transition from infinity to stationary state 1,
Substituting n1 = 3, n2 = 2, Z1 = 4 for Be3+ and Z2 = 2 for
would be (Rydberg constant = 1.097 × 107 m−1)
2
(a) 91 nm (b) 9.1 × 10−8 nm r1  3   2  9
He+, we get =   ×  =
(c) 406 nm (d) 192 nm r2  2   4  8

Chapter-2.indd 44 8/4/2016 12:07:58 PM


Solved Examples 45

25. Consider the transition from energy level n = 6 to n = 4 for n = 5. Thus, the energy difference is minimum, and so the
He+, the wavelength observed is the same as the transition in wavelength of emitted photon will be maximum.
[consider no reduced mass effect]
(a) Li2+ from n = 6 to n = 4 (b) B4+ from n = 6 to n = 4 Dual Nature of Matter
(c) H from n = 3 to n = 2 (d) Be3+ from n = 12 to n = 8 27. Uncertainty in the position of an electron (mass = 9.1 × 10−31
Solution kg) moving with a velocity 300 m s−1, accurate upto 0.001%,
1  1 1 will be
(c) Using Rydberg formula = RH × Z 2  2 − 2  , we have:
l  n1 n2  (a) 19.2 × 10−2 m (b) 5.76 × 10−2 m
−2
(c) 1.92 × 10 m (d) 3.84 × 10−2 m
For transition from energy level n = 6 to n = 4 for He+ is −34
(where h = 6.63 × 10 J s)
1  1 1
= RH × 22  2 − 2  (AIEEE 2006)
lHe + 4 6 
This can be expressed in terms of following transitions: Solution
1  1 1  1 1 (c) According to Heisenberg uncertainty principle
= RH × 22  2 − 2  = RH  2 − 2 
lHe + 4 6  2 3 
h
∆x ⋅ ∆p ≥
⇒ same as in H from n = 3 → 2 4p
1  1 1  1 1  h
= RH  2 − 2  = RH × 32  2 2 − 2 2  or ∆x ≥ (as ∆p = m∆v ) .
lHe + 2 3  3 ×2 3 ×3  4pm∆v
 1 1 Also, ∆v = 0.001% of 300. Substituting, we get
= RH × 32  2 − 2 
6 9  6.63 × 10 −34
∆x = = 0.01933 m
⇒ same as in Li2 + from n = 9 → 6 0.001
4 × 3.14 × 9.1× 10 −31 × 300 ×
1  1 1  1 1  100
= RH  2 − 2  = RH × 4 2  2 2 − 2 2  ∆x = 1.93 × 10−2 m
lHe + 2 3   4 ×2 4 ×3 
 1 1  28. In an atom, an electron is moving with a speed of 600 m s−1
= RH × 4 2  2 − 2  with an accuracy of 0.005%. Certainty with which the position
 8 12 
of the electron can be located is (h = 6.6 × 10−34 kg m2 s−1,
⇒ same as in Be3+ from n = 12 → 8 mass of electron me = 9.1 × 10−31 kg)
1  1 1  1 1  (a) 1.52 × 10−4 m (b) 5.10 × 10−3 m
= RH  2 − 2  = RH × 52  2 2 − 2 2  −3
(c) 1.92 × 10 m (d) 3.84 × 10−3 m
lHe + 2 3  5 ×2 5 ×3 
(AIEEE 2009)
 1 1 
= RH × 52  2 − 2  Solution
 10 15  h
(c) Using Heisenberg’s uncertainty principle, ∆x ⋅ m∆v =
⇒ same as in B4 + from n = 15 → 10 h 4p
Hence, the wavelength is the same in the case of H from ⇒ ∆x =
4p m∆v
n = 3 to n = 2.
Given that ∆v = 0.005% of 600 m s−1, so
26. Which of the following electron transitions in hydrogen atom 0.005
∆v = 600 × = 0.03 ms −1
will produce photon with maximum wavelength? 100
(a) n = 5 to n = 1 (b) n = 4 to n = 2 Hence,
(c) n = 6 to n = 4 (d) n = 6 to n = 5
6.6 × 10 −34
Solution ∆x = = 1.92 × 10 −3 m
4 × 3.14 × 9.1× 10 −31 × 0.03
(d) The energy of photon is calculated as the energy differ- 29. Calculate the wavelength (in nanometer) associated with
ence of the orbits, a proton moving at 1.0 × 103 ms−1 (mass of proton = 1.67 ×
Ephoton = En2 − En1 = ∆E 10−27 kg and h = 6.63 × 10−34 J s).
(a) 0.032 nm (b) 0.40 nm
hc hc
Hence, Ephoton = = ∆E ⇒ l = (c) 2.5 nm (d) 14.0 nm
l ∆E (AIEEE 2009)
So, emitted photon’s wavelength is inversely propor-
tional to the energy difference of energy levels. Since Solution
 1 1 (b) Using de Broglie relationship
∆E ∝  2 − 2  , moving away from nucleus the gap
 n2 n1  h 6.63 × 10 −34
l= = = 0.4 0 nm
between successive energy levels is suppressed. So for . × 10 −27 × 103
mv 167
the transition from highest energy level to lower energy
level will produce lowest energy photon. 30. The de Broglie wavelength of a particle of mass 6.63 g moving
For n = 6 to n = 5, transition is taking place from the with a velocity of 100 ms−1 is:
highest energy level, n = 6 to just lower energy level, (a) 10−33 m (b) 10−35 m (c) 10−31 m (d) 10−35 m
(JEE Main Online 2014)

Chapter-2.indd 45 8/4/2016 12:08:03 PM


46 Chapter 2 Atomic Structure

Solution 1 h
h 6.626 × 10 −34 ∆v x =
(a) Wavelength is given by l = = 2 2m p
mv 6.63 × 100 × 10 −3
= 10 −33 m 34. If l1 and l2 denote the de Broglie wavelengths of two par-
ticles with same masses but charges in the ratio of 1:2 after
31. Calculate the wavelength of a particle of mass 3.1 × 10−31 kg
they are accelerated from rest through the same potential
that is moving with a speed of 2.21 × 107 m s−1. (Use h = 6.626
difference, then
× 10−34 J s)
(a) l1 = l2 (b) l1 < l2 (c) l1 > l2 (d) l1 ≠ l2
(a) 6.59 × 10 −11 m (b) 7.52 × 10 −11 m
Solution
(c) 8.62 × 10 −11 m (d) 9.67 × 10 −11 m h
(c) l = ; where K.E. is in eV.
2m K.E.
Solution
(d) The wavelength of the particle can be computed using Given that the mass is same, but charges are in the ratio
de Broglie’s equation l = h/mv. 1:2, therefore, K.E.2 > K.E.1.
Given that mass of the particle (m) = 3.1 × 10−31 kg,
l1 K.E.2
velocity of particle = 2.21 × 107 m s−1, h = 6.626 × 10−34 J s Now, = ⇒ l1 > l2
= 6.626 × 10−34 kg m2 s−1. l2 K.E.1
Substituting values in equation, we get
Quantum Numbers and Rules for Filling Electrons
6.626 × 10 −34 kg m2 s −1 −11
l= = 9.67 × 10 m 35. Which of the following sets of quantum numbers represents
(3.1× 10 −31 kg) × (2.21× 107 m s −1)
the highest energy of an atom?
32. The uncertainty for the calculation of radius of the first Bohr (a) n = 3, l = 0, ml = 0, ms = +1/2
orbit is 2% for the hydrogen atom. What will be the uncer- (b) n = 3, l = 1, ml = 1, ms = +1/2
tainty in velocity of electron in the first Bohr orbit. (Given that (c) n = 3, l = 2, ml = 1, ms = +1/2
h = 6.626 × 10−34 J s and m = 9.1 × 10−31 kg.) (d) n = 4, l = 0, ml = 0, ms = +1/2
(AIEEE 2007)
(a) 5.47 × 107 m s −1 (b) 6.02 × 107 m s −1
Solution
(c) 4.71× 107 m s −1 (d) 6.85 × 107 m s −1 (c) According to (n + l) rule, more is the value of (n + l)
Solution more is the energy. Hence, when n = 3 and l = 2, then
(n + l) = (3 + 2) = 5.
(a) We know that r = 0.529 × 10−10 m. If uncertainty is 2%, we
get 36. The electrons identified by quantum numbers n and l
2 (I) n = 4, l = 1 (II) n = 4, l = 0
∆x = 0.529 × 10 −10 × = 1.06 × 10 −12 m
100 (III) n = 3, l = 2 (IV) n = 3, l = 1
From Heisenberg’s uncertainty principle, we have can be placed in order of increasing energy as:
(a) (III) < (IV) < (II) < (I) (b) (IV) < (II) < (III) < (I)
h
∆x ⋅ m∆v ≥ (c) (II) < (IV) < (I) < (III) (d) (I) < (III) < (II) < (IV)
4p (AIEEE 2012)
where ∆x is the uncertainty in position and ∆v is the Solution
uncertainty in velocity. Thus, we have
(b) Using n + l rule, that is, lower the n + l value, lower will be
6.626 × 10 −34 its energy. In case n + l value is the same, then the one
∆v ≥
4 × 3.14 × 9.1× 10 −31 × 1.06 × 10 −12 having lower n value will be lower in energy.
⇒ ∆v ≥ 5.47 × 107 m s −1 (I) n = 4, l = 1, n + 1 = 4 + 1 = 5
(II) n = 4, l = 0, n + 1 = 4 + 0 = 4
33. If uncertainty in position is twice the uncertainty in momen- (III) n = 3, l = 2; n + 1 = 3 + 2 = 5 (here the value of n is
tum, then uncertainty in velocity will be lower).
h 1 h (IV) n = 3, l = 1; n + 1 = 3 + 1 = 4 (here the value of n is
(a) (b) lower, so it is lower in energy).
p 2m p
Thus, the order is IV < II < III < I.
1 1 h
(c) h (d)
2m 2 2m p 37. The correct set of four quantum numbers for the valence
Solution electrons of rubidium atom (Z = 37) is
(d) Using the following formulas for uncertainties (a) 5, 0, 0, +1/2 (b) 5, 1, 0, +1/2
(c) 5, 1, 1, +1/2 (d) 5, 0, 1, +1/2
∆x⋅∆px = h/4p ⇒ ∆x⋅∆vx = h/4p m (JEE Main 2014)
Since ∆x is twice ∆px, Solution
2∆px⋅∆vx = h/4p m or ∆px⋅∆vx = h/8p m (a) For Rb (Z = 37), the electronic configuration is 1s2 2s2 2p6
But, p = mv. So, m⋅∆vx⋅∆vx = h/8p m. 3s2 3p6 3d10 4s2 4p6 5s1
(∆vx)2 = h/8p m2. Therefore, ∆vx = (h/8p m)1/2

Chapter-2.indd 46 8/4/2016 12:08:09 PM


Solved Examples 47

As the last electron enters 5s orbital, n = 5, l = 0 and Solution


ml = 0, and since only one electron is present in the s- (c) When n = 4, l = 3, m = +1, s = +(1/2) as l = 3, m = −l to +l and
orbital, ms = +1/2. s = +1/2 or −1/2.
38. The energy of an electron in first Bohr orbit of H-atom is 42. Consider the ground state of Cr atom (Z = 24). The number of
− 13.6 eV. The energy value of electron in the excited state of electrons with the azimuthal quantum numbers l = 1 and 2
Li2+ is are, respectively,
(a) −27.2 eV (b) 30.6 eV (c) −30.6 eV (d) 27.2 eV (a) 12 and 4 (b) 16 and 5
(JEE Main Online 2014) (c) 16 and 4 (d) 12 and 5
Solution 2 2 Solution
E1 Z1 n2
(c) The energy is given by = × 2 (d) The configuration of Cr is 1s2 2s2 2p6 3s2 3p6 4s1 3d5
E1 Z 22 n1
For l = 1, p = 6 + 6 = 12 and for l = 2, d = 5.
−13.6 12 22 −13.6 × 9
= 2 × 2 ⇒ E2 = = −30.6 eV 43. In a multi-electron atom, which of the following orbitals
E2 3 1 4 described by the three quantum number will have the same
energy in the absence of magnetic and electric fields?
39. If the principal quantum number n = 6, the correct sequence
of filling of electrons will be (I) n = 1, l = 0 , m = 0
(a) ns→np→(n − 1)d→(n − 2)f
(II) n = 2, l = 0 , m = 0
(b) ns→(n − 2)f→(n − 1)d→np
(c) ns→(n − 1)d→(n − 2)f→np (III) n = 2, l = 1, m = 1
(d) ns→(n − 2)f→np→(n − 1)d
(IV) n = 3, l = 2, m = 0
(JEE Main Online 2015)
Solution (V) n = 3, l = 2, m = −1
(b) According to Aufbau principle, the correct order of filling
(a) I and II (b) II and III (c) III and IV (d) IV and V
the electrons is shown below.
Solution
1s (d) This is because the energy levels follow n + l rule and the
values are same for IV and V, that is, 3 + 2 = 5 for both the
2s 2p levels.

44. Which of the following sets of quantum numbers is not


3s 3p 3d representing the electrons which are eliminated from Fe to
convert it into Fe3+?
4s 4p 4d 4f (a) n = 4, l = 0, m = 0, s = 1/2
(b) n = 4, l = 0, m = 0, s = −1/2
5s 5p 5d 5f (c) n = 3, l = 2, m = 0, s = −1/2
(d) n = 3, l = 1, m = 0, s = 1/2
6s 6p 6d Solution
(d) Ground state electronic configuration of Fe: 1s2 2s2 2p6
7s 7p 3s2 3p6 3d6 4s2
The electronic configuration of Fe3+: 1s2 2s2 2p6 3s2 3p6
3d5 4s0
Thus, for n = 6, ns, (n − 2) f, (n − 1) d, np is the correct order. Hence, two electrons are removed from the fourth shell
40. The orbital angular momentum for an electron revolving in and one from the third shell. Two types of orbitals (d and
an orbit is given by l(l + 1) × (h/2p). This momentum for an s) are involved. The value of l for s-orbital is 0 and for d-or-
s electron will be given by bital, l = 2.
(a) +1/2(h/2p) (b) 0 Fourth set of quantum numbers contains l = 1, that is,
(c) (h/2p) (d) 2(h/2p) p-orbital. The p-orbitals are not involved in ionization.

Solution 45. In a chromium atom, how many electrons have zero azi-
muthal quantum number?
(b) This is because for s orbital, l = 0.
(a) 1 (b) 6 (c) 7 (d) 8
41. Which of the following sets of quantum numbers is correct for Solution
an electron in 4f orbital?
(a) n = 4, l = 3, m = +4, s = +(1/2) (c) The electronic configuration of chromium is 1s2 2s2 2p6
(b) n = 3, l = 2, m = −2, s = +(1/2) 3s2 3p6 3d5 4s1.
(c) n = 4, l = 3, m = +1, s = +(1/2) The number of electrons in s-orbitals is 7, for which azi-
(d) n = 4, l = 4, m = −4, s = −(1/2) muthal quantum number is zero.

Chapter-2.indd 47 8/4/2016 12:08:12 PM


48 Chapter 2 Atomic Structure

46. Out of the following, which element is not paramagnetic in The quantum number set given in the question
its ground state? [Given: atomic number of Cu = 29, Cd = 48, belongs to second electron in 4s orbital, so, first elec-
Se = 34, Au = 79] tron’s can be considered for anyone electron in 4s-
(a) Cu (b) Se (c) Cd (d) Au orbital in Zn. Thus the only impossible quantum number
1
Solution set is n = 4 , l = 1, ml = −1, s = + which is for electron in
2
(c) Copper: The electronic configuration is 1s2 2s2 2p6 3s2 4p-orbital.
3p6 3d10 4s1. It contains one unpaired electron, so it is
49. The quantum numbers of +1/2 and −1/2 for the electron spin
paramagnetic.
represent
Selenium: The electronic configuration is [Ar] 3d10 4s2
(a) rotation of the electron in clockwise and anticlockwise
4p4, so it contains two unpaired electrons and will show
direction, respectively
paramagnetism.
(b) rotation of the electron in anticlockwise and clockwise
Cadmium: The electronic configuration is [Kr] 4d10 5s2,
direction, respectively
so no unpaired electrons exist; hence, it will not be
(c) magnetic moment of the electron pointing up and
paramagnetic.
down, respectively
Gold: It belongs to the copper family, so the electronic
(d) two quantum mechanical spin states that have no classi-
configuration is [Xe] 4f14 5d10 6s1. This one unpaired
cal analog.
electron will show paramagnetism.
Solution
47. Which of the following electronic configuration is not
possible? (a) An electron in its motion in an orbit may rotate or spin
(a) [Kr] 4d8 5s1 (b) [Kr] 3d7 4s2 about its own axis and spin may be clockwise and anti-
(c) [Xe] 4f14 5d7 6s2 (d) [Ar] 3d5 4s1 clockwise. Assuming this spin to be quantized there are
two possible values of s, that is, s = +1/2 and s = −1/2,
Solution depending upon whether the electron spins are in one
(b) The electronic configuration follows Aufbau principle, direction or in the other.
that is, electrons should be filled in increasing order 50. When the value of the azimuthal quantum number is 3, the
of energy. In [Kr], the configuration 3d7 4s2 is already maximum and the minimum values of the spin multiplicities
included with [Kr], the correct one is [Kr] 4d7 5s1 for Ru. are
All other configurations are written correctly. (a) 4, 3 (b) 8, 1 (c) 1, 3 (d) 8, 2
1 Solution
48. Consider n = 4 , l = 0 , ml = 0 , s = − is the set of four quan-
2 (d) Spin multiplicity is 2S + 1. l = 3 corresponds to s, p, d,
tum numbers for outermost electron in an atom. Which of
and f orbitals. Maximum multiplicity for the f orbitals
the following quantum set is not possible for other electrons
7
available in this atom? (7 electrons) = 2S + 1 = 2 × + 1 = 8
1 2
(a) n = 4 , l = 0 , ml = 0 , s = + Minimum multiplicity for the f orbital (1 electron)
2
1 1
(b) n = 3, l = 2, ml = −2, s = + = 2S + 1 = 2 × + 1 = 2
2 2
1 51. The number of d electrons in Cu+ (atomic number = 29) that
(c) n = 4 , l = 1, ml = −1, s = +
2 can have the spin quantum (−1/2) is
1 (a) 5 (b) 6 (c) 3 (d) 7
(d) n = 2, l = 1, ml = 1, s = −
2 Solution
Solution
(a) Cu+ = [Ar ]3d10 4 s0
(c) Outermost electron belongs to 4s orbital, so, we can 1
Five d-electrons have + spin
consider all set of quantum numbers for orbitals filled 2
before it. The set of quantum number is not possible 1
which belongs to the orbital filled after 4s orbital. Five d-electrons have − spin
2

ADVANCED LEVEL PROBLEMS


1. According to Bohr’s theory, En = total energy; Kn = kinetic (c) Angular momentum in (r) −2
energy; Vn = potential energy; Rn = radius of nth orbit. Match lowest orbital
the entries in Column I with correct values in Column II:
1
(d) ∝Zy, y =? (s) 1
Column I Column II rn
(a) Vn/Kn = ? (p) 0 (IIT-JEE 2006)
(b) If radius of nth orbit (q) −1 Solution
∝ Enx , then x = ? (a) ã r; (b) ã q; (c) ã p; (d) ã s

Chapter-2.indd 48 8/4/2016 12:08:15 PM


Advanced Level Problems 49

Potential energy = −Ze2/r and kinetic energy = Ze2/2r. Dividing 5. The orbital angular momentum quantum number of the state
the two, we get Vn/Kn = −2. S2 is
Total energy = −Ze2/2r or rn ∝ E−1 (a) 0 (b) 1 (c) 2 (d) 3
Angular momentum = nh/2p. For n = 0, angular momentum Solution
is zero. (b) Azimuthal quantum number for S2 = 1
From the expression rn = aon2/Z, therefore, rn ∝ Z−1
For S2, radial node = 1, so 1= n − l − 1 or l = n − 2
2. Match the entries in Column I with the correctly related quan-
Z2
tum number(s) in Column II. ES2 = −13.6 × = EH in ground state = −13.6
n2
Column I Column II 9
or E = −13.6 × 2 ⇒ n = 3
(a) Orbital angular momentum (p) Principal quantum n
of the electron in a hydro- number So, the state in S1 is 2s and S2 is 3p.
gen-like atomic orbital
6. The maximum number of electrons that can have principal
(b) A hydrogen-like one- (q) Azimuthal quantum
quantum number, n = 3, a spin quantum number, ms = −(1/2) is
electron wave function number
___.
obeying Pauli principle
(IIT-JEE 2011)
(c) Shape, size and orientation (r) Magnetic quantum Solution
of hydrogen like atomic number
orbitals (9) For n = 3, the total number of orbitals = n2 = 9. This means
18 electrons can be accommodated in 9 orbitals in which
(d) Probability density of (s) Electron spin quan-
9 will have spin clockwise (ms = +1/2) and 9 will have anti-
electron at the nucleus in tum number
clockwise (ms = −1/2).
hydrogen-like atom
7. The work function (W0) of some metals is listed below. The
(IIT-JEE 2008)
number of metals which will show photoelectric effect when
Solution
light of 300 nm wavelength falls on the metal is ___.
(a) ã q; (b) ã s; (c) ã p, q, r; (d) ã p, q, r
Azimuthal quantum number determines the orbital angular Metal W0 (eV) Metal W0 (eV)
momentum of electron. It also determines the shape of the Li 2.4 Ag 4.3
orbital.
Na 2.3 Fe 4.7
Pauli’s exclusion principle states that an orbital can contain
maximum of two electrons. K 2.2 Pt 6.3
Principal quantum number determines the size of the orbital. Mg 3.7 W 4.75
Magnetic quantum number determines the orientation of the Cu 4.8
orbital.
(IIT-JEE 2011)
Three of the quantum number (principal, azimuthal, magnetic)
Solution
together represents the probability density of electron.
(4) Given that wavelength is l = 300 nm = 300 × 10−9 m
Paragraph for Questions 3 to 5: The hydrogen-like species Li2+ = 3 × 10−7 m. Therefore, energy is
is in a spherically symmetric state S1 with one radial node. Upon hc 6.6 × 10 −34 × 3 × 108
absorbing light, the ion undergoes transition to a state S2. The E = hn = = = 6.6 × 10 −19 J
l 3 × 10 −7
state S2 has one radial node and its energy is equal to the ground
state energy of the hydrogen atom. 6.6 × 10 −19
= = 4.1 eV
(IIT-JEE 2010) . × 10 −19
16
3. The state S1 is For a metal to show photoelectric effect, its work function
(a) 1s (b) 2s (c) 2p (d) 3s has to be less than or equal to 4.1 eV. So, the number of met-
als having work function less than 4.1 eV are Li, Na, K and Mg.
Solution
(b) For S1 (spherically symmetrical), node = 1, or n − 1 = 1 8. The kinetic energy of an electron in the second Bohr orbit of a
⇒n=2 hydrogen atom is (a0 is the Bohr radius).
h2 h2
4. Energy of the S1 in units of the hydrogen atom ground state (a) 2 2
(b)
energy is 4p ma0 16p 2 ma02
(a) 0.75 (b) 1.50 (c) 2.25 (d) 4.50 h2 h2
(c) 2
(d)
Solution 32p ma02 64p 2 ma02
(c) The relations between energy in S1 state and that of (IIT-JEE 2012)
ground state of hydrogen atom is Solution
ES1 −13.6 × 9
= = 2.25 (c) According to Bohr’s postulate, the angular momentum of
EH 4 × ( −13.6 ) an electron is quantized, that is,

Chapter-2.indd 49 8/4/2016 12:08:17 PM


50 Chapter 2 Atomic Structure

nh nh 13. The energy of the electron in the second and the third
mvr = ⇒v = Bohr’s orbits of the hydrogen atom is −5.42 × 10−12 ergs and
2p 2p mr
−2.41 × 10−12 ergs, respectively. Calculate the wavelength of
the emitted radiation when the electron drops from the third
Now, substituting the above expression in the kinetic
to the second orbit.
energy expression (1/2) mv2, we get
1  nh 
2 Solution
K.E. = m  (1)
2  2p mr  The energy of the emitted radiation is

where r is the radius of Bohr’s orbit, that is, r = ao × n2/Z. hc hc


∆E = E3 − E2 = ⇒ −2.41× 10 −12 − (5.42 × 10 −12 ) =
For the second Bohr’s orbit of a hydrogen atom, substitut- l l
ing Z = 1 and n =2, we get r = 4a0 where a0 is a constant.
6.626 × 10 −34 × 3 × 108
Now, substituting this value of r in Eq. (1), we get 3.01× 10 −12 × 10 −7 =
l
1  4 h2  h2
K.E. = × m 2 2 = 6.626 × 10 −7 × 3
6.626 × 10 −26 × 3
2 2 2 2
 4p m ⋅16a0  32p ma0 l= =
3.01× 10 −19 3.01
9. The atomic masses of He and Ne are 4 and 20 amu, respec- −7
= 6603 Å = 6.604 × 10 m
tively. The value of the de Broglie wavelength of He gas at
−73°C is m times that of the de Broglie wavelength of Ne at 14. Calculate the wavelength in angstrom of the photon that is
727°C m is ___. emitted when an electron in the Bohr orbit, n = 2 returns to
(JEE Advanced 2013) the orbit, where n = 1 in the hydrogen atom. The ionization
Solution potential of the ground state hydrogen atom is 2.17 × 10−11
h ergs per atom.
(5) We know that l = , where E (kinetic energy) ∝ T.
2mE
Solution
h
Therefore, l = The energy of the emitted radiation is
mT
lHe h / mHeTHe mNeTNe 20 × 1000 1 1  hc  1
= = = =5 ∆E = 2.17 × 10 −18  2 − 2  ⇒ = 2.17 × 10 −18 1− 
lNe h / mNeTNe mHeTHe 4 × 200  1
n n2 l  4
10. In an atom, the total number of electrons having quantum
numbers n = 4, |ml| = 1 and ms = −1/2 is _____. 6.62 × 10 −34 × 3 × 108 3
or = 2.17 × 10 −18 ×
(JEE Advanced 2014) l 4
Solution
4 × 6.62 × 3 × 10 −26 79.44 × 10 −26
(6) As n = 4, so l = 0, 1, 2, 3 which implies the orbitals are 4s, or l= −18
= = 12.20 × 10 −8 Å
4p, 4d and 4f. 6.51× 10 6.51× 10 −18
Now, |ml| = 1 implies l = 1 that is 4p, 4d and 4f orbitals. 4s
15. Calculate the wave number for the shortest wavelength tran-
orbital not possible.
sition in the Balmer series of atomic hydrogen.
Also, given that ms = −1/2 so six orbitals are possible with
ms = −1/2. Solution
11. Not considering the electronic spin, the degeneracy of the The shortest wavelength transition in the Balmer series corre-
second excited state (n = 3) of H atom is 9, while the degen- sponds to the transition n1 = 2 to n2 = ∞. Hence, the wave
eracy of the second excited state of H− is _____. number is
(JEE Advanced 2015)
 1 1  1 1
Solution v = RH  2 − 2  = (109677 cm−1)  2 − 2  = 27419.25 cm−1
 n1 n2  2 ∞ 
(3) 2s, 2p differ in energy in multielectron systems; 2s < 2p
In the first excited state, electron moves from 1s to 2s.
16. Calculate the energy required to excite one litre of hydrogen
In the second excited state, electron moves from 1s to 2p.
gas at 1 atm and 298 K to the first excited state of atomic
So, the degeneracy of second excited state 2p = 3.
hydrogen. The energy for the dissociation of H H bond Is
234 −7a 436 kJ mol−1.
12. 92 X  → Y. Find out the atomic number, mass number
+6 b
Solution
of Y and identify the isotope.
First, we determine the number of moles of hydrogen gas,
Solution
pV 1× 1
As per the reaction, the mass number of Y is 234 − 7 × 4 n= = = 0.0409. The concerned reaction is
RT 0.082 × 298
(from a-particle) + 6 × 0 (from b particle) = 206.
The atomic number of Y is 92 − 7 × 2 (from a-particle) + 6 × 1 H2 → 2H; ∆H = 436 kJ mol−1. Energy required to bring 0.0409
206
(from b particle) = 84. Hence, the isotope is 84 Po. mol of hydrogen gas to atomic state = 436 × 0.0409 = 17.83 kJ.

Chapter-2.indd 50 8/4/2016 12:08:23 PM


Advanced Level Problems 51

Next, we calculate the total number of hydrogen atoms


 1 1  6.626 × 10 −34 × 3 × 108
in 0.0409 mol of H2 gas 1 mol H2 gas has 6.02 × 1023 mol- −2.18 × 10 −18 × Z 2  2 − 2  =
 (1) (2)  3 × 10 −8
6.02 × 1023
ecules 0.0409 mol of H2 gas = × 0.0409 mol- 3
1 −2.18 × 10 −18 × Z 2 × = 6.626 × 10 −18
4
ecules. Since 1 molecule of H2 gas has 2 hydrogen atoms,
6.626 × 10 −18 × 4
so 6.02 × 1023 × 0.0409 molecules of H2 gas will have or Z2 = =4⇒Z =2
2.18 × 10 −18 × 3
23 22
2 × 6.02 × 10 × 0.0409 = 4.92 × 10 atoms Thus, the species is He+ with Z = 2.
The energy required to excited an electron from the ground
20. Find out the number of waves made by a Bohr electron in one
state to the next excited state is
complete revolution in its third orbit.
 1 1  1 1 3
E1 = 13.6  2 − 2  = 13.6 ×  −  = 13.6 × = 10.2 eV Solution
 n1 n2   1 4  4
The number of waves that a Bohr electron makes in an orbit is
632 × 10 −21 kJ
= 1.6 in fact its principle quantum number. Now, using Bohr’s pos-
Therefore, energy required to excite 4.92 × 1022 electrons is tulate of angular momentum in the third orbit, we get
h
E2 = 1.632 × 10 −21 × 4.92 × 1022 = 8.03 × 10 = 80.3 kJ mvr = n where n = 3 for third orbit.
2p
Therefore total energy required, Etotal = 17.83 + 80.3 = 98.13 kJ
According to de Broglie’s equation, l = h/mv. Substituting this
17. Wavelength of high energy transition of hydrogen atoms in the above expression, we get
is 91.2 nm. Calculate the corresponding wavelength of He
 h  h 
atoms.   × r = 3   ⇒ 3l = 2p r
l 2p
Solution Thus, we have that the circumference of the third orbit is
For maximum energy, n1 = 1 and n2 = ∞. Hence, we have equal to three times the wavelength of the electron; or in
1  1 1 other words, the number of waves made by a Bohr electron in
= RH Z 2  2 − 2  one complete revolution in its third orbit is three.
l  n1 n2 
Since RH is a constant and transition remains the same, 21. A ball of mass 100 g is moving with 100 ms−1. Find its wave-
length.
1 l Z2 1
∝ Z 2 ⇒ He = 2H = Solution
l lH ZHe 4
The wavelength is
1
Hence, lHe = × 9.12 = 22.8 nm h 6.627 × 10 −34
4 l= = ⇒ l = 6.627 × 10 −35 m
mv 0.1× 100
18. According to Bohr’s theory, the electronic energy of hydro- = 6.627 × 10−25 Å
gen atom in the nth Bohr’s orbit is given by En = ( −21.76 × 10 −19 ) / n2
22. Find the velocity (ms−1) of electron in first Bohr’s orbit of
En = ( −21.76 × 10 −19 ) / n2 J. Calculate the longest wavelength of light that
radius a0. Also find the de Broglie’s wavelength (in m). Find
will be needed to remove an electron from the third Bohr the orbital angular momentum of 2p orbital of hydrogen
orbit of the He+ ion. atom in units of h/2p.
Solution Solution
The energy to remove an electron from the third Bohr orbit of Since mvr = nh/2p and r = a0 = 0.529 Å, we have the velocity
the He+ ion (Z2 = 4) is of the electron as
−21.76 × 10 −19 × 4 87.04 nh 6.626 × 10 −34
En= 3 = =− × 10 −19 = −9.67 × 10 −19 J v= = = 2.19 × 106 mss −1
9 9 2p mr 2 × 3.14 × 9.1× 10 −31 × 0.529 × 10 −10
Now, as ∆E = hc/l , we have as for hydrogen atom, Z = 1, n = 1. Now, the de Broglie’s wave
6.626 × 10 −34 × 3 × 108 −7 length is
l= = 2.0556 × 10 m = 2056 Å h 6.626 × 10 −34
9.67 × 10 −19 l= = = 3.32 × 10 −10 m = 3.3 Å
mv 9.1× 10 −31 × 2.19 × 106
19. At what minimum atomic number, a transition from n = 2 to For 2p, l = 1, so the orbital angular momentum is
n = 1 energy level would result in the emission of X-rays with h h
l = 3.0 × 10−6 cm? Which hydrogen atom-like species does l (l + 1) = 2
2p 2p
this atomic number correspond to? 23. An electron beam can undergo diffraction by crystals.
Solution Through what potential should a beam of electrons be accel-
hc erated so that its wavelength becomes equal to 1.54 Å.
From the expression ∆E = E1 − E1 = , we have
l

Chapter-2.indd 51 8/4/2016 12:08:30 PM


52 Chapter 2 Atomic Structure

Solution Solution
1 h
From the expressions mu2 = eV and l = , we get (b) The radius of the hydrogen-like species is calculated and
2 mv compared with that of hydrogen.
1 h2 1 h2 0.529 n2
m 2 2 = eV ⇒ V = rn = Å
2 m l 2 ml 2e Z
Substituting values, we get 0.529 × 12
r1(H) = = 0.529 Å
1× (6.62 × 10 −34 )2 1
V= = 63.3 V 0.529 × 22
2 × 9.108 × 10 −31
× (1.54 × 10 −10 )2 × 1.602 × 10 −19 r2 (Be3+ ) = = 0.529 Å
4
24. The solution of the Schrödinger wave equation for hydrogen 27. The number of radial nodes in 3p and 4s orbitals, respec-
atom is tively, are
3/2 (a) 1, 3 (b) 3, 1
1  1  r  −r / a
y 2s =  2 − a  e (c) 2, 2 (d) 2, 1
1/2  a 
4(2p )  0  0
Solution
where a0 is Bohr’s radius. Let the radial node in 2s be at r0. (a) The number of radial nodes = n − l − 1. Now, for 3p, n = 3,
Then find r in terms of a0. l = 1. Substituting, we get
Solution n−l−1=3−1−1=1
The probability of finding 2s electron will be For 4s, n = 4, l = 0. Substituting, we get n − l − 1 = 4 − 0 − 1 = 3
3 2
1  1  r  28. Ratio of time period of electrons in the second orbit of H and
y 22s dV = 2 −  e −2r / a0
32p  a0   a0  first orbit of He+ would be
Node is a point where probability of finding an electron is (a) 1 : 32 (b) 32 : 1 (c) 2 : 1 (d) 1 : 2
zero. Thus, y 22s dV = 0 when r = r0. Therefore, Solution
2 2 2p rn n2 n n3
1  1  r  (b) We have time period(T ) = ∝ × ∝ 2
   2 − 0  e −2r / a0 = 0 vn Z Z Z
32p  a0   a0 
Therefore, the ratio of time period of electrons in the
r second orbit of H and first orbit of He+ is
The only factor that can be zero is 2 − 0 = 0 ⇒ r0 = 2a0
a0
TH 23 / 13 23 22
25. What is the maximum number of electrons that may be pres- = 2 2 = 2 × 3 = 32
THe + 1 / 2 1 1
ent in all the atomic orbitals with principle quantum number
3 and azimuthal quantum number 2? 29. Naturally occurring boron consists of two isotopes whose
atomic weights are 10.01 and 11.01. The atomic weight of
Solution natural boron is 10.81. Calculate the percentage of each iso-
For n = 3 and l = 2 (i.e., 3d orbital), the values of m varies from tope in natural boron.
−2 to +2, that is, −2, −1, 0, +1, +2 and for each m there are
Solution
2 values of s, that is, +1/2 and −1/2. The maximum number
of electrons in all the five d-orbitals is 10. Average mass = Mass of isotopes’ mole fraction + Mass of iso-
topes’ mole fraction
26. Which hydrogen-like species will have same radius as that of 10.81 = 11.01x + 10.01 (1 − x) ⇒ 10.81 = 11.01x + 10.01 − 10.01x
Bohr orbit of hydrogen atom? ⇒ x = 10.81 − 10.01 = 0.80
(a) n = 2, Li2+ (b) n = 2, Be3+ Therefore, the percentage of each isotope in natural boron is
(c) n = 2, He+ (d) n = 3, Li2+ 80% and 20%.

PRACTICE EXERCISE
Level I (a) [He] 2s12p3 (b) [Kr] 3d74s2
(c) [He] 2s22p4 (d) [Xe] 4f145d86s1
Single Correct Choice Type
3. Of the following transitions in hydrogen atom, the one that
1. If n = 6, the correct sequence of filling of electrons will be
gives an absorption line of maximum wavelength is
(a) ns → np → (n − 1)d → (n − 2)f
(a) n = 1 to n = 2 (b) n = 3 to n = 8
(b) ns → (n − 2)f → (n − 1)d → np
(c) n = 2 to n = 1 (d) n = 8 to n = 3
(c) ns → (n − 1)d → (n − 2)f → np
(d) ns → (n − 2)f → np → (n − 1)d 4. In which block will copper (atomic number 29) be placed if the
Aufbau principle is not followed for filling the electrons?
2. Which one is correct among the following electron configura- (a) s-block (b) p-block
tions for atoms in their ground states? (c) d-block (d) f-block

Chapter-2.indd 52 8/4/2016 12:08:34 PM


Practice Exercise 53

5. The electron energy in hydrogen atom is given by E = 17. A compound of vanadium possesses a magnetic moment of
(−21.7 × 10−12)/n2 ergs. Calculate the energy required to 1.73 BM. The oxidation state of vanadium in this compound is
remove an electron completely from the n = 2 orbit. What is the (a) 1 (b) 2
longest wavelength (in cm) of light that can be used to cause (c) 4 (d) cannot be predicted
this transition?
18. The radius of which of the following orbit is same as that of
(a) 385 nm (b) 309 nm (c) 367 nm (d) 350 nm
the first Bohr’s orbit of hydrogen atom?
6. During the emission spectrum, the first line of Lyman series of (a) He+ (n = 2) (b) Li2+ (n = 2)
hydrogen atom occurs as l = x Å. The wavelength of the first 2+
(c) Li (n = 3) (d) Be3+ (n = 2)
line of Lyman series of He+ ion will occur at
19. An ion that has 18 electrons in the outermost shell is
(a) 4/x (b) x/4 (c) 3/x (d) 4x
(a) Cu+ (b) Th4+ (c) Cs+ (d) K+
7. Which of the following combination of quantum numbers is 20. When the frequency of light incident on a metallic plate is
not allowed? doubled, the K.E. of the emitted photoelectron will be
(a) n = 3, l = 0, ml = 0 (b) n = 4, l = 4, ml = 0 (a) doubled.
(c) n = 3, l = 1, ml = 1 (d) n = 3, l = 3, ml = 1 (b) halved.
8. The first line in the Balmer series in the hydrogen atom will (c) increased but more than double of the previous KE.
have the frequency (d) remains unchanged quantum number.
(a) 4.57 × 1014 s−1 (b) 3.29 × l015 s−1 21. The ionization energy of a hydrogen atom in terms of Rydberg
(c) 8.22 × 1015 s−1 (d) 8.05 × 1013 s−1 constant (RH ) is given by the expression
9. If the kinetic energy of a proton is increased 9 times, the wave- (a) RHhc (b) RHc
length of the de Broglie wave associated with it would become
(a) 3 times (b) 9 times (c) 2RHhc (d) RHhcNA
(c) 1/3 times (d) 1/9 times 22. The wavelength associated with a golf ball weighing 200 g
10. The total spin resulting from a d7 configuration is and moving at a speed of 5 m h−1 is of the order
(a) 3/2 (b) 1/2 (c) 2 (d) 1 (a) 10−10 m (b) 10−20 m (c) 10−30 m (d) 10−40 m

11. Four different sets of quantum numbers for four electrons are 23. For which of the species, Bohr’s theory is not applicable?
given below: (a) Be3+ (b) Li2+ (c) He2+ (d) H
E1 = 4, 0, 0, 1/2 E2 = 3, 1, 1, 1/2 24. Find the number of spectral lines that are emitted by atomic
E3 = 3, 2, 2, 1/2 E4 = 3, 0, 0, 1/2 hydrogen excited to (n + 1) the energy level.
The order of energy of E1, E2, E3, and E4 is:
(a) E1 > E2 > E3 > E4 (b) E4 > E3 > E2 > El (a) n(n +1) (b) 1 n(n − 1)
(c) E3 > E1 > E2 > E4 (d) E2 > E3 > E4 > E1 2 2
1
(c) (n + 1)(n + 2) (d) 1 (n − 1)(n − 2)
12. The first emission line in the atomic spectrum of hydrogen in 2 2
the Balmer series appears at 25. The energy associated with the first orbit in the hydrogen
9RH 7RH atom is −2.18 × 10−18 J/atom. What is the radius and energy
(a) cm−1 (b) cm−1 associated with the fifth orbit?
400 144
3RH 5RH (a) 1.3225 nm, −5.63 × 10 −20 J
(c) cm−1 (d) cm−1
4 36 (b) 1.3225 nm, −8.72 × 10 −20 J
13. Atomic number of chromium is 24, then Cr 3+ will be (c) 2.36 nm, −8.72 × 10 −20 J
(a) diamagnetic. (b) paramagnetic. (d) 3.265 nm, −5.63 × 10 −20 J
(c) ferromagnetic. (d) none of these. 26. Magnetic moments of V (Z = 23), Cr (Z = 24), Mn (Z = 25) are
14. Correct set of four quantum numbers for the valence (outer- x, y, z. Hence,
most) electron of rubidium (Z = 37) is: (a) x = y = z (b) x < y < z
(a) 5, 0, 0, +1/2 (b) 5, 1, 0, + 1/2 (c) x < z < y (d) z < y < x
(c) 5, 1, 1, + 1/2 (d) 6, 0, 0, +1/2. 27. The kinetic energy, E, of an electron is related to the kel-
3
15. Which of the following postulates does not belong to Bohr’s vin temperature through the equation, E = kT where k =
model of the atom? 2
1.38 × 10−23 J particle−1 K−1. You are given an electron with a
(a) Angular momentum is an in the orbit is stable of h/2p. de Broglie wavelength of l = 76.3 nm. What is the kelvin tem-
(b) The electron stationed in the orbit is stable. perature of electron?
(c) The path of an electron is circular. (a) 0.50 (b) 1.00 (c) 1.50 (d) 2.00
(d) The change in the energy levels of electron is continuous.
28. According to Bohr’s theory of hydrogen atom, which of the
16. Maximum number of electrons in a subshell of an atom is following is quantized for an electron?
determined by the following: (a) Angular momentum (b) Angular acceleration
(a) 2n2 (b) 4l + 2 (c) 2l + 1 (d) 4l − 2 (c) Acceleration (d) Velocity

Chapter-2.indd 53 8/4/2016 12:08:40 PM


54 Chapter 2 Atomic Structure

29. The ratio of the radii of the first three Bohr orbits is 42. How many photons having a wavelength of 3.00 mm would
(a) 1 : 0.5 : 0.33 (b) 1 : 2 : 3 have to be absorbed by 1.00 g of water to raise its tempera-
(c) 1 : 4 : 9 (d) 1 : 8 : 27 ture by 1.00°C?
(a) 5.36 × 1025 (b) 6.32 × 1022
30. The maximum number of electrons that may be present in
(c) 4.29 × 1025 (d) 8.64 × 1022
all atomic orbitals with principle quantum number 3 and azi-
muthal quantum number 2 is 43. In the spectrum of hydrogen, there is a line with a wave-
(a) 10 (b) 8 (c) 12 (d) 4 length of 410.3 nm. Use the Rydberg equation to calculate
the value of n for the higher energy Bohr orbit involved in
31. For the shortest and longest value of l of Lyman series, the
the emission of this light. Assume the value of n for the lower
type of transition is
energy orbit equals 2.
(a) n2 = 5, n1 = 1 and n2 = 2, n1 = 1.
(a) 5 (b) 7 (c) 6 (d) 8
(b) n2 = ∞ , n1 = 1 and n2 = 2, n1 = 1.
44. How fast would an electron have to be moving to eject an
(c) n2 = 1, n1 = ∞ and n2 = 1, n1 = 2. electron from an atom of neon, which has a first ionization
(d) n2 = ∞ , n1 = 2 and n2 = 2, n1 = 1. energy equal to 2080 kJ mol−1?
32. The velocity of electron in a certain Bohr’s orbit of H-atom (a) 2.754 × 106 m s−1 (b) 3.021 × 107 m s−1
bears the ratio 1:275 to the velocity of light. What is the value
(c) 1.265 × 108 m s−1 (d) 4.012 × 105 m s−1
of quantum number (n) of the orbit?
(a) 2 (b) 3 (c) 4 (d) 5 45. The removal of an electron from the hydrogen atom cor-
responds to raising the electron to the Bohr orbit that has
33. The ratio of radii of first orbits of H, He+, and Li2+ is
n = ∞. On the basis of this statement, calculate the ionization
(a) 1 : 2 : 3 (b) 6 : 3 : 2 (c) 1 : 4 : 9 (d) 9 : 4 : 1
energy of hydrogen in units of joules per atom.
34. For an electron, the product v × n (velocity × principal quan- −20
(a) 3.01 × 10 −19 J (b) 4.25 × 10 J
tum number) will be independent of the
(a) principal quantum number. (c) 2.85 × 10 −15 J (d) 2.18 × 10 −18 J
(b) velocity of the electron
(c) energy of the electron
(d) frequency of its revolution. Level II
35. Degenerate orbitals are those that contain Multiple Correct Choice Type
(a) same wave function. (b) same orientation.
46. Which of the following is the nodal plane of dxy orbital?
(c) same energy. (d) both (a) and (b).
(a) xy (b) yz
36. The angular momentum of an electron is h/p. The Bohr’s (c) zx (d) All of these
orbit in which the electron is revolving is ___.
47. Which of the following statements are correct for an electron
(a) 4 (b) 2 (c) 3 (d) 5
that has n = 2 and ml = 0?
37. If 10 −17 J of light energy is needed by the interior of human (a) The electron may be in a d-orbital.
eye to see an object, then photons of green light (l = 550 nm) (b) The electron may be in f-orbital.
needed to see the object are (c) The electron may be in a p-orbital.
(a) 27 (b) 28 (c) 29 (d) 30 (d) The electron may have the spin quantum number = +1/2.
38. In a hydrogen atom, energy of first excited state is −3.4 eV. 48. Find the ratio of la and lb for the Balmer series in the
Then, the kinetic energy of the same orbit of hydrogen atom is atomic spectra of hydrogen.
(a) +3.4 eV (b) +6.8 eV 80 90 40 20
(c) −13.6 eV (d) +13.6 eV (a) (b) (c) (d)
108 108 54 27
39. For the energy levels in an atom, which one of the following
statements is incorrect? 49. Which of the following orbitals are associated with the angu-
(a) There are 7 principal electron energy levels. lar nodes?
(b) The second principal energy level can have 4 energy (a) s-orbitals (b) p-orbitals
sublevels and contain a maximum of 8 electrons. (c) d-orbitals (d) f-orbitals
(c) The 4s energy sublevel is of higher energy than the 3d 30
50. Five valence electrons of 15 P are labeled as
energy sublevel.
(d) Both (a) and (c) are correct. pq x y z
40. When the electron of a hydrogen atom jumps from the n = 4 3s 3p
to the n = 1 state, the number of spectral lines emitted is If the spin quantum number of q and z is +(1/2), the
(a) 15 (b) 6 (c) 3 (d) 4 group of electrons with three of the quantum numbers
41. The triad of nuclei that is isotonic is same are:
(a) 146 C , 157 N, 179 F (b) 12 14 19
6 C , 7 N, 9 F (a) pq (b) (xyz),(pq)
14 14 17 14 14 19
(c) (pq),(xyz),(pz) (d) (pq),(xyz),(qy)
(c) 6 C , 7 N, 9 F (d) 6 C , 7 N, 9 F

Chapter-2.indd 54 8/4/2016 12:08:46 PM


Practice Exercise 55

51. Many elements have non-integral atomic masses because magnetic field. The paramagnetism is expressed in terms of mag-
(a) they have isotopes. netic moment.
(b) their isotopes have non-integral masses.
59. Which of the following ions has the highest magnetic
(c) their isotopes have different masses.
moment?
(d) the constituents neutrons, protons, and electrons com-
(a) Fe2+ (b) Mn2+ (c) Cr3+ (d) V3+
bine to give fractional masses.
60. Which of the following ions has magnetic moment equal to
52. In which of the following pairs the ions are isoelectronic?
that of Ti3+?
(a) N3−, Cl− (b) Al3+, O2−
+ 2− (a) Cu2+ (b) Ni2+ (c) Co2+ (d) Fe2+
(c) Na , O (d) Na+, Mg2+
53. The angular momentum of electron can have the value(s) 61. In which of the following options do both constituents of the
pair have the same magnetic moment?
h h 2h 5 h
(a) (b) (c) (d) × (a) Zn2+ and Cu2+ (b) Co2+ and Ni2+
2p p p 2 2p 4+
(c) Mn and Co 2+ (d) Mg2+ and Sc+
54. The quantum numbers that are derived from the solution of Paragraph for Questions 62 to 63: For one-electron species, the
Schrodinger’s wave equation are wave number of radiation emitted during the transition of elec-
(a) Principal quantum number tron from a higher energy state (n2) to a lower energy state (n1) is
(b) Azimuthal quantum number given by:
(c) Magnetic quantum number 1  1 1
(d) Spin quantum number n = = RH × Z 2  2 − 2  (1)
l  1
n n2
55. The well-known Schrodinger equation is
2p me k 2e 4
8p 2 m where RH = is Rydberg constant for hydrogen atom.
∇2y + (E − V )y = 0. h3c
h2 Now, considering nuclear motion, the most accurate measure-
The acceptable solutions of this wave equation, which are phys- ment would be obtained by replacing mass of electron (me) by the
ically possible, must have which of the following properties? reduced mass (m ) in the above expression, defined as
(a) y must be continuous. m × me
(b) y must be finite. m= n
mn + me
(c) y must be single valued.
(d) The probability of finding of the electron over the space where mn = mass of nucleus. For Lyman series, n1 = 1 (fixed for all
from +∞ to −∞ must be equal to one. the lines) while n2 = 2, 3, 4…. For Balmer series: n1 = 2 (fixed for all
the lines) while n2 = 3, 4, 5….
Passage Type
62. If proton in hydrogen nucleus is replaced by a positron hav-
Paragraph for Questions 56 to 58: Heisenberg gave the principle ing the same mass as that of an electron but same charge
that there is uncertainty in simultaneous measurement of position as that of proton, then considering the nuclear motion, the
and momentum of small particles. If any one of these two quan- wavenumber of the lowest energy transition of He+ ion in
tities is measured with higher accuracy, the measurement of the Lyman series will be equal to
other becomes less accurate. The product of the uncertainty in (a) 2 RH (b) 3 RH (c) 4 RH (d) RH
position (∆x) and uncertainty in momentum (∆p) is always constant 3 2 5
and is equal to or greater than h/4p, where h is Planck’s constant. 63. The ratio of the wave numbers for the highest energy transi-
tion of electron in Lyman and Balmer series of hydrogen atom
56. The uncertainty in position of an electron (m = 9.1 × 10−28 g) is
moving with a velocity 3 × 104 cm s−1 accurate upto 0.001% (a) 4 : 1 (b) 6 : 1 (c) 9 : 1 (d) 3 : 1
will be
(a) 3.84 cm (b) 1.92 cm (c) 7.68 cm (d) 5.76 cm. Matrix-Match Type
57. If uncertainty in position is twice the uncertainty in momen- 64. Match the orbitals with the number of nodes/quantum num-
tum, then uncertainty in velocity will be bers present in them.
h 1 h Column I Column II
(a) (b) (a) 2p orbital (p) Number of spherical nodes = 0
p 2m p
(b) 3d orbital (q) Number of nodal plane = 0
1 1 h (c) 2s orbital (r) Orbital angular momentum number = 0
(c) h (d)
2m 2 2m p (d) 4f orbital (s) Azimuthal quantum number = 0
58. If uncertainty in the position of an electron is zero, the uncer-
65. Match the quantum number with the property it represents.
tainty in its momentum would be
(a) zero. (b) <h/4p (c) >h/4p (d) infinite. Column I Column II
(a) Principal quantum number (p) Orientation of the orbital
Paragraph for Questions 59 to 61: The substances that contain (b) Azimuthal quantum number (q) Energy and size of orbital
species with unpaired electrons in their orbitals behave as par- (c) Magnetic quantum number (r) Spin of electron
amagnetic substances. Such substances are weakly attracted by (d) Spin quantum number (s) Shape of the orbital

Chapter-2.indd 55 8/4/2016 12:08:51 PM


56 Chapter 2 Atomic Structure

66. Match the shell with the property. Integer Type


Column I Column II
(a) K (p) Maximum number of electrons present 68. The angular momentum of an electron is h/p. The Bohr’s
is 18. orbit in which the electron is revolving is ___.
(b) L (q) There are no electrons with m value of 69. The number of electrons with l = 2 in the ground state of
zero if the shell is fully occupied chromium atom is ___.
(c) M (r) There are no electrons with l value of 2, if 70. Magnetic moment of M x+ is 24 BM. The number of unpaired
the shell is fully occupied electrons in M x+ is ___.
(d) N (s) p sub shell is present 71. The number of elements that have the last electron with
quantum numbers of n = 4 and l = 1 is ___.
67. Match the series of hydrogen spectrum with their
characteristics. 72. The number of spectral lines produced when an electron jumps
Column I Column II from 5th orbit to 2nd orbit in the hydrogen atom is ____.
(a) Lyman series (p) Visible region 73. The magnetic quantum number for valence electron of
(b) Balmer series (q) Infrared region sodium atom is ___.
(c) Paschen series (r) Ultraviolet region 74. The value of n of the highest excited state that an electron of
(d) Bracket series (s) n2 = 4 to n2 = 3 hydrogen atom in the ground state can reach when 12.09 eV
(t) n2 = 5 to n2 = 1 energy is given to the hydrogen atom is ___.

ANSWER KEY
Level I
1. (b) 2. (c) 3. (b) 4. (a) 5. (c) 6. (b)
7. (b) 8. (a) 9. (c) 10. (a) 11. (c) 12. (d)
13. (b) 14. (a) 15. (d) 16. (a) 17. (c) 18. (d)
19. (a) 20. (c) 21. (a) 22. (c) 23. (c) 24. (a)
25. (b) 26. (c) 27. (d) 28. (a) 29. (c) 30. (a)
31. (b) 32. (a) 33. (b) 34. (a) 35. (c) 36. (b)
37. (b) 38. (a) 39. (c) 40. (b) 41. (a) 42. (b)
43. (c) 44. (a) 45. (d)

Level II
46. (b), (c) 47. (c), (d) 48. (a), (c), (d) 49. (b), (c), (d) 50. (a), (b) 51. (a), (c)
52. (c), (d) 53. (a), (b), (c) 54. (a), (b), (c) 55. (a), (b), (c), (d) 56. (b) 57. (d)
58. (d) 59. (b) 60. (a) 61. (c) 62. (b) 63. (a)
64. (a) → p; (b) → p; (c) → q, r, s; (d) → p 65. (a) → q; (b) → s; (c) → p; (d) → r
66. (a) → q, r; (b) → q, r, s; (c) → p, q, s; (d) → q, s 67. (a) → r, t; (b) → p; (c) → q, s; (d) → q
68. (2) 69. (5) 70. (4) 71. (6) 72. (6) 73. (0)
74. (3)

HINTS AND EXPLANATIONS


Level I 3. (b) Absorption line in the spectra arises when energy is
absorbed, that is, electron shifts from lower to higher
Single Correct Choice Type orbit out of a and b; b will have the maximum wave-
1. (b) According to Aufbau principle, the correct sequence for length as this falls in the Paschen series.
filling up orbitals when n = 6 is 6s, 4f, 5d, 6p. 4. (a) If the Aufbau principle is not followed, the filling of elec-
2. (c) trons takes place in the sequence 1s, 2s, 2p, 3s, 3p, 3d,
Option (a): 2p cannot be filled unless 2s is completely 4s, 4p, 4d, 4f. The atomic number of Cu is 29 and so the
filled. electronic configuration will be 1s22s22p63s23p63d104s1.
Option (b): [Kr] should be replaced by [Ar] Hence Cu will be placed in s-block.
Option (d): The filling of orbitals is incorrect. According −21.7 × 10 −12
to Aufbau principle, 6s should be filled first 5. (c) E= = 5.42 × 10 −12 ergs = 5.42 × 10 −19 J
22
followed by 4f and 5d.

Chapter-2.indd 56 8/4/2016 12:08:51 PM


Hints and Explanations 57

hc 6.6 × 10 −34 × 3 × 108 Number of unpaired electrons in Cr 3+ = 3 . Therefore, it is


E = hv = =
l l paramagnetic.
6.626 × 10 −34 × 3 × 108 14. (a) Z = 37, so electronic configuration is 1s2 2s2 2p6 3s2 3p6
l= −19
= 3.67 × 10 −7 m = 367 nm
5.42 × 10 4s2 3d10 4p6 5s1.
For outermost electron, that is, 5s1 we have n = 5, l = 0,
1
6. (b) l is inversly proportional to the square of the atomic m = 0, s = + .
number. 2
7. (b) It is not possible since the value of l cannot be equal to n. 15. (d) Conceptual.
It can have values up to n − 1. 16. (a) The maximum electron population of a shell is 2n2. For
8. (a) For the first line in the Balmer series, n1 = 2 and n2 = 3. example, for n = 2, subshells are 2s and 2p with maximum
According to Rydberg’s formula, electron population 8 = 2 (2)2.
 1 1  1 1  17. (c) The number of unpaired electrons can be obtained from
n (cm−1) = 109677  2 − 2  = 109677  2 − 2 
 n1 n2   ( 2 ) ( 3)  the magnetic moment as:
5 1.73 = n(n + 2) ⇒ n = 1
= 109677 × cm−1 = 15232.9 cm−1
36
The electronic configuration of vanadium is:
The frequency can be calculated using the expression V(23) = 1s2 2s2 2p6 3s2 3p6 4s2 3d3
c = vl, as The configuration with one unpaired electron is:
c
n = = c l = 3 × 1010 cm s −1 × 15232.9 cm−1 V4+ = 1s2 2s2 2p6 3s2 3p6 4s1
l So, oxidation state is +4.
= 4.57 × 1014 s −1 0.529 × (2)2 0.529 × n2
h 18. (d) rBe3+ = = 0.529 as rn = Å
9. (c) The wavelengths are l1 = and 4 2
2 × m × (K.E. )1 19. (a) Atomic number of Cu = 29
h Electronic configuration of Cu is 1s22s22p63s23p63d104s1
l2 = Electronic configuration of Cu+ is 1s22s22p63s23p63d10
2 × m × 9 × (K.E. )1
Thus, outermost shell contains 18 electrons.
Now, (K.E.)2 = 9(K.E.)1, so
20. (c) hv1 = hv0 (Work function) + K.E.1
l2 h 9 × 2 × m × (K.E. )1 2 × m × K.E. 1 2hv1 = hv0 + K.E.1,
= = =
l1 h 2 × m × (K.E. )1 3 2 × m × K.E. 3 K.E. = hv1 − hv0
The value of kinetic energy will increase but more than
10. (a) The d 7 configuration is . double of the previous K.E.
n 1  1 1
The total spin ( S ) = where n = number of unpaired 21. (a) = RH Z 2  2 − 
2 l  1 ∞
electrons.
In d 7 configuration, the number of unpaired electrons For ionization energy, n1 = 1 and n2 = ∞.
= 3, so total spin = 3/2. hc RHhcZ 2
So, E = =
11. (c) E1: n = 4, l = 0, ml = 0, s = 1/2 represents 4s orbital. l 1
E2: n = 3, l = 1, ml = 1, s = 1/2 represents 3p orbital. h 6.6 × 10 −34
E3: n = 3, l = 2, ml = 2, s = 1/2 represents 3d orbital. 22. (c) l = = = 2.37 × 10 −30 m
mv 0.2 × 5
E4: n = 3, l = 0, ml = 0, s = 1/2 represents 3s orbital. 60 × 60
The relative energies of two orbitals can be determined
by (n + l) rule. According to this rule: 23. (c) Bohr’s theory is not applicable to He2+ as it has no
(i) Among two orbitals, the one which has a higher electrons.
value of (n + l) has more energy.
(ii) If the values of (n + l) are same, the one having a 24. (a) By definition the number of spectral lines that are emit-
higher value of n has more energy. ted by atomic hydrogen excited to nth level is n(n −1) .
So, the decreasing order of energy is E3 > E1 > E2 > E4. 2
Hence from (n + 1)th level it will be
1 1 1 (n + 1)(n + 1− 1) n(n + 1)
12. (d) According to Rydberg’s formula = RH Z 2  2 − 2  = .
l  1
n n2 2 2
25. (b) Energy is given by
For Balmer series; n1 = 2 because it is the first emission
line and n2 = 2 + 1 = 3. Z2 −2.18 × 10 −18
1 1 1 R 5 En = 2
× ( −2.18 × 10 −18 ) = = −8.72 × 10 −20 J
Hence, = RH × 12  2 − 2  = H cm−1 n 52
l  2 3  36
Radius is given by
13. (b) Cr (24) = 1s2 2s2 2p6 3s2 3p6 4s1 3d5 n2
Cr3+ = 1s2 2s2 2p6 3s2 3p6 4s0 3d3 rn = × 0.529 Å = 25 × 0.529 = 1.3225 nm
Z

Chapter-2.indd 57 8/4/2016 12:08:57 PM


58 Chapter 2 Atomic Structure

26. (c) Number of unpaired electrons in V, Cr, and Mn is 3, 6, 1


and 5. 34. (a) We know that v ∝ and n = principal quantum number.
n
V(23) = 1s2 2s2 2p6 3s2 3p6 4s2 3d3 Therefore, vn will be independent of the principal quan-
Cr(24) = 1s2 2s2 2p6 3s2 3p6 4s1 3d5 tum number, that is, n.
Mn(25) = 1s2 2s2 2p6 3s2 3p6 4s2 3d5
h h 35. (c) Orbitals belonging to a particular energy shell have the
27. (d) Given that de Broglie wavelength is l = ⇒v =
mv ml same energy are called degenerate orbitals.
1 1 h2 h2 36. (b) Angular momentum is quantized by h/2p
E = mv 2 = × m × 2 2 =
2 2 m l 2ml 2 h h h
mvr = n ⇒ =n ⇒n=2
3 2E 1 h2 2p p 2p
Therefore, E = kT ⇒ T = = × =2K
2 3k 3k ml 2 37. (b) Required energy
hc 6.626 × 10 −34 × 3 × 108
28. (a) One of the most important postulates of Bohr’s atomic = 10 −17 J = n × ⇒ n × J
l 550 × 10 −9
theory is that when an electron remains in its orbit or
stationary state, the angular momentum is given by 10 −17
Therefore, n = = 27.7  28
nh 6.626 × 10 −34 × 3 × 108
mevr =
2p 550 × 10 −9
where n = 1, 2, 3,…. Thus, an electron can move only
38. (a) Kinetic energy = − (Energy of 1st orbit)
in those orbits for which the angular momentum is
Kinetic energy = − (− 3.4 eV) = + 3.4 eV
an integral multiple of h/2p. Hence, only certain fixed
(Given energy of 1st orbit = −3.4 eV)
orbits are permitted around the nucleus in an atom.
39. (c) According to (n + l) rule, the orbital having lower (n + l)
n2
29. (c) rn = r0 × , where Z = 1. value has lower energy.
Z For 4s orbital: n + l = 4 + 0 = 4
Now, r1 = r0 , r2 = 4 r0 , r3 = 9r0 For 3d orbital: n + l = 3 + 2 = 5
Therefore, ratio = 1 : 4 : 9 Since the 4s energy sublevel has lower (n + l) value, it
has lower energy than the 3d energy sublevel.
30. (a) With principle quantum number 3 and azimuthal quan-
1 1
tum number 2, the orbital is 3d. As there are five atomic 40. (b) N = Number of lines emitted = n (n − 1) = × 4( 4 − 1) = 6
orbitals in 3d and each orbital can have a maximum of 2 2
(when electron falls from n = 4 to n = 1)
2 electrons, so the maximum number of electrons that
maybe present in all atomic orbitals of 3d is 10. 41. (a) Isotonic: Same number of neutrons, 146 C has 8 neu-
trons, 157 N has 8 neutrons, 179 F also has 8 neutrons.
31. (b) For Lyman series, all transition occurs from different
value of n to n = 1. Hence, shortest wavelength will be for 42. (b) We proceed by calculating the wavelength of a single
n2 = ∞ to n1 = 1 and longest wavelength will be for n2 = 2 photon
to n1 = 1. hc (6.626 × 10 −34 Js)(3.00 × 108 ms −1)
E = =
1 1 l (3.00 × 10 −3m)
32. (a) v e = × velocity of light = × 3 × 1010 cm s −1
275 275 = 6.63 × 10 −23 J
8 −1
= 1.09 × 10 cm s It requires 4.184 J to increase the temperature of 1.00 g
2p e2 of water by 1 Celsius degree.
Since, ve =
nh So, number of photons
2 × 3.14 × ( 4.803 × 10 −10 )2 4.184 J
1.09 × 108 = = = 6.32 × 1022
6.625 × 10 −27 × n 6.63 × 10 −23 J/photon
⇒ n=2
43. (c) 1  1 1
= 109,678 cm−1 ×  2 − 2 
0.529 × n 2 l  n2 n1 
33. (b) rn = Å
Z  1   1 nm   1 m 
= 
where n = number of orbits, Z = atomic number of  410.3 nm   1× 10 −9 m   100 cm 
element.
Here, n = 1 for H, He+, and Li2+.  1  −1
 1 1
1× 0.529  −5  = 109,678 cm ×  2 − 2 
So, rH = = 0.529 For H, Z = 1. 4.103 × 10 cm 2 n1 
1
2.437 × 10 4 cm−1 1 1
1× 0.529 = − = 0.222
For rHe+ = = 0.265 For He, Z = 2. 109,678 cm−1 4 n12
2
1× 0.529 1 1
For rLi2+ = = 0.176 For Li, Z = 3. = − 0.222
3 n12 4
rH+ : rHe + : rLi2+ = 0.529 : 0.265 : 0.176 = 3 : 1.5 : 1 = 6 : 3 : 2. n1 = 6

Chapter-2.indd 58 8/4/2016 12:09:05 PM


Hints and Explanations 59

1 2E 55. (a), (b), (c), (d) All are acceptable solutions of wave equation
44. (a) E = mv 2 ⇒ v = by postulates of the Schrödinger wave equation.
2 m  1 mol 
E = (2080 × 103 J mol−1)  23  Passage Type
 6.022 × 10 particles 
= 3.454 × 10–18 J/particle h 6.6 × 10 −27
56. (b) ∆v = = = 1.92 cm
4p m∆v 4 × 3.14 × 9.1× 10 −28 × 0.3
2(3.454 × 10 −18 J)
v= = 2.754 × 106 m s−1 h h 1 h
9.109 × 10 −31 kg 57. (d) ∆x × ∆p = ⇒ 2( ∆p )2 = ⇒ ∆p =
4p 4p 2 ×2 p
45. (d) This corresponds to the special case in the Rydberg
1 h
equation for which n1 = 1 and n2 = ∞. For a single atom, Therefore, ∆v =
we have 2 2m p
1 1 1 58. (d) When ∆x = 0, ∆p = ∞
=109,678 cm−1 ×  2 − 2  =109,678 cm−1 × (1− 0 )
l 1 ∞  59. (b) Magnetic moment = n(n + 2) , where n = number of
= 109,678 cm−1 unpaired electrons.
The configuration of Fe2+ is 3d 6 .
l = 9.12 × 10−6 cm = 91.2 nm.
Therefore, n = 4.
Converting to energy, we have
Magnetic moment = 4( 4 + 2) = 4.89 BM
hc
E= Electronic configuration of Mn2+ is 3d 5 .
l
−34 8 −1 Therefore, n = 5.
(6.626 × 10 Js)(3.00 × 10 ms )
= = 2.18 × 10 −18 J Magnetic moment = n(n + 2) = 5(5 + 2) = 5.91 BM
(91.2 × 10 −9 m)
Electronic configuration of Cr3+ is 3d 3 .
Level II Therefore, n = 3.
Magnetic moment = n(n + 2) = 3(3 + 2) = 3.812 BM
Multiple Correct Choice Type
46. (b), (c) Conceptual Electronic configuration of V3+ is d 2 .
Therefore, n = 2.
47. (c), (d) If the electron has n = 2, then the value of l = 0, 1.
Thus, if the electron has n = 2, then the electron may be in a p Magnetic moment = 2(2 + 2) = 8 = 2.82 BM
orbital and it must have the spin quantum number +1/2. Therefore, Mn2+ has the highest magnetic moment.
48. (a), (c), (d) For all these cases, the minimum ratio is 20/27. 60. (a) The electronic configuration of Ti3+ is d1
49. (b), (c), (d) Angular nodes are only associated with direction
orbitals. Therefore, n = 1.
50. (a), (b) Conceptual Magnetic moment = n + (n + 2) = 11
( + 2) = 1.73 BM
51. (a), (c) Conceptual The electronic configuration of Cu2+ is d 9

52. (c), (d) Ions having same number of electrons but different
magnitude of nuclear charge are called isoelectronic ions. Therefore, n = 1.
Na+ and O2− have 10 electrons each. Na+ and Mg2+ have 10 Magnetic moment = 11
( + 2) = 1.73 BM
electrons each.
N3− has 10 electrons and Cl− has 18 electrons, hence not isoe- 61. (c) The electronic configuration of Mn4+ is d 3
lectronic ions.
Al3+ has 10 electrons and O− has 9 electrons, hence not isoe-
lectronic ions. Therefore, n = 3.
The electronic configuration of Co2+ d7
53. (a), (b), (c) Angular momentum of the electron is given as
h
mvr = n , where n = 1, 2, 3, …
2p Therefore, n = 3.
h
If n = 1, then mvr = Magnetic moment = 3(3 + 2) = 3.872 BM
2p
2h h 62. (b) Rydberg’s constant value will change.
If n = 2, then mvr = =
2p p Rnew ( me 2) 1
= =
4 h 2h Rold me 2
If n = 4, then mvr = =
2p p
54. (a), (b), (c) Conceptual 1 R 1 1
= v = H Z2  2 − 2 
l 2  n1 n2 

Chapter-2.indd 59 8/4/2016 12:09:19 PM


60 Chapter 2 Atomic Structure

L-shell : s and p orbitals are present in it.


For lowest energy transition of He + ion in Lyman series
M-shell : s, p and d orbitals are present in it.
R 1 1 3
v = H 22  2 − 2  = RH N-shell : s, p, d and f orbitals are present in it.
2 1 2  2 The values of l for s, p, d, f are 0, 1, 2, 3 respectively.
1 1 1 The value of m for s is zero.
63. (a) Lyman series, n1 = 1, = v1 = RH ⋅12  2 − 2  The value of m for p are +1, 0, −1.
l1 1 n2 
The value of m for d are +2, +1, 0, −1, −2.
1 1 1 The value of m for f are +3, +2, +1, 0, −1, −2, −3.
Balmer series, n2 = 1, = v 2 = RH ⋅12  2 − 2 
l2  2 n2  67. (a) → r, t; (b) → p; (c) → q, s; (d) → q
For highest energy transition, Lyman series is (n1 = 1) and falls in UV region.
For Lyman series n2 = ∞ Balmer series is (n1 = 2) and falls in visible region.
For Balmer series n2 = ∞ Paschen series is (n1 = 3) and falls in infrared region.
v 4 Integer Type
Therefore, 1 =
v2 1 h h h
68. (2) mvr = n ⇒ =n ⇒n=2
Matrix-Match Type 2p p 2p
69. (5) l = 2, and the number of d-electrons = 5
64. (a) → p; (b) → p; (c) → q, r, s; (d) → p
For 2p orbital, number of spherical nodes = n − l − 1 = 2 − 1 − 1 70. (4) n( n + 2) = 24 ⇒ n( n + 2) = 24 ⇒ n = 4
= 0; nodal planes ≠ 0; l = 1; m = 1, 0, −1.
For 3d orbital, number of spherical nodes = n − l − 1 = 3 − 2 − 1 71. (6) n = 4, l = 1 represents 4p subshell containing six electrons.
= 0; nodal planes ≠ 0; l = 2; m = 2, 1, 0, −1, −2. Thus, there will be six elements having 4 p1 to 4 p 6 electronic
For 4f orbital, number of spherical nodes = n − l − 1 = 4 − 3 − 1 configuration.
= 0; nodal planes ≠ 0; l = 3; m = 3, 2, 1, 0, −1, −2, −3.
For 2s orbital, number of spherical nodes = n − l − 1 = 2 − 0 − 1 72. (6) Number of spherical lines produced if electron falls from
= 1; nodal planes ≠ 0; l = 0; m = 0. n2 to n1.

65. (a) → q; (b) → s; (c) → p; (d) → r (n2 − n1)(n2 − n1 + 1) (5 − 2)(5 − 2 + 1)


= =6
Principal quantum number (n) represents energy and 2 2
size of orbital. 73. (0) Atomic number of sodium = 11
Azimuthal quantum number (l) represents shape of
orbital. Electronic configuration is 1s22s22p63s1.
Magnetic quantum number (ml) represents spectral ori- Valence electron enters in s subshell. For s-subshell, the value
entation of orbital. of magnetic quantum number is 0.
Spin quantum number (s) represents spin of electron. It E
74. (3) En = −13.6 + 12.09 = −1.51 En = 21
is the only quantum number with a non-integral value. n
66. (a) → q, r; (b) → q, r, s; (c) → p, q, s; (d) → q, s −13.6
Therefore, n2 = =9⇒n=3
K-shell : Only s orbital is present in it. −1.51

Chapter-2.indd 60 8/4/2016 12:09:24 PM


Solved JEE 2016 Questions 61

SOLVED JEE 2016 QUESTIONS


JEE Main 2016 Thus, both the ions Na+ and F− doesn’t have the electronic config-
uration of 1s22s22p63s23p6.
1. A stream of electrons from a heated filament was passed
between two charged plates kept at a potential difference
V esu. If e and m are charge and mass of an electron respec- JEE Advanced 2016
tively, then the value of h/l (where l is wavelength associated
with electron wave) is given by 1. P is the probability of finding the 1s electron of hydrogen atom
(a) 2meV (b) meV (c) 2meV (d) meV in a spherical shell of infinitesimal thickness, dr, at a distance r
from the nucleus. The volume of this shell is 4p2dr. The qualita-
(Offline) tive sketch of the dependence of P on r is:
Solution
1 2 P P
(a) K.E. of an electron = eV = mv
2
h
According to de-Broglie relationship, l =
mv (a) (b)
1 2 m2 v 2
mv = eV ⇒ = eV ⇒ mv = 2meV
2 2m
0 r 0 r
h h
l= = 2meV
2meV ⇒ l P
P
2. The total number of orbitals associated with the principal (c) (d)
quantum number 5 is
(a) 20 (b) 25 (c) 10 (d) 5
(Online)
Solution 0 r
0 r
(b) The total number of orbitals can be determined as follows:
l ml Solution
0 0 (b) The probability of finding an electron of hydrogen atom in
a spherical shell of infinitesimal thickness, dr, at a distance
1 0, +1, −1
r from the nucleus, with volume dV = 4p 2dr is
2 +2, +1, 0, −1, −2
n=5 P = y 2 ⋅ dV = y 2 ⋅ 4p r 2dr = R 2 (r )4p r 2dr
3 +3, +2, +1, 0, −1, −2, −3
+4, +3, +2, +1, 0, −4, −3, −2, −1 Here R2(r) is the radial density function.
4 For 1s subshell, n = 1, l = 0 and n − l − 1 = 0. Therefore, num-
Total orbitals = 25
ber of radial and angular nodes = 0, Therefore the plot of
3. Aqueous solution of which salt will not contain ions with the radial probability P = R2(r)4p r2 versus r is as follows.
electronic configuration 1s22s22p63s23p6?
P
(a) NaF (b) KBr (c) NaCl (d) CaI2
(Online)
Solution
(a) Electronic configuration of Na+ is [He]2s22p6
Electronic configuration of F− is [Ne]3s23p6
0 r

Chapter-2.indd 61 8/4/2016 12:09:30 PM


Chapter-2.indd 62 8/4/2016 12:09:30 PM
Classification of Elements
3 and Periodicity in
Properties

Question Distribution in JEE (Main)

3
No. of Questions

2 JEE (Main)

0
2016 2015 2014 2013 2012 2011 2010 2009 2008 2007

Concept Distribution in JEE (Main)


Topics Covered
Year
JEE (Main)
2007
2008 Periodic Trends in Properties
2009 Periodic Trends in Properties
2010 Periodic Trends in Properties
2011 Chemical Reactivity
2012 Chemical Reactivity
2013 Ionization Enthalpy
2014 Periodic Trends in Properties
2015 Periodic Trends in Properties
2016

Chapter-3.indd 63 8/8/2016 2:12:09 PM


64 Chapter 3 Classification of Elements and Periodicity in Properties

SUMMARY
1. Periodic table is the arrangement of elements in a tabular form on the basis of their properties that facilitates the systematic study
of properties of elements.

2. Genesis of periodic classification


(a) Dobereiner’s triad: He observed that in the set of three elements having similar properties (called triads), the atomic weight of
the middle element is the arithmetic mean of the atomic weights of other two elements.
Limitations
• Only few such triads were available at that time and day by day as many more elements were discovered, the rule could no
longer be generalized
(b) Newland’s octave law: He arranged 56 elements known then in increasing order of their atomic weight and observed that “the
properties of every eighth element are similar to that of first one.” He compared this relationship to the first octave in music with
eight notes and called it Newlands’ law of octaves.
Limitations
• The inert gases were not discovered till then.
• Beyond Ca, this repetition was not observed.
(c) Lothar Meyer’s curve: He plotted the atomic volume verses atomic mass, where the atomic volume is defined as follows

Gram atomic mass of an element


Atomic volume =
Density (in g mL−1)

80
Cs

60 Rb
Atomic volume, cm3

40 Ba

Na Ca V I
CI Sr VI
III Sb Te
IV Br
I P S
20 II Zr In Sn La
Mg As Se
Li Si Mo
AI V Fe Nb Cd Nd
Be Zn Ru Rh
C Co Cu
0
10 30 50 70 90 110 130 150
Atomic mass, amu

He proposed that the physical properties of elements are periodic functions of their atomic weights and this formed the basis of
Mendeleev’s periodic table, which was also presented around the same time.
(d) Mendeleev’s periodic table: It stated that the physical and chemical properties of elements are the periodic function of their
atomic weights.
(i) Merits
• Study of properties of elements became more systematic and easier.
• Several vacant positions guided the discovery of new elements.
(ii) Demerits
• Some of the elements are wrongly placed though their atomic weights are larger compared to the next one.
• Though the isotopes have different mass yet they don’t have any different (significant) position in the periodic table.
• Placement of sub-group elements in same group inspite of lack of sufficient similarities .
• Position of hydrogen is uncertain (either in Group I A or VII B).

Chapter-3.indd 64 8/8/2016 2:12:10 PM


Summary 65

• Placement of all rare earth elements (4f and 5f series) in Group III A.
• The classification of metals and non-metals is not done in this table.
• Placement of three elements in Group VIII.
(e) Characteristics of modern version of Mendeleev’s short periodic table.
(Characteristics of modern version of Mendeleev's short periodic table)

Horizontal rows or Periods Vertical columns or Groups Other features

First period: 2 elements and is Group I B elements known as Lanthanides or Lanthanoids


known as very short period alkali metals (expect H) or Rare earths: Elements from
Ce (58) to Lu (71)
Second period: 8 elements and Group II B elements are called Actinides or Actinoids:
is known as first short period as alkaline earth metals (except Be) Elements from Th (90) to Lr (103)

Third period: 8 elements and is Group V B elements are called Coinage metals: Cu, Ag and Au
known as second short period as pnicogens
Fourth period: 18 elements and Group VI B elements are Noble metals (very low reactivity):
is known as first long period called as chalcogens Ag, Au, Pt and Hg
Fifth period: 18 elements and is Group VII B elements are Transuranium metals:
known as second long period called as halogens Elements beyond U (92)

Sixth period: 32 elements and is Group 0 elements are called as Bridging elements: elements of
known as very long period inert gases third period

Seventh period: incomplete period Diagonal relationship: Set of


elements, that is, (Li, Mg), (Be,
AI) and (B, Si), which, though,
placed in different groups, show
some similarities in their
properties

(f) Moseley’s work: He performed an experiment in which he bombarded high speed electrons on different metal surfaces and
obtained X-rays. He observed that there existed a systematic mathematical relationship between the wavelengths of the X-rays
produced and the atomic numbers of the elements. Mathematically, it is expressed as

v ∝Z

3. Modern periodic law: According to the modern periodic law, the physical and chemical properties of the elements are the periodic
functions of their atomic number.
The long form of modern periodic table and its segments is shown on the next page.

4. Classification of elements based on electronic configuration


(a) s-Block elements
(i)
If the last electron enters into s-orbital, the elements are called as s-block elements.
Electronic configuration: ns1−2.
(ii)
Alkali metals: [IG] ns1.
(iii)
(iv)Alkaline earth metals: [IG] ns2
where IG represents the inert gas core.
(b) p-Block elements
(i) If the last electron enters into the p-orbital, the elements are called as p-block elements.
(ii) Electronic configuration: ns2 np1−6.
(iii) Placement: Group number III A to VIII A (13 to 18).
(c) d-Block elements
(i) If the last electron enters into d orbital, the elements are called as d-block elements (except Thorium).
(ii) Electronic configuration: ns0−2 (n − 1)d1−10 or ns1−2 (n − 1)d1−10 (except for palladium).

Chapter-3.indd 65 8/8/2016 2:12:12 PM


66

Chapter-3.indd 66
s-Block (ns) d-Block, (n-1)d p-Block (np), (nonmetals)
Chapter 3

Group 1 2 3 4 5 6 7 8 9 10 11 12 13 14 15 16 17 18

Peroid IA IIA IIIB IVB VB VIB VIIB VIIIB IB IIB IIIA IVA VA VIA VIIA VIIIA
1 H Transition metals H He
(1s) 1 (d-block) 1 2
2 Li Be B C N O F Ne
(2s, 2p) 3 4 5 6 7 8 9 10
3 Na Mg AI Si P S Cl Ar
(3s, 3p) 11 12 13 14 15 16 17 18
4
K Ca Sc Ti V Cr Mn Fe Co Ni Cu Zn Ga Ge As Se Br Kr
(4s, 3d,
19 20 21 22 23 24 25 26 27 28 29 30 31 32 33 34 35 36
4p)
5
Rb Sr Y Zr Nb Mo Tc Ru Rh Pd Ag Cd In Sn Sb Te I Xe
(5s, 4d,
37 38 39 40 41 42 43 44 45 46 47 48 49 50 51 52 53 54
5p)
6
Cs Ba La* Hf Ta W Re Os Ir Pt Au Hg TI Pb Bi Po At Rn
(6s, 4f,
55 56 57 72 73 74 75 76 77 78 79 80 81 82 83 84 85 86
5d, 6p)
Classification of Elements and Periodicity in Properties

7 Unq Unq Unq Ung Unq Unq Unq Unq Unq


Fr Ra Ac** Uut Uuq Uup Uuh Uus Uuo
(7s, 5f 104 105 106 107 108 109 110 111 112
87 88 89 113 114 115 116 117 118
6d, 7p) (Rf ) (Db) (Sg) (Bh) (Hs) (Mt) (Ds) (Rg) (Cn)

Post transition metals

The rare earths, (n-2)f

*Lanthanide (Lanthanoid) Ce Pr Nd Pm Sm Eu Gd Tb Dy Ho Er Tm Yb Lu
series (4f ) 58 59 60 61 62 63 64 65 66 67 68 69 70 71
Period : 6
**Actinide (Actinoid) Th Pa U Np Pu Am Cm Bk Cf Es Fm Md No Lr

f-block
series (5f) 90 91 92 93 94 95 96 97 98 99 100 101 102 103
Period :7

8/8/2016 2:12:12 PM
Summary 67

(d) f-Block elements


(i) If the last electron of the elements enters into f-orbital, they are considered as f-block elements.
(ii) Electronic configuration: ns2 (n − 1)d0−1(n − 2)f 1−14.

5. Neil’s Bohr classification


(a) Representative elements: Involve outer shells consisting solely of s- and p-electrons (except inert gases).
(b) Inert elements: Noble (inert) gases, He to Rn.
(c) Transition (d-block) elements: Elements having vacant or partially vacant d-orbital in their ground state or stable oxidation
state.
(d) Inner transition (f-block) elements: 58(Ce) to 71 (Lu) and 90(Th) to 103(Lr).
6. Naming of super heavy elements
0 = nil 4 = quad 8 = oct
1 = un 5 = pent 9 = enn
2 = bi 6 = hex
3 = tri 7 = sept
When some letters are repeated
tri + ium = trium, enn + nil = ennil
7. Periodic trends in properties
(a) Atomic radius: It is the distance from the nucleus to the outermost electron or up to point at which the probability of finding of
electron is the maximum.
(i) Covalent radius
d
• For homonuclear diatomic molecule A2 is rC = A − A
2
A A

dA−A

where, dA−A is the internuclear distance between the two same atoms.
• For a heteronuclear diatomic molecule AB is dA −B = rA + rB − 0.09 ∆c
A B

dA−B
where, dA−B is the internuclear distance between A and B, rA and rB is the covalent radius of A and B expressed in A° and ∆c
is the difference in the electronegativity.
When rA and rB are expressed in picometers, then the equation is given by
dA −B = rA + rB − 9 ∆c
Later stage, the above equation was modified by Pauling and Stevenson as
dA −B = rA + rB − CA c A − CB c B
where, CA and CB are constants for two different atoms.
(ii) Metallic radius: It is defined as half of the internuclear distance separating two adjacent metal atoms in a metallic lattice.

dM−M

Chapter-3.indd 67 8/8/2016 2:12:15 PM


68 Chapter 3 Classification of Elements and Periodicity in Properties

dM −M
It is given by rM =
2
where dM−M is the internuclear distance between two metal atoms in their metallic lattice
(iii) van der Waals radius: The sum of the van der Waals radii of two atoms is the shortest distance between two immediately
adjacent atoms (either of same or different elements) in the structure of the solid compound in which they are not bonded
to each other.

dA−A
dA − A
It is given by rV =
2
(iv) Periodic variation of atomic radii
• Variation in a period: There is a general decrease of atomic radius in a period for s-and p-block elements because the
effective nuclear charge (Zeff ) increases across a row.
For van der Waals radii: Li > Be > B > C > N > O > F > Ne
For covalent radii:
Ne > Li > Be > B > C > N > O > F

van der Covalent radius


Waals
radius
For d-block elements, the atomic radius initially decreases, then remains constant and finally increases again.

Third series
Atomic radius

Second series

First series

Atomic number

• Variation in a group: For s- and p-block elements, the atomic radii increases down the group due to successive use of
orbitals with principal quantum number (n) one higher than the last.
For d-block elements, the trend in atomic radii is: r3d series < r4d series ≈ r5d series. It is because the radius increases as the shell
number increases. The radius of elements of 4d- series is about the same as elements of 5d series. This is due to the lanth-
anoid contraction that causes increase in Zeff value due to poor shielding of fourteen f electrons.
Exception: Sc < Y < La as there is no 4f electron in the electronic configuration of La.
(b) Ionic radius: Formation of positive and negative ions from an atom is shown below. The order of radius is: A− > A > A+
(i) Ionic radius depends upon following factors:
• If the charge of cation increases, the ionic radius decreases.
• If the charge of anion increases, the ionic radius increases.
• For isoelectronic species, as the number of protons increases the radius of ion decreases.

Chapter-3.indd 68 8/8/2016 2:12:22 PM


Summary 69

(ii) Periodic variation in ionic radii


• Variation in a period: The ionic radii decrease moving from left to right across any period. This is due to increased number
of charges on the nucleus and also to the increasing charge on the ions.
• Variation in a group: The ionic radii increase on descending the group, because extra shells of electrons are added.
(c) Ionization energy (enthalpy): It is the energy required to remove an electron from an isolated gaseous atom in its ground state.
IE
→ A + (g) ∆H1 = + ve
A(g) 

−e −e −e −e −e
A IE→ A + IE→ A2 + IE→ A3+ IE→ IE→ A n +
1 2 3 4 n

IE1 < IE2 < IE3 < IE 4 < …IE n

(i) Ionization energy depends upon the following factors:


• It decreases when shell number or the radius increases.
• It increases when Zeff increases.
• It is more for half-filled and fully-filled electronic configurations
• If the principal quantum number is the same, then the energy required for removal of electrons from different orbitals
shows the following order: s > p > d > f
(ii) Periodic variation in ionization energy
For representative (s- and p- block) elements
• In a period: The Zeff increases in a period, hence IE increases.
• In a group: The IE generally decreases down a group.
For d-block elements
• In a period: There is slight increase due to increase in Zeff values along the period but it is not prominent. There is a sudden
jump from copper to zinc, silver to cadmium and gold to mercury because of full-filled configurations.
• In a group: IE1 of 3d series ≈ IE1 of 4d series while IE1 of 4d series < IE1 of 5d series due to lanthanide contraction.
Exception- Sc > Y > La

(d) Electron affinity: It is the energy released when one mole of electron is added to one mole of an element in its isolated gaseous
state (ground state). It is also called electron gain (eg) enthalpy.
+e
A(g) → A − ∆H = − x kJ mol−1

The electron affinity for the first electron is greater than that of the second electron, that is, EA1 > EA2
+e +e
A → A − → A2 −

Successive electron affinity or electron gain enthalpy always decreases.

+e
O → O − ∆Heg 1 = ( − )ve
+e
O − → O2 − ∆Heg 2 = ( + )ve
+2 e
O → O2 − Heg Total = ( + )ve

The electron affinity and ionization energy are related as


EA of A
+e −

A 
A
−e
IE of A −

(i) Electron affinity depends upon the following factors:


• It decreases when radius increases.
• It increases when Zeff increases.
• It decreases when s increases.
• EA of half-filled and full-filled electronic configuration is less.
• EA order for addition of electron in different orbitals having the same principal quantum number is s > p > d > f.

Chapter-3.indd 69 8/8/2016 2:12:24 PM


70 Chapter 3 Classification of Elements and Periodicity in Properties

(ii) Periodic trends of electron affinity


For representative (s- and p-block) elements
• In a period: It increases from left to right as Zeff increases (except inert gases).
• In a group: There is decrease of electron affinity in general but for p-block elements only the EA1 of second period ele-
ment is less than EA1 of third period element.
(e) Electronegativity: It is the tendency of the atom to attract electrons (bond pair) towards itself when combined in a compound.
Some important approaches for obtaining values of electronegativity are discussed as follows:
(i) Pauling scale: This scale is based upon energy calculation and the expression is

c A ∼ c B = 0.102 ∆ A −B (when energies are expressed in kJ mol−1)

or c A ∼ c B = 0.102 ∆ A −B (when energies are expressed in kJ mol−1)


where c A is electronegativity of A, c B is electronegativity of B; ΔA–B is the resonance energy of bond A − B = |EA–B (observed)
− EA–B (theoretical)|
E (theoretical) = EA − A × EB −B
A–B

(ii) Mulliken scale: In this scale, the electronegativity is given by the relation

c A ∼ c B = 0.102 ∆ A −B

where IE and EA are expressed in eV atom−1. The values of electronegativity are about 2.8 times larger than the Pauling
cM
values. The relation is c P = ; more accurately c P = 0.336 c M − 0.2
2.8
(iii) Allred Rochow scale: Allred defined electronegativity as the attractive force between nucleus and an electron at a dis-
tance equal to covalent radius. In this scale, the electronegativity is given by
0.359 Z eff
c AR =
r2
The following equation gives the correlation of c AR with c P
c P = c AR + 0.744
(f) Application of electronegativity
Percentage ionic character in a bond can be calculated using Haney–Smith equation.

% ionic character = 16 ∆c + 3.5( ∆c )2

where ∆c is the difference in the electronegativity.


As ∆c increases, the acidic strength of oxides of different elements decreases.
(g) Lattice energy (enthalpy): It is the energy released to form one mole of ionic compound (in solid form) from their respective
isolated gaseous ions.
−U
Mn + (g) + X n − (g) 

 MX(s)
+U

(h) Hydration energy (enthalpy): It is the energy released when an isolated gaseous ion gets hydrated and it is measured in terms
of kJ mol−1 or kJ ion−1.
(i) Hydration number: It is the number of water molecules associated to a particular ion when it is hydrated.

(i) Ionic mobility: It is defined as the ability of an ion to move in an aqueous solution.
1
Ionic mobility ∝
Hydrated radius of ion

TOOLS FOR PROBLEM SOLVING


1. Total valence shell electrons of d-block elements = Total number of electrons in the outermost shell (ns orbital) and penultimate shell
[(n − 1) d orbitals]. Example Co: [Ar] 4s23d7: 2 + 7 = 9
2. Total valence shell electrons of f-block elements = Electrons present in ns, (n − 1) d and (n − 2) f orbitals or subshells.
3. Prediction of period and group number of an element
(a) For the period: The highest principal quantum number available in the electronic configuration is the period number of that ele-
ment (except in the case of Pd).

Chapter-3.indd 70 8/8/2016 2:12:29 PM


Solved Examples 71

(b) For the group number


(i) For s-block elements: Total number of valence shell electrons is the group number of it.
(ii) For p-block element: (Total number of valence shell electron + 10) = Group number of it.
(iii) For d-block element: Total number of valence shell electrons is the group number.
(iv) For f-block element: All elements are placed in the Group III or Group III B.
4. For any particular element that can provide all three kinds of radius, that is, covalent radius, metallic radius and ionic radius. Then,
rV > rM > rC . Generally, van der Waals radius > covalent radius.
5. Exceptional cases
(a) Ionization energy in a period: Be, Mg and Ca have fully-filled ns2 orbitals and hence higher IE values, while B, Al and Ga attain
fully-filled ns2 configuration on losing one electron.
Be > B Mg > Al Ca > Ga
(b) Ionization energy in a group
(i) Al ≈ Ga (due to poor shielding of ten d electrons in Ga).
(ii) In < Tl (due to lanthanoid contraction). Hence, the overall order for the group is B > Tl > Ga ≥ Al > In.
(iii) Sn < Pb (due to lanthanoid contraction in Pb). The overall order for its group elements is C > Si > Ge > Pb > Sn.
(iv) Sc > Y > La (for d-block elements).
(c) Electron affinity in a period
(i) EA of alkali metals > EA of alkaline earth metal
Alkali metals Alkaline earth metals

+e +e
ns1 → ns2 ns2 → ns2np1
• Fully-filled electronic configuration is achieved. • Fully-filled electronic configuration is disturbed.
• ∆Heg is ( − ) ve • ∆Heg is ( + ) ve

(ii) EA of C > EA of N. Similarly,


EA of Si > EA of P
EA of Ge > EA of As
EA of Sn > EA of Sb
EA of Pb < EA of Bi

SOLVED EXAMPLES
Modern Periodic Table For the group number:
For p-block element:
1. Similarity in chemical properties of the atoms of elements in a Total number of valence shell electron + 10 = 6 + 10 = 16
group of the periodic table is most closely related to
(a) atomic numbers. 3. According to the periodic law of elements, the variation in
(b) atomic masses. properties of elements is related to their
(c) number of principal energy levels. (a) atomic masses (b) nuclear masses
(d) number of valence electrons. (c) atomic numbers (d) nuclear neutron−proton
(JEE Main 2014) number ratios
Solution Solution
(d) Chemical properties of elements in group are closely (c) Because modern periodic law states that atoms or ele-
related because they have same number of valence ments are arranged according to their atomic numbers.
electron. 4. Which of the following elements belongs to Group 1 of the
periodic table?
2. In the long form of the periodic table, the valence shell elec-
(a) Beryllium (b) Aluminium
tronic configuration of 5s2 5p4 corresponds to the element
(c) Gallium (d) Rubidium
present in:
(a) Group 16 and period 6 (b) Group 17 and period 5 Solution
(c) Group 16 and period 5 (d) Group 17 and period 6 (d) Be: Group 2 (s-block); Al: Group 13 (p-Block); Ga: Group 13
(JEE Main 2015) (p-Block); Rb: Group 1 (s-block)
Solution
5. In which of the following sets of atomic numbers, all the ele-
(c) The highest principal quantum number available in the elec-
ments do not belong to the same group?
tronic configuration is the period number of that element,
(a) 3, 11, 37 (b) 12, 38, 56
therefore, the period for given electronic configuration is 5.
(c) 8, 16, 24 (d) 10, 18, 54

Chapter-3.indd 71 8/8/2016 2:12:30 PM


72 Chapter 3 Classification of Elements and Periodicity in Properties

Solution so it can lose electrons to form ionic compound. Therefore, Pb


(c) All the elements in the same group will have similar is the more metallic than the other elements in Group 14.
valence shell electronic configurations. The electronic 10. The element with atomic number 26 will be found in Group
configurations of the elements of a group are represented (a) 2 (b) 8
in terms of the noble gas configuration based on the (c) 6 (d) 10
value of n for the shell.
For option (a): All the elements belong to Group 1. Solution
(b) The electronic configuration of element with atomic
3: [He] 2s1 number 26 is 1s22s22p63s23p63d8. Therefore, group
11: [Ne] 3s1 number of the element = Number of electrons in the
37: [Kr] 4s1 valence shell = 8
For option (b): All the elements belong to Group 2
11. Write the period number, group number and block of the
12: [Ne] 3s2 element having atomic number 42.
38: [Kr] 4s2 (a) 5, 5, d (b) 5, 6, d
56: [Xe] 5s2 (c) 5, 2, d (d) 5, 15, p
For option (c): The elements do not belong to the same group.
Solution
8: [He] 2s2 2p6
16: [Ne] 3s2 3p4 (b) Mo has atomic number 42 with electronic configuration
24: [Ar] 4s2 3d4 [Kr]5s14d5. Its period number is 5 as it is the highest princi-
pal quantum number in the electronic configuration and
For option (d): All the elements are noble gases.
group number is 6 (sum of total valence shell electrons).
10: [Ne]
18: [Ar] 12. Among the elements with the following atomic numbers,
54: [Xe] which are d-block elements?
(I) 29 (II) 81 (III) 46 (IV) 58
6. Which of the following element is just below in the electronic (a) (I), (IV) (b) (IV), (II)
configuration of [Ne]3s23p64s23d104p3 (c) (I), (III) (d) (II), (III)
(a) As (b) Ge
(c) Sb (d) Sn Solution
Solution (c) For I: The atomic number is 29 and the electronic configu-
ration: [Ar]3d104s1 is that of a d-block element (Cu).
(c) The above electronic configuration given is of As. The
For II: The atomic number is 81 and the electronic config-
element just below the As is Sb.
uration: [Xe] 4f145d106s26p1 is that of p-block element (Tl).
7. Find the number of valence shell electrons in Fe. For III: The atomic number is 46 and the electronic con-
(a) 2 (b) 3 figuration: [Kr] 4d85s2 is that of a d-block element (Pd).
(c) 5 (d) 8 For IV: The atomic number is 58 and the electronic con-
Solution figuration: [Xe] 4f15d16s2 is that of a f-block element (Ce).
(d) The electronic configuration of Fe is as follows: [Ar] 4s23d6 13. If the atomic number of an inert gas element is Z, then an ele-
4s2 3d6. Thus, the valence shell electrons in Fe are 8. ment with which of the following electronic configurations
will have the highest electronegativity (according to Pauling
8. Chloride of an element A gives neutral solution in water. In scale)?
the periodic table, the element A belongs to (a) Z − 1 (b) Z − 2 (c) Z + 1 (d) Z + 2
(a) first group (b) third group
(c) fifth group (d) first transition series Solution
Solution (a) The element having atomic number (Z − 1) is a halogen
and has the highest electronegativity in that period.
(a) In the periodic table, the element A belongs to first group
and the element A may be either sodium or potassium. Periodic Trends in Properties
9. Which of the following is most metallic? 14. Which one of the following sets of ions represents a collec-
(a) C (b) Pb tion of isoelectronic species?
(c) Si (d) Sn (a) K+, Cl−, Ca2+, Sc3+ (b) Ba2+, Sr2+, K+, S2−
(c) N2−, O2−, F−, S2− (d) Li+, Na+, Mg2+, Ca2+
Solution
(AIEEE 2006)
(b) The order of metallic character is as follows: Solution
C Si Ge Sn Pb (a) Isoelectronic species have the same number of electrons
in their valence shell. Option (a) is correct as each species
Non-metal Non-metal Metalloid Metal Metal has 10 electrons in its outermost shell.

As ionization energy (IE) decreases down the group from C to Pb, 15. The increasing order of the first ionization enthalpies of the
the metallic character increases. Pb having large size, has low IE, elements B, P, S and F (lowest first) is

Chapter-3.indd 72 8/8/2016 2:12:30 PM


Solved Examples 73

(a) F < S < P < B (b) P < S < B < F (c) Li+ > Na+ > Mg2+ > Be2+ (d) Mg2+ > Be2+ > Li+ > Na+
(c) B < P < S < F (d) B < S < P < F (AIEEE 2009)
(AIEEE 2006) Solution
Solution
(b) This is because in a period, the ionic radius decreases as
(d) The Zeff increases in a period, hence IE increases. But in case we move from left to right because of increase in nuclear
of P and S, P has higher ionization enthalpy as it contains charge and also the value of principal quantum number
half-filled electronic configuration while S achieves half- remains the same. Whereas, in a group, the size increases
filled electronic configuration on the loss of one electron. as we move from top to bottom with increase in atomic
16. Following statements regarding the periodic trends of chem- number, as the atom gets larger due to increase in the
ical reactivity of the alkali metals and the halogens are given. number of shells.
Which of these statements gives the correct picture? 20. In which of the following arrangements, the sequence is not
(a) The reactivity decreases in the alkali metals but increases strictly according to the property written against it?
in the halogens with increase in atomic number down (a) CO2 < SiO2 < SnO2 < PbO2: increasing oxidizing power.
the group. (b) HF < HCl < HBr < HI: increasing acid strength.
(b) In both the alkali metals and the halogens the chemi- (c) NH3 < PH3 < AsH3 < SbH3: increasing basic strength.
cal reactivity decreases with increase in atomic number (d) B < C < O < N: increasing first ionization enthalpy.
down the group. (AIEEE 2009)
(c) Chemical reactivity increases with increase in atomic num- Solution
ber down the group in both the alkali metals and halogens.
(c)
(d) In alkali metals the reactivity increases but in the halo-
For option (a): Increasing oxidizing power: CO2 < SiO2 < SnO2
gens it decreases with increase in atomic number down
< PbO2. This is because Pb in (+ 4) oxidation
the group.
state is the strongest oxidizing agent.
(AIEEE 2006)
Solution PbO + 4HCl → PbCl + Cl2 + H2O
(d) The alkali metals are highly reactive because their first For option (b): Increasing acid strength: HF < HCl < HBr < HI.
ionization potential is very low and hence they have Higher bond dissociation energy, lower is the
great tendency to lose electron to form unipositive ion. degree of ionization and hence weaker is the acid
While the reactivity of halogen decreases on moving and the bond dissociation energy is highest for HF.
down the group due to their increase in effective nuclear For option (c): Increasing order of basic strength: SbH3 < AsH3 <
charge. PH3 < NH3. As the size of central atom increases,
the electron density on the central metal atom
17. The ionic mobility of alkali metal ions in aqueous solution is decreases and consequently its tendency to donate
maximum for a pair of electrons decreases. Hence, the basic
(a) K+ (b) Rb+ strength decreases as we move from NH3 to BiH3.
(c) Li + (d) Na+ For option (d): Increasing ionization enthalpy: B < C < O < N.
(AIEEE 2006) Nitrogen has half-filled orbitals which have
Solution extra stability so removal of electron is difficult
(b) On going down the group the ionic radii of metal ions as compared to B, C, O.
goes on increasing but the size of hydrated ion goes 21. The correct sequence which shows decreasing order of the
on decreasing. Therefore the movement of larger ion in ionic radii of the elements is
aqueous solution is easier than the movement of smaller (a) O2− > F− > Na+ > Mg2+ > Al3+
ion in aqueous solution. Thus, ionic mobility increases on (b) Al3+ > Mg2+ > Na+ > F− > O2−
going down the group. (c) Na+ > Mg2+ > Al3+ > O2− > F−
(d) Na+ > F− > Mg2+ > O2− > Al3+
18. Which one of the following constitutes a group of the isoelec-
(AIEEE 2010)
tronic species?
Solution
(a) C22 − , O2− , CO, NO (b) NO + , C22 − , CN− , N2
(a) For isoelectronic species, radii would be different because
(c) CN− , N2 , O 22 − , CO22 − (d) N2 , O2− , NO + , CO of different nuclear charges. The cation with the greater
(AIEEE 2008) positive charge will have smaller radius because of the
greater attraction of electron to the nucleus. Anions with
Solution greater negative charge will have larger radius because

(b) NO+, C2−
2 , CN and N2. here the net repulsion of electrons will outweigh the
The number of electrons is nuclear charge and ion will expand in size.
NO + = 7 + 8 − 1 = 14; C2− = 6 + 6 + 2 = 14 ; CN− = 6 + 7 + 1 = 14;
22. The increasing order of the ionic radii of the given isoelec-
N2 = 7 × 2 = 14 tronic species is:
(a) Cl−, Ca2+, K+, S2− (b) S2−, Cl−, Ca2+, K+
19. The set representing the correct order of ionic radius is: + −
2+
(c) Ca , K , Cl , S 2− (d) K+, S2−, Ca2+, Cl−
(a) Li+ > Be2+ > Na+ > Mg2+ (b) Na+ > Li+ > Mg2+ > Be2+
(AIEEE 2012)

Chapter-3.indd 73 8/8/2016 2:12:34 PM


74 Chapter 3 Classification of Elements and Periodicity in Properties

Solution 26. The ionic radii (in Å) of N3−, O2− and F− are, respectively,
1 (a) 1.36, 1.71 and 1.40 (b) 1.71, 1.40 and 1.36
(c) Size ∝
( Z /e ) (c) 1.71, 1.36 and 1.40 (d) 1.36, 1.40 and 1.71
Ca2+ K+ Cl− S2 − (JEE Main 2015)
Z 20 19 17 16 Solution
e 18 18 18 18 (b) All of these are isoelectronic species, that is, they contain
Size is increasing

 → the same number of electrons.
← 
(
Z/e ncreasing
) is i As the negative charge increases, ionic radius increases
Also, the size of anion is greater than the parent atom and in the order N3− > O2− > F−.
the size of the cation is smaller than the parent atom. 27. In which of the following arrangements the order is NOT
23. Which of the following represents the correct order of according to the property indicated against it?
increasing first ionization enthalpy for Ca, Ba, S, Se and Ar? (a) Al3+ < Mg2 + < Na+ < F − Increasing ionic size
(a) S < Se < Ca < Ba < Ar
(b) Ba < Ca < Se < S < Ar (b) B < C < N < O Increasing first ionization
(c) Ca < Ba < S < Se < Ar enthalpy
(d) Ca < S < Ba < Se < Ar (c) I < Br < F < Cl Increasing electron gain
(AIEEE 2013) enthalpy
Solution (d) Li < Na < K < Rb Increasing metallic radius
1 (with negative sign)
(b) IE ∝
size Solution
Ionization enthalpy increases moving from left to right and (b) The Zeff increases in a period, hence IE increases. Thus,
decreases moving from top to bottom in a periodic table. the correct order is B < C < O < N.
Thus the correct order of increasing first ionization enthalpy N has half-filled electronic configuration and hence
is Ba < Ca < Se < S < Ar. higher IE values, while O achieve half-filled configuration
24. Which of the following series correctly represents relations on loss of one electron.
between the elements from X to Y? 28. Which one of the following groupings represents a collection
X→Y of isoelectronic species?
(a) 3Li → 19K: Ionization enthalpy increases. (a) Na+, Ca2+, Mg2+ (b) N3−, F−, Na+
(b) 9F → 35Br: Electron gain enthalpy with negative sign (c) Be, AI , CI
3+ − (d) Ca2+, Cs+, Br
increases.
(c) 6C → 32Ge: Atomic radii increases. Solution
(d) 18Ar → 54Xe: Noble character increases. (b) Isoelectronic species contain same number of electrons.
(JEE Main 2014) For N3− number of electrons = 7 + 3 = 10
Solution For F− number of electrons = 9 + 1 = 10
For Na+ number of electrons = 11−1 = 10
(c) 3 Li → 19K: Ionization enthalpy decreases.
9 → 35Br: Electron gain enthalpy with negative sign
F 29. Which one of the following ions has the highest value of
decreases. ionic radius?
6 C → 32Ge: Moving top to bottom atomic radii increases. (a) Li+ (b) F– (c) O2– (d) B3+
18 Ar → 54Xe: Noble character decreases not increases.
Solution
25. Which of the following arrangements represents the increas-
(c) The value of ionic radius depends upon electron/proton
ing order (smallest to largest) of ionic radii of the given spe-
ratio (e/p) ratio.
cies O2−, S2−, N3−, P3−?
For O2–, the ratio is 10/8 = 1.25 which is highest among
(a) O2− < N3− < S2− < P3−
others.
(b) O2− < P3− < N3− < S2−
(c) N3−< O2− < P3− < S2− 30. Which one of the following sets of ions represents the collec-
(d) N3− < S2− < O2− < P3− tion of isoelectronic species?
(JEE Main 2014) (a) K+, Ca2+, Sc3+, Cl– (b) Na+, Mg2+, Al3+, Cl–
Solution (c) K+, Cl−, Mg2+, Sc3+ (d) Na+, Ca2+, Sc3+, F–
(a) With increase in charge, size increases. For an isoelec- Solution
tronic species, species with higher Z/e ratio has smaller
(a) Isoelectronic species contains same number of electrons.
size.
2− 3− 2− 3−
For K+ number of electrons are 18 (19 − 1)
O N S P For Ca2+ number of electrons are 18 (20 − 2)
Z 8 7 16 15 For Sc3+ number of electrons are 18 (21 − 3)
e 10 10 18 18 For Cl− number of electrons are 18 (17 + 1)

Chapter-3.indd 74 8/8/2016 2:12:36 PM


Solved Examples 75

31. Of the following sets which one does NOT contain isoelec- Solution
tronic species? (d)
(a) PO34− , SO24− , CI4− (b) CN , N2 , C22 −

Element Atomic Number Number of Electrons
(c) PO24− , CO23 − , NO3− (d) BO33 − , CO23 − , NO3− Cl 17 17

Solution P 15 15

(c) In PO42− the number of electrons = 15 + (8 × 4) + 2 = 49 Mg 12 12


In CO32− the number of electrons = 6 + (8 × 3) + 2 = 32 Ca 20 20
In NO3− the number of electrons = 7 + (8 × 3) +1 = 32
As the Zeff increases along a period, the atomic radius
Since the number of electrons is not the same, thus, these
decreases, so the order is Mg < P < Cl. Down the group, there
are not isoelectronic.
is increase in the number of shells, hence the radius increases.
32. Which of the following arrangements represents the increas- So, the order is Mg < Ca.
ing order of ionic radii?
(a) Mg2+ < F− < O2− < N3− (b) N3− < O2− < F− < Mg2+ 36. The correct order of increasing atomic radii among Se, O, S
− 2−
(c) F < O < N < Mg3− 2+ (d) O2− < F− < Mg2+ < N3− and As is
(a) As < S < Se < O (b) Se < O < S < As
Solution (c) O < S < Se < As (d) S < Se < As < O
(a) Solution
Element Number of Protons Number of Electrons (c) From O to Se group, the trend for atomic size is O < S < Se.
Mg2+ 12 10 Since As comes before Se in the period, so is bigger in size
than Se as in a period the size decreases from left to right.
F− 9 10
So, the correct trend is given in option (c).
O2− 8 10
N3− 7 10 37. Which one of the following represents the order of the
decreasing sizes in the given four isoeletronic species?
Ionic radius depends on the number of nuclear charge and (a) F− > N3− > O2− > C4− (b) C4− > N3− > O2− > F−
the number of electrons. For isoelectronic elemental species, 3− 4−
(c) N > C > F > O− 2− (d) O2− > N3− > C4− > F−
as the number of protons increase, Zeff increases and the out- Solution
ermost electrons are more strongly attracted. Hence, the size
will decrease with increasing nuclear charge for isoelectronic (b) We have
species. Nuclear Charge (Number Number of
Ions
of Protons) Electrons
33. Which of the following represents the correct order of the
least negative to the most negative electron gain enthalpy for C4− 6 10
C, Ca, Al, F and O? N3− 7 10
(a) C < O < F < Al < Ca (b) O < F < C < Ca < Al
(c) Al < Ca < F < O < C (d) Ca < Al < C < O < F O2− 8 10

Solution F− 9 10

(d) Calcium has electronic configuration of [Ar]4s2 and The order of ionic sizes is C4− > N3− > O2− > F−. For the isoe-
since it has filled s-orbital, it does not show a tendency lectronic species, greater the nuclear charge (number of
to attract electrons and has the least negative electron protons), lesser is the ionic or atomic size.
gain enthalpy. In a period, the electron gain enthalpy
with negative sign increases from left to right, and thus 38. The electronegativity of H, O and X are 2.1, 3.5 and 0.7, respec-
increases from carbon through oxygen to fluorine. tively. The correct nature of compound X−O−H is
(a) acidic. (b) basic.
34. If electronegativity of A is 2.0 and that of B is 3.0, what is the
(c) amphoteric. (d) cannot be predicted.
covalent character percentage of the bond A−B?
(a) 19.5% (b) 80.5% (c) 46% (d) 54% Solution
Solution (b) The polarity of X−O bond greater than O−H bond
(b) Percentage of ionic character = 16( cA − cB) + 3.5(cA − cB)2 because the electronegativity difference of X−O bond
= 16(1) + 3.5(1)2 = 19.5% (3.5 − 0.7 = 2.8) is greater than that of O−H bond (3.5
% covalent character = 100 − 19.5 = 80.5% − 2.1 = 0.4). Hence, X−O bond will be broken first, thus
X−O−H is basic in nature.
35. The four elements Ca, Mg, P and Cl are to be arranged in the
order of their increasing atomic sizes. Which of the following 39. Which of the following set of species are isoelectronic in
orders is correct? nature (i.e. having same number electrons)?
− −
(a) Cl < P < Mg < Ca (b) P < Cl < Ca < Mg (a) NO3 , BO3 (b) NO2− , BO3−
2− 2−
(c) Ca < Mg < Cl < P (d) Mg < Ca < P < Cl (c) SO 4 , SO3 (d) H2S, HCl

Chapter-3.indd 75 8/8/2016 2:12:39 PM


76 Chapter 3 Classification of Elements and Periodicity in Properties

Solution Statement (D): The order of IE for an electron present in a


(d) The number of electrons in each set is particular orbital having same principal quantum number
(a) NO3−NO − − −
:, BO (n) is ns > np > nd > nf because electron is closest to the
3 ,3BO3 : 30
32,
nucleus in s orbital while farthest is f orbital. Hence, (d) is
(b) NO2−NO − − −
:, 24,
BO
2 ,3BO3 : 32 incorrect.
− 2− 2− 2−
(c) SO24SO:, 50,
4 ,3SO3 : 42
SO 44. Which of the following has the largest ionic radius?
(d) H2S: 18, HCl: 18 (a) Li+ (b) K+ (c) Na+ (d) Cs+
40. Which of the following species is having maximum numeri- Solution
cal value of their electron affinity?
(d) The ionic radius is linked with the atomic radius and it
(a) Li+ (b) N+ (c) B+ (d) O+
varies accordingly. Thus, the ionic radius always increases
Solution down the group and decreases along the period provided
IE1 Li Be theBions involved
C N have O theFsameNe magnitude of charge.
(b) A  A+
EA 5.4 9.3 8.3 11.3 14.5 13.6
Increase in 17.4
ionic 21.6
radii

Hence, |IE1 of A| = |EA| of A+ Li+ Na+ K + Rb + Cs +
Ionization energy increases
Among the given species N has highest IE1 value hence N+ On moving down the group, two effects take place −
has highest numerical value of EA. increase in nuclear charge and addition of energy shells.
41. Which of the following is correct with respect to the ioniza- Out of these, the latter is more predominant. Therefore,
tion energy. atomic radii as well as ionic radii increase on going down
(a) Br > Se (b) Pb > Tl the group from top to bottom.
(c) Ca > Ga (d) All of these 45. Which of the following order for ionization enthalpy is
correct?
Solution
(a) Be > B > C > N > O (b) B < Be < C < O < N
(d) The Zeff increases in a period, hence IE increases ,thus, (a) (c) B < Be < C < N < O (d) B < Be < N < C < O
and (b) are correct.
Ca:[Ar]4s2 : Filled subshell Solution
Ga: [Ar]3d104s24p1: After losing one electron it achieves (b) In a period, as we move from left to right, the ionization
the ns2 configuration. enthalpy of the elements increases due to successive
Hence, Ca > Ga increase in the nuclear charge (i.e., atomic number) and
decrease in atomic size. However, there are certain ele-
42. The size of isoelectronic species F−, Na+, and Mg2+ is affected ments that show irregular trends.
by Li Be B C N O F Ne
(a) nuclear charge.
5.4 9.3 8.3 11.3 14.5 13.6 17.4 21.6
(b) valence principal quantum number (n).
Ionization energy increases
(c) electron−electron repulsion in the outer orbitals.
(d) none of these. The higher values of ionization potential of Be and N as
compared to B and O can be explained as follows:
Solution (i). In case of Be, it is more difficult to remove an election
(a) As the nuclear charge increases for a set of isoelectronic from the completely filled 2s orbital while in case of
ions, the attractive force between the electrons and B it is easier to remove the same from a partially-
nucleus also increases, thereby decreasing the size of the filled 2p orbital. Since to remove an electron from a
ionic species. 2s orbital of Be atom requires more energy than to
remove the same from a 2p orbital of B atom, ioniza-
43. Choose the incorrect statements from the following: tion potential of Be is higher than that of B.
(A) Ionization energy of atoms may be negative. (ii) In case of N, it is more difficult to remove an elec-
(B) Ionization energy of an ion may be negative. tron from the half-filled 2p orbital while in case of
(C) Ionization energy is inversely proportional to radius. O it is easier to remove the same from a paired 2px
(D) Electron present in p orbital is in need of less energy for orbital. Thus the ionization potential of N is higher
its removal as compared to electron present in d orbital than that of O.
having the same principal quantum number. Therefore, the correct order of ionization enthalpy is
(a) (B), (D)( (b) (A), (C) B < Be < C < O < N.
(c) (A), (D)) (d) (B), (C) 46. The first four successive ionization energies for an element
Solution are 6.113, 11.871, 50.908, 67.01 (in eV) respectively. The num-
ber of valence shell electron is
(c) Statement (A): M → M+; ∆H = (+)ve always, where M is any
(a) 3 (b) 4 (c) 2 (d) 1
elemental atom. Hence, (A) is incorrect.
Statement (B): N– → N; ∆H = (−)ve because N → N–; ∆H = Solution
(+)ve, where N is nitrogen atom. Hence, (B) is correct. (c) The sudden jump in values of successive ionization
Statement (C): IE decreases with increase in radius as the energy value indicates that valence shell electrons are
electrons become more loosely held. Hence, (C) is correct. already lost before this jump.

Chapter-3.indd 76 8/8/2016 2:12:40 PM


Solved Examples 77

47. The electronegativities of two elements A and B are cA and cB 50%. Thus, H−X bond is 50% ionic and 50% covalent
respectively and the relationship between them is given by and hence is represented as H−X.
cA ∼ cB = 0.102 ∆ (ii) When (cH − cX) < 1.7, the amount of ionic character in
A −B
where ∆A–B is the resonance energy expressed in kJ mol–1. H−X bond is less than 50% and that of covalent char-
If ∆A–B is zero, then which of the following statements is acter is more than 50%. Thus, H−X bond is predomi-
true? nantly covalent.
(a) cA > cB (b) cA < cB (iii) When (cH − cX) > 1.7, the amount of ionic character
(c) cA = cB (d) Cannot be predicted in H−X bond is more than 50% and that of covalent
character is less than 50%. Hence, H−X bond is pre-
Solution dominantly ionic and is represented as H+X−.
(c) It is mathematically correct to predict that cA = cB when
∆A–B is zero. Chemical Reactivity
48. Ease of formation of the anion is favoured by 52. Which one of the following order represents the correct
(a) lower value of ionization potential. sequence of the increasing basic nature of the given oxides?
(b) lower value of electron gain enthalpy. (a) MgO < K2O < Al2O3 < Na2O
(c) higher value of electron gain enthalpy. (b) Na2O < K2O < MgO < Al2O3
(d) lower value of electronegativity. (c) K2O < Na2O < Al2O3 < MgO
(d) Al2O3 < MgO < Na2O < K2O
Solution (AIEEE 2011)
(c) Ease of formation of the anion is favoured by higher value
Solution
of electron gain enthalpy. Halogens have the maximum
negative electron gain enthalpies in their respective peri- (d) Across a period, metallic strength decreases, and hence
ods. So, they easily form anions. the basic character also decreases.

49. On moving from left to right in a period in transition metals, 53. Which of following set of oxides are amphoteric in nature?
their atomic size (a) N2O5, B2O (b) BeO, Al2O3
(a) decreases. (c) K2O, Al2O3 (d) B2O3, SiO2
(b) increases. Solution
(c) remains the same.
(b) Be and Al shows diagonal relationship and both oxides
(d) none of these.
BeO and Al2O3 are amphoteric in nature
Solution Similarly K2O : basic = oxides of metals are in general basic.
(a) Generally, the atomic size of transition elements in a N2O5 : Acidic = Oxides of non-metals are in general acidic.
series decrease on moving from left to right in a period, B2O3 : Acidic = Oxides of non-metals are in general acidic.
but the decrease in atomic size is small after midway. SiO2 : Acidic = Oxides of non-metals are in general acidic.
50. Increasing order of atomic radii is 54. Among Al2O3, SiO2, P2O3 and SO2 the correct order of acid
(a) Mg2+ < Na+ < F− < O2− strength is
(b) Na+ < F− < Mg2+ < O2− (a) SO2 < P2O3 < SiO2 < Al2O3
(c) Mg2+ < F− < Na+ < O2− (b) Al2O3 < SiO2 < P2O3 < SO2
(d) Na+ < Mg2+ < O2− < F− (c) Al2O3 < SiO2 < SO2 < P2O3
(d) SiO2 < SO2 < Al2O3 < P2O3
Solution
(a) Ionic radii ∝ 1/(Z/e), where Z is nuclear charge and e is Solution
number of electrons. (b) In a periodic table, on moving from left to right the acidic
2+ + − 2− strength of oxides increases.
Mg Na F O
Z 12 11 9 8 55. Which of the following oxides is amphoteric in character?
(a) CaO (b) CO2 (c) SiO2 (d) SnO2
e 10 10 10 10
Zle 12/10 1110
/ 9/10 8/10 Solution
(d) CaO is basic and CO2 and SiO2 are acidic in nature while
All the given ions are isoelectronic in nature. Mg2+ having
SnO2 is amphoteric in nature.
the highest nuclear charge (12 units) has the smallest size
whereas O2− ion having the smallest nuclear charge (8 56. The acidic character of the following oxides of non-metals
units) has the largest size. (S, Cl and Si) follows the order:
51. A molecule H−X will be 50% ionic if electronegativity differ- (a) SO2 < ClO2 < SiO2 (b) SiO2 < SO2 < ClO2
ence of H and X is (c) ClO2 < SiO2 < SO2 (d) SO2 < SiO2 < ClO2
(a) 1.2 (b) 1.4 (c) l.5 (d) 1.7 Solution
Solution (b) Since the oxidation state of all central atoms is the same,
(d) (i) When (cH − cX) = 1.7, the amount of ionic character in hence as the electronegativity difference between two
H−X bond is 50% and that of covalent character is also elements decreases (for oxides), acidic character increases.

Chapter-3.indd 77 8/8/2016 2:12:41 PM


78 Chapter 3 Classification of Elements and Periodicity in Properties

57. Which of the oxides given below in the most basic? (b) SiO2 < SO2 < Al2O3 < P2O3
(a) Li2O (b) Na2O (c) K2O (d) MgO (c) Al2O3 < SiO2 < P2O3 < SO2
(d) SO2 < P2O3 < SiO2 < Al2O3
Solution
(c) Li, Na and K belong to the same group. The basic char- Solution
acter in the group increases from top to bottom in a (c) In a period of the representative elements, the basic
group. So, K2O is the most basic. As far as MgO is con- character of the oxides decreases while acidic character
cerned, it is less basic than Na2O as Mg comes after Na increases. However, the oxides of the elements midway in
in a period. On moving from left to right across a period, a period are amphoteric in nature. The trend in the oxides
the metallic character of the element decreases and the of the elements of the third period is shown as:
oxides of elements become less basic.
Na2O, MgO Al2O3 , SiO2 P2O5 , SO2 , Cl2O7
    
58. Among Al2O3, SiO2, P2O3 and SO2, the correct order of acid Basic Amphoteric Acidic
strength is
(a) P2O3 < Al2O3 < SiO2 < SO2 Hence, the correct order of acid strength is Al2O3 < SiO2
< P2O5 < SO2.

ADVANCED LEVEL PROBLEMS


1. Find the process which is endothermic. (b) → (r) because F− is having all paired up electrons and is
(a) N → N3− (b) Li+ → Li diamagnetic.
(c) H → H − (d) Ca2+ → Ca+ (b) → (s) because F has one unpaired electron while F− has no
Solution unpaired electron.
(a) The process of formation of N3− ion is endothermic. This (c) → (p) because the energy is released for H → H−.
is due to the fact that adding two more electrons cause (c) → (r) because H– has 1s2 configuration and is diamagnetic.
electrostatic repulsion between the extra electrons being (c) → (s) because H initially was paramagnetic with μ = 1.73 BM.
added and the negative charge on N− anion. (d) → (p) because it is overall exothermic as N+ → N is highly
2. Find the electronegativity difference between two atoms A exothermic while N → N− is slightly endothermic and
and B when the percentage ionic character of A B bond is overall N+ → N− is exothermic.
19.5%. 4. The first four successive ionization energies for an element are
(a) 2.45 (b) 1.5 (c) 1.0 (d) zero 6.113, 11.871, 50.908, 67.01 (in eV) respectively. The number of
Solution valence shell electron is ____
(c) According to Haney–Smith equation Solution
% ionic character = 16|Δc | + 3.5|Δc |2 ⇒ 19.5 = 16|Δc | + (2) The sudden jump in values of successive ionization
3.5|Δc |2 energy value indicates that valence shell electrons are
From the expression, it is obvious that Δc = 1, satisfies the already lost before this jump.
equation.
5. Which of the following statements are correct?
3. Match the processes with the characteristic changes. (a) The electron affinity of Si is greater than that of C.
(b) BeO is amphoteric while B2O3 is acidic.
Column I Column II
(c) The ionization energy of Tl is less than that of Al.
(a) Fe2 + → Fe3+ (p) Exothermic in nature (d) The ionization energy of elements of Cu-group is less than
that of the respective elements of Zn-group.
(b) F → F− (q) Endothermic in nature.
Solution

(c) H→H (r) Becomes diamagnetic. (a), (b), (d)
+ − For option (a): Correct because the EA of third period p-block
(d) N →N (s) Magnetic moment undergoes
a change. element is greater than that of second period
element.
Solution
For option (b): It is a fact.
(a) ãq, s; (b) ãp, r, s; (c) ãp, r, s For option (c): IE of Tl > IE of Al. So, it is incorrect.
(a) →(q) because energy is absorbed for Fe2 + → Fe3+
For option (d): Zn-group has fully filled electronic configura-
(a) → (s) because the magnetic moment for Fe2+ is 4.89 BM tion and has higher IE compared to Cu-group.
(i.e., 4 unpaired electrons) while the magnetic moment for
Fe3+ is 5.92 B.M (i.e., 5 unpaired electrons) 6. Among the elements with the following atomic numbers,
(b) → (p) because on addition of first electron to F−, the energy which are d-block elements?
gets released. (a) 29 (b) 81 (c) 46 (d) 58

Chapter-3.indd 78 8/8/2016 2:12:44 PM


Practice Exercise 79

Solution (a) IE2 − IE1 (b) IE3 − IE2


(a), (c) (c) IE3 − IE1 (d) cannot be predicted
In option (a): 29 is the atomic number of Cu. Solution
In option (c): 46 is the atomic number of Pd. IE 3
(c) A IE
1
→ A + 
IE 2
→ A2 +  → A3+
Passage for Questions 7, 8, 9: Ionization energy is always positive
for an element and successive ionization energies always gradually Since IE values are successively increasing, so IE3 − IE1 will
increase. be the maximum.
7. The successive ionization energy for an element is given below 9. Choose the incorrect ionization energy order
in kJ mol−1. (a) Al > Ga (b) TI > In (c) Pb > Sn (d) Y < Sc
0.7865, 1.5771, 3.2316, 4.3555, 16.091, 19.785, 23.786, 29252 Solution
Predict the number of valence shell electrons in the given
element. (a) The correct ionization energy order is Al < Ga.
(a) 1 (b) 2 (c) 3 (d) 4 10. The number of elements among the following atomic num-
Solution bers 83, 79, 42, 64, 37, 54, 34 that are p block elements is
_________.
(d) The sudden jump of IE values is obtained between IE4 > IE5.
Hence the valence shell electrons number is 4. Solution
8. For an element having only one valence shell electron, which (3) 83: Bi, 54: Xe and 34: Se
of the following difference will have the maximum value.

PRACTICE EXERCISE
Level I (a) Dobereiner. (b) Newland.
(c) Lothar Meyer. (d) Mendeleev.
Single Correct Choice Type
10. Who presented the law of octave?
1. The modern Mendeleev’s periodic table consists of _____
(a) Dobereiner (b) Newland
groups.
(c) Lothar Meyer (d) Mendeleev
(a) seven (b) eight (c) nine (d) ten
11. The number of elements in fifth period of periodic table is
2. How many elements were known when Mendeleev presented (a) 8. (b) 32. (c) 18. (d) 19.
the periodic table?
(a) 63 (b) 80 (c) 92 (d) 102 12. Variable valency is a general feature of
(a) s-block elements. (b) p-block elements.
3. The basis of periodic law presented by Mendeleev was (c) d-block elements. (d) f-block elements.
(a) valency. (b) atomic mass.
(c) atomic number. (d) atomic volume. 13. The radii F, F−, O and O2− are in the order of
(a) F− > O2− > F > O. (b) F > F− > O > O2−.
4. Diagonal relationship is not shown by (c) O2− > F− > O > F. (d) F > O > F− > O2−.
(a) Li and Mg. (b) C and P.
(c) B and Si. (d) Be and Al. 14. Which of the following ion has the highest value of ionic
radius?
5. The group number of element in periodic table indicates (a) Li+ (b) F− (c) O2− (d) B3+
(a) valency with respect to hydrogen.
(b) the atomicity. 15. The diagonal relationship between Li and Mg is due to
(c) the number of electrons in the outermost shell. (a) similarity of ionization potential.
(d) the atomic radii. (b) similarity of electronegativity.
(c) similarlity of ionic radii.
6. Which of the following is correct? (d) dissimilar atomic radii.
(a) rionic ∝ Z (b) rionic ∝ Zeffective
(c) rionic ∝ 1/Zeffective (d) None of these 16. The elements with atomic numbers 58 to 71 are called
(a) normal elements. (b) transition elements.
7. Which of the following relation is correct? (IP: Ionization poten- (c) lanthanides. (d) actinides.
tial; EA: Electron affinity: EN: Electronegativity)
(a) 2 IP − EA − EN = 0 (b) 2 IP − EN + EA = 0 17. Correct electronic configuration of Cr is
(c) 2 EN − IP − EA = 0 (d) EN − IP − EA = 0 (a) 1s22s22p63s23p23d44s1.
(b) 1s22s22p63s23d64s0.
8. Which of the following pairs has both members from the same (c) 1s22s22p63s23p63d54s1.
group of periodic table? (d) 1s22p22p63p03d5.
(a) Mg, Ba (b) Mg, Na (c) Mg, Cu (d) Mg, Cl
18. The elements on the right side of the periodic table are
9. The attempt for classifying elements by plotting the atomic (a) metals. (b) metalloids.
masses of elements against the volumes was made by (c) non-metals. (d) transition elements.

Chapter-3.indd 79 8/8/2016 2:12:44 PM


80 Chapter 3 Classification of Elements and Periodicity in Properties

19. ns2np4 (n-outermost orbit) represents the valence electrons. List-I List-II
The corresponding group elements would be i. 19 p. p-block
(a) F, Cl, Br (b) N, P, As ii. 22 q. f-block
(c) O, S, Se (d) C, Si, Ge iii. 32 r. d-block
iv. 64 s. s-block
20. The electronic configuration of an element C is 1s2, 2s2, 2p6.
The formula of substance containing only C will be i ii iii iv
(a) C8. (b) C4. (c) C2. (d) C. (a) p q r s
21. The electronic configuration of an element A is 1s2, 2s2, 2p6, (b) s r p q
3s2 while of the element B is 1s2, 2s2, 2p5. The formula of sub- (c) s p r q
stance containing A and B will be (d) q p r s
(a) AB. (b) A2B. (c) AB2. (d) A2B6. 33. Ionization potential does not depend upon
22. The halogen that is most easily reduced is (a) atomic size.
(a) F2. (b) Cl2. (c) Br2. (d) I2. (b) type of electron .
(c) nuclear charge.
23. Which pair is different from the others?
(d) type of bonding in crystal lattice.
(a) Li−Mg (b) Na−K (c) Ca−Mg (d) B−Al
34. The correct order of increasing order of oxidizing power is
24. The statement that is not correct for the periodic classifica-
(a) F2 < Cl2 < Br2 < I2. (b) I2 < F2 < Cl2 < Br2.
tion of element is
(c) Br2 < I2 < F2 < Cl2. (d) I2 < Br2 < Cl2 < F2.
(a) The properties of elements are periodic functions of
their atomic numbers. 35. Ionization potential of phosphorus is greater than that of sul-
(b) Non-metallic elements are lesser in number than metal- phur because
lic elements. (a) of its smaller size.
(c) Electronegativity of the elements along the period from (b) of more penetrating power of p-orbitals.
left to right increases in a regular manner. (c) its nuclear force of attraction on electrons.
(d) For transition elements the d-subshells are filled with (d) half-filled orbitals are more stable.
the electrons monotonically with increase in atomic
36. The correct order of ionization enthalpy for comparing car-
number.
bon, nitrogen and oxygen is
25. The electronic configuration (a) C < N > O. (b) C > N < O.
1s2, 2s2, 2p6, 3s2, 3p6, 3d10, 4s2, 4p6, 4d10, 5s2 is for (c) C > N > O. (d) C < N < O.
(a) f-block element. (b) d-block element.
(c) p-block element. (d) s-block element. 37. Electropositive or metallic character
(a) increases in a period.
26. Which of the following oxides is neutral? (b) decreases in a group.
(a) SnO2 (b) CO (c) Al2O3 (d) Na2O (c) decreases in a period and increases in a group.
27. Which of the following element has the maximum first ioni- (d) of an element is reflected in its tendency to form cova-
zation potential? lent compounds.
(a) V (b) Ti (c) Cr (d) Mn 38. The electronic configuration of the element which is just
above the element with atomic number 43 in the same peri-
28. Modern periodic table is based on the atomic number of the odic group is
elements. The experiment which proved the significance of (a) 1s2, 2s2, 2p6, 3s2, 3p6, 3d5, 4s2.
the atomic number was (b) 1s2, 2s2, 2p6, 3s2, 3p6, 3d10, 4s2, 4p5.
(a) Mulliken’s oil drop experiment. (c) 1s2, 2s2, 2p6, 3s2, 3p6, 3d6, 4s1.
(b) Moseley’s work on X-ray spectra. (d) 1s2, 2s2, 2p6, 3s2, 3p6, 3d10, 4s1, 4p6.
(c) Bragg’s work on X-ray diffraction.
(d) discovery of X-rays by Rontgen. 39. The heaviest atom is
(a) Ra. (b) U. (c) Pb. (d) Hg.
29. An element having electronic configuration [Ar] 3d2, 4s2
belongs to 40. The electron gain enthalpy values for the halogens show the
(a) d-block. (b) f-block. (c) s-block. (d) p-block. following trend
(a) F < Cl > Br > I. (b) F < Cl < Br < I.
30. Ce (58) is a member of (c) F > Cl > Br > I. (d) F < Cl > Br < I.
(a) s-block. (b) p-block. (c) d-block. (d) f-block.
41. The configuration 1s22s22p53s1 is valid for
31. Elements of which family form anions most readily? (a) ground state of fluorine. (b) excited state of fluorine.
(a) Halogens (b) Alkali metals (c) excited state of neon. (d) excited state of O2– ion.
(c) Oxygen (d) Nitrogen
42. In a given energy level, the order of penetration effect of dif-
32. Match List-I (atomic number of elements) with List-II (posi- ferent orbitals is
tion of element in periodic table) and select the correct (a) f > d < p < s. (b) s = p = d = f.
answer using the codes given below the list (c) s > p > d > f. (d) p > s > d > f.

Chapter-3.indd 80 8/8/2016 2:12:45 PM


Practice Exercise 81

43. Exothermic process is (a) Se2+ (b) Na+ (c) O2− (d) Ga3+
(a) Na + e−→ Na− (b) O + e− → O−
56. In which of the following set of atomic numbers, all elements
(c) O− + e− → O2− (d) Cl− → Cl + e−
are in the same group?
44. Which of the following groups of elements is assigned zero (a) 8, 16, 24 (b) 3, 11, 37
electronegativity? (c) 12, 38, 56 (d) 10, 18, 54
(a) Noble gases (b) Alkali metals 57. Which of the following statement(s) is/are true?
(c) Alkaline earth metals (d) Rare earth elements (a) Ionization enthalpy ∝ 1/(Shielding effect)
45. The electron affinity of nitrogen is lower than that of carbon (b) The first ionization enthalpies of Be and Mg are more
because than ionization enthalpies of B and Al, respectively.
(a) atomic radius of nitrogen is lower than that of carbon. (c) Atomic and ionic radii of niobium and tantalum are
(b) effective nuclear charge in carbon in greater. almost the same.
(c) addition of an electron in N gives 2p4 configuration. (d) Metallic and covalent radii of potassium are 2.3 Å and
(d) nitrogen is gaseous element. 2.03 Å, respectively.
58. Which of the following statement(s) is/are true for Group 1
46. The electronic structure (n − 1)d1−10 ns0−2 is characteristic of
elements?
(a) lanthanoids. (b) actinoids.
(a) Na is regarded as a typical/representative element.
(c) rare-earth elements. (d) transition elements.
(b) Basic character of their oxides increases down the group.
47. The electronic configurations of four elements are given (c) Among alkali metal ions, the degree of hydration is high-
below. Which element does not belong to the same family? est in Li.
(a) [Xe]4f145d106s2 (b) [Kr]4d105s2 (d) In general, electron gain enthalpy values decrease from
(c) [Ne]3s23p5 (d) [Ar]3d104s2 top to bottom in the group.
59. Mark out the correct statement(s).
48. Which of the following is true about the element with atomic (a) Atomic size is a periodic property.
number 18? (b) On moving down the group, effective nuclear charge
(a) It has a very low ionization potential. decreases and thus ionization enthalpy decreases.
(b) It has a very high electron gain enthalpy. (c) On moving across the period, effective nuclear charge
(c) Its molecules are monoatomic. increases and thus ionization enthalpy increases.
(d) Its electronegativity is very high. (d) Atomic size of alkali metals is the highest in the respec-
49. Consider three hypothetical ionic compounds AB, A2B and tive periods.
A2B3 where in all the compounds B is in (−2) oxidation state 60. IE1 of an element is lower than that of IE2 because
and A has a variable oxidation state. What is the correct order (a) it is difficult to remove an electron from cation compared
of lattice energy of these compounds? to from the same element.
(a) A2B > AB > A2B3 (b) A2B3 >AB > A2B (b) the ionization process is endothermic.
(c) AB > A2B > A2B3 (d) A2B3 > A2B > AB (c) the size of cation is smaller than its atom.
50. Choose the incorrect order for acidic strength. (d) all of the above.
(a) CO2 > CO (b) SO2 < SO3
(c) HClO2 > HOCl (d) SiO2 > CO2 Passage Type
Passage for Questions 61 to 63: Isoelectronic species have the
Level II
same number of electrons.
Multiple Correct Choice Type This property is not only restricted to atoms but also observed in
molecules and ions.
51. Which of the following species is isoelectronic with Ne?
(a) H2O (b) C4− (c) Na− (d) NH2− 61. Choose the correct order of radius.
52. Among the elements with the following atomic numbers, (a) O2− > F− (b) Al3+ < Na+
which are d-block elements? (c) Na+ < N3− (d) All of these
(a) 29 (b) 81 (c) 46 (d) 58 62. Which of the following statements is correct for the following
53. Which of the following is (are) the correct order of mobility? isoelectronic series?
(a) Li+ > Na+ > K+ (b) Li+ < Na+ < K+ S2−, Cl−, K+, Ca2+
(c) Na+ > Mg2+ > Al3+ (d) Al3+ > Mg2+ > Na+ (a) The size of the ions initially increases then decreases.
(b) The first ionization energy decreases from left to right.
54. The shielding effect of inner electrons of an atom causes
(c) The first ionization energy increases from left to right.
(a) decrease in ionization enthalpy.
(d) The size of the ions decreases.
(b) increase in atomic size.
(c) increase in ionization enthalpy. 63. Which of the following sets of isoelectronic species does not
(d) decrease in atomic size. have the same shape?
55. Which of the following atoms and ions is (are) isoelectronic (a) H2S and HCl (b) NO3− and CO32 −
with F−? (c) CIO 4− and SO24− (d) NO2+ and CO2

Chapter-3.indd 81 8/8/2016 2:12:46 PM


82 Chapter 3 Classification of Elements and Periodicity in Properties

Passage for Questions 64 and 65: Ionization potential is the (c) Electronegativity (r) Half-filled or fully filled
minimum amount of energy needed to remove the outermost configuration
electron from the gaseous isolated atom. Its unit is eV atom−1 or (s) Screening constant
kJ mol−1. Successive ionization energy is the amount of energy
needed to remove electron successively from a gaseous ion, it 67. Match the elements with their properties
may be termed as IE2, IE3,…. The difference in the values of IE1, Column I Column II
IE2… helps to determine electronic configuration of the elements. (a) K (p) One electron in the s orbital of valence
shell
Element IE1 IE2 IE3 (kcal mol-1) (b) Cu (q) Transition element
(c) La (r) One unpaired electron
P 497 947 1500 (d) Au (s) Member of the 4th period
Q 98 735 1100
68. Match the element with its electronegativity on Pauling’s
R 176 347 1850 scale.
S 296 530 2050 Column I Column II
(a) Carbon (p) 0.8
64. Which of the element (X) can make MX type compound (b) Nitrogen (q) 1.6
(where X is the halogen and M is alkali metal)? (c) Aluminium (r) 2.5
(a) P (b) Q (c) R (d) S (d) Caesium (s) 3.0
65. The order of ionic mobility in aqueous solution of the follow- 69. Match the oxide with its significant property.
ing ions will be
+ 2+ + 2+ Column I Column II
(a) R(2aq ) > S( aq ) (b) S(2aq ) > R( aq )
(a) P4O10 (p) Ionic
+ 2+ (b) SiO2 (q) Covalent
(c) S(2aq ) = R( aq ) (d) cannot be correlated.
(c) Al2O3 (r) Basic
(d) MgO (s) Amphoteric
Matrix-Match Type
Integer Type
66. Match the properties with the parameters that they are
dependent on 70. The effective nuclear charge on the last electron in Sc is ___.

Column I Column II 71. The element with atomic number 82 belongs to Group ___.
(a) Electron affinity (p) Radius
(b) Ionization potential (q) Zeff

ANSWER KEY
Level I
1. (c) 2. (a) 3. (b) 4. (b) 5. (c) 6. (c)
7. (c) 8. (a) 9. (c) 10. (b) 11. (c) 12. (c)
13. (c) 14. (c) 15. (c) 16. (d) 17. (c) 18. (c)
19. (c) 20. (d) 21. (c) 22. (a) 23. (a) 24. (d)
25. (d) 26. (b) 27. (d) 28. (b) 29. (a) 30. (d)
31. (a) 32. (b) 33. (d) 34. (d) 35. (d) 36. (a)
37. (c) 38. (a) 39. (b) 40. (a) 41. (c) 42. (c)
43. (b) 44. (a) 45. (c) 46. (d) 47. (c) 48. (c)
49. (b) 50. (d)

Level II
51. (a), (b), (d) 52. (a), (c) 53. (b), (c) 54. (a), (b) 55. (b), (c) 56. (b), (c), (d)
57. (a), (b), (c), (d) 58. (a), (b), (c), (d) 59. (a), (c), (d) 60. (a), (c) 61. (d) 62. (c)
63. (a) 64. (b) 65. (a) 66. (a)→ p, q, r, s ; (b)→ p, q, r, s; (c)→ p, q, r, s
67. (a)→ p, r, s; (b)→ p, q, r, s; (c)→ q, r; d→ p, q, r 68. (a) → r; (b) → s; (c) → q; (d)→ p
69. (a) → q; (b) → q; (c) → s; (d) → (p), (r) 70. (3) 71. (14)

Chapter-3.indd 82 8/8/2016 2:12:47 PM


Hints and Explanations 83

HINTS AND EXPLANATIONS


Level I 12. (c) d-block elements due to availability of empty d-orbitals
show variable valencies.
Single Correct Choice Type
13. (c) The radius of the anion is always larger than that of the
1. (c) The new Mendeleev’s periodic table consists of nine ver-
atom. An anion or negative ion is formed when a gase-
tical columns called groups. These are designated as
ous atom gains one or more electrons. Since the nuclear
groups 0, first, second, third, fourth, fifth, sixth, seventh,
charge remains unchanged, the electrons in the anion
and eighth. Except for groups 0 and seventh, each group
experience less attraction towards the nucleus compared
is further divided into two sub-groups designated as
with the atom. Therefore, the anionic radius is larger than
A and B.
that of the corresponding atom.
2. (a) When Mendeleev started his work on classification of ele-
14. (c)
ments, only 63 elements were known.
(i) The size of the anion increases with the increase in the
3. (b) According to periodic law presented by Mendeleev, the magnitude of negative charge.
physical and chemical properties of elements are a periodic (ii) The size of the cation decreases with the increase in the
function of their atomic weights, i.e., when the elements magnitude of positive charge.
are arranged in order of their increasing atomic weights, In O2−, the magnitude of negative charge is maximum,
elements with similar properties are repeated after certain hence the ionic radius has the highest value.
regular intervals.
15. (c) The diagonal relationship between Li and Mg is due to
4. (b) Some elements of the second period show similarities with similarity of ionic radii.
the elements of the third period placed diagonally to each
16. (d) The elements 58 to 71 are known as actinides.
other, though belonging to different groups. Lithium (of
Group 1) resembles magnesium (of group 2) and beryllium 17. (c) The electronic configuration of Cr is
(of Group 2) resembles aluminium (of group 13) and so 1s22s22p63s23p63d54s1
as. This similarity in properties of elements placed diago-
18. (c) Metals are present on the left side as well as in the centre
nally to each other is called diagonal relationship. Among
of the periodic table.
the options which do not show diagonal relationship is C
(i) Transition elements are present at the centre of the
and P.
periodic table.
5. (c) The group number of element in periodic table indicates (ii) Non-metals are mostly located on the right side of the
the number of electrons in the outermost shell. periodic table.
(iii) Metalloids constitute some border line elements.
6. (c) Ionic radius is inversely proportional to effective nuclear
charge (Zeff ). As Zeff increases the ionic radii will decrease. 19. (c) ns2np4 electronic configuration refers to Group 16. The
elements of group 16 are O, S, Se.
1
rionic ∝
Z effective 20 (d) The electronic configuration of element C refers to noble
gas. Thus, the formula of the substance will be C.
7. (c) According to Mulliken, the electronegativity of an atom is
the average of the two values, that is, electron affinity and 21. (c) The formula of substance containing A and B will be AB2.
ionization energy.
22. (a) Since F2 has high E° value of 2.87 V, it has very high oxidiz-
EN = (IP + EA)/2 ing power and thus can be easily reduced.
⇒ 2EN − IP − EA = 0
23. (a) Li−Mg shares diagonal relationship with each other.
8. (a) Mg and Ba belong to the Group 2 of the periodic table. 24. (d) d-subshells are not filled monotonically, regularity breaks
at chromium and copper.
9. (c) Lothar Meyer, a German chemist, studied the physical prop-
erties such as atomic volume, melting point and boiling 25. (d) The last electron is entering the s-orbital. So, the electronic
point of various elements. He plotted a graph between the configuration belongs to the element of the s-block.
atomic volumes (gram atomic weight divided by density)
and atomic weights of the elements and observed that the 26. (b) CO is a neutral oxide
elements with similar properties occupied similar positions SnO2 is an amphoteric oxide.
on the curve. Al2O3 is an amphoteric oxide.
Na2O is a basic oxide.
10. (b) Newland presented the law of octave.
27. (d) The increase in ionization potential is primarily due to
11. (c) There are 18 elements in the fifth period of the periodic increase in nuclear charge which would tend to attract
table. The first two of them are from s-block, the next 10 the electron cloud with greater force, thus making it diffi-
are from d-block and the last six are from p-block. cult to remove the electron.

Chapter-3.indd 83 8/8/2016 2:12:47 PM


84 Chapter 3 Classification of Elements and Periodicity in Properties

28. (b) Moseley measured the frequencies of X-rays emitted by of an atom with higher nuclear charge. This means that
a metal when bombarded with high speed electrons and greater the magnitude of nuclear charge, higher or the
concluded that atomic number was a better fundamen- value of ionization potential.
tal property of an element than its atomic weight. He, The abnormal value of I.E. shown by N can be explained as
therefore, suggested that atomic number (Z) instead of follows: According to Hund’s rule, half-filled (ns1np3nd5) or
atomic weight should be the basis of classification of the completely filled (ns2np6nd10) orbitals are compara-
elements. This forms the basis of modern periodic law. tively more stable and, hence, more energy is needed
29. (a) The given electronic configuration is of titanium. Since, to remove an electron from such orbitals. Thus, nitrogen
the last electron enters into d-orbital, thus, it belongs to (Z = 7; 1s22s22p3) has higher value of ionization potential
d-block. than oxygen (Z = 8; 1s22s22p4).

30. (d) The electronic configuration of Ce is 1s2 2s2 2p6 3s2 3p6 37. (c) Electropositive or metallic character decreases in a
4s2 3d10 4p6 5s2 4d10 5p6 6s2 5d1 4f1 period and increases in a group.
Since, the last electron goes into the f-orbit, thus, it 38. (a) The element which is just above the element with atomic
belongs to the f-block of the periodic table. number 43 in the same periodic group is manganese. Its
31. (a) X + e− → X− electronic configuration is 1s2, 2s2, 2p6, 3s2, 3p6, 3d5, 4s2.
(Halogens) (Anion)
ns2 , np 5 ns2 , np 6 39. (b) Uranium has the highest atomic mass among the given
elements, hence, it is the heaviest element.
The valence-shell configuration of halogen atoms (ns2,
np5) has an appetite for one electron to stabilize its con- 40. (a) Halogens have very high values of negative electron
figuration by attaining the stable noble gas configuration gain enthalpies. These are the maximum in their respec-
(ns2, np6). Thus, halogen atoms have a strong tendency tive periods and decrease down the group.
to accept an extra electron and hence very high values Element F Cl Br I At
of electron affinity. So, elements of halogen family form
anions most readily. Electron gain
−332.6 −348.5 −324.7 −295.5 −270
enthalpy
32. (b)
The negative electron gain enthalpy of chlorine is more
Electronic Configura-
Atomic Number Block than that of fluorine while it is expected to be less.
tion
This is because the atomic size of fluorine atom (atomic
19 3s2 3p6 4s1 s radius = 72 pm) is smaller than that of chlorine atom
22 3p6 4s2 3d2 d (atomic radius = 99 pm). As a result, there is large elec-
32 3d10 4s2 4p2 p tron crowding in the small space of the fluorine atom
and electronic repulsions are also large. The incoming
64 6s2 5d1 4f7 f
electron experiences less attraction towards the nucleus
33. (d) Ionization potential depends upon the following factors: of the fluorine compared to that of chlorine atom which
Atomic size: Ionization potential decreases as the atomic is bigger with less electron crowding.
size increases.
Penetration effect of the electrons: Ionization poten- 41. (c) Electronic configuration of Ne is 1s2 2s2 2p6. In excited
tial increases as the penetration effect of the electrons state, one electron gets transferred to the 3s orbital.
increases. In the same shell, the penetration effect Therefore, excited state electronic configuration of Ne is
decreases in the order : s > p > d > f. 1s2 2s2 2p5 3s1.
Nuclear charge: The ionization potential increases with 42. (c) s-electrons of any shell are more penetrating towards the
increase in nuclear charge. nucleus than p-electrons of the same shell followed by
34. (d) Halogens have strong electron accepting tendencies, so d and f.
they are powerful oxidizing agents. Electron gain enthal-
43. (b) When an electron is added to an oxygen atom, O− ion
pies decrease down the group, so the oxidizing power
is formed with the release of energy due to attraction
also decreases.
towards the incoming electron.
35. (d) Phosphorus contains exactly half-filled orbitals, and we
know that exactly half-filled or completely filled orbitals 44. (a) Every atom strives to gain 8 electrons in its outermost
have extra stability. So, the removal of an electron from shell. Since electronegativity measures the amount of
phosphorus atom requires more energy than sulphur. attraction between an atom and an electron, noble
gases do not have electronegativity.
36. (a) Greater is the magnitude nuclear charge (i.e., number of
protons or atomic number), greater is the electrostatic 45. (c) In nitrogen, the 2p sublevel is exactly half-filled which
force of attraction exerted by the nucleus on the outer- imparts stability, one more or less electron makes the
most electrons. Thus, it would be more and more diffi- electron distribution unsymmetrical, and destabilizes
cult to remove the electrons from the outermost shell the electron distribution.

Chapter-3.indd 84 8/8/2016 2:12:48 PM


Hints and Explanations 85

46. (d) Elements in which the last electron enters any one of 54. (a), (b) A valence-shell electron in a multi electron atom exp-
the five d-orbitals of their respective penultimate shells eriences an attractive force from the nucleus and a repulsive
are called d-block or transition elements. The atoms of force from their inner-shell electrons. The combined effect of
the elements belonging to these groups usually have 1 these two forces is that the attractive force (also called pull or
or 2 electrons in the s-orbital of the ns orbital while the hold) exerted by the nucleus on the valence-shell electron is
remaining electrons are progressively filled in the d-orbit- partially decreased or weakened by the presence of repulsive
als of their respective penultimate shells, that is, (n−1)d
force exerted by the inner-shell electrons. Thus, the electron
orbitals. General outer shell electronic configuration of
in the valence-shell experience less attraction or pull towards
d-block elements is (n−1)d1–10ns0–2, where n = 4−7.
the nucleus and hence ionization potential decreases as well
47. (c) as atomic size increases.

}
[Xe]4f145d106s2: Hg
belong to the same group and 55. (b), (c) Conceptual
[Kr]4d105s2 : Cd
10 2 same family of inert gases.
[Ar]3d 4s : Zn 56. (b), (c), (d) For atomic numbers 8, 16 and 24, the electronic
[Ne]3s23p5: Cl belongs to halogen family. configuration is
48. (c) The atomic number 18 belongs to noble gas family. 1s2 2s2 2p4; 1s2 2s2 2p63s2 3p4; 1s2 2s2 2p63s2 3p64s2 3d6
Electronic configuration is 1s22s22p63s23p6.
The noble gases are monoatomic because they have com- 57. (a), (b), (c), (d)
pletely filled shells and cannot even combine amongst In option (a): Ionization enthalpy ∝ 1/(shielding effect)
themselves. Apart from this, they have exceptionally high In option (b): Because Be and Mg has valence shell electron
ionization enthalpies, very low electron gain enthalpy completely filled (ns2). So IE requires more when
and very low electronegativity. compare to B and Al (ns2np1).
In option (c): Due to lanthanide contraction the atomic and ionic
49. (b) Lattice energy ∝ q1q2
radii of niobium and tantalum are almost the same.
∝ 1/r
Hence, it is maximum for A2B3 and minimum for A2B. In option (d): Because there are weak bonding forces between
the atoms, the metallic radii of K is higher (10−
50. (d) The difference in electronegativity of C and O atom is less 15%) than single covalent radii of potassium.
than that of Si and O atom. Hence, the acidity order is CO2
> SiO2. 58. (a), (b), (c), (d)
In option (b): Basic character of representative element oxides
Level II increases down the group because the ioniza-
Multiple Correct Choice Type tion enthalpies of these metals tend to decrease
and the tendency to from positive ion increases.
51. (a), (b), (d) Isoelectronic species are the atoms or ions that In option (c): because it has the smallest size among alkali
have the same number of electrons and electronic configura- metals.
tions but different magnitude of nuclear charge. In option (d): The electron gain enthalpies of the elements
C4− = 10 electrons H2O = 10 electrons become more negative from top to bottom in a
NH2− = 10 electrons Na− = 12 electrons group.
52. (a), (c) 59. (a), (c), (d) Conceptual
For Z = 29, the electronic configuration is 1s22s22p63s23p6 60. (a), (c) Reasons are in the statement and in option (b), ioniza-
3d104s1 tion of the same anion may be exothermic.
For Z = 81, the electronic configuration is [Xe]6s24f145d106p1
For Z = 46, the electronic configuration is [Kr]5s03d10 Passage Type
For Z = 58, the electronic configuration is [Xe]6s24f2
61. (d) The atomic radius order for O and F is O > F. Apart from that,
The elements with atomic numbers 29 and 46 are d-block ele-
ments because valence shell electron enters into d-orbital. two electrons enter into O atom while only one electron
enters into F atom, and as we know that radius increases
53. (b), (c) The correct order of ionic mobility is Li < Na+ < K+ and on addition of electron. Thus, radius order is O2– > F–. All
Na+ > Mg2+ > Al3+ the given species are isoelectronic and the radius order is
Ionic mobility is depends upon the hydration energy. Li+ ion is N3− > O2− > F− > Na+ > Al3+.
hydrated to the maximum extent due to its high charge/size
ratio compared to the other alkali metals ions. The extent of 62. (c) All are having same number of electron, that is, 18, but
hydration decreases on moving from Li+ to Cs+. Thus, Cs+ ion number of proton increases from left to right, that is, 16
is least hydrated. Thus, the ionic mobility increases from Li+ to for S while 20 for Ca. Hence, IE increases from left to right.
Cs+. 63. (a) H2S has bent shape while HCl is linear.
It also depends on the charge of the ion, if charge is more,
64. (b) For Q there is a large gap between IE1 and IE2. Hence
extent of hydration is also more. As a result, ionic mobility
element Q is an alkali metal which can form QX type of
decreases.
compound.

Chapter-3.indd 85 8/8/2016 2:12:48 PM


86 Chapter 3 Classification of Elements and Periodicity in Properties

65. (a) For R or S, there is large gap between IE2 and IE3. Hence MgO is basic in nature and thus, reacts with acids to form
both are alkaline earth elements. Since IE1 of (S) > IE1 of corresponding salts.
(R), rS2+ < rR2+ and rS2+ ( aq) > rR2+ ( aq) , where r is radius. Hence
MgO → Mg2 + + O2 −

ionic mobility order is R2 + ( aq) > S2 + ( aq). Ionic in nature

MgO + 2HCl → MgCl2 + H2O


Matrix-Match Type
MgO + H2SO4 → MgSO4 + H2O
66. (a) ã p, q, r, s; (b) → p, q, r, s; (c) → p, q, r, s
Integer Type
67. (a) ã p, r, s; (b) ã p, q, r, s; (c) ã q, r; d → p, q, r
70. (3) Zeff = Zactual − Ss
68. (a) ã r; (b) ã s; (c) ã q; (d) ã p
Caesium is electropositive metal, its electronegativity is very The electronic configuration of Sc is 1s22s22p43s23p63d14s2
low. Last electron enters into 3d orbital. So now we will calculate
Electronegativity increases along a period. the screening constant (s ) for 3d orbital
s = (1 × 18)
69. (a) ã q; (b) ã q; (c) ã s; (d) ã (p), (r)
P4O10 and silica (SiO2) are covalent in nature Zeff felt by 3d electron = 21 − (1 × 18) = 3 (there is no contri-
3+ 2−
bution from 4s electron)
Al2O3 → 2Al
 + 
3O
Ionic in nature 71. (14) For Z = 82: [Xe]6s24f145d106p2. Therefore, the element
with atomic number 82 belongs to Group 14.
Alumina (Al2O3) behaves as an amphoteric oxide.
Al2O3 + 6HCl → 2AlCl3 + 3H2O
Heat
Al2O3 + 2NaOH 
→ 2NaAlO2 + H2O

Chapter-3.indd 86 8/8/2016 2:12:49 PM


4 Chemical Bonding

Question Distribution in JEE (Main and Advanced)

3
No. of Questions

JEE (Main)
2
JEE (Adv)

0
2016 2015 2014 2013 2012 2011 2010 2009 2008 2007

Concept Distribution in JEE (Main and Advanced)

Topics Covered
Year
JEE (Main) JEE (Advanced)
Hybridization Theory, Molecular Orbital Theory, Hydrogen
2007 Molecular Orbital Theory
Bonding
2008 Bond Characteristics
2009 Molecular Orbital Theory Molecular Orbital Theory
2010 Bond Characteristics, VSEPR Theory
Polar and Non-polar Covalent Bonds, Hybridization
2011
Theory
2012 Hybridization Theory Bond Characteristics
2013 Bond Characteristics, Molecular Orbital Theory
Polar and Non-polar Covalent Bonds, VSEPR Theory,
2014 VSEPR Theory, Molecular Orbital Theory
Hybridization Theory, Molecular Orbital Theory
Polar and Non-polar Covalent Bonds, Molecular Orbital
2015 Hybridization Theory
Theory
Hybridization Theory, Intermolecular Forces (Secondary
2016 VSEPR Theory, Molecular Orbital Theory
Forces of Attraction)

Chapter-4.indd 87 8/4/2016 10:17:17 AM


88 Chapter 4 Chemical Bonding

SUMMARY
1. A bond is a force existing between atoms in a molecule that holds the atoms together.
2. Chemical bond formation
(a) Kössel and Lewis proposed the electronic theory of valence for explaining the formation of a chemical bond.
(b) According to the theory, the formation of a chemical bond between two or more atoms can be explained as the tendency of the
elements to a attain stable configuration of electrons in the outermost shell. The combination of atoms leads to rearrangement
of their electrons either by complete transfer of electrons from one atom to another or by sharing of electrons.
3. Ionic bond
Ionic compounds are formed when metals react with non-metals. For example, NaCl is formed from sodium ion, Na+, and chloride ion,
Cl−. Once formed, these ions become tightly packed together.
Modern concept of ionic bonding in crystalline solids:
(a) In the periodic table, the highly electronegative halogens and electropositive alkali metals are separated by noble gases.
(b) The formation of a positive ion from an atom is associated with loss of electrons and the formation of a negative ion is associated
with a gain of electrons.
(c) The driving force for the formation of positive and negative ions is to attain noble gas configuration.
(d) The bond formation between negative and positive ions is stabilized by electrostatic attraction.
(e) The bond that results from electrostatic attraction between the positive and negative ions is called the electrovalent bond or the
ionic bond.

Tip The term electrovalence was used to represent the number of units of charge involved in the bond formation or the charge
on the participating ions. For example, in sodium chloride, sodium is assigned an electrovalence of one.

4. Lewis representation
(a) It is a simple notation to represent the atom with its valence electrons using dots. This notation is known as Lewis symbol. The
structure of a molecule using the Lewis symbols is known as Lewis structure.

Na + Cl [Na] + + [ Cl ]
Sodium Chlorine Sodium Chloride
atom atom ion ion

Tip The number of dots around the symbol represents the number of valence electrons. The number of valence electrons helps
in calculating the group valence of the elements, which is generally equal to the number of dots around the symbol or 8 minus the
number of dots.

(b) Octet rule


Atoms tend to gain or lose electrons until they have achieved an outer shell that contains an octet of electrons (eight
electrons).
5. Concept of covalent bond
(a) When non-metals combine with each other to form molecules, the energy is lowered due to electron sharing. According to the
Lewis–Langmuir theory, when two atoms come together, the valence electrons of each of the atoms will be attracted and
shared by the nuclei of both the atoms; as a result, a net strong bonding occurs between the two atoms and is called the cova-
lent bond (electron pair bond or a homopolar bond).
(b) Lewis representation
(i) For Lewis dot structures for covalent bonds, the bonds are formed by electron sharing, as a result of which, the atoms form-
ing covalent bonds attain the outer shell noble gas configurations.
The formation of H2 can be depicted as: H ⋅ + H ⋅ → H : H
(ii) Octet rule: The octet rule can also be used to explain the number of covalent bonds an atom forms. This number normally
equals the number of electrons the atom must acquire to have a total of eight (an octet) in its outer shell.
Types of covalent bonds

Single bond Bond produced by the sharing of one pair of electrons between two atoms. CH4

Double bond Bond produced by sharing two of the electrons of an atom with adjacent atoms. O C O

Triple bond Bond produced when three pairs of electrons are shared between two atoms. N N

Chapter-4.indd 88 8/4/2016 10:17:19 AM


Summary 89

(iii) Procedure to draw Lewis structures


• Decide which atoms are bonded to each other.
• Count all of the valence electrons. Add or remove electrons to account for charges on ions.
• Place two electrons in each bond.
• Complete the octets of the atoms attached to the central atom by adding electron in pairs.
• Place any remaining electrons on the central atom in pairs.
• If central atom has less than an octet, form double bonds. If necessary form triple bonds.
6. Formal charge
(a) It is the charge that each atom in a molecule would have if the electrons in the bonds were divided equally between the two
atoms. It considers all bonds as if they were non-polar. It does not imply that it is the actual charge on the atom.

Tip Formal charge is a method of electron book-keeping and does not imply that it is the actual charge on the atom in question.

(b) Calculating the formal charge on an atom


(i) Number of electrons in the valence shell
of the atom in the Lewis structure

Formal Number of e− in valence Number of bonds Number of


charge = shell of the isolated atom − on the atom
+
unshared e−

For example, the formal charge on sulphur in the given structure is calculated as
The electrons in a bond Unshared electrons belong
are divided equally exclusively to one atom.
between the two atoms.
O
H O S O H

There are 4 bonds


to the sulphur atom.

formal charge on S = 6 − (4 + 0) = +2

An isolated S atom has There are no unshared


6 electrons. pairs on the sulphur.

Ba
(ii) The formal charge (FCa) on an atom is also calculated by the formula: FC a = Va − Na −
2
where Va is the number of valence electrons on a neutral atom of the element, Na is the number of non-bonding electrons
on the atom and Ba is the number of bonding electrons on the atom.

Tips
(a) The sum of the formal charges in the molecule is zero.
(b) The formal charges in any Lewis structure add up to the charge on the particle.
(c) When several Lewis structures are possible, one with formal charges closest to zero is the most stable and preferred.

7. Ionic bonding
(a) Ionic solids are held together by the electrostatic attraction between the positive and negative ions. The formation of an ionic
compound and its stability primarily depend on (i) the ease of formation of ions; (ii) the lattice structure of the crystal.
(b) Formation of ionic compound will depend upon:
(i) Ionization enthalpy
• Enthalpy change that accompanies the removal of an electron from the outermost shell of an isolated gaseous atom to
form a positive ion in gas phase.
• Lesser the ionization enthalpy, greater will be the ease of formation of an ionic bond.

Chapter-4.indd 89 8/4/2016 10:17:20 AM


90 Chapter 4 Chemical Bonding

(ii) Electron gain enthalpy


• Enthalpy change that accompanies the addition of an extra electron to a gaseous isolated atom to form a negative ion
in gas phase.
• Higher the electron gain enthalpy, greater will be the energy released on formation of a negative ion and greater will be
the ease of formation of an ionic bond.
(iii) Lattice enthalpy
• It is defined as the change in energy that occurs when isolated ions in the gas phase combine to form one mole of ionic
compound. It is determined by the size of the ions and the charge on them.
• Greater the value of lattice enthalpy of the resulting ionic compound, greater will be the ease of its formation and its
stability.
• Lattice enthalpy for ionic compounds can be calculated theoretically from Born Lande’s equation as

NA Z + Z − e 2 A  1 
∆Hlattice = − 1 − 
4p ∈0 r0  n 
where NA = Avogadro constant; A = Madelung constant, relating to the geometry of the crystal; Z+ = charge number of cation;
Z− = charge number of anion; e = charge of electron 1.6022 × 10−19 C and ∈0 = permittivity of free space; r0 = distance to
closest ion; n = Born exponent, typically a number between 5 and 12.

Tip For ionic bonding to occur, the energy-lowering effect of the lattice enthalpy must be greater than the combined net energy-
raising effects of the ionization enthalpy and electron gain enthalpy.

(iv) Hydration energy


• It is the energy released when the ions of an ionic compound are attached to water molecules.
8. Born–Haber cycle
(a) We cannot measure the lattice enthalpy for sodium chloride directly, but we can use Hess’ law and some other experimental data
to calculate the lattice enthalpy indirectly.
(b) A set of alternate paths from the free elements to the solid ionic compound in order to measure the lattice enthalpy is called a
Born–Haber cycle.
(c) We begin with the free elements, sodium and chlorine. The direct path at the bottom left has as its enthalpy change the heat of
formation of NaCl, ∆ f H o .
Na(s) + 21 Cl2 (g) → NaCl(s); ∆ f H o = − 411.3 kJ

(d) The alternative path (Born–Haber cycle) is divided into a number of following steps. The enthalpy changes accompanying each
step can be listed as:
(i) Sublimation of sodium metal: Na(s) → Na(g); ∆ subH o = 107.8 kJ mol−1

(ii) Ionization of sodium atoms: Na(g) → Na+ (g) + e − ; ∆ ionH o = 495.4 kJ mol−1
(iii) Dissociation of chlorine into atoms: 1
Cl (g) → Cl(g);
2 2
1

2 bond
H o = 121.3 kJ mol−1
− −
(iv) Gain of electron by chlorine: Cl(g) + e → Cl (g); ∆ electron gainH o = − 348.8 kJ mol−1
(v) The calculated lattice enthalpy of NaCl:
Na+ (g) + Cl− (g) → Na+Cl− (s)
∆ lattice H o = 411.3 + 107.8 + 121.3 + 495.4 − 348.8 kJ mol−1 = 787.0 kJ mol−1
9. Covalent bonding
Covalent bond is formed between two atoms by mutual sharing of their outer electrons so as to complete their octets or duplets
(in case of hydrogen which has only one shell).
Every covalent bond is characterized by two quantities:
(i) The average distance between the nuclei held together by the bond.
(ii) The amount of energy needed to separate the two atoms to produce neutral atoms again.
10. Polar and non-polar covalent bonds
(a) When two identical atoms form a covalent bond, each atom has an equal share of the bonding electron pair. The electron den-
sity at both ends of the bond is the same. The bond is known as a non-polar bond.
(b) However, when different kinds of atoms combine, as in HCl, one nucleus usually attracts the electrons in the bond more strongly
than the other. The result of unequal attraction for the bonding electrons causes one atom to have slightly positive charge and
the other to have slightly negative charge. This is known as polar bond.
(c) When the bonding electrons are around one atom all the time, the bond is ionic.

Chapter-4.indd 90 8/4/2016 10:17:22 AM


Summary 91

(d) If two atoms have similar electronegativities, the bond formed between them will be covalent.
(e) A large difference in electronegativity leads to formation of an ionic bond.
11. Fajans’ rules
(a) Ionic and covalent bonds are the two extreme types of bonding, and almost always the bonds formed are intermediate in type.
This is explained in terms of polarizing power of the cation and polarizabilty of the anion (i.e., deforming) and the shape of the
ions.
(b) Fajans put forward four rules which summarize the factors favouring polarization, and hence covalency.
(i) A small positive ion favours covalency
In small ions, the positive charge is concentrated over a small area. The high charge density of the small cation distorts
the electron cloud of the anion to such an extent that the electron clouds of the anion and the cation overlap, making the
bond resemble a covalent bond. This explains why LiCl is more covalent than KCl.
(ii) A large negative ion favours covalency
Large ions are highly polarizable, that is, easily distorted by the positive ion. This explains why covalent character of lith-
ium halides is in the order:
LiI > LiBr > LiF
(iii) Large charges on either ion, or on both ions, favour covalency
This is because a high charge increases the amount of polarization. This explains why the covalent character in halides is
in the order:
Na+Cl− < Mg2+Cl2 < Al3+Cl3

(iv) Polarization, and hence covalency, is favoured if the positive ion does not have a noble gas configuration.
(c) Applications of Fajans’ rule
(i) The compound with higher covalent character will have a lower melting and boiling point. For example, CaCl2
(m. p. = 782°C) > BeCl2 (m. p. = 450°C)
(ii) Greater the covalent character, lower is its solubility in polar solvents. For example, AgF > AgCl > AgBr > AgI.
(iii) With increase in the covalent character, thermal stability decreases. For example, BeCO3 < MgCO3 < CaCO3 < BaCO3.
12. Dipole moment
(a) The dipole moment (µ), is equal to the amount of charge on either end of the molecule q, multiplied by the distance between
the charges r.
m =q×r

It measures the polarity of the bond which is caused by the electronegativity difference between two atoms involved in that
bond. It is measured in Debye unit (symbol D), where 1 D = 10−18 esu cm = 3.33 × 10−30 coulomb metre.
(b) Bond moment
(i) It is the dipole moment of a particular bond. The dipole moment of a molecule is determined by the bond moments of the
bonds.
(ii) For diatomic heteroatomic molecules, the bond moment is equal to the dipole moment and for diatomic homoatomic
molecule it is zero because there is no shifting of bond pair towards any atom. Hence q is zero in above equation.
(iii) For polyatomic molecule (molecule having more than two atoms), the dipole moment is equal to the vector sum of all
bond moments and lone pair moments.
(c) Lone pair moment
(i) The lone pair has no contribution to the dipole moment of the molecule, if the lone pair is present in the pure s or p orbital.
However, when the lone pair is present in a hybrid orbital, the electronic density is not equal in two lobes of the hybrid
orbital.
(ii) The value of lone pair moment depends upon the %s character of the hybrid orbital and as the %s character increases, the
lone pair moment increases upto 50% and then falls down as it becomes more and more spherical. The lone pair moment
order for different hybrid orbitals (consisting of s or p orbitals) is given as sp > sp2 > sp3.
(d) Applications of dipole moment
(i) Determining the extent of polar character in a covalent bond.
(ii) Calculating the percentage ionic character of the bond.
(iii) Predicting the shape of molecules and their symmetry.
(iv) Distinguishing between cis and trans isomers.
(v) Identifying the position of the substituent in substituted aromatic rings.
13. Coordinate or dative bond
(a) During the formation of a bond by sharing of electrons between atoms, when both electrons of the shared pair are provided by
one of the combining atoms, then the bond formed is called a coordinate bond or a dative bond.

Chapter-4.indd 91 8/4/2016 10:17:23 AM


92 Chapter 4 Chemical Bonding

(b) The atom which provides the electron pair is termed as the donor atom, while the atom which accepts it is termed as the
acceptor atom.
(c) An arrow pointing from donor towards the acceptor atom represents a coordinate bond. An example of coordinate bond is the
formation of ammonium (NH4+) ion.
+
H H
×
×
+
H × N +H H× N H
× ×
Coordinate bond
H Lone pair H
of electrons
Ammonia Ammonium ion
molecule
Here nitrogen is the donor atom, while H+ is the acceptor.
14. Resonance structures
(a) The concept of resonance implies that whenever the molecule cannot be described accurately by a single Lewis structure, a
number of structures with similar energy, position of nuclei and number of bonding and non-bonding pair of electrons are used
to describe the structure. These are known as resonance structures.
S S
O O O O
2− 2− 2−
O O O
C C C
O O O O O O

The actual structure of the molecule is a hybrid of these canonical structures and is known as resonance hybrid.
(b) Some important points about resonance structures are as follows:
(i) Resonance is exhibited only in molecules whose atoms are in one plane.
(ii) The resonance hybrid structure averages the bond characteristics of various resonance structures.
(iii) The canonical structures have no real existence but only help visualize the possible resonance structures.
(iv) There is no equilibrium between the canonical structures.
(v) All the canonical structures must have the same number of unpaired electrons.
15. Exceptions to the octet rule
(a) The incomplete octet of the central atom
In some molecules, the central atom behaves as though it has less than an octet. The most common examples involve com-
pounds of beryllium and boron.
Be + 2 CI CI Be CI
Four electrons around Be

CI
B + 3 CI CI B CI
Six electrons around B
(b) Odd-electron molecules
(i) Some molecules have an odd number of valence electrons, such as nitric oxide (NO –11 valence electrons) and nitrogen
dioxide (NO2 – 17 valence electrons).
(ii) The expanded octet: Elements in rows below second row, such as phosphorus and sulphur, sometimes do exceed an
octet, because their valence shells can hold more than 8 electrons.
(c) Other exceptions to the octet theory
(i) The octet rule does not work in molecules which have an odd number of electrons.
(ii) The rule was postulated on the basis of inert behaviour and lack of reactivity of noble gases. However, some compounds
of noble gases are known.
(iii) It does not account for the shapes of molecules and the relative stability of structures.
16. Valence shell electron pair repulsion (VSEPR) theory
(a) The main postulates of VSPER theory are as follows:
(i) The shape of the molecule is determined by repulsions between all the electron pairs present in the valence shell.
(ii) The electron pairs in the valence shell experience electrostatic repulsions due to negatively charged electron clouds.
(iii) The electron pairs tend to occupy positions that minimize these repulsions.
(iv) A multiple bond is treated like a single bond and the electron pairs involved in multiple bonding are treated as single super
pair.
(v) The double bond needs more space as compared to the single bond. The repulsion order in relation to the bonds is as
follows:

Chapter-4.indd 92 8/4/2016 10:17:25 AM


Summary 93

Double bond–double bond > Double bond–single bond > Single bond–single bond
(vi) If a molecule has two or more resonance structures, then VSEPR model can be applied to any one structure.
(vii) The magnitude of repulsions between bonding pairs of electrons depends on the electronegativity difference between
the central atom and the other atoms.
(viii) A lone pair (lp) of electrons takes up more space round the central atom than a bond pair (bp). The repulsion order:
Lone pair–lone pair > Lone pair–bond pair > Bond pair–bond pair
(b) Molecular geometry based on number of electron pairs
Number of electron pairs Shape Example
2 Linear BeCl2 180°
M
CI Be CI

3 Trigonal planar BCl3


CI

M 120°
B
CI CI

4 Tetrahedral CH4 H

109.5°
M C
H
H
H

5 Trigonal bipyramidal PCl5


CI

CI
M P CI
CI

CI

6 Octahedral SF6
F

F F
M S
F F

(c) Molecular geometry in presence of one or more lone pairs on central atom
No. of
Characteris- electron Expected Observed
Type tics Example pairs geometry geometry Reason
AB2E Two bp and 3 Trigonal Non-linear As lp–bp > lp–lp, the Sn–Cl bonds are
one lp. planar or bent or pushed slightly the bond angle becomes
V-shaped less than 120°.
Sn
CI CI

(Continued)

Chapter-4.indd 93 8/4/2016 10:17:30 AM


94 Chapter 4 Chemical Bonding

(Continued)
No. of
Characteris- electron Expected Observed
Type tics Example pairs geometry geometry Reason
AB3E One 4 Tetrahedral Trigonal Presence of l.p at the apex. Since the lp re-
non-bonding pyramidal pels three bonded N H pairs more strong-
and three ly, these are pushed together and the bond
bonding elec- H N angle becomes less than 109.5°.
H
tron pairs
H

AB2E2 Two bond- 4 Tetrahedral Non-linear, Presence of two lp on central oxygen atom
ing and two bent causes greater repulsion and the two O H
non-bonding bonding pairs move closer to each other. The
electron pairs H O bond angle between the two shows a great-
er distortion from normal tetrahedral angle
H
(109.5°) than in the case of ammonia (104.5°).
AB4E Four bonding 5 Trigonal See-saw The lp can occupy either equatorial or axial
F
and one bipyramidal position or results in different distortion in
non-bonding geometries. The repulsion is smaller when
electron pairs S F the lone pair is at the equatorial position
F and it is more stable. This is the experimen-
F
tally determined structure and the structure
is a distorted tetrahedron or see–saw like.
AB3E2 Three bonding 5 Trigonal T-shaped Presence of two lp at the equatorial or axi-
F
and two bipyramidal al positions causes greater lp–bp repulsion.
non-bonding The equatorial position is favoured by the
electron pairs CI F lone pairs as it is more stable and there is less
lp-bp repulsion as compared to when the
F
one or both the lp are at the axial position.
The structure of the molecule is T-shaped.
AB2E3 Two bonding I 5 Trigonal Linear The lp(s) occupy the three equatorial posi-
and three bipyramidal tions to minimize repulsion.
non-bonding
electron pairs I

AB5E Five bonding F 6 Octahedral Square The lp is present at the apex.


and one pyramidal
non-bonding F F
electron pairs Br
F F

AB4E2 Four bond- 6 Octahedral Square The lp occupy the axial positions.
ing and two planar
F F
non-bonding Xe
electron pairs F F

Tip Molecular shape describes the arrangement of atoms, not the arrangement of electron pairs.

17. Quantum mechanical theory of chemical bonding


There are fundamentally two theories of covalent bonding that have evolved based on quantum theory, the valence bond (VB)
theory and the molecular orbital (MO) theory.

Chapter-4.indd 94 8/4/2016 10:17:32 AM


Summary 95

18. Valence bond theory


The basic idea of the theory is that a bond between two atoms is formed when two electrons with their spins paired are shared by
two overlapping atomic orbitals, one orbital from each of the atoms joined by the bond.
(a) Orbital overlap concept
(i) A covalent bond is formed between two atoms in a molecule, when the half-filled valence atomic orbitals (AOs) of the two
atoms containing unpaired electron overlap with one another and the electrons pair up in the overlap region.
(ii) When the attractive forces between the atoms are stronger than the repulsive forces, energy is released; lowering of
energy makes the molecule stable.
(iii) Covalent bonds are classified into two main types – sigma (s) bonds and pi (p) bonds.
(b) Types of overlapping and nature of covalent bonds
The following bonds are observed based on the type of overlapping.
(i) r bond: It is formed by head-on overlap of orbitals. Different types of overlap for s bonds:
• s–s overlap, such as in H2 molecule.

• s–p overlap, such as in HX molecule (X = F, Cl, Br, I)

• p–p overlap, such as in X2 molecule (X = F, Cl, Br, I)

The relative strength of bond is: s–s < s–p < p–p

Tip The shape of the molecule is determined by the s bonds (and lone pairs) but not by the p bonds.

(ii) o bond: It is formed by sideways overlap of orbitals and it merely shortens the bond length. The relative strength of the
p bonds increases when the intermolecular distance increases and the order is:

pp − dp
pp – pp : or or
(or) dp − pp

2pp – 2pp > 2pp – 3dp > 2pp – 3pp > 3pp – 3pp

The strength of the bond depends on the extent of overlap between the orbitals participating in the bond formation. The
p bond is thus weaker than the s bond. Also, p bond is always formed in addition to the s bond in molecules containing
multiple bond (double or triple bonds).
(c) Hybridization
(i) Combination of the different atomic orbitals in the valence shell of the same atom resulting in more stable set of orbitals
are called hybridized orbitals and the process is known as hybridization.
(ii) Hybridization results in a set of atomic orbitals which have equal energies (degenerate) and are intermediate between
those of the basic orbitals used to construct them. These hybridized atomic orbitals can overlap more effectively with the
atomic orbitals of other atoms in the molecule, thus providing an overall molecular structure which has stronger bonds
and lower energy.
(iii) Hybridization can be calculated by the following expression

(V + M − c + a)
Hybridization =
2
where V is the number of valence electrons of the metal, M is the number of monovalent atoms attached, c is the charge
on cation and a is the charge on anion.
(d) Salient features of hybridization
(i) The number of hybridized orbitals formed is equal to the number of atomic orbitals that get hybridized.
(ii) The hybridized orbitals are always equivalent in energy and shape.
(iii) The atomic orbitals involved in hybridization can be empty, half-filled or completely filled.

Chapter-4.indd 95 8/4/2016 10:17:34 AM


96 Chapter 4 Chemical Bonding

(iv) The hybridized orbitals have two lobes, one larger and one smaller; larger lobes are more effective in overlapping with
other atomic orbitals and forming stable bonds.
(v) The hybridized orbitals are directed in certain directions in space which will have stable arrangements. This gives the
geometry of the molecule.
(vi) Hybridized orbitals are usually involved only in s bonds; p bonds involve the overlap of unhybridized orbitals.
(vii) Hybridization is not invoked in isolated atoms, only in covalent bond formation with other atoms.

Tip (a) As the %s character increases in a hybrid orbital, the hybrid orbital becomes bulkier and shorter. On the other
hand, as the %p or %d character increases, the orbital becomes longer and thinner.
(b) As the %s character increases, the energy of the hybrid orbital decreases and as the %p or %d character
increases, the energy of the hybrid orbital increases.

(e) Important conditions of hybridization


(i) Only the orbitals present in the valence shell undergo hybridization.
(ii) For effective hybridization, the energy of mixing orbitals must nearly be the same.
(iii) Promotion of electrons to empty orbitals is not a necessary step in hybridization.
(iv) It is not essential that only half-filled orbitals take part in hybridization. In some cases, even filled orbitals of the valence
shell participate in hybridization.
(f) Types of hybridization

Type of hybridization Orbitals involved % Character Orientation and shape Example


sp One s orbital and one p 50% s and Two hybridized sp orbitals are in a lin- BeF2
orbital on a central atom 50% p ear arrangement, that is, 180° apart.
hybridize to give rise to 180° 180°
two equivalent sp hybrid
orbitals
F Be F

sp2 One s orbital and two p 33% s and The three sp2 hybridized orbitals are BF3
orbitals of a central atom 66% p trigonal and planar, 120° apart.
F
hybridize to give rise to
three sp2 orbitals 120°
120°
B
F F

sp3 One s and all three p 75% p and Four sp3 hybrid orbitals are tetra CH4
atomic orbitals hybridize 25% s hedral with an angle of 109.28° to
to give four equivalent sp3 each other. H
hybridized atomic orbitals
109.5°
H
H
H

(g) Hybridization of elements involving d orbitals


The type of hybridization possible with s, p and d orbitals along with their geometries and some examples are shown below:

Type of hybridization Number of atomic orbitals Shapes of molecules Examples


dsp2 4[d + s + p(2)] Square planar [Ni(CN)4]2−
sp3d 5[s + p(3) + d] Trigonal bipyramidal PF5, PCl5
6[s + p(3) + d(2)] or SF6, [CoF6]3−
sp3d2 Octahedron
[d(2) + s + p(3)] [Co(NH3)6]3+
sp3d3 7[s + p(3) + d(3)] Pentagonal bipyramidal IF7

Chapter-4.indd 96 8/4/2016 10:17:36 AM


Summary 97

19. Molecular orbital theory


(a) The molecular orbital theory is based on the following assumptions:
(i) When two atoms approach each other, their atomic orbitals lose their identity and mutually overlap to form new orbitals
called molecular orbitals.
(ii) The molecular orbitals are polycentric and are associated with the nuclei of all the atoms constituting the molecule.
(iii) Only atomic orbitals of about the same energy and same symmetry interact significantly.
(iv) The total number of molecular orbitals produced is always equal to the total number of atomic orbitals contributed by the
atoms that have combined.
(v) When two atomic orbitals overlap, they interact to form two molecular orbitals–a bonding molecular orbital and an
antibonding molecular orbital. The amount of stabilization of the bonding orbital equals the amount of destabilization
of the antibonding orbital.
(vi) Electrons of the combining atoms are then assigned to these molecular orbitals. Each molecular orbital can accommodate
a maximum of two electrons of successively higher energy.
(b) Formation of molecular orbitals
(i) The formation of molecular orbitals can be explained by linear combination of atomic orbitals (LCAO).
(ii) Consider two atoms A and B which have atomic orbitals described by the wave functions ΨA and ΨB. When the electron
clouds of these two atoms overlap as the atoms approach, the wave function for the molecule can be obtained by a linear
combination of the atomic orbitals ΨA and ΨB. The atomic orbitals ΨA and ΨB combine to give rise to a pair of molecular
orbitals Ψg (bonding molecular orbital [Ψg = ΨA + ΨB]) and Ψu (antibonding molecular orbital which is higher in energy
[Ψu = ΨA−ΨB]).

Atomic orbitals Molecular orbitals

+ + + Bonding orbital

s(g)
s s
ψ(g)

Node Antibonding orbital


+ − + −
s ∗(u)

s s ψ(u)

(iii) The energy of the bonding molecular orbital is lower than that of the atomic orbital by an amount Δ. This is known as the
stabilization energy. Similarly the energy of the antibonding molecular orbital is increased by Δ. Molecular orbitals may
hold up to two electrons (with opposite spins).
(c) Conditions for the combination of atomic orbitals
(i) The atomic orbitals must be roughly of the same energy.
(ii) The orbitals must overlap one another as much as possible.
(iii) In order to produce bonding and antibonding MOs, either the symmetry of the two atomic orbitals must remain unchanged
when rotated about the internuclear line, or both atomic orbitals must change symmetry in an identical manner.
(d) Types of atomic and molecular orbitals
Atomic 1sA + 1sB 1sA− 1sB 2sA + 2sB 2sA− 2sB 2pxA + 2pxB 2pxA− 2pxB 2pyA + 2pyB 2pyA− 2pyB 2pzA + 2pzB 2pzA− 2pzB
orbitals
Molec- s 1s s *1s s 2s s * 2s s 2p x s * 2p x p 2p y p * 2p y p 2pz p * 2pz
ular
orbitals
(e) Energy level diagram for molecular orbitals
(i) The order of energy of molecular orbitals in simple homonuclear diatomic molecules is:

 p 2 p x ,  p * 2 px , s * 2 pz
s 1s , s * 1s , s 2s , s * 2s , s 2 pz ,   p * 2p
 p 2 py ,  y

Increasing energy
→

(ii) The bonding p 2px and p 2py MOs have exactly the same energy and are said to be doubly degenerate. In a
similar way, the antibonding p *2px and p *2py MOs have the same energy and are also doubly degenerate.

Chapter-4.indd 97 8/4/2016 10:17:40 AM


98 Chapter 4 Chemical Bonding

(iii) For the lighter elements boron, carbon and nitrogen the p 2px and p 2py energy is lower than that of s 2pz. For these
atoms the order is:
p 2 p x p * 2 px
s 1s , s * 1s , s 2 s , s * 2 s ,  , s 2 pz , s * 2 pz , p * 2 p
p 2 py  y

Increasing energy
 →

20. Concept of bond order, bond length and bond energy


(a) The distribution of electrons in various molecular orbitals is known as electronic configuration of the molecule.
(b) Stability of the molecules
The stability of molecules can be determined by the relative number of electrons in the bonding (Nb) and non-bonding (Na)
molecular orbitals.
(i) The molecule is stable if Nb > Na.
(ii) The molecule is unstable if Nb < Na.
(c) Bond order is defined as the number of pairs of electrons shared between two atoms. It is given by:
1
Bond order = (Nb − Na )
2
where Nb and Na are the number of electrons in the bonding and antibonding molecular orbitals.
(d) Bond length is the distance between the atomic nuclei involved in bonding. These are also known as bond distances. The bond
length decreases with increase in bond order.
(e) The amount of energy required to break a bond is called bond dissociation energy or simply bond energy.
B.E. = ∆ = (Actual bond energy) − EA − A − EB −B
(f) The relation between bond order, bond length and bond energy can be summarized as follows:
Lower bond order ⇒ Longer bond length ⇒ Lower bond energy ⇒ Weaker bond
Higher bond order ⇒ Shorter bond length ⇒ Higher bond energy ⇒ Stronger bond
21. Magnetic properties
In filling up the molecular orbitals, if a molecule contains no unpaired electrons then it is diamagnetic and if it contains one or more
unpaired electrons, then it is paramagnetic.
22. Bonding in some homonuclear diatomic molecules

Bond Bond energy Bond length


Configuration order Magnetic properties (kJ mol−1) (pm)
H2 s 1s2 1 Diamagnetic
He2 (s 1s)2 (s *1s)2 0 Noble gas -
Li2 s 1s2, s *1s2, s 2s2 1 Diamagnetic 110 267

 p 2 p2x
C2 s 1s2 , s * 1s2 , s 2 s2 , s * 2 s2 ,  2
2 Diamagnetic 612 124
 p 2 p y

 p 2 p2x
N2 s 1s2 , s * 1s2 , s 2 s2 , s * 2 s2 ,  2
s 2 p2z 3 Diamagnetic 953 109
 p 2 p y

 p 2 p2x ,  p * 2 p1x
O2 s 1s2 , s * 1s2 , s 2 s2 , s * 2 s2 , s 2 p2z ,  2  1 2 Paramagnetic 501 121
 p 2 p y ,  p * 2 p y

MO theory predicts that molecules of Be2 and Ne2 should not exist at all because they have bond orders of zero.
23. Heteronuclear diatomic molecules
(a) The MO theory can also be extended to heteronuclear diatomic molecules.
(b) Heteronuclear diatomic molecules can have non-bonding orbitals, in addition to bonding and antibonding orbitals.

Chapter-4.indd 98 8/4/2016 10:17:41 AM


Summary 99

(c) Molecular orbital diagram for some heteronuclear molecules

HF s∗

1s
H atom

Energy
2p
F atom

s
HF molecule

Li–F s∗

2s
Li atom
Energy

2px 2py

Non-bonding pairs
s
located on F

Li–F 2s
molecule F atom

CO Atomic Molecular Atomic


Energy orbitals orbitals orbitals

s ∗2px

p ∗2py p 2pz

2px 2py 2pz

s 2px

2py 2pz 2px


p 2py p 2pz

s ∗2s
2s

2s
s 2s

C atom CO molecule C atom

24. Back bonding


(a) It is the bonding of p-conjugated ligands to a transition metal which involves a synergic process with donation of electrons
from the filled p-orbital or lone electron pair orbital of the ligand into an empty orbital of the metal (donor–acceptor bond),
together with the release (back donation) of electrons from a d orbital of the metal (which is of p-symmetry with respect to the
metal–ligand axis) into the empty p* antibonding orbital of the ligand.

Chapter-4.indd 99 8/4/2016 10:17:43 AM


100 Chapter 4 Chemical Bonding

(b) Properties of certain molecules are explained by back bonding −


(i) CHCl3 is more acidic than CHF3 because the lone pair on carbon atom gets delocalized through 2pp− 3dp bonding in C Cl3 ,

which is not possible in case of CF3.
− −
(ii) R3 C is pyramidal while (CN)3 C is planar due to the back bonding from the lone pair of carbon atom into the p * orbital of
the CN group.
(iii) The Lewis acidity order of BX3 molecules is explained in terms of back donation from X atom (where X = F, Cl, Br, I) to boron
atom. The ability of back donation order for halogens towards boron atom is F  Cl > Br > I.
(iii) The relative stability order for the diradicals is: CH2 < : CCl2 < : CF2 . In :CH2, the carbon atom is electron deficient due to
incomplete octet and no back bonding, while in :CF2 and :CCl2 back donation takes place to stabilize them.
25. Metallic bonding
(a) Electron sea model
(i) In the case of metals, every atom achieves a more stable configuration by sharing the outer shell electrons with the various
other atoms in the metal lattice. The electrons are said to be delocalized. The metal lattice is held together by the strong
forces of attraction between the positive nuclei and the delocalized electrons. This is described as “an array of positive ions
in a sea of electrons”.
(ii) The strength of the metallic bond depends on the number of valence electrons contributed by the atoms to the delocal-
ized electrons and the packing arrangement of the metal atom. More number of delocalized electrons and more closely
packed atoms result in a stronger bond and a higher melting point.
(iii) This explains relatively low melting points of Group I metals have compared to other metals, the properties of the metals
like malleability and ductility and metallic shine and lusture.
(b) Band theory
In a multi-atom system, the number of molecular orbital states will be equal to the number (n) of the atomic orbitals combining.
Since the number of molecular orbitals is large, the spacing between them decreases to become almost negligible and we get
a “band” of continuous energy levels. These molecular orbitals extend in all three dimensions over all the atoms in the metal
piece. The molecular orbital theory can explain most of the physical properties of metals as well as the classification of materials
into conductors, insulators and semiconductors.
26. Hydrogen bonding
(a) The electrostatic force of attraction between hydrogen atom and another electronegative atom like O, N or F is called hydrogen
bonding.
(b) There are two types of hydrogen bonds:
(i) Intermolecular hydrogen bond: When the hydrogen bonding occurs between an H atom of one molecule and an elec-
tronegative atom of a second molecule. For example HF, alcohol, H2O, NH3 etc.
(ii) Intramolecular hydrogen bond: In this type, the hydrogen bond is formed between hydrogen and an electronegative
atom (F, O, N), within the same molecule. For example, intramolecular hydrogen bonds are present in molecules such as
o-chlorophenol, salicylaldehyde, o-nitrobenzoic acid, etc.
(c) Consequences of hydrogen bonding
(i) Physical state: Intermolecular hydrogen bonding causes two or more molecules of a compound to exist as associated or
grouped molecules. This results in an increase in the size as well as the molecular mass of the compound which in turn is
reflected in the physical state of the substance.
(ii) Melting and boiling points: Due to intermolecular hydrogen bonding and a consequent association of molecules, larger
energy is required to separate these molecules before they can melt or boil. Hence there is an elevation in the melting and
boiling points of these compounds.
(iii) Solubility: Covalent compounds which can form a hydrogen bond with water readily dissolve in it. For example, alcohols
like ethanol, ammonia, amines, lower aldehydes and ketones are soluble in water.

SOLVED EXAMPLES
KÖSSEL–LEWIS Approach to Chemical Bonding Solution
(b) After sharing three electrons pair with three oxygen atoms,
1. Which of the following compounds contains the maximum two electrons are left as non-bonding electrons on Xe in
number of lone pairs at the central atom in its best Lewis XeO3, so only one lone pair exists on the central atom.
structure?
(a) XeO3 (b) ClO2− Xe
O O
(c) SOCl2 (d) IO 4− O

Chapter-4.indd 100 8/4/2016 10:17:45 AM


Solved Examples 101

In ClO2− , chlorine shares three electrons with two oxygen Solution


atoms, four electrons are left as non-bonding electrons (a) In benzene, due to the resonance C C bond length is in
on Cl, so two lone pairs exist on the central atom. between double bond and single bond. However, if we con-
− sider the six carbon atoms to be linked by three single and
Cl three double bonds, then the circumference of benzene
O O ring is 3 × (C C) + 3 × (C C) = 3 × 1.33 + 3 × 1.54 = 8.61 Å.

In SOCl2, sulphur shares two electrons with oxygen and 5. The correct order about of C O bond length in the following is
one with each chlorine atom, two electrons are left as (I) CO (II) CO2 (III) CO2−
3
non-bonding, so only one lone pair exists on the central
(a) II < I < III (b) III < II < I
atom.
(c) I < II < III (d) I < III < II
S
O Cl Cl Solution
(c) The order is as follows:
In IO 4− , iodine shares two electrons each with three oxy-
gen atoms and one electron with one oxygen atom, so no O−
electron is left as non-bonding and zero lone pairs exist C O <O C O<O C
O−
on the central atom.
− 6. Which of the following compounds does not follow the octet
O rule for electron distribution?
I (a) H2O (b) PH3 (c) PCl3 (d) PCl5
O O O Solution
(d) P has 10 electrons in its valence shell in PCl5.
2. Which of the following contains minimum dots in the Lewis
Cl
structure?
Cl
(a) HIO3 (b) H2CO3 (c) OF2 (d) HPO2−
3 Cl P
Solution Cl
(b) HIO3: each O atom contains 4 and iodine atom 2 dots, so, Cl
total are 14.
7. Element A has three electrons in the outermost orbit and B has
H2CO3: each O contains 4 dots, so, total are 12 dots.
six electrons in the outermost orbit. The formula of the com-
OF2: O atoms contain 4 and 2 ‘F’ atoms contains 2 × 6, so,
pound will be
total are 16 dots.
(a) A2B3 (b) A2B6 (c) A2B (d) A3B2
HPO2− 3 : 2 O atoms contain 2 × 6 dots and one oxygen
atom contains 4, so, total are 16 dots. Solution
(a) A has three electrons in the outermost orbit, it needs five
3. Which of the following molecules contains the maximum S–O more electrons to complete it octet and B has six electrons
bond length? so it would need two more electrons to complete the octet.
(a) SO2 (b) SO3 (c) SO2−
4 (d) SO2−
3 So each A will form two bonds with two B atoms, and sin-
gle electron remaining on each A will form bond with the
Solution same B atom such that one B atom is shared between the
(d) In SO2 and SO3, all the bonds are double bonds. In SO2− 3
two A atoms.
and SO2−4 also, all the bonds are of the same type due to 8. What are the formal charges of the atoms in the nitrite ion in
resonance. But in SO2−
3 , 2− charge is delocalized over three the order of nitrogen, oxygen 1 (O1) and oxygen 2 (O2)?
atoms whereas it is delocalized over four atoms in SO2− 4 . 1 2
So, bond order is lowest for SO2− 2−
3 . Thus, SO3 contains the O N O
maximum S–O bond length.
(a) 0, 0, −1 (b) 0, −1, −1 (c) −1, 0, −1 (d) −1, 0, 0
O O 2−
149 pm 151 pm Solution
S 142 pm
S 143.1 pm
S S O
O− O 106° (a) The nitrite ion can be represented as
O 120° O O 119° O O O− O
1 + 2

4. The C C bond length is 1.54 Å, C C bond length is 1.33 Å. O N O
What is the circumference of benzene ring? Formal charge on any atom = Number of valence electrons
(a) 8.61 Å (b) 8.82 Å − Number of non-bonding electrons − 1/2 (number of
(c) 8.43 Å (d) 8.05 Å bonding electrons)

Chapter-4.indd 101 8/4/2016 10:17:52 AM


102 Chapter 4 Chemical Bonding

1 0.759 × 10 −20
Formal charge on nitrogen = 5 − 2 − (6) = 0 With respect to electron, q = = 0.05
2 1.6 × 10 −19
1
Formal charge on oxygen atom 1 = 6 − 4 − (4) = 0 13. The correct order of increasing covalent character of the
2
1 following is
Formal charge on oxygen atom 2 = 6 − 6 − (2) = − 1 (a) KCl < CaCl2 < AlCl3 < SiCl4
2
Ionic Bond (b) SiCl4 < AlCl3 < CaCl2 < KCl
(c) AlCl3 < CaCl2 < KCl < SiCl4
9. Lattice energy of an ionic compound depends upon (d) CaCl2 < SiCl4 < KCl < AlCl3
(a) charge on the ions only.
(b) size of the ions only. Solution
(c) packing of ions only. (a) According to Fajans’ rule, as the charge on the cation
(d) charge on the ions and size of the ions. increases, its effective nuclear charge as well as polariz-
Solution ing power increases. Hence, covalent character will also
increase. The increasing order should be
(d) Lattice energy depends on both charge and size of the KCl < CaCl2 < AlCl3 < SiCl4
ions.
14. Which of these contains both polar and non-polar bonds?
Polar and Non-Polar Covalent Bonds (a) HCN (b) CO2 (c) H2O2 (d) CH4
10. Among the following the maximum covalent character is Solution
shown by the compound
(c) H O O H. Here, O H bonds are polar and O O bond
(a) SnCl2 (b) AlCl3 (c) MgCl2 (d) FeCl2
is non-polar.
(AIEEE 2011)
Solution 15. Which of the following is arranged in order of increasing
dipole moment?
(b) According to Fajans’ rule, greater charge and small size (a) BCl3 < NH3 < H2O < SO2
of cation causes more polarization and hence more is (b) BCl3 < NH3 < SO2 < H2O
the covalent character. That is why AlCl3 has maximum (c) NH3 < SO2 < H2O < BCl3
covalent character. (d) H2O < SO2 < NH3 < BCl3
11. Amongst LiCl, RbCl, BeCl2 and MgCl2 the compounds with Solution
the greatest and the least ionic character, respectively are:
(b) Dipole moment is defined as the product of the magni-
(a) LiCl and RbCl (b) RbCl and BeCl2
tude of charge on any one of the atoms and the distance
(c) MgCl2 and BeCl2 (d) RbCl and MgCl2
between them.
(JEE Main Online 2014)
Solution Cl O
N S
(b) According to Fajans’ rule high positive charge, small cat- Cl B H H O O H H
ion and large anion favours covalency. RbCl has great- Cl H
est ionic character and least covalent character among m=0 m = 1.49 D m≠0 m = 1.83 D
given species as both the ions in the compound are of The dipole moment of BCl3 molecule is zero. The bond
comparable sizes which favours the ionic character. dipoles of three B Cl bonds give a net sum of zero
BeCl2 has least ionic character and greatest covalent because the resultant of any two is equal and opposite
character. As Be2+ is small cation and Cl– is a large anion of the third as shown above. In other molecules dipole
which favours the covalent character. moment shows increasing trend due to the presence
12. Molecule AB has a bond length of 1.617Å and a dipole of central atom that is electronegative in nature. As the
moment of 0.38 D. The fractional charge on each atom (abso- electronegativity of the central metal atom increases
lute magnitude) is: (e0 = 4.802 × 10−10 esu) (N < S < O), dipole moment of the resulting molecule
(a) 0 (b) 0.05 (c) 0.5 (d) 1.0 also increases. Hence, the trend is
(JEE Main Online 2015) BCl3 < NH3 < SO2 < H2O
Solution
Bond Characteristics
(b) m = q×l
16. Which one of the following pairs of species has the same
−30 −10 bond order?
0.38 × 3.335 × 10 Cm = q × 1.617 × 10 m
(a) CN– and NO+ (b) CN– and CN+

0.38 × 3.335 × 10 −30 Cm (c) O2 and CN– (d) NO+ and CN+
q= = 0.759 × 10 −20 C (AIEEE 2008)
1.67 × 10 −10 m
Solution
(1D = 3.335 × 10 −30 Cm) (a) CN− and NO+ has 14e– each, hence same bond order = 3.

Chapter-4.indd 102 8/4/2016 10:17:55 AM


Solved Examples 103

(c) Bond length in NO+ in equal to that in NO.


17. Stability of the species Li2, Li2− and Li2+ increases in the order of
(d) Bond length in NO is greater than in NO+.
(a) Li2− < Li2+ < Li2 (b) Li2 < Li2− < Li2+
Solution
(c) Li2− < Li2 < Li2+ (d) Li2 < Li2+ < Li2− (d) Bond length in NO is greater than in NO+. As we know
(JEE Main 2013) that the bond order is inversely proportional to the bond
Solution length. So, greater is the bond order and shorter is the
1 bond length.
(a) Bond order = (bonding electrons − antibonding electrons)
2 20. The correct order of increasing C O bond length of CO,
Li2 (atomic number = 6) = s 1s2s * 1s2s 2 s2 CO23 − , CO2 is
4−2
Bond order = =1 (a) CO < CO2 < CO32 − (b) CO < CO23 − < CO2
2
+ (c) CO2 < CO23 − < CO (d) CO23 − < CO2 < CO
Li2 (atomic number = 5) = s 1s2s * 1s2s 2 s1
3−2 Solution
Bond order = = 0.5 (a) All bonds in CO2−
2 3 are equivalent because it is resonance
hybrid.
Li2− (atomic number = 7) = s 1s2s * 1s2s 2 s2s * 2 s1
− −
4−3 O O O
Bond order = = 0.5
2
C C C
O O− O O− O O
Stability ∝ Bond order. So, although bond order of both −
Li2− and Li2+ are same but Li2+ will be more stable as com- So, the bond order of CO2−3 is 4/3.

pared to Li2 as the number of antibonding electrons are Bond order in CO is 3 and of CO2 it is 2.
more in Li2− than that of Li2+ . Since bond length is inversely proportional to the bond
18. In which of the following pairs of molecules/ions, both the order, bond length order is CO < CO2 < CO32 − .
species are not likely to exist?
(a) H2− , He22 − (b) H22 + , He2 VSEPR Theory
(c) H2− , He22 + (d) H2+ , He22 − 21. In which of the following molecules/ions are all the bonds not
(JEE Main 2013) equal?
Solution (a) SF4 (b) SiF4 (c) XeF4 (d) BF4−
(AIEEE 2006)
(b) The bond order of various species is as follows:
Solution
1
(a) H2− = [3 − 1] = 0.5 can exist (a) S has 6 valence electrons. However, in SF4 there are
2 4 bonds, so 4 of the valence electrons from S are used in
1
He22 − = [2 − 2] = 0 cannot exist bonding. Since two of them are not involved in bonding,
2 they exist as a lone pair. SF4 is based on trigonal bipyram-
1 idal structure. The lone pair present will repel the 4 bond
(b) H22 + = [0 − 0] = 0 cannot exist pairs, thereby distorting the trigonal bipyramidal shape
2
1 and leading to non-equal bonds.
He2 = [2 − 2] = 0 cannot exist
2 22. The correct order of bond dissociation energy among N2, O2,
− 1 O2− is shown in which of the following arrangements?
(c) H2 = [2 − 1] = 0.5 can exist
2 (a) N2 > O2− > O2 (b) O2− > O2 > N2
2+ 1
He2 = [2 − 0] = 1 can exist
2 (c) N2 > O2 > O2− (d) O2 > O2− > N2
(JEE Main Online 2014)
(d) H+ = 1 [1 − 0] = 0.5 can exist
2 Solution
2
2− 1 (c) Bond energy ∝ Bond strength ∝ Bond order
He2 = [ 4 − 2] = 1 can exist
2
1
Bond order of H2+ Bond order = [Bonding electrons − Antibonding electrons]
2 and He2 is zero, thus their existence is 2
not possible. Bond order of N2 is 3.0; bond order of O2 is 2.0; bond
19. The bond order in NO is 2.5 whereas that in NO+ is 3. Which of order of O2− is 1.5.
the following statements is true for these two species? Thus, the decreasing order of bond energy is
(a) Bond length in NO+ is greater than in NO. N2 > O2 > O2−
(b) Bond length is unpredictable.

Chapter-4.indd 103 8/4/2016 10:18:03 AM


104 Chapter 4 Chemical Bonding

23. The molecular shapes of SF4, CF4 and XeF4 are causes trigonal bipyramidal geometry. Now the VSEPR
(a) the same with 2, 0 and 1 lone pairs of electrons. theory implies that lone pair should occupy one equa-
(b) the same with 1, 1 and 1 lone pair of electrons on the torial position due to which molecular shape becomes
central atoms, respectively. see-saw. The shapes of the other molecules are: IO 4−
(c) different with 0, 1 and 2 lone pairs of electrons on the (tetrahedral); ICl4− (square planar) and SnCl4 (tetrahedral).
central atom, respectively. − − −
(d) different with 1, 0 and 2 lone pairs of electrons on the F O Cl Cl Cl
central atom, respectively.
F Br F I I Sn
Solution O O Cl
F O Cl Cl Cl Cl
(d) The structures are as follows:
F F F F
F See–saw Tetrahedral Square planar Tetrahedral
S C Xe
F 26. Which of the following molecule has linear shape?
F F F F F F (a) BF3 (b) PCl5 (c) ClF3 (d) I3−
SF4 CF4 XeF4 Solution
See-saw Tetrahedral Square planer (d) BF3: Central atom is B
1 lone pair No lone pair Two lone pairs. Valence electrons = 3
24. Which of the following molecules has the greatest number of So, three fluorine atoms will form 3 bonds.
lone pair of electrons around the central atom? Therefore, number of electron pairs = 3 and all are bond
(a) IF7 (b) XeF2 (c) BrF3 (d) NF3 pairs.
Solution Thus, the shape is trigonal planar.
(b) The structures of the compounds are as follows: F F
F B
IF7 F
F F
No lone pairs
F I
PCl5: Central atom is P
Valence electrons = 5
F There are 5 Cl atoms, so the total number of electrons is
F
10.
F Number of electron pairs = 10/2 = 5 (all bond pairs)
XeF2 F Therefore, the shape is trigonal bipyramidal.
Cl
Cl
3 lone pairs P
Xe Cl Cl
Cl
ClF3: Central atom is Cl
F
Valence electrons = 7
F There are 3F atoms, so the number of electron pairs
BrF3 shared is 3.
Total number of electrons = 10
2 lone pairs Number of bonds = 10/2 = 5 (3 bond pairs +2 lone pairs)
Br F
The shape is trigonal bipyramidal.
F

F
Cl F
NF3 N

F 1 lone pair
F F
F
I3− : Central atom is I
25. Which of the following molecules has a see-saw shape?
Valence electrons = 7
(a) IO 4− (b) ICl4− (c) SnCl4 (d) BrF4−
Number of peripheral iodine atoms = 2
Solution Negative charge = 1
(d) In BrF4− , the central atom, Br contains four bond pair elec- Total number of electrons = 7 + 2 + 1 = 10 (2 bond pairs
trons and one lone pair electrons. Thus five steric number and 3 lone pairs)

Chapter-4.indd 104 8/4/2016 10:18:08 AM


Solved Examples 105

The shape is linear. It undergoes sp3d3 hybridization to form a pentagonal


− bipyramidal structure of IF7.
I I I
5s 5p
Ground
27. In OF2, number of bond pairs and lone pairs of electrons are, state
respectively,
(a) 2, 6 (b) 2, 10 (c) 2, 8 (d) 2, 9 5s 5p 5d
Excited
Solution
state
(c)
O sp3d3 hybridization
F F F
F
In F O F; Bond pairs = 2. Lone pairs around oxygen = 2,
lone pairs around two fluorine atoms = 3 × 2 = 6. Therefore,
total lone pairs are 6 + 2 = 8. F
I
Hybridization Theory F
28. The charge/size ratio of a cation determines its polarizing F
power. Which one of the following sequences represents the F
increasing order of the polarizing power of the cationic spe-
F
cies, K+, Ca2+, Mg2+ and Be2+?
31. In which of the following pairs the two species are not
2+ 2+ 2+ + 2+ 2+ + 2+
(a) Ca < Mg < Be < K (b) Mg < Be < K < Ca isostructural?

2+ + 2+
(c) Be < K < Ca < Mg
2+ + 2+ 2+
(d) K < Ca < Mg < Be
2+ (a) CO2−
3 and NO3 (b) PCl+4 and SiCl4
(AIEEE 2007) (c) PF5 and BrF5 (d) AlF63− and SF6
Solution (AIEEE 2012)
(d) Higher is the charge/size ratio, higher is the polarizing Solution
power. −
(c) CO2− 2
3 and NO3 both are sp hybridized and the shape is
29. The hybridization of orbital of N atom in NO3− , NO2+ and NH+4 trigonal planar.
are, respectively:
PCl+4 and SiCl4 both are sp3 hybridized and the shape is
(a) sp2, sp, sp3 (b) sp, sp3, sp2
2 3 tetrahedral.
(c) sp , sp , sp (d) sp, sp2, sp3
(AIEEE 2011) PF5 is sp3d hybridized and the shape is trigonal bipyram-
Solution idal, whereas BrF5 is sp3d2 hybridized and its shape is
1 square pyramidal.
(a) The expression can be written as H = [V − M − c + a]
2
AlF63− and SF6 both are sp3d2 hybridized and the shape is
where V is the number of valence electrons around
octahedral.
central atom, M is the number of monovalent groups
attached, c is the charge on cation and a is the charge on 32. The number and type of bonds in C2−
2 ion in CaC2 are:
anion. (a) One s bond and one p−bond
1
For NO3−, H = [5 + 0 − 0 + 1] = 3 so the hybridization is (b) One s bond and two p−bonds
sp2. 2 (c) Two s bonds and two p−bonds
1 (d) Two s bonds and one p−bond
For NO2+, H = [5 + 0 − 1 + 0] = 2 so the hybridization is
sp. 2 (JEE Main Online 2014)
1 Solution
For NH+4 , H = [5 + 4 − 1 + 0] = 4 so the hybridization is
sp3. 2 (b) As can be seen from the figure below, there is one sigma
and two pi bonds between the two carbon atoms.
30. The structure of IF7 is:
C C
(a) trigonal bipyramidal (b) octahedral
(c) pentagonal bipyramidal (d) square pyramidal Ca
(AIEEE 2011) 33. Shapes of certain interhalogen compounds are stated
Solution below. Which one of them is not correctly stated?
(c) The structure can be explained on the basis of VSEPR (a) IF7 : pentagonal bipyramid
theory. The electronic configurations of iodine in the (b) BrF5: trigonal bipyramid
ground and excited state are: (c) BrF3 : planar T- shaped
Ground state: [Kr] 5s2 5p5 (d) ICl3 : planar dimeric
Excited state: [Kr] 5s15p35d3 (JEE Main Online 2014)

Chapter-4.indd 105 8/4/2016 10:18:15 AM


106 Chapter 4 Chemical Bonding

Solution
(b) BrF5 is square pyramidal with sp3d2 hybridization. PCl3 + Cl2 → PCl5 ; ⇒ sp3 → sp3d
F
S2O3− + I− → I2 + S4 O26 − ; ⇒ sp3 → sp3
F F
Only in option (d) hybridization is not changing.
Br 38. Which of the following is the hybridization state of the cen-
tral atom in a molecule that has a trigonal planar shape?
F F (a) sp3d (b) sp3 (c) sp2 (d) sp
Solution
34. The maximum number of 90° angles between bond pair of
electrons is observed in (c) The shapes corresponding to the hybridization states
(a) dsp3 hybridization. are
(b) sp3d2 hybridization. sp3d: Trigonal bipynamidal; sp3: Tetrahedral; sp2: Trigonal
(c) dsp2 hybridization. planar; sp: Linear
(d) sp3d hybridization. 39. Which one of the following compounds is non-polar?
Solution (a) CHCl3 (b) SiCl4 (c) SnCl2 (d) NH3
(b) In sp3d2 hybridization, the geometry is octahedral and Solution
all the bonds are at 90° to each other. (b) Molecules with either non-polar bonds or symmetric
35. The states of hybridization of boron and oxygen atoms in polar bonds are non-polar.
boric acid (H3BO3) are, respectively, H Cl
(a) sp2 and sp2 (b) sp3 and sp3
3
(c) sp and sp 2 (d) sp2 and sp3
C Si
Solution
(d) Ground state configuration of boron is 1s2 2 s2 2 p1x Cl Cl Cl Cl
Cl Cl
Excited state configuration is 1s2 2 s1 2 p1x 2 p1y
(Net dipole moment is downward) (Net dipole moment is 0)
This means that boron atom undergoes sp2 hybridiza-
tion and each of these three sp2 orbitals overlap with 2p
orbitals of O− ion to form three BO− bonds.
Sn N
36. Which one of the following has the regular tetrahedral
Cl Cl H H
structure?
(a) XeF4 (b) [Ni(CN)4]2− (c) BF4− (d) SF4 H
(Net dipole moment is upward) (Net dipole moment is upward)
Solution
In SiCl4, the bond moments are cancelled out completely
(c) The expression is so the net dipole moment is zero and the molecule is
(V + M − c + a) non-polar.
Hybridization =
2 Quick Tip/Alternate Solution: Just look for a molecule that
where V is the number of valence electrons of the metal, contains all bonds as non-polar or molecules with symmetric
M is the number of monovalent atoms attached, c is the polar bonds.
charge on cation and a is the charge on anion, we get
(3 + 4 − 0 + 1) 8 40. Which of the following molecules is non-polar in nature?
Hybridization of BF4− = = =4 (a) POCl3 (b) CH2O (c) SbCl5 (d) NO2
2 2
3
Therefore, the hybridization is sp . Solution
37. In which of the following chemical change, the hybridiza- (c) POCl3 has sp3 hybridization, is tetrahedral in shape; but
tion of the central atom of the polyatomic molecule is not contains different polar bonds due to which net dipole
changing? moment is not zero and the molecule is polar.
CH2O (or HCHO) has sp2 hybridization, is triangular
(a) ICl3 + Cl− → ICl4− (b) BF3 + F − → BF4− planar in shape; but contains one non-polar bond (i.e.,
(c) PCl3 + Cl2 → PCl5 (d) S2O3− + I− → I2 + S4 O26 − C O) due to which net dipole moment is not zero, and
the molecule is polar.
Solution
SbCl5 has sp3d hybridization, is trigonal bipyramidal
(d) ICl3 + Cl− → ICl4− ; ⇒ sp3d → sp3d 2 in shape; all Sb Cl bonds are polar but due to regular
geometry net dipole moment is zero, thus the molecule
BF3 + F − → BF4− ; ⇒ sp2 → sp3 is non-polar.

Chapter-4.indd 106 8/4/2016 10:18:20 AM


Solved Examples 107

NO2 has sp2 hybridization, bent shape because it contains There is no unpaired electron, so it is diamagnetic.
one non-bonding electron on central atom; each N O
O2+ = s 1s2 s * 1s2 s 2 s2 s * 2 s2 s 2 pz2 p 2 p2x = p 2 p2y p * 2 p1x
bond is polar, so net dipole moment is not zero and the
molecule is polar. There is one unpaired electron, so it is paramagnetic.
41. Hybridization, shape, and number of p bonds in XeOF2 mole- O2 = s 1s2 s * 1s2 s 2s2 s * 2s2 s 2p2z p 2p2x
cule are, respectively,
(a) sp3, pyramidal, 1 = p 2p2y p * 2p1x = p * 2p1y
(b) sp3d, trigonal bipyramidal, 2
There are two unpaired electrons, so it is paramagnetic.
(c) sp3d, T-shaped, 1
NO is isoelectronic with O2+ , so it also has one unpaired
(d) sp3d, square pyramidal, 1
electron, and is paramagnetic.
Solution 44. In which of the following ionization processes, the bond order
(c) The structure of XeOF2 molecule is T-shaped. has increased and the magnetic behaviour has changed?
(a) N2 → N2+ (b) C2 → C2+
F Xe F
(c) NO → NO + (d) O2 → O2+
O (AIEEE 2007)
The central atom in XeOF2 is Xe which carries 8 electrons Solution
in the valence shell. It forms three bond pairs (with two (c) In NO → NO +
atoms of F and one of O) and has two lone pairs. There are
NO → s 1s2 s * 1s2 s 2 s2 s * 2 s2 s 2 p2z p 2 p2x = p 2 p2y
a total of five electron pairs, so hybridization will be sp3d
and basic structure is expected to be trigonal bipyramidal. p * 2 p1x = p 2 p 0y
The presence of two lone pairs at the corners of the tri-
angle gives the molecule T-shape. There is one double (p) NO + → s 1s2 s * 1s2 s 2 s2 s * 2 s2 s 2 p2z p 2 p2x = p 2 p2y
bond present between Xe and O due to dp-pp bonding. .
The electron has been removed from antibonding orbital,
Molecular Orbital Theory so bond order increases and nature changes from para-
42. Which of the following molecules/ions does not contain magnetic to diamagnetic.
unpaired electrons? 45. Using MO theory, predict which of the following species has
(a) O2−
2 (b) B2 (c) N2+ (d) O2 the shortest bond length?
(AIEEE 2006)
Solution (a) O2− (b) O2−
2 (c) O2+
2 (d) O2+
(AIEEE 2009)
(a) The electronic configuration of O2− 2 2 2
s2 s 2 p2z p 2 p2x = p 2 p2y p * 2 p2x = p * 2 p2y .
2 is s 1s s * 1s s 2 s s * 2Solution
s 1s2 s * 1s2 s 2 s2 s * 2 s2 s 2 p2z p 2 p2x = p 2 p2y p * 2 p2x = p * 2 p2y . (c) We know, bond order = (Nb − Na)/2.
Hence, there are no unpaired electrons. The molecular orbital configuration of O2− is
2 2 2 2 2 2 2 2 1
(b) The electronic configuration of B2 is s 1s2 s * 1s2 s 2 s2 s * 2 s2 ps21ps1x s= *p12sp1ys. 2s s * 2s s 2pz p 2p x = p 2p y p * 2p x = p * 2p y
s 1s2 s * 1s2 s 2 s2 s * 2 s2 p 2 p1x = p 2 p1y . Hence, there are two unpaired Bond order of O2− = (1/2) (10 − 7) = 1.5.
electrons.
The molecular orbital configuration of O2− 2 is
(c) The electronic configuration of N2+ is s 1s2 s * 1s2 s 2 s2 s * 2 s2 p 2 p22x = p 2 p22y s 2 p2 1z .
s 1s s * 1s s 2s s * 2s s 2pz p 2p x = p 2p y p * 2p2x = p * 2p2y
2 2 2 2
s 1s2 s * 1s2 s 2 s2 s * 2 s2 p 2 p2x = p 2 p2y s 2 p1z . Hence, there is one
unpaired electron. Bond order of O2− 2 = (1/2) (10 − 8) = 1.
The molecular orbital configuration of O2+ 2 is
(d) The electronic configuration of O2 is s 1s2 s * 1s2 s 2 s2 s * 2 s2 s 2 p2x = p 2 p2y p * 2 p1x = p * 2 p1y .
s 1s2 s * 1s2 s 2 s2 s * 2 s2 s 2 p2x = p 2 p2y p * 2 p1x = p * 2 p1y . Hence, there are s 1s s * 1s s 2 s s * 2 s s 2 pz p 2 p x = p 2 p2y
2 2 2 2 2 2

two unpaired electron. Bond order of O2+ 2 = (1/2) (10 − 4) = 3.


The molecular orbital configuration of O2+
43. Which of the following species exhibits the diamagnetic
behaviour?
s 1s2 s * 1s2 s 2 s2 s * 2 s2 s 2 pz2 p 2 p2x = p 2 p2y p * 2 p1x
(a) NO (b) O2−
2 (c) O2+ (d) O2
(AIEEE 2007) Bond order of O2+ = (1/2) (10 − 5) = 2.5.
Solution
(b) The electronic configurations are as follows: Out of these, O2+
2 has the highest bond order, so the low-
est bond length as bond length is inversely proportional
O22 − = s 1s2 s * 1s2 s 2s2 s * 2s2 s 2pz2 p 2p2x
to bond order.
= p 2p2y p * 2p2x = p * 2p2y

Chapter-4.indd 107 8/4/2016 10:18:30 AM


108 Chapter 4 Chemical Bonding

46. The bond dissociation energy of B F in BF3 is 646 kJ mol−1, Solution


whereas that of C F in CF4 is 515 kJ mol−1. The correct rea- (b) The electronic configuration of given species are:
son for higher B F bond dissociation energy as compared to
that of C F is the
N2 = 14 e − = s 1s2 ,s * 1s2 ,s 2 s2 ,s * 2 s2, p 2 p2y , s 2 p2x p 2 pz2
(a) smaller size of B atom as compared to that of C atom.
(b) stronger s bond between B and F in BF3 as compared to O2− = 17e − = s 1s2 ,s * 1s2 ,s 2 s2 ,
that between C and F in CF4.
(c) significant pp–pp interactions between B and F in BF3, s * 2 s2s 2 p2x p 2 p2y p * 2 p2y p 2 p2z p * 2 p1z
whereas there is no possibility of such interaction N22 + = 12e − = s 1s2 ,s * 1s2 ,s 2 s2 ,s * 2 s2 , p 2 p2x p 2 p2y
between C and F in CF4.
(d) lower degree of pp–pp interactions between B and F in O22 − = 18e − = s 1s2 ,s * 1s2 ,s 2 s2 ,
BF3 than that between C and F in CF4. (AIEEE 2009)
s * 2 s2 s 2 p2x p 2 p2y p * 2 p2y p 2 p2z p * 2 p2z
Solution
(c) Bond strength of B F bond is higher in BF3 as compared
to C F bond in CF4 due to formation of extra pp -pp Only O2− has one unpaired electron in anti-bonding
bond between B and F. molecular orbital.
F
50. Which one of the following molecules is paramagnetic?
+ B−
F F (a) N2 (b) NO (c) CO (d) O3
pp -pp bonding is not possible in CF4. (JEE Main Online 2014)
Solution
47. Which one of the following molecules is expected to exhibit
(b) N2, CO, O3 all diamagnetic in nature while (NO) is par-
diamagnetic behaviour?
amagnetic because it has one unpaired electron in its
(a) N2 (b) O2 (c) S2 (d) C2
molecular orbital.
(JEE Main 2013)
Solution
= s 1s2 ,s * 1s2 ,s 2 s2 ,s * 2 s2 ,s 2 pz2 p 2 p2x = p 2 p2y , p * 2 p1x
(a), (d)
p 2 p2y
N2 = s 1s2s *1s2 , s 2 s2s * 2 s2 , 
2
s 2px 51. After understanding the Statement 1 and Statement 2,
2 choose the correct option.
p 2 pz
Statement 1: In the bonding molecular orbital (MO) of H2,
p 2 p y2 p * 2 p1y electron density is increased between the nuclei.
O2 = s 1s2s * 1s2 , s 2 s2s * 2 s2s 2 p x ,  
2
p * 2 p1z Statement 2: The bonding MO is ψ A + ψ B ′ which shows
p 2 pz
destructive interference of the combining electrons waves.
p 2 p y2 (a) Statement 1 and Statement 2 are correct and State-
C2 = s 1s2s * 1s2 , s 2s2s * 2s2  ment 2 is the correct explanation for the Statement 1.
2
p 2 pz (b) Statement 1 and Statement 2 are correct, but State-
N2 and C2 exhibit diamagnetic behaviour as they do not ment 2 is not the correct explanation for the Statement 1.
possess unpaired electrons, while O2 and S2 exhibit par- (c) Statement 1 is correct, Statement 2 is incorrect.
amagnetic behaviour due to the presence of unpaired (d) Statement 1 is incorrect, Statement 2 is correct.
electrons. (JEE Main Online 2015)
48. Which one of the following properties is not shown by NO? Solution
(a) It is diamagnetic in gaseous state. (c) H2 has bond order 1, is formed by bonding molecular
(b) It is a neutral oxide. orbitals. So Statement 1 is correct.
(c) It combines with oxygen to form nitrogen dioxide. ψA + ψB give the constructive interference. So
(d) Its bond order is 2.5. (JEE Main 2014) Statement 2 is wrong.
Solution
52. Which one of the following species is diamagnetic in nature?
(a) The MO configuration of NO is s 1s2 s * 1s2 s 2s2 s * 2s2s 2p2z p 2p(a)
2
p2y+p * 2p1x (b) H
x p 2He (c) H+ (d) H−
2 2 2 2
s 1s2 s * 1s2 s 2s2 s * 2s2s 2p2z p 2p2x p 2p2y p * 2p1x
Solution
As can be seen, it is an odd electron species and so para-
(b) According to MO theory, configuration of H2 is
magnetic in gaseous state with one unpaired electron.
s 1s2 s *1s2. There are no unpaired electrons, so it is
Bond order = (10 − 5)/2 = 2.5
diamagnetic.
49. Which of the following has unpaired electron(s)?
53. Which of the following is not correct regarding C2 molecule?
(a) N2 (b) O2− (c) N2+
2 (d) O2+
2 (a) C2 molecule has been found to exist in vapour phase.
(JEE Main Online 2014)

Chapter-4.indd 108 8/4/2016 10:18:34 AM


Advanced Level Problems 109

(b) It has total 12 electrons, out of which 8 electrons occupy 55. The bond order of CO and NO is
bonding orbitals, while 4 electrons occupy antibonding (a) 3 and 2 (b) 3 and 2.5
orbitals. (c) 3 and 1.5 (d) 3 and 3.5
(c) The molecule is paramagnetic.
Solution
(d) C2 molecule contains double bond and both are
p bonds. (b) The electronic configuration of CO is

Solution s 1s2 s * 1s2 s 2 s2 s * 2 s2 p 2 p2x = p 2 p2y s 2 p2z


Nb − Na 10 − 4
(
(c) C2 = (s 1s )2 (s * 1s )2 (s 2 s )2 (s * 2 s )2 p 2 p2x = p 2 p2y ) Bond order =
2
=
2
=3
Four electrons are present in 2p molecular orbitals; that The electronic configuration of NO is
is why double bond contains both p bonds.
s 1s2 s * 1s2 s 2 s2 s * 2 s2 p 2 p2x = p 2 p2y s 2 p2z p * 2 p1x
Molecule is diamagnetic as it does not have any unpaired
electron. 10 − 5
Bond order = = 2.5
2
54. Select the incorrect statement among the following:
(a) NF3 is a weaker base than NH3.
(b) AlCl3 is largely covalent while AlF3 is largely ionic. Hydrogen Bonding
(c) B2H6⋅2NH3 is ionic.
(d) NO has one unpaired electron in bonding molecular 56. Which of the following hydrogen bonds is the strong-
orbital. est?
(a) O H F (b) O H N
Solution (c) F H F (d) O H O (AIEEE 2007)
(d) MO configuration of NO: Solution
(c) The strength of hydrogen bonding depends on the elec-
s 1s2 s * 1s2 s 2s2 s * 2s2s 2p2x p 2p2x p 2p2y p * 2p1y tronegativity of the other atom. The hydrogen bond in HF
NO has one unpaired electron in antibonding p molecu- is the strongest because fluorine is the most electronega-
lar orbital. tive element.

ADVANCED LEVEL PROBLEMS


1. Among the following, the paramagnetic compound is 2. The species having bond order different from that in CO is
(a) Na2O2 (b) O3 (c) N2O (d) KO2 (a) NO− (b) NO+ (c) CN− (d) N2
(IIT-JEE 2007) (IIT-JEE 2007)
Solution Solution
(d) In Na2O2, oxygen is present as peroxide, that is, O2− 2 . (a) NO− is a 16 electron system (7 + 8 + 1 = 16) with bond order
Considering molecular orbital theory for peroxide, we have 2 whereas the other molecules like CO, NO+, CN− and N2
O22− = s 1s2 s * 1s2 s 2 s2 s * 2 s2 s 2 pz2 p 2 p2x = p 2 p2y are 14 electron systems with bond order of 3.

p * 2 p2x = p * 2 p2y 3. Match each of the diatomic molecules in Column I with its
property/properties in Column II.
We can see that there are no unpaired electrons.
Column I Column II
Considering the structure of O3, there are no unpaired
electrons. (a) B2 (p) Paramagnetic
+ +
O − O (b) N2 (q) Undergoes oxidation

O O O O (c) O2− (r) Undergoes reduction
In N2O, there are no unpaired electrons
(d) O2 (s) Bond order 2
N N O
(t) Mixing of s and p orbitals
N2O = s 1s2 s * 1s2 s 2 s2 s * 2 s2 s 2 pz2 p 2 p2x = p 2 p2y (IIT-JEE 2009)
p * 2 p2x s * 2 pz2 2
s 3s s * 3s 2 Solution
(a) → p, r, t; (b) → s, t; (c) → p, q; (d) → p, q, s
In KO2, oxygen is present as O2−, that is, superoxide. The
The electronic configuration B2 is s 1s2 s * 1s2 s 2 s2 s * 2 s2 p 2 p1x = p 2 p1y
molecular orbital theory for superoxide ion is 2
s 1s s * 1s s 2 s s * 2 s p 2 p1x = p 2 p1y. Thus, it contains two unpaired electrons,
2 2 2

O2− = s 1s2 s * 1s2 s 2 s2 s * 2 s2 s 2 pz2 p 2 p2x = p 2 p2y p * 2 p2x undergoes reduction and there is mixing of s and p orbitals
because they are equal in energies.
= p * 2 p1y
The electronic configuration N2 is s 1s2 s * 1s2 s 2 s2 s * 2 s2 p 2 p2x = p 2 p2y s 2 p2z
There is one unpaired electron in p*2py orbital. s 1s Thus,
2
s * 1s itsis2 s s * 2 s p 2 p2x = p 2 p2y s 2 p2z . Thus, the bond order is (10 − 4)/2
2 2 2
paramagnetic in nature.

Chapter-4.indd 109 8/4/2016 10:18:38 AM


110 Chapter 4 Chemical Bonding

= 6/2 = 3. There is mixing of s and p orbitals because they 8. Using the data provided, calculate the multiple bond energy
are equal in energies. (kJ mol−1) of a C C bond in C2H2. That energy is (take the
The electronic configuration O2− is s 1s2 s * 1s2 s 2 s2 s * 2 s2 bond energy of C H bond as 350 kJ mol−1).

s 2 p2z p 2 p2x = p 2 p2y p * 2 p2x p * 2 p1y . Thus, it contains 2C(s) + H2 (g) → C2H2 ; ∆H = 225 kJ mol−1
1 unpaired electron and undergoes oxidation. 2C(s) → 2C(g); ∆H = 1410 kJ mol−1
2 2 2 2
The electronic configuration O2 is s 1s s * 1s s 2 s s * 2 s H2 (g) → 2H(g); ∆H = 330 kJ mol−1
s 2 p2z p 2 p2x = p 2 p2y p * 2 p1x p * 2 p1y . Thus, it contains
(a) 1165 kJ mol−1 (b) 837 kJ mol−1
two unpaired electrons and undergoes oxidation. (c) 865 kJ mol−1 (d) 815 kJ mol−1
Therefore, the bond order is (10 − 6)/2 = 4/2 = 2. (IIT-JEE 2012)
Solution
4. The bond energy (in kcal mol−1) of C C single bond is,
approximately, (d) We know that
(a) 1 (b) 10 Binding energy = Total energy of reactants − Total energy
(c) 100 (d) 1000 of products.
(IIT-JEE 2010) So, we have
Solution 225 = 2(C C) + 1(H H) − {2(C H) + (C C)}
(c) The bond energy of C C single bond is approximately 225 = 1410 + 330 − {2 × 350 + (C C)}
100 kcal mol−1. Energy of (C C) bond =1740 − 925 = 815 kJ mol−1
Thus, solving, we get the value of C C as 815 kJ mol−1.
5. The species having pyramidal shape is
(a) SO3 (b) BrF3 9. A list of species having the formula XZ4 is given below.
(c) SiO2−
3 (d) OSF2 XeF4, SF4, SiF4, BF4−, BrF4−, [Cu(NH3)4]2+, [FeCl4]2−, [CoCl4]2−
(IIT-JEE 2010)
and [PtCl4]2–.
Solution
Defining shape on the basis of the location of X and Z atoms,
(d) OSF2 has 3 bond pairs, one lone pair, with hybridization
the total number of species having a square planar shape
sp3 and pyramidal geometry.
is ___.
6. Based on VSEPR theory, the number of 90° F Br F angles in (JEE Advanced 2014)
BrF5 is ___.
Solution
(IIT-JEE 2010) (4) The structures are as follows. Out of these, XeF4, BrF4−,
Solution
[Cu(NH3)4]2+ and [PtCl4]2– have square planar shape.
(0) The structure of BrF5 is
F F
F F F
Xe S Si
Br F F F F F
F F
F F
2+ Cl 2−
F F F F H3N NH3
F Br Cu Fe
F F H3N NH3 Cl Cl
All the four bonds (F Br F) will reduce from 90o to 84.8o Cl
after lone pair–bond pair repulsion.
Cl 2− 2−
7. Assuming that Hund’s rule is violated, the bond order and Cl Cl
magnetic nature of the diatomic molecule B2 is Co Pt
(a) 1 and diamagnetic. (b) 0 and diamagnetic. Cl Cl Cl Cl
(c) 1 and paramagnetic. (d) 0 and paramagnetic Cl

(IIT-JEE 2010) 10. Assuming 2s-2p mixing is NOT operative, the paramagnetic
Solution species among the following is
(a) Be2 (b) B2 (c) C2 (d) N2
(a) The configuration of
(JEE Advanced 2014)
B2 (10 ) = s 1s2 s * 1s2 s 2 s2 s * 2 s2p 2 p x 2
Solution
6−4
Therefore, the bond is = = 1. (c) Out of the options, C2 has MO configuration
2
So, the nature is diamagnetic as there are no unpaired s 1s2 s * 1s2 s 2s2 s * 2s2 s 2p2z p 2p1x = p 2p1y
electrons. with two unpaired electrons, so it is paramagnetic.

Chapter-4.indd 110 8/4/2016 10:18:43 AM


Advanced Level Problems 111

11. Match the orbital overlap figures shown in List-I with the Cl Cl Cl Cl
description given in List-II and select the correct answer using Al Al C2H5
the code given below the lists. Cl O H Cl O
C2H5
H
List-I List-II
Hydrated AlCl3 Anhydrous AlCl3

P. 1. p–dp antibonding 14. Using VSEPR theory deduces the structures of PCl5 and BrF5.
Solution
In PCl5 : P [Ne] 3s2 3p3
2. d–ds bonding 3s 3p 3d
Q.
Excited
state of P
sp3d
3. p–dp bonding
R. Cl
Cl
Cl P Trigonal bipyramidal
Cl
S. 4. d–ds antibonding Cl

In BrF5 : Br [Ar] 4s2 4p5


Code: 4s 4p 4d
P Q R S
(a) 2 1 3 4
(b) 4 3 1 2 sp3d2
Lone pair
(c) 2 3 1 4
(d) 4 1 3 2 (JEE Advanced 2014)
Solution F F
Br Square pyramidal
(c) The atomic orbital overlap is the key feature that explains F F
the bond formation. Bonding orbitals are formed by F
in-phase overlap of atomic orbitals while antibonding
orbitals are formed by out-of-phase overlap between
atomic orbitals. 15. Draw the shape of XeF4 and OsF4 according to VSEPR theory.
Show the lone pair of electrons on the central atom.
12. BeCl2, N3− , N2O, NO2+, O3, SCl2, ICl2, I 3−
and XeF2, the total
number of linear molecules(s)/ions(s) where the hybridization Solution
of the central atom does not have contribution from the d-
orbital(s) is ___.
F F F
[Atomic number: S = 16, Cl = 17, I = 53 and Xe = 54] O
Xe
(JEE Advanced 2015) F S
F F
Solution F
(4) BeCl2, N3− ,
N2O and NO2+ are
linear molecules where the F
hybridization of the central atom does not have contribu-
Two lone pairs No lone pair
tion from the d-orbital(s).
Square planar Trigonal bipyramidal
13. Which of the two, anhydrous or hydrated AlCl3 is more soluble sp3d2 hybridization sp3d hybridization
in diethyl ether? Justify using the concepts of bonding in not
more than 2 or 3 sentences. 16. On the basis of ground state electronic configuration arrange
the following molecules in increasing O O bond length
Solution order.
Anhydrous AlCl3 is more soluble in diethyl ether because the KO2, O2 and O2[AsF6].
oxygen atom of the ether molecule donates its lone pair of
electrons to the vacant orbital of electron deficient alumin- Solution
ium of AlCl3 through coordinate bond formation. However, In KO2, the anion is O2− with electronic configuration
in case of hydrated AlCl3, the aluminium atom is not electron
deficient as the oxygen atom from the water molecule has s 1s2 s * 1s2 s 2 s2 s * 2 s2 s 2 p2z p 2 p2x = p 2 p2y p * 2 p2x = p * 2 p1y
already donated its electron pairs to fulfill the electron defi- Nb − Na 10 − 7 3
ciency of Al. Bond order = = =
2 2 2

Chapter-4.indd 111 8/4/2016 10:18:47 AM


112 Chapter 4 Chemical Bonding

The electronic configuration of O2 is


The configuration of O2+ is s 1s2 s * 1s2 s 2 s2 s * 2 s2 s 2 p2x p 2 p2x = p 2 p2y p * 2 p1y
s 1s2 s * 1s2 s 2 s2 s * 2 s2 s 2 p2z p 2 p2x = p 2 p2y p * 2 p1x = p * 2 p1y
s 1s2 s * 1s2 s 2 s2 s * 2 s2 s 2 p2x p 2 p2x = p 2 p2y p * 2 p1y
N − Na 10 − 6 4
Bond order = b == = =2 Since it contains one unpaired electron, so O2+ are
2 2 2 paramagnetic.
In O2[AsF6], the cation is O2+ with electronic configuration Nb − Na 10 − 5 5
Bond order = = = = 2.5
O2+ = s 152 s * 152 s 2 s2 s * 2 s2 s 2 p2z p 2 p2x = p 2 p2y p * 2 p1x 2 2 2

= p * 2 p 0y Hence, bond order of O2+ is greater than O2.


10 − 5 5 18. Why does R3NO have higher dipole moment as compared to
Bond order == = 2.5
2 2 R3PO?
Bond length is inversely proportional to bond order. So, Solution
Bond length order is O2+ < O2 < O2− .
This is due to back bonding from O to P atom.
17. According to MO theory,
(a) O2+ is paramagnetic and bond order greater than O2. + − + −
R3N O; R3P O R 3P O
(b) O2+ is paramagnetic and bond order less than O2.
(c) O2+ is diamagnetic and bond order less than O2. (m = 5.0 D) (m = 4.37 D) (3d) (2p)
(d) O2+ is diamagnetic and bond order more than O2. 19. Although electronegativities of N and Cl are the same, but N
Solution shows hydrogen bonding and Cl does not. Explain.
(a) The configuration of O2 is Solution
s 1s2 s * 1s2 s 2 s2 s * 2 s2 s 2 p2z p 2 p2x = p 2 p2y The size of N atom (2s2 2p3) is smaller than Cl atom (3s2 3p3).
This gives rise to more electron density on N atom which is
p * 2 p1x =p * 2 p1y
Nb − Na 10 − 6 4 responsible for H bonding in addition to electronegativity.
Bond order = = = =2
2 2 2

PRACTICE EXERCISE
Level I 6. The correct order of decreasing polarizable ions is
(a) Cl−, Br−, I−, F− (b) F−, I−, Br−, Cl−
Single Correct Choice Type (c) F−, Cl−, Br−, I− (d) I−, Br−, Cl−, F−
1. Structure of IF4+ and hybridization of iodine in this structure are,
7. The concept that electron pairs in the valence shell of an atom
respectively,
bonded to other atoms stay as far apart as possible to mini-
(a) linear, sp3d (b) K-shaped, sp3d2
mize repulsions between them is part of the
(c) irregular tetrahedral, sp3d (d) square planar, sp3d 2
(a) Pauli principle.
2. The chlorine end of the chlorine monoxide molecule carries a (b) Heisenberg uncertainty principle.
charge of +0.167e. The bond length is 154.6 pm. Calculate the (c) valence shell electron pair repulsion theory.
dipole moment of the molecule in Debye units. (d) electronegativity and polar bonds theory.
(a) 2.35 D (b) 1.24 D (c) 1.59 D (d) 2.05 D
8. Most favourable conditions for the formation of ionic bonds
3. The dipole moment of KCl is 3.336 × 10–29 Cm. The interatomic are
distance K+ and Cl− ion in KCl is 260 pm. Calculate the dipole (a) large cation and small anion.
moments of KCl, if there were opposite charges of the funda- (b) large cation and large anion.
mental unit located at each nucleus. (c) small cation and small anion.
(a) 4.165 × 10–29 Cm (b) 4.325 × 10–27 Cm (d) small cation and large anion.
–29
(c) 5.321 × 10 Cm (d) 5.012 × 10–27 Cm
9. Predict the molecular geometry and polarity of the SO2 mole-
4. A molecule has a central atom surrounded by 2 lone pairs and cule by applying VSEPR theory.
3 atoms. The best description for the shape of the molecule is (a) Linear, nonpolar (b) Linear, polar
(a) trigonal bipyramidal (b) octahedral. (c) Bent, 109.5° angle, polar (d) Bent, 120° angle, polar
(c) bent. (d) T-shaped.
10. Polarization of ions is governed by
5. Which of the following has pp–dp bonding? (a) hybridization. (b) VSEPR theory.
(a) NO3− (b) SO2− (c) BO2− (d) CO2− (c) Fajans’ rules. (d) Pauling’s rule.
3 3 3

Chapter-4.indd 112 8/4/2016 10:18:53 AM


Practice Exercise 113

11. Calculate the lattice enthalpy of CaCl2, given that the enthalpy 21. The bond angle in ammonia molecule is
of (a) 93°3’ (b) 91°8’ (c) 106°45’ (d) 109.5°
(i) sublimation of Ca is 121 kJ mol−1,
22. Which of the following can be calculated from the Born–
(ii) dissociation of Cl2 to Cl is 242.8 kJ mol−1,
Haber cycle of formation of Al2O3?
(iii) ionization of Ca to Ca2+ is 2422 kJ mol−1,
(a) Lattice enthalpy of Al2O3
(iv) electron gain enthalpy for Cl to Cl− is −355 kJ mol−1 and
(b) Electron gain enthalpy of O atom
(v) formation of CaCl2 is −795 kJ mol−1.
(c) Hydration enthalpy of Al3+
(a) −1985.6 kJ mol−1 (b) −1785.3 kJ mol−1
(d) Ionization enthalpy of Al
(c) −2870.8 kJ mol−1 (d) −2089.6 kJ mol−1
23. Lattice enthalpy of an ionic compound depends on
12. Which statement is true for the BrF5 molecule?
(a) charge on the ion only.
(a) There are two lone pairs in the valence shell of the bro-
(b) size of the ion only.
mine atom.
(c) packing of ions only.
(b) The molecule is nonpolar.
(d) charge on the ion and size of the ion.
(c) Some of the F Br F bond angles are close to 90 degrees.
(d) At least one of the bond angles, F Br F, is about 109.5 24. Based on conclusions from application of the VSEPR theory,
degrees. which one of the following species is linear?
(a) BF3 (b) I 3− (c) H2CO (d) H2S
13. As compared to covalent compounds, electrovalent com-
pounds generally have 25. Which of the following is least ionic?
(a) low melting points and low boiling points. (a) CaBr2 (b) CaI2 (c) CaCl2 (d) CaF2
(b) high melting points and high boiling points.
(c) low melting points and high boiling points. 26. The compound with no dipole moment is
(d) high melting points and low boiling points. (a) methyl chloride. (b) carbon tetrachloride.
(c) methylene chloride. (d) chloroform.
14. Which basic arrangement would best accommodate three
electron domains in the valence shell of a covalently bonded 27. When two ice cubes are pressed together, they join to
atom? form one cube. Which of the following forces holds them
(a) Planar triangular (b) Octahedral together?
(c) Tetrahedral (d) Linear (a) Hydrogen bond formation
(b) van der Waals forces
15. The correct order of Cl O bond lengths in ClO − , ClO2− , ClO3− (c) Covalent attraction
and ClO 4− is (d) Dipole interaction
(a) ClO − < ClO2− < ClO3− < ClO 4−
28. Carbon dioxide is isostructural with
(b) ClO 4− < ClO3− < ClO2− < ClO − (a) HgCl2 (b) SnCl2 (c) C2H4 (d) NO2−
(c) ClO3− < ClO 4− < ClO2− < ClO − 29. The correct order of dipole moment is
(d) ClO 4− = ClO3− = ClO2− < ClO − (a) CH4 < NF3 < NH3 < H2O (b) NF3 < CH4 < NH3 < H2O
(c) NH3 < NF3 < CH4 < H2O (d) H2O < NH3 < NF3 < CH4
16. On hybridization of one s-orbital and one p-orbital, we get
(a) mutual perpendicular orbitals. 30. The bond present in N2O5 is
(b) two orbitals at 180° to each other. (a) only ionic.
(c) four orbitals directed tetrahedrally. (b) covalent and coordinate.
(d) three orbitals in one plane. (c) only covalent.
(d) covalent and ionic.
17. What fraction of charge is present on iodine in covalently
bonded Hδ+ − Iδ− if the dipole moment of HI is 0.38 D and the 31. The pair of elements from ionic bond is
bond length is 1.61 Å. (a) C + Cl (b) H + F (c) Na + Br (d) O + H
(a) 5% (b) 16% (c) 33% (d) 79%
32. The types of bonds present in CuSO4⋅5H2O are only
18. The central atom in BrF5 uses sp3d2 hybridization. It must (a) electrovalent and covalent.
therefore have _____ s bonds and _____ lone pair(s) of elec- (b) electrovalent and coordinate covalent.
trons in the valence shell of the central atom respectively. (c) electrovalent, covalent and coordinate covalent.
(a) 1, 5 (b) 3, 3 (c) 5, 0 (d) 5, 1 (d) covalent and coordinate covalent.

19. The species in which central atom uses sp2 hybrid orbitals in 33. What is the nature of chemical bonding between Cs and F?
its bonding is (a) Ionic (b) Covalent
(a) PH3 (b) NH3 (c) CH3+ (d) SbH3 (c) Coordinate (d) Metallic
20. The shape of sulphate ion is 34. The smallest F—S—F bond angle in SF6 is
(a) hexagonal. (b) square planar. (a) 90 degrees. (b) 109.5 degrees.
(c) trigonal bipyramidal. (d) tetrahedral. (c) 120 degrees. (d) 145 degrees.

Chapter-4.indd 113 8/4/2016 10:18:56 AM


114 Chapter 4 Chemical Bonding

35. What is the effect of more electronegative atom on the 48. According to VSEPR theory, the most probable shape of the
strength of ionic bond? molecule having 4 electron pairs in the outermost shell of
(a) Increases the central atom is
(b) Decreases (a) linear. (b) tetrahedral.
(c) Remains the same (c) hexagonal. (d) octahedral.
(d) First increases then decreases.
49. Which of the following have same shape as NH2+?
36. Formal charge on the central oxygen atom in a molecule of (a) CO2 (b) SnCl2 (c) SO2 (d) BeCl2
ozone is
(a) 0 (b) 1+ (c) 1− (d) 2− Level II
37. The HF molecule has a dipole moment of 1.83 D and a bond Multiple Correct Choice Type
length of 91.7 pm. What is the amount of charge, in elec- 50. Polarity of a molecule is calculated in terms of dipole moment
tronic charge units, on either end of the bond? pointing by an arrow from +ve to −ve pole. Find out the cor-
(a) +0.416e, −0.416e (b) +0.356e, −0.416e rect statement among the following.
(c) +0.416e, −0.356e (d) +0.256e, −0.532e (a) Carbonyl group shows greater dipole moment than car-
38. The bond order in N2+ ion is bon monoxide.
(a) 1 (b) 2 (c) 2.5 (d) 3 (b) In carbon monoxide C atom forms negative pole of
39. In N2H4, nitrogen is dipole whereas O atom forms positive pole even though
(a) sp hybridized. (b) sp2 hybridized. O is more electronegative than C.
(c) sp3 hybridized. (d) dsp2 hybridized. (c) Dipole moment of CH3Cl is greater than CH3F.
(d) All homonuclear polyatomic molecules are non-polar.
40. Which of the following ions has a pseudo-inert gas
configuration? 51. Which of the following combination(s) of atomic orbitals
(a) Zn2+ (b) Cu+ suggest non-bonding combinations of atomic orbitals?
(c) Ag+ (d) All of these +
+ + +
(a) (b)
41. The Se C Se molecule is non-polar because − −
(a) the bonds in the molecule are all non-polar.
(b) the bonds in the molecule are polar but their effect on
+ + +
the overall polarity is canceled by the effect of lone pairs − +
(c) − (d)
in the valence shell of the carbon atom. −
(c) the bonds in the molecule are polar but their effect on
the overall polarity is cancelled by the fact that they are 52. CO2 is isostructural with
equal in magnitude and oppositely directed. (a) HgCl2 (b) SnCl2 (c) C2H2 (d) NO2
(d) the bonds in the molecule are polar but the polar effect
53. Which of the following conditions apply to resonating
is canceled by the resonance hybrids which distribute
structures?
the charge evenly.
(a) Identical arrangement of atoms
42. In XeF2, XeF4 and XeF6, the number of lone pairs of Xe is, (b) Nearly same energy content
respectively, (c) Identical number of bonds
(a) 2, 3, 1 (b) 1, 2, 3 (c) 4, 1, 2 (d) 3, 2, 1 (d) Same number of unpaired electrons.
43. The bond in the formation of fluorine molecule will be 54. Mark out the incorrect match of shape.
(a) due to s–s overlapping. (b) due to s–p overlapping. (a) XeOF2 : Trigonal planar
(c) due to p–p overlapping. (d) due to hybridization. (b) ICl4− : Square planar
44. Shape of NH3 is very similar to (c) [SbF5]2− : Square bipyramidal
(a) CH4 (b) CH3− (c) BH3 (d) CH3+ (d) NH2− : Pyramidal
45. In a homonuclear molecule, which of the following set of 55. Which among the following species have zero bond order?
orbitals are degenerate?
(a) s 2s and s 1s (b) p 2px and p 2py (a) F22− (b) Ar2 (c) He2+ (d) H2+
(c) p 2px and s 2pz (d) s 2pz and s *2px 56. The linear structure is assumed by
46. Which of the following factors do not favour electrovalency? (a) SnCl2 (b) NCO− (c) NO2+ (d) CS2
(a) Low charge on ions 57. Which of the following set correctly represent sp3 hybridiza-
(b) High charge on ions tion and tetrahedral geometry?
(c) Large cation and small anion
(a) ClO 4− , SO2−
4 , Diamond (b) SiO 4− +
4 , [PCl4] , SiC
(d) Small cation and large anion
(c) Ni(CO)4, SnCl4, Graphite (d) Ice, FClO3, [Zn(H2O)4]2+
47. Which of the following molecules has dipole moment?
(a) ClF3 (b) TeCl4 58. Which among the following have regular geometry?
(c) SF4 (d) All of these (a) BF3 (b) NF3 (c) PF3 (d) BF4−

Chapter-4.indd 114 8/4/2016 10:19:01 AM


Practice Exercise 115

59. Which of the following molecules are polar and sp3d2 68. The bond angle is Cl2O more than that in F2O because of so
hybridized? many reasons like
(a) IF5 (b) XeF4 (c) XeOF4 (d) ICl4− (I) pp–dp overlap in Cl2O, which is not possible in F2O.
(II) There is no lone pair–lone pair repulsion in Cl2O, which is
60. Among the following bonds, which are more ionic than
present in F2O.
covalent?
(III) F is more electronegative than oxygen while Cl is less
(a) Si O (b) Ba O (c) Se Cl (d) K Br
electronegative than oxygen.
61. Which among the following have bond order 2.5? The correct reason is (are)
(a) O2 (b) N2− 2 (c) N2+ (d) O2+ (a) Both I and II. (b) II and III.
(c) Only II. (d) All of these.
62. In O2 molecule, which of the following molecular orbitals are
lower in energy than s2pz orbital? 69. Which is correct about the C F bond?
(a) p 2px (b) s 2s (c) s *2s (d) p 2py (a) C F is shorter than expected because of lone pair–lone
pair repulsion.
63. Which of the following have identical bond order? (b) C F is longer than expected because of lone pair–lone
(a) CN− (b) O2− (c) NO+ (d) CN+ pair repulsion.
(c) C F is shorter than expected because of high ionic
Passage Type character.
Paragraph for Questions 64 to 66: “No ionic compound is 100% (d) C F is longer than expected because of high ionic
ionic as well as no covalent compound is 100% covalent.” character.

64. Correct solubility order is 70. In which of the following cases, the central atom–F bond has
(a) AgCl < AgF (b) BeCO3 < BaCO3 partial double bond character?
(a) NF3 (b) CF4 (c) PF3 (d) OF2
(c) LiNO3 < CsNO3 (d) NaClO4 < KCIO4
65. Choose the incorrect order of the given properties. Paragraph for Questions 71 to 73: Valence shell electron pair
repulsion (VSEPR) theory can be used to predict the approximate
(a) BeCl2 > MgF2 > CaF2 : Electrical conductivity
shape of a molecule. Electrons in bonds and in lone pairs can
(b) NaF< MgF2< AlF3 : Covalent character order be thought of as “charge cloud” that repel one another and stay
(c) BeSO4 < MgSO4 < CaCO4 : Thermal stability order as far apart possible, thus causing molecules to assume specific
(d) HgCl2< HgBr2< HgI2 : Solubility order in water shapes.

66. Which of the following option gives incorrect melting point The repulsive interactions of electron pairs decrease in the order:
order? Lone pair–lone pair > Lone pair–bond pair > Bond pair–bond
(a) H2 < T2 (b) He > T2 (c) D2 > He (d) T2 > D2 pair.
These repulsions result in deviations from idealized shapes and
Paragraph for Questions 67 to 70: The average distance between alteration in bond angles in molecules.
the nuclei of the two atoms held by a bond is called bond length.
This represents the internuclear distance corresponding to the 71. Molecular shape of XeF3+ , SF3+ , and CF3+ are
minimum potential energy for the system. The main factors which (a) the same with 2, 1, and 0 lone pairs of electrons,
affect the bond length are as follows: respectively.
(i) Double/triple bonds are shorter than corresponding sin- (b) different with 2, 1, and 0 lone pairs of electrons,
gle bonds. respectively.
(ii) Sometimes, bond distances of single bonds are some- (c) different with 0, 1, and 2 lone pairs of electrons,
what larger than those of double bonds of their respective respectively.
covalent radii (e.g., F2). It is due to strong repulsive interac- (d) the same with 2, 0, and 1 lone pairs of electrons,
tion between the lone-pair electrons on adjacent atoms. respectively.
(iii) Sometimes single bond distances are somewhat shorter 72. Which of the following statement is correct with respect to
than double of their respective covalent radii because bond angle?
bonds acquire some partial double bond character. This (a) The F S F angle in SF2 is more than 109.5°.
occurs when one atom has vacant orbital and another (b) The H N N angle in N2H2 is approximately 180°.
atom has a lone pair. The bond length may also become (c) The F Kr F angle in KrF4 is 90°.
shorter due to high ionic character in the covalent bond. (d) The Cl N O angle in NOCl is more than 120°.
67. Which is not true about the N N bond length among the fol- 73. Which of the following statements is incorrect?
lowing species? (a) In ClF3, the axial Cl F bond length is larger than equato-
(I) H2N NH2 (II) N2 (III) HN+ NH2 (IV) N2O rial Cl F bond length.
(a) N N bond length is the shortest in II (b) In SF4, F S F equatorial bond angle is not 120° and 104°
(b) N N bond length in I is shorter than that of in III due to lone pair–bond pair repulsions.
(c) N N bond length in III is shorter than that of in I (c) In ICI4− , bond angles is 90°.
(d) N N bond length IV is intermediate between I and III (d) In OBr2, the bond angle is less than OCl2.

Chapter-4.indd 115 8/4/2016 10:19:04 AM


116 Chapter 4 Chemical Bonding

Matrix-Match Type 77. Match the molecules/species with the properties.


74. Match the species with their properties. Column I Column II
Column I Column I (a) BrF5 (p) It has at least one angle less
than 90°.
(a) O2+ (p) Isoelectronic with N2
(b) ICl3 (q) The central atom is sp3d
(b) CO (q) Fractional bond order hybridized.
(c) NO (r) Paramagnetic (c) H3O+ (r) It is non-planar.
(d) He2+ (s) Diamagnetic (s) The central atom is having only
one lone pair.
75. Match the molecules with hybridization of central atom.
Integer Type
Column I Column II
78. The number of 90° bond angles present in the molecule of
(a) IO2F2− (p) sp3d
SF4 is___.
(b) F2SeO (q) sp3
79. The number of ions among the following having pseudo-
(c) ClOF3 (r) sp2 inert gas configuration is ___.
(d) XeF5+ (s) sp3d2
Zn2+, Cd2+, Hg2+, Cu+, Ag+, Au+

80. The sum of oxidation states of N atom in the following com-


76. Match the pair of compounds with their identical property. pounds of nitrogen is ___.
Column I Column II
NH3, N2H4, NH2OH, N2, N2O, NO, N2O3, NO2, N2O5
(a) PCl3F2, PCl2F3 (p) Hybridization of central atom
(b) BF3, BCl3 (q) Shape of molecule/ion 81. The number of lone pair(s) present in the structure of HNO3
is___.
(c) CO2, CN2−
2 (r) µ (dipole moment)
(d) C6H6, B3N3H6 (s) Total number of electrons 82. In [Ag(CN)2]−, the number of p bonds is ___.

ANSWER KEY
Level I
1. (c) 2. (b) 3. (a) 4. (d) 5. (b) 6. (a)
7. (c) 8. (a) 9. (d) 10. (c) 11. (c) 12. (c)
13. (b) 14. (a) 15. (b) 16. (b) 17. (a) 18. (d)
19. (c) 20. (d) 21. (c) 22. (a) 23. (d) 24. (b)
25. (b) 26. (b) 27. (a) 28. (a) 29. (a) 30. (b)
31. (c) 32. (c) 33. (a) 34. (b) 35. (a) 36. (b)
37. (a) 38. (c) 39. (c) 40. (d) 41. (c) 42. (d)
43. (c) 44. (b) 45. (b) 46. (d) 47. (d) 48. (b)
49. (b)

Level II
50. (a), (b), (c) 51. (c), (d) 52. (a), (c) 53. (a), (b), (d) 54. (a), (d) 55. (a), (b)
56. (b), (c), (d) 57. (a), (b), (d) 58. (a), (d) 59. (a), (c) 60. (b), (d) 61. (c), (d)
62. (b), (c) 63. (a), (c) 64. (a) 65. (d) 66. (b) 67. (b)
68. (d) 69. (c) 70. (c) 71. (b) 72. (c) 73. (d)
74. (a) → q, r; (b) → p, s; (c) → p, s; (d)→ q, r 75. (a) → p; (b) → q; (c) → p; (d) → s
76. (a) → p, q; (b) → p, q, r; (c) → p, q, r, s; (d) → p, q, r, s 77. (a) → p, r, s; (b) → p, q; (c) → r, s 78. (0)
79. (6) 80. (9) 81. (7) 82. (4)

Chapter-4.indd 116 8/4/2016 10:19:05 AM


Hints and Explanations 117

HINTS AND EXPLANATIONS


Level I Configuration in oxygen ion: 2s2 2p5
2s 2p x 2 p y 2pz
Single Correct Choice Type
↑↓ ↑↓ ↑↓ ↑
1. (c) IF4+ hybridization; sp3d, structure is irregular tetrahedral. sp3d2 hybridization with four electrons from four sulphur
2. (b) For µ = q × r, we use the given values
 1.602 × 10 −19 C 
q = 0.167 e–  –20
 = 2.675 × 10 C S
 1 e−  − −
O O O
r = 154.6 pm = 154.6 × 10–12 m
µ = (2.675 × 10–20 C) × (154.6 × 10–12 m) 6. (a) Larger the anion, greater is its polarizability (because the
= 4.136 × 10–30 Cm hold on the electron cloud by the nucleus decreases).
In Debye units, we have: 7. (c) Groups of electrons in the valence shell of an atom repel
 1D  each other and position themselves in the valence shell
q = 4.136 × 10–30 Cm = 1.24 D
 3.34 × 10 −30 Cm  so that they are as far apart as possible in order to mini-
3. (a) From the given data mize the repulsion.
q =1.602 × 10–19 C 8. (a) Favourable conditions for ionic bond formation are:
r = 260 pm = 260 × 10–12 m = 2.6 × 10–10 m (i) Low ionization energy (IE) values of atoms-forming
Magnitude of dipole moment for 100% ionic character cation, that is, down the group, as size of the cation
|μ| = qr = (1.602 × 10–19) (2.6 × 10–10) = 4.165 × 10–29 Cm increases, IE decreases.
(ii) High electron affinity (EA) values of atoms-forming
4. (d) F
anions, that is, down the group EA values decreases
and size of the anion increases.
CI F 9. (d) The molecular geometry of SO2 molecule is bent. The
angle between S O S is 120°. SO2 is a polar molecule
F due to asymmetrical charge distribution.

The shape of the molecule is best described by T-shaped. 10. (c) Fajans’ rule: Greater the polarization of anion in a mole-
cule, more covalent the molecule will be.
5. (b) The outer electronic configuration of sulphur is 3s23p4 in
11. (c) The given values can be represented in the chemical
the ground state and 3s2 3 p1x 3 p1y 3 p1z 3d1xy in the excited reaction as:
state. One 3s and three 3p orbitals of the sulphur atom −795 kJ mol−1
hybridize to give four sp3 hybrid orbitals. Sulphur forms Ca(s) + Cl2(g) CaCl2(s)
three sigma covalent bonds by overlapping three of the 121 kJ mol−1 242.8 kJ mol−1 ∆H
four sp3 hybrid orbitals with three oxygen atoms and
−355 × 2 kJ mol−1
also one p bond by lateral overlapping of 3d orbitals of Ca(g) 2Cl(g) 2Cl−(g) + Ca2+
sulphur with the half-filled p orbitals of one of the oxy-
gen atoms as shown below. 2422 kJ mol−1

Sulphur (ground state): 3s2 3p4 Applying Hess’s law of constant heat summation, we get
3s 3 px 3 py 3 pz 3d −795 = 121+ 242.8 + 2422 + ( −355 × 2) + ∆H
↑↓ ↑↓ ↑ ↑ Therefore, ∆H = −2870.8 kJ mol−1.
Sulphur (excited state): 3s2 3p3 3d1 12. (c) The shape of BrF5 molecule is square pyramidal, where
3s 3 px 3 py 3 pz
some of the F—Br—F bond angles are close to 90 degrees.
3d

↑↓ ↑ ↑ ↑ ↑ F

Since the central S atom forms three sigma bonds with F F


oxygen atoms, the SO2− 3 ion should have a planar trigo-
Br
nal geometry. But the geometry of SO2− F F
3 ion is actually
pyramidal. This is due to the fact that one of the sp3
hybrid orbitals is occupied by a lone pair of electrons, as
is evident from figure. 13. (b) There is a very strong electrostatic force of attraction
between oppositely charged ions. As a result, a large
Configuration in oxygen atom: 2s2 2p4
amount of energy is required to overcome strong attrac-
2s 2 px 2 py 2 pz
tive interionic forces. Hence, ionic compounds have high
↑↓ ↑↓ ↑ ↑ boiling and melting points.

Chapter-4.indd 117 8/4/2016 10:19:10 AM


118 Chapter 4 Chemical Bonding

14. (a) The best arrangement to accommodate three bonding 22. (a) Lattice enthalpy of Al2O3 is calculated from Born–Haber
domains in the valence shell of a covalently bonded cycle using enthalpy of formation of Al(g), first, second
atom is planar triangular. and third ionization energies of Al(g), enthalpy of forma-
tion of O(g), first and second electron gain enthalpies of
15. (b) Bond length is inversely proportional to bond order.
O(g) and enthalpy of formation of Al2O3(s).
Greater the delocalization of p-electrons (−ve charge)
shorter is the bond length. 23. (d) For lattice enthalpy to be high, the force of electrostatic
attraction between the constituent ions should be high;
16. (b) s and p orbitals hybridize to form two sp hybrid orbitals
thus, the cation and anion should be small in size and
oriented at an angle of 180°.
should have high electric charge. So, it depends on
17. (a) Given that the dipole is 0.38 D = 0.38 × 3.34 × 10−30 Cm charge on the ion and size of the ion.
= 1.26 × 10−30 Cm.
If the bond was 100% ionic, the magnitude of the dipole 24. (b) I 3− should have a linear shape. Although there are five
moment will be electron pairs around the central iodine atom, only two
are being used for bonding.
m = qr = 1.6 × 10 −19 × 1.61× 10 −10 Cm = 2.57 × 10 −29 Cm
25. (b) CaI2 is least ionic; this is according to Fajans’ rule. Larger
Therefore, the fraction of charge covalently bonded is the anion, greater is its polarizability (because the hold
1.26 × 10 −30 on the electron cloud by the nucleus decreases).
= 0.05
2.57 × 10 −29 26. (b) CCl4 does not have a dipole moment because, the dipole
Therefore, % fractional charge = 5% moment from the centre (C) to Cl cancel out each other.

18. (d) From the figure shown below, it can be concluded that Cl
BrF5 have 5 s -bonds and one lone pair of electrons in
the valence shell of the central atom. Cl C Cl

F
Cl
F F 27. (a) Ice cube contains water molecules and these are present
Br at a particular lattice. When the two cubes are pressed
F F
together, they join to form one cube due to the forma-
tion of hydrogen bonding between the water molecules
19. (c) of two cubes, which results in forming one cube.
( 5 + 3)
In PH3: H = = 4 = sp3
2 28. (a) O C O (linear) Cl Hg Cl (linear)
( 5 + 3)
In NH3: H = = 4 = sp3 Sn H H N
2 C C −
( 4 + 3 − 1) Cl Cl (bent) H H (trigonal) O O (bent)
In CH3+ : H = = sp2
2 29. (a) In CH4: Net dipole moment is zero.
In SbH3: H = (5 + 3) = 4 = sp3 In NF3: Resultant dipole moment toward nucleus of
2
nitrogen.
1
20. (d) SO 4 : H = 4 +   (6 – 8 + 2) = 4 = sp3
2−  In NH3: Resultant dipole moment toward lone pair.
 2
O− In H2O: Resultant dipole moment toward oxygen.

S
30. (b) O O
O O− O
N O N
Tetrahedral
O O
21. (c) The structure of ammonia is
31. (c) Ionization potential of sodium is less, electron gain
enthalpy of Br is more. So, the type of bond formed is
ionic bond.
32. (c) In CuSO4⋅5H2O, electrovalent, covalent, and coordinate
N bonds are present.
H 33. (a) Cs is the highest electropositive element and F is the
H 106°45′
highest electronegative element. As a result, ionic bond
is formed between them.

34. (b) SF6 have an octahedral shape because it has six electron
H pairs around the central atom. The smallest F—S—F
bond angle in SF6 is 109.5 degrees.

Chapter-4.indd 118 8/4/2016 10:19:14 AM


Hints and Explanations 119

35. (a) As the electronegativity of the atom increases, the 43. (c) Due to overlapping of p–p orbitals, bond exists between
strength of the ionic bond increases because electron- two fluorine atoms to form fluorine molecule.
egative atom attracts the electron pair toward itself. If
44. (b) CH3− is sp3 hybridized with three bond pairs and one lone
electronegativity is more, strength of ionic bond is also
pair. Thus, the shape is trigonal pyramidal like NH3.
more.
45. (b) In a homonuclear molecule,
36. (b) O O1+ O p 2 p x = p 2 p y and p *2 p x = p *2 p y are two sets of
For charge on central O = 6 − (1/2) (6) − 2 = 6 − 3 − 2 = +1 degenerate orbitals.
m
37. (a) We use the expression q = to solve for q. The debye 46. (d) Larger the size of cation and smaller the size of anion
r
unit, D, equals 3.34 × 10−30 C m, so the dipole moment of favour the electrovalency or formation of ionic bond.
HF is 47. (d) ClF3, TeCl4, and SF4 all molecules have unsymmetri-
m = 1.83 × (3.34 × 10 −30 Cm)= 6.11× 10 −30 Cm cal structure and polar bonds; hence, all of them show
dipole moment.
The SI prefix p (pico) means × 10−12, so the bond length
r = 91.7 × 10−12 m. Substituting in the above equation 48. (b) Tetrahedral shape is formed with the molecule having
gives four electron pairs in the outermost shell of the central
atom. H = 4 ⇒ sp3 hybridization.
6.11× 10 −30 Cm
q= = 6.66 × 10 −20 C
91.17 × 10 −12 m 49. (b) Both SnCl2 and NH2+ has non-linear or bent or V-shape.
The amount of charge on an electron (i.e., an electronic
charge unit) equals 1.602 × 10−19 C, which we can express
as 1 e = 1.602 × 10−19 C. Sn N+
The value of q in electronic charge units is, therefore Cl Cl H H
 1e 
q = 6.66 × 10 −20 C  = 0.416 e Level II
−19 
 1.602 × 10 C 
Multiple Correct Choice Type
As in HF, the hydrogen carries the positive charge, so the
charge on the hydrogen end of the molecule is +0.416e 50. (a), (b), (c) Conceptual
and the charge on the fluorine end is −0.416e.
51. (c), (d) Non-bonding combinations of atomic orbitals.
38. (c) N2+ has a total 13 electrons.
52. (a), (c) CO2 is isostructural with HgCl2 and C2H2.
MO configuration N2+ is: s 1s2 s * 1s2 s 2s2 s * 2s2 p 2p2y p 2p2z s All
2p1xthese are linear in structure and possess sp hybridization.
2 2 2 2 2 2 1
s 1s s * 1s s 2s s * 2s p 2p y p 2p z s 2p x 53. (a), (b), (d) Conceptual
Bond order = (1/2) (9 – 4) = 5/2 = 2.5
54. (a), (d) XeOF2 has trigonal bipyramidal geometry (more spe-
39. (c) N2H4: H = (5 + 3 + 0 + 0 ) = 4 = sp3 cifically T-shaped since two positions in the plane are occu-
2 pied by lone pairs) whereas NH2− is angular bent molecule.
H H
N N 55. (a), (b) MO configuration of F22− :
H H
40. (d) Zn2+: 28 electrons: 1s2, 2s2, 2p6, 3s2, 3p6, 3d10. s 1s2 s * 1s2 s 2 s2 s * 2 s2 s 2 p2x p 2 2 p y p 2 2 pz p * 2 p2y p * 2 p2z s * 2 p2x
Cu+: 28 electrons: 1s2, 2s2, 2p6, 3s2, 3p6, 3d10. Bond order = (1/2) (10 – 10) = 0
Ag+: 46 electrons: [Kr] 4d10 Ar2 also has zero bond order because it is a noble gas; it exists
In these all the d-orbitals are completely filled, hence all only monoatomic in nature.
of these are pseudo inert gas configuration.
56. (b), (c), (d) SnCl2 is bent or V-shaped; NCO−, NO2+ and CS2 are
41. (c) Conceptual linear (sp hybridization).
42. (d) The structures are as follows: 57. (a), (b), (d) Conceptual
F F
F F 58. (a), (d) BF3 has a regular trigonal planar structure and BF4−
F
has a regular tetrahedral structure. Both NF3 and PF3 have
Xe Xe F Xe F pyramidal structure.

F F F 59. (a), (c)


F F IF5: H = (7 + 5 – 0 + 0)/2 = 6 = sp3d2
XeF4: H = (8 + 4 – 0 + 0)/2 = 6 = sp3d2
Xenon Xenon Xenon
XeOF4: H = (8 + 4 – 0 + 0)/2 = 6 = sp3d2
difluoride tetrafluoride hexafluoride
XeF2 XeF4 XeF6 ICl4− : H = (7 + 4 – 0 + 1)/2 = 6 = sp3d2

Chapter-4.indd 119 8/4/2016 10:19:20 AM


120 Chapter 4 Chemical Bonding

But XeF4 is a square structure that is symmetrical in nature. 72. (c) The structure of KrF4 is
IF5 and XeOF4 both are sp3 d 2 hybridized but structurally
unsymmetrical, therefore, these two will be polar. F
90°
60. (b), (d) If the electronegativity difference between the two F Kr F
atoms is more than 1.9, the bond is taken as ionic. Here, (b)
and (d) have an electronegativity difference greater than 1.9.
F
61. (c), (d) N2+ : s 1s2 s * 1s2 s 2s2 s * 2s2 p 2p2y p 2p2z s 2p1x 73. (d) In OBr2, the bond angle is more than OCl2 because Br is
Bond order = (1/2)(9 – 4) = 5/2 = 2.5 less electronegative element than chlorine.
O2+ : s 1s2 s * 1s2 s 2s2 s * 2s2 p 2p2y p 2p2z s 2p2x p * 2p1y
Matrix-Match Type
Bond order = (1/2)(10 – 5) = 5/2 = 2.5
74. (a) ã q, r; (b) ã p, s; (c) ã p, s; (d) ã q, r
62. (b) (c) s 1s < s * 1s < s 2 s < s * 2 s < s 2 p x < p 2 p y
O2−: Bond order is 1.5, so paramagnetic.
= p 2 pz < p * 2 p y = p * 2 pz < s * 2 p x
CO: Bond order is 3, so diamagnetic.
Energy order of molecular orbitals in oxygen molecule. NO+: Bond order is 3, so diamagnetic.
63. (a), (c) CN–: Total electrons = 6 + 7 + 1 = 14 He2+ : Bond order is 0.5, so paramagnetic.
MO configuration of CN–: s 1s2 s * 1s2 s 2s2 s * 2s2 p 2p2y p 2p2z s 75.
2p2x (a) ã p; (b) ã q; (c) ã p; (d) ã s
2 2 2
s * 1s s 2s s * 2s p 2p2y 2 2
p 2p z s 2p x (a) IO2F2− : H = (7 + 2 – 0 +1)/2 = 5 = sp3d
Bond order = (1/2)(10 – 4) = 3 (b) F2SeO: H = (6 + 2 – 0 + 0)/2 = 4 = sp3
NO+: Total electrons = 7 + 8 − 1 = 14
(c) ClOF3: H = (7 + 3 – 0 + 0)/2 = 5 = sp3d
MO configuration of NO+ : s 1s2 s * 1s2 s 2s2 s * 2s2s 2p2x p 2p2x p 2p2y
(d) XeF5+ : H = (8 + 5 – 1 + 0)/2 = 6 = sp3d2
s * 1s2 s 2s2 s * 2s2s 2p2x p 2p2x p 2p2y
Bond order = (1/2)(10 − 4) = 3 76. (a) ã p, q; (b) ã p, q, r; (c) ã p, q, r, s; (d) ã p, q, r, s
Both PCl3F2 and PCl2F3 have sp3d hybridization with
Passage Type
trigonal bipyramidal shape.
64. (a) Conceptual Both BF3 and BCl3 have sp2 hybridization with trigonal
planar shape and also the dipole moments are zero in
65. (d) Conceptual
both the cases (dipole moment cancels each other).
66. (b) H2 > He (according to higher surface area) and T2 > D2 > 2−
Both CO2 and CN2 have sp hybridization with linear
H2 (according to higher molecular weight). geometry, zero dipole moment, and also have same
67. (b) Because of positive charge on N, the electrons are closer number of electrons, that is, 22.
to the N, so bond is slightly shorter. C6H6 known as benzene and B3N3H6 known as borazine
68. (d) Conceptual are planar molecules with aromatic character; B and C
69. (c) Conceptual atoms undergo sp2 hybridization, have equal dipole
moments and total number of electrons is 30.
70. (c) The PF3 molecule has lone pair of electrons on F atom
and P has vacant d-orbitals. The small size of F atom leads 77. (a) ã p, r, s; (b) ã p, q; (c) ã r, s
to pp–dp bonding in PF3.
BrF5: H = (7 + 5 – 0 +0)/2 = sp3d2
Partial double bond It contains five bond pairs and one lone pairs and the
pp −dp structure is square pyramidal.
P P
+
F F F F F F

Br
71. (b) XeF3+ : H = (8 + 3 – 1 + 0)/2 = 5 = sp3d
F F
AB5 → AB3L2 → Two lone pairs
SF3+ = (6 + 3 – 1 + 0)/2 = 4 = sp3 F F
F
AB4 → AB3L → One lone pair
It has at least one angle less than 90° and it is non-planar
CF3+ : (4 + 3 – 1 + 0)/2 = 3 = sp2 in structure and central atom Br has only one lone pair
of electron.
AB3 → Zero lone pair

Chapter-4.indd 120 8/4/2016 10:19:26 AM


Hints and Explanations 121

ICl3: H = (7 + 3 – 0 + 0)/2 = 5 = sp3d It is non-planar structure, and central atom oxygen has
It contains three bond pairs and two lone pairs and the one lone pair of electrons.
structure is T-shaped.
Integer Type
Cl
78. (0) SF4 is see-saw shaped. Its bond angles are 186° and 116°.

Cl l 79. (6) All the given ions show pseudo-inert gas configuration.
80. (9) Total = 1 − 3 + (−2) + (−1) + 0 + 1 + 2 + 3 + 4 + 5 = +9
Cl −3 −2 −1 0 +1 +2 +3 +4 +5
N H3 , N2 H4 , NH2OH, N2 , N2 O, N O, N2 O3 , N O2 , N2 O5
In the above structure due to the presence of two lone
pair electrons, it has at least one angle less than 90°. 81. (7) The structure showing lone pairs is
O
H3O+: H = (6 + 3 – 1 + 0)/2 = 4 = sp3
It has three bond pairs and one lone pair. So, the structure N
is pyramidal.
O O H
1 3
82. (4) [N C Ag C N]−
2 4
O+
H H H

Chapter-4.indd 121 8/4/2016 10:19:29 AM


122 Chapter 4 Chemical Bonding

SOLVED JEE 2016 QUESTIONS


JEE Main 2016 (d) He2+ has the same energy as two isolated He atoms.
1. The group of molecules having identical shape is Solution
(a) PCl5, IF5, XeO2F2. (b) BF3, PCl3, XeO3. (a), (c) According to molecular orbital theory, the arrangement
(c) SF4, XeF4, CCl4. (d) ClF3, XeOF2, XeF + . of the electrons in the molecular orbitals is as follows.
3

(Online) For C2−


2 : The total number of electrons is 14. The molecu-
lar orbital configuration is
Solution s 1s2 s *1s2 s 2s2 s *2s2 p 2px2= p 2py2s 2pz2
(d) The central atom in the molecules ClF3, XeOF2, XeF3+ is There is no unpaired electron and thus it is diamagnetic.
sp2d hybridized with two lone pairs, so their geometry For O2+ 2 : The total number of electrons is 14. The molec-
gets distorted to T-shaped molecule.
ular orbital configuration is
F F F s 1s2 s *1s2 s 2s2 s *2s2 p 2px2 = p 2py2s 2pz2
+ Thus the bond order = (10 − 4)/ 2 = 3; for O2, bond order
Cl F O Xe F Xe = 2. As bond order is inversely proportional to bond length,
thus bond length of O2+ 2 is less than the bond length
F F F of O2.
For N2+ , the total number of electrons is 13. The molecular
2. Which intermolecular force is most responsible in allowing orbital configuration is
xenon gas to liquefy?
s 1s2 s *1s2 s 2s2 s *2s2 p 2px2= p 2py2s 2pz1
(a) Instantaneous dipole – induced dipole
Bond order of N2+ = (9 – 4)/2 = 2.5
(b) Ion – dipole −
(c) Ionic For N2 , the total number of electrons is 15. The molecular
(d) Dipole – dipole orbital configuration is
σ1s2 s *1s2 s 2s2 s *2s2 p 2px2 = p 2py2s 2pz2 p ∗2px1
(Online)
Bond order of N2− = (10 – 5)/2 = 2.5
Solution Thus the bond orders for N2+ and N2 are not the same.
(a) Instantaneous dipole–induced dipole or temporary force For He2+ , as some energy is released during the forma-
is most responsible in allowing xenon gas to liquefy. tion of He2+ from two isolated He atoms, thus it has lesser
energy as compared to two isolated He atoms.
JEE Advanced 2016
2. The compound(s) with TWO lone pairs of electrons on the
1. According to molecular orbital theory. central atom is(are)
2−
(a) C2 is expected to be diamagnetic. (a) BrF3 (b) CIF3
(c) XeF4 (d) SF4
(b) O2+
2 is expected to have a longer bond length than O2.
Solution
+ −
(c) N2 and N2 have the same bond order. (b), (c)

Compound Lone pair Bond pair Structure


F
F F
Square py-
BrF5 1 5 Br
ramidal

F F

Cl
ClF3 2 3 T-shape

F F
F
(Continued)

Chapter-4.indd 122 8/4/2016 10:19:33 AM


Solved JEE 2016 Questions 123

(Continued)

F F

XeF4 2 4 Square planar Xe

F F

F F

SF4 1 4 See saw S

F F

Chapter-4.indd 123 8/4/2016 10:19:34 AM


Chapter-4.indd 124 8/4/2016 10:19:34 AM
5 States of Matter

Question Distribution in JEE (Main and Advanced)

3
No. of Questions

JEE (Main)
2
JEE (Adv)

0
2016 2015 2014 2013 2012 2011 2010 2009 2008 2007

Concept Distribution in JEE (Main and Advanced)

Topics Covered
Year
JEE (Main) JEE (Advanced)
2007 Real Gases
2008 Real Gases
2009 Real Gases, Kinetic Theory of Gases
2010 Gas Laws
2011 Real Gases Gas Laws, Kinetic Theory of Gases
2012 Real Gases Real Gases
2013 Kinetic Theory of Gases
2014 Real Gases, Kinetic Theory of Gases, Mixture of Gases
Gas Laws, Mixture of Gases
2015 Kinetic Theory of Gases, Real Gases
Real Gases, Kinetic Theory of Gases, Kinetic Theory of Gases, Properties
2016
Gas Laws of Liquids

Chapter-5.indd 125 8/8/2016 2:37:25 PM


126 Chapter 5 States of Matter

SUMMARY
1. Intermolecular forces are the bonds which hold atoms in molecules together. They are classified as London or dispersion forces,
dipole–dipole, dipole–induced dipole and hydrogen bonds.
2. The gas laws
(a) Boyle’s law states that the volume of a fixed amount of gas is inversely proportional to the pressure at a constant temperature.
1
p∝
V
k1
⇒p= or pV = k1
V
or p1V1 = p2V2
(b) Charles’ law gives temperature–volume relationship at a constant pressure (V ∝ T ). It states that at constant pressure, the volume
of a fixed amount of gas is directly proportional to its absolute temperature (in K).
V
V1 = k2T or = k2
T
At absolute zero temperature, translational motion (motion from point to point) ceases. This absolute zero corresponds to the
zero point on the Kelvin temperature scale or Absolute temperature scale. To obtain a Kelvin temperature, we add 273.15°C
to the Celsius temperature.
TKelvin = 273.15 + TCelsius
(c) Gay–Lussac’s law states that at a constant volume, pressure of a fixed amount of gas is directly proportional to the temperature.
p
p = k3T or k3 =
T
(d) Avogadro’s law explains that volume of all the gases under the same pressure and temperature condition is directly propor-
tional to the amount of substance. At constant T and p
V ∝n
V = k4n

3. Combined gas law states that the ratio of pV/T is constant for a fixed amount of gas.
p1V1 p2V2
=
T1 T2
V1 > V2 > V3
V3

V2

p V1

4. Ideal gas equation can be derived from the above gas laws, that is, pV = nRT, where R is called the universal gas constant. The value
of R depends on the units to express the four variables p, v, n and T. The value of R = 0.082057 L atm mol−1 K−1 if we express volume
in litres, pressure in atmosphere and temperature in Kelvin.
Under the condition of temperature 273.15 K and 1 bar pressure, we get R = 8.314 JK−1mol−1
5. Density and molar mass measurement of a gas: Using the ideal gas equation, pV = nRT, and substituting n = m/M, where m is the
given mass and M is the molar mass, we get
m
pV = RT
M
m
Rearranging, we get pM = RT ⇒ pM = rRT
V
where r is the density of the gas. Hence, knowing density of a substance, molar mass can be calculated as
rRT
M=
p

Chapter-5.indd 126 8/8/2016 2:37:27 PM


Summary 127

6. Dalton’s law of partial pressure states that the total pressure of a non-reacting gaseous mixture in a system is the sum of the par-
tial pressures of each component gas.
ptotal = pA + pB + pC +

where ptotal is the total pressure of the mixture, and pA, pB, pC, etc. are the individual pressures of components A, B, C, etc.
7. Relative humidity is defined as the ratio of the partial pressure of moisture in air to its vapour pressure.
pH2 O in air
%Relative humidity = × 100
Vapor pressure of H2O

8. Graham’s law of effusion and diffusion states that rate of effusion of different gases is inversely proportional to the square roots
of their densities at the same temperature and pressure.
k
Effusion rate(r ) = ( when compared at the same T and P )
r

Effusion rate (A) r MB


= B =
Effusion rate (B) rA MA

where rA and rB as the densities of gases A and B respectively.


9. The kinetic molecular theory states that the temperature of the gas is proportional to the average kinetic energy of its molecules.
1
T ∝ Average molecular kinetic energy ∝ mv 2
2
(a) Relationship between most probable ump(or c*), average (uavg c ) and root mean square velocities (urms or c).

2RT 8RT 3RT


ump: uavg: urms = : : = 1.414 : 1.596 : 1.732 = 1: 1.128 : 1.224
M pM M

At a given temperature, the three molecular speeds can be arranged in the order: ump > uavg > urms .
(b) The average kinetic energy of gas molecules, however, is constant at a given temperature and is related to the absolute tem-
perature by the relation
3RT 3
EK = = kT
2NA 2

where k is the Boltzmann constant = 1.38 × 10−23 J K−1.


10. Maxwell–Boltzmann distribution of molecular speeds states that the fraction of molecules having too low or too high speed is
very small. At a certain speed, the fraction of molecules possessing that speed is the maximum. This is known as the most probable
speed. Maxwell’s speed distribution law can be stated as
3/ 2
 M 
v 2e − Mv
2
/ 2 RT
P(v ) = 4p 
 2p RT 
where P(v) is probability distribution function, M is the molar mass of the gas.
Maxwell distribution curve for molecular speeds in oxygen gas at temperature of 300 K and 1200 K is shown below.

Most probable speed is near 400 ms−1


Percentage of molecules per

300 K
unit speed interval

Most probable speed is near 800 ms−1

1200 K

400 800 1200 1600


Molecular speed, m/s

Chapter-5.indd 127 8/8/2016 2:37:30 PM


128 Chapter 5 States of Matter

11. Deviation from ideal behaviour (real gas): The molecules of an ideal gas have negligible volume and no attraction or repulsion for
each other. The molecules of a ‘real’ gas obviously do have a volume, and there is some interaction between molecules, especially at
high pressures (i.e., the molecules are pressed close together) and low temperatures (i.e., the molecules move more slowly).
(a) According to the ideal gas law, the ratio pV/T equals a product of two constants, nR. But, experimentally, for real gases pV/T is
actually not quite a constant. When we use experimental values of p, V and T for a real gas, such as O2, to plot actual values of
pV/T as a function of p, we get the curve as shown in below figure. The horizontal line at pV/T = 1 in below figure is what we
should see if pV/T were truly constant over all values of p, as it would be for an ideal gas. It is observed that there is a negative
deviation from ideal behaviour first followed by an increase in the pV value.
N2
2.0 CH4
O2CO2
H2
1.5

Ideal
1.0
pV
RT

gas

0.5

0 200 400 600 800 1000


p (atm)
(b) The plots for nitrogen, methane and carbon dioxide are similar to the plot for oxygen. The pV/T value first decreases with
increase in pressure, reaches a minimum value that is characteristic of the gas and then rises.
12. van der Waals equation
The measured The measured
pressure volume

n 2a
pmeas + (Vmeas − n b ) = n RT
V2

Correction to bring Correction to reduce


measured p up to the measured V to the
pressure an ideal gas volume an ideal gas
would exert would have

Constants a and b are called van der Waals constants.


13. Compressibility factor
The ratio of the observed volume of a gas to the calculated volume under given conditions of temperature and pressure is known
as compressibility factor.
V pV V
Z = real = real ⇒ Z = real
nRT /p nRT Videal
14. Boyle’s temperature
The temperature at which gases show ideal gas behaviour for an appreciable range of pressure is known as Boyle’s temperature or point.
(Real gas)

Z
TB
Z = 1 (Ideal gas)
T1
T2

p
The Boyle’s temperature is given by
a
TB =
Rb
where TB is Boyle’s temperature

Chapter-5.indd 128 8/8/2016 2:37:31 PM


Summary 129

15. Virial equation


The equation of state (developed for real gases based on the different pressure, volume and temperature values) in terms of com-
pressibility factor is called the virial equation.
pVm B C
Z= = 1+ + 2 +
RT Vm Vm

This equation is a power series expansion in variable V. The coefficients B, C… are functions of temperature and are called first virial
coefficient, second virial coefficient and so on. These are substance and temperature specific.
16. Liquefaction of gases
(a) Critical temperature (TC) is the temperature above which the substance in gas phase cannot be converted into a liquid, no
matter how much pressure is applied. It is, thus, the highest temperature at which a substance can exist as a liquid.
8a
TC =
27Rb
(b) Critical pressure (pC) is the minimum pressure that must be applied to liquefy a gas at the critical temperature.
a
pC =
27b2
(c) Volume of 1 mol of gas under the condition of critical temperature and critical pressure is known as critical volume (VC).
VC = 3b

If the values of van der waals constants a and b are known, it possible to calculate pC, VC, and TC from above equation.
pCVC 3
The critical compressibility factor, Z = =
RTC 8
17. The law of corresponding states says that the two gases at same reduced temperature and pressure have same reduced volume.
We can calculate a, b and R in terms of pC, VC and TC from above equation as
VC 8a 8p V
a = pC × 3 × 9b2 = 3pCVC2 ; b = ; R= = C C
3 27TCb 3TC

Using the above values, the van der Waals equation becomes
 3 pCVC2   VC   8 pCVC2 
p+ 2  

V −  =  ×T
 V  3  3TC 
p V T
On rearranging the above equation and substituting a = ,b = and g = , we get
pC VC TC

 3 
 a + 2  (3 b − 1) = 8g
 b 
where a, b and g are known as reduced variable of states and above equation is known as reduced equation of state.
18. Joule–Thomson effect: In this effect, when a compressed gas is allowed to expand into a low pressure under adiabatic conditions,
lowering of temperature is observed. The gas molecules do work at the cost of kinetic energy to overcome the intermolecular forces,
which causes cooling to occur. The relation is given by
 ∂T 
m= 
 ∂p  H
The pressure at which gases neither cool nor warm up on expansion, that is, m = 0 is called the inversion point. This temperature is
related to the van der Waals constants by the relation:
2a
Tinv =
Rb
19. Properties of liquid state: Liquids have high density, are essentially incompressible, undergo little thermal expansion, have a fixed
volume but no definite shape. When a liquid evapourates, the molecules that enter the vapour exert a pressure called the vapour
pressure.
20. Surface tension is the force that causes the surface of a liquid to contract. Surface tension is a property that varies with the strength
of intermolecular attractive forces. As the temperature is increased, the kinetic energy of the molecules increases and the strength
of intermolecular forces decreases. Thus, the surface tension decreases with the increase in temperature.

Chapter-5.indd 129 8/8/2016 2:37:33 PM


130 Chapter 5 States of Matter

21. Flowing is a change in the form of the liquid, and such resistance to a change in form is called the liquid’s viscosity.
dv dv
F∝A or F = hA
dx dx
where h is a constant that depends on the nature of the liquid and is known as coefficient of viscosity. The SI unit of viscosity coef-
ficient is Newton second per square metre (N sm−2) or Pascal second (Pas = 1 kg m−1 s−1). In the CGS system, the unit for viscosity
coefficient is poise (P). Viscosity also depends on the temperature as the temperature drops, the viscosity increases.

SOLVED EXAMPLES
Gas Laws (Boyle’s Law, Charles’ Law, Gay–Lussac’s Law, (3) (4)
Avogadro’s Law)
V = constant
1. Which scientist made the following proposal: “Equal volumes
of gases, under the same conditions of temperature and pres- p pV
sure, contain the same number of particles”?
(a) Gay-Lussac (b) Dalton
(c) Berzelius (d) Avogadro
Solution T T

(d) Avogadro’s law states that equal volumes of gases, under (a) (2), (3) (b) (1), (2) (c) (3), (4) (d) (1), (3)
the same conditions of temperature and pressure, contain
Solution
the same number of particles.
(a) The graph (2) explains the Charles’ law which states that
2. A gas has a volume of 3.86 L at a temperature of 45°C. What the volume of a given mass of gas is directly proportional
will the volume of gas be if its temperature is raised to 80°C, to the absolute temperature at a constant pressure.
while its pressure is kept constant? The graph (3) explains the Gay–Lussac–Charles’ law which
(a) 5.26 L (b) 4.28 L (c) 6.25 L (d) 3.27 L states that the pressure of a given mass of gas is directly
Solution proportional to absolute temperature at a constant
volume.
(b) Using Charles’s law to solve the second volume:
5. For a given 1 mol of ideal gas kept at 6.5 atm in a container
V1T2 3.86 L (353 K )
V2 = = = 4.28 L of capacity 2.463 L, the Avogadro proportionality constant for
T1 318 K the hypothesis is
(a) 22.4 (b) 2.46
3. The vapour pressure of water at 80°C is 355 mm Hg. A one-litre (c) 0.406 (d) 3.25
vessel contains O2 at 80°C, saturated with water vapour the
total pressure being 760 mm Hg. The contents of the vessel Solution
were pumped into 0.3 L vessel at the same temperature. What (c) From Avogadro’s hypothesis,
is the partial pressure of O2?
n ∝ V ⇒ n = kV (1)
(a) 1350 mm Hg (b) 2263.3 mm Hg
(c) 123.5 mm Hg (d) 455 mm Hg For 1 mol of an ideal gas, pV = RT
Solution pV 6.5 × 2.463
or T= = = 195 K
(a) Given that pH2O = 355 mmHg at 80°C , pO2 = 405 mmHg. R 0.0821
According to Boyle’s law, p1V1 = p2V2. Substituting values, we get From ideal gas law, pV = nRT, we have
405 × 1
p1 = = 1350 mmHg pV
0.3 n= (2)
RT
4. Which of the following plots are correct? From Eqs. (1) and (2), we get
(1) (2)
p 6.5
p = constant
k= = = 0.406
T = constant RT 0.0821× 195

V
V 6. For a closed (not rigid) container containing n = 10 mol of
an ideal gas fitted with movable, frictionless, weightless pis-
ton operating such that pressure of the gas remains constant
at 0.821 atm. Which graph represents correct variation of
p T log V vs. log T, where V is in litre and T is in kelvin?

Chapter-5.indd 130 8/8/2016 2:37:36 PM


Solved Examples 131

(a) (b) 9. If 10−4 dm3 of water is introduced into a flask at 300 K, how
many moles of water are in the vapour phase when equilib-
rium is established? (Given: Vapour pressure of H2O at 300 K
log V

is 3170 Pa; R = 0.0821 J K−1 mol−1.)

log V
(a) 1.27 × 10−3 mol (b) 5.56 × 10−3 mol
−2
(c) 1.53 × 10 mol (d) 4.46 × 10−2 mol
45° 35° (AIEEE 2010)
log T log T Solution
(c) (d) (a) Using ideal gas equation pV = nRT, we get

pV 3170 × 10 −5 atm × 1 L
= 1.27 × 10 −3 mol
log V

n= =
RT 0.0821 L atm K −1 mol−1 × 300 K
log T
45°

1 10. The initial volume of a gas cylinder is 750.0 mL. If the pres-
sure of gas inside the cylinder changes from 840.0 mm Hg to
log T 45°
360.0 mm Hg, the final volume of the gas will be
log V
(a) 1.750 L (b) 3.60 L
Solution (c) 4.032 L (d) 7.50 L
(c) From Charles law, we have V ∝ T. Taking log on both sides, (JEE Main 2014 Online)
we get
Solution
log V = log T + log constant (c) ⇒ log V = log T + c
Comparing with y = mx + c, we get, m = 1 and c = 1. So, (a) The ideal gas equation is
θ = 45°. pV = nRT
Ideal Gas Equation Since n, R, T = constant, the equation becomes
7. An ideal gas is allowed to expand both reversibly and irre- pV = k
versibly in an isolated system. If Ti is the initial temperature p1V1 = p2V2
and Tf is the final temperature, which of the following state- 750 × 840 = 360 × V2
ments is correct? 750 × 840
(a) (Tf )irrev > (Tf )rev V2 =
360
(b) Tf > Ti for reversible process but Tf = Ti for irreversible
process. V2 = 1750 mL = 1.750 L
(c) (Tf )rev = (Tf )irrev
(d) Tf = Ti for both reversible and irreversible processes. 11. The average oxygen content of arterial blood is approxi-
(AIEEE 2006) mately 0.25 g of O2 per litre. Assuming a body temperature
of 37°C, how many moles of oxygen are transported by each
Solution litre of arterial blood and how many millilitres?
(d) In an ideal gas, there are no intermolecular forces of (a) 7.8 × 10−3 and 200 mL (b) 6.8 × 10−3 and 200 mL
attraction. So, Tf = Ti for both reversible and irreversible −3
(c) 7.8 × 10 and 100 mL (d) 6.8 × 10−3 and 100 mL
processes.
Solution
8. Boron forms a variety of unusual compounds with hydrogen. (a) The number of moles of oxygen per litre of blood is
A chemist isolated 6.3 mg of one of the boron hydrides in a
1 mol O2
glass bulb with a volume of 385 mL at 25°C and a bulb pres- × 0.25 gO2 = 7.8 × 10 −3 mol O2
sure of 11 torr. The molecular mass of the boron hydride in 32 g O2
g mol−1 is Therefore, the volume of oxygen is
(a) 3.68 (b) 27.6
nRT 7.8 × 10 −3 × 0.0821× 310
(c) 36.35 (d) 56.52 V= = = 0.2 L = 200 mL O2
p 1
Solution
(b) Assuming ideal gas law: 12. Assume that you take a flask, evacuate it to remove all the
w wRT air, and find its mass to be 478.1 g. You then fill the flask with
pV = nRT = RT ⇒ M = . Substituting values, we get
M pV argon to a pressure of 2.15 atm and weigh it again. What
would the balance read (in grams) if the flask has a volume of
6.3 × 10 −3 g × 0.082 L atm mol−1 K −1 × 298 K 7.35 L and the temperature is 20.0°C?
M= = 27.6 gmol−1
11/ 760 atm × 385 × 10 −3 L (a) 203.6 g (b) 504.3 g
(c) 471.1 g (d) 523.3 g
Quick Tip/Alternate Solution The first option can be
Solution
neglected easily because boron’s atomic weight is 10.8. So,
molar mass cannot be less than 10.8. (b) Mass of empty flask = 478.1 g, Ar p = 2.15 atm, V = 7.35 L,
T = 293 K. The number of moles of argon is

Chapter-5.indd 131 8/8/2016 2:37:39 PM


132 Chapter 5 States of Matter

pV 2.15 × 7.35 NH3 (g) + O2 (g) → N2 (g) + H2O(g)


n= = = 0.657 mol
RT 0.0821× 293 (a) 75 mL (b) 125 mL
Mass of Argon = Number of moles × Atomic mass of Ar (c) 140.625 mL (d) 241.4 mL
= 0.657 × 40 = 26.27 g Solution
Balance reading = 478.1 g + 26.27 g = 504.37 g
(c) We first require volume of ammonia at the same condi-
Combined Gas Law tion as it is for water vapour. Hence, we use combined
gas law equation
13. Use the combined gas law to determine by what factor the
pressure of an ideal gas must change if the Kelvin tempera- p2V2 p1V1  p T 
= ⇒ V2 =  1   2  V1
ture is doubled and the volume is tripled. T2 T1  p2   T1 
(a) 2/3 (b) 4/3 (c) 1/3 (d) 3/4 950 600
= × × 50 mL = 93.75 mL
Solution 760 400
(a) From combined gas law Now considering the following balanced chemical equa-
p1V1 p2V2 tion, we get
=
T1 T2 4 NH3 (g) + 3 O2 (g) → 2 N2 (g) + 6 H2O(g)
V2 = 3V1 and T2 = 2T1
4 vol of NH3 ≡ 6 vol of H2O
p1V1 p2 3V1 ⇒ 93.75 mL of NH3 will produce
= ⇒ p2 = 2/3 p1
T1 2T1
6
Thus, the pressure must change by 2/3. = × 93.75 mL = 140.625 mL H2O vapour
4
14. A sample of nitrogen has a volume of 880 mL and a pressure
of 740 torr. What pressure will change the volume to 870 mL 17. Butane (C4H10) is the fuel in cigarette lighters. It burns in oxy-
at the same temperature? gen according to the equation
(a) 325 torr (b) 230 torr (c) 750 torr (d) 689 torr 2C 4H10 (g) + 13O2 (g) → 8CO2 (g) + 10H2O(g)
Solution
How many millilitres of O2 at 35°C and 725 torr are needed
(c) Since volume is to decrease, pressure must increase, and to react completely with 75.0 mL of C4H10 measured at 45°C
we multiply the starting pressure by a volume ratio that and 760 torr?
is larger than one. Also, since p1V1 = p2V2, we can solve (a) 345 mL (b) 432 mL
for p2: (c) 495 mL (d) 385 mL
p1V1 (740 torr )(880 mL ) Solution
p2 = = = 750 torr
V2 (870 mL ) (c) Starting with the combined gas law:
15. A bubble of gas released at the bottom of a lake increases p1V1 pV
to four times its original volume when it reaches the surface. = 2 2
T1 T2
Assuming that atmospheric pressure is equivalent to the
pressure exerted by a column of water 10 m high, what is the Rearranging the equation to find a new volume (V2)
depth of the lake? p1V1T2
(a) 15 m (b) 45 m (c) 30 m (d) 60 m V2 =
T1p2
Solution
Now, we calculate the new volume for the butane (C4H10)
(c) At the bottom of the lake, we have V1 = V and p1 = p. using the temperature and pressure of oxygen (O2)
At the surface, we have V2 = 4V and p2 is the pressure
exerted at 10 m water height = 9.8 × 104 Pa where (760 torr )(75.0 mL )(308 K )
V2 = = 76.1 mL C4H10
p = hpg and g = 9.8 ms−2. (318 K )(725 torr )
Therefore, on substituting values, we get
Thus, the number of millilitres of O2 required to react
p2V2 9.8 × 10 4 × 4V
p1 = = = 39.2 × 10 4 Pa with 76.1 mL of C4H10 is
V1 V
Now, ∆p = 39.2 × 104 – 9.8 × 104 = 29.4 × 104 Pa. Thus, the 76.1 mL C4H10  13 volume O2  = 495 mL O2
 2 volume C H 
depth of the lake is 4 10

29.4 × 10 4
h= = 30 m
1000 × 9.8 Mixtures of Gases (Dalton’s Law, Graham’s Law)
16. How many millilitres of H2O vapour measured at 327°C and 18. The ratio of rate of effusion of O2 and H2 from a container
760 torr are formed when 50 mL of ammonia at 950 torr and containing 16 g O2 and 4 g H2 is
127°C reacts with oxygen according to the following reaction? (a) 1:16 (b) 1:4 (c) 1:1 (d) 4:1

Chapter-5.indd 132 8/8/2016 2:37:42 PM


Solved Examples 133

Solution 21. A certain gas diffuses from two different vessels A and B. The
(a) From Graham’s law of effusion, vessel A has a circular orifice, while vessel B has a square ori-
fice of length equal to the radius of the orifice of vessel A.
rO2 pO2 MH2 nO2 MH2 Calculate the ratio of the rates of diffusion of the gas from ves-
= × = ×
rH2 pH2 MO2 nH2 MO2 sel A to vessel B, assuming same temperature and pressure.
(a) 1:p (b) p
Since in a closed container, the pressure ratio is equal to (c) 1:1 (d) 2:1
the ratio of number of moles, we have
Solution
rO2 16 / 32 2 0.5 1 1 (b) r ∝ area of orifice
= = × = r1 ∝ p r2 (circle)
rH2 4 / 2 32 2 4 16
r2 ∝ r2 (square)
r1
19. What volume of wet methane (saturated with water vapour) Therefore, =p
would you have to collect at 29°C and 1 atm pressure to be r2
sure that the sample contains 244 mL of dry methane at 950 22. A small quantity of gaseous NH3 and HBr are introduced
torr. [Given that vapour pressure of water at 29°C = 30 torr.] simultaneously into the opposite ends of an open tube that
(a) 317.5 mL (b) 187.5 mL is 1 m long. Calculate the distance of the while solid NH4Br
(c) 305 mL (d) 244 mL formed from the end that was used to introduce NH3.
(a) 68.55 cm (b) 63.5 cm
Solution
(c) 60.09 cm (d) 65.24 cm
(a) The moles of methane gas required is
Solution
pV 950 × 244 (a) By Graham’s law of diffusion
n= = mol
RT RT
rNH3 MHBr 81
Same number of moles of methane should be present = = = 2.18
rHBr MNH3 17
in wet methane, which is saturated with water vapour.
So, wet methane contains water vapour and the partial Thus, NH3 travels 2.18 times faster than HBr. In other words,
pressure of water vapour in it should be equal to the NH3 will travel 2.18 cm in the same time in which HBr trav-
vapour pressure of water for given conditions. els 1 cm. Given that the length of the tube = 100 cm.
Therefore, partial pressure of methane in wet methane =
760 − 30 = 730 torr. Therefore, distance travelled in the tube by NH3
Thus, 2.18
= × 100 = 68.55 cm
nRT 950 × 244 RT 2.18 + 1
V= = = 317.5 mL
pCH4 RT 730 Thus, NH4Br will first appear at a distance of 68.55 cm
from NH3 end.
20. Equal masses of methane and oxygen are mixed in an empty
container at 25°C. The fraction of the total pressure exerted
Kinetic Theory of Gases (rms Velocity, Most Probable
by oxygen is
Velocity, Average Velocity, Most Probable Kinetic
1 273 Energy)
(a) 1/2 (b) 2/3 (c) × (d) 1/3
3 298
23. According to the kinetic theory of gases, in an ideal gas,
(AIEEE 2007)
between two successive collisions a gas molecule travels
Solution (a) in a circular path.
(d) Let the mass of methane and oxygen be x because they (b) in a wavy path.
have same masses (c) in a straight line path.
(d) with an accelerated velocity.
Number of moles of oxygen
Mole fraction of oxygen = Solution
Total number of moles
(c) This is one of the assumptions of kinetic theory of gases.
where number of moles of oxygen = x/32 (molecular
mass of O2 is 32) 24. As the temperature is raised from 20°C to 40°C, the average
Number of moles of methane = x/14 (molecular mass of kinetic energy of neon atoms changes by a factor of which of
CH4 is 16) the following?
x / 32 1 (a) 1/2 (b) 2
Therefore, mole fraction of oxygen = =
( x / 32) + ( x /16 ) 3 (c) 313/293 (d) 313/ 293
Let the total pressure be p. Then, partial pressure of oxy-
gen is Solution
1 p (c) T1 = 273 + 20 = 293 K and T2 = 273 + 40 = 313 K. As the
pO2 = Mole fraction of oxygen × ptotal = × p = kinetic energy is proportional to the temperature, we have
3 3

Chapter-5.indd 133 8/8/2016 2:37:45 PM


134 Chapter 5 States of Matter

E1 T1 293 E2 313 (a) c * : c : c = 1.128 : 1.225 : 1 (b) c * : c : c = 1: 1.128 : 1.225


= = ⇒ =
E2 T2 313 E1 293
(c) c * : c : c = 1: 1.125 : 1.128 (d) c * : c : c = 1.225 : 1.128 : 1
25. Select the incorrect statement: (JEE Main 2013)
(a) The most probable speed increases with increase in Solution
temperature.
(b) The fraction of gaseous molecules moving with most 2RT 8RT 3RT
(b) We know that c* = , c= ,c=
probable speed decreases with increase in temperature. M pM M
(c) Fraction of gaseous molecules moving with average 8
speed is less than the fraction of molecules moving with Thus, we have c * : c : c = 2: : 3
p
root mean square speed.
Solving, we get c * : c : c = 1.414:1.595:1.732 = 1:1.128:1.225
(d) In a mixture of gases, heavy molecules are unlikely to be
found with very high speeds. 28. Which of the following statements is incorrect about H2 and
CO2 gas considering them as ideal gases?
Solution
(a) The average kinetic energies of H2 and CO2 molecules
2RT are the same at a given temperature.
(c) Since vmp = , on increasing temperature vmp will
M (b) The root mean square velocities of H2 and CO2 mole-
increase. cules are the same at a given temperature.
On increasing temperature, the distribution graph tends (c) The fraction of H2 and CO2 molecules with the most prob-
to flatten, which indicates that fraction of molecules with able velocity decreases with increase in temperature.
most probable velocity decreases. (d) The density of H2 is less than CO2 at a given temperature
and pressure.
Most probable Solution
Average (b) The average kinetic energy per mole is equal to kinetic
energy per molecule multiplied by Avogadro number,
Root mean square
that is
3 3
Fraction

Eavg = NA × kT = RT
2 2
Therefore,
300 K Eavg ∝ T
100 K Since, for any ideal gas, average kinetic energy depends
only on the temperature, so, at a given temperature it is
equal for every ideal gas.
Speed The root mean square velocity depends on temperature
as well as on molar mass, so, for two different ideal gases,
From Maxwell distribution, it is clear that average speed it is different at a given temperature.
fraction is greater than root mean square speed.
3RT T
Since speed is inversely proportional to the root of molar vrms = or vrms ∝
mass, heavier molecules move relatively with less speed. M M
Maxwell’s distribution curve of velocity graph flattens on
26. Which one of the following statements is NOT true about the increasing temperature, so, the most probable velocity
effect of an increase in temperature on the distribution of fraction is decreased.
molecular speeds in a gas? Since density is directly proportional to the molar mass,
(a) The most probable speed increases. and molar mass of hydrogen is lower than carbon diox-
(b) The fraction of the molecules with the most probable ide, so, density of hydrogen is less than carbon dioxide.
speed increases.
(c) The distribution becomes broader. 29. Which of the following is not an assumption of the kinetic
(d) The area under the distribution curve remains the same theory of gases?
as under the lower temperature. (a) A gas consists of many identical particles which are in
continual motion.
Solution (b) Gas particles have negligible volume.
(b) According to Maxwell’s distribution, the most probable (c) At high pressure, gas particles are difficult to compress.
velocity increases as the fraction of molecules possess- (d) Collisions of gas particles are perfectly elastic.
ing most probable velocity decreases. (JEE Main 2015 Online)

27. For gaseous state, if the most probable speed is denoted by Solution
c*, average speed by c and mean square speed by c, then for (c) According the kinetic theory of gases, gases is highly
a large number of molecules the ratio of these speeds are compressible in nature thus option (c) is not correct.

Chapter-5.indd 134 8/8/2016 2:37:49 PM


Solved Examples 135

Real Gases (van der Waals Equation, Compressibility So if the gas is ideal, then its pressure is 1 atm. However,
Factor) it is a real gas, so at different conditions its pressure may
vary from the ideal pressure.
30. In van der Waals equation of state of the gas law, the con- If its compressibility factor Z > 1, this indicates that repul-
stant b is a measure of sive forces dominate inside the gas. These repulsive
(a) intermolecular repulsions forces imply that the pressure should be more than ideal
(b) intermolecular collisions per unit volume. pressure, that is, more than 1 atm pressure.
(c) volume occupied by the molecules. pV pb
(d) intermolecular attractions If b dominates, then p(Vm − b ) = RT ⇒ Z = m = 1+
RT RT
Solution This equation implies that Z will be more than one
(c) Volume occupied by the molecules always, and pressure should be more than 1 atm.
The van der Waals equation for 1 mol of gas is: If ‘a’ dominates, then
 a pVm a
 a
 p + 2  Vm = RT ⇒ Z = = 1−
 p + 2  (V − b ) = RT  Vm  RT RTVm
V
where b is the measure of effective size of the gas mole- This equation implies that Z < 1, so the pressure should be
cules. It is also called co-volume or excluded volume. less than 1 atm as the attraction forces will dominate here.

31. Chlorine is more easily liquefied than ethane because (a and 34. If Z is a compressibility factor, van der Waals equation at low
b are van der Waals constants for gases). pressure can be written as
(a) a and b for Cl2 < a and b for C2H6 (a) Z = 1 + RT/pb (b) Z = 1 − a/VRT
(b) a for Cl2 < a for C2H6 but b for Cl2 > a for C2H6 (c) Z = 1 − pb/RT (d) Z = 1 + pb/RT
(c) a for Cl2 > a for C2H6 but b for Cl2 < a for C2H6 (JEE Main 2014)
(d) a and b for Cl2 > a and b for C2H6 Solution
(AIEEE 2011) pV
(b) We know that compressibility factor is given by Z =
Solution For 1 mol of gas, we have RT
(c) Ease of liquefaction ∝ a/b  a
For easily liquefiable gas, more should be the value of a  p + 2  (V − b ) = RT
V
and less should be the value of b.
At low pressure, volume is very high, so V  b and thus
32. The compressibility factor for a real gas at high pressure is V − b ≈ V.
(a) 1 + RT/pb (b) 1
 a a pV a a
(c) 1 + pb/RT (d) 1 − pb/RT  p + 2  (V ) = RT ⇒ pV = RT − ⇒ = 1− ⇒ Z = 1−
(AIEEE 2012) V V RT VRT VRT
Solution
35. van der Waals equation for a gas is stated as
(c) According to van der Waals equation, we have 2
nRT  n
 n2a  p= − a 
+ V − nb V
 p  (V − nb ) = nRT
 V2  nRT
This equation reduces to the perfect gas equation, p =
At high pressure, a/V2 is very large so it is neglected in when, V
comparison to p. Now, (a) temperature is sufficiently high and pressure is low.
p(V − b) = RT for 1 mol of a gas, that is, n = 1. (b) temperature is sufficiently low and pressure is high.
pV = RT + pb (c) both temperature and pressure are very high.
(d) both temperature and pressure are very low.
pV pb (JEE Main 2014 Online)
or = 1+
RT RT Solution
pb (a) If temperature is sufficiently high and pressure is low,
Z= +1
RT Vm will be sufficiently large and thus term a/Vm2 will be
where Z is known as the compressibility factor. negligibly small. At this stage b also becomes negligible
compared to Vm, thus
33. Select the correct statement about compressibility factor of
1 mol gas which is kept inside a 22.4 L vessel at 273 K. pV = nRT
(a) For Z > 1 the pressure of gas will be less than 1 atm. nRT
(b) For Z > 1 the pressure of gas will be more than 1 atm. p=
V
(c) If ‘b’ dominates, pressure will be less than 1 atm.
36. A real gas obeys the equation of state p(V − nb) = nRT where
(d) If ‘a’ dominates, pressure will be greater than 1 atm.
b is van der Waals constant and R is the gas constant. If the
Solution pressure and temperature are such that the molar volume of
nRT 1× 0.082 × 273 the gas is 10b, what is the value of compressibility factor?
(b) Using ideal gas law: p = = = 1 atm
V 22.4 (a) 10/9 (b) 8/9 (c) 12/11 (d) 10/11

Chapter-5.indd 135 8/8/2016 2:37:52 PM


136 Chapter 5 States of Matter

Solution (III) At room temperature, attractive forces never dominate


(a) We know that for helium and hydrogen gas.
(IV) The value of van der Waals constant b for SF6 is more
p(V − nb ) = nRT ⇒ p(Vm − b ) = RT (where Vm = V/n)
than that for NH3.
The compressibility factor is given by (a) I, II only (b) II, III and IV only
pV pb (c) I, II, IV only (d) All are correct
Z = m = 1+
RT RT Solution
Suppose pressure and temperature are p′ and T ′ when (d) The graph of Z vs. p for He lies close to Z = 1 line for
its molar volume is 10b, then greater range of pressure, so, it shows ideal behaviour
p′ 1 1 for greater range of pressure.
p′(Vm − b ) = RT ′ ⇒ = =
RT ′ 10b − b 9b At room temperature, compressibility factor for hydro-
p ′b 1 10 gen and helium is always greater than 1, so, these gases
or Z = 1+ = 1+ = never show negative deviation. Since their critical points
RT ′ 9 9
lie at very low temperature, so, attractive forces never
37. Two moles of a gas confined in a 4 L flask exert a pressure of dominate at room temperature.
11.0 atm at 300 K temperature. The value of b is 0.05 L mol, As temperature is increased real gases tend to behave
the value of a is ideally but at Boyle’s temperature, gases show ideal
(a) 6.00 L atm mol−2 (b) 6.52 L atm mol−2 behaviour for maximum range of pressure.
(c) 6.46 L atm mol −2 (d) 6.46 L atm−1 mol−1 van der Waals constant a is the measurement of attrac-
tive forces, in general, its value is more for polar mole-
Solution
cule but for SF6 dispersion forces are very large, so, value
(b) Using van der Waals equation, we have of a as well as b of SF6 are greater than that of NH3.
 an2  39. When does a gas deviate the most from its ideal behaviour?
 p + 2  (V − nb ) = nRT
 V  (a) At low pressure and low temperature
(b) At low pressure and high temperature
an2 nRT
= −p (c) At high pressure and low temperature
V 2 V − nb (d) At high pressure and high temperature
a × 4 2 × 0.0821× 300 a×4 (JEE Main 2015 Online)
= − 11⇒ = 1.631
16 4 − 2 × 0.05 16 Solution
or
1.631× 16 (c) At high pressure and low temperature, gas will deviate most
a= = 6.523 L2 atm mol−2
4 from ideal behaviour. At high pressure, the space taken by
38. Which of the following statement(s) is/are correct? the molecules themselves is a significant part of the total
(I) Helium gas shows ideal behaviour for bigger range of volume, so doubling the pressure cannot halve the total vol-
pressure in comparison of H2 gas. ume. As a result, the actual volume of a real gas is larger than
(II) At Boyle temperature real gases behave ideally for maxi- expected for an ideal gas, and the ratio pV/T is larger than if
mum range of pressure. the gas were ideal.

ADVANCED LEVEL PROBLEMS


1. Negative deviation from ideal behaviour is due to the fact that (a) 4.0 (b) 2.0 (c) 1.0 (d) 0.5
(a) there exist molecular interactions and pV/nRT > 1.
(b) there exist molecular interactions and pV/nRT < 1. Solution
(c) atoms have finite size and pV/nRT > 1. (d) The ratio is
(d) atoms have finite size and pV/nRT < 1. 1/ 2
Rate of diffusion of methane  4  1
Solution =  = = 0.5
Rate of diffusion of helium  16
6 2
(b) For positive deviation, pV = nRT + npb. Thus, the factor
pb/RT is responsible for increasing the pV/RT value above
ideal value. The quantity b is actually the effective volume 3. For a monoatomic gas kinetic energy = E. The relation with rms
of molecule. So, it is the finite size of molecules that leads velocity is
to the origin of b and hence positive deviation at high 1/ 2 1/ 2
 2E   3E 
(a) urms =   (b) urms = 
pressure. For negative deviation, pV/nRT = Z = 1 − a/VRT,  M  2M 
where a accounts for the molecular interactions.
2. If methane and helium are allowed to diffuse out of a con- 1/ 2 1/ 2
 E   E 
(c) urms =  (d) urms = 
tainer under identical conditions of temperature and pressure,  2M   3M 
the ratio of rate of diffusion of methane to helium is

Chapter-5.indd 136 8/8/2016 2:37:55 PM


Advanced Level Problems 137

Solution
1/ 2 (c) CO ( p = 1 atm, T = 273 K ) (r) pV = nRT
2
 3RT 
(a) Since urms = 
 M 
(d) Real gas with very large (s) p(V − nb ) = nRT
1/ 2
molar volume
 3kT  (IIT-JEE 2007)
or urms =  (1)
 M  Solution

The kinetic energy = 3/2 kT or 2E = 3kT. Substituting in (a) → p, s; (b) → r; (c) → p, q; (d) → p, s
Eq. (1), we get In option (a): Z = 1 for ideal gas. For real gases, Z is not
equal to 1. At low temperature and high
 2E 
1/ 2 pressure, van der Waals equation (p + an2/
urms =   V2) (V − nb) = nRT reduces to p(V − nb) = nRT.
 M
In option (b): For H2 gas, the value of Z = 1 at p = 0 and it
4. The given graph in the figure below represents the var- increases continuously on increasing p.
iation of Z (compressibility factor = pV/nRT) vs. p, for 1.6 H2
three real gases A, B and C. Identify the only incorrect 1.4
statement. 1.2

nRT
Ideal gas

pV
A 1.0
C 0.8

Z=
Ideal gas
A 0.6
B
0.4
1 C
0 200 400 600 800
Z B p/bar
In option (c): CO2 molecules have larger attractive forces,
0 under normal conditions.
In option (d): As Z = pVm/RT, at very large molar volume Z is
p (atm) not equal to 1.

(a) For the gas A, a = 0 and its dependence on p is linear at all 6. A gas described by van der Waals equation
pressures. (a) behaves similar to an ideal gas in the limit of large molar
(b) For the gas B, b = 0 and its dependence on p is linear at all volumes.
pressures. (b) behaves similar to an ideal gas in the limit of large pressures.
(c) For the gas C, which is typical real gas for which nei- (c) is characterized by van der Waals coefficients that are
ther a nor b = 0. By knowing the minima and the dependent on the identity of the gas but are independent
point of intersection, with Z = 1, a and b can be of the temperature.
calculated. (d) has the pressure that is lower than the pressure exerted
(d) At high pressure, the slope is positive for all real gases. by the same gas behaving ideally.
(IIT-JEE 2006) (IIT-JEE 2008)
Solution
Solution (a), (c), (d) The van der Waals equation is
(b) b relates to the incompressible volume of the molecules  n2a 
and measures the effective size of gas molecules.  p + 2  (V − nb ) = RT
 V 
Z = pV / RT = 1 for an ideal gas.
At low pressure, when the sample occupies a large vol-
5. Match gases under specified conditions listed in Column I ume, the intermolecular forces do not play any significant
with their properties/laws in Column II. role and the gas behaves like an ideal.
Constants a and b are called van der Waals constants and
Column I Column II their value depends on the characteristic of a gas. Value of
a is a measure of magnitude of intermolecular attractive
(a) Hydrogen gas (p) Compression factor ≠ 1 forces within the gas and is independent of temperature
( p = 200 atm, T = 273 K ) and pressure.
The observed p correction term is added as
(b) Hydrogen gas (q) Attractive forces are
( p ~ 0 , T = 273 K ) dominant an2
pideal = preal +
V2

Chapter-5.indd 137 8/8/2016 2:37:58 PM


138 Chapter 5 States of Matter

Pressure exerted by real gas is lower than the pressure (c) only a small number of molecules have very high velocity.
exerted by the ideal gas. (d) between collisions, the molecules move in straight lines
with constant velocities.
7. The term that corrects for the attractive forces present in a
(IIT-JEE 2011)
real gas in the van der Waals equation is
an2 an2 Solution
(a) nb (b) 2 (c) − (d) −nb
V V2 (a), (b), (c), (d)
(IIT-JEE 2009) Collisions of gas molecules are perfectly elastic which means
that the total energy of the molecules before and after the
Solution collision remains the same.
(b) The measure of forces of attraction for n mol of real gas Heavier particles transfer more momentum because momen-
(n2a/V2) in the following van der Waals equation tum depends on the mass.
Only a small number of molecules possess very high velocity.
 n2a  Particles of gas move in all possible directions in straight
 p + 2  (V − nb ) = nRT
 V  lines. The distribution of speed remains constant at a par-
ticular temperature.
8. At 400 K, the root mean square (rms) speed of a gas X (molec-
ular weight = 40) is equal to the most probable speed of gas 11. For 1 mol of a van der Waals gas when b = 0 and T = 300 K, the
Y at 60 K. The molecular weight of the gas Y is _________. pV vs. 1/V plot is shown in figure below. The value of the van
(IIT-JEE 2009) der Waals constant a (atm L2 mol−2) is

Solution
(4) Given that urms (X gas) at 400 K = ump (Y gas) at 60 K, (Graph not to scale)
24.6
molecular weight M1 = 40, M2 = ?, T1 = 400 K and

pV (L atm mol−1)
T2 = 60 K. Therefore, 23.1
21.6
3RT1 2RT2 20.1
=
M1 M2

400 × 3 × R 2 × R × 60
=
40 M2

Solving, we get M2 = 4. 0 2.0 3.0


1
9. To an evacuated vessel with movable piston under external (mol L−1)
V
pressure of 1 atm, 0.1 mol of He and 1.0 mol of an unknown
compound (vapour pressure 0.68 atm at 0°C) are introduced. (a) 1.0 (b) 4.5 (c) 1.5 (d) 3.0
Considering the ideal gas behaviour, the total volume (in (IIT-JEE 2012)
litres) of the gases at 0°C is close to ________.
Solution
(IIT-JEE 2011)
(c) van der Waals equation is
Solution
(7) Let V be the volume of the gas. Applying ideal gas equa-  a
tion pV = nRT, we get  p + 2  (V − b ) = RT
V
pV V
n= = 0.68 × but it is given that b = 0. So, the equation reduces to
RT RT
So, the total moles of gas present in the vessel will be =  a a
(0.1 + 0.68 × V/RT)  p + 2  V = RT ⇒ pV = − + RT
V V
Again applying ideal gas equation pV = nRT, we get
Comparing it with a straight line equation we get slope
 V 
1× V =  0.1+ 0.68 ×  RT = 0.1RT + 0.68V as −a. Calculating the slope, we get
 RT 
273 24.6 − 20.1
Solving, we get V = 0.1× 0.082 × =7 = 1.5 ⇒ a = 1.5
0.32 3.0 − 0

10. According to kinetic theory of gases, Passage for Questions 12 and 13: X and Y are two volatile liquids
(a) collisions are always elastic. with molar weights of 10 g mol−1 and 40 g mol−1, respectively.
(b) heavier molecules transfer more momentum of the wall Two cotton plugs, one soaked in X and the other soaked in Y, are
of the container. simultaneously placed at the ends of a tube of length L = 24 cm,

Chapter-5.indd 138 8/8/2016 2:38:01 PM


Advanced Level Problems 139

as shown in the figure. The tube is filled with an inert gas at 1


(a) (n1T1 + n2T2 ) (b) (n1n2 + T1T2 )
atmosphere pressure and a temperature of 300 K. Vapours of X (n1 + n2 )
(n1n2 )
and Y react to form a product which is first observed at a distance
d cm from the plug soaked in X. Take X and Y to have equal molec-
(c) (n1T2 + n2T1) (d) (n1T1 + n2T2 )
ular diameters and assume ideal behaviour for the inert gas and (n1 + n2 ) (n1 + n2 )
the two vapours.
(JEE Advanced 2014) Solution
L = 24 cm (d) According to the kinetic theory of gases, the kinetic
energy of an ideal gas molecule at temperature T is
given by K.E. = (3/2)kT. As there is no force of attraction
Cotton wool among the molecules of a perfect gas, P.E. of the mole-
Cotton wool
soaked in X d Initial formation soaked in Y cule is zero. So the energy of a molecule of perfect gas is
of the product
3 3
E = K.E. + P.E. = kT + 0 = kT
12. The value of d in cm (shown in the figure), as estimated from 2 2
Graham’s law, is  Now, if T is the temperature of the mixture, by conserva-
(a) 8 (b) 12 (c) 16 (d) 20 tion of energy, we have
Solution n1E1 + n2E2 = (n1 + n2 )E
(c) Using Graham’s law of diffusion we have
3  3  3 (n T + n T )
or n1  kT1 + n2  kT2  = (n1 + n2 ) kT ⇒ T = 1 1 2 2
x  40 
1/ 2 2  2  2 (n1 + n2 )
=  ⇒ x = 16
24 − x  10  16. Calculate the pressure exerted by one mole of CO2 gas at
273 K, if the van der Waals constant a = 3.592 dm6 atm mol−2.
13. The experimental value of d is found to be smaller than the Assume that the volume occupied by CO2 molecules is
estimate obtained using Graham’s law. This is due to negligible.
(a) Larger mean free path for X as compared to that of Y. (a) 0.865 atm (b) 0.992 atm
(b) Larger mean free path for Y as compared to that of X. (c) 0.963 atm (d) 0.824 atm
(c) Increased collision frequency of Y with the inert gas as
compared to that of X with the inert gas. Solution
(d) Increased collision frequency of X with the inert gas as (b) The van der Waals equation for 1 mol of gas is given by
compared to that of Y with the inert gas.
 a
Solution  p + 2  (V − b ) = RT
V
(d) Collision frequency of X is greater than Y.
The volume occupies by CO2 molecules is negligible
14. A graph is plotted for a real gas which follows van der Waals
implies that b = 0. Substituting given values and consid-
equation with pVm taken on y-axis and p on x-axis. Find the
ering V = 22.4 L for 1 mol of gas at 1 atm, we get
intercept of the line where Vm is the molar volume.
Solution RT a 0.082 × 273 3.592 501.45 − 3.592
p= − = − =
For a real gas, the plot of pVm vs. p can be of two types A or V V2 22.4 22.4 2 501.76
B, but at the point of intercept, that is, p = 0 or when p is low, 497.8
85
= = 0.9922 atm
van der Waals equation reduces to ideal gas equation pV = 501.76
nRT. For 1 molar, comparing pVm = RT with y = mx + C, we get
the intercept on y-axis as R. 17. The compression factor (compressibility factor) for one mole
of a van der Waals gas at 0°C and 100 atm pressure is found
B
to be 0.5. Assuming that the volume of a gas molecule is neg-
A ligible, calculate the van der Waals constant a.
pVm Solution
The compressibility factor is given by Z = pV/nRT. Therefore,
substituting values, we get the volume as
RT
Z × n × R × T 0.5 × 1× 0.082 × 273
V= = = 0.111 L
p 100
p
When the volume of the gas is negligible, the van der Waals
15. Two perfect gases at absolute temperatures T1 and T2 are constant b = 0, so the equation becomes
mixed. Assuming there is no loss of energy, find the tempera-
ture of the mixture if the masses of the molecules are m1 and a
pV = RT −
m2 and the number of molecules in the gases are n1 and n2. V

Chapter-5.indd 139 8/8/2016 2:38:04 PM


140 Chapter 5 States of Matter

or a = RTV − pV 2 = 0.082 × 273 × 0.111− 100(0.111)2 Solution


(a) Using the expression pV = nRT, we have
= 2.48 − 1.23 = 1.25 atm L2 mol−2

18. Oxygen is present in 1 L flask at a pressure of 7.6 × 10−10 mm  7.6 × 10 −10  
  atm (1L )
Hg. Calculate the number of oxygen molecules in the flask 760 
n=
pV
=   = 4.459 × 10 −14 mol
at 0°C.
RT (0.0821 atm L K −1 mol−2 )(273.15 K )
(a) 2.686 × 1010 (b) 26.86 × 1010
(c) 0.626 × 10 12 (d) 4.123 × 108
N = nNA = (4.459 × 10−14 mol−1) = 2.686 × 1010

PRACTICE EXERCISE
Level I (c) its pressure is more than critical pressure.
(d) its temperature is more than Boyle’s temperature.
Single Correct Choice Type
10. Which of the following statements is correct as shown in the
1. The ratio between the root mean square speed of H2 at 50 K graph?
and root mean square speed of O2 at 800 K is
H2
(a) 1:2 (b) 1:1 (c) 2:1 (d) 8:1 at 0°C
N2 CO2
He CH4
2. A real gas is supposed to obey the gas equation p(V − b) = RT
at STP. If 1 mol of a gas occupies 25 dm3 volume at STP, then its
compressibility factor is (b = 2.586 L mol−1) Z=1 Ideal gas
(a) 1.115 (b) 0.115 (c) 0.784 (d) 0.885

3. 1 mol of N2 gas at 0.8 atm takes 38 s to diffuse through a pin-


hole, whereas 1 mol of an unknown gas at 1.6 atm takes 57 s p
to diffuse through the same pinhole. The molecular weight of
(a) The slope of Z vs. p at constant temperature for all real
unknown gas is
gases, is b /RT .
(a) 126 (b) 64 (c) 80 (d) 252
(b) The slope of Z vs. p at constant temperature for both He
4. A small bubble rises from the bottom of a lake, where the tem- and H2 is b /RT .
perature and pressure are 8°C and 6.0 atm, to the water’s sur- (c) The slope of Z vs. p at low pressure for all real gases, at
face, where the temperature is 25°C and pressure is 1.0 atm. constant temperature is b /RT .
Calculate the final volume of the bubble if its initial volume (d) The slope of Z vs. p at high pressure and at constant tem-
was 2 mL. perature for real gases is −b /RT .
(a) 14 mL (b) 12.72 mL (c) 11.21 mL (d) 15 mL 11. Two flasks X and Y of volumes 250 mL and 300 mL respec-
tively at the same temperature are connected by a stop cock
5. Calculate relative rate of effusion of O2 to CH4 through a
of negligible volume. The flask X contains nitrogen gas at a
container containing O2 and CH4 in 3 : 2 mass ratio
pressure of 660 torr and the flask Y contains neon gas at a
(a) 3 2 / 4 (b) 3/ 4 2 (c) 3/ 2 2 (d) 2 3 / 4 pressure of 825 torr. If the stop cock is opened to allow the
two gases to mix, the partial pressure of neon gas and total
6. 4 g of sulphur dioxide gas diffuses from a container in 8 min. pressure of the system will be
Mass of helium gas diffusing from the same container over (a) 300 torr, 700 torr (b) 400 torr, 700 torr
the same time interval is (c) 450 torr, 750 torr (d) 300 torr, 750 torr
(a) 0.5 g (b) 1 g (c) 2 g (d) 4 g
12. The time taken for a certain volume of gas to diffuse through
7. A gas obeys the equation of state p(V − b) = RT. (The parame- a small hole was 2 min. Under similar conditions an equal
ter b is a constant.) The slope for an isochore will be: volume of oxygen took 5.65 min to pass. The molecular mass
(a) Negative (b) Zero (c) R/(V − b) (d) R/p of the gas is
(a) 32.0 (b) 11.33 (c) 4.0 (d) 8.0
8. For non-zero value of force of attraction between gas mole-
cules, gas equation will be 13. What will be the molecular diameter of helium if van der
2 Waals constant, b = 24 mL mol–1?
(a) pV = nRT − (n a /V ) (b) pV = nRT + nbP
(a) 2.71 Å (b) 2.71 nm (c) 5.42 Å (d) 542 Å
(c) p = (nRT /V − b ) (d) pV = nRT
14. The rms speed of the molecules of a gas of density 4 kgm–3
9. An ideal gas obeying kinetic gas equation can be liquefied if and pressure 1.2 × 105 Nm–2 is:
(a) its temperature is more than critical temperature. (a) 120 ms–1 (b) 300 ms–1
(b) It cannot be liquefied at any value of p and T. (c) 600 ms–1 (d) 900 ms–1

Chapter-5.indd 140 8/8/2016 2:38:09 PM


Practice Exercise 141

15. The compressibility factor Z = ( pV /RT ) for 1 mol of a real gas 25. The rms velocity of hydrogen is 7 times the rms velocity of
is greater than unity at a pressure of 1 atm and 273.15 K. The nitrogen. If T is the temperature of the gas
molar volume of the gas at STP will be (a) T(H2) = T(N2) (b) T(H2) > T(N2)
(a) less than 22.4 L. (b) greater than 22.4 L. (c) T(H2) < T(N2) (d) T(H2) > 7 T(N2)
(c) equal to 22.4 L. (d) none of these.
16. 20 mL of O2 gas passed through a square pin hole of side x in 26. The root mean square velocity of an ideal gas at constant
5 s at 4 atm pressure, whereas 40 mL of unknown gas passed pressure varies with density (d) as
through a circular pin hole of radius x in 2 s at 7 atm pressure. (a) d 2 (b) d (c) d (d) 1/ d
The molecular mass of unknown gas is
27. Helium atom is two times heavier than a hydrogen mol-
(a) 3.8 g (b) 38.72 g (c) 387.2 g (d) 19.36 g
ecule. At 298 K, the average kinetic energy of a helium
17. Compressibility factor for 1 mol of a van der Waals gas at atom is
0  °C and 100 atm pressure is found to be 0.5. The volume (a) two times that of a hydrogen molecule.
occupied by the gas is (b) same as that of a hydrogen molecule.
(a) 2.0224 L (b) 1.4666 L (c) four times that of a hydrogen molecule.
(c) 0.8542 L (d) 0.1119 L (d) half that of a hydrogen molecule.
18. Rate of effusion of LPG (a mixture of n-butane and propane) is 28. When an ideal gas undergoes unrestrained expansion, no
1.25 times that of SO3. The mole fraction of n-butane in LPG is cooling occurs because the molecules
(a) 0.75 (b) 0.25 (c) 0.50 (d) 0.67 (a) are above the inversion temperature.
19. The ratio of speeds of diffusion of two gases A and B is 1:4. (b) exert no attractive forces on each other.
If the ratio of their mass present in the mixture is 2:3, then (c) do work equal to loss in kinetic energy.
which of the following is the ratio of their mole-fractions? (d) collide without loss of energy.
(a) 24:1 (b) 1:24 (c) 32:1 (d) 3:17
29. The graph of Maxwell–Boltzmann distribution of molecular
20. At 47°C and 16.0 atm, the molar volume of NH3 gas is about velocities is shown below
10% less than the molar volume of an ideal gas. Which of the
following is the true reason? T1
Fraction of molecules

(a) NH3 decomposes to N2 and H2 at 47°C. T2


(b) The force of attraction between NH3 molecules is signifi-
T3
cant at this temperature and pressure.
(c) The volume occupied by NH3 molecules themselves is a
significant fraction of the volume of the container at this
temperature and pressure.
(d) At 16 atm, NH3 molecules no longer move randomly.
Velocity
21. At 748.0 torr and 20.65°C, what is the density of argon (in g L−1)?
(a) 2.06 (b) 1.63 (c) 1.86 (d) 3.02 Which of the following is the correct order of temperature?
(a) T1 < T2 < T3 (b) T3 < T2 < T1
22. A chemist isolated a gas in a glass bulb with a volume of
(c) T2 < T1 < T3 (d) T2 = T1 < T3
225 mL at a temperature of 25°C and a pressure (in the bulb)
of 10.0 torr. The gas weighed 12.1 mg. What is the molecular 30. The graphs shown below the behaviour of various gases.
mass of the gas? Which of these gases exhibit behaviour that deviates most
(a) 78.9 g mol−1 (b) 35.2 g mol−1 significantly from that expected for an ideal gas?
(c) 88.2 g mol −1 (d) 96.3 g mol−1
Gas B
(a) Gas A (b)
23. A mixture of N2, O2 and CO2 has a total pressure of 740 torr.
In this mixture the partial pressure of N2 is 120 torr and the pV
V
partial pressure of O2 is 400 torr. What is the partial pressure
of the CO2?
(n, T, const.)
(a) 220 torr (b) 300 torr (p, const.)
(c) 225 torr (d) 339 torr T p

24. A sample of carbon monoxide was prepared and collected (c) (d) Gas D
Gas C
over water at a temperature of 20°C and a total pressure of
754 torr. It occupied a volume of 268 mL. Calculate the dry p pV
volume (in mL) of CO under a pressure of 1 atm and 20°C.
(Partial pressure of water at 20°C is 17.54 torr).
(n, V, const.) (n, const.)
(a) 239 mL (b) 260 mL
T
(c) 285 mL (d) 290 mL V

Chapter-5.indd 141 8/8/2016 2:38:11 PM


142 Chapter 5 States of Matter

31. The temperature of an ideal gas can be raised by (c) Bulb B weighs more than bulb A.
(a) decreasing the volume but keeping the quantity and (d) Bulb A contains more number of molecules than bulb B.
pressure fixed.
39. Which of the following statements are wrong?
(b) decreasing the quantity but keeping the volume and
(a) In mixture of equal weights of O2 and H2 gases, the fraction
pressure fixed.
of the partial pressure exerted by O2 is more than that of H2.
(c) decreasing the pressure but keeping the quantity and
(b) At high p and low T, gases approach ideal behaviour.
volume fixed.
(c) When gases are collected over water, pobserved = pdry gas
(d) decreasing the pressure and volume but keeping the
+ Aqueous tension.
quantity fixed.
(d) At critical state the compressibility factor for 1 mol of
32. The pressure exerted by 12 g of an ideal gas at temperature gas = 1.
T(°C) in a vessel of volume V is 1 atm. When the tempera-
40. Which of the following indicate kinetic gas equation?
ture is increased by 10°C at the same volume, the pressure
increases by 10%. If molecular weight of the gas is 120, the (a) pV = (1/ 3)mnu2 (b) pV = nRT
temperature (T°C) and volume (V) are 2
(a) T = −273°C, V = 0.082 L (b) T = −173°C, V = 0.82 L (c) 3 p = du2 (d) pV = (1/ 3)Mu
(c) T = 0°C, V = 22.4 L (d) T = 27°C, V = 22.4 L 41. If temperature of a gas is raised, which of the following would
33. At what temperature will the rate of effusion of N2 be 1.625 be true?
times the rate of effusion of SO2 at 500°C? (a) Fraction of the molecules possessing most probable
(a) 273 K (b) 830 K velocity will increase.
(c) 110 K (d) 173 K (b) Fraction of the molecules possessing most probable
velocity will decrease.
34. Many laboratory gases are sold in steel cylinders with a vol-
(c) Fraction possessing very low velocity will decrease.
ume of 43.8 L. What mass (in grams) of argon is inside a cylin-
(d) Fraction possessing very high velocity will increase.
der whose pressure is 17,180 kPa at 20°C?
(a) 1.33 × 104 g (b) 1.23 × 104 g 42. What conclusion would you draw from the following graphs
4
(c) 1.13 × 10 g (d) 1.03 × 104 g for an ideal gas?
35. Two glass bulbs A and B at same temperature are connected V p
by a very small tube having a stop cock. Bulb A has a volume
of 100 cm3 and contained the gas, while bulb B was empty.
On opening the stop cock, the pressure fell down to 20%. The
volume of the bulb B is
(a) 1200 cm3 (b) 800 cm3 T(°C) T(°C)
(c) 600 cm 3 (d) 400 cm3
(a) As the temperature is reduced, the volume as well as the
36. Dry ice (solid CO2) has occasionally been used as an explo- pressure increases.
sive in mining. A hole is drilled, dry ice and a small amount of
(b) As the temperature is reduced, the volume becomes
gun powder are placed in the hole, a fuse is added, and the
zero and the pressure reaches infinity.
hole is plugged. When lit, it explodes up with an immense
(c) As the temperature is reduced, the pressure decreases.
pressure. Assume that 500.0 g of dry ice is placed in a cavity
(d) A point is reached where, theoretically, the volume
with a volume of 0.800 L and the ignited gunpowder heats
becomes zero.
the CO2 to 700 K. What is the final pressure inside the hole?
(a) 416 atm (b) 816 atm 43. Which of the following statements are correct?
(c) 616 atm (d) 1216 atm
(a) Z (compressibility factor) for ideal gas is independent of
temperature and pressure.
Level II (b) Z for ideal gas is greater than one.
Multiple Correct Choice Type (c) Z for non-ideal gas is either greater than one or less than
one as well as dependent on temperature and pressure.
37. A gas obeys p(V − b) = RT. Which of the following are correct
(d) When Z > 1, force of attraction dominates over force of
about this gas?
repulsion.
(a) Isochoric curves have slope = R/(V − b ).
(b) Isobaric curves have slope R/p and intercept b 44. Which of the following are correct?
(c) For the gas the compressibility factor = 1+ ( pb / RT ) .
(d) The attractive forces are overcome by repulsive forces. (a) Ne > O2 > F2 > SO 2 (Order of rate of diffusion at same
T and p).
38. A bulb A of volume V contains H2 gas and another bulb B
(b) 1 g H2 > 1 g He > 1 g CH4 > 1 g O 2 (Total K.E. at same T).
of same volume contains CH4 gas at same T exerting same
pressure. Which of the following are correct? (c) ump > uaverage > urms (order of value of velocities).
(a) Both the bulbs contain equal number of moles.
(b) The average kinetic energy of molecules in both the (d) N2 > O2 > O3 > SO2 (order of average K.E. per molecule
bulbs is same. at same T).

Chapter-5.indd 142 8/8/2016 2:38:14 PM


Practice Exercise 143

45. Precisely 1 mol of helium and 1 mol of neon are placed in a (c) For hydrogen gas b dominates over a at all temperatures.
container. Indicate the correct statements about the system: (d) At high pressure van der Waals constant b dominates
(a) Molecules of the two gases strike the wall of the con- over a.
tainer with same frequency.
52. In the equation pV = RT, the value of R will not depend on
(b) Molecules of helium strike the wall more frequently.
(a) the nature of the gas.
(c) Molecules of helium has greater average molecular
(b) the temperature of the gas.
speed.
(c) the pressure of the gas.
(d) Helium exerts larger pressure.
(d) units of measurement.
46. Which of the following statement(s) is are correct?
53. Which of the following statements is(are) correct?
(a) Gas molecules never come to rest except at 0 K.
(b) At T > 0, some gas molecules are unfortunate not to pos- (a) At constant temperature, the gas density is directly pro-
sess translational kinetic energy. portional to pressure.
(c) On raising temperature, fraction of molecules possess- (b) At higher pressures, gases deviate from Boyle’s law.
ing most probable velocity increases. (c) Plots of p vs. T at constant volumes for an ideal gas are
(d) At a given temperature the most probable velocity var- parabolic.
ies inversely with the square root of vapour density of (d) At −273 K gases have zero volume which corresponds to
the gas. solid state.
47. Which of the following statements are correct? 54. Which of the following statements are not correct?
(a) It is not possible to compress a gas at a temperature (a) Surface tension of liquids decreases with increase in
below TC. temperature.
(b) At a temperature below TC, the molecules are close (b) The viscosity of a liquid increases with decrease of
enough for the attractive forces to act, and condensa- temperature.
tion occurs. (c) Liquids diffuse faster than gases.
(c) No condensation takes place above TC. (d) Gases can be liquefied only above their critical
(d) The kinetic energy of the gas molecules is higher above temperatures.
TC, is considered as super critical fluid.
48. According to Graham’s law at a given temperature, the ratio
Passage Type
of the rates of diffusion rA rB of gases A and B is given by Passage for Questions 55–57: Density (d ) of gas is inversely pro-
1/ 2 1/ 2 portional to absolute temperature and directly proportional to
pA  MA  MA  pA 
(a) × (b) × pressure
pB  MB  MB  pB 
p dT dT d T
1/ 2 ⇒d ∝ ⇒ = constant ⇒ 1 1 = 2 2
pA  MB  p dB T p p1 p2
(c) × (d) A
pB  MA  pB dA
Density at a particular temperature and pressure can be calcu-
49. If a gas is expanded at constant temperature lated by using ideal gas equation
(a) the pressure decreases. Mass
(b) the kinetic energy of the molecules remains the same. pV = nRT ⇒ pV = × RT
Molar mass ( M )
(c) the kinetic energy of the molecules decreases.
(d) the number of molecules of the gas increases. Mass
p×M = × RT ⇒ p × M = d × RT
50. The van der Waals parameters of two gases are given as: Volume

Gas a (dm6 bar mol−2) b (dm3 mol−1) pM


d=
Gas A 6.5 0.056 RT
Gas B 18.0 0.011
55. Which of the following has maximum density?
Considering the value of parameters, which of the following (a) O2 at 25°C and 1 atm
statements is(are) correct?
(a) Critical volume of A < Critical volume of B. (b) O2 at 0°C and 2 atm
(b) Critical pressure of A > Critical pressure of B. (c) O2 at 273°C and 1 atm
(c) Critical temperature of A < Critical temperature of B.
(d) Ease of liquefaction of A > Ease of liquefaction of B. (d) O2 at 0°C and 1 atm
51. Select the correct statement(s).
56. The density of CO2 at 1 atm and 273 K is
(a) At Boyle’s temperature, a real gas behaves like an ideal
gas at low pressure. (a) 1.96 gL−1 (b) 2.12 gL−1
(b) Above critical conditions, a real gas behaves like an ideal
gas. (c) 1.09 gL−1 (d) 2.02 gL−1

Chapter-5.indd 143 8/8/2016 2:38:19 PM


144 Chapter 5 States of Matter

62. Compressibility factor (Z) for a real gas at moderately low


57. The density of gas is 3.8 gL−1 at STP. The density at 27°C and
pressure is given as:
700 mm Hg pressure will be
PV  bp 
(a) 3.185 g L−1 (b) 3.185 g m L−1 (a) (b) 1+ 
RT  RT 
(c) 3.185 kg L−1 (d) 3.185 kg m L−1  a 
(c) 1−  (d) 1− bp 
 RTV   RT 
 
Passage for Questions 58–60: It is known that equation of state
63. At critical state, compressibility factor (Z) is equal to
pV = nRT derived from postulates of kinetic theory of gases, is
(a) 3 / 8 (b) 8 / 3 (c) 4 / 3 (d) 3 / 4
valid only for ideal gas. Real gases tend to obey this equation only
under conditions of low pressure and high temperature. At higher
pressure and lower temperature greater would be the deviation Passage for Questions 64–66: Graham’s law of diffusion: The phe-
from ideal behaviour. nomenon of spontaneous intermixing of gases against the law of
pV gravitation is known as diffusion. If diffusion occurs through small
Compressibility factor, Z = orifice of the container, then it is known as effusion.
nRT
For ideal gas Z = 1 and for real gas Z > 1 or Z < 1. pA
The rate of effusion is defined as g =
CH4 CO 2p RTM
H2 where p is partial pressure of the gas, A is the area of cross-section
He of the orifice of the container; M is the molar mass of the gas.
NH3 Volume effused Moles effused
Rate of diffusion = =
t t
Pressure drop Distance travelled
= =
Z Ideal gas t t
CH4
64. 1 mol of gas A and 4 mol of gas O2 is taken inside the vessel,
CO which effuse through the small orifice of the vessel at the same
NH3
temperature, then which is the correct % of effused volume
of gas A and O2 initially, respectively? (Assume that the gas A
does not react with O2 gas and molar mass of gas A is 2 g.)
p (a) 50%, 50% (b) 60%, 40%
(c) 30%, 70% (d) 10%, 90%
Based on the information provided in the graph, answer the fol-
lowing questions: 65. 1 mol of gas X and 2 mol of gas Y enters from the end P and
Q of the cylinder, respectively. The cylinder has the same area
58. The correct order for van der Waals constant a of gases given is of cross-section throughout.
(a) CO > H2 > CH4 > NH3 > He
A
(b) NH3 > CH4 > CO > He > H2 P Q
(c) H2 > He > CO > CH4 > NH3 X Y
(d) NH3 > CO > CH4 > He > H2
59. For NH3 (g), Z < 1, so volume occupied by 1 mol of NH3 at The length of the cylinder is 150 cm. The gas X intermixes
STP is with gas Y at the point A. If the molecular weight of the gases
(a) = 22.4 L (b) > 22.4 L (c) < 22.4 L (d) = 0 X and Y is 20 and 80 respectively, then what will distance of
point A from Q?
60. In which of the following gases repulsive forces are dominat- (a) 75 cm (b) 50 cm
ing between molecules? (c) 100 cm (d) 90 cm
(a) NH3 (b) CO (c) CH4 (d) H2
66. The density of the gaseous mixture in a vessel (CH4 and
Passage for Questions 61–63: The extent of deviation of real He) at 2 atm and 300 K is 0.9756 g  L−1. If a small pin-hole
gases from ideal behaviour can be expressed in terms of com- is made on the wall of the vessel, through which gas effuse,
pressibility factor (Z). Corrected ideal gas equation is written as: then which of the following is the correct composition of the
pV = nZRT gases CH4 and He effusing out initially?
(a) 1:1 (b) 2:1
where symbols have their usual meaning. The reason why real gases
(c) 3:1 (d) 1:4
deviate from ideal behaviour can be explained by kinetic theory of
gases. For a real gas, compressibility factors (Z) have different values Paragraph for Questions 67–69: The following figure shows
in different pressure zones, but its value is fixed at critical state. three glass chambers that are connected by valves of negligible
61. Compressibility factor of a real gas at very high temperature volume. At the outset of an experiment, the valves are closed and
and very low pressure is: the chambers contain the gases as detailed in the diagram. All the
(a) < 1 (b) = 1 chambers are at the temperature of 300 K and external pressure
(c) > 1 (d) slightly more than one of 1.0 atm.

Chapter-5.indd 144 8/8/2016 2:38:24 PM


Practice Exercise 145

pext = 1.0 atm Throughout (c) pT vs. T2 for ideal gas at (r)
Valve 1 Valve 2 T = 300 K constant V and n
He N2
0.0 atm
4.1 atm 0.82 atm

6.0 L 4.0 L 1 (s)


(d) V vs. for ideal gas at
10.0 L p2
A B C constant T and n
67. Which of the following describes the relation between the
average velocity of gas molecules in chambers A and B
before valve 1 is opened. 74. Column I Column II
(a) vA ≠ vB (b) vA < vB (Constant) (Unit)
(c) vA > vB (d) Insufficient information (a) R (universal gas constant) (p) ergs/degree K
(b) a (van der Waals constant) (q) Joule mol–1 K–1
68. What is the total pressure in chamber B after valve 1 is (c) b (van der Waals constant) (r) atm L2 mol–2
opened? (d) k (Boltzmann) constant (s) m3 mol–1
(a) 0.31 atm (b) 2.05 atm
(c) 2.46 atm (d) 3.10 atm 75. The van der Waals constants a and b of a real gas are
3.6 L2 atm mol−2 and 0.05 L  Lmol−1 respectively. If 200 g of
69. Which of the following represents the total kinetic energy of gas (molecular mass 40) is placed in 10 L vessel at 300 K,
all the gas molecules after both valves are opened? (R = 0.082 Match the following:-
atm L K–1 mol–1 = 8.314 J K–1 mol–1)
(a) 2836 J (b) 3280 J (c) 4520 J (d) 4988 J Column I Column II
(a) Pressure correction (atm) (p) 0.25
Matrix – Match Type
(b) Free space for the molecules to move (q) 0.0125
70. Column I Column II about (L)
(a) Critical temperature (p) a / Rb
(c) Actual volume of the gas molecules per (r) 0.9
(b) Boyle temperature (q) 2a / Rb
mole (L)
(c) Inversion temperature (r) a / 27b2
(d) Effective volume occupied by total gas (s) 9.75
(d) Critical pressure (s) 8a / 27Rb molecules (L)
71. Column I Column II 76. Column I Column II
Vt M2 (a) Most probable speed (p) (3/2) kT
(a) Boyle’s law (p) 1 2 = (at constant
V2t1 M1 (b) Average kinetic energy of gas (q) Vreal/Videal
T and p) (c) van der Waals constant a (r) correction factor for
(b) Charle’s law (q) pA = x A ptotal the force of attrac-
tion
(c) Dalton’s law (r) pV = K (at constant T for a (d) Compression factor (s) 2RT /M
fixed amount of gas)
(d) Graham’s law
V T
(s) 1 = 1 (at constant p for
Integer Type
V2 T2
77. Two moles of a gas are confined to a 5 L flask at 27°C. Its
a fixed amount of gas)
pressure using van der Waals equation is ____ atm. (a = 6.25
72. Column I Column II atm L2 mol–2 and b = 0.037 L mol–1).
(Unit of R) (Value) 78. Nitrogen is present in a flask of 0.56 L capacity at a pressure
(a) erg K mol−1 −1
(p) 8.314 of 7.6 × 10 −10 mm of Hg at 0°C. The number of atoms pres-
(b) mL atm K −1 mol−1 (q) 8.314 × 107 ent in the flask is m ×1010 . The value of m is ____.
(c) kPaL K −1 mol−1 (r) 82.05 79. The temperature at which the rate of effusion of O2 is 1.6
times of SO2 at 127°C is ( m × 128 )K . The value of m is ____.
(d) L atm mol−1K −1 (s) 0.0821
80. The compressibility factor for N2 at 330 K and 800 atm is
73. Column I Column II 1.90 and at 570 K and 200 atm is 1.10. A. Certain mass of N2
1 occupies a volume of 1 dm3 at 330 K and 800 atm. Volume
(a) 2 vs. p for ideal gas at (p)
occupied by same quantity of N2 gas at 570 K and 200 atm
V
in L is ____.
constant T and n
81. A flask contains a mixture of N2 and CO2 at total pressure of
1 (q) 1.5 atm and at the temperature 27°C. If CO2 is removed, the
(b) V vs. for ideal gas at
T pressure falls to 0.5 atm and the weight of the flask drops by
constant p and n
22 g. The weight of N2 in the mixture in grams is ____.

Chapter-5.indd 145 8/8/2016 2:38:30 PM


146 Chapter 5 States of Matter

82. How many of the following factors can affect the transla-
84. Root mean square speed of a gas at 300 K is 3 R ms −1. The
tional kinetic energy of an ideal gas?
molar mass of gas in g is expressed as 10 x . Find x.
(a) temperature (b) pressure
(c) molar mass (d) force 85. A 4:1 molar mixture of He and CH4 is contained in a vessel
(e) molecular diameter at 20 atm pressure. Due to hole in the vessel the gas mix-
ture leaks out. The molar ratio of He and CH4 effusing out
83. Find the number of correct statements among the initially is y:1. Find the value of y.
following. 86. The stopcock, connecting the two bulbs of volumes 5 L and
(a) The compressibility factor for He is [1+ ( pb / RT )] . 10 L containing an ideal gas at 9 atm and 6 atm respectively,
(b) The numerical value of van der Waals constant ‘a’ for is opened. What is the final pressure in the two bulbs if the
H2O is higher than C6H6 due to H-bonding. temperature remained the same?
(c) Ideal gas does not show Joule–Thomson effect.
(d) All molecules of an ideal gas move with same speed. 87. If 1023 gas molecules each having a mass of 10–25 kg, placed
(e) Gases like CO2 and NH3 can never show positive in a 1 L container, move with rms speed of 105 cm s−1 then
deviation. the total kinetic energy of gaseous molecules in kJ is

ANSWER KEY

Level I
1. (b) 2. (a) 3. (d) 4. (b) 5. (b) 6. (b)
7. (c) 8. (a) 9. (c) 10. (b) 11. (c) 12. (c)
13. (a) 14. (b) 15. (b) 16. (b) 17. (d) 18. (c)
19. (b) 20. (b) 21. (b) 22. (c) 23. (a) 24. (b)
25. (c) 26. (d) 27. (b) 28. (b) 29. (a) 30. (d)
31. (b) 32. (b) 33. (b) 34. (b) 35. (d) 36. (b)

Level II
37. (a), (b), (c), (d) 38. (a), (b), (c) 39. (a), (b), (d) 40. (a), (c), (d) 41. (b), (c), (d) 42. (c), (d)
43. (a), (c) 44. (a), (b) 45. (b), (c) 46. (a), (d) 47. (b), (c), (d) 48. (c), (d)
49. (a), (b) 50. (a), (c) 51. (a), (b), (d) 52. (a), (b), (c) 53. (a) (b) 54. (c), (d)
55. (b) 56. (a) 57. (a) 58. (d) 59. (c) 60. (d)
61. (b) 62. (c) 63. (a) 64. (a) 65. (a) 66. (a)
67. (c) 68. (b) 69. (d) 70. (a) → s; (b) → p; (c) → q; (d) → r
71. (a)→ r; (b) →s; (c) → q; (d) →p 72. (a)→ q; (b) →r; (c) →p; (d) →s 73. (a) →r; (b)→ s; (c)→p; (d)→ q
74. (a) → p; (b) → r; (c) → s; (d) → q 75. (a)→r; (b)→s; (c)→q; (d)→p 76. (a) → s; (b) →p; (c) → r; (d) →q
77. (9) 78. (3) 79. (4) 80. (4) 81. (7) 82. (1)
83. (2) 84. (5) 85. (8) 86. (7) 87. (5)

HINTS AND EXPLANATIONS


Level I 4. (b) The amount of the gas in the bubble remains constant,
so that n1 = n2 . To calculate the final volume, V2
Single Correct Choice Type
p1 T2 6.0 298
cH2 TH2 MO2 50 32 1 V2 = V1 × × = 2.0 × × = 12.72 mL
1. (b) = × = × = p2 T1 1.0 281
c O2 MH2 TO2 2 800 1
rO2 nO2 MCH4 3 16 16 3
pb 1× 2.586 5. (b) = × = × × =
2. (a) Z = 1+ = 1+ = 1.115 rCH4 nCH4 MO2 2 32 32 4 2
nRT 1× 0.0821× 273
6. (b)
n1 t2 p1 M2
3. (d) × = × n1.t2 M2 w1 M2 M2
t1 n2 p2 M1 = ; × =
n2 .t1 M1 M1 w2 M1
1 57 0.8 M2 w1 M1 w1 4
× = ⇒ M2 = 252 = ; = ⇒ w1 = 1 g
38 1 1.6 28 w2 M2 4 64

Chapter-5.indd 146 8/8/2016 2:38:34 PM


Hints and Explanations 147

7. (c) 12. (c) According to Graham’s law of diffusion


p(V − b ) = RT V1 t2 M2
⋅ =
RT  R  t1 V2 M1
p= = T
V − b  V − b Since, V1 = V2
Since, V is constant for isochoric process Therefore,
R 2
Slope =  t2  M2 5.65 × 5.65 32 32 × 4
V −b  t  = M = 2 × 2 = M ⇒ M1 = 5.65 × 5.65 = 4.0
1 1 1
8. (a) For n moles of a real gas, van der Waals equation is
13. (a) Since, approximately
 n2a  b = 4Vm
 p + 2  (V − nb ) = nRT
 V  4
24 = 4 × NA × p r 3
3
1/ 3
n2a  3 × 24 
When ≠0 ⇒r =  23 
V2  16 × ( 22 / 7 ) × 6 .023 × 10 
 n2a  ⇒ r = 1.355 × 10 −8 cm
 p + 2  V = nRT
 V  Therefore,

n2a d = 2r = 2 × 1.355 × 10 −8 cm = 2.71 Å


pV = nRT −
V
3RT 3p 3 × 1.2 × 105
9. (c) Conceptual 14. (b) c = = = = 300 ms −1
M d 4
10. (b) van der Waals equation is given by 15. (b) The gas is less compressible than ideal gas. Hence,
 n a
2 Vm > 22.4 L .
 p + 2  (V − b ) = RT
 V  16. (b)
For H2 and He, force of attraction is zero. Therefore, a = 0. (V1 / t1) p1A1 M2
=
Substituting the value of a, van der Waals equation (V2 / t2 ) p2 A2 M1
becomes (20 / 5) 4 x2 M2
p(V − b ) = RT = ⇒ M2 = 38.72
⇒ pV = RT + pb ( 40 / 2) 7 (22 / 7) x 2 32
pV RT pb 17. (d)
Dividing the above equation
= + by RT on both the sides,
RT RT RT pV
we get Z=
pV pb nRT
⇒ = 1+ 100 × V
RT RT 0.5 =
pb 1× 0.0821× 273
⇒ Z = 1+ V = 0.1119 L
RT
 b rSO3
⇒ Z =   p +1 Mmix
 RT  18. (c) Since, =
rmix MSO3
Comparing with equation of the straight line, we get slope
b 1 Mmix
= Therefore , =
RT 1.25 80
11. (c) VX + VY = 250 + 300 = 550 mL ⇒ Mmix = 51.2

Since, the amount of nitrogen remains unchanged, therefore, Since Mmix = xbutane Mbutane + xpropane Mpropane
partial pressure of Therefore,
p V
N2 = X X 51.2 = 58 xbutane + 44(1− xbutane )
VX + VY
⇒ 7.2 = 14 xbutane
660 × 250
= 300 torr ⇒ xbutane = 0.5
550
pYVY 825 × 300 19. (b) Since,
Partial pressure of Ne = = = 450 torr rA 1 MB
VX + VY 550 = =
Therefore, total pressure = (300 + 450 ) = 750 torr rB 4 MA

Chapter-5.indd 147 8/8/2016 2:38:40 PM


148 Chapter 5 States of Matter

Therefore, that is, TH2 < TN2


2 3
nA : nB = :
MA MB 3RT
26. (d) Root mean square velocity(c) = (1)
2 3 1 M
: = :3
16 MB MB 8 pM = dRT (2)
nA : nB = 1: 24 From Eqs. (1) and (2), we get
x A nA 1
= = 3p 1
xB nB 24 c= ⇒c∝
d d
Vm, real
20. (b) Z= = 0.9 3
Vm, ideal 27. (b) Average kinetic energy = kBT ; Since, it is independent
2
Therefore, there will be attractive molecular forces as Z < 1. of molar mass, it will be same for He and H2 at a given
21. (b) In general pV = nRT, where n = mass/molecular mass. temperature.
Thus, 28. (b) When an ideal gas undergoes unrestricted expansion,
Mass
pV = × RT no cooling occurs as no attractive interaction exists
Molar mass
among molecules.
We arrive at the formula for the density (mass divided by
volume) of a gas as: 29. (a) As the peak of the curve decreases, temperature
increases.
p×M
d=
RT 30. (d) Graphs (a) represents Charle’s law; (b) represents Boyle’s

=
(748.0 torr ) (1 atm
760 torr ) (39.95 g mol−1) law; (c) represents Gay–Lussac’s law and (d) represents
non-ideal gas curve.
( L atm
0.0821 mol K )(293.80 K) 31. (b) From the ideal gas equation pV = nRT, we find that decreas-
= 1.63 gL−1 ing the quantity but keeping the volume and pressure
22. (c) First determine the number of moles from the ideal gas fixed leads to rise in the temperature of an ideal gas.
law: 12
32. (b) Moles of gas = 0.1
n=
pV (10.0
=
torr) 760(
1 atm
torr )(
255 mL 1000 1L
mL )( ) = 1.37 × 10−4 mol 120
RT ( L atm
0.0821 mol K
( 298 .2 K )
) pV = nRT or 1 × V = 0.1 × R (273 + t) (1)

Now, calculate the molecular mass 10


After temperature change, p = 1+ = 1.1 atm
1g  100
(12.1 mg) 
Mass  1000 mg 
Molecular mass = = = 88.2 g mol−1 T = (t + 273 + 10) = (283 + t) K
Number of moles 1.37 × 10 −4 mol 1.1 × V = 0.1 × R × (283 + t) (2)
1g 
(12.1 mg) 
Mass  1000 mg  Dividing Eq. (2) by Eq. (1), we get
Molecular mass = = = 88.2 g mol−1
Number of moles 1.37 × 10 −4 mol 1.1 283 + t
23. (a) pTotal = pN + pO + pCO = ⇒ t = −173 °C
2 2 2 1 273 + t
pCO = pTot − pN − pO
2 2 2 Substituting the value of t in Eq. (2), we get
pCO = 740 torr − 120 torr − 400 torr = 220 torr
2
1 × V = 1.1 (273–173) × 0.082 ⇒ V = 0.82 L
24. (b) ptotal = (pCO + pH O)
2
pH O = 17.54 torr at 20 °C
2 33. (b)
pCO = 754 – 17.54 torr = 736 torr
The temperature stays constant so, p1V1 = p2V2, and rSO2 MN2 TSO2 x 28 773
p V (736 torr )(268 mL ) = × or = ×
V2 = 1 1 = = 260 mL rN2 MSO2 TN2 1.625 x 64 TN2
p2 (760 torr )
28 773 × 1.625
3RT TN2 = × 773 × 1.625 = 7 × = 830 K
25. (c) Root mean square velocity (c) = 64 4
M
cH 2 TH 2 28 as Rate ∝ Temperature
= 7= ×
cN 2 2 TN 2 34. (b) Given that V = 43.8 L, Ar mass = ?, p = 17180 kPa = 169.6
atm, T = 293 K. Using pV = nRT, We get
On squaring both the sides, we get
14TH 2 pV 169.6 × 43.8
7= ⇒ TN 2 = 2TH 2 n= = = 308.8 mol
TN 2 RT 0.082 × 293

Chapter-5.indd 148 8/8/2016 2:38:45 PM


Hints and Explanations 149

Mass of Argon = moles (n) × Atomic mass of Ar = 308.8 51. (a), (b), (d) The temperature at which real gas approaches
× 40 = 12,352 g = 1.2352 × 104 g. ideal behaviour for appreciable range of pressure is called
Boyle’s temperature.
35. (d) Given that V1 = 100 cm3, p1= 100 atm and p2 = 20 atm.
For hydrogen gas b does not dominate over a at all
Thus, V2 can be found as
temperatures.
p1V1 100 × 100 At high pressure, van der Waals constant b dominates over a.
V2 = = = 500 cm3
p2 20 For 1 mol of ideal gas piVi = RT (1)

Thus, volume of B = 500 − 100 = 400 cm3  a


For 1 mol of real gas,  p + 2  (V − b ) = RT
36. (b) We know, that 44 g of CO2 = 1 mol V
At high pressure, the term a/V2 can be neglected in compar-
500
Therefore, 500 g of CO2 = = 11.36 mol ison to p.
44
Thus, p (V – b) = RT = piVi
Also given that T = 700 K, V = 0.8 L. Now, using the ideal
gas equation, we get or pV – pb = piVi ⇒ pV = (piVi + pb)
nRT 11.36 × 0.082 × 700 52. (a), (b), (c) The value of R is independent of the amount as well
p= = = 816 atm
V 0.8 as nature of the gas and is known as universal gas constant.

Level II Its value does not depend on the temperature and pressure,
hence its value is constant but it depends on the units of
Multiple Correct Choice Type measurement.
37. (a), (b), (c), (d) Conceptual pV
R=
38. (a), (b), (c) Conceptual nT

39. (a), (b), (d) Conceptual In S.I. units, pressure is expressed in N m−2 and volume in m3,
then R value is
40. (a), (c), (d) Conceptual
1, 01, 325 × 22.4 × 10 −3
41. (b), (c), (d) On increasing temperature, Maxwell’s curve of R= = 8.31 JK −1 mol−1
distribution of molecular velocities is flattened and maxi- 1× 273.15
mum is shifted to higher velocity. Likewise, R values have the different values like 0.0821
42. (c), (d) Conceptual L  atm  K−1  mol−1, 0.0831 bar L  K−1  mol−1, all depending on
the unit of measurement.
43. (a), (c) Conceptual
44. (a), (b) Conceptual 53. (a), (b) At constant temperature, the density is directly pro-
portional to pressure
45. (b), (c) Conceptual
r
46. (a), (d) Conceptual p= RT ⇒ p ∝ r
M
47. (b), (c), (d) It is possible to compress a gas at a temperature where ρ is the density of gas. Since V ∝ 1/p, if pressure is high,
below TC. the gas deviates from ideal behaviour and it deviates from
48. (c), (d) Conceptual Boyle’s law. Plot of p vs. T at constant volumes for an ideal gas
is a straight line. This is based on Gay-Lussac law.
49. (a), (b) Pressure is inversely proportional to volume, thus,
option (a) is correct. Average kinetic energy of a gas is directly
At constant V
proportional to absolute temperature, thus, option (b) is correct.
p
50. (a), (c) The critical temperature is given by
8a
TC =
27 Rb
T
8 × 6.5
For gas A: TC = = 413.85 dm3mol−1
27 × 0.0831× 0.056 54. (c), (d) Gases diffuse faster than liquid because in gases, the
intermolecular distance is quite large. As a result, there is less
8 × 18 force of attraction between the molecules. In liquids, inter-
For gas B: TC = = 5834.6 dm3mol−1
27 × 0.0831× 0.011 molecular distance is less in comparison to gases and weak
forces of attraction are present.
The critical volume is given by VC = 3b
Gases can be liquefied only at or below their critical tem-
Gas A: VC = 3 × 0.056 = 0.168 dm3 mol−1 perature (the temperature above which it cannot be lique-
Gas B: VC = 3 × 0.011 = 0.033 dm3 mol−1 fied by application of pressure).

Chapter-5.indd 149 8/8/2016 2:38:47 PM


150 Chapter 5 States of Matter

Passage Type 66. (a)


pMmix = dRT
55. (b) This option is correct because temperature is lowest
and pressure is the highest and density (d) it given by dRT 0.9756 × 0.082 × 300
2 × 32 64 Mmix = = ≈ 12
d= = P 2
R × 273 273R
1× 32 32 Let x , y are the moles of He and CH4 respectively,
For option (a): d = = ;
R × 298 298R
x × 4 + y × 16
1× 32 32 Mmix = = 12
For option (c): d = = ; x+y
R × 546 546R
1× 32 32 x 1
For option (d): d = = Therefore, =
R × 273 273R y 2
pM 1× 44 44
56. (a) d = = = = 1.96 gL−1 rHe n MCH4 1 16
RT 0.0821× 273 22.4 Since = He = = 1: 1
rCH4 nCH4 MHe 2 4
57. (a)
d1T1 d2T2 67. (c) Since the temperature of both the gases is the same
=
p1 p2 (300 K), average kinetic energy per molecule of either
3.80 × 273 d2 × 300 gas will be the same. But since He atoms are lighter, they
⇒ = will have higher average velocity.
760 700
⇒ d2 = 3.185 gL−1 68. (b)
pH2 × Vinital + pN2 × Vinital
58. (d) The attractive forces (determined by van der Waals con- pfinal =
stant a) between molecules reveal themselves by caus- Vfinal
ing the pressure of real gas to be slightly lower than that 4.1 × 6.0 + 0.82 × 10
= = 2.05 atm
expected for an ideal gas. 6 + 10
59. (c) For NH3 (g), Z < 1, which indicates the force of attraction 69. (d)
between the gas molecules are higher, so the volume
occupied by 1 mol of NH3 at STP is < 22.4 L. pV 4.1× 6
Moles of He , n = = =1
RT 0.082 × 300
60. (d) For Z > 1, the force of attraction between gas molecules
PV 0.82 × 10 1
becomes weak. Moles of N2 , n = = =
RT 0.082 × 300 3
61. (b) At very high temperature and very low pressure, a real
gas behaves in ideal manner. 3
The average kinetic energy of 1 mol of a gas is given by RT.
2
62. (c) van der Waals equation for 1 mol of gas:
3
So the total kinetic energy of n mol = nRT
 a a ab 2
 p + 2  (V − b ) = RT or pV + − bp − 2 = RT
V V V
3  1
a =  1+  × 8.314 × 300 = 4988.4 J
At very low pressure  bp 2  3
V
a pV a
Hence, pV = RT − ; Z = = 1−
V RT RTV Matrix-Match Type
63. (a) Conceptual
70. (a) ã s; (b) ã p; (c) ã q; d ã r
64. (a) Since, 71. (a) ã r; (b) ã s; (c) ã q; (d) ã p
rA n MO2 1 32 1
= A = = 72. (a) ã q; (b) ã r; (c) ã p; (d) ã s
rO2 nO2 MA 4 2 1
73. (a) ã r; (b) ã s; (c) ã p; (d) ã q
Vol. effused A 1
⇒ = 74. (a) ã p; (b) ã r; (c) ã s; (d) ã q
Vol. effused O2 1
Therefore, ratio of initially effused gases = 50%, 50% 75. (a) ã r; (b) ã s; (c) ã q; (d) ã p
2
rX nX MY 1 80 an2 3.6  200  3.6
65. (a) = = = 1: 1 Pressure correction = = ×  = × 25 = 0.9 atm
rY nY MX 2 20 V 2 100  40  100
2
distance travelled by gas X an2 3.6  200  3.6
⇒ = 1: 1 = ×  = × 25 = 0.9 atm
distancce travelled by gas Y V 2
100  40 100
⇒ distance of A from point Q = 75cm Free space = V − nb = 10 − 5 × 0.05 = 9.75 L

Chapter-5.indd 150 8/8/2016 2:38:53 PM


Hints and Explanations 151

Actual volume of the gas molecules per mole = no attractions among their molecules
b 0.05
= L = 0.0125 L dT 
4 4  d p  = 0 .
H
76. (a) ã s; (b) ã p; (c) ã r; (d) ã q
For satatement (d): All molecules of an ideal gas do not
Integer Type move with same speed but their aver-
age speed is same.
nRT an2 2 × 0.0821× 300 6.25 × 2 × 2 For satatement (e): Gases like CO2 and NH3 will show posi-
77. (9) p = − 2 = − =9
V − nb V 5 − (2 × 0.037) 5×5 tive deviation at high pressure.
84. (5)
7.6 × 10 −10 × 0.56 22.4 3RT
78. (3) = n× × 273 ⇒ n = 2.5 × 10 −14 urms =
760 273 M
Therefore, number of atoms = 2.5 × 10 −14 × 2 × 6 × 1023 = 3 × 1010 3RT
3 R=
−14 23 10 M
2.5 × 10 × 2 × 6 × 10 = 3 × 10
3RT
r ( O2 ) T 64 T or 9R =
79. (4) = 1.6 = × ⇒ 1.6 = M
r (SO2 ) 32 400 200 3T 3 × 300
or M = = = 100 kg mol−1 = 105 g mol−1
T 9 9
1.6 × 1.6 = ⇒ 2.56 × 200 = T ⇒ 128 × 4 = T
200
Hence, x = 5
80. (4) Number of moles is given by
pV rHe MCH4 n 16 4
n= 85. (8) For effusion = × He = × = 8 :1
ZRT rCH4 MHe nCH4 4 1
1× 800
n330 K = 86. (7) For the first bulb
1.9 × R × 330
p1V1 = n1RT
V × 200 For the second bulb,
n570 K =
1.1× R × 570 p2V2 = n2RT
By equalizing these two, we get V = 4 L
On adding: p1V1 + p2V2 = (n1 + n2 )RT (1)
p w M
81. (7) 1 = 1 × 2 When the stopcock is opened, the total volume is (V1 + V2)
p2 M1 w2
and total number of moles is (n1 + n2). Suppose the equilib-
1.0 22 28 rium pressure at each bulb is p atm. Then,
= × ⇒ w2 = 7 g
0.5 44 w2 p (V1 + V2 ) = (n1 + n2 ) RT (2)
From Eqs. (1) and (2), we have
3
82. (1) Translational K .E. = RT ( for 1 mol). It depends only on
2 p1V1 + p2V2 = p(V1 + V2 ) (3)
temperature.
Given that, p1 = 9 atm, V1 = 5 L, p2 = 6 atm, V2 = 10 L.
83. (2) Statement (a) and (c) are correct.
Substituting the given values in Eq. (3), we get

 a 9 × 5 + 6 × 10 = p × 15
For satatement (a):  P + 2  (V − b ) = RT
 V  p = 7 atm
pV  pb 
Therefore, Z = =  1+  87. (5) According to kinetic energy of gaseous molecules
RT  RT 
1
For satatement (b): a is more for C6H6 due to high molecular K.E. total = m(urms )2 × N
mass. 2
= 5 kJ
For satatement (c): Internal energy of an ideal gas depends
on temperature only. Since, they have

Chapter-5.indd 151 8/8/2016 2:39:00 PM


152 Chapter 5 States of Matter

SOLVED JEE 2016 QUESTIONS


JEE Main 2016 3. Two closed bulbs of equal volume (V) containing an ideal
gas initially at pressure pi and temperature T1 are connected
1. At very high pressures, the compressibility factor of one mole
through a narrow tube of negligible volume as shown in the
of a gas is given by
figure below. The temperature of one of the bulbs is then raised
pb pb pb b to T2. The final pressure pf is
(a) 1+ (b) (c) 1− (d) 1−
RT RT RT (VRT ) T1 T1 T1 T1
(Online) p i, V p i, V p f, V p f, V

Solution

   T1T2 
(a) According to van der Waals equation, for one mole of a gas (a) 2pi T1T2 (b) pi 
 T + T   T1 + T2 
1 2
 a
 p + 2  (V − b ) = RT (1)  T1   T 
V (c) 2pi  (d) 2pi  2 
 T1 + T2   T1 + T2 
At high pressure, a can be neglected
V2 (Offline)
Solution
a
So, p+ ≈p (2) (d) Applying combined gas law (for both initial and final
V2
states)
From Eqs. (1) and (2), we get (Initial moles) n1 = n2 (Final moles)
p(V − b) = RT pV p × V pf × V pf × V
i
pV − pb = RT + i = +
RT1 RT1 RT2 RT1
pV = RT + pb
V  pi pi  V  pf pf 
Dividing both the sides by RT + = +
R  T1 T1  R  T2 T1 
pV pb
= 1+ 2 pi  1 1
RT RT = pf  + 
T1  T2 T1 
pb
Z = 1+
RT 2 pi T +T 
= pf  1 2 
T1  T1T2 
2. Initially, the root mean square (rms) velocity of N2 molecules
at certain temperature is u. If this temperature is doubled and  T 
all the nitrogen molecules dissociate into nitrogen atoms, then pf = 2 pi  2 
 T1 + T2 
the new rms velocity will be
(a) 2u. (b) 14u. (c) 4u. (d) u/2.
JEE Advanced 2016
(Online)
Solution 1. The diffusion coefficient of an ideal gas is proportional to its
3RT mean free path and mean speed. The absolute temperature of
(a) Root mean square (rms) velocity of N2 molecule, u = an ideal gas is increased 4 times and its pressure is increased 2
M
For N2 molecule, molecular mass, M = 28 times. As a result, the diffusion coefficient of this gas increases
x times. The value of x is
Therefore, u= 3RT (1)
28 Solution
(4) Given that the diffusion coefficient (D) is directly propor-
When temperature becomes doubled 2T
tional to mean free path (l) and mean free speed (v).
Let the new root mean square (rms) velocity of N atom be u′
Thus, D ∝ l and D ∝ v
For N atom, molecular mass, M = 14
D∝l×v (1)
3RT
Therefore, u′ = (2) We know that the mean free path is inversely proportional
14
to the size of molecules, pressure and density of the gas. It
On dividing Eq. (1) by Eq. (2), we get increases with increase in temperature. Therefore,

u 1 T
= l∝ (2)
u′ 4 p
1 The mean free speed increases with increase in tempera-
= ⇒ u′ = 2u
2 ture and is inversely proportional to the molecular mass of

Chapter-5.indd 152 8/8/2016 2:39:05 PM


I II

Surface tension

Surface tension
Solved JEE 2016 Questions 153
Concentration Concentration
the molecules. Therefore, from the expressions for differ- III
ent molecular speeds, we have

Surface tension
1/ 2
T (3)
v ∝ 
 M

From Eqs. (1), (2) and (3), we have

1/ 2
T T T 3/ 2 (4)
D∝ ×   = Concentration
p  M p( M )1/ 2
(a) I : KCl II : CH3OH III : CH3(CH2)11OSO3−Na+
Let the new temperature be T ′ = 4T and the new pressure (b) I : CH3(CH2)11OSO3−Na+ II : CH3OH III : KCl
p′ = 2p. Then from Eq. (4), we have the new diffusion coef- (c) I : KCl II : CH3(CH2)11OSO3−Na+ III : CH3OH
ficient as (d) I : CH3OH II : KCl III : CH3(CH2)11OSO3−Na+
(T ’)3 / 2 ( 4T )3 / 2 (5) Solution
D’ ∝ 1/ 2
=
p ’( M ) 2 p( M )1/ 2
(d) The surface tension of water is large because of strong
On dividing Eq. (5) by Eq. (4), we get intermolecular interactions. On addition of solutes the sur-
face tension changes. Compounds that have a tendency to
D ’ [( 4T )3 / 2 / 2 p( M )1/ 2 4
= = concentrate on the surface of the liquids as compared to
D [(T )3 / 2 / p( M )1/ 2 1
its bulk are found to lower the surface tension. Thus,
2. The following qualitative sketches I, II and III show the varia- • Substance like detergents, soaps (CH3(CH2)11SO3−Na+)
tion of surface tension with molar concentration of three differ- decrease the surface tension sharply due to hydropho-
ent aqueous solutions of KCl, CH3OH and CH3(CH2)11OSO3−Na+ bic interaction (Graph III).
at room temperature. The correct assignment of the sketches is • Substances like alcohols (CH3OH, C2H5OH) lower the
I II surface tension slightly as they have smaller dielectric
constant. Dielectric constant is directly proportional to
Surface tension

Surface tension

the surface tension,so, on addition of CH3OH to water,


the overall dielectric constant of the solution decreases
and thus surface tension decreases (Graph I).
• Inorganic impurities like KCl present in the bulk of a
liquid increase the intermolecular forces and thus are
likely to increase the surface tension of water (Graph II).
Concentration Concentration
III
Surface tension

Concentration

Chapter-5.indd 153 8/8/2016 2:39:07 PM


Chapter-5.indd 154 8/8/2016 2:39:07 PM
6 Thermodynamics

Question Distribution in JEE (Main and Advanced)

3
No. of Questions

JEE (Main)
2
JEE (Adv)

0
2016 2015 2014 2013 2012 2011 2010 2009 2008 2007

Concept Distribution in JEE (Main and Advanced)

Topics Covered
Year
JEE (Main) JEE (Advanced)
2007 First Law of Thermodynamics, Gibb’s Free Energy Gibb’s Free Energy
2008 Enthalpy, Gibb’s Free Energy Second Law of Thermodynamics, Gibb’s Free Energy
2009 Thermochemistry, Gibb’s Free Energy Fundamental of Thermodynamics, Enthalpy
Fundamental of Thermodynamics, Pressure-Volume
2010 Thermochemistry, Gibb’s Free Energy
Work, Enthalpy
2011 Entropy Enthalpy
2012 Gibb’s Free Energy Entropy, Pressure-Volume Work
2013 First Law of Thermodynamics Thermochemistry, Pressure-Volume Work
2014 Enthalpy, Thermochemistry, Entropy Pressure-Volume Work, Entropy
2015 Gibb’s Free Energy Entropy, Pressure-Volume Work, Gibb’s Free Energy
2016 Enthalpy, Gibb’s Free Energy, Pressure Volume Work Entropy

Chapter-6.indd 155 8/4/2016 10:31:57 AM


156 Chapter 6 Thermodynamics

SUMMARY
1. The study of flow of heat into or out of a system during its physical or chemical transformation is known as thermodynamics.
2. Fundamentals of thermodynamics
(a) System
It is any region of space which is under thermodynamic investigation. Depending on the movement of matter and energy, the
system can be of the following types.

Closed system In such a system, matter can neither be added nor removed, since Energy
the boundary of the system is sealed. The system is however, not
insulated. In such a system, matter cannot be transferred but energy
can be transformed in the form of heat or work to and from the Closed
surroundings. system

Open system In such a system, matter can be added to or removed from the Energy, matter
system, since the boundary is open. The system is also not insulated.
In such a system, both energy and matter can be transferred to the
system and from the surroundings to the system. Open
system

Isolated system In such a system, neither energy nor matter can be transferred to or Energy, matter
from the surroundings, since the boundary is both sealed and insu-
lated. Such a system has no interaction with the surroundings.
Isolated
system

(b) Surrounding
It comprises the rest of the universe apart from the system. Usually, the region in space in the neighborhood of the system con-
stitutes the surroundings.
(c) Universe
It comprises the system and the surroundings together.
(d) Boundary
A wall or layer separating the system from the surroundings is called boundary.

Surrounding
System Boundary

(e) Thermodynamic properties


(i) Intensive properties: Properties which are independent of the quantity of matter present in the system. For example, tem-
perature, pressure, density, concentration, etc.
(ii) Extensive properties: Properties that depend on the quantity of matter present in the system. For example, volume,
number of moles, entropy, Gibbs free energy, enthalpy, etc. These properties are additive, while intensive properties
are not.
(iii) State of the system: Set of variables (such as temperature, pressure, composition, etc) that describe the system is
known as the state of the system. If any of these variables undergo a change, the system is said to undergo change of
state.
(iv) State function: Any property of the system that defines the state of the system, but is independent of the path that brings
about that change in state is known as state function. For example, pressure (p), temperature (T), volume (V) and composi-
tion or amount (n) are called state variables.
(v) Path functions: Any property of the system that depends on the initial and final states of the system as well as the
path that brings about that change in state is known as path function. For example work done (w) and heat (q) are path
functions.

Chapter-6.indd 156 8/4/2016 10:31:58 AM


Summary 157

Tip
Difference between state function and path function:
State function Path function
Independent of path taken to establish property or value. Dependent on path taken to establish property or value.
Can integrate using final and initial values. Need multiple integrals and limits of integration in order
to integrate.
Multiple steps result in same value. Multiple steps result in different value.
Based on established state of systems Based on how state of system was established.
(temperature, pressure, amount and identity of systems).

Normally represented by an uppercase letter. Normally represented by a lowercase letter.

(vi) Perfect differential: If we have any single-valued variable (property) z, such that its value at any point of time
is determined by any two variables (properties) x and y of the system at that point in a given state, then z is a perfect
differential if:
• The change dz between any two states is independent of the path by which the change is done, that is, it is a state
function.
• The value of dz for a cyclic process is zero.
• The second differential of z with respect to x and y, carried out in any order is the same.
(f) Thermodynamic processes
A thermodynamic process is said to occur when the state of a system changes from one state (initial state) to another (final state).
Various thermodynamic processes are:

Process Condition
Constant temperature is maintained by the exchange
Isothermal Constant temperature, dt = 0
of heat with the thermostat.
The system is thermally insulated from the surround-
Adiabatic Constant heat, dq = 0 ings. However, the temperature of the system may
increase or decrease.
Isobaric Constant pressure, dp = 0 Volume change always takes place.
All reactions carried out at atmospheric pressure are
Isochoric Volume of the system is kept constant (dV = 0)
examples of isobaric process.
The net work done is zero in a cyclic process, w = 0.
Cyclic Initial and final states are identical
Also, dU = 0 and dH = 0.
Energy change in each step of the process can be The change must be performed at an infinitesimal
Reversible reversed in direction by making a small change in slow rate. There must be no loss of energy due to
any property of the system, such as temperature, friction and no finite temperature differences.
pressure, etc.
The system or surroundings are not restored to their They always tend to proceed in a definite direction;
Irreversible initial state at the end of the process. All processes and do not proceed in the opposite direction without
occurring spontaneously in nature are irreversible. the action of an external force.

(g) The zeroth law of thermodynamics


“Two systems individually in thermal equilibrium with a third system are in thermal equilibrium with each other”. The zeroth law
establishes temperature as the indicator of thermal equilibrium.
(h) Energy, work and heat
(i) Internal energy
• The total energy contained in a thermodynamic system is called internal energy (U).
• The change in internal energy is given by ∆U = Ufinal − Uinitial .
• The SI unit of internal energy is joule (J).
• It can also be expressed in calories, where 1 cal = 4.184 J.

Chapter-6.indd 157 8/4/2016 10:31:59 AM


158 Chapter 6 Thermodynamics

(ii) Work
• The energy of the system is its capacity to do work and the system may transfer energy to another system or surroundings
through work.
• When work (w) is done by the system, its energy decreases; and when work is done on the system, its energy increases.
• Internal energy can also be defined as a state function, where change from one state (U1) to another (U2) can be brought
about by doing adiabatic work on the system.
∆U = U2 − U1 = w adiabatic
• Work may also be in the form of expansion or compression of system known as the pressure-volume work. Work done
by gas in the system is equal to the change in volume against pressure p.
w = p∆V
(iii) Heat
• The internal energy of the system can also be changed by supplying heat to the system or removing heat from it.
∆U = q
• The amount of heat transferred (q) is proportional to the difference in temperatures.
• The quantity of heat is also expressed in calories. 1 cal is defined as the amount of heat required to raise the temperature
of 1 g of water by 1°C at 15 °C.

Tip
Work and heat conventions:
q is (+) Heat is absorbed by the system. q is (−) Heat is released by the system.
w is (+) Work is done on the system. w is (−) Work is done by the system.

3. The first law of thermodynamics


The first law of thermodynamics states that energy can neither be created nor destroyed, that is, the total energy of the system
remains constant, though it may change from one form to another.
Mathematical expressions for the first law of thermodynamics: ∆U = q + w
4. Pressure volume work
If the process take place in a manner that pressure at each stage decreases by an infinitesimal amount dp and the volume increases
by an infinitesimal amount dV, then the total work done by the gas can be obtained by the relation
Vf
w = ∫ pext dV
Vi
Vf
Work done by the gas wrev = 2.303nRT log
Vi
Vf
Work done on the gas wrev = − 2.303nRT log
Vi

Tip
(a) For an isothermal expansion, Vf > Vi, work done, w, by an ideal gas is positive.
(b) For a compression, Vf < Vi, work done, w, by an ideal gas is negative.

(a) Expression of first law for different thermodynamic processes


(i) For an isochoric process: w = −pext ∆V = 0; U = qV
(ii) For reversible and irreversible isothermal processes
• For irreversible change, q = − w = pext (Vf − Vi )
Vf V
• For reversible change, q = − w = nRT ln = 2.303nRT log f
Vi Vi
(iii) For an adiabatic process: In this process q = 0, so we get ∆U = w.
(iv) Free expansion
• The expansion of a gas in vacuum is called free expansion and during the process, no work is done on or by the system,
irrespective of whether the process is reversible or irreversible.

Chapter-6.indd 158 8/4/2016 10:32:01 AM


Summary 159

• When an ideal gas undergoes free expansion under isothermal conditions, no change in internal energy takes place
because no work is done and no heat is exchanged (q = 0).

5. Enthalpy or heat content (H)


The enthalpy of a system may be defined as the sum of the internal energy and the product of its pressure and volume. It is denoted
by the symbol H
∆H = ∆U + w
∆H = ∆U + p∆V = ∆U + ∆ngRT
where ∆n is the difference between stoichiometric coefficients of all gaseous products and all gaseous reactants.
6. Joule–Thomson effect
(a) When real gases are allowed to expand into vacuum or low-pressure region, the temperature of the gas is lowered. This effect is
known as Joule–Thomson effect.
(b) The rate of change of temperature with pressure at constant enthalpy is called Joule–Thomson coefficient and is represented
by m.
 ∂T 
m= 
 ∂p  H
(c) The pressure at which gases neither cool nor warm up on expansion, that is, m = 0 is called the inversion point.
(d) For a given pressure, the temperature at which a gas on passage from higher to lower pressure does not undergo heating or
cooling (m = 0) is known as inversion temperature.
(e) At constant pressure, each gas shows two inversion temperatures, the upper and lower one, between which it undergoes cooling
on expansion and beyond which it undergoes heating.
7. Heat capacity
Heat capacity of a system refers to its capacity to absorb and store heat. Heat capacity can be determined by the rate of change of
dq
heat with temperature C = .
dT
 ∂U 
Heat capacity at constant volume, CV =  
 ∂T  V
 ∂H 
Heat capacity at constant pressure, C p =  
 ∂T  p

(a) Molar heat capacity (Cm)


(i) Amount of heat required to raise the temperature of one mole of the substance by 1°C (or K).
(ii) Molar heat capacity equals the specific heat times the molar mass and has units of J mol−1 °C−1.
(iii) The molar heat capacity at constant volume and constant pressure are expressed as Cp,m and CV, m respectively.
(b) Specific heat or specific heat capacity
(i) Heat required to raise the temperature of a unit mass by 1°C (or K), at a specified temperature.
(ii) The amount of heat required to bring about a temperature change (∆T) in mass m of a substance with specific heat c, can be
obtained by the relation q = c × m × ∆T
(c) The relation between Cp and CV for an ideal gas: C p − CV = R
For solids and liquids, there is no volume change, so ∆V = 0. ∆H = ∆U , therefore, Cp = CV.
(d) Ratio of heat capacities
The ratio of heat capacities is given by g = Cp/CV. It depends on the atomicity of the gas.

Atomicity of gas CV Cp(= CV + R) f = Cp/CV

3 5 (5/2)R
Monoatomic R R = 1.66
2 2 (3/2)R

5 7 (7/2)R
Diatomic R R = 1.40
2 2 (5/2)R

4R
Triatomic 3R 4R = 1.33
3R

Chapter-6.indd 159 8/4/2016 10:32:05 AM


160 Chapter 6 Thermodynamics

n1(CV )1 + n2 (CV )2
(e) Heat capacity for a mixture of gases: CVmixture =
n1 + n2
8. Expansion and compression of an ideal gas
(a) Reversible isothermal expansion or compression
(i) Since, U depends only on temperature, therefore, ∆U = 0. Using this first law of thermodynamics become q = −w
V2
dV V
(ii) The expression for work involved is w = − nRT ∫ = − nRT ln 2
V1
V V1
p2
(iii) In terms of pressure, w = − nRT ln
p1

(b) Reversible adiabatic expansion or compression


(i) Heat is neither allowed to enter nor leave the system. Therefore, q = 0.
nR
(ii) w = ∆U = nCV , m (T2 − T1) = − (T2 − T1); ∆H = nC p , m (T2 − T1)
g −1
where CV,m is the molar heat capacity at constant volume.

(iii) Since, dU = dw , ∆U = nCV , mdT and dw = − popp dV therefore

nCV , mdT = − popp dV .


For a reversible process, popp = pint ± dp ≈ pint
For an ideal gas, pint = (nRT/V)
RT
or CV , m dT = − dV
V
T2 V
dT 2
dV
∫ CV , m T
=−∫R
V
T1 V 1

CV , m −R R
 T2  V  V 
or  T  =  2 =  1
1  V1   V2 
R / CV , m CV , m / R
TV = constant, T V = constant

or TV g −1 = constant where g = Cp,m/CV,m

− R / C p ,m − C p ,m / R
(c) Relationship between T and p: Tp = constant, T p = constant, Tp(1−g )/g = constant
g (1−g )
Therefore, T p = constant.
C V ,m
 pV 
(d) Relationship between p and V:   VR = constant
 R 
C p ,m
pCV , mV = constant

pV g = constant, whereg = Cp,m/CV,m


(e) Irreversible isothermal expansion or compression
q = −w = popp (V2 − V1) and ∆U = ∆H = 0
(f) Irreversible adiabatic expansion or compression
(i) Free expansion: dw = −poppdV = 0.
For finite change we will have w = 0, ∆U = 0, ∆T = 0, ∆H = 0.

Tip Adiabatic irreversible free expansion of an ideal gas is identical with the isothermal free expansion.

(ii) Intermediate expansion or compression


w = ∆U = − popp (V2 − V1)
For an ideal gas, ∆U is given by ∆U = nCV , m ∆T = nCV , m (T2 − T1)

Chapter-6.indd 160 8/4/2016 10:32:10 AM


Summary 161

The expression for ∆H is ∆H = nC p , m ∆T = nC p , m (T2 − T1)


Eliminating ∆T from above equation, we get

∆V nC p , m
∆H = nC p , m = ∆V = g ∆V
nCV , m nCV , m

C + Rp2 p1 
T2 = T1  V , m 
 C p,m 

where popp = p2 (the pressure of the gas after expansion).


(g) Isobaric processes
For an isobaric process, the pressure (p) remains constant, when volume changes from V1 to V2 at temperatures T1 and T2.

w = p(V2 − V1) = p∆V


(h) Isochoric processes
For an isochoric process, volume remains constant, therefore dV = 0 ⇒ w = pdV = 0
9. Calorimetry
(a) The amount of heat absorbed or released in a chemical reaction (heat of reaction) or a physical process can be measured by a
technique called calorimetry.
(b) The heat of reaction at constant volume (qv) is measured in a bomb calorimeter, ∆U = qV
(c) An apparatus to measure heat change at constant pressure (qp) is a calorimeter, qp = C p ∆T = ∆H
10. Thermochemistry
The study of energy or heat changes accompanying a chemical reaction or a physical change is called thermochemistry. Reactions
which involve absorption of heat are called endothermic reactions, whereas reactions involving release of heat are called exother-
mic reactions.
11. Enthalpy changes during phase transformations
(a) The enthalpy changes accompanying different phase changes are described as follows:

Enthalpy change Definition Phase change

Enthalpy change of the system when one mole of a solid sub- H2O(s) → H2O(l)
Enthalpy of fusion
stance is converted into the liquid state at its melting point. ∆Hfus = +1.43 kcal

Enthalpy change of the system when one mole of liquid is con- H2O(l) → H2O(g)
Enthalpy of vapourization
verted into vapour or gaseous state at its boiling point. ∆Hvap = +9.71 kcal

Enthalpy change when one mole of a solid is directly converted I2 (s) → I2 (g)
Enthalpy of sublimation
into the gaseous form at a temperature below its melting point. ∆Hsub = +14.92 kcal

(b) Enthalpy of reaction


(i) ∆ r H = Enthalpies of products − Enthalpies of reactants
= ∑ ai Hproducts − ∑ bi Hreactants
i i

(ii) The enthalpy of reaction varies with the temperature.


(iii) When Hproducts > Hreactants, ∆H is positive; Reaction will be endothermic.
(iv) When Hreactants > Hproducts, ∆H is negative, Reaction will be exothermic.
(v) The heat change for a reaction taking place at standard conditions of temperature and pressure, that is, 298 K and 1 bar is
called standard heat of reaction or standard enthalpy change (∆H°). All the participating substances (reactants and prod-
ucts) are in their standard states.

1
CO(g) + O2 (g) → CO2 (g), ∆H° = −284.5 k J
2

Chapter-6.indd 161 8/4/2016 10:32:13 AM


162 Chapter 6 Thermodynamics

(vi) Factor affecting enthalpy of reaction


• Heat of reaction will vary when the reactants are in different physical states.
• The heat of reaction varies with the temperature at which it is determined and the variation in the heat of reaction with
temperature is given by Kirchhoff’s equation.
H2 T2

∫ d( ∆H ) = ∫ ∆C p dT
H1 T1

( ∆H2 − ∆H1) = ∆C p (T2 − T1)

Similarly, for reactions at constant volume, the relation can be expressed for change in internal energy with temperature as

( ∆U2 − ∆U1) = ∆CV (T2 − T1) .

(b) Enthalpy of formation


It is the enthalpy change of a system when one mole of compound is formed from its constituent elements. It is represented
as ∆f H°.
(c) Relation between enthalpy of formation and enthalpy of reaction
∆ r H ° = n∆ f H °

where n is number of moles of a substance formed from its constituent elements.

Tip All values of ∆ f H° for the elements in their standard states are zero.

(d) Enthalpies of different types of reactions

Enthalpy Definition Example


Enthalpy change of a system when 1 g -equiv. of an acid
Enthalpy of neutrali- HCl(aq) + NaOH(aq) → NaCl(aq) + H2O(l)
is neutralized by 1 g-equiv. of a base (or vice versa) in a
zation
dilute solution. ∆H = −13 .7 kcal mol−1

Burning of carbon in presence of excess


Standard enthalpy of It is the enthalpy change when one mole of the sub- oxygen to give CO2
combustion (∆cH°) stance is completely burnt in excess of air. C(graphite) + O2(g) → CO2(g)
∆H = −94.05 kcal mol−1
It is the amount of energy needed to break all the chem- CH4 (g) → C(g) + 4H(g)
Enthalpy of atomization
ical bonds in 1 mol of molecules to give gaseous atoms
(∆aH°) ∆ aH° = 1665 kJ mol−1
as products.
Enthalpy of solution The total energy absorbed or released when a solute dis-
(∆solH°) solves in a solvent at constant pressure to make a solution.

(e) Bond enthalpy and bond dissociation enthalpies


(i) Bond enthalpy (∆bondH°) is the amount of energy required to break all the bonds present in one mole of a compound.
Likewise, the energy liberated when a bond is formed between two atoms to form one mole of a compound is also called
bond enthalpy.
(ii) Bond enthalpy depends on many factors, such as electronegativity of the atoms, size of the atoms, bond length, etc.
(iii) Bond breaking is an endothermic process, and value of bond dissociation enthalpy is positive
(iv) Bond formation is an exothermic process, and value of bond formation enthalpy is negative.
(v) For all diatomic molecules, the bond dissociation enthalpy is same as the enthalpy of atomization.
(vi) For complex polyatomic molecules, the bond dissociation enthalpy for different bonds is different within the same
molecule.
(vii) The bond enthalpies can be used for
• Determination of enthalpy of reaction
• Determination of enthalpy of formation
• Determination of resonance energy
(f) Lattice enthalpy
The lattice enthalpy is the energy required to completely separate the ions in one mole of a solid compound from each other to
form a cloud of gaseous ions.

Chapter-6.indd 162 8/4/2016 10:32:14 AM


Summary 163

12. Laws of thermochemistry


(a) Lavoisier–Laplace law
It states that the enthalpy change accompanying a chemical process is same in magnitude but opposite in sign to the enthalpy
change accompanying the same process in the reverse direction. This is also known as the first law of thermochemistry.
(b) Hess’s law of constant heat summation
(i) The heat absorbed or liberated in a given chemical equation is always constant and is the same whether the process occurs
in one step or in several steps.
(ii) Enthalpy of overall reaction is ∆rH.
(iii) If the reaction can also take place in n number of steps, with reaction enthalpy ∆rH1, ∆rH2, …, ∆rHn, then,
∆ r H = ∆ r H1 + ∆ r H2 … ∆ r Hn

Tip
• When an equation is reversed—written in the opposite direction—the sign of ∆H° must also be reversed.
• Formulas cancelled from both sides of an equation must be for the substance in identical physical states.
• If all the coefficients of an equation are multiplied or divided by the same factor, the value of ∆H° must likewise be multiplied
or divided by that factor.

(c) Born–Haber cycle


A set of alternate paths from the free elements to the solid ionic compound.
13. Entropy
(a) It represents the state of disorderliness or randomness of a system. Entropy is a state function and depends only on the state of
the system. Entropy change, DS, is independent of the path from start to finish.
∆S = Sfinal − Sinitial

For a chemical system, ∆S = Sproducts − Sreactants

Tip When Sfinal > Sinitial (or Sproducts > Sreactants), ∆S is positive.

(b) Disorder in particles in a system is directly proportional to the heat (q), and inversely proportional to the temperature (T).

qrev
∆S =
T
(c) When heat is absorbed, q is positive and ∆S is also positive indicating an increase in entropy of the system.
(d) In case of an isolated system, total entropy change is considered which is the sum total of entropy of system and
surroundings.
∆Stotal = ∆Ssystem + ∆Ssurrondings

(e) For a change to be spontaneous, the final state should have greater randomness, and hence greater entropy than the initial
state, therefore, ∆Stotal > 0.
(f) For a system undergoing reversible change, ∆S = 0.
(g) If the system undergoes an irreversible change, then in absence of any change in internal energy or volume, all spontaneous
processes will result in increase in entropy.
(h) Entropy change in reversible and irreversible processes
(i) For all processes carried out under isothermal and reversible conditions in an isolated system, the total entropy change is
zero.
∆Srev = ∆S1 + ∆S2 = 0

(ii) For an irreversible process occurring isothermally in an isolated system, an increase in the total entropy of the system is
observed.
∆Sirrev > 0 (as ∆S1 > 0 )

(i) Factors affecting entropy


(i) For gases, the entropy increases with increasing volume.
(ii) The higher the temperature, the larger is the entropy.
(iii) A gas has a much larger entropy compared to a liquid or solid.
(iv) As the number of particles increases, the entropy increases.

Chapter-6.indd 163 8/4/2016 10:32:17 AM


164 Chapter 6 Thermodynamics

(j) Entropy of physical and chemical changes


Entropy of Entropy of fusion It is the increase in entropy when a solid substance ∆Hfusion
physical melts at its melting point. It is always a positive ∆Sfusion = Sliquid − Ssolid =
Tm
changes value.
(Tm is the melting point of the solid)
Entropy of It is the increase in entropy when a liquid substance ∆Hvap
vapourization vapourizes at its boiling point. It is always a positive ∆Svap = Svapor − Sliquid =
Tb
value.
(Tb is the boiling point of the liquid)
Entropy of subli- It is the increase in entropy when a solid substance ∆Hsub
mation converts directly into vapour at any temperature. It ∆Ssub = Svapor − Ssolid =
T
is always a positive quantity
Entropy of chemical changes The entropy change accompanying a chemical ∆S = ∑ Sproducts − ∑ Sreactants
reaction is given by the difference between the sum
of entropies of all products and sum of entropies of
all reactants

(k) Standard entropy of a substance: The entropy of a substance at 25°C (298 K) and 1 atm pressure. It is represented as S°.
∆S ° = ∑ Sproducts
° − ∑ Sreactants
°

14. Fundamental equations


(a) From first and second law we have, dU = dq − pextdV and dqrev = TdS
For a reversible process, pext = p and dq = dqrev = TdS, therefore, dU = TdS − pdV.

 ∂U   ∂U 
  = T ;   = − p
∂S V ∂V S

(b) Differentiating the equation H = U + pV on both the sides we get dH = dU + pdV + Vdp
Using dU = TdS – pdV, we get dH = TdS + Vdp
 ∂S  1  ∂H  Cp
  = ;   =
∂H p T ∂S p T

15. Gibbs energy and spontaneity


(a) Spontaneity
Processes which occur on their own accord are called spontaneous processes. These do not necessarily take place instantane-
ously, but are favourable and feasible.
(b) Gibbs energy of a system at any moment in time is defined as the enthalpy of the system minus the product of the temperature
times the entropy of the system. It is a state function and has a unit of energy.
G = H − TS
(c) Change in Gibbs energy can be expressed as
∆Gsystem = ∆Hsystem − T ∆Ssystem
(d) Reactions that occur with a Gibbs energy decrease are said to be exergonic. Those that occur with a Gibbs energy increase are
said to be endergonic.
(e) Decrease in Gibbs energy during a thermodynamic process is equal to the useful work obtainable from the system.
−(∆G )p ,T = wuseful work
(f) The spontaneity of a process can be predicted by taking two factors together i.e. enthalpy and entropy.
The equation ∆G = ∆H − T∆S takes both the factors into consideration.
(i) If ∆Gsystem < 0, the reaction is spontaneous.
(ii) If ∆Gsystem > 0, the reaction is non-spontaneous.
(iii) If ∆Gsystem = 0, the reaction is at equilibrium.

DH DS DG Spontaneity
−ve +ve Always −ve Reaction is spontaneous
+ve −ve Always +ve Reaction is non-spontaneous

Chapter-6.indd 164 8/4/2016 10:32:19 AM


Summary 165

At low temperature, ∆G = +ve Non-spontaneous


+ve +ve
At high temperature, ∆G = −ve Spontaneous
At low temperature, ∆G = −ve Spontaneous
−ve −ve
At high temperature, ∆G = +ve Non-spontaneous

(g) Standard Gibbs energy change (DGç)


The Gibbs energy change for a system when the process is carried out at 25°C (298 K) and 1 atm pressure.
°
∆ G ° = ΣGproducts °
− ΣGreactants
(h) Relationship between DrG and DGç
∆ rG = ∆G ° + RT ln Q

where R is the gas constant (8.314 J mol−1 K−1), T is the Kelvin temperature, and ln Q is the natural logarithm of the reaction
quotient.
At equilibrium, ∆rG = 0 and Q = K,
∆ rG ° = − RT ln K = − 2.303RT log K

(i) Relation between equilibrium constant to enthalpy change and entropy

∆ rG ° = ∆ r H ° − T ∆ r S ° = − RT ln K = − 2.303RT log K

∆rH° is large and Equilibrium constant Reaction does not proceed to a great extent
Endothermic reactions
positive K 1 towards formation of products
∆rH° is large and Equilibrium constant
Exothermic reactions Reaction would proceed towards completion
negative K1

Tip The value for ∆rGο also depends on the changes of entropy (∆rSο) accompanying the reaction, so the value of equilibrium con-
stant will be affected by whether the entropy change is positive or negative.

16. Chemical potential of multicomponent system


(a) In a multicomponent system, change in free energy (dG) at constant temperature and pressure may be expressed in terms of
partial molar free energy (G i ) and number of moles of component n as

(dG )T ,p = ∑ G i dni (i = number of constituents)


i

(b) When temperature and pressure are not constant, free energy change is given by

dG = Vdp − SdT + ∑ G i dni


i

(c) The partial molar free energy was also called chemical potential by Gibbs and represented as m, therefore, free energy
change is
dG = Vdp − SdT + ∑ mi dni
i
(d) Gibbs–Duhem equation
(i) It gives the contribution of chemical potential of each component of the mixture per mole to the total free energy of the
system at constant temperature and pressure.
(ii) When a small amount of substance i (dni ) is added to the system, keeping other variables constant, the change in free energy
with change in composition is given by
(dG )T , p = ∑ mi dni
i

(e) The variation of chemical potential of component i of a multicomponent system, as a function of temperature is related to par-
tial molar entropy of the component by the expression
 ∂mi 
  = − Si
∂T  p ,n
i

Chapter-6.indd 165 8/4/2016 10:32:23 AM


166 Chapter 6 Thermodynamics

(f) The variation of chemical potential of the component i of a multicomponent system, as a function of pressure is related to par-
tial molar volume of the component by the expression
 ∂mi 
 ∂p  = − Vi
T ,n
i

17. vant Hoff reaction isotherm and equations


(a) vant Hoff equation
The variation in equilibrium constant K with temperature is given
∆G ° = − RT ln K

∆G °
lnK = −
RT
Differentiating with respect to T, we get
∂ ln K 1  ∂( ∆G / T ) 
=−  
∂T R  ∂T 
1  − ∆H ° 
= −  2  (by Gibbs− Helmholttz equation)
R T 
∆H
= (as ∆H does not vary much with pressure, ∆H ° = ∆H )
RT 2

This equation is another form as van′t Hoff equation. Integrating this equation, we get

− ∆H
log K = + Constant
2.303RT

If integration is carried out between the temperatures T1 and T2, we get another useful form of van′t Hoff equation.
K  − ∆H  T2 − T1 
log  2  =
 K1  2.303R  T1T2 
(b) vant Hoff isochore
The variation in equilibrium constant K with pressure is given.
∂ lnK ∆E
At constant pressure, = 2
∂T RT

18. Second law of thermodynamics


(a) It states that whenever a spontaneous event takes place in our universe, the total entropy of the universe increases (∆Stotal > 0).

Tip All natural processes are spontaneous and irreversible, thus, they are accompanied by a net increase in entropy.

(b) Another statement for second law of thermodynamics:


The entropy of the universe tends always to a maximum.

∆Suniverse = ∆Ssystem + ∆Ssurroundings

The spontaneity of the process can be predicted on the basis of the following
(i) If ∆Suniverse > 0, the reaction is spontaneous.
(ii) If ∆Suniverse < 0, the reaction is non-spontaneous.
(iii) If ∆Suniverse = 0, the reaction is at equilibrium.
(c) Carnot cycle
(i) It is the ideal cycle of operation of a heat engine, used to calculate the maximum extent to which heat can be converted to
work.
(ii) The direction of a particular spontaneous transformation can be ascertained with the help of the second law of
thermodynamics.
(iii) Carnot cycle was assumed to be a different reversible system operating between two temperatures. In one complete cycle,
it involves two isothermal and two adiabatic processes.

Chapter-6.indd 166 8/4/2016 10:32:25 AM


Summary 167

a (p1, V1, T1)


b (p , V , T )
2 2 2

d c
(p4, V4, T4) (p3, V3, T3)

0 V

Steps Change Work done Heat change


Step 1 Isothermal a (p1, V1, T1) to b (p2, V2, T2) V  Heat (q1) is absorbed by the gas
expansion w1 = m RT ln  2  from the reservoir at temperature
 V1  T1
Step 2 Adiabatic b (p2, V2, T2) to c (p3, V3, T3)  T −T  q= 0
expansion w2 = mR  1 2 
 g −1 
Step 3 Isothermal c (p3, V3, T3) to d (p4, V4, T4) V  Heat (q2) is released by the gas to
compression w3 = m RT ln  3  the reservoir at temperature T2.
 V2 

Step 4 Adiabatic com- d (p4, V4, T4) to a (p1, V1, T1)  T −T  q= 0


pression w 4 = mR  1 2 
 g −1 

(iv) Total work done during the complete one cycle is given by
w = w1 + w2 − w3 − w 4
V  V 
w = mRT1 ln  2  − mRT2 ln  3  .
 V1   V4 
(v) Now, step 2 is an adiabatic process, therefore,
T1V2g −1 = T2V3g −1
1/(g −1)
V2  T2 
= .
V3  T1 
Since step 4 is an adiabatic process, therefore,
1/(g −1)
V T 
T2V4g −1 = T1V1g −1 ⇒ 1 =  2
V4 T  1

Equating these two, we get,


V3 V2
=
V4 V1
(vi) Efficiency of the Carnot’s engine
w Q  T  ln(V3 / V4 )
h= = 1− 2 = 1−  2 
Q1 Q1  T1  ln(V2 / V1)
T2
or, h = 1−
T1
19. Third law of thermodynamics
(a) All substances in their normal crystalline state at the absolute zero temperature would be in the condition of maximum
orderly arrangement because all motion has essentially ceased at 0 K. In other words, entropy of a substance at 0 K is
minimum (tends to be zero).
(b) There are certain substances that possess entropy even when they are cooled close to absolute zero and this entropy is known
as residual entropy.

Chapter-6.indd 167 8/4/2016 10:32:29 AM


168 Chapter 6 Thermodynamics

TOOLS FOR PROBLEM SOLVING


1. Work for isothermal processes
(a) For irreversible change, q = − w = pext (Vf − Vi )
V V
(b) For reversible change, q = −w = nRT ln f = 2.303nRT log f
Vi Vi
2. Enthalpy or heat content (H): ∆H = ∆U + p∆V = ∆U + ∆ngRT
 îT 
3. Joule–Thomson coefficient: m = 
 î p  H
4. Heat capacity
 îU 
(a) At constant volume, CV =  
 îT V
 îH
(b) At constant pressure, C p = 
 î T  p
5. Variation of internal energy with temperature: dU = CVdT
6. Variation of enthalpy with temperature: dH = CPdT
7. Relation between Cp and CV for an ideal gas: C p − CV = R
8. Ratio of heat capacities g = Cp/CV
9. Work for adiabatic processes: w = ∆U = CVdT
10. Expansion or compression of ideal gas

Process Work done by system Heat into system Change in internal energy
Reversible isothermal V2 V2 ∆U = 0
dV V dV
expansion or com- w = − nRT ∫ = − nRT ln 2 q = nRT ∫
pression V1
V V1 V1
V
V2
= − nRT ln
V1
Reversible adiabatic w = nCV , m (T2 − T1) q=0 ∆U = nCV , m (T2 − T1)
expansion or com-
pression nR nR
=− (T2 − T1) =− (T2 − T1)
g −1 g −1

Irreversible isother- w = − popp (V2 − V1) q = popp (V2 − V1) ∆U = 0


mal expansion or
compression
Irreversible adiabatic w=0 q=0 ∆U = 0
expansion or com-
pression

CV , m −R R
T  V  V 
11. Temperature-volume relation for adiabatic change:  2  =  2 =  1
 T1   V1   V2 

TV g −1 = constant
where g = Cp,m/CV,m.
− R / C p ,m − C p ,m / R
12. Temperature-pressure relation for adiabatic change: Tp = constant, T p = constant, Tp(1−g )/g = constant

T g (1−g ) p = constant
C V ,m
 pV 
13. Pressure-volume relation for adiabatic change:   VR = constant
R
C p ,m
pCV , mV = constant

Chapter-6.indd 168 8/4/2016 10:32:35 AM


Solved Examples 169

pV g = constant
where g = Cp,m/CV,m.
H2 T2
14. Kirchhoff’s law for temperature dependence of enthalpies: ∫ d( ∆H ) = ∫ ∆C p dT
H1 T1

( ∆H2 − ∆H1) = ∆C p (T2 − T1)


qrev
15. Entropy change: ∆S =
T
16. Change in Gibb’s free energy: ∆Gsystem = ∆Hsystem − T ∆Ssystem
17. Relationship between DrG and DGç at equilibrium: ∆ rG ° = − RT ln K = −2.303RT log K
18. Relation between equilibrium constant to enthalpy change and entropy: ∆ rG ° = ∆r H ° − T ∆r S ° = − RT ln K = −2.303RT log K
19. Free energy change for a multicomponent system: dG = Vdp − SdT + ∑ mi dni
i
 ∂m 
(a) Variation of m as a function of temperature:  i  = − Si
 ∂ T  p ,n
i

 ∂m 
(b) Variation of m as a function of pressure:  i  = −Vi
 ∂ p  T ,n
i

20. van’t Hoff equation: The variation in equilibrium constant K with temperature
∆G ° = − RT ln K
K  − ∆H  T2 − T1 
log  2  =
 K1  2.303R  T1T2 
∂ lnK ∆E
21. van’t Hoff isochore: = 2
∂T RT V  V 
22. Total work done during the one complete Carnot cycle: w = mRT1 ln  2  − mRT2 ln  3 
T  V1   V4 
23. Efficiency of the Carnot’s engine: h = 1− 2
T1

SOLVED EXAMPLES
First Law of Thermodynamics 100°C. (Given: Molar enthalpy of vapourization of water at
1 bar and 373 K = 41 k J mol−1 and R = 8.3 J mol−1 K−1) will be
1. (∆H − ∆U) for the formation of carbon monoxide (CO) from its (a) 4.1 k J mol−1
elements at 298 K is (R = 8.314 J K−1 mol−1) (b) 3.7904 k J mol−1
(a) −1238.78 J mol−1 (c) 37.904 k J mol−1
(b) 1238.78 J mol−1 (d) 41.0 k J mol−1
(c) −2477.57 J mol−1 (AIEEE 2007)
(d) 2477.57 J mol−1
(AIEEE 2006) Solution
(c) The reaction involved is H2O(l) → H2O(g), where ∆ng =
Solution
1 mol, ∆vapH = 41 k J mol−1. Now,
(d) H = U + pV ; ∆H = ∆U + ∆(pV) ⇒ ΔH − ΔU = Δ(pV)
Also, ∆(pV) = ∆ngRT for an ideal gas, therefore, ∆U = ∆H − ∆ngRT
ΔH − ΔU = ∆ngRT = 41 kJ mol−1 − 1× 8.3 × 10 −3 kJ K −1 mol−1 × 373 K
∆ng = 2 for the reaction O2 (g) + 2C(s) → 2CO(g) = 41− 3.096 kJ mol−1 = 37.904 kJ mol−1

3. A piston filled with 0.04 mol of an ideal gas expands reversibly


∆H − ∆U = 1 × 8.314 × 298 = 2477.57 J mol−1 from 50.0 mL to 375 mL at a constant temperature of 37.0°C.
As it does so, it absorbs 208 J of heat. The value of q and w for
2. Assuming that water vapour is an ideal gas, the internal energy
the process will be (R = 8.314 J mol−1 K−1) (ln 7.5 = 2.01)
(∆U) when 1 mol of water is vapourized at 1 bar pressure and

Chapter-6.indd 169 8/4/2016 10:32:41 AM


170 Chapter 6 Thermodynamics

(a) q = −208 J, w = −208 J are 1000 kg m−3 and 0.6 kg m−3 respectively. The latent heat of
(b) q = −208 J, w = +208 J vapourization of water = 2.25 × 106 J kg−1
(c) q = +208 J, w = +208 J (a) 2.99 × 106 J (b) 3.26 × 106 J
(d) q = +208 J, w = −208 J 6
(c) 3.02 × 10 J (d) 2.08 × 106 J
(JEE Main 2013)
Solution
Solution
(d) Change in internal energy can be calculated using heat
(d) Process is isothermal reversible expansion, hence and work relation.
∆U = 0. Therefore, according to first law of thermodynamics The volume of 1 kg of water
q = −w. As q = +208 J, hence w = −208 J. 1 1 3
= m3 and of 1 kg of steam = m
4. The internal energy change when a system goes from state 1000 0.6
A to B is 40 k J mol−1. If the system goes from A to B by a revers- The increase in volume
ible path and returns to state A by an irreversible path what 1 3 1
= m − m3 = (1.7 − 0.001 m3 ≈ 1.7 m3 )
would be the net change in internal energy? 0.6 1000
(a) 40 k J (b) > 40 k J (c) < 40 k J (d) Zero The work done by the system is
Solution p∆V = (100 kPa)(1.7 m3 ) = 1.7 × 105 J
(d) ∆U = 0, as the change in internal energy is zero for a cyclic The heat given to convert 1 kg of water into steam
process.
= 2.25 × 106 J
5. Consider the reaction: N2 + 3H2 → 2NH3 carried out at a con- The change in internal energy is
stant temperature and pressure. If ∆H and ∆U are the enthalpy
∆U = ∆q − ∆w = 2.25 × 106 − 1.7 × 105 = 2.08 × 106 J
and internal energy changes for the reaction, which of the fol-
lowing expressions is true?
(a) ∆H = 0 (b) ∆H = ∆U Pressure Volume Work
(c) ∆H < ∆U (d) ∆H > ∆U 9. An ideal gas expands in volume from 1 × 10−3 m3 to 1 × 10−2 m3
at 300 K against a constant pressure of 1 × 105 Nm−2. The work
Solution
done is
(c) ∆H = ∆U + ∆nRT . Since, ∆n = 2 − 4 = −2, therefore, (a) −900 J (b) 900 k J
∆H = ∆U − 2RT ⇒ ∆H < ∆U (c) 2780 k J (d) −900 k J
6. The value of ∆U  for a certain change is −1455 J. During the Solution
change, the system absorbs 812 J of heat. How much work, (d) Work done (under constant pressure) = −p∆V = −1 × 105
expressed in joules, was involved? [10−2 − 10−3] = −900 J
(a) −2267 J (b) −1325 J
(c) −3021 J (d) −2856 J 10. Which of the following statements is not correct?
(a) Final temperature in reversible adiabatic expansion is
Solution
greater than that in irreversible adiabatic expansion.
(a) ∆U = q + w ⇒ −1455 J = 812 J + w ⇒ w = −2267 J (b) When heat is supplied to an ideal gas in isothermal pro-
Since w is defined to be the work done on the system by cess, kinetic energy of gas remains constant.
the surroundings, then in this case, a negative amount of (c) When an ideal gas is subjected to adiabatic expansion, it
work is done on the system by the surroundings. The sys- gets cooled.
tem, in fact, does work on the surroundings. (d) Entropy increases when an ideal gas expands
7. Which one of the following equations does not correctly isothermally.
represent the first law of thermodynamics for the given Solution
process? w
(a) For adiabatic process, ∆U = w ⇒ w = CV ∆T ⇒ ∆T =
(a) Isothermal process: q = −w CV
(b) Cyclic process: q = −w Since work done in reversible process is more than that
(c) Isochoric process: ∆U = q in irreversible process, the temperature change is more
(d) Adiabatic process: ∆U = −w in reversible process, that is, the temperature decrease
will be more.
Solution
Since kinetic energy of ideal gases is proportional
(d) According to the first law of thermodynamics, ∆U = q + w to root of temperature, for isothermal it will not
For isothermal process, ∆U = 0. Hence, q = −w. change.
For cyclic process, ∆U = 0. Hence, q = −w. From the above relations, it is clear that when an ideal
For isochoric process, ∆V = 0. Hence, ∆U = q (w = p∆V = 0). gas is subjected to adiabatic expansion, its temperature
For adiabatic process, q = 0. Hence ∆U = w. decreases.
8. Calculate the increase in internal energy of 1 kg of water at Entropy increases when an ideal gas expands isother-
100°C when it is converted into steam at the same temper- mally because increasing the volume causes increase in
ature and at 1 atm (100 kPa). The density of water and steam randomness.

Chapter-6.indd 170 8/4/2016 10:32:45 AM


Solved Examples 171

11. Calculate the work, in joules, done by a gas as it expands at Therefore,


constant temperature from a volume of 3.00 L and a pressure ∆ H1 240
of 5.00 atm to a volume of 8.00 L. The external pressure against ∆H = + ∆H2 + ∆H3 = − 349 − 381 = −610 kJ mol−1
2 2
which the gas expands is 1.00 atm. (1 atm = 101,325 Pa.)
(a) − 405 J (b) − 532 J 14. For complete combustion of ethanol,
(c) − 458 J (d) − 507 J
C2H5OH(l) + 3O2 (g) → 2CO2 (g) + 3H2O(l)
Solution
the amount of heat produced as measured in bomb cal-
(d) w = −p∆V
orimeter, is 1364.47 k J mol−1 at 25°C. Assuming ideality
∆V = 8.00 L − 3.00 L = 5.00 L
the enthalpy of combustion, ∆cH, for the reaction will be
w = −(1 atm) × (5.00 L) = −5.00 L atm
(R = 8.314 J K−1 mol−1)
 101.325 J  (a) −1366.95 k J mol−1 (b) −1361.95 kJ mol−1
In joules: −5.00 L atm  = −507 J
 1 L atm  (c) −1460.50 kJ mol−1 (d) −1350.50 kJ mol−1
12. Two separate experiments were carried out involving adia- (JEE Main 2014)
batic reversible expansion, taking monoatomic gas in one Solution
case and a diatomic gas in the second case. In each case, the (a) For the given reaction,
initial temperature was Ti and expansion was from volume ( −1 × 8.314 × 298 )
V1 to volume V2. The final temperatures attained were TM for ∆H = ∆U + ∆ngRT = −1364.47 + × 10 −3 = −1366.95 kJ mol−1
1000
monoatomic gas and T∆ for= diatomic ( −1 × 8.314 × 298 )
DH ∆U + ∆nggas. RT =Then−1364.47 + × 10 −3 = −1366.95 kJ mol−1
(a) TM = TD < Ti (b) TM < TD < Ti 1000
(c) TD > TM > Ti (d) TD < TM < Tf
Solution 15. The reaction 2N2O(g) → 2N2 (g) + O2 (g); ∆H° = −163 kJ. What
g −1
T2  V1  is the change in internal energy for the decomposition of 176
(b) For adiabatic expansion, =   , g = 1.66 for mon- g of N2O at 25°C?
T1  V2 
(a) −331 k J (b) −657 k J
oatomic gas and 1.40 for diatomic gas. (c) −642 k J (d) −168 k J
0.66
T V 
Hence, for monoatomic gas, M =  1  and for dia- Solution
0.40 Ti  V2 
T V  (a) We have ∆H ° = ∆U ° + ∆ngRT ⇒ ∆U = ∆H ° − ∆ngRT
tomic gas, D =  1 
Ti  V2   1 × 8.314 × 300 
V1 TM TD =  −163 −  = ( −163 − 2.494 )
As < 1 (since V2 > V1), < 1. i.e., TM < Ti and <1  1000
V2 Ti Ti = − 165.5 kJ
i.e., TD < Ti This change of ∆U is for two moles of N2O therefore,
Suppose V2 = 10 V1. Then, ∆U for four moles = −2 × 165.5 kJ mol−1
0.66 − 0.40 0.26 = −331.0 kJ mol−1
TM  1   1
=  =  i.e., TM < TD. 16. Standard molar enthalpy of formation of CO2 is equal to
TD  10   10 
(a) zero.
Hence, TM < TD < Ti (b) the standard molar enthalpy of combustion of gaseous
Enthalpy carbon.
(c) the sum of standard molar enthalpies of formation of CO
13. Oxidizing power of chlorine in aqueous solution can be and O2.
determined by the parameters indicated below: (d) the standard molar enthalpy of combustion of carbon
1 ∆ H° ∆ eg H ° ∆ hyd H ° (graphite).
1
Cl (g) 2 → Cl(g)  → Cl− (g)  → Cl− (aq)
diss

2 2
The energy involved in the conversion of 1 Cl (g) to Solution
2 2
(d) Standard molar heat enthalpy (H°) of a compound is
Cl (g) (using the data, ∆ dissH (Cl2 ) = 240 kJ mol−1 ,
− o
equal to its standard heat of formation from most stable
states of initial components.
∆ egH o (Cl) = −349 kJ mol−1 ∆ hydH o (Cl) = −381 kJ mol−1 )
(a) +152 k J mol−1 (b) −610 kJ mol−1 Heat Capacities
(c) −850 k J mol−1 (d) +120 kJ mol−1
17. When 0.1 mol of a gas absorbs 41.75 J of heat, the rise in tem-
(AIEEE 2008)
perature that occurs is equal to 20 K. The gas must be
Solution
(a) triatomic. (b) diatomic.
(b) Given that
(c) polyatomic. (d) monoatomic.
1
 Cl− (aq);
Cl (g) →
2 2
∆H = ?
Solution
 2Cl; ∆H1 = 240 kJ mol−1
Cl2 → (b) CV (heat absorbed per degree rise per mole)
 Cl− ; ∆H2 = −349 kJ mol−1
Cl → 41.75 J
= = 20.875 JK −1 mol−1
Cl− Aqueous
medium
 → Cl− (aq); ∆H3 = −381kJ mol −1 0.1 mol × 20 K

Chapter-6.indd 171 8/4/2016 10:32:50 AM


172 Chapter 6 Thermodynamics

∆H =  21 ∆HI2(s) →I2(g ) + 21 ∆HI −I + 21 ∆HCl− Cl  − [ ∆HI − Cl ]


 
Cp = CV + R = (20.875 + 8.314) J K−1mol−1
=  2 × 62.76 + 2 × 151.0 + 2 × 242.3 − [211.3]
1 1 1
= 29.189 J K−1 mol−1
Cp ∆H = 228.03 − 211.3 = 16.73 kJ mol−1
29.189
= = 1.40
CV 20.875
Hence, the gas is diatomic. 21. On the basis of the following thermochemical data:
∆ f G°[H+ (aq)] = 0
18. What is the molar heat capacity of ethyl alcohol, C2H5OH, in
units of J mol−1 °C−1, if its specific heat is 0.586 cal g−1 °C−1? H2O(l) → H+ (aq) + OH− (aq); ∆H = 57.32 kJ
(a) 215 J mol−1 ° C −1 (b) 135 J mol−1 ° C −1 H2 (g) + 21 O2 (g) → H2O(l); ∆H = −286.20 kJ
(c) 113 J mol−1 ° C −1 (d) 256 J mol−1 ° C −1 The value of enthalpy of formation of OH− ion at 25°C is
Solution (a) +228.88 k J (b) −343.52 k J
(c) 1 cal = 4.184 J; 1 mol C2H5OH = 46.08 g (c) −22.88 k J (d) −228.88 k J
(AIEEE 2009)
J
Molar heat capacity = Solution
mol °C
(d) From the first reaction,
 0.586 cal   4.184 J   46.08 g C2H5OH 
=   ∆ r H = [ ∆ f H(H+ ) + ∆ f H °(OH− )] − ∆ f H °(H2O )
 g °C   cal   1 mol C2H5OH 
57.32 = 0 + ∆ f H °(OH− ) − ( −286.2)
= 113 J mol−1 ° C −1
or ∆ f H°(OH− ) = 57.32 − 286.2 = −228.88 kJ
19. A steam boiler made up of steel weighs 900 kg. The boiler
contains 400 kg of water. Assuming 70% of the heat is deliv- 22. The standard enthalpy of formation of NH3 is −46.0 kJ mol−1.
ered to boiler and water, how much heat is required to raise If the enthalpy of formation of H2 from its atoms is
the temperature of the whole from 10°C to 100°C? Heat −436 k J mol−1 and that of N2 is −712 kJ mol−1, the average
capacity of steel is 0.11 kcal kg−1 K−1 and heat capacity of bond enthalpy of N−H bond in NH3 is
water is 1 kcal kg−1 K−1. (a) −1102 k J mol−1 (b) −964 kJ mol−1
(a) 65321 kcal (b) 64157 kcal (c) +352 k J mol −1 (d) +1056 kJ mol−1
(c) 6869 kcal (d) 66549 kcal (AIEEE 2010)
Solution Solution
(b) By using the equation ∆H = mboilerCboiler ∆T + mwater (c) The formation of NH3 is represented as
Cwater ∆T first find out the ∆H required for complete heat- 1
N + 31H → 3NH
2 2 2 N22 + H23 → NH3 ; r
∆ H = −46 kJ mol−1
2 2
ing and then by using the value of ∆H calculate the actual
Given that the bond enthalpy (∆bondH) of H2 = +436
heat required when 70% is transferred to the system.
k J mol−1 and that of N2 = +712 kJ mol−1. Therefore, the
∆H required for heating = mboilerCboiler ∆T + mwaterCwater ∆T
average bond enthalpy of N H bond is
= 900 × 0.11 × 90 + 400 × 1 × 90 = 44910 kcal
Since, only 70% of heat given is used up to do so. ∆ r H =  21 ∆bondH(N2 ) + 23 ∆bondH(H2 ) − 3∆bondH(N H)
44910 × 100
Thus, actual heat required = = 64157 kcal
70 −46 =  21 × 712 + 23 × 436  − 3∆bondH(N H)
Thermochemistry
= (356 + 654) − 3∆bondH(N H)
20. The enthalpy changes for the following processes are listed
below: or 3 ∆ bondH (N H) = 1010 + 46 = 1056
Cl2(g) = 2Cl(g), ∆H = 242.3 k J mol−1 Therefore, ∆ bondH (N H) = 352 kJ mol−1
I2(g) = 2I(g), ∆H =151.0 k J mol−1
ICl(g) = I(g) + Cl(g), ∆H = 211.3 k J mol−1
23. The standard enthalpy of formation of NH3 is − 46.0 kJ mol−1.
I2(s) = I2(g), ∆H = 62.76 k J mol−1
If the enthalpy of formation of H2 from its atoms is
Given that the standard states for iodine and chlorine are I2(s) −436 k J mol−1 and that of N2 is −712 kJ mol−1, the average
and Cl2(g), the standard enthalpy of formation for ICl(g) is bond enthalpy of N H bond in NH3 is
(a) −14.6 k J mol−1 (b) −16.8 k J mol−1 (a) −1102 k J mol−1 (b) −964 kJ mol−1
(c) +16.8 k J mol −1 (d) +244.8 k J mol−1 (c) +352 kJ mol−1 (d) +1056 kJ mol−1
(AIEEE 2006) (JEE Main Online 2014)
Solution Solution
(c) The required equation is 21 I2 (s) + 21 Cl2 (g) → ICl(g) ∆H
(c) NH3 (g) → N(g) + 3H(g) ∆N−HH ° =
3
∆H =  21 ∆HI2(s) →I2(g ) + 21 ∆HI −I + 21 ∆HCl− Cl  − [ ∆HI − Cl ] The value of ∆H for this reaction can be obtained from
  summation of following equations.
=  2 × 62.76 + 2 × 151.0 + 2 × 242.3 − [211.3]
1 1 1

Chapter-6.indd 172 8/4/2016 10:33:02 AM


Solved Examples 173

∆H Solution
2NH3 → 2N + 6H ∆N−HH ° = (b) Consider the reactions:
6
2NH3 → N2 + 3H2 ∆ r H ° = 92 C + O2 → CO2 ; ∆H = − 393.5 kJ mol−1 (1)
1 (2)
N2 → 2N ∆rH ° = 712 CO + O2 → CO2 ; ∆H = + 283 kJ mol−1
2
3H2 → 6H ∆r H ° = 436 × 3 1
We have to find C + O2 → CO; ∆H = ?
_________________ 2
2NH3 → 2N + 6H ∆r H ° = 2112 Reversing Eq. (2) and then adding it to Eq. (1), we get
_________________ ∆H = −393.5 + 283 = −110.5 k J mol−1
2112 28. If the bond dissociation energies of XY, X2 and Y2 (all diatomic
Thus, at this condition ∆N−HH ° = = 352 kJ mol−1
6 molecules) are in the ratio of 1 : 1 : 0.5 and ∆f H for the forma-
24. The standard enthalpy of formation (∆ f H°298) for meth- tion of XY is −200 k J mol−1. The bond dissociation energy of
ane, CH4 is −74.9 k J mol−1. In order to calculate the average X2 will be
energy given out in the formation of a C H bond from this it (a) 100 k J mol−1 (b) 800 kJ mol−1
is necessary to know which one of the following? (c) 300 kJ mol−1 (d) 400 kJ mol−1
(a) The dissociation energy of the hydrogen molecule, H2. Solution
(b) The first four ionization energies of carbon. (b) Since, XY, X2 and Y2 have dissociation energies in the
(c) The dissociation energy of H2 and enthalpy of sublima- ratio of 1:1:0.5. Thus, let us assume that,
tion of carbon (graphite).
XY → X(g) + Y(g); ∆H = +a kJ mol−1
(d) The first four ionization energies of carbon and electron
affinity of hydrogen. X 2 → 2X ; ∆H = +a kJ mol−1
(JEE Main Online 2014) Y2 → 2Y ; ∆H = +0.5a kJ mol−1
Solution
Given that heat of formation of XY is −200 kJ mol−1. Thus,
∆H
(c) CH4 → C(g) + 4H(g) ∆ C − HH ° =
4 1 11 1
X2 + YX22→ + XYY2 → XY
C (graphite ) + 2H2 → CH4 ∆ r H ° = −74.9 2 22 2
CH4 → C (graphite ) + 2H2 ∆ r H ° = 74.9  Ha = 0.5+ a + 0.5 a − a= −200−kJ −1
∆H = ∆ + +  a − a = −200 kJ mol 1 mol
o  2 2 2 2  
2H2 (g) → 4H ∆r HBDE =? −1
800−kJ
a = 800 akJ= mol 1 mol
C (graphite) → C(g) ∆ r H sub = ?
____________________________________ 29. The enthalpy of neutralization of HCl by NaOH is
CH4 → C(g) + 4H(g) −55.9 k J mol−1. If the enthalpy of neutralization of HCN by
____________________________________
NaOH is −12.1 k J mol−1, the enthalpy of dissociation of HCN is
25. If at 298 K the bond energies of C H, C C, C C and H H (a) 43.8 k J (b) −43.8 k J
are respectively 414, 347, 615, and 435 k J mol−1, the value of (c) 68 k J (d) −68 k J
enthalpy change for the reaction
H2C CH2(g) + H2(g) → H3C CH3(g) at 298 K will be, Solution
(a) +250 k J (b) −250 k J (c) +125 k J (d) −125 k J (a) HCl + NaOH → NaCl + H2O ∆H1 = −55.9 kJ mol−1

Solution HCN + NaOH → NaCN + H2O ∆H2 = −12.1 kJ mol−1


(d) ∆H = (Σ B.E. of the reactants − Σ B.E. of the products) Enthalpy of dissociation of HCN = ∆H2 − ∆H1 = −12.1 −
= [615 + (4 × 414) + 435] − [347 + (414 × 6)] (−55.9) = +43.8 k J.
= 2706 − 2831 = −125 k J
30. The lattice energy of NaCl is −780 k J mol−1. The enthalp-
26. The enthalpy change for a reaction does not depend upon ies of hydration of Na+(g) and Cl−(g) ions are −406 kJ mol−1
(a) the physical states of reactants and products. and −364 kJ mol−1 respectively. The enthalpy of solution of
(b) use of different reactants for the same product. NaCl(s) in water is
(c) the nature of intermediate reaction steps. (a) 738 k J mol−1 (b) 10 kJ mol−1
(d) the differences in initial or final temperatures of involved (c) −10 kJ mol −1 (d) −822 kJ mol−1
substances.
Solution
Solution
(b) NaCl(s) → Na+ (g) + Cl− (g) ∆H = +780 kJ (Lattice energy)
(c) Because according to Hess law, the enthalpy change
does not depend upon the path followed. Na+ (g) + H2O(l) → Na+ (aq) ∆H = −406 kJ (Hydration)

27. The enthalpies of combustion of carbon and carbon monox- Cl− (g) + H2O(l) → Cl− (aq) ∆H = −364 kJ (Hydration)
ide are −393.5 and −283 k J mol−1 respectively. The enthalpy The net reaction of dissolution of NaCl(s) is
of formation of carbon monoxide per mole is NaCl(s) + 2H2O(l) → Na+ (aq) + Cl− (aq)
(a) 110.5 k J (b) −110.5 k J
(c) −676.5 k J (d) 676.5 k J ∆ solH = 780 − ( 406 + 364 ) = 10 kJ mol−1

Chapter-6.indd 173 8/4/2016 10:33:07 AM


174 Chapter 6 Thermodynamics

31. The bond energies of C C, C H, H H, and C C are 198, 35. In an irreversible process taking place at constant T and p
98, 103, and 145 kcal, respectively. The enthalpy change of and in which only pressure − volume work is being done, the
the reaction HC CH + H2 → C2H4 in k J is change in Gibbs free energy (dG) and change in entropy (dS),
(a) −325.6 k J (b) 100.32 k J satisfy the criteria
(c) −192.56 k J (d) −100.32 k J (a) (dS)V,U < 0, (dG)T,p < 0 (b) (dS)V,U > 0, (dG)T,p < 0
(c) (dS)V,U = 0, (dG)T,p = 0 (d) (dS)V,U = 0, (dG)T,p > 0
Solution
(c) Enthalpy of reaction = ∑ ∆H °reactants − ∑ ∆H °products Solution
Therefore, (b) For an irreversible process, the value of dS is greater than
∆H∆H
= [2(C − H)++(C(C ≡C)
= [2(C H)] C )++(H )] −−[ 4[4(C
(H − HH)] + (+CC= C)]C)]
( C − H)H) zero.
= [2(98 ) + 198 + 103] − [ 4(98 ) + 145] = −40 kcal 36. For which of the following reactions, the change in entropy
Enthalpy in k J is − 40 × 4.814 = −192.56 k J is not negative?
(a) Ni(s) + 2HCl(aq) → H2 (g) + NiCl2 (aq)
Entropy
(b) Br2 (g) + 3Cl2 (g) → 2BrCl3 (g)
32. The entropy change involved in the isothermal reversible
expansion of 2 mol of an ideal gas from a volume of 10 dm3 (c) NH3 (g) + HCl(g) → NH4 Cl(s)
to a volume of 100 dm3 at 27°C is
(a) 35.8 J mol−1 K−1 (b) 32.3 J mol−1 K−1 (d) CaO(s) + H2O(l) → Ca(OH)2 (s)
−1
(c) 42.3 J mol K −1 (d) 38.3 J mol−1 K−1 Solution
(AIEEE 2011)
(a) Entropy is just the measurement of randomness of
Solution
a system, if randomness increases entropy will also
(d) For an ideal gas, for isothermal reversible process, increase. Therefore, during phase transformation, when
system moves from solid state to liquid or liquid to
V   100  gaseous state, randomness increases, so entropy also
∆S = 2.303 nR log  2  = 2.303 × 2 × 8.314 × log 
 V1   10  increases.
During the chemical reaction, product may appear in
= 38.3 J mol−1 K −1
any state of matter, so entropy will increase if more and
33. The molar heat capacity (Cp) of CD2O is 10 cals at 1000 K. The more gaseous products are formed from solid or liquid
change in entropy associated with cooling of 32 g of CD2O state.
vapour from 1000 K to 100 K at constant pressure will be: In all given options, only in option (a), solid Ni reacts
(D = deuterium, at. mass = 2 u) with liquid HCl which produces hydrogen gas. So, it has
(a) 23.03 cal deg−1 (b) −23.03 cal deg−1 positive change in entropy. All other cases have negative
(c) 2.303 cal deg −1 (d) −2.303 cal deg−1 change of entropy.
(JEE Main Online 2014) 37. When one mole of an ideal gas is compressed to half of its
Solution initial volume and simultaneously heated to twice its initial
(b) For a finite change of temperature at constant pressure, temperature, the change in entropy is
T (a) CV ln 2 (b) Cp ln 2
∆S = 2.303 nC P log10 2
T1 (c) R ln 2 (d) (CV − R) ln 2
100 Solution
∆S = 1 × 2.303 × 10 × log10 = −23.03 cal deg−1
1000 (d) The change in entropy is given by

34. The entropy (S°) of the following substances is: T2 V 1


∆S = CV ln + R ln 2 = CV ln 2 + R ln = CV ln 2 − R ln 2
CH4(g) 186.2 J K−1 mol−1; O2(g) 205.0 J K−1 mol−1; CO2(g) T1 V1 2
213.6 J K−1 mol−1; H2O(l) 69.9 J K−1 mol−1 = (CV − R )ln 2
The entropy change (∆S°) for the reaction
CH4(g) + 2O2(g) → CO2(g) + 2H2O(l) is 38. The entropy change can be calculated by using the expres-
(a) −312.5 J K−1 mol−1 (b) −242.8 J K−1 mol−1 sion ∆S = qrev/T. When water freezes in a glass beaker, choose
−1
(c) −108.1 J K mol −1 (d) −37.6 J K−1 mol−1 the correct statement amongst the following.
(JEE Main Online 2014) (a) ∆S (system) decrease but ∆S (surroundings) remains the
Solution same.
(b) CH4 (g) + 2O2 (g) → CO2 (g) + 2H2O (b) ∆S (system) increase but ∆S (surroundings) decreases.
(c) ∆S (system) decrease but ∆S (surroundings) increases.
°
∆S = Sproduct °
− Sreactant (d) ∆S (system) decrease and ∆S (surroundings) also
decreases.
° + 2SH° O ) − ( SCH
∆S = ( SCO ° + 2SO° )
2 2 4 2
Solution
∆S = (213.6 + 2 × 69.9) − (186.2 + 2 × 205)
(c) ∆S (system) decreases because molecules in solid ice are
= −242.8 J K−1 mol−1 less random than in liquid water. However, when water

Chapter-6.indd 174 8/4/2016 10:33:10 AM


Solved Examples 175

freezes to ice, heat is released, which is absorbed by the Gibb’s Free Energy
surroundings. Hence, the entropy of the surroundings
increases. 42. In conversion of limestone to lime,

39. The heat of vapourization of ammonia is 21.7 kJ mol−1 and CaCO3 (s) → CaO(s) + CO2 (g)
the boiling point of ammonia is −33°C. Estimate the entropy the values of ∆H° and ∆S° are +179.1 k J mol−1 and 160.2 J K−1,
change for the vapourization of liquid ammonia. respectively, at 298 K and 1 bar. Assuming that ∆H° and ∆S°
(a) 101.5 J K−1 (b) 88.9 J K−1 do not change with temperature, the temperature above
(c) 90.5 J K −1 (d) 56.8 J K−1 which conversion of limestone to lime will be spontaneous is
Solution (a) 1008 K (b) 1200 K
(c) 845 K (d) 1118 K
∆H o
(c) T≈ (AIEEE 2007)
∆S o
 1000 J  Solution
. kJ mol−1 
∆H = 217 o
 1 kJ  (d) We know ∆G = ∆H − T∆S. We will find out the equilibrium
= 217
. × 10 J 3 temperature at which ∆G = 0. Therefore, ∆H = T ∆S
21.7 × 103 J 1000
239.9 K ≈ T = 179.1× = 1118 K
160.2
∆S o
So, above this temperature the reaction will become
∆S o = 90.5 J K −1 spontaneous.

40. Oxygen gas weighting 64 is expanded from 1 atm to 0.25 43. Identify the correct statement regarding a spontaneous
atm at 30°C. Calculate entropy change, assuming the gas to process:
be ideal. (a) Lowering of energy in the reaction process is the only
(a) 23.1 J K−1 (b) 25.0 J K−1 criterion for spontaneity.
(c) 26.1 J K−1 (b) 22.4 J K−1 (b) For a spontaneous process in an isolated system, the
change in entropy is positive.
Solution
(c) Endothermic processes are never spontaneous.
p  (d) Exothermic processes are always spontaneous.
(a) ∆S = 2.303nR log  1 
 p2  (AIEEE 2007)
weight 64 Solution
n= = =2
molecular weight 32 (b) Change in entropy for an isolated system should be pos-
itive, ∆Ssystem + ∆Ssurroundings = 0.
p   1 
∆S = 2.303nR log  1  = 2.303 × 2 × 8.314 log 
 p2   0.25  44. Standard entropies of X2, Y2 and XY3 are 60, 40 and 50 J K−1 mol−1,
respectively. For the reaction,
= 23.053 J K −1
1 3
X2 + Y2 → XY3 ; ∆H = −30 kJ
41. If for a given substance melting point is TB and freezing point 2 2
is TA, then correct variation shown by graph between entropy to be at equilibrium, the temperature will be
change and temperature is (a) 1250 K (b) 500 K
(c) 750 K (d) 1000 K
TB (AIEEE 2008)
∆S ∆S Solution
TB 1 3
(a) (b) (c) For the reaction, X2 + Y2 → XY3
2 2
TA
TA
1 3 
∆ r S = 50 −  × 60 + × 40 = −40 J mol−1
T T 2 2 
Since, ∆G = ∆H − T ∆S and at equilibrium ∆G = 0, so,
TB ∆H −30
∆S ∆S
TA T= = × 103 = 750 K
TB ∆S −40
TA
(c) (d)
45. In a fuel cell, methanol is used as fuel and oxygen gas is used
as an oxidizer. The reaction is
T T 3
CH3OH(l) + O2 (g) → CO2 (g) + 2H2O(l)
Solution 2
(a) For a pure substance, TA and TB represent the same tem- At 298 K, standard Gibbs energies of formation for CH3OH(l),
perature. Hence, (a) is a correct choice. H2O(l) and CO2(g) are −166.2 k J mol−1, −237.2 k J mol−1 and

Chapter-6.indd 175 8/4/2016 10:33:18 AM


176 Chapter 6 Thermodynamics

−394.4 k J mol−1, respectively. If the standard enthalpy of Solution


combustion of methanol is −726 k J mol−1, efficiency of the
(c) ∆G ° = 2 ∆G(°NO2 ) − [2 ∆G(NO)
° °
+ ∆G(O 2)
] = 2 x − [2 × 86 , 600 + 0]
fuel cell will be
(a) 90% (b) 97% (c) 80% (d) 87% Also, ∆G° = −RT ln Kp. Substituting values, we get
(AIEEE 2009)
Solution − R(298 )ln(1.6 × 1012 ) = 2 x − 2 × 86 , 600
(b) The reaction involved is ⇒ x = 0.5[2 × 86 , 600 − R(298 )ln(1.6 × 1012 )]

CH3OH(l) + 23 O2 (g) → CO2 (g) + 2H2O(l) ; ∆H = − 726 kJ mol−1 49. The correct relationship between free energy change in a
reaction and the corresponding equilibrium constant Kc is
Now,
(a) ∆G = RT ln Kc (b) − ∆G = RT ln Kc
∆G = ∆G °(CO2 ) + 2∆G °(H2O) − ∆G °(CH3OH) (c) ∆Gο = RT ln Kc (d) − ∆Gο = RT ln Kc
= −394 − 2 × (237.2) + 166.2 = −702.6 kJ mol−1 Solution
Efficiency of fuel cell is given by (d) ΔG° = − RT ln Kc or, −ΔG° = RT ln Kc
∆G 702.6
× 100 = × 100 = 97% 50. For a spontaneous reaction, the ∆G, equilibrium constant (K)
∆H 726 and E°cell will be respectively
46. For a particular reversible reaction at temperature T, ∆H and (a) −ve, >1, +ve (b) +ve, >1, −ve
∆S were found to be both positive. If Te is the temperature at (c) −ve, <1, −ve (d) −ve, >1, −ve
equilibrium, the reaction would be spontaneous when Solution
(a) T = Te (b) Te > T (c) T > Te (d) Te = 5T (a) For a spontaneous process ∆G has to be negative always,
°
(AIEEE 2010) so that K >1 because ∆G = −2.303 RT log K and Ecell has
Solution to be positive because ∆G = − nFE°cell
(c) ∆G = ∆H − T∆S. At equilibrium ∆G = 0. Also, ∆G should 51. Using the data given below, pick the correct statement about
be negative for a spontaneous reaction. So, T > Te in the reaction: TiO2 (s) + 2Cl2 (g) → TiCl4 (l) + O2 (g) [Given that
order to make ∆G negative, because both ∆H and ∆S are ∆ f H° for TiO2(s), TiCl4(l), Cl2(g) and O2(g) are −944.7, −804.22,
positive. 0.0, 0.0 kJ mol−1 respectively. Also S° for TiO2(s), TiCl4(l), Cl2(g)
47. The incorrect expression among the following is and O2(g) are 50.3, 252.3, 233.0, 205.1 J mol−1 K−1, respectively.]
(a) The reaction is exothermic at standard conditions.
∆G
(a) = −T (b) The standard entropy is favourable for the reaction in
∆Stotal forward direction.
Vf
(b) In isothermal process wreversible = − nRT ln (c) The reaction is not spontaneous at standard condition at
Vi 25°C.
(c) lnK = ∆H ° − T ∆S ° (d) Reaction becomes spontaneous at higher temperature.
RT
Solution
(d) K = e −∆G °/ RT (AIEEE 2012) ° ° ° °
(c) ∆H ° = [HTiCl4 (l) + HO2 ( g) ] − [HTiO2 ( s ) + HCl2 ( g) ]
Solution = ( −804.2 + 0 ) − ( −944.7 + 0 ) = 140.5 kJ mol−1
(c) We know that ΔG° = ΔH° − TΔS° and ΔG° = −RT ln K. °
∆S ° = [ STiCl + SO° 2 ( g) ] − [ STiO
° °
+ SCl ]
4 ( l) 2 (s) 2 ( g)
Equating both the equations, we get
= ( +252.3 + 205.1) − ( +50.3 + 2 × 233)
∆H ° − T ∆S °
− RT ln K = ∆H ° − T ∆S ° ⇒ ln K = − = − 58.9 J mol−1 K −1 = − 0.0589 kJ mol−1 K −1
RT
∆G ° = ∆H ° − T ∆S ° = 140.5 − 298 × ( −0.0589 )
48. The following reaction is performed at 298 K.
= 158.06 kJ mol−1
2NO(g) + O2 (g)  2NO2 (g)
Since change in free energy is positive, the reaction is
The standard free energy of formation of NO(g) is 86.6 k J mol−1 not spontaneous.
at 298 K. What is the standard free energy of formation of
NO2(g) at 298 K? (Kp = 1.6 × 1012) 52. Given these reactions and their ∆G° values,
(a) 86600 + R(298) ln(1.6 × 1012) COCl2(g) + 4NH3(g) → CO(NH2)2(s) + 2NH4Cl(s), ∆G° = –332.0 kJ
12 COCl2(g) + H2O(l) → CO2(g) + 2HCl(g), ∆G° = –141.8 kJ
ln(1.6 × 10 )
(b) 86600 − NH3(g) + HCl(g) → NH4Cl(s), ∆G° = –91.96 kJ
R(298 )
Calculate the value of ∆G° for the reaction
(c) 0.5[2 × 86,600 − R(298)ln (1.6 × 1012 )] CO(NH2)2(s) + H2O(l) → CO2(g) + 2NH3(g)
(d) R(298 ) ln(1.6 × 1012 ) − 86600 (a) −9.2 k J (b) + 6.3 k J
(JEE Main 2015) (c) +8.5 k J (d) − 7.8 k J

Chapter-6.indd 176 8/4/2016 10:33:26 AM


Advanced Level Problems 177

Solution ∆G° = { ∆ f G° [CO2(g)] + 2 ∆ f G° [H2O(g)]} − { ∆ f G°


(b) On adding the reverse of the first equation to the sec-
[CH4(g)] + 2 ∆ f G° [O2(g)]}
ond equation plus twice, the third equation, we get the
required equation ∆G° = {1 mol × (−394.4 k J mol−1) + 2 mol × (−228.6 k J
mol−1)} − {1 mol × (−50.79 k J mol−1) + 2 mol ×
CO(NH2)2(s) + 2NH4Cl(s) → COCl2(g) + 4NH3(g), ∆G° = +332.0 kJ (0.0 k J mol−1)}
COCl2(g) + H2O(l) → CO2(g) + 2HCl(g), ∆G° = −141.8 kJ ∆G° = −800.8 k J mol−1
2NH3(g) + 2HCl(g) → 2NH4Cl(s), ∆G° = −183.9 kJ Maximum amount of useful work that could possibly be
CO(NH2)2(s) + H2O(l) → 2NH3(g) + CO2(g), ∆G° = +6.3 kJ obtained from the combustion of 48.0 g of methane is
− (∆G ) = wuseful work kJ
53. At 1500°C, Kc = 5.67 for the reaction
 1 mol CH4   80 00.8 kJ 
CH4(g) + H2O(g) CO(g) + 3H2(g) = ( 48.0 g CH4 )   
 16.0 g CH4   1 mol CH4 
Calculate the value of ΔG°1773  for the reaction at that
= 2.40 × 103 kJ
temperature.
(a) −25.6 k J (b) −30.2 k J 55. For a reaction at equilibrium,
(c) −22.7 k J (d) −35.6 k J (a) ∆G = ∆G° = 0
(b) ΔG = 0 but not ΔG°
Solution
(c) ∆G° = 0 but not ∆G
° = −RT ln K = −(8.314 J mol−1 K−1)(1773 K)(ln 5.67)
(a) ∆G1773 (d) ∆G = ∆G°≠ 0
= −25,600 J = −25.6 k J Solution
54. What is the maximum amount of useful work that could pos- (b) ∆G = ∆G° + RT ln Q at equilibrium, ∆G = 0 and ∆G° = −RT
sibly be obtained at 25°C and 1 atm from the combustion of ln Keq
48.0 g of methane, to give CO2 and H2O
56. Methanol, a potential replacement for gasoline as an auto-
∆ f G°[CO2(g)] = −394.4 kJ mol−1, ∆ f G°[H2O(g)] = −228.6 kJ
motive fuel, can be made from H2 and CO by the reaction
mol−1, ∆ f G° [CH4(g)] = −50.79 kJ mol−1 CO(g) + 2H2 (g)  CH3OH(g)
(a) 2.40 × 103 kJ (b) −3.1 × 10 4 kJ At 500 K, this reaction has  KP  = 6.25 × 10−3. Calculate
3 ΔG°500 for this reaction in units of kJ.
(c) 4.01 × 10 kJ (d) −5.2 × 103 kJ (a) 25.6 k J (b) 36.9 k J
Solution (c) 85.2 k J (d) 21.1 k J
(a) We must first determine ∆G° for the reaction: Solution
CH4(g) + 2O2(g) → CO2(g) + 2H2O(g) (d) ∆G°500 = −RT ln Kp
∆G° = (sum ∆ f G° [products]) − (sum ∆ f G° [reactants]) = −(8.314 J K−1 mol−1)(500 K) ln (6.25 × 10−3)
= 2.11 × 104 J = 21.1 k J

ADVANCED LEVEL PROBLEMS


1. The direct conversion of A to B is difficult; hence it is carried 2. For the reaction,
out by the following shown path: 2CO + O2 → 2CO2 ; ∆H = −560 kJ
C D
Two moles of CO and one mole of O2 are taken in a container
of volume 1 L. They completely form two moles of CO2, the
gases deviated appreciably from ideal behaviour. If the pres-
sure in the vessel changes from 70 to 40 atm, find the magni-
A B
tude (absolute value) of ∆U at 500 K. (1 L atm = 0.1 k J)
Given (IIT-JEE 2006)
∆S(A → C) = 50 eu Solution
∆S(C→D) = 30 eu From the definition of enthalpy,
∆S(B→D) = 20 eu ΔH = ΔU + Δ(pV) ⇒ ΔH = ΔU + VΔp + pΔV
(at constant volume ΔV = 0)
where eu is entropy unit, then ∆S(A →B) is ΔU = ΔH − VΔp = −560 + 1 × 30 × 0.1 = −557 k J
(a) +100 eu (b) +60 eu 3. For the process H2O(L , 1 bar , 373 K) → H2O(g, 1 bar, 373 K),
(c) −100 eu (d) −60 eu the correct set of thermodynamic parameters is
(IIT-JEE 2006) (a) ∆G = 0, ∆S = + ve
Solution (b) ∆G = 0, ∆S = −ve
(b) ∆SA →B = ∆SA → C + ∆S(C→D) − ∆SB→D (c) ∆G = + ve , ∆S = 0
(d) ∆G = −ve , ∆S = + ve
= 50 + 30 − 20 = +60 eu
(IIT-JEE 2007)

Chapter-6.indd 177 8/4/2016 10:33:33 AM


178 Chapter 6 Thermodynamics

Solution Solution
(a) At 100°C, H2O in liquid state has equilibrium with H2O in (a), (d) Both internal energy and molar enthalpy depend
vapour phase; therefore, ∆G = 0. Also, the liquid molecules only on the initial and final states, not on the path followed.
are converting into gaseous phase and we know that the
randomness of system increases from the liquid to gase- 8. In a constant-volume calorimeter, 3.5 g of a gas with molar
ous phase, so, ∆S = positive. mass 28 g mol−1 was burnt in excess oxygen at 298.0 K. The
temperature of the calorimeter was found to increase from
4. The value of log10K for a reaction A → B is 298.0 K to 298.45 K due to the combustion process. Given
(Given: ∆ r H298K = − 54.07 kJ mol−1 and R = 8.314 J K −1 mol−1; T = 298 Kthat
, ∆S =the −1 capacity of the calorimeter is 2.5 k J K−1, the
heat
10 JK )
−1 −1
R = 8.314 J K mol ; T = 298 K , ∆S = 10 JK )−1 . numerical value for the change in internal energy of com-
bustion of the gas in k J mol−1 is ___.
(a) 5 (b) 10 (c) 95 (d) 100 (IIT-JEE 2009)
(IIT-JEE 2007) Solution
Solution
(a) Energy released at constant volume due to combustion of
(b) ∆G ° = ∆H ° − T ∆S ° = − 54.07 − 298 × 10 = − 54070 − 2980 3.5 g of a gas = 2.5 × 0.45. Hence, energy released due to
= − 57050 J mol−1 combustion of 28 g (i.e., 1 mol) of a gas
2.5 × 0.45 × 28
Note, ∆G ° = −2.303RT log K . Substituting values, we get = = 9 kJ mol−1
3.5
57050 57050
log K = = = 10 9. Among the following, the intensive property is (properties
2.303 × 8.314 × 298 5705
are)
5. Statement 1: There is a natural asymmetry between convert- (a) molar conductivity. (b) electromotive force.
ing work to heat and converting heat to work. (c) resistance. (d) heat capacity.
Statement 2: No process is possible in which the sole result is (IIT-JEE 2010)
the absorption of heat from a reservoir and its complete con- Solution
version into work. (a), (b) Both molar conductivity and electromotive force are
(a) Statement 1 is True. Statement 2 is True; Statement 2 is a mass-independent properties. Resistance and heat capacity
correct explanation for Statement 1. are mass-dependent properties, hence extensive.
(b) Statement 1 is True, Statement 2 is True; Statement 2 is
10. One mole of an ideal gas is taken from a to b along two paths
NOT a correct explanation for Statement 1.
denoted by the solid and the dashed lines as shown in the
(c) Statement 1 is True, Statement 2 is False.
below graph. If the work done along the solid line path is ws
(d) Statement 1 is False, Statement 2 is True.
and that dotted line path is wd, then the integer closest to
(IIT-JEE 2008)
the ratio wd/ws is ___.
Solution
(IIT-JEE 2010)
(a) Conceptual; based on second law of thermodynamics
4.5
6. Statement 1: For every chemical reaction at equilibrium, 4.0 a
standard Gibbs energy of reaction is zero and 3.5
Statement 2: At constant temperature and pressure, chemi- 3.0
p (atm)

cal reactions are spontaneous in the direction of decreasing 2.5


Gibbs energy. 2.0
(a) Statement 1 is True, Statement 2 is True: Statement 2 is a 1.5
correct explanation for Statement 1. 1.0
(b) Statement 1 is True, Statement 2 is True; Statement 2 is 0.5 b
NOT a correct explanation for Statement 1. 0.0
(c) Statement 1 is True, Statement 2 is False. 0.0 0.5 1.0 1.5 2.0 2.5 3.0 3.5 4.0 4.5 5.0 5.5 6.0
(d) Statement 1 is False, Statement 2 is True. V(L)
(IIT-JEE 2008)
Solution
Solution
(d) At equilibrium ΔG = 0, standard Gibb’s energy, ΔGo of a
(2) The work done along the dotted line wd is calculated as
reaction may or may not be zero. For a spontaneous reac-
tion, ΔG < 0. w d = 4 × 15
. + 1 × 1 + 2.5 × 0.75 = 8.667 L atm
7. Among the following, the state function(s) is (are)
(a) internal energy. 5.5 5.5 5.5
Process
w s =is−isothermal
2 × 2.303 =w−s 2= ×−22.303
w slog × 2.303
log log 0.5
(b) irreversible expansion work. 0.5 0.5
(c) reversible expansion work. 2 × 2.303 × log2 ×22×.303
11×=22−.303 ×2.log×11
303 log
×=111 2= ×−22.303
.−0414 × 2.303 × 1.0414
× 1.0414
(d) molar enthalpy. = − 4.79 =
= − 4.79− 4 .79
(IIT-JEE 2009)

Chapter-6.indd 178 8/4/2016 10:33:36 AM


Advanced Level Problems 179

The ratio is 14. The reversible expansion of an ideal gas under adiabatic and
w d 8.65 isothermal conditions is shown in the below figure. Which of
= = 1.80 = 2 the following statement(s) is (are) correct?
w s 4.79
(p1, V1, T1)
11. The species which by definition has zero standard molar
enthalpy of formation at 298 K is
(a) Br2(g) (b) Cl2(g) (c) H2O(g) (d) CH4(g) isothermal
p
(IIT-JEE 2010) adiabatic (p2, V2, T2)
Solution (p3, V3, T3)
(b) Cl2 is a gas at 298 K while Br2 is a liquid.
12. Match the transformations in Column I with appropriate
V
options in Column II

Column I Column II (a) T1 = T2 (b) T3 > T1


(c) wisothermal > wadiabatic (d) ∆Uisothermal > ∆Uadiabatic
(a) CO2 (s) → CO2 (g) (p) Phase transition
(IIT-JEE 2012)
(b) CaCO3 (s) → CaO(s) + CO2 (g) (q) Allotropic change Solution
(c) 2H(g) → H2 (g) (r) ΔH is positive (a), (d) For isothermal process T1 = T2. Work done in iso-
thermal process is less than adiabatic process. In case of
(d) P( white , solid) → P(red, solid) (s) ΔS is positive isothermal process, the temperature remains constant so
(t) ΔS is negative there is no change in the internal energy, whereas in case
of adiabatic process expansion occurs through internal
(IIT-JEE 2011) energy.
Solution
(a) → p, r, s; (b) → r, s; (c) → t; (d) → q, r, t 15. The standard enthalpies of formation of CO2(g), H2O(l) and
glucose(s) at 25°C are −400 k J mol−1, −300 k J mol−1 and −1300
CO2 (s) → CO2 (g) ; ΔH = +ve, phase transition and ΔS = +ve.
k J mol−1, respectively. The standard enthalpy of combustion
CaCO3 (s) → CaO(s) + CO2 (g) ; ΔH = +ve, no phase transition per gram of glucose at 25°C is
and ΔS = +ve. (a) +2900 k J (b) −2900 k J
2H(g) → H2 (g); Two particles give one gaseous particle, so ΔS (c) −16.11 k J (d) +16.11 k J
= −ve. (JEE Advanced 2013)
P( white , solid) → P(red, solid) ; ΔH = +ve, ΔS = +ve, allotropic Solution
change, that is, phosphorus is getting changed from one allo-
trope form into another. (c) The reaction involved is
C6H12O6 (s) + 6O2 (g) → 6CO2 (g) + 6H2O(l)
13. For an ideal gas, consider only p−V work in going from an ini-
tial state X to the final state Z. The final state Z can be reached
∆H = Σ∆H(products) − Σ∆H(reactants)
by either of the two paths shown in the below figure. Which
of the following choice(s) is(are) correct? [Take ∆S as change = 6( −400 ) + 6( −300 ) − ( −1300) = − 2900 kJ mol−1
in entropy and w as work done.] Now, 1 mol of glucose = 180 g of glucose, so enthalpy of
X Y combustion per gram of glucose at 25°C is

−2900
∆H = = − 16.11 kJ g−1
p (atm)

180
Z
Paragraph for Questions 16 and 17: A fixed mass m of a gas is
subjected to transformation of states from K to L to M to N and
V(L) back to K as shown in the figure
(a) ∆SX → Z = ∆SX → Y + ∆SY → Z (b) w X → Z = w X → Y + w Y → Z
K L
(c) w X→ Y →Z = w X→ Y (d) ∆SX → Y → Z = ∆SX → Y
(IIT-JEE 2012)
Solution p
(a), (c) As entropy is a state function and is additive
∆SX → Z = ∆SX → Y + ∆SY → Z
N M
On moving from Y to Z, the work done is zero as the volume is
kept constant (isochoric process) V
w X→ Y →Z = w X→ Y (JEE Advanced 2013)

Chapter-6.indd 179 8/4/2016 10:33:42 AM


180 Chapter 6 Thermodynamics

16. The succeeding operations that enable this transformation Solution


of states are (a), (b), (c) As the vessel is thermally insulated (so, q = 0).
(a) heating, cooling, heating, cooling. Therefore, ΔU = 0, ΔT = 0 ⇒ T1 = T2. According to combined
(b) cooling, heating, cooling, heating.
(c) heating, cooling, cooling, heating. gas law equation,
(d) cooling, heating, heating, cooling. p2V2 p1V1
= ⇒ p2V2 = p1V1
T2 T1
Solution
(c) Heating, cooling, cooling, heating 19. For the process H2O(l) → H2O(g) at T = 100°C and 1 atmos-
phere pressure, the correct choice is
17. The pair of isochoric processes among the transformation of
(a) ∆Ssystem > 0 and ∆Ssurroundings > 0
states is
(a) K to L and L to M (b) L to M and N to K (b) ∆Ssystem > 0 and ∆Ssurroundings< 0
(c) L to M and M to N (d) M to N and N to K (c) ∆Ssystem < 0 and ∆Ssurroundings > 0
Solution (d) ∆Ssystem < 0 and ∆Ssurroundings < 0
(b) (JEE Advanced 2014)
Solution
K L (b) We know that ΔSsystem + ΔSsurroundings = 0
⇒ ΔSsystem = −ΔSsurroundings. Therefore, ΔSsystem > 0 and
p ΔSsurroundings < 0.
20. Match the thermodynamic processes given under Column I
N M with the expressions given in Column II
Column I Column II
V
(a) Freezing of water at 273 K and 1 atm (p) q = 0
As the term pV increases, the temperature
(b) Expansion of 1 mol of an ideal gas into a (q) w = 0
increases; and when pV decreases with decrease in
vacuum under isolated conditions
temperature.
The term pV increases K → L, so this involves heating and (c) Mixing of equal volumes of two ideal (r) ∆Ssys < 0
is an isobaric process. gases at constant temperature and pres-
It increases from N → K, so this involves heating and is an sure in an isolated container
isochoric process. (d) Reversible heating of H2(g) at 1 atm (s) ∆U = 0
It decreases from M → N, so this involves cooling and is from 300 K to 600 K, followed by reversi-
an isobaric process. ble cooling to 300 K at 1 atm (t) ∆G = 0
It decreases from L → M, so this involves cooling and is
an isochoric process. (JEE Advanced 2015)

18. An ideal gas in a thermally insulated vessel at internal Solution


pressure = p1, volume = V1 and absolute temperature = T1 (a) → r, t; (b) → p, q, s; (c) → p, q, s; (d) → p, q, s, t
expands irreversibly against zero external pressure, as shown During freezing of water entropy decreases, ∆Ssys < 0, ∆G = 0.
in the below diagram. The final internal pressure, volume and In vacuum, free expansion work done = 0. So, q = 0 and ∆U = 0.
absolute temperature of the gas are p2, V2 and T2, respec-
For an ideal gas in isolated condition q = 0, DU = 0, so as per
tively. For this expansion,
first law w = 0.
The process is cyclic through unique reversible path, hence
pext = 0 ∆U = 0 (cyclic), ∆G = 0.
w = 0 (positive and negative work are same and opposite)
pext = 0 Irreversible and q = 0.
21. 100 mL of a liquid contained in an isolated container at a
pressure of 1 bar. The pressure is steeply increased to 100
(p1, V1, T1) Thermal (p2, V2, T2)
bars. The volume of the liquid is decreased by 1 mL at this
insulation constant pressure. Find the ΔH and ΔU.
Solution
(a) q = 0 (b) T2 = T1
(c) p2V2 = p1V1 (d) p2V2g = p1V1g
∆H = ∆U + ∆( pV ) ⇒ ∆pV = p2V2 − p1V1
(JEE Advanced 2014) p1 = 1 bar , V1 = 100 mL

Chapter-6.indd 180 8/4/2016 10:33:46 AM


Advanced Level Problems 181

p2 = 100 bar , V2 = 99 mL In adiabatic condition, ∆q = 0; w = ∆U . Therefore,


3
∆pV = 100 × 99 − 1 × 100 = 9800 bar mL = 980 J (1bar L = 100 J) − pext (V1 − V2 ) = R(T1 − T2 ) (Since the expansion work
2
∆H = 10 + 980 = 990 J is negative)
where T2 is final temperature, T1 is initial temperature
22. For which one of the following equations is ∆rHo equal to ∆f Ho?
3 3
(a) CH4 (g) + 2Cl2 (g) → CH2Cl2 (l) + 2HCl(g) − pext (V1 − V2 ) = RT1 − RT2
2 2
(b) Xe(g) + 2F2 (g) → XeF4 (g) pext = 1 atm; V2 = 3 L ; V1 = 1 L
2 2
(c) 2CO(g) + O2 (g) → 2CO2 ( g) Solving, T2 = T1 − = T1 −
(3/2)R 1.5 × 0.0821
(d) N2 (g) + O3 (g)  N2O3 (g)
2
Therefore, Tfinal = T −
Solution 1.5 × 0.082
(b) For this reaction, ΔrH = Δf H as it involves the formation of
one mole of product from its elements in their standard 27. The heat of vapourization of mercury is 60.7 k J mol−1. For
state. Hg(l), S° = 76.1 J mol−1 K−1 and for Hg(g), S° = 175 J mol−1 K−1.
Estimate the normal boiling point of liquid mercury.
23. Spontaneous adsorption of a gas on solid surface is an exo-
Solution
thermic process because
(a) ΔH increases for system. For the vapourization process in particular, and for any pro-
(b) ΔS increases for gas. cess in general, we have:
(c) ΔS decreases for gas. ∆G = ∆H − T∆S
(d) ΔG increases for gas.
If the temperature is taken to be that at which equilibrium is
Solution obtained, that is, the temperature of the boiling point (where
(c) For a spontaneous process, ΔG should be negative. Also, liquid and vapour are in equilibrium with one another), then
the process is exothermic (means ΔH is also negative). we also have the result that ∆G is equal to zero:
For adsorption of gas to be spontaneous, the entropy ∆H
∆G = 0 = ∆H − T∆S, or Teq =
should decrease in accordance with the relation ∆S
ΔG = ΔH − TΔS. We know ∆H to be 60.7 k J mol−1; we need the value for ∆S in
units k J mol−1 K−1:
24. 2 mol of an ideal gas expanded isothermally and reversibly
∆S° = (sum S°[Products]) − (sum S°[Reactants])
from 1 L to 10 L at 300 K. What is the enthalpy change?
= S°[Hg(g)] − S°[Hg(l)]
(a) 4.98 k J (b) 11.47 k J (c) −11.47 k J (d) 0 k J
= (175 × 10−3 k J mol−1 K−1) − (76.1 × 10−3 k J mol−1 K−1)
Solution = 98.9 × 10−3 k J mol−1 K−1
(d) H = U + pV; ∆H = ∆U + ∆( pV ) ⇒ ∆H = ∆U + nR ∆T So, the normal boiling point of liquid mercury is
For isothermal and reversible expansion ∆U = 0 and 60.7 kJ mol−1
Teq = = 614 K(341°C)
∆T = 0, so, ∆H = 0 + 0 = 0 98.9 × 10 −3 mol−1 K
25. ΔHvap = 30 k J mol−1 and ΔSvap = 75 J mol−1 K−1. Find tempera- 28. Assume an ideal gas obeys p/V = constant. This gas confined
ture of vapour, at one atmosphere. in piston fitted cylinder having initial volume and pressure
(a) 400 K (b) 350 K (c) 298 K (d) 250 K 2 litre and 1 atm is allowed to expand to occupy finally 6 litres.
Calculate the work done by the system.
Solution
(a) ∆G = ∆H − T ∆S ; At equilibrium ΔG = 0, Solution

From the given data calculate work done in terms of volume


∆H 30 × 103 J mol−1
T= = = 4 00 K
∆S 75 J mol−1 K −1 pressure relationship, as
V2
26. One mole of a monatomic ideal gas expands adiabatically at w = − ∫ p∆V
initial temperature T against a constant external pressure of 1 V1
atm from one litre to three litres. The final temperature of the 1
gas is Given, p/V = K, therefore, K = atm L−1 (Initial condition)
2
(a) T (b) T/3(5R/2) V2 V2 V2
(c) T−2/(1.5 × 0.082) (d) T + 2/(1.5 × 0.082) V2  V2 V2 
w = − ∫ p∆V = − ∫ KV ∆V = − K ∫ = −K  2 − 1 
Solution V1 V1 V1
2  2 2

(c) Work done against constant pressure p = pext (V1 − V2 ) 1  36 4 


= −  −  = − 8 L atm
2  2 2

Chapter-6.indd 181 8/4/2016 10:33:54 AM


182 Chapter 6 Thermodynamics

PRACTICE EXERCISE
Level I 10. Which one of the following statements is false?
(a) Work is a state function.
Single Correct Choice Type (b) Temperature is a state function.
1. The molar entropy of vapourization of acetic acid is (c) Work appears at the boundary of the system.
14.4 cal K−1 mol−1 at its boiling point 118°C. The latent heat of (d) Change in the state is completely defined when the ini-
vapourization of acetic acid is tial and final states are specified.
(a) 49 cal g−1 (b) 64 cal g−1
−1 11. The bond enthalpies of F2, Cl2, Br2 and I2 are 155.4 k J mol−1,
(c) 94 cal g (d) 84 cal g−1
243.6 k J mol−1, 193.2 k J mol−1, and 151.2 k J mol−1, respec-
2. The enthalpy of a reaction at 273 K is −3.57 k J. What will be the tively. The strongest bond is
enthalpy of reaction at 373 K if ΔCp = 0? (a) F−F (b) Cl−Cl (c) Br−Br (d) I−I
(a) −3.57 (b) 0 12. For A → B, ∆H = 4 kcal mol−1, ∆S = 10 cal mol−1 K−1. Reaction
373
(c) −3.57 × (d) −375 is spontaneous when temperature can be
273
(a) 400 K (b) 300 K (c) 500 K (d) 200 K
3. The sublimation energy of I2(s) is 57.3 k J mol−1 and the
13. Which of the reaction defines ∆ f H °?
enthalpy of fusion is 15.5 k J mol−1. The enthalpy of vapouriza-
tion of I2 is 1 1
(a) H2 (g) + F2 (g) → HF(g)
(a) 41.8 k J mol−1 (b) −41.8 k J mol−1 2 2
(c) 72.8 k J mol−1 (d) −72.8 k J mol−1 1
(b) CO(g) + O2 (g) → CO2 (g)
4. In thermodynamics, a process is called reversible when 2
(a) surroundings and system change into each other. (c) C + O2 (g) → CO2 (g)
(b) there is no boundary between system and surroundings. Diamond
(c) the surroundings are always in equilibrium with the
system. (d) N2 (g) + 3H2 (g) → 2NH3 (g)
(d) the system changes into the surroundings spontaneously.
14. The gas absorbs 100 J heat and is simultaneously com-
5. Which process is accompanied by a decrease in the entropy of pressed by a constant external pressure of 1.50 atm from 8 L
the system? to 2 L in volume. Hence, ∆U will be:
(a) A cup of juice spreading across the floor after being (a) −812 J (b) 812 J (c) 1011 J (d) 911 J
spilled
(b) Soaking up a chemical spill with an absorbent material 15. An ideal gas is allowed to expand both reversibly and irre-
(c) The sublimation of iodine crystals versibly in an isolated system. If Ti is the initial temperature
(d) Water boiling in a steam kettle and Tf is the final temperature, which of the following state-
ments is correct?
6. Which one of the following is not applicable for a thermo- (a) (Tf )rev = (Tf )irrev.
chemical equation? (b) Tf = Ti for both reversible and irreversible processes.
(a) It tells about the physical state of the reactants and (c) (Tf )irrev > (Tf )rev.
products. (d) Tf > Ti for a reversible process but Tf = Ti for an irreversi-
(b) It tells whether the reaction is spontaneous. ble process.
(c) It tells whether the reaction is exothermic or endothermic.
(d) It tells about the allotropic form (if any) of the reactants. 16. Which of the following provide exceptions to the third law of
thermodynamics?
7. If the standard Gibbs energy change for a reaction is (a) CO (b) Ice
1.546 k J mol−1 at 500°C, then the value of standard equilib- (c) CO2 (d) All of these
rium constant for the reaction is
(a) antilog(0.105) (b) antilog(−0.105) 17. The heat evolved in the combustion of 112 L at STP of water
(c) antilog(0.241) (d) antilog(−0.241) gas (CO + H2 in the ratio of 1:1 by volume) is
(Given: Heat of combustion of H2 and CO is −241.8 k J mol−1
8. If water kept in an insulated vessel at −10°C suddenly freezes, and −283 k J mol−1, respectively.)
the entropy change of the system (a) −1414.0 k J (b) −12090 k J
(a) decreases. (c) −1312.0 k J (d) −524.8 k J
(b) increases.
(c) is zero. 18. Among mass, volume, density and specific volume of a gas,
(d) equals to that of the surroundings. the intensive properties are
(a) density and specific volume.
9. What is the sign of ∆G for the process of melting of ice at 280 K? (b) volume and density.
(a) ΔG > 0 (b) ΔG = 0 (c) specific volume and mass.
(c) ΔG < 0 (d) None of these (d) density only.

Chapter-6.indd 182 8/4/2016 10:33:56 AM


Practice Exercise 183

19. H2 (g) + 21 O2 (g) → H2O(l); ∆H298K = − 68.32 kcal 29. Some of the thermodynamic parameters are state variables,
while some are process variables. Some groups of the param-
Enthalpy of vapourization of water at 1 atm and 25°C is eters are given. Choose the correct one.
10.52 kcal. The standard enthalpy of formation (in kcal) of 1 (a) State variables: Temperature, Number of moles
mol of water vapour at 25°C is Process variables: Internal energy, work done by the gas
(a) 10.52 (b) −78.84 (c) +57.80 (d) −57.80 (b) State variables: Volume, temperature
Process variables: Internal energy, work done by the gas
20. The heat of formation of HCl(g) from the reaction
(c) State variables: work done by the gas, heat rejected by
H2(g) + Cl2(g) → 2HCl(g); ΔH = −44 kcal is
the gas.
(a) +44 kcal (b) −44 kcal (c) +22 kcal (d) −22 kcal
Process variables: Temperature, volume
21. The bond energies of C C and C C at 298 K are 590 and (d) State variables: Internal energy, volume
331 k J mol−1, respectively. The enthalpy of polymerization per Process variables: Work done by the gas, heat absorbed
mole of ethylene is by the gas
(a) −70 k J (b) −72 k J (c) 72 k J (d) −68 k J 30. If 1 kcal of heat is added to 1.2 L of oxygen in a cylinder at con-
22. Given that stant pressure of 1 atm, the volume increases to 1.5 L. Hence,
C + O2 → CO2; ∆H° = −x k J ΔU for this process is
2CO + O2 → 2CO2; ∆H° = −y k J (a) 0.993 kcal (b) 1.0073 kcal
(c) 0.0993 kcal (d) 1.00073 kcal
The enthalpy of formation of carbon monoxide will be
31. The enthalpy of neutralization of HCl by NaOH is −55.9 k J
2x − y y − 2x mol−1. If the enthalpy of neutralization of HCN by NaOH is
(a) y − 2x (b) (c) (d) 2x − y
2 2 −12.1 k J mol−1, the enthalpy of dissociation of HCN is
(a) −43.8 k J (b) 43.8 k J (c) 68 k J (d) −68 k J
23. In an adiabatic process, no transfer of heat takes place
32. One mole of a non-ideal gas undergoes a change of state
between the system and surroundings. Choose the correct
(2.0 atm, 3.0 L, 95 K) → (4.0 atm, 5.0 L, 245 K) with a change
option for free expansion of an ideal gas under adiabatic con-
in internal energy, ∆U = 30.0 L atm. The change in enthalpy in
dition from the following:
the process in litre atm is
(a) q = 0, ΔT ≠ 0, w = 0 (b) q ≠ 0, ΔT = 0, w = 0
(a) 40.0
(c) q = 0, ΔT = 0, w = 0 (d) q = 0, ΔT < 0, w ≠ 0
(b) 42.3
24. On heating 128 g of oxygen gas from 0°C to 100°C, CV and Cp (c) 44.0
on an average are 5 cal mol−1 deg−1 and 7 cal mol−1 deg−1. (d) not defined, because pressure is not constant.

The value of ΔU and ΔH are, respectively, 33. When 1 mol of CO2 (g) occupying a volume of 10 L at 27°C is
allowed to expand under adiabatic conditions, temperature
(a) 2800 cal, 2000 cal (b) 2000 cal, 2800 cal
falls to −123°C. Hence, the final volume of the gas will be
(c) 280 cal, 200 cal (d) 2369 cal, 2589 cal
(a) 20 L (b) 40 L (c) 60 L (d) 80 L
25. Given that
34. Find temperature of vapour at 1 atm. Given that ∆H =
A(s) → A(l); ∆H = x 30 k J mol−1 and ∆S = 75 J K−1 mol−1.
A(l) → A(g); ∆H = − y (a) 400 K (b) 350 K (c) 298 K (d) 250 K

The enthalpy of sublimation of A will be 35. The intermediate SiH2 is formed in the thermal decomposi-
(a) x + y (b) x − y (c) x or y (d) −(x + y) tion of silicon hydrides. Calculate ∆ f H ° of SiH2 with the help
of following reactions
26. If the enthalpy change for the transition of liquid water to Si2H6(g) + H2(g) → 2SiH4(g); ∆H° = −11.7 kJ mol−1
steam is 30 k J mol−1 at 27°C, the entropy change for the pro- SiH4(g) → SiH2(g) + H2(g); ΔH° = + 239.7 kJ mol−1;
cess would be (in J mol−1 K−1) (Given Δf H of Si2H6(g) = +80.3 kJ mol−1)
(a) 100 (b) 10 (c) 1.0 (d) 0.1 (a) 353 k J mol−1 (b) 321 kJ mol−1
(c) 198 kJ mol −1 (d) 274 kJ mol−1
27. The standard enthalpy of formation of NO2(g) and N2O4(g)
are 8.0 kcal mol−1 and 2.0 kcal mol−1, respectively. The heat of 36. Consider the below figure for a gas.
dimerization of NO2 in kcal is
(a) 10.0 (b) −6.0 (c) +14.0 (d) −14.0 1 B
28. Over a wide temperature range, the reaction, M2O3(s) + C(s) 2
→ M(s) + CO2(g), is spontaneous at low temperatures but 3
p
non-spontaneous at high temperatures. If we assume that,
since the physical states do not change, the values of ∆HTo
and ∆STo are constant over this temperature range, we can 4
then deduce that 5
A
(a) ∆H < 0 and ∆S > 0 (b) ∆H < 0 and ∆ S < 0
(c) ∆ H > 0 and ∆ S < 0 (d) ∆ H > 0 and ∆ S > 0
V

Chapter-6.indd 183 8/4/2016 10:33:58 AM


184 Chapter 6 Thermodynamics

The change in entropy ∆S in going from state A to state B 46. 1 mol of NH3 gas at 27°C is expanded under adiabatic con-
(a) is the same for path 1 through 3. ditions to make volume 8 times (g = 1.33). Final temperature
(b) is different for each of the paths. and work done, respectively, are
(c) is smallest for path 3 because it is the shortest path. (a) 150 K, 900 cal (b) 150 K, 400 cal
(d) is the maximum for path 5 and smallest for path 3. (c) 250 K, 1000 cal (d) 200 K, 800 cal

37. The enthalpy of dissociation of benzene to elements is 47. An ideal gas can be expanded from an initial state to a cer-
5535 k J mol−1. The bond enthalpies of C C, C C, and C H tain volume through two different processes, (I) pV2 = K and
are 347.3 k J, 615.0 k J, and 416.2 k J, respectively. The reso- (II) p = KV2, where K is a positive constant. Then, choose the
nance energy of benzene is correct option from the following:
(a) 1.51 k J (b) 15.1 k J (c) 151 k J (d) 1511 k J (a) Final temperature in (I) will be greater than in (II).
(b) Final temperature in (II) will be greater than in (I).
38. For a reversible isothermal expansion of an ideal gas (c) Work done by the gas in both the processes would be
(a) ΔSsys = ΔSsurr = positive (b) ΔSsys = −ΔSsurr equal.
(c) ΔSsys = ΔSsurr = negative (d) ΔSsys = ΔSsurr = 0 (d) Total heat given to the gas in (I) is greater than in (II).
39. Thermodynamics is not concerned about 48. Calculate the free energy change for the following reac-
(a) energy changes involved in a chemical reaction. tion at 298 K and 1 atm, given that free energy of formation
(b) the extent to which a chemical reaction proceeds. ( ∆ f G o ) in k J mol−1 is CH4(g) = −50.752, O2(g) = 0, CO2(g)
(c) the rate at which a reaction proceeds. = −394.359 and H2O(g) = −228.572:
(d) the feasibility of a chemical reaction.
CH4 (g) + 2O2 (g) → CO2 (g) + 2 H2O(g)
40. Which statement is true?
(a) −868.859 k J mol−1
(a) Spontaneous changes are always accompanied by an
(b) −815.751 k J mol−1
increase in the entropy of the system.
(c) −985.25 k J mol−1
(b) Spontaneous changes are always accompanied by a
(d) −800.751 k J mol−1
decrease in the entropy of the system.
(c) Spontaneous changes are always accompanied by an 49. If So for H2, Cl2, and HCl are 0.13 k J K−1 mol−1, 0.22 k J K−1
increase in the enthalpy of the system. mol−1, and 0.19 k J K−1mol−1, respectively, then the total
(d) Most highly exothermic chemical reactions are also change in standard entropy for the following reaction is
spontaneous chemical reactions H2(g) + Cl2(g) → 2HCl(g)
(a) 30 J K−1 mol−1 (b) 40 J K−1 mol−1
41. Using only the following data: (c) 60 J K−1 mol−1 (d) 20 J K−1 mol−1
(I) Fe2O3 (s) + 3CO(g)  2Fe(s) + 3CO2 (g); ∆H° = − 26.8 kJ
50. The densities of graphite and diamond at 298 K are 2.25 g cm−3
(II) Fe(s) + CO(g)  FeO(s) + CO(g); ∆H° = + 16.5kJ and 3.31 g cm−3, respectively. If the standard Gibbs energy dif-
ference (ΔG°) is equal to 1895 J mol−1, the pressure at which
the ∆H° value, in kilojoules, for the reaction Fe2O3(s) + graphite will be transformed into diamond at 298 K is
CO(g) → 2FeO(s) + CO2(g) is calculated as (a) 9.92 × 108 Pa (b) 9.92 × 107 Pa
(a) −43.3 (b) −10.3 (c) +6.2 (d) +10.3 (c) 9.92 × 106 Pa (d) 9.92 × 1010 Pa
42. The enthalpy of combustion of C2H4, C2H6, and H2 are 1
51. The enthalpy of reaction for A + O2 → AO is −50 kcal and
−1409.5 k J, −1558.3 k J, and −285.6 k J, respectively. The heat 2
of hydrogenation of ethane is 1
for A O + O2 → AO2 is −100 kcal. The enthalpy of reaction
(a) −136.8 k J (b) −13.68 k J (c) 273.6 k J (d) 1.368 k J 2
for A + O2 → AO2 is
43. A gas is allowed to expand under reversible adiabatic condi- (a) −50 kcal (b) −150 kcal
tions, then what quantity is zero for such a process? (c) 100 kcal (d) 150 kcal
(a) ΔG = 0 (b) ΔT = 0
52. For which change ∆H ≠ ∆U?
(c) ΔS = 0 (d) ΔU = 0
(a) H2 (g) + I2 (g)  2HI(g)
44. For the reaction, 2N2(g) + O2(g) → 2N2O, at 298 K ∆H is 164 k J (b) HCl(aq) + NaOH(aq) → NaCl(aq) + H2O(l)
mol−1. The ∆U of the reaction is (c) C(s) + O2 (g)  2CO2 (g)
(a) 166.5 k J mol−1 (b) 161.5 k J mol−1 (d) N2 (g) + 3H2 (g) → 2NH3 (g)
(c) 164.0 k J mol −1 (d) 169 k J mol−1
53. Given that
1 NH3 (g) + 3Cl2 (g)  NCl3 (g) + 3HCl(g); − ∆H1
45. Consider the reaction, H2 (g) + O2 (g) → H2O(l); ΔCp = 32
2
J K−1; ΔH at 27°C = −285.8 k J mol−1. N2 (g) + 3H2 (g)  2NH3 (g); − ∆H2
What will be the value of ΔH at 127°C? H2 (g) + Cl2 (g)  2HCl(g); ∆H3
(a) −289.0 k J mol−1 (b) −282.6 k J mol−1
(c) −317.0 k J mol−1 (d) −253.8 k J mol−1 The enthalpy of formation of NCl3(g) in terms of ΔH1, ΔH2,
and ΔH3 is

Chapter-6.indd 184 8/4/2016 10:34:04 AM


Practice Exercise 185

∆H2 3 p
(a) ∆ f H = − ∆H1 − − ∆H3
2 2 A B
∆H2 3 1 atm
(b) ∆ f H = ∆H1 + − ∆H3
2 2
∆H2 3
(c) ∆ f H = ∆H1 − − ∆H3
2 2 0.5 atm C
∆H2 3
(d) ∆ f H = − ∆H1 + + ∆H3
2 2
V
54. Find the maximum work available, in kilojoules, from the oxi- 20 L 40 L
dation of 1 mol of octane, C8H18(l), by oxygen to give CO2(g) (a) The entropy change for the overall process is 0 (zero).
and H2O(l) at 25°C and 1 atm. Given that free energy of forma- (b) ΔH > ΔU for the overall process.
tion (∆ f G°) in k J mol−1 is C8H18(g) = +17.3, O2(g) = 0, CO2(g) (c) Total work, w = q.
= −394.359 and H2O(l) = −237.2. (d) Total work = − 620.77 J.
(a) −5307 k J (b) −2575 k J (c) −5277 k J (d) −3327 k J
60. An adiabatic process involves
(a) no exchange of heat.
Level II (b) exchange of heat with change in volume.
Multiple Correct Choice Type (c) work done equal to change in internal energy.
(d) increase in temperature of the system under reversible
55. When a real gas expands adiabatically against a finite compression.
pressure,
(a) its internal energy increases. 61. Which of the following statement(s) is/are correct?
(b) its internal energy decreases. (a) The system of constant entropy and constant volume will
(c) its temperature always decreases. attain the equilibrium in a state of minimum energy.
(d) its entropy always remains constant. (b) The entropy of the universe is increasing.
(c) The process would be spontaneous when (∆S)U,V < 0,
56. For the C(s) + H2O(g) → CO(g) + H2 (g) reaction; ΔHo = +131 kJ; (∆U)S,V > 0.
ΔSo = +134 J K−1 (d) The process would be spontaneous when (∆S)U,V > 0,
Mark out the correct statement(s). (∆U)S,V < 0.
(a) Reaction is spontaneous even at room temperature. 62. Which of the following statements are correct?
(b) Reaction is not spontaneous at room temperature. (a) When ∆G = 0, the system is at equilibrium.
(c) Reaction is spontaneous above 705°C. (b) When ∆G < 0, the process will be spontaneous.
(d) ΔH outweighs the entropy factor TΔS at room (c) When ∆G is negative, the process is said to be exergonic.
temperature. (d) When ∆G is positive, the process is said to be
57. Choose the incorrect combination(s). endergonic.
63. When a solid melts there will be
ΔH ΔS Temperature (T) Spontaneity (a) an increase in enthalpy.
(a) + − Any T Non-spontaneous (b) a decrease in free energy.
(c) no change in enthalpy.
(b) − − Low T Non-spontaneous (d) a decrease in internal energy.
(c) + + Low T Spontaneous
64. Which statements are correct for neutralization of a strong
(d) − + Any T Spontaneous acid by a strong alkali in aqueous solution at 25°C.
(a) It is an endothermic process.
58. One mole of an ideal diatomic gas (CV = 5 cal) was trans- (b) It can be represented as H3O + (aq) + OH− (aq) → 2H2O(l)
formed from initial 25°C and 1 L to the state when temper- (c) The enthalpy change per mole of H2O formed is inde-
ature is 100°C and volume 10 L. Then for this process (R = 2 pendent of the acid or alkali used.
cal mol−1 K−1) (take calories as unit of energy and kelvin for (d) Some sort of energy is consumed to make the acid or
temperature). Mark out the correct one(s). alkali completely ionized.
(a) ∆H = 525 1
65. C(s) + O2 → CO(g); ∆H ° = − 26 kcal mol−1
2
373
(b) ∆S = 5 ln + 2 ln 10 1
298 CO(g) + O2 (g) → CO2 (g); ∆H ° = − 68 kcal mol−1
2
(c) ∆U = 525
Which is/are correct statement(s)?
(d) ∆G of the process cannot be calculated using given
(a) Heat of formation of CO2 is −68 kcal mol−1
information.
(b) Heat of combustion of C(s) is −26 kcal mol−1
59. Mark out the correct one(s) about the pV plot system shown (c) Heat of combustion of CO(g) is −68 kcal mol−1
in the following figure. (d) Heat of formation of CO(g) is −26 kcal mol−1

Chapter-6.indd 185 8/4/2016 10:34:07 AM


186 Chapter 6 Thermodynamics

66. Which of the following sets show the intensive properties? 72. When 2 mol of NO2 change into equilibrium mixture with
(a) Temperature and pressure N2O4, then ∆G° is
(b) Viscosity and surface tension (a) −5.40 k J (b) +5.40 k J (c) −6.24 k J (d) 6.24 k J
(c) Refractive index and specific heat
73. When 1 mol of N2O4 changes into equilibrium mixture with
(d) Volume and heat capacity
NO2, then ∆G° is
67. Which of the following process take(s) place with decrease in (a) −5.40 k J (b) 0.84 k J (c) −0.84 k J (d) 6.24
entropy?
74. Select out the correct statement.
(a) 3CO2(g) → 2O3(g)
(a) The conversion of N2O4 into 2NO2 is spontaneous.
(b) Sugar + Water → Solution
(b) The conversion of 2NO2 into N2O4 is non-spontaneous.
(c) 2SO2(g) + O2(g) → 2SO3(g)
(c) The attainment of equilibrium from N2O4 and from NO2
(d) H2(g) + Cl2(g) → 2HCl(g)
are both equally spontaneous.
68. In a given process on an ideal gas, dw = 0 and dq < 0, then for (d) The attainment of equilibrium from 2 mol of NO2 with
the gas N2O4 is more spontaneous than the conversion of 2NO2
(a) the volume will increase. into N2O4.
(b) the temperature will decrease.
Paragraph for Questions 75 to 77: Enthalpy of neutralization is
(c) the pressure will remain constant.
defined as the enthalpy change when 1 mol of H+ in dilute solution
(d) the internal energy will decrease.
combines with 1 mol of OH− to give rise to undissociated water,
Passage Type that is, H+(aq) + OH−(aq) → H2O(l) ∆H = −57.54 kJ mol−1 or −13.7
kcal mol−1. When 1 mol of strong monoprotic acid is mixed with
Paragraph for Questions 69 to 71: Figure below is a volume vs.
1 mol of strong base, the above neutralization reaction takes place.
temperature plot for one mole of gas.
When a weak acid/base reacts with a strong base/acid, the
A B release of heat is less than 57.54 k J mol−1. It is because of the fact
20 L
that these acids/bases are not completely ionized in the solution.
Some heat is consumed in ionizing them.
V
10 L 75. When 1 g equiv. of NaOH reacts with 0.33 g equiv. of HCl,
C enthalpy change is −4.6 cal. When 0.66 g equiv. of HCl is fur-
Graph for one mole gas
ther added, enthalpy change would be
(a) −13.7 cal (b) −9.2 cal (c) 13.7 cal (d) 9.2 cal
200 K 400 K
76. In the following reaction involving neutralization of HF
T
(a weak acid) with NaOH (a strong base).
69. The process A → B is HF(aq) + NaOH(aq) → NaF(aq) + H2O(l)
(a) isobaric. (b) isochoric. the heat of neutralization is −68.6 k J mol−1. This value is
(c) isothermal. (d) adiabatic. much higher than the heat of neutralization of strong acid
70. The pressure at C is with strong base (= −57.3 k J mol−1). This is because
(a) 3.284 atm (b) 1.642 atm (a) in aqueous solution, HF behaves as a strong acid.
(c) 0.0821 atm (d) 0.821 atm (b) there is hydration of F− ion in aqueous solution, and pro-
cess is exothermic.
71. Work done in the process C → A is (c) non-solubility of HF in water.
(a) zero (b) 8.21 L atm (d) formation of hydrogen bonding among HF molecules.
(c) 16.2 L atm (d) cannot be predicted
77. Heat of dissociation of CH3COOH is 0.005 kcal g−1; hence,
Paragraph for Questions 72 to 74: Consider the following reac- enthalpy change when 1 mol of Ca(OH)2 is completely neu-
tion N2O4(g) →2NO2(g). The Gibbs energy of the reaction occur- tralized by CH3COOH is
ring at 298 K and 1 atm has been plotted against the fraction of (a) −27.4 kcal (b) −13.6 kcal
N2O4 dissociated as shown in below figure. (c) −26.8 kcal (d) −27.1 kcal
6 Paragraph for Questions 78 to 80: The thermodynamic property
that measures the extent of molecular disorder is called entropy.
5 The direction of a spontaneous process for which the energy is
∆G° (kJ mol−1)

constant is always the one that increases the molecular disorder.


∆G° = 5.40 kJ

4
Entropy change of phase transformation can be calculated using
3 Trouton’s formula (∆S = ∆H/T). In the reversible adiabatic pro-
cess, however, ∆S will be zero. The rise in temperature in isobaric
2 and isochoric conditions is found to increase the randomness or
1 N2O4 entropy of the system.
0.84 kJ
T 
∆S = 2.303C log  1 
0.2 0.4 0.6 0.8 1.0  T2 
Fraction of N2O4 dissociated where C = Cp or CV .

Chapter-6.indd 186 8/4/2016 10:34:10 AM


Answer Key 187

78. The entropy change in an adiabatic process is 83. Match the process with the expression.
(a) zero.
(b) always positive. Column I Column II
(c) always negative. (a) For spontaneous
(p) ∑ ∆ bondH (reactants) − ∑ ∆bond
(d) sometimes positive and sometimes negative.
H (products)
79. If water in an insulated vessel at −10°C suddenly freezes, the (b) For endothermic (q) ∆ r H = ∆ rU
entropy change of the system will be
(a) +10 J K−1 mol−1 (c) Bond dissociation energy (r) ∆G < 0
(b) −10 J K−1 mol−1 (d) For solids and liquids (s) ∑ H (products) > ∑ H (reactants)
(c) zero. in a thermochemical
(d) equal to that of surroundings. reaction
80. The melting point of a solid is 300 K and its latent heat of
fusion is 600 cal mol−1. The entropy change for the fusion of 84.
1 mol of the solid (in cal K−1) at the same temperature would Column I Column II
be
(a) Reversible isothermal (p) w = −2.303nRT log (p2/p1)
(a) 200 (b) 2 (c) 0.2 (d) 20
process
Matrix-Match Type (b) Reversible adiabatic (q) w = 0
process
81. Match the column correctly:
(c) Irreversible adiabatic (r) w = −2.303nRT log (V2/V1)
Column I Column II process
(a) Extensive properties (p) Density (at constant p, V, (d) Irreversible isothermal (s) w = nC (T − T )
and T) process
V ,m 2 1

(b) Closed system (q) Bursting of tire


(t) w = − popp (V2 − V1)
(c) Path function (r) Additive in nature
(d) Adiabatic (s) U ≠ constant Integer Type
(e) Intensive properties (t) Heat
85. Standard Gibbs energy change for a reaction is zero. The value
82. Match the signs of ∆H and ∆S with the nature of the reaction. of equilibrium constant is ___.
86. A gas expands against a constant external pressure so that
Column I Column II the work done is 607.8 J. The work done in litre atmosphere is
(a) Negative and negative (p) Spontaneous only at low ___.
temperature.
87. The amount of ice (in g) that remains when 52 g of ice is
(b) Negative and positive (q) Spontaneous only at high added to 100 g of water at 40°C is ___.
temperature.
88. One mole of monatomic perfect gas, initially at 1.0 atm pres-
(c) Positive and positive (r) Spontaneous at all sure and at 300 K is heated reversibly to 400 K at constant vol-
temperature. ume. Calculate the change in internal energy and the work
(d) Positive and negative (s) Non-spontaneous at all done.
temperature.

ANSWER KEY

Level I
1. (c) 2. (a) 3. (a) 4. (c) 5. (b) 6. (b)
7. (b) 8. (c) 9. (a) 10. (a) 11. (b) 12. (c)
13. (a) 14. (c) 15. (b) 16. (a) 17. (c) 18. (a)
19. (d) 20. (d) 21. (b) 22. (c) 23. (c) 24. (b)
25. (b) 26. (a) 27. (d) 28. (b) 29. (d) 30. (a)
31. (b) 32. (c) 33. (d) 34. (a) 35. (d) 36. (a)
37. (c) 38. (b) 39. (c) 40. (d) 41. (c) 42. (a)
43. (c) 44. (a) 45. (b) 46. (a) 47. (b) 48. (d)
49. (a) 50. (d) 51. (b) 52. (d) 53. (a) 54. (a)

Chapter-6.indd 187 8/4/2016 10:34:11 AM


188 Chapter 6 Thermodynamics

Level II
55. (b), (c), (d) 56. (b), (c), (d) 57. (b), (c) 58. (a), (b) 59. (a), (d) 60. (a), (b),(c)
61. (a), (b), (d) 62. (a), (b), (c), (d) 63. (a), (b) 64. (b), (c) 65. (c), (d) 66. (a), (b), (c)
67. (a), (c) 68. (b), (d) 69. (b) 70. (b) 71. (c) 72. (c)
73. (c) 74. (d) 75. (b) 76. (b) 77. (c) 78. (a)
79. (c) 80. (b) 81. (a) → r; (b) → t; (c) → s; (d) → q; (e) → p 82. (a) → p, (b) → r, (c) → q, (d) → s
83. (a) → r; (b) → s; (c) → p; (d) → q 84. (a) → p, r; (b) → s; (c) → q; (d) → t 85. (1) 86. (6)
87. (44) 88. (0)

HINTS AND EXPLANATIONS


Level I 13. (a) ∆ f H° is the enthalpy of formation; in standard state of
Single Correct Choice Type 1 mol of a substance from its constituent elements in their
1 1
∆H ∆H standard states, for example, H2 (g) + F2 (g) → HF(g) .
1. (c) ∆S = ⇒ 14.4 = ⇒ ∆H = 391 × 14.4 2 2
T (K ) 391 14. (c) ΔH = ΔU + pΔV ⇒
5630
= 5630 cal mol−1 = = 93.8 caal g−1 8.314 JK −1 mol−1
60 100 = ∆U + 1.5 atm (2 − 8 ) L ×
2. (a) Using Kirchhoff’s equation, 0.0821 L atm K −1 mol−1
⇒ ΔU = 1011.4 J
H2 = H1 + (T2 – T1)ΔCp
= –3.57 + (373 – 273) × 0 15. (b) In an ideal gas, there are no intermolecular forces of
= –3.57 kJ mol–1 attraction. Hence, Tf = Ti for both reversible and irreversi-
ble processes.
3. (a) ΔHsub = ΔHfus + ΔHvap
16. (a) The third law of thermodynamics states that the entropy
ΔHvap = ΔHsub – ΔHfus = (57.3 – 15.5) kJ mol–1 = 41.8 kJ mol–1.
of a perfectly crystalline solid approaches zero as the
4. (c) In a reversible process, the driving force and opposing force temperature approaches absolute zero. However, some
of that process are nearly equal, and hence, the system and solids do not have zero entropy even at absolute zero, for
the surrounding always remain in equilibrium with each other. example, CO, NO, H2O, etc.
5. (b) Conceptual 17. (c) 1 112
∆Hcombustion = ( −241.8 − 283) kJ mol−1 × mol
2 22.4
6. (b) A balanced chemical equation that indicates the
= − 1312.0 kJ
enthalpy change accompanying the reaction is called a
thermochemical equation. It gives the physical states of
18. (a) Density is mass per unit volume and specific volume is
the reactants and products, and its ΔH° value is true only
volume per unit mass.
when the coefficients of the reactants and products are
19. (d) The required equation is
taken to mean moles of the corresponding substances.
This ΔH° value provides the information that the reac- 1
H2 (g) + O2 (g) → H2O(g)
tion is exothermic or endothermic. It gives information 2
about the allotropic form (if any) of the reactants. Thus, The equation can be obtained as
a thermochemical reaction does not tell about whether 1
H2 (g) + O2 (g) → H2O(l); ∆H298K = − 68.32 kcal (1)
the reaction is spontaneous or not. 2
7. (b) ΔG = –2.303 RT log K H2O(l) → H2O(g); ∆H = 10.52 kcal (2)
1.546 × 10–3 = −2.303 × 8.314 × 773 × log K Adding Eqs. (1) and (2),
K = antilog (−0.105). 1
H2 (g) + O2 (g) → H2O(g); ∆H = − 57.80 kcal
8. (c) As the system is insulated from the surroundings, no 2
heat enters or leaves the system. Hence, ΔS = q/T = 0. 20. (d) The heat of formation of HCl is to be calculated on per
mole of HCl produced
9. (a) Melting of ice at 280 K is a non-spontaneous process. 44
Heat of formation of HCl = = − 22 kcal
10. (a) Work is not a state function; instead it is dependent on 2
the path of the reaction. (as ∆H = −44 kcal per 2 mol of HCl)

11. (b) The larger the value of bond enthalpy, the stronger the 21. (b) Polymerization reaction nCH2 CH2 → (CH2 CH2)n
bond. 1 mol of C C bond is broken and 2 mol of C C bonds
are formed per mole of ethylene.
12. (c) As ΔG = −ve for spontaneous change and ΔG = ΔH − TΔS.
∆H = 590 − 2 × 331 = 590 − 662 (590 is spent, and 2 × 331
Thus, we have
is formed).
∆H 4000
T ∆S > ∆H ⇒ T > ⇒T > ⇒ T > 400 K = −72 k J mol−1 of ethylene.
∆S 10

Chapter-6.indd 188 8/4/2016 10:34:15 AM


Hints and Explanations 189

22. (c) C + O2 → C O2 ∆H o = − x kJ (1) Enthalpy of dissociation of HCN


o = ∆H2 − ∆H1 = −12.1 − (−55.9) = +43.8 kJ
2CO + O2 → 2C O2 ∆H = − y kJ (2)
32. (c) ∆H = ∆U + p∆V = 30 + (4 × 5) − (2 × 3) = 44 L atm
Subtracting Eq. (2) from 2 × Eq. (1) results
33. (d) For adiabatic expansion,
o
2C + O2 → 2C O ∆H = y − 2 x ( 3) T2  V1 
g −1
=
Dividing Eq. (3) by Eq. (2) gives the desired equation T1  V2 
1 y − 2x For CO2 (triatomic gas), g = 1.33.
C + O2 → C O ; ∆H o =
2 2 0.33 1/ 3
150  10   10 
23. (c) For free expansion, pext = 0, w = pextΔV = 0. = = 
300  V2   V2 
For adiabatic process, q = 0. 1/ 3
1  10  10 1
= ⇒ = ⇒ V2 = 80 L
Therefore, ∆U = q + w = 0. This means that internal energy 2  V2  V2 8
remains constant, which is so at constant temperature.
Hence, ∆T = 0. ∆H 30 × 103
34. (a) ∆S = ⇒T = = 400 K
T 75
24. (b) Number of moles in 128 g of O2 = 128/32 = 4 mol.
35. (d) For the first reaction,
∆U = CV ∆T = 5 cal mol−1 deg−1 × 100 deg Si2H6(g) + H2(g) → 2SiH4(g); ∆H° = −11.7 k J mol−1
= 500 cal mol−1 × 4 mol = 2000 cal Given that Δf H of Si2H6(g) = + 80.3 k J mol−1. Let Δf H of
H2(g) = x, and Δf H of SiH2(g) = y. Therefore,
∆H = C p ∆T = 7 cal mol−1 deg−1 × 100 deg
2x − 80.3 = −11.7 ⇒ 2x = 80.3 − 11.7 = 68.6 k J mol−1 ⇒ x
= 700 cal mol−1 × 4 mol = 2800 cal = 34.3 k J mol−1
25. (b) Enthalpy of sublimation is the enthalpy change accom- For the second reaction, SiH4(g) → SiH2(g) + H2(g);
panying the conversion of one mole of a solid directly ∆H = +239.7 k J mol−1
into vapour phase at a given temperature below its melt- Therefore, y − 34.3 = 239.7 ⇒ y = (239.7 + 34.3) k J mol−1
ing point. Sublimation is nothing but fusion and vapour- = 274 k J mol−1
ization carried out in one step. 36. (a) Entropy is a state function whose value depends only
upon the state of the system and not upon the path by
Thus, ΔHsub = ΔHfus + ΔHvap = x + (−y) = x − y.
which the state has been attained.
26. (a) H2O(l)  H2O(g); ΔH = 30 k J mol−1 where T = 300 K,
37. (c) Δf H° will be calculated with the bond energy given and
therefore,
that comes out as 5384.1 kJ.
∆ transH 30 × 103 Δf H° = Σ B.E. (reactants) − Σ B.E. (products)
∆S = = = 100 J mol−1 K −1
T 300 = 0 − 5384.1 = − 5384.1 k J
Resonance energy = Observed Δf H° − Calculated Δf H°
1
27. (d) N + O2
2 2
→
 NO2 ∆H = 8 kcal (1) = 5535 k J − 5384.1 k J = 151 k J.
N2 +2O2 →
 N2O 4 ∆H = 2 kcal (2) 38. (b) Heat is transferred to the system from the surroundings
(Ssurr < 0) so that
2NO2 →
 N2O 4 ∆H = ? kcal
qsurr
Subtracting twice of Eq. (1) from Eq. (2) gives ∆Ssurr = <0
T
2NO2 →
 N2O 4 ∆H = 2 − 2 (8 ) = − 14.0 kcal The heat transferred from the surroundings, however, is
equal to the heat received by the system: qsurr = qsys = w.
28. (b) Conceptual q −w
∆Ssurr = surr =
29. (d) Internal energy and volume depend on initial and final T T
states. The total change in entropy (system plus surroundings)
q q
30. (a) ∆H = ∆U + p∆V is therefore, ∆Stotal = ∆Ssys + ∆Ssurr = − = 0
T T
∆U = ∆H − p∆V
The reversible process has zero total change in entropy.
∆U = 1 kcal − 1 × 0.3 L atm (1 L atm = 24.217 cal)
= 1 kcal − 0.3 L atm = 1 kcal − 0.3 × 0.024 kcal 39. (c) Thermodynamics is not concerned with the rate at which
= 1 kcal − 0.0072 kcal = 0.993 kcal a reaction proceeds.
40. (d) Conceptual
 NaCl + H2O ∆H1 = −55.9 kJ mol−1
31. (b) HCl + NaOH →
41. (c) Multiplying Eq. (II) by 2 and adding to Eq. (I) gives the
 NaCN + H2O ∆H2 = −12.1kJ mol−1
HCN + NaOH → required reaction with ∆H° = −26.8 + 2(+16.5) = +6.2 k J.

Chapter-6.indd 189 8/4/2016 10:34:20 AM


190 Chapter 6 Thermodynamics

42. (a) Chemical equation for hydrogenation of ethane is Process (II):


C2H4 (g) + H2 (g) →
 C2H6 (g) ∆H = ? nRT T
p = KV 2 ⇒ = KV 2 ⇒ 3 = constant ⇒ T ∝ V 3
V V
Given:
48. (d) Gibbs energy for the given reaction can be calculated as
 2CO2 (g) + 2H2O(l) ∆H = − 1409.5 kJ mol−1 (1)
C2H4 (g) + 3O2 (g) → follows:
 2CO2 (g) + 3H2O(l) ∆H = − 1558.3 kJ mol−1 (2)
C2H6 (g) + 72 O2 (g) →
∆G ° = [2( ∆ f G ° )H2 O(g) + ( ∆ f G ° )CO2 (g) ] − [2( ∆ f G ° )O2 (g) + ( ∆ f G ° )CH4 (g) ]
1 −1
H2 (g) + O (g) →
 H2O(l) ∆H = − 285.6 kJ mol ( 3)
2 2 ∆G° = [2( −228.527) − (394.359 )] − [0 + ( −50.752)]
The desired equation can be obtained by keeping = −800.751 kJ mol−1
Eqs. (1) and (3) as such and reversing Eq. (2) so as to have
C2H6(g) as the product and adding up the three equa- 49. (a) ∆S ° = ΣS °P − ΣS °R = 2 × 0.19 − (0.13 + 0.22) = −0.03 kJ K−1
tions. Thus, mol−1 = 30 J K−1 mol−1
 2CO2 (g) + 2H2O(l) ∆H = − 1409.5 kJ mol−1
C2H4 (g) + 3O2 (g) → 50. (d) − ∆G = p∆V
2CO2 (g) + 3H2O(l) → 7
 C2H6 (g) + O2 (g) ∆H = + 1558.3 kJ mol −1  12 12  −6 2 
2 1895 = p  −  × 10 m 
  2 .25 3 .31 
 H2O(l) ∆H = − 285.6 kJ mol−1
H2 (g) + 21 O2 (g) →
1895
Adding up the above three equations, we get
⇒p= = 1.1147 × 109 Pa = 11.1 × 1010 Pa
1.7 × 10 −6
C2H4 (g) + H2 (g) →
 C2H6 (g) 51. (b) The required equation can be obtained by adding the
∆H = [ −1409.5 + 1558.3 − 285.6] kJ mol −1 given equations.
Thus, ∆H = −50 + (−100) = −150 kcal.
= − 136.8 kJ mol−1
52. (d) ∆H = ∆E +∆ngRT
43. (c) For reversible adiabatic expansion, q = 0. Hence, For option (d), ∆ng ≠ 0. Hence, for this reaction, ∆H ≠ ∆E.
∆S = q/T = 0.
53. (a) The given equations are
44. (a) ∆H = ∆U + ∆nRT where ∆n = 2 − 3 = −1. Substituting
NH3 (g) + 3Cl2 (g)  NCl3 (g) + 3HCl(g); − ∆H1 (1)
given values, we get
N2 (g) + 3H2 (g)  2NH3 (g); − ∆H2 (2)
164000 = ∆U − 1 × 8.314 × 298
∆U = 164000 + 2477.5 = 166.5 k J mol−1 H2 (g) + Cl2 (g)  2HCl(g); ∆H3 (3)

45. (b) According to Kirchhoff’s equation, We aim at: N2 (g) + 3Cl2 (g)  2NCl3 (g) ∆ f H = ?

∆H2 − ∆H1 Adding 2 × Eq. (1) + Eq. (2) −3 × Eq. (3), we get
= ∆C p
T2 − T1 ∆H2 3
2 ∆ f H = − 2 ∆H1 + [ − ∆H2 − 3∆H3 ] ⇒ ∆ f H = − ∆H1 − − ∆H3
∆H2 − ∆H1 = ∆C p (T2 − T1) 2 2

∆H2 = ∆H1 + ∆C p (T2 − T1) 54. (a) The maximum work is equal to ΔG for the reaction, so
we need to calculate ΔG ° using the balanced equa-
8 + (32 × 10 −3 )( 400 − 300 ) = − 282.6 kJ mol−1
= − 285.8 tion for the reaction of combustion of one mole of
C8H18 .
46. (a) Temperature and volume are related for adiabatic pro-
cess as: T2V2g −1 = T1V1g −1. Therefore, C8H18 (l) + 12 21 O2 (g) → 8CO2 (g) + 9H2O(l)

g −1 1/ 3
Then,
T1V1g −1 V   1 1
T2 = = T1  1  = 300   = 300 × = 150 K o o
∆G = [8 ∆ f GCO + 9 ∆ f GHo2 O(l) ] − [ ∆ f GCo8H18 (l) + 12.5∆ f GOo 2( g) ]
V2g −1 V 
2  8 2 2 ( g)

Substituting values, we get


Cp 4 ∆G° = [18 × ( −394.4 ) kJ + 9 × ( −237.2) kJ] − [1× ( + 17.3) kJ + 12.5 × (0 ) kJ]
Also, = g = and Cp − CV = R. Solving, we get
CV 3 = ( −5290 kJ) − ( +17.3 kJ) = −5307 kJ
Cp = 4R and CV = 3R
R = 1.987 cal mol−1 K−1 ≈ 2 cal mol−1 K−1 Level II
Therefore, adiabatic work done is Multiple Correct Choice Type
∆w = −CV ∆T = −3 × 2 × (150 − 300) = 900 cal
55. (b), (c), (d) ∆H = ∆U + ∆w. If ∆H = 0, ∆w = −∆U. Thus, internal
47. (b) Process (I): energy decreases. The temperature of the reversible adia-
nRT 2 1 batic process always decreases, and is also known as isoen-
pV 2 = K ⇒ V = K ⇒ TV = constant ⇒ T ∝
V V tropic process.

Chapter-6.indd 190 8/4/2016 10:34:35 AM


Hints and Explanations 191

56. (b), (c), (d) A process can be assumed to be at equilibrium in Passage Type
between a spontaneous and non-spontaneous situation.
∆G = ∆H − T∆S 69. (b) The process is isochoric because volume remains con-
At equilibrium, ∆G = 0. stant with increase in temperature.
∆H = T∆S 0.0821× 200
70. (b) pV = RT ⇒ p = = 1.642 atm
T = ∆H/∆S = 978 K = 705 °C. 10
57. (b), (c) ∆G = ∆H − T∆S
71. (c) w = p∆V = 1.642 × 10 = 16.42 L atm
ΔH ΔS Temperature (T) Spontaneity
72. (c) ΔG ° = (−5.40) + (−0.84) = −6.24 k J
− − Low T Spontaneous
+ + Low T Non-spontaneous 73. (c) ΔG ° = −0.84 k J
74. (d) 2NO2  N2O 4 ∆G ° = − 6.24 kJ
T  V   373 
58. (a), (b) ∆S = nCV ln  f  + nR ln  f  = 5 ln  + 2 ln10 2NO2 → N2O 4 ∆G ° = − 5.40 kJ
T  i V  i  298 
One reaction is in equilibrium the other is not.
Also, Cp − CV = R ⇒ Cp = 7. 75. (b) The reaction involved is
Now, ∆H = nCp∆T = 1 × 7 × 75 = 525 cal
HCl + NaOH → NaCl + H2O
and ∆U = nCV∆T = 1 × 5 × 75 = 375 cal
Initial g equivalent. 0.33 1 0 0
59. (a), (d) The overall process is cyclic.
g equivalent at equilibrium 0 0.67 0.33 0.33
Thus, ∆U = 0, ∆H = 0, ∆S = 0.
g equivalent added 0.66 0.01 0.66 0.66
Now, ∆U = q + w ⇒ 0 = q + w ⇒ q = −w
∆H = −4.6 cal. On further addition ∆H = −9.2 cal
Total work = wAB + wBC + wCA
VC 76. (b) The observed value of heat of neutralization of HF is
w = − p(VB − VA ) + 0 + 2.303 × nRT log more due to extensive solvation of F− ion because of
VA
its small size. Hydration reactions are exothermic also in
VC nature.
= −1 × ( 40 − 20 ) + 0 + 2.303 × pAVa log
VA
= −6.13 L atm = −621.1 J 77. (c) 2CH3COOH + Ca(OH)2 → (CH3COO)2Ca + 2H2O
2 mol 1 mol
(as 1 L atm = 101.325 J)
∆ dissH of CH3COOH = 0.005 kcal g−1 × 60 = 0.3 kcal mol−1
60. (a), (b), (c) Adiabatic means no exchange of heat energy
from the surrounding. ∆U = q + w. When q = 0, ∆U = w. For 2 moles = 0.6 K cal
Enthalpy change for neutralization of 2 moles of
61. (a), (b), (d) For a spontaneous process in an isolated system,
∆SU,V > 0. For a reversible adiabatic process, ∆SU,V = 0. CH3COOH = (−13.7 K cal × 2 + 0.6 K cal) = −26.8 K cal

62. (a), (b), (c), (d) ∆G = 0 indicates that the system is at equilib- 78. (a) As q = 0, so ∆S = q/T = 0
rium and ∆G is negative for all spontaneous processes. 79. (c) As there is decrease in randomness, ∆S = 0.
Exergonic process will occur spontaneously and is thermo-
∆Hfus 600
dynamically favorable. 80. (b) ∆S = = = 2 cal K −1
Enodergonic processes are not spontaneous at standard Tmelt 300
conditions of pressure, temperature.
Matrix-Match Type
63. (a), (b) Melting is an endothermic process.
81. (a) → r; (b) → t; (c) → s; (d) → q; (e) → p
64. (b), (c) Strong acids and alkalis are completely ionized.
Enthalpy change depends upon the equivalent of acid or Extensive properties depend on the quantity of matter pres-
alkali used and not upon their nature. ent in the system.
If the system exchanges only energy with the surroundings
65. (c), (d) Heat of combustion is the amount of heat released and not the matter.
when 1 mol of that substance burns in excess of air. Path functions are those that depend only on the path and
not the state of the system.
66. (a), (b), (c) Intensive properties are those properties that
In adiabatic process, no heat can flow from the system to the
depend only upon the nature of the substance and are inde-
surroundings and vice versa.
pendent of the amount of substance present in the system.
Temperature, pressure, viscosity, surface tension, refractive Intensive properties are those that do not depend on the
index and specific heat are examples of intensive properties. matter present in the system.

67. (a), (c) In both the reactions, large number of molecules 82. (a) → p, (b) → r, (c) → q, (d) → s
combine together to form lesser number of molecules. Using the equation ∆G = ∆H − T∆S
68. (b), (d) ∆U = q + w; work done = 0, heat given out = −q. 83. (a) → r; (b) → s; (c) → p; (d) → q.
Temperature falls, thus the internal energy decreases.

Chapter-6.indd 191 8/4/2016 10:34:41 AM


192 Chapter 6 Thermodynamics

Gibbs free energy is negative for spontaneous process. Now, heat gained by ice = heat lost by water
Heat is absorbed in case of endothermic process. So, 52 × 80 + mass of ice at 40 °C × 0.5 × 40 =
Bond dissociation enthalpy is the energy needed to break 1 mol 100 × 1 × 40 °C
of the bond. 100 × 1× 40 − 52 × 80
Mass of ice at 40 °C = =8g
∆rH = ∆rU + ∆nRT . Now, ∆n = 0 for solids and liquids, so ∆rH = ∆rU. 0.5 × 40
Therefore, the mass of ice that remains = 52 − 8 = 44 g
84. (a) → p, r; (b) → s; (c) → q; (d) → t
3
Integer Type 88. (0) For a monoatomic perfect gas CV = R
2
85. (1) K = e−∆G °/RT = e0 = 1 The change in internal energy, ∆U = CV ∆T
86. (6) 1 atm L = 101.325 J. Therefore, 3
1 ∆U = × 8.314 × 100 = 1247.1 J = 1.247 kJ
607.8 J = × 607.8 = 5.99 ≈ 6 L atm 2
101.325
87. (44) We know that specific heat of ice = (1/2) of specific heat Since the volume remains constant, w = 0.SOL
of water = 0.5 cal g−1 deg−1.
Latent heat of fusion of ice = 80 cal g−1

Chapter-6.indd 192 8/4/2016 10:34:45 AM


Solved JEE 2016 Questions 193

SOLVED JEE 2016 QUESTIONS


JEE Main 2016 At low temperature: ΔG is positive (non-spontaneous pro-
cess), so ΔH is positive and ΔS is positive (TΔS < ΔH as T is
1. The heats of combustion of carbon and carbon monoxide are low).
– 393.5 and – 285.5 kJ mol–1, respectively. The heat of forma- At high temperature: ΔG is negative (spontaneous pro-
tion (in kJ) of carbon monoxide per mole is cess), so ΔH is positive and ΔS is positive (TΔS > ΔH as T
(a) – 110.5 (b) 110.5 (c) 676.5 (d) – 676.5 is high).
(Offline)
4. If 100 mol of H2O2 decompose at 1 bar and 300 K, the work done
Solution (kJ) by one mole of O2(g) as it expands against 1 bar pressure is:
(a) The given equations are 2H2O2 (l)  2H2O(l) + O2 (g)
C(s) + O2 (g) → CO2 (g) (1) ΔH = −393.5 kJ mol−1
1 (R = 8.3 J K−1 mol−1)
CO(g) + O2 (g) → CO2 (g) (2) ΔH = −283.5 kJ mol−1 (a) 124.50 (b) 249.0 (c) 498.0 (d) 62.25
2
(Online)
The required equation is
1 Solution
C(s) + O2 (g) → CO(g) ΔH = ? (b) For reaction: 2H2O2 (l)  2H2O(l) + O2 (g)
2
Given, p = 1 bar, T = 300 K, n = 100 mol, R = 8.3 J K−1 mol−1
On reversing equation (2), we get
We know that,
1
CO2 (g) → CO(g) + O2 (g) (3) ΔH = 283.5 kJ mol−1 w = p∆V = nRT
2
On adding Eq. (1) and Eq. (3), we get the required equation = 100 × 8.3 × 300
= 249000 J or 249 kJ
1
C(s) + O2 (g) → CO2 (g) JEE Advanced 2016
2
ΔH = −393.5 + 283.5 = −110.0 kJ mol−1 1. One mole of an ideal gas at 300 K in thermal contact with
surroundings expands isothermally from 1.0 L to 2.0 L against
2. For the reaction,
a constant pressure of 3.0 atm. In this process, the change in
A(g) + B(g) → C(g) + D(g), ΔH° and ΔS° are, respectively,
entropy of surroundings (ΔSsurr) in J K−1 is
−29.8 kJ mol−1 and −0.100 kJ K−1 mol−1 at 298 K. The equilib-
(1 L atm = 101.3 J)
rium constant for the reaction at 298 K is
(a) 5.763 (b) 1.013
(a) 1.0 × 10−10 (b) 10
(c) −1.013 (d) −5.763
(c) 1 (d) 1.0 × 1010
(Online) Solution
Solution (c) From the first law of thermodynamics
(c) ΔG° = ΔH° − TΔS° ∆U = qsys + w sys (1)
ΔG° = −29.8 – 298 (−0.1)
=0 For an ideal gas, during an isothermal process (expansion
ΔG° = −2.302 RT log keq or compression), the temperature of the system remains
log keq = 0 ⇒ keq = 1 constant, so ∆U = 0 (as U depends only on temperature).
Thus, for pressure volume work, Eq. (1) can be written as
3. A reaction at 1 bar is non-spontaneous at low temperature but
becomes spontaneous at high temperature. Identify the cor- 0 = q − pext ∆V
rect statement about the reaction among the following:
(a) ΔH is negative while ΔS is positive. Substituting values, we get
(b) Both ΔH and ΔS are negative. q = 3 (2 − 1) = 3 atm L = (3 × 101.3) =303.9 J
(c) ΔH is positive while ΔS is negative.
(d) Both ΔH and ΔS are positive. −q 303.9
∆Ssurr = =− = −1.013 JK −1
(Online) t 300
Solution
(d) We know that ΔG = ΔH – TΔS
= +ve – T (+ve)

Chapter-6.indd 193 8/4/2016 10:34:50 AM


Chapter-6.indd 194 8/4/2016 10:34:50 AM
7 Equilibrium

Questions Distribution in JEE (Main and Advanced)


4

3
No. of Questions

JEE (Main)
2
JEE (Adv)

0
2016 2015 2014 2013 2012 2011 2010 2009 2008 2007

Concept Distribution in JEE (Main and Advanced)

Topics Covered
Year
JEE (Main) JEE (Advanced)
Equilibrium in Physical and Chemical Processes, Hydrolysis of Salts
2007 and pH Determination of their Aqueous Solutions, Buffer Solutions,
Equilibria for Sparingly Soluble Salts
Equilibrium in Physical and Chemical Processes, Acid-Base Concept, Acid Base Titrations, Equilibria for Sparingly Soluble
2008
Hydrolysis of Salts and pH Determination of their Aqueous Solutions Salts
Hydrolysis of Salts and pH Determination of their
2009 Equilibria for Sparingly Soluble Salts
Aqueous Solutions
Acid-Base Concept, Acid-Base Equilibria, Equilibria for Sparingly
2010 Buffer Solutions , Acid-Base Concept
Soluble Salts
Common Ion Effect, Equilibria for Sparingly Soluble
2011 Equilibrium in Physical and Chemical Processes
Salts
2012 Equilibrium in Physical and Chemical Processes, Acid-Base Equilibria
Hydrolysis of Salts and pH Determination of their
2013 Aqueous Solutions, Equilibria for Sparingly Soluble
Salts, Equilibrium in Physical and Chemical Processes
Equilibrium in Physical and Chemical Processes, Factors Affecting
Equilibria and Le Chatelier’s Principle, Acid-Base Concept, Hydrolysis
2014
of Salts and pH Determination of their Aqueous Solutions, Buffer
Solutions, Equilibria for Sparingly Soluble Salts
Equilibrium in Physical and Chemical Processes, Factors Affecting
2015
Equilibria and Le Chatelier’s Principle

2016 Equilibrium in Chemical Processes Equilibrium in Chemical Processes

Chapter-7.indd 195 7/29/2016 3:15:31 PM


196 Chapter 7 Equilibrium

SUMMARY
1. An equilibrium system is the system in which the macroscopic properties of the system, such as temperature, pressure, concentra-
tion, etc., do not change. An equilibrium is attained in a closed system. Any increase in these properties is balanced by a decrease in
that property within the system. There are two kinds of equilibrium (a) Physical equilibrium and (b) Chemical equilibrium.
2. Physical equilibrium
An equilibrium which exists between different physical states or when phases undergo transition. The physical equilibrium can be
attained in closed system. The rate of change between the phases is the same at equilibrium. Thus, the equilibrium is stable yet
dynamic because two opposing processes take place simultaneously and at the same rate.
(a) Types of physical equilibrium
(i) Solid–liquid equilibrium: The state when solid and liquid phases of a substance coexist is called solid-liquid equilibrium.
Solid  Liquid
If no heat is exchanged with the surroundings, then the temperature and the mass of the two phases (solid and liquid)
remain constant.
(ii) Liquid–vapour equilibrium: When a liquid is transformed into a gas is known as evaporation or vapourization.
Water (liquid)  Water (vapour)
(iii) Solid–vapour equilibrium: The process by which a solid is directly converted into vapour or gaseous state is known as
sublimation. The stage of dynamic equilibrium wherein the rate of conversion of solid to its vapour is the same as the rate
at which vapours condense back to solid is solid-vapour equilibrium.
Iodine (solid)  Iodine (vapour)
(b) Equilibrium in dissolution of solids and gases in liquids and Henry’s law
(i) Solids in liquids: When the maximum amount of solid is dissolved in a given amount of liquid at a given temperature, the
solution is said to be a saturated solution. The dynamic equilibrium that exists between solute molecules dissolved in the
solution and molecules in solid form in a saturated solution is as follows:
Sugar (solution)  Sugar (solid)
(ii) Gases in liquids: At a given pressure, equilibrium exists between the gas (e.g. CO2) molecules dissolved in the liquid and
those in gaseous state, is represented as
CO2 (gas)  CO2 (in solution)
The amount of gas dissolved is determined by the pressure.
(c) Henry’s law (also called the Pressure–solubility law) states that the concentration of a gas dissolved in a given volume of a
liquid at any given temperature is directly proportional to the partial pressure of the gas over the solution.
C gas = K H pgas (K H is a constant)

where Cgas is the concentration of the gas and pgas is the partial pressure of the gas above the solution. The proportionality con-
stant, KH, called the Henry’s law constant, is unique to each gas.
An alternate (and commonly used) form of Henry’s law is

C1 C2
=
p1 p2
3. Chemical equilibrium
The state at which the concentration of reactants and products do not change with time is called a state of chemical equilibrium. It is
a dynamic system in which two or more opposing chemical reactions are going on at the same time and at the same rate. Chemical
equilibrium can be attained only in a closed system.
(a) Law of chemical equilibrium and equilibrium constant
The law of mass action correlates the rate of a chemical reaction and the concentration of the reactants. It states that, at a con-
stant temperature and pressure, the rate of a chemical reaction is directly proportional to the product of the molar concentration
of the reactants each raised to a power equal to the corresponding stoichiometric coefficient, which appears in the balanced
chemical equation.
Consider a reversible reaction, aA + bB  c C + d D
At equilibrium, the two rates become equal, i.e., Rate of forward reaction (Rf ) = Rate of reverse reaction (Rr)
So, at equilibrium, kf [A]a [B]b = kr [C]c [D]d

Chapter-7.indd 196 7/29/2016 3:15:33 PM


Summary 197

k f [C]c [D]d
= = KC
kr [A ]a [B]b
The ratio of kf/kr is the equilibrium constant KC.
The above equation is called law of chemical equilibrium or the equilibrium law.
(b) Characteristics of equilibrium constant
(i) For any reaction, KC is constant for a specific temperature.
(ii) It is independent of the initial concentrations of the reactants.
(iii) It is independent of presence of catalyst and inert materials in the reaction.
(iv) It is independent of the direction from which the equilibrium is attained.
(v) It is dependent on the representation of the reaction. The equilibrium of the reverse reaction is the reciprocal of the equilib-
rium constant of the forward reaction.
[C][D]
A + B  C + D KC =
[A][B]
[A][B]
C + D  A + B K C′ =
[C][D]
1
Therefore, K C′ =
KC
(vi) If a chemical equation is multiplied by a common factor (n), then the new equilibrium constant should also reflect the
change. For example,
[C][D]
A + B  C + D KC =
[A][B]
[C]n [D]n
nA + nB  nC + nD K C′ = = (K C )n
[A]n [B]n
(c) Equilibrium constant for gaseous reactions (Kp)
For gaseous reactions, since gas pressures are more conveniently measured, the equilibrium law expressions for gaseous reac-
tions are written using partial pressures instead of molar concentrations.
[ pC ]c × [ pD ]d
Therefore, Kp =
[ pA ]a × [ pB ]b
where, pA, pB, pC and pD the partial pressures of A, B, C and D, respectively and Kp is called pressure equilibrium constant.

(d) Units of KC and Kp


(i) When the total number of moles of reactants and products are equal, for such reactions, KC and Kp have no units.
(ii) When the total number of moles of reactants and products are unequal, KC will have units (mol L−1)∆n and Kp will have the
units (atm)∆n where, ∆n = total number of moles of products − total number of moles of reactants.
(e) Relationship between Kp and KC
∆ng
K p = K C × (RT )
where ∆ng = total number of moles of gaseous products − total number of moles of gaseous reactants.
(i) For ∆ng = 0; Kp = KC
(ii) For ∆ng > 0; Kp > KC
(iii) For ∆ng < 0, Kp < KC
(f) Homogeneous and heterogeneous equilibria
(i) A homogeneous equilibrium is established in a system where all reactants and products are in the same phase, such as in
a system containing only gases or liquids or solids. Some examples are as follows:
2SO2 (g) + O2 (g)  2SO3 (g)
CH3COOH(l) + C2H5OH(l)  CH3COOC2H5 (l) + H2O(l)

(ii) A heterogeneous equilibria is established in a system where the reactants and products are in more than one phase, such
as between solid and liquid, liquid and gas, solid and gas, etc.
CaCO3 (s)  CaO(s) + CO2 (g)
CuO(s) + H2 (g)  Cu(s) + HO2 (g)

Chapter-7.indd 197 7/29/2016 3:15:36 PM


198 Chapter 7 Equilibrium

(iii) Simultaneous equilibrium occurs when more than one equilibrium exists in the same container simultaneously. For
example,
CaCO3 (s)  CaO(s) + CO2 (g)
CO2 (g) + C(s)  2CO(g)
Hence, CO2(g) is the common species in both these equilibria and its concentration at equilibrium will be the same for both
finally.
(g) Applications of equilibrium constants
(i) Predicting the extent of a reaction
• When K is very large, the reaction proceeds far towards completion. The position of equilibrium lies far to the right,
towards the products.
• When K is very small, extremely small amounts of products are formed. The position of equilibrium lies far to the left,
towards the reactants.
• A special case is when K ≈ 1, then the concentrations of reactants and products are nearly the same at equilibrium. The
position of equilibrium lies approximately midway between reactants and products.
(ii) Predicting the direction of reaction
• Reaction quotient (Q) is expressed for any reaction in the same way as for equilibrium constant, except that the concen-
trations may not necessarily be equilibrium. In general, for a reaction of the type
aA + bB  cC + dD

[C]c [D]d
Q=
[A]a [B]b

(pC )c (pD )d
or in terms of pressure, we have Q=
(pA )a (pB )b
• Relation between QC and KC values and the direction of reaction

QC and KC values Relative concentration Direction of net reaction


QC > KC Products more than reactants From right to left
QC < KC Reactants more than products From left to right
QC = KC Equilibrium concentrations No net reaction
(iii) Calculating equilibrium concentrations and degree of dissociation
• The degree of dissociation is the fraction that undergoes dissociation in a reaction, and is represented by a. Hence,
Moles dissociated
a=
Initial moles
(h) Relation between Gibbs energy change and chemical equilibrium
(i) The relation between ∆G° and the position of equilibrium can be listed as
∆G = ∆G ° + RT ln Q
(ii) For a reaction in equilibrium, ∆G = 0 and Q = KC
∆G = ∆G ° + RT ln K = 0
∆G °
∆G ° = − RT ln K ⇒ ln K = −
RT
−∆G °/ RT
Therefore, K = e

Tip
−∆G °/RT
(i) If ΔG° < 0, then −ΔG°/RT is positive, and therefore, e > 1 or K > 1. Reaction is spontaneous and it pro-
ceeds in the forward direction to an extent that products are present predominantly.
(ii) If ΔG° > 0, then −ΔG°/RT is negative, and therefore, e −∆G °/RT < 1 or K < 1. Reaction is non-spontaneous and it
proceeds in the forward direction to a small extent, such that very small quantities of products are formed.

(iii) Le Chatelier’s principle and factors affecting equilibria


Le Chatelier’s Principle states that “if a system at equilibrium is subjected to a change in one or more variables such as, pres-
sure, temperature, and/or concentration, then the equilibrium shifts in such a way so as to undo the effect of the change”.

Chapter-7.indd 198 7/29/2016 3:15:39 PM


Summary 199

(i) Factors affecting equilibria


(a) Effect of change in concentration: An increase in the concentration of any of the substances on one side of the equilibrium
shifts the equilibrium to produce more of the substances on the other side of it.
(b) Effect of change in pressure
• The equilibrium involving only solids are not affected by a change of pressure.
• The equilibrium involving liquids and/or gases, where the number of molecules before and after the attainment of equi-
librium remains the same (where n = 0), the reactions are not affected by a change of pressure.
• However, in gaseous reactions where there is a change in the number of molecules in going from reactants to products
or vice-versa, pressure plays an important role. For example,
N2O 4 (g) → 2NO2 (g)
1 mol 2 mol (∆n = +1)(Number of moles increases)
1 vol 2 vol (Volume of the system increases)

An increase of pressure will cause a decrease in volume so, an increase of pressure on a gaseous system will shift the equilib-
rium in a direction where there is a decrease in number of moles of gaseous state i.e. in the reverse direction and vice-versa.
(c) Effect of addition of an inert gas
• Inert gas added keeping pressure of the system constant: The addition of an inert gas increases the volume of the system,
which in turn, causes the equilibrium position of the system to move in the direction of larger number of gaseous molecules.
• Inert gas added keeping volume of the system constant: Addition of an inert gas into a system at equilibrium under con-
stant volume causes an increase in the pressure of the system and an increase in the total number of moles in the system.
V n
Total pressure of a system is then given by, ptotalV = ntotal RT or = total
RT ptotal
Under constant volume, at any temperature, the ratio ntotal /ptotal remains constant even on the addition of an inert gas i.e.,
the equilibrium position of the reaction remains unaffected. As a result, there is no change in any of the variable and the
amount of the substances at equilibrium by the addition of inert gas.
(d) Effect of change in temperature
• Increasing the temperature shifts a reaction in a direction that produces an endothermic (heat-absorbing) change.
• Decreasing the temperature shifts a reaction in a direction that produces an exothermic (heat-releasing) change.
(e) Effect of adding catalyst: A catalyst in a system that reaches a point of equilibrium increases the rate of both the forward
and reverse reactions proportionally, but does not change the point of equilibrium. The function of a catalyst is simply to
reach the point of equilibrium in less time.
4. Concepts of acids and bases
(a) Arrhenius concept
(i) Acids are compounds that produce H+ ions in water. For example,
HCl(aq) + H2O(l) → H3O + (aq) + Cl− (aq)
H3O+ is the acid species known as the hydronium ion.
(ii) Bases are compounds that produce OH− ions in water forming basic solutions (or alkaline/caustic solutions).
HO
NaOH(s) 
2
→ Na+ (aq) + OH− (aq)
Limitations
(i) Applicable only to aqueous solutions.
(ii) Cannot explain the basic nature of substances that do not contain a hydroxyl group or for the acidic nature of substances
that do not contain a hydrogen (hydronium) ion.
(b) Brönsted–Lowry concept
(i) Acids are species that donate protons and bases are species that accept protons.
(ii) HCl is a Brönsted acid (proton donor) because when it reacts with NH3 (Brönsted base, a proton acceptor), HCl molecules
donate protons to NH3 molecules.
NH3 (aq) + HCl(l)  NH+4 (aq) + Cl− (aq)
In the reverse reaction, NH+4 behaves as a Brönsted acid, while Cl− ion behaves as a Brönsted base.
(iii) Two substances that differ from each other only by one proton are referred to as a conjugate acid–base pair.
conjugate pair

HCHO2 + H2O H3O+ + CHO2−


acid base acid base

conjugate pair

Chapter-7.indd 199 7/29/2016 3:15:41 PM


200 Chapter 7 Equilibrium

Limitation
(i) Some reactions which do not involve proton transfer and have properties similar to acid–base reactions.
(c) Lewis concept
(i) Lewis acid is any ionic or molecular species that can accept a pair of electrons in the formation of a coordinate covalent
bond. These substances have an incomplete valence shell, such as BF3, H+, CO2 and SO2.
(ii) Lewis base is any ionic or molecular species that can donate a pair of electrons in the formation of a coordinate covalent
bond. These substances have completed valence shells and unshared pairs of electrons, such as NH3, H2O, and O2−.
Limitations
(i) Fails to explain the behaviour of monoprotic and polyprotic acids, such as HCl, H2SO4, etc.
(ii) Does not help in predicting the relative strength of acids and bases.
5. Ionic equilibrium
In a system involving ionic species, if the number of ions formed is equal to the number of ions consumed, the system is said to be in
ionic equilibrium. The substances that form ions in aqueous solutions or in fused state are called electrolytes. Substances that ion-
ized almost completely are known as strong electrolytes. Examples of strong electrolytes are NaCl, KCl, KNO3 and K2SO4. Substances
that ionize to a small extent are known as weak electrolytes. For example CH3COOH, HCN, etc.
(a) Ionization of electrolytes
(i) When an electrolyte is dissolved in water, it ionizes to produce ions in the solution, which are free to move throughout the
bulk of the solution.
(ii) The total number of positive charges is equal to the total number of negative charges in any solution of any electrolyte. The
ions tend to recombine to form the unionized electrolyte. Hence, a dynamic equilibrium exists between the ionized and
unionized form of the electrolyte.
AB  A+ + B+
The ionization constant (K) is then given by,
[A+ ][B − ]
K=
[AB]
(iii) The fraction of the total number of molecules present as free ions in the solution is known as the degree of ionization (a).
Number of molecules present as ions
a=
Total number of molecu
ules of the electrolyte

The extent of dissociation increases with increase in dilution and degree of dissociation of an electrolyte at infinite dilution
is assumed to be unity (i.e. a = 1).
(b) Ostwald’s dilution law
The expression that correlates the variation of the degree of dissociation of an electrolyte with dilution is known as Ostwald’s
dilution law.
a2
KC =
(1− a )V

where C is the initial concentration of the electrolyte in mol L−1 and a is the degree of dissociation.
(i) For weak electrolytes: The degree of dissociation is small, that is, a  1, therefore, 1 – a ≈ 1.
a2
KC =
V
KC
or a = KC V =
C
1
a =K ’ V =K ’
C
where K ’ = K C .
(ii) For strong electrolytes: a is large and cannot be neglected in comparison to 1.
a2
KC =
(1− a )V
or a 2 = K C V − K Ca V

or a 2 + a K CV − K CV = 0
Solution of this quadratic equation gives the value of a.

Chapter-7.indd 200 7/29/2016 3:15:44 PM


Summary 201

(c) Common ion effect


It is defined as the suppression in degree of dissociation of an electrolyte by addition of an electrolyte containing a common
ion.
6. Acid–Base equilibria and ionization constants
(a) Ionization constants of weak acids
HA(aq) + H2O(l)  A − + H3O+
C (1− a ) Ca Ca

[H3O+ ][A − ]
K C′ =
[HA ][H2O]

Since in dilute aqueous solutions [H2O] can be considered a constant, so it can be combined with K C′ to give a new equilibrium
constant.
[H3O+ ][A − ]
K C′ × [H2O] = = Ka
[HA ]

[H+ ][A − ]
or Ka =
[HA ]
This new constant Ka is called an acid dissociation or ionization constant.
k1C (1 − a ) = k2Ca × Ca

Ca × Ca k1
= = Ka
C (1− a ) k2

a 2C
Ka = mol L−1
1− a

Tip pKa of an acid can be represented as


pK a = −log K a
The larger the Ka, the stronger and more fully ionized the acid and smaller is its value of pKa.

(b) Ionization constants of weak bases


The weak Brönsted bases are proton acceptors. Let B be any weak base

B + H2O  BH+ + OH−


C (1− a ) Ca Ca

The base dissociation or ionization constant, Kb is given by

[BH+ ][OH− ] (Ca )2 Ca 2


Kb = = =
[B] C (1− a ) 1− a

Tip pKb of a base can be represented as


pK b = −log K b

(c) Relation between Ka and Kb


pKa + pKb = pKw = 14.00 (at 25 °C)

Chapter-7.indd 201 7/29/2016 3:15:47 PM


202 Chapter 7 Equilibrium

(d) Relative strength of acids and bases

HClO4 ClO4− Very weak bases;


do not react with

Acid strength increases


Very strong acids;
HNO3 NO3− water to a
100% ionized in water
measurable
extent
HCl Cl−

Strongest proton donor


that can exist in water H3O+ H2O

HF F−

HNO2 NO2−
Weak bases
C2H3O2−

Base strength increases


Weak acids in water HC2H3O2 in water

HOCl OCl−

NH4+ NH3

H 2O OH− Strongest proton


Very weak acid; acceptor that can
does not react with NH3 NH2− exist in water
water as an acid

Very strong base;


reacts 100%
with water
(e) Ionization of polyprotic acids
(i) Polybasic or polyprotic acids are those acids supplying more than one H+ per molecule. For example, H2SO4, H3PO4, H2CO3.
(ii) In water, the weak diprotic acid H2CO3 ionizes in two steps, each of which is an equilibrium that transfers an H+ ion to a water
molecule.
H2CO3  H+ + HCO3−
HCO3−  H+ + CO23 −
Each step has its own ionization constant, Ka, which we identify as K a1 for the first step and K a2 for the second.
(f) Ionic product of water
K w = [H3O + ][OH− ] = [H+ ][OH− ]
where Kw is called the ionic product or ionization constant of water.
In pure water, the concentrations of H+ and OH− produced by the autoionization are equal because the ions are formed in equal
numbers.
[H+ ] = [OH− ] = 1.0 × 10 −7 mol L−1
Therefore, at 25°C, K w = (1.0 × 10 −7 ) × (1.0 × 10 −7 ) = 1.0 × 10 −14 (at 25 °C).
(g) pH concept
[H+ ]
pH = −log aH+ = − log
mol L−1

For dilute solutions, pH = − log[H+ ] ⇒[H+ ] = 10 −pH = antilog(pH)


(i) pOH of a solution can be defined as
pOH = − log[OH− ]
(ii) pKw can be defined as
pK w = −log K w
The numerical value of pKw at 25°C equals −log (1.0 × 10−14) or −(−14.00).
(iii) pH + pOH = pK w = 14.00 (at 25 °C)

Chapter-7.indd 202 7/29/2016 3:15:50 PM


Summary 203

(h) pH of a mixture of two or more weak monobasic acids in water


HA + H2O  H3O + + A− 

C1 0 0 
H O + being common ion
C1(1− a 1) C1a 1 + C2a 2 C1a 1  3
 the con
ncentration would be the
HB + H2O  H3O+ + B− 
same.
C2 0 0 

C2 (1− a 1) C2a 2 + C1a 1 C2a 2 

[H3O + ][A − ] [C1a 1 + C2a 2 ][C1a 1]


Now, K a (HA ) = =
[HA] [C1(1− a 1)]

[H3O + ][B − ] [C1a 1 + C2a 2 ][C2a 2 ]


Similarly, K a (HB) = =
[HB] [C2 (1− a 2 )]

pH = − log(H+ ) = −log K a1C1 + K a2C2

where K a1 = K a (HA ) and K a2 = K a (HB)

7. Hydrolysis of salts and the pH of their solutions


(a) Salt of a weak acid and a strong base
CH3COONa is a salt of weak acid (CH3COOH) and a strong base (NaOH). It undergoes complete dissociation in aqueous solution.

CH3COONa(aq) → CH3COO − (aq) + Na+ (aq)

CH3COO − (aq) + H2O(l)  CH3COOH(aq) + OH− (aq)

[CH3COOH][OH− ]
The hydrolysis constant Kh is expressed as K h =
[CH3COO − ]

[CH3COO − ][H+ ] K
We know that Kw = [H+][OH−] and Ka = . Comparing with above equation we get K h = w
pH is expressed as CH3COOH Ka

[Salt]
pH = pK a + log
[Acid]
In hydrolysis of salts, the degree of dissociation is replaced by degree of hydrolysis, which is represented by h.

CH3COO − (aq) + H2O(l)  CH3COOH(aq) + OH− (aq)


C (1− h) Ch Ch

Ch × Ch Ch2
We get Kh = =
C (1− h) 1− h

Kw Kw
As h  1, we have Ch2 = ⇔h=
Ka K aC

kw k C
[OH− ] = Ch = C × = w
K aC Ka

Kw K wK a
[H+ ] = =
[OH− ] C
Taking log on both sides and solving, we get
1 1 1 1
− log[H+ ] = log k w − log K a + log C = ( pK w + pka + log C )
2 2 2 2
1 1
pH = 7 + pK a + log C
2 2

Chapter-7.indd 203 7/29/2016 3:15:55 PM


204 Chapter 7 Equilibrium

(b) Salt of a strong acid and a weak base


NH4Cl is a salt of a strong acid (HCl) and a weak base (NH4OH).

NH4 Cl(aq) → NH+4 (aq) + Cl− (aq)

NH+4 (aq) + H2O(l)  NH4 OH(aq) + H+


pOH is expressed as
[Salt]
pOH = pK b + log
[Base]

NH+4 (aq) + H2O(l)  NH4 OH(aq) + H+


C(1− h) Ch Ch
2
Ch × Ch Ch
we get Kh = =
C (1− h) 1− h
Kw Kw
As h  1, we have Ch2 = ⇔h=
Kb K bC

Kw K wC
Now, [H+ ] = Ch = C × =
K bC Kb

Taking log on both sides and solving, we get


1 1
pH = 7 − logC − pK b
2 2
(c) Salt of a weak acid and a weak base
CH3COONH4 is a salt of a weak acid (CH3COOH) and a weak base (NH4OH)

CH3COONH4 (aq) → NH4+ (aq) + CH3COO − (aq)

NH+4 (aq) + CH3COO − (aq) + H2O(l)  CH3COOH + NH4 OH


Acetic acid and ammonium hydroxide being a weak acid and base, respectively, will also exist in solution in partly dissociated
form.
CH3COOH  CH3COO − + H+
NH4 OH  NH+4 + OH−

Kw
The hydrolysis constant can be expressed as K h =
K aK b

NH+4 (aq) + CH3COO − (aq) + H2O(l)  CH3COOH + NH4 OH


C(1− h) C (1− h) Ch Ch

Ch × Ch h2
we get Kh = =
C 2 (1− h)2 (1− h)2

Kw Kw
As h  1, we have h2 = ⇔h=
K aK b K aK b
Now, due to common ion effect, the acetate ion coming from CH3COOH can be neglected, thus from the following reaction,
CH3COOH  CH3COO − + H+
Ch C (1− h)

[CH3COO − ][H+ ] C (1− h)[H+ ]


we have, Ka = =
[CH3COOH] Ch

[H+ ] Kw K aK w
As h  1, we have Ka = ⇒ [H+ ] = K ah = K a × =
h K aK b Kb

Chapter-7.indd 204 7/29/2016 3:15:59 PM


Summary 205

This expression is independent of the concentration of the salt.


(i) if Ka = Kb, pH = 7 solution will be neutral.
(ii) if Ka > Kb, pH < 7 acidic solution.
(iii) if Ka < Kb, then pH > 7 alkaline solution.
Taking log on both sides and solving, we get
1 1
pH = 7 + logK a − pK b
2 2

Tip The degree of hydrolysis of salts of weak acids and weak bases is unaffected by the dilution because there is no
concentration term in the expression of degree of hydrolysis.

(d) Salt of strong acid and strong base does not undergo salt hydrolysis. The pH in such cases is
1
pH = pK w
2

Tip Degree of hydrolysis always increases with the increase in temperature because at an elevated temperature, the
increase in Kw is greater as compared to Ka and Kb.

8. Buffer solutions
(a) It contains solutes that enable it to resist large changes in pH when small amounts of either strong acid or strong base are added
to it.
(b) Buffers generally consist of mixtures of weak acids and their salts or weak bases and their salts. If the pH of the buffer solution
is less than 7, it is called an acidic buffer and if the pH of the solution is more than 7, it is called a basic buffer.
9. pH of buffer solutions: Henderson–Hasselblach equation
(a) pH of an acidic buffer
[Salt]
pH = pK a + log
[Acid]
(b) pH of a basic buffer
[Salt]
pOH = pK b + log
[Base]

(c) If pOH of an aqueous solution is known, the pH can be calculated as follows.


pH + pOH = 14,
pH = 14 − pOH
[Salt] [Base]
pH = 14 − pK b − log = pK a + log (as pK a + pK b = 14)
[Base] [Salt]
(d) Salt buffer: When a solution of a salt itself acts as a buffer, it is known as a salt buffer. These are salts of a weak acid and weak
base.
(e) Buffer capacity: The buffer capacity (buffer index), is the change in the concentration of buffer acid (or base) required for
changing the pH of its solution by one, while keeping the concentrations of [salt] + [acid] and [base] + [salt] constant.
y (x − y )
The buffer index is given by B.I. = 2.303 ×
x
where x is number of moles of acid and y is number of moles of salt.
(f) Buffer range: It is generally accepted that a solution has useful buffer index provided that the value of [Salt]/[Acid] lies within
the range 10–0.1. Hence, from Henderson equation we have
pKa + log(0.1) < pH < pKa + log10
or (pKa − 1) < pH < (pKa + 1)
Beyond this range, the buffer index is very small for any practical application.
10. Solubility equilibria of sparingly soluble salts
(a) Solubility is defined as the concentration of a saturated solution of a solute in a given solvent at a particular temperature. It
depends on the nature of the solute and solvent, and the temperature and pressure conditions.
(b) Solubility product: The solubility product (Ksp) of an electrolyte may be defined as the maximum product of the concentra-
tions of the constituent ions each raised to the power equal to its ions present in the equation representing the solution of one
molecule of the electrolyte.

Chapter-7.indd 205 7/29/2016 3:16:00 PM


206 Chapter 7 Equilibrium

Consider the dissociation of any sparingly soluble electrolyte, AxBy

A xB y  xA y + + yB x −
Applying the law of chemical equilibrium, we get
[A y + ] x [B x − ] y
K=
[A xB y ]

In saturated solution, [A xB y ] = constant = K ′


Therefore, K sp = [A y + ] x [B x − ] y
(c) Effect of common ion on solubility: The presence of a common ion in the solution decreases the solubility of a given substance.
(d) Effect of pH on solubility: When pH is lowered, the concentration of anion is lowered due to its protonation which in turn
increases the solubility of the salt.

Tip For a given electrolyte, solubility product is always constant at a particular temperature.

(e) Ionic product and relation with solubility product


For a salt solution at a specified concentration, the product of the concentration of the ions, each raised to the appropriate
power, is called as the ionic product (Qsp) for a saturated solution in equilibrium with excess of solid.
(i) If ionic product = solubility product, the solution is in equilibrium.
(ii) If ionic product < solubility product, the solution is unsaturated and more salt can be dissolved in it.
(iii) If ionic product > solubility product, then the solution is containing more salt than it can dissolve, therefore, precipitation
starts and continues till ionic product becomes equal to solubility product.
(f) Preferential precipitation of salts
(i) Selective precipitation means causing one metal ion to precipitate while holding another in solution.
(ii) If the stoichiometry of the precipitated salts is the same, then the salts are precipitated in increasing order of Ksp.
(iii) If the stoichiometry of the precipitated salts is not the same, then with Ksp alone, we cannot predict the ion that will get
precipitated first.
11. Theory of indicators
(a) Ostwald theory
The acid–base indicator shows different colour in ionized and unionized forms.
(i) Phenolphthalein
• It is an acid and can be represented as HPh. Its ionization can be represented as

HPh  H+ + Ph−
(Colourless) (Pink)

From law of chemical equilibrium, we have


[H+ ][Ph− ]
KC =
[HPh]
• In the presence of acid, the dissociation is almost negligible due to common ion H+. So, the equilibrium shifts towards
left and no colour change is observed. However, when alkali is added, the OH− ions remove the excess H+ ions and shift
the equilibrium to the right, thereby increasing the concentration of pink Ph− ions. The ionization constant of indicator is
[Ph− ][H+ ]
K In =
[HPh]
[HPh]
or [H+ ] = K In ×
[Ph− ]
[HPh]
pH = − log10 [H+ ] = − log10 [K In ] − log10
[Ph− ]
[Ph− ]
pH = pK In + log10
[HPh]
This is the Henderson equation for phenophthalien. Now, at the equivalence point, we have [Ph−] = [HPh] and pH = pKIn.
(ii) Methyl orange
• It is a very weak base. It is ionized to give Me+ and OH− ions.
MeOH  Me+ + OH−
Yellow Red

Chapter-7.indd 206 7/29/2016 3:16:03 PM


Summary 207

The ionization constant is


[Me + ][OH]
K=
[MeOH]
• In the presence of acid, the OH− are removed and the equilibrium shifts to the right, thereby increasing the concentration
of Me+ and imparting red colour to the solution.
• By adding alkali, the concentration of OH− ions increases, and the equilibrium shifts to the left, thus ionization of MeOH
reduces, and when it stops altogether, yellow colour appears.
(b) Quinonoid theory
Acid-base indicators are aromatic organic compounds that can exist as tautomers in equilibrium with each other. One of the
tautomer is in quinoid form while the other is in benzenoid form.
(i) Phenolphthalein
OH O
OH−
C+ C
H+
OH OH
O
C COO−
O
Benzenoid form Quinonoid form
(Stable in acid, (Stable in alkali,
colourless) pink in colour)
(ii) Methyl orange
CH3
−O
3S NH N N

CH3
Benzenoid form
(Stable in alkali,
colourless or yellow)

CH3
−O
3S N N N

CH3
Quinonoid form
(Stable in acid,
red in colour)

12. Acid Base titrations curves


Titration curve Features of curve
Strong acid vs. strong base • The curve starts from low pH characteristic of strong
15 acid, and ends at high pH characteristic of strong
14 base.
13 • The pH at the equivalence point of all strong acid–
12 strong base titrations is 7.
11 • Indicators such as phenolphthalein, methyl red and
10
9 methyl orange are suitable for such titration.
Equivalence point,
8 pH = 7.0
pH
7
6
5
4 Volume of NaOH used
3 to reach equivalence
2 point is 25.00 mL.
1
0
0 5 10 15 20 25 30 35 40 45 50
Volume (mL) of 0.2000 M NaOH added
0.2 M HCl is titrated with 0.2 M NaOH.
(Continued)

Chapter-7.indd 207 7/29/2016 3:16:07 PM


208 Chapter 7 Equilibrium

(Continued)
Weak acid vs. strong base • The curve starts from a higher acidic pH and ends at
15 high basic pH.
14 • The pH change at the equivalence point (pH > 7) is
13 not so great.
12 • Phenolphthalein is the suitable indicator for such
11 titration.
Equivalence point,
10
pH = 8.88
9
8
pH
7
6
5
4 Volume of NaOH used
3 to reach equivalence
2 point is 25.00 mL.
1
0
0 5 10 15 20 25 30 35 40 45 50
Volume (mL) of 0.2000 M NaOH added
0.2 M acetic acid is titrated with 0.2 M NaOH.
Weak base vs. strong acid • The curve starts from a higher acidic pH and ends at
15 low basic pH.
14 • There is no sharp pH change at the equivalence
13 point (pH~7) and makes very difficult for titration to
12 perform.
11 • Methyl red and methyl orange are suitable indicators
10
for such titration.
9
8
pH
7 Equivalence point,
6 pH = 5.12
5
4 Volume of HCl used
3 to reach equivalence
2 point is 25.00 mL.
1
0
0 5 10 15 20 25 30 35 40 45 50
Volume (mL) of 0.2000 M HCl added
0.2 M ammonia is titrated with 0.2 M HCl.

13. Titration curve for diprotic acids


When a weak diprotic acid such as ascorbic acid (vitamin C) is titrated with a strong base, there are two protons to be neutralized
and there are two equivalence points.

Second equivalence
point

A2−

pH
HA−

First equivalence
point
H2A

Volume of base added

Chapter-7.indd 208 7/29/2016 3:16:09 PM


Tools for Problem Solving 209

TOOLS FOR PROBLEM SOLVING


1. Estimation of the position of equilibrium
(a) When K is very large, the position of equilibrium lies far to the right (towards the products).
(b) When K is very small, the position of equilibrium lies far to the left (towards the reactants).
2. Almost any problem in which you are given a value of Ka or Kb falls into one of three categories:
(a) The aqueous solution contains a weak acid as its only solute.
(b) The solution contains a weak base as its only solute.
(c) The solution contains both a weak acid and its conjugate base.

Determine the nature of the solute.

The solute consists


The only solute The only solute of both a weak acid
is a weak acid. is a weak base. and a weak base.

Write the equation for


Write the equation for Write the equation for
the ionization of either
the ionization of an acid the ionization of a base
the acid or the base

HA H+ + A− B + H2O BH+ + OH− HA H+ + A−


or
A− + H2O HA + OH−

Write the equilibrium law Write the equilibrium law Write the equilibrium law
based on the equation. based on the equation. based on the equation.

Solve for [OH−] Solve for [H+] or


Solve for [H+]
and pOH, then [OH−], then for pH
and pH.
for pH. (or pOH, then pH).

3. If the percentage ionization is known, you can calculate the change in the concentration of the acid or base and then use that infor-
mation to calculate the Ka or Kb.
moles per litre ionized
Percentage Ionization = × 100
moles per litre available
4. Relationship between Kp and KC
∆ng
K p = K C × (RT )

5. Degree of ionization Number of molecules present as ions


a=
ules of the electrolyte
Total number of molecu
6. Relation between Gibbs energy change and chemical equilibrium

K = e −∆G °/RT

7. Ostwald’s dilution law


a2
KC =
(1− a )V
For weak electrolytes
1
a =K′ V =K′
C
where K ′ = K C .

Chapter-7.indd 209 7/29/2016 3:16:13 PM


210 Chapter 7 Equilibrium

8. Acid dissociation or ionization constant


a 2C
Ka = mol L−1
1− a
9. Solubility product
(a) For 1:1 type salts like AgCl, AgI, BaSO4, PbSO4, etc.
S = K sp
(b) For 1:2 or 2:1 type salts like Ag2CO3, Ag2CrO4, PbCl2, CaF2, etc.
1/ 3
 K sp 
S= 
 4 
(c) For 1:3 type salts like AlI3, Fe(OH)3, Cr(OH)3, Al(OH)3, etc.
1/ 4
 K sp 
S= 
 27 
10. The pH of a mixture of two weak acids can be obtained as:

pH= K a1 × C1 + K a2 × C2
However, if both acids are too weak (pH ranging from 10−10 to 10−14) or the solutions are too dilute the alternate formula from
charge-balance method is given as

[H+ ] = K a1 + C1 + K a2 + C2 + K w
11. Hydrolysis at a glance
Hydrolysis
Salt Degree of hydrolysis constant pH
(i) NaCl (Strong acid + Strong base) No hydrolysis
Kw Kw 1
(ii) CH3COONa (Weak acid + Strong base) h= Kh= pH = [pKw + pKa + logC]
K aC Ka 2

Kw Kw 1
(iii) NH4Cl (Strong acid + Weak base) h= Kh = pH = [pKw – pKb – logC]
K bC Kb 2

(iv) CH3COONH4 Kw Kw 1
h= Kh = pH = [pKw + pKa – pKb]
(Weak acid + Weak base) K aK b K aK b 2

SOLVED EXAMPLES
 x 
Equilibrium in Physical and Chemical Processes Therefore, the partial pressure of PCl3 is p(PCl3 ) =  p
 x + 1
1. Phosphorus pentachloride dissociates as follows, in a closed
reaction vessel, PCl5 (g)  PCl3 (g) + Cl2 (g) . If total pressure at 2. The equilibrium for the reaction SO3 (g)  SO2 (g) + 21 O2 (g)
equilibrium of the reaction mixture is p and degree of dissocia- is KC = 4.9 × 10−2. The value of KC for the reaction
tion of PCl5 is x, the partial pressure of PCl3 will be 2SO2 (g) + O2 (g)  2SO3 (g) will be
 x   2x   x   x  (a) 416 (b) 2.40 × 10−3
(a)  p (b)  p (c)  p (d)  p
 x + 1  1− x   x − 1  1− x  (c) 9.8 × 10−2 (d) 4.9 × 10−2
(AIEEE 2006)
(AIEEE 2006) Solution
Solution
(a) For SO2 + 21 O2  SO3 K C′ = (1/K C )
(a) The reaction can be represented as
PCl5 (g)  PCl3 (g) + Cl2 (g) For 2SO2 + O2  2SO3 K C′ = (1/K C )2
Initial moles 1 0 0 The equilibrium constant is
Moles at equilibrium 1− x x x 2
 1  10 4 10000
The total number of moles at equilibrium = 1 − x + x + x = 1 + x K C′ =   = = = 4.1649 × 100 = 416.49
 4.9 × 10 −2  4.9 × 4.9 24.01

Chapter-7.indd 210 7/29/2016 3:16:20 PM


Solved Examples 211

3. The first and second dissociation constants of an acid H2A are p 1


1.0 × 10−5 and 5.0 × 10−10, respectively. The overall dissociation or =
p′ 36
constant of the acid will be
5. For the following three reactions I, II and III, the equilibrium
(a) 0.2 × 105 (b) 5.0 × 105
constants are given:
(c) 5.0 × 1015 (d) 5.0 × 10−15 K1
(AIEEE 2007) 
(I) CO(g) + H2O(g)  CO2 (g) + H2 (g)
Solution K2

(II) CH4 (g) + H2O(g)  CO(g) + 3H2 (g)
(d) Let us first consider the dissociation of H2A and HA
K3
[HA − ][H+ ] 
(III) CH4 (g) + 2H2O(g)  CO2 (g) + 4H2 (g)
− + K1 =
H2A  HA + H [HA] Which of the following relations is correct?
− + 2− [H+ ][A2 − ]
HA  H + A K2 = (a) K1(K2)1/2 = K3 (b) K2K3 = K1
[HA − ] (c) K3 = K1K2 (d) K3(K2)3 = (K1)2
For the reaction
(AIEEE 2008)
H2A  2H+ + A2 −
we have Solution
+ 2 2− (c) The expression for K3 is
[H ] [A ]
K= = K1 × K 2 = 1× 10 −5 × 5 × 10 −10 = 5 × 10 −15
[H2A ] [CO2 ][H2 ]4
K3 =
[CH4 ][H2O]2
4. The equilibrium constants K p1 and K p2 for the reactions
X  2Y and Z  P + Q, respectively, are in the ratio of 1:9. If which can be obtained by K1 × K2
the degree of dissociation of X and Z be equal, then the ratio of
[CO2 ][H2 ] [CO][H2 ]3 [CO2 ][H2 ]4
total pressure at these equilibria is K 2 × K1 = × = = K3
(a) 1:36 (b) 1:1 (c) 1:3 (d) 1:9 [CO][H2O] [CH4 ][H2O] [CH4 ][H2O]2
(AIEEE 2008)
6. A vessel at 100 K contains CO2 with a pressure of 0.5 atm. Some
Solution
of the CO2 is converted into CO on the addition of graphite. If
(a) The reactions can be expressed as the total pressure at equilibrium is 0.8 atm, the value of Kp is
X  2Y (1) (a) 3 atm (b) 0.3 atm (c) 0.18 atm (d) 1.8 atm
Initial moles 1 0 (AIEEE 2011)
Moles at equilibrium 1− x 2 x Solution
Z  P+Q (2) (d) The reaction can be expressed as
Initial moles 1 0 0
CO2 (g) + C  2CO(g)
At equilibrium 1− x x x
Initial moles p p 0
For reaction (1), let the total pressure be equal to p. The Moles at equilibrium p−x p − x 2x
total number of moles in reaction = 1 − x + 2x = 1 + x.
For reaction (2), let the total pressure be equal to p′. The Total pressure at equilibrium = 0.8 atm (given)
total number of moles in reaction = 1 − x + x + x = 1 + x. Total number of moles at equilibrium = p − x + 2x = p + x
Therefore, Also, since p ∝ n,
2 p 0.5
 2x  =
 p p + x 0.8
1+ x  4 px 2 p2
K p1 = = and Solving, we get x = 0.3. Now, K p = CO
 1− x  1− x 2 pCO2
 p 
1+ x 
where pCO2 = p − x = 0.5 − 0.3 = 0.2 atm and pCO = 2x = 2
 x  x 
 p′  p ′ 2 × 0.3 = 0.6. Therefore,
1+ x   1+ x  p ′ x
K p2 = =
 1− x  1− x 2 0.6 × 0.6
 p ′ Kp = = 1.8 atm
1+ x  0.2
Given that the ratio of K p1/K p2 = 1/ 9 , therefore,
7. The equilibrium constant (KC) for the reaction N2(g) + O2(g)
 4p x
2 2
→ 2NO(g) at temperature T is 4 × 10−4. The value of K′C for the
 
K P1 1  1− x 2 
4p reaction, NO(g) → 21 N2 (g) + 21 O2 (g) at the same temperature is:
= = =
K P 2 9  p ′x 2  2 p ′
(a) 0.02 (b) 2.5 × 102 (c) 4 × 10−4 (d) 50.0
  (AIEEE 2012)
 1− x 2 

Chapter-7.indd 211 7/30/2016 11:58:43 AM


212 Chapter 7 Equilibrium

Solution Total pressure = 2 p + p


(d) N2 (g) + O2 (g)  2NO(g) K C = 4 × 10 −4 = 3p = 3 × 1.94 × 10 −2 = 5.82 × 10 −2 atm
1 1
NO(g)  N2 (g) + O2 (g) K C′ 11. Gaseous N2O4 dissociates into gaseous NO2 according to the
2 2
Relation between KC and K C′ is reaction
N2O 4 (g)  2NO2 (g)
1
K C′ =
KC At 300 K and 1 atm pressure, the degree of dissociation of
N2O4 is 0.2. If one mole of N2O4 gas is contained in a vessel,
1
K C′ = = 50 then the density of the equilibrium mixture is
4 × 10 −4 (a) 1.56 g L−1 (b) 3.11 g L−1
(c) 4.56 g L −1 (d) 6.22 g L−1
(JEE Main Online 2015)
8. For the reaction SO2 (g) + 21 O2 (g)  SO3 (g) if Kp = KC(RT)x Solution
where the symbols have usual meaning then the value of x is
(assuming ideality) (b)
(a) −1 (b) −1/2 (c) 1/2 (d) 1 N2O 4  2NO2
(JEE Main 2014) Initial moles 1 0
Solution Moles at equilibrium 1− x 2x
(b) For the reaction, x = ∆ng = 1 − (1 + 1/2) = −1/2. = 0.8 = 0.4
9. At a certain temperature, only 50% HI is dissociated into H2 Total gaseous moles at equilibrium = 0.8 + 0.4 = 1.2
and I2 at equilibrium. The equilibrium constant is Average molar mass of mixture = xN2 O4 MN2 O4 + xNO2 MNO2
(a) 1.0 (b) 3.0 (c) 0.5 (d) 0.25
(JEE Main Online 2014) 0.8 0.4
Mavg = × 92 + × 46
1.2 1.2
Solution
(d) 2Hl  H2 + I2 = 61.33 + 15.33

At t = 0 1 0 0 x = 0.5 = 76.66
At t = equil. 1 − x x/2 x/2 dRT pMavg 1× 76.66
1 – 0.5 0.25 0.25 p= ⇒ d= = = 3.11 gL−1
Mavg RT 0.0821× 300
The equilibrium constant for dissociation of HI is

[H2 ][I2 ] 0.25 × 0.25 1 12. For the reaction equilibrium, N2O 4 (g)  2NO2 (g) the con-
KC = 2
= = = 0.25 centrations of N2O4 and NO2 at equilibrium are 4.8 × 10−2 and
[HI] 0.5 × 0.5 4
1.2 × 10−2 mol L−1, respectively. The value of KC for the reac-
10. For the decomposition of the compound, represented as tion is
NH2COONH4(s)  2NH3(g) + CO2(g), Kp = 2.9 × 10−5 atm3. (a) 3.3 × 10−2 mol L−1 (b) 3 × 10−1 mol L−1
−3
(c) 3 × 10 mol L −1 (d) 3 × 103 mol L−1
If the reaction is started with 1 mol of the compound, the
total pressure at equilibrium would be: Solution
(a) 1.94 × 10−2 atm (b) 5.82 × 10−2 atm
(c) The equilibrium concentration is
(c) 7.66 × 10−2 atm (d) 38.8 × 10−2 atm
(JEE Main Online 2014) . × 10 −2 )2
[NO2 ]2 (12
KC = = = 3 × 10 −3 mol L−1
Solution [N2O 4 ] (4.8 × 10 −2 )
(b) 13. What is the equilibrium expression for the reaction
NH2COONH4 (s)  2NH3 (g) + CO2 (g) P4 (s) + 5O2 (g)  P4 O10 (s) ?
At t = 0 1 0 0 [P4 O10 ] 1
(a) K C = 5
(b) K C =
At t = equil. 2p p (partial pressure ) [P4 ][O2 ] [O2 ]5
K p = [2p]2 [ p] [P O ]
(c) K C = [O2 ]5 (d) K C = 4 10
5[P4 ][O2 ]
K p = 4 p3
Solution
1/ 3 1/ 3
 Kp   29 × 10 −6  [P4 O10 ]
p = =  (b) The equilibrium constant is given by K C = .
 4   4  [P4 ][O2 ]5
As the concentration of solids is taken as unity, we have
p = (7.25 × 10 −6 )1/ 3 atm
1
p = 1.94 × 10 −2 atm KC =
[O2 ]5

Chapter-7.indd 212 7/30/2016 11:58:50 AM


Solved Examples 213

14. For the reaction CO(g) + Cl2 (g)  COCl2 (g) the ratio Kp/KC is For option (a): ∆ng = 2 − 1 = 1⇒ K p = K C × (RT )
equal to
For option (b): ∆ng = 2 − 2 = 0 ⇒ K p = K C × (RT )0 ⇒ K p = K C
(a) (1/RT) (b) 1.0 (c) RT (d) RT
Solution For option (c): ∆ng = 5 − 0 = 5 ⇒ K p = K C × (RT )5
(a) We know that Kp = KC (RT)∆ng For option (d): ∆ng = 1− 0 = 1⇒ K p = K C × (RT )
where ∆ng = number of moles of products in gaseous
phase − number of moles of reactants in gaseous phase 18. At a certain temperature, KC = 0.64 for the reaction
= 1 − 2 = −1 Kp NO2 (g) + NO(g)  N2O(g) + O2 (g). Equal moles of NO and
1
Substituting, we get K p = K C (RT )−1 ⇒ = NO2 are to be placed in a 5.0 L container until the N2O con-
K C RT
centration at equilibrium is 0.05 M. How many moles of gases
15. An amount of solid NH4SH is placed in a flask already contain- exist in the vessel?
ing ammonia gas at a certain temperature and 0.50 atm pres- (a) 0.225 mol (b) 0.5625 mol
sure. Ammonium hydrogen sulphide decomposes to yield (c) 1.125 mol (d) 2.62 mol
NH3 and H2S gases in the flask. When the decomposition reac- Solution
tion reaches equilibrium, the total pressure in the flask rises to
0.84 atm. The equilibrium constant for NH4SH decomposition (d)
at this temperature is NO2 (g) + NO(g)  N2O (g) + O2 (g)
(a) 0.30 (b) 0.18 (c) 0.17 (d) 0.11 At t = 0 a a 0 0
Solution At equilibrium a− x a− x x x
(d) The reaction involved is However, x/5 = 0.05 ⇒ x = 0.25 mol
NH4HS(s)  NH3 (g) + H2S(g) (0.25/5)2
Kp = = 0.64 ⇒ a − 0.25 = 1 ⇒ a = 1+ 0.25 = 1.31
Initial pressure p0 0.5 0 [(a − 0.25)/5]2 0.25 0.8 0.8
Pressure at equilibrium p0 − p 0.5 + p p
Total moles existing = a + a = 1.31+ 1.31 = 2.62 mol
The total pressure at equilibrium is 0.5 + 2x = 0.84 (given)
Solving, we get x = 0.17. Therefore,
19. At 460°C, the reaction, SO2 ( g) + NO2 ( g)  NO( g) + SO3 (g);
K p = pNH3 × pH2 S = (0.67) × (0.17) = 0.1139 has KC = 85. A reaction flask at 460°C contains these gases
at the following concentrations: [SO2] = 0.0025 M, [NO2] =
0.0035 M, [NO] = 0.025 M and [SO3] = 0.04 M.
16. For the reaction 2NO2 (g)  2NO(g) + O2 (g), KC = 1.8 × 10−6 at
Which of the following statements is correct?
184°C and R = 0.0821 kJ mol−1 K−1. When Kp and KC are com-
(a) Reaction is proceeding towards forward direction.
pared at 184°C it is found that
(b) Reaction is proceeding towards reverse direction.
(a) Kp is greater than KC.
(c) Equilibrium would be affected on addition of inert gas at
(b) Kp is less than KC.
constant pressure.
(c) Kp = KC.
(d) The value of Kp is greater than KC.
(d) Whether Kp is greater than, less than or equal to KC
depends upon the total gas pressure. Solution
Solution (b) The reaction quotient is given by
(a) We know that Kp = KC (RT)∆ng [SO3 ][NO] 0.025 × 0.04
where ∆ng = number of moles of products in gaseous Q= = ≈ 114
[SO2 ][NO2 ] 0.0025 × 0.0035
phase − number of moles of reactants in gaseous phase
=3−2=1 Since Q > KC, this implies that the product concentration
Substituting, we get K p = K C (RT ) ⇒ K p > K C is more than the equilibrium concentration, so, reverse
reaction takes place.
17. For which of the following equilibrium, the value of Kp is not The number of moles of reactants and products are
greater than the value of KC? the same, hence, change in pressure will not affect the
(a) 2C(s) + O2 (g)  2CO(g) equilibrium.
The addition of an inert gas at constant pressure will also
(b) 2C(s) + 2H2O(g)  CH4 (g) + CO2 (g) not affect the equilibrium.
(c) CuSO 4 ⋅ 5H2O(s)  CuSO 4 (s) + 5H2O(g) The two equilibrium constants are related as:
(d) Mg(OH)2 (s)  MgO(s) + H2O(g) K p = K C (RT )
∆ng

Solution where ∆ng = coefficient of gaseous products − coefficient


(b) We know that K p = K C × (RT )∆ng ; where ∆ng = product of gaseous reactants. Since coefficient of reactants and
side gaseous moles − reactant side gaseous moles. products is the same, ∆ng = 0 and so Kp = KC.

Chapter-7.indd 213 7/30/2016 11:58:58 AM


214 Chapter 7 Equilibrium

20. At 25°C, 0.056 mol O2 and 0.02 mol N2O were placed in a 1 L 22. During an experiment, 0.200 mol of H2 and 0.200 mol of I2
container where the following equilibrium was established: were placed into a 1.00 litre vessel where the reaction
2N2O(g) + 3O2 (g)  4NO2 (g).
H2 (g) + I2 (g)  2HI(g)
At equilibrium, the NO2 concentration was 0.02 M. What is
the value of KC for this reaction? came to equilibrium. For this reaction, Kc = 49.5 at the tem-
(a) 23.2 (b) 48.78 (c) 34.33 (d) 0.043 perature of the experiment. What were the equilibrium con-
centrations of H2, I2, and HI?
Solution (a) 0.044 M, 0.044 M, 0.312 M
(a) The reaction involved is (b) 0.013 M, 0.013 M, 0.342 M
(c) 0.023 M, 0.023 M, 0.128 M
2N2O(g) + 3O2 (g)  4NO2 (g) (d) 0.015 M, 0.015 M, 0.285 M
Initial conc. 0.02 0.056 0
Solution
Final conc. 0.02 − 2 x 0.056 − 3 x 4x
(a) Initially we have [H2] = [I2] = 0.200 M.
Now, as [NO2] = 0.02 ⇒ x = 0.005 M. Therefore, [O2] =
[H2] [I2] [HI]
0.056 − 3 (0.005) = 0.041 M, and
[N2O] = 0.02 − 2(0.005) = 0.01M Initial concentration 0.200 0.200 0
Concentration at
[NO2 ]4 (0.02)4 0.200 − x 0.200 – x +2x
KC = = = 23.2 equilibrium
2 3 2 3
[N2O] [O2 ] (0.01) × (0.041)
Substituting the above values for equilibrium concentra-
tions into the expression for mass action gives:
21. At 27°C, K p = 1.5 × 1018 for the reaction,
3NO (g)  N2O (g) + NO2 (g). If 0.03 mol of NO were placed [HI]2 (2x )2
KC = = = 49.5
[H2 ][I2 ] (0.200 − x )(0.200 − x )
in a 1 L vessel and equilibrium were established, what would
be the equilibrium concentration of NO? Take the square root of both the sides of this equation
[R = 0.082 L atm mol−1 K−1] we get
(a) 0 (b) 4 × 10 −8 M 2x
= 7.04
−7
(c) 1.2 × 10 M (d) 1.4 × 10 −8 M (0.200 − x )

Solution This equation is easily solved giving x = 0.156. The


(d) The pressure and concentration equilibrium constants substances then have the following concentrations at
of a reaction are related by the equation equilibrium: [H2] = [I2] = 0.200 – 0.156 = 0.044 M, [HI] =
2 × (0.156) = 0.312 M.
∆ng
K p = K C × (RT ) 23. Equilibrium constant, KC, for the reaction N2(g) +3H2(g) 
For the given reaction, the equation becomes 2NH3(g) at 500 K is 0.061. At a particular time, the analysis
shows that the composition of the reaction mixture is 3.00
K p = K C × (RT )2 − 3 mol L−1 N2, 2.00 mol L−1 H2, and 0.500 mol L−1 NH3. Is the
reaction at equilibrium? If not, in which direction does the
Substituting values, we get
reaction tend to proceed to reach equilibrium?
K C = 1.5 × 1018 × 0.082 × 300 = 3.7 × 1019 (a) Yes, It will remain at equilibrium.
(b) No, the reaction tends to move toward products side.
3NO(g)  N2O(g) + NO2 (g) (c) No, the reaction tends to move toward reactant side.
Initial conc. 0.03 mol 0 0 (d) None of these.
Final conc. 0 0.01 mol 0.01 mol Solution
Equilibrium for the reverse reaction (b) Given that [N2] = 3.0 mol L−1, [H2] = 2.0 mol L−1, [NH3]
3 x mol 0.01− x 0.01− x = 0.50 mol L−1
The reaction quotient for the reaction, N2(g) + 3H2(g)
(0.01− x )2  2NH3(g) is
K C = 3.7 × 1019 =
(3 x )3
[NH3 ]2 (0.50 )2
3 1× 10 −4 −9 QC = = = 0.0104
27 x = ⇒ x = 4.6 × 10 [N2 ][H2 ]3 3 × (2)3
3.7 × 1019
(as K C is very large ⇒ 0.01− x ≈ 0.01) We have KC = 0.061. Thus, at this stage of the reaction,
QC < KC. Therefore, the reaction is not at equilibrium. The
Since volume of vessel is 1 L
reaction is on the left of the equilibrium state. That is, the
[NO] = 3 x = 1.4 × 10 −8 M reaction tends to move toward products.

Chapter-7.indd 214 7/30/2016 11:59:04 AM


Solved Examples 215

24. Example of physical equilibrium is Solution


(a) H2 + I2  2HI (a) The reaction may be expressed as
(b) CaCO3(s)  CaO(s) + CO2(g)
(c) H2O(s)  H2O(l) AB2 (g)  AB(g) + B(g)
(d) PCl5(s)  PCl3 (s) + Cl2(g) 1 0 0
Solution −a +a +a
1− a a a
(c) Physical equilibrium does not involve any chemical
change. It only involves a change in the physical state of 1− a a a
the substance side to form more products. pAB2 = p ; pAB = p ; pB = p
1+ a 1+ a 1+ a
25. Which of the following is incorrect about the chemical a 2 p2
equilibrium? 2 a 2p a 2p
(a) (∆G)T,p = 0 K = (1+ a ) = =
(1− a ) 2
p (1+ a )(1− a ) 1− a
(b) Equilibrium constant is independent of initial concentra- (1+ a )
tion of reactants.
K 1
(c) Catalyst has no effect on the equilibrium state. If 1−a 2  1, then K = a 2 p ⇒ a 2 = or a ∝ or a ∝ V
(d) Reaction stops at equilibrium. p p

Solution
28. For the reversible reaction N2 (g) + 3H2 (g)  2NH3 (g) at
(d) At equilibrium, ∆G = 0 and equilibrium constant is inde- 500°C, the value of Kp is 1.44 × 10−5 when partial pressure is
pendent of initial concentration of reactant. The catalyst measured in atmosphere. The corresponding value of KC, with
does not affect the equilibrium state. The reaction does concentration in mol L−1, is
not stop at equilibrium, but a dynamic balance is there
between the forward and reverse reactions. Thus, the rate 1.44 × 10 −5 1.44 × 10 −5
of forward reaction is equal to the rate of reverse reaction (a) (b)
(0.082 × 500 )−2 (8.314 × 773)−2
at equilibrium.
26. At a certain temperature, the equilibrium constant KC is 16 for 1.44 × 10 −5 1.44 × 10 −5
(c) (d)
the reaction, (0.082 × 773)2 (0.082 × 773)−2
SO2 (g) + NO2 (g)  SO3 (g) + NO(g)
Solution
If 1.0 mol each of all the four gases is taken in a 1 L container, (b) Kp = KC (RT)∆n where Kp = 1.44 × 10−5, ∆n = 2 − 4 = −2.
the concentration of NO2 at equilibrium would be
Hence,
(a) 1.6 mol L−1 (b) 0.8 mol L−1
(c) 0.4 mol L−1 (d) 0.6 mol L−1 1.44 × 10 −5
KC =
Solution (8.314 × 773)−2
(c) The reaction is
29. At temperature T, a compound AB2(g) dissociates according
SO2 (g) + NO2 (g)  SO3 (g) + NO(g) to the reaction
Initialconcentration 1 1 1 1 2AB2 (g)  2AB( g) + B2 (g)
At equilibrium 1− x 1− x 1+ x 1+ x
with degree of dissociation a, which is small compared with
[SO3 ][NO] (1+ x )(1+ x ) unity. The expression for Kp in terms of a and the total pres-
KC = = sure pT is
[SO2 ][NO2 ] (1− x )(1− x )
p a3 p a2
(a) T (b) T
(1+ x )2 (1+ x ) 2 3
16 = ⇒ = 4 ⇒ x = 0.6
(1− x )2 (1− x )
pTa 2 pTa
−1
(c) (d)
[NO2 ] = 1− x = 1− 0.6 = 0.4 mol L 2 3

27. For the reaction AB2 (g)  AB(g) + B(g), if a is negligible with Solution
respect to 1 then degree of dissociation (a ) of AB2 is propor- (a) For the given equilibrium, the equilibrium concentration
tional to is

1 1 1 2AB2(g)  2AB(g) + B2(g)


(I) (II) (III) (IV) V
p V p Equilibrium
c(1 − a) ca ca
(a) (III, IV) (b) (I, II) (c) (I, III) (d) (II, III) concentration 2

Chapter-7.indd 215 7/30/2016 11:59:12 AM


216 Chapter 7 Equilibrium

( pB2 )( pAB )2 Factors Affecting Equilibria and Le Chatelier’s Principle


Therefore, K p =
( pAB2 )2 32. What happens when an inert gas is added to an equilibrium
keeping volume unchanged?
Substituting for pAB, pAB2 and pB2 in terms of mole frac- (a) More product will form
tion and pT, we get (b) Less product will form
(c) More reactant will form
(ca / 2) × (ca )2 pT a 3 × pT (d) Equilibrium will remain unchanged
Kp = × =
[c (1− a )] 2 [c (1+ a /2)]  a (JEE Main Online 2014)
2(1− a )2  1+ 
 2
Solution
As a is small compared with unity, so (d) Addition of inert gas into gas phase equilibrium at con-
a a 3 × pT stant volume does not result in shift. This is because the
1− a  1 and 1+  1 , therefore, K p = addition of non-reactive gas does not change in partial
2 2
pressure of other gases in container. While total pres-
30. In a system, A(s)  2B(g) + 3C(g). If the concentration of C sure increases which does not affect the equilibrium
at equilibrium is increased by a factor of 2, it will cause the constant.
equilibrium concentration of B to change to 33. The increase of pressure on ice water system at constant
(a) two times of its original value. temperature will lead to
(b) one half of its original value. (a) no effect on the equilibrium.
(c) 2 2 times of its original value. (b) a decrease in the entropy of the system.
(d) 1/2 2 times of its original value. (c) a shift of the equilibrium in the forward direction.
(d) an increase in the Gibbs energy of the system.
Solution
(JEE Main Online 2015)
(d) A(s)  2B(g) + 3C(g) Solution
1− x 2x 3x
(c) Ice  Water
K c = 4 x 2 × 27 x 3 = 108 x 5
Increases in pressure favours the reaction towards
A(s) 2B(g) + 3C(g) decrease in volume. Since, volume of ice is greater than
2x × C 6x that of liquid water, thus equilibrium will shift in the for-
ward direction.
108 x 5 = 4 x 2 × C 2 × 216 x 3 ⇒ 108 = C 2 × 216 × 4
34. Consider the reaction equilibrium, 2SO2 (g) + O2 (g)  2SO3 (g)
108 1
C2 = ⇒ C= 2SO2 (g) + O2 (g)  2SO3 (g) with ∆H° = −198 kJ. On the basis of Le Chatelier’s
4 × 216 2 2
principle, the condition favourable for forward reaction is
(a) lowering of temperature as well as pressure.
31. Calculate the equilibrium concentration ratio of C to A if (b) increasing temperature as well as pressure.
2.0 mol each of A and B were allowed to come to equilibrium (c) lowering the temperature and increasing the pressure.
at 300 K (d) any value of temperature and pressure.
o
A + B  C + D; ∆G = 460 cal
Solution
(a) 1.0 (b) 0.5 (c) 0.8 (d) 0.68
(c) According to Le Chatelier’s principle, for an exothermic
Solution reaction, on lowering the temperature will shift the reac-
tion in forward direction. Also, as the number of gaseous
(d) ∆G o = −2.303RT log K moles is decreasing, so on increasing the pressure, the
reaction will shift in forward direction.
460
log K = − = − 0.3329 35. In a cell that utilizes the reaction Zn(s) + 2H+ (aq) → Zn2 + (aq) + H2 (g),
2 × 2.303 × 300
+ 2+
Zn(s) + 2H (aq) → Zn (aq) + H2 (g), addition of H2SO4 to cathode compartment, will
log K = 0.3329 ⇒ K = Antilog of 0.3329 = 0.4646 (a) lower the E and shift equilibrium to the left.
(b) increases the E and shift equilibrium to the left.
A + B  C + D (c) increase the E and shift equilibrium to the right.
Initial concentration 2 2 0 0 (d) Lower the E and shift equilibrium to the right.
At equilibrium 2− x 2− x x x
Solution
[C] (c) This is because according to Le Chatelier’s principle,
x2 =
x
= 0.68
K= = 0.4646 ⇒ increase in H+ ions will shift the equilibrium in the for-
(2 − x )2 [A] 2 − x
ward direction.

Chapter-7.indd 216 7/30/2016 11:59:19 AM


Solved Examples 217

36. The exothermic formation of CIF3 is represented by the Since reaction is endothermic, increasing the tempera-
equation: ture will increase the production of hydrogen gas. Adding
inert gas at constant volume will not affect the partial
Cl2 (g) + 3F2 (g)  2ClF3 (g); ∆ r H = −329 kJ
pressure of gases inside the container, so no change is
observed.
Which of the following will increase the quantity of ClF3 in an
equilibrium mixture of Cl2, F2 and ClF3? 39. In presence of a catalyst, what happens to the chemical
(a) Increasing the temperature equilibrium?
(b) Removing Cl2 (I) Energy of activation of the forward and reverse reactions
(c) Increasing the volume of the container is lowered by the same amount.
(d) Adding F2 (II) Equilibrium amount is not disturbed.
(III) Rates of forward and reverse reaction increase by the
Solution same factor.
(d) According to Le Chatelier’s principle, on increasing the (IV) More product is formed.
concentration of reactants (i.e., adding F2) will shift the (a) (II, III, IV) (b) (I, II, IV)
equilibrium in the forward direction. (c) (I, II, III) (d) (I, III, IV)
37. Consider the equilibrium: Solution
2NaHSO3 (s)  Na2SO3 (s) + H2O(g) + SO2 (g) . (c) Catalyst does not enhance the productivity of the reac-
tion. It speeds up the reaction by lowering the energy of
Which of the following will affect this equilibrium?
activation of the forward and reverse reactions
(I) Adding NaHSO3 to the reaction vessel.
without disturbing the state of the equilibrium.
(II) Removing Na2SO3 from the reaction mixture.
(III) Adding H2O (g) to the reaction vessel. 40. The following reactions are known to occur in a system,
(IV) Increasing the volume of the reaction vessel. CO2 + H2O  H2CO3  H+ + HCO3− . If CO2 escapes from the
(a) I and III only (b) I, II and IV only system, then
(c) III and IV only (d) I, II, III and IV (a) pH will decrease.
(b) hydrogen ion concentration will diminish.
Solution
(c) H2CO3 concentration will remain unchanged.
(c) The reaction exhibits a heterogeneous equilibrium, so, (d) the forward reaction will be favoured.
solid state species would not contribute to the equilib-
rium constant. Solution
2NaHSO3 (s)  Na2SO3 (s) + H2O(g) + SO2 (g) (b) According to Le Chatelier’s principle, in the given reaction
at equilibrium, if CO2 escapes from system, the equilib-
On the product side, there are SO2 and H2O gases which rium will shift more towards the reverse reaction and
contribute to the equilibrium constant. Any factor that hydrogen ion concentration will diminish from solution.
changes the concentration of these gases will affect
the equilibrium. Therefore, addition of water vapour 41. Under what conditions of temperature and pressure, the for-
and increasing the volume of vessel will affect the mation of atomic hydrogen from molecular hydrogen will be
equilibrium. favoured most?
(a) High temperature and high pressure
38. The amount of H2 gas produced by the reaction, (b) High temperature and low pressure
CH4 (g) + 2H2S(g)  CS2 (g) + 4H2 (g) − Heat (c) Low temperature and low pressure
(d) Low temperature and high pressure
can be increased by
(I) increasing pressure. Solution
(II) increasing temperature. (b) H2(g) 
Endothermic
 
  2H
(III) increasing volume of container. 1 mol
Exothermic 2 mol
(IV) adding inert gas by keeping volume constant.
According to Le Chatelier’s principle, with decrease in
(a) I and II only (b) II and III only the pressure, the volume occupied by the system will
(c) 1, II and III only (d) I, II, III and IV increase. As such there will be lesser number of moles
Solution per unit volume. The effect of this change can be counter
balanced if the equilibrium shifts in the direction involv-
(b) CH4 (g) + 2H2S(g)  CS2 (g) + 4H2 (g) − Heat ing increase in number of moles. Hence, in conversion
In this reaction, gaseous moles are increasing and sign of molecular hydrogen to atomic hydrogen decrease in
of heat is indicating that reaction is endothermic. So, the pressure favours the forward reaction.
production of hydrogen gas can be increased by chang- According to Le Chatelier’s principle, the increase in tem-
ing any factor which will shift the equilibrium to the right perature will favour the forward (endothermic) reaction.
hand side. This equilibrium will shift to the right hand This is because the equilibrium shifts towards the direc-
side if pressure is decreased or volume of the container is tion in which heat is absorbed to relieve the system of the
increased. stress of added heat.

Chapter-7.indd 217 7/30/2016 11:59:20 AM


218 Chapter 7 Equilibrium

Acid-Base Concept 45. Out of which of the following pair of acids, the first one is
more acidic?
42. Four species are listed below
(a) HSeO3 and HBrO3
(I) HCO3− (II) H3O + (III) HSO 4− (IV) HSO3F (b) HClO3 and HClO2
Which one of the following is the correct sequence of their (c) H3AsO4 and H3PO4
acid strength? (d) H2CO3 and HNO3
(a) IV < II < III < I (b) II < III < I < IV
(c) I < III < II < IV (d) III < I < IV < II Solution
(AIEEE 2008) (b) In oxoacids, acidity depends on the electronegativ-
ity of central atom and the number of oxygen atoms
Solution
attached.
(c) Acidic strength of compounds can be determined by If central atom is more electronegative, then its acid
comparing stability of conjugate base of given acid. strength is more and presence of more oxygen atoms
Higher the stability of conjugate base higher is acidic causes more acidity.
strength. In the second option, HClO3 contains more oxygen
(I) HCO3−  H+ + CO23 − atoms than HClO2, so HClO3 is more acidic. Therefore
HSeO3 < HBrO3 HClO3 > HClO2
(II) H3O+  H+ + H2O H3AsO4 < H3PO4 H2CO3 < HNO3

(III) HSO 4−  H+ + SO24− 46. The anions of sodium hydrogen sulphate, HSO 4− react with
the phosphate ion, PO3−
4
. Identify the two conjugate acid−
base pairs.
(IV) HSO3F  H+ + SO 3 F −

Thus, the increasing order of stability of conjugate base is HSO 4− (aq)+PO34− (aq) → SO24− (aq)+HPO24− (aq)

CO23 − < SO24− < H2O < SO3F − or I < III < II < IV (a) HSO 4−, PO3− 2− 2−
4 and HPO 4 , SO 4

(b) HSO 4−, SO2− 2− 3−


4 and HPO 4 , PO 4
43. Three reactions involving H2PO 4− are given below:
(c) HSO 4− , HPO2− 2− 2−
4 and HPO 4 , SO 4
(i) H3PO 4 + H2O → H2PO 4− + H3O +
(d) HSO 4− , HPO24− and PO34− , SO24−
(ii) H2PO 4− + H2O → HPO24− + H3O +
Solution
(iii) H2PO 4− + OH− → H3PO 4 + O2 −
(b) The one with the greater number of hydrogens, HPO2− 4 ,
In which of the above does H2PO 4− act as an acid? must be the Brönsted acid, and the other, PO3−
4 , must be
(a) (i) only (b) (ii) only the Brönsted base. Therefore, one conjugate acid–base

(c) (i) and (ii) (d) (iii) only pair is HPO2− 3−
4 and PO 4 . The other two ions, HSO 4 and
(AIEEE 2010) 2−
SO 4 , belong to the second conjugate acid–base pair;
Solution HSO 4− is the conjugate acid and SO2−4 is the conjugate
base.
(b) (i) H3PO 4 + H2O → H2PO 4− + H3O +
Acid Conjugate base 47. Which of the following is the strongest acid?
(a) HClO4 dissolved in glacial CH3COOH.
(ii) H2PO 4− + H2O → HPO24− + H3O + (b) HClO4 dissolved in H2O.
Acid Conjugate base (c) H2SO4 dissolved in H2O.
(d) Glacial CH3COOH dissolved in H2O.
(iii) H2PO 4− + OH− → H3PO 4 + O2 − Solution
Base Conjugate acid
(a) Due to leveling effect of water both HClO4 and H2SO4
Only in reaction (ii) H2PO 4− act as an acid. are equally strong acid in water but in glacial acetic acid
which is differentiating solvent for strong acids HClO4 is
44. The conjugate base of hydrazoic acid is the strongest acid.
(a) N3− (b) N3− (c) N2− (d) HN3− 48. Strength of an acid can be explained on the basis of
(JEE Main Online 2014) (a) its concentration in solution.
(b) its degree of ionization.
Solution (c) both (a) and (b).
(b) (d) It is an inherent property of acid.
HN3 + OH−  N3− + H2O
Solution
Acid Base Conjugate Conjugate
(c) For strong acids (completely ionized) only concentration
base acid is the measure of strength but for weak (incompletely

Chapter-7.indd 218 7/30/2016 11:59:30 AM


Solved Examples 219

ionized) acids both degree of ionization (or Ka) and con- Given that pH = 3, this means −log[H+] = 3 or [H+] = 10−3 M.
centration will be required. Substituting, we get
49. Which of the following indicates the correct order of acidic
strength of the following acids? (10 −3 )2
Ka = = 10 −5
10 −1
(I) HCOOH (Ka = 1.8 × 10–4) (II) CH3COOH (Ka = 1.8 × 10–5)
(III) HCN (Ka = 4.8 × 10–10) (IV) HS– (Ka = 1× 10–14) 52. Hydrogen ion concentration in mol L−1 in a solution of pH
= 5.4 will be
(a) I > II > III > IV (b) IV > III > II > I (a) 3.98 × 108 (b) 3.88 × 106
(c) I > III > II > IV (d) II > III > IV > I (c) 3.68 × 10 −6 (d) 3.98 × 10−6
Solution Solution
(a) Greater the value of Ka for acids more will be the acidic
strength. Thus, the correct order of acidic strength is, (d) The pH is pH = −log[H+ ] ⇒ 5.4 = −log[H+ ].
HCOOH > CH3COOH > HCN > HS– So, [H+ ] = antilog (6.06 ) = 3.98 × 10 −6 .
Acid-Base Equilibria 53. For the reaction: NH3 + N2H+5  N2H4 + NH+4 , which of the fol-
lowing statements is correct?
50. In aqueous solution the ionization constants for carbonic acid
are K1 = 4.2 × 10−7 and K2 = 4.8 × 10−11. (a) N2H5+ is acting as base and ammonia is acting as acid.
Select the correct statement for a saturated 0.034 M solution (b) NH+4 is the conjugate base of N2H5+ .
of the carbonic acid.
(c) N2H4 is the conjugate base of NH3.
(a) The concentration of H+ is double that of CO2−
3 .
(b) The concentration of CO2− (d) N2H5+ is the conjugate acid of N2H4.
3 is 0.034 M.

(c) The concentration of CO2−3 is greater than of HCO3 . Solution
+ −
(d) The concentrations of H and HCO3 are approximately
equal. (d) According to Brönsted, any compound which can furnish
(AIEEE 2010) H+ ions is an acid and the compound which can accept
Solution H+ ions is a base. So, acid and base actions are just oppo-
site to each other. Also, removal of one H+ from any com-
(d) The reactions are pound produces its conjugate base and addition of H+
H2CO3  H+ + HCO3− K1 = 4.2 × 10 −7 (1) produces its conjugate acid.
In the equilibrium, NH3 + N2H+5  N2H4 + NH+4 , ammonia
HCO3− H +
+ CO32 −
K 2 = 4.8 × 10 −11
(2) is gaining protons, and hence acting as a base. Similarly,
 +
As K2 K1, all the H ions (in total) are due to Eq. (1) only. N2H5+ is losing protons, so it is acting as an acid. In this
From the first equation, way, conjugate acid of ammonia is NH+4 and conjugate
base of N2H5+ is N2H4. So, N2H+5 is the conjugate acid of
[H+ ]A = [HCO3 − ] = [H+ ]total N2H4.
[CO2− 2−
3 ] is negligible as compared to [HCO3 ] or [H ]total.
+ 54. Correct options for the following reaction is
+ −
So, the concentrations of H and HCO3 are approximately NH3 + H2O  NH4+ + OH−
equal.
1
51. The pH of a 0.1 molar solution of the acid HQ is 3. The value of (a) K a(NH+ ) + K b(NH3 ) = 10 −14 (b) K a(NH+ ) × K b(NH3 ) =
4 4
Kw
the ionization constant, Ka of this acid is:
(a) 3 × 10−1 (b) 1 × 10−3 (c) K a(NH+ ) × K b(NH3 ) = K w (d) pK a(NH+ ) + pK b(NH3 ) = 10
(c) 1 × 10−5 (d) 1 × 10−7 4 4

(AIEEE 2012)
Solution
Solution
(c) For weak acids and weak bases K a × K b = K w .
(c) Consider the dissociation of acid HQ
55. The self-ionization constant for pure formic acid,
HQ  H+ + Q − K = [HCOOH+2 ][HCOO − ] has been estimated as 10−6 at room
The acid dissociation constant is temperature. What percentage of formic acid molecules in
pure formic acid are converted to formate ion? The density of
[H+ ][Q − ] [H+ ]2 formic acid is 1.22 g cm−3.
Ka = =
[HQ] [HQ] (a) 0.0037 (b) 0.037 (c) 0.37 (d) 1.037
+ −
As at equilibrium [H ] = [Q ]. Solution
(a) Given density of formic acid = 1.22 g cm–3
[H+ ]2
or Ka = ⇒ K a ⋅ C = [H+ ]2 Weight of formic acid in 1 L solution = 1.22 × 103 g
C

Chapter-7.indd 219 7/30/2016 11:59:38 AM


220 Chapter 7 Equilibrium

1.22 × 103 (NH4 )2 SO 4 → 2NH+4 + SO24−


[HCOOH] = = 26.5 M
46 NH4+ + H2O  NH4 OH + H+
In the case of autoionization, [HCOOH2+ ] = [HCOO − ]
58. The pKa of a weak acid, HA, is 4.80. The pKb of a weak base,
So, [HCOOH2+ ][HCOO − ] = 10 −6 −
⇒ [HCOO ] = 10 −3 BOH, is 4.78. The pH of an aqueous solution of the corre-
sponding salt, BA, will be
Now percentage dissociation of (a) 9.58 (b) 4.79 (c) 7.01 (d) 9.22
(AIEEE 2008)
[HCOO − ] × 100 10 −3 × 100
HCOOH = = = 0.00377
[HCOOH] 265 Solution

56. Ascorbic acid (vitamin C) is a diprotic acid, H2C6H6O6. [H+], (c) The reaction is
HO
pH and [C6H6O2− 6 ] in a 0.10 M solution of ascorbic acid will be BA  B+ + A − 
2
→ AH + OH−

 →
respectively H 2O
(Given K a1 = 6.8 × 10 −5 and K a2 = 2.7 × 10 −12 for ascorbic acid).
(a) 3.6 × 10−5 M, 2.58, 2.7 × 10 −12 M BOH + H+
(b) 2.6 × 10−3 M, 5.18, 1.5 × 10 −10 M Now since,
(c) 2.6 × 10−3 M, 2.58, 2.7 × 10 −12
M Kw + Ka 1 1
[H+ ] = = 7 + pK a − pK b
(d) 1.5 × 10−4 M, 3.67, 1.7 × 10 −14 M Kb 2 2

Solution Therefore, pH = 7.01.


(c) [H+] is determined by the first protic equilibrium 59. Assuming that the degree of hydrolysis is small, the pH of 0.1 M
solution of sodium acetate (Ka = 1.0 × 10−5) will be
H2C6H6O6  H +
+ HC6H6O6−
(a) 5.0 (b) 6.0 (c) 8.0 (d) 9.0
x2 (JEE Main Online 2014)
The mass action expression is: K a1 = 6.8 × 10 −5 =
0.10 Solution
So, x = [H+] = 2.6 × 10−3 M and pH = –log (2.6 × 10−3) (d) pH is given by
= 2.58.
1
The concentration of the anion, [HC6H6O6− ], is given pH = (p K w + p K a + log C )
almost entirely by the second ionization equilibrium 2
1
HC6H6O6−  H+ + C6H6O26 − pH = (14 + 5 − 1) = 9
2
for which the mass action expression is
60. The aqueous solution of NH4F is
[H+ ][C6H6O26 − ]
K a2 = = 2.7 × 10 −12 [Given that: K a (NH+4 ) = 5.6 × 10 −10 and K a (HF ) = 6.8 × 10 −4 ]
[HC6H6O6− ]
(a) acidic (b) basic
Using the value of x from the first step above gives (c) neutral (d) data insufficient
(2.6 × 10 −3 )[C6H6O26 − ] Solution
2.7 × 10 −12 =
(2.6 × 10 −3 ) (a) In the aqueous solution of NH4F, NH+4 and F−, both will
hydrolyzed because both come from weak base and
So, [C6H6O26 − ] = 2.7 × 10 −12 M .
weak acid, respectively.
Hydrolysis of F− produces hydroxide ions, so it works as
Hydrolysis of Salts and pH Determination of their base F − + H2O  HF + HO −
Aqueous Solutions Kw
The base dissociation constant, K b = = 1.5 × 10 −11
57. Regular use of which of the following fertilizers increases the K a (HF )
acidity of soil?
Now it is clear that NH+4 is a slightly stronger acid than
(a) Ammonium sulphate
basic strength of F− ions. So, the solution will be acidic.
(b) Potassium nitrate
(c) Urea 61. The pH of a 0.1 M aqueous solution of Na2CO3 is adjusted to
(d) Superphosphate of lime 12 using a strong base. What is the degree of hydrolysis of car-
(AIEEE 2007) bonate ions? [Given: K a1 = 4.5 × 10 −7 and K a2 = 4.7 × 10 −11
Solution for H2CO3]
(a) Ammonium sulphate is a salt of strong acid and weak (a) 2.1× 10 −2 (b) 4.6 × 10 −2
base. On hydrolysis, it produces H+ ions which increase
the acidity of soil. (c) 2.2 × 10 −4 (d) 4.7 × 10 −10

Chapter-7.indd 220 7/30/2016 11:59:46 AM


Solved Examples 221

Solution pH = 4.5 + log 1 = 4.5


(a) Na2CO3 is the salt of weak acid and strong base, so, hydrol-
ysis of anions, CO2− Therefore, pOH = 14 − pH = 14 − 4.5 = 9.5
3 occurs. But since strong base is also
present in the solution, hydrolysis of CO2−3 ions is sup- 64. In some solutions, the concentration of H3O+ remains con-
pressed; further hydrolysis of HCO3− is suppressed, and
stant even when small amounts of strong acid or strong base
becomes negligible. Let the degree of hydrolysis be h, now
are added to them. These solutions are known as:
CO23 − (aq) + H2O  HCO3− (aq) + OH− (aq) (a) Ideal solutions (b) Colloidal solutions
(c) True solutions (d) Buffer solutions
Initial 0.1 0 0
(JEE Main Online 2014)
Final ( − h)
0.11 0.1h 10 −2
Solution
Kw 10 −14 (d) Buffer is an aqueous solution consisting mixture of weak
Kh = = = 2.12 × 10 −4 acid and its conjugate base, or vice versa. Buffer solution
K a2 4.7 × 10 −11
Now, resists change in pH when a small quantity of acid or base
is added.
0.1h × 10 −2 0.1h × 10 −2
Kh = ⇒ 2.12 × 10 −4 = ⇒ h ≈ 2.12 × 10 −2 Equilibria for Sparingly Soluble Salts
( − h)
0.11 ( − h)
0.11
(as 1  h) 65. Given the data at 25°C,
62. What is the pH of the mixture obtained by mixing 50 mL of Ag + I−  AgI + e − ; E o = 0.152 V
0.05 M NH4OH and 50 mL of 0.05 M CH3COOH [Ka = 1.8 × 10−5
and Kb = 1.8 × 10−5]? Ag  Ag+ + e − ; E o = −0.800 V
(a) 2 (b) 7 (c) 5 (d) 6
Solution What is the value of log Ksp for AgI?
(a) −8.12 (b) +8.612 (c) −37.83 (d) −16.13
(b) The reaction involved is (AIEEE 2006)
NH4 OH + CH3COOH  CH3COONH4 + H2O
Solution
Before hydrolysis 2.5 2.5 0 0
(d) From Nernst equation, we have
After hydrolysis 0 0 2.5 2.5
o 2.303RT
In this case, pH is decided on the basis of hydrolysis of Ecell = log K sp (1)
nF
salt.
2.5 AgI + e −  Ag + I− ; E o = −0.152 V
[CH3COONH4 ] = = 2.5 × 10 −2 M
100 Ag  Ag+ + e − ; E o = −0.800 V
Therefore, AgI(s) → Ag+ + I− ; E o = −0.952
1 Substituting in Eq. (1), we have
pH = + [log K b − log K w − log K a ]
2
1 0.059 0.952
= [pK w + pK a − pK b ] −0.952 = log K sp ⇒ logK sp = − = − 16.135
2 1 0.059
1
= [log (1.8 × 10 −5 ) + log10 −14 − log(1.8 × 10 −5 )] 66. In a saturated solution of the sparingly soluble strong elec-
2 trolyte AgIO3 (Molecular mass = 283 g mol−1) the equilibrium
=7 which sets in is
AgIO3 (s)  Ag+ (aq) + IO3− (aq)
Buffer Solutions
If the solubility product constant Ksp of AgIO3 at a given tem-
63. The pKa of a weak acid (HA) is 4.5. The pOH of an aqueous
perature is 1.0 × 10−8, what is the mass of AgIO3 contained in
buffered solution of HA in which 50% of the acid is ionized is
100 mL of its saturated solution?
(a) 7.0 (b) 4.5 (c) 2.5 (d) 9.5
(a) 1.0 × 10−4 g (b) 28.3 × 10−2 g
(AIEEE 2007) −3
(c) 2.83 × 10 g (d) 1.0 × 10−7 g
Solution (AIEEE 2007)
(d) For buffer solution (Henderson’s equation) Solution
[Salt] [Salt] (c) The reaction involved is
pH = pK a + log = 4.5 + log
[Acid] [Acid]
As the acid is 50% ionized, which means that [Salt] AgIO3 (s)  Ag+ + IO3−
= [Acid] Let S be the solubility product of AgIO3, then

Chapter-7.indd 221 7/30/2016 11:59:52 AM


222 Chapter 7 Equilibrium

K sp = [Ag+ ][IO3− ] K sp = [Mg2 + ][OH− ]2


or 1.0 × 10 −8 = S 2 ⇒ S = 10 −4 mol L−1 1/ 2 1/ 2
 K sp   1× 10 −11 
−4 100 Therefore, [OH− ] =   =  = 10 −4 M
or S = 10 × 283 × = 283 × 10 −5 = 2.83 × 10 −3 g  [Mg2 + ]   10 −3 
1000
per 100 mL Therefore, pOH = −log[OH−] = 4.
Now, since pH + pOH = 14, we get pH = 10.

67. Solid Ba(NO3)2 is gradually dissolved in a 1.0 × 10−4 M Na2CO3 70. Zirconium phosphate [Zr3(PO4)4] dissociates into three zir-
solution. At what concentration of Ba2+ will a precipitate conium cations of charge +4 and four phosphate anions
begin to form? (Given Ksp for BaCO3 = 5.1 × 10−9.) of charge −3. If molar solubility of zirconium phosphate is
(a) 8.1 × 10−8 M (b) 8.1 × 10−7 M denoted by S and its solubility product by Ksp then which of
−5
(c) 4.1 × 10 M (d) 5.1 × 10−5 M the following relationship between S and Ksp is correct?
(AIEEE 2009) (a) S = {Ksp/(6912)1/7} (b) S = {Ksp/144}1/7
(c) S = {Ksp/6912}1/7 (d) S = {Ksp/(6912)7}
Solution
(d) The reaction is Ba(NO3 )2 + CaCO3  BaCO3 + 2NaNO3 (JEE Main Online 2014)
Solution
Here [CO2− −4
3 ] = [Na2CO3] = 10 M. Therefore, the solubil- (c) The solubility product is calculated as
ity product is
K sp = [Ba2 + ][CO23 − ] [Zr3 (PO 4 )]4  3Zr 4 + + 4 PO 43 −
or 5.1× 10 −9 = [Ba2 + ] × 10 −4 (1− S ) 3S 4S

we get [Ba2+] = 5.1 × 10−5. K sp = [Zr ] 4+ 3


[PO34− ]4
At this value, precipitation just starts. K sp = [3S ]3 [ 4 S ]4
68. Solubility product of silver bromide is 5.0 × 10−13. The quan- K sp = 27 × 256S 7
tity of potassium bromide (molar mass taken as 120 g mol−1
1/ 7
to be added to 1 L of 0.05 M solution of silver nitrate to start  K sp 
S=
the precipitation of AgBr is  6912 
(a) 5.0 × 10−8 g (b) 1.2 × 10−10 g
(c) 1.2 × 10−9 g (d) 6.2 × 10−5 g 71. The molar solubility of a sparingly soluble salt MX4
(AIEEE 2010) (in mol L−1) is S. The corresponding solubility product Ksp in
terms of S is given by the relation
Solution
1/ 4 1/ 5
(c) The reaction involved is Ag+ + Br −  AgBr  K sp   K sp 
(a) S =  (b) S = 
Precipitation will start only if the ionic product just  128   256 
exceeds the solubility product
(c) S = (256 K sp )1/ 5 (d) S = (256 K sp )1/ 4
K sp = [Ag+ ][Br − ]
Solution
K sp 5 × 10−13 (b) Consider the reaction: MX 4  M4 + + 4X −
Therefore, [Br − ] = = = 10 −11 M S 4S 1/ 5
[Ag+ ] 0.05 4  K sp 
5
The solubility product is K sp = S( 4 S ) = 256 S ⇒ S = 
That is, precipitation just starts when 10−11 mol of KBr is  256 
added to 1 L of AgNO3 solution. Now, number of moles 72. The solubility product of a salt having general formula MX2,
of KBr to be added = 10−11. Therefore, in water is: 4 ×10−12. The concentration of M2+ ions in the
Weight of KBr to be added = Number of moles × Molar mass aqueous solution of the salt is
(a) 2.0 × 10−6 M (b) 1.0 × 10−4 M
= 10 −11 × 120 = 1.2 × 10 −9 g −4
(c) 1.6 × 10 M (d) 4.0 × 10−10 M
Solution
69. At 25°C, the solubility product of Mg(OH)2 is 1.0 × 10−11. At 2+ −
(b) The reaction is MX2  M + 2X .
which pH will Mg2+ ions start precipitating in the form of S 2S
Mg(OH)2 from a solution of 0.001 M Mg2+ ions? So, the solubility product is K sp = S(2S )2 = 4 S 3
(a) 8 (b) 9 (c) 10 (d) 11 1/ 3
(AIEEE 2010)  K sp 
or S= = 1× 10 −4 M
Solution  4 
(c) The reaction is Mg2 + + 2OH−  Mg(OH)2 . The solubility 73. The precipitate of CaF2 (Ksp = 1.7 × 10−10) is observed when
product is equal volumes of the following are mixed.

Chapter-7.indd 222 7/30/2016 11:59:57 AM


Advanced Level Problems 223

(a) 10 −4 M Ca2+ + 10 − 4 M F −
1.2 × 10 −12 = (2S )2 (0.2 + S ) ≈ 4 S 2 × 0.2
−2 2+ −3 −
(b) 10 M Ca + 10 MF
(c) 10 −5
M Ca2+
+ 10 −3
M F− ⇒ S = 1.5 × 10 −12 = 1.22 × 10 −6 mol L−1

(d) 10 −3 M Ca2+ + 10 −5 M F − (as 0.2  S, so S + 0.2 ≈ 0.2)

Solution 75. What is the calculated molar solubility in water at 25°C of


(b) The reaction is AgBr (Ksp = 5.0 × 10−13)?
CaF2  Ca2 + + 2F − (a) 3.1 × 10–8 M (b) 5.1 × 10–7 M
(c) 7.1 × 10–7 M (d) 4.1 × 10–5 M
The solubility product is K sp = [Ca2 + ][F − ]2 = 1.7 × 10 −10
Solution
The ionic product is K I = [Ca2 + ][F − ]2 (c) AgBr(s)  Ag+(aq) + Br–(aq) Ksp = [Ag+][Br–] = 5.0 × 10−13
For option (a): K I = [Ca2 + ][F − ]2 = 0.5 × 10 −4 × (0.5 × 10 −4 )2 = 1.25 × 10 −13 AgBr(s)  Ag+ (aq) + Br − (aq)
2+ − 2 −4 −4 2 −13
][F ] = 0.5 × 10 × (0.5 × 10 ) = 1.25 × 10 Initial conc. 1 0 0
For option (b): K I = [Ca2 + ][F − ]2 = 0.5 × 10 −2 × (0.5 × 10 −3 )2 = 1.25 × 10 −9 Conc. at equilibrium 0 x x
2+
][F − ]2 = 0.5 × 10 −2 × (0.5 × 10 −3 )2 = 1.25 × 10 −9 Substituting the above values for equilibrium concentra-
2+ − 2
For option (c): K I = [Ca ][F ] = 0.5 × 10 −5 −3 2
× (0.5 × 10 ) = 1.25 × 10 −12 tions into the expression for Ksp gives
2+
][F − ]2 = 0.5 × 10 −5 × (0.5 × 10 −3 )2 = 1.25 × 10 −12 Ksp = 5.0 × 10–13 = [Ag+][Br–] = (x)(x)

For option (d): K I = [Ca2 + ][F − ]2 = 0.5 × 10 −3 × (0.5 × 10 −5 )2 = 1.25 × 10 −14 x = 5.0 × 10 −13 = 7.1 × 10 −7 M
2+ − 2 −3
][F ] = 0.5 × 10 × (0.5 × 10 −5 )2 = 1.25 × 10 −14 Thus, the solubility is 7.1 × 10−7 M.
In the case of (b), ionic product is greater than the solubil- 76. The [Ag+] ion in a saturated solution of Ag2CrO4 at 25°C is
ity product, so precipitation will be observed in this case. 1.5 × 10−4 M. What is the Ksp of Ag2CrO4 at 25°C?
74. The solubility of Ag2CrO4 at 25°C in 0.2 M Na2CrO4 is (a) 2.5 × 10−12 mol3 L−3 (b) 1.688 × 10−12 mol3 L−3
(Given Ksp (Ag2CrO4) at 25°C = 1.2 × 10−12) (c) 3.9 × 10−14 mol3 L−3 (d) 1.9 × 10−10 mol3 L−3

(a) 1.22 × 10−6 (b) 2.45 × 10−6 Solution


(c) 3.0 × 10−12 (d) 6.0 × 10−12 (b) For saturated solution of Ag2CrO4, if solubility is S mol L−1.
Solution Then,
AgCrO 4 (s)  2Ag+ (aq) + CrO 4− (aq)
(a) The reaction involved is 2S S
Ag2CrO 4 ( s )  2Ag+ (aq) + CrO 4− (aq) AgCrCO 4 (s)  2Ag (aq) + CrO24− (aq)
+

The solubility product is Ksp = [Ag+][CrO 4− ] = (2S)2 (S) = 4S3.


K sp = [Ag+ (aq)]2 [CrO 4− (aq)]
Given that [Ag+] = 2S = 1.5 × 10−4 ⇒ S = 0.75 × 10−4.
In the solution of 0.2 M Na2CrO4, [CrO 4− (aq)] = 0.2 M Substituting in the above expression, we get
Let the solubility of Ag2CrO4 in sodium chromate solution Ksp = 4(0.75 × 10−4)3 = 1.688 × 10−12 mol3 L−3
is S mol L−1. Then,

ADVANCED LEVEL PROBLEMS


[Ag(NH3 )+ ]
1. Ag+ + NH3  [Ag(NH3)]+; K1 = 3.5 × 10-3 K1 = = 3.5 × 10 −3 (1)
[Ag+ ][NH3 ]
[Ag(NH3)]+ + NH3  [Ag(NH3)2]+; K2 = 1.7 × 10−3
Then the formation constant of [Ag(NH3)2]+ is [Ag(NH3 )]+ + NH3  [Ag(NH3 )2 ]+
(a) 6.08 × 10−6 (b) 6.08 × 106
(c) 6.08 × 10−9 (d) None of these [Ag(NH3 )2 ]+
K2 = = 1.7 × 10 −3 (2)
(IIT-JEE 2006) [Ag(NH3 )]+ [NH3 ]
Solution From Eq. (1) and (2), we get
(a) We have to calculate
[Ag(NH3 )+ ] [Ag(NH3 )2 ]+
Ag+ + 2NH3  [Ag(NH3 )2 ]+ K1 ⋅ K 2 = ⋅
[Ag ][NH3 ] [Ag(NH3 )]+ [NH3 ]
+
+
[Ag(NH3 )2 ]
K= [Ag(NH3 )2 ]+
[Ag+ ][NH3 ]2 K1 ⋅ K 2 = =K
+ + [Ag+ ][NH3 ]
Ag + NH3  [Ag(NH3 ) ]

Chapter-7.indd 223 7/30/2016 12:00:05 PM


224 Chapter 7 Equilibrium

Substituting the values of K1 and K2, we get Also, we have M1V1(salt) = M2V2 (BOH), that is,
−3 −3 2
K = 3.5 × 10 × 1.73 × 10 M1 × (2.5 + 7.5) = 2.5 × ⇒ M1 = 0.1 M
−6
5
= 6.08 × 10 Now, Ka × Kb = Kw where Ka = Ca 2/(1 − a). Therefore,
2. Which is correct statement if N2 is added at equilibrium
condition? Kw a2
= 0.1× ⇒ a = 0.27
N2 + 3H2  2NH3 Kb 1− a
Hence, [H+] = Ca = 2.7 × 10−2 M.
(a) The equilibrium will shift to forward direction because
according to IInd law of thermodynamics the entropy 5. Solubility product constants (Ksp) of salts of types MX, MX2 and
must increases in the direction of spontaneous reaction. MX3 at temperature T are 4.0 × 10–8 M2, 3.2 × 10–14 M3 and
(b) The condition for equilibrium is 2.7 × 10–15 M4, respectively. Solubilities (mol dm–3) of the salts
at temperature T are in the order
GN2 + 3GH2 = 2GNH3
(a) MX > MX2 > MX3 (b) MX3 > MX2 > MX
where G is Gibbs free energy per mole of the gaseous
(c) MX2 > MX 3 > MX (d) MX > MX3 > MX 2
species measured at that partial pressure. The condition
of equilibrium is unaffected by the use of catalyst, which (IIT-JEE 2008)
increases the rate of both the forward and reverse reac- Solution
tions to the same extent. (d) Solubility of MX = (4.0 × 10–8)1/2 = 2 × 10–4 mol L–1
(c) The catalyst will increase the rate of forward reaction by a
1/ 3
and that of reverse reaction by b.  3.2 × 10 −14 
(d) Catalyst will not alter the rate of either of the reaction. Solubility of MX2 =   = (8 × 10 −15 )1/ 3 = 2 × 10 −5 mol L−1
 4 
(IIT-JEE 2006)
1/ 4
 2.7 × 10 −15 
Solution Solubility of MX3 =   = (1× 10 −16 )1/ 4 = 1× 10 −4 mol L−1
 27 
(b) Because change in Gibbs free energy is given by:
∆G (reaction) = ∆G (products) − ∆G (reactants) Therefore, MX > MX3 > MX2
Also, catalyst only speeds up the reaction without affect-
ing the equilibrium conditions. 6. The dissociation constant of a substituted benzoic acid at
25°C is 1.0 × 10–4. The pH of a 0.01 M solution of its sodium salt
3. The species present in solution when CO2 is dissolved in water is . (IIT-JEE 2009)
are
Solution
(a) CO2 , H2CO3 , HCO3− , CO23 (b) H2CO3 , CO23 (8) Given that Ka (C6H5COOH) = 1 × 10−4 and pH of 0.01 M
(c) CO23 − , HCO3− (d) CO2 , H2CO3 C6H5COONa is to be calculated. The reaction can be
(IIT-JEE 2006) expressed as
Solution
C6H5COO − + H2O  C6H5COOH + OH −
(a) CO2 + H2O  H2CO3
Initial moles 0.01 0 0
H2CO3  H+ + HCO3−
Moles at 1 − h)
0.01(1 0.01h 0.01h
HCO3−  H+ + CO23 − equilibrium

4. 2.5 mL of (2/5) M weak monoacidic base (Kb = 1 × 10–12 at 25°C) The hydrolysis constant is given by
is titrated with (2/15) M HCl in water at 25°C. The concentra- K w 0.01h2 10 −14 10 −2 h2
tion of H+ at equivalence point is (Kw = 1 × 10–14 at 25°C). Kh = = ⇒ −4 =
Ka 1− h 10 1− h
(a) 3.7 × 10–13 M (b) 3.2 × 10–7 M
Now, 1 − h is approximately equal to 1, so
(c) 3.2 × 10–2 M (d) 2.7 × 10–2 M
h = (10−14 + 4 + 2)1/2 = 10−4
(IIT-JEE 2008)
− −
Solution ] = ]0=.001.h01=h0=.001.×010×−140=
[OH[OH
Hence, −410 −6 −
= 10 M6 M
(d) The reactions involved are and [H+[H −14 14 −6 −6 −8
] = 10 −/10
] =+ 10 = 10 −M8 M
/10 = 10
BOH + HCl → BCl + H2O Therefore, pH = 8.
7. Aqueous solutions of HNO3, KOH, CH3COOH, and CH3COONa
B+ + H2O  BOH + H+ of identical concentrations are provided. The pair(s) of solu-
Initial moles C a a tions which form a buffer upon mixing is (are)
Moles at equilibrium C − Ca Ca Ca (a) HNO3 and CH3COOH.
(b) KOH and CH3COONa.
Using M1V1(BOH) = M2V2(HCl), we get (c) HNO3 and CH3COONa.
2 2 (d) CH3COOH and CH3COONa.
2.5 × = × V2 ⇒ V2 = 7.5 mL (IIT-JEE 2010)
5 15

Chapter-7.indd 224 7/30/2016 12:00:11 PM


Advanced Level Problems 225

Solution
x 1.6 × 10 −10 −4 10 4
(c), (d) In (c), if HNO3 is present in limiting amount then this = = 1 .6 × 10 ⇒ y =
y 1.0 × 10 −6 1.6
mixture will be a buffer and in (d), the mixture contains a
weak acid, that is, acetic acid and its salt of strong base, that is, Putting this value of y in Eq. (1), we get
sodium acetate.  10 4 
x ×x + x  = 1.6 × 10 −10 ⇒ x 2 = (1.6 )2 × 10 −14 ≈ 1.6 × 10 −14
 1.6 
8. The total number of diprotic acids among the following is
. or x = 1.6 × 10 −7 = [Ag+ ]. Therefore, n = 7.
H3PO4 H2SO4 H3PO3 H2CO3 H2S2O7
H3BO3 H3PO2 H2CrO4 H2SO3 11. The initial rate of hydrolysis of methyl acetate (1 M) by a weak
acid (HA, 1 M) is 1/100th of that of a strong acid (HX, 1 M), at
(IIT-JEE 2010)
25°C. The Ka of HA is
Solution
(a) 1 × 10−4 (b) 1 × 10−5 (c) 1 × 10−6 (d) 1 × 10−3
(6) The diprotic acids are: H2SO4, H3PO3, H2CO3, H2S2O7,
(JEE Advanced 2013)
H2CrO4, H2SO3
H3PO3 is a diprotic acid since one of the protons (hydro- Solution
gen) is bound to the phosphorous atom. (a) The rate with respect to weak acid is
9. The equilibrium rate1 = k[H+ ]weak [Ester ] and rate with respect to strong acid
2Cu (I)  Cu (0) + Cu (II) is rate2 = k[H+ ]strong[Ester ]
in aqueous medium at 25°C shifts towards the left in the
presence of Now, given that
(a) NO3− (b) Cl− (c) SCN− (d) CN−
[H+ ]strong 1
(IIT-JEE 2011) [H+ ]weak = = = 0.01 M
Solution 100 100
(b), (c), (d) The reaction of Cu2+ ions with CN−, SCN− and CuCl2 The reaction involved is
will lead to the formation of [Cu(CN)4]3−, [Cu(SCN)4]3− and CuCl
HA  H+ + A −
which will shift the reaction in the reverse direction. 1− 0.01≈1 0.01 0.01

Cu2 + + 2CN−  Cu(CN)2 [H+ ][A − ] 0.01× 0.01


Ka = = = 10 −4
2Cu(CN)2  2CuCN + (CN)2 [HA ] 1
CuCN + 3CN−  [Cu(CN)4 ]3 − 12. The Ksp of Ag2CrO4 is 1.1 × 10–12 at 298 K. The solubility
Cu2 + + 4SCN−  [Cu(SCN)4 ]3 − (in mol L−1) of Ag2CrO4 in a 0.1 M AgNO3 solution is
CuCl2 + Cu  2CuCl (a) 1.1 × 10−11 (b) 1.1 × 10−10
(c) 1.1 × 10−12 (d) 1.1 × 10−9
10. In 1 L saturated solution of AgCl (Ksp (AgCl) = 1.6 × 10−10),
(JEE Advanced 2013)
0.1 mol of CuCl (Ksp (CuCl) = 1.0 × 10−6) is added. The resultant
concentration of Ag+ in the solution is 1.6 × 10−n]. The value Solution
of n is (IIT-JEE 2011) (b) The reaction involved is
Solution
(7) The reaction involved is Ag2CrO 4  2Ag+ + CrO24−
S 0.1+ 2 S ≈ 0.1 S
AgCl(s)  Ag+ + Cl−
The solubility product is
x x
CuCl(s)  Cu+ + Cl− K sp = [Ag+ ]2 [CrO24− ] ⇒ 1.1× 10 −12 = (0.1)2 × S
y y
Net Cl− in solution = (x + y). Now, the solubility product of 1.1× 10 −12
Therefore, S = [CrO24− ] = = 1.1× 10 −10 mol L−1
AgCl is (0.1)2
13. The thermal dissociation equilibrium of CaCO3(s) is studied
. × 10 −10 = [Ag+ ][Cl− ] = x ( x + y )
K sp (AgCl) = 16
under different conditions
or . × 10 −10
x ( x + y ) = 16 (1) CaCO3 (s)  CaO(s) + CO2 (g)
Now, the solubility product of CuCl is
For this equilibrium, the correct statement(s) is(are)
(a) ∆H is dependent on T.
K sp (CuCl) = 1.0 × 10 −6 = [Cu+ ][Cl− ] (b) K is independent of the initial amount of CaCO3.
(c) K is dependent on the pressure of CO2 at a given T.
or y ( x + y ) = 1.0 × 10 −6 (2) (d) ∆H is independent of catalyst, if any.
Dividing Eq. (1) by Eq. (2), we get (JEE Advanced 2013)

Chapter-7.indd 225 7/30/2016 2:44:30 PM


226 Chapter 7 Equilibrium

Solution
K2
(a), (c), (d) For the reaction CaCO3 (s)  CaO(s) + CO2 (g), N2 + O2  2NO
∆H is dependent on temperature according to Kirchhoff’s K3
equation ∆H2 − ∆H2 = C p (T2 − T1), but it is independent of H2 + 21 O2  H2O
addition of catalyst.
The equilibrium constant of the reaction 2NH3 + 25 O2  2NO + 3H2O
The equilibrium constant (K) is independent of initial amount 5
2NH3 + 2 O2  2NO + 3H2O in terms of K1, K2 and K3 is
of CaCO3 but dependent on pressure of CO2 by the relation
K = pCO2 . KK K K2 K K3
(a) 1 2 (b) 1 3 (c) 2 3 (d) K1K2K3
14. In the dissociation of HI, 20% of HI is dissociated at equilib- K3 K2 K1
rium. Calculate Kp for
Solution
HI  21 H2 + 21 I2
(c) The reaction can be rearranged and summed to obtain
Solution the resultant reaction, as
The reaction can be represented as 2NH3  N2 + 3H2 (1/K1)
2HI  H2 + I2 3H2 + 23 O2  3H2O (K 3 )3
Initial moles 1 0 0
Moles at equilibrium 1− x x/2 x/2 N2 + O2  2NO K 2
2NH3 + 25 O2  2NO + 3H2O
Hence, x = 0.2 and ∆n = 0. Therefore, Kp = KC. Now,

( x / 2V )1/ 2 ( x / 2V )1/ 2 x 0.2 K 2 (K 3 )3


K p = KC = = = = 0.125 The equilibrium constant is .
(1− x ) / V 2(1− x ) 2 × 0.8 K1

For a general reaction xA(g) + yB(g)  pC(g) + qD(g), Kp


17. Arrange the following oxides in the increasing order of
can also be expressed as
Brönsted basicity.
p ∆n Cl2O7 , BaO, SO3 , CO2 , B2O3
(nC )eq (nD )qeq  pT  g
Kp = x y n  Solution
(nA )eq (nB )eq  T  eq
BaO + H2O  Ba(OH)2 (basic )
where nC, nD, nA and nB are the respective moles of A, B,
C and D at equilibrium and pT is the total pressure and nT CO2 + H2O  H2CO3 ( weakly acidic)
are the total gaseous moles at equilibrium. SO3 + H2O  H2SO 4 (strongly acidic)
15. In the following equilibrium N2O 4 (g)  2NO2 (g), when B2O3 + 3H2O  2H3BO3 ( very weakly acidic)
5 mol of each are taken, the temperature is kept at Cl2O7 + H2O  2HClO 4 ( very strongly acidic)
298 K the total pressure was found to be 20 bar. Given that
∆ f G °(N2O 4 ) = 100 kJ and ∆ f G °(NO2 ) = 50 kJ Hence, the order is Cl2O7 < SO3 < CO2 < B2O3 < BaO .

(a) Find ∆G of the reaction. 18. A solution contains a mixture of Ag+ (0.10 M) and Hg2+ 2
(b) The direction of the reaction in which the equilibrium (0.10 M) which are to be separated by selective precipita-
shifts. tion. Calculate the maximum concentration of iodide ion at
Solution which one of them gets precipitated almost completely. What
percentage of that metal ion is precipitated?
(a) For the reaction N2O 4 (g)  2NO2 (g), the reaction quo-
tient is Solution
2
pNO 100 For the precipitation, the ionic product should be greater
Q= 2
= = 10 atm
pN2 O4 10 than solubility product. Also, Ksp of Hg2I2 = 2.5 × 10−26 is
much less than that of AgI = 8.5 × 10−17.
∆G o reaction = 2∆ f G o (NO2 ) − ∆ f G o (N2O 4 ) ⇒ 0 = 100 − 100
For the reaction Hg2I2  Hg22 + + 2I− , the solubility prod-
∆G = ∆G o + RT ln K
Therefore, uct is K sp = [Hg22 + ][I− ]2 = 2.5 × 10 −26

∆G = RT ln Q = 2.303 × 0.082 × 298 × log 9.9 K sp 2.5 × 10 −26


Therefore, [I−] = = = 5.0 × 10 −13 M
= 56.0304 L atm ⇒ Positive quan
ntity [Hg22 + ] 0.1
(b) Therefore, the reaction will shift in the reverse direction.
For the reaction AgI  Ag+ + 2I− , the solubility product is
16. The following equilibria are given:
K1 K sp = [Ag+ ][I− ] = 8.5 × 10 −17
N2 + 3H2  2NH3

Chapter-7.indd 226 7/30/2016 2:44:40 PM


Advanced Level Problems 227

K sp 8.5 × 10 −17 (a) 3.6 × 10 −13 (b) 4 × 10 −12


Therefore, [I−] = = = 8.5 × 10 −16 M
[Ag+ ] 0.1 (c) 3.24 × 10 −14 (d) 3.6 × 10 −12
On comparing, we find that [I−] required to precipitate
AgI is less than that required to precipitate Hg2I2. So, AgI Solution
starts precipitating first and Hg2I2 gets precipitated only (a) The reaction is
when [I−] reaches 5.0 × 10−13 M. The concentration of AgI
left when Hg2I2 starts precipitating is AgCN(s)  Ag+ (aq) + CN− (aq)
Since CN− comes from weak acid, so its hydrolysis will
K sp of AgI 8.5 × 10 −17 −4 take place, therefore
= = 1.7 × 10
[I− ] for Hg2I2 5.0 × 10 −13
CN− (aq) + H2O  HCN(aq) + OH− (aq)
Therefore, % of [Ag+] precipitated
At equilibrium 2 × 10 −6 (1 − h) 2 × 10 −6 h 2 × 10 −6 h
−4
1.7 × 10 (where h is the degree of hydrolysis)
= × 100 = 0.17%
0.1
K w [HCN][OH− ] (2 × 10 −6 h)2
Kh = = −
=
So, the % of Ag+ precipitated is 100 − 0.17 = 99.83%. Ka [CN ] (2 × 10 −6 )(1− h)

19. HX is a weak acid (K a =10 −5 ) . It forms a salt NaX(0.1 M) on 10 −14 2 × 10 −6 h2


or = 2 × 10 −5 =
reaction with caustic soda. The degree of hydrolysis of NaX is 5 × 10 −10 (1− h)
(a) 0.01% (b) 0.0001% (c) 0.1% (d) 0.5%
h2
Solution or = 10 ⇒ h2 + 10h − 10 = 0 ⇒ h = 0.91
(1− h)
(a) The reaction is NaX + H2O → NaOH + HX . The hydrolysis
constant is Therefore, [CN− ] = 2 × 10 −6 (1− 0.91) = 1.8 × 10 −7 mol L−1
K and
K h = w = 10 −9
Ka
K sp = [Ag+ ][CN− ] = 2 × 10 −6 × 1.8 × 10 −7 = 3.6 × 10 −13
Now, Kh = Ch2 = 10−9 where h is the degree of hydrolysis
and C = 0.1 M. Solving, we get
22. Two bulbs each of one litre are filled with 0.500 atm of F2 and
h2 = 10 −8 ⇒ h = 10 −4 PF3 gases as illustrated in the figure below. At the particular
a = 10 −4 temperature, Kp = 4.0 for the reaction of the gases to form
Therefore, % hydrolysis = 0.01%. PF5(g):
F2 (g) + PF3 (g)  PF5 (g)
20. 0.1 mol of CH3NH2 (Kb = 6 × 10−4) is added to 0.08 mol of HCl
and the solution is diluted to 1 L. The hydrogen-ion concen-
tration in the solution is F2
(a) 5 × 10−5 M (b) 8 × 10−2 M
(c) 1.6 × 10−11 M (d) 6.7 × 10−11 M
PF3
Solution
(d) The reaction is

CH3NH2 + HCl → CH3 NH2+ + Cl−


Initial concentration 0.1 0.08 0 0 The stopcock between the bulbs is opened and allows reach-
Concentration in solution 0.02 0.08 ing the equilibrium. Select the incorrect statement about
the equilibrium.
The concentrations of [OH−] and [H+] are (a) Partial pressure of PF5 is 0.25 atm at equilibrium.
K b [CH3NH2 ] 6 × 10 −4 × 0.02 3 (b) Total pressure at equilibrium is 0.75 atm.
[OH− ] = = = × 10 −4 (c) If whole content is forced into one bulb, the total pres-
0.08 2
[CH3 NH3]+ sure becomes 1.39 atm.
Kw (d) If whole content is forced into one bulb, partial pressures
[H+ ] = = 0.666 × 10 −10 = 6.7 × 10 −11 of gases remain constant.
[OH− ]
Solution
21. Silver cyanide, AgCN is one of the important salts used in the (d)
extraction of silver metal. It is sparingly soluble in water and F2 (g) + PF3 (g)  PF5 (g)
its solubility is approximately 2 × 10−6 mol L−1 at 25°C. If the Initial pressure (atm) 0.5 0.5 0
dissociation constant of HCN at 25°C is 5 × 10−10, what is the Pressure at equilibrium (atm) 0.5 − x 0.5 − x x
solubility product of AgCN at this temperature?

Chapter-7.indd 227 7/30/2016 2:44:46 PM


228 Chapter 7 Equilibrium

x 25. The value of Kp is 1 × 10−3 atm−1 at 25°C for the reaction:


Kp = = 4 ⇒ 4 x 2 − 5 x + 1 = 0 ⇒ x = 0.25 atm 2NO + Cl2  2NOCl. A flask contains NO at 0.02 atm and at
(0.5 − x )2
25°C. Calculate the moles of Cl2 that must be added if 1% of the
Given that NO is to be converted to NOCl at equilibrium. The volume of
pF2 = 0.5 − 0.25 = 0.25 atm; pPF3 = 0.25 atm; pPF5 = 0.25 atm the flask is such that 0.2 mol of gas produces 1 atm pressure at
25°C. (Assume that there is no association of NO to form N2O2.)
Hence, ptotal = 0.25 × 3 = 0.75 atm
Solution
On shifting whole content into one bulb
Consider the reaction:
F2 (g) + PF3 (g)  PF5 (g)
2NO + Cl2  2NOCl
Initial pressure (atm)
0.5 0.5 0.5
Pressure at equilibrium (atm) 0.5 − x ′ 0.5 − x ′ 0.5 + x ′ Initial pressure 0.02 p 0
At equilibrium (0.02 − 0.02/100) (p − 0.01/100) 0.02/100
0.5 + x 198 × 10−4 p − 10−4 2 × 10−4
Kp = = 4 ⇒ 8 x 2 − 10 x + 1 = 0 ⇒ x = 0.11 atm
(0.5 − x )2
2
pNOCl (2 × 10 −4 )2
Given that: Kp = ⇒ 10 −3 =
2
pNO × pCl2 (198 × 10 −4 )2 × ( p − 10 −4 )2
pF2 = 0.5 − 0.11 = 0.39 atm; pPF3 = 0.39 atm; pPF5 = 0.61 atm
⇒ p = 0.1021 atm
Hence, ptotal = 0.39 + 0.39 + 0.61 = 1.39 atm We know that pV = nRT, so
nRT 0.2 × 0.0821× 273
23. At 540 K, 0.10 mol of PCl5 is heated in an 8 L flask. The pressure V= = = 4.88 L
p 1
of the equilibrium mixture is found to be 1.0 atm. Calculate
Kp and KC for reaction. pV 0.1021× 4.887
n= = = 0.0204 mol
RT 0.0821× 298
Solution
Consider the reaction 26. The equilibrium constant KC for the reaction N2O 4  2NO2 in
chloroform at 291 K is 1.14. Calculate the free energy change
PCl5  PCl3 + Cl2 of the reaction when the concentration of the two gases is
Initial moles 0.1 0 0 0.5 mol dm−3 each at the same temperature. (R = 0.082 L atm
K−1 mol−1)
At equilibrium 0.1 − x x x
Total number of moles = 0.1 − x + x + x = 0.1 + x Solution
From the given data, we have T = 291 K; R = 0.082 L atm
As pV = nRT, we have K−1 mol−1, KC = 1.14; C = 0.5 mol dm−3.
pV 1× 8 As Qp = QC (RT )∆ ng and ∆ng = 2 − 1 = 1 in this case, the
n= = = 0.18 reaction quotient QC for the reaction is
RT 0.0821× 540
[NO2 ]2
So, 0.1 + x = 0.18 ⇒ x = 0.08 Qp = × 0.082 × 291
[N2O 4 ]
(0.08 / 8 )(0.08 / 8 ) (0.5)2
KC = = 0.04 = × 0.082 × 291 = 11.93
(0.02 / 8 ) 0.5
The equilibrium constant is
K p = K C (RT )∆n = 0.04 × (0.0821× 540 )1 = 1.77
K p = K C (RT )∆
ng
= 1.14 × (0.082 × 291) = 27.1 atm
24. The K value for the reaction: H2 + I2  2HI , at 460°C is 49. If Substituting these values in the equation
the initial pressure of H2 and I2 is 0.5 atm, respectively, deter-
mine the partial pressure of each gas at equilibrium. ∆G = ∆G o + RT ln Qp = − RT ln K p + RT ln Qp
Solution = −2.303 RT (log K p − log Qp )
Consider the reaction We get
H2 + I2  2HI ∆G = −(0.082 × 291× 2.303)[log 27.2 − log11.93]
= −54.95(1.4346 − 1.076
66 ) = −19.67 L atm
Initial number of moles 0.5 0.5 0
At equilibrium 0.5 − x 0.5 − x 2x 27. At 817°C, Kp for the reaction between CO2(g) and excess hot
2 graphite C(s) is 10 atm.
(2 x ) 2x (a) What is the equilibrium concentration of the gases at
K= 2
= 49 ⇒ 7 = ⇒ x = 0.39
(0.5 − x ) 0.5 − x 817°C and a total pressure of 5 atm?
Pressure of H2 and I2 at equilibrium = 0.5 − 0.39 = 0.11 atm (b) At what total pressure, the gas contains 5% CO2 by
Pressure of HI at equilibrium = 2 × 0.39 = 0.78 atm volume?

Chapter-7.indd 228 7/30/2016 2:44:52 PM


Practice Exercise 229

Solution Moles of CO at equilibrium = 2 × 0.577 = 1.154 and total


(a) The reaction involved is moles = 0.423 + 1.154 = 1.577
Using pV = nRT at equilibrium, we have
CO2(g) + C(s)  2CO(g)
5 × V = 1.577 × 0.0821× 1090 ⇒ V = 28.23 L
Initial moles 1 0
Hence, the equilibrium concentration of the gases is
Final moles 1−a 2a
[CO2] = 0.423/28.23 = 0.015 M and [CO] = 1.154/28.23
= 0.041 M
(nCO )2 p (2a )2 5 (b) At 5% CO2 by volume pCO2 = 
 5   95 
p and pCO =  p
Kp = × = ×
nCO2 1+ a (1− a ) 1+ a  100   100 
Therefore,
Given that Kp = 10 atm, so ( pCO )2 (95/100 )2 p
Kp = =
pCO2 (5/100 )
20a 2
10 = ⇒ 10 − 10a 2 = 20a 2 ⇒ a = 0.577 5 × 10 × 100
1− a 2 ⇒p= = 0.554 atm
95 × 95
Moles of CO2 at equilibrium = 1 − 0.577 = 0.423

PRACTICE EXERCISE
Level I 6. Which of the following is not Lewis acid?
(a) BF3 (b) AlCl3 (c) FeCl3 (d) PH3
Single Correct Choice Type
1. Acetic acid undergoes dimerization in benzene solution. When 7. The equilibrium constant KC for the reaction N2 +3H2  2NH3
the solution is diluted to twice the original volume, the posi- is 0.5 L2 mol−2 at 400 K. Find Kp, given that R = 0.0821 atm K−1
tion of equilibrium in the reaction 2CH3COOH  (CH3COOH)2 mol−1.
is shifted (a) 3.25 × 10−5 atm2 (b) 5.06 × 10−3 atm2
(a) to the right. (c) 6.25 × 10−4 atm2 (d) 4.37 × 10−4 atm2
(b) to the left.
8. The pH of a solution obtained by mixing 50 mL of 0.4 N HCl
(c) neither to left nor to right.
and 50 mL of 0.2 N NaOH is
(d) none of these.
(a) −log 2 (b) −log 0.2 (c) 1.0 (d) 2.0
2. Which of the following can act both as a Brönsted acid and a
9. Molten sodium chloride conducts electricity due to the
Brönsted base?
presence of
(a) O2− (b) HCl (c) HSO 4− (d) Na2CO3
(a) free electrons.
3. The pH of a buffer solution containing 25 mL of 1 M CH3COONa (b) free ions.
and 25 mL of 1 M CH3COOH will be appreciably affected by (c) free molecules.
5 mL of which solution? (d) atoms of sodium and chlorine.
(a) 1 M CH3COOH (b) 5 M CH3COOH
10. Nitrous oxide, N2O, sometimes called laughing gas has a
(c) 5 M HCl (d) 1 M NH4OH
strong tendency to decompose into nitrogen and oxygen
4. In a gas phase reaction 2A + B  3C + 2D, it was found that following the equation
when 1.0 mol of A, 2.0 mol of B, and 1.0 mol of D were mixed 2N2O(g)  2N2 (g) + O2 (g)
in 1 L flask and came to equilibrium, the resulting mixture con-
tained 0.9 mol of C. Calculate equilibrium constant KC. but the reaction is so slow that the gas appears to be stable
(a) 2.86 (b) 1.86 (c) 4.86 (d) 6.86 at room temperature (25°C). The decomposition reaction has
KC = 7.3 × 1034. What is the value of Kp for this reaction at
5. An endothermic reaction is represented by the graph 25°C?
(a) (b) (a) 3.1 × 1036 (b) 1.8 × 1036
(c) 2.01 × 1035 (d) 2.6 × 1034
In Kp In Kp
11. Dichloroacetic acid Cl2CHCOOH is not so weak acid with an
ionization constant: Ka = 5.5 × 10−2. Calculate the [H+] and the
1/T 1/T pH of a 0.100 molar solution of this acid?
(c) (d) (a) 1.9 × 10−2, 1.72 (b) 5.2 × 10−2, 1.29
(c) 2.7 × 10−2, 1.57 (d) 0.1 × 10−3, 2.98
In Kp In Kp
12. NaOH solution and 50 mL of M/5 HCl solution chloride ion
(Cl−) in aqueous solution is a
(a) weak base. (b) strong base.
1/T 1/T
(c) weak acid. (d) strong acid.

Chapter-7.indd 229 7/30/2016 2:44:55 PM


230 Chapter 7 Equilibrium

13. In which of the following reaction, the value of Kp will be sufficient water to make 1.40 L of solution. Calculate the pH
equal to KC? of the solution.
(a) PCl5  PCl3 + Cl2 (b) 2NH3  N2 + 3H2 (a) 4.25 (b) 4.45 (c) 5.00 (d) 5.27

(c) H2 + I2  2HI (d) 2SO2 + O2  2SO3 24. During an electrolysis of conc. H2SO4, perdisulphuric acid
(H2S2O8) and O2 form in equimolar amount. The moles of H2
14. What would be the pH of H3PO4 if it loses two protons in that will form simultaneously will be
aqueous solution? (a) equal to that of O2. (b) twice that of O2.
(a) 6 (b) 7 (c) 6.5 (d) 7.8 (c) thrice that of O2. (d) half of that O2.

15. For which of the following reaction is product formation 25. The preparation of SO3(g) by reaction SO2 (g) + 21 O2 (g)  SO3 (g)
favoured by low pressure and high temperature? SO (g) + 1 O (g)  SO (g) is an exothermic reaction. If the preparation follows
2 2 2 3
(a) H2 (g) + I2 (g)  2HI(g); ∆H° = −9.4 kJ the following temperature–pressure relationship for its %
yield (figure below), then for temperatures T1, T2 and T3, the
(b) CO2 (g) + C(s)  2CO(g); ∆H° = 172.5 kJ
correct option is
(c) CO(g) + 2H2 (g)  CH3OH(l); ∆H° = −21.7 kJ
(d) 3O2 (g)  2O3 (l); ∆H° = 285 kJ 50
40 T3
16. A weak acid HX has the dissociation constant 1 × 10−5. It forms

% Yield
a salt NaX on reaction with alkali. The percentage hydrolysis 30 T2
of 0.1 M solution of NaX is 20 T1
(a) 0.0001 (b) 0.01 (c) 0.1 (d) 0.15
10
17. Which anion is the weakest base?
(a) C2H5O− (b) NO3− 1 2 3 4
− Pressure (atm)
(c) F (d) CH3COO−
18. A compound HB is formed from H and B according to the (a) T3 > T2 > T1 (b) T1 > T2 > T3
following reaction: H + B  HB. A solution was prepared by (c) T1 = T2 = T3 (d) Cannot be predicted.
dissolving 0.1 mol of H and 0.1 mol of B in enough water to
make the total volume equal to 1 L. After equilibrium had 26. The equilibrium constant for the reaction N2 (g) + 3H2 (g)  2NH3 (g)
been reached, it was found that 20% of H had beenNreacted.
2 ( g) + 3H2 ( g)  2NH3 ( g) is K, then the equilibrium constant for the equilib-
What is the equilibrium constant, K for this reaction?
rium, NH3 (g)  21 N2 (g) + 23 H2 (g) is
(a) 3.125 (b) 4.125 (c) 5.125 (d) 2.125
19. Pure PCl5 is introduced into an evacuated chamber and 1 1 1
(a) (b) K (c) (d)
comes to equilibrium at 247°C and 2.0 atm. The equilibrium K K2 K
gaseous mixture contains 40% chlorine by volume. Calculate
Kp at 247°C for the reaction 27. Which of the following reactions will tend to proceed farthest
towards completion?
PCl5 (g)  PCl3 (g) + Cl2 (g)
(a) H2 (g) + Br2 (g)  2HBr(g); K C = 1.4 × 10 −21
(a) 1 atm (b) 3 atm (c) 1.6 atm (d) 4 atm
(b) 2NO(g)  N2 (g) + O2 (g); K C = 2.1× 1030
20. In which case(s), vapour density remains unchanged on
dissociation? (c) 2BrCl(g)  Br2 (g) + Cl2 (in CCl4 solution); K C = 0.145
(a) PCl5 (g)  PCl3 (g) + Cl2 (g) (d) Both (a) and (c)
(b) N2O 4 (g)  2NO2 (g)
(c) N2 (g) + O2 (g)  2NO(g) 28. Which of the following statements is not true?
(a) pH + pOH = 4 for all aqueous solutions.
(d) All of these.
(b) pH of 1 × 10−8 M HCl = 8.
21. The pKa of a weak acid HA is 4.80. The pKb of a weak base BOH (c) The conjugate base of H2PO 4− is HPO2−
4 .
is 4.78. The pH of an aqueous solution of the corresponding (d) All are correct.
salt, BA, will be
29. An aqueous solution of ammonia consists of
(a) 7.01 (b) 9.22 (c) 9.58 (d) 4.79
(a) H+ (b) OH−
22. Which of the following is the strongest acid? (c) NH4+
(d) NH+4 and OH−
(a) H3PO4 (b) H2SO4
(c) HNO2 (d) CH3COOH 30. For the reaction, H2 (g) + I2 (g)  2HI(g) equilibrium concen-
tration of H2, I2 and HI are 8.0, 3.0 and 28.0 mol L−1, respec-
23. A buffer solution is prepared by taking 0.25 mol of ace- tively. The equilibrium constant of the reaction is
tic acid (pKa = 4.761) and 0.40 mol of barium acetate in (a) 32.66 (b) 34.33 (c) 22.44 (d) 11.22

Chapter-7.indd 230 7/30/2016 2:45:03 PM


Practice Exercise 231

31. The number of H+ ions present in 1 cm3 of a solution whose (a) Kp does not change significantly with pressure.
pH is 10 is (b) a does not change with pressure.
(a) 10−10 (b) 10−13 (c) concentration of NH3 does not change with pressure.
(c) 6.02 × 1010 (d) 6.02 × 1013 (d) concentration of H2 is less than that of N2.
32. A nitrogen–hydrogen mixture initially in the molar ratio of 42. The decomposition of N2O4 to NO2 is carried out at 280 K in
1:3 reached equilibrium to form ammonia when 25% of the chloroform. When equilibrium has been established, 0.2 mol
N2 and H2 had reacted. If the total pressure of the system was of N2O4 and 2 × 10−3 mol of NO2 are present in 2 L solution.
21 atm, the partial pressure of ammonia at the equilibrium The equilibrium constant for reaction, N2O 4  2NO2 is
was (a) 1 × 10−2 (b) 1 × 10−3
(a) 3.5 atm (b) 3.0 atm (c) 2.5 atm (d) 1.5 atm (c) 1 × 10 −4 (d) 1 × 10−5
33. Calculate the equilibrium constant for the reaction 43. In a chemical equilibrium, the rate constant of the reverse
+ 2+ reaction is 7.5 × 10−4 and the equilibrium constant is 1.5. So,
Cu(s) + 2Ag (aq)  Cu (aq) + 2Ag(s)
the rate constant of the forward reaction is
Given that equilibrium concentrations are [Ag+] = 1.0 ×10−11 (a) 1.125 × 10−3 (b) 2.225 × 10−3
mol L−1 and [Cu2+] = 2.0 ×10−7 mol L−1 at 300 K. (c) 3.335 × 10−5 (d) 1.125 × 10−1
(a) 2.5 × 1015 mol L−1
44. For the reaction: 2HI(g)  H2(g) + I2(g), the degree of disso-
(b) 1.5 × 1015 mol L−1
ciation (a) of HI(g) is related to equilibrium constant Kp by the
(c) 2 × 1015 mol L−1
expression
(d) 1.08 × 1015 mol L−1
1+ 2 K p 1 + 2K p
34. For the reaction PCl3 (g) + Cl2 (g)  PCl5 (g), the value of KC at (a) (b)
250°C is 26. The value of Kp at this temperature will be 2 2
(a) 0.41 (b) 0.51 (c) 0.61 (d) 0.71 2K p 2 Kp
(c) (d)
35. The formation of SO3 takes place according to the following 1 + 2K p 1+ 2 K p
reaction 2SO2 (g) + O2 (g)  2SO3 (g), ∆H = −45.2 kcal. The
formation of SO3 is favoured by 45. A reversible chemical reaction is having two reactants in
(a) increase of volume. equilibrium. If the concentration of the reactants is doubled,
(b) increase of pressure. then the equilibrium constant will
(c) increase in temperature. (a) also be doubled. (b) be halved.
(d) removal of oxygen. (c) become one-fourth. (d) remain the same.
36. The decomposition of N2O4 at 25°C, 46. When 5 mol of SO2 and 5 mol of O2 are allowed to react to
N2O 4 (g)  2NO2 (g) form SO3 in a closed vessel, at the equilibrium stage 60% of
SO2 is used up. The total number of moles of SO2, O2 and SO3
has  KC = 4.61 × 10−3. A 2.00 L vessel contained 0.0466 mol in the vessel now is
N2O4 at equilibrium. What was the concentration of NO2 in (a) 8.5 (b) 9.5 (c) 10 (d) 10.5
the vessel?
(a) 1.04 × 10–3 M (b) 1.04 × 10–2 M 47. 1 M NaCl and 1 M HCl are present in an aqueous solution. The
(c) 3.15 × 10–5 M (d) 2.03 × 10–2 M solution is
(a) not a buffer solution with pH < 7.
37. An acidic buffer solution can be prepared by mixing (b) a buffer solution with pH < 7.
(a) solution of acetate and acetic acid. (c) not a buffer solution with pH > 7.
(b) ammonium chloride and ammonium hydroxide. (d) a buffer solution with pH > 7.
(c) sulphuric acid and sodium sulphate.
(d) sodium chloride and sodium hydroxide. 48. When baking soda is heated in a sealed tube, following equi-
librium exits:
38. NaOH is a strong base because
(a) it gives OH− ion. (b) it can be oxidized. 2NaHCO3 (s)  Na2CO3 (s) + CO2 (g) + H2O(g)
(c) it can be easily ionized. (d) both (a) and (c).
If the equilibrium pressure is 1.04 atm at 398 K, calculate the
39. Aqueous solution of sodium cyanide is
equilibrium constant for the reaction at 398 K.
(a) acidic. (b) amphoteric.
(a) 0.135 atm (b) 0.271 atm
(c) basic. (d) neutral.
(c) 0.156 atm (d) 0.185 atm
40. At infinite dilution, the percentage ionization for both strong 49. The degree of ionization of a compound depends on
and weak electrolyte is (a) size of solute molecules.
(a) 1% (b) 20% (c) 50% (d) 100% (b) nature of solute molecules.
41. Pure ammonia is placed in a vessel at a temperature where (c) nature of vessel used.
its dissociation constant (a) is appreciable at equilibrium, (d) quantity of electricity passed.

Chapter-7.indd 231 7/30/2016 2:45:05 PM


232 Chapter 7 Equilibrium

50. The following equilibrium exists in aqueous solution 61. On addition of ammonium chloride to a solution of ammo-
CH3COOH  CH3COO − + H+ . If dil. HCl is added without nium hydroxide,
change in temperature, the (a) dissociation of NH4OH increases.
(a) concentration of CH3COO− will increase. (b) concentration of OH− decreases.
(b) concentration of CH3COO− will decrease. (c) concentration of NH+4 and OH− increases.
(c) the equilibrium constant will increase. (d) concentration of NH+4 decreases.
(d) the equilibrium constant will decrease.
62. The species that acts as a Lewis but not a Brönsted acid is
51. The degree of dissociation of 0.1 M HCN solution is 0.01% its
(a) NH2− (b) O2− (c) BF3 (d) OH−
ionization constant would be
(a) 10−3 (b) 10−5 (c) 10−7 (d) 10−9 63. Let the solubility of an aqueous solution of Mg(OH)2 be S,
then its Ksp is
52. The partial pressures of CH3OH, CO and H2 in the equilibrium
(a) 4S3 (b) 108S5 (c) 27S4 (d) 9S
mixture for the reaction
CO + 2H2  CH3OH 64. What is the pH value of N/1000 KOH solution?
(a) 10−11 (b) 3 (c) 2 (d) 11
at 427°C are 2.0, 1.0 and 0.1 atm, respectively. The value of Kp
for the decomposition of CH3OH to CO and H2 is 65. Which of the following salts is least soluble in water?
(a) 1 × 102 atm (a) AgCl (b) AgF (c) AgI (d) Ag2S
(b) 4 × 102 atm
(c) 5 × 10−3 atm (d) 5 × 103 atm 66. The solubility product of a sparingly soluble salt AB at room
temperature is 1.21 × 10−6. Its molar solubility is
53. Ammonium carbamate dissociates as
(a) 1.21 × 10−6 (b) 1.21 × 10−3
(c) 1.1 × 10 −4 (d) 1.1 × 10−3
NH2COONH4 (s)  2NH3 (g) + CO2 (g)

in a closed vessel. If the equilibrium total pressure is 9 atm. 67. Ionic product in water increases if
What is the value of Kp? (a) pressure is reduced. (b) H+ is added.
(a) 132 atm3 (b) 108 atm3 (c) OH− is added. (d) temperature is increased.
(c) 152 atm 3 (d) 128 atm3 68. The degree of hydrolysis of 0.2 M solution of NH4Cl will be
54. At 25°C the dissociation constant of HCN is 4.9 × 10−10 M. [Given Kb for NH4OH is 1.8 × 10−5]
Calculate the degree of dissociation of HCN if the concentra- (a) 4.25 × 10−5 (b) 5.27 × 10−5
(c) 2.98 × 10 −5 (d) 3.56 × 10−5
tions is 0.1 M.
(a) 7 × 10−5 (b) 5 × 10−5 69. The pH of a 10−9 M solution of HCl in water is
(c) 6 × 10−5 (d) 8 × 10−5 (a) 8 (b) −8
55. In the reaction SnCl2 + 2Cl− → SnCl4, the Lewis acid is (c) between 7 and 8 (d) between 6 and 7
(a) SnCl2 (b) Cl− 70. What is the percentage hydrolysis in 0.0003 M aqueous solu-
(c) SnCl4 (d) None of these. tion of NaOCN? [Ka for HOCN = 3.33 × 10−4]
56. The pKa for acid A is greater than pKa for acid B. The strong (a) 10−2 (b) 10−3 (c) 10−4 (d) 10−5
acid is
71. A buffer solution can be prepared from a mixture of
(a) acid B. (b) acid A.
(I) sodium acetate and acetic acid in water.
(c) both A and B. (d) neither A nor B.
(II) sodium acetate and hydrochloric acid in water.
57. Which among the following is strongest acid? (III) ammonia and ammonium chloride in water.
(a) H(ClO)O2 (b) H(ClO)O3 (c) H(ClO)O (d) H(ClO) (IV) ammonia and sodium hydroxide in water.
(a) (I), (II) (b) (II), (III) (c) (III), (IV) (d) (I), (III)
58. An aqueous solution of sodium carbonate is alkaline because
sodium carbonate is a salt of 72. The solubility product of Ag2CrO4 is 32 × 10−12. What is the
(a) weak acid and weak base. concentration of (CrO 4 )− ions in that solution?
(b) strong acid and weak base. (a) (2 × 10−4 M) (b) (16 × 10−4 M)
(c) weak acid and strong base. (c) (8 × 10−4 M) (d) (8 × 10−8 M)
(d) strong acid and strong base.
73. The solubility product of PbCl2 at 20°C is 1.5 × 10−4. Calculate
59. Which indicator is used in the titration of iodine against the solubility.
sodium thiosulphate? (a) 3.75 × 10−4 (b) 3.34 × 10−2
(a) Starch (b) K3Fe(CN)6 (c) 3.34 × 10 2 (d) 3.95 × 103
(c) K2CrO4 (d) Potassium
74. At 298 K, the solubility of PbCl2 is 2 × 10−2 mol L−1, then its Ksp
60. One litre of HCl (pH = 2) is mixed with two litres of an acid will be
HNO3 (pH = 3). Calculate the pH of the resultant acid mixture. (a) 1 × 10−7 (b) 3.2 × l0−7
(a) 2.4 (b) 3.2 (c) 4.5 (d) 1.9 (c) 1 × 10 −5 (d) 3.2 × 10−5

Chapter-7.indd 232 7/30/2016 2:45:07 PM


Practice Exercise 233

75. A sample of HI was found to be 22% dissociated when equi- 87. When solid potassium cyanide is added in water, then
librium was reached. What will be the degree of dissociation (a) pH will increase.
if hydrogen is added in the proportion of 1 mol for every (b) pH will decrease.
mole of HI present originally? Assume temperature and pres- (c) pH will remain the same.
sure to be constant. (d) electrical conductivity will not change.
(a) 0.065 (b) 0.085
(c) 0.037 (d) 0.052 88. What is the pH of a buffer solution prepared by 30 g of
Na2CO3 in 1000 mL and 150 mL of M HCl? [Ka for HCO3− = 5.63
76. Calculate the molar solubility of Fe(OH)3 in a solution where
× 10−11]?
the OH− concentration is initially 0.050 M. Assume the disso- (a) 10.2 (b) 9.5 (c) 8.9 (d) 7.9
ciation of Fe(OH)3 is 100%.
(a) 1.5 × 10−36 M (b) 1.9 × 10−34 M 89. Dissociation constant of a weak acid is 1 × 10−4. Equilibrium
−36
(c) 2.1 × 10 M (d) 1.3 × 10−35 M constant of its reaction with strong base is
(a) 1 × 10−4 (b) 1 × 1010
77. A 0.1 M solution of a weak acid HA is 1% dissociated. The (c) 1 × 10−10 (d) 1 × 104
approximate value of dissociation constant is
(a) 1.0 × l0−4 mol L−1 (b) 0.1 × 10−2 mol L−1 90. Salicylic acid reacts with acetic acid to form aspirin, acetylsal-
(c) 1 × l0−3 mol L−1 (d) 0.1 × 10−4 mol L−1 icylic acid. A 0.200 molar solution of salicylic acid has a pH of
1.836. Calculate the Ka and pKa of salicylic acid.
78. Which hydroxide will have lowest value of solubility product (a) 1.9 × 10−3, 3.0 (b) 2.05 × 10−3, 3.2
at normal temperature (25°C)? (c) 2.5 × 10−3, 2.6 (d) 1.15 × 10−3, 2.9
(a) Mg(OH)2 (b) Ca(OH)2 (c) Ba(OH)2 (d) Be(OH)2
91. The suitable indicator for strong acid and weak base is
79. Orthoboric acid in aqueous medium is
(a) methyl orange. (b) methyl red.
(a) monobasic. (b) dibasic.
(c) phenol red. (d) phenolphthalein.
(c) tribasic. (d) all of these.
92. Calculate the [H+] in a solution containing 0.1 M HCOOH and
80. A precipitate of AgCl is formed when equal volumes of the
0.1 M HOCN. [Ka for HCOOH and HOCN are 1.8 × 10−4 and
following are mixed [Ksp for AgCl = 10−10]
3.3 × 10−4, respectively].
(a) 10−4 M AgNO3 and 10−7 M HCl.
(a) 8.02 × 10−3 M (b) 5.98 × 10−3 M
(b) 10−5 M AgNO3 and 10−6 M HCl.
(c) 7.13 × 10−3 M (d) 6.25 × 10−3 M
(c) 10−5 M AgNO3 and 10−5 M HCl.
(d) 10−6 M AgNO3 and 10−6 M HCl. 93. Vapour density of the equilibrium mixture of NO2 and N2O4 is
81. A 90°C pure water has [H3O+] = 10−6 M, the value of Kw at this found to be 40 for the equilibrium:
temperature will be N2O 4 (g)  2NO2 (g)
(a) 10−6 (b) 10−12 (c) 10−14 (d) 10−8 Calculate percentage of NO2 in the mixture.
(a) 26.08% (b) 21.52% (c) 19.24% (d) 24.62%
82. A litre of solution is saturated with AgCl. To this solution, if
1.0 × 10−4 mol of solid NaCl is added, what will be the [Ag+] 94. Ammonia under a pressure of 15 atm at 27°C is heated to
assuming no volume change? 347°C in a closed vessel in the presence of catalyst. Under
(a) More (b) Less (c) Equal (d) Zero the conditions, NH3 is partially decomposed according
83. Given pH of a solution A is 3 and it is mixed with an equal to the equation 2NH3  N2 + 3H2 . The vessel is such that
volume of another solution B having pH 2. If both are mixed, the volume remains effectively constant, whereas pressure
then the resultant pH of the solution will be increases at 50 atm. Calculate the percentage of NH3 actually
(a) 3.2 (b) 2.26 (c) 3.4 (d) 3.5 decomposed.
(a) 61.3% (b) 63.5%
84. NaOH(aq), HCl(aq) and NaCl(aq) concentration of each is (c) 65.3% (d) 66.6%
10−3 M. Their pH will be, respectively,
(a) 10, 6, 2 (b) 11, 3, 7 (c) 10, 2, 6 (d) 3, 4, 7 95. One possible way of removing NO from the exhaust of a gas-
oline engine is to cause it to react with CO in the presence of
85. The pH of a solution is increased from 3 to 6, its H+ ion con- a suitable catalyst.
centration will be
(a) reduced to half. (b) doubled. 2NO(g) + 2CO(g)  N2 (g) + 2CO2 (g)
(c) reduced by 1000 times. (d) increased by 1000 times. At 300°C, the reaction has KC = 2.2 × 1059. What is Kp at 300°C?
86. Assuming complete dissociation, which of the following (a) 3.52 × 1057 (b) 4.7 × 1050
aqueous solutions will have the same pH value? (c) 6.23 × 1050 (d) 4.7 × 1057
(I) 100 mL of 0.01 M HCl
96. For the following equilibrium, van’t Hoff factor is 0.80,
(II) 100 mL of 0.01 M H2SO4
2A(g)  A2 (g) . Hence, volume % of A2(g) in the total mix-
(III) 50 mL of 0.01 M HCl ture is:
(IV) Mixture of 50 mL of 0.02 M H2SO4 and 50 mL of 0.02 M NaOH (a) 25% (b) 50% (c) 75% (d) 100%
(a) (I), (II) (b) (I), (III) (c) (II), (IV) (d) (I), (IV)

Chapter-7.indd 233 7/30/2016 2:45:08 PM


234 Chapter 7 Equilibrium

97. A mixture containing 8.07 mol of hydrogen and 9.08 mol 0.1 bar at that temperature. If 12.7 g of solid iodine are
of iodine was heated at 448°C till equilibrium was attained placed in a 10 L vessel at 40°C, what is the minimum amount
when 13.38 mol of hydrogen iodide was obtained. Calculate of hydrogen gas that must be introduced in order to remove
the percentage of dissociation of hydrogen iodide at 448°C. all solid iodine?
(a) 13.2% (b) 19.8% (c) 18.9% (d) 21.4% (a) 0.024 g (b) 0.012 g (c) 0.036 g (d) 0.018 g
98. Let the solubilities of AgCl in H2O, 0.01 M CaCl2; 0.01 M NaCl 105. At 500°C, the reaction between N2 and H2 to form ammonia
and 0.05 M AgNO3 be S1, S2, S3, S4, respectively. What is the has KC = 6.0 × 10−2. What is the numerical value of Kp for the
correct relationship between these quantities? reaction N2 (g) + 3H2 (g)  2NH3 (g) ?
(a) S1 > S2 > S3 > S4 (b) S1 > S2 = S3 > S4 (a) 1.9 × 10−4 (b) 2.23 × 10−4
(c) S1 > S3 > S2 > S4 (d) S4 > S2 > S3 > S1 (c) 1.5 × 10−5 (d) 2.01 × 10−5

99. The reaction COCl2 (g)  CO(g) + Cl2 (g) has Kp = 4.6 × 10−2 at 106. Equal moles of F2(g) and Cl2(g) are introduced into a sealed
container and heated to certain temperature where the fol-
395°C. What is the value of KC at that temperature? lowing equilibria were established
(a) 7.56 × 10–5 (b) 4.32 × 10–3
(c) 8.4 × 10 –4 (d) 6.25 × 10–2 Cl2 (g) + F2 (g)  2ClF(g); K p = 3.2

100. One of the reaction that takes place in producing steel from Cl2 (g) + 3F2 (g)  2ClF3 (g)
iron ore is the reduction of iron (II) oxide by carbon monox-
ide to give iron metal and CO2. If partial pressures of ClF and ClF3 at equilibrium are 0.2 and
FeO(s) + CO(g)  Fe(s) + CO2 (g) K p = 0.265 at 1050 K 0.04, respectively, calculate Kp for the second equilibrium
reaction.
What are the equilibrium partial pressures of CO and CO2 at (a) 7.05 (b) 14.95 (c) 12.05 (d) 16.05
1050 K if the initial partial pressures are: pCO = 1.4 atm and
107. A solid substance A decomposes into two gaseous products
pCO = 0.80 atm?
B and C as:
(a) 1.96 atm, 0.35 atm (b) 1.74 atm, 0.46 atm
(c) 2.01 atm, 0.52 atm (d) 1.56 atm, 0.67 atm A(s)  2B(g) + C(g)

101. For a fairly concentrated solution of a weak electrolyte AxBy, If at equilibrium, some C(g) at 1.0 atm is added in constant
the degree of dissociation is given by volume condition, 10% of B(g) is solidified before the equi-
librium was re-established. Determine the total pressure at
K eq xy
(a) a = final equilibrium.
C (a) 9.06 (b) 5.06 (c) 6.06 (d) 4.06
K eqC 108. The first equilibrium expression (KC) as represented by the
(b) a =
xy first chemical equation is 5.81 × 10−7 at 700 K. Calculate the
value of the equilibrium expression for the second equation
K eqC
(c) a = at the same temperature.
(x + y)
2SO3 (g)  2SO2 (g) + O2 (g)
1/( x + y )
 K eq  SO3 (g)  SO2 (g) + 21 O2 (g)
(d) a =  x + y −1 x y 
C x y 
(a) 7.62 × 10−4 (b) 6.32 × 10−4
102. When 100 mL of M/10 NaOH solution and 50 mL of M/5 (c) 5.90 × 10−5 (d) 4.28 × 10−5
HCl solution are mixed, the pH of the resulting solution
109. The equilibrium 2SO3 (g)  2SO2 (g) + O2 (g) is established
would be
in a vessel of volume 1 L at 600 °C tanking SO3 initially in the
(a) 0 (b) 7
vessel. If 1 mol of SO3 is formed under equilibrium condi-
(c) less than 7 (d) more than 7
tion then how much of SO3 was taken initially? (Given KC for
103. Bromine monochloride, BrCl, decomposes into bromine and 2SO2 + O2  2SO3 is 4.5 at 600 °C.)
chlorine and reaches the equilibrium (a) 4.5 mol (b) 3.91 mol (c) 4.62 mol (d) 6.24 mol
2BrCl(g)  Br2 (g) + Cl2 (g)
110. Suppose 20.85 g of PCl5(g) is introduced in a vessel washed
For which KC = 32 at 500 K. If initially, pure BrCl is present with a non-volatile solvent (b.p. = 350 K, molar mass =
at a concentration of 3.30 × 103 mol L−1, what is its molar 154 g mol−1). The equilibrium is established at 523 K
concentration? when PCl5(g) is 52% dissociated and a total pressure was
(a) 3.03 × 10−3 mol L−1 found to be 5.5 bar. If Kp for the decomposition reaction:
(b) 9.06 × 10−3 mol L−1 PCl5  PCl3 + Cl2 is 1.78, calculate the weight of solvent left
(c) 12.10 × 10−3 mol L−1 in the vessel during washing.
(d) 10.28 × 10−3 mol L−1 (a) 2.188 g (b) 1.128 g (c) 3.388 g (d) 4.212 g

104. The equilibrium constant for the reaction H2 (g) + I2 (g)  2HI111.
(g) The two bulbs R and S connected by a mercury manometer
H2 (g) + I2 (g)  2HI(g) is 20 at 40 °C, and vapour pressure of solid iodine is are held in a thermostat (to maintain constant temperature)

Chapter-7.indd 234 7/30/2016 2:45:12 PM


Practice Exercise 235

shown in the following figure. Volume of R is twice the vol- 117. Which of the following is correct about the chemical
ume of S. R contains a gas X at the same pressure of nitro- equilibrium?
gen in S. The mercury level in the right hand limb rises (a) (∆G)T,p = 0
when temperature is increased if the gas X is (b) Equilibrium constant is the independent of initial con-
R centration of reactants.
S (c) Catalyst has no effect on equilibrium state.
X N2 (d) Reaction stops at equilibrium.

118. Consider the reaction: N2F4 (g)  2NF2 (g); ∆H o = 38.5 kJ.
Which of the following conditions will favour the formation
of NF2?
(a) Adding He to the equilibrium mixture at constant tem-
(a) nitrogen.
perature and volume.
(b) an equilibrium mixture,
(b) Increasing the temperature.
CH3CN(g)  CH3NC( g); ∆H is negative.
(c) NF2 gas is removed from the reaction mixture.
(c) an equilibrium mixture, (d) Decreasing the pressure at constant temperature.
N2F2 (g)  2NF(g); ∆H is positive.
(d) hydrogen. 119. Consider the reactions given below. In which cases, will
the reaction proceed toward the right by increasing the
Level II pressure?
Multiple Correct Choice Type (a) 4HCl(g) + O2(g) → 2Cl2(g) + 2H2O(g)
112. For the reaction: Cl2(g) + 3F2(g)  2ClF3(g); ∆H = − 329 kJ, (b) Cl2 (g) + H2O(g) → 2HCl(g) + 21 O2 (g)
dissociation of ClF3(g) will be favoured by (c) CO2(g) + 4H2(g) → CH4(g) + 2H2O(g)
(a) increasing the temperature. (d) N2(g) + O2(g) → 2NO(g)
(b) increasing the volume of the container.
(c) adding of F2 gas. 120. For the equilibrium N2O4  2NO2, the value of equilibrium
(d) adding of inert gas at constant pressure. constant is 2 × 10−4 at a particular temperature if equilib-
rium is established by taking 2 mol of N2O4 in 1 L container
113. Ammonia is a weak base that reacts with water according then
to the equation (a) concentration of NO2 at equilibrium is 0.02 M
NH3 (aq) + H2O(l)  NH+4 (aq) + OH− (aq) approximately.
(b) concentration of NO2 at equilibrium is 0.01 M
Select the correct option(s) that can increase the concentra- approximately.
tion of ammonium ion in water. (c) concentration of N2O4 at equilibrium is 2 M
(a) Addition of HCl (b) Addition of NaOH approximately.
(c) Addition of NH4Cl (d) Addition of H2O (d) concentration of N2O4 at equilibrium is 1 M
114. For the reaction AB2 (g)  AB(g) + B(g) , if a is negligible approximately.
w.r.t. 1, then degree of dissociation (a ) of AB2 is propor- 121. What will be the effect of addition of catalyst at constant
tional to temperature?
1 1 1 (a) The equilibrium constant will remain constant.
(a) (b) (c) (d) V
p V p (b) ∆H of the reaction will remain constant.
115. A catalyst (c) Kf and pKr will increase upto same extent.
(a) increases the average kinetic energy of reacting (d) The equilibrium composition will change.
molecules.
122. Increase in the pressure for the following equilibrium:
(b) decreases the activation energy.
(c) can alter the reaction mechanism. H2O(l)  H2O(g), result in the
(d) can change pre-exponential factor. (a) formation of more H2O(l)
116. Solid ammonium carbamate, NH4CO2NH2(s), dissociates (b) formation of more H2O(g).
into ammonia and carbon dioxide when it evaporates as (c) increase in boiling point of H2O(l)
shown by NH4CO2NH2(s)  2NH3(g) + CO2(g). At 25°C, the (d) decrease in boiling point of H2O(l).
total pressure of the gases in equilibrium with the solid is
0.116 atm. If 0.1 atm of CO2 is introduced after equilibrium 123. For the reaction PCl5 (g)  PCl3 (g) + Cl2 (g) the forward
is reached, then reaction at constant temperature is favoured by
(a) final pressure of CO2 will be less than 0.1 atm. (a) introducing an inert gas at constant volume.
(b) final pressure of CO2 will be more than 0.1 atm. (b) introducing chlorine gas at constant volume.
(c) pressure of NH3 will decrease due to addition of CO2. (c) introducing an inert gas at constant pressure.
(d) pressure of NH3 will increase due to addition of CO2. (d) increasing the volume of the container.

Chapter-7.indd 235 7/30/2016 2:45:16 PM


236 Chapter 7 Equilibrium

124. At a particular temperature and atmospheric pressure, 129. For the reaction SO2Cl2 (g)  SO2 (g) + Cl2 (g) , Kp > Kx is
the solid and liquid phases of a pure substance can exist obtained at
in equilibrium. Which of the following term defines this (a) 0.5 atm. (b) 0.7 atm. (c) 1 atm. (d) 2 atm.
temperature?
(a) Normal melting point (b) Equilibrium temperature 130. For the following equilibrium, relation between KC and Kx (in
(c) Boiling point (d) Freezing point terms of mole fraction) is

125. The following reaction attains equilibrium at high PCl3 (g) + Cl2 (g)  PCl5 (g)
temperature (a) KC = Kx(RT)−1 (b) KC = Kx(RT)
N2(g) + 2H2O(g) + heat  2NO(g) + 2H2(g) (c) KC = Kx(RT/p) (d) Kp = Kx(p/RT)
The yield of NO is affected by
(a) increasing the nitrogen concentration. Paragraph for Questions 131 to 134: Ostwald dilution law states
(b) decreasing the hydrogen concentration. that the degree of dissociation (a ) of weak electrolyte is inversely
(c) compressing the reaction mixture. proportional to square root of concentration a = K a /C . As the
(d) None of these. temperature increases, a will increase.
126. For the reaction N2O 4 (g)  2NO2 (g) , the value of K is 50 at a1 K a1
If concentration is the same, then =
400 K and 1700 at 500 K. Which of the following options is a2 K a2
correct? a1 C2
(a) The reaction is endothermic. If acid is same, then =
a2 C1
(b) The reaction is exothermic.
(c) If NO2(g) and N2O4(g) are mixed at 400 K at partial pres- 131. 0.01 M CH3COOH has 4.24% degree of dissociation. The
sures 20 bar and 2 bar, respectively, more N2O4(g) will be degree of dissociation of 0.1 M CH3COOH will be
formed. (a) 1.34% (b) 4.24% (c) 5.24% (d) 0.33%
(d) The entropy of the system increases.
132. pH of 0.005 M HCOOH with Ka = 2 × 10−4 is equal to
127. 2CaSO4(s)  2CaO(s) + 2SO2(g) + O2(g), ∆H > 0. (a) 4 (b) 2 (c) 3 (d) 5
Above equilibrium is established by taking sufficient amount 133. Which of the following is strongest acidic solution?
of CaSO4(s) in a closed container at 1600 K. Then which of (a) pH = 1 (b) pH = 2 (c) pH = 3 (d) pH = 4
the following may be a correct option? (Assume that solid
CaSO4 is present in the container in each case.) 134. If a1 and a2 are in ratio of 1:2 and K a1 = 2 × 10−4, then what
(a) Moles of CaO(s) will increase with the increase in will be the value of K a2?
temperature. (a) 4 × 10−4 (b) 2 × 10−4 (c) 8 × 10−4 (d) 1 × 10−4
(b) If the volume of the container is doubled at equilib-
rium, then partial pressure of SO2(g) will change at new Paragraph for Questions 135 to 136: The dissociation of nitrosyl
equilibrium. chloride into nitric oxide and chlorine takes place according to
(c) If the volume of the container is halved, partial pressure the equation: 2NOCl(g)  2NO(g) + Cl2 (g). Varying amount of the
of O2(g) at new equilibrium will remain same. three gases were placed in a container and allowed to come to
(d) If 2 mol of the He gas are added at constant pressure, equilibrium at two different temperatures. The equilibrium con-
then the moles of CaO(s) will increase. centrations of the three gases obtained are tabulated below:

Passage Type Concentration in mol L−1


Temperature NOCl NO Cl2
Paragraph for Questions 128 to 130: For a gaseous reaction
aA(g) + bB(g)  c C(g) + d D( g) , equilibrium constant KC, Kp and Kx 230°C 2.33 × 10−3 1.46 × 10−3 1.15 × 10−2
are represented by the following relations: 465°C 3.68 × 10−4 7.63 × 10−3 2.14 × 10−4
c c
[C ]c [D ]d pC ⋅ pDd x C ⋅ x Dd
KC = , Kp = , and K x =
a
[ A] [B ] b
pAa ⋅ pBb x Aa ⋅ x Bb 135. In one experiment, the volume of the reaction vessel was
halved at constant temperature, 230°C. The equilibrium
where [A] represents molar concentration of A, pA represents par- constant will be
tial pressure of A and p represents total pressure, xA represents (a) 4.514 × 10−3 mol L−1 (b) 1.614 × 10−2 mol L−1
mole fraction of A. −3
(c) 2.225 × 10 mol L −1 (d) 0.211 mol L−1
128. On the basis of the above calculations, the correct relation is 136. Which of the following statements is correct about the
(a) K p = K C (RT )
∆ng
; K x = K p (RT )
∆ng equilibrium?
− ∆ng ∆ng
(a) Increase in pressure promotes formation of NOCl.
(b) K C = K p (RT ) ; Kp = Kxp (b) Equilibrium constant for NOCl(g)  NO(g) + 21 Cl2 (g) is
(c) K C = K x p
∆ng
; Kp = Kxp
∆ng equal to that of 2NOCl(g)  2NO(g) + Cl2 (g)
− ∆ng ∆ng (c) Increase in temperature favours dissociation of NOCl.
(d) K C = K p (RT ) ; K x = K p (RT ) (d) For the reaction, Kp > KC.

Chapter-7.indd 236 7/30/2016 2:45:21 PM


Practice Exercise 237

Paragraph for Questions 137 to 140: Chemical equilibrium is (c) Solubility of calcium oxalate in 0.1 (r) 10000 Ksp
a dynamic state in which the composition of a system does not M oxalic acid solution.
change, and net reaction is zero. In reversible reactions, a stage is
reached when the rate of transformation of reactants into prod- (d) Solubility of Hg2Cl2 in 0.1 M NaCl (s) Ksp
ucts equals the rate of transformation of products into reactants. solution.
At this stage, the composition of reactants and products does not
change with time. The reaction has not stopped, but the rates of 142. Match the following equilibria with the corresponding
forward and reverse reactions are equal. For a general reaction condition.
A(g) + B(g)  C(g) + D(g), we have
Column I Column II
[C][D] p ×p (p) Saturated solution
KC = and K p = C D (a) Liquid  Vapour
[A ][B] pA × pB
(b) Solid  Liquid (q) Boiling point
where KC is the equilibrium constant for the ratio of the concen-
trations of the product to reactants and Kp is the equilibrium con- (c) Solid  Vapour (r) Sublimation point
stant for the ratio of partial pressure and of products to reactants.
The relation between Kp and KC is as follows. (d) Solute(s)  Solution(l) (s) Melting point
∆ng
K p = K C (RT )
143. Match the reactions with the factors.
137. The mass ratio of steam and hydrogen is found to be 1:2 at
equilibrium. Column I Column II
(p) Forward shift by
3Fe(s) + 4H2O(g)  Fe3O 4 (s) + 4H2 (g) (a) N2 (g) + 3H2 (g)  2NH3 (g)
rise in pressure.
(Exothermic )
The value of equilibrium constant (KC) for the above reac-
(q) Unaffected by
tion is (b) 2SO2 (g) + O2 (g)  2SO3 (g)
(a) 3.05 × 103 (b) 1.05 × 105 change in press-
(Exothermic )
(c) 0.75 × 102 (d) 2.42 × l07 ure.

138. For the reaction NH2COONH4 (s)  2NH3 (g) + CO2 (g) , the (c) N2 (g) + O2 (g)  2NO(g) (r) Forward shift by
equilibrium constant Kp = 2.9 × 10−5 atm3. The total pres- rise in temper-
sure of gases at equilibrium when 1 mol of reactant was ature.
heated will be Exothermic (s) Forward shift by

(d) PCl3 (g) + Cl2 (g)   PCl5 (g)
(a) 0.0194 atm (b) 0.0388 atm Endothermic lowering the
(c) 0.0580 atm (d) 0.0667 atm temperature.
139. KC for the reaction 21 N2 (g) + 21 O2 (g) + 21 Br2 (g)  NOBr(g)
from the following information at 298 K is 144. Match the conditions with effect on equilibrium.
30
2NO(g)  N2 (g) + O2 (g); K1 = 2.4 × 10 Column I Column II
1
NO(g) + Br (g)  NOBr(g);
2 2
K 2 = 1.4 (a) Pressure increased in (p) Equilibrium shifted in
forward direction.
(a) 3.15 × 10−9 (b) 6.35 × 10−18 2NO(g)  N2 (g)+ O2 (g)
(c) 9.03 × 10−16 (d) 17 × 10−17
(b) Pressure increased in (q) Equilibrium shifted in
140. The equilibrium constant for the reaction 2SO2 + O2  2SO3 reverse direction.
at 1000 K is 3.5. What would the partial pressure of oxygen CH4 (g) + H2O(g)  CO(g) + 3H2 (g)
gas to give equal moles of SO2 and SO3?
(a) 0.29 atm (b) 3.5 atm (c) Temperature increased and (r) Equilibrium remains
(c) 0.53 atm (d) 1.87 atm pressure increased in unaffected.

Matrix-Match Type 3O2 (g)  2O3 (g) ; ∆H = 285 kJ


141. Match the expression of solubility for various substances in
(d) Pressure decrease and moles (s) Theoretically we can-
terms of their solubility products.
of N2 increased in not predict.
Column I Column II
N2 (g) + 2O2 (g)  2NO2 (g) ;
(a) Solubility of Ag2CrO4 in 0.25 M (p) 10 Ksp
∆H = 66.4 kJ
K2CrO4 solution.
(b) Solubility of PbI2 in 0.01 M KI (q) 100 Ksp 145. Match the effect of change administered to its effect on
solution. equilibrium.

Chapter-7.indd 237 7/30/2016 2:45:27 PM


238 Chapter 7 Equilibrium

Column I Column II 148. The pH for a salt of weak acid and weak base having Ka = Kb
at 298 K is ____.
(a) For the equilibrium NH4I(s)  (p) Forward shift
NH3(g) + HI(g), if pressure is 149. If concentrations of SO2 and O2 in the equilibrium reaction,
increased at equilibrium 2SO2 (g) + O2 (g)  2SO3 (g) are quadrupled, the concentra-
(b) For the equilibrium N2(g) + 3H2(g) (q) No Change tion of SO3 now will be ___ times.
 2NH3(g) 150. The pH of an aqueous solution of a sparingly soluble metal
Volume is increased at equilibrium hydroxide M(OH)2 with solubility product 5 × 10−16 mol3
(c) For the equilibrium H2O(g) + (r) Reverse shift dm−9 at 298 K is ____ times.
CO(g)  H2(g) + CO2(g) inert gas 151. The pH at which Mg(OH)2 begins to precipitate from a solution
is added at constant pressure at containing 0.10 M Mg2+ ions will be ____. [Ksp of Mg(OH)2 =
equilibrium 1 × 10−11]
(d) For the equilibrium PCl5  PCl3 + (s) Final pressure is
152. Equilibrium constant for the reaction A3 (g) + 3B2 (g)  3AB2 (g)
Cl2, Cl2 is removed at equilibrium more than initial
A3 (g) + 3B2 (g)  3AB2 (g) is 64.0. Then the equilibrium constant for the
pressure
reaction 31 A3 (g) + B2 (g)  AB2 (g) will be ____.

Integer Type 153. If pKa of a weak acid is 5, then pKb of the conjugate base will
be ____.
146. A certain buffer solution contains equal concentration of X− 154. The pH of a buffer solution containing equal concentration
and HX− (Kb for X− is 10−10). What is the pH of the buffer? of B− and HB when Kb for B− is 1 × 10−10 is ____.
147. For the reaction involving oxidation of ammonia by oxygen 155. What is the pH of mixture obtained by mixing 50 mL of 0.05
to form nitric oxide and water vapour, the equilibrium con- M NH4OH and 50 mL of 0.05 M CH3COOH? [Ka = 1.8 × 10−5 and
stant has the units (bar)n. Then n is ____. Kb = 1.8 × 10−5]

ANSWER KEY
Level I
1. (b) 2. (c) 3. (b) 4. (d) 5. (b) 6. (d)
7. (d) 8. (c) 9. (b) 10. (b) 11. (c) 12. (a)
13. (c) 14. (a) 15. (b) 16. (a) 17. (b) 18. (a)
19. (c) 20. (c) 21. (a) 22. (b) 23. (c) 24. (c)
25. (b) 26. (d) 27. (b) 28. (b) 29. (d) 30. (a)
31. (c) 32. (b) 33. (c) 34. (c) 35. (b) 36. (b)
37. (a) 38. (d) 39. (c) 40. (d) 41. (a) 42. (d)
43. (a) 44. (d) 45. (d) 46. (a) 47. (b) 48. (b)
49. (b) 50. (b) 51. (d) 52. (c) 53. (b) 54. (a)
55. (a) 56. (a) 57. (b) 58. (c) 59. (a) 60. (a)
61. (b) 62. (c) 63. (a) 64. (d) 65. (d) 66. (d)
67. (d) 68. (b) 69. (d) 70. (a) 71. (d) 72. (a)
73. (b) 74. (d) 75. (c) 76. (d) 77. (d) 78. (d)
79. (a) 80. (c) 81. (b) 82. (b) 83. (b) 84. (b)
85. (c) 86. (d) 87. (a) 88. (a) 89. (b) 90. (d)
91. (a) 92. (c) 93. (a) 94. (a) 95. (d) 96. (a)
97. (d) 98. (c) 99. (c) 100. (b) 101. (d) 102. (b)
103. (a) 104. (b) 105. (c) 106. (b) 107. (a) 108. (a)
109. (d) 110. (c) 111. (c)

Level II
112. (a), (b), (d) 113. (a), (d) 114. (c), (d) 115. (b), (c), (d) 116. (b), (c) 117. (a), (b), (c)
118. (b), (c), (d) 119. (a), (c) 120. (b), (c) 121. (a), (b), (c) 122. (a), (c) 123. (c), (d)
124. (a), (d) 125. (a), (b), (c) 126. (a), (c), (d) 127. (a), (c), (d) 128. (b) 129. (b)
130. (c) 131. (a) 132. (c) 133. (a) 134. (c) 135. (a)
136. (a) 137. (b) 138. (a) 139. (c) 140. (a)
141. (a) → s; (b) → r; (c) → p; (d) → q 142. (a) → q; (b) → s; (c) → r; (d) → p
143. (a) → p, s; (b) → p, s; (c) → q, r; (d) → p, r 144. (a) → r; (b) → q; (c) → p; (d) → s 145. (a) → r, s; (b) →r; (c) → q; (d) → p, s
146. (4) 147. (1) 148. (7) 149. (8) 150. (9) 151. (9)
152. (4) 153. (9) 154. (4) 155. (7)

Chapter-7.indd 238 7/30/2016 2:45:29 PM


Hints and Explanations 239

HINTS AND EXPLANATIONS


Level I Excess of H+ or H3O + ion = (2 × 10 −2 − 1 × 10 −2 ) mol
Single Correct Choice Type = 1 × 10 −2 mol
1. (b) 2CH3COOH  (CH3COOH)2 1× 10 −2
Moles of H+ = = 1× 10 −1
As volume increases, pressure decreases. Therefore, equi 100 × 10 −3 L
librium will shift where the number of moles increases
pH = − log[H+ ] = − log[1× 10 −1] = 1
(i.e., shifts to the left).
2. (c) HSO 4− can act both as Brönsted acid and a Brönsted 9. (b) Molten sodium chloride conducts electricity due to the
base. presence of free ions.

3. (b) By increasing the concentration of CH3COOH, the pH of 10. (b) Use the equation K p = K C (RT )∆ng . In this reaction,
the buffer solution will decrease. ∆ng = 3 − 2 = 1, so
∆ng
4. (d) The reaction can be expressed as K p = K C (RT ) = (7.3 × 1034 )(0.0821× 298)1 = 1.8 × 1036

2A + B  3C 2D 11. (c) The required reaction is


Initial 1 2 0 1
Cl2CHCOOH  Cl2CHCOO − + H+
Reacted −0.6 −0.3 +0.9 +1.6
Initial moles 0.1 0 0
At equilibrium 0.4 1.7 0.9 1.6
Moles at equilibrium 0.1− x x x
pC3 × pD2 (0.9 )3 (1.6 )2 [H+ ][Cl2CHCOO − ]
Kp = = = 6.86 Ka =
pA2 × pB ( 0.4 )2 (1.7) [Cl2CHCOOH]

5. (b) In this graph, equilibrium constant decreases and the Given that Ka = 5.5 × 10−2, therefore,
temperature increases; hence, the reaction is endother- x×x
mic. As the temperature increases, the equilibrium shifts 5.5 × 10 −2 = ⇒ 0.005.5 − 0.055 x = x 2
0.1− x
in the direction of endothermic reaction.
6. (d) PH3 is not a Lewis acid. It cannot accept electrons. Solving the quadratic equation, we get x = [H+]
Instead, it acts as weak Lewis base as it can donate a = 2.7 × 10−2. Therefore,
lone pair of electrons. pH = −log[H] = −log(0.027) = 1.57

12. (a) NaOH + HCl → NaCl + H2O


P Weak base
Cl− ion is the conjugate base of strong acid HCl and does
H H H
not hydrolyze.
93.6°
PH3 molecule
(pyramidal) 13. (c) K p = K C (RT )∆n. If ∆n = 0, then Kp = KC . So, for H2 + I2  2HI,
∆n = 0.
BF3, AlCl3, and FeCl3 have vacant p and d orbitals; these 14. (a) The reactions involved are
can accept electron pair from Lewis bases and act as
strong Lewis acids. H3PO 4  H2PO 4− + H+
7. (d) The given equilibrium is N2 (g) + 3H2 (g)  2NH3 (g) H2PO 4−  HPO24− + H+
where ∆ng = 2 − (1 + 3) = −2. Substituting the values of
∆ng , KC, R and T we get Kp as Since H2PO 4− is amphiprotic and HPO2−
4 is its conjugate
base, pH > 7.
∆ng
K p = K C (RT ) = 0.5 × (0.082 × 400)−2 = 4.325 × 10 −4 atm2
15. (b) A product formation reaction is favoured when it is
8. (c) When 50 mL of 0.4 N HCl and 50 mL of 0.2 N NaOH are endothermic and ∆n = +ve at low pressure and high
mixed temperature.
50 × 0.4 10 −14
Moles of H3O + = = 2 × 10 −2 mol 16. (a) h=
Kw
= = 10 −8 = 10 −4
1000 Ka × C 10 5 × 10 −1

50 × 0.2
Moles of OH− = = 1× 10 −2 mol
1000 17. (b) NO3− is the weakest base because its conjugate acid
All of the OH− reacts leaving an excess of H+ ions. HNO3 is the strong acid.

Chapter-7.indd 239 7/30/2016 2:45:36 PM


240 Chapter 7 Equilibrium

18. (a) The reaction can be expressed as So, if x moles of O2 are being produced, electricity being
passed at anode is:
H + B  HB
4x (For O2) + 2x (3 for S2O2−
8 )= 6x F
Initial 0.1 0.1 0
At the cathode:
At equilibrium −0.02 −0.02 +0.02
2 H+ + 2 e − → H2
Total 0.08 0.08 0.02
2 F electricity ≡ 1 mol of H2 produced
[HB] 0.02 Therefore, 6x F electricity ≡ 3x mol of H2 is produced
K= = = 3.125
[H][B] (0.08 )(0.08 ) Hence, moles of H2 produced at cathode = 3 moles of O2
produced at anode.
19. (c) PCl5 (g)  PCl3 (g) + Cl2 (g)
25. (b) The value of K of an exothermic reaction decreases and
The gaseous mixture contains 40% Cl2 and 40% PCl3. that of an endothermic reaction increases with increase
Since they are produced in 1:1 molar ratio, PCl5 is 20%. in temperature. Here, the given reaction is exothermic. So
For ideal gases, % mole = volume %, so yield is good at lower temperature. Hence, T1 > T2 > T3.
pCl2 = pPCl3 = 2 × 0.40 = 0.8 atm 26. (d) N2 (g) + 3H2 (g)  2NH3 (g)
pPCl5 = 2 × 0.20 = 0.40 atm
Dividing by 2 and reversing the reaction, we get:
( p PCl3 )( pCl2 ) 0.8 × 0.8 1
Kp = = = 1.6 atm NH3 (g)  21 N2 (g) + 23 H2 (g); K’ =
( p PCl5 ) 0.4 K

27. (b) Reaction (b) will proceed farthest to completion since it


pM has the largest value for KC.
20. (c) Using d = ; we have that for a mixture at equilibrium,
d ∝ p. RT
28. (b) Since 1 × 10−8 M HCl is a strong acid, the pH is below 7.
Among the given reactions at equilibrium, the one that 29. (d) The reaction of ammonia with water is
is not affected by p (i.e., having same number of moles
of reactants and product) is option (c) only. So, vapour NH3 + H2O  NH4+ + OH−
density remains unchanged for N2 (g) + O2 (g)  2NO(g)
[HI]2 [28.0]2
21. (a) The pH is calculated as 30. (a) H2 + I2  2HI; K = = = 32.66
[H2 ][I2 ] [8][3]
1 1
pH = [pK w + pK a − pK b ] = [14 + 4.80 − 4.78] = 7.01 31. (c) pH = 10 means
2 2
[H+] ions = 10−10 mol L−1 = 10−13 g ions cm−3
22. (b) H2SO4 is the strongest acid because it is completely ion-
= 10−13 × (6.02 × 1023) ions cm−3
ized when dissolved in water and its Ka value is larger
= 6.02 × 1010 ions cm−3
when compared to the other three compounds.
32. (b) The reaction can be expressed as
H2SO 4 → 2H+ + SO24−
N2 (g) + 3H2 (g)  2NH3 (g)
H3PO4, CH3COOH, and HNO2 are weak acids. Initial moles 1 3 0
23. (c) According to Henderson equation, At equilibrium −0.25 −0.75 +0.5
[Salt] Total 0.75 2.25 0.5
pH = pK a + log
[Acid]
Total number of moles = 3.5
From the given values, we have
nNH3 0.5
0.4 0.25 pNH3 = × Total pressure = × 21 = 3 atm
[Salt] = M and [Acid] = M Total no. of moles 3.5
1.4 1.4
33. (c) The reaction represents a heterogeneous equilibrium,
0.4
So, pH = pK a + log = pK a + log1.6 = 4.761+ 0.204  5.0 so the equilibrium constant will depend only on con-
0.25 centrations of Ag+ and Cu2+ ions, while activity of solid
24. (c) At the anode: substances is taken as unity. Thus, for the reaction
4 OH− → O2 + 2H2O+ 4e − Cu(s) + 2Ag+ (aq)  Cu2+ (aq) + 2Ag(s)
2 SO24− → S2O28 − + 2e − The equilibrium constant expression is

1 mol of O2 requires 4 F electricity and 1 mol S2O2−


8 [Cu2+ (aq)][Ag(s)]2 2 × 10 −7 mol L−1
KC = = = 2 × 1015 mol L−1
(= H2S2O2−
8 ) requires 2 F electricity [Cu(s)][Ag+ ]2 (1× 10 −11mol L−1)2

Chapter-7.indd 240 7/30/2016 2:45:43 PM


Hints and Explanations 241

34. (c) For the reaction PCl3 (g) + Cl2 (g)  PCl5 (g), [2 × 10 −3 / 2]2 4 × 10 −6
KC = = = 1× 10 −5
K p = K C (RT )∆n 0.2 / 2 4 × 10 −1
where kf kf
43. (a) = KC ⇒ = 1.5 ⇒ k f = 1.125 × 10 −3
∆n = 1− 2 = −1 and T = 250 + 273 = 523 K kb 7.5 × 10 −4
Therefore, 44. (d) 2HI(g)  H2(g) + I2(g)
26 a a
K p = 26 × (0.821× 523)−1 =  0.61 1−a
(0.821× 523) 2 2
2
35. (b) The change in pressure has effect only on those equilib- a 
 pT  a 2 Kp
ria that involve gaseous substances and proceed with a 2
Kp = or = 2 K p or a =
change in the number of moles of the gases. According (1− a )2 pT2 1− a 1+ 2 K p
to Le Chatelier’s principle, increase in external pressure
should affect the equilibrium in such a way as to reduce 45. (d) KC is a characteristic constant for the given reaction.
the pressure. This implies that equilibrium will shift in 46. (a) Since 60% of SO2 is used up, the number of moles of SO2
the direction that has smaller number of moles of the that reacted is
gaseous substances. This can be easily understood from
the equilibrium representing the formation of SO3: 60
× 5 = 3 mol
2SO2 (g) + O2 (g)  2SO3 (g) 100
2SO2 (g) + O2 (g)  2SO3 (g)
On increasing the pressure, the volume occupied by the Initial moles 5 5 0
system will decrease. As such there will be greater num-
Moles reacted 3 1.5 3
ber of moles per unit volume. The effect of this change
can be counter balanced if the equilibrium shifts in the Molesat equilibrium 2 3.5 3
direction involving a decrease in the number of moles. Total number of moles at equilibrium = 2 + 3.5 + 3
This can happen only if SO2 and O2 combine to form = 8.5 mol
SO3. Thus, increase in pressure in this case will favour
47. (b) The aqueous solution containing 1 M NaCl and 1 M HCl
the forward reaction.
is a buffer solution of strong acid HCl and its salt NaCl
36. (b) with strong base. Thus, the pH is less than 7 due to HCl.
[NO2 ]2
KC = 48. (b) Since, there are only two gaseous species in the above
[N2O 4 ]
equilibrium, so we have their partial pressures as
[NO2 ]2 ptotal 1.04
4.61 × 10–3 = pCO2 = pH2 O = = = 0.52 atm
 0.0466  2 2
 
2  K p = pCO2 × pH2 O = [0.52]2 = 0.2704 atm
[NO2] = 1.04 × 10–2 M
49. (b) The degree of ionization of a compound depends on
37. (a) An acidic buffer is the solution of a mixture of a weak
nature of solute molecules. If the compound is strong
acid and a salt of this weak acid with a strong base.
acid, then it ionizes completely when dissolved in water.
38. (d) NaOH is a strong base because it gives OH− ion and can If the compound is weak acid, then it ionizes partially in
be easily ionized. aqueous solutions.
39. (c) In aqueous solution of NaCN; NaCN + H2O → NaOH + HCN 50. (b) CH3COOH  CH3COO − + H+
Since both acids and bases are present in the aqueous
If dil. HCl is added to the above equilibrium reaction,
solution of NaCN, it is amphoteric.
the concentration of CH3COO– will decrease due to
40. (d) At infinite dilution, the percentage ionization of both the common ion effect, that is, suppression in the
strong and weak electrolytes is 100%. degree of dissociation of a weak electrolyte by adding
a strong electrolyte containing a common ion. Here,
41. (a) N2 + 3H2  2NH3
HCl is a strong electrolyte, when it is added it splits
At appreciable dissociation constant (a), the equilibrium into H+ and Cl– ions. As a result, H+ is the common
constant does not change significantly with pressure. ion in the equilibrium and hence the concentration of
CH3COO– decreases.
42. (d) N2O 4  2NO2 0.01
51. (d) a = 0.01% = = 1× 10 −4
Moles at equilibrium 0.2 2 × 10 −3 100
0.2 2 × 10 −3 K a = a 2C = (10 −4 )2 × 0.1 = 10 −9
Conc. at equilibrium
2 2

Chapter-7.indd 241 7/30/2016 3:27:09 PM


242 Chapter 7 Equilibrium

( pCH3 OH ) 2 59. (a) Iodine solution can be titrated against sodium thiosul-
52. (c) CO + 2H2  CH3OH; K p = = = 200 phate (as also against sodium sulphite, sodium arsenite,
( pCO )( pH2 )2 (1)(0.1)2 etc.). For starch as an indicator, iodine gives an intense
Since the decomposition reaction is reverse of the given blue colour with starch due to formation of a surface
reaction, that is, complex. At the end point, the colour changes from blue
CH3OH  CO + 2H2 to colourless.
1 1 The redox reaction involved in the titration is
K p′ = = = 5 × 10 −3 atm
K p 200
2S2O23 − + I2  S4 O23 − + 2I−
2 mol 1mol
53. (b) The reaction can be represented as
NH2COONH4 (s)  2NH3 (g) + CO2 (g)
60. (a) We know that N1V1 + N2V2 = Nf Vf . Substituting given val-
Initial moles 1 0 0
Moles at equilibrium 0 2p p ues and Vf = V1 + V2 in this equation, we get
10 −2 × 1+ 10 −3 × 2 = Nf × 3 ⇒ Nf = 4 × 10 −3
The total pressure = 2p + p = 3p = 9 atm (given) ⇒ p = 3
atm.
Therefore, pH = − log( 4 × 10 −3 ) = 2.3980
2 2 2 3
Now, K p = ( pNH3 ) ( pCO2 ) = (2 p) ( p ) = (6 ) (3) = 108 atm
[NH4+ ][OH− ]
54. (a) The reaction can be represented as 61. (b) NH4 OH(aq)  NH4+ + OH− ; K b =
[NH4 OH]
HCN + H2O  H3O + + CN−
C 0 0 If a small quantity of a strong electrolyte such as NH4Cl
(which has a common ion, i.e., NH+4 ion) is added to the
C (1− a ) aC aC above solution, [ NH+4 ] increases. In order to maintain
Ca 2 KC the value of Kb constant, the reverse reaction is favoured.
Therefore, K C = ; 1− a  1; a = Thus, concentration of [OH–] decreases and [ NH+4 ]
1− a C
increases, that is, the forward reaction is suppressed due
4.9 × 10 −10 to common ion effect.
a= = 7 × 10 −5
0.1 62. (c) BF3 acts as Lewis acid but not as a Brönsted acid.

According to Lewis, acid is a substance that accepts a
55. (a) SnCl2 + 2Cl → SnCl4 lone pair of electrons. According to Brönsted, acid is a
SnCl2 is a Lewis acid because it can accept a pair of substance that can donate proton (H+). But in BF3 there
electrons. are no protons; hence, it is not a Brönsted acid.

56. (a) Acidity constant, Ka, is usually expressed in terms of pKa, 63. (a) Salt type Relation between Ksp and S
that is, negative logarithm of Ka. AB2 Ksp = (S)(2S)2 = 4S3
pK a = −log K a
64. (d) Given that normality of KOH = N/1000 = 10−3 N, so
Thus, strength of acid is inversely released to pKa value,
that is, the larger is the value of pKa, the weaker is acid. pOH = −log[OH− ] = − log[10 −3 ] = 3
Hence, the strong is acid B. pH = 14 − pOH = 14 − 3 = 11
57. (b) H(ClO)O3 is the strongest acid. The acidic character of 65. (d) The solubility product of a sparingly soluble salt at a
oxoacid containing the same halogen atom in different given temperature is the product of concentration of
oxidation state is H(ClO) < H(ClO)O < H(ClO)O2 < H(ClO)O3. its ions in the saturated solution, with each concentra-
It increases with increase in oxidation number of halo- tion term raised to equal to the number of times the ion
gen atom. occurs in the equation, representing the dissociation of
As the number of oxygen atom increases, the negative electrolyte. Lower the solubility product lesser will be
charge dispersal becomes more and more from Cl atom the solubility of the sparingly soluble salt.
due to more electronegative oxygen atom. Thus, lesser
is the charge on Cl atom, more will be its stability and it AgCl  Ag+ + Cl− ; K sp = 1.8 × 10 −10
easily furnishes proton and acts as the strongest acid.
AgF  Ag+ + F − ; K sp = 2.8 × 10 −5
58. (c) Na2CO3 + 2H2O  2NaOH + H2CO3
Strong base Weak acid AgI  Ag+ + I− ; K sp = 8.3 × 10 −7
When sodium carbonate is dissolved in water it forms Ag2S  2Ag+ + S2 − ; K sp = [Ag+ ]2 [S2 − ]; K sp = 6 × 10 −51
sodium hydroxide and carbonic acid. NaOH is the
strong base; hence, the aqueous solution of Na2CO3 is Since Ag2S has the lowest solubility product, it is least
alkaline. soluble.

Chapter-7.indd 242 7/30/2016 3:27:13 PM


Hints and Explanations 243

The value for solubility product (Ksp) of Ag2S is very less. The ability of a solution to resist change in pH on addi-
Hence, it is least soluble in water. tion of acid or base is called buffer action.
Buffer solution can be prepared by mixing:
66. (d) For a saturated solution of a sparingly soluble salt AB,
• A weak acid and its salt with a strong base or weak
the following solubility equilibrium would exist.
acid and its conjugate base, for example, CH3COOH
AB (Saturated salt solution)  A + (aq) + B − (aq) and CH3COONa.
• Weak base and its salt, for example, NH3 and NH4Cl.
If the molar solubility of the salt is S, then
72. (a) The reaction can be expressed as
[A + ] = S mol dm−3 , [B − ] = S mol dm−3 . Therefore, solu-
Ag2CrO 4 (s)  2Ag+ (aq) + CrO24−
bility product is given by
2S S
K sp = [A + ][B − ] = ( S )( S ) = S 2 + 2
K sp = [Ag ] [CrO24− ]
Given that Ksp = 1.21 × 10−6, so the molar solubility is
32 × 10 −12 = [2S ]2 [ S ]
−6 −3
S = K sp = 1.21× 10 = 1.1× 10
4[ S ]3 = 32 × 10 −12
67. (d) The reaction involved is [ S ] = 2 × 10 −4 mol L−1
73. (b) PbCl2 is AB2 salt type. It solubility product is Ksp = 4S3.
[H+ ][OH− ]
H2O(l)  H+ (aq) + OH− (aq); K a = From the given value of Ksp, we have
[H2O]
1.5 × 10 −4
K a [H2O] = [H+ ][OH− ] = K w 1.5 × 10 −4 = 4 S 3 ⇒ S 3 =
4
The constant Kw is called ionic product of water. Since
1.5 × 10 −4
as the temperature increases, dissociation of water also ⇒S=3 = 3.34 × 10 −2
increases, therefore value of ionic product of water 4
increases with increase in temperature. 74. (d) We have
68. (b) The reaction can be represented as
PbCl2  Pb2 + (aq) + 2Cl− ( aq)
NH4 Cl + H2O  H+ (aq) + CH3COOH− (aq) [Pb2 + ] = [PbCl2 ] = 2 × 10 −2 mol L−1
Before hydrolysis C 0 0 [Cl− ] = 2[P
PbCl2 ] = 2[2 × 10 −2 ] mol L−1
After hydrolysis C (1− h) x x
K sp = [Pb2 + ][Cl− ]2 = (2 × 10 −2 )(2 × 2 × 10 −2 )2 = 3.2 × 10 −5
where h is the degree of hydrolysis.

K  K  10 −14 75. (c) Given that the percentage dissociation = 22, so degree
h=  h =  w  = = 5.27 × 10 −5 of dissociation = 0.22. The reaction involved is
C  K bC  1.8 × 10 −5 × 0.2
2HI  H2 + I2
Initial moles 1 0 0
69. (d) [H+ ] = 10 −9 M + 10 −7 M
Moles at equilibrium 1− a = 0.78 a / 2 = 0.11 a / 2 = 0.11
At low concentrations, the contribution of H+ ions from
water should also be considered. Here, 10−7 M is from [H2 ][I2 ] 0.11× 0.11
Therefore, KC = = = 0.0199
ionization or dissociation of water. Then [HI]2 (0.78 )2

[H+ ] = 0.01× 10 −7 + 10 −7 = 1.01× 10 −7 M When 1 mol of hydrogen is added starting with 1 mol of
HI, we get
pH = − log[H+ ] = − log[1.01× 10 −7 ] = 6.9956 [H2 ][I2 ] (a / 2) × (a / 2 + 1)
KC = = = 0.0199
[HI]2 (1− a )2
70. (a) The reaction involved is
Solving, we get a = 0.037. So, the addition of 1 mol of H2
NaOCN + H2O  NaOH + HCN suppresses the dissociation of HI.
The degree of hydrolysis is 76. (d) Fe(OH)3(s)  Fe3+(aq) + 3OH–(aq)
K  K  10 −14 Ksp = [Fe3+][OH–]3 = 1.6 × 10–39
h =  h =  w  = = 10 −4
 C  K aC  3.33 × 10 −4 × 0.003 [Fe3+] [OH–]
Therefore, % hydrolysis = 10 −4 × 100 = 10 −2. Intial conc. – 0.050
+x + 3x
71. (d) A buffer solution is the solution that can resist the
change in pH on addition of small amount of acid or Final conc. +x 0.050 + 3x
base.

Chapter-7.indd 243 7/30/2016 3:27:18 PM


244 Chapter 7 Equilibrium

Substituting the above values for equilibrium concentra-


tions into the expression for Ksp gives: Before the addition of NaCl, [Ag+ ] = S = 10 −10 = 10 −5 M
Ksp = 1.6 × 10–39 = [Fe3+][OH–]3 = (x)[0.050 + 3x]3
We try to simplify by making the approximation that 10 −10
After the addition of NaCl, [Ag+ ] = = 10 −6 M
(0.050 + 3x) ≈ 0.050 10 −4
1.6 × 10–39 = (x)(0.050)3 or x = 1.3 × 10–35 Thus, the concentration of [Ag+] gets decreased.
Clearly the assumption that (0.050 + 3x) ≈ 0.050 is
justified. 83. (b) For solution A, pH = 3 ⇒ [H+ ] = 10 −3 M and for solution B,
Thus 1.3 × 10–35 M of Fe(OH)3 will dissolve in a 0.050 M
pH = 2 ⇒ [H+ ] = 10 −2 M.
sodium hydroxide solution.
Ka For the resultant solution,
77. (d) We know that a = where a = 0.01 and C = 0.1. So,
C 10 −3 M + 10 −2 M
Ka [H+ ] = = 5.5 × 10 −3
0.01 =
0.1 2
Taking square on both sides, we get pH = − log[H+ ] = − log[5.5 × 10 −3 ] = 2.26

K a = (0.01)2 × 0.1 = 0.1 × 10 −4 mol L−1


84. (b) Given that [NaOH] = 10 −3 M, [OH− ] = 10 −3 M ⇒ pOH = 3.
78. (d) Be(OH)2 has the lowest value of solubility product at 25°C. Now, pH + pOH = 14 ⇒ pH = 14 − pOH = 14 − 3 = 11
The trend of solubility of these hydroxides depends on
the values of lattice enthalpy and hydration energy. The Given that [HCl] = 10−3 M, so [H+] = 10−3 M ⇒ pH = 3
magnitude of hydration energy remains almost same
Given that [NaCl] = 10−3 M, so as per option (b), pH = 7.
whereas lattice enthalpy decreases appreciably down
the group, leading to more negative values for ∆Hsolution 85. (c) When the pH of a solution is increased from 3 to 6, its H+
down the group. ion concentration will be reduced by 1000 times.

Be(OH)2
Solubility 86. (d) In 100 mL of 0.01 M HCl:
decreases
Mg(OH)2 100 × 0.01
Moles of [H3O + ] = = 1× 10 −3 ⇒ pH = 3
Ca(OH)2 1000
Ba(OH)2 In mixture of 50 mL of 0.02 M H2SO4 and 50 mL of 0.02 M
NaOH:
50 × 0.02
Moles of [H3O + ] = = 1× 10 −3 × 2 = 2 × 10 −3
∆Hsolution = ∆Hlattice enthalpy + ∆Hhydration energy 1000
50 × 0.02
Moles of [OH− ] = = 1× 10 −3
1000
The more negative ∆Hsolution is, more is the solubility Excess [H3O + ] = 2 × 10 −3 − 1× 10 −3 = 1× 10 −3
of compounds. The solubility of hydroxide can also be
pH = − log[1× 10 −3 ] = 3
explained in terms of Ksp, which increases from Be(OH)2
to Ba(OH)2. 87. (a) When KCN is added in water, then pH will increase.
79. (a) H3BO3 is a weak acid and ionizes mainly as monobasic KCN + H2O → KOH + HCl
acid. It does not liberate H+ ion but accepts OH− ion and
behaves as weak Lewis acid. As KCN will react with water to form KOH, which is a
strong base.
H3BO3 + H2O → B(OH)4− + H+
88. (a) The reaction is represented as
+ − Na2CO3 + HCl  NaCl + NaHCO3
80. (c) The reaction involved is AgCl  Ag + Cl
Milliequiv. 30
The solubility product is Ksp = (S) × (S) = S2. But it is given before × 100 = 283 150 × 1 = 150 0 0
that Ksp = 10−10. Therefore, S = 10−5 and so [Ag+] = [Cl−] reaction 106
= 10−5 M.
Milliequiv.
81. (b) For pure water [H3O + ] = K w after 133 0 150 150
reaction
K w = [H3O + ]2 = (10 −6 )2 = 10 −12 The solution contains Na2CO3 and HCO3− and thus, acts
as buffer. The pH is
82. (b) [Ag+ ][Cl− ] = 10 −10 [Cl− ] = 10 −4 ( from NaCl)
[CO23 − ] 133
−10 −10 pH = − log K a + log = − log(5.63 × 10 −11) + log
10 10 [HCO3− ] 150
Then [Ag+ ] = = = 10 −6 M
[Cl− ] 10 −4 = 10.249 − 0.052 = 10.197

Chapter-7.indd 244 7/30/2016 3:27:23 PM


Hints and Explanations 245

89. (b) Given that Ka of weak acid, HA = 10−4. The reaction equi- 91. (a)
librium is as follows:
10 Phenophthalien
+ −
HA + H2O  H3O + A
7
From the reaction we have,
pH

[H3O + ][A − ] 4 Methyl orange


Ka = = 10 −4 (1)
[HA]
The reaction of strong acid with strong base is
0 50 100
HA + NaOH  NaA + H2O Volume of base (cm3)

The equilibrium constant is The graph shows how pH changes during titration of
50 cm3 of 0.1 M HCl with 0.1 M NH3
[NaA ] [Na+ ][A − ] HCl(aq) + NH4 OH(aq) → NH4 Cl(aq) + H2O(l)
K= =
[HA ][NaOH] [HA ][Na+ ][OH− ] In this case the pH changes rapidly from 3.5 to 7 at the
where both NaOH and NaA are strong electrolytes. Thus, equivalence point. Methyl orange is suitable indicator for
we have this type of titration. Phenolphthalein is unsuitable because
[A − ] its pH range lies outside the vertical portion of the curve.
K= (2)
[HA ][OH− ] 92. (c) The reaction is

[A − ] 10 −4 HCOOH  H+ + HCOO −
From Eq. (1), we get = 0.1− a a+b a
[HA] [H3O + ]

Substituting in Eq. (2), we get and HOCN  H+ + OCN−


0.1− b b+a b
[A − ] 10 −4 10 −4 (a + b )a
K= −
= + −
= K a(HCOOH) = (1)
[HA ][OH ] [H3O ][OH ] 10 −14 0.1− a
(as [H3O + ][OH− ] = K w = 10 −14 ) (a + b )b
K a(HOCN) = (2)
0.1− b
Solving, we get K = 10 −4 × 1014 = 1× 1010.
Dividing Eq. (1) by Eq. (2), we get
90. (d) The reaction involved is HSal  H+ + Sal−, with the ioni-
a 1.8
zation constant as = ⇒ b = 1.83a
b 3.3
[H+ ][Sal− ] Now solving, we get a = 2.52 × 10 −3 and b = 4.61× 10 −3 .
Ka = (1)
[HSal] Therefore, [H+ ] = a + b = 7.13 × 10 −3 M.
D−d
[H+] = 10−pH = 10−1.836 = 0.0146 M. With the given values, 93. (a) We know that a = (1)
(n − 1)d
the reaction can be expressed as
where D = MN2 O4 /2 = 92/2 = 46; d = 40 and x = 0.15. The
HSal  H+ + Sal− reaction can be represented as
Initial concentration (M) 0.200 0 0 N2O 4  2NO2
Change in concentration caused −x +x +x Initial moles 1 0
by the ionization Moles at equilibrium (1− x ) 2x
Concentration at equilibrium (0.200 − x ) x x
Hence, the total moles at equilibrium = (1 + x) = 1 + 0.15
= 1.15
We know the [H+] is 0.0146 M, so x = 0.0146. Thus, [HSal]
= 0.200 − 0.0146 = 0.185 M and [Sal−] = 0.0146 M. Substituting all values in Eq. (1), we get

[H+ ][Sal− ] (0.0146 )(0.0146 ) 46 − 40 6


Ka = = = 1.15 × 10 −3 a= = =1
[HSal] (0.185) (1.15 − 1)40 6

Therefore, pKa = –log (Ka) = –log (1.15 × 10−3) = 2.938. Percentage of NO2 = 26.08%

Chapter-7.indd 245 7/30/2016 3:27:29 PM


246 Chapter 7 Equilibrium

94. (a) Let a be the degree of dissociation of ammonia under [HI]2 (2 x )2


these conditions. According to the balanced equation K C′ = =
[H2 ][I2 ] (9.08 − x )(8.07 − x )
a moles of NH3 decompose to produce a/2 mol of N2
and 3a/2 mol of H2. (13.38 )2
=
(9.08 − 6.69)) × (8.07 − 6.69 )
2NH3  N2 + 3H2
= 54.279
Initial moles 1 0 0
To calculate the dissociation constant of HI, consider the
Moles at equilibrium 1− a a / 2 3a / 2 following reaction
Total number of moles after decomposition = 1 + α 2HI  H2 + I2
0 0 0
1− 2 x x x
Using Gay Lussac’s law
From the above data, the dissociation constant is given
p1 p2 by
=
T1 T2 x×x x2
T 273 + 347 620 KC = 2
=
p2 = p1 × 2 = 15 × = 15 × = 31 atm (1− 2 x ) (1− 2 x )2
T1 273 + 27 300
1 1
Number of moles without decomposition at temperature T p But KC =
= = 0.01842
= a K C′ 54.279
er decomposition at the same temprature T pb
Number of moles afte
Therefore, x can be calculated as
1 31atm
= = 0.613 2
1+ a 50 atm  x  1 x 1 1
  = ⇒ = =
% decomposition of ammonia = 0.613 × 100 = 61.3 % 1− 2 x 54.279 1− 2 x 54.279 7.3674

or 7.3674 x = 1− 2 x
95. (d) K = K × (RT )∆ng 1
p C Therefore, x = = 106.75 × 10 −3 mol
9.3674
= (2.2 × 1059 ) × [(0.0821 L atm/K /mol)(573 K )]−1
= 4.7 × 1057 a = 2 x = 2 × 106.75 × 10 −3 = 213.5 × 10 −3

96. (a) Given that i = 0.80, van’t Hoff factor, i = 1 + na – a, and Percentage dissociation = 21.35%
2A  A 2 .
98. (c) AgCl in water : [Ag+ ][Cl− ] = 10 −10
Two molecules get associated to form one molecule.
1 a S × S = S 2 = 10 −10
Hence, n = ; then i = 1+ na − a becomes 0.80 = 1+ − a
2 2 S = 10 −5 mol dm−3 = S1

a a a AgCl in 0.01 M CaCl2 : [Ag+ ][0.02] = 10 −10


0.80 = 1− ⇒ = 1− 0.80 ⇒ = 0.2 or a = 0.40
2 2 2 10 −10
[Ag+ ] = = 0.5 × 10 −8 = 5 × 10 −9 mol dm−3
2A  A2 2 × 10 −2
= S2
Initial 1 0
At equilibrium 1− 0.4 0.2 AgCl in 0.01M NaCl : [Ag ][10 −2 ] = 10 −10+

= 0.6 10 −10
[Ag+ ] = = 10 −8 mol dm−3 = S3
Total volume = 0.8(0.6 + 0.2) 10 −2
Volume of A2 0.2 AgCl in 0.05 M AgCO3 : [0.05][Cl− ] = 10 −10
Volume % of A2 = × 100 = × 100 = 25%
Total volume 0.8 10 −10
[Cl− ] = = 0.2 × 10 −8 = 2 × 10 −9 mol dm−3 = S4
5 × 10 −2
97. (d) The reaction of formation of HI is as follows: Hence, S1 > S3 > S2 > S4.

H2 + I2  2HI 99. (c) K p = K C × (RT )∆ng


8.07 9.08 0
8.07 − x 9.08 − x 2x Kp
KC = ∆ng
Therefore, from the above data x can be calculated as (RT )
4.6 × 10 −2
13.38 =
2 x = 13.38 ⇒ x = = 6.69 (0.0821 L atm
)(668
1
K )
2  mol K

The equilibrium constant can be calculated as = 8.4 × 10 −4

Chapter-7.indd 246 7/30/2016 3:27:36 PM


Hints and Explanations 247

100. (b) For this reaction at equilibrium So, [BrCl]eq = 3.03 × 10−3 mol L−1
pCO2 104. (b) H2 (g) + I2 (g)  2HI(g); K C = 20 ; p = 0.1 bar = 0.0987 atm
Kp = = 0.265 ⇒ pCO2 = K p pCO = 0.265 pCO
pCO pV 0.0987 × 10
Sum of initial partial pressures pV = nRT ⇒ n = = = 0.0384 mol
RT 0.0821× 313
= pCO + pCO2 = 1.4 atm + 0.8 atm = 2.2 atm Initial 12.7 g of I2 = 0.1 mol of I2
Sum of partial pressures at equilibrium = pCO + 0.265pCO At equilibrium 0.0384 mol of I2
= 1.265 pCO
Number of moles of I2 reacted = 0.1 − 0.0384 = 0.0616 mol
As there is no change in the number of moles of gase-
ous components during the reaction, we have 0.0616
[H2 ] to be introduced = = 0.00616 M = 0.00616 mol = 0.00616 × 2 = 0.012
10
2.2 atm
1.265 pCO = 2.2 atm ⇒ pCO = = 1.74 atm 0.0616
[H2 ] to be introduced =
1.265 = 0.00616 M = 0.00616 mol = 0.00616 × 2 = 0.0123 g
10
So, pCO2 = 0.265 × pCO = 0.265 × 1.74 atm = 0.46 atm. 105. (c) The tool we need to convert between Kp and KC is
101. (d) The reaction can be expressed as
∆ng
K p = K C (RT )
y+ x−
A xB y  xA + yB
From the given data, then, we have KC = 6.0 × 10−2, ∆ng
Initial concentration C 0 0 = −2 mol, T = 500 + 273 = 773 K and R = 0.0821 L atm
Conc. at equilibrium C (1−a ) xCa yCa mol−1 K−1. Substituting these into the equation for Kp
gives
( xCa ) x ( yCa ) y ( xCa ) x ( yCa ) y
K eq = = ( Taking 1− a = 1) K p = (6.0 × 10 −2 ) × [(0.0821) × (773)]−2
C (1− a ) C
x x y y C x + ya x + y = (6.0 × 10 −2 ) / (63.5)2 = 1.5 × 10 −5
= = x x y y C x + y −1a x + y
C In this case, Kp has a numerical value quite different from
1/( x + y ) that of KC.
 K eq 
a =  x y x + y −1 
x y C  106. (b) Let us consider p0 be the initial partial pressure. Then at
equilibrium, partial pressure of each gas would be
102. (b) The moles of NaOH and HCl are
pCl2 = p0 − 0.12, pF2 = p0 − 0.16
0.1 mol
× 100 mL = 10 −2 mol of NaOH 2
1000 mL pClF (0.2)2
3.2 = =
0.2 mol pCl2 × pF2 ( p0 − 0.12)( p0 − 0.16 )
× 50 mL = 10 −2 mol of HCl
1000 mL 3.2 [ p02 − 0.28 p0 + 0.0192] = 0.04
So, NaOH + HCl → NaCl + H2O p02 − 0.28 p0 + 6.7 × 10 −3 = 0 or p02 − 0.28 p0 + 0.0067 = 0
10 −2 mol
−2
10 mol
Thus, NaOH and HCl exactly neutralize each other. So −b ± b2 − 4 ac 0.28 ± ( 0.28 )2 − 4(1)(0.0067)
pH = pH of pure water = 7. p0 = = = 0.253
2a 2
103. (a) From the reaction stoichiometry Kp for the second reaction is
2
2BrCl (g)  Br2 (g) + Cl2 (g) pClF (0.04 )
Kp = 3
= = 14.95
Initial moles 2 mol 1 mol 1 mol pCl2 × pF32 (0.133)(0.093)3

Moles at equilibrium (3.30 × 10 −3 ) − x x x 107. (a) The reaction can be expressed as


2 2
A( s)  2B(g) + C(g)
Then, 2 2 3
2 2p p ⇒ K p = [B ][C] = [2 p] [ p] = 4 p
[Br2 ][Cl2 ] ( x / 2) × ( x / 2) ( x / 2)
KC = = −3
= 2 p − 0.2 p p + 1− 0.1p
[BrCl] 2
(3.30 × 10 − x)2
(3.30 × 10 −3 − x)2
x 1.8 p 0.9 p + 1 ⇒ K p = (1.8 p )2 (0.9 p + 1) = 4 p3
or (32)1/2 =
2(3.30 × 10 −3 − x )
x (3.24 p2 )(0.9 p + 1) = 4 p3
This gives, 5.66 × 2 =
3.30 × 10 −3 − x 2.916 p3 + 3.24 p2 = 4 p3
x 1.084 p3 = 3.24 p2
11.32 = −3
⇒ x = 3.30 × 10 −3
3.30 × 10 −x p2 (1.084 p − 3.24 ) = 0

Chapter-7.indd 247 7/30/2016 3:27:42 PM


248 Chapter 7 Equilibrium

So, we have p = 0 or 1.084 p − 3.24 = 0 . Solving, we get So, p = 4.8 (due to PCl5, PCl3 and Cl2 ), psolvent = 5.5 − 4.8 = 0.7
3.24 p = 4.8 (due to PCl5, PCl3 and Cl2 ), psolvent = 5.5 − 4.8 = 0.7
1.084 p = 3.24 ⇒ p = = 2.988 bar
1.084 Since p ∝ number of moles, we have
ptotal = 1.8 p + 0.9 p + 1 = 1.8 × 2.988 + 0.9 × 2.988 + 1 = 9.0676 bar p( Cl2 , PCl3 and PCl5 ) Total moles of Cl2 , PCl3 and PCl5
=
psolvent nsolvent
108. (a) The equilibrium expression for
22SO
SO33(g
(g) )
22SO (g) )++OO22(g
SO22(g (g) ) is, 4.8 0.152
= ⇒ nsolvent = 0.0222 mol = 0.022 × 154 = 3.388 g
0.7 nsolvent
[SO2 ]2 [O2 ]
KC =
[SO3 ]2 111. (c) N2F2 (g)  2NF(g); ∆H is positive.
The equilibrium expression for the second equation, For the above reaction ∆H is positive. On increasing
SO3 (g)  SO2 (g) + 21 O2 (g) is the temperature, reaction will proceed in a direction
where the temperature is decreasing that is in the for-
[SO2 ][O2 ]1/ 2
KC = ward direction. As the reaction proceeds in the forward
[SO3 ] direction the number of moles of gases increases. Due
Notice the relationship between the two equilibrium to which the pressure increases and the mercury level in
expressions the right hand limb rises.
[SO2 ]2 [O2 ] [SO2 ][O2 ]1/ 2 Level II
=
[SO3 ] [SO3 ] Multiple Correct Choice Type
The equilibrium constant of the second equation is the 112. (a), (b), (d) Cl2 (g) + 3F2 (g)  2ClF3 (g); ∆H = −329 kJ
square root of the first.
Increase in temperature favours endothermic reaction (here
5.87 × 10 −7 = 7.62 × 10 −4 reverse reaction).
Increase in volume of container means decreases in pres-
109. (d) The reaction can be expressed as sure; so, the equilibrium shifts to the side where the num-
2SO3 (g)  2SO2 (g) + O2 (g) ber of molecules increases (i.e., reverse reaction is favoured).
Addition of inert gas at constant pressure means increasing
x 0 0
the volume of the container.
−2 +2 +1
( x − 2) +2 +1 113. (a), (d) NH3 (aq) + H2O(l)  NH+4 (aq) + OH− (aq)
Addition of HCl removes OH−; hence, [NH+4 ] increases.
[O ][SO2 ]2 1× 4 Addition of water increases the dissociation of weak electro-
K= 2 = = 0.222
[SO3 ] ( x − 2)2 lyte; hence, [NH+4 ] increases.
x 2 − 4 x + 4 = 18 114. (c), (d) The reaction may be expressed as
x 2 − 4 x − 14 = 0 AB2 (g)  AB(g) + B(g)
4 ± 16 − 4( −14 ) 1 0 0
Hence, x = = 6.24 mol
2 −a +a +a
1− a a a
110. (c) The reaction can be expressed as
1− a a a
PCl5 (g)  PCl3 (g) + Cl2 (g) pAB2 = p pAB = p pB = p
1+ a 1+ a 1+ a
20.85 g
a 2 p2
(0.1 mol) 0 0 2 a 2p a 2p
−0.052 +0.052 +0.052 K = (1+ a ) = =
(1− a ) 2
p (1+ a )(1− a ) 1− a
0.048 0.052 0.052 (1+ a )
Total number of moles = 0.048 + 0.052 + 0.052 = 0.152 mol K 1
If 1−a 2  1, then K = a 2 p ⇒ a 2 = or a ∝ or a ∝ V
p p
If p is pressure to PCl5, PCl3 and Cl2, then
115. (b), (c), (d) A catalyst lowers the activation energy of a
 0.052   0.052  reaction.
pPCl3 . pPCl2   p ×  p
0.152  0.152 
Kp = = 116. (b), (c) The pressure of NH3 will decrease due to addition
pPCl5  0.048 
 p of CO2, (shifting of equilibrium in reverse direction as per
0.152 
Le Chatelier’s principle). The pressure of CO2 will be more
0.052 × 0.052 than 0.1 atm.
1.78 = p ⇒ p = 4.8
0.048 × 0.152

Chapter-7.indd 248 7/30/2016 3:27:49 PM


Hints and Explanations 249

117. (a), (b), (c) At equilibrium, ∆G = 0 and equilibrium constant  n


is independent of initial concentration of reactant. pV = nRT ⇒ p =   RT ⇒ p = CRT
V
118. (b), (c), (d) In accordance with Le Chatelier’s principle.
pA = C A RT = [A ]RT pC = C CRT = [C]RT
119. (a), (c) With the increase in pressure, forward reaction is
pB = CBRT = [B]RT pD = CDRT = [D]RT
favoured, because in forward reaction, lesser number of
molecules is involved. Therefore,
120. (b), (c) N2O4  2NO2
{[C]RT }c × {[D]RT }d [C]c [D]d (RT )c + d [C]c [D]d
Kp = = × = (RT )( c + d ) −( a + b ) = K C (R
2−x 2x a
{[A ]RT } × {[B]RT } b a
[A ] [B] b
(RT ) a+b
[A ]a [B]b
4 x2
Kp = 01]RT }c × {[D]RT }d [C]c [D]d (RT )c + d [C]c [D]d
= 2 × 10 −4 ⇒ x = 0{[.C ∆n
2 Kp = a b
= a b× a+b
= a b (RT )( c + d ) −( a + b ) = K C (RT ) g
{[A ]RT } × {[B]RT } [A ] [B] (RT ) [A ] [B]
[NO2] = 2x = 0.02 M, [N2O4] = 2/1 = 2 M
 Total number of   Total number of 
121. (a), (b), (c) Addition of catalyst does not change the where ∆ng = (c + d ) − (a + b ) = moles of gaseous − moles of gaseous
   
composition.
 products   reactants 
122. (a), (c) Increase in pressure for H2O(l)  H2O(g) causes − ∆ng
condensation of vapours into liquid, that is, more H2O(l) or K C = K p (RT )
will be formed. Increase in pressure increases the boiling
Now,
point.
123. (c), (d) Introducing an inert gas at constant pressure and Partial pressure = Mole fraction × Total pressure
increasing the volume of the container both favours the p = xp then pAa = x Aa p a pCc = x Cc p c
decomposition of PCl5.
pBb = xBb pb pDd = xDd p d
124. (a), (d) Solid  Liquid.
pCc × pDd x Cc p c × xDd p d x Cc × xDd pc + d
Kp = = = ×
125. (a), (b), (c) As the concentration of reaction (gaseous) pAa × pBb x Aa p a × xBb pb x Aa × xBb pa + b
is increased at equilibrium, reaction will go in the for-
ward direction. Also, the above reaction is endothermic. Thus , K p = K x p( c + d ) − ( a + b )
Therefore, an increase in temperature will favour it.
∆ng
or Kp = Kxp
126. (a), (c), (d)

N2O 4 (g)  2NO2 (g); K = 50 at 400 K and K = 1700 at 500 K ∆ng


129. (b) SO2Cl2 (g)  SO2 (g) + Cl2 (g); K p = K x p
As temperature increases, the value of equilibrium constant
For this reaction, ∆ng = 2 − 1 = 1
(K) increases. So, the forward reaction is endothermic.
2
Now, K p = K x p .
pNO 400
Q= 2
= = 200 at 400 K When p = 2 atm, Kp > Kx.
pN2 O4 2
130. (c) PCl3 (g) + Cl2 (g)  PCl5 (g)
Here, Q > K; so, reverse (reverse) reaction is favoured, that is,
more N2O4 will be formed.
∆ng ∆ng ∆ng
Kp = Kxp ⇒ K C (RT ) = Kxp
127. (a), (c), (d)
(a) As the reaction is endothermic, therefore it will go
where ∆ng = 1− 2 = −1. So,
in the forward direction. Hence, moles of CaO will
increase.
(c) With the increase or decrease of volume, partial pres- K C (RT )−1 = K x p −1
sure of the gases will remain same.
K x p −1 (RT )
(d) Due to the addition of inert gas at constant pressure, KC = = Kx ×
reaction will proceed in the direction in which more (RT )−1 ( p)
number of gaseous moles are formed.
a1 C2
131. (a) Here the acid is the same, so =
Passage Type a2 C1

[C]c [D]d pCC × pDd x CC × xDd 4.24


128. (b) K C = , Kp = , Kx = Given that a 1 = 4.24% = = 4.24 × 10 −2. Substituting, we
a
[A ] [B] b
pAa × pBb x Aa × xBb get 100

Chapter-7.indd 249 7/30/2016 3:27:55 PM


250 Chapter 7 Equilibrium

4.24 × 10 −2 0.1  1 
1/ 2
= ⇒ a 2 = 0.0134 × 100 = 1.34 % 139. (c) For 21 N2 (g) + 21 O2 (g)  NO(g); K C = 
a2 0.01  2.4 × 1030 

132. (c) The reaction can be expressed as NO(g) + 21 Br2 (g)  NOBr(g); K 2 = 1.4

HCOOH + H2O  HCOO − + H3O + 1


N (g) + 21 O2 (g) + 21 Br2 (g)  NOBr(g); K = K C × K 2 = 9.0 × 10 −16
2 2
[HCOOH] = 0.005 (1− a )  0.005 M 2
pSO
+ − 140. (a) K p =
[H3O ] = [HCOO ] = 0.005a
3
2
pSO 2
⋅ pO2
Substituting values, we get 1 1
Therefore pO2 = = atm = 0.29 atm
(0.005a )(0.005a ) K p 3.5
Ka = ⇒ 2 × 10 −4 = 5 × 10 −3 × a 2 ⇒ a = 0.2
0.005
Matrix-Match Type
+ −3
[H3O ] = 0.005a = 0.005 × 0.2 = 1× 10 141. (a) → s; (b) → r; (c) → p; (d) → q
−3
pH = − log[1× 10 ] = 3
(a) [2Ag+ ]2 [Cr2O24− ] = K sp Substituting, we get
133. (a) pH = 1 is the strongest acidic solution. Lower the pH more
K sp
acidic the solution. [2Ag+ ]2 [0.25] = K sp ⇒ 4[Ag+ ]2 =
0.25
134. (c) The ratio is K sp
[Ag+ ]2 = = K sp ⇒ [Ag+ ] = K sp
a1 K a1 1 2 × 10 −4 0.25 × 4
= ⇒ =
a2 K a2 2 K a2
(b) [Pb +2 ][2I− ]2 = K sp ; But [I− ] = 10 −2
Squaring on both sides, we get
K sp
1 2 × 10 −4 [Pb +2 ] = Solubility = = 10000 K sp
= ⇒ K a2 = 8 × 10 −4 [10 −2 ]2
4 K a2
(c) [Ca2+ ][C2O24− ] = K sp
135. (a) At 230°C,
K sp K sp
(1.15 × 10 −2 )(1.46 × 10 −3 )2 [Ca+2 ] = = = 10 K sp
K= −3 2
= 4.514 × 10 −3 molL−1 [C2O24− ] 0.1
(2.33 × 10 )
At 465°C, (d) K sp = [Hg2 + ][2Cl− ]2 But [Cl− ] = 0.1
(2.14 × 10 −4 )(7.63 × 10 −3 )2
K= = 0.092 mol L−1 [Hg2 + ][0.1]2 = K sp
(3.68 × 10 −4 )2
K sp
[Hg2 + ] = Solubility = = 100 K sp
As the volume of the container is halved at 230°C, K remains 10 −2
constant, that is,
142. (a) → q; (b) → s; (c) → r; (d) → p
K = 4.514 × 10 −3 molL−1
(a) Liquid  Vapour
The pressure exerted by the vapour in equilibrium with
136. (a) As we increase the pressure, the equilibrium will shift to
the liquid at a particular temperature is called vapour
the direction where number of moles get decreased.
pressure of the liquid.
137. (b) As ∆n = 0, so KC = Kp = Kx The boiling point of a liquid may be defined as the
temperature at which vapour pressure of the liquid
xH42 ( 2 / 2 )4 becomes equal to atmospheric pressure.
Kx = = = 18 4 = 1.05 × 105
xH42 O (1/18 )4 The above equilibrium represents the boiling point of
the liquid.
138. (a)
(b) Solid  Liquid
2 2 3 For any pure substance at atmospheric pressure the
K p = ( pNH3 ) × pCO2 = (2 p ) × p = 4 p
temperature at which the solid and liquid states can
1/ 3 1/ 3 co-exist is called the normal melting point or the nor-
 2.9 × 10 −5 
 Kp 
( )
1/ 3
p=  =  = 0.72 × 10 −6 = 0.0193 atm mal freezing point of the substance.
 4  4 
(c) Solid  Vapour

Chapter-7.indd 250 7/30/2016 3:28:02 PM


Hints and Explanations 251

For any pure substance at atmospheric pressure the Increase in pressure will favour the forward reaction.
temperature at which the solid and vapour states can With increase in temperature, the equilibrium shifts
co-exist is called sublimation point. toward reverse reaction and decrease in temperature
(d) Solute(s)  Solution(l) shifts the equilibrium toward forward direction.
A solution in which no more solute can be dissolved is 144. (a) → r; (b) → q; (c) → p; (d) → s
called saturated solution. (a) 2NO(g)  N2 (g) + O2 (g)
In a saturated solution, a dynamic equilibrium exists 2 mol 1mol 1mol
between the molecules in the solution. In the above reaction which does not involve any change
143. (a) → p, s; (b) → p, s; (c) → q, r; (d) → p, r in number of moles of gaseous species, in such reactions
pressure does not have any effect on equilibrium.
(a) N2 (g) + 3H2 (g)  2NH3 (g) (Exothermic)
(b) CH4 (g) + H2O(g)  CO(g) + 3H2 (g)
On increasing the pressure, the volume occupied by the 1mol 1mol 1mol 3 mol
system will decrease. As such there will be greater num- On increasing the pressure, the volume occupied by
ber of moles per unit volume. The effect of this change the system will decrease. As such there will be greater
can be counter balanced if the equilibrium shifts in the number moles per unit volume. The effect of this
direction involving a decrease in the number of moles. change can be counter balanced if the equilibrium
This can happen only if nitrogen and hydrogen com- shifts in the direction involving a decrease in the num-
bine to form ammonia. Thus, increase in pressure in ber of moles. In the above reaction the number of
this case will favour the forward reaction. moles decrease in the reactant side; hence, the equi-
Effect of temperature: librium shifts in toward the reverse direction.
Exothermic

N2 (g) + 3H2 (g)  
 2NH3 (g) ; ∆H
H = −93.6 kJ Exothermic
Endothermic (c) 
3O2 (g)  2O3 (g) ; ∆H = 285 kJ
In this equilibrium, the forward reaction is exothermic Endothermic
3 mol 2 mol
while the reverse reaction is endothermic. Now, if the On increasing the pressure, the equilibrium is shifted
temperature is increased, the equilibrium will shift in toward forward direction because in the product side,
the direction of endothermic reaction, which tends there is decrease in the number of moles. Hence, it is
to undo the effect of added heat. Since the reverse favoured.
reaction is endothermic, so the equilibrium shifts on With increase in temperature, the equilibrium shifts
favour of reverse reaction. In other words, it will result toward the exothermic reaction; hence, the equilib-
into lesser amount of ammonia. On the other hand, rium is shifted to the forward direction.
if temperature is decreased, the equilibrium will shift
towards exothermic reaction. So, low temperature (d) N2 (g) + 2O2 (g)  2NO2 (g); ∆H = 66.4 kJ
1mol 2 mol 2 mol
favours the formation of ammonia.
If pressure is decreased, the equilibrium will shift
(b) 2SO2 (g) + O2 (g)  2SO3 (g) (Exothermic ) toward reverse direction.
This reaction is also similar to formation of NH3, that is, But at a time when pressure is decreased and the
increase in pressure favours the forward reaction and number of moles of N2 gets increased, then we theo-
low temperature favours the formation of SO3. retically cannot predict the equilibrium shift.

(c) N2 (g) + O2 (g)  2NO(g) 145. (a) ã r, s; (b) ã r; (c) ã q; (d) ã p, s


In the above reaction which do not involve change in (a) As ∆ng > 0 therefore if p increases, reaction will go in
number of moles of gaseous species. In such reactions the reverse direction.
pressure does not have any effect on equilibrium. (b) As ∆ng < 0 therefore if V increases, p decreases reac-
Effect of temperature: tion will go in the direction in which more number of
Exothermic gaseous moles are formed that is reverse direction.

N2 (g) + O2 (g)   2NO(g); ∆H = 180 kJ. (c) As ∆ng = 0, hence no effect.
Endothermic
In the case of reaction between nitrogen and oxy- (d) If concentration of the product is decreased, reaction
gen, the forward reaction is endothermic while will go in the forward direction.
the reverse reaction is exothermic. According to Integer Type
Le Chatelier’s principle, the increase in temperature
will favour the forward (endothermic) reaction. This 146. (4) Given that Kb for X− = 10−10. For conjugate acid–base
is because the equilibrium shifts toward the direction pair, we have
in which heat is absorbed to relieve the system of the
10 −14
stress of added heat. On the other hand, decrease in K a(HX ) × K b(X −1 ) = 10 −14 ⇒ K a(HX ) = = 10 −4
temperature shifts the equilibrium in the direction of 10 −10
exothermic reaction, that is, in favour of reactants in But [HX] = [X−], so pH is
the above reaction. [Salt]
Exothermic pH = − log K a + log = − log 10 −4 = 4
(d) 
PCl3 (g) + Cl2 (g)    PCl5 (g) [Acid]
Endothermic

Chapter-7.indd 251 7/30/2016 3:28:05 PM


252 Chapter 7 Equilibrium

147. (1) For the reactions 151. (9) When Mg(OH)2 starts precipitation, the solubility product
5 is
2NH3 (g) + O (g) → 2NO(g) + 3H2O(g)
2 2 Ksp = [ Mg2+][OH−]2
4NH3 (g) + 5O2 (g) → 4NO(g) + 6H2O(g)
(bar)4 (bar )6 [0.1][OH− ]2 = 1× 10 −11 ⇒ [OH− ] = 10 −5 M
K= = (bar )1 ⇒ n = 1
(bar)4 (bar )5 Thus, pOH = 5 and pH = 14 − pOH = 14 − 5 = 9.
148. (7) Ka = Kb. So, pOH = pH. Therefore, pH = pOH = 7.
152. (4) As number of moles of A3 is 1/3, so, K C = 3 K C = 3 64 = 4.
149. (8) 2SO2 (g) + O2 (g)  2SO3 (g)
153. (9) pH = pK a + pK b . Suppose pH = 14, and given that
[SO3 ]2 pKa = 5, then pKb = 9.
KC = ⇒ [SO3 ]2 = K C [SO2 ]2 [O2 ]
[SO2 ]2 [O2 ]
154. (4) Given that [Salt] = [Acid], so pOH = pKb. Given that
[SO3 ]2 ∝ [SO2 ]2 [O2 ] or [SO3 ] ∝ [SO2 ]2 [O2 ] pOH = 10, so pH = 14 − 10 = 4.

When concentration is in equilibrium, 155. (7) The reaction involved is


[SO3 ] ∝ [ 4 SO2 ]2 [ 4 O2 ]
NH4 OH + CH3COOH  CH3COONH4 + H2O
2 2 Before hydrolysis 2.5 2.5 0 0
[SO3 ] ∝ 16[SO2 ] 4[O2 ] ∝ 8 [SO2 ] [O2 ]
After hydrolysis 0 0 2.5 2.5
It increases 8 times.
In this case, pH is decided on the basis of hydrolysis of salt.
2+ − 2.5
150. (9) M(OH)2 → M + 2 OH
[CH3COONH4 ] = = 2.5 × 10 −2 M
2+ − 2
K sp = [M ][OH ] = 5 × 10 −16 100

If M2+ = S, then [OH−] = 2S, thus Therefore,


1 1
K sp = S × (2S )2 ⇒ K sp = 4 S 3 pH = + [log K b − log K w − log K a ] = [pK w + pK a − pK b ]
2 2
1/ 3 1
 5 × 10 −16 
( )
−5 −14
⇒S= = 0.125 × 10 −15
1/ 3
= 0.5 × 10 −5 M = + [log (1.8 × 10 ) − log10 − log(1.8 × 10 −5 )]
 2
 4 
=7
Therefore,

[OH− ] = 2S = 2 × 0.5 × 10 −5 M = 1× 10 −5 M

(
pOH = −log 1× 10 −5 = 5 )
pH = 14 − 5 = 9

Chapter-7.indd 252 7/30/2016 3:28:09 PM


Solved JEE 2016 Questions 253

SOLVED JEE 2016 QUESTIONS


JEE Main 2016 Solution
1. The equilibrium constants at 298 K for a reaction A + B  C + D (b) The give n reaction for thermal decomposition is
is 100. If the initial concentration of all the four species were 1 M
each, then equilibrium concentration of D (in mol L−1) will be:
(a) 1.182 (b) 0.182 (c) 0.818 (d) 1.818 X2 (g)  2X (g)
(Offline) At t = 0 , 1 0
Solution At t = t , (1− a ) 2a
A +B  C+D
(d) Initial mole 1 1 1 1
At t = t eq (1− b /2) b (Given 2a = b )
Let x be the amount of substance dissociated Total number of moles at equilibrium = (1 + a) = (1 +βeq/2)
At teq 1–x 1–x 1+x 1+x Therefore,
[C][D]
keq =  
[A ][B]  b eq 
pX =  pT 
(1+ x )2 9 b
 1+ eq 
100 = ⇒1 + x = 10 – 10x ⇒ x =
(1− x )2 11  2 
9  b eq 
[D] = 1 + x = 1+ = 1.818 mol L−1  1− 2 
11 pX2 =  p
b eq T 
 1+ 
2. A solid XY kept in an evacuated sealed container undergoes  2 
decomposition to form a mixture of gases X and Y at temper- Therefore, equilibrium constant Kp is given by
ature T. The equilibrium pressure is 10 bar in this vessel. Kp for
2
this reaction is:  2
(a) 25 (b) 100 (c) 10 (d) 5  b eq 
 bb pT 
(Online) eq
 1+ eq pT 
pX2  1+ b2eq 
Solution Kp = p = 
2

(b) For reaction: XY (s)  X(g) + Y(g) K p = pXX2 =  b2eq 
 1 − b
pX2 2eq 
pX = 10 bar and pY = 10 bar  1− b2 pT 
K p = ( pX )( pY ) = 10 × 10 = 100  1+ b eq pT 
 1+ 2eq 
 2 2  2 2
b eq b eq 8 b eq
= b eq2 pT = b eq
2 2
2
×2 = 2
8 b eq2
JEE Advanced 2016 = b eq pT = b eq ×2 = 4 − b eq
1− b eq 2 1− b eq
2 2
4 − b eq
Paragraph for Questions 1 to 2: Thermal decomposition of gase- 1− 4 1− 4
ous X2 to gaseous X at 298 K takes place according to the following 4 4
equation : 2. The INCORRECT statement among the following , for this reac-
X2 (g)  2X(g) tion, is
(a) decrease in the total pressure will result in formation of
The standard reaction Gibbs energy, ΔrG°, of this reaction is pos- more moles of gaseous X.
itive. At the start of the reaction, there is one mole of X2 and no (b) at the start of the reaction, dissociation of gaseous X2 takes
X. As the reaction proceeds, the number of moles of X formed is place spontaneously.
given by b. Thus, bequilibrium is the number of moles of X formed (c) bequilibrium = 0.7
at equilibrium. The reaction is carried out at a constant total pres- (d) KC < 1
sure of 2 bar. Consider the gases to behave ideally.
(Given : R = 0.083 L bar K−1mol−1) Solution
(d) On decreasing total pressure, the reaction will move in
1. The equilibrium constant Kp for this reaction at 298 K, in terms
the forward direction where number of gaseous mole-
of bequilibrium, is
2 cules is less. So option (a) is correct.
2 8 b equilibrium
(a) 8 b equilibrium (b) 2
At the start of reaction Q = 0. From ∆G = ∆Go + RT lnQ, we
2−b 4 − b equilibrium have that for Q = 0 at the start of reaction ∆rG is nega-
equilibrium
tive, this causes dissociation of X2 to take place sponta-
2 2
4 b equilibrium 4 b equilibrium neously. So option (b) is correct.
(c) (d) 2 If bequilibrium = 0.7, then value of equilibrium constant is
2 − b equilibrium 4 − b equilibrium

Chapter-7.indd 253 7/30/2016 3:28:13 PM


254 Chapter 7 Equilibrium

2 ∆Go > 1, so Kp should be less than 1.


8 b eq 8(0.7)2
Kp = = We know that, ∆n
2
4 − b eq 7 − (0.7)2 K p = K C (RT ) g

The value of Kp is greater than 1, which is not possible as Kp


KC = ∆ng
given that ∆G°> 0 for the reaction. (RT )
Therefore, option (c) is incorrect.
As ∆Go > 0 and Therefore, KC < Kp. As Kp is less than 1, so KC is also less
than 1.
∆Go = −RT lnKp

Chapter-7.indd 254 7/30/2016 3:28:14 PM


8 Redox Reactions

Question Distribution in JEE (Main and Advanced)

3
No. of Questions

JEE (Main)
2
JEE (Adv)

0
2016 2015 2014 2013 2012 2011 2010 2009 2008 2007

Concept Distribution in JEE (Main and Advanced)


Topics Covered
Year
JEE (Main) JEE (Advanced)
2007 Redox Reaction Titrations, Types of Redox Reactions
2008 Redox Reaction Titrations
2009 Modern Approach to Redox Reactions
2010 Modern Approach to Redox Reactions
2011 Modern Approach to Redox Reactions, Balancing of Ionic
Redox Reactions
2012 Types of Redox Reactions
2013 Balancing of Ionic Redox Reactions
2014 Modern Approach to Redox Reactions, Balancing of Ionic Modern Approach to Redox Reactions
Redox Reactions
2015 Modern Approach to Redox Reactions
2016 Stoichiometry of Redox Reaction

Chapter-8.indd 255 8/4/2016 10:41:56 AM


256 Chapter 8 Redox Reactions

SUMMARY
1. Electron transfer reactions are called oxidation–reduction reactions, or simply redox reactions.
2. Classical concept of oxidation and reduction
(a) Oxidation is defined as addition of oxygen or electronegative element to a substance or removal of hydrogen/electropositive
element from a substance. Some examples of oxidation:

Oxidation reactions Examples

Addition of oxygen S(s) + O2(g) → SO2(g)

CH4 + 2O2 → CO2 + 2H2O


Removal of hydrogen
H2S + Cl2 → 2HCl + S

Addition of electronegative elements 2FeCl3 + SnCl2 → 2FeCl2 + SnCl4

Removal of electropositive elements 2KI + Cl2 → 2KCl + I2

(b) Reduction is defined as removal of oxygen/electronegative element from a substance or addition of hydrogen/electropositive
element to a substance. Some examples of reduction are:

Reduction reactions Examples


Removal of oxygen CuO + H2 → Cu + H2O
Copper is reduced as oxygen from CuO is removed.

Removal of electronegative element 2FeCl3 + SnCl2 → 2FeCl2 + SnCl4


FeCl3 is reduced as one chlorine atom is removed.

Addition of hydrogen C2H4 (g) + H2 (g)  C2H6 (g)


Ethene is reduced to ethane in the presence of hydrogen.

Addition of electropositive element 1


Na + Cl2 → NaCl
Cl2 is reduced as the addition of electropositive element 2

sodium takes place.

(c) Oxidation and reduction always occur together, that is, no substance is ever oxidized unless something else is reduced. Therefore,
these reactions are collectively called oxidation–reduction or redox reactions.
3. Oxidation and reduction as electron transfer reactions
(a) Oxidation is the loss of electrons by one reactant; Reduction is the gain of electrons by another reactant.

(b) Na → Na+ + e − (oxidation)


− −
Cl2 + 2e → 2Cl (reduction)

Overall reaction: 2Na(s) + Cl2 (g) → 2Na+ Cl− (s)


(c) The total number of electrons lost by one substance is always the same as the total number gained by the other.
4. Oxidizing and reducing agents
(a) Oxidizing agent is a substance that accepts electrons, and causes the other substance to lose electrons and get oxidized. (It
undergoes decrease in valence.)
(b) Reducing agent is a substance that donates electrons, and causes the other substance to gain electrons and get reduced. (It
undergoes an increase in valence.)
(c) In a redox reaction, the oxidizing agent gets reduced and the reducing agent gets oxidized.
For example, zinc metal donates electrons to the Ag+ ions, and therefore reduces the Ag+ ions. Zinc, therefore, acts as a reducing
agent in the reaction. The same electrons that are lost by the zinc are gained by the silver. Ag+ ion is, therefore, an oxidizing
agent.

Chapter-8.indd 256 8/4/2016 10:41:58 AM


Summary 257

Zn(s) + 2Ag+ (aq)  Zn2 + (aq) + 2Ag(s)


Reducing Oxidizing
agent agent

5. Displacement reactions and activity series


(a) In displacement reactions, one metal displaces another metal from its compounds. For example, displacement of copper ions
(from copper sulphate) by zinc ions (from zinc metal).
Zn(s) + Cu2 + (aq) → Cu( s) + Zn2 + (aq)

Metallic zinc is oxidized as copper ion is reduced. In the process, Zn2+ ions have taken the place of the Cu2+ ions, so a solution of
copper sulphate is changed to a solution of zinc sulphate.
(b) An element that is more easily oxidized will displace the one that is less easily oxidized from its compounds. The metals are
placed in order of their ease of oxidation. Thus emerges the activity series shown in the table below. In this table, metals at the
bottom are more easily oxidized (are more active) than those at the top.
Element Oxidation Product

Least active Gold Au3+


Mercury Hg2+
Silver Ag+
Copper Increasing case of oxidation of the metal Cu2+
Hydrogen H+

Increasing case of reduction of the ion


Lead Pb2+
Tin Sn2+
Cobalt Co2+
Cadmium Cd2+
Iron Fe2+
Chromium Cr3+
Zinc Zn2+
Manganese Mn2+
Aluminum Al3+
Magnesium Mg2+
Sodium Na+
Calcium Ca2+
Strontium Sr2+
Barium Ba2+
Potassium K+
Rubidium Rb+
Most active Cesium Cs+

Tip Important observations from activation series:


(a) An element will be displaced from its compounds by any metal below it in the table.
(b) Metals below hydrogen in the series can displace hydrogen from solutions containing H+.
(c) The metals above hydrogen in the table do not react with acids in which H+ is the strongest oxidizing agent.
(d) Metals at the very bottom of the table are very easily oxidized and hence are extremely strong reducing agents. They are so
reactive that they are able to reduce the hydrogen in water molecules.
(e) For metals below hydrogen in the activity series, there is a parallel between the ease of oxidation of the metal and the
speed with which it reacts with H+.

6. Oxidation number and oxidation state


(a) Oxidation number
The oxidation number of an element indicates the number of electrons lost, gained, or shared when the element changes from
free-state to form that compound (i.e., as a result of chemical bonding). The change in the oxidation number of a species helps
us identify if the element has undergone oxidation or reduction.
(b) Oxidation state
A term that is frequently used interchangeably with oxidation number is oxidation state. Oxidation state refers to the degree
of oxidation of an atom in a molecule. Each atom of the molecule have a distinct oxidation state for that molecule where the sum
of all the oxidation states is equal to the overall electrical charge of the molecule or ion.
(c) Oxidation number is the total charge on all the atoms of the same kind constituting a compound, whereas oxidation state is the
charge per atom of all the atoms of the same kind constituting a compound.
Oxidation state = oxidation number × total number of atoms of the element in the compound.

Chapter-8.indd 257 8/4/2016 10:41:59 AM


258 Chapter 8 Redox Reactions

(d) Stock notation


(i) Oxidation state of an element can be specified by writing the oxidation number as a roman numeral in parentheses after the
name of the element. For example, “iron (III)” means iron is in the +3 oxidation state.
(ii) In molecular compounds, the oxidation number is expressed by putting the roman numeral in parenthesis after the symbol
of the metal in the molecular formula. For example, ferric chloride can be represented as Fe(III)Cl3 and ferrous chloride as
Fe(II)Cl2.
(iii) The oxidation number helps in identifying in which compound the element is present in reduced and oxidized states. For
example, stannous chloride Sn(II)Cl2 is obtained by the reduction of stannic chloride Sn(IV)Cl4.

Tip It is assumed that if two or more than two atoms of an element are present in a molecule or ion (e.g., two atoms of sulphur in
Na2S2O3), then the oxidation number of that element is the average of the oxidation number of all the atoms of that element.

7. Rules for assigning the oxidation number


(a) For simple, monatomic ions in a compound, the oxidation numbers are the same as the charges on the ions. For example, in NaCl
the oxidation number of sodium is +1 and the oxidation number of chlorine is −1.

Tip To differentiate oxidation numbers from actual electrical charges, the sign is specified before the number when writing oxida-
tion numbers, and after the number when writing electrical charges. Thus, a sodium ion has a charge of 1+ and an oxidation number
of +1.

(b) In covalent compounds, the following set of rules is followed to determine the oxidation numbers.
(i) The oxidation number of any free element is zero. For example, each atom in molecules such as H2, O2, S8, P4, Ca, K, etc. has
oxidation number zero.
(ii) The oxidation number for any simple, monatomic ion (e.g., Na+ or Cl−) is equal to the charge on the ion. The oxidation num-
ber of a polyatomic ion is equal to the charge on the ion. For example, the oxidation state of SO2−
4 is −2 and the oxidation
state of PO3−
4 is −3.
(iii) The sum of all the oxidation numbers of the atoms in a molecule/polyatomic ion must equal the charge on the particle.
For example, in Cr2O72− , the sum of oxidation numbers of two chromium and seven oxygen atoms must be equal to −2. The
algebraic sum of oxidation numbers of all the atoms in a compound must be zero.
(iv) Fluorine has an oxidation number of −1 in all its compounds, other halogens also have −1 oxidation number in their halides
but the oxoacids and oxoanions of halogens have positive oxidation numbers.
(v) Hydrogen has an oxidation number of +1 in most of its compounds. When combined with a less electronegative element
(usually a metal), hydrogen has oxidation number −1 (e.g., LiH).
(vi) Oxygen has an oxidation number of −2 in most of its compounds; Exceptions include molecules and polyatomic ions that
contain O O bonds, such as O2, O3, H2O2 and the O2− 2 ion. Oxygen is positive only when bound to the more electronegative
fluorine. When it is bonded to fluorine (OF2 or O2F2), the oxidation numbers are +2 and +1, respectively.
(vii) In binary ionic compounds with metals, the non-metals have oxidation numbers equal to the charges on their anions. For
example, Mg3P contains the phosphide ion P3− which has an oxidation number of −3.

(c) Oxidation numbers for different types of elements


(i) All the metals in Group 1 form ions with a 1+ charge, and all those in Group 2 form ions with a 2+ charge.
(ii) Some elements, particularly transition elements and some p-block elements show variable oxidation states.
(iii) When non-metals are combined with H and O in compounds/polyatomic ions, their oxidation number varies and have to be
calculated using the above rules. Non-metals can have both +ve or −ve oxidation numbers.
(iv) The folllowing figure shows the common oxidation numbers for many of the elements in the periodic table. The following
observations can be made:
• Elements in the same group often have the same oxidation numbers.
• The largest, or most positive, oxidation number of an atom is often equal to the group number of the element.
• The smallest, or most negative, oxidation number of a non-metal can often be found by subtracting 8 from the group
number.
• The highest value of oxidation number for the first two groups is the group number while for the rest of the groups
(except transition metals), it is group number minus 10.
• The maximum value of oxidation number of an element increases across the period.

Chapter-8.indd 258 8/4/2016 10:42:00 AM


Summary 259

1 2 3 4 5 6 7 8 9 10 11 12 13 14 15 16 17 18

H H He
+1 +1 0
1 −1 −1

Li Be B C N O F Ne
+1 +2 +3 −4 −3 −2 −1 0
2 −2 +3
+2 +5
+4

Na Mg Al Si P S Cl Ar
+1 +2 +3 +4 −3 −2 −1 0
3 +3 +2 +1
+5 +4 +3
+6 +5
+7
K Ca Sc Ti V Cr Mn Fe Co Ni Cu Zn Ga Ge As Se Br Kr
Periods

+1 +2 +3 +2 +2 +2 +2 +2 +2 +2 +1 +2 +3 −4 −3 −2 −1 +2
4 +4 +4 +3 +4 +3 +3 +2 +2 +3 +4 +1
+5 +6 +7 +4 +5 +6 +5

Rb Sr Y Zr Nb Mo Tc Ru Rh Pd Ag Cd In Sn Sb Te I Xe
+1 +2 +3 +4 +3 +2 +4 +2 +1 +2 +1 +2 +1 +2 −3 −2 −1 +2
5 +4 +3 +7 +3 +3 +4 +3 +4 +3 +4 +1 +4
+5 +4 +4 +5 +6 +5 +6
+5 +7 +8
+6
Cs Ba La Hf Ta W Re Os Ir Pt Au Hg Tl Pb Bi Po At Rn
+1 +2 +3 +4 +3 +2 +3 +4 +3 +2 +1 +1 +1 +2 +3 +2 −1 0
+4 +3 +4 +4 +3 +2 +3 +4 +5 +4 +1
6 +5 +4 +7 +3
+5 +5
+6 +7
Fr Ra
+1 +2
7

(d) Fractional oxidation number


Certain compounds have fractional oxidation number. The element for which fractional oxidation number is observed is present
in different oxidation states. For example, Fe in Fe3O4 has an oxidation number 8/3.
Fractional oxidation states actually represent the average oxidation states of several atoms in a structure.
(e) Limitations of concept of oxidation number
The definitions of oxidation and reduction are under constant change depending on the new discoveries and insights into the
redox processes. Apart from transfer of electrons, the reactions are now viewed in terms of electron densities wherein, oxidation
involves decrease in electron density while reduction involves an increase in it.

8. Types of redox reactions

Type Definition General reaction Example


Combination reac- Combining of two or A + B → AB , either A or 0 0 +1 −3
tions more reactants to form a B or both must be in the 3Li(s)+ 21 N2 (g) → Li3 N(s)
single product. elemental form. 0 0 +4 − 2

C(s) + O2 (g) 
→ CO2 (g)
Decomposition Breakdown of reactants AB → A + B +1+5 −2 +1 −1 0

reactions into two or more prod- 2 K Cl O 3 (s) 
→ 2 K Cl(s) + 3O2 (g)
At least one of the com-
ucts. ponents formed should
+1 −2

0 0
2 H 2 O 2 (l) 
→ 2H2 (g) + O2 (g)
be in the elemental state.

Displacement One element of the AB + C → AC + B


reactions compound is replaced by Necessary condition for
atom or ion from another the reaction is that C is
element. more reactive than B.
(Continued)

Chapter-8.indd 259 8/4/2016 10:42:02 AM


260 Chapter 8 Redox Reactions

(Continued)
(a) Metal dis- If one metal is more AX + M → MX + A 0 +2 +2 +2 −2 0
placement easily oxidized than Zn + Hg O → Zn O + Hg
Single displacement
another, it can displace reactions. 0 +1 −1 0 +2 −1
the other metal from its Cu(s) + 2AgNO3 (aq) → 2A g(s) + Cu(NO3 )2 (aq)
compounds by a redox
reaction.

(b) Non-metal The non-metal displace- Metals replacing hydro- 0 +1 −2 +1 −1 0


displacement ment reactions involve gen from cold water. 2K (s) + 2H2 O → 2KO H (aq) + H2 (g)
displacement of hydro-
gen and in some cases Metals replacing hydro- 0 +1 −2 +3 −2 0
oxygen. gen from steam. 2Fe(s) + 3H2 O → 2Fe2 O 3 (s) + 3H2 (g)
Metals replacing hydro- 0 +1 −1 +2 −1 0
gen from acid. Zn(s) + 2HCl( aq) → ZnCl2 (aq) + H2 (g)

Disproportiona- Redox reactions in 0 −3 +1


tion reactions which an element in a P 4 (s) + 3NaOH(aq) + 3H2O(l) → P H3 (g) + 3NaH2 PO2
compound is simultane-
ously oxidized as well as
reduced.

9. Balancing of redox reactions


(a) Oxidation number method
This method focuses on the atoms of the elements undergoing a change in oxidation state. The methodology is as follows:
(i) Represent each reactant and product with correct formula.

HNO3 (aq) + H2S(aq) → NO(g) + S(s) + H2O(l)

(ii) Identify the atoms whose oxidation states have changed.


+5 −2 +2 0
HNO3 + H2 S → N O + S + H2O

(iii) Find the increase (or decrease) in the oxidation number for the number of atoms involved.
(iv) Draw a bridge between the same atoms whose oxidation states have changed, indicating the electrons gained or lost. This
corresponds to the change in oxidation number of atom.
+3e−

+5 +2
HNO3 + H2S NO + S + H2
−2 0
−2e−

(v) Multiply the molecule, atoms or ions oxidized or reduced by an appropriate number so as to make the increase in the oxida-
tion number of the atom(s) oxidized equal to the decrease in the oxidation number of the atom(s) reduced.

+3e− × 2 = + 6e−

2HNO3 + 3H2S 2NO + 3S + H2O

−2e− × 3 = + 6e−

(vi) Balance the rest of the equation by inspection.


The final balanced equation is
2HNO3 + 3H2S → 2NO + 3S + 4H2O
(b) Half-reaction (ion electron) method
(i) Write the skeleton equation, which shows only the ions/molecules involved in the redox changes.

Chapter-8.indd 260 8/4/2016 10:42:06 AM


Summary 261

Cr2O72 − (aq) + Fe2 + (aq) → Cr 3+ (aq) + Fe3+ (aq)


(ii) Divide the skeleton equation into half-reactions.
Cr2O72 − (aq) → Cr 3+ (aq)

Fe2 + (aq) → Fe3+ (aq)

(iii) Balance atoms other than H and O.

Cr2O72 − (aq) → 2Cr 3+ (aq)


Fe2 + (aq) → Fe3+ (aq)
(iv) Balance oxygen by adding H2O to the side that needs O.
Cr2O72 − → 2Cr 3+ + 7H2O
Fe2 + (aq) → Fe3+ (aq)

(v) Balance hydrogen by adding H+ to the side that needs H.


14H+ + Cr2O72− → 2Cr 3+ + 7H2O
Fe2 + → Fe3+
(vi) Balance the charge by adding electrons.
14H+ + Cr2O72 − → 2Cr 3+ + 7H2O
    
Net charge = (14 + ) + (2 − ) = 12+ Net charge = 2(3+ ) + 0 = 6 +

6e − + 14H+ + Cr2O72 − → 2Cr 3+ + 7H2O


(vii) Make the number of electrons gained equal to the number lost and then add the two half-reactions.

6e − + 14H+ + Cr2O72 − → 2Cr 3+ + 7H2O


Fe2+ → Fe3+ + e −

6e − + 14H+ + Cr2O72 − → 2Cr 3+ + 7H2O


Fe2+ → Fe3+ + e − ] × 6
Sum: 6e − + 14H+ + Cr2O72 − + 6Fe2+ → 2Cr 3+ + 7H2O + 6Fe3+ + 6e −
(viii) Cancel anything that is the same on both sides: This is the final step. Six electrons cancel from both sides to give the final
balanced equation.
14H+ + Cr2O72 − + 6Fe2+ → 2Cr 3+ + 7H2O + 6Fe3+
On following Steps (i)–(viii) for acidic solutions, if we get

2H+ + 3SO23 − + 2MnO 4− → 3SO24− + 2MnO2 + H2O


Then, conversion of this equation to one appropriate for a basic solution proceeds as follows.
The additional steps in the half-reaction method for basic solutions are as follows:
(ix) Add to both sides of the equation the same number of OH− as there are H+.

2OH− + 2H+ + 3SO23 − + 2MnO 4− → 3SO24− + 2MnO2 + H2O + 2OH−


(x) Combine H+ and OH− to form H2O.

2OH + 2H + + 3SO23 − + 2MnO 4− → 3SO24− + 2MnO2 + H2O + 2OH−


2H2O + 3SO23 − + 2MnO 4− → 3SO24− + 2MnO2 + H2O + 2OH−

(xi) Cancel any H2O that you can.


H2O + 3SO23 − + 2MnO 4− → 3SO24− + 2MnO2 + 2OH−

Chapter-8.indd 261 8/4/2016 10:42:09 AM


262 Chapter 8 Redox Reactions

10. Applications of redox reactions


(a) In industrial manufacture of chemicals.
(b) In metallurgy to reduce metal oxides to metals by suitable reducing agents through redox reactions.
(c) In photosynthesis: The photosynthesis reaction that converts carbon dioxide and water in green plants in the presence of chlo-
rophyll and sunlight to glucose and oxygen is a redox reaction.
Chlorophyll
6CO2 (g) + 6H2O(l) 
→ C6H12O6 (aq) + 6O2 (g)
Sunlight
(d) In combustion of fuels.
(e) The electrode processes in batteries or electrochemical cells take place through redox reactions, between metal electrodes and
metal salts as electrolytes.
(f) In quantitative estimations: Redox reactions form the basis of the titrations carried out for quantitative analysis of different
species in solution.
11. Redox titrations
A redox titration involves a redox reaction between the titrant and the analyte, either one of which can act as an oxidant or a reduct-
ant. These redox titrations are used to determine the strength of the oxidizing or reducing agent.
(a) Redox indicators
There are no simple indicators that can be used to conveniently detect the end points in redox titrations, so detection of end
points in such titrations is based on colour changes among the reactants themselves. Some examples of indicators used in
redox titrations are listed below

Indicator Property Example


Potassium permanganate, It serves as a good redox indicator as MnO 4− In the redox reaction with oxalate or ferrous ions,
KMnO4 (self-indicator) ion has an intense purple colour and its as the KMnO4 solution is added, the purple colour
reduction product in acidic solution (Mn2+ continues to disappear as long as there is any
ion) is the almost colourless. reducing agent is left in the solution. After the last
trace of the reducing agent has been consumed,
the MnO 4− ion in the next drop of titrant turns the
solution pink indicating end point.

Diphenylamine Used in conjunction with potassium dichro- When the equivalence point of the reaction is
(An external indicator) mate which is a strong oxidizing agent, but reached and all the reactant is consumed, the next
is not a self-indicator. drop of potassium dichromate added produces
intense blue colour with diphenylamine and marks
the end point of the titration.
Starch In iodometric titrations, the amount of iodine lib-
erated is estimated by titrating against a reducing
agent such as sodium thiosulphate which reduces
it to iodide ions (I−). Starch forms an intensely blue
coloured complex with iodine. When all iodine
is reduced to iodide ions, the blue colour of the
solution disappears and marks the end point of
the reaction.

(b) Iodine titrations


(i) Iodometry: Titrations in which iodine liberated during the oxidation of iodide ions by strong oxidizing agent, is titrated
with a reducing agent and the strength of the oxidizing agent is estimated are called iodometric titrations. For example,
estimation of Cu2+ with thiosulphate involving the following reactions:

2Cu2 + + 4I− → Cu2I2 + I2

I2 + 2S2O23 − → S4 O26− + 2I−

In the first step, Cu2+ is reduced to Cu+ by the addition of potassium iodide in excess in the pH range 4.0–5.5. The iodine
liberated is titrated against sodium thiosulphate using starch as indicator. The estimated amount of iodine is stoichiomet-
rically equal to the amount of copper present.
(ii) Iodimetry: Titrations involving the direct titration of iodine with a reducing agent are termed as iodimetric titrations. Examples
of iodimetric titration include the determination of strength of ferrous ions, sulphite ions, arsenite ions and thiosulphate ions.

Chapter-8.indd 262 8/4/2016 10:42:10 AM


Summary 263

12. Stoichiometry of redox reactions and concept of gram equivalents


(a) The first mandatory step is to write a balanced chemical equation for the redox reaction.
The coefficient of the reductant and oxidant will tell us the exact number of moles of each taking part in the redox reaction.
If the stoichiometric coefficients of the reductant and oxidant are n1 and n2, respectively, then in a volumetric estimation, the
molarity equation may be written as
M1V1 M2V2
=
n1 n2
The equation can be used for the calculation of any one variable out of the four, if the other three are known.
(b) The equivalent weights for oxidizing and reducing agents can be found as follows:

Molecular weight
Equivalent weight =
n
where n is the number of electrons lost/gained by 1 mol of oxidizing/reducing agent in their balanced half reaction. Here n is
known as n factor or valence factor.
Some examples are as follows:
Equivalent weights Fe2(SO4)3 as oxidizing agent (COOH)2 as reducing agent
of oxidants and 3+
+ 1e − → Fe2 +
Fe C2O24− → 2CO2 + 2e −
reductants
90
1 mol of Fe2(SO4)3 = 2 mol of Fe2+ n = 2, so Equiv. weight = = 45
So n = 2 for 1 mol of Fe2(SO4)3 and 2
400
Equiv. weight = = 200
2
Equivalent weight KMnO4 as oxidizing agent in acidic medium. Na2S2O3 as reducing agent in acidic medium.
based on medium
MnO 4− + 8H+ + 5e − → Mn2 + + 4H2O 2S2O23 − → S4 O26 − +2e −
n = 2/2 = 1 for 1 mol of Na2S2O3,
158 158
n = 5, so Equiv. weight = = 31.6 so Equiv. weight = = 158
5 1
KMnO4 as oxidizing agent in alkaline medium. Na2S2O3 as reducing agent in alkaline medium.

MnO 4− + e − → MnO 42 − S2O23 − + 10OH− → 2SO24− + 5H2O + 8e −

158 n = 8 for 1 mol of Na2S2O3,


n = 1, so Equiv. weight = = 158 158
1 so Equiv. weight = = 19.75
8
Equivalent weight conc. HNO3 dil. HNO3
based on dilution
NO3− + 2H+ + 1e − → NO2 + H2O NO3− + 4H+ + 3e − → NO + 2H2O
63 63
n = 1, so Equiv. weight = = 63 n = 3, so Equiv. weight = = 21
1 3
Equivalent weight of Mol. wt. of AX in + Mol. wt. of AX in
a substance under- Effective mol. wt. of AX oxidation half-reaction reduction half-reaction
Equivalent weight = =
going dispropor- Number of e − transferred Number of e − transferred
tionation
(a) When number of transferred electrons is (b) When number of transferred electrons is differ-
same: Consider the following reaction ent: Consider the following reaction
H O →H O+O Br2 + OH− → Br − + BrO3− + H2O
2 2 2 2
Oxidation: Br2 + 12OH− → 2BrO3− + 6H2O + 10e −
The half cell reactions are as follows:
Reduction: Br2 + 2e − → 2Br − × 5
Oxidation: H2O2 → O2 + 2H+ + 2e −
So, n = 10 and effective molecular weight = 79.9
Reduction: H2O2 + 2H+ + 2e − → 2H2O + 5 × 79.9 = 479.4. Thus,
So, n = 2 and effective molecular weight = 34 479.4
Equivalent weight = = 47.94
68 10
+ 34 = 68. Thus, equivalent weight = = 34 The factor 10/6 is called the n-factor.
2

Chapter-8.indd 263 8/4/2016 10:42:16 AM


264 Chapter 8 Redox Reactions

(c) n-factor calculation


The n-factor of oxidizing agent or reducing agent is calculated depending upon the change in oxidation state of the species
considered. Some of the cases are discussed as follows:
Condition Example n-factor
When only one atom undergoes either +7 +2 5
H+
reduction or oxidation. MnO 4− 
→ Mn2 +

When only one atom undergoes change +6 × 2 +3 3+ 3


in oxidation state but appears in two Cr2O72 − →
 Cr 3+ + Cr 3+
products with the same oxidation state.
When only one atom undergoes change +7 +2 +6
n-factor of MnO 4− = 11/3
in oxidation state but forms two products 3MnO 4− →
 2Mn2 + + Mn6 +
with different oxidation states because of
either only oxidation or only reduction.
When only one atom undergoes change K 2Cr2O7 + 14HCl → 2KCl + 2CrCl3 + 3Cl2 + 7H2O the n-factor of HCl is 3/7
in oxidation state in two products with
changed oxidation state in one product
and same oxidation state as reactant in
the other product.
When two or more atoms in the salt un- n-factor of FeC2O4 is 3
FeC2O 4 → Fe3+ + 2CO2
dergo change in oxidation state because
of either oxidation or reduction.
When two atoms undergo change in −3 × 2 +6 × 2 0 ×2 +3 × 2 n-factor of (NH4)2Cr2O7 is 6
oxidation state with one undergoing (N H4 )2 Cr 2 O7 →
 N2 + Cr2 O3 + 4H2O
oxidation and other reduction.
When the species undergoes dispropor- 2H2O2 → 2H2O + O2 n-factor of H2O2 either consid-
tionation reaction. ering oxidation or reduction
reaction is same, that is, 2

SOLVED EXAMPLES
Modern Approach to Redox Reactions (c) H2SO3 is the reducing agent because it undergoes
oxidation.
1. Which of the following chemical reactions depicts the oxidiz- (d) H2SO3 is the reducing agent because it undergoes
ing behaviour of H2SO4? reduction.
(a) 2HI + H2SO4 → I2+ SO2+ 2H2O (JEE Main Online 2014)
(b) Ca(OH)2 + H2SO4 → CaSO4 + 2H2O Solution
(c) NaCl + H2SO4 → NaHSO4 + HCl (c)
(d) 2PCl5 + H2SO4 → 2POCl3 + 2HCl + SO2Cl2 Oxidation (2e−)
(AIEEE 2006)
Solution H2SO4 + Sn4+ + H2O Sn2+ + HSO−4 + 3H+
(a) 2HI + H2SO 4 → I2 + SO2 + 2H2O
+4 +6
On left hand side oxidation state of sulphur = 6 Reduction (2e−)
On right hand side oxidation state of sulphur = 4
There is a decrease of oxidation state of sulphur from +6 H2SO3 itself gets oxidized hence acts as a reducing agent
in sulphuric acid to +4 in sulphur dioxide. It means that while Sn4+ being reduced to +2 oxidation state acts as an
H2SO4 is reduced and iodide ions are oxidized to iodine oxidizing agent.
by conc. H2SO4. 3. Excess of KI reacts with CuSO4 solution and then Na2S2O3
solution is added to it. Which of the statements is incorrect for
2. Consider the reaction: this reaction?
4+ 2+ − + (a) Cu2I2 is reduced (b) Evolved I2 is reduced
H2SO3 (aq) + Sn (aq) + H2O(l) → Sn (aq) + HSO 4 (aq) + 3H (aq)
(c) Na2S2O3 is oxidized (d) CuI2 is formed
Which of the following statements is correct? Solution
(a) Sn4+ is the oxidizing agent because it undergoes oxidation.
(a) CuI2 is formed as 2CuSO 4 + 4KI → 2K 2SO 4 + CuI2 + I2
(b) Sn4+ is the reducing agent because it undergoes oxidation. (excess )

Chapter-8.indd 264 8/4/2016 10:42:18 AM


Solved Examples 265

4. The oxidation number of iodine in Ca(IO3)2 is Solution


(a) +7 (b) +5 (c) +2 (d) + 3 (d) The reaction is

Solution Zn(s) + 2HNO3(aq) → Zn2+(s) + 2NO3 (aq) + H2(g)
Oxidation is an increase in oxidation number. Thus, the
(b) Let the oxidation number of iodine be x, then
species being oxidized in the above reaction is Zn.
2 + 2x − 12 = 0 ⇒ x = +5
9. The correct order of compounds in the decreasing order based
5. In which of the following changes, there is transfer of five on the oxidation state of oxygen in them is
electrons? (a) OF2 > HOF > KO2 > Sr(IO3 )2
(a) MnO 4− → Mn2 + (b) CrO24− → Cr 3+ (b) OF2 > KO2 > HOF > Sr(IO3 )2
(c) MnO24− → MnO2 (d) Cr2O72 − → 2Cr 3+ (c) HOF > KO2 > Sr(IO3 )2 > OF2
(d) KO2 > OF2 > HOF > Sr(IO3 )2
Solution
(a) For the reaction in option (a), the number of electrons Solution
involved is 5. (a) Oxidation state of oxygen in OF2 = +2
+7 Oxidation state of oxygen in HOF = 0
MnO 4− + 5e − → Mn2 + Oxidation state of oxygen in KO2 = –1/2
Oxidation state of oxygen in Sr(IO3)2 = −2
For the reaction on option (b), number of electrons So, the order is OF2 > HOF > KO2 > Sr(IO3 )2
involved is 3.
+6
10. In the reaction, Na2S2O3 + I2 → Na2S4O6 + NaI (not balanced),
Cr O24− + 3e − → Cr 3+
which of the following is/are true?
For the reaction in option (c), number of electrons involved (I) Na2S2O3 is a reducing agent.
is 2. (II) Iodine is an oxidizing agent.
+6 +4 (III) It is an example of intermolecular redox reaction.
MnO24− + 2e − → MnO2
(IV) In Na2S4O6, the average oxidation state of S is (+5/2).
For the reaction in option (d), number of electrons involved (a) I, II, IV (b) I, II
is 3. (c) I, II, III (d) I, II, III, IV
+6
Cr 2 O72 − + 3e − → 2Cr 3+ Solution
+2 +2.5
6. The following reaction describes the production of metallic (d) S → S ; I0 → I−1
iron: The oxidation number of sulphur increases from +2 to
2Fe2O3 + 3C → 4Fe + 3CO2 +2.5 thus it undergoes oxidation and acts as a reducing
agent while the oxidation number of iodine decreases
How many electrons are lost by each carbon atom in this
from 0 to −1 hence undergoes reduction and acts as an
reaction?
oxidizing agent.
(a) 1 (b) 2 (c) 3 (d) 4
Solution Types of Redox Reactions
(d) In the given reaction, there is a change involved in 11. What products are expected from the disproportionation
the number of electrons of C atom from 0 to 4 which reaction of hypochlorous acid?
indicates the oxidation of C atom. (a) HClO3 and Cl2O
7. Consider the redox equation below, (b) HClO2 and HClO4
(c) HCl and Cl2O
2 MnO 4− + 5H2SO3 → 2Mn2+ + 3H2O + 5SO24− + 4H+ (d) HCl and HClO3
The reducing agent is (AIEEE 2006)
(a) H2SO3 (b) MnO4− (c) H+ (d) SO2− Solution
4
(d) Exchange of chlorine atoms of reactants occurs during
Solution
the reaction.
(a) In the above reaction the reducing agent is H2SO3 as the
oxidation number of sulphur increases from +4 to +6. Thus, 2ClO2 + HOCl + H2O → 3H+ + 2ClO3− + 3Cl−
H2SO3 is getting oxidized and act as reducing agent. 12. The metals that can undergo disproportionation are
8. Zinc metal reacts with nitric acid solution to produce zinc (i) Sn (ii) Na (iii) Cu (iv) Ca
nitrate (in solution), ammonium nitrate (in solution), and water. (a) (i) and (ii) (b) (i) and (iii)
The species being oxidized in this reaction is (c) (iii) and (iv) (d) (ii) and (iii)
(a) HNO3(aq) (b) NH+4 (aq) Solution
2+
(c) Zn (aq) (d) Zn(s) (b) Sn and Cu can undergo disproportionation reaction.

Chapter-8.indd 265 8/4/2016 10:42:22 AM


266 Chapter 8 Redox Reactions

For Cu : Cu+ → Cu2 + + e − For Sn : Sn2 + → Sn4 + + 2e − Cr2O72 − + 14H+ + 3e − → 2Cr 3+ + 7H2O] × 5
Cu+ + e − → Cu Sn2 + + 2e − → Sn I2 + 6H2O → 2IO3− + 5e − + 12H+ ] × 3
Overall 2Cu+ → Cu2 + + Cu Overall 2Sn2 + → Sn4 + + Sn
5Cr2O72 − + 3I2 + 34H+ → 10Cr 3+ + 6IO3− + 17H2O

Balancing of Ionic Redox Reactions 16. Complete the balancing of the following half reaction, taking
place in basic media,
13. Consider the following reaction
Br − (aq) → BrO3− (aq)
xMnO 4− + yC2O24− + zH+ → xMn2 + + 2 yCO2 + 2z H2O
How many electrons are needed to balance it?
The values of x, y and z in the reaction are, respectively (a) 2 electrons, left side (b) 2 electrons, right side
(a) 2, 5 and 8 (b) 2, 5 and 16 (c) 4 electrons, right side (d) 6 electrons, right side
(c) 5, 2 and 8 (d) 5, 2 and 16 Solution
(JEE Main 2013)
Solution (d) The balanced equation in basic media is
(b) The balanced equation is Br − + 6OH − → BrO3− + 3H2O + 6e −
Thus, six electrons are required on the right hand side to
2MnO 4− + 5C2O24 − + 16H+ → 2Mn2 + + 10CO2 + 8H2O balance the reaction.
Hence, x = 2, y = 5, z = 16 17. After balancing of the equation for the reaction,
14. How many electrons are involved in the following redox Sn2 + + H+ + TcO 4− → Tc 4+ + H2O+Sn4 +
reaction?
3+ 3+
what is the sum of ALL the coefficients in the equation?
Cr2O72 − + Fe2 + + C2O24− → Cr + Fe + CO2 (Unbalanced) (a) 6 (b) 22 (c) 30 (d) 34
(a) 3 (b) 4 (c) 6 (d) 5 Solution
(JEE Main Online 2014)
Solution (d) TcO 4− − Sn2 + → Tc 4+ + Sn4 +
First, we break the reaction above into half-reactions:
(c)
Reduction (3e−) TcO 4− → Tc4+
Sn2+ → Sn4+
Cr2O27− + Fe2+ + C2O24− Cr3+ + Fe3+ + CO2 Each half-reaction is already balanced with respect to
+6 +2 +3 +3 +3 +4 atoms other than O and H, so next we balance the O
1e− atoms by using water:

1e− TcO 4− → Tc4+ + 4H2O


2 × 3e− = 6e− Sn2+ → Sn4+
Now we balance H by using H+:
Cr2O27− + Fe2+ + C2O24− 2 Cr3+ + Fe3+ + 2CO2
8H+ + TcO 4− → Tc4+ + 4H2O
2× 1e− = 2e− Sn2+ → Sn4+
2 × 1e− × 2 Next, we add electrons to balance the charges on both
The balanced reaction is sides of the equations:

Cr2O72 − + 2Fe2 + + 2C2O24− → 2Cr 3+ + 2Fe3+ + 4CO2 3e– + 8H+ + TcO 4− → Tc4+ + 4H2O
Sn2+ → Sn4+ + 2e–
Total 6e− are involved in given redox reaction.
We multiply the equations so that the electrons gained
15. Consider the following redox reaction occurring in acidic equals the electrons lost,
solution:
2(3e– + 8H+ + TcO 4− → Tc4+ + 4H2O)
Cr2O72 − + I2 → IO3− + Cr 3+ 3(Sn2+ → Sn4+ + 2e–)
On balancing the above equation, the coefficients of I2 and which gives us:
H+ are, respectively, 6e– + 16H+ + 2TcO 4− → 2Tc4+ + 8H2O
(a) 5 and 16 (b) 3 and 16
(c) 3 and 34 (d) 5 and 34 3Sn2+ → 3Sn4+ + 6e–
Solution Now, by adding the half-reactions back together, we
(c) Writing the half reactions, we have have our balanced equation:

Chapter-8.indd 266 8/4/2016 10:42:27 AM


Solved Examples 267

(a) Mg, Mn, Zn are more reactive than Ag, Cu; but less reac-
3Sn2+ + 16H+ + 2TcO 4− → 2Tc4+ + 8H2O + 3Sn4+
tive than H2.
Therefore, the sum of all the coefficients is = 3 + 16 + 2 + 2 (b) Mg, Mn, Zn are less reactive than Ag, Cu; but more reac-
+ 8 + 3 = 34 tive than H2.
(c) Mg, Mn, Zn are more reactive than Ag, Cu; and more reac-
18. Consider the chemical change which is occurring in basic
tive than H2.
medium: ClO3− + N2H4 → NO + Cl− . The correct statement is
(d) Mg, Mn, Zn are less reactive than Ag, Cu; and less reactive
(a) N2H4 is acting as an oxidizing agent in the reaction.
than H2.
(b) 3 mol of N2H4 are required per mole of ClO3− .
(c) The electrons transferred per mol of N2H4 reduce 1.33 Solution
mol of ClO3− . (c) The increasing reactivity order of given metals is
(d) 5 mol of electrons are gained by per mole ofClO3− . Ag < Cu < Zn < Mn < Mg.
Solution 21. Consider the metals: Mn, Mg, Zn, Ag, Cu. Based on their reac-
(c) The balanced chemical equation is tivity order, pick the correct statement.
(a) All five metals produce hydrogen gas on reacting with
4 ClO3− + 3N2H4 → 4 Cl− + 6NO + 6H2O acids.
Since nitrogen’s oxidation state increases from −2 to +2, (b) Ag will substitute Cu from its aqueous solution.
N2H4 is oxidizing and is working as a reducing agent. (c) Mn will substitute Zn from its aqueous solution.
The equation clearly tells us that four moles of ClO3− is (d) Cu will substitute Mg from its aqueous solution.
reduced by 3 mol of N2H4, so, per mole of ClO3− will require
Solution
(3/4) = 0.75 mol of N2H4 or 1 mol of N2H4 will reduce 4/3
=1.33 mol of ClO3− . (c) The order of reactivity of these metals is Mg > Mn > Zn
The oxidation state of chlorine is changing from +5 to −1, > Cu > Ag. Out of these metals, Cu and Ag are less reactive
and per mole of ClO3− contains one mole of chlorine. So, and their reaction with acid will not produce hydrogen
one mole of ClO3− will gain 6 mol of electrons. gas.
Highly reactive metal can substitute the less reactive
19. Consider the redox reaction in acidic medium: metal from their aqueous solution, so, Mn2+ will substi-
tute Zn2+ from aqueous solution.
Mn2 + + BiO3− → MnO 4− + Bi3+. Choose the correct statement
from the following: Stoichiometry of Redox Reactions
(a) The total electrons involved in the balanced chemical 22. When 1 mol of KMnO4 reacts with HCl, the volume of chlorine
equation is 5. liberated at STP will be
(b) Two moles of electrons are donated by per mole of BiO3− . (a) 11.2 L (b) 22.4 L (c) 44.8 L (d) 56.0 L
(c) 2.8 mol of acid are required for the change from BiO3− to
Bi3+. Solution
(d) Acid is used as reducing agent in this reaction. (d) The reactions involved are
Solution [2Cl− → Cl2 + 2e − ] × 5
(c) In this reaction, BiO3− is reduced by accepting electrons [MnO 4− + 8H+ + 5e − → Mn+2 + 4H2O] × 2
which are donated by Mn2+ that is getting oxidized. 10 Cl− + 2MnO 4− + 16H+ → 5Cl2 + 2Mn+2 + 8H2O
Balancing the redox equation, we get:
Hence, 2 mol of KMnO4 liberates 5 mol of Cl2 or volume of
[Mn2 + + 4H2O → MnO 4− + 8H+ + 5e − ] × 2 Cl2 = 5 × 22.4 L at STP
1 mol of KMnO4 will liberates volume of Cl2 = 5 × 22.4/2
[BiO3− + 6H+ + 2e − → Bi3+ + 3H2O] × 5
= 56.0 L at STP
2Mn2 + + 5BiO3− + 14H+ → 2MnO 4− + 5Bi3+ + 7H2O 23. How many mL of 0.05 M KMnO4 is required to oxidize 2.0 g of
FeSO4 in dilute solution?
From the balanced equation, it is clear that total of 10 (a) 52.63 mL (b) 52.96 mL
electrons are involved in the reaction, and 2.8 mol of acid (c) 67.63 mL (d) 70.96 mL
are required by per mole of BiO3− .
Solution
Applications of Redox Reactions (a) The redox reactions are
MnO 4− + 8H+ + 5e − → Mn2 + + 4H2O
20. In a series of chemical test reactions the following observations
were made when several metallic element samples were treated Fe2 + → Fe3+ + e −
with 3.00 M hydrochloric acid solution. Zn: metal dissolves, Therefore, the overall reaction is
hydrogen gas is emitted. Cu: no reaction. Ag: no reaction. Mg:
metal dissolves, hydrogen gas is emitted. Mn: metal dissolves, MnO 4− + 8H+ + 5Fe2 + → Mn2 + + 4H2O + 5Fe3+
hydrogen gas is emitted. Which of the following statements is Now, by the relation, M1V1 = M2V2 we have
true? n1 n2

Chapter-8.indd 267 8/4/2016 10:42:32 AM


268 Chapter 8 Redox Reactions

2 Na2SO3 was required to reduce the CrO24− to CrO2− in a basic


5 × 0.05 × V = × 1000
152 solution, with SO2− 2−
3 getting oxidized to SO 4 . Then which
Solving, we get V = 52.63 mL. of the following statement is incorrect?
(a) The percentage of chromium in alloy is 23.9%.
24. The number of electrons involved in the reduction of nitrate
(b) Oxidation state of chromium changes from +6 to +4.
ion (NO3− ) to hydrazine (N2H4) is
(c) 1.5 moles of SO2− 2−
3 are consumed per mole of CrO 4 .
(a) 8 (b) 7 (c) 5 (d) 3
(d) One mole of base is involved to convert per mole of
Solution CrO2−
4 in this redox reaction.
(b) The reaction is 2NO3− + 7e − + 10H+ → N2H4 + 3H2O Solution
25. NaIO3 reacts with NaHSO3 according to equation: (b) The reaction involved is
IO3− + 3HSO3− −
→ I + 3H +
+ 3SO24−
. The weight of NaHSO3 2CrO24− + 3SO23 − + H2O → 2CrO2− + 3SO24− + 2OH−
required to react with 100 mL of solution containing 0.66 g
In the reaction, the oxidation state of chromium changes
of NaIO3 is
from +6 to +3 and sulphur from +4 to +6. Also 1.5 mol of
(a) 5.2 g (b) 4.57 g (c) 2.3 g (d) 1.04 g
SO2− 2−
3 are oxidized per mol of CrO 4 and one mole of
Solution base is used per mole of CrO2− 4 in the redox reaction.
(d) In the reaction Also,

I5+ + 6e − → I−  1 mol Na2SO3   1 mol SO23 − 


mol CrO24− = (3.0 g Na2SO3 ) ×  × 
S 6 + + 2e − → S 4 +  126 g Na2SO3   1 mol Na2SO3 

Therefore,  2 mol CrO24− 


× 2− 
MequivNaHSO3 = MequivNaIO3  3 mol SO3 

N1V1(NaHSO3 ) = N2V2 (NaIO3 ) = 1.58 × 10 −2 mol CrO24−


w 0.66 Since there is one mole of Cr in each mole of CrO2− 4 ,
× 100 = × 100
104/2 198/6 then the above number of moles CrO2− 4 is also equal to
w = 1.04 g the number of moles of Cr that were present:
0.0158 mol Cr × 52.00 g mol−1 = 0.825 g Cr in the original
− alloy.
26. The n-factor of the reduction of nitrate ion (NO3 ) to hydrazine % of Cr = (0.825 g Cr/3.450 g sample) × 100% = 23.9% Cr
(N2H4) is Alternately,
(a) 8 (b) 7 (c) 5 (d) 3 2−
gram equiv of CrO2−4 = gram equiv of SO3
Solution 3
⇒ (Moles of Cr or CrO24− ) × 3 =
(b) The reaction involved is 126/2
+5 −2
2 NO3− + 7e − + 16H+ → N2 H4 + 6H2O 1
⇒ Moles of Cr =
63
The n-factor for reducing agent is total net change in oxi- 52/63
dation number per formula unit, so n-factor = 7. Therefore, % of Cr in alloy = × 100 = 23.92%
3.45
27. How many grams of aluminium must react to displace all the
silver from 25 g of silver nitrate? [At. wt. of Ag = 108 and Al = 27] 29. If 2.68 × 10–3 mol of a solution containing an ion An+ requires
(a) 1.32 g (b) 2.08 g (c) 3.97 g (d) 1.98 g 1.61 × 10–3 mol of MnO 4− for the oxidation of An+ to AO3− in
acid medium, what is the value of n?
Solution
(a) 3 (b) 5 (c) 2 (d) 4
+ 3+
(a) The reaction is 3Ag + Al → Al + 3Ag Solution
25 (c) Since in acidic medium, An+, is oxidized to AO3−, the
Moles of silver nitrate = mol change in oxidation state from (+5) to (+n) = 5 − n
170
Total number of electrons that have been given out
1 25 during oxidation of 2.68 × 10−3 mol of An+ = 2.68 × 10–3
Therefore, moles of Al required = × mol
3 170 × (5 – n)
1 25 Thus, the number of electrons added to reduce
Mass of Al required = × × 27 = 1.32 g 1.61 × 10−3 mol of MnO 4− to Mn2+, that is, (+7) to (+2) =
3 170
1.61 × 10−3 × 5
Therefore,
28. A sample of a chromium-containing alloy weighing 3.45 g
was dissolved in acid, and all the chromium in the sam- 1.61 × 10–3 × 5 = 2.68 × 10–3 × (5 − n)
ple was oxidized to CrO2−
4 . It was then found that 3.0 g of
Solving, we get n = 2.

Chapter-8.indd 268 8/4/2016 10:42:39 AM


Advanced Level Problems 269

Redox Reaction Titrations Hence, 5 mol of FeSO4 are completely oxidized by 1 mol
of KMnO4 and 15 mol of FeSO4 are completely oxidized
30. Amount of oxalic acid present in a solution can be deter- 1 × 15
mined by its titration with KMnO4 solution in the presence of by = 3 mol of KMnO4
5
H2SO4. The titration gives unsatisfactory result when carried
33. When iodine is passed through sodium thiosulphate, it is
out in the presence of HCl, because HCl
decolourized and sodium tetrathionate is formed. What is
(a) gets oxidized by oxalic acid to chlorine.
the change in the oxidation number of a sulphur atom in the
(b) furnishes H+ ions in addition to those from oxalic acid.
reaction?
(c) reduces permanganate to Mn2+.
(a) 1/2 (b) 3/2 (c) 2 (d) 1
(d) oxidizes oxalic acid to carbon dioxide and water.
(AIEEE 2008) Solution
Solution (a) The reaction involved is
(c) MnO 4− + C2O24− + H+ → Mn2 + + CO2 + H2O 0 +2 −1 +2.5
I2 + 2 Na2 S2 O3 → 2Na I + Na2 S4 O6
MnO 4− −
+ Cl → Cl2 + Mn 2+
Sodium thiosulphate Sodium tetrathionate
Change in oxidation number of sulphur = 1/2.
MnO 4− gets easily reduced to Mn2+ in presence of HCl,
hence not used. 34. Metallic tin in the presence of HCl is oxidized by K2Cr2O7 to
stannic chloride. What volume of decinormal dichromate
31. A 0.46 g sample of As2O3 required 25.0 mL of KMnO4 solution
solution will be reduced by 1 g of tin? (Atomic weight of
for its titration. The molarity of KMnO4 solution is
Sn = 118.7 u)
(a) 0.016 (b) 0.074 (c) 0.032 (d) 0.128
(a) 336.9 mL (b) 235.6 mL
Solution (c) 465.23 mL (d) 398.56 mL
(b) The reaction involved is Solution
5As2O3 + 4MnO4− + 9H2O + 12H+ → 10HAsO34− + 4Mn2+ (a) Balanced chemical equation is
5 mol of As2O3 reacts with 4 mol of MnO 4− 2K 2Cr2O 7 + 3Sn + 28HCl → 4KCl + 4 CrCl3 + 3SnCl4 + 14H2O
2(2 × 39 + 2 3 × 118.7
Therefore, 0.46/198 mol of As2O3 will reacts with
× 52 + 7 × 16 ) = 356.1 g
4 0.46
× = 0.00186 mol of MnO 4− = 2 × 294 = 588 g
5 198
Now 356.1 g Sn react with K2Cr2O7 = 588 g
0.00186
Hence, molarity of KMnO4 = × 1000 = 0.074 M 588
25 1 g Sn will react with K2Cr2O7 = g = 1.651 g
356.1
32. How many moles of KMnO4 are required in the acidic medium Equivalent weight of K2Cr2O7
for complete oxidation of 15 mol of FeSO4?
Molecular weight 294
(a) 2 (b) 3 (c) 4 (d) 5 = = = 49
6 6
Solution Decinormal K2Cr2O7 solution means 1000 mL solution
(b) The reactions involved are contains 4.9 g K2Cr2O7, that is, 4.9 g K2Cr2O7 are present
in 1000 mL solution.
MnO 4− + 5e − + 8H+ → Mn2 + + 4H2O Thus, 1.651 g K2Cr2O7 will be present in
[Fe2 + → Fe3+ + e − ] × 5 1000
= × 1.651 = 336.9 mL
MnO 4− + 5Fe2 + + 8H+ → Mn2 + + 5Fe3+ + 4H2O 4.9

ADVANCED LEVEL PROBLEMS


1. Consider a titration of potassium dichromate solution with Oxidation number
acidified Mohr’s salt solution using diphenylamine as indica- increases by
tor. The number of moles of Mohr’s salt required per mole of 1 unit (oxidation)
dichromate is
(a) 3 (b) 4 (c) 5 (d) 6 +6
(IIT-JEE 2007) Cr2O72− + Fe2+ Fe3+ + Cr3+

Solution
Oxidation number
(d) The skeletal ionic equation is decreases by
Cr2O7 + Fe2 + → Fe3+ + Cr 3+ 3 unit (reduction)

In Cr2O72−, the decrease in oxidation number is by 3 units


per atom and since 2 atoms of chromium are involved so
the total decrease is 3 × 2 = 6.

Chapter-8.indd 269 8/4/2016 10:42:44 AM


270 Chapter 8 Redox Reactions

Solution
Cr2O7 + 6e − → Cr 3+
(6) 2MnO2 + 4KOH + O2 → 2K 2MnO 4 + 2H2O
Fe2 + → Fe3+ + e − ] × 6 Potassium
manganate

We need to multiply this equation by 6. So the n factor Let the oxidation number of Mn be x, then we have
for Cr2O7 = 6 and for Fe2+ = 1. Thus, to reduce 1 mol of
dichromate, 6 mol of Fe2+ is required. 2( +1) + x + 4( −2) = 0 ⇒ 2 + x − 8 = 0 ⇒ x − 6 = 0 ⇒ x = + 6

2. Match the reaction with the type of reaction/nature of 4. Among the following, the number of elements showing only
product. one non-zero oxidation state is ___.

Column I Column II O, Cl, F, N, P, Sn, Tl, Na, Ti


(IIT-JEE 2010)
(a) O2− → O2 + O22 − (p) Redox reaction Solution
(q) One of the products has (2) F and Na show only one non-zero oxidation state.
+
(b) CrO2−
4 +H → trigonal planar structure. 5. The difference in the oxidation number of the two types of sul-
(r) Dimeric bridged tetrahe- phur atoms in Na2S4O6 is ___.
(c) MnO 4− + NO2− +H →+
dral metal ion. (IIT-JEE 2011)
Solution
(d) NO3− + H2SO 4 + Fe2 + → (s) Disproportionation
(5) S∗ are in 0 oxidation state and S are in +5 oxidation state.
(IIT JEE 2007) Their difference is 5.
Solution O O
(a) → p, s; (b) → r; (c) → p, q; (d) → p +5 ∗ ∗ +5
Na+O− S S S S O−
(a) Consider the reaction O2− → O2 + O22 − 0 0
O O
This is a redox reaction and to be more specific this is a
disproportionation reaction because in this case, both the 6. Reaction of Br2 with Na2CO3 in aqueous solution gives sodium
oxidation number of oxygen (from −1 to 0 and −2) is both bromide and sodium bromate with evolution of CO2 gas. The
increasing as well as decreasing at the same time. number of sodium bromide molecules involved in the bal-
(b) The chromate ion has tetrahedral structure in which four anced chemical equation is ___.
atoms are around chromium atom and are oriented in tet- (IIT-JEE 2011)
rahedral arrangement. Solution
O 2−
(5) The balanced chemical equation is:
Cr
3Na2CO3 + 3Br2 → 5NaBr + NaBrO3 + 3CO2
O O O
CrO4
2− 7. Reaction of the metal centre in aqueous permanganate ion
involves
The dichromate ion consists of two tetrahedral units shar- (a) 3 electrons in neutral medium.
ing one corner with Cr O Cr bond angle of 126°. (b) 5 electrons in neutral medium.
(c) 3 electrons in alkaline medium.
2−
O 179 pm O 163 pm O (d) 5 electrons in acidic medium.
O Cr 126° Cr O (IIT-JEE 2011)
O O Solution
2− (a), (c), (d) In acidic media, permanganate acts as a strong oxi-
Cr2O7 dizing agent:
(c) This reaction is also a redox reaction, since both reduction 8H+ + 5e − + MnO 4− → Mn2 + + 4H2O (i.e. 5 electrons in
and oxidation are involved and also the product formed, acidic media)
that is, NO3− has a trigonal planar structure. In neutral and alkaline media, potassium permanganate
2MnO4− + 5NO2− + 6H+ → Mn2 + + 3H2O + 5NO3− acts as a moderate oxidizing agent:

(d) This reaction is also a redox reaction since both reduction 2H2O + 3e − + MnO −4 → MnO2 + 4 OH− (i.e. 3 electrons in
and oxidation are involved. neutral and alkaline media)

3. The oxidation number of Mn in the product of alkaline oxida- 8. The reaction of white phosphorous with aqueous NaOH gives
tive fusion of MnO2 is __. phosphine along with another phosphorus containing com-
pound. The reaction type; the oxidation states of phosphorus
(IIT-JEE 2009) in phosphine and the other product are, respectively,

Chapter-8.indd 270 8/4/2016 10:42:49 AM


Advanced Level Problems 271

(a) redox reaction; −3 and −5. Solution


(b) redox reaction; +3 and +5. (a) , (b) Fe3+ is reduced to Fe2+ by H2O2/NaOH and Na2O2/H2O.
(c) disproportionation reaction; −3 and +1.
(d) disproportionation reaction; −3 and +3. 2Fe3+ + H2O2 + 2OH− → 2Fe2 + + H2O + O2
(IIT-JEE 2012)
Na2O2 + H2O → H2O2 + NaOH
Solution
(c) According to the reaction 12. Which of the following is not oxidized by O3?
P4 + NaOH + 3H2O → PH3 + 3NaH2PO2 (a) KI (b) KMnO4 (c) K2MnO4 (d) FeSO4
Oxidation state of phosphorus in P4 is zero while in PH3
it is −3 and in NaH2PO2, it is +1. This shows that this is a Solution
type of disproportionation reaction because there is (b) KMnO4 is not oxidized by O3 because Mn in KMnO4 is in
an increase as well as decrease in the oxidation state of its maximum possible oxidation state (i.e. +6). It cannot
phosphorus. increase its oxidation state beyond that.

9. Consider the following list of reagents: 13. When I− is oxidized by MnO 4− in alkaline medium, I− converts
Acidified K2Cr2O7, alkaline KMnO4, CuSO4, H2O2, Cl2, O3, FeCl3, into
HNO3, and Na2S2O3. (a) IO3− (b) I2 (c) IO 4− (d) IO −
The total number of reagents that can oxidize aqueous iodide Solution
to iodine is _____. (a) The reactions involved are
(JEE Advanced 2014)
Solution 2KMnO 4 + 2KOH → 2K 2MnO 4 + H2O + O
(7) The reactions involved are 2K 2MnO 4 + 2H2O → 2MnO2 + 4KOH + 2O
2KMnO 4 + H2O → 2MnO2 + 2KOH + 3[O]
K2Cr2O7 + I− + H+ → I2 CuSO4 + I− → I2
KI + [O] → KIO3
H2O2 + I− → I2 Cl2 + I− → I2
2KMnO 4 + KI + H2O → 2KOH + 2MnO2 + KIO3
FeCl3 + I− → I2 HNO3 + I− → I2
O3 + I−→ I2 14. In the standardization of Na2S2O3 using K2Cr2O7 by iodome-
try, the equivalent mass of K2Cr2O7 is
10. Hydrogen peroxide in its reaction with KIO4 and NH2OH (a) (molar mass)/2 (b) (molar mass)/6
respectively, is acting as a (c) (molar mass)/3 (d) same as molar mass
(a) Reducing agent, oxidizing agent. Solution
(b) Reducing agent, reducing agent.
(c) Oxidizing agent, oxidizing agent. (b) As this is a redox titration involving gain of electrons,
(d) Oxidizing agent, reducing agent. equivalent mass = (molar mass)/number of electrons
(JEE Advanced 2014) involved = (molar mass)/6. The reaction is
Solution
Cr2O72− + 14H+ + 6e − → 2Cr 3+ + 7H2O
(a) The reactions involved are
+7 +5 15. 0.5 g of sample containing MnO2 is treated with HCl, liberat-
H2O2 + K I O 4 → K I O3 + H2O + O2 ing Cl2. The Cl2 is passed into a solution of KI and 30.0 cm3
of 0.1 M Na2S2O3 which is required to titrate the liberated
In above reaction, the oxidation state of I decreases from iodine. Calculate the percentage of MnO2 in the sample
+7 to +5. Therefore, KIO4 gets reduced and H2O2 acts as (atomic weight of Mn = 55 u).
reducing agent. (a) 19.63% (b) 26.1% (c) 32.5% (d) 16.98%
−1 +3 Solution
H2O2 + NH2OH → H2O + N 2 O3
(b) The reaction involved is
In the above reaction, the oxidation state of N increases
HCl KI Na S O
from −1 to +3. Therefore, NH2OH gets oxidized and H2O2 MnO2 → Cl2 
→ I2 
2 2 2
→ NaI + Na2S4 O6
acts as oxidizing agent.
Redox changes involved are
11. Fe3+ is reduced to Fe2+ by using
(a) H2O2 is presence of NaOH. 2e − + I20 → 2I−
(b) Na2O2 is water. +
(c) H2O2 in presence of H2SO4. 2S22 + → S(45 / 2 ) + 2e −
(d) Na2O2 in presence of H2SO4.
(JEE Advanced 2015) Mn4 + + 2e − → Mn2 +

Chapter-8.indd 271 8/4/2016 10:42:52 AM


272 Chapter 8 Redox Reactions

The reactions suggest that Milliequiv. of MnO2 = 0.0320 M × 0.02344 L = 7.50 × 10−4 mol e–
Milliequiv. of Cl2 formed = Milliequiv. of I2 liberated =
The ratio of moles of electrons to moles of Ce4+ reactant
Milliequiv. of Na2S2O3 used
is, therefore, 2:1, and we conclude that the product is
Ce2+.
w
Therefore, × 1000 = 0.1 × 1 × 30
M/2 17. A 1.362 g sample of an iron ore that obtained Fe3O4 was dis-
solved in acid and all the iron was reduced to Fe2+. The solu-
(as NNa2 S2 O3 = MNa2 S2 O3 since valence factor = 1, see tion was then acidified with sulphuric acid and titrated with
39.42 ml of 0.0281 M KMnO4, which oxidized Fe2+ to Fe3+. The
redox changes for Na2S2O3)
percentage of Fe3O4 in the sample is
(a) 31.45% (b) 94.3% (c) 65 % (d) 22.7%
0.1 × 1 × 30 × M 0.1 × 1 × 30 × 87
or wMnO2 = = = 0.1305 g Solution
2000 2000
(a) In sample, iron is in Fe2+ and Fe3+ state, but all iron is con-
(as MMnO2 = 87 g mol−1 ) verted into Fe2+ before reaction with KMnO4.
Hence, percentage purity of MnO2 is
Fe2+ + Fe3+ → Fe2+
0.1305 Since iron is oxidized in the presence of acidic KMnO4,
Percentage of MnO2 = × 100 = 26.1%
0.5 the balanced equation for this redox reaction is
5Fe2+ + MnO 4− + 8H+ → 5Fe3+ + Mn2 + + 4H2O
16. A solution contains Ce(SO 4 )2−
3 at a concentration of 0.0150 M.
It was found that in a titration, 25.00 mL of this solu- 1 mol MnO 4− = 5 mol Fe2+ . Therefore,
tion reacted completely with 23.44 mL of 0.032 M FeSO4
solution. The reaction gave Fe3+ as a product in the solu- 39.42 × 0.0281
tion. In this reaction, what is the final oxidation state of Mole of iron in sample = × 5 = 5.5 × 10 −3 mol
1000
the Ce?
(a) +1 (b) +2 (c) +7 (d) +4 as 3 mol of Fe is present in 1 mol of Fe3O4. Hence,
Solution 5.5 × 10 −3
Moles of Fe3O 4 in sample = = 1.83 × 10 −3 mol
(b) The oxidation state of cerium in the reactant ion is +4. 3
The number of moles of this ion in the reactant solution
is Mass of Fe3O 4 in sample = 1.83 × 10 −3 × 232 = 0.43 g

0.0150 M × 0.02500 L = 3.75 × 10−4 mol Ce4+ 0.43


or % Fe3O 4 = × 100 = 31.5%
1.362
The number of moles of electrons that come from the
Fe2+ reducing agent is

PRACTICE EXERCISE
Level I (c) S2O24− < S2O26 − < SO23 − (d) S2O26 − < S2O24− < SO23 −
Single Correct Choice Type 5. In the reaction, C4H10(l) + Cr2O 27− (aq) + H+(aq) → H6C4O4(s)
1. The oxidation number of carbon in CH2Cl2 is + Cr3+(aq) + H2O(l) the change in the oxidation number of the
(a) −4 (b) +4 (c) 0 (d) −2 chromium atom is
(a) a decrease by six units (b) a decrease by three units
2. SO2 under atmospheric condition changes to SO2− x . If oxida- (c) an increase by three units (d) an increase by five units
tion number of S in is +6, what is the value of x in SO2−x ?
(a) 1 (b) 2 (c) 3 (d) 4 6. Which of the following is not a reducing agent?
(a) SO2 (b) H2O2 (c) CO2 (d) NO2
3. Equivalent weight of H3PO2 (molecular weight = M) when it
disproportionates into PH3 and H3PO3 is 7. On the basis of structure, the oxidation of two Cl atoms in
(a) M (b) M/2 (c) M/4 (d) 3M/4 CaOCl2, respectively, are
(a) −1 and +1. (b) +2 and −2.
4. The oxidation states of sulphur in the anions (c) −2 and +2. (d) −1 and +3.
SO23 − , S2O24− and S2O26 − follow the order 8. In the reaction 4I− + Hg2 + → HgI24− , 1 mol each of Hg2+ and
I− will form:
(a) S2O24− < SO23 − < S2O26 − (b) SO23 − < S2O24− < S2O26 − (a) 1 mol (b) 0.5 mol (c) 0.25 mol (d) 2 mol

Chapter-8.indd 272 8/4/2016 10:42:57 AM


Practice Exercise 273

9. After balancing of the equation for the reaction, 22. In the alkaline medium, the colour of potassium dichromate
solution changes from orange to yellow due to the formation
H+ + Cr2O72 − + Fe2 + → Cr 3+ + H2O + Fe3+ of potassium chromate. What is the change in oxidation state
of chromium?
what is the sum of ALL the coefficients in the equation? (a) 1 (b) 2 (c) 3 (d) 0
(a) 34 (b) 38 (c) 39 (d) 36
23. Amongst the following, identify the species with an atom in
10. White P reacts with caustic soda; the products are PH3 and +6 oxidation state.
NaH2PO2. This reaction is an example of (a) MnO 4− (b) Cr(CN)3−
6 (c) NiF62− (d) CrO2Cl2
(a) oxidation. (b) reduction.
(c) disproportionation. (d) neutralization. 24. Which of the following behaves as both oxidizing and reduc-
ing agents?
11. The reaction 3ClO − (aq) → ClO3− (aq) + 2Cl− (aq) is an example of (a) H2SO4 (b) SO2 (c) H2S (d) HNO3
(a) oxidation reaction.
25. Which oxyacid of Cl cannot undergo disproportionation?
(b) reduction reaction.
(a) HClO (b) HClO2 (c) HClO3 (d) HClO4
(c) disproportionation reaction.
(d) decomposition reaction. 26. Which of these will not be oxidized by ozone?
(a) KI (b) FeSO4 (c) KMnO4 (d) K2MnO4
12. When SO2 is passed through a solution of potassium iodate,
the oxidation state of iodine changes from 27. Among the following molecules, in which does bromine show
(a) +5 to 0 (b) +5 to −1 the maximum oxidation number?
(c) −5 to 0 (d) −7 to −1 (a) Hg2(BrO3)2 (b) BrCl
(c) KBrO4 (d) Br2
13. When copper is treated with a certain concentration of nitric
acid, nitric oxide, and nitrogen dioxide is liberated in equal 28. How many moles of KMnO4 are required in the acidic medium
volumes according to the equation, for complete oxidation of 15 mol of FeSO4?
xCu + yHNO3 → Cu(NO3 )2 + NO + NO2 + H2O (a) 2 (b) 3 (c) 4 (d) 5

The coefficients of x and y are 29. A sample of 2.5 mol of hydrazine (N2H4) loses 25 mol of elec-
(a) 2 and 3. (b) 2 and 6. (c) 1 and 3. (d) 3 and 8. trons on being converted to a new compound X. Assuming
that there is no loss of nitrogen in the formation of the new
14. Which one of the following is an example of dispropor- compound, what is the oxidation number of nitrogen in com-
tionation? pound X?
(a) NH3 + 3CuO → 3Cu + 3H2O + N2 (a) −1 (b) −2 (c) +3 (d) +4
(b) 5HI + HIO3 → 3H2O + 3I2
(c) I2 + 2Na2S2O3 → 2NaI + Na2S4O6 30. Which reaction involves neither oxidation nor reduction?
(d) P4 + 3NaOH + 3H2O → 3NaH2PO2 + PH3
(a) CrO24− → Cr2O72 − (b) Cr → CrCl3
15. The equivalent weight of phosphoric acid (H3PO4) in the reac- (c) Na → Na +
(d) 2S2O23 − → S4 O26 −
tion NaOH + H3PO4 → NaH2PO4 + H2O is
(a) 59 (b) 49 (c) 25 (d) 98
31. A compound contains atoms of three elements A, B, and C. If
16. Which of the following agents is the most reducing? the oxidation number of A is +2, B is +5 and that of C is −2, the
(a) Mg (b) Na (c) K (d) Br2 possible formula of the compound is
(a) A3(BC4)2 (b) A3(B4C)2
17. The average oxidation number of sulphur in Na2S4O6 is (c) ABC2 (d) A3(BC3)2
(a) 1.5 (b) 2.5 (c) 3 (d) 2
32. No disproportionation is possible for
18. Which of the following compounds contain the metal atom in (a) AsH3 (b) SF4 (c) H5IO6 (d) PCl3
its highest possible oxidation state?
(a) KSCN (b) MnO2 (c) Cr2O3 (d) H2SnCl4 33. Which of the following agents is most oxidizing?
(a) O3 (b) KMnO4 (c) H2O2 (d) K2Cr2O7
19. Number of moles of K2Cr2O7 reduced by 1 mol of Sn2+ is
(a) 1/3 (b) 1/6 (c) 2/3 (d) 1 34. The oxidation state of chlorine in perchloric acid is
(a) −1 (b) 0 (c) −7 (d) +7
20. Which of the following undergoes disproportionation in solution?
(a) Fe2+ (b) Cu+ (c) Cr3+ (d) Ni2+ 35. H2O2 acts as a reducing agent in
(a) FeCl2 + HCl + H2O2 → FeCl3 + H2O
21. The oxidation number of sulphur in S8, S2F2, and H2S, respec-
tively, are (b) Cl2 + H2O2 → 2HCl + O2
(a) 0, +1 and −2 (b) +2, +1 and −2 (c) 2HI + H2O2 → I2 + H2O
(c) 0, +1 and +2 (d) −2, +1 and −2 (d) H2SO3 + H2O2 → H2SO4 + H2O

Chapter-8.indd 273 8/4/2016 10:43:00 AM


274 Chapter 8 Redox Reactions

Level II of electrons. The redox reaction may involve combination of


atoms or molecules, displacement of metals or non-metals and
Multiple Correct Choice Type disproportionation reactions. Redox reactions may be balanced
36. H2O2 can act as by two methods, oxidation number method or ion–electron
(a) an oxidizing agent. (b) a reducing agent. method.
(c) a bleaching agent. (d) None of these.
45. Which one of the compound cannot decolourize an acidified
37. Which of the following represent redox reactions? solution of KMnO4?
(a) SO2 (b) FeCl3 (c) H2O2 (d) FeSO4
(a) Cr2O72 − + 2OH− → 2CrO24− + H2O
46. When SO2 is passed in a solution of potassium iodate, the
(b) 2CrO24− + 2H+ → Cr2O72 − + H2O
oxidation state of iodine changes from
(c) 2MnO 4− + 3Mn2 + + 4 OH− → 5MnO2 + 2H2O (a) +5 to 0. (b) +5 to −1.
(c) −5 to 0. (d) −7 to −1.
(d) IO 4− + I− + H+ → I2 + H2O
47. The oxidation state of the most electronegative element in
38. Which of the following substances undergo disproportiona- the products of the reaction between BaO2 and H2SO4 are,
tion reactions under basic medium? respectively,
(a) F2 (b) P4 (c) S8 (d) Br2 (a) 0 and −1 (b) −2 and −1
(c) −2 and 0 (d) −2 and +1
39. H2C2O4 and NaHC2O4 behave as acids as well as reducing
agents. Which are the correct statements? Matrix-Match Type
(a) Equivalent weight of H2C2O4 and NaHC2O4 are equal
to their molecular weights when acting as reducing 48. Match the compound with the average oxidation state of Fe.
agents. Column I Column II
(b) Equivalent weight of H2C2O4 and NaHC2O4 are equal to half
(a) Fe 4 [Fe(CN)6 ]3 (p) 12/5
their molecular weights when acting as reducing agents.
(c) 100 mL of 1 M solution of each is neutralized by equal (b) [Fe(NO)(H2O)5 ]SO 4 (q) 2
volumes of 1 N Ca(OH)2.
(c) Fe3 [Fe(CN)6 ]2 (r) 18/7
(d) 100 mL of 1 M solution of each is oxidized by equal vol-
umes of 1 M KMnO4. (d) Na2 [Fe(CN)5 NO] (s) 1
40. In which of the following, oxidation number of oxygen is
fractional? 49. Match the bold atoms in compounds with their oxidation
(a) B4O10 (b) B2H6 (c) CsO2 (d) KO3 numbers.

41. Which of the following statements about the following reac- Column I Column II
tions are wrong? (a) H2O2 (p) +1
2Cu2O(s) + Cu2S(s) → 6Cu(s) + SO2(g) (b) MnSO4 (q) +3

(a) Both Cu2O and Cu2S are reduced. (c) AlCl3 (r) +2
(b) Only Cu2S is reduced. (d) S2Cl2 (s) +6
(c) Cu2S is the oxidant.
(d) Only Cu2O is reduced. 50. Match the reaction with its type.

42. When Cl2 is passed through NaOH in cold, the oxidation Column I Column II
number of Cl changes from
(a) V2O5 + Ca → 2V + 5CaO (p) Non-metal displace-
(a) 0 to −1 (b) 0 to +2 (c) 0 to −2 (d) 0 to +1
ment reaction
43. Which of the following statements are correct? (b) 2NaH → 2Na + H2 (q) Disproportionation
(a) All reactions are oxidation and reduction reactions. reaction
(b) Oxidizing agent is itself reduced.
(c) Oxidation and reduction always go side by side. (c) P4 + 3OH− + 3H2O → PH3 + (r) Decomposition reac-
(d) Oxidation number during reduction decreases. 3H2PO2 tion
(d) Ca + 2H2O → Ca(OH)2 + H2 (s) Redox reaction
44. A reducing agent in a redox reaction undergoes
(a) a decrease in oxidation number.
(b) an increase in oxidation number. Integer Type
(c) loss of electrons. 51. The number of electrons involved in the conversion of
(d) gain of electrons. MnO 4− to MnO2 is ____.
Paragraph for Question 45 to 47: Oxidation number of an atom 52. If 6 × 10−3 mol of K2Cr2O7 reacts completely with 9 × 10−3 mol
increases during oxidation and decreases during reduction, since of Xn+ to give XO3− and Cr3+ then the value of n is ___.
oxidation involves loss of electrons while reduction involves gain

Chapter-8.indd 274 8/4/2016 10:43:02 AM


Hints and Explanations 275

53. In the redox reaction, 55. Given balanced chemical equation for oxidation of phosphorus
xNO3− + yAs2S3 + zH2O → AsO34− + NO+ + SO24− + H+ (III) sulphide by nitric acid. The products include NO and SO2.

what is the value of (x/y)? aP4 S6 + bH+ + cNO3− → dNO + eH3PO 4 + f SO2 + gH2O
54. In Ba(H2PO2 )2 the oxidation number of phosphorus is ____. What is the value of (a + g)?

ANSWER KEY
Level I
1. (c) 2. (d) 3. (d) 4. (a) 5. (b) 6. (c)
7. (a) 8. (c) 9. (d) 10. (c) 11. (c) 12. (b)
13. (b) 14. (d) 15. (d) 16. (c) 17. (b) 18. (a)
19. (a) 20. (b) 21. (a) 22. (d) 23. (d) 24. (b)
25. (d) 26. (c) 27. (c) 28. (b) 29. (c) 30. (a)
31. (a) 32. (c) 33. (a) 34. (d) 35. (b)

Level II
36. (a), (b), (c) 37. (c), (d) 38. (b), (c), (d) 39. (b), (c), (d) 40. (a), (c), (d) 41. (b), (c), (d)
42. (a), (b) 43. (b), (c), (d) 44. (b), (c) 45. (b) 46. (a) 47. (b)
48. (a) → r; (b) → s; (c) → p; (d) → q 49. (a)→ r; (b) → s; (c) → q; (d) → p 50. (a) → s; (b) → r, s; (c) → q, s; (d) → p, s
51. (3) 52. (1) 53. (14) 54. (1) 55. (7)

HINTS AND EXPLANATIONS


Level I Ca2+ (OCl)− Cl− where (OCl)− is the hypochlorite ion and
Cl− chloride ion. In the hypochlorite ion, chlorine is in +1
Single Correct Choice Type state, while in chloride ion, it is in −1 state.

1. (c) Suppose oxidation number of C = x, then x + 2 – 2 = 0 8. (c) 1 mol each of Hg2+ and I− will form 1/4 = 0.25 mol
⇒x=0 9. (d) The balanced chemical equation is
2. (d) SO2−
4 ⇒ +6 + x(−2) = −2 ⇒ x = 4.
14H+ + Cr2O72 − + 6Fe2 + → 2Cr 3+ + 7H2O + 6Fe3+ .

3. (d) The reaction involved is Therefore, the sum of all the coefficients is = 14 + 6 + 1
+ 2 + 7 + 6 = 36
+1 −3 +3
3H3 P O2  P H3 + 2H3 P O3 10. (c) White phosphorus dissolves in boiling NaOH solution (in a
CO2 atmosphere) to give phosphine, PH3.
The number of electrons transferred in oxidation half reac- 0 −3 +1
CO atmosphere
tion is 4 and in the reduction half reaction is 2. P 4 + 3NaOH+3H2O 
2
→ P H3 + 3NaH2 P O2
Hence, the effective number of electrons transferred = 4, Phosphine Sodium
and total number of moles involved = 3. Therefore, hypophosphite
Molecular weight M 3M It is an example of disproportionation reaction.
Equivalent weight = = =
n-factor 4/3 4 +1 +5 −1
11. (c) 3 ClO − (aq) → ClO3− (aq) + 2 Cl− (aq)
+3 +4 +5
4. (a) S2 O24− < S O23 − < S2 O26 − It is an example of disproportionation reaction.
5. (b) The oxidation state of Cr changes from +6 to +3. Thus, it 12. (b) The reaction involved is
+5 −1
decreases by three units. − −
KlO3 + SO2 I + KSO4
6. (c) CO2 is not a reducing agent. It is an oxidizing agent.
Reducing agent must undergo oxidation SO2, H2O2 and
NO2 can undergo oxidation (i.e., they act as reducing 13. (b) Balanced equations for producing NO and NO2, respec-
agent). But CO2 not as carbon is present in its highest pos- tively, are
sible oxidation state of +4. 3Cu + 8HNO3 → 3Cu(NO3 )2 + 2NO + 4H2O (1)
7. (a) CaOCl2 has two chlorine atoms having different oxidation Cu + 4HNO3 → Cu(NO3 )2 + 2NO2 + 2H2O (2)
states.
Adding Eqs. (1) and (2), we have

Chapter-8.indd 275 8/4/2016 10:43:05 AM


276 Chapter 8 Redox Reactions

4 Cu + 12HNO3 → 4 Cu(NO3 )2 + 2NO2 + 2NO + 6H2O 25. (d) Disproportionation is the process in which an element
undergoes both oxidation and reduction in a reaction.
2Cu + 6HNO3 → 2Cu(NO3 )2 + NO2 + NO + 3H2O
Moreover, an element can show a maximum oxida-
Thus, coefficients x and y of Cu and HNO3 are 2 and 6, tion state equal to its “group number” and a minimum
respectively. oxidation state equal to its “group number 8”. Among
the oxyacids of chlorine given, in HClO4 chlorine is in
14. (d) Disproportionation is a type of reaction in which the
its highest oxidation state (+7). So, it cannot undergo
same element is oxidized and reduced as well.
further oxidation, and hence, HClO4 cannot undergo
0 +1 −3
P 4 + 2NaOH → 3NaH2 P O2 + P H3 disproportionation.

In the above reaction, P is oxidized to PH3 and reduced to 26. (c) KMnO4 cannot be oxidized further because Mn is already
NaH2PO2. in its maximum oxidation state of +7 in KMnO4.
+5 +5 27. (c) The maximum oxidation state possible for Br = +7. In
15. (d) In the reaction NaOH + H3 P O 4 → NaH2 PO 4 + H2O,
KBrO4, Br exists in +7 oxidation state.
no change in oxidation state of phosphorus occurs. So,
28. (b) The reactions involved are
equivalent weight = molecular mass = H3PO4 = 3 × 1 + 31
× 1 + 16 × 4 = 98 g. MnO 4− + 5e − + 8H+ → Mn2 + + 4H2O

16. (c) K is at topmost position in the electrochemical series in [Fe2 + → Fe3+ + e − ] × 5


comparison with other elements (Mg, Na, Br2). Therefore, MnO 4− + 5Fe 2+
+ 8H+ → Mn2 + + 5Fe3+ + 4H2O
it will be the most strongly reducing substance.
Hence, 5 mol of FeSO4 are completely oxidized by KMnO4
17. (b) Suppose oxidation number of sulphur = x = 1 mol
2 × 1 + 4 × (x) + 6 × (−2) = 0 ⇒ 2 + 4x − 12 = 0 15 mol of FeSO4 are completely oxidized by KMnO4
4x − 10 = 0 ⇒ 4x = 10 ⇒ x = +2.5 1 × 15
= = 3 mol
5
18. (a) In this compound K is present in the highest possible oxi-
dation state. 29. (c) The total oxidation of 2 nitrogen atoms in N2H4 is −4.
+1 −2 +4 −3 Since it loses 25 mol of electrons, the total oxidation
K SCN
number of two N atoms in Y increases by

19. (a) Cr2O72 − + 3Sn2 + + 14H+ → 3Sn4 + + 2Cr 3+ + 7H2O −2


N2 H4 → N2x + ne −
3 mol of Sn2+ react with 1 mol of Cr2O72− . 2.5 mol 25 mol
1 mol 10 mol
So, 1 mol of Sn2+ will react with (1/3) mol of Cr2O72− .
In hydrazine, the oxidation number of N is 2x + 4 (+1) = 0
20. (b) The reaction involved is 2Cu+ → Cu2 + + Cu0. ⇒ x = −2. The change in oxidation number per N atom
= number of electrons lost, so 2 (x + 2) = 10 ⇒ x = +3.
21. (a) The oxidation state of sulphur in S8 = 0; S2F2 is 2x − 2 = 0
+6 2− +6 2−
⇒ x = +1; and H2S is +2 + x = 0 ⇒ x = −2. 30. (a) C r O 4 → C r2 O7
+6 +6 No change in oxidation number.
Alkaline
22. (d) K 2 Cr2 O7 medium
 → K 2 Cr O 4
Potassium dichromate Potassium chromate 31. (a) Sum of oxidation numbers of all the elements in the
No change in the oxidation state of chromium takes molecular formula of any compound is zero. Putting the
place. oxidation number of all the elements in the molecular
formula of all the compounds, we find that in molecular
23. (d) Let x be the oxidation state of Cr. Then x + 2(−2) + 2(−1) A3(BC4)2 the sum of oxidation number of all the elements
= 0 ⇒ x = +6. Similary for the others the oxidation states comes out to zero.
+7 +3 +4
are MnO 4− , Cr(CN)36− , NiF62 − . 3 × ( + 2) + 2 × ( + 5 + 4× − 2) = 0

24. (b) SO2 behaves as both oxidizing and reducing agent. In 32. (c) The compound in which an element is present in its high-
sulphur dioxide (SO2), oxidation number of sulphur is est or lowest oxidation state cannot undergo dispropor-
+4. It can increase its oxidation number to +6 and get tionation reaction. In H5IO6, the oxidation number of I is
oxidized. It can also decrease its oxidation number to +2 +7 (highest oxidation state).
or even 0 and get reduced.
33. (a) Among the given oxidizing agents, ozone is the strong-
As reducing agent, SO2 + 2H2O → SO24− + 4H+ + 2e −
est oxidizing agent because on decomposition it forms
As oxidizing agent, SO2 + 4H+ + 4 e − → S + 2H2O atomic oxygen.

Chapter-8.indd 276 8/4/2016 10:43:10 AM


Hints and Explanations 277

34. (d) Perchloric acid is HClO4. Suppose oxidation state of Cl be 40. (a), (c), (d) Suppose oxidation state of oxygen = x, then we
x, then have
1 + x + 4 (–2) = 0 ⇒ x – 7 = 0 ⇒ x = +7 B4O10 CsO2 KO3
35. (b) The reaction can be represented as 4 × 1 + 10 (x) = 0 +1 × 2 (x) = 0 +1 × 3 (x) = 0
⇒ 4 + 10x = 0 ⇒ 1 × 2x = 0 ⇒ 1 = –3x
4 1
⇒ 4 = –10x ⇒ x = − ⇒ 1 = −2x ⇒ x = − 1
0 +1 −1 +1 −1 0 10 2 ⇒ x=−
Cl2 + H2O2(aq) 2HCl + O2 3
Oxidizing Reducing
+1 −2 +1 −2 0 +4 −2
agent agent 41. (b), (c), (d) 2Cu2 O (s) + Cu2 S (s) → 6 Cu(s) + S O2 (g)
Oxidation by Both Cu2O and Cu2S are reduced to Cu. Also, Cu2O acts as the
1 unit oxidant while Cu2S acts as the reductant as it gets oxidized to
SO2.
0 +1 −1 +1 −1 −1+2
42. (a), (b) The reaction is Cl2 + NaOH → NaCl + H Cl + O Cl .
Level II
43. (b), (c), (d) Statement (a) is not correct. For example, during
Multiple Correct Choice Type neutralization reactions and precipitation reactions oxidation
36. (a), (b), (c) In H2O2, the oxidation numbers of O is −1, it can number does not change. Hence, these reactions (collectively
increase its oxidation number to 0 and get oxidized. Similarly, known as metathesis reaction) are not redox reactions.
it can decrease its oxidation number to −2 and get reduced. 44. (b), (c) A reducing agent in a redox reaction undergoes an
increase in the oxidation number with loss of electrons.
H2O2 → 2H+ + O2 + 2e − (As reducing agent)
Passage Type
H2O2 + 2H+ + 2e − → 2H2O (As oxidizing agent)
45. (b) FeCl3 cannot decolourize an acidified solution of KMnO4
H2O2 is used extensively as a mild bleaching agent for textiles because it is present in Fe3+ state. So it cannot act as reducing
and paper/wood pulp. agent.
+5 0
46. (a) 2K I O3 + 5SO2 + 4H2O → 2KHSO 4 + 3H2SO 4 + I2
H2O2 + Cl2 → 2HCl + O2
47. (b) The reaction is BaO2 + H2SO 4 → BaSO 4 + H2O2 . Oxygen is
37. (c), (d) Both (c) and (d) are disproportionation reactions. the most electronegative element, and it has oxidation states
Redox reactions are those in which both reduction and oxida- of −2 and −1 in BaSO4 and H2O2, respectively.
tion occur simultaneously.
Matrix–Match Type
38. (b), (c), (d) Except F2 (which shows oxidation state of –1 48. (a) ã r; (b) ã s; (c) ã p; (d) ã q
only), all the other substances show both +ve and −ve oxida- (a) 7 x + ( − 18 ) = 0 (b) x +1 − 2 = 0
tion states and hence undergo disproportionation. (c) 5x − 12 = 0 (d) +2 +x − 5 + 1 = 0

39. (b), (c), (d) In the reduction reactions, H2C2O4 and NaHC2O4 49. (a) ã r; (b) ã s; (c) ã q; (d) ã p
are converted to CO2, so (a) 2x + 2(−2) = 0 so x = +2
(b) x + (+2) + (−2) × 4 = 0 so x = +6
(C3+ )2 → 2C 4 + + 2e − (c) x + (−1) × 3 = 0 so x = +3
(d) 2x + (−1)2 = 0 so x = +1
Therefore, equivalent weight = molecular weight/2. 50. (a) ãs; (b) ã r, s; (c) ã q, s; (d) ã p, s
On titration with 1 M KMnO4, +5 0 0 +2
(a) V2 O5 + 5 Ca → 2 V + 5 CaO
7+ 2+
Mn + 5e → Mn
This is a redox reaction.
Therefore, +1 −1 0 0
(b) 2 Na H → 2 Na + H2
MequivH2C2O 4 = MequivKMnO 4
This is an example of decomposition as well as redox
100 mL × 1 M × 2 = V mL × 1 M × 5 reaction.
0 −3 +2
(c) P4 + 3OH− + 3H2O → P H3 + 3H2 P O2
MequivNaHC2O 4 = MequivKMnO 4
Disproportionation is a reaction in which the same ele-
100 mL × 1 M × 2 = V mL × 1 M × 5 ment undergoes oxidation as well as reduction. It is also a
type of redox reaction.
So 100 mL of 1 M solution of each is neutralized by equal vol- 0 +1 −2 +2 0
umes of 1 M KMnO4. (d) Ca + 2 H2 O → Ca(OH)2 + H2

Chapter-8.indd 277 8/4/2016 10:43:16 AM


278 Chapter 8 Redox Reactions

This reaction is an example of non-metal displacement 53. (14) The half reactions are
reaction. It is also a redox reaction. +5 +3
2 N O3− + 8H+ + 2e − → 2 N O + + 4H2O] × 5
Integer Type
+3 −2 +5 +6
51. (3) MnO 4− + 2H2O + 3e − → MnO2 + 4 OH− As2 S3 + 20H2O → 2 AsO34− + 3 S O24− + 10e − + 40H+

52. (1) 2Cr 6 + + 6e − → 2Cr 3+ Overall balanced reaction


+5 +3 −2 +3 +5 +6
X n + → X5+ + (5 − n)e − 14 N O3− + As2 S3 + 16H+ → 14 N O + + 2 AsO34− + 3 S O24− + 8H2O
Milliequiv. of Xn+ = Milliequiv. of K2Cr2O7
54. (1) (H2PO2 )2−
2
−3 −3
9 × 10 × (5 − n) × 1000 = 6 × 10 × 6 × 1000
2(2( +1)) + x + 2( −2) = − 2 ⇒ x = +1
−3
6 × 10 × 6 × 1000 36
5−n= =
9 × 10 −3 × 103 9 55. (7) The balanced redox reaction is
Solving, we get n = 1. 3P4 S6 + 44H+ + 44NO3− → 44NO + 12H3PO 4 + 18SO2 + 4H2O

Chapter-8.indd 278 8/4/2016 10:43:19 AM


Solved JEE 2016 Questions 279

SOLVED JEE 2016 QUESTIONS


JEE Advanced 2016 Thus, 8 mol of permanganate anion oxidize thiosulphate
to produce 6 mol of sulphate ions.
1. In neutral or faintly alkaline solution, 8 mol permanganate
anion quantitatively oxidize thiosulphate anions to produce X
mol of a sulphur containing product, the magnitude of X is
Solution
(6) The balanced oxidation reaction of permanganate anion
in neutral (or faintly alkaline) medium is
8MnO 4− + 3S2O32 − + H2O → 8MnO2 + 6SO24− + 2OH−

Chapter-8.indd 279 8/4/2016 10:43:19 AM


Chapter-8.indd 280 8/4/2016 10:43:19 AM
9 Hydrogen

Question Distribution in JEE (Main and Advanced)

3
No. of Questions

JEE (Main)
2
JEE (Adv)

0
2016 2015 2014 2013 2012 2011 2010 2009 2008 2007

Concept Distribution in JEE (Main and Advanced)

Topics Covered
Year
JEE (Main) JEE (Advanced)
2008 Physical and Chemical Properties of Water and
Heavy Water
2010 Physical and Chemical Properties of
Water and Heavy Water
2012 Preparation, Properties and Uses of Hydrogen
2014 Hydrogen Peroxide Physical and Chemical Properties of
Water and Heavy Water
Preparation, Properties and Uses of Hydrogen,
2015 Physical and Chemical Properties of Water and
Heavy Water, Hydrogen Peroxide
2016 Physical Properties of Water, Heavy Water and
Chemical Properties of Water

Chapter-9.indd 281 7/29/2016 3:02:11 PM


282 Chapter 9 Hydrogen

SUMMARY
1. Hydrogen has the simplest atomic structure of all the elements, and consists of a nucleus containing one proton and one orbital
electron. Hydrogen is the first element in the periodic table, the electronic structure is written as 1s1.
2. Properties of atomic hydrogen
(a) The structure of hydrogen atom is in some ways like that of the alkali metals:
(i) Like alkali metals, it has just one electron in the outer shell.
(ii) The loss of one electron produces a cation, H+, like M+ ions formed by alkali metals, but hydrogen has a much greater ten-
dency to pair the electron to form a covalent bond.
(iii) Like alkali metals, hydrogen forms oxides, sulphides and halides. However, it does not show metallic properties and unlike
alkali metals has very high ionization enthalpy.
(b) The structure of hydrogen atom is in some ways like that of the halogens (Group 17):
(i) Both hydrogen and halogens are one electron short of a noble gas structure.
(ii) Halogens gain an electron to form X− ions, whereas hydrogen generally does not form H− but does form ionic hydrides M+
H− (e.g., LiH and CaH2) with highly electropositive metals.
(c) In some ways, the structure of hydrogen resembles that of the Group 14 elements:
(i) Both have a half-filled outer shell of electrons.
(ii) Hydrides and organometallic compounds show a number of similarities, the hydride is often considered as part of a series of
organometallic compounds. For example, LiH, LiMe, LiEt; NH3, NMe3, NEt3; or SiH4, CH3SiH3, (CH3)2SiCl2, (CH3)3SiCl, (CH3)4Si.
3. Isotopes
(a) Protium 11H or H, deuterium 21D or D and tritium 31T or T are three isotopes of naturally occurring hydrogen. Each of the three
isotopes contains one proton and 0, 1 or 2 neutrons, respectively. The relative abundance of isotopes is:

Isotope Abundance
1
1H 99.986%
2
1D 0.014%
3
1T 7 × 10−16%

Since protium is most abundant, the properties of hydrogen are those of the lightest isotope.

Tip Among the three isotopes, only tritium is radioactive and emits low energy b − particle.

(b) These isotopes have the same electronic configuration and hence same chemical properties, which differ only in the rates of
reactions and equilibrium constants. For example, H2 is more rapidly adsorbed onto surfaces than D2 and reacts over 13 times
faster with Cl2 than D2.
(c) The isotopes of hydrogen show a large variation in physical properties that arise due to difference in mass (isotope effect).

Tip This difference is largest among any group of isotopes is because the percentage difference in mass between protium,
deuterium and tritium is greater than that between the isotopes of any other element.

(d) In the H2 molecule, the two nuclei can spin in either the same direction, or in opposite directions (spin isomerism) to give rise to
two different forms of H2 called ortho and para hydrogens, respectively. The two forms show difference in physical properties due
to difference in internal energy.

Tip The para form has the lower energy, and hydrogen gas at absolute zero contains 100% of the para form. As the tem-
perature is raised, some of the para form changes into the ortho form and at high temperatures; about 75% ortho hydrogen
is present.

4. Preparation
(a) Laboratory preparation
(i) Reaction of dilute acid with metal or an alkali with metal. For example,

Zn + H2SO 4 → ZnSO 4 + H2
Zn + 2NaOH → Na2ZnO2 + H2
(ii) Action of water on metal hydrides
LiH + H2O → LiOH + H2

Chapter-9.indd 282 7/29/2016 3:02:13 PM


Summary 283

(b) Commercial preparation


(i) By passing steam over red hot coke to get water gas (also known as synthesis or syn gas), which is a mixture of CO and H2.
The process is also known as coal gasification.
1000 C
C + H2O  → CO + H2

Water gas

To obtain pure H2 from water (syn) gas:


• CO can be liquefied at a low temperature under pressure and removed.
• It can be mixed with steam, cooled to 400°C and passed over iron oxide in a shift converter, giving H2 and CO2 (water
gas shift reaction).
(ii) Steam reformer process: Light hydrocarbons such as methane are mixed with steam and passed over a nickel catalyst at
800–900°C.
CH4 + H2O → CO + 3H2
CH4 + 2H2O → CO2 + 4H2
Pure hydrogen is obtained from the mixture of CO + CO2 + H2 by water gas shift reaction.
(iii) By the electrolysis of water using platinum electrodes in the presence of traces of acid or base.
Anode : 2OH− → H2 × O + 21 O2 + 2e −

Cathode : 2H2O + 2e − → 2OH− + H2

Overall: H2O → H2 + 21 O2
(iv) As a byproduct from the chlor-alkali industry, in which aqueous NaCl (brine) is electrolyzed to produce NaOH, Cl2 and H2.
Anode: 2Cl− → Cl2 + 2e −
 + −
 Na + e → Na
Cathode: 
 2Na + 2H2O → 2NaOH + H2


(v) As a byproduct of cracking of natural hydrocarbon mixtures such as naptha and fuel oil.
5. Properties
(a) Hydrogen gas is the lightest gas known and is colourless, odourless and almost insoluble in water.

Tip Being lighter, hydrogen is used instead of helium to fill balloons for meteorology.
(b) The two hydrogen atoms in dihydrogen gas are joined by very strong covalent bonds (bond energy is 435.9 kJ mol−1). This high
bond energy is the cause for low reactivity of hydrogen under normal conditions. As a result many reactions are slow, or require
high temperatures, or catalysts (generally transition metals).
(c) Atomic hydrogen is obtained by cleavage of H H bond of dihydrogen at high temperature using electric arc or under ultraviolet
radiations. The 1s1 electron in the orbital can then react:
(i) By forming an electron pair (covalent) bond with another atom: In reactions with non-metals, such as in formation of
H2O, HCl (gas) or CH4.
(ii) By losing an electron to form H+: These always exist associated with other atoms or molecules such as H3O+, H9O4+ or
H(H2O)n+ ions
(iii) By gaining an electron to form H: Formation of metal halides with electropositive metals.
(d) Reaction with halogens
(i) With fluorine: The reaction is violent, even at low temperatures.
H2 + F2 → 2HF
(ii) With chlorine: The reaction is slow in the dark, but is catalyzed by light (photocatalysis). It becomes faster in daylight and
explosive in sunlight.
H2 + Cl2 → 2HCl
(iii) With bromine: The reaction proceeds at temperatures between 200°C and 400°C in the presence of platinum as catalyst.
H2 + Br2 → 2HBr
(iv) With iodine: The reaction proceeds in the presence of catalyst or ultraviolet radiation
H2 + I2 → 2HI

Chapter-9.indd 283 7/29/2016 3:02:15 PM


284 Chapter 9 Hydrogen

(e) Reaction with dioxygen


Used in oxy-hydrogen flame.
2H2 (g) + O2 (g) → 2H2O(l); ∆H o = − 485 kJ mol−1
(f) Reaction with dinitrogen
Used in Haber’s process for manufacture of ammonia.
Fe catalyst
N2 (g) + 3H2 (g) (673
 → 2NH3 (g) ∆H o = −92.22 kJmol−1
K, 200 atm)
(g) Reaction with metals
Leads to formation of metal hydrides.
H2 (g) + 2M(g) → 2MH
(h) Reaction with metal ions and oxides
These are reduced to corresponding metals.
H2 (g) + Pd2+ (aq) → Pd(s) + 2H+ (aq)
MO2 + 2H2 → M + 2H2O
M2O3 + 3H2 → 2M + 3H2O
MO + H2 → M + H2O
(i) Reaction with organic compounds
These react in presence of catalysts to give hydrogenated products.
CH3 (CH2)n CH CH COOH + H2 CH3 (CH2)n CH2 CH2 COOH

Tip Hydroformylation is an important industrial process for the production of aldehydes from alkenes. It involves treating
an alkene at high pressures (between 10 atm and 100 atm) with carbon monoxide and hydrogen at temperatures between
40°C and 200°C, using transition metals as catalysts.

6. Uses
(a) In production of ammonia using Haber’s process.
(b) In hydrogenation of alkenes and other double bonds. For example, in hardening of fats and oils by converting unsaturated
hydrocarbons (liquid at room temperature) to saturated hydrocarbons (solids at room temperature).
(c) In catalytic reduction of benzene and reduction of nitrobenzene to aniline.
(d) In preparation of many useful chemicals such as hydrogen chloride.
(e) In metallurgical processes for reduction of metal oxides to metals.
(f) As rocket fuel.
(g) In fuel cells for generation of electrical energy.
(h) In synthesis of organic compounds such as methyl alcohol.
Catalyst
CO + 2H2 
→ CH3OH
7. Hydrides
Hydrogen reacts with almost all elements (except noble gases) to form binary compounds called hydrides. Based on the type and
nature of bond, these can be classified as ionic, covalent or metallic hydrides.
(a) Ionic or salt-like hydrides
(i) These are formed by alkali (Group 1) and alkaline-earth (Group 2) metals.
(ii) They are only formed by the elements with an electronegativity value much lower than the value of 2.1 for hydrogen.
(iii) Lighter metal hydrides, such as LiH, BeH2 and MgH2 show significant covalent character, where the latter two show poly-
meric structure.
(iv) They are solids with high melting point.
(v) They are non-conducting in solids state.
(vi) They have high heats of formation, and are always stoichiometric.
(vii) Group 1 hydrides are more reactive than the corresponding Group 2 hydrides, and reactivity increases down the group.
(viii) They react with water and liberate hydrogen.
LiH + H2O → LiOH + H2
CaH2 + 2H2O → Ca(OH)2 + 2H2
(ix) They are powerful reducing agents, especially at high temperatures. For example,
SiCl4 + 4NaH → SiH4 + 4NaCl
PbSO4 + 2CaH2 → PbS + 2Ca(OH)2

Chapter-9.indd 284 7/29/2016 3:02:17 PM


Summary 285

Other hydrides obtained from them (such as lithium aluminium hydride and sodium borohydride) are used as important reduc-
ing agents in synthetic reactions.
8LiH + Al2Cl6 → 2Li[AlH4 ] + 6LiCl
2LiH + B2H6 → 2Li[BH4 ]

Tip The density of ionic hydrides is greater than that of the metal from which they were formed. This is explained by H− ions
occupying holes in the lattice of the metal, without distorting the metal lattice.

(b) Covalent or molecular halides


(i) These are generally hydrides of p-block elements, where the difference in electronegativity of elements and hydrogen is very
small. The formula of these hydrides is XHn or XH(8–n) where n is the group in the periodic table to which X belongs.
(ii) They consist of discrete covalent molecules, with only weak van der Waals forces holding the molecules together and so they
are usually volatile, and have low melting and boiling points.
(iii) They are prepared by direct action of hydrogen on the elements. For example,
High temp & pressure
3H2 + N2 
Catalyst
→ 2NH3

Electric spark
2H2 + O2 → 2H2O
(iv) They can be further classified as:
• Electron-deficient: These hydrides contain lesser number of electrons than that required for representing the correct
Lewis structure and can act as Lewis acids. For example, boron hydride and all Group 13 hydrides.
• Electron-precise: These hydrides contain the number of electrons required for representing the Lewis structure. For
example, methane.
• Electron-rich: These have excess electrons (in form of lone pairs) than that required for representing the Lewis structure
and thus act as Lewis bases. For example, ammonia, water. Hydrogen fluoride and other elements of Groups 15–17.
(c) Metallic or non-stoichiometric (or interstitial) hydrides
(i) These hydrides are formed by elements in the d-block, and the lanthanoid and actinoid elements in the f-block on reaction
with hydrogen.

Tip The elements in the middle of the d-block, that is, Groups 7, 8 and 9, do not form hydrides. The absence of
hydrides in this part of the periodic table is sometimes called: the hydride gap.
(ii) These are usually prepared by heating the metal with hydrogen under high pressure.
(iii) Like the parent metals, these hydrides are hard, have a metallic luster, conduct electricity and have magnetic properties. The
thermal and electric conduction is however, less than the parent metal.
(iv) The inclusion of hydrogen expands the crystal so these hydrides are less dense than parent metal and the distorted crystal
lattice makes hydrides brittle.
(v) Many of the hydrides are non-stoichiometric, for example, LaHn, TiHn and PdHn, where the chemical composition is variable.
Typical formulas are LaH2.87, YbH2.55, TiH1.8, ZrH1.9, VH1.6, NbH0.7 and PdH0.7.

Tip These hydrides were also called interstitial hydrides, and it was thought that a varying number of interstitial
positions in the metal lattice could be filled by hydrogen.
(d) Intermediate hydrides
These hydrides do not fit completely into any of the three classifications. For example,
(i) (BeH2)n is a chain polymer with hydrogen bridges.
(ii) MgH2 has properties in between those of ionic and covalent hydrides. CuH, ZnH2, CdH2 and HgH2 have properties interme-
diate between metallic.
8. Physical properties of water
(a) Water exists in nature in all three common states of matter as water vapour, liquid water (in sea, rivers, oceans and ground lique-
fier) and ice (icebergs, glaciers, polar caps, etc.).
(b) Generally the boiling points increase as the atoms become larger and heavier and the volatility decreases. This trend is shown by
the boiling points of H2S, H2Se, H2Te and H2Po, but the boiling point of water does not follow this trend. Water has an abnormally
low volatility, high boiling point and melting point because its molecules are associated with each other by means of hydrogen
bonds in both the solid and liquid states.
(c) Water is often called the universal solvent because it can dissolve many substances through polar bonds.

Tip A hydrogen bond is a weak chemical bond (strength 20 kJ mol−1) between a hydrogen atom in one polar molecule and
a very electronegative atom of the second polar molecule. Liquid water contains about 4–8 water molecules associated by
hydrogen bonding.

Chapter-9.indd 285 7/29/2016 3:02:17 PM


286 Chapter 9 Hydrogen

(d) Structure of H2O


(i) In water: The two bonding pairs between hydrogen and oxygen and two lone pairs on oxygen arrange in a tetrahedral
geometry. However, the H O H bond angle in water is 104°28’, which is slightly less than that for a tetrahedral structure
and the molecule has a bent structure.
(ii) In ice: The crystal structure is characterized by hexagonal symmetry. The distance between oxygen atoms along each bond
is about 275 pm and that is maintained throughout the lattice. The angle between bonds is very close to the tetrahedral
bond angle of 109.5°.
(e) Density
A unique property of water is that the solid is less dense than the liquid. The maximum density of water is shown at 4°C.

Tip Ice has a rather open structure, with quite large cavities. On partial melting some “free” water molecules occupy some
of these cavities, and hence the density increases. This effect outweighs the effect of thermal expansion up to 4°C, but above
this temperature, expansion has the larger effect so the density decreases.

9. Chemical properties of water


(a) Amphoteric nature
Water has the ability to act both as an acid (proton donor) as well as a base (proton acceptor).
H2O(l) + HCl(aq)  H3O + (aq) + Cl− (aq) (acts as a base)

H2O(l) + NH3 (aq)  NH+4 (aq) + OH− (aq) (acts as an acid)

H2O(l) + H2O(l)  H3O+ (aq) + OH− (aq) (self-ionization)


Acid Base Conjugate Conjugate
acid base

(b) Redox reactions


Water can act both as oxidizing and reducing agent:
• Reduction to dihydrogen
2 Na(s) + 2H2O(l) → 2 NaOH(aq) + H2 (g)
• Oxidation to oxygen gas
2F2 (g) + 2H2O(l) → 4H+ (aq) + 4F − (aq) + O2 (g)
4AgF2 (s) + 2H2O(l) → 4AgF(aq) + 4HF(aq) + O2 (g)
(c) Hydrolysis
It is the decomposition of a compound on reaction with water in which H+ gets incorporated in one part of the compound and
OH− in the other.
CO2 (g) + H2O(l) → H2CO3 (aq)
NH4 + (aq) + H2O(l) → NH3 (g) + H3O + (aq)
Br2 (l) + H2O(l) → HBr(aq) + HOBr(aq)
(d) Hydrate formation
These are solid compounds in which discrete water molecules are either bound to cations through the oxygen atom or bound to
the anions or other electron-rich atoms through hydrogen bonds or both. For example,
(i) [Cr(H2O)6]3+, 3Br−, [Zn(H2O)4]2+, 2Cl− (Coordinated with metal ion)
(ii) BaCl2 ⋅ 2H2O, ScCl3 ⋅ 6H2O, (Interstitial water)
(iii) CuSO4 ⋅ 5H2O, (Hydrogen bonded water)
10. Hard and soft water
Hard water contains a large quantity of dissolved minerals like calcium and magnesium. These minerals are not removed during
purification of water by physical processes such as filtration and sedimentation. Hardness of water is classified into two types, tem-
porary hardness and permanent hardness.
(a) Temporary hardness results in water that contains bicarbonates of calcium and magnesium. It can be removed by:
(i) Boiling ∆
Ca(HCO3 )2  → CaCO3 ↓ + H2O + CO2

Mg(HCO3 )2 
→ Mg(OH)2 ↓ + 2CO2
(ii) Clarke’s method: Involves addition of calculated amounts of calcium hydroxide.
Ca(HCO3 )2 + Ca( OH)2 → 2CaCO3 ↓ + 2H2O
Mg(HCO3 )2 + 2Ca(OH)2 → 2CaCO3 ↓ + Mg(OH)2 ↓ + 2H2O

Chapter-9.indd 286 7/29/2016 3:02:19 PM


Summary 287

(b) Permanent hardness is due to presence of chlorides and sulphates of calcium and magnesium in water and also salts of heav-
ier elements such as iron and aluminium. It can be removed by:
(i) Addition of calculated amount of sodium carbonate (washing soda).
Na2CO3 + MgCl2 → MgCO3 (↓ ) + 2NaCl

Na2CO3 + CaSO 4 → CaCO3 (↓ ) + Na2SO 4


(ii) Addition of Calgon (sodium hexametaphosphate) which forms soluble complexes with calcium and magnesium ions at
pH 10 (Calgon’s method).
2Ca2 + + [Na4 (PO3 )6 ]2 − → 4Na+ + [Ca2 (PO3 )6 ]2 −

(iii) Percolating through the bed of zeolite (Na2Z) which are naturally occurring hydrated aluminosilicate minerals (ion
exchange method or permutit process).
Na2Z + CaSO 4 (or CaCl2 ) → CaZ + Na2SO 4 (or 2NaCl)
Na2Z + MgCl2 (or MgSO 4 ) → MgZ + 2NaCl (or Na2SO 4 )
Na2Z + Ca(HCO3 )2 (or Mg(HCO3 )2 ) → CaZ (or MgZ) + 2NaHCO3
(iv) Using synthetic ion exchange resins for removal of cations causing hardness in water (demineralization method).
nRCOOH+ + Ca2 + → n[(RCOO )2 Ca2 + ] + 2H+ (cation exchange resin)
+ − +
n[R − N(CH3 )3 OH] + Cl → n[R − N(CH3 )3 ]Cl + OH−
− −
(anion exchange resin)

11. Heavy water (D2O)


(a) It is prepared by exhaustive electrolysis of water and is obtained as a byproduct in some fertilizer industries.
(b) The comparison of physical properties with water is given in below table.

Physical Constant H2O D2O

Freezing point (°C) 0 3.82


Boiling point (°C) 100 101.42
Density at 20°C (g cm–3) 0.917 1.017
Temperature of maximum density (°C) 4 11.6
Ionic product Kw at 25°C 1.0 × 10–14 3.0 × 10–15
Dielectric constant at 20°C 82 80.5
Solubility grams of NaCl/100 g water at 25°C 35.9 30.5
Solubility grams of BaCl2/100 g water at 25 °C 35.7 28.9
Enthalpy of vaporization (at 100°C, kJ mol−1) 40.66 41.61

12. Hydrogen peroxide


It is an oxygen–oxygen single bond. It is considered a highly reactive oxygen species due to its strong oxidizing properties
(a) Structure
(i) It has a skew structure with following dimensions

H H
95.0 pm 94.8° 111.5° 98.8 pm 101.9° 90.2°
O O O O
147.4 pm 145.8 pm
H H

(a) (b)

Structure of H2O2 In (a) gaseous and (b) solid states

Chapter-9.indd 287 7/29/2016 3:02:21 PM


288 Chapter 9 Hydrogen

(ii) It shows restricted rotation about the O O bond, presumably due to repulsion between the OH groups.
13. Preparation of hydrogen peroxide
(a) H2O2 was earlier obtained by electrolysis of H2SO4 or (NH4)2SO4 at a high current density to form peroxosulphates, which were
then hydrolyzed.
Electrolysis
2SO24−  → S2O28 − + 2e −
H2S2O8 + H2O → H2SO5 + H2SO 4
Peroxodisulphuric Peroxomonosulphuric
acid acid
H2SO5 + H2O → H2SO 4 + H2O2
(b) In the laboratory, it can be prepared by adding calculated amount of sodium peroxide to ice cold dilute (20%) solution of H2SO4.
Na2O2 + H2SO 4 → Na2SO 4 + H2O2
(c) It can be prepared by the addition of acid to a peroxide or a persulphide salt. This is known as Merck’s process.
BaO2 ⋅ 8H2O(s) + H2SO 4 (aq) → H2O2 (aq) + BaSO 4 (s) + 8H2O(l)
(d) It is produced on an industrial scale by a cyclic process where 2-ethyl anthroquinol is oxidized by air to the corresponding qui-
none and H2O2.
OH O2 O
C2H5 C2H5
+ H2O2

OH H2/Pd O
2-Ethyl anthraquinol Quinone
Tip H2O2 is extracted with water as a 1% solution. This is concentrated by distillation under reduced pressure, and sold
as a 30% (by weight) solution. It can be further concentrated to about 85% by careful distillation and pure H2O2 can be
obtained by removal of water on freezing.

14. Physical properties of hydrogen peroxide


(a) Pure H2O2 is almost a colorless liquid with a tinge of pale blue and resembles water quite closely. It is more hydrogen bonded
than is water and so has a higher boiling point (b.p. 152°C).
(b) It is completely miscible with water in all proportions and forms hydrate structure of the type H2O2 ⋅ H2O. Concentrated aque-
ous solution is also soluble in organic solvents.
(c) Hydrogen peroxide and water do not form azeotropic mixtures and can be completely separated by distillation.

Tip The hydrogen peroxide solution is available commercially as a 30 % solution and 1 mL of solution can give 100 V of
oxygen. The solution is also known as “100 volume” hydrogen peroxide.

15. Chemical reactions of hydrogen peroxide


(a) As oxidizing agent
H2O2 acts as a strong oxidizing agent. The reaction is slow in acidic medium and fast in basic medium.
2Fe2+ + 2H+ + H2O2 → 2Fe3+ + 2H2O (Slow)

2Fe2+ + H2O2 → 2Fe3+ + 2OH− (Fast)


(b) As reducing agent
H2O2 is reduced in presence of stronger oxidizing agents and oxygen is evolved.
For example,
In acidic medium: 2KMnO 4 + 5H2O2 + 3H2SO 4 → 2MnSO 4 + K 2SO 4 + 5O2 + 8H2O

In basic medium: 2KMnO 4 + 3H2O2 → 2MnO2 + 3O2 + 2H2O + 2OH−


(c) H2O2 is fairly stable and decomposes only slowly on exposure to light.
2H2O2 → 2H2O + O2
However, in the presence of impurities and catalysts, it decomposes (disproportionates) and the rate of decomposition depends
on the temperature and concentration.

Chapter-9.indd 288 7/29/2016 3:02:24 PM


Tools for Problem Solving 289

Tip H2O2 solutions are stored in plastic or wax coated glass vessels, often with negative catalysts such as urea or sodium
stannate added as stabilizers.

16. Uses of hydrogen peroxide


(a) As a mild bleaching agent for textiles and paper/wood pulp. Smaller amounts are used to bleach hair, feathers, fats and waxes.
(b) For making other chemicals and as rocket fuel.
(c) It is used as an oxidizing agent in the laboratory, and as an antiseptic to treat wounds.
(d) It is useful to counteract chlorine, and in this reaction H2O2 behaves as a reducing agent.
H2O2 + Cl2 → 2HCl + O2

17. Hydrogen as a fuel


(a) Burning hydrogen in air or dioxygen forms water and liberates a great deal of energy.
(b) Unlike other fuels like coal, petrol or diesel, burning of hydrogen as a fuel does not produce and is environmental friendly.
(c) It can be produced readily by electrolysis and chemical methods.
(d) It can be transported as gas in cylinders and in liquid form in cryogenic cans.
(e) Liquid hydrogen is used as a fuel in space rockets and space shuttles in space program.
(f) It can be used in H2–O2 fuel cells to convert the chemical energy of fuel directly into electrical energy through catalytically acti-
vated redox reactions.
(g) Hydrogen has low mass and high enthalpy of combustion, therefore it is considered as an ideal rocket fuel. In addition to hydro-
carbon fuels, other hydrogen-containing compounds used are aniline, lithium hydride, hydrazine, liquid ammonia and alkyl or
aryl boranes.
(h) Hydrogen bomb releases energy by fusion of nuclei like tritium and deuterium. A sequence of reaction steps take place, thereby
converting more and more matter into energy, resulting in an explosion.
18. Limitations of use of hydrogen as a fuel
(a) Hydrogen is highly combustible and is difficult to handle safely. Because of its combustible nature it burns with explosion.
(b) It is not easy to store or transport hydrogen safely from one place to another.

TOOLS FOR PROBLEM SOLVING


Some Reactions of Hydrogen
NaBH4 LiAlH4

+B2H6 +Al2Cl6
CaH2

NaH LiH
+Ca

+Li UH3
LaH2.87 +Na
+La +U

catalyst catalyst
CH4 + H2O H2 H2O + CO
−CO −CO2

catalyst
CuO +N2

+WO3 +Fe3O4

NH3
Cu0

W0 Fe0

Chapter-9.indd 289 7/29/2016 3:02:24 PM


290 Chapter 9 Hydrogen

SOLVED EXAMPLES
Preparation, Properties and Uses of Hydrogen (c) Tasteless gas (d) Non-inflammable gas
(JEE Main Online 2015)
1. Which one the following statement regarding helium is
incorrect? Solution
(a) It is used to fill gas balloons instead of hydrogen because (d) Dihydrogen is highly flammable gas.
it is lighter and non-inflammable.
(b) It is used in gas – cooled nuclear reactors. 5. Hydrogen resembles the properties of
(c) It is used to produce and sustain powerful superconduct- (a) alkali metals. (b) alkaline earth metals.
ing reagents. (c) halogens. (d) both (a) and (c).
(d) It is used as cryogenic agent for carrying out experiments Solution
at low temperatures.
(d) Though hydrogen shows some similarity with both alkali
Solution metals and halogens, it differs from them on some grounds.
(a) It is used to fill gas balloons instead of hydrogen because Unlike alkali metals, hydrogen does not possess metallic
it is lighter and non-inflammable. characteristics. On the other hand, it possesses a high ioni-
zation enthalpy. Also, it is less reactive than halogen.
2. Which of the following statements in relation to the hydrogen Owing to these reasons, hydrogen cannot be placed
atom is correct? with alkali metals (Group 1) or with halogens (Group 17).
(a) 3s orbital is lower in energy than 3p orbital.
(b) 3p orbital is lower in energy than 3d orbital. 6. Hydrogen atom may attain stability by
(c) 3s and 3p orbitals are of lower energy than 3d orbital. (a) forming an electron pair (covalent) bond with another atom.
(d) 3s, 3p and 3d orbitals all have the same energy. (b) losing an electron to form H+.
(c) gaining an electron to form H–.
Solution (d) All of these.
(d) This is because in case of H atom, the only force of inter- Solution
action is the force of attraction between positively and
(d) H + H → H − H → Stable
negatively charged nucleus.
H → H+ + e− → Stable
3. Very pure hydrogen (99.9%) can be made by which of the fol- H + e− → H− → Stable
lowing processes?
(a) Reaction of methane with steam. 7. Dihydrogen is obtained from
(b) Mixing natural hydrocarbons of high molecular weight. (a) water. (b) dilute acids.
(c) Electrolysis of water. (c) alkalis. (d) all of these.
(d) Reaction of salt like hydrides with water. Solution
(AIEEE 2012) (d) Very pure hydrogen (99.9% pure) is made by the electrol-
Solution ysis of water using platinum electrodes in the presence of
(c)
800 − 900° C
CH4 + H2O 
→ CO + 3H2 traces of acid or base.
The usual laboratory preparation is the reaction of dilute
In this method mixture of CO and H2 are obtained. acids with metals.
High molecular weight hydrocarbon on cracking Zn + H2SO4 → ZnSO4 + H2
gives H2 but this mixture also contain small amount of
The usual laboratory preparation is the reaction of an
hydrocarbon.
alkali with metal.
Electrolysis of H2O is best method formation of 99.97 %
pure H2. H2O is mixed with NaOH or KOH. 2Al + 2NaOH + 6H2O → 2Na[Al(OH)4] + 3H2
At the cathode Zn + 2NaOH → Na2ZnO2 + H2
2H2O + 2e − → 2OH− + H2 8. The conversion of atomic hydrogen into ordinary hydrogen is
(a) exothermic change. (b) endothermic change.
At the anode
(c) nuclear change. (d) photochemical change.
1
2OH− → 2H2O + O2 + 2e −
2 Solution
Overall reaction (a) Atomic hydrogen combines to form ordinary hydrogen
1 with the evolution of 104.2 kcal of heat (exothermic pro-
H2O → H2 + O2 cess). This recombination is catalyzed by certain metals.
2
LiH + H2O → LiOH + H2 H + H → H2; ΔH = −104.2 kcal
This method is difficult to handle. 9. The bond dissociation enthalpy is highest in
4. Which physical property of dihydrogen is wrong? (a) H H (b) D D
(a) Colourless gas (b) Odourless gas (c) F F (d) Cannot be determined.

Chapter-9.indd 290 7/29/2016 3:02:26 PM


Solved Examples 291

Solution Solution
(b) The stronger the attraction, the greater will be the bond (d) In dihydrogen gas, the hydrogen atoms have already
strength and the higher is the bond dissociation enthalpy. shared their electrons to form the molecule, so H2 mole-
Hence, the bond dissociation enthalpy of D D is higher cule is not very reactive.
than H H. However, bond dissociation enthalpy is mini-
12. In context with the industrial preparation of hydrogen from
mum in the case of F F. The bond pair experiences strong
water gas (CO + H2), which of the following is the correct
repulsion from the lone pairs present on each F-center.
statement?
Therefore, the increasing order of bond dissociation
(a) CO and H2 are fractionally separated using differences in
enthalpy is as follows:
their densities.
F F<H H<D D (b) CO is removed by absorption in aqueous Cu2Cl2 solution.
10. Very pure hydrogen can be prepared by which of the follow- (c) H2 is removed through occlusion with Pd.
ing processes? (d) CO is oxidized to CO2 with steam in the presence of a
(a) Electrolysis of brine solution. catalyst followed by absorption of CO2 in alkali.
(b) Passing steam over red hot coke. (AIEEE 2008)
(c) Water gas shift reaction. Solution
(d) Electrolysis of warm aqueous barium hydroxide solution
(d) Hydrogen is made in large amounts by the steam
between nickel electrodes.
reformer process. When water gas is passed over iron or
Solution copper catalyst at 400°C, CO gets converted into CO2.
(d) Hydrogen of high purity can be made by electrolysis of Finally CO2 is absorbed in a solution of alkali.
water using platinum electrodes. However, since water H2O
CO + H2 CO2 (Acidic oxide) + 2H2
does not conduct electricity very well, hydrogen of high
KOH (base)
purity (more than 99.95%) is prepared by electrolysis Water gas
of warm aqueous barium hydroxide solution between K2CO3
nickel electrodes. The reactions taking place at the cath-
ode and anode are: Isotopes
− − 13. Which isotope of hydrogen is the most abundant?
At the anode: 2OH → H2O + 21 O2 + 2e
At the cathode: 2H2O +2e − → 2OH− + H2 (a) Protium 11H (b) Deuterium 12 H

Overall recation: H2O → H2 + 21 O2 (c) Tritium 13 H (d) Both (a) and (b)
Solution
All other given methods will produce hydrogen but that
may not be very pure. (a) Naturally occurring hydrogen contains 99.986% of the
1 2 −16% of 3 T.
Electrolysis of brine solution is accompanied by for- 1H isotope, 0.014% of 1 D and 7 × 10 1
mation of NaOH and Cl2. 14. There are three isotopes of hydrogen and three naturally
At the anode : 2Cl− → Cl2 + 2e − occurring isotopes of oxygen (16O, 17O, and 18O). How many
kinds of water are possible?
At the cathode : Na+ + e − → Na (a) 6 (b) 3 (c) 18 (d) 9
2Na + 2H2O → 2NaOH + H2
Solution
Passing steam over red hot coke leads to formation of (c) 18 kinds are possible.
water gas from which it is difficult to obtain pure hydro-
gen as CO is difficult to remove. 15. The isotope effect arises due to
1000° C
(a) difference in number of electrons
C(s) + H2 (g) → CO + H2 (b) difference in number of protons.
Water gas
(c) difference in mass
In water gas shift reaction, the water gas mixture is (d) difference in properties.
mixed with steam and passed over iron oxide to obtain Solution
hydrogen and carbon dioxide which is easier to remove
(c) The isotopic effect arises due to difference in atomic
than CO.
masses of the isotopes of an element. Protium 11H has
450° C, Fe O
CO + H2 2 3
→ 2H2 (g) + CO2 (g) mass number 1; deuterium 12 H has mass number 2 and
HO
Water gas 2
Tritium 13 H has mass number 3.
11. Of the following statements regarding dihydrogen, identify 16. At absolute zero,
the statement which is not correct? (a) only para-hydrogen exists
(a) It is a colorless, odorless, tasteless gas. (b) only ortho-hydrogen exists
(b) It has very low solubility in water. (c) Both (a) and (b)
(c) It forms more compounds than any other element. (d) None of these exists
(d) It is a highly reactive gas.

Chapter-9.indd 291 7/29/2016 3:02:29 PM


292 Chapter 9 Hydrogen

Solution 22. Which of the following has the highest extent of hydrogen
(a) At absolute zero only para-hydrogen exists. As the tem- bonding?
perature is raised, some of it changes into ortho form. (a) NH3 (b) H2O
At high temperatures the gas contains about 75% (c) HF (d) Same in all
ortho-hydrogen. Solution
Hydrides (b) Among nitrogen, fluorine and oxygen, the increasing
order of their electronegativities are N < O < F. Hence,
17. Which one of the following substances has the highest pro- the expected order of the extent of hydrogen bonding is
ton affinity? HF > H2O > NH3. But, the actual order is H2O > HF > NH3.
(a) H2O (b) H2S (c) NH3 (d) PH3 Although fluorine is more electronegative than oxy-
Solution gen, the extent of hydrogen bonding is higher in water.
(c) This is because NH3 is a strong base (means it will accept There is a shortage of hydrogens in HF, whereas there are
H+ ions easily). exactly the right numbers of hydrogens in water.
In case of ammonia, the extent of hydrogen bond-
18. The correct order of the thermal stability of hydrogen halides ing is limited because nitrogen has only one lone pair.
(H–X) is Therefore, it cannot satisfy all hydrogens.
(a) HI > HBr > HCl > HF (b) HF > HCl > HBr > HI
(c) HCl < HF > HBr < HI (c) HI > HCl < HF > HBr 23. Dihydrogen forms interstitial compounds with
(a) alkali metals.
Solution (b) alkaline earth metals.
(b) Because thermal stability ∝ bond dissociation energy. (c) non-metals.
Since, the bond dissociation energy of HF is the highest (d) some transition metals, such as Pt and Pd.
and HI is the least. So, HF is the most stable halogen acid Solution
while HI is the least stable.
(d) Only transition metals form interstitial compounds.
19. Electron-deficient, electron-precise and electron-rich hydrides Alkali, alkaline earth and non-metals will not form inter-
are types of stitial compounds with dihydrogen. This formation of
(a) ionic hydrides. (b) molecular hydrides. interstitial compounds depends on the size of voids
(c) covalent hydrides. (d) metallic hydrides. formed and radius of the atom to be trapped. Hence the
Solution option (d) is correct.

(b) These are the types of molecular hydrides. 24. The carbon hydrides of the type (CnH2n+2) act as
(a) Lewis acid. (b) Lewis base.
20. Saline hydrides are formed by (c) both (a) and (b). (d) Brönsted acid.
(a) alkali metals.
(b) alkaline earth metals. Solution
(c) both (a) and (b). (c) For carbon hydrides of the type CnH2n+2, the following
(d) none of these. hydrides are possible for
Solution n = 1⇒ CH4
(c) At high temperatures, the metals of Group 1 (alkali met- n = 2 ⇒ C2H6
als) and the heavier Group 2 metals (alkaline earth met- n = 3 ⇒ C3H8
als) Ca, Sr and Ba form ionic hydrides such as NaH and

CaH2. These compounds are solids with high melting


As these are electron-precise hydrides, having all atoms
points, non-conducting in solid state and are called ionic
with complete octets, they can neither donate nor
(salt-like) hydrides.
accept electrons to act as a Lewis acid or Lewis base.
21. Which of the properties of interstitial hydrides is correct?
25. Which of the following hydrides is used for the storage of
(a) They generally form non-stoichiometric species.
hydrogen and serve as a source of energy?
(b) The hydrogen dissolved in titanium improves its
(a) Ionic hydride (b) Covalent hydride
mechanical properties.
(c) Metallic hydride (d) Polymeric hydride
(c) They give rise to metals fit for fabrication.
(d) On thermal decomposition, they afford a source of pure Solution
hydrogen. (c) Metallic hydrides are hydrogen-deficient, that is, they
Solution do not hold the law of constant composition. Metals like
Pd, Pt have the capacity to accommodate a large volume
(a) The interstitial hydrides are generally non-stoichiomet-
of hydrogen. Therefore, they are used for the storage of
ric and their compositions vary with temperature and
hydrogen and serve as a source of energy.
pressure. For example, TiH1.73, CeH2.7, LaH2.8, etc.

Chapter-9.indd 292 7/29/2016 3:02:29 PM


Solved Examples 293

Physical and Chemical Properties of Water and Heavy 31. What is formed when calcium carbide reacts with heavy
Water water?
(a) C2D2 (b) CaD2 (c) Ca2D2O (d) CD2
26. Which one of the following processes will produce hard
water? Solution
(a) saturation of water with CaCO3 (a) When calcium carbide reacts with water, it forms calcium
(b) saturation of water with MgCO3 hydroxide and acetylene.
(c) saturation of water with CaSO4
(d) addition of Na2SO4 to water CaC2 + 2H2O → Ca(OH)2 + C2H2

Solution When calcium carbide reacts with heavy water, it forms


deuterated acetylene and calcium hydroxide.
(c) Hardness of water is due to sulphates of calcium and
magnesium, that is, CaSO4 and MgSO4. CaC2 + 2D2O → Ca(OD)2 + C2D2
27. Permanent hardness in water cannot be cured by
32. When hard water is passed through permutit, which ions are
(a) boiling.
exchanged with Ca2+ and Mg2+?
(b) ion exchange method.
(a) Na+ (b) Al3+
(c) Calgon’s method.
(c) H+ (d) k+
(d) Treatment with washing soda.
(JEE Main Online 2015) Solution
Solution (a) For the softening of water by permutit process, the water
(a) Permanent hardness is due to presence of chlorides and is percolated through the bed of zeolite packed in a ver-
sulphates of calcium and magnesium in water and also tical cylinder. The zeolite may be represented as Na2Z
salts of heavier elements such as iron and aluminium. It and the following reactions occur in the zeolite bed:
cannot be cured by boiling. Na2Z + CaSO 4 (or CaCl2 ) → CaZ + Na2SO 4 (or 2NaCl)
28. Chemical A is used for water softening to remove temporary Na2Z + MgCl2 (or MgSO 4 ) → MgZ + 2NaCl (or Na2SO 4 )
hardness. Chemical A reacts with sodium carbonate to gen- Na2Z + Ca(HCO3 )2 (or Mg(HCO3 )2 ) → CaZ (or MgZ) + 2NaHCO3
erate caustic soda. When CO2 is bubbled through a solution
The ion exchange process will continue till all the
A, it turns cloudy. What is the chemical formula of A?
exchangeable sodium ions are used up. Regeneration of
(a) CaO (b) Ca(OH)2
the bed is carried out by treating the bed with a concen-
(c) CaCO3 (d) Ca(HCO3)2
trated solution of sodium chloride:
Solution CaZ + 2NaCl → Na2Z + CaCl2
(b) The chemical A is Ca(OH)2. MgZ + 2NaCl → Na2Z + MgCl2
Chemical A reacts with sodium carbonate to generate
caustic soda. 33. The decomposition of a compound on reaction with water is
Ca(OH)2 + Na2CO3 → 2NaOH + CaCO3 called
Caustic soda
(a) hydration. (b) hydrolysis.
When CO2 is bubbled through a solution A, it turns
(c) dehydration. (d) breaking.
cloudy.
Ca(OH)2 + CO2 → CaCO3 ↓ + H2O Solution
Cloudy
(b) Hydrolysis is the decomposition of a compound on reac-
29. What does [H9O4]+ stand for? tion with water in which H+ gets incorporated in one
(a) Acidic hydrogen (b) Hydrated proton part of the compound and OH− in the other.
(c) Trimer (d) Basic hydrogen Hydrogen Peroxide
Solution 34. In which of the following reactions H2O2 acts as a reducing
(b) [H9O4]+ stands for hydrated proton. In [H9O4]+, H+ is tet- agent?
rahedrally surrounded by four water molecules. (i) H2O2 + 2H+ + 2e − → 2H2O
30. Which of the following has lower value for D2O than for H2O? (ii) H2O2 − 2e − → O2 + 2H+
(a) Molecular mass (b) Dielectric constant
(c) Viscosity (d) Ionization constant (iii) H2O2 + 2e − → 2OH−
Solution (iv) H2O2 + 2OH− − 2e − → O2 + 2H2O
(d) Ionization constant of H2O 1.008 × 10−14 is lower than (a) (i), (ii) (b) (iii), (iv) (c) (i), (iii) (d) (ii), (iv)
that of D2O 1.95 × 10−15. This is because of strong associ- (JEE Main 2014)
ation (H-bonding) in D2O molecule than water molecule.

Chapter-9.indd 293 7/29/2016 3:02:31 PM


294 Chapter 9 Hydrogen

Solution H2S2O8 + 2H2O → 2H2SO4 + H2O2


(d) The oxidation states of the species are as follows:
−1 −2
38. Which of the following is not reduced by hydrogen peroxide?
(i) H2 O2 + 2H+ + 2e − → 2H2 O (a) KI (b) KIO4
−1 0
(c) Chlorine (d) Silver oxide
(ii) H2 O2 − 2e − → O2 + 2H+ Solution
−1 −2
(a) Hydrogen peroxide oxidizes potassium iodide (KI) to lib-
(iii) H2 O2 + 2e − → 2 O H−
−1 −2 0 −2
erate iodine.
(iv) H2 O2 + 2 O H− − 2e − → O2 + 2H2 O H2O2 → H2O + [O]
In (ii) and (iv) H2O2 acts as a reducing agent as it loses 2KI + H2O + [O] → 2KOH + I2
electrons to reduces the other species and itself under-
goes oxidation. 2KI + H2O2 → 2KOH + I2

35. Hydrogen peroxide acts both as an oxidizing and as a reduc- 39. Find the percentage strength of H2O2 in a sample marked “10
ing agent depending upon the nature of the reacting spe- volumes.”
cies. In which of the following cases H2O2 acts as a reducing (a) 0.3% (b) 0.5% (c) 0.1% (d) 0.15%
agent in acidic medium? Solution
(a) MnO4− (b) Cr2O72− (c) SO2−
3 (d) KI
(a) 1 L of 10 volume H2O2 solution on decomposition gives
(JEE Main Online 2014)
10 L of oxygen at STP.
Solution Now, 22.4 L of O2 at STP will be obtained from H2O2 = 68 g
(a) 2MnO 4− + 6H+ + 5H2O2 → 2Mn2 + + SO2 + 8H2O Therefore, 10 L of O2 at STP will be obtained from H2O2 =
68 × 10
In this reaction KMnO4 is reduced from +7 to +2 oxida- = 30.36 g
22.4
tion state.Thus, it acts as a oxidizing agent and H2O2 act
as a reducing agent in acidic medium. % strength of H2O2 = 0.303%

36. From the following statements regarding H2O2, choose the 40. A mixture of hydrazine and H2O2 with which of the following
incorrect statement. catalyst is used as rocket propellant?
(a) It decomposes on exposure to light. (a) Fe(II) (b) Cu(II) (c) Cr(III) (d) Fe(III)
(b) It has to be stored in plastic or wax lined glass bottles in Solution
dark.
(b) The reaction between hydrazine and H2O2 is highly exo-
(c) It has to be kept away from dust.
thermic and is accompanied by a large increase in the
(d) It can act only as an oxidizing agent.
volume of the products and hence this mixture is used
(JEE Main 2015)
as a rocket propellant.
Solution
Cu(II)
(d) The oxidation state of oxygen in H2O2 is −1 (peroxide). NH2NH2 (l)+2H2O2 (l) 
→ N2 (g)+ 4H2O(g)
So, oxygen can increase and decrease its oxidation num-
ber which means it can act as a reducing as well as an 41. The O O H bond angle in H2O2 in the gas phase is
oxidizing agent. (a) 106° (b) 109°28′ (c) 120° (d) 94.8°
37. H2O2 is manufactured industrially by Solution
(a) the action of H2O2 on BaO2 (d) The structure of H2O2 in the gaseous phase is
(b) the action of H2SO4 on Na2O2
(c) electrolysis of 50% H2SO4
(d) burning hydrogen in excess of oxygen
Solution
(c) Industrial preparation of H2O2 O 95.0 pm
A 50% solution of H2SO4 is electrolyzed in a cell. As a H
147.4 pm 94.8°
result peroxodisulphuric acid is formed at the anode and
O
hydrogen is evolved at cathode. H
H2SO 4 → HSO 4− + H+

At anode: 2HSO 4− → H2S2O8 + 2e − 111.5°


Peroxodisulphuric acid

At cathode: 2H+ + 2e– → H2


Hydrogen peroxide is a dihydroxy compound (H O
Peroxodisulphuric acid is taken out from the cell and is
O H) and the O O linkage is known as a peroxide
then hydrolyzed with water to give hydrogen peroxide
linkage. It is a non-linear molecule as the two O H
as follows.

Chapter-9.indd 294 7/29/2016 3:02:34 PM


Advanced Level Problems 295

bonds are in different planes. The interplanar (dihedral) Hydrogen as a Fuel


angle is 111.5° in the gaseous phase, but it is reduced
44. Which of the following could act as a propellant for rockets?
to 90.2° in the crystalline state because of hydrogen
(a) Liquid hydrogen + liquid nitrogen
bonding.
(b) Liquid oxygen + liquid argon
42. The bleaching property of hydrogen peroxide is due to its (c) Liquid hydrogen + liquid oxygen
(a) acidic nature (d) Liquid nitrogen + liquid oxygen
(b) ability to liberate nascent oxygen
Solution
(c) reducing nature
(d) ability to liberate nascent hydrogen (c) Liquid hydrogen has already been used as rocket fuel.
The chemical reaction involved is:
Solution
H2 (g) + 21 O2 (g) → H2O(l) + 286 kJ
(b) The bleaching property of hydrogen peroxide is due to
its ability to liberate nascent oxygen. Bleaching nature of Both reactants H2 and O2 are stored as liquids in separate
hydrogen peroxide is due to its oxidation. tanks. The tank holds 1.5 × 106 L of liquid hydrogen. The
oxygen tank carries 5.4 × 105 L of liquid oxygen. During
H2O2 → H2O + [O]
Nascent the “lift off” operation, these propellants power shuttle’s
oxygen main engine for about 8.5 min. Here, liquid hydrogen is
consumed at the rate nearly 3000 L s−1.
Coloured matter + [O] → Colourless
Nascent (Bleached)
oxygen 45. Hydrogen bomb is based on the principle of
(a) nuclear fission (b) natural radioactivity.
43. H2O2 used in rockets has the concentration (c) nuclear fusion (d) artificial radioactivity
(a) 50% (b) 70% (c) 30% (d) 90%
Solution
Solution (c) Hydrogen bomb releases a greater amount of energy
(a) H2O2 used in rockets has the concentration of 50%. through the process of nuclear fusion.

ADVANCED LEVEL PROBLEMS


1. The reagent(s) used for softening the temporary hardness of 3. Which of the following processes can be used for preparation
water is (are) of H2 gas?
(a) Ca3(PO4)2 (b) Ca(OH)2 (c) Na2CO3 (d) NaOCl (a) Dissolving LiH in water.
(IIT-JEE 2010) (b) Reaction of Al with NaOH solution.
(c) Reaction of Zn with dilute H2SO4.
Solution (d) Electrolysis of H2O in the presence of KOH.
(b), (c), (d) Ca(OH)2 is used in Clark’s method.
Solution
Ca(HCO3 )2 + Ca(OH)2 → 2CaCO3 + 2H2O (a), (b), (c), (d) All of the above methods can be used for the
Other methods are: preparation of H2 gas.

NaOCl + H2O ⇔ HOCl + NaOH (a) LiH(s) + H2O(l) → H2 (g) + LiOH(aq)

OH− + HCO−3 → CO23 − + H2O (b) 2Al + 2NaOH + 6H2O → 2Na + 2[Al(OH)4 ]− + 3H2 (g)

Ca(HCO3 )2 + Na2CO3 → CaCO + 2NaHCO3 (c) Zn(s) + H2SO 4 (aq) → ZnSO 4 (aq) + H2 (g)

2. Hydrogen bonding plays a central role in the following Electrolysis of H2O is best method formation of 99.97%
phenomena. pure H2. H2O is mixed with NaOH or KOH.
(a) Ice floats in water. At the cathode.
(b) Higher Lewis basicity of primary amines than tertiary
amines in aqueous solutions. 2H2O + 2e − → 2OH− + H2
(c) Formic acid is more acidic than acetic acid.
At the anode
(d) Dimerization of acetic acid in benzene.
(JEE Advanced 2014) 1
2OH− → 2H2O + O2 + 2e −
Solution 2
(a), (b), (d) Due to hydrogen bonding, ice has cage-like struc- Overall reaction
ture. In the case of amines, the order is 1° > 2° > 3° as primary
1
amine forms maximum hydrogen bonds compared to sec- H2O → H2 + O2
ondary and tertiary. Acetic acid dimerize in benzene due to 2
hydrogen bonding.

Chapter-9.indd 295 7/29/2016 3:02:37 PM


296 Chapter 9 Hydrogen

PRACTICE EXERCISE
Level I 11. Hydrogen has the tendency to gain one electron to acquire
helium configuration. In this respect, it resembles
Single Correct Choice Type (a) alkali metals. (b) carbon.
1. Reaction between the following pairs produces hydrogen (c) alkaline earth metals. (d) halogens.
except
12. Which gas is passed through in Merck’s process for making
(a) Cu + HCl (b) Fe + dil. H2SO4
hydrogen peroxide?
(c) Mg + Steam (d) Na + Alcohol
(a) CO (b) O2 (c) CO2 (d) O3
2. Which is more reactive?
13. H2O2 reduces K3[Fe(CN)6] in
(a) Ordinary hydrogen (b) ortho-Hydrogen
(a) neutral solution. (b) acidic solution.
(c) Nascent hydrogen (d) Heavy hydrogen
(c) alkaline solution. (d) non-polar medium.
3. Calgon used as a water softener is 14. Both temporary and permanent hardness in water is
(a) Na2[Na4(PO3)6] (b) Na4[Na2(PO3)6] removed by
(c) Na2[Na4(PO4)5] (d) Na4[Na4(PO4)6] (a) boiling. (b) filtration.
4. Non-combustible hydride is (c) distillation. (d) decantation.
(a) NH3 (b) PH3 (c) AsH3 (d) SbH3 15. ortho-Hydrogen and para-hydrogen resemble in which of
5. ortho- and para-hydrogens differ in the following property?
(a) nuclear charge. (b) nuclear reaction. (a) Thermal conductivity
(c) electron spin. (d) proton spin. (b) Magnetic properties
(c) Chemical properties
6. Hydrogen molecule differs from chlorine molecule in the fol- (d) Heat capacity
lowing respect:
(a) Hydrogen molecule is non-polar but chlorine molecule 16. The ionization energy order for H, H+, H− is
is polar. (a) H < H+< H− (b) H− > H > H+
(b) Hydrogen molecule is polar while chlorine molecule is (c) H− < H (d) H+ > H > H−
non-polar. 17. The most dangerous method of preparing hydrogen would
(c) Hydrogen molecule can form intermolecular hydrogen be by the action of HCl and
bonds but chlorine molecule does not. (a) Zn (b) Fe (c) K (d) Al
(d) Hydrogen molecule cannot participate in coordination
bond formation but chlorine molecule can. 18. Which of the following is not a water softener?
(a) Calgon (b) Permutit
7. Permutit is the technical name given to (c) Na2CO3 (d) Na2SO4
(a) aluminates of calcium and sodium.
(b) hydrated silicate of aluminium and sodium. 19. The most common way to attain stability by the H atom is
(c) silicates of calcium and magnesium. (a) forming covalent bond.
(d) silicates of calcium and sodium. (b) forming H+.
(c) forming H−.
8. When electric current is passed through an ionic hydride in (d) both (b) and (c).
the molten state,
(a) hydrogen is liberated at the cathode. 20. On burning hydrogen in air, the color of flame is
(b) hydrogen is liberated at the anode. (a) green. (b) light blue.
(c) no reaction takes place. (c) yellow. (d) None of these.
(d) hydride ion migrates towards the cathode. 21. What is more for water than for heavy water?
9. When two ice cubes are pressed over each other they unite (a) Surface tension
to form one cube. Which of the following forces are responsi- (b) Viscosity
ble to hold them together? (c) Latent heat of vaporization
(a) Ionic interaction (d) Melting point
(b) van der Waals forces 22. Proton and deuteron are differentiated by their
(c) Covalent interaction (a) number of protons.
(d) Hydrogen bond formation (b) number of neutrons.
10. Chemical A is used for water softening to remove temporary (c) number of electrons.
hardness. Chemical A reacts with sodium carbonate to gen- (d) charges.
erate caustic soda. When CO2 is bubbled through a solution 23. The bond angle and dipole moment of water, respectively,
A, it turns cloudy. What is the chemical formula of A? are
(a) CaO (b) Ca(OH)2 (a) 109.5°, 1.84 D. (b) 107.5°, 1.56 D.
(c) CaCO3 (d) Ca(HCO3)2 (c) 104.5°, 1.84 D. (d) 102.5°, 1.56 D.

Chapter-9.indd 296 7/29/2016 3:02:37 PM


Practice Exercise 297

24. Pure hydrogen is obtained by electrolysis of a solution of (c) At the top of carbon family
(a) KOH (b) NaOH (d) None of these
(c) Ba(OH)2 (d) All of these 39. TiH1.73 is an example of
25. Surface water contains (a) ionic hydride. (b) covalent hydride.
(a) suspended impurities. (c) metallic hydride. (d) polymeric hydride.
(b) organic impurities. 40. The ionization constant for acetic acid is
(c) salt. (a) three times larger than deuteric acid.
(d) salt and organic compound. (b) one-third of the deuteric acid.
26. The component present in greater proportion in water gas is (c) equal to deuteric acid.
(a) CH4 (b) CO2 (c) CO (d) H2 (d) two times larger than deuteric acid.
27. The alum used for purifying water is 41. Ordinary hydrogen at room temperature is a mixture of
(a) ferric alum. (b) chrome alum. (a) 75% o-Hydrogen + 25% p-Hydrogen.
(c) potash alum. (d) ammonium alum. (b) 25% o-Hydrogen + 75% p-Hydrogen.
28. When CO2 is bubbled through a solution of barium peroxide (c) 50% o-Hydrogen + 50% p-Hydrogen.
in water, (d) 1% o-Hydrogen + 99% p-Hydrogen.
(a) O2 is released. 42. The name of the perhydrol is given to
(b) carbonic acid is formed. (a) solution of Na2O2 in ether.
(c) H2O2 is formed. (b) dilute solution of phenol in ether.
(d) no reaction occurs. (c) dilute solution of H2O2 in water.
29. Which of the following on oxidation gives H2O2? (d) dilute solution of CrO4 in ether.
(a) 2-Ethylanthraquinol (b) 2-Ethylanthraquinone 43. H2O2 is always stored in black bottles because
(c) Anthracene (d) 2-Ethylanthracene (a) it is highly unstable.
30. Hydrogen is evolved by the action of cold dilute HNO3 on (b) its enthalpy of decomposition is high.
(a) Mn. (b) Cu. (c) Fe. (d) Al. (c) it undergoes auto-oxidation on prolonged standing.
(d) None of these.
31. The velocity of neutrons in nuclear reactor is slowed down by
(a) heavy water (D2O). (b) ordinary water (H2O). 44. Heavy water is called as such because it
(c) zinc rod. (d) fused caustic soda. (a) is a dimer of common water.
(b) is an oxide of protium.
32. Which of the following is not hard water? (c) has a heavy or bad taste.
(a) Water containing CaCl2 (d) has a heavier isotope of hydrogen.
(b) Water containing dil. HCl
45. Hydrides of elements of Groups 3–5 are generally called
(c) Water containing MgSO4
(a) interstitial hydrides. (b) ionic hydrides.
(d) Water containing Ca(HCO3)2
(c) polymeric hydrides. (d) complex hydrides.
33. Alkali metal hydrides react with water to give
46. Which of the halogens has maximum affinity for hydrogen?
(a) acidic solution. (b) basic solution.
(a) F2 (b) Cl2 (c) Br2 (d) I2
(c) neutral solution. (d) hydride ion.
47. What is formed when calcium carbide reacts with heavy
34. The colour of hydrogen is
water?
(a) black. (b) yellow.
(a) C2D2 (b) CaD2 (c) Ca2D2O (d) CD2
(c) orange. (d) colourless.
48. The percentage by weight of hydrogen in H2O2 is
35. Which of the following produces hydrolith with dihydrogen?
(a) 5.88 (b) 6.25 (c) 25 (d) 50
(a) Mg (b) Al (c) Cu (d) Ca
49. Which of the following species has highest bond enthalpy?
36. Limiting compositions of f-block hydrides are
(a) H2 (b) T2 (c) D2 (d) Cl2
(a) MH2 and MH3 (b) MH3 and MH5
(c) MH2 and MH8 (d) MH2 and MH6 50. Ionization enthalpy of hydrogen is
(a) equal to that of chlorine.
37. Which of the following is a true structure of H2O2?
97°
(b) lesser than that of chlorine.
180°
(a) (b) (c) slightly higher than that of chlorine.
H O O H H (d) much higher than that of chlorine.
O O
H 51. Decomposition of H2O2 is prevented by
H (a) NaOH (b) MnO2
(c) O O (d) H O
H O (c) acetanilide (d) oxalic acid
H
52. The compound sodium polymetaphosphate (Na[Na4(PO3)6])
38. Which position for hydrogen explains all its properties?
is called calgon because
(a) At the top of halogen
(a) it was developed by the scientist named Challaghan.
(b) At the top of alkali metals
(b) it was developed first in California.

Chapter-9.indd 297 7/29/2016 3:02:39 PM


298 Chapter 9 Hydrogen

(c) it refers to calcium gone. 64. In the preparation of dihydrogen by the action of steam on
(d) it is based on the name of the company which devel- coke, a mixture of CO and H2 gas is formed. This mixture is
oped it. known as
(a) syngas (b) producer gas
53. High dipole moment of water (1.03 D) justifies that
(c) water gas (d) industrial gas
(a) it is not linear molecule.
(b) it is a universal solvent. 65. Which of the following elements form metallic hydrides?
(c) it has higher density than ice. (a) Cu (b) Pd (c) Li (d) Sc
(d) it is neutral toward litmus.
66. Which of the properties of water given below is/are false?
54. Which one of the following removes temporary hardness of (a) Water is a universal solvent.
water? (b) Hydrogen bonding is present to a large extent in liquid
(a) Slaked lime (b) Plaster of Paris water.
(c) Cuprous oxide (d) Hydrolith (c) There is no hydrogen bonding in the frozen state of water.
(d) Frozen water is heavier than liquid water.
55. Which is the poorest reducing agent?
(a) Nascent hydrogen 67. Permanent hardness of water is due to the presence of
(b) Atomic hydrogen (a) chlorides of Ca and Mg in water.
(c) Dihydrogen (b) sulphates of Ca and Mg in water.
(d) All of these have the same reducing strength (c) hydrogen carbonates of Ca and Mg in water.
(d) carbonates of alkali metals in water.
56. Of the two solvents, H2O and D2O, sodium chloride dissolves
(a) equally in both (b) more in D2O 68. Temporary hardness of water can be removed by which of
(c) more in H2O (d) only in H2O the following processes?
(a) Lime–soda process (b) Boiling
57. The approximate mass of tritium oxide molecule is
(c) Clark’s process (d) Ion exchange method
(a) 18 u (b) 20 u (c) 22 u (d) 24 u
69. Which of the following properties of metallic hydrides are
58. Systematic name of H2O is
similar to those of their parent metal?
(a) water (b) hydrogen oxide
(a) Hardness (b) Metallic luster
(c) oxidane (d) None of these
(c) Electrical conductivity (d) Magnetic property
59. Which of the following is correct about heavy water?
70. Hydrogen can be obtained from water by
(a) Water at 4°C having maximum density is known as heavy
(a) reaction with metal oxides.
water.
(b) reaction with non-metal oxides.
(b) It is heavier than water.
(c) reaction with metals.
(c) It is formed by the combination of heavier isotope of
(d) reaction with metal hydrides.
hydrogen and oxygen.
(d) None of these. 71. Pick out the correct statement(s).
(a) Natural hard water is toxic in nature.
60. Free hydrogen is found in
(b) Natural hard water does not produce lather with soap.
(a) acid (b) water
(c) Water containing some potash alum is hard water.
(c) marsh gas (d) water gas
(d) Water obtained by zeolite process is not pure water.
61. The atom of oxygen lost by H2O2 molecule during oxidation
72. The s-block hydrides with covalent polymeric structure are
reaction is that which is linked through
(a) LiH (b) BeH2 (c) NaH (d) MgH2
(a) an electrovalent bond (b) a covalent bond
(c) a coordinate bond (d) a hydrogen bond 73. When zeolite, which is hydrated sodium aluminium silicate,
is treated with hard water, the sodium ions are exchanged
62. The oxidation states of the most electronegative element in
with
the products of the reaction, BaO2 with dil. H2SO4 are
(a) H+ ions (b) Ca2+ ions
(a) 0 and −1 (b) −1 and −2
(c) SO2−4 ions (d) Mg2+ ions
(c) −2 and 0 (d) −2 and + 1
Passage Type
Level II
Paragraph for Questions 74 to 77: Hydrogen has three isotopes,
Multiple Correct Choice Type
protium, 11H, deuterium, 21H or D and tritium, 31H or T, which differ from
63. Hydrogen atom may attain stability by one another in the number of neutrons. Naturally occurring hydro-
(a) forming an electron pair (covalent) bond with another gen contains 99.986% of the 11H isotope, 0.014% of 21H and 7 × 10−16
atom. % of 31H; so, the properties of hydrogen are essentially those of the
(b) losing an electron to form H+. lightest isotope. Protium is by far the most abundant and tritium is
(c) gaining an electron to form H−. radioactive and decays by b emission. Their only differences in chem-
(d) None of these ical properties are the rates of reactions and equilibrium constants.

Chapter-9.indd 298 7/29/2016 3:02:41 PM


Answer Key 299

74. The isotope effect arises due to (c) Hydrogen peroxide (r) Strong reducing agent
(a) difference in number of electrons. (d) Heavy water (s) Concentrated by distilla-
(b) difference in number of protons. tion under reduced pres-
(c) difference in mass. sure
(d) difference in properties.
81. Match the compound with the composition/reaction it
75. Which of the following pairs shows maximum isotope effect? undergoes.
(a) 1
and 21D (b) 35 37 Column I Column II
1H 17 Cl and 17 Cl
(a) Syngas (p) Causes sequestration of Ca2+ and
12 14
(c) 6 C and 6 C (d) None of these Mg2+ ions
(b) Hydride gap (q) Softening hard water
76. Which of the following properties has incorrect order?
(c) Calgon (r) Elements of Groups 7, 8 and 9
(a) H2 < D2 < T2: Boiling point order.
(d) Permutit (s) CO + H2
(b) H2 < D2 < T2: Freezing point order.
(c) H2 < D2 < T2: Latent heat of vapourization. 82. Match the compound with its characteristics.
(d) T2O > H2O > D2O: Equilibrium constant for dissociation. Column I Column II
(a) H2O (p) Complex metal hydride
77. The properties of hydrogen are essentially those of
(b) NaBH4 (q) Oxidizing agent
3 2
(a) 1H (b) 1H (c) H2O2 (r) Amphoteric
(c) 1
(d) None of these. (d) H2 (s) Reducing agent
1H
Integer Type
Matrix-Match Type
83. The number of neutrons in deuterium is ___.
78. Match the hydride with its property.
Column I Column II 84. The number of protons that can be accepted by hydrazine is
(a) CaH2 (p) Conducts electricity when melted ___.
(b) CuH (q) Reaction of formation is endothermic
85. The number of electron-rich hydrides among the following is
(c) SiH4 (r) Powerful reducing agent but very less
___.
reaction with water
(d) LiH (s) Produces hydrogen when dissolved in CH4, NH3, PH3, H2O, H2S, BH3, HF, AlH3, AsH3
water
86. The number of isotopes of hydrogen is ___.
79. Match the hydride with its type.
87. The mass number of the element obtained when tritium
Column I Column II
undergoes b-decay is ___.
(a) CaH2 (p) Intermediate
(b) B2H6 (q) Interstitial 88. Maximum number of hydrogen bonding in H2O is ___.
(c) LaH3 (r) Covalent
89. The sum of protons, electrons and neutrons in the heaviest
(d) LiAlH4 (s) Ionic
isotope of hydrogen is ___.
80. Match the compound with its reaction/process.
90. The number of nucleons in D2 molecule is ___.
Column I Column II
(a) Hard water (p) Calgons’ method
(b) Atomic hydrogen (q) Used as tracer in the study
of reaction mechanism

ANSWER KEY
Level I
1. (a) 2. (c) 3. (a) 4. (a) 5. (d) 6. (d)
7. (b) 8. (b) 9. (d) 10. (b) 11. (d) 12. (c)
13. (c) 14. (c) 15. (c) 16. (c) 17. (c) 18. (d)
19. (a) 20. (b) 21. (a) 22. (b) 23. (c) 24. (c)
25. (a) 26. (d) 27. (c) 28. (c) 29. (a) 30. (a)
31. (a) 32. (b) 33. (b) 34. (d) 35. (d) 36. (a)
37. (b) 38. (d) 39. (c) 40. (a) 41. (a) 42. (c)
43. (c) 44. (d) 45. (a) 46. (a) 47. (a) 48. (a)
49. (b) 50. (c) 51. (c) 52. (c) 53. (a) 54. (a)
55. (c) 56. (c) 57. (c) 58. (c) 59. (c) 60. (d)
61. (c) 62. (b)

Chapter-9.indd 299 7/29/2016 3:02:42 PM


300 Chapter 9 Hydrogen

Level II
63. (a), (b), (c) 64. (a), (c) 65. (a), (b), (d) 66. (c), (d) 67. (a), (b) 68. (a), (b), (c)
69. (a), (b), (c), (d) 70. (c), (d) 71. (b), (c), (d) 72. (b), (d) 73. (b), (d) 74. (c)
75. (a) 76. (d) 77. (c) 78. (a) → p, s; (b) → q; (c) → r; (d) → p, s
79. (a) → s; (b) → r; (c) → q; (d) → p 80. (a) → p; (b) → r; (c) → s; (d) → q 81. (a) → s; (b) → r; (c) → p, q; (d) → q
82. (a) → q, r, s; (b) → p, s; (c) → q, s; (d) → q, s 83. (1) 84. (2) 85. (6)
86. (3) 87. (3) 88. (4) 89. (4) 90. (4)

HINTS AND EXPLANATIONS


Level I 6. (d) Hydrogen molecule cannot participate in coordination
bond formation but chlorine molecule can because
Single Correct Choice Type there is no unshared pair of electrons in hydrogen mol-
1. (a) Cu + HCl → No evolution of hydrogen gas because cop- ecule (H2) whereas chlorine molecule has six unshared
per lies below hydrogen in electrochemical series. Hence, electron pairs as shown below.
it acts as an oxidizing agent. For the rest of the pairs, the H H Cl Cl
reactions are as follows:
Coordinate bond is formed by sharing of an electron
Fe + dil.H2SO 4 → FeSO 4 + H2 pair between the atoms where the shared pair of elec-
trons is contributed by only one of the atom.
Heat
Mg + H2O(steam) 
→ MgO + H2 7. (b) Permutit can be represented by the general formula,
Na2Z where Z is Al2Si2O8 ⋅ xH2O.
C2H5OH + Na → C2H5O −Na+ + 21 H2
Alcohol Alkoxide 8. (b) When the salts of hydrogen gas and halogens with
2. (c) Nascent hydrogen is a form of hydrogen at the moment alkali metals in the molten state are electrolyzed, both
of its generation from chemical reactions in aqueous solu- of them are liberated at the anode.
tion. It is also called newly born hydrogen. This form of Electrolysis
Anode: 2NaCl(l) 
→ 2Na + Cl2
hydrogen is more reactive and better reducing agent than
ordinary dihydrogen. Electrolysis
Anode: 2NaCl(l) 
→ 2Na + H2
Nascent hydrogen reduces KMnO4:
9. (d) The intermolecular hydrogen bonding is responsible for
Zn + H2SO 4 → ZnSO 4 + 2[H] the formation of one cube.

2KMnO 4 + 3H2SO 4 + 2[H] → K 2SO 4 + 2MnSO 4 + 4H2O 10. (b) The chemical formula of A is Ca(OH)2. Chemical A reacts
with sodium carbonate to generate caustic soda.
Molecular dihydrogen does not reduce KMnO4.
Ca(OH)2 + Na2CO3 → 2NaOH + CaCO3
3. (a) Calgon is sodium hexametaphosphate Na2[Na4(PO3)6]. It Caustic soda
is used as water softener. When CO2 is bubbled through a solution A, it turns
4. (a) The thermal stability of the hydrides decreases in the cloudy.
order: Ca(OH)2 + CO2 → CaCO3 + H2O
NH3 > PH3 > AsH3 > SbH3 > BiH3 Cloudy

11. (d) Just like halogens, hydrogen needs one electron to


This is because the strength of M H bond decreases
attain the configuration of nearest noble gas.
down the group due to increase in the size of central atom.
NH3 is non-combustible because the N H bond strength 12. (c) In Merck’s process, H2O2 is obtained by passing a cur-
is maximum due to small size of nitrogen. rent of CO2 through a cold solution of barium peroxide
in water. Barium carbonate formed is insoluble and is
5. (d) ortho- and para-hydrogen differ in proton spin. filtered off.
ortho-hydrogen: Spin of protons or nucleus in same
BaO2 + CO2 + H2O → H2O2 + BaCO3
direction.
para-hydrogen: Spin of protons or nucleus in opposite 13. (c) H2O2 reduces K3[Fe(CN)6] in alkaline solution. H2O2
direction. reduces potassium ferricyanide in alkaline solution to
potassium ferrocyanide.
2K 3Fe(CN)6 + 2KOH → 2K 4Fe(CN)6 + H2O + [O]
H2O2 + [O] → H2O + O2
ortho-hydrogen para-hydrogen 2K 3Fe(CN)6 + 2KOH + H2O2 → 2K 4Fe(CN)6 + 2H2O + O2

Chapter-9.indd 300 7/29/2016 3:02:45 PM


Hints and Explanations 301

14. (c) The best methods would be distillation or ion-exchange OH O


for washing water. For drinking water, reverse osmosis is C2H5 O2 C2H5
the best method.
+ H2O2
15. (c) The two forms differ in physical properties such as spe-
H2/Pd
cific heat, thermal conductivity, boiling point, and oth-
ers, but their chemical properties are the same. OH O
2-Ethyl anthraquinol Quinone
16. (c) Ionization energy term is not associated with H+.
17. (c) Potassium (K) is a very reactive metal. The reaction of 30. (a) Manganese liberates hydrogen with cold dilute nitric
HCl with K is highly vigorous and H2 liberated immedi- acid.
ately catches fire, which can cause accident. Mn + 2HNO3 → Mn(NO3 )2 + H2
2K + 2HCl → H2 + 2KCl 3Cu + 8HNO3 → 3Cu(NO3 )2 + 2NO + 4H2O
18. (d) Na2SO4 is not a water softener because it cannot precip-
4Fe + 10HNO3 → 4Fe(NO3 )2 + N2O + 5H2O
itate calcium or magnesium ions.
CaCl2 + Na2CO3 → CaCO3 + 2NaCl 8Al + 3NO3− + 18H2O + 5OH− → 8Al(OH)4 + 3NH3
(ppt. )
CaCl2 + Na2SO 4 → CaSO 4 + 2NaCl
(No ppt. ) 31. (a) Heavy water is used in nuclear reactor for slowing down
the fast-moving neutrons. Neutrons are used for bring-
CaSO4 formed from Na2SO4 is soluble in water. So we ing about fission of uranium atoms. But, for this purpose
cannot remove Ca2+ or Mg2+ ions present in water. their speed should be slowed down. This is done by
19. (a) Most of the compound of H-atom is covalent in nature. passing them through heavy water.

20. (b) Hydrogen gas is highly combustible and burns in air or 32. (b) Water that does not lather with soap is known as hard
oxygen with pale blue flame to form water. water. Hardness of water is due to the presence of bicar-
bonates, chlorides and sulphates of calcium and mag-
2H2 + O2 → 2H2O nesium in it. However, when hard water is treated with
dil. HCl, the soluble bicarbonates of Ca and Mg get pre-
21. (a) Surface tension is more for water than for heavy water
cipitated as CaCl2 and MgCl2, respectively, which can be
because H2O is more rapidly adsorbed on to the surface
removed.
than D2O. The remaining physical properties, such as
viscosity, latent heat of vaporization, and melting point 33. (b) Alkali metal hydrides react with water to give basic
are less for water than heavy water. solution.
NaH + H2O → NaOH + H2
22. (b) Proton: +1p1 while deuteron: +1p2. (Basic )

23. (c) Dipole moment of water is 1.84 D. Its bond angle is 34. (d) Hydrogen is colourless, odourless and almost insoluble
104.5°. in water.

24. (c) By the electrolysis of warm aqueous solution of Ba(OH)2 35. (d) Calcium reacts with dihydrogen to produce hydrolith.
between nickel electrodes, hydrogen of very high purity Ca + H2 → CaH2.
(more than 99.95%) is obtained. 36. (a) General f-block elements form interstitial hydrides with
25. (a) Surface water contains suspended impurities because the limiting composition of MH2 and MH3, because
water usually carries many soluble impurities along f-block elements form stable +3 oxidation state.
with insoluble impurities in suspension as well as in col- 37. (b) The structure of H2O2 in the gaseous phase is
loidal state.
26. (d) Water gas is a mixture of CO and H2. It contains greater
proportion of H2 than CO.
1000 ° C
C(s) +H2O(g) 
→ CO +H2

  O 95.0 pm
Water gas H
147.4 pm 94.8°
27. (c) Potash alum is used for purifying water. O
H
28. (c) The reaction involved is
BaO2 + CO2 + H2O → H2O2 + BaCO3
111.5°
29. (a) When 2-ethylanthraquinol is dissolved in a mixture of
benzene and cyclohexanol and air is passed, it is oxi-
dized to 2-ethylanthraquinone and hydrogen peroxide.

Chapter-9.indd 301 7/29/2016 3:02:48 PM


302 Chapter 9 Hydrogen

Hydrogen peroxide is a dihydroxy compound (H O 49. (b) Conceptual.


O H) and the O O linkage is known as a peroxide link-
50. (c) Ionization enthalpy of hydrogen is slightly higher than
age. It is a non-linear molecule as the two O H bonds
that of chlorine. In hydrogen, only one electron is pres-
are in different planes. The interplanar (dihedral) angle is
ent but in chlorine there are 17 electrons present; hence,
111.5° in the gaseous phase, but it is reduced to 90.2° in
ionization enthalpy is slightly higher of hydrogen than
the crystalline state because of hydrogen bonding.
that of chlorine.
38. (d) H2 resembles in some of its properties with alkali met- 51. (c) A small amount of phosphoric acid or glycerol or acet-
als of Group 1 as well as with halogens of Group 17. anilide is added in H2O2 during its storage. These addi-
However, the oxide of hydrogen, that is, H2O has neu- tives act as negative catalyst for the decomposition of
tral character that neither resembles with Na2O or K2O, H2O2 and thus decomposition of H2O2 is checked.
which are basic in nature, nor with halogen oxides,
Cl2O7, which are acidic in nature. Thus, H2 has been 52. (c) The compound sodium hexametaphosphate
placed on the top of periodic table. (Na2[Na4(PO3)6]) is called calgon because it refers to
“calcium gone”. This compound is used to remove Ca2+
39. (c) Transition elements form interstitial or metallic hydrides
and Mg2+ ions present in hard water.
with hydrogen as hydrogen occupies interstitial places
in the metal lattice. These are non-stoichiometric com- 53. (a) High dipole moment of water suggests that it is not a
pounds and lose H2 easily; and thus act as strong reduc- linear molecule.
ing agents, for example, TiH1.73, GeH2.7, PdH0.60, LaH2.8,
54. (a) In Clark’s method that is used for removing tempo-
among others.
rary hardness of water, calculated amounts of calcium
40. (a) The ionization constant for acetic acid is three times hydroxide (lime) are added to the water. It reacts with
larger than deuteric acid due to isotopic effect, that is, soluble calcium and magnesium bicarbonates to form
the difference in atomic sizes and atomic masses of ace- insoluble precipitates of calcium carbonate and magne-
tic acid and deuteric acid. sium hydroxide that are removed by filtration.
41. (a) At room temperature, ordinary hydrogen contains Ca(HCO3 )2 + Ca( OH)2 → 2CaCO3 ↓ + 2 H2O
about 75% ortho-hydrogen and 25% para-hydrogen.
Mg(HCO3 )2 + 2Ca(OH)2 → 2CaCO3 ↓ + Mg(OH)2 ↓ + 2H2O
42. (c) Perhydrol is the trade name for H2O2, which is used as
an antiseptic for washing wounds, teeth and ears. 55. (c) Dihydrogen (H2) is a poorer reducing agent than nas-
cent and atomic hydrogen.
43. (c) H2O2 readily undergoes auto-oxidation on prolonged
standing due to its unstable nature. The decomposition 56. (c) The rate of dissolution of NaCl in water is more than that
or auto-oxidation is further accelerated by sunlight and of D2O (heavy water) due to isotopic effect.
rough surface. In order to check it, hydrogen peroxide is
stored in black bottles. H2O D2O
44. (d) Heavy water is called as such because it has a heavier Solubility of NaCl 359 g L−1 305 g L−1
isotope of hydrogen.
57. (c) The atomic mass of an element may be defined as the
45. (a) Hydrides of elements of Groups 3–5 are generally called average relative mass of an atom of the element as com-
interstitial hydrides because in these hydrides, hydro- pared with the mass of an atom of carbon (12C) taken as
gen atoms, being small in size, occupy some interstitial 12 u.
sites in the metallic lattice producing distortion without Now, mass number of O = 16 and of 13 H = 3, so mass of
any change in its type. T2O = 2 × 3 + 16 = 6 + 16 = 22 u.
46. (a) Owing to its small size, high electronegativity, low bond 58. (c) Systematic name of H2O is oxidane.
energy and high oxidation potential, F2 has maximum
affinity for hydrogen. 59. (c) Heavy water is formed by the combination of heavier
isotope of hydrogen and oxygen.
47. (a) When calcium carbide reacts with water, it forms cal-
cium hydroxide and acetylene. 2D2 + O2 → 2D2O
Deutirium Heavy water
CaC2 + 2H2O → Ca(OH)2 + C2H2
When calcium carbide reacts with heavy water, it forms 60. (d) Free hydrogen is found in water gas
deuterated acetylene and calcium hydroxide. 1000 ° C
C(s) +H2O(g) 
→ CO +H2

  
CaC2 + 2D2O → Ca(OD)2 + C2D2 Water gas

48. (a) Molecular weight of H2O2 = 2 × 1 + 16 × 2 = 34 g 61. (c) The reaction involved is
2 g of hydrogen is present in H2O2 = 34 g H2O2 H2O O (Oxidation)
2 Coordinate
% weight of hydrogen = × 100 = 5.88%
34 bond

Chapter-9.indd 302 7/29/2016 3:02:50 PM


Hints and Explanations 303

62. (b) H2O2 can be prepared by adding barium peroxide to If water reacts with non-metal oxides then also H2 gas is not
dilute H2SO4 at 0 °C. Anhydrous BaO2 reacts slowly with formed.
H2SO4 and the reaction ceases after some time due to
CO2 + H2O → H2CO3 (carbonic acid)
the formation of a protective film of barium sulphate on
barium peroxide. It is therefore always advised to use SO2 + H2O → H2SO3 (sulphurous acid)
hydrated barium peroxide.
71. (b), (c), (d) Natural hard water is non-toxic in nature. Natural
BaO2 ⋅ 8H2O + H2SO 4 → BaSO 4 + 8H2O + H2O2 hard water does not produce lather with soap easily. Potash
alum is Al2(SO4)3 ⋅ K2SO4 ⋅ 24H2O.
The products are BaSO4, water and hydrogen peroxide. Water containing some potash alum is hard water
In these, oxygen is the most electronegative element because sulphates present in the potash alum are responsi-
and oxidation states are −1 in H2O2 and −2 in H2O. ble for hardness of water. Water obtained by zeolite process
−2
+1
O is not pure water. It removes only Ca2+ and Mg2+ ions, that is,
H −1 −1 hardness of water but it cannot remove any microbes pres-
O O +1 +1 +1 ent in the water.
H H H
72. (b), (d) BeH2 and MgH2 have covalent polymeric structure
Level II in which each H atom is involved in a three centered bond
and exists as a bridge between two beryllium atoms on both
Multiple Correct Choice Type sides.
63. (a), (b), (c)
H H H
(a) H + H → H − H (stable) (b) H → H+ + e − (stable)
Be Be Be Be
− −
(c) H + e → H (stable) H H H
64. (a), (c) The reaction is The small size and high charge makes Be2+ a highly polar-
C + H2O → CO + H2 izing ion that effectively polarizes the electron charge
Coke Steam

   cloud of H− ion permitting partial overlapping of orbitals
Water gas or syngas
of H− and Be2+ ions. This induces considerable covalent
Producer gas is a mixture of carbon monoxide and nitrogen. character in BeH2. The same is true in MgH2 but to a lesser
65. (a), (b), (d) Many transition metals absorb hydrogen to form extent.
metallic hydrides. Cu, Pd, and Sc belong to transition series. 73. (b), (d) The reactions involved are
So, these elements form metallic hydrides. Li belongs to
s-block and it forms ionic or saline hydrides. Na2Z + CaSO 4 (or CaCl2 ) → CaZ + Na2SO 4 ( or 2NaCl)
66. (c), (d) Hydrogen bonding is present in frozen state of water. Na2Z + MgCl2 (or MgSO 4 ) → MgZ + 2NaCl (or Na2SO 4 )
Frozen water is lighter than liquid water due to its lesser density. Na2Z + Ca(HCO3 )2 (or Mg(HCO3 )2 ) → CaZ (or MgZ) + 2NaHCO3
67. (a), (b) Temporary hardness is due to the presence of soluble
bicarbonates of calcium and magnesium. Permanent hard- Passage Type
ness is due to the presence of chlorides and sulphates of cal-
74. (c) The isotopic effect arises due to difference in atomic
cium and magnesium.
masses of the isotopes of an element. For isotopes of
68. (a), (b), (c) Temporary hardness of water can be removed by hydrogen:
lime-soda process, boiling and Clark’s process.
Isotope Mass Number
69. (a), (b), (c), (d) All the given properties of metallic hydrides
Protium 1
are similar to those of their parent metal because when metal
combines with hydride these properties do not change. Deuterium 2
Tritium 3
70. (c), (d) Reaction with metals
Zn + H2O → ZnO + H2 ↑
75. (a) Protium (11H) and deuterium (12 D) pairs show maximum
3Fe + 4H2O → Fe3O 4 + 4H2 ↑ isotopic effect. Protium reacts with chlorine nearly 13.4
Reaction with metal hydrides times faster than deuterium.
It is adsorbed more readily on activated charcoal than
LiH + H2O → LiOH + H2 ↑ deuterium.
If water reacts with metal oxides, then there is no liberation 76. (d) The correct order of equilibrium constant for dissocia-
of H2 gas. tion is H2O > D2O > T2O.
Na2O + H2O → 2NaOH
CaO + H2O → Ca(OH)2 77. (c) The properties of hydrogen are essentially those of the
lightest isotope, that is, 11H (protium).

Chapter-9.indd 303 7/29/2016 3:02:53 PM


304 Chapter 9 Hydrogen

Matrix-Match Type As oxidizing agent: The oxidizing character is because of


its electron-accepting tendency.
78. (a) ã p, s; (b) ã q; (c) ã r; (d) ã p, s
LiH(s)+H2O(l) → LiOH2 (aq) +H2 (g) 2H2O + 2e− → 2OH− + H2
CaH2 (s) + 2H2O(l) → Ca(OH)2 (aq) + 2H2 (g) 2H2O + 2Na → 2NaOH + H2

They are powerful reducing agents especially at high tem- As reducing agent: The reducing character of water is
peratures, though their reactivity towards water limits their due to its electron-releasing tendency.
usefulness.
2H2O → 4H+ + 4e− + O2
SiH4 is a powerful reducing agent.
2F2 + 2H2O → 4HF + O2
SiCl4 + 4NaH → SiH4 + 4NaCl
Reaction of formation of CuH is endothermic, and it is formed (b) NaBH4 is a complex metal hydride that can be prepared
by reducing Cu2+ with hypophosphorous acid. by reaction of sodium hydride (NaH) with aluminium
trichloride (AlCl3) in THF. NaBH4 acts as reducing agent.
79. (a) ã s; (b) ã r; (c) ã q; (d) ã p (c) Hydrogen peroxide can act both as an oxidizing as well
(a) CaH2 is ionic hydride. (b) B2H6 is intermediate. (c) LaH3 is as reducing agent. The change on oxidation number
an interstitial hydride. (d) LiAlH4 is a covalent hydride. accompanying the decomposition of H2O2 is as follows:
80. (a) ã p; (b) ã r; (c) ã s; (d) ã q Increase in
(a) Hard water is treated with Calgon’s method to remove oxidation
Ca2+ and Mg2+ ions. (b) Atomic hydrogen is used as tracer number
in the study of reaction mechanism. (c) Hydrogen peroxide
is a strong reducing agent. It is concentrated by distillation +1 −1 +1 −2 0
under reduced pressure. (d) Heavy water is used as tracer in H2O2 H2 O + 1 O2
2
the study of reaction mechanism.
Decrease in
81. (a) ã s; (b) ã r; (c) ã (p), (q); (d) ã (q)
oxidation
(a) Syngas or water gas is mainly a mixture of CO and H2. It is
number
made by blowing steam through the layers of red hot coal at
temperature 1000–1400 °C.
Since oxygen atom in H2O2 can undergo an increase as
C + H2O → CO + H2 well as decrease in oxidation number, therefore, it can
Coke Steam

  
Water gas or syngas act both as reducing as well as oxidizing agent. This is
(b) The metals of Groups 7, 8 and 9 do not form hydrides and supported by the following reactions:
form a gap. This is known as hydride gap. As reducing agent: Ag2O + H2O2 → 2Ag + H2O + O2
(c) Ca2+ and Mg2+ ions present in hard water react with cal- In this reaction, H2O2 acts as reducing agent and has
gon to form soluble complexes. reduced Ag2O to metallic Ag.
2Ca2 + + Na2 [Na4 (PO3 )6 ]2 − → 4Na+ + Na2 [C
Ca2 (PO3 )6 ]2 − As oxidizing agent: PbS + 4H2O2 → PbSO4 + 4H2O
Hardness Calgon Soluble complex
causing ion In this reaction, H2O2 has oxidized PbS to PbSO4
2Mg2 + + Na2 [Na4 (PO3 )6 ]2 − → 4Na+ + Na2 [Mg2 (PO3 )6 ]2 − (d) H2 acts as an oxidizing as well reducing agent.
Hardness Calgon Soluble complex
causing ion Oxidizing nature: H2 reacts with metals such as Na, Ca, Li
The complexes of calcium and magnesium so formed and others to form respective hydrides.
remain dissolved in water, but they do not cause hin- 525 K
2Na + H2 
→ 2NaH
drance in the formation of lather. This is because calcium 525 K
and magnesium ions are not free to react with soap, but Ca + H2 
→ CaH2
these have been tied up to form stable complexes. This Reducing nature: The oxides of less electropositive met-
is also known as sequestration of Ca2+ and Mg2+ ions. als such as copper, tin, iron, lead, and others are reduced
(d) Permutit process is used for treatment of hard water. to metal when treated with hydrogen.
82. (a) ã q, r, s; (b) ã p, s; (c) ã q, s; (d) ã q, s PbO + H2 → Pb + H2O
(a) H2O can act as amphoteric, oxidizing and reducing agent.
CuO + H2 → Cu + H2O
Amphoteric character: According to Lowry Bronsted
concept of acids and bases, H2O is amphoteric, that is, it Fe3O4 + 4H2 → 3Fe + 4H2O
can act both as an acid as well as base.
Integer Type
HNO3 + H2O  H3O +
+ NO3− 83. (1) Deuterium (12 D): Mass number = 2
Base

NH3 + H2O  NH+4 + OH−


Acid

Chapter-9.indd 304 7/29/2016 3:02:55 PM


Hints and Explanations 305

Mass number = Number of protons + Number of neutrons H H


⇒2 = 1 + x ⇒ x = 1. O 4 O
84. (2) Hydrazine is represented as H2N − NH2 . Since there are H H 2 H
two lone pairs on nitrogen, it can accept two protons. O 1

85. (6) NH3, PH3, H2O, H2S, HF, and AsH3 are electron-rich H H
3
hydrides due to the presence of lone pairs of electrons, so O O
these are expected to behave as Lewis bases. CH4, BH3 and H H H
AlH3 are electron-deficient hydrides. O O
86. (3) Hydrogen exists in three isotopic forms. These are pro- H H
tium, deuterium and tritium.
87. (3) When tritium undergoes β-decay it forms element with 89. (4) Heaviest isotope of hydrogen is tritium (13 H). It has 3 neu-
mass number 3. trons and one electron but no proton.
3
1H → 32He + −10e Sum of neutron and electron is 4.
Tritium
90. (4) The number of protons and neutrons are collectively
88. (4) The structure is as follows: called nucleons. In D2 molecule, the number of nucleons is 4.

Chapter-9.indd 305 7/29/2016 3:02:57 PM


306 Chapter 9 Hydrogen

SOLVED JEE 2016 QUESTIONS


JEE Main 2016 3. Identify the reaction which does not liberate hydrogen.
(a) Reaction of lithium hydride with B2H6.
1. Which one of the following statements about water is FALSE?
(b) Electrolysis of acidified water using Pt electrodes.
(a) Ice formed by heavy water sinks in normal water.
(c) Reaction of zinc with aqueous alkali.
(b) Water is oxidized to oxygen during photosynthesis.
(d) Allowing a solution of sodium in liquid ammonia to stand.
(c) Water can act both as an acid as a base.
(Online)
(d) There is extensive intramolecular hydrogen bonding in the
condensed phase. Solution
(Offline) (a) The reactions are as follows:
Solution (a) Reaction of lithium hydride with B2H6 forms a complex
and does not liberate hydrogen gas.
(d) In case of H2O molecule, there is extensive intermolecular
hydrogen bonding in the condensed phase. 2LiH+B2H6→2Li[BH4]

2. Identify the incorrect statement regarding heavy water: (b) Electrolysis of acidified water produces hydrogen and
(a) It reacts with SO3 to form deuterated sulphuric acid (D2SO4). water at the cathode.
(b) It is used as a coolant in nuclear reactors. 2H3O+ + 2e− →H2 (g) + 2H2O (1)
(c) It reacts with CaC2 to produce C2D2 and Ca(OD)2.
(d) It reacts with Al4C3 to produce CD4 and Al(OD)3. (c) Zinc metal dissolves in aqueous alkalis such as potas-
(Online) sium hydroxide, KOH, to form zincates and hydrogen.

Solution Zn + KOH → K2ZnO2 + H2


(c) Heavy water is used as a moderator in nuclear reactors. (d) Na(s) + 2NH3(1) →2NaNH2(aq) + H2(g)

Chapter-9.indd 306 7/29/2016 3:02:57 PM


10 The s-Block Elements

Question Distribution in JEE (Main and Advanced)

3
No. of Questions

JEE (Main)
2
JEE (Adv)

0
2016 2015 2014 2013 2012 2011 2010 2009 2008 2007

Concept Distribution in JEE (Main and Advanced)

Topics Covered
Year
JEE (Main) JEE (Advanced)
2007 General Trends in Physical and Chemical Properties
of Elements
2009 Preparation and Properties of Some Important Com-
pounds of Sodium
2010 General Trends in Physical and Chemical Properties
of Elements
2014 General Trends in Physical and Chemical Properties of General Trends in Physical and Chemical Properties
Elements, Preparation and Properties of Some Important of Elements
Compounds of Sodium
General Trends in Physical and Chemical Properties of
2015
Elements
General Properties of Alkali Metals, Chemical Properties,
2016 Chemical Properties and General Characteristics of Com-
pounds, Compounds of Calcium

Chapter-10.indd 307 8/8/2016 7:11:10 PM


308 Chapter 10 The s-Block Elements

SUMMARY
1. Group 1 and 2 elements
(a) The elements in which the highest energy (last) electron is present in the s orbital are called s-block elements and include alkali
metals and alkaline earth metals.
(b) Group 1 elements: Alkali metals

Element Symbol Electronic Configuration Occurence


Lithium Li 1s22sl or [He] 2s1 LiAl (SiO3)2, Li2 Al2 (SiO3)3 (FOH)2
Sodium Na 1s22s22p63s1 or [Ne] 3sl NaCl, Na2B4O7⋅10H2O(borox), NaNO3, Na2SO4
Potassium K 1s22s22p63s23p64s1 or [Ar] 4sl KCl, KCl⋅MgCl2⋅6H2O
Rubidium Rb 1s22s22p63s23p63d104s24p65s1 or [Kr] 5s1 No Convenient source
Caesium Cs 1s22s22p63s23p63d104s24p64d105s25p66s1 or [Xe] 6s1 Obtained as by products from lithium processing
Francium Fr [Rn] 7s1 Radioactive

(c) Group 2 elements: Alkaline earth metals

Element Symbol Electronic Configuration Occurence


Beryllium Be 1s22s2 or [He] 2s2 Be3Al2Si6O18(Beryl), Be2SiO4(phenacite)
Magnesium Mg 1s22s22p63s2 or [Ne] 3s2 [MgCO3⋅CaCO3] etc.
Calcium Ca 1s22s22p63s23p64s2 or [Ar] 4s2 CaCO3
Strontium Sr 1s22s22p63s23p63d104s24p65s2 or [Kr] 5s2 SrSO4, SrCO3
Barium Ba 1s22s22p63s23p63d104s24p64d105s25p66s2 or [Xe] 6s2 BaSO4
Radium Ra 1s22s2 [Rn] 7s2 Radioactive

2. General trends in physical properties of elements


(a) Alkali metals
On reaction with water, they form hydroxides which are strongly alkaline in nature
(i) Appearance: Li, Na, K and Rb are silvery, but Cs has a golden yellow appearance.
(ii) Density: They have low densities due to their large sizes. The density increases as we move down the group. However, potas-
sium has lower density than sodium.
(iii) Boiling and melting points: They have low melting and boiling points because weak metallic bonding exists due to pres-
ence of single valence electron.
(iv) Flame colour: The elements show characteristic colours when heated in a flame. The colour actually arises from electronic
transition in short-lived species which are formed momentarily in the flame. Li (670.8 nm) → crimson, Na (589.2 nm)→ yellow,
K (766.5 nm) → Lilac, Rb (780.0 nm)→ red violet, Cs (455.5 nm) → blue
When the excited electron drops back to its original energy level it gives out the extra energy it obtained corresponding to
their wavelength.
(v) Atomic and ionic radii:
• Have the largest size in the horizontal period they are present in.
• Formation of ion by removal of electron reduces the size considerably due to removal of one orbital shell.
• Ionic radii is smaller than the atomic radii but increases down the group from Li+ to Fr+ due to addition of extra shell of
electrons.
(vi) Ionization enthalpy:
• The first ionization enthalpy is low because the atoms are very large and the outer electrons are only held weakly by the
nucleus.
• On descending the group from Li to Cs, the size of the atoms increases: the outermost electrons become less strongly held,
so the ionization enthalpy decreases.
• Second ionization enthalpy is very high because the electron needs to be removed from stable noble gas configuration.
(vii) Hydration enthalpy:
• The salts of alkali metals dissolve in aqueous solutions to form ions and conduct electricity.
• The order of ionic mobility or conductivity observed is Cs+ > Rb+ > K+ > Na+ > Li+. Li+ being the smallest ion is most heavily
hydrated which increases its radius and reduces its mobility and conductivity.
• In general, hydration energy of alkali metals decreases with increase in ionic radii.

Chapter-10.indd 308 8/8/2016 7:11:10 PM


Summary 309

(b) Alkaline earth metals


They are so called because their oxides and hydroxides are alkaline in nature and these are found in earth’s crust.
(i) Appearance: They are silvery white in colour.
(ii) Density: Group 2 metals are harder, have higher cohesive energy because they have two electrons which participate in
metallic bonding.
(iii) Melting and boiling points: They have much higher melting points and boiling points than Group 1 elements but the
metals are relatively soft.
(iv) Flame test: They impart characteristic colour to the flame in which they are heated. Calcium-brick red, Strontium-crimson,
Barium-apple green.
(v) Atomic and ionic radii:
• They are smaller than the corresponding Group 1 elements due to the extra charge on the nucleus of these elements
draws the orbital electrons inside atoms.
• Similarly, the ions are large, but smaller than those of Group 1, especially because the removal of two orbital electrons
increases the effective nuclear charge even further.
(vi) Ionization enthalpies: The total energy required to produce gaseous divalent ions for Group 2 elements (first ionization
energy + second ionization energy) is over four times greater than the energy required to produce M+ from Group 1 metals.

Tip The ionization energy for Be2+ is high, and it forms covalent compounds. Mg also forms some covalent compounds but the
compounds formed by Mg, Ca, Sr and Ba are mainly divalent and ionic.

(vii) Hydration enthalpy: The hydration enthalpies of the Group 2 ions are four or five times greater than for Group 1 ions. This
is largely due to their smaller size and increased charge. The hydration enthalpy decreases down the group as the size of the
ions increases.
3. General trends in chemical properties of elements
(a) Alkali metals
Lithium is usually the least and Cs is the most reactive. Some important reactions are summarized in the table below.
Important reactions of alkali metals
Reaction Observation Comment
With water The hydroxides are the strongest The reaction becomes increasingly violent on descend-
M + H2O → MOH + H2 bases known ing the group.
With excess dioxygen The fundamental difference in reactivity towards air is
Li + O2 → Li2O Monoxide is formed by Li and to a attributable to the cation size. The increasing stability of
small extent by Na the peroxides and superoxides as the size of alkali metal
increases is due to stabilization of larger anions by larger
cations through lattice energy effects.
Na + O2 → Na2O2 Peroxide formed by Na and to a
small extent by Li
K + O2 → KO2 Superoxides formed by K, Rb, Cs

With hydrogen Ionic “salt-like” hydrides The ease of hydride formation decreases from lithium to
M + H2 → MH caesium.
With nitrogen Nitride formed only by Li Li3N is ionic (3Li3+ and N3−) and is ruby red.
Li + N2 → Li3N
With halogens All the metals form halide The halides are ionic in nature except lithium halides
M + X2 → MX which is due to small size of lithium ion and its tendency to
Where, X is F, Cl, Br, I distort the electron cloud around the anion (polarization).
With ammonia All the metals form amides in pres- Dilute solutions of alkali metals in liquid ammonia are
M + NH3 → MNH2 + 21 H2 ence of impurities or Fe as catalyst dark blue in colour. At concentration above 3M, solu-
tions are copper-bronze coloured and have a metalic
M + (x + y )NH3 → lusture because metal ion clusters are formed.
The main species present in the solution are solvated
[M(NH3 ) x ]+ + [e(NH3 ) y )]− metal ions and solvated electrons. The solution is para-
magnetic in nature.
M+ (aq) + e − (aq) + NH3
→ MNH2 + 21 H2 (g)

(Continued)

Chapter-10.indd 309 8/8/2016 7:11:11 PM


310 Chapter 10 The s-Block Elements

(Continued)
Reaction Observation Comment
As reducing agent Alkali metals are strong reducing Lithium has the highest negative value for electrode
M(s) 
sublimation
→ M(g) agents. The reducing character potential in the group, and is the strongest reducing
increases down the group from Li agent.
ionization
→ M+ (g) + e − to Cs due to decreasing ionization
enthalpies.
hydration
M+ (g) + H2O →
M+ (aq)

(b) Alkaline earth elements


(i) Important reactions of alkaline earth metals

Reactions Observations Comments


Be probably reacts with steam, Mg with hot Ca, Sr and Ba have reduction potentials simi-
With water
water, and Ca, Sr and Ba react rapidly with lar to corresponding Group 1 metals and are
M + 2H2O → M(OH)2 + H2
cold water. high in electrochemical series.
With air Normal oxide formed by all group members Mg burns with dazzling brillance in air, a lot
2M + O2 → 2MO with excess dioxygen. of heat is evolved.
Ba + O2 → BaO2 Ba also forms the peroxide.
3M + N2 → M3N2 All form nitrides at high temperatures.
All the hydrides are hydrolyzed by water and
dilute acids with the evolution of hydrogen.
With hydrogen Ionic “salt-like” hydrides formed at high
CaH2, SrH2 and BaH2 are ionic, and contain
M + H2 → MH2 temperatures by Ca, Sr and Ba.
the hydride ion H−. Beryllium and magnesi-
um hydrides are covalent and polymeric.
With base
Mg, Ca, Sr and Ba do not react with NaOH,
Be + NaOH → Na2[Be(OH)4] Be is amphoteric.
and are purely basic
+ H2
With acid All the metals react with acids, liberating The basic properties increase on descending
M + 2HCl → MCl2 + H2 hydrogen. the group.

With halogens
All the metals form halides. Chloride and bromide are best obtained by
M + X2 → MX2
direct interaction of elements in hot tube.
where, X is F, Cl, Br, I

With ammonia All metals dissolve in liquid ammonia and On evaporation of ammonia Group 2 metals
M + (x + y )NH3 → [M(NH3 )x ]2 + dilute solutions are bright blue in colour. give hexammoniates of the metals. These
+ 2[e(NH3 )y ]− All the metals form amides at high temper- slowly decompose to give amides.

atures. M(NH3 )6 → M(NH2 )2 + 4NH3 + H2


3M + 2NH3 → 2M(NH2)2
With carbon Group 2 metals typically form ionic carbides The metals Mg, Ca, Sr and Ba form carbides of
M + 2C → MC2 of formula MC2. formula MC2.
Reducing nature
The reducing property increases as we move
These elements show a strong
They show strong reducing properties down the group from Be to Ba because the
tendency to lose two valence
which are however, weaker as compared to ionization enthalpies decrease and electrode
shell electrons to form M2+
alkali metals. potential values become more negative with
ions, except for beryllium.
increasing atomic number.
M → M2+ + 2e−

(ii) Salts of oxoacids


• Carbonates
 These are all rather insoluble in water and the solubility products decrease with increasing size of M2+ ion.
 The carbonates are all ionic, but BeCO3 is unusual because it contains the hydrated ion [Be(H2O)4]2+ rather than Be2+.

Chapter-10.indd 310 8/8/2016 7:11:12 PM


Summary 311

• Sulphates
 The solubility of the sulphates in water decreases down the group. Be > Mg  Ca > Sr > Ba.
• Nitrates
 These can all be prepared in solution and can be crystallized as hydrated salts by the reaction of HNO3 with carbonates,
oxides or hydroxides.
4. Anomalous properties of the first element of each group
(a) Anomalous properties of lithium
(i) Lithium compounds show closer similarities with Group 2 elements (particularly magnesium) than they show towards their
own group.
(ii) Differences with alkali metals:
• Higher melting and boiling points.
• Forms nitride with nitrogen and ionic carbide with carbon.
• Forms only normal oxide with oxygen, does not form peroxide or superoxide.
• Alkyls and halides of lithium show more covalent character and are hence soluble in organic solvents.
• LiOH is sparingly soluble and less basic than other hydroxides of the group.
(iii) Similarities with magnesium:
• Both react with oxygen to form oxides but do not give any peroxides or superoxide even in presence of excess oxygen.
• Both react directly with nitrogen forming nitrides (LiN3 and Mg3N2).
• Carbonates decompose on heating, do not form bicarbonates.
• Halides are covalent in character and soluble in organic solvents.
• Both have tendency to form complexes with ammonia and water.
(b) Anomalous properties of beryllium
(i) Differences between beryllium and other alkaline earth metals:
• Form covalent compounds because of its small size and high electronegativity.
• Can form a maximum of four conventional electron pair bonds. The other elements can have more than eight outer elec-
trons, and may attain a coordination number of 6 using one s, three p and two d orbitals for bonding.
• Beryllium salts are acidic when dissolved in pure water because the hydrated ion hydrolyzes, producing H3O+. The other
Group 2 salts do not interact so strongly with water, and do not hydrolyze appreciably.
(ii) Similarities with aluminium
• Beryllium forms a protective coating of oxide on its surface like aluminium.
• The chlorides of beryllium and aluminium have bridged chloride ions in their structures.
• Beryllium hydroxide forms beryllate ion in excess alkali just like aluminate ion formed by aluminium hydroxide.
• Both aluminium and beryllium ions have a tendency to form complexes.
5. Diagonal relationship
The similarity in properties between lithium (first member in Group 1) and magnesium (second element in Group 2); between beryl-
lium (first element in Group 2) and aluminium (second element in Group 3) and between boron (first element in Group 3) and silicon
(second element in Group 4) is called a diagonal relationship. It arises because of the effects of both size and charge.
Li Be B C

Na Mg Al Si

6. Preparation and properties of some important compounds of sodium


(a) Sodium carbonate (Na2CO3)
(i) Preparation: It is prepared by the Solvay (ammonia–soda) process. Following reactions take place during the process.
2NH3 +H2O + CO2 → (NH4 )2 CO3
(NH4 )2 CO3 + H2O + CO → 2NH4HCO3
NaCl + NH4HCO3 → NaHCO3 + NH4 Cl
150 ° C
2NaHCO3 → Na2CO3 + CO2 + H2O
(ii) Properties: It is odourless white powder that absorbs moisture from air to form colourless and transparent decahydrate salt
(Na2CO3 ·10 H2O). The anhydrous salt obtained on heating is called soda ash and forms strongly alkaline solution in water.

CO23 − + H2O → HCO3− + OH−


(b) Sodium hydroxide (NaOH)
(i) Preparation: It is produced by electrolysis of an aqueous solution of NaCl (brine) using either a diaphragm or mercury cath-
ode (Castner–Kellner cell) cell.

Chapter-10.indd 311 8/8/2016 7:11:13 PM


312 Chapter 10 The s-Block Elements

At the anode : 2Cl− → Cl2 + 2e −


At the cathode: Na+ + e − → Na
Overall cell reaction: 2Na + H2O → 2NaOH + H2

• Some side reactions taking place are:

2NaOH + Cl2 → NaCl + NaOCl + H2O


4OH− → O2 + 2H2O + 4e − (to a small extent)
(c) Sodium hydrogen carbonate (NaHCO3)
It is also known as baking soda. It causes the cakes and bread to rise because on heating it decomposes to give bubbles of carbon
dioxide.
Gentle heat
2NaHCO3 
→ Na2CO3 + H2O + CO2
(i) Preparation: It is prepared by bubbling carbon dioxide through a solution of sodium carbonate, wherein it separates out as
white crystalline solid.
Na2CO3 + H2O + CO2 → 2NaHCO3

7. Preparation and properties of some important compounds of calcium


(a) Lime (CaO)
(i) Preparation: By heating CaCO3 in lime kilns at temperatures between 1070 K and 1270 K.

CaCO3  CaO + CO2


(ii) Properties:
• It is a high-melting white solid.
• It is a basic oxide, so it reacts with acids and acidic oxides at high temperatures to form salts.
CaO + 2HCl → CaCl2 + H2O
CaO + SO2 → CaSO3
• It absorbs water to form calcium hydroxide. The process is call slaking of lime and is important in use of lime as a binding
material in construction.

CaO + H2O → Ca(OH)2


(b) Limestone (CaCO3)
(i) Preparation:
• On commercial scale: Ca(OH)2 + CO2 → CaCO3 + H2O
• Calcining crude calcium carbonate:
CaCO3 → CaO + CO2
CaO + H2O → Ca(OH)2
Ca(OH)2 + CO2 → CaCO3 + H2O
• Reaction of sodium carbonate with calcium chloride:

CaCl2 + Na2CO3 → CaCO3 + H2O


(ii) Properties:
• Decomposes on heating
CaCO3 → CaO + CO2

• Reacts with acids to release carbon dioxide

CaCO3 + H2SO 4 → CaSO 4 + CO2 + H2O


• Reacts with water that is saturated with carbon dioxide

CaCO3 + CO2 + H2O → Ca(HCO3 )2

Chapter-10.indd 312 8/8/2016 7:11:17 PM


Tools for Problem Solving 313

(c) Plaster of paris (CaSO 4 ⋅ 21 H2O)


(i) Preparation: By dehydration of gypsum
150 °C 200 °C 1100 °C
CaSO 4 ⋅ 2H2O → CaSO 4 ⋅ 21 H2O → CaSO 4 
→ CaO + SO3
Gypsum Plaster of paris Anhydride
(ii) Properties: When powdered plaster of Paris, CaSO 4 ⋅ 21 H2O, is mixed with correct amount of water it sets into a solid mass
of CaSO4⋅2H2O.
(d) Cements
(i) The principal constituents of cement used for constructional purposes are compounds of lime (i.e., calcareous).
(ii) Portland cement: It is made from finely ground limestone and finely divided clay with composition: Lime-60–69%, Silica
(SiO2)-17–25%, Alumina (Al2O3)-3–8 %, Iron oxide (Fe2O3)-2–4%, Magnesium oxide (MgO)-1–5%, Sulphur trioxide (SO3)-
1–3%, Alkali oxides (Na2O + K2O)-0.3–1.5%
(iii) Setting and hardening:
• When cement is mixed with water to a plastic mass, called “cement paste”, hydrating reaction begins, resulting in the for-
mation of gel and crystalline products. The process of solidification comprises of setting, and then hardening.
• Setting is defined as stiffening of the original plastic mass due to initial gel formation.
• Hardening is the development of strength due to crystallization
8. Biological significance of Na, K, Mg and Ca
(a) Biological functions of Na and K are very different. Na+ ions are actively expelled from cells, whereas K+ ions are not. This ion
transport is sometimes called a sodium pump, and it involves both the active expulsion of Na+ and the active take-up of K+.
(b) The different ratio of Na+ to K+ inside and outside cells produces an electrical potential across the cell membrane, which is essen-
tial for the functioning of nerve and muscle cells.
(c) K+ ions inside the cell are essential for the metabolism of glucose, the synthesis of proteins, and the activation of some enzymes.
(d) Mg2+ is concentrated in animal cells and Ca2+ is concentrated in the body fluids outside the cell.
(e) Mg2+ ions form a complex with ATP, and are constituents of enzymes for reactions involving ATP and energy release. They are also
essential for the transmission of impulses along nerve fibers.
(f) Mg2+ is an important constituent of chlorophyll in the green plants.
(g) Ca2+ is important in bones and teeth as apatite Ca3(PO4)2, and the enamel on teeth as fluoroapatite [3(Ca3(PO4)2) · CaF2].
(h) Ca2+ ions are important in blood clotting, and are required to trigger the contraction of muscles and to maintain the regular
beating of the heart.

TOOLS FOR PROBLEM SOLVING


Reactions of Alkali Metals
NaNH2

+FeCl3 cat.

−H2

LiOH NaNHR
NaL3Cl(s)
CH3Li +H2O ∆ Na+(am) + e− (am)
LiNHR
+RNH2
+CH3I Na2O2
Li2O −H2 +L = chelate
−RI +NH3 +Cl−
+RNH2 +H2O
LiR −H2 +O2
+O2 −H2O2
+RX
+e−
NaCl(l) Na NaOH
−LiX
Li +H2 +H2O
LiH +H2 −H2
+e−
+ROH +Mg(UO2)3OA
−H2 NaH
LiCl(l) +ROH +N2
+MCl +C222
−H2 NaOR −NaCl
Li3N
+Na[Mg(UO2)3OAc9](s)
LiOR M = K, Rb, Cs [NaC222]+Na−

Chapter-10.indd 313 8/8/2016 7:11:19 PM


314 Chapter 10 The s-Block Elements

MNHR

MHCO3(s)
+RNH2
MNH2 −H2
MO2
+NH3
+H2O +CO2
−H2 +O2
−H2O2

+Na
MCl −NaCl M = K, Rb, Cs MOH

+H2O
+ROH +H2
−H2 −H2
+NaB(C6H5)4

MH
MOR
MB(C6H5)4(s)

SOLVED EXAMPLES
General Trends in Physical and Chemical Properties of 4. Which of the following has the highest first ionization enthalpy?
Elements (a) Ba (b) Mg (c) Ca (d) Be

1. The correct statement for the molecule, CsI3, is Solution


(a) It is a covalent molecule. (d) Be has the highest first ionization enthalpy because it
(b) It contains Cs+ and I3− ions. is the first element in Group 2 and ionization enthalpy
(c) It contains Cs3+ and I− ions. decreases down the group.
(d) It contains Cs+, I− and lattice I2 molecule.
5. Of the metals Be, Mg, Ca, and Sr of Group 2 in the periodic
(JEE Main 2014)
table, the least ionic chloride would be formed by
Solution (a) Ca (b) Mg (c) Be (d) Sr
(b) CsI3 contains Cs+ and I3− ions because Cs cannot show +3 Solution
oxidation states and it is difficult to accommodate I2 mol-
ecules into the lattice because of their large size. (c) Be is the metal that formed least ionic chloride compound
because of its more polarizing power towards anion and
2. Which of the alkaline earth metal halides given below is essen- thus developing covalent character in molecule.
tially covalent in nature?
(a) MgCl2 (b) BeCl2 (c) SrCl2 (d) CaCl2 6. Magnesium burns in air to give
(JEE Main Online 2015) (a) MgO (b) Mg3N2
(c) MgCO3 (d) both (a) and (b)
Solution
Solution
(b) BeCl2 is covalent in nature because of small size and high
electronegativity of Be. (d) The reactions are
Air
3. Several alkali metals and alkaline earth metals when dissolved 2Mg + O2 → 2MgO
in NH3 produce bright blue solution. On addition of more Air
metal to this solution which of the following characteristics 6Mg + N2 → 2Mg3N2
get changed?
(a) Electrical conductivity 7. The solubility of carbonates decrease down the magnesium
(b) Colour group due to a decrease in
(c) Magnetic behaviour (a) lattice energies of solids.
(d) All of these (b) hydration energies of cations.
(c) inter-ionic attraction.
Solution (d) entropy of solution formation.
(d) On addition of more metal to this solution, the solvated
electrons gets paired up within the solvent cage, by which Solution
electrical conductivity decreases, colour gets changed (b) Due to decrease in hydration energy of cation, the solubil-
and paramagnetic behaviour is changed to diamagnetic. ity decreases down the group.

Chapter-10.indd 314 8/8/2016 7:11:21 PM


Solved Examples 315

8. One mole of magnesium nitride on the reaction with an Solution


excess of water gives (a) The hydroxides of BeO and Al2O3 , that is Be(OH)2 and
(a) one mole of ammonia. (b) two moles of nitric acid. Al(OH)3 are amphoteric in nature.
(c) two moles of ammonia. (d) one mole of nitric acid. Beryllium forms an unusual carbide Be2C, which, like
Solution Al4C3 yields methane on hydrolysis.
(c) The reaction is Be2C + 2H2O →
 2Be(OH)2 + CH4
Mg3N2 + 3H2O → 3Mg(OH)2 + 2NH3 Al4 C3 + 12H2O →
 4 Al(OH)3 + 3CH4
9. Based on lattice energy and other considerations which one Be forms many complexes like Al. Examples: [Be(OH)4]2–,
of the following alkali metal chlorides is expected to have the [BeF4]2–, [Al(OH)4]–, [AlCl4]–.
highest melting point? BeCl2 usually forms chains but also exists as the dimer.
(a) LiCl (b) NaCl (c) KCl (d) RbCl AlCl3 is dimeric.
Solution 14. Which of the following form nitride?
(b) Although lattice energy of LiCl > NaCl but LiCl is cova- (a) Na (b) Li (c) Cs (d) K
lent in nature and NaCl is ionic. Therefore, the melting Solution
point decreases as we move because the lattice energy
(b) Lithium forms a nitride Li3N, whereas rest of the Group
decreases as the size of alkali metal atom increases (lat-
1 elements do not form nitride. This is due to diagonal
tice energy is proportional to melting point of alkyl metal
relationship of Li with Mg.
halide).
15. Which of the following pairs form a diagonal relationship?
10. Which of the following statements is not correct? (a) Be and Mg (b) Li and Al
(a) Common salt absorbs water because it is hygroscopic. (c) Be and Al (d) Be and Na
(b) Common salt is used to clear snow on the road.
(c) Anhydrous MgCl2 can be prepared by heating a double Solution
salt of it, i.e. MgCl2 ⋅ NH4Cl ⋅ 6H2O. (c) Be and Al and Li and Mg show diagonal relationship due
(d) CaSO4 and BaSO4 are reacted with coke to produce CaS to similar charge to radius ratio.
and BaS respectively.
Preparation and Properties of Some Important
Solution Compounds of Sodium
(a) Common (NaCl) adsorbs water because it is contami-
nated with MgCl2 which is hygroscopic in nature. 16. Which of the following statements about Na2O2 is not correct?
(a) It is diamagnetic in nature.
11. KOH is preferably used to absorb CO2 because (b) It is a derivative of H2O2.
(a) KOH is more soluble than NaOH in water. (c) Na2O2 oxidizes Cr3+ to CrO2− 4 in acidic medium.
(b) KOH is a stronger base than NaOH. (d) It is the super oxide of sodium.
(c) KHCO3 is soluble in water and NaHCO3 is insoluble in water. (JEE Main Online 2014)
(d) KOH is cheaper than NaOH. Solution
Solution (d) (a) In Na2O2 all electrons are paired hence diamagnetic.
(c) The produced insoluble NaHCO3 chokes the reactor. (b) H2O2 + NaOH → Na2O2 (Na2O2 is derivative of H2O2)

12. In which of following cases is the value of x maximum? (c) Na2O2 + Cr3+→ Na2CrO4 (Na2O2 oxidize Cr3+ to CrO42−)
(a) CaSO4 ⋅ xH2O (b) BaSO4 ⋅ xH2O (d) Na2O2 has peroxy linkage hence peroxide.
(c) MgSO4 ⋅ xH2O (d) All have the same value of x. 17. Sodium carbonate is manufactured by Solvay’s process. The
Solution products that are recycled are
(a) CO2 and NH3 (b) CO2 and NH4Cl
(c) Efficient packing of larger size cation and larger size
(c) NaCl and CaO (d) CaCl2 and CaO
anion leads to the formation of thermally more stable
compound. Being hydrated sufficiently, Mg2+ accepts Solution
maximum number of water molecule to match its size (a) CO2 and NH3 can be recycled during the Solvay process
with SO42– ion. for manufacturing of sodium carbonate.

Anomalous Properties of the First Element of Each 18. In the following reactions
Group and Diagonal Relationship A + H2O → NaOH
400 ° C HO
13. Which of the following statement is incorrect regarding the A → B 
2
→ NaOH + O2
at 25 °C
diagonal relationship between the Al and Be?
(a) BeO and Al2O3 are basic in nature. B is used for oxygenating in submarines. Then A and B are
(b) Carbides of both on hydrolysis produce the same gas. respectively:
(c) Both can form complexes. (a) Na2O2 and Na2O (b) Na2O and Na2O2
(d) Both BeCl2 and AlCl3 can exist as dimer. (c) Na2O2 and O2 (d) Na2O and O2

Chapter-10.indd 315 8/8/2016 7:11:24 PM


316 Chapter 10 The s-Block Elements

Solution Compounds of Calcium


400 °C 1
H2 O at 25 °C
(b) Na2O → Na2O2 
→ 2NaOH + O2 ↑ 24. Which of the following on thermal decomposition yields a
(A ) (B ) 2
basic as well as an acidic oxide?
(a) NaNO3 (b) KClO3 (c) CaCO3 (d) NH4NO3
19. NaOH is manufactured by electrolysis of brine solution. The
products of the reaction are Solution
(a) Cl2 and H2 (b) Cl2 and Na-Hg (c) CaCO3 on decomposition gives CaO (which is a basic
(c) Cl2 and Na (d) Cl2 and O2 oxide) and CO2 (which is an acidic oxide).
Solution CaCO3 → CaO + CO2
(a) Electrolysis of brine solution 25. Gypsum CaSO4⋅2H2O on heating to about 150°C forms plas-
At the anode: 2Cl− → Cl2 + 2e− ter of Paris which has chemical composition represented by
At the cathode: 2H+ + 2e− → H2 (a) 2CaSO4⋅3H2O (b) CaSO4⋅H2O
Overall cell reaction: 2Cl− + 2H+ → Cl2 + H2 1
(c) 2CaSO4 ⋅ H2O (d) CaSO4
2
20. What would you observe if excess of dilute NaOH solution is
added and shaken with an aqueous solution of aluminium Solution
chloride? (c) Calcium sulphate hemihydrate forms by partially dehy-
(a) A permanent white precipitate is formed immediately. drating gypsum at 150°C.
(b) No change at first but a white precipitate is formed on 150 °C
2CaSO 4 ⋅ 2H2O → 2CaSO 4 ⋅ 21 H2O + 3H2O
standing. Gypsum Plaster of paris
(c) A white precipitate is formed which later dissolves.
(d) A green precipitate is formed which turns red on stand- 26. The substance not likely to contain CaCO3 is
ing in air. (a) a marble statue.
(b) calcined gypsum.
Solution
(c) sea shells.
(c) AlCl3 + 3NaOH → Al(OH)3 + 3NaCl (d) dolomite.
( White ppt )
Solution
21. Some large white transparent crystals are left out in a bowl
for several days. They are then observed to have changed (b) Formula for calcinated gypsum is CaSO 4 ⋅ 21 H2O .
their form into white powder. The crystals may have been of 27. In curing cement plasters, water is sprinkled from time to
(a) ammonium chloride (b) sodium chloride time. This helps in
(c) sodium carbonate (d) calcium oxide (a) keeping it cool.
Solution (b) developing interlocking needle-like crystals of hydrated
(c) Sodium carbonate normally occurs as a crystalline silicates.
heptahydrate, which readily undergo efflorescence to (c) hydrating sand and gravel mixed with cement.
form a white powder. (d) converting sand into silicic acid

22. Which one is formed in the Solvay’s process? Solution


(a) Na2CO3 (b) NaHCO3 (b) As setting of cement is an exothermic reaction, inter-
(c) CaCl2 (d) All of these locking crystals of hydrated silicates are developed.
Solution 28. The function of sand in mortar is
(d) In Solvay’s process, Na2CO3, NaHCO3 and CaCl2 are (a) to decrease the hardness.
formed from the raw materials NaCl, CaCO3 and NH3. The (b) to prevent excessive shrinkage because of which cracks
reactions involved are may result.
(NH4 )2 CO3 + H2O + CO2 → 2NH4HCO3 (c) to increase hardness.
NH4HCO3 + NaCl → NH4 Cl + NaHCO3 (d) to make the mass compact.

2NaHCO3 → Na2CO3 + CO2 + H2O Solution


2NH4 Cl + Ca(OH)2 → 2NH3 + CaCl2 + 2H2O (b) During the course of drying, it prevents excessive shrink-
age, and thus prevents appearance of cracks.
23. Na2CO3 can be manufactured by Solvay’s process but K2CO3
29. Which of the following can dissolve limestone?
cannot be prepared because
(a) CO2 + H2O (b) NaOH + H2O
(a) K2CO3 is highly soluble in water.
(c) NH3 + H2O (d) None of these
(b) K2CO3 is less soluble in water.
(c) KHCO3 is highly soluble in water as compared to NaHCO3. Solution
(d) KHCO3 is less soluble as compared to NaHCO3. (a) Limestone (CaCO3) dissolves in water in the presence of
Solution CO2 due to formation of calcium bicarbonate, otherwise
insoluble.
(c) This is because KHCO3 formed in the process is fairly sol-
uble in water. CaCO3 + H2O + CO2 → Ca(HCO3)2

Chapter-10.indd 316 8/8/2016 7:11:27 PM


Practice Exercise 317

Biological Significance of Na, K, Mg and Ca 31. The metal used for blood coagulation is
(a) Ca (b) K
30. Which of the following metals is present in chlorophyll? (c) Na (d) Mg
(a) Mg (b) Be
(c) Ca (d) Al Solution
(a) Ca plays important role in neuromuscular function,
Solution
inter-neuronal transmission, cell membrane integrity and
(a) Mg2+ is an important constituent of chlorophyll in the blood coagulation.
green plants.

ADVANCED LEVEL PROBLEMS


1. Statement 1: Alkali metals dissolve in liquid ammonia to give Solution
blue solutions.
Statement 2: Alkali metals in liquid ammonia give solvated (a), (b) Na2O2 is the major product, and small amount of Na2O
species of the type [M(NH3)n]+ (M = alkali metals). is also formed.
(a) Statement 1 is True, Statement 2 is True; Statement 2 is a 3. Amongest the following the total number of compounds
correct explanation for Statement 1. whose aqueous solution turns red paper blue is ___.
(b) Statement 1 is True; Statement 2; is True Statement 2 is not KCN; K2SO4; (NH4)2C2O4; NaCl; Zn(NO3)2; FeCl3; K2CO3; NH4NO3;
a correct explanation for Statement 1. LiCN
(c) Statement 1 is True, Statement 2 is False. (IIT-JEE 2010)
(d) Statement 1 is False, Statement 2 is True. Solution
(IIT-JEE 2007) (3) KCN, K2CO3 and LiCN are basic in nature and their aque-
Solution ous solution turns red litmus paper blue.
(b) The alkali metals dissolve in liquid ammonia giving deep 4. The pair(s) of reagents that yield paramagnetic species is/are
blue solutions which are conducting in nature. (a) Na and excess of NH3.
(b) K and excess of O2.
M + ( x + y )NH3 → [M(NH3 )]+ + [e(NH3 ) y ]− (c) Cu and dilute HNO3.
(d) O2 and 2-ethylanthraquinol.
The blue colour is due to the ammoniated electron which (JEE Advanced 2014)
absorbs energy in the visible region of light and thus Solution
imparts blue colour to the solution.
(a), (b), (c) The solution of Na in liquid ammonia is paramag-
netic due to the presence of ammoniated electrons.
2. The compound(s) formed upon combustion of sodium metal
K + O2 → KO2
in excess air is (are)
− (superoxide) is paramagnetic due to unpaired electron in p*
(a) Na2O2 (b) Na2O O2
(c) NaO2 (d) NaOH orbital.

(IIT-JEE 2009) Cu + 8HNO3 → 3Cu(NO3 )2 + 2NO + 4H2O


NO is paramagnetic as it is an odd electron species.

PRACTICE EXERCISE
Level I 4. Washing soda is
(a) Na2CO3⋅7H2O (b) Na2CO3⋅10H2O
Single Correct Choice Type (c) Na2CO3⋅H2O (d) Na2CO3
1. The principal products obtained on heating iodine with con-
5. Which of the following when introduced into the Bunsen’s
centrated caustic soda solution are
flame gives pink violet colour?
(a) NaOI + NaI (b) NaIO3 + NaI
(a) NaCl (b) BaCl2 (c) CsCl (d) KCl
(c) NaOI + NaIO3 + NaI (d) NaIO4 + NaI
6. When a substance (A) reacts with water, it produces a com-
2. Which has minimum hydration enthalpy? bustible gas (B) and a solution of substance (C) in water. When
(a) K+ (b) Li+ another substance (D) reacts with the solution of (C), it pro-
(c) Ca 2+ (d) Al3+ duces the same gas (B) on warming, but (D) can produce (B) on
reaction with dilute H2SO4 at room temperature, (A) imparts a
3. On heating sodium metal in a current of dry ammonia gas, the golden yellow colour to brown flame. The compounds (A), (B),
compound formed is (C), and (D) are
(a) sodium nitrate (b) sodium hydride (a) K, H2, KOH, Al (b) Na, H2, NaOH, Zn
(c) sodium amide (d) sodium azide (c) CaC2, C2H2, Ca(OH)2, Fe (d) Ca, H2, Ca(OH)2, Sn

Chapter-10.indd 317 8/8/2016 7:11:28 PM


318 Chapter 10 The s-Block Elements

7. Which of the following alkali metal emits light of longest (a) Blue coloured solution is obtained.
wavelength in the flame test? (b) Na ions are formed in the solution.
(a) Na (b) Li (c) K (d) Cs (c) Liquid NH3 becomes a good conductor of electricity.
(d) Liquid ammonia remains diamagnetic.
8. Baking soda is
(a) Na2CO3 (b) NaHCO3 (c) Na2SO4 (d) K2CO3 23. When NaCl is dissolved in water, the sodium ion is
(a) oxidized. (b) reduced.
9. Mark the false statement. (c) hydrolyzed. (d) hydrated.
(a) The electropositive character of alkali metal decreases
with increase in the atomic number. 24. Which of the following imparts violet colouration to the
(b) Lithium is a hard metal and cannot be cut with a knife. Bunsen burner non-luminous flame?
(c) Alkali metals are strong reducing agents. (a) NaCl (b) BaCl2 (c) CaCl2 (d) KCl
(d) Electronegativities of all alkali metals lie between 1.0 25. Epsom salt is
and 0.7. (a) MgSO4⋅7H2O (b) MgSO4⋅2H2O
10. Which one of the following will react most vigorously with (c) 2CaSO4⋅H2O (d) BaSO4⋅2H2O
water? 26. What is X in the equation?
(a) V (b) Ti (c) Ca (d) Rb 3273 K
CaO + 3C → X + CO
11. When sodium metal is dissolved in liquid ammonia, a blue
(a) CaCO3 (b) CaC2 (c) CO2 (d) CaCl2
solution is formed. The blue colour is due to
(a) solvated Na+ ions. (b) solvated electrons. 27. Electrolysis of molten sodium chloride leads to the formation
(c) solvated NH2− ions. (d) solvated protons. of
12. Chile saltpeter is (a) Na and H2 (b) Na and O2
(a) NaNO3 (b) Na2SO4 (c) KNO3 (d) Na2CO3 (c) H2 and O2 (d) Na and Cl2

13. Commonly used laboratory desiccant is 28. Soda ash is


(a) calcium chloride (b) sodium carbonate (a) Na2CO3⋅H2O (b) NaOH
(c) sodium chloride (d) potassium nitrate (c) Na2CO3 (d) NaHCO3
14. The wire of flash bulb is made of 29. The hydration enthalpy of Mg2+ ions is larger than that of
(a) Mg (b) Cu (c) Ba (d) Ag (a) Al3+ (b) Na+ (c) Be2+ (d) Li+
15. Na2CO3⋅10H2O is prepared by 30. Which of the following is used as antacid?
(a) Castner process (b) Solvay’s process (a) MgO (b) Mg(OH)2
(c) Ostwald’s process (d) Haber’s process (c) MgSO4 (d) MgCO3
16. Electrolysis of KH produces H2 31. An amphoteric oxide dissolves HCl to form a salt. The salt
(a) at the cathode. does not impart any colour to the flame and fumes in moist
(b) at the anode. air. The oxide is
(c) either at the cathode or at the anode. (a) BaO2 (b) MgO (c) BeO (d) CaO
(d) Cannot be predicted
Level II
17. The metal that can be extracted from seawater is
(a) Cl (b) Br Multiple Correct Choice Type
(c) Mg (d) All of these 32. The alkali metals are characterized by which of the following
18. The density of Ca is less than that of Mg because properties?
(a) nuclear charge of Ca is more than Mg (a) High boiling point
(b) vacant 3d orbital is present in Ca (b) High negative standard electrode potential
(c) size of Ca is less than Mg (c) High density
(d) None of these (d) Large atomic size
19. Mixture of MgCl2 and MgO is called 33. Which of the following compounds of sodium are used for
(a) Portland cement (b) Sorel cement textile industry?
(c) Double salt (d) Complex (a) Na2CO3 (b) NaHCO3 (c) NaOH (d) NaCl
20. The useful byproduct obtained in Solvay’s process of manu- 34. Identify the correct formula of halides of alkaline earth met-
facture of sodium carbonate is als from the following:
(a) Quicklime (b) NH4Cl (a) BaCl2⋅2H2O (b) BaCl2⋅4H2O
(c) CaCl2 (d) Ca(OH)2 (c) CaCl2⋅6H2O (d) SrCl2⋅4H2O

21. Which one of the following is ionic? 35. Which of the following statements are true about beryllium
(a) Sodium chloride (b) Beryllium chloride and its compounds?
(c) Lithium chloride (d) Carbon tetrachloride (a) Beryllium exhibits coordination number more than four.
(b) Beryllium sulphate is readily soluble in water as the
22. On dissolving moderate amount of sodium metal in liquid NH3 greater hydration enthalpy of Be2+ overcomes the lattice
at low temperature, which one of the following does not occur? enthalpy factor.

Chapter-10.indd 318 8/8/2016 7:11:29 PM


Answer Key 319

(c) Beryllium is not readily attacked by acids because of the (c) BeCl2 < LiCl < NaCl
presence of an oxide film on the surface of the metal. (d) LiCl < NaCl < BeCl2
(d) Beryllium oxide is purely acidic in nature.
42. Which ion has the minimum polarizing power?
36. Which of the following are not the correct reasons for anoma- (a) Li+ (b) Na+ (c) K+ (d) Mg2+
lous behaviour of lithium?
Matrix-Match Type
(a) Exceptionally small size of its atom.
(b) Its low polarizing power. 43. Match the property with the compound/element.
(c) It has high degree of hydration. Column I Column II
(d) Exceptionally low ionization enthalpy. (a) Flame colouration (p) Na
(b) Stored under kerosene (q) KF
37. When KO2 reacts with water, the product formed is (c) Soluble in organic solvents (r) BeCl2
(a) KOH (b) H2O2 (c) K2O2 (d) O2 (d) Soluble in water without
38. Na2O cannot be prepared by any reaction with it (s) LiCl
(a) Na2O2 + CO → (b) Na + NaNO3 → 44. Match the compounds with their properties.
(c) Na + NaNO2 → (d) Na2O2 + Na → Column I Column II
Passage Type (a) CaCO3 (p) Insoluble in water
(b) CaSO4⋅2H2O (q) Amphoteric nature
Paragraph for Questions 39 to 43: Lithium exhibits the charac- (c) BaCO3 (r) Insoluble in water but dissolves
teristic properties of Group 1 elements but also differs from the in the presence of CO2
remaining alkali metals in a number of ways. This anomalous
(d) Be(OH)2 (s) Gives precipitate with dil. H2SO4
behaviour of lithium can be attributed to the small size of lith-
ium atom and ion and high charge density. Lithium ions possess Integer Type
highest polarizing power among all alkali metal ions and hence
45. The number of compounds among the following that do not
cause maximum distortion of the negative ion. Lithium ions show
produce acidic or basic solutions when dissolved in water is
greater resemblance in properties to magnesium ions and this
___.
relationship is known as diagonal relationship.
NaCl, BeCl2, BaCl2, Li2O, MgO, CaH2, CaSO4
39. Which of the following undergoes hydrolysis by cold or hot
water? 46. The number of elements among the following that do not
(a) LiCl (b) NaCl (c) KCl (d) RbCl give characteristic colour in flame test is ___.
40. The small size of lithium ion cannot explain Ca, Li, Mg, Be, Ba, Sr, Na, K
(a) covalent nature of lithium hydrides.
(b) solubility of methyl lithium in alcohol. 47. The ratio of the number of water of crystallization in gypsum
(c) high dipole moment of lithium iodide. and plaster of Paris is ___.
(d) instability of lithium carbonate.
48. The number of elements among the following that will form
41. The correct sequence of increasing covalent character is rep- nitrides when heated in an atmosphere of nitrogen is ___.
resented by
(a) BeCl2 < NaCl < LiCl Li, Cs, Rb, K, Ca, Ba, Na, Sr, Mg
(b) NaCl < LiCl < BeCl2

ANSWER KEY
Level I
1. (b) 2. (a) 3. (c) 4. (b) 5. (d) 6. (b)
7. (c) 8. (b) 9. (a) 10. (d) 11. (b) 12. (a)
13. (a) 14. (a) 15. (b) 16. (b) 17. (c) 18. (b)
19. (b) 20. (c) 21. (a) 22. (d) 23. (d) 24. (d)
25. (a) 26. (b) 27. (d) 28. (c) 29. (b) 30. (b)
31. (c)

Level II
32. (b), (d) 33. (a), (c) 34. (a), (c) 35. (b), (c) 36. (b), (d) 37. (a), (b), (d)
38. (b), (c), (d) 39. (c) 40. (c) 41. (b) 42. (c)
43. (a) → s; (b) → p; (c) → r, s; (d) → q 44. (a) → r, s; (b) → p; (c) → r, s; (d) → q 45. (3) 46. (2)
47. (4) 48. (5)

Chapter-10.indd 319 8/8/2016 7:11:29 PM


320 Chapter 10 The s-Block Elements

HINTS AND EXPLANATIONS


Level I 21. (a) Sodium chloride is ionic and rest of the compounds con-
tain covalent character.
Single Correct Choice Type
22. (d) On dissolving moderate amount of sodium metal in liq-
1. (b) 3I2 + 6NaOH → 5NaI + NaIO3 + 3H2O uid NH3 at low temperature, liquid ammonia does not
2. (a) K+ has minimum hydration enthalpy. Smaller the cation, remain diamagnetic. It changes into paramagnetic.
greater is the degree of hydration. 23. (d) NaCl + H2O → Na + + OH− + H+ Cl−
(Hydrated)
3. (c) 2Na + NH3 → 2NaNH2 + H2
Sodium amide 24. (d) KCl → violet colour; NaCl → golden yellow;
4. (b) Washing soda is Na2CO3⋅10H2O BaCl2 → apple green and CaCl2 → brick red.

5. (d) NaCl gives golden yellow, BaCl2 gives apple green, CsCl 25. (a) Epsom salt is MgSO4⋅7H2O
gives violet, and KCl gives pink violet colour. 3273 K
26. (b) CaO + 3C → CaC2 + CO
6. (b) The reactions are
27. (d) Electrolysis of molten sodium chloride leads to the for-
2Na + 2H2O → 2NaOH + H2 mation of Na and Cl2
(A) (C) (B )
NaCl(aq)  Na+ + Cl−
2NaOH + Zn → Na2ZnO2 + H2 H2O  H+ + OH−
(C) (D ) (B )
At the cathode: 2H+ + 2e − → H2
Zn + H2SO 4 → ZnSO 4 + H2
(D) (B) At the anode: 2Cl− → Cl2 + 2e −
Sodium imparts golden colour to the flame. 28. (c) Anhydrous Na2CO3 is called soda ash.
7. (c) Potassium emits light of longest wavelength in the flame 29. (b) Due to the smaller size and increased charge on Na+ ions.
test because it gives lilac (violet) colour in the flame.
30. (b) A suspension of Mg(OH)2 in water is used in medicine as
8. (b) Baking soda is NaHCO3 an antacid under the name of milk of magnesia.
9. (a) The electropositive character of alkali metals increases 31. (c) Due to small size and somewhat high ionization enthalpy
with increase in the atomic number. of Be, BeO is amphoteric and dissolves in both acids and
10. (d) Rubidium will react most vigorously with water. alkalies. The flame test is not shown both by beryllium and
magnesium oxides (basic oxide) as due to the smaller size
11. (b) Ammoniated electron (solvated electrons) is responsible of these elements, the valence shell electrons are strongly
for blue colour solution. bound to the nucleus. The energy from the flame of a
burner is not sufficient to raise them to the excited state.
12. (a) Chile saltpeter is NaNO3
Level II
13. (a) Calcium chloride is used as laboratory desiccant because
it is easily available, less toxic, and costs less. Multiple Correct Choice Type
14. (a) Magnesium is used in flash bulbs for photography. In 32. (b), (d) Alkali metals are strong reducing agents. Hence, they
flash bulbs magnesium powder is mixed with potassium have high negative standard reduction potential. Since alkali
chlorate. metals are the first elements in the respective periods, they
have large atomic size.
15. (b) Pure Na2CO3 is prepared by Solvay’s process, absorbs
moisture from air and normally exists as a decahydrate 33. (a), (c) Sodium carbonate is used in textile industry for dye-
salt (Na2CO3·10H2O) in form of transparent and colour- ing and printing operations. Sodium hydroxide is used in tex-
less crystals. tile industry for mercerizing cotton fabrics.

16. (b) Conceptual 34. (a), (c) Except BeCl2, all other chlorides of Group 2 form
hydrates but their tendency to form hydrates decreases,
17. (c) Mg is present in seawater in amounts of about 1300 ppm for example, MgCl2⋅6H2O, CaCl2⋅6H2O, SrCl2⋅2H2O and
and is the most commonly found cation in oceans after BaCl2⋅2H2O.
sodium. 35. (b), (c) Conceptual
18. (b) Ca has less density than that of Mg because of presence 36. (b), (d) Conceptual
of vacant 3d orbitals in Ca.
37. (a), (b), (d) KO2 + H2O →
 KOH + H2O2 + O2 [Reaction not
19. (b) The composition of Sorel’s cement is MgCl2⋅5MgO⋅xH2O balanced].

20. (c) 2NH4Cl + Ca(OH)2 → 2NH3 + 2H2O + CaCl2 38. (b), (c), (d) Conceptual

Chapter-10.indd 320 8/8/2016 7:11:31 PM


Hints and Explanations 321

Passage Type BaCO3 or CaCO3 + dil H2SO 4 → BaSO 4 or CaSO 4 + H2O + CO2
White ppt
39. (c) Conceptual
When powdered plaster of Paris CaSO 4 ⋅ 21 H2O is mixed
40. (c) Conceptual with the correct amount of water, it sets into a solid mass of
41. (b) As the size of the cation increases, the covalent character CaSO4⋅2H2O (gypsum) due to hydration of molecules and
decreases. their rearrangement.
Due to anomalous behaviour, Be(OH)2 resembles Al(OH)3 and
42. (c) Larger the cation, Lower its polarizing power. is amphoteric in nature.
Integer Type
Matrix-Match Type
45. (3) NaCl, BaCl2, CaSO4 are ionic and produce neutral solu-
43. (a) → s; (b) → p; (c) → r, s; (d) → q tions; BeCl2 produces acidic solution and the remaining basic
LiCl gives characteristic red colour. solutions.
Alkali metals react with oxygen, moisture and carbon dioxide
46. (2) Be and Mg atoms are smaller in size and their electrons
present in the air, forming oxides, hydroxides and carbonates.
are tightly bound to the nucleus. They need large amounts
In order to prevent these reactions, the alkali metals are
of energy for excitation of electrons to higher energy levels
stored in kerosene, example, Na.
which is not available in the Bunsen flame. So they do not
The halides and alkyls of lithium are far more covalent than
impart colour to the flame.
the corresponding sodium compounds, and because of this
covalency they are soluble in organic solvents. 47. (4) The formula for gypsum is CaSO4⋅2H2O and that of plaster
Be halides are all soluble in organic solvents and hydrolyze in of Paris is CaSO 4 ⋅ 21 H2O.
water. 48. (5) Li, Ca, Ba, Sr and Mg form nitrides. In Group 1, only Li is
KF dissociates into potassium ions and fluoride ions KF → K+ + F− able to form nitride.
44. (a) → r, s; (b) → p; (c) → r, s; (d) → q
Excess CO
CaCO3 or BaCO3 + H2O 
2
→ Ca2+ (HCO3− )2 or Ba2+ (HCO3− )2
Insoluble Soluble bicarbonate
white ppt

Chapter-10.indd 321 8/8/2016 7:11:32 PM


322 Chapter 10 The s-Block Elements

SOLVED JEE 2016 QUESTIONS


JEE Main 2016 Solution
1. The hottest region of Bunsen flame shown in the figure (a)
below is: Li + O2 (g) → Li2O (Monoxide)
( excess )

Region 4 Na + O2 (g) → Na2O2 (Peroxide)


( excess )
Region 3
K + O2 (g) → KO2 (Superoxide)
Region 2 ( excess )

Region 1 3. The correct order of the solubility of alkaline earth metal sul-
phates in water is
(a) Mg > Ca > Sr > Ba. (b) Mg > Sr > Ca > Ba.
(c) Mg < Ca < Sr < Ba. (d) Mg < Sr < Ca < Ba.
(Online)
Solution
(a) region 4 (b) region 1 (c) region 2 (d) region 3 (a) The solubility of the sulphates in water decreases down the
(Offline) group. Mg > Ca > Sr > Ba. MgSO4 is soluble, but CaSO4 is
sparingly soluble, and the sulphates of Sr, Ba and Ra are vir-
Solution tually insoluble. The significantly high solubility of MgSO4
(c) is due to the high enthalpy of solvation of the smaller Mg2+
ion.
Reactive gases 4. The commercial name for calcium oxide is
incomplete combustion (a) quick lime. (b) milk of lime.
Hottest part of flame (c) slaked lime. (d) limestone.
(Online)
Unburnt gas and air Solution
(a) The commercial name of calcium oxide (CaO) is quick lime.
5. Which of the following atoms has the highest first ionization
energy?
(a) Sc (b) Na
(c) K (d) Rb
(Online)
Solution
2. The main oxides formed on combustion of Li, Na and K in (a) The effective nuclear charge increases in a period, hence
excess of air is respectively ionization energy increases while it generally decreases
(a) Li2O, Na2O2 and KO2. (b) Li2O, Na2O and KO2. down the group. Thus, the increasing order of first ioniza-
(c) LiO2, Na2O2 and K2O. (d) Li2O2, Na2O2 and KO2. tion enthalpy as follows:
(Offline) Rb < K < Na < Sc

Chapter-10.indd 322 8/8/2016 7:11:34 PM


11 The p-Block Elements

Question Distribution in JEE (Main and Advanced)


4

3
No. of Questions

JEE (Main)
2
JEE (Adv)

0
2016 2015 2014 2013 2012 2011 2010 2009 2008 2007

Concept Distribution in JEE (Main and Advanced)


Topics Covered
Year
JEE (Main) JEE (Advanced)
2007 General Properties of Group 14 Elements, Chemical Properties Phosphorus, Boron, General Properties of Group 18
of Group 17 Elements
2008 Compounds of Aluminium, Silicon Chemical Properties of Group 14 Elements, Phosphorus,
Sodium Thiosulphate
2009 General Properties of Group 15 Elements, Phosphorus, Oxides of Nitrogen
Compounds of Xenon
2010 Compounds of Boron, Oxoacids of Phosphorus, Chemical
Properties of Groups 14, 17 and 18 Elements
2011 Compounds of Boron, General Properties of Group 15 Dinitrogen, Phosphorus halides
Elements, General Properties of Group 16 Elements
2012 General Properties of Group 15 Elements Nitrogen, Halides, Compounds of Xenon, Allotrope of
Carbon
2013 Silicon, Allotrope and Compounds of Oxygen Allotrope and Compounds of Oxygen, Chemical Proper-
ties of Groups 14, 15, 16 and 18 Elements
2014 General Properties of Group 13 Elements, Compounds of Chemical Properties of Group 13 Elements, Chemical
Aluminium, Silicon, General Properties of Group 15 Elements, Properties of Group 15 Elements, Compounds of Xenon
Oxoacids of Halogens, Compounds of Xenon
2015 Compounds of Phosphorus, General Properties of Group 18 Oxides of Nitrogen
Elements
2016 Oxides of Nitrogen, Oxoacids of Phosphorus, Oxoacids of General Properties of Group 13 Elements, Compounds of
Nitrogen, General Properties of Group 17 Elements, General Boron, Chemical Properties of Group 15 Elements
Properties of Group 14 Elements, Allotropes of Sulphur, General
Properties of Group 16 Elements

Chapter-11.indd 323 7/30/2016 12:57:42 PM


324 Chapter 11 The p-Block Elements

SUMMARY
1. The elements in which the highest energy electron enters the outermost p orbital are known as p-block elements. There are six
groups (ns2 np1−6) of p-block elements in the periodic table, named as Group13–18 as shown below.
Group 13 14 15 16 17 18
First element Boron (B) Carbon (C) Nitrogen (N) Oxygen (O) Fluorine (F) Helium (He)
Electronic configuration ns2 np1 ns2 np2 ns2 np3 ns2 np4 ns2 np5 ns2 np6
(He: 1s2)
Group oxidation state +3 +4 +5 +6 +7 +8
Other oxidation states +1 +2, −4 +3, −3 +4, +2, −2 +5, +3, +1, −1 +6, +4, +2
2. This occurrence of an oxidation state two units below the Group oxidation state is sometimes attributed to the inert pair effect. The
term refers to the resistance of a pair of s electrons to be lost or to participate in covalent bond formation.
3. General properties of Group13 elements

Ionization enthalpy
(kJ mol−1)
Atomic Electronic Oxidation m.p. b.p.
Elements Occurrence number configuration states 1st 2nd 3rd (çC) (çC)
Boron (B) Concentrated deposits of borax 5 [He] 2s2 2p1 3 801 2427 3659 2180 3650
Na2[B4O5(OH)4] ⋅ 8H2O and
kernite Na2[B4O5(OH)4] ⋅ 2H2O
Aluminium (Al) Aluminosilicate rocks, such as 13 [Ne] 3s2 3p1 1, 3 577 1816 2744 660 2467
feldspars and micas
Gallium (Ga) 31 [Ar] 3d10 4s2 4p1 1, 3 579 1979 2962 30 2403
Indium (In) Occur as sulphides, in trace 49 [Kr] 4d10 5s2 5p1 1, 3 558 1820 2704 157 2080
Thallium (Tl) amounts 81 [Xe] 4f14 5d10 6s2 6p1 1, 3 589 1971 2877 303 1457

(a) The metallic radii of the atoms do not increase regularly on descending the Group. The ionic radii for M3+ increase down the Group.
(b) The electronegativity decreases from B to Al due to difference in atomic sizes of the elements.
(c) The ionization enthalpies increase as expected (first ionization enthalpy < second ionization enthalpy < third ionization enthalpy).
The ionization enthalpy values do not decrease smoothly down the Group. The decrease from B to Al is the usual trend on
descending a Group associated with increased size. The poor shielding by d electrons and the resulting d-block contraction affect
the values for the later elements.
(d) The melting points of the Group 13 elements do not show a regular trend.
(e) Boron has an unusual crystal structure while Al, In and Tl all have close-packed metal structures. Gallium has an unusual structure
with an incredibly low melting point of gallium of 30°C and also because the liquid expands when it forms the solid, that is, the
solid is less dense than the liquid.
(f) The boiling point of B is unusually high, but the values for Ga, In and Tl decrease on descending the Group as expected.
(g) The densities of elements increase down the Group.
(h) The electropositive or metallic nature of the elements increases from B to Al, but then decreases from Al to Tl.
4. Chemical properties of Group13 elements
(a) Oxidation state
(i) The elements all have three outer electrons. Apart from Tl, they normally use these to form three bonds, giving an oxidation
state of (+3).
(ii) The atoms in this group have an outer electronic configuration of s2p1. Monovalency is explained by the inert pair effect.
(iii) The trivalent compounds of Group 13 elements act as electron acceptors or Lewis acids and the donor of electrons such as
ammonia and water act as Lewis bases and form compounds like BF3 and NH3. The tendency of Group13 elements to behave
as Lewis acids increases as we move down the group.
(b) Reaction with water
Since the compounds in trivalent state are covalent, they are hydrolyzed in water. The trihalides on hydrolysis in water form
tetrahedral species [M(OH)4]−.
BCl3 + 3H2O → H3BO3 + 3HCl  [B(OH)4 ]+Cl−

Chapter-11.indd 324 7/30/2016 12:57:42 PM


Summary 325

Tip In acidified aqueous solution, aluminium chloride forms a complex ion [Al(H2O)6]3+ which has aluminium in sp3d2 hybridization
and is octahedral in shape. This difference in behaviour from BCl3 on hydrolysis can be explained on the basis of vacant 3d orbitals in
Al3+ ions, which allow it to expand its coordination number from 4 to 6 and coordinate with water molecules that are present in larger
excess than OH− ions.

(c) Some important reactions


(i) Reactions of boron
Reaction Comment

4B + 3O2 → 2B2O3 At high temperature.

2B + 3S → B2S3 At 1200°C.

2B + N2 → 2BN At very high temperature.

2B + 3X2 → 2BX3 (X = F , Cl, Br , I) At high temperature.


Boron halides thus act as Lewis acids. The Lewis acid character, however,
decreases in the order: BI3 > BBr3 > BCl3 > BF3.
2B + 6NaOH → 2Na3BO3 + 3H2 On fusion with alkali.

2B + 2NH3 → 2BN + 3H2 At very high temperature.

B + M → M xB y Many metals form borides (not Group 1) often non-stoichiometric.

Tip Boron nitride (BN)x is a slippery white solid with a layer structure resembling graphite, in which hexagonal rings of boron and
nitrogen atoms alternate.

(ii) Reactions of other Group 13 metals


Reaction Comment
All react at high temperature. Al is very strongly exothermic. Ga only superficially
4M + 3O2 → M2O3 oxidized. Tl forms some Tl2O as well.
2Al + N2 → 2AlN Only Al reacts at high temperature.
2M + 3X2 → 2MX3 All the metals 
where X = F, Cl, Br, I  +
form  Tl also formed
trihalides 
Trivalent iodides formed by Al, Ga, In only. Thallium (I) triiodide is formed.

2M + 6HCl → 2MCl3 + 3H2 All react with dilute mineral acids, Al passivated by HNO3 particularly when concen-
trated.
Al + NaOH + H2O → NaAlO2 + H2 Al and Ga only.
Na3AlO3 + H2

M + NH3 → MNH2 All the metals form amides.

5. (a) Important trends and anomalous properties of boron

Boron Aluminium Gallium, indium and thallium


Forms no simple B3+ cation. Electropositive and readily forms Al3+. Readily forms M3+ ions in solution.
Exclusively forms covalent compounds Shows greater metallic character than boron Show increasing metallic character and
and forms great number of ionic compounds. form coordination compounds charac-
teristic of metals.
Obeys octet rule and shows maximum Forms compounds with coordination number The lower oxidation state (+1) becomes
covalence of four. six or more. more stable especially in Tl+ and lower
valent compounds are formed.

(Continued)

Chapter-11.indd 325 7/30/2016 12:57:44 PM


326 Chapter 11 The p-Block Elements

(Continued)
Boron Aluminium Gallium, indium and thallium
Forms trivalent compounds that act as Forms trivalent halides that dimerize to com- Monovalent compounds are formed
Lewis acids and form adducts with Lewis plete their octet by bridging through halogen and nature of covalent bond becomes
bases to complete their octets. atom. weaker.
Forms acidic oxide (B2O3) and hydroxide Forms oxide Al2O3 and hydroxide Al(OH)3 that Oxide and hydroxide of gallium are
B(OH)3. are amphoteric in nature. amphoteric, while that of In and Tl are
basic.
Forms covalent halides that are easily Forms solid halides that are only partially Form trivalent halides (except from TlI3)
hydrolyzed. hydrolyzed. which are aggregated in the solid state
through halide ion bridge and have
coordination numbers 4, 6 or higher.

Forms stable hydride ion BH4− . Forms anionic hydride AlH4− which is more Only Ga forms hydride ion GaH4− .
reactive than BH4− .
Frequently forms polyhedral com- – –
pounds like boranes and borates.

(b) Some important compounds of boron


Borax OH Na2B4 O7 + 7H2O → 2NaOH + 4H3BO3
(Na2B4O7 ¥ Orthoboric acid
10H2O) B− Heat Heat
O O Na2B4 O7 ⋅10H2O 
→ Na2B4 O7 

B O B OH 2 NaBO2 + B2 O 3
HO Sodium metaborate Boric anhydride
O O Glassy borax bead
B− Borax is a useful primary standard for titrating against acids.

OH Na2 [B4 O5 (OH)4 ]⋅ 8H2O + 2HCl → 2NaCl + 4H3BO3 + 5H2O

Boric Acid H H Preparation


(Orthoboric Na2 [B4 O5 (OH)4 ]⋅ 8H2O + 2HCl → 2NaCl + 4H3BO3 + 5H2O
acid, B(OH)3 or O O
H
H3BO3) H B H H Properties
O O
B(OH)3 (or H3BO3 ) + 2H2O  H3O + + [B(OH)4 ]− pK a = 9.25
O
H B H H B H B(OH) (or H BO )  H O + + [B O (OH) ]− + H O pK = 6.84
3 3 3 3 3 3 4 2 a
O O O O
100°C Red heat
H H H H H3BO3 → HBO2 
→ B2 O 3
Boric Metaboric Boron
O O O O acid acid sesquioxide
H H H H
B B
O
O O
H H B H H
O O

H H
Diborane H H1 H Preparation
Heat
B B 119 pm Mg3B2 + H3PO 4 → Mixture of boranes → B2H6
Magnesium (Mainly B4H10 ) Diborane
H 133 H H borate
pm
Diglyme solution
Overlap of approximately sp2 hybrid orbitals 2Na[BH4 ] + I2 → B2H6 + H2 + 2NaI
from B with an s orbital from H to give a
Ether
“banana-shaped” three-center two-electron 4[Et2O ⋅ BF3 ] + 3Li[AlH4 ] 
→ 2B2H6 + 3Li[AlF4 ] + 4Et2O
bond.
(Continued)

Chapter-11.indd 326 7/30/2016 12:57:48 PM


Summary 327

(Continued)

Diglyme
[Et2O ⋅ BF3 ] + 3Na[BH4 ] → 2B2H6 + 3Na[BF4 ] + 4Et2O
H 750 atm, 150°C
H B2O3 + 3H2 + 2Al → B2H6 + Al2O3
B H Boron
H B trioxide

H 2BF3 (g) + 6NaH 


180°C
→ B2H6 (g) + 6NaF
H
Boron
fluoride
3NaBH4 + 4BF3 → 2B2H6 + 3NaBF4
Properties
B2H6 + 3O2 → B2O3 + 3H2O ∆H = − 2165 kJ mol−1
H H
B2H6 + 6H2O → 2H3BO3 + 3H2
B H
H B B2H6 + 6ROH → 2B(OR)3 + 6H2
H B2H6 + 2(Me )3 N → 2[Me3N ⋅ BH3 ]
H
B2H6 + 2CO → 2BH3 ⋅ CO
Excess NH
B2H6 + NH3 Low
 3

temperature
→ B2H6 ⋅ 2NH3
Excess NH
Higher
 3

temperature
→ (BN) x
Boron nitride
Ratio 2NH : 1B H
Higher
 3 2 6

temperature
→ B3N3H6
Borazine

Borazine H H
(B3N3H6)
H H H B− H
B +
N N N N+
B B B− B−
N N+
H H H H
H H
Borazine
Borazine (B3N3H6) is sometimes called “inor-
ganic benzene” because its structure shows
some formal similarity with benzene, with
delocalized electrons and aromatic character.
Tetrahydridob- The tetrahydridoborate ion [BH4]− is tetrahe- Preparation
orates dral. 4B(OMe)3 + 4NaH 250 °C, high pressure
 → Na[BH4 ] + 3Na[B(OMe)4 ]
H THF solvent
Trimethylborate

B− 2LiH + B2H6 


Ether
→ 2LiBH4
H H
H Heat
AlCl3 + 3NaBH4 
→ Al(BH4 )3 + 3NaCl

6. General properties of Group 14 elements


Ionization enthalpy
(kJ mol−1)
Oxi-
Atomic Electronic configu- dation 1st 2nd 3rd 4th m.p. b.p.
Elements Occurrence number ration states (çC) (çC)
Carbon (C) Carbon occurs mainly as 6 [He] 2s2 2p2 4 1086 2354 4622 6223 4100
coal, crude oil and car-
bonates in rocks such as
calcite CaCO3, magnesite
MgCO3 and dolomite
[MgCO3∙CaCO3].

(Continued)

Chapter-11.indd 327 7/30/2016 12:57:51 PM


328 Chapter 11 The p-Block Elements

(Continued)

Ionization enthalpy
(kJ mol−1)
Oxi-
Atomic Electronic configu- dation 1st 2nd 3rd 4th m.p. b.p.
Elements Occurrence number ration states (çC) (çC)
Silicon (Si) As silica SiO2 (sand and 14 [Ne] 3s2 3p2 2, 4 786 1573 3232 4351 1420 3280
quartz), and in a wide
variety of silicate miner-
als and clays.
Germani- As traces in some silver 32 [Ar] 3d10 4s2 4p2 2, 4 760 1534 3300 4409 945 2850
um (Ge) and zinc ores, and in
some types of coal.
Tin (Sn) Tin is mined as cassiter- 50 [Kr] 4d10 5s2 5p2 2, 4 707 1409 2943 3821 232 2623
ite SnO2
Lead (Pb) Found as the ore galena 82 [Xe] 4f14 5d10 6s2 6p2 2, 4 715 1447 3087 4081 327 1751
PbS

(a) The covalent radii increase down the group.


(b) The difference in size between Si and Ge is less than might be otherwise expected because Ge has a full 3d shell which shields
the nuclear charge rather ineffectively. In a similar way, the small difference in size between Sn and Pb is because of the filling of
the 4f shell.
(c) The ionization enthalpies decrease from C to Si, but then change in an irregular way because of the effects of filling the d and f shells.
(d) The increase in ionization enthalpy from tin to lead is because of their large size as well as poor shielding effect of intervening
d and f orbitals.
(e) The first ionization enthalpies of Group 13 are lower than that of Group 14. The amount of energy required to form M4+ ions is
extremely large, and hence simple ionic compounds are rare.
(f) The electronegativity decreases from carbon to silicon but remain the same till lead. In general, since the sizes of these elements
are small, these elements are more electronegative than Group 13 elements.
(g) The melting points decrease on descending the Group because the M M bonds become weaker as atoms increase in size.
(h) Metallic character increases down the group.
7. Chemical properties of Group 14 elements
(a) Oxidation states and trends in chemical reactivity
(i) The elements in Group 14 are relatively unreactive but reactivity increases down the group.
(ii) The most common oxidation states are (+2) and (+4).
(iii) The (+2) oxidation state becomes increasingly stable on descending the group. The inert pair effect shows itself increasingly
in the heavier members of the group. There is a decrease in the stability of the (+4) oxidation state and an increase in the
stability of the (+ 2) state on descending the group.
(b) Reactivity towards oxygen
Carbon reacts with oxygen to form two oxides of C, CO and CO2 while Si also forms two oxides of silicon, SiO and SiO2. The lower
oxides GeO, SnO and PbO have layer lattices rather than the typical ionic structures. The basicity of the oxides usually increases
down the group. Thus, CO2 and SiO2 are purely acidic. GeO2 is not as strongly acidic as SiO2, and SnO2 and PbO2 are amphoteric.
(c) Reactivity towards water
C, Si, Ge, Pb are unaffected by water. Sn reacts with steam to give SnO2 and H2.
Heat
Sn + 2H2O 
→ SnO2 + 2H2
(d) Reactivity towards halogen
(i) Two series of halides, MX4 and MX2, are formed by the elements. All the tetrahalides are known except PbI4. All halides are
typically covalent, tetrahedral (central atom is sp3 hydridized), and very volatile. The exceptions are SnF4 and PbF4, which
have three-dimensional structures and are high melting solids.

Tip The elements after C have d orbitals available, and the Si–F, Si–Cl and Si–O bonds are stronger than the corresponding bonds
with C. This is thought to be due to the donation of electrons from F, Cl or O to Si, giving rise to pp–dp bonding.

(ii) The carbon halides are not hydrolyzed under normal conditions because they have no d orbitals, and cannot form a five-
coordinate hydrolysis intermediate.
The silicon halides are rapidly hydrolyzed by water to give silicic acid.

Chapter-11.indd 328 7/30/2016 12:57:52 PM


Summary 329

SiCl4 + 4H2O → Si(OH)4 + 4HCl


Silicic acid
(e) Reactivity towards acids and alkalis
(i) Si is oxidized and fluorinated by concentrated HF/HNO3. Ge dissolves slowly in hot concentrated H2SO4 and in HNO3. Sn dis-
solves in several concentrated acids. Pb does not dissolve in concentrated HCl because a surface coating of PbCl2 is formed.
(ii) Carbon is unaffected by alkalis. Si reacts slowly with cold aqueous solutions of NaOH and readily with hot solutions,
giving solutions of silicates [SiO4]4–. Sn and Pb are slowly attacked by cold alkali, and rapidly by hot alkali, giving stannates
Na2[Sn(OH)6] and plumbates Na2[Pb(OH)6]. Thus, Sn and Pb are amphoteric.
8. (a) Important trends and anomalous behaviour of carbon
(i) Higher ionization enthalpy, being more covalent and being less metallic.
(ii) Unique ability to form pp –pp multiple bonds, such as C C, C C, C O, C S and C N.
(iii) Forms many chains of great length.
(b) Allotropes of carbon

Allotrope Structure Uses


Diamond Tetrahedral structure; Gemstones
sp3 hybridization. Abrasive powder for cutting and polishing

Graphite Two-dimensional sheets Lubricant, electrodes, fibers, pencils, brake linings and
of carbon atoms. brushes for electric motors

(a) (b)
Fullerenes It consists of a fused Superconductor
system of five- and
six-membered rings.

(c) Some important compounds of carbon and silicon


Oxides of carbon and silicon
Preparation Reactions
O C 2C(s) + O2 (g) 
Heat
→ 2CO(g)
Heat
Fe2O3 (s) + 3CO(g) 
→ 2Fe(s) + 3CO2 (g)
H − COOH + H2SO 4 → CO + H2O CuO(s) + CO(g) →
 Cu(s) + CO2 (g)
Carbon monoxide

Red heat 2CO + O2 → 2CO2 ∆H o = − 565 kJ mol−1


→ CO + H2 ∆H o = + 131 kJ mol−1
C + H2O  1
Water gas CO + O2 → CO2
2
and ∆S o = + 134 kJ mol−1 CO + S → COS
Carbonyl sulphide
+C
C + O2 + 4N2 → CO2 + 4N2 → 2CO + 4N2
Air Producer gas
CO + Cl2 → COCl2
Carbonyl chloride
(Phosphene)

O C O CH4 (g) + 2O2 (g) → CO2 (g) + 2H2O CO2 + H2O  H2CO3
C(s) + O2 (g) → CO2 (g) Sunlight
6CO2 + 6H2O → C6H12O6 + 6O2
Carbon dioxide

Glucose
CaCO3 (s) + 2HCl(aq) → CaCl2 (aq) + CO2 (g) + H2O(l) 180°C
CO2 + 2NH3 Pressure
 → NH4 CO2NH2 → CO(NH2 )2 + H2O
CO + H2O  CO2 + H2 Ammonium Urea
carbamate
CH4 + 2H2O → CO2 + 4H2
Ca(OH)2 + CO2 → CaCO3 + H2O
Yeast under
C6H12O6 anaerobic
 conditions
→ 2C2H5OH + 2CO2 White
precipitate
Strong heat CaCO3 + CO2 + H2O → Ca(HCO3 )2
CaCO3 
→ CaO + CO2
Soluble

(Continued)

Chapter-11.indd 329 7/30/2016 12:57:58 PM


330 Chapter 11 The p-Block Elements

(Continued)
Preparation Reactions
Silica in any form is unreactive due to high bond enthalpy
Silicon dioxide

Si O Si O Si of Si O bond.
O O O SiO2 + NaOH → (Na2SiO3 )n and Na4 SiO 4
Si O Si O Si +SiO2
H2SO 4 + CaF 2 → HF  

O O O
Si O Si O Si
SiF 4
+H2 O
 → HF +
{ Si(OH)4 or
SiO2 ⋅ 2H2O

SiO2 + 2F2 → SiF4 + O2

Cu catalyst 280 − 300° C Hydrolysis under carefully controlled conditions can pro-
Si + 2CH3Cl → (CH3 )2SiCl2
duce cyclic structures, with rings containing three, four,
+2H2 O
five or six Si atoms.
− → (CH3 )2Si(OH)2
2HCl
R R R R
−H2O
HO Si OH + HO Si OH HO Si O Si OH
R R R R
Silicones

R R R
HO Si OH + HO Si O Si OH
R R R

R R R
HO Si O Si O Si OH, etc.
R R R
1400° C +SiO
Na2CO3 → CO2 + Na2O 
2
→ Na4 SiO 4 , (Na2SiO3 )n and others
Silicates

Si

(d) Types of silicates


Pyrosilicates
Orthosilicates (soro-silicates, disil- Chain silicates Sheet silicates
(neso-silicates) icates) Cyclic silicates (SiO3 )2nn- (Si2O5 )2nn−

− − −
− − −
− − −
− − − − −
− − −
Si SiO4 ion Si −
O
O − − and − − − −
O O − − − −
− − − − − − −
Si Si2O6− − − − −
O 7 ion Si3O96− ion − − − −
12−
Si6O18 ion
(Si2O5)n2n− sheet

(e) Zeolites
(i) Sharing all four corners of a SiO4 tetrahedron results in a three-dimensional lattice of formula SiO2 (quartz, tridymite, cristo-
balite, etc.). These contain no metal ions, but three-dimensional structures can form the basis of silicate structures if there is
a replacement of some of the Si4+ by Al3+ plus an additional metal ion. This gives an infinite three-dimensional lattice, and
the additional cations occupy holes in the lattice.

Chapter-11.indd 330 7/30/2016 12:58:02 PM


Summary 331

(ii) Replacing one quarter of the Si4+ in SiO2 with Al3+ gives a framework ion AlSi3O8−. These are known as aluminosilicates.
Cations are usually the larger metal ions such as K+, Na+, Ca2+ or Ba2+. Replacements of one-fourth or one-half of the Si atoms
are quite common, giving structures

MI [AlSi3O8 ] and MII [Al2Si2O8 ]

Such replacements result in three groups of minerals: Feldspars, zeolites and ultramarines.
9. General properties of Group15 elements

Ionization enthalpy
(kJ mol-1)
Oxi-
Atomic Electronic configu- dation 1st 2nd 3rd m.p. b.p.
Element Occurrence number ration states (çC) (çC)
78% of the earth’s
atmosphere;
Nitrate deposits in −3, −2,
Nitrogen (N) desert regions; 7 [He] 2s2 2p3 −1, 0, 1, 1403 2857 4578 −210 −195.8
Essential constit- 2, 3, 5
uent of proteins/
amino acids
Nucleic acids such
as DNA and RNA;
Phosphorus (P) Fluoroapatite, 15 [Ne] 3s2 3p3 3, 5 1012 1897 2910 44 281
hydroxyapatite and
chloroapatite
Arsenic (As) As sulphides 33 [Ar] 3d104s2 4p3 3 947 1950 2732 816 615
Antimony (Sb) occurring as traces 51 [Kr] 4d10 5s2 5p2 3 834 1590 2440 631 1587
in other ores;
Metallurgical
Bismuth (Bi) byproducts from 83 [Xe]4f14 5d10 6s2 6p3 3 703 1610 2467 271 1564
roasting sulphide
ores in a smelter

(a) The atomic and ionic radii increase as one descends down the group.
(b) The ionization enthalpy of Group 15 elements is much higher than that expected because of extra stability associated
with electronic configuration of half-filled p orbitals and the smaller atomic size of the elements as compared to Group 14
elements.
(c) The ionization enthalpy decreases down the group due to increase in atomic size. The successive ionization energies increase
due to greater energy required in removal of successive electrons
(d) The electronegativity value decreases with increase in atomic size down the group.
10. Chemical properties of Group 15 elements
(a) Oxidation states
(i) They exhibit a maximum oxidation state of (+5) towards oxygen by using all five outer electrons in forming bonds.
(ii) The tendency for the pair of s electrons to remain inert (the inert pair effect) increases with increasing atomic weight. Thus,
only the p electrons are used in bonding and trivalency results.
(iii) Valencies of 3 and 5 are shown with the halogens and with sulphur. The hydrides are trivalent.

Tip Sb and Bi can lose three electrons, forming M3+ ions, with very high ionization energy. Both SbF3 and BiF3 exist as ionic solids.
The M3+ ions are not very stable in solution. They can exist in fairly strong acid solutions, but are rapidly hydrolyzed in water to give
SbO+ and BiO+.

(b) Anomalous properties of nitrogen


(i) Ability to form strong pp–pp multiple bonds itself and other elements having small size and high electronegativity. This
ability to form these bonds diminishes down the Group. Nitrogen forms a number of compounds like nitrates, nitrites, azides;
oxides of nitrogen; cyanides; and azo and diazo compounds of which there are no P, As, Sb or Bi analogues.
(ii) The absence of lower lying 3d orbitals in nitrogen.

Chapter-11.indd 331 7/30/2016 12:58:02 PM


332 Chapter 11 The p-Block Elements

(c) Important reactions


(i) Reactivity towards hydrogen: The elements form volatile hydrides of formula EH3 which are strong reducing agents. On
descending the Group from NH3 to BiH3, we observe that
• Their stability decreases.
• Their reducing power increases.
• The ease of replacing the hydrogen atoms by other groups such as Cl or Me decreases.
• Their ability to act as lone pair donors for coordinate bond formation decreases.
• The bond energy and bond angle decreases, bond length increases.
• The boiling and melting points show no specific trend.
(ii) Reactivity towards oxygen: All elements of the Group form oxides of the type E2O3 and E2O5. In case of nitrogen, oxides
such as N2O, NO, NO2, N2O4 are also known.

Tip The oxide in higher oxidation state of an element is more acidic than oxide in lower oxidation state.

(iii) Reactivity towards halogens: All the possible trihalides of N, P, As, Sb and Bi are known. An excess of halogen gives penta-
halide (EX5).
• These trihalides are predominantly covalent and, like NH3, have a tetrahedral structure with one position occupied by
a lone pair. The exception includes BiF3 which is ionic and the other halides of Bi and SbF3 which are intermediate in
character.
• Nitrogen is unable to form pentahalides because the second shell contains a maximum of eight electrons, that is, four
bonds. The subsequent elements have suitable d orbitals, and form pentahalides.
(iv) Reactivity towards metals: They react with metals to form binary compounds in which their oxidation state is −3.
11. (a) Dinitrogen
(i) Nitrogen is a colourless, odourless, tasteless gas which is diamagnetic and exists as diatomic molecule.
(ii) It has two stable isotopes 14N and 15N. N2 molecule contains a triple bond N N (bond length =1.09 pm), which is very stable
with high dissociation energy.
(iii) N2 is inert at room temperature, but at elevated temperatures N2 becomes increasingly reactive.

Tip Other isoelectronic species such as CO, CN− and NO+ are much more reactive than N2, and this is because the bonds are partly
polar, while in N2 they are not.

(b) Ammonia
(i) Preparation
• Decay of nitrogenous organic matter.

NH2CONH2 + 2H2O → (NH4 )2CO3 + NH3 + H2O + CO2

• Heating an ammonium salt with Ca(OH)2.

2NH4 Cl + Ca(OH)2 → 2NH3 + 2H2O + CaCl2

• Heating an ammonium salt with NaOH. This is a standard test in the laboratory for NH+4 compounds.

NH4 Cl + NaOH → NaCl + NH3 + H2O

• Hydrolysis of calcium cyanamide.


CaNCN + 3H2O → 2NH3 + CaCO3

• The commercial production of ammonia is done synthetically from H2 and N2 by the Haber–Bosch process.


N2 + 3H2  2NH3 + heat
  
4 volumes 2 volumes

The reaction is reversible, and Le Chatelier’s principle suggests that high pressure and low temperature are required to
drive the reaction to the right, and thus form NH3.
(ii) Properties
• The boiling and melting points of ammonia are −34.5°C and −77.8°C, respectively. NH3 has a higher boiling point and is
much less volatile because it is hydrogen bonded in the liquid state.

Chapter-11.indd 332 7/30/2016 12:58:03 PM


Summary 333

• It dissolves very readily in water with the evolution of heat. In solution, it forms NH4OH, a weak base.
• NH3 acts as a good Lewis base and donates its lone pair of electrons to form complexes. For example, the [Co(NH3)6]3+.
(iii) Uses
• As a fertilizer; making HNO3; making caprolactam; making hexamethylenediamine; transporting H2 (liquid ammonia);
refrigerator coolant.
(c) Oxides of Nitrogen

Oxi-
Oxides dation
of ni- num- Bond Bond Physical
trogen ber Shape length order property Preparation
N2O +I Linear, asymmetricalN N= N N= Stable, rela- (i) Thermal decomposition of molten
molecule 112.6 pm 2.73 and tively unreac- ammonium nitrate at about 280°C
N O= N O= tive, neutral
N N O NH4NO3 → N2O + 2H2O
Dinitrogen monoxide 118.6 pm 1.61 colourless
(nitrous oxide) gas.
(ii) Heating a solution of NH4NO3
acidified with HCl
NH4NO3 → N2O + 2H2O
NO +II N O Colourless (i) 3Cu + 8HNO3 → 2NO + 3Cu(NO3 )2 + 4H2O
Nitrogen monoxide gas, para-
3Cu + 8HNO3 → 2NO + 3Cu(NO3 )2 + 4H2O
(nitric oxide) magnetic;
diamagnetic
in the liquid (ii) 2HNO2 + 2H+ → 2NO + I2 + 2H2O
and solid (iii)
states.
2NaNO2 + 2FeSO 4 + 3H2SO 4 → 2NO
+ Fe2 (SO 4 )3 + 2NaHSO 4 + 2H2O

N2O3 +III O N N N= (i) NO + NO2 → N2O3


N O 186.4 pm
in the (ii) 4NO + O2 → 2N2O3
O asymmet-
rical form, (iii) 2NO + N2O 4 → 2N2O3
weak
N2O3 can only be obtained at low tem-
peratures. It can be made by condens-
ing equimolar of NO and NO2 together.
It gives a blue liquid or solid which is
unstable and dissociates into NO and
NO2 at −30°C.
NO2 +IV N Red–brown (i) Large scale by oxidizing NO in the
O O poisonous Ostwald process.
gas; para-
Nitrogen dioxide (ii) 2Pb(NO3 )2 → 2PbO + 4NO2 + O2
magnetic.
N2O4 +IV O O O N of angular Diamagnetic; NO2 dimerizes into colourless N2O4.
N N 120 pm with an mixed anhy-
O O N N = 164 O N O dride.
pm angle of
132°.
N2O5 +V O O Colourless Dehydration of HNO3 with P2O5 at low
N O N deliquescent temperatures treating N2O5 with O3
O O solid, which
4HNO3 + P4 O10 → 4HPO3 + 2N2O5
is highly
reactive; light
sensitive.

Chapter-11.indd 333 7/30/2016 12:58:07 PM


334 Chapter 11 The p-Block Elements

• NO readily forms coordination complexes with transition metal ions. These complexes are called nitrosyls.
Tip Fe2+(aq) and NO form the complex [Fe(H2O)5NO]2+, which is responsible for the colour in the “brown-ring test” for nitrates.
• NO often acts as a three-electron donor. Thus, three CO groups may be replaced by two NO groups:

[Fe(CO)5 ] + 2NO → [Fe(CO)2 (NO )2 ] + 3CO


(d) Nitric Acid
(i) Preparation
• HNO3 was originally made from NaNO3 or KNO3 and concentrated H2SO4.

NaNO3 + H2SO 4 → NaHSO 4 + HNO3


• Birkeland–Eyde process
spark +O HO
N2 + O2 
→ NO 
2
→ NO2 
2
→ 4HNO3 (Process is now obsolete, because of the high cost of electricity)
• The Ostwald process
NH3 + 2O2 → HNO3 + H2O
(ii) Structure
The structure of the nitrate ion is a planar triangle.

H O
102° pm
121
96

O140.6 pmN 130°


pm

O
(iii) Properties
• On exposure to light it turns slightly brown because of slight decomposition into NO2 and O2.

4HNO3 → 4NO2 + O2 + 2H2O


• In aqueous solutions, nitric acid behaves as a strong acid and gives rise to hydronium and nitrate ions. It forms a large
number of salts called nitrates, which are typically very soluble in water.

HNO3 (aq) + H2O(l) → H3O+ (aq) + NO3− (aq)


• Oxidation of metals
Cold dilute < 1 M
3Cu + 8HNO3 
→ 2NO + 3Cu(NO3 )2 + 4H2O

Strong acid
Cu + 4HNO3 → 2NO2 + Cu(NO3 )2 + 2H2O

Tip The enhanced ability to dissolve metals such as gold shown by aqua regia arises from the oxidizing power of HNO3 coupled with
the ability of Cl− (from HCl) to form complexes.

• Oxidation of non-metals
P4 + 20HNO3 → 4H3PO 4 + 20NO2 + 4H2O

C + 4HNO3 → CO2 + 4NO2 + 2H2O


• Brown ring test: The test depends on the ability of Fe2+ to reduce nitrates to nitric oxide which reacts with Fe2+ to form
a brown coloured complex.
NO3− + 3Fe2 + + 4H+ →
 NO + 3Fe3+ + 2H2O

Fe2 + + NO + 5H2O → [Fe(H2O )5 (NO )]2 +


Brown ring
12. (a) Phosphorus
(i) It is a solid at room temperature. The 3 allotropic forms of P are white, red, and black.

Chapter-11.indd 334 7/30/2016 12:58:09 PM


Summary 335

Allotropic forms Structure Physical property Chemiluminescence


White phosphorus Soft, waxy and reactive; highly Reacts with moist air
toxic. and gives out light
60° (chemiluminescence)
It ignites spontane-
ously in air at about
35°C forming P4O10.

Red phosphorus P P P Polymeric solid, which is much less Does not ignite unless
(heating white P to P P P P P P reactive. it is heated to 400°C
about 250°C, or a and is not chemilumi-
lower temperature P P P nescent.
in sunlight) Tetrahedral P4 molecules linked together

Black phosphorus Thermodynamically the most stable


(heating white phos- allotrope; inert;
phorus under high Black, flaky, has sheet-like struc-
pressure) ture and is a good conductor of
electricity;
Layer structure Exists in two allotropic forms,
a -black P and b -black P.

(b) Phosphine
(i) Preparation
• Hydrolyzing metal phosphides such as Na3P or Ca3P2 with water or dilute acids.
Ca3P2 + 6H2O → 2PH3 + 3Ca(OH)2
• Hydrolyzing white phosphorus with NaOH solution in an inert atmosphere.
P4 + 3NaOH + 3H2O → PH3 + 3NaH2PO2
• Reaction of phosphonium iodide with KOH
PH4I + KOH → PH3 + KI + H2O
(ii) Properties
• Phosphine is a highly reactive and causes explosion when treated with traces of HNO3, Cl2 and Br2 vapours.
• It is a very weak base, but however reacts with acids to give phosphonium compounds.
PH3 + HBr → PH4Br
• Phosphine reacts with copper sulphate or mercuric chloride to form their respective phosphides.
3CuSO 4 + 2PH3 → Cu3P2 + 3 H2SO 4
• Pure PH3 is stable in air, but it catches fire when heated to about 150°C.
PH3 + 2O2 → H3PO 4
(iii) Uses
• Fumigant, smoke screens, synthesis of organophosphorus compounds and some alloys.
(c) Phosphorus halides
Two types of halides (X = F, Cl, Br, I), trihalides (PX3) and penathalides (PX5) formed.

Structure Preparation Reactions


PCl3 P4 + 6Cl2 → 4PCl3 PCl3 + 3H2O → H3PO3 + 3HCl
P P4 + 8SOCl2 → 4PCl3 + 4SO2 PCl3 + 3RCOOH → 3RCOCl + H3PO3
+ 2S2Cl2
Cl Cl Cl PCl3 + 3ROH → 3RCl + H3PO3

PCl3 + Cl2 (or SO2Cl2 ) → PCl5

2PCl3 + O2 → 2POCl3
6PCl3 + P4 O10 + 6Cl2 → 10POCl3

(Continued)

Chapter-11.indd 335 7/30/2016 12:58:15 PM


336 Chapter 11 The p-Block Elements

(Continued)
Structure Preparation Reactions
PCl5 Cl CCl PCl5 + 4H2O → H3PO 4 + 5HCl
PCl3 + Cl2 
4
→ PCl5
Cl Phosphoric acid
240 pm
PCl5 + H2O → POCl3 + 2HCl
202 PCl5 
heat
→ PCl3 + Cl2
P pm
Cl Cl 2Ag + PCl5 →
 2AgCl + PCl3

PCl5 + 4RCOOH → 4RCOCl + H3PO 4 + HCl


Cl PCl5 + 4ROH → 4RCl + H3PO 4 + HCl
6PCl5 + P4 O10 → 10POCl3

PCl5 + SO2 → POCl3 + SOCl2

(d) Oxoacids of Phosphorus


O

The acid contains three replaceable


Orthophosphoric acids P H atoms in form of three P OH
H3PO4 HO OH
bonds, and is tribasic.

OH
O O

Pyro- P P
O OH The acid contains four replaceable H
phosphoric HO
atoms in form of four P OH bonds,
acid
compounds have oxidizing properties.

and is tetrabasic.
H4P2O7 OH OH
Oxidation state of P is (+5);

H4P2O7
Phosphoric acid series

Polyphos- Pyrophosphoric acid


phoric
acids O O O

Tri- P P P
polyphos- HO O O OH The acid contains five replaceable H
atoms in form of five P OH bonds,
phoric acid
and is pentabasic.
H5P2O10 OH OH OH
H5P3O10
Tripolyphosphoric acid

O O−
P
O O O
O O
P P O O
Metaphosphates form a family of
Metaphosphoric acids −O P P
O O− ring compounds;
−O O O−

Di-metaphosphate ion Tri-metaphosphate ion


(cyclo-diphosphate) (cyclo-triphosphate)
O O
(Continued)
−O P O P O−

O O

−O P O P O−
Chapter-11.indd 336 7/30/2016 12:58:18 PM
O O−
P
O O O
O O
P P O O
−O O O− P P
−O O O− Summary 337

Di-metaphosphate ion Tri-metaphosphate ion


(Continued) (cyclo-diphosphate) (cyclo-triphosphate)
O O

−O P O P O−

O O

−O P O P O−

O O
Tetra-metaphosphate ion
(cyclo-tetraphosphate)
OH OH
O P P O
Hypophosphoric acid,
Tetrabasic
H4P2O6 OH OH
Hypophosphoric
acid
H These are very strong reducing
compounds are reducing agents.

Orthophosphorous HO P OH agents in basic solutions.


Oxidation state of P is (+3);
Phosphorous acid series

acid H3PO3
O The salts are known as phosphites.

O
The acid is monobasic and a very
Hypophosphorous
P strong reducing agent.
acid H3PO2 H H
OH
H H
Pyrophosphorous acid It is dibasic in nature.
HO P O P OH
H4P2O5
O O

13. General properties of Group16 elements

First ion-
ization
Atomic Electronic config- Oxidation enthalpy m.p. b.p.
Elements Occurrence number uration states (kJ mol−1) (çC) (çC)
It exists in the free form as dioxygen molecules;
Oxygen Constituent of silicate minerals on the earth’s crust; –2, –1,
8 [He] 2s2 2p4 1314 –229 –183
(O) Also occurs as many metal oxide ores, and as +1, 2
deposits of oxosalts.
Sulphide ores, and as sulphates (particularly gyp-
Sulphur sum CaSO4∙2H2O.
16 [Ne] 3s2 3p4 –2, 2, 4, 6 999 114 445
(S) Occurring as traces in ores (MgSO4 ⋅ 7H2O), galena
(PbS), zinc blende (ZnS), copper pyrite (CuFeS2).
Selenium
Very scarce. They occur in small amounts as sele- 34 [Ar] 3d10 4s2 4p4 2, 4, 6 941 221 685
(Se)
nides and tellurides in sulphide ores, particularly
Tellurium
those of Ag and Au; 52 [Kr] 4d10 5s2 5p4 2, 4, 6 869 452 1087
(Te)
Pollonium occurs as the decay product of thorium
Polonium and uranium minerals 84 [Xe] 4f14 5d10 6s2 6p4 2, 4 813 254 962
(Po)

Chapter-11.indd 337 7/30/2016 12:58:19 PM


338 Chapter 11 The p-Block Elements

(a) Oxygen, sulphur, selenium and tellurium are non-metals. Collectively, they are called “the chalcogens”.
(b) The atomic and ionic radii increase down the group.
(c) The ionization enthalpy decreases down the group because of increase in size. The values of ionization enthalpies are how-
ever much lower than the corresponding members of group 15 because of the stability associated with half-filled orbitals in
Group 15, which make the removal of electrons difficult.
(d) The electron gain enthalpy value becomes less negative down the group from sulphur onwards.
(e) From top to bottom in the group, the electronegativity decreases with increasing atomic size.

Tip Oxygen is the most electronegative among all elements, second only to fluorine.

14. Chemical properties of Group 16 elements


(a) Oxidation states
(i) The oxidation state of O is (–2) in most of its compounds.
(ii) The elements S, Se and Te show oxidation states of (+4) and (+6), and these are more stable than the +2 state.
(b) Anomalous behaviour of oxygen
(i) Oxygen can use pp orbitals to form strong double bonds. The other elements can also form double bonds, but these become
weaker as the atomic number increases.
(ii) Due to absence of d orbitals, oxygen cannot expand its covalency beyond four, though two is most common. In case of the
other elements (S, Se, Te, Po), the valence shell can be expanded beyond four by participation of d orbitals.
(c) Reactivity with hydrogen
(i) All the elements of Group 16 form covalent hydrides of the type H2 E.
(ii) The hydrides decrease in stability from H2O to H2S to H2Se to H2Te because the bonding orbitals become larger and diffuse more.
(iii) The hydrides are all very weak acids and there is an increase in acidic strength from H2O to H2Te.
(d) Reactivity with oxygen
All elements form dioxides of the type EO2 and trioxides of the type EO3, where E = S, Se, Te, Po.
(e) Reactivity towards halogens
(i) A large number of halides of the type EX6, EX4 and EX2 are formed.
(ii) Since F is more electronegative than O, binary compounds are oxygen fluorides, whereas similar chlorine compounds are
chlorine oxides.
(iii) The stability of hexahalides decreases in the order F > Cl > Br > I.
(iv) In contrast to the relatively stable hexafluorides, the tetrahalides are very sensitive to water.
15. (a) Dioxygen
(i) Preparation
• Prepared by the electrolysis of water with a trace of H2SO4 or barium hydroxide solution.
• Thermal decomposition of KClO3:
150°C
2KClO3 MnOcatalyst
→ 2KCl + 3O2
2

• Catalytic decomposition of hypochlorites:


Co2+
2HOCl 
→ 2HCl + O2
• Thermal decomposition of metal oxides:
2BaO2 → 2BaO + O2
3MnO2 → Mn3O 4 + O2
• The decomposition of hydrogen peroxide in the presence of MnO2 and finely divided metals as catalysts.
2H2O2 → 2H2O + O2
(ii) Properties
• Dioxygen is paramagnetic in all phases and has rather high dissociation energy of 496 kJ mol−1.
• Oxygen has three isotopes: 16O (99.75%), 17O (0.0374%) and 18O (0.204%).
(iii) Uses
For respiration by both animals and plants, preparation of TiO2 from TiCl4, manufacture of HNO3 and oxirane, oxidant in
rockets.
(b) Simple Oxides

These are usually covalent. SO3 + H2O → H2SO 4


Non-metallic Oxides
They are all acidic in nature and form acids when dissolved in water. B2O3 + 3H2O → 2H3BO3
Most metal oxides are ionic and contain the O2– ion; ionic oxides typically have
Metallic Oxides Na2O + H2O → 2NaOH
high melting points and are generally basic in nature.

Chapter-11.indd 338 7/30/2016 12:58:21 PM


Summary 339

(i) Many metal oxides with formulae M2O3 and MO2, though ionic, do not react with water. Examples include Tl2O3, Bi2O3 and
ThO2. These react with acids to form salts, and so are basic.
(ii) In some cases where a metal can exist in more than one oxidation state, and thus form more than one oxide, for example,
CrO, Cr2O3, CrO3, PbO, PbO2, and Sb4O6, Sb4O10.
(iii) Many metals yield oxides which are amphoteric, and react with both strong acids and strong bases. Examples include BeO,
Al2O3, Ga2O3, SnO and ZnO.
(iv) A few covalent oxides have no acidic or basic properties (N2O, NO, CO) and are called neutral oxides.
(c) Ozone
(i) Ozone O3 is the triatomic allotrope of oxygen.

O
Single valence bond structure
O O

O O
Resonance hybrids
O O O O

(ii) Preparation
Electric discharge
3O2 
→ 2O3 ∆H(25°C) = +142 kJ
(iii) Properties
• O3 has a characteristic sharp smell, often associated with sparking in electrical equipment.
• O3 is an extremely powerful oxidizing agent, which can be attributed to the formation of nascent oxygen.
• Some reactions depicting oxidizing nature of ozone are given as follows:

3PbS(s) + 4O3 (g) → 3PbSO4 (s)

2NO2 (g) + O3 (g) → N2O5 (g) + O2 (g)

• O3 adds to unsaturated organic compounds at room temperature, forming ozonides.


• Potassium ozonide KO3 is an orange-coloured solid and contains the paramagnetic O3− ion.

2KOH + 5O3 → 2KO3 + 5O2 + H2O

(iv) Uses
Disinfectant, mild bleaching agent and an oxidizing agent.
16. Sulphur
(a) Allotropic forms
Melting Specific grav-
Allotrope point (K) ity (g cm–3) Preparation Arrangement of S atoms
Rhombic
Occurs naturally as large yel- All three forms
sulphur or 385.8 2.069
low crystals in volcanic areas. contain puckered S8
a-sulphur
rings with a crown
Made by melting the rhombic
Monoclinic or conformation, and
393 1.94–2.01 sulphur in a dish and then
b -sulphur differ only in the
allowing it to cool.
overall packing
Made by chilling hot concen- of the rings in the
g -Monoclinic 2.19
trated solutions of S in solvents crystal.
sulphur
such as CS2, toluene or EtOH.

205.7 pm
S S
Engel’s Contains S6 rings
sulphur arranged in chair
(e -sulphur) conformation. 102.2°
S S

Chapter-11.indd 339 7/30/2016 12:58:22 PM


340 Chapter 11 The p-Block Elements

Tip Transition temperature (369 K) is the temperature at which both a and b forms are stable. Above the transition
temperature, b form is stable and gets converted into a form below it and vice versa.

(b) Sulphur dioxide


(i) SO2 gas forms discrete V-shaped molecules with bond angle is 119°30′. The molecule can be considered as resonance hybrid
of the following two structures.

O
S
O
+ +
S S
− −
O O O O

(ii) Preparation
• By burning S in air.
S(s) + O2 (g) → SO2 (g)
• By burning H2S in air.
H2S(g) + O2 (g) → SO2 (g) + H2 (g)
• By roasting various metal sulphide ores with air in smelters.
4FeS2 (s) + 11O2 (g) → 2Fe2O3 (s) + 8SO2 (g)
• By treating a sulphite with dilute sulphuric acid.
+
SO2−
3 ( aq) + 2H ( aq) → H2O + SO2 ( g)

(iii) Properties
• Sulphur dioxide is a colourless gas (b.p. −10°C, m.p. −75.5°C) and is very soluble in water.
• It reacts with aqueous NaOH to form sodium sulphite and later bisulphate.
2NaOH + SO2 → Na2SO3 + H2O
Na2SO3 + H2O + SO2 → 2NaHSO3
• It reacts with chlorine in the presence of charcoal as catalyst to form sulphuryl chloride.

SO2 (g) + Cl2 (g) → SO2Cl2 (l)

• In the presence of V2O5, it reacts with oxygen to give sulphur trioxide.


VO
2SO2 (g) + O2 (g) 
2 5
→ 2SO3 (g)

• Laboratory test for SO2: It turns a moist acidified potassium dichromate solution green, due to the formation of Cr3+ and
acts as a reducing agent.
K 2Cr2O7 + 3SO2 + H2SO 4 → Cr2 (SO 4 )3 + K 2SO 4 + H2O
It also turns starch iodate paper blue (due to starch and I2).
2KIO3 + 5SO2 + 4H2O → I2 + 2KHSO 4 + 3H2SO 4
(iv) Uses
Manufacture of H2SO4, making sulphites SO32− (for bleaching, and for preserving food and wine), as a non-aqueous solvent.
(c) Oxoacids of sulphur
(i) Sulphurous acid series

Name Formula Structure Oxidation number

S
Sulphurous acid H2SO3 HO O +IV
HO

(Continued)

Chapter-11.indd 340 7/30/2016 12:58:25 PM


Summary 341

(Continued)
Name Formula Structure Oxidation number
S
Thiosulphurous acid H2S2O2 +IV, −II
HO S OH

Dithionous acid or Disul- O O


H2S2O4 +III
phurous acid HO S S OH

O O
Di or pyrosulphurous acid H2S2O5 HO S S +V, +III
OH
O

(ii) Sulphuric acid series

Name Formula Structure Oxidation number


O
Sulphuric acid H2SO4 S +VI
HO
O
HO

S
Thiosulphuric acid H2S2O3 HO S OH +VI, −II
O

O O
Di or pyrosulphuric H2S2O7 S S +VI
O O O
OH OH

(iii) Thionic acid series

Name Formula Structure Oxidation number


O O
Dithionic acid H2S2O6 S S +V
O
O OH OH

O O

Polythionic acid H2(S)n+2O6 HO S (S)n S OH +V, 0


O O

(iv) Peroxoacid series

Name Formula Structure Oxidation number


O
Peroxomono sulphuric acid S
H2SO5 +VI
or Caro’s acid HO O OH
O

O O
Peroxo disulphuric acid or
H2S2O8 S S +VI
Marshall’s acid O O O O
HO OH

Chapter-11.indd 341 7/30/2016 12:58:27 PM


342 Chapter 11 The p-Block Elements

(d) Sulphuric acid


(i) Preparation
Contact process
Concentrated
Air Air sulphuric acid

Combustion Sulphur Sulphur


trioxide Oleum
furnace dioxide

Sulphur V2O5 catalyst


Heat is 450°C Heat is Water
recovered recovered
Sulphuric
acid
VO
• SO2 (g) + O2 (g) 
2 5
→ 2SO3 (g) ∆H = − 196.6 kJ mol−1
• The SO3 is passed into 98% H2SO4, forming pyrosulphuric acid H2S2O7, sometimes called oleum.
• Oleum is diluted with water to give concentrated sulphuric acid which is a 98% mixture with water (18 M solution).
H2S2O7 + H2O → 2H2SO 4
(ii) Properties
• It is strongly hydrogen bonded, and in the absence of water it does not react with metals to produce H2.
• Strong acidic character:
K > 10
H2SO 4 (aq) + H2O(l) 
a1
→ H3O+ (aq) + HSO 4− (aq)
K = 1.2 × 10 −2
HSO 4− (aq) + H2O(l) 
a2
→ H3O+ (aq) + SO24− (aq)

• Strong affinity for water: HNO3 + 2H2SO 4 → NO2+ + H3O + + 2HSO 4−


H SO
• Charring of organic compounds: C12H22O11 
2 4
→12C + 11H2O
• Strong oxidizing agent: Cu + 2H2SO 4 (conc ) → CuSO 4 + SO2 + 2H2O
(iii) Uses
Converting calcium phosphate into superphosphate, sulphonation of fatty acids, catalyst in the production of high octane
fuels, electrolyte in lead storage batteries as a non-aqueous solvent.
17. General properties of Group 17 elements
First Electron Hydration Standard
Oxi- ionization gain enthalpy electrode
Atomic Electronic config- dation enthalpy enthalpy of X− potential m.p. b.p.
Elements Occurrence number uration state (kJ mol−1) (kJ mol–1) (kJ mol–1) Eç (V) (çC) (çC)
Fluorspar or fluorite
Fluorine (CaF2)
9 [He] 2s2 2p5 –1 1681 –333 –513 +2.87 –219 –188
(F) fluoroapatite
[3(Ca3(PO4)2 ∙ CaF2]
Dried-up beds of
inland lakes and seas –1, +1,
Chlorine containing deposits +3, +4,
17 [Ne] 3s2 3p5 1256 –349 –370 +1.40 –101 –34
(Cl) of NaCl, with smaller +5, +6,
amounts of CaCl2, +7
KCl and MgCl2
–1, +1,
Bromine
Sea water 35 [Ar] 3d10 4s2 4p5 +3, +4, 1143 –325 –339 +1.09 –7 60
(Br)
+5, +6
Natural brine
–1, +1,
Sodium iodate
Iodine (I) 53 [Kr] 4d10 5s2 5p5 +3, +5, 1009 –296 –274 +0.62 114 185
(NaIO3) and sodium
+7
periodate (NaIO4)
Astatine [Xe] 4f14
85 –270 +0.3
(At) 5d10 6s2 6p5

Chapter-11.indd 342 7/30/2016 12:58:30 PM


Summary 343

(a) The atomic and ionic radii increase from F to I as the number of quantum shells increase.
(b) The atomic radii of halogens are the smallest in the respective periods due to maximum effective attraction of electrons by the
nuclear charge.
(c) The ionization enthalpies of the halogens decreases as the atoms increase in size. There is little tendency for the atoms to lose
electrons and form positive ions.

Tip The ionization enthalpy for F is appreciably higher than for the others, because of its small size.

(d) The electron gain enthalpies for all the halogens are negative.
(e) The electronegativity of the halogens is very high and it decreases from F to I. Fluorine is the most electronegative element in
the Group.
(f) The melting and boiling points of the elements increase with increased atomic number.
(g) All the elements exist as diatomic molecules, and they are all coloured.

Tip The colours arise from the absorption of light on promoting an electron from the ground state to a higher state.

(h) Bond energy in the X2 molecules decrease as the atoms become larger, other than F2. The bond energy in F2 is abnormally low
contributing to its very high reactivity.
18. Chemical properties of Group 17 elements
(a) Oxidation states
(i) Fluorine is always univalent while in Cl, Br and I, the oxidation state may be either (–1) or (+1).
(ii) Cl, Br and I also exhibit higher valencies, with oxidation numbers of (+III), (+V) and (+VII).
(iii) The oxidation states (+4) and (+6) occur in the oxides ClO2, BrO2, Cl2O6 and BrO3.
(iv) Chlorine has the highest electron gain enthalpy with a negative sign, so gaseous Cl atoms accept electrons most readily.
However, Cl is not the strongest oxidizing agent.
(v) Fluorine accepts electrons more readily than chlorine, so fluorine is the strongest oxidizing agent. There are two main
reasons for this change of order:
• F2 has a low enthalpy of dissociation (arising from the weakness of the F F bond).
• F2 has a high enthalpy of hydration (arising from the smaller size of the F− ion).
(b) Anomalous behaviour of fluorine
The reasons for the difference of fluorine from remaining group are:
(i) The first element is smaller than the rest, and holds its electrons more firmly.
(ii) It has no low-lying d orbitals which may be used for bonding.
(iii) HF is a liquid, while HCl and HBr are gases.

Tip The great reactivity of fluorine leading to exothermic reactions is attributable to two factors:
(i) The low dissociation energy of the F F bond (which results in low activation energy for the reaction).
(ii) The very strong bonds which are formed with other elements.

(c) Some important reactions of halogens


Reaction Comment
2F2 + 2H2O → 4H+ + 4F− + O2 Vigorous reaction with F.
2I2 + 2H2O → 4H+ + 4X− + O2 I reaction in reverse direction.
X2 + H2O → H+ + X− + HOX Cl > Br > I (F not at all).
All the halogens.
Reactivity towards hydrogen decreases down the Group. Hydrogen and fluorine react
violently.
X2 + H2 → 2HX
The acidic strength in aqueous solution increases from HF to HI.
The stability of halides decreases from HF to HI as the bond dissociation enthalpy
decreases.
Most metals form halides.
nX2 + 2M → 2MXn F the most vigorous.
The ionic character of metal (monovalent) halides decreases as follows: F > Cl > Br > I.
(Continued)

Chapter-11.indd 343 7/30/2016 12:58:30 PM


344 Chapter 11 The p-Block Elements

(Continued)
Reaction Comment
All the halogens form trihalides.
3X2 + 2P → PX3 As, Sb and Bi also form trihalides.
5X2 + 2P → PX5 F, Cl and Br form pentahalides.
AsF5, SbF5, BiF5, SbCl5
X2 + 2S → S2X2 Cl and Br
2Cl2 + S → SCl4 Cl only
3F2 + S → SF6 F only
X2 + H2S → 2HX + S All the halogens oxidize S2– to S.

3X2 + 8NH3 → N2 + 6NH4X F, Cl and Br

X2 + X′ 2→ 2XX’ Interhalogen compounds formed higher interhalogen compounds.


X2 + X’X → X’X3

(iv) Reactivity towards oxygen


• Binary compounds of F and O are fluorides of oxygen rather than oxides of fluorine (e.g. OF2 and O2F2).
• The other halogens are less electronegative than oxygen and thus form oxides.
• The general order of decreasing stability of halogen oxides is I > Cl > Br with higher oxides being more stable than the
lower ones. The higher oxidation states are more stable than the lower states.
• Most of the chlorine oxides, for example, Cl2O, ClO2, Cl2O6 and Cl2O7 are unstable, are highly reactive and strong oxidiz-
ing agents. ClO2 is used in disinfection of water and as a bleaching agent for paper pulp and textiles.
• The iodine oxides (I2O4, I2O5, I2O7) are the most stable. Bromine oxides (e.g., Br2O, BrO2 and BrO3) are the least stable of
halogen oxides but are also strong oxidizing agents.
19. (a) Chlorine
(i) Preparation
• By heating MnO2 with concentrated hydrochloric acid.
4HCl + MnO2 → MnCl2 + 2H2O + Cl2
• By heating MnO2 and NaCl with concentrated hydrochloric acid.
4NaCl+MnO2 + 4H2SO 4 → MnCl2 + 4NaHSO 4 + 2H2O2
• By the reaction of potassium permanganate with concentrated hydrochloric acid.
16HCl + KMnO 4 → 2MnCl2 + 8H2O + 5Cl2 + 2KCl

• By the electrolysis of aqueous NaCl solutions where it is liberated at anode in the manufacture of NaOH.
Electrolysis
2NaCl + 2H2O 
→ 2NaOH + Cl2 + 2H2

• By Deacon’s process where HCl gas is oxidized by atmospheric oxygen in the presence of CuCl2 (catalyst) at 723 K.
CuCl
4HCl + O2 
2
→ 2Cl2 + 2H2O
(ii) Properties
Reaction with Example
Cl2 + H2O → HCl + HOCl
Water
As a result of this reaction, chlorine water loses its yellow colour on standing.
Bleaching action
Cl2 + H2O → 2HCl + O
Metals Cl2 + 2Na → 2NaCl

Non-metals Cl2 + 2S → S2Cl2


Hydrogen and
Cl2 + H2 → 2HCl
compounds con-
taining hydrogen Cl2 + H2S → 2HCl + S

(Continued)

Chapter-11.indd 344 7/30/2016 12:58:32 PM


Summary 345

(Continued)
Reaction with Example
3Cl2 + 8NH3 → N2 + 6NH4 Cl
Ammonia
3Cl2 (excess) + NH3 → 3HCl + NCl3

NaOH Cl2 + 2NaOH → NaCl + NaOCl + H2O


6NaOH + 3Cl2 → NaClO3 + 5NaCl + 3H2O

Slaked lime 2Cl2 + 2Ca(OH)2 → CaCl2 + Ca(OCl)2 + 2H2O

Carbon monox- Cl2 + CO → COCl2


ide and sulphur
dioxide Cl2 + SO2 → SO2Cl2

Oxidation of
sulphur dioxide Cl2 + SO2 + 2H2O → H2SO 4 + 2HCl

Oxidation of Na2SO3 + Cl2 + H2O → Na2SO 4 + 2HCl


sulphite salts
2FeSO 4 + H2SO 4 + Cl2 → Fe2 (SO 4 )3 + 2HCl
Saturated hydro- Cl2 + CH4 → CH3Cl + HCl
carbons
Cl2 + C2H4 → C2H4 Cl2

(b) Hydrogen Chloride


(i) Preparation
HCl is made by adding concentrated H2SO4 to rock salt (NaCl).
150°C 550°C
NaCl + H2SO 4 → HCl(g) + NaHSO 4 ; NaCl + NaHSO 4 → HCl(g) + Na2SO 4

(ii) Properties
K = 107
• Reaction with water: HCl(g) + H2O(l) 
a
→ H3O+ (aq) + Cl− (aq)

• Decomposition of salts of weaker acids: K 2CO3 + 2HCl → 2KCl + H2O + CO2
• Reaction with ammonia: HCl + NH3 → NH4 Cl
(c) Oxoacids of halogens
Four series of oxoacids formed by halogens are HOX, HXO2, HXO3 and HXO4.
− − − O −

x x x x
O O O O O O
O O O
OX XO−2 XO3− XO4−
(d) Interhalogen compounds
The halogens react with each other to form interhalogen compounds. These are divided into four types XX′, XX′ 3 , XX′ 5 and XX′7.
(X = halogen of larger size, X′ = halogen of smaller size).
(i) Preparation
They can all be prepared by direct reaction between the halogens
200° C
Cl2 + F2 (equal volumes) → 2ClF

I2 + Cl2 liquid (excess Cl2) → (ICl3)2


(ii) Properties
• Their melting and boiling points are a little higher than expected, and the melting and boiling points increase as the
difference in electronegativity between two atoms increases.
• The bonds are essentially covalent because of the small electronegativity difference.
• These are diamagnetic in nature.
• The interhalogens are generally more reactive than the halogens (except F2). This is because the X X′ bond in interhal-
ogens is weaker than the X X bond in the halogens.
• The reactions of interhalogens are similar to those of the halogens.

Chapter-11.indd 345 7/30/2016 12:58:35 PM


346 Chapter 11 The p-Block Elements

(e) A few ions are known, consisting of two or more atoms of which at least one is N, that have properties similar to those of the
halide ions. They are therefore called pseudohalide ions as shown in the below table.
Anion Acid Dimer
CN− Cyanide ion HCN Hydrogen cyanide (CN)2 Cyanogen
SCN− Thiocyanate ion HSCN Thiocyanic acid (SCN)2 Thiocyanogen
SeCN− Selenocyanate ion HOCN Cyanic acid (SeCN)2 Selenocyanogen
OCN− Cyanate ion H2NCN Cyanamide
NCN2− Cyanamide ion HOCN Fulminic acid
ONC− Fulminate ion HN3 Hydrogen azide
N3− Azide ion
The best known pseudohalide is CN−. This resembles Cl−, Br− and I− in the following respects:
(i) It forms an acid HCN.
(ii) It can be oxidized to form a molecule cyanogen (CN)2.
(iii) It forms insoluble salts with Ag+, Pb2+ and Hg+.
(iv) Interpseudohalogen compounds ClCN, BrCN and ICN can be formed.
(v) AgCN is insoluble in water but soluble in ammonia, as is AgCl.
(vi) It forms a large number of complexes similar to halide complexes, e.g. [Cu(CN)4]2− and [CuCl4]2−, and [Co(CN)6]3− and [CoCl6]3−.
20. General properties of Group 18 elements
The elements of Group 18 have been called the inert gases or the rare gases.
Atomic Atomic First ionization m.p. b.p.
Elements Occurrence number Electronic configuration radius (pm) enthalpy (kJ mol−1) (çC) (çC)
He 2 1s2 120 2372 –269.0
Ne 10 [He] 2s2 2p6 160 2080 –248.6 –246.0
Ar Atmosphere 18 [Ne] 3s2 3p6 191 1521 –189.4 –186.0
Kr 36 [Ar] 3d10 4s2 4p6 200 1351 –157.2 –153.6
Xe 54 [Kr] 4d10 5s2 5p6 220 1170 –111.8 –108.1
Decay of radioactive radi-
Rn 86 [Xe] 4ƒ14 5d10 6s2 6p6 1037 –71 –62
um and thorium minerals
(a) Helium has two electrons which form a complete shell 1s2. The other noble gases have a closed octet of electrons in their outer
shell ns2 np6. This electronic configuration is very stable explaining their inertness.
(b) These atoms have an electron affinity of zero and have very high ionization energies.
(c) Under normal conditions, the noble gas atoms have little tendency to gain or lose electrons. Thus, they have high electron gain
enthalpy, little tendency to form bonds, and so they exist as single atoms.
(d) They have high ionization enthalpy which decreases on descending the group due to increase in atomic size.
(e) The atomic radii of the elements are all very large, and increase on descending the group.
21. (a) Chemical properties of Group 18 elements
(i) Xenon–fluorine and Xenon–oxygen compounds
Oxida- m.p.
Formula Name tion state (çC) Structure Preparation
F
400°C, 1 bar
2 : 1 mixture XeF2
Linear 600°C, 7 bar

Xenon diflu-
XeF2 +2 129 (RnF2 and XeCl2 Xe Xe + F2 → 1 : 5 mixture XeF4
oride
are similar)

300°C, 60-70 bar


1 : 20 mixture XeF6
F

F F
Xenon Square planar Xe
XeF4 +4 117
tetrafluoride (XeCl4 is similar)
F F

(Continued)

Chapter-11.indd 346 7/30/2016 12:58:36 PM


Summary 347

(Continued)
Oxida- m.p.
Formula Name tion state (çC) Structure Preparation
F

Xenon hex- Distorted octa- F


XeF6 +6 49.6 Xe F
afluoride hedron F
F

Pyramidal
3XeF4 + 6H2O → 2Xe + XeO3 + 12HF
Xenon Ex- (tetrahedral Xe
XeO3 +6 + 121 O2
trioxide plodes with one corner O O
unoccupied)
XeF6 + 6H2O → XeO3 + 6HF
O
Pyramidal

Trigonal bipyr- O
amid Xe
Xenon dioxy
XeO2F2 +6 30.8 (with one
difluoride
position unoc- O
cupied)
F
XeO2F2

O
Square pyram- F F 2XeF6 + SiO2 → XeOF4 + SiF4
Xenon oxy idal
XeOF4 tetra fluo- +6 –46 (octahedral with Xe
XeF6 + H2O → XeOF4 + 2HF
ride one position F F XeF6 + 2H2O → XeO2F4 + 4HF
unoccupied)
Square pyramidal
O
Xenon Xe
XeO4 +8 –35.9 Tetrahedral
tetroxide O
O O
F
Xenon trioxy Trigonal bipyr- O
XeO3F2 +8 –54.1 O Xe
difluoride amid O
F
Barium dec. >
Ba2[XeO6]4– +8 Octahedral
perxenate 300

(ii) Reactions of XeF2, XeF4 and XeF6

Reaction type Example


XeF2 + H2 → 2HF + Xe
With hydrogen
XeF4 + 2H2 → 4HF + Xe
XeF6 + 3H2 → 6HF + Xe

(Continued)

Chapter-11.indd 347 7/30/2016 12:58:39 PM


348 Chapter 11 The p-Block Elements

(Continued)
Reaction type Example
XeF2 + 2HCl → 2HF + Xe + Cl2
Oxidation
XeF4 + 4KI → 4KF + Xe + 2I2
SO + XeF2 + Ce(SO 4 )3 → 2CeIV (SO 4 )2 + Xe + F2

Fluorination XeF4 + 2SF4 → Xe + 2SF6


XeF4 + Pt → Xe + PtF4

Hydrolysis 2XeF2 + 2H2O → 2Xe + 4HF + O2

SOLVED EXAMPLES
General Properties of Group 13 Elements (a) The atomic radius of gallium (Ga) is larger than that of alu-
minium (Al) because it lies below Al in the Group.
1. Which of these statements is not true? (b) Compounds such as AlCl3 and GaCl3 are covalent when
(a) NO + is not isoelectronic with O2. anhydrous but form cations in solution.
(b) B is always covalent in its compounds. (c) Both aluminium and thallium form trihalides only.
(c) In aqueous solution, the Tl (I) ion is much more stable than (d) Aluminium compounds are detected in solution when
Tl (III). addition of NH4OH gives white precipitate of Al(OH)3 that
(d) LiAlH4 is a versatile reducing agent in organic synthesis. dissolves in excess NH4OH.
(JEE Main Online 2014)
Solution
Solution
(b) The elements of Group 13 generally form covalent com-
(a), (b), (c), (d) All the options are correct. pounds because the sum of three ionization enthalpies
(a) The number of electrons in NO+ is 14 while O2 has 16 elec- for each element is high. The change from covalent to
trons. Thus, they are not isoelectronic. ionic happens for compounds such as AlCl3 and GaCl3
(b) Due to small size and high ionization enthalpy boron when the compounds are hydrated and the amount of
always form covalent compounds. hydration enthalpy is more than the ionization enthalpy.
(c) The Tl (I) ion is much more stable than Tl (III) due to inert The compounds thus ionize in solution.
pair effect. For option (a): The atomic radius generally decreases
(d) LiAlH4 is widely used as reducing agent in organic synthesis. on moving down a Group. However, atomic radius Ga
is smaller than that of Al due to ineffective shielding by
2. Which one of the following is an electron-deficient species?
− intervening 3d orbitals.
(a) B2H6 (b) NH3 (c) AIH4 (d) CH4
For option (c): Aluminium halide is trivalent, while thal-
Solution lium halide is monovalent. This is because of the inert
pair effect which causes the two s-electrons of the out-
(a) After having 3 center-2 electron bonds also, B2H6 is still
ermost electronic shell to remain non-participative in
electron deficient. The octet/duplet is complete for the
bonding. The trend increases down the Group, so Ga that
central atom in the rest of the hydrides, so they are not
comes after Al in the Group also forms trihalides (e.g.,
electron deficient.
GaF3) but thallium (the last element in the Group) has
H N only one p-electron available for bond forming and forms
H B B H TlCl only.
H H NH3: H H H
For option (d): The precipitate of Al(OH)3 formed does not
B2H6: H dissolve in excess NH4OH.
− H
H 4. The BCl3 is a planar molecule, whereas NCl3 is pyramidal
Al C because
(a) N Cl bond is more covalent than B Cl bond.
AlH−4: H H H CH4: H H H
(b) B Cl bond is more polar than N Cl bond.
(c) Nitrogen atom is smaller than boron.
3. Which of the following statements is correct regarding Group (d) BCl3 has no lone pair but NCl3 has one lone pair of
13 elements/compounds? electrons.

Chapter-11.indd 348 7/30/2016 12:58:41 PM


Solved Examples 349

Solution 9. Orthoboric acid contains


(d) BCl3 has no lone pair as B has only three valance electrons (a) triangular BO3−3 units.
thus its shape is trigonal planar; while NCl3 has one lone (b) pyramidal BO3− 3 units.
pair of electron as N has 5 valence electrons, so its shape (c) T-shaped BO3− 3 units.
is pyramidal.
(d) irregular tetrahedral BO3−
3 units.
Cl Solution
N
Cl Cl B (a) Two-dimensional sheet structure of boric acid is shown
Cl Cl Cl below. Dotted lines represent hydrogen bonds.
5. Which of the following has the lowest melting point? H H
(a) B (b) Ga (c) Al (d) Tl O O
Solution H H B H H
(b) Gallium has an unusual structure. Each metal atom has one O O
close neighbour at a distance of 2.43 Å and six more dis- O
tant neighbours at distances between 2.70 Å and 2.79 Å. H B H H B H
This remarkable structure tends towards discrete dia- O O O O
tomic molecules rather than a metallic structure. This
accounts for the incredibly low melting point of gallium H H H H
of 30°C. O O O O
H H H H
B B
Compounds of Boron O

6. Boron cannot form which one of the following anions? O O


H H B H H
(a) BH4− (b) B(OH)4− (c) BO2− (d) BF63− O O
(AIEEE 2011)
H H
Solution
10. In diborane, the two H B H angles are nearly
(d) As boron has only four orbitals in the valence shell (i.e., 2s, (a) 97°, 120° (b) 60°, 120° (c) 120°, 180° (d) 95°, 150°
2px, 2py, 2pz), due to non-availability of d orbitals, it cannot
expand its covalency beyond four. Therefore, [BF6]3− is not Solution
possible. (a) Due to the presence of bridging hydrogens.
7. The maximum number of atoms is lying in one plan for B2H6 is 178 pm
(a) 4 (b) 5 (c) 6 (d) 8
Solution
H
H
(c) 178 pm
B 97° H
H B
120°
H H
H H
B 97° H
H B
120° 133 pm 119 pm
H
H 11. How many double bonds between B and N are there in inor-
ganic benzene?
(a) Zero (b) 6 (c) 3 (d) 4
133 pm 119 pm
Solution
From the structure it is clear that maximum 6 atoms are (c) The structure of borazine is
lying in one plane.
H
8. Which of the following does not exist in free form?
H B H
(a) BF3 (b) BH3 (c) BCl3 (d) BBr3
N N
Solution B B
N
(b) BH3 does not exist in free form because this compound is H H
unstable at room temperature. As a result, it forms higher H
hydrides such as B2H6, B4H10, B10H14 among others. Borazine

Chapter-11.indd 349 7/30/2016 12:58:45 PM


350 Chapter 11 The p-Block Elements

12. Number of hydroxyl groups attached to boron in borax are (a) 1 and 2 (b) 1 and 3
(a) four. (b) five. (c) six. (d) ten. (c) 2 and 4 (d) 3 and 4
(JEE Main Online 2014)
Solution
Solution
(a) Borax (Na2[B4O5(OH)4]·8H2O) is made up of two triangu-
(b) The amphoteric behaviour of Al(OH)3 can be explained
lar and two tetrahedral units. This ion is [B4O5(OH)4]2– and
when it reacts with acid and base. Al2O3∙ xH2O exist in
the other water molecules are associated with the metal
hydrated form. The reactions are
ions.
Al2O3∙ xH2O + H+ → [Al(H2O)6]3+
OH Base Acid
Al2O3∙xH2O + OH− → NaAl(OH)4
B− Acid Base Sodium aluminate
O O Thus, 1 and 3 set of reactions explain amphoteric charac-
HO B O B OH ter of Al2O3∙xH2O.
O O 16. Which of the following is formed when aluminium oxide and
B− carbon are strongly heated in dry chlorine gas?
(a) Aluminium chloride
OH
(b) Hydrated aluminium chloride
Thus, four OH groups are attached to boron in borax. (c) Anhydrous aluminium chloride
13. The structure of diborane (B2H6) contains (d) Aluminium carbide
(a) four 2c-2e bonds and two 3c-2e bonds. Solution
(b) two 2c-2e bonds and four 3c-2e bonds. (c) When aluminium oxide and carbon are strongly heated
(c) two 2c-2e bonds and two 3c-3e bonds. in dry chlorine gas anhydrous aluminium chloride is
(d) four 2c-2e bonds and four 3c-2e bonds. formed. The reaction involved is
Solution 100°C
Al2O3 + 3C + 3Cl2 → 2AlCl3 + 3CO
(a) Aluminium Anhydrous
2c-2e 3c-2e
H oxide aluminium chloride
bond bond
H H 17. Aluminium chloride exists as dimer, Al2Cl6, in solid state as
well as in solution of non-polar solvents such as benzene.
B B 2c-2e When dissolved in water, it gives
bond (a) [Al(OH)6]3− + 3HCl (b) Al2O3 + 6HCl
H H (c) Al3+ + 3Cl− (d) [Al(H2O)6]3+ + 3Cl−
H
Solution
2c-2e 3c-2e 2c-2e
bond bond bond (d) AlCl3 is covalent, but in water it becomes ionic due to
large hydration energy of Al3+.
Compounds of Aluminium Al2Cl6 + 6H2O → 2AlCl3∙ 6H2O or [Al(H2O)6]3+ + 3Cl−
14. Which one of the following is the correct statement? 18. Heating an aqueous solution of aluminium chloride to dry-
(a) Boric acid is a protonic acid. ness will give
(b) Beryllium exhibits coordination number of six. (a) AlCl3 (b) Al2Cl6
(c) Chlorides of both beryllium and aluminium have bridged (c) Al2O3 (d) Al(OH)Cl2
chloride structures in solid phase. Solution
(d) B2H6⋅2NH3 is known as “inorganic benzene”.
(c) Al2Cl6∙ 6H2O → Al2O3 + 6HCl + 3H2O
(AIEEE 2008)
Solution General Properties of Group 14 Elements
(c) The structures of AlCl3 and BeCl2 are as follows: 19. The stability of dihalides of Si, Ge, Sn and Pb increases steadily
Cl Cl Cl Cl Cl Cl in the sequence:
Al Al Be Be Be (a) PbX2  SnX2  GeX2  SiX2
Cl Cl Cl Cl Cl Cl
(b) GeX2  SiX2  SnX2  PbX2
B3N3 is called inorganic benzene and boric acid is a Lewis acid.
(c) SiX2  GeX2  PbX2  SnX2
15. In the following sets of reactants, which two sets best exhibit
the amphoteric character of Al2O3⋅ xH2O? (d) SiX2  GeX2  SnX2  PbX2
(AIEEE 2007)
Set1: Al2O3. xH2O (s) and OH−(aq) Solution
Set2: Al2O3. xH2O (s) and H2O (l) (d) Due to inert pair effect, the stability of +2 oxidation state
Set3: Al2O3. xH2O (s) and H+ (aq) increases as we move down the group. Therefore,
Set4: A12O3. xH2O (s) and NH3 (aq) SiX2  GeX2  SnX2  PbX2

Chapter-11.indd 350 7/30/2016 12:58:47 PM


Solved Examples 351

20. In which of the following pairs, the two species are not 22. Which of the following exists as covalent crystals in the solid
isostructural? state?
(a) PCl+4 and SiCl4 (b) BF4− and SF4 (a) Silicon (b) Sulphur
(c) SF6 and AlF63− (d) CO2 and NO2+ (c) Phosphorus (d) Iodine
Solution (JEE Main 2013)
(b) Solution
For option (a): Tetrahedral, tetrahedral (a) Silicon (Si) exists as covalent solid, sulphur (S8) exists as
+ molecular solid, phosphorus (P4) exists as molecular solid
Cl Cl and iodine (I2) exists as molecular solid.
P Si 23. The gas evolved on heating CaF2 and SiO2 with concentrated
Cl Cl Cl Cl Cl Cl H2SO4, on hydrolysis gives a white gelatinous precipitate. The
precipitate is
For option (b): Tetrahedral, see-saw (a) hydrofluosilicic acid. (b) silica gel.
BF4− has 4 bond pairs and 0 lone pairs on cen- (c) silicic acid. (d) calciumfluorosilicate.
tral atom. Hence, boron is sp3 hybridized and (JEE Main Online 2014)
tetrahedral in shape.
− Solution
F
(c) Silica is unreactive but its product when it reacts with
B H2SO4 and CaF2 followed by hydrolysis is
F F F H2SO4 + CaF2 → HF + CaSO4
SF4 has 4 bond pairs and 1 lone pair on the central atom. HF + SiO2 → SiF4 + H2O
Hence, sulphur is sp3d hybridized and see-saw in shape.
SiF4 + H2O → HF + Si(OH)4 or SiO2∙ 2H2O
F
Silicic acid
S F
F SiF4 comes in contact with a drop of H2O and hydrolyzed
F to silicic acid. This can be seen as white solid forming on
the surface of drop of water.
For option (c): Octahedral, octahedral
3−
24. Example of a three-dimensional silicate is
F F (a) zeolites. (b) ultramarines.
F F F F (c) feldspars. (d) beryls.
(JEE Main Online 2014)
S Al
Solution

F F F F (a), (b), (c) Zeolites, ultramarines and feldspars are all three
dimensional silicates.
F F
For option (d): Linear, linear 25. The water repellant characteristics of silicone is because
+ (a) Si atom has no vacant d-orbital available for nucleophillic
O C O O N O
attack by water molecule.
Compounds of Silicon (b) Silicone chain is surrounded by organic side groups and
looks like an alkane from outside.
21. Among the following substituted silanes, the one which will
(c) The average chain lengths of silicones are very high.
give rise to crosslinked silicone polymer on hydrolysis is
(d) Low thermal stability.
(a) R4Si (b) RSiCl3
(c) R2SiCl2 (d) R3SiCl Solution
(AIEEE 2008) (b)
Solution R R R R
(b) Hydrolysis of alkyl trichlorosilane RSiCl3 gives complex Si O Si O Si O Si
cross linked polymer.
R R R R
Si Si The Si O Si skeleton is protected by water repelling
O O alkyl groups that orient themselves towards the surface
Cl H2O
OH Condensation creating a water proof structure, like an alkane.
R Si Cl R Si OH R Si O Si R Option (a) is incorrect as Si has vacant 3d orbital.
Polymerization
Cl OH Option (c) is incorrect as there is no relation between
O O
chain length and water repelling characteristics.
Si Si
n

Chapter-11.indd 351 7/30/2016 12:58:50 PM


352 Chapter 11 The p-Block Elements

26. How many oxygen atoms are shared by silicate units in form- basic strength decreases as we move from NH3
ing pyrosilicates? to BiH3.
(a) Zero (b) 1 (c) 2 (d) 3 In option (d): Increasing ionization enthalpy: B < C < O < N.
Nitrogen has half-filled orbitals which have
Solution
extra stability so removal of electron is difficult
(b) The structure is as compared to B, C, O.
− −
29. Which of the following statements is wrong?
− − O
Si (a) Nitrogen cannot form dp–pp bond.
− − (b) Single N N bond is weaker than the single P P bond.
Si2O6−
7 ion (c) N2O4 has two resonance structures.
(d) The stability of hydrides increases from NH3 to BiH3 in
27. In silicon dioxide
Group 15 of the periodic table.
(a) each silicon atom is surrounded by four oxygen atoms
(AIEEE 2011)
and each oxygen atom is bonded to two silicon atoms.
Solution
(b) each silicon atom is surrounded by two oxygen atoms
and each oxygen atom is bonded to two silicon atoms. (d) Except statement (d) all the statements are correct.
(c) silicon atom is bonded to two oxygen atoms. Stability of hydrides decreases down the group from
(d) there are double bonds between silicon and oxygen NH3 to BiH3 as the M H bond energy decreases. In other
atoms. words, as we move down the group the size of the central
atom increases, and hence its tendency to form covalent
Solution bond with small H atom decreases.
(a) Si is tetrahedrally surrounded by four O atoms. Each cor-
30. The molecule having smallest bond angle is
ner is shared with another tetrahedron, thus giving an
(a) NCl3 (b) AsCl3
infinite array. The difference between these structures is
(c) SbCl3 (d) PCl3
the way in which the tetrahedral SiO4 units are arranged.
(AIEEE 2012)
Si O Si O Si Solution
(c) On moving down in a group from nitrogen to antimony
O O O (Sb), the size of the central metal atom increases and
Si O Si O Si electronegativity decreases. As a result, lone pair–bond
pair repulsions increase and bond angle decreases.
O O O The order of increase in bond angle is
Si O Si O Si NCl3 > PCl3 > AsCl3 > SbCl3
31. Which one of the following does not have a pyramidal shape?
General Properties of Group 15 Elements (a) (CH3)3 N (b) (SiH3)3N
(c) P(CH3)3 (d) P(SiH3)3
28. In which of the following arrangements, the sequence is not (JEE Main Online 2014)
strictly according to the property written against it?
(a) CO2 < SiO2 < SnO2 < PbO2: Increasing oxidizing power. Solution
(b) HF < HCl < HBr < HI: Increasing acidic strength.
(b)
(c) NH3 < PH3 < AsH3 < SbH3: Increasing basic strength.
H3Si
(d) B < C < O < N: Increasing first ionization enthalpy.
(AIEEE 2009)
Solution N SiH3
(c)
SiH3
In option (a): Increasing oxidizing power: CO2 < SiO2 <
SnO2 < PbO2. This is because Pb in (+4) oxidation
state is the strongest oxidizing agent. Due to back bonding between N and Si atoms, (SIH3)3N
becomes planar.
PbO + 4HCl → PbCl + Cl2 + H2O
32. The best reducing agent among the following is
In option (b): Increasing acid strength: HF < HCl < HBr < HI. (a) NH3 (b) SbH3 (c) PH3 (d) AsH3
Higher bond dissociation energy, lower is the
degree of ionization and hence weaker is the Solution
acid and the bond dissociation energy is high- (b) In moving from top to bottom in a group, the reducing
est for HF. nature of hydride increases. Also, in moving from top to
In option (c): Increasing order of basic strength: SbH3 < AsH3 < bottom in a group, the bond length increases as size of
PH3 < NH3. As the size of central atom increases, central atom increases hence bond energy decreases and
the electron density on the central metal atom bond gets easily cleaved.
decreases and consequently its tendency to The increasing order of reducing agent is NH3 < PH3 <
donate a pair of electrons decreases. Hence, the AsH3 < SbH3.

Chapter-11.indd 352 7/30/2016 12:58:51 PM


Solved Examples 353

33. Which of the following is a solid? Solution


(a) Nitrogen trioxide (b) Nitrogen pentoxide (c) Reaction of PH3 with calcium hypochlorite.
(c) Dinitrogen tetroxide (d) Nitric oxide
PH3 + Ca(OCl)2 → PCl3
Solution Reaction of NH3 with calcium hypochlorite.
(b) As N2O5 is a colourless solid, whereas all the others exist
in solid and liquid form both or as gases. 4NH3 + 3Ca(OCl)2 → 3CaCl2 + N2 + H2O

34. The element that has the least tendency to show the inert- 38. (NH4)2CrO7 on heating liberates a gas. The same gas will be
pair effect is obtained by
(a) B (b) P (c) S (d) N (a) heating NH4NO2. (b) heating NH4NO3.
Solution (c) treating Mg3N2 with H2O. (d) heating H2O2 on NaNO2.
(d) Nitrogen since it can remove s electron only by remov- Solution
ing 3p electron. Therefore, net energy required is very (a) The reactions involved are
high which is unable to be overcome by lattice enthalpy, ∆
which is even less than B. (NH4 )Cr2O7 
→ N2 + Cr2O3 + 4H2O
NH4NO2 → N2 + 2H2O
Chemical Properties of Group 15 Elements
Both evolve N2 gas.
35. The acid strength of the following hydrides increases in the
order: 39. N2 forms NCl3 whereas P can form both PCl3 and PCl5. Why?
(a) SiH4 < PH3 < H2S < HCl (b) H2S < PH3 < HCl < SiH4 (a) P has d orbitals which can be used for bonding but N2
(c) PH3 < H2S < HCl < SiH4 (d) H2S < SiH4 < PH3 < HCl does not have d orbitals.
(b) N atom is larger than P in size.
Solution
(c) P is more reactive towards Cl than N.
(a) As the polarity of the bond increases with increase in (d) The size of N is comparable to Cl while P size is greater
∆ c (electronegativity difference in a period from left to than that of Cl.
right with hydrogen), the tendency to lose H+ increases
in polar solvent. Solution
(a) P has empty d orbitals which can be used for bonding
36. Choose the correct options from the following orders. whereas N2 does not have d orbitals.
(a) Basicity order: NH3 > NH2 NH2 > NH2OH > NF3
(b) Melting point order: NH3 < SbH3 < AsH3 < PH3 40. What may be expected to happen when phospine gas is
(c) Boiling point order: NH3 > SbH3 > AsH3 > PH3 mixed with chlorine gas?
(d) Thermal stability: NH3 < PH3 < AsH3 < SbH3 (a) The mixture only cools down.
(b) PCl3 and HCl are formed and the mixture warms up.
Solution
(c) PCl5 and HCl are formed and the mixture cools down.
(a) (d) PH3∙ Cl2 is formed with warming up.
Option (a) is correct as all the three bond pairs are
attracted towards N-atom and increase the electron den- Solution
sity on N-atom in the case of NH3. While in the case of (b) The reaction is
NH2 NH2, bond pair is not attracted towards any
PH3 + Cl2 → PCl3 + HCl
N-atom; for NH2OH oxygen withdraws the bond pair
towards itself; and finally in NF3, three F-atom having −I
effect decrease the electron density on N-atom. Compounds of Phosphorus
Option (b) is incorrect as due to H-bonding in NH3, the 41. Which of the following compounds has a P P bond?
effective molecular mass increases, and overcomes all (a) H3PO4 (b) H4P2O6
other molecular masses. Thus, the melting point is high- (c) H4P2O7 (d) (HPO3)3
est in NH3.
(JEE Main Online 2015)
Option (c) is incorrect as at the boiling point temperature,
the extent of H-bonding is less and cannot overcome the Solution
molecular weight of SbH3. (b) Hypophosphoric acid contains P P linkage.
Option (d) is incorrect as the thermal stability depends O O
upon bond energy of M H bond, which is the highest
(shortest bond length) for NH3 and thus it has the highest P P
stability. HO OH
HO OH
37. PH3 and NH3 on separate reaction with bleaching powder Pyrophosphoric acid (H4P2O7) contains four P OH, two
produce respectively P O and one P O P bonds.
(a) P and N2. (b) PCl3 and NCl3. Orthophosphoric acid (H3PO4) contains three P OH and
(c) PCl3 and N2. (d) PCl5 and NCl3. one P O bonds.

Chapter-11.indd 353 7/30/2016 12:58:52 PM


354 Chapter 11 The p-Block Elements

Metaphosphoric acid (HPO3)3 contains three P OH, Solution


three P O and three P O P bonds. (c) Sulphur can exhibit a minimum oxidation state of (−2).
42. The number of P O P bonds in cyclic metaphosphoric acid is The electronic configuration of sulphur is 3s2 3p4.
(a) zero. (b) two. (c) three. (d) four. 3s 3p
Solution
(c) Cyclic metaphosphoric acid has 3 P O P bonds. So, it can accept two more electrons to attain noble gas
O O configuration. Hence, a minimum of (−2) oxidation state
P is possible. For example, H2S, Na2S.
O O
O O
P P Allotrope of Oxygen
O O O
47. Which of the following is the correct statement?
43. Which acid on heating produces phosphine? (a) O3 molecule is bent.
(a) Phosphoric acid (b) Ozone is violet-black in solid state.
(b) Phosphorous acid (c) Ozone is diamagnetic gas.
(c) Peroxymonophosphoric acid (d) ONCl and ONO− are not isoelectronic.
(d) Metaphosphoric acid (JEE Main 2013)
Solution Solution

(b) Phosphorous acid on heating produces phosphine. (a), (b), (c), (d)
O
3H3PO2 

→ PH3 + 2H3PO3 Option (a): Correct, as O O is bent.
Option (b): Correct, as ozone is violet-black in solid state.
44. Which of the following statements is not correct regarding Option (c): Correct, as ozone is a diamagnetic gas.
PH3 molecule? Option (d): Correct, as ONCl contains 32 electrons and
(a) It is extremely toxic and colorless gas ONO− contains 24 electrons. Hence they are not
(b) It has rotten fish smell isoelectronic.
(c) It is formed by hydrolysis of Na3P
(d) PH3 is highly soluble in water 48. Which of the following is the hybridized state of oxygen atom
in ozone molecule?
Solution (a) sp (b) sp2 (c) sp3 (d) sp3d
(d)
Solution
Option (a): is correct (information based)
Option (b): is correct (information based) (b) Ozone can be represented in terms of the following reso-
Option (c): is correct (Concept based) nance structures.

Na3P + 3 H2O → PH3 + 3 NaOH O O


O O O O
Option (d): is incorrect because
The structure is described as the central O atom using
PH3 + H2O → PH4+ + OH−
sp2 hybrid orbitals to bond to the terminal O atoms.
The donating ability of lone pair of P-atom in PH3 is much The central O atom has one lone pair and the terminal
less because it is present in the almost pure ‘s’ orbital O atoms have two lone pairs. The sigma bonds and lone
according to Drago’s rule. pairs together account for 14 out of 18 electrons avail-
able in the valence shell (six electrons contributed by
45. Which of the following is tetrabasic acid?
each O atom). The remaining four electrons are involved
(a) Metaphosphoric acid (b) Orthophosphoric acid
in p-bonding using 2pz orbitals on each of the three O
(c) Hypophosphoric acid (d) Hypophosphorous acid
atoms.
Solution
(c) Hypophosphoric acid has four acidic hydrogens. 49. Which of the following gases causes tailing of mercury?
(a) Ozone (b) Nitrogen
General Properties of Group 16 Elements (c) Sulphur dioxide (d) Hydrogen sulphide

46. Which of the following statements regarding sulphur is Solution


incorrect? (a) O3 is extremely powerful oxidizing agent.
(a) The vapour at 200°C consists mostly of S8 rings.
(b) At 600°C the gas mainly consists of S2 molecules. Compounds of Sulphur
(c) The oxidation state of sulphur is never less than (+4) in its
compounds. 50. The number of S S bonds in sulphur trioxide trimer, (S3O9) is
(d) S2 molecule is paramagnetic. (a) 3 (b) 2 (c) 1 (d) 0
(AIEEE 2011)

Chapter-11.indd 354 7/30/2016 12:58:54 PM


Solved Examples 355

Solution 56. Which of the following is formed when HgO is treated with
(d) The structure of S3O9 is chlorine?
(a) ClO2 (b) Cl2O (c) Cl2O6 (d) Cl2O7
O O
S Solution
O O
O O (b) HgO + Cl2 → HgCl2∙ HgO + Cl2O
S S
O O O 57. What is the correct order of oxidizing nature among halogens?
(a) F2 > Cl2 > Br2 > I2 (b) F2 < Cl2 < Br2 < I2
51. Which of the following species does not contain S S linkage?
(c) Cl2 > Br2 > F2 > I2 (d) I2 < Br < F2 < Cl2
(a) S2O22− (b) S2 O32− (c) S2 O52− (d) SO32−
Solution
Solution
(a) The electrode potential value for F2 is the maximum and
(d)
that for I2 is the minimum.
O O
S O 58. Which of the following is formed when ICl3 is treated with
− −
S O O S S O water?
S S S (a) HIO (b) HIO2 (c) HIO3 (d) HIO4
− O−
O
− O
− O
− O
− O O
Solution
52. Hydrolysis of one mole of peroxydisulphuric acid produces (c) ICl3 + H2O → ICl + HIO3 + HCl
(a) two moles of sulphuric acid.
(b) two moles of peroxymonosulphuric acid. Oxoacids of Halogens
(c) one mole of sulphuric acid and one mole of peroxymono- 59. Among the following oxoacids, the correct decreasing order
sulphuric acid. of acid strength is
(d) one mole of sulphuric acid, one mole of peroxymonosul- (a) HOCl > HClO2 > HClO3 > HClO4
phuric acid and one mole of hydrogen peroxide. (b) HClO4 > HOCl > HClO2 > HClO3
Solution (c) HClO4 > HClO3 > HClO2 > HOCl
(d) HClO2 > HClO4 > HClO3 > HOCl
(a) Hydrolysis of peroxydisulphuric acid yields H2SO4
(JEE Main 2014)
O O O Solution
HO S O O S OH 2 HO S OH (c) Let us consider the stabilities of the conjugate bases,
ClO−, ClO2−, ClO3− and ClO4− formed from the acids, HClO,
O O O HClO2, HClO3 and HClO4, respectively. These anions are
stabilized by the delocalization of the charge over oxy-
53. The number of S S bonds in sulphur trioxide trimer S3O9 is gen atoms. More oxygen atom causes more delocaliza-
(a) three. (b) two. (c) one. (d) zero. tion of charge on conjugate base. So, the stability order
of respective conjugate base is:
Solution
(d) No S S bond exists in S3O9. ClO − < ClO2− < ClO3− < ClO 4−

54. Which of the following allotropic forms of sulphur is thermo- Now from conjugate acid base pair definition, more
dynamically most stable? stable anion implies weaker is the conjugate base and
(a) Orthorhombic (b) b -monoclinic stronger is the respective acid. Thus, the acidic strength
(c) g -monoclinic (d) Plastic sulphur decreases in the order

Solution HClO4 > HClO3 > HClO2 > HOCl

(a) Orthorhombic sulphur has very less Δf H so it is very


General Properties of Group 18 Elements
stable.
60. Which one has the highest boiling point?
Chemical Properties of Group 17 Elements (a) Ne (b) Kr (c) Xe (d) He
55. Identify the incorrect statement among the following: (JEE Main 2015)
(a) Br2 reacts with hot and strong NaOH solution to give Solution
NaBr, NaBrO4 and H2O. (c) London dispersion forces increase from He to Xe because
(b) Ozone reacts with SO2 to give SO3. the molecular mass increases, so the boiling point also
(c) Silicon reacts with NaOH(aq) in the presence of air to give increases from He to Xe.
Na2SiO3 and H2O. Compounds of Xenon
(d) Cl2 reacts with excess of NH3 to give N2 and HCl.
(AIEEE 2007) 61. Which one of the following reactions of xenon compounds is
Solution not feasible?
(a) Br2 reacts with hot and strong NaOH solution to give (a) 2XeF2 + 2H2O → 2Xe + 4HF + O2
NaBr, NaBrO3 and H2O. (b) XeF6 + RbF → Rb[XeF7]

Chapter-11.indd 355 7/30/2016 12:58:55 PM


356 Chapter 11 The p-Block Elements

(c) XeO3 + 6HF → XeF6 + 3H2O Column I Column II


(d) 3XeF4 + 6H2O → 2Xe + XeO3 + 12HF + 1.5O2
(AIEEE 2009) (A) XeOF4 (p) Distorted octahedron
Solution (B) XeF6 (q) Square pyramidal
(c) XeO3 + 6HF → XeF6 + 3H2O (C) XeO2F2 (r) Octahedral
The reaction is not feasible because XeO3 is a powerful
(D) [XeO6]4− (s) Trigonal bipyramid
oxidizing agent.
62. Which of the following xenon compounds may not be (a) (A) → (p); (B) → (q); (C) → (r); (D) → (s)
obtained by hydrolysis of xenon fluorides? (b) (A) → (q); (B) → (p); (C) → (s); (D) → (r)
(a) XeO2F2 (b) XeOF4 (c) XeO3 (d) XeO4 (c) (A) → (q); (B) → (s); (C) → (p); (D) → (r)
(JEE Main Online 2014) (d) (A) → (s); (B) → (r); (C) → (q); (D) → (p)
Solution Solution
(b) Xenon reacts directly only with fluorine. However, oxy-
(d) 3XeF4 + 6H2O → 2Xe + XeO3 + 12HF + 1 O2 gen compounds can be obtained from fluorides. The
2
structures of the given compounds are as follows:
[XeO6]4− + H2SO4 → XeO4
XeOF4 Square pyramidal (that is octahedral
Concentrated H2SO4 react with per-xenate to give XeO4. with one position unoccupied)
63. Which of the following is planar? XeF6 Distorted octahedron
(a) XeF4 (b) XeO4 (c) XeO2F2 (d) XeOF4
XeO2F2 Trigonal pyramidal (with one position
Solution unoccupied)
(a) XeF4 has 2 lone pairs and 4 bond pairs, thus the shape is [XeO6]4− Octahedral
based on octahedral geometry. To minimize the repulsive
forces, the lone pairs occupy the axial position, and thus 65. In XeO3 and XeF6 the oxidation state of Xe is
the shape of the ion is square planar. (a) +4 (b) +6 (c) +1 (d) +3
Solution
F F (b) The oxidation state of Xe is +6 in XeO3 and XeF6.
Xe 66. Which one of the following is the correct pair with respect
to molecular formula of xenon compound and hybridization
F F state of xenon in it?
(a) XeF4, sp3 (b) XeF2, sp (c) XeF2, sp3d (d) XeF4, sp2
64. In the following table, the first column lists some compounds
of xenon with oxygen and fluorine and the second column Solution
lists the shapes. Then which of the following options is (c) The hybridization in XeF2 is sp3d with three lone pairs
correct: (linear shape).

ADVANCED LEVEL PROBLEMS


1. Match the following: 2. The percentage of p-character in the orbitals forming P P
Column I Column II bonds in P4 is
(a) 25 (b) 33 (c) 50 (d) 75
(a) Bi3+ → (BiO)+ (p) Heat (IIT-JEE 2007)
(b) [AlO2]− → Al(OH)3 (q) Hydrolysis Solution
(c) SiO 44 − → Si2O76 − (r) Acidification (d) Because the hybridization in P4 is sp3 which involves 75%
p-character and 25% s-character.
(d) (B4 O72 − ) → [B(OH)3 ] (s) Dilution by water
(IIT-JEE 2006) P
Solution
(a) → q; (b) → r; (c) → p; (d) → q, r
(a) BiCl3 + H2O →  BiOCl + 2HCl P P
(b) NaAlO2 + H2O →  Al(OH)3 + NaOH P
(c) 4−
SiO4 (orthosilicates) → 2−
 (heat) Si2O7 (pyrosilicates)
(d) Na2B4O7 + 2HCl + 5H2O →  4H3BO3 + 2NaCl (hydrolysis 3. Statement 1: In water, orthoboric acid behaves as a weak
and acidification) monobasic acid.

Chapter-11.indd 356 7/30/2016 12:58:56 PM


Advanced Level Problems 357

Statement 2: In water, orthoboric acid acts as a proton


donor.
(a) Statement 1 is True, Statement 2 is True; Statement 2 is a
Xe
correct explanation for Statement 1
(b) Statement 1 is True, Statement 2 is True; Statement 2 is O O
O
NOT a correct explanation for Statement 1
(c) Statement 1 is True, Statement 2 is False 5c. XeF4 and XeF6 are expected to be
(d) Statement 1 is False, Statement 2 is True (a) oxidizing. (b) reducing.
(c) unreactive. (d) strongly basic.
Solution
Solution
(c) The correct reason is that in water orthoboric acid
(a) Both are oxidizing as decrease in oxidation number of
behaves as a weak monobasic Lewis acid.
xenon is taking place in their reactions:
B(OH)3 + 2HOH → [B(OH)4]− + H3O+ 6XeF4 + 12H2O → 4Xe + 2XeO3 + 24HF + 3O2
XeF6 + 3H2O → XeO3 + 6HF
4. Statement 1: Boron always forms covalent bond.
Statement 2: The small size of B3+ favours formation of cova- 6. Statement 1: Pb4+ compounds are stronger oxidizing agents
lent bond. than Sn4+ compounds.
(a) Statement 1 is True, Statement 2 is True; Statement 2 is a Statement 2: The higher oxidation states for the Group 14
correct explanation for Statement 1. elements are more stable for the heavier members of the
(b) Statement 1 is True, Statement 2 is True; Statement 2 is group due to inert pair effect.
NOT a correct explanation for Statement 1. (a) Statement 1 and Statement 2 are true and the
(c) Statement 1 is True, Statement 2 is False. Statement 2 is the correct explanation of the
(d) Statement 1 is False, Statement 2 is True. Statement 1.
(IIT-JEE 2007) (b) Statement 1 and Statement 2 are true but the
Statement 2 is NOT a correct explanation of the
Solution
Statement 1.
(a) Due to small size and high charge on B3+, it is highly (c) Statement 1 is true but the Statement 2 is false.
polarizing and therefore, favours formation of covalent (d) Statement 1 is false, Statement 2 is true.
bond. (IIT-JEE 2008)
5. Paragraph for Questions 5a–5c: The noble gases have Solution
closed-shell electronic configuration and are monoatomic (c) The correct reason is that the lower oxidation states for
gases under normal conditions. The low boiling points of the the Group 14 elements are more stable for heavier mem-
lighter noble gases are due to weak dispersion forces between bers due to inert pair effect.
the atoms and the absence of other interatomic interactions.
7. Paragraph for Questions 7a–7c: There are some deposits
The direct reaction of xenon with fluorine leads to a series
of nitrates and phosphates in earth’s crust. Nitrates are more
of compounds with oxidation numbers +2, +4 and +6. XeF4
soluble in water. Nitrates are difficult to reduce under the
reacts violently with water to give XeO3. The compounds of
laboratory conditions but microbes do it easily. Ammonia
xenon exhibit rich stereochemistry and their geometries can
forms larger number of complexes with transition metal ions.
be deduced considering the total number of electron pairs in
Hybridization easily explains the ease of sigma donation
the valence shell.
capability of NH3 and PH3. Phosphine is a flammable gas and
(IIT-JEE 2007)
is prepared from white phosphorus.
(IIT-JEE 2008)
5a. Argon is used in arc welding because of its
(a) low relativity with metal. 7a. Among the following, the correct statements is
(b) ability to lower the melting point of metal. (a) Phosphates have no biological significance in humans.
(c) flammability. (b) Between nitrates and phosphates, phosphates are less
(d) high temperature calorific value. abundant in earth’s crust.
(c) Between nitrates and phosphates, nitrates are less abun-
Solution
dant in earth’s crust.
(a) Argon is mainly used to provide an inert atmosphere at (d) Oxidation of nitrates is possible in soil.
high temperature metallurgical extraction, that is, in arc
welding of metals or alloys. Solution
(c) This is because nitrates are more soluble and prone to
5b. The structure of XeO3 is microbial action.
(a) linear. (b) planar.
(c) pyramidal. (d) T-shaped. 7b. Among the following, the correct statement is
(a) Between NH3 and PH3, NH3 is a better electron donor
Solution because the lone pair of electrons occupies spherical
(c) The structure is pyramidal with sp3 hybridization. s-orbital and is less directional.

Chapter-11.indd 357 7/30/2016 12:58:57 PM


358 Chapter 11 The p-Block Elements

(b) Between NH3 and PH3, PH3 is a better electron donor 11. The value of n in the molecular formula BenAl2Si6O18 is
because the lone pair of electrons occupies sp3 orbital (IIT-JEE 2010)
and is more directional. Solution
(c) Between NH3 and PH3, NH3 is a better electron donor (c) Be3Al2Si6O18 (Beryl)
because the lone pair of electrons occupies sp3 orbital According to charge balance in a molecule
and is more directional.
2n + 2(+3) + 6(+4) − 18 (2) = 0
(d) Between NH3 and PH3, PH3 is a better electron donor
n=3
because the lone pair of electrons occupies spherical
s-orbital and is less directional. 12. The total number of diprotic acids among the following is
Solution ____.

(c) On moving from top to bottom in a group, the bond H3PO4 H2SO4 H3PO3 H2CO3 H2S2O7
angle decreases due to more p character in the bond
pair and subsequently more s character in the lone pair H3BO3 H3PO2 H2CrO4 H2SO3
orbital. (IIT-JEE 2010)
7c. White phosphorus on reaction with NaOH gives PH3 as one of Solution
the products. This is a (6) H2SO4, H3PO3, H2CO3, H2S2O7, H2CrO4, H2SO3
(a) dimerization reaction. H3PO3 is a diprotic acid since one of the proton (hydro-
(b) disproportionation reaction. gen) is bound to the phosphorus atom.
(c) condensation reaction.
13. All the compounds listed Column I react with water. Match
(d) precipitation reaction.
the result of the respective reactions with the appropriate
Solution options listed in Column II.
(b) P4 + 3NaOH + 3H2O → 3NaH2PO2 + PH3
There is a change (increase as well as decrease) in the oxi- Column I Column II
dation state of phosphorus from (0) to (+1) and (–3). (a) (CH3)2SiCl2 (p) Hydrogen halide formation
8. Aqueous solution of Na2S2O3 on reaction with Cl2 gives (b) XeF4 (q) Redox reaction
(a) Na2S4O6 (b) NaHSO4 (c) NaCl (d) NaOH (c) Cl2 (r) Reacts with glass
(IIT-JEE 2008)
(d) VCl5 (s) Polymerization
Solution
(t) O2 formation
(b) Na2S2O3 + 4Cl2 + 5H2O → 2NaHSO4 + 8HCl
(IIT-JEE 2010)
9. The reaction of P4 with X leads selectively to P4O6. Then X is Solution
(a) Dry O2 (b) A mixture of O2 and N2 (a) → p, s; (b) → p, q, r, t; (c) → p, q, t; (d) → p
(c) Moist O2 (d) O2 in the presence of
aqueous NaOH (a) (CH3)2SiCl2 + 2H2O → (CH3)Si(OH)2 + 2HCl
(IIT-JEE 2009) (hydrogen halide formation)
Solution (CH3)Si(OH)2 can undergo polymerization to form
silicones.
(b) P4 + 3O2 → P4O6 3
N2 is used to retard further oxidation. (b) 3XeF4 + 6H2O → XeO3 + 2Xe + 12HF + O2
2
10. The nitrogen oxide(s) that contain(s) N N bond(s) is (are) (hydrogen halide formation, O2 formation, redox reaction)
(a) N2O (b) N2O3 (c) N2O4 (d) N2O5 (c) Cl2 + H2O → 2HCl + 21 O2
(IIT-JEE 2009) (hydrogen halide formation, O2 formation, redox reaction)
Solution (d) VCl5 + H2O → VOCl3 + 2HCl
(hydrogen halide formation)
(a), (b), (c)
14. Extra pure N2 can be obtained by heating
N2O N N O (a) NH3 with CuO (b) NH4NO3
O (c) (NH4)2CrO7 (d) Ba(N3)2
N2O3 N O N O N N (IIT-JEE 2011)
O O O Solution
N2O4 O O (d) It can be obtained by the thermal decomposition of bar-
N N ium nitride, that is,
O O Ba(N3)2 → Ba + 3N2(g)
O O 15. Among the following, the number of compounds that can
N2O5
N O N react with PCl5 to give POCl3 is ____.
O O O2, CO2, SO2, H2O, H2SO4, P4O10 (IIT-JEE 2011)

Chapter-11.indd 358 7/30/2016 12:58:58 PM


Advanced Level Problems 359

Solution Solution
(5) (b), (d) Diamond is hard and graphite is soft. Graphite is a
CO2 + PCl5 → POCl3 + COCl2 good conductor of electricity as it has one free electron
PCl5 + H2O → POCl3 + 2HCl which is responsible for the conduction.
PCl5 + SO2 → POCl3 + SOCl2 Diamond has higher thermal conductivity than graphite
because the structure of diamond is precise and thus the
PCl5 + H2SO 4 → POCl3 + HSO2Cl + HCl
Chlorosulphonic acid
transfer of heat is faster in it.
In case of graphite, the C C bond has a double bond
PCl5 + P4 O10 → 10POCl3 character so its bond order becomes higher than that of
diamond which has only single C C bonds.
16. Which ordering of compounds is according to the decreasing
order of the oxidation state of nitrogen? 20. The correct statement(s) about O3 is (are)
(a) HNO3, NO, NH4Cl, N2 (b) HNO3, NO, N2, NH4Cl (a) O O bond lengths are equal.
(c) HNO3, NH4Cl, NO, N2 (d) NO, HNO3, NH4Cl, N2 (b) Thermal decomposition of O3 is endothermic.
(IIT-JEE 2012) (c) O3 is diamagnetic in nature.
Solution (d) O3 has a bent structure.
(JEE Advanced 2013)
(b) In HNO3, the oxidation state of N is +5 while in NO, it is
Solution
+2. In N2, it is zero and in NH4Cl, the oxidation state of
nitrogen is −3. So, the decreasing order of oxidation state (a), (c), (d) As all electrons are paired, ozone is diamagnetic
of nitrogen is HNO3 > NO > N2 > NH4Cl. in nature. The structure is bent or V-shaped. The struc-
ture of ozone is resonance hybrid of the two structures
17. Which of the following hydrogen halides react(s) with with a delocalized p-orbital which covers all three atoms.
AgNO3(aq) to give a precipitate that dissolves in Na2S2O3(aq)? Because of this, the two O O bond lengths are equal.
(a) HCl (b) HF (c) HBr (d) HI
(IIT-JEE 2012) + +
Solution O O O
− −
(a), (c), (d) Only HF does not react with AgNO3. All others O O O O O O
react with AgNO3 to give precipitate of AgX which is sol-
21. The unbalanced chemical reactions given in Column I show
uble in Na2S2O3
missing reagent or condition (?) which are provided in
HX + AgNO3 → AgX ↓ + HNO3 (X = Cl, Br , I) Column II. Match Column I with Column II and select the cor-
rect answer using the code given below the lists.
AgX + Na2S2O3 → Na3 [Ag(S2O3 )2 ] + NaX
Soluble
Column I Column II

18. The shape of XeO2F2 molecule is ?


(P) PbO2 + H2SO 4 →
 PbSO4 + O2 (1) NO
(a) trigonal bipyramidal. (b) square planar.
+ Other products
(c) tetrahedral. (d) see-saw.
(IIT-JEE 2012) ?
Solution (Q) Na2S2O3 + H2O → NaHSO 4 (2) I2
(d) Actual shape is trigonal bipyramidal but due to the pres- + Other products
ence of lone pair it gets distorted to see-saw. The elec- ?
tronic configuration of Xe is 5s2 5p6; and that of Xe in (R) N2H4 →
 N2 + Other products (3) Warm
excited state is 5s25p55d1. The hybridization is sp3d and
? (4) Cl2
geometry is see-saw. (S) XeF2 →
 Xe + Other products

5s 5p 5d P Q R S
↑↓ ↑↓ ↑↓ ↑ ↑ (a) 4 2 3 1
F O (b) 3 2 1 4
Xe (c) 1 4 2 3
F O (d) 3 4 2 1
(JEE Advanced 2013)
19. With respect to graphite and diamond, which of the state- Solution
ment(s) given below is (are) correct?
(d) The reactions involved are
(a) Graphite is harder than diamond. warm
(b) Graphite has higher electrical conductivity than diamond. (P) PbO2 + H2SO 4  → PbSO4 + H2O + 21 O2
(c) Graphite has higher thermal conductivity than diamond. (Q) 2Na2S2O3 + 2H2O + Cl2 → 2NaCl + 2NaHSO4 + 2S
(d) Graphite has higher C C bond order than diamond. (R) N2H4 + 2I2 → N2 + 4HI
(IIT-JEE 2012) (S) XeF2 + 2NO → Xe + 2NOF

Chapter-11.indd 359 7/30/2016 12:59:01 PM


360 Chapter 11 The p-Block Elements

22. Paragraph for Questions 22a and 22b: The reactions of Cl2 Complete
gas with cold-dilute and hot-concentrated NaOH in water hydrolysis
XeF6 P + Other product
give sodium salts of two (different) oxoacids of chlorine, P
and Q, respectively. The Cl2 gas reacts with SO2 gas, in pres-
ence of charcoal, to give a product R. R reacts with white OH−/H2O

phosphorus to give a compound S. On hydrolysis, S gives an


Q
oxoacid of phosphorus, T.
(JEE Advanced 2013)
Slow disproportionation in OH−/H2O
22a. P and Q, respectively, are the sodium salts of
(a) hypochlorous and chloric acids. Products
(b) hypochlorus and chlorous acids.
(c) chloric and perchloric acids. (a) 0 (b) 1 (c) 2 (d) 3
(d) chloric and hypochlorous acids. (JEE Advanced 2014)
Solution
Solution
(c) The reaction involved is
(a) The reactions involved are
Complete
Cl2 + 2NaOH (cold, dil.) → NaOCl + NaCl + H2O Hydrolysis
(P) XeF6 XeO3 + H2F2
3Cl2 + 6NaOH (hot, conc.) → NaClO3 + 5NaCl + 3H2O (P)
(Q)
OH−/H2O
where NaOCl and NaClO3 are salts of hypochlorous acid
(HOCl) and chloric acid (HClO3), respectively. HXeO4− XeO6 + Xe(g) + H2O + O2(g)
(Q)
22b. R, S and T respectively, are
(a) SO2Cl2, PCl5 and H3PO4 (b) SO2Cl2, PCl3 and H3PO3 26. The number of lone pairs of electrons in N2O3 is __________.
(c) SOCl2, PCl3 and H3PO2 (d) SOCl2, PCl5 and H3PO4 (JEE Advanced 2015)
Solution Solution

(a) The reactions involved are (8) The structure of N2O3 is


Charocal
SO2 + Cl2  → SO2Cl2 O O
(R) N N
10SO2Cl2 + P4 → 4PCl5 +10SO2 O
(R) (S)
PCl5 + 4H2O → H3PO 4 + 5HCl Therefore, the total number of lone pairs is 8.
(S) (T) 27. Which of the following statements is correct?
23. The correct statement(s) for orthoboric acid is/are (a) H3PO3 is dibasic and reducing.
(a) It behaves as a weak acid in water due to self-ionization. (b) H3PO3 is tribasic and reducing.
(b) Acidity of its aqueous solution increases upon addition of (c) H3PO3 is tribasic and non-reducing
ethylene glycol. (d) H3PO3 is dibasic and non-reducing.
(c) It has a three dimensional structure due to hydrogen Solution
bonding. (a) There are only two OH groups in orthophosphorous
(d) It is a weak electrolyte in water. acid thus is dibasic. The oxidation number of P in this
(JEE Advanced 2014) acid is +3. Whereas P may have +5 oxidation state also.
Solution Therefore, H3PO3 can be oxidized which means H3PO3 is
(b), (d) The reactions involved are a reducing agent.
CH2OH CH2 O O CH2 H
[B(OH)4]− + B−
CH2OH H2 C O O CH2 HO P OH
H3BO3 + H2O  − +

 [B(OH)4] + H (weak acid)

O
24. The product formed in the reaction of SOCl2 with white phos-
phorus is 28. Boric acid is a
(a) PCl3 (b) SO2Cl2 (c) SCl2 (d) POCl3 (a) weak monobasic Lewis acid.
(JEE Advanced 2014) (b) weak monobasic Arrhenius acid only.
Solution (c) weak monobasic Bronsted acid only.
(d) weak monobasic Arrhenius as well as Bronsted acid.
(a) The reaction involved is
P4 + 8SOCl2 → 4PCl3 + 4SO2 + 2S2Cl2 Solution

25. Under ambient conditions, the total number of gases released as (a) The central boron atom in boric acid, H3BO3 is elec-
products in the final step of the reaction scheme shown below is tron deficient, so boric acid is a Lewis acid with one

Chapter-11.indd 360 7/30/2016 12:59:03 PM


Practice Exercise 361

p-orbital vacant. There is no d-orbital of suitable energy in Oxidation state of sulphur in compounds (A), (B), (C) and
boron atom. So, it can accommodate only one additional (D) are as follows:
electron pair in its outermost shell. NaHSO3 = (+1) + (+1) + x + 3(−2) = +4
Na2SO3 = 2(+1) + x + 3(−2) = +4
H2O + B(OH)3 OH+2 The oxidation state of S in Na2S2O3 is +6, and −2.
Base Acid B− S−2
HO OH −
O
OH S+6
O− O
[B(OH)4] − + H+
The oxidation state of S in Na2S4O6 is +5 and 0.
29. (CH3)2 SiCl2 on hydrolysis produces O O
(a) (CH3)2Si(OH)2 (b) (CH3)2Si O +5 0 0 +5
(c) [ O (CH3)2Si O ]n (d) (CH3)2SiCl(OH) O− S S S S O−

Solution O O
(c) It appears at the first sight that Me2SiCl2 on hydrolysis will 31. Which of the following isomers of phosphorus are thermo-
produce Me2Si(OH)2 which ultimately upon loss of water, dynamically least and most stable?
will form Me2Si O. But silicon atom, because of its very (a) White (least), Red (most) (b) Yellow (least), Red (most)
large size in comparison to oxygen, is unable to form (c) Red (least), White (most) (d) White (least), Black (most)
π-bond. Thus, the product of hydrolysis is polymeric in
nature. Solution
(d) Black phosphorous is most layered because it has lay-
SO2 Na2 CO3 Elemental I2 ered structure, whereas white is the least because it is
30. Na2CO3 → (A ) 
→ (B) Sulphur
 /∆
→ (C ) 
→ (D)
very reactive and catches fire in air (so, it is stored under
Identify the compounds (A), (B), (C), (D), and give oxidation water).
state of sulphur in each compound.
32. A pale blue liquid obtained by equimolar mixture of two
Solution
gases at −30°C is
Na2CO3 + 2SO2 + H2O → 2NaHSO3 + CO2 (a) nitric oxide (b) nitrogen trioxide
(A) (c) nitrogen tetroxide (d) nitrogen pentoxide
2NaHSO3 + Na2CO3 → 2Na2SO3 + H2O + CO2 Solution
(B )
(b) N2O3 can only be obtained at low temperatures. It can be
Na2SO3 + S → 2Na2S2O3
made by condensing equimolar of NO and NO2 together.
(C)
This gives a blue liquid or solid which is unstable and dis-
2Na2S2O3 + I2 → 2Na2S4 O6 + 2NaI sociates into NO and NO2 at −30°C.
(D )

NO + NO2 → N2O3

PRACTICE EXERCISE
Level I 5. Salicylic acid reacts with two equivalent of ICl vapour to
produce
Single Correct Choice Type (a) iodinated product. (b) chlorinated product.
1. Which of the following is not a peroxy acid? (c) mixture of both. (d) virtually no reaction.
(a) Caro’s acid (b) Marshall’s acid 6. Which of the following has the smallest bond angle?
(c) Thiosulphuric acid (d) Sulphurous acid (a) H2S (b) H2O (c) H2Se (d) H2Te
7. The most stable hydrate of noble gases is
2. Which of the following pairs of species are non-linear? (a) Kr ⋅ 4H2O (b) Kr ⋅ 6H2O
(a) OCN− and Br3− (b) (SCN)2 and I5− (c) Xe ⋅ 4H2O (d) Xe ⋅ 6H2O
2−
(c) NCN and N3 − (d) HN3 and (CN)2
8. Which of the following is an organo silicon polymer?
3. Xenon difluoride is (a) Silica (b) Orthosilicic acid
(a) linear. (b) angular. (c) Silicon carbide (d) Silicic acid
(c) trigonal. (d) pyramidal. 9. How many peroxy linkages are present in pyrophosphoric
acid?
4. The reagent used to distinguish between H2O2 and O3 is (a) 0 (b) 1 (c) 2 (d) 3
(a) PbS. (b) starch and iodine. 10. A + SbF5 → B
(c) KMnO4. (d) bleaching powder. B + tert-butane → [tert-butyl]+ + X− + H2

Chapter-11.indd 361 7/30/2016 12:59:06 PM


362 Chapter 11 The p-Block Elements

Then A is 24. Diborane cannot be obtained from


(a) HCl (b) HF (c) HBr (d) HI (a) Na2B4O7 + HCl (b) NaBH4 + I2
(c) BF3 + LiAlH4 (d) BF3 + NaH
11. Caro’s acid is
(a) H2SO5 (b) H2SO3 (c) H3S2O5 (d) H2S2O8 25. The compounds of S obtained by reaction of S and conc. hot
KOH when reacted separately with dil. HCl produce
12. When tin is treated with concentrated nitric acid, (a) different gases, SO2 and H2S.
(a) it is converted into stannous nitrate. (b) same gas SO2.
(b) it becomes passive. (c) S is obtained back.
(c) it converted into stannic nitrate. (d) same gas H2S.
(d) it is converted into metastannic acid.
26. Which of the following ions is not a pseudohalide ion?
13. The most basic oxide of elements in Group 14 of the periodic (a) CN− (b) SCN−
table is (c) SH− (d) CNO−
(a) SiO2 (b) GeO (c) SnO2 (d) PbO HNO I
27. White compound A 
3
→ B 2
→ C (used to estimate CO).
14. Which of the following options are true (T) and which are
The compound A is
false (F)?
(a) P2O5 (b) I2O5 (c) I4O9 (d) S3O9
(i) Ionic mobility is the highest for I− in water as compared
to other halides. 28. In which of the following reactions is PbSO4 formed?
(ii) Stability order is: Cl3− > Br3− > I3− (a) PbO2 + SO2 (b) PbS + O3
(iii) Reactivity order is: F < Cl < Br < I. (c) PbS + H2O2 (d) All of the above.
(iv) Oxidizing power order is: F2 < Cl2 < Br2 < I2.
29. Which of the following is a pseudohalogen?
(a) TFTF (b) TFFF (c) TFFT (d) FTFT (a) IF7 (b) (CN)2 (c) ICl2 (d) I3−
15. Which of the following is most stable M F bond, where M is 30. Which of the following pairs of substances are structurally not
Group 14 element? similar?
(a) PbF4 (b) SnF4 (c) CF4 (d) SiF4 (a) Diamond and silicon carbide
(b) Boron nitride and graphite
16. XeF4 exists as _____ under ordinary atmospheric
(c) Borazine and benzene
conditions.
(d) Diborane and hydrazine
(a) solid (b) liquid
(c) gas (d) semi-solid 31. Which of the following pseudohalides does not form dimer
like halogen X2?
17. SO2 is passed through strongly acidic solution of SnCl2. The
(a) CN− (b) SCN− (c) SeCN− (d) OCN−
precipitate obtained is consisting of S and oxidation state of S
in the precipitate is 32. The element which forms oxides in all the oxidation states
(a) −1 from +1 to +5 is
(b) zero (a) N (b) P (c) As (d) Sb
(c) −2
33. Which of the following is the electron-deficient molecule?
(d) precipitate does not contain S.
(a) PH3 (b) C2H6 (c) SiH4 (d) B2H6
18. The most abundant and common oxidation state of 34. Which of the following oxides has three-dimensional
sulphur is structure?
(a) −2 (b) +4 (c) +2 (d) +6 (a) CO (b) CO2 (c) SiO2 (d) SO2
19. Which of the following series of fluorides is known? 35. Which of the following pseudohalides best resembles Cl−, Br−
(a) XeF2, XeF4, XeF3 (b) XeF2, XeF4, XeF6 and I−
(c) XeF2, XeF3, XeF6 (d) XeF2, XeF4, XeF5 (a) CN− (b) N3−
(c) ONC − (d) Both N3− and ONC−
20. Boron nitride is isoelectronic with
(a) C2 (b) B2 (c) N2 (d) O2 36. Lead pencil contains
(a) PbS. (b) FeS. (c) graphite. (d) Pb.
21. PbCl2 is more ionic than PbO2 because
(a) the radius of Pb2+ is more than that of Pb4+ 37. Which of the halides of carbon is the least stable?
(b) of inert pair effect (a) Iodide (b) Bromide
(c) chlorine is more electronegative than oxygen (c) Chloride (d) Fluoride
(d) chlorine atom is smaller than oxygen atom.
38. Which of the following has the properties of metals as well as
22. Electron affinity of noble gas element is non-metals?
(a) very high. (b) high. (c) low. (d) zero. (a) C (b) Pb (c) Sn (d) Ge
23. Bond angle in XeO3 is 39. Which of the following polyhalides is not known to exist?
(a) 107° (b) 103° (c) 119° (d) 92° (a) Br3− (b) Cl3− (c) F3− (d) I3−

Chapter-11.indd 362 7/30/2016 12:59:07 PM


Practice Exercise 363

40. Which of the following statements about H3BO3 is not correct? 51. Which of the following is thermally the most stable?
(a) It is prepared by acidifying an aqueous solution of borax. (a) H2S (b) H2O (c) H2Se (d) H2Te
(b) It has a layered structure in which planar B(OH)3 units are
52. Ozone is
joined by hydrogen bonds.
(a) an unstable, dark blue, diamagnetic gas.
(c) It does not act as proton donor, but acts as Lewis acid by
(b) an unstable, dark blue, paramagnetic gas.
accepting hydroxyl ions.
(c) a stable, dark blue, paramagnetic gas.
(d) It is a strong tribasic acid.
(d) found in the upper atmosphere where it absorbs UV
41. In which of the following elements, +1 oxidation state is more radiation.
stable than +3? Level II
(a) B (b) Al (c) Ga (d) Tl
Multiple Correct Choice Type
42. Which gas is evolved when urea is treated with NaOH?
(a) Ammonia (b) Nitrogen 53. Boric acid
(c) Laughing gas (d) NO (a) exists in polymeric form due to intermolecular hydrogen
bonding.
43. Which one of the following is correct statement? (b) is used in the manufacture of optical glasses.
(a) The hydroxide of aluminium is more acidic than that of (c) is a tribasic acid.
boron. (d) with borax, it is used in the preparation of a buffer
(b) The hydroxide of boron is basic, while that of aluminium solution.
is amphoteric.
(c) The hydroxide of boron is acidic, while that of aluminium 54. The oxidation states exhibited by oxygen in its compounds
is amphoteric. are
(d) The hydroxides of boron and aluminium are amphoteric. (a) −2 (b) −1 (c) +1 (d) +2
44. O3 reacts with KOH solution to produce 55. Which of the following statements are correct regarding
(a) O2 and KO2. (b) O3− and K2O. B2H6?
(c) O2 only. (d) O2 and KO3. (a) Banana bonds are longer but stronger than normal B H
45. Which of the following is the correct regarding B2H6? bonds.
(a) There is direct boron–boron bond. (b) B2H6 is also known as 3c–2e compound.
(b) The boron atoms are linked through hydrogen bridges. (c) The hybridization of B in B2H6 is sp3 while that of sp2 in
(c) The structure is similar to C2H6. BH3.
(d) All the atoms are in one plane. (d) It cannot be prepared by reacting BF3 with LiBH3 in the
presence of dry ether.
46. Silicon tetrafluoride reacts with HF to form
(a) SiF2 (b) SiF64− (c) SiF62− (d) H2[SiF4] 56. Which of the following statements are incorrect?
(a) Solid PCl5 exists as tetrahedral [PCl4]+ and octahedral
47. Which of the following interhalogen compound is used as [PCl6]− ions.
Wij's reagent? (b) Oxides of phosphorus, P2O3 and P2O5 exist as monomers.
(a) ClF3 (b) ICI (c) CIF (d) IBr (c) Solid PCl5 exists as [PCl4]+ Cl−.
48. Which of the following does not exist? (d) Solid N2O5 exists as NO+2 NO−3.
(a) [CCl6]2− (b) [SiCl6]2− (c) [GeF6]2− (d) [SnCl6]2−
57. In the reaction Al2(SO4)3 + NH4OH → X
49. Which of the following lead oxides is used as vermillion (a) X is a white-coloured compound.
(sindur)? (b) X is insoluble in excess of NH4OH.
(a) PbO (b) Pb2O3 (c) PbO2 (d) Pb3O4 (c) X is soluble in NaOH.
(d) X cannot be used as an antacid.
50. Hydrolysis of dimethyldichlorosilane, (CH3)2SiCl2, followed by
condensation polymerization yields straight-chain polymer of 58. White phosphorus has
O O (a) P P P bond angle is 60°.
(a) (b) four P P single bonds.
O Si Si O
(c) polymeric structure.
O O (d) six P P single bonds.
(b) Si O O Si 59. When an inorganic compound (X) having 3c–2e as well as
CH3 CH3 2c–2e bonds reacts with ammonia gas at a certain temper-
ature, gives a compound (Y) isostructural with benzene.
(c) O Si O Si O Compound (X) with ammonia at a high temperature, pro-
CH3 CH3 duces a hard substance (Z). Then
(a) (X) is B2H6.
CH3 CH3
(d) (b) (Z) is known as inorganic graphite.
O Si Si O (c) (Z) having structure similar to graphite.
(d) (Z) having structure similar to (X).
CH3 CH3

Chapter-11.indd 363 7/30/2016 12:59:09 PM


364 Chapter 11 The p-Block Elements

60. Which of the following reactions are feasible? The mineral also contains Ca2+ ions, Cu2+ ions and water mol-
(a) Cl2 + 2Br− → 2Cl− + Br2 (b) Br2 + 2F− → 2Br− + F2 ecules in a 1:1:1 ratio mineral is represented as
− −
(c) I2 + 2Br → 2l + Br2 (d) F2 + 2Br− → 2F− + Br2 (a) CaCuSi3O10 ⋅ H2O
(b) CaCuSi3O10 ⋅ 2H2O
61. Which of the following statements is not correct?
(c) Ca2Cu2Si3O10 ⋅ 2H2O
(a) Aluminium is among the best conductors of electricity.
(d) CaCuSi3O10 ⋅ 3H2O
(b) Physical properties of aluminium are characteristic of a
true metal. Paragraph for Questions 68 to 70: The elements of Group 18
(c) Aluminium does not dissolve in aqueous sodium hydrox- have been called “the inert gases” or “the noble gases”. The name
ide solution. “noble gases” implies that they tend to be unreactive. Helium has
(d) Aluminium reacts vigorously with hot conc. HNO3, oxides two electrons which form a complete shell 1s2. The other noble
of nitrogen are evolved. gases have a closed octet of electrons in their outer shell ns2 np6.
62. Which of the following are peroxoacids of sulphur? This electronic configuration is very stable and is related to their
(a) H2S2O8 (b) H2SO3 (c) H2SO5 (d) H2S2O3 chemical inactivity. The low boiling points of the lighter noble
gases are due to weak dispersion forces between the atoms and
63. On the basis of structure of graphite, which of the following is the absence of other interatomic interactions. Xe reacts directly
(are) correct? only with F2 to give a series of compounds with oxidation states
(a) It is a diamagnetic substance. (+2), (+4) and (+6). Xenon–oxygen compounds can be obtained
(b) It behaves like metal as well as semiconductor. from fluorides. XeF4 reacts violently with water to give XeO3. The
(c) C C bond length in it is more than the diamond. compounds of xenon exhibit varied geometries that can be pre-
(d) It is thermodynamically more stable as well as more dicted on the basis of the total number of electron pairs in the
dense than diamond. valence shell.
64. The noble gases which are lighter than air are 68. The structure of XeOF4 is
(a) Ar. (b) He. (c) Ne. (d) Kr. (a) linear. (b) planar.
(c) pyramidal. (d) T-shaped.
Passage Type
69. XeF2 and XeF4 are
Paragraph for Questions 65 to 67: Silica covers an entire group of (a) oxidizing. (b) reducing.
minerals, which have the general formula SiO2, the most common (c) unreactive. (d) strongly basic.
of which is quartz, which is a framework silicate with SiO4 tetrahe-
dra arranged in spirals. The spirals can turn in a clockwise or anti- 70. Argon is used in metallurgical processes because of its
clockwise direction resulting in two mirror images, optically active, (a) low reactivity with metal.
varieties of quartz. (b) ability to lower the melting point of metal.
(c) flammability.
65. The following structures represent various silicate anions. (d) high calorific value.
Their formulas are respectively
Matrix-Match Type
Silicon
Oxygen 71. Match the characteristics with the compound.
Column I Column II
(a) Lower oxidation state (+1) is (p) InCl
more stable
(a) SiO23 − and Si3O72 − (b) SiO 44 − and Si3O10
8−
(b) Covalent when anhydrous (q) Tl

(c) SiO24− and Si3O26 − (d) SiO34 − and Si3O78 − (c) Shows disproportionation (r) Al
reaction
(d) Undergoes thermite reaction (s) GaCl3
66. Si3O2−
9 (having three tetrahedral) is represented as
72. Match the reactions with the characteristic of products.

(a) (b) Column I Column II


∆ (p) Gives highly explosive solid
(a) H3PO3 

− − (b) XeF6 + 6H2O → (q) One of the products is a
tribasic non-reducing acid
(c) − − (d) Both (a) and (b)
(c) NO2 + H2O → (r) Dehydration
− −
∆ (s) In one of the products, cen-
(d) HNO3 + P4 O10 

67. The silicate anion in the mineral kinoite is a chain of three tral atom has (+5) oxidation
SiO44− tetrahedra that share corners with adjacent tetrahedra. state

Chapter-11.indd 364 7/30/2016 12:59:11 PM


Hints and Explanations 365

73. Match the compound with its characteristics. 75. The most common oxidation state of sulphur is ___.
Column I Column II 76. The number of isomers possible for distributed borazine,
(a) Inorganic benzene (p) An allotrope of carbon B3N3H4X2 is ______.
(b) Graphite (q) Orthosilicate 77. Amongst the following, the number of interhalogen mole-
(c) Phenacite (r) An ore of tin cules/ions is___.
(d) Cassiterite (s) Borazine (B3N3H6) XeO3, BrO3−, ClF, XeF2, OF2, XeF4, ICI4−, ClO−, IBr2−
78. Total number of elements of Groups 13 and 14 which form
Integer Type
basic oxides is ______.
74. The number of electrons in sp3 hybridized atom in one mole-
cule of borax is ______.

ANSWER KEY
Level I
1. (c) 2. (b) 3. (a) 4. (c) 5. (b) 6. (d)
7. (d) 8. (b) 9. (a) 10. (b) 11. (a) 12. (d)
13. (d) 14. (b) 15. (d) 16. (a) 17. (c) 18. (d)
19. (b) 20. (a) 21. (a) 22. (d) 23. (b) 24. (a)
25. (a) 26. (c) 27. (a) 28. (d) 29. (b) 30. (d)
31. (d) 32. (a) 33. (d) 34. (c) 35. (a) 36. (c)
37. (a) 38. (d) 39. (c) 40. (d) 41. (d) 42. (a)
43. (c) 44. (d) 45. (b) 46. (c) 47. (b) 48. (a)
49. (d) 50. (c) 51. (b) 52. (a)

Level II
53. (a), (b), (d) 54. (a), (b), (c), (d) 55. (a), (b), (c) 56. (b), (c) 57. (a), (b), (c) 58. (a), (d)
59. (a), (b), (c) 60. (a), (d) 61. (c), (d) 62. (a), (c) 63. (a), (b) 64. (b), (c)
65. (a) 66. (a) 67. (c) 68. (c) 69. (a) 70. (a)
71. (a) → q; (b) → s; (c) → p; (d) → r 72. (a) → q, s; (b) → p; (c) → s; (d) → r, s
73. (a) → s; (b) → p; (c) → q; (d) → r 74. (2) 75. (6) 76. (4) 77. (3)
78. (2)

HINTS AND EXPLANATIONS


Level I 5. (b) When ICI reacts with organic compounds it often iodi-
nates them, though chlorination may occur depending
Single Correct Choice Type on the conditions.
1. (c) Thiosulphuric acid, that is, H2S2O3 does not have peroxide +ICl
Vapour
Chlorination
linkage O O. Salicyclic acid
2. (b) +ICl in
Iodination
nitrobenzene

N I
S 6. (d) H2Te has the smallest bond angle as Te is very less electron-
C and
C S I egative therefore, there is very less electron pair repulsion.
N I I I 7. (d) Xe ⋅6H2O. Noble gas hydrates are clathrate compounds
which are solid crystalline addition compounds having
(SCN)2 I5− the formula 6H2O:1 gas atom. Due to its large size, Xe is
3. (a) XeF2 is linear. trapped easily and thus has the most stable hydrate.
8. (b) Conceptual
F
9. (a) 0 as the structure below shows
Xe O O
O
F P P
HO OH
4. (c) Ozone does not reduce acidified solution of KMnO4. OH OH

Chapter-11.indd 365 7/30/2016 12:59:12 PM


366 Chapter 11 The p-Block Elements

10. (b) HF reacts with SbF5 to produce super acid H+[SbF6]– and
H H H H H
this super acid reacts with tertiary H-atom of a hydrocar-
B B N N
bon producing H2.
H H H H H
11. (a) H2SO5 is known as Caro’s acid. Diborane Hydrazine

12. (d) Sn + 4HNO3 → H2SnO3 + 4NO2 + H2O 31. (d) Some of the pseudohalide ions combine to form dimers
13. (d) Conceptual comparable with the halogen molecules X2. These
include cyanogen (CN)2, thiocyanogen (SCN)2 and sele-
14. (b) Correct Ionic mobility order: F−(aq) < Cl−(aq) < Br−(aq) nocyanogen (SeCN)2.
< I−(aq).
Correct stability order: Cl3− < Br3− < I3−. 32. (a) N has the oxidation states from +1 to +5 in the following
Correct reactivity order: F > Cl > Br > I. compounds: N2O, NO, NO2, N2O3, N2O3, N2O5.
Correct oxidizing powder order: F2 > Cl2 > Br2 > I2. 33. (d) Due to electron-deficient nature of BH3, it forms three
15. (d) Conceptual electron two bonds and dimerizes to B2H6. For normal
covalent bond, 16 electrons are required, but in B2H6
16. (a) XeF4 is solid under ordinary conditions. (diborane) there are only 12 valence electrons: three from
17. (c) SO2 + SnCl2 + HCl → SnS2 + SnCl4 + H2O each boron atom and six from the hydrogen atoms. In
this way, it is an electron-deficient compound.
18. (d) +6. For example, H2SO4, SO3, etc.
34. (c) The structures are as follows:
19. (b) XeF2 (+2), XeF4 (+4), XeF6 (+6) as these oxidation states of
Xe are known. Si O Si O Si
20. (a) Boron nitride contains 12 electrons and C2 also contains O O O
12 electrons.
Si O Si O Si
21. (a) Conceptual
O O O
S
22. (d) This is because they have completely filled orbitals.
C OO C O O O Si O Si O Si
23. (b) Xenon trioxide XeO3 has pyramidal structure with bond
angle = 103°. 35. (a) The best known pseudohalide is CN− which resembles
Cl−, Br− and I− in various respects. It forms dimers, dis-
proportionates in alkaline medium similar to halogens,
whereas N3− and ONC− do not show these characteristics.
Xe
O O 36. (c) Lead pencil contains graphite.
O
24. (a) 37. (a) The simple halides of carbon are CF4, CCl4, CBr4 and CI4.
(a) Na2B4 O7 + 2HCl + 5H2O → 2NaCl + 4H3BO3 All these halides are known to exist. However, the stability
Boric acid of these tetrahalides decreases as the size of the halogen
(b) 2NaBH4 + I2 → B2H6 + 2NaI + H2 atom increases, that is, CF4 > CCl4 > CBr4 > CI4. This is due
(c) 4BF3 + LiAlH4 → 2B2H6 + 3AlF3 + 3LiF to the reason that the bond energies of the carbon–halo-
(d) 2BF3 + 6NaH → B2H6 + 6NaF gen bonds decreases in order: C F > C Cl > C Br > C I.
25. (a) S + KOH → K2S + K2S2O3 + H2O Hence, iodide is least stable carbon halide.
S2− + H+ → H2S 38. (d) Germanium acts as metalloid.
S2O23 − + H+ → H2O + SO2 + S ↓
39. (c) The Br3− ion is much less stable and less common than I3−.
26. (c) Pseudohalide ions must contain N-atom (at least one) A few unstable Cl3− compounds are known, and the ion is
having one negative charge (in general). As SH− does not formed in concentrated solution. No F3− compounds are
contain N-atom so it is not a pseudohalide ion. known to exist.
27. (a) P2O5 + 2HNO3 → 2HPO3 + N2O5 40. (d) Boric acid (H3BO3) is soluble in water, and behaves as a
weak monobasic acid. It does not donate protons like
5N2 O5 + I2 → I2O5 + 10NO2
most acids, but rather accepts OH− ions. It is, therefore, a
Lewis acid and is better written as B(OH)3.
28. (d) Conceptual
B(OH)3 + 2H2O  H3O + + [B(OH)4 ]−
29. (b) A pseudohalogen is any inorganic compound structurally
similar to halogens, but containing atleast one Hence, it does not behave as strong tribasic acid.
non-halogen radical such as cyanide. 41. (d) Tl+ shows +1 ionic state due to inert pair effect. The outer
30. (d) The structures are as follows: shell s electrons (ns2) penetrate to (n − 1)d electrons and
thus become closer to nucleus and are more effectively

Chapter-11.indd 366 7/30/2016 12:59:14 PM


Hints and Explanations 367

pulled towards the nucleus. This results in less availabil- Level II


ity of ns2 electrons pair for bonding or ns2 electron pair
becomes inert. The inert pair effect begins after n ≥ 4 and
Multiple Correct Choice Type
increases with increasing value of n. The tendency to form 53. (a), (b), (d)
M+ ion increases down the group from Ga to Tl. Boric acid exists in polymeric form due to intermolecular
hydrogen bonding. The structure is as follows:
42. (a) The reaction involved is
H
NH2CONH2 + 2NaOH → 2NH3 ↑ + Na2CO3. O
43. (c) The hydroxide of boron is acidic, while that of aluminium
H B
is amphoteric. O O
Boric acid is a hydroxide of boron, which is soluble in
water and behaves as a weak monobasic acid. It does not H H
donate proton like most acids. But rather it accepts OH− O O Hydrogen bonds
ions. It is, therefore, a Lewis acid. H B H H
O O
O
B(OH)3 + 2H2O  H3O + + [B(OH)4 ]− B H H B
H
O O O O
Hydroxide of aluminium [Al(OH)3] is amphoteric. It reacts H H
principally as a base, that is, it reacts with acids to form
As an antiseptic and eye wash under the name “boric lotion”
salts that contain the [Al(H2O)6]3+ ion. However, Al(OH)3
and it is also used in manufacturing of optical glasses.
shows some acidic properties when it is dissolved in
It is a weak acid and ionizes mainly as monobasic acid. It does
NaOH forming sodium aluminate.
not liberate H+ ion but accepts OH– ion, that is, it behaves as
44. (d) O2 and KO3 are produced. The reaction involved is Lewis acid.

2KOH + 5O3 → 2KO3 + 5O2 + H2O H3BO3 + H2O → B(OH)4− + H+

(d) With borax, it is used in the preparation of buffer solution.


45. (b) The structure is
54. (a), (b), (c), (d) This is because it has variable valency from −2
H H H to +6.
B B
55. (a), (b), (c)
H H H
2s 2p
B (ground state)
46. (c) SiF4 + 2HF → SiF62 − + H2 ↑
2s 2p
47. (b) Iodine monochloride ICI is well known. It is used as Wij's
BH3 (hybridized state) × × × ×
reagent in the estimation of the iodine number of fats
and oils. sp3 hybridization
48. (a) Because carbon forms tetravalency, hence [CCl6]2– does
not exist.
H H
49. (d) Pb3O4 is known as red lead or sindur.
B H
H B
50. (c) CH3 CH3 CH3 H
H2O H
Cl Si Cl HO Si OH + HO Si OH
CH3 CH3 CH3
3c–2e bond
−H2O

CH3 CH3 H H
B H
O Si O Si O H B
CH3 CH3 H
H

51. (b) As the size of the atom increases, the H X bond becomes
weaker and breaks easily. B2H6 can be prepared by reacting BF3 with LiBH3 in the pres-
ence of dry ether.
52. (a) Ozone is that it is an unstable, dark blue, diamagnetic gas.
4BF3 + 3LiBH3 → 2B2H6 + 3BF3 + 3LiF

Chapter-11.indd 367 7/30/2016 12:59:17 PM


368 Chapter 11 The p-Block Elements

56. (b), (c) Oxides of phosphorus P2O3 and P2O5 exist as dimers, melts at 660°C and boils at 1800°C. The density of aluminium is
that is, P2O6 and P4O10. 2.7 g cm–3 (light metal).
Solid PCl5 exists as [PCl4]+ and [PCl6]−. Aluminium dissolves in aqueous sodium hydroxide solution
and liberates hydrogen gas.
57. (a), (b), (c) Al2(SO4)3 + NH4OH → (NH4)3 [Al(OH)6]
X 2Al + 2NaOH + 2H2O → 2NaAlO2 + 3H2 ↑
 Sodium meta
Solution aluminate (soluble)
(NH4)3 [Al(OH)6] is a white coloured compound that is insolu-
ble in excess NH4 OH by soluble in NaOH. 2Al + 6NaOH →
Fused
2Na3AlO3 + 3H2 ↑
Sodium aluminate
58. (a), (d) The structure of white phosphorus is
P Aluminium does not react with hot conc. HNO3 because Al is
60° rendered passive by nitric acid. This is due to oxidation and
formation of a thin film of oxide on its surface.
62. (a), (c)
P P
P O O O
White phosphorus HO S O OH HO S O O S OH

O O O
59. (a), (b), (c) B2H6 contains 3c 2e as well as 2c 2e bonds.
450 K (H2SO3) (H2S2O3)
B2H6 + 6NH3 
→ 3B3N3H6 + 12H2 Peroxomonosulphuric acid Peroxydisulphuric acid
Borazine also known as Caro’s acid also known as Marshall’s acid
(Inorganic benzene)
(X) (Y) 63. (a), (b)
Excess of NH3 Graphite is a paramagnetic substance because the fourth
B2H6 + NH3 High

temperature
→ (BN) x valency of each carbon atom remains unsatisfied, that is, the
(Boron nitride)
fourth valence electron remains unpaired.
(X) (Z) It behaves like metal as well as semiconductor. It conducts
electricity due to unpaired electron in the structure.
H
B B B C C bond length in it is less than the diamond. In graph-
N N N N ite C C covalent distance is 1.42 Å and in diamond the
H B H
N N B B B B C C bond distance is 1.54 Å.
N N N
B B It is thermodynamically more stable as well as less dense than
N B B
H H N N B diamond. Density of graphite is 2.25 g cm–3 and that of dia-
H mond is 3.51 g cm–3.
Borazine Boron nitride 64. (b), (c) Density of the noble gases decreases down the group.
structure

Passage Type
C C C
C C C C 65. (a) Conceptual
C C C C 66. (a) Conceptual
C C C
67. (c) The mineral kinoite contains Ca2+ ions, Cu2+ ions and
C C C water molecules in a 1:1:1 ratio. So, the compound is
C C
Ca2Cu2Si3O10 ⋅ 2H2O.
Graphite structure 68. (c) The structure of xenon oxyfluoride is as follows.
60. (a), (d) F2 is the strongest oxidizing agent; while Cl2 will oxi- O
dize Br2 and I2, Br2 will oxidize only I2 and I2 is the weakest
F F
oxidizing agent.
61. (c), (d) Xe
Aluminium is among the best conductors of electricity,
because it easily loses electrons and acts as a reducing agent. F F
Al → Al3+ + 3e− (responsible for conduction)
XeOF4
Physical properties of aluminium are characteristics of a true Square pyramidal
metal because aluminium has a bluish white luster metal and (Octahedral with one
loses its luster on long exposure to air due to the formation of position unoccupied)
a thin film of oxide on the surface. It is malleable and ductile. It

Chapter-11.indd 368 7/30/2016 12:59:20 PM


Hints and Explanations 369

69. (a) XeF2 oxidizes Cl− to Cl2 while XeF4 oxidizes I− to I2. Borax is made up of two triangular and two tetrahedral units.
This ion is [B4O5(OH)4]2– and the other water molecules are
XeF2 + 2HCl → 2HF + Xe + Cl2
associated with the metal ions. Hence, the number of elec-
XeF4 + 4KI → 4KF + Xe + 2I2
trons is 2.
70. (a) Ar provides inert atmosphere and thus has low reactivity
75. (6) S has 3s2 3p4, that is, it can have a maximum oxidation
with metal.
state of +6. For example, in SF6.
Matrix-Match Type 76. (4) Four isomers are possible for B3N3H4X2.
71. (a) → q; (b)→ s; (c)→ p; (d)→ r
X X
Unipositive ion in case of Tl is more stable due to inert pair
effect. H B− X H B− H
+ +
The type of bonds formed by Group 13 elements depends N N+ N N+
on which is the most favourable in terms of energy. For AlCl3 B− B− B− B−
and GaCl3, the hydration enthalpy exceeds the ionization N+ N+
H H H X
enthalpy, the compounds are covalent in nature.
H H
The reaction is In+(aq) → In(s) + In3+(aq)
72. (a) → q, s; (b) → p; (c) → s; (d) → r, s
∆ H X
(a) H3PO3 
→ PH3 + 3H3PO 4
X H B− H
H B− +
(phosphorus is in +5 oxidation state, it is a tribasic non- + N N+
reducing acid). N N+
(b) XeF6 + 6H2O → XeO3 (explosive solid) + 6HF B− B− B− B−
N+ N+
(c) NO2 + H2O → 2HNO3 + NO H H H H
(nitrogen is in +5 oxidation state in HNO3) X
X
(d) 2HNO3 + P4O10 → 6N2O5 + 4H3PO4
(where P is in +5 oxidation state). −
77. (3) ClF, ICI4 and IBr −2 . The compounds formed when hal-
73. (a) → s; (b) → p; (c) → q; (d) → r ogens react with each other are called interhalogen
(a) Borazine is also called inorganic benzene because its compounds.
structure is similar to benzene.
78. (2) Only two basic oxides are formed from Group 13 and
H H Group 14; they are In2O3 and Tl2O3.
H B− H H C H
+
N N +
C C Group 13 B2O3 > Al2O3 > Ga2O3 > In2O3 > Tl2O3
Acidic nature Acidic Amphoteric Amphoteric Basic Strongly
B− B− C C basic
N+ C
H H H H Group 14 CO; Ge SnO PbO SiO2
H H Neutral   ↓
Amphoteric Acidic
Borazine Benzene

(b) Graphite is an allotrope of carbon.


(c) Phenacite (Be2SiO4) is an example of orthosilicate.
(d) Cassiterite is an ore of tin.
Integer Type
74. (2) The structure is
OH

B−
O O
HO B O B OH
O O
B−

OH

Chapter-11.indd 369 7/30/2016 12:59:22 PM


370 Chapter 11 The p-Block Elements

SOLVED JEE 2016 QUESTIONS


JEE Main 2016 5. Match the items in Column I with its main use listed in Column
II:
1. The species in which the N atom is in a state of sp hybridization
is Column I Column II

(a) NO2 (b) NO2+ (c) NO2 (d) NO3− (A) Silica gel (i) Transistor
(Offline) (B) Silicon (ii) Ion-exchanger
Solution (C) Silicone (iii) Drying agent
(b) The shape and hybridization of N atom in the given species
(D) Silicate (iv) Sealant
is as follows: NO2− = sp2(bent); NO2+ = sp (linear); NO2 = sp2
(bent); NO3− = sp2 (trigonal planar). (a) (A)-(iii), (B)-(i), (C)-(iv), (D)-(ii)
(b) (A)-(iv), (B)-(i), (C)-(ii), (D)-(iii)
2. The pair in which phosphorous atoms have a formal oxidation (c) (A)-(ii), (B)-(i), (C)-(iv), (D)-(iii)
state of +3 is (d) (A)-(ii), (B)-(iv), (C)-(i), (D)-(iii)
(a) pyrophosphorous and pyrophosphoric acids. (Online)
(b) orthophosphorous and pyrophosphorous acids.
Solution
(c) pyrophosphorous and hypophosphoric acids.
(d) orthophosphorous and hypophosphoric acids. (a) Silica gel is used as a drying agent.
(Offline) Silicon is used in transistor as a semiconductor, etc.
Solution Silicone is used as a sealant. It is an elastomer (polymer).
Silicate is used in ion-exchanger. Hydrated sodium alumin-
(b) Orthophosphorous acid H3PO3
ium silicate is zeolite/permutit.
3+ x −6 = 0
6. The bond angle H−X−H is the greatest in the compound:
x = +3 (a) PH3 (b) CH4 (c) NH3 (d) H2O
Pyrophosphorous acid H4P2O5 (Online)

4 + 2x − 10 = 0 Solution
2x − 6 = 0 (b) The central atom is sp3 hybridized in all the given
compounds.
x = +3
However, the bond angles are:

3. The reaction of zinc with dilute and concentrated nitric acid, Compound Hybridization Bond angle
respectively produces: PH3 sp3 98ο
(a) NO2 and N2O (b) N2O and NO2
(c) NO2 and NO (d) NO and N2O CH4 sp3 109. 28ο
(Offline) NH3 sp3 107ο
Solution H2O sp3 104.5ο
(b) 4Zn + 10HNO3 (dil.) → 4Zn (NO3)2 (aq) + N2O + 5H2O
Zn + 4HNO3 (conc.) → Zn (NO3)2 (aq) + 2NO2 + 2H2O In CH4, the geometry is tetrahedral with bond angle 109.
28°. In other compounds presence of lone pairs distorts
4. The non-metal that does not exhibit positive oxidation state is the geometry.
(a) chlorine. (b) iodine. (c) fluorine. (d) oxygen.
(Online) 7. Identify the incorrect statement:
(a) The S-S-S bond angles in the S8 and S6 rings are the same.
Solution (b) Rhombic and monoclinic sulphur have S8 molecules.
(c) Fluorine is the most electronegative element. It can gain (c) S2 is paramagnetic like oxygen.
one electron and hence shows the oxidation state −I only. (d) S8 ring has a crown shape.
The other halogen elements show positive oxidation (Online)
states of +I, +III, +V, +VII as shown below.
Solution
Element Oxidation state (a) Crystals of both rhombic and monoclinic allotropic
Cl HOCl (+I), HClO2(+III), HClO3(+V), HClO4(+VII) forms of sulphur are built up from zigzag rings of eight
Br HOBr (+I), HBrO3(+V), HBrO4(+VII) sulphur atoms. In these S8 has crown shaped, puckered
conformation. Another allotropic form, ε-sulphur, also
I HOI (+I), HIO3(+V), HIO4(+VII) known as Engel’s sulphur, contains S6 rings arranged in a

Chapter-11.indd 370 7/30/2016 12:59:25 PM


Solved JEE 2016 Questions 371

chair conformation. S-S-S bond angle in S8 is 107.8o, while Solution


in S6 it is 102.2o. S2 is paramagnetic as it has unpaired (b) The increasing order of atomic radii is as follow:
electron.
Ga < Al < In < Tl
8. The following statements concern elements in the periodic The atomic radius generally increases on moving down a
table. Which of the following is true? group in the periodic table. As an anomaly, the atomic radius of
(a) For Group 15 elements, the stability of +5 oxidation state Ga is less than that of aluminium because of poor shielding of
increases down the group. nuclear charge by ten 3d electrons. As a result, the outer shell
(b) Elements of Group 16 have lower ionization enthalpy val- electrons are held more firmly by the nucleus and contraction
ues compared to those of Group 15 in the corresponding of radius is observed. This contraction is also called d-block
periods. contraction. The size of Tl is similarly effected by fourteen 4f
(c) The Group 13 elements are all metals. electrons (lanthanoid contraction) and the atomic radius of Tl
(d) All the elements in Group 17 are gases. is almost similar in size to In.
(Online)
2. The crystalline form of borax has
Solution (a) tetranuclear [B4O5(OH)4]2− unit.
(b) Consider the given statements: (b) all boron atoms in the same plane.
(a) In Group 15 elements, due to the inert pair affect the (c) equal number of sp2 and sp3 hybridized boron atoms.
stability of +5 oxidation state decreases down the (d) one terminal hydroxide per boron atom.
group. Solution
(b) Group 15 elements have half-filled electronic config-
uration, thus have high values of ionization enthalpy, (a), (c), (d) Molecular formula for borax is Na2[B4O5(OH)4]⋅8H2O
while Group 16 elements achieve half-filled configura- OH
tion on loss of one electron therefore have lower ioni-
zation enthalpy. B−
(c) Except for the lightest element (boron), all other Group sp3
O O
13 elements are relatively electropositive and are thus O B OH
HO B sp2
metals. sp2
(d) Group 17 elements are halogens; they exist in all three O O
B−
states of matter at room temperature. sp3

9. Assertion: Among the carbon allotropes, diamond is an insula- OH


tor, whereas, graphite is a good conductor of electricity. It has tetranuclear (B4O5(OH)4)2− unit. Two boron atoms
Reason: Hybridization of carbon in diamond and graphite are are sp2 hybridized and other two atoms are sp3 hybridized.
sp3 and sp2, respectively. Only two boron atoms lie in the same plane. Each boron
(a) Both assertion and reason are correct, but the reason is not atom has one terminal hydroxide.
the correct explanation for the assertion. 3. The nitrogen containing compound produced in the reaction
(b) Both assertion and reason are correct, and the reason is of HNO3 with P4O10
the correct explanation for the assertion. (a) can also be prepared by reaction of P4 and HNO3.
(c) Both assertion and reason are incorrect. (b) is diamagnetic.
(d) Assertion is incorrect statement, but the reason is correct. (c) contains one N N bond.
(Online) (d) reacts with Na metal producing a brown gas.
Solution
Solution
(b) In diamond, each carbon is sp3 hybridized and is tetra-
hedrally surrounded by four other carbon atoms. The (b), (d) The reaction of HNO3 and P4O10 produces N2O5.
tetrahedral carbon are linked together into a three dimen-
4HNO3 + P4 O10 → 2N2O5 + 4HPO3
sional giant molecule. Strong covalent bonds extend in all
directions. The reaction of HNO3 with P4 does not yield N2O5.
In graphite only three of the valency electrons of each
carbon atom are involved in forming, σ bonds (using sp2 P4 + 20HNO3 → 4H3PO 4 + 20NO2 + 4H2O
hybrid orbitals). The fourth electron forms a p bond. The p
electrons are delocalized over the whole sheet, and as they The structure of N2O5 has one N–O–N bond, but no N–N bond.
are mobile, graphite conducts electricity. It is diamagnetic in nature.
O O
JEE Advanced 2016 N N
O O O
1. The increasing order of atomic radii of the following Group 13
elements is N2O5 reacts with sodium metal to produce NO2 (brown gas).
(a) Al < Ga < In < Tl (b) Ga < Al < In < Tl N2O5 + Na → NaNO3 + NO2 ↑
(c) Al < In < Ga < Tl (d) Al < Ga < Tl < In Brown gas

Chapter-11.indd 371 7/30/2016 12:59:28 PM


Chapter-11.indd 372 7/30/2016 12:59:28 PM
12 Some Basic Principles in
Organic Chemistry

Question Distribution in JEE (Main and Advanced)

3
No. of Questions

JEE (Main)
2
JEE (Adv)

0
2016 2015 2014 2013 2012 2011 2010 2009 2008 2007

Concept Distribution in JEE (Main and Advanced)

Topics Covered
Year
JEE (Main) JEE (Advanced)
Hybridization, Shapes of Simple Molecules and Nomen- Electronic Displacement in a Covalent Bond, Isomerism
2007
clature, Isomerism – Structural and Stereoisomerism – Structural and Stereoisomerism
Hybridization, Shapes of Simple Molecules and Nomen- Electronic Displacement in a Covalent Bond, Isomerism
2008
clature – Structural and Stereoisomerism
Reaction Intermediates, Electronic Displacement in a
Isomerism – Structural and Stereoisomerism, Reaction
2009 Covalent Bond, Hybridization, Shapes of Simple Mole-
Intermediates
cules and Nomenclature
2010 Isomerism – Structural and Stereoisomerism
2011 Electronic Displacement in a Covalent Bond Electronic Displacement in a Covalent Bond
Hybridization, Shapes of Simple Molecules and Nomen-
2012
clature, Isomerism – Structural and Stereoisomerism
2013 Reaction Intermediates
Hybridization, Shapes of Simple Molecules and Nomen- Electronic Displacement in a Covalent Bond
clature, Isomerism – Structural and Stereoisomerism,
2014
Reaction Intermediates, Electronic Displacement in a
Covalent Bond
2015 Isomerism – Structural and Stereoisomerism Isomerism – Structural and Stereoisomerism
2016 Optical Stereoisomerism

Chapter-12.indd 373 7/30/2016 4:14:18 PM


374 Chapter 12 Some Basic Principles in Organic Chemistry

SUMMARY
1. Hybridization and shapes of molecules
(a) Hybridization refers to the mixing of atomic orbitals to form newly hybridized orbitals, which are involved in bonding and influ-
ence bonding properties as well as the overall molecular geometry.
(b) Orbital hybridization is a mathematical approach that involves the combining of individual wave functions for s and p orbitals to
obtain wave functions for new orbitals of equivalent energy (degenerate) and uniform shape.
(c) The new orbitals have the properties of the original orbitals taken separately in varying proportions and are called hybrid atomic
orbitals.
(d) sp3 hybridization
(i) Mixing one s and all three p atomic orbitals produces a set of four equivalent sp3 hybridized atomic orbitals. The four sp3
hybrid orbitals having 75% p character and 25% s character point towards the vertices of a tetrahedron.
(ii) In case of methane, CH4, each sp3 hybrid orbital of carbon containing 1 electron overlaps with 1s orbital of H atom to form a
s bond.
(e) sp2 hybridization
(i) Mixing of one s orbital and two p orbitals of a carbon atom gives rise to three sp2 orbitals. These three orbitals are equivalent
and are distributed at 120° to each other giving a planar triangle. Each orbital consists of 67% p and 33% s character struc-
ture. The remaining unchanged p orbital lies perpendicular to the plane of the three sp2 orbitals.
(ii) In case of ethene, each carbon uses two of its sp2 hybrids to form s bonds to hydrogen atoms. The third sp2 hybrid on each
carbon is used to form a s bond between the two carbon atoms. The remaining unhybridized 2p orbitals, one from each
carbon atom, overlap to produce a p bond, which accounts for the second bond of the double bond.
(f) sp hybridization
(i) The hybridization of one s orbital and one p orbital on a carbon atom gives rise to two equivalent sp hybrid orbitals. Generally
the pz orbital is involved in overlap with s orbital and the hybrid sp orbital has 50% s and 50% p character. The two hybridized
sp orbitals are in a linear arrangement and are 180° apart.
(ii) In case of ethyne the sp orbitals point in opposite directions and are used to form the s bonds. The unhybridized 2px and 2py
orbitals are perpendicular to the C C bond axis and overlap sideways to form two separate p bonds that surround the C C
s bond.
Tip The shortest C H bonds are associated with those carbon orbitals with the greatest s character. The sp orbitals
of ethyne with 50% s (and 50% p) in character form the shortest C H bonds. The sp3 orbitals of ethane with 25% s (and
75% p) in character form the longest C H bonds.

2. Index of hydrogen deficiency or Degree of unsaturation


(a) Unsaturated and cyclic compounds
(i) The index of hydrogen deficiency (IHD) or degree of unsaturation is defined as the number of pairs of hydrogen atoms
that must be subtracted from the molecular formula of the corresponding alkane to give the molecular formula of the com-
pound under consideration.
(ii) For example, both cyclohexane and 1-hexene have an index of hydrogen deficiency equal to 1 (meaning one pair of hydro-
gen atoms). The corresponding alkane (i.e., the alkane with the same number of carbon atoms) is hexane:
C6H14 = formula of corresponding alkane (hexane)
C6H12 = formula of compound (1-hexene or cyclohexane)
--------
H2 = difference = 1 pair of hydrogen atoms
Index of hydrogen deficiency = 1
• The index of hydrogen deficiency of ethyne (acetylene) or of 1,3-butadiene equals 2; the index of hydrogen deficiency of
1,3,5-hexatriene equals 3.
(iii) Each double bond consumes one molar equivalent of hydrogen; each triple bond consumes two.
(b) Compounds containing halogens, oxygen, or nitrogen
(i) For compounds containing halogen atoms, we simply count the halogen atoms as though they were hydrogen atoms. For
example, we treat a compound with the formula C4H6Cl2 as though it were C4H8, and we get an IHD = 1.
(ii) For compounds containing oxygen, we simply ignore the oxygen atoms and calculate the IHD from the remainder of the
formula. For example, we treat a compound with the formula C4H8O as though it were C4H8, and we get an IHD = 1.
(iii) For compounds containing nitrogen atoms we subtract one hydrogen for each nitrogen atom, and then we ignore the nitro-
gen atoms. For example, we treat a compound with the formula C4H9N as though it were C4H8, and we get an IHD = 1.
3. Electronegativity
(i) Electronegativity is the ability of an element to attract electrons that it is sharing in a covalent bond.
(ii) The electronegativity of carbon in its compounds is also affected by the state of hybridization. The greater the s character of
the hybrid orbital, greater is the electronegativity. Thus, carbon atom with sp hybridized orbital with 50% s character is more
electronegative than sp2 or sp3 hybridized carbon atom.

Chapter-12.indd 374 7/30/2016 4:14:19 PM


Summary 375

(iii) The relative acidities of ethyne, ethene, and ethane parallel the effective electronegativity of the carbon atom in each
compound.
(iv) The acidity constant, Ka, is expressed as the negative logarithm of pKa.
pKa = − log Ka
There is an inverse relationship between the magnitude of the pKa and the strength of the acid. The larger the value of the
pKa, the weaker is the acid.
(v) The protons of ethyne are more acidic than those of ethane, which in turn are more acidic than those of ethane.
H
H H H H

H C C H C C C C
H
H H H
H
Ethyne Ethene Ethane
pKa = 25 pKa = 44 pKa = 50

4. Dipole moment
(i) The dipole moment is defined as the product of the magnitude of the charge in electrostatic units (esu) and the distance that
separates them in centimeters (cm):
Dipole moment = charge (in esu) × distance (in cm)
m=e ×d
(ii) Any diatomic molecule in which the two atoms are different (and thus have different electronegativities) will, of necessity,
have a dipole moment. Because the electronegativity of chlorine is greater than that of carbon, each of the carbon–chlorine
bonds in CCl4 is polar.
5. Representation of structure
(a) Two dimensions
H H H
H H H
H OH
H C C C OH H C C C O CH3CH2CH2OH
H H H
H H H
Electron-dot formula Dash formula Condensed formula Bond-line formula
(b) Three dimensions
(i) The representation of three dimensional molecules in two dimensions can be done by the following ways: Wedge formula;
Sawhorse formula, Fischer projection and Newman projection formula.
CH3 H
H H H H
H H Br H
H
H H
H H Br H H
H H
H H CH3 H
Wedge and dash Sawhorse Fischer projection Newman projection

(ii) Fischer projection


• The Fischer projection consists of both horizontal and vertical lines, vertical lines represent bonds that project behind the
plane of the paper (or that lie in it). Horizontal lines represent bonds that project out of the plane of the paper.
• Fischer projections are written with the main carbon chain extending from top to bottom and with all groups eclipsed.
CHO CHO CHO

OH H OH H OH
HOH2C
H
CH2OH CH2OH
(R)-(+)-Glyceraldehyde Fischer projection
6. Classification of organic compounds
(a) Open chain or acyclic or aliphatic compounds
These compounds comprise of an open chain system of carbon atoms, which can be either non-branched or branched.
Straight chain compounds: CH3 CH2 CH2 CH3 H2C CH2
Butane Ethene

Chapter-12.indd 375 7/30/2016 4:14:20 PM


376 Chapter 12 Some Basic Principles in Organic Chemistry

Branched chain compounds:


CH3

CH3 CH CH3 CH3 C CH3

CH3 CH3
2-Methylpropane 2,2-Dimethylpropane
(iso-butane) (neo-pentane)

(b) Closed chain or cyclic (or ring) compounds


These compounds contain one or more closed chains (rings) of atoms. These are of two types:
(i) Homocyclic or carbocyclic compounds: Compounds in which the ring consists of only carbon atoms. These can be are fur-
ther divided into:
• Alicyclic compounds: When three or more carbon atoms resembling aliphatic compounds are contained within a ring. The
saturated alicyclic hydrocarbons have the general formula CnH2n.

H2 H2 H2
C C C
CH2 H2C CH2 H2C CH2 CH H2C CH HC CH

H2C CH2 H2C CH2 H2C CH2 H2C CH H2C CH HC CH


C C C
H2 H2 H2
Cyclopropane Cyclobutane Cyclohexane Cyclopropene Cyclohexene Cyclohexa-1,4-diene

• Aromatic compounds: Carbocylic compounds which contain at least one benzene ring. The aromatic compounds may also
have a side-chain or a functional group attached directly to the ring or may contain more than one benzene rings fused
together.
H
CH3 OH NO2 CHO
C
H C C H

H C C H
C
Toluene Phenol Nitrobenzene Benzaldehyde Naphthalene Anthracene
H
Benzene

(ii) Heterocyclic compounds


When atoms of more than one kind make up the ring in the compounds. In these compounds generally one or more atoms
of elements such as N, O or S (other than C) are present, and are called heteroatoms.
H
C
HC CH HC CH HC CH HC CH

HC CH HC CH HC CH HC CH
N O S N
H
Pyridine Furan Thiophene Pyrrole

7. Nomenclature of organic compounds


Word root-represents number of carbon atoms in a chain. For example, Meth-(1 C), Eth-(2 C), Prop-(3 C).
Prefix: Part of name appearing before word root.
Primary suffix: Part of name used to indicate presence of single (saturation) or double/triple (unsaturation) bonds in carbon
chain.
Secondary suffix: Part of name used to indicate presence of functional groups in organic compound. It is usually added to primary
suffix.
IUPAC name = Prefix (1 or more) + Word root + Primary suffix + Secondary suffix

Chapter-12.indd 376 7/30/2016 4:14:21 PM


Summary 377

(a) Alkanes: In the IUPAC nomenclature, alkanes end with suffix ‘-ane’. For example, methane (CH4), ethane (C2H6), propane (C3H8).
Rings are designated by the prefix “cyclo”, for example, cyclopropane (C3H6), cyclobutane (C4H8).
(b) Alkenes: In the IUPAC nomenclature, alkenes end with suffix ‘-ene’. For example, ethene (C2H4), propene (C3H6), butene
(C4H8).
(c) Alkynes: In the IUPAC nomenclature, alkynes end with suffix ‘-yne’. For example, ethyne (C2H2), propyne (C3H4), butyne (C4H6).
(d) Nomenclature of molecules containing substituents and functional groups

Family of compounds Structure Prefix Suffix


Carboxylic Acid O carboxy- -oic acid (-carboxylic acid)

R C OH
Aldehyde O oxo-(formyl) -al (carbaldehyde)

R C H
Ketone O oxo- -one

R C R
Alcohol R O H hydroxy- -ol
Amine amino- -amine
R N

Alkyl R alkyl- -------------


Alkoxy R O alkoxy- -------------
Haolgen F− fluoro- -------------
Cl− chloro- -------------
Br− bromo- -------------
Ι− iodo- -------------
Imine NH imino
Thioalcohol R SH mercapto- -thiol
Amide R CONH2 carbamyl- -amide
Acid chloride R COCl chloroformyl- -oyl
Ester R COOR carbalkoxy- -oate
Nitrile R CN cyano- -nitrile
Isonitrile R NC isocyano- -isonitrile
Sulphonic acid R SO3H sulpho- sulphonic acid
Anhydride R (CO)2O anhydride
Nitro NO2 nitro
Other groups CH CH 2 vinyl
CH2CH CH2 allyl
C6H5 phenyl
CH3 methyl
C2H5 ethyl
CH2CH2CH3 n-propyl
CH(CH3)2 iso-propyl
CH2CH2CH2CH3 n-butyl
C(CH3)3 tertiary butyl
NO nitroso
SR alkylthio

Chapter-12.indd 377 7/30/2016 4:14:23 PM


378 Chapter 12 Some Basic Principles in Organic Chemistry

(e) Naming of saturated compounds


(i) Identify the longest continuous parent carbon chain and determine its root name.
(ii) For cyclic compounds, the ring is usually considered the parent chain, unless it is attached to a longer chain of carbons;
indicate a ring with the prefix “cyclo” before the root name.
(iii) Number the chain in the direction so as to provide the smallest number to the first substituent.
(iv) Determine the name and position number of each substituent.
(v) Indicate the number of identical groups by the prefixes di, tri, tetra, etc.
(vi) Place the position numbers and names of the substituent groups, in alphabetical order, before the root name. In alphabet-
izing, ignore prefixes like sec-, tert-, di, tri, etc., but include iso and cyclo.
6 7 8
H3C CHCH2CH3
1 2 3
CH3 CH2CH2CH3 1 1-sec-butyl-3-nitrocyclohexane
6 2 (numbering determined by the
CH3 CH CH C CH CH2CH3
4 5 alphabetical order of substituents)
5 NO2
Cl Br CH3 3
4
3-Bromo-2-chloro-5-ethyl-4,4-dimethyloctane

(f) Naming of unsaturated compounds


(i) For carbon chain containing double bond.
• Number the carbon chain so that the C C has the lower position number.
• Change “ane” to “ene” and assign a position number to the first carbon of the C C.
• Designate geometrical isomers with a cis, trans or E,Z prefix.
F F
CH CH CH CH2 C C CH CH2
F CH3 F CH3
4,4-Difluoro-3-methylbut-1-ene 1,1-Difluoro-2-methyl-
buta-1,3-diene
(ii) For carbon chain containing triple bond.
• Number the carbon chain so that the C C has the lower position number.
• Change “ane” to “yne” and assign a position number to the first carbon of the C C.
F H
1 2 3 4 5
CH CH C CH HC C C CHCH3
F CH3
4,4-Difluoro-3-methylbut-1-yne Pent-3-en-1-yne
(iii) For chain containing both double and triple bond.
• Both alkene and alkyne are considered to have equal priority.
• Whichever (double and a triple bond) is closer to the end of the chain determines the direction of numbering. In the case
where each would have the same position number, the double bond takes the lower number.
• In the name, “ene” comes before “yne” because of alphabetization.
5 4 3 2 1
HC C CH2CH CH2
Pent-1-en-4-yne

(g) Naming of compounds containing functional groups


• Identify longest C chain containing maximum number of carbon.
• Number the C chain such that lowest number is assigned to functional group (and not to any unsaturation present)
• If the carbon chain includes multiple bonds, replace “ane” with “ene” for an alkene or “yne” for an alkyne.
• Designate the position of the multiple bond with the number of the first carbon of the multiple bond.
CH3CH2 CH2CH3
N

CH3CH2CH2 NH2 CH2 CH CHCH3


Propan-1-amine N,N-Diethylbut-3-en-2-amine

Chapter-12.indd 378 7/30/2016 4:14:25 PM


Summary 379

• For compound containing multiple functional groups, the principal functional group determines suffix name and secondary
functional group determines prefix name.
Tip (a) The order of decreasing priority for some of the functional groups is
COOH, SO3H, COOR, COCl, CONH2, CN, HC O, >C O, OH, NH2, >C C<, C C
(b) The functional groups, alkyl ( R), phenyl (C6H5 ), alkoxy ( OR), halogens (F, Cl, Br, I) and nitro ( NO2) are
always treated as subordinate functional groups in presence of other functional groups.
8. Isomerism – structural and stereoisomerism
(a) Structural isomerism
Structural isomerism occurs when two or more organic compounds have the same molecular formula but different structural
formulae.
(i) Chain isomers: These differ in the order in which groups are bonded in a chain. For example, butane and isobutane.
(ii) Position isomers: These are compounds that differ in the position of the substituent or the functional group on the parent
carbon chain. For example, 1-bromopropane and 2-bromopropane.
(iii) Functional group isomers: These are compounds that have the same molecular formula but have different functional
groups. For example, ethanol and dimethyl ether.

Tip Tautomerism is a special case of functional isomerism, where an equilibrium exists between enol and a ketone. The
enol and ketone iosmers rapidly inter-convert via the migration of a proton.

(iv) Metamers: These are compounds that have different alkyl chains attached to the functional group. For example, ethers,
thioethers ketones, secondary amines, etc., show metamerism.
(v) Ring-chain isomerism: These are compounds that have the same molecular formula but possess open-chain or cyclic struc-
tures. This isomerism is generally exhibited by alkenes and cyclic compounds. For example, propene and cyclopropane.
(b) Stereoisomerism
Isomers that have the same molecular formula and ligands, but differ in the arrangement of those ligands in three dimensional
space.
(i) Geometrical isomerism
Geometric isomers are those which have the same structural formulas but differ in the arrangement of groups at a single
atom, at double bonds, or in rings. For example 2-butene can be represented by two structures: cis-2-butene (similar groups
on same side) and trans-2-butene (similar groups on opposite side).

cis trans
(ii) E & Z notation for geometrical isomers
• The Cahn-Ingold and Prlog sequence rule are used for naming geometric isomers.
ο Assign priority to the groups attached to the two double bonded carbons.
ο If groups of higher priority are on same side of double bond, the isomer is “Z” isomer (zusammen= together); if groups
of higher priority are on opposite side of double bond, the isomer is “E” isomer (entgegen= across).
• Rules for assigning priority are based on the highest atomic number of the atom directly bonded to the carbon.
1 17 high 1 6
H Cl priority H CH3
high
priority
H3C CH3 high H3C Cl high
6 6 priority 6 17 priority
(E)-2-Chloro-2-butene (Z)-2-Chloro-2-butene
ο If carbon is bonded to same atoms, rules are applied at first point of difference.
ο If carbon is bonded to isotopes of same element, isotope with higher mass number is given higher priority.
ο If carbon is bonded to double or triple bond, it is considered equivalent to the same number of single bonded atoms.
(iii) Optical isomerism
These are substances that can rotate the plane of plane-polarized light and generally contain a carbon atom attached to
four different groups (chiral centre). The stereoisomers that can exhibit optical isomerism can be either enantiomers or
diastereomers.

Tip (a) The number of optically active forms of a compound depends on the number of chiral carbon atoms. If there
are n chiral carbons, the number of optically active form will be 2n, provided the chiral atoms are not identically
substituted.
(b) In case of bridge/bicyclic compounds, the number of stereoisomers is equal to the number of chiral centres
because no carbon centres rotation possible.

Chapter-12.indd 379 7/30/2016 4:14:26 PM


380 Chapter 12 Some Basic Principles in Organic Chemistry

• Enantiomers: These are optical isomers that are non-superimposable mirror images of each other.
CH3 CH3

H C Br Br C H

H C Br Br C H

C2H5 C2H5
• Diastereomers are one of a set of stereoisomers that are not enantiomers i.e. stereoisomers that are not mirror images of
one another and are  non-superimposable  on one another. Stereoisomers with two or more stereocentres can be
diastereomers.
CHO CHO
H OH HO H
Enantiomers
H OH HO H
CH2 OH CH2 OH

Diastereoisomers

CHO CHO
H OH H OH
Enantiomers
HO H HO H
CH2 OH CH2 OH
• Mesomers: These are optically inactive substances having two or more chiral centres and a plane of symmetry, which
bisects the compound into two equal parts that are mirror images of each other.
(iv) Chiral molecules without chiral centre
Sometimes, a compound shows optical activity even when there is no chiral centre present. For example, allenes (two ends
of allene are perpendicular causing molecular dissymmetry), biphenyls (plane of two phenyl rings are perpendicular to each
other and mirror images non-superimposable) and spiranes (similar to allenes).
Br
H Cl
CH3 H
C C C
H CH3
Cl H
Br
Dimethylallene Dibromobiphenyl Dicholorospiroheptane

9. Nomenclature of chiral compounds


(a) D and L nomenclature
(i) The main carbon chain is oriented vertically (Fischer projection) with the lowest numbered carbon at the top.
(ii) Next, arrange the structure around the particular chiral carbon whose configuration is to be assigned such the horizontal
bonds to that carbon extend towards observer and the vertical bonds extend away from observer.
(iii) The relative positions of the substituents on the horizontal bonds at the chiral centres are examined. If the main substitu-
ent is on the left of the main chain, the L configuration is assigned; if this substituent is on the right, the D configuration is
assigned.

Main substitutent Main substitutent


on left on left
CHO CHO CHO CHO

HO H = HO H H OH = H OH

CH2OH CH2OH CH2OH CH2OH

L D

Chapter-12.indd 380 7/30/2016 4:14:27 PM


Summary 381

(b) (R) and (S) configurations are assigned on the basis of the following procedure.
The Cahn-Ingold-Prelog priority rules are used for naming chirality centres.
(i) Each of the four groups attached to the stereogenic carbon is assigned a priority or preference a, b, c, or d. Priority is first
assigned on the basis of the atomic number of the atom that is directly attached to the stereogenic carbon. The group with
the lowest atomic number is given the lowest priority, d; the group with next higher atomic number is given the next higher
priority, c; and so on.
We can illustrate the application of the rule with the 2-butanol enantiomer, I:
CH3 (b or c)

(a) HO C H (d)

CH2 (b or c)

CH3
I
(ii) When a priority cannot be assigned on the basis of the atomic number of the atoms that are directly attached to the stereo-
genic carbon, then the next set of atoms in the unassigned groups is examined. This process is continued until a decision can
be made.
H
(c)
H C H (H, H, H)

(a) HO C H (d)

H C H
(b) (C, H, H)
H C H

H
I
(iii) We now rotate the formula (or model) so that the group with lowest priority (d) is directed away from us. Then we trace a path
from a to b to c. If, as we do this, the direction of our finger (or pencil) is clockwise, the enantiomer is designated (R). If the direc-
tion is counterclockwise, the enantiomer is designated (S). On this basis the 2-butanol enantiomer I is (R)-2-butanol
(c)
CH3

CH3

HO C H (d) H
OH
(a)
CH2 Viewer
CH3CH2
CH3 (b)
Arrows are clockwise
(R)-2-Butanol

Tip Groups containing double or triple bonds are assigned priorities as if both atoms were duplicated or triplicated.
Thus, the vinyl group, CH CH2, is of higher priority than the isopropyl group, CH(CH3)2. That is
H H H H

C C H> C C H

(C) (C) H

H C H

H
C, H, H > H, H, H
Vinyl group Isopropyl group

Chapter-12.indd 381 7/30/2016 4:14:28 PM


382 Chapter 12 Some Basic Principles in Organic Chemistry

10. Elements of symmetry


Optically active compounds are those compounds which are not superimposable on their mirror images. If the molecule is sym-
metric then it is superposable and so achiral. If the molecule is asymmetric then it is not superposable, and so it is optically active.
For an asymmetric molecule the following elements of symmetry must be absent.
(a) Plane of symmetry (r ): A plane of symmetry is an imaginary plane which bisects the molecule into two equal halves that are
mirror images of each other.

COOH
H OH
Plane of symmetry
H OH s
COOH

(b) Centre of symmetry (i): An imaginary point through which a line is drawn in a direction from an atom or group to the same atom
or group placed at the same distance just opposite to the imaginary centre. This rule is applicable for each atom of the molecule.
Cl CH3
C C
CH3 Cl

(c) Rotational axis of symmetry (Cn): It is an imaginary axis through which rotation of molecule, by certain angle will result the
same molecule. Examples are water (C2) and ammonia (C3).
(d) Alternating axis of symmetry (Sn): This operation involves rotation followed by reflection, that is, rotation by an angle of
360°/n about the axis and followed by reflection in a plane perpendicular to the axis of rotation. The resultant molecule is iden-
tical to the original molecule.
11. Electronic displacement in a covalent bond
(a) Inductive effect (+I or -I)
This effect arises because in a single covalent bond between two unlike atoms, the shared electron is displaced more towards
electronegative atom.
(i) The effect is said to be –I if a group pulls the electrons away from the carbon atom. Such a group is called an electron-with-
drawing group. Examples:

R3N+ > NO2 > CN− > COO− > F− > Cl− > Br− > I− > OAr− > OH− > Phenyl
(ii) The effect is said to be +I if a group pushes the electrons towards the carbon atom. Such a group is called electron-releasing
group. Examples:
O− > COO− > (CH3C) > (CH3C)2CH > CH3CH2CH2 > CH3CH2 > CH3
(iii) Inductive effect decreases with the increase in distance from the bond.
(iv) Inductive effect causes development of a dipole moment in the molecule, which increases with increase in inductive
effect.
(v) The bond length of the organic molecules decreases with increase in inductive effect.
(b) Electromeric effect (+E or -E)
This effect involves the displacement of p electrons in a multiple bond towards one of the bonded atoms at the demand of an
attacking species. It is a temporary displacement effect and ends when attacking reagent is removed from the reaction.
(i) If the transference of p electrons occurs towards the atom where attacking species attaches, it is called +E effect.
+
C C + H+ C C

H
(ii) If the transference of p electrons occurs away from the atom where the attacking group attaches, it is called −E effect.

O

C O + CN C
CN

Tip When inductive and electromeric effects are both operative in opposite directions, the electromeric effect
predominates.

Chapter-12.indd 382 7/30/2016 4:14:29 PM


Summary 383

(c) Resonance structure


Whenever a molecule or ion can be represented by two or more Lewis structures that differ only in the positions of the elec-
trons, they can be represented by resonance structures or contributors.
2− 2− 2−
O O O
C C C
O O O O O O
(i) The energy of the actual molecule is lower than the energy that might be estimated for any contributing structure and is
called resonance stabilization.
(ii) Equivalent resonance structures make equal contributions to the hybrid, and a system described by them has large reso-
nance stabilization.
(d) Resonance (mesomeric) effect (R or M effect)
This effect arises when electron redistributions take place in unsaturated, especially in conjugated systems via their p orbitals
thereby generating centres of high- and low-electron densities. It is a permanent effect and produces polarity in the molecule
by the interaction of two p bonds or between a p bond and a lone pair.
(i) Groups which tend to withdraw electrons from the conjugated system possess −R or −M effect, for example, NO2, CN,
CO,
(ii) Groups which tend to donate electrons to the conjugated system possess +R or +M effect, for example, Cl, Br, I, NH2,
OH, OCH3, etc.
(e) Steric Inhibition of resonance (SIR) effect
Restriction in exhibition of resonance due to steric hindrance is known as SIR effect.

CH3 CH3 CH3 CH3

N N
CH3 1 CH3
6
2

5 3
4
(P) (Q)

In compound (P) due to steric hindrance across nitrogen, bond rotation across C1 and nitrogen will be hindered and the lone
pair of electrons on nitrogen will not be involved in resonance with the phenyl ring. As a result, the compound (P) is more basic
than compound (Q).
(i) SIR can affect dipole moment of the compounds.
CH3 CH3 CH3 CH3
N N NO2 NO2
CH3 CH3 CH3 CH3
and
CH3 CH3 CH3 CH3
NO2 NO2
m = 6.87 D m = 4.11 D m = 3.95 D m = 3.39 D

(ii) SIR can also affect bond length.


NO2 NO2
a b
CH3O OCH3

(X) (Y)

• Bond lengths of C N are of the order b > a because of the SIR effect in compound (Y). NO2 group will not participate in
resonance with the phenyl ring in (Y).
• NO2 group is involved in resonance with phenyl ring lending double bond character to bond a.

Chapter-12.indd 383 7/30/2016 4:14:30 PM


384 Chapter 12 Some Basic Principles in Organic Chemistry

(f) Ortho- and para effects


(i) Ortho-substituted benzoic acid is stronger than benzoic acid.

CO2H CO2H
CH3
>

Benzoic acid o-Toluic acid


Due to steric crowding between carboxylic and the methyl group, resonance does not take place between them. The inhi-
bition of +M effect in o-toluic acid, makes it a stronger acid compared to benzoic acid.
(ii) Ortho-substituted aniline is a weaker base than aniline.

NH2 NH2
CH3
>

Aniline o-Toluidine
(iii) Para-substituted aniline is a stronger base than aniline.
NH2 NH2

>

OCH3

(g) Hyperconjugation
When s -electrons of a C H bond are in conjugation with an adjacent p bond, then this type of conjugation is called
hyperconjugation.
(i) Condition for Hyperconjugation
The presence of s -hydrogen at an alkene, alkyl carbocation, alkyl free radical or a phenyl group is the main condition for
hyperconjugation.
Hyperconjugation can be divided into five categories:
Type I: s -p conjugation in alkene
• The examples of this type of hyperconjugation are as follows:
a a a
CH3 CH CH2 CH3 CH2 CH CH2 CH3 CH CH CH2
Propene 1-Butene 3-Methyl-1-butene
CH3
(3a -hydrogens) (2a -hydrogens) (1a -hydrogen)

Hyperconjugation explains the stability of alkenes. More the number of a-hydrogens in an alkene, more will be its stability.
Thus, the order of stability some alkenes is as follows

CH3 CH3 CH3 CH3 CH3


C C > C C > C CH2
CH3 CH3 CH3 H CH3

(12a -hydrogens) (9a -hydrogens) (6a -hydrogens)

Tip Number of resonating structures = Number of a-hydrogens + 1


This expression is applicable to alkenes only.

Type II: In 1935, Baker Nathan effect explained that hyperconjugation dominates inductive effect. The decreasing order of
electron density on phenyl ring is as follows:

Chapter-12.indd 384 7/30/2016 4:14:32 PM


Summary 385

CH3 CH3

CH3 CH2 CH3 CH CH3 CH3 C CH3

> > >

(3a -hydrogens) (2a -hydrogens) (1a -hydrogens) (0a -hydrogens)


Here, more the number of a-hydrogens, more will be the electron density on the phenyl ring. The electron donation of the alkyl
groups is dominated by hyperconjugation effect rather than inductive effect.
Type III: Hyperconjugation in carbocations
In the carbocations, the conjugation is between s-bond of C H and positive charge on carbon.
a + a + a + a
CH3 CH2 CH3 CH CH3 CH3 C CH3
a
CH3
1° 2° 3°
(3a -hydrogens) (6a -hydrogens) (9a -hydrogens)
(A) (B) (C)

The stability of the above noted carbocations follows the order 3° > 2° > 1° i.e., (C) > (B) > (A)
Type IV: Hyperconjugation in free radical
a a a
CH3 CH2 CH3 C CH3
a
CH3
(A) (B)
(3a -hydrogens) (9a -hydrogens)

Here again, more the number of a-hydrogens, more will be the stability of the radical. So, (B) is more stable than (A).
Type V: Hyperconjugation is also observed in systems of the following type
X

C C C

(X = halogen)

Cl Cl
Cl + + Cl +
Cl C CH CH2 C CH CH2 Cl − C CH CH2 Cl C CH CH2
Cl Cl
Cl Cl − Cl

12. Reaction intermediates


(a) Heterolytic fission
In this fission, the covalent bond is cleaved in such a manner that both the electrons of the bonding pair remain attached with
only one of the two originally bonded atoms resulting in the formation of positive and negative ions.
d+ heterolysis
C Zd − C+ + :Z −
Carbocation
d− heterolysis
C Zd + C :− + Z +

Carbanion

(i) Carbocation
• It is a chemical species in which the trivalent carbon atom bears a positive charge. The carbocations are primary, second-
ary or tertiary depending on the number of carbons attached directly to the positively charged carbon.

Chapter-12.indd 385 7/30/2016 4:14:33 PM


386 Chapter 12 Some Basic Principles in Organic Chemistry

(ii) Carbanion
• It is a chemical species in which a carbon atom bears a formal negative charge and possesses an unshared pair of
electrons.
• A carbanion is generated when carbon is attached to a less electronegative element or by nucleophilic attack on a
carbon–carbon multiple bond.
• The stability order can be explained on the basis of +I (inductive) effect of alkyl groups. The stability also increases in the
presence of electron withdrawing groups. Between the carbanions of an alkane, alkene and alkyne, the stability of the
carbanion having maximum s character is the maximum. This is because the electronegativity of carbon and hence its
ability to carry a negative charge increases with increase in s character.
(b) Homolytic fission
It is the fission of covalent bonds with the production of intermediates possessing unpaired electrons called radicals (or free
radicals). Free radical is generally a planar species in which the carbon atom lies in the centre of the triangle and the three
bonds are directed towards the three corners of planar trigonal geometry. The relative stability can be explained on the basis of
ease of formation. Lower the homolytic bond dissociation enthalpy, greater is the ease of formation of free radicals.
(i) Carbenes and nitrenes
• Carbenes are uncharged, electron deficient molecular species that contain a divalent carbon atom surrounded by a sextet
of electrons.
• Nitrenes are nitrogen analogues of carbenes. These are also uncharged, electron deficient molecular species that contain
a monovalent nitrogen atom surrounded by a sextet of electrons.
• Carbenes and nitrenes exists in two states:
Singlet state: wherein they are carbocation-like in nature with trigonal planar geometry and exhibit electrophilic
character;
Triplet state: diradical-like in nature, linear geometry.

+ +

R′ Y Y
R′ N N
C C Z Z
R R
Singlet carbene Triplet carbene Single nitrene Triplet nitrene

Reactive intermediates and their characteristics

Reactive Hybridization of carbon and


intermediate Charge and nature geometry Order of stability
Carbocation Positive, electron deficient, six elec- sp2 hybridized state, trigonal planar. Tertiary > secondary > primary
trons in outermost orbital > methyl
Carbanion Negative, unshared pair of electrons sp3 hybridized state, tetrahedral Tertiary < secondary < primary
with the unshared electron pair < methyl
occupying the apex of the tetrahe- Alkyne (sp) > alkene (sp2) >
dron. alkane (sp3)
Free radicals+ Unpaired electron sp2 hybridized state with trigonal Tertiary > secondary > primary
planar geometry; odd electron in > methyl > vinyl
unhybridized p orbital or sp3 hybrid-
ized with trigonal pyramidal
geometry; odd electron in
sp3 hybrid orbital.

Tip Energy required for homolysis of covalent bond is supplied either by heating or irradiation with light.

13. Electrophiles, nucleophiles and types of reactions


(a) Electrophiles: These are electron-deficient species (meaning electron-loving) seeking a site of high electron density.
Electrophiles have an incomplete outer electron shell and are very reactive. They could be:
(i) Neutral species in which the central atom has six electrons, for example,

BF3, AlCl3, SO3, FeCl3, SiCl4, CH2, NR

Chapter-12.indd 386 7/30/2016 4:14:34 PM


Summary 387

(ii) Positively charged species, both of which are deficient in electrons for example,
+
H+, NO+, NO+2, R C, R+, Cl+, Br+, I+
O
(iii) Carbocations are also electrophiles because they are electron seeking species. They can fill their valence shell by accepting
a pair of electrons from a nucleophile.
(b) Nucleophiles: Any negative ion or any neutral molecule that has at least one unshared electron pair is called a nucleophile
(meaning nucleus loving).
(i) They are electron-rich species with a tendency to donate a pair of electrons.
(ii) They attack electron-deficient sites and could be neutral or negatively charged.
(iii) Examples of nucleophiles are:
NH3, RNH2, R2NH, R3N, H2O, ROH, ROR (Neutral)

CN− , OH− , X − , RCOO − , HC C − (Negatively charged)

(iv) Carbanions are nucleophiles, they seek a proton or some other positive centre to which they can donate their electron pair
and thereby neutralize their negative charge.
(c) Classification of organic reactions
(i) Substitution reactions: In these reactions, one group is replaced by another. These are characteristic reactions of saturated
compounds such as alkanes and alkyl halides and of aromatic compounds. For example,
CH3Cl + OH− → CH3OH + Cl−
(ii) Elimination reactions: In these reactions, one molecule loses the elements of another small molecule. These reactions are
used for preparing compounds with double and triple bonds.
• An elimination reaction in which both the groups or atoms are removed from the same carbon of the molecule is
called a-elimination reaction. This reaction is mainly given by gem dihalides and gem trihalides having at least one
a-hydrogen.
• An elimination reaction in which functional group (i.e., leaving group) is removed from a-carbon and other group (gener-
ally a hydrogen atom) from the b-carbon is called b-elimination reaction. For example,
H H
b a H H
KOH
H C C H C C
(−HBr)
H H
H Br

(iii) Addition reactions: Reactions in which the components of a species A–B are added to adjacent atoms across a carbon–
carbon multiple bond. An addition reaction is the reverse of an elimination reaction.
HCl + CH2 CH2 → CH3CH2Cl
(iv) Rearrangement reactions: In these reactions, a molecule undergoes a reorganization of its constituent parts.
For example,

H H H3C CH3
Acid
H3C C C C C
Catalyst
C H H3C CH3
H3C CH3
(d) Reaction mechanism:
The mechanism of different ionic reactions can be illustrated by following four characteristic patterns of electron flow.
(i) Nucleophilic attack
Br
− +
Br
Nucleophile Electrophile
(ii) Loss of leaving group
Br

+ Br
+

Chapter-12.indd 387 7/30/2016 4:14:36 PM


388 Chapter 12 Some Basic Principles in Organic Chemistry

(iii) Proton transfers


+ H
O H Cl O

+ Cl

+ H
O −H+ O

− −
O OH O
H

(iv) Rearrangements

H H
+
+

H3C CH3
+ +

SOLVED EXAMPLES
Hybridization, Shapes of Simple Molecules and (c) 4,4-dimethyl-5,5-diethylpentane.
Nomenclature (d) 5,5-diethyl-4,4-dimethylpentane.
(AIEEE 2007)
1. The IUPAC name of the compound shown below is Solution
Cl (a) Naming of the given organic compound is done by follow-
ing a step-wise process:
(i) Selection of longest chain.
(ii) Numbering is done from that end having lowest set of
Br locants for substitutents.
(a) 2-bromo-6-chlorocyclohex-1-ene. (iii) Writing the name by using the below formula:
(b) 6-bromo-2-chlorocyclohexene. Name of substituent with position in alphabetical order
(c) 3-bromo-1-chlorocyclohexene. + Root word + Suffix (nature of functional group)
(d) 1-bromo-3-chlorocyclohexene.
(AIEEE 2006) 6 2
4
7 5 3 1
Solution
(c) Cl 3-ethyl-4, 4-dimethylheptane
1
6 2
3. The correct decreasing order of priority for the functional
5 3 groups of organic compounds in the IUPAC system of nomen-
Br clature is
4
(a) COOH, SO3H, CONH2, CHO
According to the order of precedence of groups the sub- (b) SO3H, COOH, CONH2, CHO
stituents are ordered alphabetically and the position is (c) CHO, COOH, SO3H, CONH2
mentioned by the first carbon of the double bond. (d) CONH2, CHO, SO3H, COOH
(AIEEE 2008)
2. The IUPAC name of the following compound is
Solution
(a) According to the IUPAC system priority order of various
functional group is
(a) 3-ethyl-4, 4-dimethylheptane . COOH > SO3H > CONH2 > CHO
(b) 1,1-diethyl-2,2-dimethylpentane.

Chapter-12.indd 388 7/30/2016 4:14:38 PM


Solved Examples 389

4. The IUPAC name of neopentane is Solution


(a) 2-methylbutane. (b) The IUPAC name of the compound is 3-ethyl-4-methyl
(b) 2,2-dimethylpropane. hexane.
(c) 2-methylpropane. 2 6
(d) 2,2-dimethylbutane. 1
3 4
5
Solution
(b) The structure is of neopentane is
8. Which one of the following does not have sp2 hybridized
CH3
3 2 1 carbon?
H 3C C CH3 (a) Acetone (b) Acetamide
(c) Acetonitrile (d) Acetic acid
CH3
The IUPAC name is 2,2-dimethylpropane. Solution
sp 3 sp
(c) Acetonitrile ( CH3 C N) has no double bonds.
5. In allene (C3H4), the type(s) of hybridization of the carbon
atoms is (are): 9. The general formula CnH2nO2 could be for open chain
(a) sp and sp3 (b) sp2 and sp (a) diketones. (b) carboxylic acids.
(c) only sp 2 (d) sp2 and sp3 (c) diols. (d) dialdehydes.
(JEE Main Online 2014)
Solution
Solution (b) This is a general formula for carboxylic acids. For example,
(b) In allene molecule (C3H4), the central carbon atom is for n = 2, we have CH3COOH
sp hybridized and the terminal carbon atoms are sp2
hybridized. 10. The correct IUPAC name of the following compound is
H p p H
C C
C
s s
H 2 H (a) 1-cyclopropylcyclobutane.
sp sp sp2 (b) 1,1′-dicyclobutane.
6. Which of the following molecules has two sigma (s) and two pi (c) 1-cyclobutane-1-cyclopropane.
(p) bonds? (d) None of these.
(a) C2H4 (b) N2F2 (c) C2H2Cl2 (d) HCN Solution
(JEE Main Online 2014) (a) 1-Cyclopropylcyclobutane
Solution 11. Which of the molecules is not planar?
(d) The number of s and p bonds in the given molecule are: (a) HCONH2 (b) CH2 CH CH CH2
(c) H2C C C CH2 (d) H2C C CH2
H H
(a) C C 5s and 1p Solution
H H (d) The sp2 hybridization ensures the planar structure but
F presence of sp3 hybridized atom makes the structure
(b) N N 3s and 1p three-dimensional.
F Formamide (HCONH2) is planar because delocalization of
lone pair at nitrogen through resonance makes it planar.
Cl Cl
(c) C C 5s and 1p −
H H O O

(d) H C N 2s and 2p C H C H
H N H N+
7. The correct IUPAC name of the following compound is
H H

Similarly, 1,3-butadiene (CH2 CH CH CH2) is also


planar because all carbon atoms are sp2 hybridized and
(a) 4-methyl-3-ethylhexane. resonance make it planar.
(b) 3-ethyl-4-methylhexane. But in the case of adjacent double bonds (H2C C
(c) 3,4-ethylmethylhexane. C CH2 or H2C C CH2), the molecule may be planar or
(d) 4-ethyl-3-methylhexane. non-planar depending on the number of adjacent double
(JEE Main Online 2014) bonds. When even number of such double bonds exist,

Chapter-12.indd 389 7/30/2016 4:14:42 PM


390 Chapter 12 Some Basic Principles in Organic Chemistry

the molecule is non-planar; but if it contains odd number (c) H2N NH2
of adjacent double bonds, the molecule is planar.
H H
12. The shape of a carbanion is
(a) linear. (b) planar. Ph Ph
Plane of symmetry Achiral
(c) pyramidal. (d) tetrahedral.
(d) COOH
Solution
(c) The carbon atom bearing the negative charge in the H 2N H Two identical
groups attached
carbanion is in sp3 hybridized state. It is bonded to three
other atoms and the unshared electron pair occupies H Achiral
the apex of the tetrahedron.Thus, the shape of carboan- 15. The alkene that exhibits geometrical isomerism is
ion is pyramidal. (a) propene. (b) 2-methylpropene.
R (c) 2-butene. (d) 2-methyl-2- butane.
R C− (AIEEE 2009)
R Solution
sp3 hybridized carbon (c) 2-Butene exhibits geometrical isomerism. In rest of the
13. What is the correct IUPAC name for the following structure? compounds, two identical groups are attached to the
doubly bonded carbon atom, so their geometrical isom-
erism is not possible.
H H H CH3
C C C C
H3C CH3 H3C H
(a) 2−Ethylheptane (b) 2−Methylhexane cis trans
(c) 6−Methylhexane (d) 2−Methylpentane 16. The number of stereoisomers possible for a compound of
Solution the molecular formula CH3 CH CH CH(OH) CH3 is
(a) 3 (b) 2 (c) 4 (d) 6
(b) The IUPAC name of the compound is 2−methylhexane.
(AIEEE 2009)
5 3 1 Solution
(c) The compound has one chiral centre. Therefore,
6 4 2
2n = 21 = 2 optically active compounds are possible. Also,
two geometrical isomers (cis and trans) are possible. So,
total of four isomers are possible.
Isomerism – Structural and Stereoisomerism
H H H
14. Which of the following molecules is expected to rotate the CH3 C C
*
C OH
plane of plane-polarized light?
(a) (b) CH3
CHO
SH 17. Out of the following, the alkene that exhibits optical isomer-
HO H
ism is
CH2OH (a) 2-methyl-2-pentene. (b) 3-methyl-2-pentene.
(c) 4-methyl-1-pentene. (d) 3-methyl-1-pentene.
(c) H2N NH2 (d) COOH (AIEEE 2010)

H 2N H Solution
H H
(b) The alkene that has chiral centre exhibits optical iso-
Ph Ph H merism.
(AIEEE 2007) CH3CH2 C CH2CH3 CH3CHCH2CH CH2
Solution
CH3 CH3
(a) The compound has a chiral carbon atom.
Chiral carbon 3-Methyl-2-pentene 4-Methyl-1-pentene
CHO (A carbon attached (No chiral carbon)
to four different
(No chiral carbon)
substituents)
HO * H H CH3CH2CH C CH3

CH2OH H 2C CH C* CH2CH3 CH3


Plane of symmetry
2-Methyl-2-pentene
(b) CH3
(which divides the (No chiral carbon)
molecule into two 3-Methyl-1-pentene
equal halves) (One chiral carbon)
SH Achiral

Chapter-12.indd 390 7/30/2016 4:14:46 PM


Solved Examples 391

18. Which one of the following acids does not exhibit optical (c) CH3 CH2 CH2 C CH3 and
isomerism?
(a) Lactic acid (b) Tartaric acid O
(c) Maleic acid (d) a-Amino acids CH3 CH2 C CH2 CH3
(JEE Main Online 2014)
O
Solution (d) CH3 CH2 C CH2 CH3 and
(c) Except maleic acid all compounds are optically active.
O
CH COOH CH3
CH CH2 CHO
CH COOH CH3
(JEE Main Online 2015)
Since maleic acid has plane of symmetry, so it does not Solution
exhibit optical isomerism. (c) Compounds that differ in the position of the substituent
19. Which of the following compounds will exhibit geometrical or the functional group on the parent carbon chain are
isomerism? known as the positional isomers.
(a) 3-Phenyl-1-butene (b) 2-Phenyl-1-butene O
(c) 1,1-Diphenyl-1-propane (d) 1-Phenyl-2-butene Thus, CH3 CH2 CH2 C CH3 and
(JEE Main 2015)
O
Solution
(d) 1-Phenyl-2-butene CH3 CH2 C CH2 CH3 are positional isomers.

Ph-H2C CH3 Ph-H2C H 22. The number of structural isomers for C6H14 is
C C C C (a) 3 (b) 4 (c) 5 (d) 6
H H H CH3 (JEE Main Online 2015, IIT-JEE 2007)
(cis) (trans)
Solution
20. The optically inactive compound from the following is (c) Since the structural isomers refers to the compounds that
(a) 2-chloropropanal. have same molecular formula but different arrangement
(b) 2-chloropentane. of atoms or groups within the molecule. Based on this,
(c) 2-chlorobutane. following are the structural isomers for C6H14:
(d) 2-chloro-2–methylbutane.
(JEE Main Online 2015) CH3 CH2 CH2 CH2 CH2 CH3
n-Hexane
Solution
(d) Among the given compounds, 2-chloro-2–methylbu- H3C CH CH2 CH2 CH3 H3 C CH2 CH CH2 CH3
tane does not contain any chiral centre hence is optically
CH3 CH3
inactive.
2-Methyl pentane 3-Methyl pentane
Cl
* CH3
CH3 CH CHO CH3 CH2 CH2 CH CH3
* H3C C CH2 CH3 H3 C CH CH CH3
Cl
2-Chloropropanal 2-Chloropentane CH3 CH3 CH3
2,2-Dimethyl butane 2,3-Dimethyl butane
CH3
* 23. Among the following four structures I to IV, it is true that,
CH3 CH2 CH CH3 CH3 CH2 C CH3
CH3 O CH3
Cl Cl
2-Chlorobutane 2-Chloro-2-methylbutane C2H5 CH C3H7 CH3 C CH C 2 H5

21. Which of the following pairs of compounds are positional I II


isomers? H CH3
(a) CH3 CH2 CH2 CH2 CHO +
H C C2H5 CH C 3 H7
O
H
and CH3 CH2 CH2 C CH3
III IV
(b) CH3 CH2 CH2 C CH3 and (a) all four are chiral compounds.
(b) only I and II are chiral compounds.
O
(c) only III is a chiral compound.
CH3 CH CH2 CHO (d) only II and IV are chiral compounds.
CH3

Chapter-12.indd 391 7/30/2016 4:14:49 PM


392 Chapter 12 Some Basic Principles in Organic Chemistry

Solution 28. Which types of isomerism is shown by 2,3-dichlorobutane?


(b) Because in structure I and II, there is asymmetric centre (a) Diastereomers (b) Optical
present, that is, carbon atom having four different sub- (c) Geometric (d) Structural
stituent attached. Solution
24. Which of following compound can show geometrical (b) Optical isomerism is shown by 2,3-dichlorobutane.
isomerism? CH3 CH3 CH3
(a) CH2 CHCl (b) CH3CH2 CH2Cl H Cl H Cl Cl H
(c) CH CCl (d) (Cl)(Br)C CH(l) H Cl Cl H H Cl
Solution
(d) Geometrical isomerism is not observed in a molecule CH3 CH3 CH3
if either of the doubly bounded atoms has two similar
groups. 29. Consider the Newman projection formulas shown below
CH3 CH3 CH3
25. Which of the following will have mesoisomer also? H Cl H Cl Cl H
(a) 2-Chlorobutane
(b) 2-Hydroxypropanoic acid
Cl H H Cl Cl H
(c) 2,3-Dichloropentane
(d) 2,3-Dichlorobutane CH3 CH3 CH3

Solution A B C

(d) 2,3-Dichlorobutane has a plane of symmetry that divides Which of the following statements is the correct?
the molecule into halves that are mirror images of each (a) Compounds A and B are enantiomers.
other. (b) Compounds A and C are identical.
CH3 (c) Compounds B and C are enantiomers.
(d) Compounds A, B and C are optically active.
H Cl
Plane of symmetry Solution
H Cl (c) The Fischer projection of the given Newmann projection
are as drawn as follows.
CH3
CH3 CH3
26. Amongst the following compound, the optically active H Cl
alkane having lowest molecular mass is Cl H
(a) H3C (b) Cl H
CH3 H3C Cl H
CH CH3 CH3
(c) CH3 (d) CH3 A
H3C CH3 CH3
H Cl
C2H5 H3C CH3 H Cl
Cl H
Solution H Cl
(c) Among the given compounds, there is only one optically CH3
CH3
active alkane.
CH3 B
CH3 CH3
H3C *
Cl H
Cl H
C2H5 Chiral centre H Cl
Cl H
Four different substituents attached (chiral carbon)
CH3 CH3
27. Which of the following compound is not chiral? C
(a) 1-Chloropentane From Fischer projection, it is clear that compound A is
(b) 3-Chloro-2-methylpentane a meso compound since it has plane of symmetry, and
(c) 1-Chloro-2-methylpentane therefore, optically inactive. Compounds B and C are mir-
(d) 2-Chloropentane ror images of each other, and hence, are enantiomers.
Solution
30. The stereochemical relationship between the following mol-
(a) 1-Chloropentane is not chiral because it does not have a ecules is
chiral or asymmetric carbon atom.

Chapter-12.indd 392 7/30/2016 4:14:52 PM


Solved Examples 393

I Br 33. Arrange the carbanions, (CH3 )3 C , CCl3 , (CH3 )2 CH, C6H5 CH2
in order of their decreasing stability:
(a) C6H5 CH2 > CCl3 > (CH3 )3 C > (CH3 )2 CH

Br I (b) (CH3 )2 CH > CCl3 > C6H5 CH2 > ( CH3 )3 C


(a) identical. (b) diastereomers.
(c) CCl3 > C6H5 CH2 > (CH3 )2 CH > (CH3 )3 C
(c) enantiomers. (d) constitutional isomers.
Solution (d) (CH3 )3 C > (CH3 )2 CH > C6H5 CH2 > CCl3
(AIEEE 2009)
(a) On 180° rotation, the configuration of both the mole-
cules is identical, that is, 2S, 3S. Hence, the molecules are Solution
identical. (c) Because in CCl3− , Cl− being electronegative stabilizes the
(1) carbanion, in C6H5CH2− , the presence of phenyl group sta-
I (4) Br bilizes it due to resonance and in case of secondary and
H H
(4) (3) tertiary, the order of reactivity is 2° > 3°.
(3) 3 (2) (3) (3) 2 3 4
4 (2) 2 1 2
CH2−
H H CH3 CH3
(4) Br (4) I
(1) (1) −
CCl3 CH3 CH

CH3 C− CH3
2S, 3S 2S, 3S

31. Indicate the relationship of the pair of molecules shown below. Resonance +I Effect
H H 34. The order of stability of the following carbocations is
+
CH2
I I + +
(a) Constitutional isomers (b) Enantiomers H2C CH CH2 ; H3C CH2 CH2 ;
(c) Diastereomers (d) Different molecules
I II III
Solution
(a) II > III > I (b) I > II > III
(c) There is two stereocentres, so disastereomers are not elim- (c) III > I > II (d) III > II > I
inated. The molecules have the same formula and connec- (JEE Main 2013)
tivity so different molecules and constitutional isomers are
eliminated. The two molecules are not mirror images of Solution
each other and not superposable; therefore, enantiomers (c) Stability is directly proportional to extent of delocaliza-
are eliminated. The first molecule is (2R, 3S) and the second tion of positive charge. Thus, order of stability is
is (2R, 3R). So the molecules are disatereomers of each other. +
CH2
Reaction Intermediates
+ +
> CH2 CH CH2 > CH3 CH2 CH2
32. The increasing order of stability of the following free radicals
is
i i i i
(a) (CH3 )2 CH < (CH3 )3 C < (C6H5 )2 CH < (C6H5 )3 C 35. In which of the following pairs A is more stable than B?
i i i i
A B
(b) (C6H5 ) 3C < (C6H5 )2 CH < (CH3 )3 C < (CH3 )2 CH
i i i i (a)
(c) (C6H5 )2 CH < (C6H5 )3 C < (CH3 )3 C < (CH3 )2 CH
i i i i
(b) (CH3 )2 CH < (CH3 )3 C < (C6H5 )3 C < (C6H5 )2 CH
(b)
(AIEEE 2006)
Solution
(a) The relative stability of free radicals is measured through
(c)
the homolytic bond dissociation energies of R H
bonds. Greater is the value of bond dissociation energy
the lesser is the stability. Tertiary C H bond is easier to
(d) Ph3C• (CH3)3C•
break than secondary C H bond. Therefore, the order is
3° > 2° > 1° > methyl.
(JEE Main Online 2014)

Chapter-12.indd 393 7/30/2016 4:14:57 PM


394 Chapter 12 Some Basic Principles in Organic Chemistry

Solution CH3
CH3 H
(d) Stability of radicals can be compared on the basis of +
(b) +
electronic effects and angle strain.
In option (a): B is more stable than A due to resonance.
CH3
CH3
< + +
(d) CH CH3 CH3
A B
In option (b): 3° radical is more stable as compared to 2°. 38. The correct order of stability of the following free radicals is

<

In option (c): Since A has more angle strain than B, hence


I II III IV
B is more stable than A.
(a) III > I > II > IV (b) I > II > IV > III
(c) IV > II > I > III (d) I > III > II > IV
<
Solution
In option (d): A is more stable due to resonance. (a) The free radicals are electron-deficient species whose
CH3 stability is enhanced by electron donating groups.
Ph C Ph > CH3 C Therefore in (III), allylic free radical is most stable since
electron density can be transferred by both inductive as
Ph CH3 well as resonance effects. As (I), (II) and (IV) are 3°, 2° and
1° free radicals, respectively, the correct order of stability
36. Due to the presence of an unpaired electron, free radicals are is III > I > II > IV.
(a) chemically reactive. (b) chemically inactive.
(c) anions. (d) cations. 39. Which of the following is the most stable carbocation?
OMe OMe
Solution
(a) Free radicals are highly reactive due to the presence of (a) + (b)
unpaired electron. +

R H → R• + H• (where R is an alkyl group)


OMe
37. Which of the following carbocations will not rearrange? OMe
+
CH2 CH3 (c) + (d) +

(a) (b) +
Solution
(a) A group of organic species having a positively charged
CH3 carbon atom bearing only six electrons around the car-
CH3
+ + bon are called carbocations. The stability order of carbo-
(c) (d) CH CH3
CH3 cations is 3° > 2° > 1°.
From the given structures, compound in option (a) is a
Solution tertiary carbocation, which is resonance stabilized and
(c) The rearrangement of a carbocation is possible only if it one of its resonance structure has an additional covalent
can form a more stable carbocation. Sometimes, it can bond with complete octet of all the atoms
also release angle strain present in relatively smaller or
bigger rings. Hence, (a), (b) and (d) rearrange as shown +
OMe OMe
below. The compound in (c) cannot rearrange because
formation of more stable carbocation is not possible by
+
shifting alkyl or halide group.
+ CH3
H CH2
+
(a)
Quick Tip From the stability order of a carbocation, you can
easily find the correct option (a).

Chapter-12.indd 394 7/30/2016 4:15:01 PM


Solved Examples 395

40. Which of the following pairs contains identical compounds? 42. Which one of the following substituents at para-position is
CHO CHO O−
most effective in stabilizing the phenoxide ion?
H OH HO H
(a) and
H OH H OH
(a) CH3 (b) OCH3
CH2OH CH2OH (c) COCH3 (d) CH2OH
CHO CHO (JEE Main 2014)
Solution
HO H H OH
and (c) The substituent COCH3 is electron withdrawing due to
(b) H OH HO H (−I) and (−M) effect and increases stability of phenoxide
ion.
CH2OH CH2OH
O−
CHO CHO
H OH HO H
(c) and
H OH HO H
C
CH2OH CH2OH CH3 O
O
CH2OH CHO
Due to extended conjugation C CH3 group incre-
H OH HO H
and ases stability of the phenoxide ion.
(d) H OH HO H
43. In the anion HCOO−, the two carbon–oxygen bonds are found
CHO CH2OH to be of equal length. What is the reason for it?
(a) Electronic orbitals of carbon atom are hybridized.
Solution (b) The C O bond is weaker than the C O bond.
(c) The anion HCOO– has two resonating structures.
(d) In option (a) – diastereomers.
(d) The anion is obtained by removal of a proton from the
Option (b) – mirror images, so enantiomers.
acid molecule.
Option (c) – mirror images, so enantiomers.
Option (d) – rotating anyone compound by 180o in plane Solution
of paper produces second structure, so, both the struc- (c) The anion HCOO− can be represented as following
tures are identical. resonating structures.
Electronic Displacement in a Covalent Bond O O−

41. The strongest acid amongst the following compounds is H C O− H C O


(a) HCOOH
44. The order of decreasing basicity of following anions is
(b) CH3CH2CH(Cl)COOH
(c) ClCH2CH2CH2COOH O
(d) CH3COOH
(AIEEE 2011) CH3C O− CH3 O−

Solution
I II
(b) Cl– being an electron withdrawing group withdraws electron
density, thereby facilitating the release of proton. As the dis-
tance between Cl− and COOH increases, the dispersal of neg-
CH3O O− Br O−
ative charge of the corresponding carboxylate ion becomes
less pronounced, and hence acidic strength decreases.
The pKa values of the compounds are: III IV

Compound pKa (a) II > III > IV > I (b) I > IV > II > III
(c) III > II > IV > I (d) III > II > I > IV
HCOOH 3.75
Solution
CH3 CH2 CH COOH
2.86 (c) All given anions are basic in nature because they contain
Cl negative charge. Now this strength is enhanced by elec-
tron donating groups and decreases by electron with-
CH2 CH2 CH2 COOH drawing groups.
4.52
Cl Since COCH3 and Br groups are electron withdrawing,
they decrease the basic strength. CH3 and OCH3 are elec-
CH3COOH 4.76 tron donating groups, so they increase the basic strength.

Chapter-12.indd 395 7/30/2016 4:15:04 PM


396 Chapter 12 Some Basic Principles in Organic Chemistry

finally attached, the effect is called +E effect. For exam-


Quick Tip/Alternate Solution The basic strength of given ple, addition of acids to alkenes.
anions is reverse of the acidic strength of their conjugate acids.
The acidic strength depends on ease of deprotonation of com-
+
pound. A compound will easily deprotonate if its anion is easily C C + H+ C C
stabilized. Now these anions can be stabilized by electron with-
drawing groups and destabilized by electron donating groups. H

So, the order of acidic strength of their conjugate acids is I > IV > 47. The heterolysis of O forms a
II > III and order of basic strength of anions is III > II > IV > I. (a) carbanion (b) carbocation
(c) free radical (d) cleavage not possible
45. Consider the structures given below, the pair which repre-
sents the resonance structures of each other is Solution
N O N OH (a) The heterolysis takes place as follows:
(a) and − Heterolysis
N N OH + O O + H2O

H H
H Carbanion
N O N OH
(b) and
N N Electrophiles, Nucleophiles and Type of Organic
Reactions

N O N O
(c) 48. The lowest nucleophilicity among the nucleophiles
and
N N
+ (a) CH3 C O O− (b) CH3O−
N OH N OH O
O
(d) and
N N (c) CN− (d) H3C S O−
Solution O
(c) For two structures to be in resonance only movement of
electrons takes place but there should be no change in Solution
the skeleton of the molecule. (d)
The structures given in option (c) are resonance hybrids O
of each other since they satisfy both the conditions.
CN− > CH 3 O− > CH3 C O O− > H3 C S O−

N O N O O O

N N 49. Which of the following reactions involves a nucleophile?


+
(I) CH3COOH + OH− → CH3COO − + H2O

(II) CH3COCH3 + CN− → (CH3 )2 C(CN) + (OH)



N + O +
(III) C6H5 + CH3 C O → C6H5COCH3
N (a) I and II (b) I and III (c) III only (d) II and III

However, in structures given in options (a), (b) and (d), Solution


the position of hydrogen atom changes, so the mole- (a) The reactions I and II involve nucleophiles, OH− being
cule undergoes a change in the skeleton of the structure. the nucleophile in (I) and CN− being the ambident
Hence, these pairs do not represent resonance structures. nucleophile in (II).
In option (c), there is only delocalization of electrons in
both structures, and no change in the skeleton of structure 50. The reaction between electrophile and nucleophile involves
is observed; so both represent the resonance structures. (a) an ionic bond.
(b) a covalent bond.
46. Addition of acids to alkenes is an example of (c) a coordinate covalent bond.
(a) +I effect (b) −I effect (c) +E effect (d) −E effect (d) None of these.
Solution Solution
(c) If the electrons of the p bond are transferred to that (c) A reaction between a nucleophile and electrophile
atom of the double bond to which the reagent gets involves formation of a coordinate covalent bond.

Chapter-12.indd 396 7/30/2016 4:15:08 PM


Advanced Level Problems 397

H+ − Cl− Solution
51. The type of reaction CHCl3 
→ : CCl3  → : CCl2
is Dichloro carbon (a) Two groups are removed from the same carbon. Thus,
(a) a-elimination (b) b-elimination this is a a-elimination reaction.
(c) g-elimination (d) elimination

ADVANCED LEVEL PROBLEMS


1. Among the following, the least stable resonance structure is Solution
(a) +
+ O (b) − + O (d)
− N + N CH3 CH3
+ +
O O
− C CH2 C CH2
− (I) +
H3C O CH3 H 3C O CH3
(c) − + O (d) −
+ O
+ N + N + CH3
O (II) H3C CH CH2 HC
− O
− CH3
(IIT-JEE 2007) (III)
+ CH3 + CH3
Solution
H3C CH O HC H3C CH O HC
(a) The least stable structure is (a) as the same charges (posi- CH3 CH3
tive charges) are present at nearest positions.
+ CH3
2. Statement 1: Molecules that are not superimposable on their (IV) H2C CH2 CH2 CH
mirror images are chiral. CH3
Statement 2: All chiral molecules have chiral centres. Structure I stabilizes by resonance and has six a-hydrogen
(a) Statement 1 is True, Statement 2 is True; Statement 2 is a atoms.
correct explanation for Statement 1. Structure II has only five a-hydrogen atoms for stabiliza-
(b) Statement 1 is True, Statement 2 is True, Statement 2 is tion through hyperconjugation.
NOT a correct explanation for Statement 1. Structure III stabilizes by resonance and has only three
(c) Statement 1 is True, Statement 2 is False. a-hydrogen atoms.
(d) Statement 1 is False, Statement 2 is True Structure IV has only two a-hydrogen atoms for stabiliza-
(IIT-JEE 2007) tion through hyperconjugation.
Solution So, the order of stability is I > III > II > IV.
(c) A chiral molecule is defined as one that is not superimposa 5. The correct statement(s) concerning the structures E, F and G
ble on its mirror image. Example, enantiomers. For a mole- is (are)
cule to be chiral, one of the following conditions should be O OH CH3
satisfy: H3C H3C H3 C
(i) Presence of chiral centre
(ii) Absence of symmetry elements in the molecule (plane
H3C CH3 H 3C CH3 H3C OH
of symmetry, axis of symmetry, etc.,)
(E) (F) (G)
3. Hyperconjugation involves overlap of the following orbitals
(a) E, F and G are resonance structures.
(a) s–s (b) s–p
(b) E, F and E, G are tautomers.
(c) p–p (d) p–p
(c) F and G are geometrical isomers.
(IIT-JEE 2008)
(d) F and G are diastereomers.
Solution (IIT-JEE 2008)
(b) Hyperconjugation involves the overlapping of s–p Solution
orbitals.
(b), (c), (d) E, F and E, G represent keto–enol tautomerism,
4. The correct stability order of the following species is while F and G exhibit geometrical isomerism.
F and G are diastereomers because both the structures are not
mirror images of each other.
+ + +
O O + 6. The correct statement(s) about the compound given below is
I II III IV (are)
Cl H
(a) (II) > (IV) > (I) > (III) (b) (I) > (II) > (III) > (IV)
(c) (II) > (I) > (IV) > (III) (d) (I) > (III) > (II) > (IV) CH3
H3C
(IIT-JEE 2008) Cl H

Chapter-12.indd 397 7/30/2016 4:15:11 PM


398 Chapter 12 Some Basic Principles in Organic Chemistry

(a) The compound is optically active. Solution


(b) The compound possesses centre of symmetry. (b) The IUPAC name of the compound is 2-bromo-5-hydroxy-
(c) The compound possesses plane of symmetry. benzonitrile. CN has the highest priority.
(d) The compound possesses axis of symmetry.
(IIT-JEE 2008) 10. The total number of contributing structures showing hyper-
conjugation (involving C H bonds) for the following carbo-
Solution cation is ___.
(a), (d) The compound is optically active as it has two chiral H3C + CH2CH3
centres. It also possess centre of symmetry.
7. In the following carbocation, H/CH3 that is most likely to
migrate to the positively charged carbon is
H H (IIT-JEE 2011)
1 2 + 4 5
H 3C C C C CH3 Solution
3
(6) Since there are six a-hydrogens. So, the contributing
OH H CH3
hyperconjugating structures = 6
(a) CH3 at C4 (b) H at C4 (c) CH3 at C2 (d) H at C2
H H H
(IIT-JEE 2009)
+C + CH2CH3
Solution H C CH2CH3 H C CH2CH3
C
(d) Here the driving force is conjugation with oxygen. H H H+ H+
H
H H H
1 2 +3 4 5 H− shift from C2 to C3 1 +2 3 4 5
H 3C C C C CH3 H3 C C C CH CH3
I II
OH H CH3 OH H CH3

8. The correct stability order of the following resonance struc-


tures is H
H+ H+
H 3C C CH3 H+ C CH2CH3 H C CH2CH3
+ − + − − + − +
H2C N N H2C N N H2C N N H2C N N H H H
I II III IV

(a) I > II > IV > III (b) I > III > II > IV
(c) II > I > III > IV (d) III > I > IV > II V IV III
(IIT-JEE 2009)
Solution
(b) Greater the number of p bonds, greater is the stability. H
Out of structures (II) and (IV), (II) is more stable as negative
charge is on more electronegative atom, that is, nitrogen. H 3C C CH3
Thus, the correct order is I > III > II > IV. H+

9. The IUPAC name of the following compound is


OH
VI

11. In allene (C3H4), the type(s)of hybridization of the carbon


CN atoms is (are)
(a) sp and sp3 (b) sp and sp2
Br
(c) only sp2 (d) sp2 and sp3
(IIT-JEE 2012)
(a) 4-bromo-3-cyanophenol.
Solution
(b) 2-bromo-5-hydroxybenzonitrile.
(c) 2-cyano-4-hydroxybromobenzene. (b) The structure of allene is CH2 C CH2 both the terminal
(d) 6-bromo-3-hydroxybenzonitrile. carbons are sp2 hybridized while the central carbon is sp
(IIT-JEE 2009) hybridized.

Chapter-12.indd 398 7/30/2016 4:15:13 PM


Advanced Level Problems 399

12. Which of the given statement(s) about N, O, P and Q with nine a-hydrogens and hence nine hyperconjugation
respect to M is (are) correct? structures.
H
HO C1
HO + H+
H H H C C CH3 H C C CH3
H
OH
HO C1 H CH3 HO H CH3 H CH3
OH H
H

CH3 C1 CH3 H H
M N O
H
C C CH3 H+
C C CH3
H+
CH3 CH3 CH3 H CH3
(Plus six more such structures from the other methyl groups.)
H OH HO H

The hyperconjugation for 2-butene involves s → p∗ electron


delocalization. It has six a-hydrogens and hence six hyper-
HO H HO H conjugation structures.
C1 C1 H H H H
P Q H+ −
H C C C C H H C CH CH2 CH3
(a) M and N are non-mirror image stereoisomers .
(b) M and O are identical. H H H
(c) M and P are enantiomers.
(d) None of these.
H H
(IIT-JEE 2012)
− −
Solution H C CH CH2 CH3 H+ C CH CH2 CH3
H+
(a), (b), (c) The Fischer projections for the given compounds
H
can be represented as follows:
(Similarly three more structures from the other methyl groups.)

Cl Cl Cl Cl
14. The total number of stereoisomers that can exist for M is
HO H HO H HO H H OH
_____.
HO H H OH HO H H OH HC CH3
3

CH3 CH3 CH3 CH3


M N O P

M and N are diastereomers.


M and O are identical. H3C O
M and P are enantiomers (non-superimposable mirror M (JEE Advanced 2015)
images).
Solution
M and Q are diastereomers.
(2) Total number of stereoisomers = 2n = 22 = 4, where n is the
13. The hyperconjugative stabilities of tert-butyl cation and number of chiral (stereogenic) carbons.
2-butene, respectively, are due to But in bridge/bicyclic compounds, the number of stereoi-
(a) s → p (empty) and s → p∗ electron delocalizations. somers is equal to the number of chiral centres because
(b) s → s∗ and s → p electron delocalizations. no carbon centres rotation possible. Therefore, two stere-
(c) s → p (filled) and s →p electron delocalizations. oisomers would be possible for the given compound.
(d) p (filled) → s∗ and s → p∗ electron delocalizations.
(JEE Advanced 2013) 15. The IUPAC name of the following compound is

Solution
(a) The hyperconjugation for tert-butyl carbocation involves
s → p (empty) electron delocalization. This is because
carbon having + charge has one vacant p-orbital. It has

Chapter-12.indd 399 7/30/2016 4:15:15 PM


400 Chapter 12 Some Basic Principles in Organic Chemistry

(a) bis-cyclobutylcyclobutane. (b) cyclododecane. Solution


(c) 1,1′,2′,1″-tercyclobutane. (d) None of these. (d) After deprotonation, the dianion obtained is stabilized
Solution by resonance and four membered ring becomes aro-
matic due to which it is more acidic than acetic acid.
(c) 1,1′,2′,1″-tercyclobutane
Thus, all four resonance structures are equally stable,
3 2 4′ 3′ which implies that all C C and C O bonds are equal.
In general, first deprotonation is easier than the second,
1″ 2″ and pKa1 < pKa2.
4 1 1′ 2′ 18. Select the correct statement(s) given below:
(I) Both Cl group as well as OCOCH3 group deactivate
4″ 3″ benzene ring towards electrophilic aromatic substitution
reactions. −
16. Arrange the following carbon acids in the increasing order of + O
their pKa values? (II) N is more stable resonance

O O O O O O O O
NO2 + O−
H OEt form than + N
I II III IV O−
(a) IV < II < I < III (b) III < I < II < IV (III) N and
(c) IV < III < II < I (d) III < IV < I < II
Solution NH are functional isomerism.
(a) Each of these compounds contains methylene ( CH2 ) CH3
group clamped between two electron withdrawing (IV) The combustion enthalpy of CH3C CH2 is less than
groups. So, after deprotonation from methylene group, H 3C H
negative charge can be delocalized and conjugate base
can be stabilized. If stronger electron withdrawing group C C
is attached with this methylene group, it will give more sta- H CH3.
ble conjugate base and is produced from a stronger acid.
(a) I and II only (b) II and III only
H+ (c) II, III and IV only (d) All four are correct.
− +
O O O OH O OH
Solution
OEt OEt OEt (c) (I) In the case of Cl, (−I) effect is the dominating fac-
H H H H tor, so it deactivates the benzene ring weakly. But
OCOCH3 activates the benzene ring because lone pair
Since compound (IV) contains NO2 group which is most on oxygen atom increases electron density on the
electron withdrawing through −M effect and − I effect, its benzene ring. −
+ O
presence makes the compound most acidic. Compound
(II) N is more stable reso-
(III) contains COOEt group which is least electron with-
drawing. The OEt group shows mesomeric effect (+M) O
nance form because it contain more chemical bonds
and inductive effect (− I) but +M effect is more dominant
than − I effect. In the case of compound (I) and com- + O−
in comparison + N
pound (II), group COCH3 and CHO are more electron
withdrawing than COOEt. Therefore acidic order is IV O−
> II > I > III. Since pKa = − log Ka, so increasing order is
(III) N and
IV < II < I < III.
17. Consider the compound shown in the figure. NH
O OH are functional isomerism because chemical proper-
ties of these two amines are different.
CH3
O OH (IV) The combustion enthalpy of is less than
CH3C CH2
Which of the following statements is not correct? H 3C H
(a) Both protons are more acidic than acetic acid. CH3
C C because is a more stable
(b) All C C bonds in the dianion, C 4 O2−
4 are equal. CH3C CH2
H CH3
(c) All C O bonds in the dianion, C 4 O2−
4 are equal.
(d) The pKa1 and pKa2 values are equal for both alkene in comparison of trans alkene.
deprotonations.

Chapter-12.indd 400 7/30/2016 4:15:19 PM


Practice Exercise 401

PRACTICE EXERCISE
Level I 8. How many meso forms are possible for the following organic
compound?
Single Correct Choice Type CH3 OH Br OH CH3
1. How many chiral centres are there in the following compound? HOOC COOH
O H H H H H
(a) Zero (b) 3 (c) 2 (d) 4
O
9. The compound that gives the most stable carbocation ion on
dehydration is
(a) CH3CH(CH3)CH2OH (b) (CH3)3COH
(c) CH2 CHCH2CH2OH (d) CH3CHOHCH2CH3

(a) 4 (b) 5 (c) 6 (d) 7 10. Which of the following statements would be true about this
compound?
2. Which of the following has a tendency to undergo
NO2
disproportionation? 5
(a) Tertiary carbocation (b) Tertiary carbanion
(c) Tertiary free radical (d) All of these 1 3
3. Which of the following carbon will give fastest deprotonation? NO2 NO2
Br
4 2
3 1 H
5
(a) All three C N bonds are of the same length.
O (b) C1 N and C3 N bonds are of the same length but
N shorter than C5 N bond.
(c) C1 N and C3 N bonds are of the same length but longer
(a) 4 (b) 5 (c) 2 (d) 3
than C5 N bond.
4. How many geometrical isomers are possible for the following (d) C1 N and C3 N bonds are of different length but both
compound? are longer than C5 N bond.

COOH 11. Which of following compound can show geometrical


H3C isomerism?
Br Cl CH3
(a) 6 (b) 8 (c) 10 (d) 4
(a) C C (b) C
5. Identify the nature of the product in the following reaction. l Cl CH 3

a b F Et
syn addition
C C (c) C C (d) None of these
Cl Et
b a
(a) Dextrorotatory (b) Laevorotatory 12. Which of the following is optically inactive?
(c) Racemic mixture (d) Meso form H CH3
6. How are the following compounds related?
(a) O (b)
H3C Cl CH3
Cl CH3
Cl
CH3 CH3
H Cl H
(c) (d)
(a) Diastereomers (b) Enantiomers CH3 CH3
(c) Meso compounds (d) Identical OH

7. The number of 3° carbon atoms in 2,2,4,4-tetramethylpentane 13. Which of the following has minimum enol content?
is (a) Phenol (b) Acetaldehyde
(a) 1 (b) 2 (c) 3 (d) 4 (c) Acetoacetic ester (d) Pentane-2,4-dione

Chapter-12.indd 401 7/30/2016 4:15:22 PM


402 Chapter 12 Some Basic Principles in Organic Chemistry

14. How many ethers are possible for the formula C5H12O includ- (III) The symbol D indicates that hydrogen atom lies left to
ing stereoisomers? the chiral centre in the Fischer projection diagram.
(a) 5 (b) 6 (c) 7 (d) 8 (IV) The symbol D indicates that hydrogen atom lies right to
the a chiral centre in the Fischer projection diagram.
15. How many degrees of unsaturation are there in the following
(a) I only (b) I and IV (c) I and III (d) II only
compound?
OH 22. What type of isomerism is shown by following pairs?
O O
O
CH3 CH2 C O CH3 and CH3 C O C2H5
OH (a) Position isomers (b) Metamers
(c) Functional isomers (d) All of these
(a) 6 (b) 7 (c) 10 (d) 11 23. Among the following molecules, the correct order of C C
bond length is
16. If degree of unsaturation is three then the compound shows (a) C2H6 > C6H6 > C2H2
(a) one triple bond and one double bond. (b) C2H6 > C6H6 > C2H4 > C2H2 (C6H6 is benzene)
(b) one double bond and two rings. (c) C2H4 > C2H6 > C2H2 > C6H6
(c) one ring and two double bonds (d) C2H6 > C2H4 > C2H2 > C6H6
(d) All of these are correct.
24. Which of the following structures belongs to R-configuration?
17. (I) CH2 CH CH CH CH CH2
+ + F H 3C
(II) CH2 CH CH CH CH CH2
+ + (a) Br (b) H
(III) CH2 CH CH CH CH2
H 2N Cl H5C2 COOH
Among these three canonical structures (though more are
possible) what would be their relative contribution in the H 2N
hybrid? COOEt
(c) (d) None of these
(a) I > II > III (b) III > II > I
(c) I > III > II (d) III > I > II H Br

18. Which is not the position isomer of following compound? 25. Most stable carbocation among the given examples is

CH2Cl CH3 +
(a) + (b)

CH3 +
(a) (b) (c) CH2 (d) +CH
3
CH2Cl Cl

26. What is the absolute configuration when the following com-


CH3 CH3
pound is rotated through 270° clockwise?
(c) (d)
H
Cl Cl
Cl CH3
19. Hyperconjugation is most useful for stabilizing which of the
following carbocations? NH2
(a) Neopentyl (b) tert-Butyl
(c) Isopropyl (d) Methyl (a) R (b) S (c) D (d) L

20. Heterolytic fission of a covalent bond in organic molecules 27. Select the most stable carbocation from among the
gives following.
(a) free radicals. (b) cations and anions. (a) +
(c) only cations. (d) only anions.
+
21. Which of the following statement(s) is/are not correct for (b)
D(+)glyceraldehyde? +
(I) The symbol D indicates the dextrorotatory nature of the (c)
compound. +
(II) The sign (+) indicates the dextrorotatory nature of the (d)
compound.

Chapter-12.indd 402 7/30/2016 4:15:27 PM


Practice Exercise 403

28. These are three canonical structures of naphthalene. Examine 34. Which of the following ions is most stable?
them and find correct statement among the following: + +
(a) CH3CH2 CH2 (b) CH3 CHCH2CH3
1 1 1
2 2 2
+ CH3
3 3 3 CH3 C CH3 +
(c) (d) CH C CH2
3
4 4 4 CH3
CH3
(a) All C C bonds are of same length.
(b) C1 C2 bond is shorter than C2 C3 bond. 35. How many cyclic and acyclic isomers can be made by the for-
(c) C1 C2 bond is longer than C2 C3 bond. mula C3H6O?
(d) None of these. (a) 10 (b) 7 (c) 5 (d) 4
29. Five alcohols can be drawn for formula C4H10O. How many of 36. Which of the following is the least stable carbocation?
these are optically active?
+
(a) 1 (b) 2 (c) 3 (d) 4 CH2
+
30. Which of the following carbonyl compounds will form car- (a) (b) C
banion most readily?
H
H3C O O
(a) (b) H3C
+
H3C CH3 CH3 (c) C (d)
+
C
O O
H3C H
(c) (d) H3C 37. HNCO in isocyanic acid exists as following resonating
CH3 CH3 structures

H N C O H N C O+
31. Which of the following can be applied to explain relative
I II
order of stability of carbocations?
(a) Resonance (b) Inductive effect
(c) Hyperconjugation (d) All of these + −
H N C O
+ III
32. If the carbocation CH2 rearranges to gain The correct order of stability is
H2C (a) I > III > II (b) I > II > III (c) II > III > I (d) II > I > III
stability, it will rearrange to 38. What kind of isomerism is possible in the following organic
CH3 +
compound?
(a) (b)
H2C +
+ H3C CH3
+ CH2
(c) CH2 (d) CH3
H3C
(a) Optical (b) Geometrical
33. Which of these shows incorrect representation of inductive (c) Both (a) and (b) (d) None of these
effect?
CH3 39. Which of the following is an example of elimination reaction?
O (a) Chlorination of methane.
(b) Dehydration of ethanol.
C OH (c) Nitration of benzene.
(a) (b) (d) Hydroxylation of ethylene.
40. How many enantiomeric pairs are obtained by monochlorin-
ation of 2,3-dimethylbutane?
(a) One (b) Four
CH3 (c) Two (d) Three
O 41. The total number of contributing structures for hyperconju-

CH3 C CH2 CH3 gation in CH3CH CH2 is
C O
(c) (d) (a) 2 (b) 3 (c) 4 (d) 5
O

Chapter-12.indd 403 7/30/2016 4:15:33 PM


404 Chapter 12 Some Basic Principles in Organic Chemistry

42. Which of the following structures represents 2-bromo- 48. For the formula C5H12O, eight alcohols can be drawn. How
6-isobutyl-4-methyldec-3-en-1-ol? many tertiary alcohols of these are optically active?
Br (a) 1 (b) 2
(c) 3 (d) None of these

(a) 49. Electrophilic reagents are


(a) electron pair donors. (b) Lewis acids.
(c) odd electron molecules. (d) None of these.
OH
OH 50. The inductive effect
(a) decreases with increase of distance.
(b) increases with increase of distance.
(c) indicates the transfer of pair of electron from less elec-
(b) tronegative atom to more electronegative atom in a
Br molecule.
(d) shows the transfer of lone pair of electrons.
OH
51. Nucleophiles are
(a) Lewis bases. (b) Lewis acids.
(c) (c) cations. (d) radicals.
52. p-Chlorophenol is a stronger acid than phenol because
Br (a) Cl is less electronegative than oxygen atom.
OH (b) of the −I effect of a halogen, which is greater than its +R
effect.
(c) of +R effect of Cl, which is stronger than its effect −I
(d) effect.
Br (d) of +R effect of Cl.
53. Which will not show tautomerism?
43. The decreasing order of electronegativity of the hybrid orbitals is (a) CH3CN (b) CH3COCH2COCH3
O
(a) sp > sp2 > sp3. (b) sp3 > sp2 > sp.
2 3 (c) CH3CH2N (d) CCl3CHO
(c) sp > sp > sp. (d) sp2 > sp > sp3. O
44. The IUPAC name of the following compound is 54. In hyperconjugation of alkene there is an overlap between
CO NH (a) p and p orbitals. (b) 2p orbitals.
(c) d and p orbitals. (d) s and p orbitals.
COOH
55. Which one of the following molecules exists as a resonance
NO2 hybrid?
(a) 4-(benzoylamino)-2-nitrobenzoic acid. (a) (b) CH2 CH CH2 CHO
(b) 4-(benzamide)-6-nitrobenzoic acid.
(c) 4-(benzoylamino)-6-nitrobenzoic acid. (c) CH2 CH CH2OH (d) CH2 CH NO2
(d) 4-(benzamide)-6-nitrobenzoic acid.
45. Identify the nature of the product in the below reaction 56. How many resonance structures are possible for CO2−
3 ?
(a) 4 (b) 3 (c) 5 (d) 2
a b
anti addition 57. The most stable species in the following sets of ions is
C C + + + +
(a) CH3 (b) CH2Br (c) CHBr2 (d) CBr3
b a
58. How many chiral carbons are there in the following
(a) Dextrorotatory (b) Laevorotatory
compound?
(c) Racemic mixture (d) Meso form
H 3C
46. The total number of benzene derivatives with the molecular
CH3
formula C6H3Cl3 is CH3
(a) 2 (b) 3 (c) 4 (d) 5 CH3
47. What is the simplest alkane, that is, the one with the smallest H3C
molecular weight, which possesses primary, secondary and
tertiary carbon atoms?
(a) 2-Methylpropane (b) 2-Methylbutane HO
(c) 2-Methylpentane (d) 3-Methylpentane (a) 6 (b) 7 (c) 8 (d) 9

Chapter-12.indd 404 7/30/2016 4:15:37 PM


Practice Exercise 405

59. Removal of a hydride ion from methane molecule gives 69. Decreasing order of C C bond length is
(a) methyl radical (b) carbocation (I) C2H4 (II) C2H2 (III) C6H6 (IV) C6H12
(c) carbanion (d) methyl group (a) IV > III > I > II (b) I > II > IV > III
(c) II > I > IV > III (d) IV> I > III > II
60. IUPAC name of 4-isopropyl-m-xylene is
(a) 1-isopropyl-2,4-dimethylbenzene. 70. Name the family to which each belong:
(b) 4-isopropyl-m-xylene. O
(c) 4-isopropyl-3,5-dimethylbenzene. CH3CH2CH2NH2 HOCH2CH2CH3
CH3CH2CH2COH
(d) 4-isopropyl-3,5-dimethylbenzene.
(a) Acid, alkane, alcohol (b) Alkene, alcohol, ester
61. The number of functional groups in aspartame is (c) Acid, amine, alcohol (d) Amine, acid, alcohol
O
71. The structural formula of cyclohexyl alcohol is
O OH
N OH
NH2 H OCH3 (a) (b)
OH
O
CH2OH
(a) 4 (b) 6 (c) 5 (d) 7
(c) (d) None of these
62. Which type of isomerism is not possible in alkenes?
(a) Chain isomerism (b) Geometrical isomerism
(c) Metamerism (d) Positional isomerism 72. Name the following substance.
COOH
63. The reactive intermediate formed when CH3 Cu undergoes
Cl
heterolytic cleavage is
(a) carbocation. (b) carbanion.
(c) free radical. (d) none of these.
64. The IUPAC name for the compound is CH2CH3
CH2CH3 (a) 1-Ethyl-3-chlorobenz-4-oic acid
CH3 CH2 N CH2CH3 (b) 6-Chloro-4-ethyl benzoic acid
(c) 4-Ethyl-2-chlorobenzoic acid
(a) triethylamine. (b) ethyltriamine. (d) 2-Chloro-4-ethyl benzoic acid
(c) N,N-diethylethanamine. (d) None of these. 73. Which of the following is the correct bond line formula for the
65. Resonance effect involves given compound? (CH3)2C CHCH2CH3?
(a) delocalization of p electrons along a conjugated system. (a) (b)
(b) delocalization of n-electrons along a conjugated system.
(c) delocalization of s electrons into an adjacent p bond.
(d) All of these are correct.
(c) (d)
66. The compounds with structural formula R C N and R N C
are 74. The structure of the functional groups corresponding to acid,
(a) position isomers. (b) functional isomers. ester and ether are
(c) metamers. (d) chain isomers. (a) COR, COOR, COOH (b) O , COR, COOR
(c) COH, COOH, COOR (d) COOH, COOR, O
67. Identify the number of sp3 hybridized carbon atoms in the
given compounds (CH3)C CHC(CH3)2
(a) 1 (b) 2 (c) 3 (d) 4 75. The heterolysis of forms a
68. The set which does not have structural isomers is Br
(a) carbanion. (b) carbocation.
(a) CH3CH2OH and CH3OCH3 (c) free radical. (d) cleavage not possible.
O O 76. Which of the following keto compounds will give the least
(b)
stable enol structure?
CH3CCH3 and CH3CH2CH
O O
O O
(c) (a) (b)
CH3COCH3 and CH3CH2COH O
O
O
OH O
(d) O
CH3CHCH2CH3 and CH3CCH2CH3 (c) (d)
O

Chapter-12.indd 405 7/30/2016 4:15:42 PM


406 Chapter 12 Some Basic Principles in Organic Chemistry

77. The functional isomer of propan-1-ol is (c) It is difficult to replace chlorine from chlorobenzene than
(a) ethoxypropane. (b) dipropyl ether. from benzoyl chloride.
(c) methoxyethane. (d) methoxymethane. (d) Chlorobenzene on further chlorination gives
m-dichlorobenzene.
78. How many s and p bonds are present in the following mol-
ecule: CH2 CH CH2 C CH? 86. Which of the following represent the resonating structure of
(a) 7s bonds and 6p bond (b) 8s bonds and 5p bond
(c) 10s bonds and 3p bond (d) 9s bonds and 4p bond
79. The IUPAC name of the following compound is
+
CH3
(a) (b)

− + + −
(c) (d)
(a) 3-methyl cyclohexane.
(b) 1-methylcyclohex-2-ene. 87. Which of the following contain same number of linearly
(c) 6-methyl cyclohexane. arranged carbon atoms in their respective molecules?
(d) 1-methylcyclohex-5-ene. (a) CH2 CH C CH (b) HC C C CH
(c) H2C CH CH CH2 (d) CH3 C CH3
80. What is the shape of HCHO?
(a) Trigonal planar (b) Tetrahedral 88. Isovalent hyperconjugation explains the stability of
(c) Linear (d) Pyramidal (a) carbocations. (b) carbanions.
(c) free radicals. (d) carbenes.
81. Write formulas for the first three members of the homolo-
gous series beginning with H CH CH2 89. The greater stability of tert-butyl carbocation than methyl
(a) H CH CH2, CH3 CH2 CH CH2, CH3 CH CH CH3 carbocation can be explained on the basis of
(b) H CH CH, H3C CH CH2, CH3 CH2 CH CH2 (a) +I effect of the methyl groups.
(c) H CH CH2, CH3 CH2 CH CH2, CH3–CH2 CH2 CH CH2 (b) electromeric effect of the methyl groups.
(d) H CH CH2, CH3 CH2 CH CH2, CH3 CH CH2 (c) hyperconjugation effect of the methyl groups.
(d) −I effect of the methyl groups.
82. In the organic compound CH2 CH CH2 CH2 C CH, the
pair of hybridized orbitals involved in the formation of C2 C3 90. Which of the following species contain six electrons around
bond is the central carbon atom?
(a) sp–sp2 (b) sp–sp3 (c) sp2–sp3 (d) sp3–sp3 (a) Carbanion (b) Carbocation
(c) Carbene (d) Free radical
83. The number of resonance contributing structure(s) for the
following compound is 91. Homolytic fission yields
H (a) electrophiles.
H
H (b) nucleophiles.
C C H (c) free radicals.

C C (d) electrophiles and nucleophiles.
HO
H H 92. Which of the following are position isomers?
(a) 3 (b) 2 (c) 4 (d) 0 CH2 CH CH3 C CH3
(a) (b)
84. Which of the following is the highest priority in the CH2 CH3 CH2
Cahn-Ingold-Prelog system of prioritizing groups for R/S CH3 CH C CH
nomenclature? (c) (d) None of these
Br Cl CH3
Passage Type

(a) Propyl group (b) Methyl group Paragraph for Questions 93 to 94: A mixture of two or more
(c) Bromine (d) Chlorine structurally distinct compounds are present in many compounds.
Those in rapid equilibrium are called tautomers and the phenom-
Level II enon as tautomerism. For example, tautomerism between a car-
Multiple Correct Choice Type bonyl compound containing on a hydrogen and its enol form.
H H
85. Which of the following statements are correct?
(a) The C Cl bond in chlorobenzene is shorter than in R C C R R C C R
chloroethane.
(b) The C Cl bond in chlorobenzene has some double H O O H
bond character.

Chapter-12.indd 406 7/30/2016 4:15:47 PM


Answer Key 407

Generally, keto form is more stable, but in the compound in which 97. The correct statement about T is
the enolic double bond is in conjugation with another double (a) It has two stereoisomers.
bond or in which enol is stabilized by internal hydrogen bonding, (b) It gives an achiral product (W).
enol form predominates. (c) It has two asymmetric carbon atoms.
(d) It has four stereoisomers and all are optically active.
93. In which of the following enol content is maximum?
(a) Ph2CHCHO (b) NCCH2COOC2H5 Matrix-Match Type
(c) CH3COCH2COCH3 (d) PhCOCH3
98. Match the compound with its characteristic.
94. Which structure (I or II) is more stable? Column I Column II

R2 C C R R2 C C R (a) Benzene (p) Aromatic

O O− (b) Neopentane (q) Only sp2 carbons


(I) (II) (c) 1,3-Butadiene (r) Exclusive monochloro derivative
(a) I (b) II (d) Naphthalene (s) 1,4-addition at room temperature
(c) Equal stability (d) Cannot be compared
99. Match the intermediates with the potential molecular species.
Paragraph for Questions 95 to 97: Three optically active acyclic Column I Column II
compounds P, Q and T (molecular formula C5H9Br) give the follow- (a) Stable carbocation (p) Trifluoromethyl
ing reactions. (b) Stable carbanion (q) Trichloromethyl
H / catalyst (1 mol) (c) Stable free radical (r) Divinyl methyl
2
P → R (achiral) (C5H11Br ) (d) Most stable carbocation (s) Acetylenic
H / catalyst (1 mol)
2
Q → S (Chiral) (C5H11Br )
Integer Type
O3 100. The total number of isomers formed by C5H10 is ___.
W + CH3 CHO
T 101. The number of double bonds present in hexachlorobenzene
U (Chrial) is ___.
H2/catalyst (1 mol)
102. The number of unhybridized orbitals present in vinyl acety-
95. The compound P can be lene is ___.
(a) 2-bromopent-1-ene (b) 3-bromopent-1-ene
(c) 1-bromopent-1-ene (d) 4-bromopent-1-ene 103. The number of sp3 hybridized carbon atoms present in
cyclohexene is ___.
96. Which of the following is not Q?
CH3 104. The number of optical isomers of lactic acid is ___.
CH3Br
H Br 105. The number of p bonds present in naphthalene molecule is
H CH3 ___.
(a) (b)
CH CH CH3
CH CH2 106. The total number of isomers of a disubstituted benzene
compound is ___.
107. The possible number of optical isomers for a compound
CH3 containing 2-dissimilar asymmetric carbon atom is
CH2CH3 ___.
H Br
(c) (d) Br CH CH2 108. The number of methyl groups present in 2,5-dime-
CH2 CH CH2
H thyl-4-ethylheptane is ___.

ANSWER KEY
Level I
1. (c) 2. (c) 3. (a) 4. (b) 5. (c) 6. (a)
7. (b) 8. (d) 9. (b) 10. (c) 11. (d) 12. (b)
13. (b) 14. (c) 15. (b) 16. (d) 17. (c) 18. (a)
19. (b) 20. (b) 21. (b) 22. (b) 23. (b) 24. (d)
25. (a) 26. (a) 27. (b) 28. (b) 29. (b) 30. (c)
31. (d) 32. (c) 33. (d) 34. (c) 35. (b) 36. (b)
37. (a) 38. (c) 39. (b) 40. (a) 41. (c) 42. (d)
43. (a) 44. (a) 45. (d) 46. (b) 47. (b) 48. (b)
49. (b) 50. (a) 51. (a) 52. (b) 53. (d) 54. (d)
55. (d) 56. (b) 57. (a) 58. (c) 59. (b) 60. (a)

Chapter-12.indd 407 7/30/2016 4:15:49 PM


408 Chapter 12 Some Basic Principles in Organic Chemistry

61. (c) 62. (c) 63. (b) 64. (c) 65. (d) 66. (b)
67. (d) 68. (d) 69. (a) 70. (c) 71. (a) 72. (d)
73. (b) 74. (d) 75. (b) 76. (a) 77. (c) 78. (c)
79. (b) 80. (a) 81. (b) 82. (b) 83. (b) 84. (c)

Level II
85. (a), (b), (c) 86. (a), (b), (c), (d) 87. (b), (d) 88. (a), (c) 89. (a), (c) 90. (b), (c)
91. (b), (c) 92. (a), (b) 93. (c) 94. (b) 95. (b) 96. (d)
97. (d) 98. (a) → p, q, r; (b) → r; (c) → q, s; (d) → p, q 99. (a) → s; (b) → p; (c) → q; (d) → r 100. (6)
101. (3) 102. (6) 103. (4) 104. (2) 105. (5) 106. (3)
107. (4) 108. (5)

HINTS AND EXPLANATIONS


Level I side of the double bond, that is, either above or below the
plane. This is called syn addition.
Single Correct Choice Type a b
1. (c) There are six chiral centres present in the given syn addition
C C Racemic mixture
compound.
O b a
6. (a) The stereoisomers that are optically active isomers but not
O mirror images are called diastereoisomers or diastereomers.
7. (b) 2,2,4,4-tetramethylpentane has two tertiary carbon atoms.
CH3 CH3
CH3 C CH2 C CH3
3° 3°
2. (c) Tertiary carbocation is stable due to electron-releasing CH3 CH3
effect of the three alkyl groups. Free radicals are very reac-
tive reaction intermediates because the odd electron pres- 8. (d) Meso compounds are the inactive compound because the
ent has a natural tendency to pair its odd electrons with rotation by upper half of molecule is internally compen-
another electron from any available source. sated by lower half.
H 4 H 2 4 2 COOH COOH COOH COOH
3. (a) 3 1 H − 3 1 H
5 5 H3C H H CH3 H CH3 H3C H
O −H+ O
N N Carbanion HO H HO H H OH H OH

Br H Br H Br H Br H
C 4 will give fastest deprotonation due to stabilization of
negative charge through conjugation. HO H HO H H OH H OH
4. (b) The number of geometrical isomers is calculated by the
formula 2n2 (where n is the number of double bonds) in H3C H H CH3 H CH3 H3C H
case both ends are identical. COOH COOH COOH COOH
For example, H3C CH CH CH3
However, if both ends are different, the number of geomet- All these four molecules possess symmetry; so, all are
rical isomers as in the given compound, can be calculated meso compounds.
by the formula 9. (b) (CH3)3COH gives (CH3)3C+ which is a tertiary carbocation,
2n−1 + 2p and thus very stable.
where p = n/2 for even number and p = (n + 1)/2 for odd 10. (c) C1 N and C3 N bonds are of the same length as they
number of double bonds. occupy symmetrical position in the molecule and due
In the given compound, n = 3. So, to the close proximity of electronegative atom Br they
3 +1 have longer bond length than C5 N bond.
p= =2 11. (d) Two or more organic compound having the same molec-
2
ular formula but different spatial arrangement of atoms
Substituting, we get 23−1 + 2(p) = 22 + 4 = 8
or groups, around doubly bonded (carbon) atoms, are
5. (c) We know that an alkene molecule is planar with the p cloud known as geometrical isomers and the phenomenon
above and below the plane. Thus, the addition of electro- as geometrical isomerism. Geometrical isomerism is not
phile to alkenes is done in two distinct pathways. In one, observed in a molecule if either of the doubly bonded
both parts of the reagent add to the p bond from the same atoms has two similar groups.

Chapter-12.indd 408 7/30/2016 4:15:51 PM


Hints and Explanations 409

In options (a) and (c), the groups attached to doubly OH


bonded carbon are same. In option (c) one of the substit- 1
uent on doubly bonded carbon is a cyclic structure; so, O
3
geometrical isomerism is not possible. 7
5 4
12. (b) Optical activity (the property to rotate the plane of polar- 2
OH
ized light) is observed only when molecules have chirality. 6
Optically inactive forms are known as meso forms and
the molecules are superimposable on their mirror 16. (d) Degree unsaturation means number of double or triple
images even though they contain chiral centre. So com- bond present in the compound.
pound (b) is optically inactive. Degree of unsaturation = Number of double bonds +
Number of rings in the molecule + Twice the number of
13. (b) Enol where en = double bond and ol = alcohol. triple bonds.
Enol content means compound contains double bond as
well as OH (alcoholic group). 17. (c) Compounds with greater number of covalent bonds are
The enol content is minimum in acetaldehyde. more stable. Compound containing symmetrical charge
Phenol C6H5OH exhibits keto–enol tautomerism, but distribution is more stable than unsymmetrical charge
only a tiny fraction of phenol exists as the keto form. distribution. Therefore, the order is I > III > II
+ +
O O H CH2 CH CH CH CH CH2 CH2 CH CH CH CH CH2
Acetaldehyde: H3C C H H2C C H + +
Acetoacetic ester: CH2 CH CH CH CH CH2
O O H O O This is the relative contribution in the hybrid orbital.
H3C C CH2 C O C2H5 H3C C CH2 C O C2H5 18. (a) Two or more organic compounds having the same
molecular formula but different structural formula due to
different positions of substituents, alkyl groups, unsatu-
O H O ration or functional groups in carbon chain are known as
position isomers and the phenomenon as position isom-
H2 C C CH2 C O C2H5 erism. In the given compound
Pentane-2,4-dione:
O H O O O CH2Cl and

H 2C C CH2 C CH3 H3 C C CH2 C CH3


CH2Cl

O O H both are similar structures; hence, they are not position


isomers.
H3C C CH2 C CH2 19. (b) In the given carbocations:
Hence, acetaldehyde has minimum enol content. CH3 H
14. (c) + + +
H3 C C CH2 H3C C CH3 H3C C CH2 CH3
(I) CH3 CH2 CH2 CH2 OCH3 +

CH3 CH3 CH3


Neopentyl (1°) tert-Butyl (3°) Isopropyl (2°) Methyl (1°)
(II and III) CH3 CH2 CH OCH3
∗ There are four hyperconjugating structures for primary
CH3 carbocation; similarly, the number of hyperconjugat-
(IV) CH3 CH CH2 OCH3 ing structures for secondary and tertiary carbocation
is 7 and 10, respectively. As the number of hyperconju-
CH3
gating structures increases, the stability of carbocation
(V) CH C CH3 increases. Hence, tert-butyl carbocation is the most sta-
3
bilizing hyperconjugation effect.
OCH3 20. (b) Heterolytic fission of covalent bond in organic molecules
(VI) CH3 CH2 CH2 OC2H5 results in the formation of a carbocation, in which the
carbon atom bears a positive charge and has six elec-
(VII) CH CH CH3
3 trons in the valence shell.
OC2H5 R R
15. (b) There are 7 double/triple bonds in the compound; hence, Heterolytic
R C X fission R C+ + X −
degree of unsaturation is 7. Carbanion
R R
Carbocation

Chapter-12.indd 409 7/30/2016 4:15:54 PM


410 Chapter 12 Some Basic Principles in Organic Chemistry

21. (b) CHO CHO X-ray diffraction studies show that, unlike benzene, all
carbon–carbon bonds in naphthalene are not of the
H C OH HO C H same length. In particular, the C1 C2 bond is consider-
ably shorter (1.36 Å) than the C2 C3 bond (1.40 Å). This
CH2OH CH2OH
difference can be understood if we examine the three
(+)-Glyceraldehyde (−)-Glyceraldehyde resonance forms given above. Notice that the C1 C2
D configuration L configuration bond is double in two structures (A and B) and single
in only one (C); whereas the C2 C3 bond is double in
The symbol D indicates the dextrorotatory nature of the
two structures (A and B) and single in only one (C). We
compound, that is, hydrogen atom lies right to the a chi-
would, therefore, except the C1 C2 bond to have more
ral centre in the Fischer projection formula.
double-bond character (shorter bond length), and
22. (b) Two or more organic compounds having the same
the C2 C3 bond to have more single-bond character
molecular formula but different structural formula due
(longer bond length).
to different nature of alkyl groups, around a polyvalent
29. (b)
functional group, are known as metamers and the phe-
nomenon as metamerism. Metamers include same class H CH3 H
of compounds. ∗
H 3C C CH2 CH3 H3C C OH H3C C CH2OH
23. (b) As the s-character increases in hybridized orbital, the
tendency of overlapping becomes more, giving rise to OH CH3 CH3
smaller bond length and higher bond energy.
C2H6 (1.54 Å) > C6H6 (1.39 Å) > C2H4 (1.34 Å) > C2H2 (1.20 Å) ∗
24. (d) CH3 CH2 CH2 CH2OH H3C CH2 CH CH3
3
F 1 OH
1
Br 2 3
(a) 30. (c)
H 2N Cl 4 H H
4 2 O O
H 3C C2H5O− H 3C
S-configuration
3 −
H 3C 2
4 H CH3 CH3
H 3 1
(b) 2
Carbanion
H5C2 COOH 4
1 Because in the above compound electron-withdrawing
S-configuration group is present adjacent to carbanion. Hence, it is the
2
most readily formed carbanion.
H2N 3
3 31. (d) Stability of carbocations can be explained by resonance,
COOEt 1 2 inductive effect and hyperconjugation effect.
(c) 4
H Br 4 (a) Resonance: For example consider allyl carbocation
1 + +
CH2 CH CH2 CH2 CH CH2
S-configuration
(b) Inductive effect: The order of stability of carbocation
25. (a) As it contains tertiary carbon atom. can also be explained on the basis of +ve inductive
26. (a) effect of alkyl group. Lesser the positive formal charge
4 3 on the carbon atom of the carbocation due to +I of
H 3 CH3 2
1 3 4 alkyl group, greater is the stability of carbocation.
Rotation
Cl CH3 2 1 H NH2 1 3 (c) Hyperconjugation: The stability of carbocations can be
270° 2
Cl explained on the basis of number of hyperconjugating
NH2 4
1
4
2 structures. As the number of hyperconjugating struc-
R-configuration R-configuration ture increases, the stability of carbocation increases.
32. (c)
27. (b) The most stable carbocation is the tertiary carbocation. +
+ CH2 +
CH2
H2C
28. (b)
1 1 1 33. (d) Inductive effect may be defined as the induction of
2 2 2 polarity in an otherwise covalent bond, due to incom-
plete shifting of the electron pair between the two
3 3 3
atoms that have different electronegativity. It is a per-
4 4 4
manent effect. It is represented by the symbol →. It
A B C propagates through the carbon chain.

Chapter-12.indd 410 7/30/2016 4:15:58 PM


Hints and Explanations 411

CH3 C CH2 CH3 Geometrical isomers


due to double bond
O Chiral carbon
This is the incorrect representation of inductive effect, ∗
the correct representation is H3C CH3
CH3 C CH2 CH3 CH3
O 39. (b) The reaction proceeds with elimination of water
34. (c) The order of stability of carbocations is 3° > 2° > 1°. So, molecule.
the carbcation in option (c) is most stable. H+ + −H2O +
+
H 3C CH2 OH H3 C CH2 OH2 H 3C CH2
CH3 C CH3
−H+
CH3
H 2C CH2
35. (b) Molecular formula of C3H6O represents following seven
isomers.
40. (a)
(i) Acyclic isomers: Asymmetric carbon atom
O OH O
H H H H
CH3 C CH3 CH3 C CH2, CH3 CH2 C H, Cl2
H 3C C C CH3 H3C C C CH2Cl
Propanone 2-Hydroxypropene Propanal
CH3 CH3 CH3 CH3
CH2 CH CH2OH, CH2 CHOCH3 +
H Cl
Prop-2-en-1-ol Methoxyethene
(ii) Cyclic isomers: H 3C C C CH3
CH2 O CH2 CH OH CH2 CH CH3 CH3 CH3
CH2 CH2 CH2 O
H H H H
Oxetane Cyclopropanal 2-Methyloxirane
H 3C C C CH2Cl ClCH2 C C CH3
36. (b)
Secondary CH3 CH3 CH3 CH3
carbocation Mirror plane

+ 41. (c) There are four contributing structures for hyperconjuga-


C
tion of CH3CH CH2.
H +
The order of stability can be explained on the basis of dis- H H H H H H
persal of positive charge by electron releasing +I effect
H C C C H H C C C H
of groups. Greater the number of alkyl groups on the −
carbon carrying positive charge, greater would H H
be the dispersal of charge, and hence, more stable will I II
be the carbocation.
Benzyl > Tertiary > Secondary > Primary H H H H H H

37. (a) H C C C H H+ C C C H
− −
− + + −
H N C O H N C O H N C O H H
+
I II III IV III
Order of stability: I > III > II. In compound II the electron- 42. (d)
egative oxygen atom carries +ve charge hence this struc- OH
ture is unstable.
9 7 5 3
38. (c) The compound shows both optical and geometrical 10 8 6 4 2 1
isomerism. Optical isomerism is shown by those com-
pounds that contain at least one chiral carbon and Br
geometrical isomerism shown by the compound con-
taining double bond.

Chapter-12.indd 411 7/30/2016 4:16:01 PM


412 Chapter 12 Some Basic Principles in Organic Chemistry

49. (b) Electron-deficient species that attack a negative centre


43. (a) % of s character : sp > sp2 > sp3
50% 33% 25% or a carbanion (C−) are known as electrophiles. These
accept electron pairs thus act as Lewis acids.
The hybrid state of carbon atom also affects the elec-
tronegativity of the atom. Greater the s-character of the 50. (a) The inductive effect decreases with increase of distance.
hybrid orbitals, greater is the electronegativity of the
atom. Thus, sp-hybridized carbon is more electronega- ddd + dd + d+ d−
C4 C3 C2 C1 Cl
tive than sp2 or sp3 hybridized carbon atom.
44. (a) 4-(Benzoylamino)-2-nitrobenzoic acid 51. (a) Nucleophile is a Lewis base, an electron pair donor that
CO NH seeks a positive centre in a molecule.
4
3
1
COOH 52. (b)
2 OH OH
NO2

45. (d) Attack of the electrophilic reagent from the opposite


side to the p bond is called anti-addition.
a a Cl
a b Br b b Br
anti addition
p-Chlorophenol Phenol
C C −Br2 Br b b Br
The acidity of phenols is due to resonance stabilization
b a of the phenoxide ion relative to phenol. A group that
a a
stabilizes the phenoxide ion by more dispersal of neg-
Meso compounds ative charge will increase the acidity of phenols. Thus,
46. (b) There are three derivatives for formula C6H3Cl3. electron-withdrawing groups that stabilize the phenox-
Cl H Cl ide ion by dispersing the negative charge relative to
phenol increases the acid strength of phenols. Hence,
H Cl Cl Cl Cl H p-chlorophenol is more acidic than phenol.
53. (d) CCl3CHO will not show tautomerism because a-carbon
H Cl H H H H atom does not contain hydrogen atom. A molecule
undergoes tautomerism, if the a-carbon contains at
H Cl Cl
least one hydrogen atom.
47. (b) 2-Methylbutane is the alkane with smallest molecular
weight which possesses primary, secondary and tertiary 54. (d) Hyperconjugation is a special kind of resonance in
carbon atoms. which delocalization of electrons takes place through
overlap between s bond orbital and p bond orbital or
p-orbital.
Hyperconjugation arises due to partial overlap of a
48. (b) In the isomeric alcohols with the molecular formula
sp3–s sigma bond orbital with the empty p orbital or p
C5H12O, there are three structures with chiral carbons.
bond orbital of an adjacent carbon atom.
CH3
CH3 CH2 CH2 CH2 CH2 OH CH3 CH CH2 CH2 OH 55. (d)
Pentan-1-ol 3-Methylbutan-1-ol O O−
CH2 CH2 N CH2 CH2 N
CH3 CH3 O O

CH3 CH2 CH CH2 OH CH3 C CH2 OH 56. (b) Three different but equivalent structures are possible
*
CH3 for CO2−
3 .
2-Methylbutan-1-ol 2,2-Dimethylpropan-1-ol
− −
* * O O O
CH3 CH2 CH2 CH CH3 CH3 CH CH CH3
C C C
OH CH3 OH −O O− −O O O O−
Pentan-2-ol 3-Methylbutan-2-ol
1 2 3
CH3
57. (a) CH3+ is the most stable. The replacement of H by Br
CH3 C CH2 CH3 CH3 CH2 CH CH2 CH3 increases positive charge on carbon atom as Br is more
3
electronegative than H, and consequently the species
OH OH
becomes less stable.
2,2-Methyl-2-butan-2-ol Pentan-3-ol

Chapter-12.indd 412 7/30/2016 4:16:04 PM


Hints and Explanations 413

58. (c) There are 8 chiral carbon atoms present. 73. (b) The carbon skeleton and the bond line formula are rep-
H 3C resented as
CH3 H3 C H CH3
CH3 C C
CH3 H2
H3C CH3

74. (d) The functional groups are represented as acid ( COOH),


HO ether ( O ) and ester ( COOR) where R is an alkyl or
aryl group.
59. (b) H3C H → H3C +
Carbocation 75. (b) The heterolysis takes place as follows:
60. (a) 1-Isopropyl-2,4-dimethylbenzene
Heterolysis
+ + Br−
61. (c) It has phenyl (C6H5), carboxyl ( COOH), amide ( CONH2),
ester ( COOCH3) and primary amine ( NH2) as func- Br
tional groups. Carbocation
62. (c) Metamers can be possible only if the functional group is
76. (a) The enol form is antiaromatic as it has 4np electrons. So,
attached to different alkyl groups, which is not possible
it is the least stable.
in case of double bonded C.
63. (b) The heterolytic cleavage takes place as follows: O O H

CH3 Cu CH3 + Cu+ H
H
Carbanion Least stable alcohol
64. (c) N,N-Diethylethanamine 77. (c) The functional isomer of propan-1-ol is CH3 O CH2CH3
65. (d) Conceptual and its name is methoxyethane.

66. (b) R C N (cyanide) and R N C (isocyanide) are func- 78. (c) It has 10s and 3p bonds
tional isomers.
H H H
67. (d) The hybridization is as follows: s s s
p s s
3
sp each sp 2
sp sp 2 3
sp each H s C s C C C C s H
s 2p
( CH3)2 C = C H C (CH3 ) 2 1s
H
68. (d) They are not structural isomers because their molecular
79. (b) 1-Methylcyclohex-2-ene
formula is different. The rest are because they are func-
tional group isomers. 80. (a) Since the carbon is sp2 hybridized, the shape is trigonal
planar.
69. (a) The order of C C bond length is
81. (b) CH CH2 (Ethene); H3C CH CH2 (Propene);
> > H 3C CH2 > HC CH CH3 CH2 CH CH2 (But-1-ene) or CH3 CH CH CH3
(But-2-ene) followed by CH3 CH2 CH2 CH CH2 (Pent-1-
ene) and so on.
1.54 A° 1.39 A° 1.34 A° 1.20 A°
6 5 4 3 2 1
As the s-character increases in hybridized orbitals, the ten- 82. (b) CH2 CH CH2 CH2 C CH. In the compound,
dency of overlapping becomes more giving rise to smaller
C2 is sp and C3 is sp3 hybridized
bond length and higher bond energy.
C6H12 > C6H6 > C2H4 > C2H2 83. (b) The resonating structures are
70. (c) Carboxylic acid, amine, alcohol H H
H H
71. (a) Cyclohexyl alcohol H H H H
C C
− −
72. (d) The name of the compound is 2-chloro-4-ethyl benzoic HO C C C HO C C C
acid. H H H H
COOH
1 84. (c) Priority is determined by atomic number, Br = 35, Cl = 17,
6
Cl
2 C = 6, C = 6. Therefore, Br has the highest priority.
5 3
4
CH2CH3

Chapter-12.indd 413 7/30/2016 4:16:08 PM


414 Chapter 12 Some Basic Principles in Organic Chemistry

Level II H +
+ H
Multiple Correct Choice Type H C C H H C C H
85. (a), (b), (c)
H H H H
The C Cl bond in chlorobenzene is shorter than in chlo-
roethane because chlorobenzene undergoes resonance; as
a result, chlorine and C bonds have double bond character. H H
So, the C Cl bond is shorter. But in chloroethane, there is no
resonance; hence, C Cl bond is longer. H C C H H+ C C H
+
H
C6H5 Cl and C2H5 Cl H H H
In chlorobenzene, chlorine atom is involved in resonance
and in benzoyl chloride oxygen atom is involved in reso- (CH3 )3 C+ > H3C +
89. (a), (c)
nance. Hence, Cl from chlorobenzene is difficult to remove tert -Butyl carbocation Methyl carbocation
than from benzoyl chloride.
The stability of tertiary butyl carbocation can be explained on
C6H5 Cl and C6H5 COCl
the basis of (+I) inductive effect of methyl group. It is clear that
Chlorine group is ortho- and para-directing group; hence, lesser the positive formal charge on the carbon atom of the
incoming nucleophile enters into either para or ortho posi- carbocation due to +I of methyl group, greater is the stability
tion. Para position is more stable and forms major compound of carbocation.
because there is no steric hindrance between the two chlorine As the number of hyperconjugating structures increases, the
atoms. stability of carbocation increases. There are 4 hyperconjugat-
ing structures for methyl carbocation and 10 hyperconjugat-
Cl Cl Cl ing structures for tertiary butyl carbocation. Hence, it is more
Cl stable.
Cl2
+ 90. (b), (c)
hv

H H H
Cl H
(major) H C+ +C H C− H C
H
86. (a), (b), (c), (d) H H H
Carbocation Singlet carbene Carbanion Carbon free radical
+ (6 electrons) (6 electrons) (8 electrons) (7 electrons)

91. (b), (c) In homolysis, the covalent bond breaks in such a way

that each resulting species get its own electron. The neutral
fragments A and B are called free radicals.

Homolytic fission
+ − − + A : B 
→ A i + Bi
Free radicals
Hence, homolytic fission yields free radicals and they also act
as nucleophiles.
92. (a), (b) These are position isomers because they have differ-
ent position of the double bond. In (c), the molecular formula
changes, and hence it is not an isomer.
87. (b), (d) The following two compounds contain same num- Passage Type
ber of linearly arranged carbon atoms.
93. (c)
H H O O O H O
H C C H H C C C H
H H H 3C C CH2 C CH3 H3 C C CH2 C CH3
Keto form Enol form
88. (a), (c) Isovalent hyperconjugation explain the stability of
carbocations and free radicals.
In the free radicals and carbocations, the canonical forms do O H OH
not display more charge separation than the main form.
H2 C C CH C CH3

Chapter-12.indd 414 7/30/2016 4:16:10 PM


Hints and Explanations 415

94. (b) In compound (II) the negative charge is present on elec- F


tronegative oxygen atom; hence, it is more stable.
F C− F H C C+

R2 C C R R2 C C R Trifluoromethyl carbanion Acetylenic carbocation

O O
+
(I) (II) CCl3 CH2 CH CH CH CH2
95. (b) Trichloromethyl free radical Divinyl methyl carbocation
H H
H2/Catalyst (1 mol) Integer Type
H3C CH2 C CH CH2 H3 C CH2 C CH2 CH3
100. (6)
Br Br
CH3 CH2 CH2 CH CH2 CH3 CH2 CH CH CH3 H2
3- Bromopent-1-ene Achiral
(P) 1-Pentene 2-Pentene C
H2 C C H2
96. (d)
CH3 C CH CH3 CH3 CH CH CH2 CH3 C CH2 CH3 H2 C C H2
97. (d)
CH3 CH3 CH2
CH3 3-Methylpent-2-ene 3-Methylpentene 2-Methylpent-1-ene
O3
H Br + CH CHO
CH3 CH3 CH2 CH2 CH CH2 3CH3 CH2 CH CH CH3 H2
H Br CHO
1-Pentene 2-Pentene C
(W)
H2 C C H2
CH CH CH3
CH
CH3 C CH CH3 3CH3 CH CH CH2 CH3 C CH2 CH3 H2 C C H2
H2/catalyst (1 mol) H Br
CH3 CH3 CH2
3-Methylpent-2-ene CH2 3-Methylpentene
CH2 CH3 101. (3) Hexachlorobenzene
2-Methylpent-1-ene
U (Chiral) Cl
Cl Cl
Matrix-Match Type
98. (a) → p, q, r; (b) → r; (c) → q, s; (d) → p, q Cl Cl
(a) Benzene is aromatic in nature, contains sp2 carbon Cl
atoms; and is an exclusive monochloro derivative.
CH3
(b) CH 102. (6)
C CH
3 3 H
H C
CH3
C C
Neopentane is an exclusive monochloro derivative C
H
because all the atoms is in similar position.
H
(c) H2C CH CH CH2
One unhybridized orbital on each carbon involved in dou-
1,3-butadiene has sp2 hybrid carbon atoms, and under- ble bond and two each on carbon involved in triple bond.
goes 1,4 addition at room temperature.
HCl 103. (4)
H 2C CH CH CH2 CH2 CH CH CH3
sp3 hybridized carbon atom
H Cl
1,4-addition

(d)
104. (2) Two optical isomers of lactic acid are as follows:
COOH COOH
Naphthalene is aromatic in nature. It only has sp2 carbon
atoms. H C OH HO C H
99. (a) → s; (b) → p; (c) → q; (d) → r
CH3 CH3

Chapter-12.indd 415 7/30/2016 4:16:13 PM


416 Chapter 12 Some Basic Principles in Organic Chemistry

105. (5) There are 5p bonds in the naphthalene structure. 107. (4) a = 2n where a = number of optically active isomers and
n = number of asymmetric carbon atoms
As n = 2, so a = 22 = 4.
108. (5) 2,5-Dimethyl-4-ethylheptane
106. (3) Three isomers are possible for disubstituted benzene.
X X X CH2 CH3
7 6 5 4 3 2 1
X CH3 CH2 CH CH CH2 CH CH3

CH3 CH3
X
X

SOLVED JEE 2016 QUESTIONS


JEE Main 2016 Solution
(c)
1. The absolute configuration of:
3 3
CO2H COOH
4
4 1 2S
1
H OH H OH
2 2
4
H Cl 2 2
H Cl
3 1
CH3 CH3 4 1 3R
3
(a) (2R, 3R) (b) (2R, 3S) (c) (2S, 3R) (d) (2S, 3S)
(Offline)

Chapter-12.indd 416 7/30/2016 4:16:15 PM


13 Aliphatic Hydrocarbons

Question Distribution in JEE (Main and Advanced)

3
No. of Questions

JEE (Main)
2
JEE (Adv)

0
2016 2015 2014 2013 2012 2011 2010 2009 2008 2007

Concept Distribution in JEE (Main and Advanced)

Topics Covered
Year
JEE (Main) JEE (Advanced)
Conformations of Alkanes,
2007 Chemical Properties of Alkenes
Chemical Properties of Alkynes
Chemical Properties of Alkenes,
2008
Chemical Properties of Alkynes
2009
2010 Chemical Properties of Alkenes Conformations of Alkanes
Chemical Properties of Alkanes,
2011 Chemical Properties of Alkenes
Methods of Preparation of Alkenes
Chemical Properties of Alkanes,
2012 Chemical Properties of Alkenes
Chemical Properties of Alkynes
2013 Chemical Properties of Alkenes
Chemical Properties of Alkanes,
Chemical Properties of Alkenes, Conformations of Alkanes,
2014
Chemical Properties of Alkynes, Physical Properties of Alkanes
General Method of Preparation of Alkenes
2015 Chemical Properties of Alkenes
General Method of Preparation of Alkenes, Chemical Prop-
2016
erties of Alkenes, Nomenclature

Chapter-13.indd 417 8/8/2016 2:25:54 PM


418 Chapter 13 Aliphatic Hydrocarbons

SUMMARY
Alkanes
1. Alkanes are those organic compounds that comprise of single-bonded carbon and hydrogen atoms entirely and are devoid of any
other functional groups. Alkanes have the general formula CnH2n + 2 and are also saturated hydrocarbons.
2. General methods of preparation
(a) Hydrogenation of alkenes and alkynes

C H Pt, Pd, or Ni C H C Pt H C H
+ + 2H2
Solvent, Solvent,
C H pressure
C H C pressure
H C H

Alkene Alkane Alkyne Alkane

(b) From alkyl halides


(i) Heating of alkyl halides with Na metal in the presence of dry ether (Wurtz reaction for the preparation of symmetrical
alkanes). If Zn is used in place of Na, the reaction is named as Frankland method.
Dry ether
R − X +2Na +X − R → R − R + 2NaX

(ii) Reduction of alkyl halides: Alkanes are obtained when alkyl halides are heated with reducing agents like LiAlH4/ether, Pd/H2,
Pt/H2, Zn/conc. HCl.
R − X → R − H + HX
(c) From Grignard reagents
Grignard reagent readily reacts with compounds containing active hydrogen, such as alcohols, acids, amines, water, ammonia,
etc. to form alkanes.
Dry ether
RMgX + H2O 
→ R − H + Mg(OH)X
(d) From carboxylic acids
Two methods using these are
(i) Decarboxylation of sodium salt of carboxylic acid in presence of soda lime.
CaO
RCOO − Na+ + NaOH 

→ CH4 + Na2CO3

(ii) Kolbe’s electrolytic method


Electrolysis of aqueous solution of sodium or potassium salts of carboxylic acids.
Electrolysis
2RCOO − Na+ + H2O 
→ R − R + 2CO2 + H2 + 2NaOH

The reaction proceeds by the following path:


i i
−2 e −
At the anode: 2RCOO − 
→ 2R − CO O →
 2 R + 2CO2 ↑

i i
R + R →R − R ↑
i
At the cathode: H2O + e − → OH− + H
i
2 H → H2
(e) Corey–House synthesis: This method involves coupling of two alkyl halides as in Wurtz reaction. However, one of the alkyl
halide is converted into lithium dialkylcuprate (R2CuLi) and then treated with second alkyl halide (R′—X). It can thus be used for
synthesis of symmetrical and unsymmetrical alkanes.
R2CuLi + R′ X R R′ + RCu + LiX
Lithium Alkyl halide Alkane
dialkylcuprate

Chapter-13.indd 418 8/8/2016 2:25:58 PM


Summary 419

3. Physical properties
(a) Alkanes are non-polar compounds with weak intermolecular interactions. They are insoluble in water due to low polarity and
inability to form hydrogen bonds.
(b) The boiling points of the unbranched alkanes show a regular increase with increasing molecular weight. Branching of the alkane
chain, however, lowers the boiling point. This can be explained on the basis of van der Waals forces.
(c) The unbranched alkanes do not show smooth increase in melting point with increase in atomic number. However, individually
even and odd carbon atom alkanes show a smooth increase with increasing molecular weight.
4. Chemical properties
(a) Oxidation
Alkanes undergo combustion reaction with oxygen when ignited (as in automobiles and furnaces). A complex series of reactions
takes place, ultimately converting the alkane to carbon dioxide and water. General reaction is

 3n + 1
CnH2n + 2 +  O → nCO2 + (n + 1)H2O
 2  2
(b) Substitution reactions
(i) Halogenation: Alkanes react with molecular halogens to produce alkyl halides by substitution reaction called radical halo-
genation. The halogenation is catalyzed by dibenzoyl peroxide upon exposure to UV light or at elevated temperature.
R − H + X2 → R − X + HX
The reactivity order for halogens shows the order: F2  > Cl2  > Br2  > I2. The reactivity order for H atom in alkane shows the order:
3° > 2° > 1° > CH4.
Mechanism for radical chlorination of methane
Heat
CH4 +Cl2 Light
 → CH3Cl + HCl
Step 1: Halogen dissociation—Under the influence of heat or light a molecule of chlorine dissociates; each atom takes one of the bond-
ing electrons.
Heat
Cl Cl Cl + Cl
or Light

Step 2: Hydrogen abstraction


Step 3: Halogen abstraction

H H
Cl + H C H Cl H + C H
H H
A chlorine atom This step produces a
abstracts a hydrogen molecule of hydrogen
atom from a chloride and a methyl
methane molecule. radical.

H H
H C + Cl Cl H C Cl + Cl
H H
A methyl radical This step produces a
abstracts a chlorine molecule of methyl
atom from a chlorine chloride and a chlorine
molecule. atom. The chlorine atom
can now cause a repetition
of Step 2.

(ii) Nitration
CH4 + HNO3→ CH3NO2 + H2O (at 400°C)
The reactivity order for nitric acid is: 3° > 2° > 1° > CH4

Chapter-13.indd 419 8/8/2016 2:26:00 PM


420 Chapter 13 Aliphatic Hydrocarbons

(iii) Sulphonation
C6H14 + H2SO4 → C6H13SO3H + H2O (at 400°C)
The reactivity order for sulphonation is 3° > 2° > 1° > CH4
(c) Isomerization
Straight chain alkanes on heating with AlCl3 + HCl at 200°C and 35 atm pressure isomerize into branched chain alkanes.
Anhy. AlCl3/HCl
CH3(CH2)4CH3 CH3CH (CH2)2 CH3 + CH3CH2 CH CH2 CH3
n-hexane
CH3 CH3
2-Methylpentane 3-Methylpentane

(d) Aromatization
CH3
Cr2O3 or V2O5
CH2 CH3 or Mo2O3
773K,
CH2 CH2 10–20 atm
CH2

(e) Pyrolysis
Pyrolysis (cracking) occurs when alkane vapours on passing through red hot metal tube in absence of air decomposes to simpler
hydrocarbons.
C6H12 + H2
C6H14 773K
C4H8 + C2H6
C3H6 + C2H4 + CH4

5. Conformations
(a) Conformations of ethane
The eclipsed and staggered conformation of ethane is shown below:
H HH
H H
H
H H H H
H H
Eclipsed Staggered

(b) Conformations of butane


(i) Rotation about the C2—C3 bond of butane give rise to six important conformers (I–VI).

H3C CH3

HCH3 H H H CH3
H H
Eclipsed IV
H H H H
H3C H H CH3
Eclipsed II Eclipsed VI
Potential energy

19 kJ mol−1
16 kJ mol−1 16 kJ mol−1
3.8 kJ mol−1 3.8 kJ mol−1
CH3 CH3 CH3 CH3
H H H3C H H CH3 H H

H H H H H H H H
CH3 H H CH3
Anti I Gauche III Gauche V Anti I

0° 60° 120° 180° 240° 300° 360°


Rotation

(ii) The anti-conformation (I) does not have torsional strain from steric hindrance because the groups are staggered and the
methyl groups are far apart. The anti-conformation is the most stable.

Chapter-13.indd 420 8/8/2016 2:26:04 PM


Summary 421

(iii) The methyl groups in the gauche conformations III and V are so close that they repel each other. 
(iv) The eclipsed conformations (II, IV, and VI) represent energy maxima in the potential energy diagram.
(v) Eclipsed conformations II and VI have repulsive dispersion forces arising from the eclipsed methyl groups and hydrogen atoms.
(vi) Eclipsed conformation IV has the greatest energy of all because of the added large repulsive dispersion forces between the
eclipsed methyl groups as compared to II and VI.

Tip (a) Torsional strain (also called eclipsed interaction strain) is the strain that arises when non-bonded atoms separated by
three bonds are forced from a staggered conformation to an eclipsed conformation.
(b) The angle strain (also known as Baeyer’s strain) arises when the bond angles deviate from the ideal bond angles to
achieve maximum bond strength.

(b) Conformations of cyclohexane


(i) The most stable conformation of cyclohexane is the chair conformation. In this, all of the carbon–carbon bond angles are
109.5°, and are thereby free of angle strain. There is no angle or torsional strain in the chair form of cyclohexane.
H
H H 4 H
H H
H H H CH2 H
5 6 3
H 2
H H CH2 H
H H 1
H H H
H
(ii) By partial rotations about the carbon–carbon single bonds of the ring, the chair conformation can assume another shape
called the boat conformation. The boat conformation has no angle strain, but it does have torsional strain.
H H
H H HH
H H H H
H H
H H H
H H
H H
H H H H
H
(iii) The boat conformation is quite flexible. By flexing to a new form, that is, twist conformation, the boat conformation can
relieve some of its torsional strain and, at the same time, reduce the flagpole interaction.

Twist boat conformation


(iv) The relative energies of the various conformations of cyclohexane are given below. The positions of maximum energy are
conformations called half-chair conformations, in which the carbon atoms of one end of the ring have become coplanar.
Half- Half-
chair chair
40
Relative energy, kJ mol−1

Boat
Twist Twist
45.2 kJ mol−1
20

30 kJ mol−1
23 kJ mol−1
Chair
Chair
0

Chair Half- Twist Boat Twist Half- Chair


chair boat boat chair
Because of the greater stability of the chair, more than 99% of the molecules are estimated to be in a chair conformation at
any given moment.

Tip The reactivity of cycloalkanes is proportional to the angle strain. The reactivity decreases as the number of carbon atoms in the
ring increases. Thus, the order of reactivity is
Cyclopropane > Cyclobutane > Cyclopentane > Cyclohexane

Chapter-13.indd 421 8/8/2016 2:26:10 PM


422 Chapter 13 Aliphatic Hydrocarbons

(c) Axial and equatorial groups


(i) Steric strain arises from an interaction between an axial group on carbon atom 1 and an axial hydrogen on carbon atom 3 (or
5) is called a 1,3-diaxial interaction.
CH3 (Axial) H H
H
H H H H
H H H H

H H H CH3 (Equatorial)
H H H H
H H
H H
(Less stable) (More stable by 7.6 kJ mol−1)
(ii) In cyclohexane derivatives with larger alkyl substituents, the strain caused by 1,3-diaxial interactions is even more pro-
nounced. The conformation of tert-butylcyclohexane with the tert-butyl group equatorial is estimated to be approximately
21 kJ mol−1 more stable than the axial form.
CH3

CH3 C CH3 H H
H
H H H H
H H H H CH3

H H H C CH3
H H H H
H H
H H CH
3
Axial tert-butylcyclohexane Equatorial tert-butylcyclohexane
(iii) In case of cis-1,4-disubstituted cyclohexane, when one ring substituent group is larger than the other and they cannot both
be equatorial, the conformation with the larger group equatorial will be more stable.
H3C
CH3 CH3
H3C
CH3 ring
H3C CH3
flip
H3C
(More stable because large (Less stable because large
group is equatorial) group is axial)
(iv) In case of trans-1,3-disubstituted cyclohexane, lower energy conformation is that with the largest group equatorial.
H3C
CH3
H3C
CH3 ring
H3C
flip
H3C CH3
CH3
(More stable because large (Less stable because large
group is equatorial) group is axial)

Alkenes

1. Alkenes are unsaturated hydrocarbons containing a carbon–carbon double bond. They are also known as olefins and are represented
by the general formula CnH2n.
2. General methods of preparation
(a) By dehydration of alcohols
Al2O3, 623 K
RCH2 CH2OH −H2O
RCH CH2
Alcohol Alkene

conc. H2SO4, 453 K


RCH2 CH2OH −H2O
R CH CH2
Alcohol Alkene
Other dehydrating agents like H3PO4 and anhydrous ZnCl2 may also be used. The order of the ease of dehydration of alcohols is,
tertiary > secondary > primary.

Chapter-13.indd 422 8/8/2016 2:26:12 PM


Summary 423

(i) Acid catalyzed dehydration of secondary or tertiary alcohols: An E1 mechanism


Step 1: The alcohol accepts a proton from the acid in a fast step.
R R H
+
C C O H + H A C C O H + A−
fast
H R′ H R′
2° or 3° Alcohol Acid catalyst Protonated alcohol Conjugate
(R′ may be H) (typically sulphuric or base
phosphoric acid)

Step 2: The protonated alcohol loses a molecule of water to become a carbocation. This step is slow and rate determining.

R H R H
+
C C O H C C+ + O H
slow
(rate determining)
H R′ H R′
Step 3: The carbocation loses a proton to a base. 
R R
C C+ + A− C C + H A
fast
H R′ R′
Alkene
(ii) Dehydration primary alcohols: An E2 mechanism
Step 1: The alcohol accepts a proton from the acid in a fast step.
H H H
fast +
C C O H + H A C C O H + A−
H H H H
Primary Acid catalyst Protonated alcohol Conjugate
alcohol (typically sulphuric or base
phosphoric acid)

Step 2: A base removes a hydrogen from the b carbon as the double bond forms and the protonated hydroxyl group
departs. 
H H H H
+ slow
A− + C C O H C C +H A+ O H
(rate determining)
H H H
Alkene

(b) From haloalkanes


(i) Dehydrohalogenation of alkyl halides
X H
Heat
R C C R + KOH (alcoholic) R C C R + KX + H2O
H H H H
Haloalkane Alkene

If two alkenes may be formed due to dehydrohalogenation of an alkyl halide, the one which is most substituted is the main
and preferred product. This is known as Saytzeff’s rule (Zaitsev’s rule). For example, dehydrohalogenation of 2-bromobu-
tane gives,

Br H3C CH CH CH3
Diethylethene (disubtitued): Main product
KOH (alcoholic)
H3C CH CH2 CH3

H2C CH CH2 CH3


Ethylethene (monosubtitued): Minor product

Chapter-13.indd 423 8/8/2016 2:26:16 PM


424 Chapter 13 Aliphatic Hydrocarbons

The order of reactivity of haloalkanes in dehydrohalogenation is: Tertiary > Secondary > Primary.
Mechanism
Elimination occurs by E2 mechanism.

B H
b a E2
C C C C + B H + X−
X

Reaction conditions that favor elimination by an E1 mechanism should be avoided because the carbocation intermediate in
an E1 reaction can undergo rearrangement of the carbon skeleton, or it can also undergo substitution by an SN1 mechanism,
which competes strongly with product formation by E1 path.
(ii) From vicinal dihaloalkanes
H2C Br Heat CH2
+ Zn (methanol) + ZnBr2
H2C Br CH2

(c) From carboxylic acids: Kolbe’s electrolytic method

H2C COO− CH2


+ 2CO2(g) + 2e −
At the anode: H2C COO− CH2
At the cathode: 2H2O + 2e − 2OH− + H2(g)
(d) From alkynes
Ni, 200° C
CnH2n − 2 + H2 
→ CnH2n

(e) From carbonyl compounds


Reaction of (aldehydes and ketones) with phosphorus ylides yields alkenes and triphenylphosphine oxide as a by-product. This
reaction is known as the Wittig reaction.
R + − R″ R R″
C O + (C6H5)3P C C C + O P(C6H5)3
R′ R″ R′ R″
Aldehyde or Phosphorus ylide Alkene Triphenyl-
ketone (or phosphorane) [(E) and (Z) isomers] phosphine oxide

Mechanism
R″ R′ R″ R″
R R′ R′
+ − +
C C R′″ R C C R′″ C C O P(C6H5)3
P(C6H5)3 O P(C6H5)3 R R′″
O +
Aldehyde Ylide Oxaphosphetane Alkene Triphenylphosphine
or ketone (+ diastereomer) oxide

(f) The Cope elimination


Elimination of a dialkylhydroxylamine from tertiary amine oxides when heated yields alkenes.

R + Me H Me
N +
Me 150°C R O N
H O Me
A tertiary amine oxide An alkene N,N-Dimethylhydroxylamine

(g) The Hofmann elimination


Tetraalkylammonium hydroxide, upon heating strongly undergoes elimination of a b-hydrogen to give alkene along with a ter-
tiary amine.

Chapter-13.indd 424 8/8/2016 2:26:21 PM


Summary 425

HO − H
+ + HOH + NR3
NR3 Heat

A quaternary ammonium An alkene A tertiary


hydroxide amine

3. Physical properties
(a) Alkenes are non-polar compounds and the only attractive forces between their molecules are dispersion forces.
(b) They are weakly polar because the p- electrons of the double bond and have higher dipole moment than the corresponding
alkanes.
(c) The boiling points of unbranched alkenes are higher than the branched chain compounds.
(d) They are slightly soluble in water but dissolve in non-polar or low polarity solvents.
4. Chemical properties
Due to the presence of carbon-carbon double bond, they are more reactive than the alkanes. The most characteristic reaction of
alkenes is addition to the carbon–carbon double bond in which the p-bond is broken and, in its place, s -bonds are formed to two
new atoms or groups of atoms.
(a) Addition reactions

+d −d slow
C C + E − Nu C C + Nu−
+
E

Nu−
fast
C C + C C
+
E Nu E

(i) Addition of halogens


Alkenes react with halogens in an inert solvent like carbon tetrachloride to form 1, 2-dihalogenated products. The addition of
halogen to an alkene is anti-addition. The order of reactivity is, chlorine > bromine > iodine.

H H X X
R C C R + X2 R C C R
H H

Mechanism
Step 1: The alkene functions as a nucleophile and attacks molecular bromine, expelling bromide as a leaving group and forming
a bridged intermediate, called bromonium ion.


C C C C + Br

d + Br Br
+
d − Br Bromonium ion Bromide ion

Step 2: Bromide functions as a nucleophile and attacks the bromonium ion in an SN2 process.

Br

C C + Br C C + enantiomer
Br Br
+
Bromonium ion Bromide ion vic-Dibromide

Tip The reaction with bromine is used for detection of double bond. The colour of bromine water disappears on formation of dibro-
moalkane if carbon–carbon double bond is present.

Chapter-13.indd 425 8/8/2016 2:26:24 PM


426 Chapter 13 Aliphatic Hydrocarbons

(ii) Addition of hydrogen halides

C C + HX C C
H X

The order of reactivity is, HI > HBr > HCl


• Symmetrical alkenes give only one product, upon addition of HX.
CH2 H H2C X
+
CH2 X CH3

• In asymmetrical alkenes, the addition of a HX takes place in a manner where by the halogen atom adds to the carbon atom,
which has lesser number of hydrogen atoms on it (Markovnikov’s rule).

Alkene carbon CH2 CHCH3 CH2 CHCH3


atom with the
greater number of H Br
hydrogen atoms
H Br Markovnikov addition
product

• The addition of HBr to unsymmetrical alkenes in the presence of oxygen and peroxides is contrary to Markovnikov’s rule. This
is known as Peroxide effect or Kharasch effect. For example, propene reacts with HBr in the presence of peroxides, to form
1-bromopropane instead of 2-brompropane.
CH3−CH CH2 + HBr CH3−CH2−CH2Br
The mode of addition of hydrogen chloride or hydrogen iodide is not affected by the presence of peroxides.
(iii) Addition of sulphuric acid (in accordance to Markovnikov’s rule).
• Alkene donates a pair of electrons to a proton from sulphuric acid to form a carbocation.
• The carbocation reacts with a hydrogen sulphate ion to form an alkyl hydrogen sulphate

O H O HO3SO H
+ −
C C + H O S O H C C + O S O H C C
O O
Alkene Sulphuric Carbocation Hydrogen Alkyl hydrogen
acid sulphate ion sulphate

Soluble in sulphuric acid

(iv) Addition of water


Addition of water across a double in the presence of an acid is called acid-catalyzed hydration. The reaction proceeds via
Markovnikov’s addition.
H3PO4
CH3CH CH2 + H2O CH3 CH CH3
300οC, 65 atm
OH

Mechanism
Step 1: The alkene is protonated forming a carbocation intermediate.
Step 2: Water functions as a nucleophile an attacks the carbocation intermediate.
Step 3: Water functions as a base and deprotonates the oxonium ion, yielding the product.
Proton transfer Nucleophilic attack Proton transfer
H
H O+ H
O H O H O HO H
H H H H + H H
+

Carbocation Oxonium ion

Chapter-13.indd 426 8/8/2016 2:26:28 PM


Summary 427

(v) Addition of hypohalous acids (in accordance with Markovnikov’s rule)


CH2 H2C Cl
+ HOCl
CH2 H2C OH

(vi) Addition of oxygen


CH2 Ag catalyst H2 C
+ 1/2O2(g) ο
O
CH2 200–400 C
H 2C

(vii) Oxymercuration-Demercuration (in accordance with Markovnikov’s rule)


Step 1: In the first step, oxymercuration, water and mercuric acetate add to the double bond.

O O
CH3 H
C C + H2O + Hg OCCH3 C C O + CH3COH
2 THF
H H
HO Hg OCCH3

Step 2: In the second step, demercuration, sodium borohydride reduces the acetoxymercury group and replaces it with
hydrogen.

CH3 H CH3 H O

H C C HO + OH− + NaBH4 H C C H + Hg + CH3CO−

HO Hg OCCH3 HO H

(viii) Hydroboration-oxidation
• Anti-Markovnikov hydration of a double bond can be achieved through the use of diborane (B2H6) or a solution of
borane in tetrahydrofuran (BH3: THF).

BH3: THF H2O2/OH−


3 hydroboration B oxidation 3 OH
3
Propene 1-Propanol
Tripropylborane

• Hydroboration–oxidation takes place with syn stereochemistry, as well as anti-Markovnikov regiochemistry.


CH3
1. BH3: THF
CH3 H
2. H2O2, OH−
H
H OH

(ix) Addition of carbenes


R R
R
+ CCI2 + enantiomer
R
Cl Cl

(x) The Simmons–Smith reaction


The diiodomethane and zinc react to produce a carbene-like species called a carbenoid. The carbenoid then brings about
the stereospecific addition of a CH2 group directly to the double bond.
CH2I2 + Zn(Cu) → ICH2Znl
A carbenoid

Chapter-13.indd 427 8/8/2016 2:26:33 PM


428 Chapter 13 Aliphatic Hydrocarbons

(b) Oxidative cleavage of alkenes


(i) Treatment with hot basic potassium permanganate oxidatively cleaves the double bond of an alkene. (Bayer’s test for
unsaturation)

O O
KMnO4, OH−, H2O H3O+
CH3CH CHCH3 heat
2 C 2 C
CH3 O− CH3 OH
(cis or trans) Acetate ion Acetic acid

(ii) Cyclic product called ozonides is formed when alkenes add a molecule of ozone across the double bond in an inert solvent
like CCl4 or chloroform at low temperature. Ozonides on hydrolysis with water in the presence of a reducing agent give alde-
hydes. This reaction is termed as ozonolysis.

CH2 H2C O CH2 H2O/Zn


+ O3 2HCHO + H2O2
low temperature
CH2 O O

(c) Oxidation of alkenes


Alkenes undergo a number of reactions in which the carbon–carbon double bond is oxidized. 1,2-Dihydroxylation is an impor-
tant oxidative addition reaction of alkenes.
NaHSO3
C C Pyridine C C H2O C C
+ O O OH OH
O O Os +
Os Os
O O
O O
An osmate ester

(d) Polymerization
O2
nCH2 CH2 ( CH2 CH2 )n
(trace)

(e) Isomerization

500–700°C
CH3 CH2 CH CH2 CH3 C CH2
200–300°C
But-1-ene Cat.
CH3
Al2(SO4)3
2-Methylpropene
(Isobutane)

5. Reactions of conjugated dienes


(a) Conjugated dienes undergo both 1,2-additions and 1,4-addition reactions.
(b) Addition occurs via an allylic intermediate which is common to both.

1,2-Addition 1,4-Addition
Cl
HCl H + H
25°C Cl
1,3-Butadiene 3-Chloro-1-butene 1-Chloro-2-butene
(78%) (22%, primarily E)

Chapter-13.indd 428 8/8/2016 2:26:38 PM


Summary 429

Mechanism
Step 1:

+ +
+ −
Cl H + Cl

An allylic cation
equivalent to
d+ d+

Step 2:

Cl
(a) (a)
1,2-Addition
d+ d + (b) −
+ Cl
(b)
Cl 1,4-Addition

(c) Allylic substitution


When propene reacts with chlorine or bromine at very high temperatures or under conditions in which the concentration of the
halogen is very small, the reaction that occurs is a substitution.

high temperature
CH3 CH CH3 + X2 or low concentration of X2 CH3 CH CH2X + HX
(substitution reaction)

In this substitution a halogen atom replaces one of the hydrogen atoms of the methyl group of propene. These hydrogen atoms
are called the allylic hydrogen atoms, and the substitution reaction is known as an allylic substitution

H H
C C H
H
C Allylic hrydrogen atoms
H H

(i) Allylic chlorination


Propene undergoes allylic chlorination when propene and chlorine react in the gas phase at 400°C. This method for synthe-
sizing allyl chloride is called the “Shell process.”

400° C
CH3 = CH − CH3 + Cl2 gas
 phase
→ CH3 = CH − CH2Cl + HCl
3-Chloropropene

(ii) Allylic bromination


Propene undergoes allylic bromination when it is treated with N-bromosuccinimide (NBS) in CCl4 in the presence of perox-
ides or light.
O O

light or ROOR
CH2 CH CH3 + N Br CH2 CH CH2Br + N H
CCl4

O O
N-Bromosuccinimide 3-Bromopropane Succinimide
(NBS) (allyl bromide)

Chapter-13.indd 429 8/8/2016 2:26:42 PM


430 Chapter 13 Aliphatic Hydrocarbons

Tip N-Bromosuccinimide is nearly insoluble in CCl4 and provides a constant but very low concentration of bromine in the reaction
mixture. It does this by reacting very rapidly with the HBr formed in the substitution reaction. Each molecule of HBr is replaced
by one molecule of Br2:
O O

N Br + HBr N H + Br2

O O

Alkynes

1. Alkynes are unsaturated hydrocarbons containing carbon–carbon triple bonds. They have the general formula of CnH2n−2.
2. General methods of preparation
(a) From alkyl halides
(i) By dehydrohalogenation of vicinal dihalides (1,2-dihalides).
Vicinal dihalide Double dehydrohalogenation
formation H H
2NaNH2
RCH CHR + Br2 R C C R R C C R + 2NH3 + 2NaBr
Br Br
A vic-dibromide

(ii) By dehydrohalogenation of germinal dihalides (1,1-dihalides).


O CI
CH
1. 3 equiv.
C PCl5 C CH3 NaNH2
C
CH3
0°C CI mineral oil,
(−POCl3) heat
2. HA

Cyclohexyl methyl A gem-dichloride Cyclohexylacetylene


ketone (70–80%) (46%)

(b) By dehalogenation of vicinal tetrahalides


Zn/alcohol
CH3CBr2CHBr2 CH3C CH

(c) From calcium carbide


Ethyne can be synthesized commercially by reaction of calcium carbide with water.

CaCO3 
→ CaO + CO2
CaO + 3C → CaC2 + CO
CaCO2 + 2H2O → Ca(OH)2 + C2H2

3. Physical properties
(a) Alkynes also have properties similar to corresponding alkanes. Alkynes up to three carbons are gases at room temperature.
(b) The boiling points, melting points and densities increase with increase in molecular mass.
(c) They are non-polar and dissolve in non-polar or low polarity solvents.
(d) They are only slightly soluble in water but show better solubility than corresponding alkenes.
4. Chemical properties
(a) Acidic character of alkynes
(i) The electronegativity of differently hybridized carbon atoms follow the order, sp > sp2 > sp3. This increase in the electronega-
tivity of an alkyne carbon, (in comparison to that in alkenes and alkanes) polarize the C H electron bond towards carbon and
facilitate the release of proton(s). Therefore, the acid strength of hydrogens will follow the order, Alkynes > Alkenes > Alkanes.

Chapter-13.indd 430 8/8/2016 2:26:46 PM


Summary 431

(ii) The stabilities of the anion left after the removal of proton, that is, carbanions follow the order, RC C− > RCH CH− > R CH2
CH2− .
(iii) Sodium ethynide and other sodium alkynides can be prepared by treating terminal alkynes with sodium amide in liquid
ammonia.
CH3C C H + NaNH2 CH3C C: −Na+ + NH3
liq. NH3

(b) Addition reactions


(i) Hydrogenation
when a platinum catalyst is used, the alkyne generally reacts with two molar equivalents of hydrogen to give an alkane.
+H2 +H2
R C CH Pt/Ni
R CH CH2 Pt/Ni
R CH2 CH3
Alkyne Alkene Alkane

• Syn addition of hydrogen: Synthesis of cis-alkenes


Metallic palladium deposited on calcium carbonate can cause syn addition of hydrogen after it has been conditioned
with lead acetate and quinoline. This special catalyst is known as Lindlar’s catalyst.
R R
H2, Pd/CaCO3
(Lindlar’s catalyst)
R C C R C C
quinoline
(syn addition) H H

• Anti addition of hydrogen: Synthesis of trans-alkenes


An anti-addition of hydrogen atoms to the triple bond occurs when alkynes are reduced with lithium or sodium metal
in ammonia or ethylamine at low temperatures. This reaction, called dissolving metal reduction, takes place in solution
and produces an (E )- or trans-alkene.
CH3(CH2)2 H
1. Li, C2H5NH2, −78°C
CH3(CH2)2 C C (CH2)2CH3 C C
2. NH4Cl
H (CH2)2CH3
4-Octyne (E)-4-Octene
(trans-4-octene)
(52%)

(ii) Addition of halogens


• Alkynes show the same kind of addition reactions with chlorine and bromine that alkenes do.

Br Br
Br
Br2 Br2
C C C C C C
CCl4 CCl4
Br Br Br
Dibromoalkene Tetrabromoalkane

• Addition of one molar equivalent of chlorine or bromine to an alkyne generally results in anti-addition and yields
a trans-dihaloalkene.
HO2C Br
Br2
HO2C C C CO2H C C
(1 mol)
Br CO2H
Acetylenedicarboxylic (70%)
acid

(iii) Addition of hydrogen halides (in accordance with the Markovnikov’s rule)

+ HX
R C + HX R CX R CX2
CH CH2 CH3

Chapter-13.indd 431 8/8/2016 2:27:01 PM


432 Chapter 13 Aliphatic Hydrocarbons

• The rate of addition of halogen acids follows the order, HI > HBr > HCl.
• In the presence of peroxides, the addition of HBr takes place according to the anti-Markovnikov’s rule.
(iv) Addition of hypochlorous acid (in accordance with the Markovnikov’s rule).

HOCl +HOCl
R C + R C OH R C (OH)2 −H2O
R C O

CH CHX CHX2 CHX2


Alkyne Dihaloketone

(v) Addition of sulphuric acid

CH CH2 +H2SO4 CH3


+ H2SO4
CH CH·O·SO2·OH CH(O·SO2·OH)2
Ethyne Vinyl hydrogen Ethylidene
(acetylene) sulphate hydrogen

(vi) Addition of water

OH O
H2SO4, H2O
R HgSO4
R R
Enol Ketone
(Not isolated)

Mechanism

Proton transfer Proton transfer


H
H O+
H + H
OH O O O O
H H H
+
Enol Resonance-stabilized Ketone
intermediate

• The enol and ketone are said to be tautomers, which are constitutional isomers that rapidly interconvert via the migra-
tion of a proton. The interconversion between an enol and a ketone is called keto-enol tautomerization.
• Tautomerization is an equilibrium process, which means that the equilibrium will establish specific concentrations for
both the enol and the ketone. Generally, the ketone is highly favored, and the concentration of enol will be quite small.
(vii) Addition of HCN
Ba(CN)2
R C + HCN R C CN

CH CH2

(viii) Hydroboration-oxidation
H OH O
1. BH3•
R 2. H2O2, NaOH
R R H

(c) Oxidative cleavage of alkynes


Treating alkynes with ozone followed by acetic acid, or with basic potassium permanganate followed by acid, leads to cleavage
at the carbon–carbon triple bond. 
O
H2O
R C + O3 R C C R′ R C C R′ + H2O2 RCOOH + R′COOH

CR′ O O O O

Chapter-13.indd 432 8/8/2016 2:27:07 PM


Tools for Problem Solving 433

(d) Polymerization

Hot Cu tube
CH C6H6 or
3
CH

CH Cu2Cl2/NH4Cl HC CH
H2C CH C CH H2C CH C C CH CH2
CH
Vinyl acetylene Divinyl acetylene

(e) Oxidation
Alkaline KMnO4 oxidizes ethyne to oxalic acid:
CH COOH
+ O2
CH COOH

On treating with chromic acid, ethyne is oxidized to acetic acid.


CH CH3
+ [O] + H2O
CH COOH
Homologues of ethyne on oxidation with alkaline KMnO4 give mixture of acids.
R C KMnO4(alkaline)
+ [O] + H2O RCOOH + R′COOH
C R′

TOOLS FOR PROBLEM SOLVING


H X H R2 X R2 X X
HX X2
R1 C C R2 (2nd molar equivalent) C C C C R1 C C R2
(2nd molar
Hydro halogenation R X R1 X equivalent)
H X (If R1 in alkyne is H X X
Halogenation
than addition is
Markovnikov) HX X2
(1st molar
equivalent) (1st molar
Hydro halogenation equivalent)
halogenation
(1) KMnO4, HO−, heat; (2) H3O+
R1 C C R2 or
1. X2 1. X2 (1) O3; (2) HOAc
2. NaNH2 2. NaNH2 Ozonolysis
(2 equiv.). (2 equiv.).
heat heat
H2 or
Lindlar’s 1. Na EtNH2
R1 R2 R1 2. NH4Cl R1 H 1. KMnO4. R1 OH
1. O3 R2 catalyst
HO−, heat
C O + O C C C C C C O + O C
2. Me2S Hydrogenation 2. H3O+
H H H H H R2 HO R2
Ozonolysis (Z) (E)
H2 Catalyst
CH2I2, Zn(Cu) (Pt, Pd, CH2I2.Zn(Cu)
(or other methods) Rh, Ni) (or other methods)
Carbene Carbene
addition H H addition

R1 C C R2
R1 R2 R1 H
H H
H H H R2

Chapter-13.indd 433 8/8/2016 2:27:10 PM


434 Chapter 13 Aliphatic Hydrocarbons

SOLVED EXAMPLES
Alkanes In option (b) methyl group is equatorial while chloro
group is in axial position.
General Methods of Preparation In option (c) both the methyl and chloro groups are
1. Which of the following reaction would not produce saturated equatorial.
hydrocarbon as a major product? In option (d) methyl group is axial while chloro group is
(a) CH3CH2Br + Na 

ether equatorial.

Zn(Hg)/HCl, ∆ Physical Properties


(b) CH3COCH3 →
4. Which of following have zero dipole moment?
N H /KOH
(c) CH3CHO 
2 4

LiAlH (a) Cl Cl (b) Cl Cl
(d) CH3COOCH3 
4

Cl
Solution
(d) Major products of the reactions are as follows:
ether (c) Cl (d) None of these
CH3CH2Br + Na 
→ CH3CH2CH2CH3
Zn(Hg)/HCl, ∆
CH3COCH3 → CH3CH2CH3 Solution
N H /KOH
CH3CHO 
2 4
→ CH3CH3 (d) Cyclobutane exists in puckered conformation to eliminate
LiAlH4 its torsional strain with two hydrogen atom in this con-
CH3COOCH3 → CH3CH2OH
formation. The individual dipole moments of C Cl bonds
in trans-1,3-dichlorocyclobutane do not fully cancel each
Conformations
other hence net resultant dipole moment is non-zero.
2. Which one of the following conformations of cyclohexane is Cl H
chiral?
(a) Twist boat (b) Rigid
H Cl m≠0
(c) Chair (d) Boat
(AIEEE 2007)
Solution
Similarly in cis-1,3-dichlorobutane net dipole is non-zero.
(a) The twist boat conformation is chiral as it does not have Cl Cl
plane of symmetry.
1 4
H H m≠0
6 3

5 2 In 1,2-dichlorocyclobutane resultant dipole moment is


also non-zero.
3. Which represents the chair structure of the following with Cl
methyl group axial? Cl
H
H
m≠0
Cl

(a) Cl (b) Cl So in none of the option net dipole moment is zero.

Tip Cyclobutane exist in puckered conformation which is non-


planner, and hence none of individual dipole cancels out each
other in disubstituted product.

(c) Cl (d) Cl Chemical Properties


5. How many chiral compounds are possible on monochlorina-
Solution tion of 2-methylbutane?
(d) In option (a) both the methyl and chloro groups are in (a) 8 (b) 2 (c) 4 (d) 6
axial position. (AIEEE 2012)

Chapter-13.indd 434 8/8/2016 2:27:21 PM


Solved Examples 435

Solution CH2CH3 CH2


(c) The reaction involved is (c) (d)
1 2 3 4
CH3 CH CH2 CH3 CH2CH3 CH2CH3

CH3 Solution
2-Methylbutane (d) All alkenes contain single double bond, so one equiv-
alent of hydrogen can be added to each alkene. After
Cl2/hv or heat hydrogenation, almost all products are equally stable but
reactants are not equally stable. The hydrogenation reac-
Cl tion is exothermic, so more energy will release from the
most unstable alkene.
CH3 CH CH2 CH2Cl + CH3 C CH2 CH3 The stability of alkenes can be explained by hypercon-
jugation which is directly proportional to alpha hydrogen
CH3 CH3
atoms. The alpha hydrogen atoms in (a), (b), (c) and (d)
(Achiral) (Achiral) options are 5, 8, 4 and 3, respectively. Hence, the alkene
∗ ∗ given in option (d) is the least stable for which heat of
+ H3C CH CH2 CH3 + CH3 CH CH CH3
hydrogenation will be the maximum.
CH2Cl CH3 Cl Tip/Alternate Solution Just look for the least stable alkene by
(Chiral) (Chiral) counting the least number of alpha hydrogens.
6. Which branched chain isomer of the hydrocarbon with molec-
ular mass 72 u gives only one isomer of monosubstituted alkyl 9. For the following reaction
halide? C5H12 + Cl2 
hn
→ C5H11Cl →(C )
Na / ether
(a) Tertiary butyl chloride (b) Neopentane (A ) (B)
(c) Isohexane (d) Neohexane
(AIEEE 2012) the products (A), (B) and (C) are
Solution CH3 CH3
(b) The reaction involved is (a) CH3 C CH3 CH3 C CH2Cl
CH3 CH3
CH3 CH3
Cl2/hv
CH3 C CH3 CH3 C CH2Cl
CH3 CH3
CH3 CH3
CH3 C CH2 CH2 C CH3
Neopentane Monosubstituted alkyl halide
All hydrogens are equivalent CH3 CH3
Molecular mass = C5H12 = 12 × 5 + 12 = 72 u
CH3 CH3
7. The major product obtained in the photocatalyzed bromina-
(b) CH CH CH2CH3 CH3 C CH2CH3
tion of 2-methylbutane is 3
(a) 1-bromo-2-methylbutane.
Cl
(b) 1-bromo-3-methylbutane. CH3 CH3
(c) 2-bromo-3-methylbutane.
(d) 2-bromo-2-methylbutane. CH3 CH2 C C CH2CH3
(JEE Main Online 2014)
CH3 CH3
Solution
(c) both (a) and (b)
(d) The reaction is
CH3 (d) None of these
CH3
Br2 Solution
CH3 CH CH2 CH3 CH3 C CH2 CH3 CH3 CH3
hv
Cl2 Na/ether
Br (a) CH3 C CH3 CH3 C CH2Cl
hn Wurtz
2-Bromo-2-methylbutane CH3 CH3
reaction
(major)
Neopentane Neopentyl chloride
8. Which one of the following alkenes has the maximum heat of (A) (B)
hydrogenation?
CH3 CH3
CH2CH3 CH2CH3
(a) (b) CH3 C CH2 CH2 C CH3
CH2CH3 CH2CH3 CH3 CH3
(C)

Chapter-13.indd 435 8/8/2016 2:27:29 PM


436 Chapter 13 Aliphatic Hydrocarbons

Alkenes Solution
General Methods of Preparation (c) For elimination, strong base is required. But more hin-
dered base will produce less substituted alkene.
10.
Me t-BuOK
+
∆ t-BuOH, 75°C
Me Br
+
N 2-Methyl-2-butene 2-Methyl-1-butene
Et (27.5%) (72.5%)
OH− n-Bu (more substituted) (less substituted)
The alkene formed as a major product in the above elimina- 13. What is the reagent needed for the following reaction?
tion reaction is OH
(a) Me (b) CH2 CH2
Me Me
(c) (d)
(a) concentrated sulphuric acid and high heat.
(b) 85% phosphoric acid and high heat.
(AIEEE 2006) (c) dilute sulphuric acid and heat.
(d) Both (a) and (b).
Solution
Solution
(d) From Hofmann’s rule, the alkene having least alkylated
(b) The temperature and concentration of acid required to
double bond is the major product.
dehydrate an alcohol depend on the structure of the
11. The gas liberated by the electrolysis of dipotassium succinate alcohol substrate.
solution is Primary alcohols are the most difficult to dehydrate. They
(a) ethane. (b) ethyne. require concentrated sulphuric acid and a temperature of
(c) ethene. (d) propene. 180°C in order to dehydrate.
(JEE Main Online 2014) Secondary alcohols can be dehydrated using 85% phos-
phoric acid at 165–170°C.
Solution
OH
(c)
85% H3PO4
CH2 COOK CH2 COO− 165−170°C
+ 2K+
CH2 COOK CH2 COO−
Chemical Properties
H2O  OH− + H+
14. HBr reacts with CH2 CH OCH3 under anhydrous condi-
+ 1 − tions at room temperature to give
At the cathode: H + e → H2
2 (a) CH3CHO and CH3Br (b) BrCH2CHO and CH3
At the anode: (c) BrCH2 CH2 OCH3 (d) H3C CHBr OCH3
O (AIEEE 2006)
Solution
CH2 COO− CH2 C O CH2
(d) The reaction follows Markovnikov’s rule.
(g) + 2CO2(g)
CH2 COO− CH2 C O CH2 HBr +
H2C CH OCH3 H2C CH OCH3
O
H Br −
12. Which of the following conditions/reagents would you Br
employ to obtain the best yields in the following reaction?
H3C CH OCH3

15. In the following sequence of reactions, the alkene forms the


Br
compound B
(a) NaOH(aq), heat O HO
CH3CH CHCH3 
3
→ A 2
Zn
→B
(b) C2H5ONa/C2H5OH, heat
The compound B is
(c) (CH3 )3 COK/(CH3 )3 COH, heat (a) CH3CH2CHO (b) CH3COCH3
(c) CH3CH2COCH3 (d) CH3CHO
(d) H2SO 4 , heat (AIEEE 2008)

Chapter-13.indd 436 8/8/2016 2:27:40 PM


Solved Examples 437

Solution The product formed is opposite to Markovnikov’s regio-


(d) This is an example of reductive ozonolysis. selectivity.

O 19. In the presence of peroxide, HCl and HI do not give anti-


O3
H3C CH CH CH3 H3C CH CH CH3 (Ozonide) Markovnikov’s rule addition to alkenes because
(A) (a) one of the steps is endothermic in HCl and HI.
O O (b) both HCl and HI are strong acids.
H2O/Zn (c) HCl is oxidizing and the HI is reducing.
O (d) all the steps are exothermic in HCl and HI.
(2 mol) (JEE Main Online 2014)
C Solution
H3C H
(B) (a) Peroxides do not give anti-Markovnikov’s addition
because one of the steps is endothermic in HCl and HI.
16. One mole of a symmetrical alkene on ozonolysis gives two moles
of an aldehyde having a molecular mass of 44 u. The alkene is 20. Which of the following products is not possible when
(a) ethane. (b) propene. cyclohexene reacts with bromine in the presence of brine?
(c) 1-butene. (d) 2-butene. Br Br
(AIEEE 2010)
(a) (b)
Solution
(d) The reaction is OH
Br
O3 Br Cl
CH3 CH CH CH3 Zn/H2O
2CH3CHO
2-Butane (c) (d)
Molecular mass
(Symmetrical (24 + 16 + 4 = 44 u)
alkene) Cl Cl

H H H H Solution
(d) Brine is 10% aqueous solution of NaCl, so the reaction
CH3 C O+O C CH3 CH3 C C CH3
mixture contains Br2, Cl− and H2O, out of which Br2 can
2-Butene initiate the reaction since the first step of addition is elec-
17. Ozonolysis of an organic compound gives formaldehyde as trophilic in nature. For the second step, a nucleophile is
one of the products. This confirms the presence of required for which Br−, Cl− and H2O are available and lead
(a) a vinyl group. to the following products.
(b) an isopropyl group.
Br Br Br
(c) an acetylenic triple bond. Br2, NaCl, H2O
(d) two ethylenic double bonds. + +
(AIEEE 2011)
Br OH Cl
Solution
(a) Vinyl group gives formaldehyde on ozonolysis which Formation of 1,2-dichlorocyclohexane is not possible in
confirms the presence of vinylic group –CH CH2 in the this reaction since the first step in the addition to double
compound. bond involves an electrophilic attack and chlorine is not
O available as Cl+ (electrophile) in the reaction mixture.
Zn
CH2 CH2 + O3 CH2 CH HCHO + HCHO 21. 1-Butene reacts with the following reagents given in the
H2O
options. Out of these, which will react through free radical
O O mechanism?
(a) HCl + peroxide (b) (i) BH3, (ii) H2O2, OH−
18. In the hydroboration – oxidation reaction of propene with (c) HBr (d) HBr + peroxide
diborane, H2O2 and NaOH, the organic compound formed is
(a) CH3CH2OH (b) CH3CHOHCH3 Solution
(c) CH3CH2CH2OH (d) (CH3)3COH (d) Electrophilic addition reactions are the characteristic
(JEE Main Online 2014) reactions of alkenes and these reactions are ionic in
nature, so, the reaction is completed by electrophiles and
Solution
nucleophiles. However, in the case of presence of perox-
(c) ide, reaction takes place through free radical formation.
1. B2H6 This behaviour is observed only in the case of addition
CH3 CH CH2 CH3 CH2 CH2 OH
2. H2O2/OH− of HBr.

Chapter-13.indd 437 8/8/2016 2:27:55 PM


438 Chapter 13 Aliphatic Hydrocarbons

Solution
light (c)
RO OR or
2RO
∆ H+
an alkyl peroxide an alkoxy radical
+

R O + H Br R O H + Br
a bromine
radical
OH
Br + CH2 CHCH2CH3 CH2CHCH2CH3 +
OH−
Br

CH2CHCH2CH3 + H Br CH2 CHCH2CH3 + Br Tip Addition is according Markovnikov’s rule but we have to
Br Br H look for rearrangement of carbocation if it gives more stable
carbocation.
22. The correct order of reactivity of the following alkenes to 24. 3-Phenylpropene on reaction with HBr gives (as a major product)
acid-catalyzed hydration is (a) C6H5CH2CH(Br)CH3 (b) C6H5CH(Br)CH2CH3
(a) (CH3)3C CH2 > H3CHC CH2 > H2C CH2 (c) C6H5CH2CH2CH2Br (d) C6H5CH(Br)CH CH2
Solution
(b) H3CHC CH2 > (CH3)3C CH2 > H2C CH2
(b)
(c) H2C CH2 > (CH3)3C CH2 > H3CHC CH2 H
+
(d) H2C CH2 > H3CHC CH2 > (CH3)3C CH2 CH2 CH CH2 CH CH CH3
HBr
(2°)
Solution
(a) Acid catalyzed hydration is the addition of water mole-
cule in the presence of acid. It occurs through carboca- Br
+
tion formation in the first step which is also slow step, CH CH3 CH CH2 CH3
so, more stable carbocation formation indicates faster CH2 Br−
reaction.
+
H+ (Major product)
H2C = CH2 
→ H2 C CH3 (1° carbocation, least stable)
+
H+
H3CHC = CH2 → CH3 CHCH3 (2° carbocation, moder- 25. An unsaturated hydrocarbon upon ozonolysis gives one
ately stable) mole each of formaldehyde, acetaldehyde and methylglyoxal
+ (CH3COCHO). The structure of the hydrocarbon is
((H )22C = CH2 H→ (CH3 )2 C CH3
+

H33CC) (3° carbocation, most (a) CH2 CH CH2 CH CH2


stable) (b) CH2 CH C(CH3) CH CH3
Thus the order of reactivity (CH33)3C = CH2 >
is (CH (c) (CH3)2C CH CH3
H3CHC = CH2 > H2C = CH2 (d) CH3 CH C(CH3) CH3

23. The major product of the reaction given below is Solution


(b) The reaction is
H2SO4, H2O
O3/Zn
H2C CH C(CH3) CH CH3 H2C O
Formaldehyde
OH
+
CH3
(a) OH (b) O CH CH3 + O CH C O
OH
Acetaldehyde Methyl glyoxal
OH
(c) (d)
26. Which of the following will react with sodium metal?
(a) Ethene (b) Propyne
(c) But-2-yne (d) Ethane

Chapter-13.indd 438 8/8/2016 2:28:04 PM


Solved Examples 439

Solution Solution
(b) Terminal alkynes react with sodium metal to form sodium (d)
alkynides. +
Na
− + H+ +
→ H3C C C Na + 21 H2
H3C C C − H 
Propyne NGP H
Sodium propylene

27. What is the product of the following reaction? Hydride Hydride


shift shift
1. O3, CH2Cl2, − 78°C
2. Zn/AcOH +
H H H +
(a) + (b) + CO2
O O O 3° Carbocation
more stable
H OH OH
(c) + (d) +
Br− Br−
O O O O

Solution Br
(c) Ozonolysis of a trisubstituted alkene produces an alde- Br
hyde and a ketone.
H O 30. What is the major product of the following reaction?
1. O3, CH2Cl2
O 2. Zn/AcOH
−78°C
O O +H
O
NaOMe/MeOH
(Ozonide)

Cl
28. What product would you expect from addition of deuterium
chloride to 2-cyclohexyl-4-methyl-2-pentene?

Cl (a) (b)
D
(a) (b)
D H
Cl
D Cl

(c) (d) (c) (d)


Cl
D
Solution
(b) According to Markovnikov’s rule, deuterium (D) will get
attached to the carbon with fewer alkyl substituents and Solution
the chloride (Cl) will get attached to the carbon with
more alkyl substituents. (b) In a cyclic structure elimination occurs when both the
Cl leaving group and the b hydrogen are axial. The major
product arises from the elimination of the only b hydro-
DCl gen that is trans to the leaving group.
D H
Quick Tip If your leaving group is wedged it will need
2-Cyclohexyl-4-methyl-2-pentene dashed hydrogen for the anti-coplanar elimination. Likewise,
29. What are possible products for the following reaction? if your leaving group is dashed it will need wedged hydrogen
for anti-coplanar elimination.
HBr
31. What is the product of the following reaction?
Br Br2

Br Br Br
(a) (b)
+ Enantiomer

Br (a) (b)

(c) (d) Both (a) and (c) Br Br

Chapter-13.indd 439 8/8/2016 2:28:18 PM


440 Chapter 13 Aliphatic Hydrocarbons

Br OH Thus, CH3MgX acts as a base to abstract acidic


hydrogen.
(c) (d) 35. 2-Hexyne gives trans-2-hexene on treatment with
(a) Pt/H2 (b) Li/NH3 (c) Pd/BaSO4 (d) LiAlH4
Br Br (AIEEE 2012)
Solution Solution
(c) When a dihalide is added to a double bond, the double (b) An anti addition of hydrogen atoms to the triple bond
bond is replaced by the two halogens, which add anti to occurs when alkynes are reduced with lithium or sodium
each other. metal in ammonia at low temperature.
Br H CH3
Li/NH3
CH3(CH2)2 C C CH3 C C
Br2
2-Hexyne CH3 CH2 CH2 H

Br trans-2-Hexane
In this reaction, trans product is also formed because of
Alkynes an equilibrium between cis and trans alkenyl radicals. The
Chemical Properties trans radical is formed because it is more stable.
36. The reagent needed for converting
32. Which of the following reactions will yield
2,2-dibromopropane? Ph H
(a) CH3 C CH + 2HBr C C
Ph C C Ph
(b) CH3CH CHBr + HBr
(c) CH CH + 2HBr H Ph
(d) CH3 CH CH2+ HBr (AIEEE 2007) is
Solution (a) cat. hydrogenation (b) H2/Lindlar Catalyst
(c) Li/NH3 (d) LiAlH4
(a) The reaction is
(Markovnikov’s addition) (JEE Main Online 2014)
CH3 C CH + HBr CH3 C CH2
Solution
Br (c) The reaction is
(Markovnikov’s
addition)
HBr Ph Ph H
Li/NH3 _ NH3
Br Ph C C Ph _ C C C C
Ph H Ph
CH3 C CH3
Br 37. Which of the following ketones could not be prepared by an
2,2-Dibromopropane acid catalyzed hydration of an alkyne?
O O
33. The hydrocarbon which can react with sodium in liquid
ammonia is (a) (b)
(a) CH3CH2CH2C CCH2CH2CH3
(b) CH3CH2C CH O
O
(c) CH3CH CHCH3
(c) (d)
(d) CH3CH2C CCH2CH3
(AIEEE 2008)
Solution
(b) Terminals alkyne are acidic hence they will react with Na Solution
in presence of liquid ammonia. (c) Alkynes on reacting with water in the presence of water
NH3 initially form an enol, which on tautomerization gives
CH3 CH2 C CH + Na 
→ CH3 CH2 C C−Na+
ketone. So, in this structure we have to check whether an
34. The treatment of CH3MgX with CH3C C H produces alkyne is possible for the given structure of ketone.
(a) CH3 CH CH2 (b) CH3C C CH3 H O+ Tautomerization
(c) CH3CH CHCH3 (d) CH4 Alkynes 
3
→ Enol → Ketone
(AIEEE 2008) R C C R + H2O
Solution H+
(d) Grignard reagents react with active H compound and lib- R C C R R CH2 C R
erate alkane from it.
CH3 MgX + CH3 C CH→CH4↑ + CH3 C C−MgX+ H OH O

Chapter-13.indd 440 8/8/2016 2:28:26 PM


Advanced Level Problems 441

For the given ketones, all alkynes should be internal Solution


alkynes. Overall change during this reaction is that triple (c) The reaction is
bond converts into single bond and one double bond cold KMnO4 (alk.)
appears at one carbon of triple bond and two hydrogen H3C CH2 C CH H3 C CH2 COOH + CO2
atoms attached with second carbon atom of triple bond.
So, alkynes can be drawn by placing triple bond at car- 40. Addition of hypochlorous acid to propyne gives
bonyl carbon. In this way, we cannot place a triple bond Cl
O
in the compound given in option (c). So, this ketone can- (a) Cl2HC (b) H3C
not be formed by alkynes on hydration.
H
Tip/Alternate Solution Just look for minimum two hydrogen Cl
atoms at alpha position. If they exist, then alkyne formation
O OH Cl
is possible with the same skeleton and we can also form a
ketone from it. (c) H3C (d) H3C
Cl Cl OH
38. The major result of treating 1-butyne with 6 M aqueous NaOH Cl
would be
Solution
(a) the production of the sodium acetylide.
(b) the production of an alkene. (c) The reaction is
(c) the production of an enol. CH3 C CH + HOCl H3C C CH
(d) nothing, as the alkyne would not react. Propyne
OH Cl
Solution
HOCl
(d) The terminal alkynes are very less acidic, the pKa value is
around 25. NaOH is not as much as basic so that it can O Cl OH Cl
deprotonate alkynes. Hence, there are no major changes −H2O
H3C C CH Cl H3C C CH
occurring in this reaction.
1,1-Dichloropropan-2-one
39. 1-Butyne reacts with cold alkaline KMnO4 to produce OH Cl
(a) CH3CH2CH2COOH (b) CH3CH2COOH Unstable
(c) CH3CH2COOH + CO2 (d) CH3CH2COOH + HCOOH

ADVANCED LEVEL PROBLEMS

1. Indentify the adduct formed in the following reaction. 2. The number of stereoisomers obtained by bromination of
trans-2-butene is
CH3 CH CH2 + NOCl
(a) 1 (b) 2 (c) 3 (d) 4
CH3 CH CH2 CH3 CH CH2 (IIT-JEE 2007)
(a) (b) Solution
Cl NO NO Cl (a)
NO
CH3 CH3
CH3 CH2 CH CH2 CH2 CH2 H3C H
(c) (d) + Br2 H C C H
Cl NO Cl C C

(IIT-JEE 2006) H CH3 Br Br


Solution
or
(a)
H H
CH3 CH CH2 + NO+ Cl− CH3 CH CH2
H3C C C CH3
Cl NO
Br Br
This reaction follows Markonikov’s addition according
to which the negative part (i.e. Cl−) goes to the carbon
The compound obtained is a meso compound. Meso
atom having lesser number of hydrogen atoms and the
compounds are those which are superimposable on
positive part goes to the carbon having more number of
their mirror image even though they contain chiral
hydrogen atoms.
centres.

Chapter-13.indd 441 8/8/2016 2:28:32 PM


442 Chapter 13 Aliphatic Hydrocarbons

3. In the Newman projection for 2,2-dimethylbutane X and Y can CH3


respectively be
(a) H and H (b) H and C2H5 C
(c) C2H5 and H (d) CH3 and CH3 CH3CH2 CH2CH3
H
X
H3C CH3 (IIT-JEE 2011)
Solution
H H
(8)
Y
H3C CH2 CH CH2 CH3 + Cl2
(IIT-JEE 2010)
Solution CH3 Monochlorination

(b), (d) ∗
CH3 CH2 CH2 CH CH2 CH3
4 3 2 1
H3C CH2 C CH3 Cl CH3
(1 chiral carbon)
CH3 No. of enantiomeric
pair = 21 = 2
On C2–C3 bond axis X = CH3, Y = CH3
Cl
On C1–C2 bond axis X = H, Y = C2H5 ∗ ∗
+ H3C CH CH CH2 CH3 + H3C CH2 C CH2 CH3
4. The total number of alkenes possible by dehydrobromination
of 3-bromo-3-cyclopentylhexane using alcoholic KOH is ___. Cl CH3 CH3
(IIT-JEE 2011) (2 chiral carbons) (No chiral carbon)
Solution No. of enantiomeric 20 = 1
pair = 22 = 4
(5) The structures of five possible alkenes are as follows:

H3C CH2 CH2 H3C CH2 CH CH2CH3


H
CH2Cl
C C
(No chiral carbon)
CH3 20 = 1

H3C CH2 CH2 Total = 2 + 4 + 1 + 1 = 8


CH3
6. The number of optically active products obtained from the
C C
complete ozonolysis of the given compound is ___.
H
CH3 H
H3C CH2 CH2 CH3
H3C CH CH C CH CH C CH CH CH3
C C
H CH3
H
(a) 0 (b) 1 (c) 2 (d) 4
H CH2 CH3 (IIT-JEE 2012)
C C Solution
CH3 CH2 (a) The reaction is

H3C CH2 CH2 C CH2 CH3 CH3 H

H3C CH CH C CH CH C CH CH CH3

H CH3
5. The maximum number of isomers (including stereoisomers)
that are possible on monochlorination of the following com- O3
pound, is ___.
CH3 H
O O O
H3C CH CH C CH CH C CH CH CH3

O O H O O CH3 O O
Chapter-13.indd 442 8/8/2016 2:28:37 PM
CH3 H

H3C CH CH C CH CH C CH CH CH3

H CH3
Advanced Level Problems 443
O3

CH3 H Cl
O O O Br CH3
H3C CH CH C CH CH C CH CH CH3 Br Cl

O O O O CH3 O O CH3
H

Zn/H2O (JEE Advanced 2014)


Solution
CH3
CH3 CH3 CH3
H3C CH O + O CH C CH O Br Cl H3C Br Cl CH3
(3)
H Br Cl Br Cl Br Cl
Achiral Achiral CH3 Cl Br
H
9. Isomers of hexane, based on their branching, can be divided
+ O CH C CH O + CH3 CH O into three distinct classes as shown in the figure.

CH3
Achiral Achiral and

7. Match the chemical conversions in Column I with the appro-


priate reagents in Column II. (I)

Column I Column II
(p) 1. Hg(OAc)2; and
(a) Cl
2. NaBH4
(q) NaOEt
(b) ONa OEt (II) (III)

OH (r) EtBr The correct order of their boiling point is


(c) (a) I > II > III (b) III > II > I
(c) II > III > I (d) III > I > II
(s) 1. BH3; (JEE Advanced 2014)
(d) 2. H2O2/NaOH Solution
(b) As the branching increases, the boiling point decreases. This
OH is because the van der Waals forces of attraction decrease.

(IIT-JEE 2013) 10. Compound(s) that on hydrogenation produce(s) optically


inactive compound(s) is (are)
Solution
(a) → q; (b) → r; (c) → p; (d) → s H Br
H Br
The reactions involved are
CH3 H2C CH3
NaOEt (a) H3C (b)
(a) Cl E2 reaction
H Br
EtBr H2C
(b) ONa OEt SN2 reaction CH3 Br H
CH3 H2C CH3
OH (c) (d)
1. Hg(OAc)2
(c) Markovnikov addition
2. NaBH4 (JEE Advanced 2015)
Solution
1. BH3
(d) 2. H2O2/NaOH Anti-Markovnikov addition (b), (d)
H Br H Br
H2/Ni
OH
(a) CH CH3 CH3 * CH3
3
8. The total number of stable conformers with non-zero dipole
Optically active
moment for the following compound is ___.

Chapter-13.indd 443 8/8/2016 2:28:54 PM


444 Chapter 13 Aliphatic Hydrocarbons

H Br H2/Ni H Br meso compound and optically inactive. The product


(b) H2C CH3 structures are as follows:
H Cl H Cl Cl Cl
Optically inactive Cl2
Cl +
H Br H Br
H2/Ni 2-Chlorobutane A B
(c) H2C H3C
* CH3
CH3 +
CH3 H Cl Cl Cl H Cl H Cl
Optically active H H
+ +
Br Br H Cl
H2/Ni E D C
(d) H C CH3
2
Hence, a total of five compounds are produced. Since all
Optically inactive five have different boiling points, they can be separated
by distillation, so five fractions are obtained in fractional
11. In the following reaction, the major product is
distillation.
CH3
1 equivalent HBr 13. Find the major product formed in the given reaction.
CH2
H2C
H2SO2 (Conc.)
CH3 CH3
CH3 O
(a) H2C (b) H C
3
Br
Br
(a) HO (b)
H3C CH3
(c) (d)
H2C Br H3C Br
(JEE Advanced 2015)
Solution (c) (d)
(d) The 1,4-addition is the major product as it is the result of O
the formation of a stable allylic carbocation.
Solution
CH3 CH3
1 equivalent
CH2 (a) H2SO 4  H+ + HSO 4−
H2C
HBr H3C Br

12. Chlorination of 2-chlorobutane yields a mixture of isomers


with the formula C4H8Cl2. The correct statements about the H+
products is (are) +
O O
H Cl H
H+ HSO−4
2-Chlorobutane H
+
(I) Total six isomers are produced
(II) On fractional distillation of products mixture, five frac-
HO
tions are obtained.
HO
(III) Two isomers are optically inactive
(IV) Four isomers are optically active 14. Consider the reaction given below
(a) II and III only (b) I, II, and III only 1.Na/NH 2. OsO
(c) II, III and IV only (d) I and III only CH3C CCH3 →
3
NaHSO
 4

,H O

3 2

Solution Select the correct statement about the final product(s).


(a) In the reactant, there are four different types of hydrogen (a) Only one product is obtained which is optically inactive.
atoms which can be substituted by chlorine atom. In this (b) Four optically active products are obtained in equal amount.
way, four different structural isomers are possible in this (c) Two optical active compounds which are enantiomers
reaction. But one structural isomer contains two chiral are obtained in equal amount.
centres, so it will show one more stereoisomer, which is a (d) Two diastereomers in unequal amount are obtained.

Chapter-13.indd 444 8/8/2016 2:29:03 PM


Practice Exercise 445

Solution HO OH HO H
(c) First step of the reaction is reduction of alkyne in dis- CH3
solved metal from which trans alkene is obtained. But the H CH3 H
H CH3 OH
oxidation of alkene by OsO4 is syn addition which pro- 1. OsO4 CH3 H CH3
duces two enantiomers in equal amount, so, a recemic 2. NaHSO3, H2O
(R,R)
mixture is produced in this reaction. CH3 H
H CH3 H OH
trans-2-Butene H
CH3 H CH3 CH3
Na or Li H
CH3C CCH3 C C HO OH HO
NH3(liq) CH3
−78°C H CH3
2-butyne (S,S)
trans-2-Butene
Enantiomeric 2,3-butanediols

PRACTICE EXERCISE
Level I 4. CH3 C C CH3
1. X
CH3 C C CH3
2. Zn/HO2
Single Correct Choice O O
In this reaction X is
1. NaNH 1. B H /THF
1. CH3 C CH 2. 2
→ A 2.
 
2 6
→B
CH CH Br 3 2 CH COOH 3 (a) HNO3 (b) O2 (c) O3 (d) KMnO4
The compound B is 5. Which of the following compound does not dissolve in conc.
CH3 CH2 CH3 CH3 H H2SO4 even on warming?
(a) (b) C C (a) Ethylene (b) Propylene
C C
H CH2 CH3 (c) Methane (d) 1,3-Butadiene
H H
CH3 CH3 6. Identify the hydrocarbon that gives:
CH3 H
(I) one mole each of ethanedial and butanedial on ozonolysis.
(c) C C (d) C C (II) mixture of ethanol and pentan-3-one.
H H H CH3 (a) Cyclohepta-1,3-diene; 3-methyl-pent-2-ene
(b) Cyclohexa-1,3-diene; 3-ethyl-pent-2-ene
(c) Cyclohex-1,3,5-triene; 3-methyl-pent-3-ene
Br2/hv Alc. KOH HBr/Peroxide (d) Cyclohepta-1,3-diene; 3-ethyl-pent-1-ene
2. X Y Z
7. Which of the following compounds on oxidative ozonolysis
The compound Z is gives malonic acid as only product?
Br Br (a) CH3CH2CH2C CH (b) CH3CH2C C CH3
(a) (b)
(c) Cyclohexa-1,3–diyne (d) Cyclohexa-1,4–diyne

CH2Br 8. Ethyne forms vinyl chloride on treating it with


(a) dil. H2SO4 in presence of mercury salts.
Br (b) dil. HCl in presence of mercury salts.
(c) (d) (c) Cl2 in presence of hv.
(d) None of these.
1. BH3/THF
O 9. CH3 C C H A
2. H2O2/OH−
3. 18
C6H5 C O O H The compound A is
R CH CH R X
The compound X is (a) CH3CH2CHO (b) CH3COCH3
R CH CH R (c) CH3CHO (d) CH3CH2CH2OH
(a)
H2 O+
O18 10. Ph C C CH3 A
HgSO4
R CH CH R
(b) The compound A is
O O Ph
Ph
(c) both (a) and (b) (a) (b) O
(d) depends upon the reaction condition. H3C
H3C

Chapter-13.indd 445 8/8/2016 2:29:21 PM


446 Chapter 13 Aliphatic Hydrocarbons

OH 20. An alkyne C7H12 on reaction with alkaline KMnO 4 and


Ph
Ph subsequent acidification with HCl yields a mixture of
OH CH3 CH COOH + CH3CH2COOH. The alkyne is
(c) (d)
H3C
H3C CH3
11. Among the following, which has the shortest carbon–carbon (a) 3-heptyne. (b) 2-methyl- 3-hexyne.
single bond? (c) 5-methyl -2-hexyne. (d) 4-methyl-2-hexyne.
(a) CH3 CH3 (b) CH2 CH CH3
(c) HC C C CH (d) CH2 CH C CH 21. The most stable conformation of the product of following
reaction is
12. From which one of the following, both ethylene and acety-
HBr/R2O2 HBr/Dark
lene could be prepared in a single step reaction? C CH X Product
(1 equivalent)
(a) CH3CH2OH (b) Br CH2 CH2 Br
(c) CH3CH2Br (d) Br CH2 CH2 OH H Ph
Ph Br H Br
13. The most acidic among benzene, n-hexane and ethyne is
(a) benzene. (b) n-hexane. (a) (b)
(c) ethyne. (d) none of these. H H H H
Br Br
14. In the given reaction sequence, compound Z is
H
Ph Br H
C6H5COOOH H3O+ PCl5 H H
X Y Z
(c) (d)
Ph Br
H Br
H
Cl Cl Br
(a) (b) 22. An alkene on reductive ozonolysis gives two molecules of
Cl Cl CH2(CHO)2. The alkene is
(a) 2,4-hexadiene.
Cl (b) 1,3-cyclohexadiene.
(c) (d) mixture of all (c) 1,4-cyclohexadiene.
Cl (d) 1-methyl-1,3-cyclopentadiene.
15. Which of the following reactions will give an alkyne? 23. Name the reagent X in the reaction
Zn/alc. KOH X
(a) CH3CBr2CHBr2 
→ 1, 2-Dibromoethane 
→ Acetylene
alc. KOH (a) aq. KOH (b) aq. KMnO4
(b) CH3CH2CHBr2 

(c) alc. KOH (d) alc. KMnO4
NaOH
(c) CH3CHBrCH2Br 
→ 24. Choose the correct statement:
(d) All of the above (a) Acetylene is more reactive than ethylene to an electro-
philic attack.
16. In the following sequence of reactions, the product D is (b) Acetylene and ethylene show similar reactivities towards
HBr HBr alc. KOH NaNH2 an electrophilic attack with different rates.
CH CH A B C D
(c) The reactivities of acetylene and ethylene towards an
(a) ethanol (b) ethyne electrophilic attack depend on the electrophilic reagent.
(c) ethanal (d) ethene (d) Acetylene is less reactive than ethylene to an electro-
philic attack.
17. The final product in following sequence of reaction is
25. Which one of the following has the smallest heat of hydro-
CH3Br
HC CH NaNH
2→ A  →B genation per mole?
(a) 1-Butene (b) trans-But-2-ene
(a) CH2 CH CH CH2 (b) HC C CH3 (c) cis-But-2-ene (d) Buta-1,3-diene
(c) CH2 CH CH3 (d) CH3 CH2 CH3
26. The electrophilic addition of buta-1,3-diene produces
18. The number of structural and configurational isomers of a (a) 1,2-addition product only.
bromo compound, C5H9Br, formed by the addition of HBr to (b) 1,4-addition product.
2-pentyne, respectively are (c) both 1,2- and 1,4-addition products which are not at
(a) 1 and 2 (b) 2 and 4 (c) 4 and 2 (d) 2 and 1 equilibrium with each other.
(d) both 1,2- and 1,4-addition products which are at equilib-
19. An alkene on treatment with NaIO4, KMnO4 and Na2O2 gives
rium with each other.
two moles of acetone. The alkene is
(a) 2,3-dimethylbut-2-ene. (b) 2-methyl but-2-ene. 27. The reduction of an alkyne to alkene using lithium metal in
(c) 3-methylbut-1-ene. (d) 3,3-dimethylbut-1-ene. liquid ammonia as solvent results into

Chapter-13.indd 446 8/8/2016 2:29:30 PM


Practice Exercise 447

(a) cis addition of hydrogen atoms. 39. In the oxidation of alkenes with a dilute KMnO4 solution fol-
(b) trans addition of hydrogen atoms. lowed by acidification with dilute H2SO4 to give diol:
(c) both cis and trans additions of hydrogen atoms. The rela-
tive amount of the two depends on temperature. C OH− dil. H2SO4 C OH
+ KMnO4 intermediate
(d) both cis and trans additions of hydrogen atoms. The rela- C C OH
tive amounts depend on the nature of alkyne.
The OH groups come from
28. Which of the following cycloalkanes is under the highest (a) NaOH (b) H2O
Baeyer’s strain? (c) KMnO4 (d) H2SO4
(a) Cyclobutane (b) Cyclopentane
40. How many eclipsed conformations are possible in butane?
(c) Cyclopropane (d) All are equal
(a) 4 (b) 3 (c) 2 (d) 6
29. What are the products obtained by the ozonolysis of
41. Ozonolysis of the following compound gives:
RCH CR1R2?
(a) R1CH2CH2R (b) R2CO
(c) R1COR2 (d) None of these
30. Increasing order of stability among the three main conforma-
tions (i.e., eclipse, anti, gauche) of 2-fluoroethanol is
O
(a) eclipse, anti, gauche. (b) anti, gauche, eclipse.
(a) 2 (b) O
(c) eclipse, gauche, anti. (d) gauche, eclipse, anti.
31. Polymerization reaction is shown by O
(a) disubstituted alkenes. (c) (d) 2 O
(b) higher molecular weight alkenes.
(c) only ethene.
(d) lower molecular weight alkenes. 42. Water can be added across a triple bond in the presence of
(a) acidic medium. (b) alkaline medium.
32. When 2-butyne is treated with H2 in presence of Pd-BaSO4, (c) neutral medium. (d) acid and HgSO4.
the product formed will be
(a) cis-2-butene. (b) trans-2-butene. 43. Addition of hypochlorous acid to propyne gives
(c) 1-butene. (d) 2-hydroxybutane. Cl
O
H3C
33. Which of the following the is most acidic? (a) Cl2HC (b)
(a) Butane (b) But-1-ene
H Cl
(c) But-1-yne (d) But-2-yne
O OH
34. Which among the following compounds will give 1,4-addi- Cl
tion reaction? (c) H3C (d) H C
Cl 3
OH
CH2 Cl
(a) H2C (b) H C Cl
2 CN
CH3 44. How many different staggered conformations are there for
(c) H C (d) All of these
2 2-methylpropane?
O (a) 1 (b) 2 (c) 3 (d) 4
35. The treatment of ethene with cold alkaline potassium per-
45. Which of the following form alkynide?
manganate produces
H C C
(a) ethylene glycol. (b) formaldehyde.
(c) formic acid. (d) carbon dioxide and water. C C
(a) (b)
36. Which of these will not react with acetylene?
(a) NaOH (b) Ammoniacal AgNO3
(c) Na (d) HCl C C CH3

37. Which of the following reactions can be used to prepare (c) C C (d)
methane?
(a) Clemmensen reduction
(b) Wurtz reaction
(c) Reduction of CH2 CH2 by LiAlH4. 46. Which of the following statement is correct about acetylene
(d) Reduction of methyl iodide by using a zinc–copper. molecule?
(a) its C C bond distance being 121 pm.
38. Wurtz reaction between two moles of 1-bromopropane gives (b) its C H bond distance being 108 pm.
(a) hexane. (b) heptane. (c) its H C C bond angle being 180°.
(c) octane. (d) all of these. (d) all of these.

Chapter-13.indd 447 8/8/2016 2:29:51 PM


448 Chapter 13 Aliphatic Hydrocarbons

47. The highest boiling point is expected for 56. Which of the following isomer of C6H8 shall give
(a) iso-octane. (b) n-octane. OHC CH2 CHO on ozonolysis in presence of Zn?
(c) 2,2,3,3-tetramethylbutane. (d) n-butane.
H
48. A hydrocarbon (A), of formula C8H10, on ozonolysis gives
compound (B), C4H6O2 only. The compound (B) can also be CH3
obtained from alkyl bromide, (C) C3H5Br upon treatment with
magnesium in dry ether, followed by carbon dioxide and (a) (b) D
acidification. The compound (A) is D
(a) CH3 CH2 C C C C CH3
(b) CH3 C C CH2 C C CH2 CH3 CH3
(c) CH3 C C CH2 C C CH3 (c) (d) CD2 CH CH2 CH C CH2
D
(d) C3H5 C C C3H5
49. A conjugated alkadiene having molecular formula C13H22 on
ozonolysis yielded ethyl methyl ketone and cyclohexanecar- 57. Of the four isomeric hexanes, the isomer which can give two
baldehyde. Identify the diene. monochlorinated compounds is
(a) 1-cycloheptyl-4-methylhexa-1,3,5-triene (a) 2-methylpentane. (b) 2,2-dimethylbutane.
(b) 1-cyclohexyl-3-methylhexa-1ene (c) 2,3-dimethylbutane. (d) n-hexane.
(c) 1-cyclohexyl-4-methylhexa-1,3-diene 58. Which hydrocarbon cannot be obtained by the Kolbe’s elec-
(d) 1-cycloheptyl-3-methylhexa-1,3-diene trolytic method?
50. The final product in following sequence of reaction is (a) Ethane (b) Methane (c) Butane (d) Propane
NaNH2 CH3Br 59. The reactant A can be
CH CH A B
CH3 CH3
(a) CH2 CH CH CH2 (b) HC C CH3 HBr
(c) CH2 CH CH3 (d) CH3 CH2 CH3 A H3C CH3

51. When 2-butyne is treated with dil. H2SO4/HgSO4, the product Br H


formed is CH3 CH3
(a) 1-butanol. (b) 2-butanol. H3C
CH3
(c) 2-butanone. (d) butanal. H3C
(a) (b) H3C
52. On halogenation, an alkane (C5H12) gives only one monohal- CH3 CH2 OH
ogenated product. The alkane is CH3 H3C CH3
(a) n-pentane. (b) 2-methyl butane.
(c) 2,2-dimethyl propane. (d) cyclopentane. H3C
(c) (d) H C CH3
OH CH2 3
53. What is the correct order of reagents needed for the following
reaction? 60. The products of dehydration of
Br
CH2OH

Br CH2 CH3
(a) (b)
(a) 1. 3 eq. NaNH2 2. butyl chloride 3. H2/Lindlar’s catalyst
(b) 1. 3 eq. NaNH2 2. butyl chloride 3. Li in NH3 4. NH4Cl
(c) 1. 3 eq. NaNH2 2. butyl chloride 3. H2/Pt
(d) 1. 3 eq. NaNH2 2. propyl chloride 3. Li in NH3 4. NH4Cl (c) (d) CH3

54. Which of the following is false?


(a) Alkanes are very reactive towards chemical reagents. 61. Which one of the following has the minimum boiling point?
(b) Chlorination of methane does not occur in dark. (a) n-butane (b) isobutane
(c) A tertiary carbon atom can be oxidized with relative ease. (c) 1-butene (d) 1-butyne
(d) None of these.
62. Which of the following statements is true for conformations
55. What reagent is needed for the following reaction? of ethane?
(a) No staggered form is possible for ethane.
(b) Eclipsed form is more stable than staggered.
(a) H2/Lindlar’s catalyst (b) (i) Li in NH3 (ii) NH4Cl (c) Staggered form is more stable than eclipsed.
(c) H2/Pt (d) H2/Ni2B (d) None of these.

Chapter-13.indd 448 8/8/2016 2:30:09 PM


Practice Exercise 449

63. The reagent used in the conversion of isopropyl chloride to 72. Which of the following on treatment with warm dil. H2SO4 in
alkene is the presence of HgSO4 will give but -2-one?
(a) CH3COOH (b) CH3CH2ONa (a) But-1-yne (b) But-2-yne
(c) CH3CH2OH (d) Both (b) and (c) (c) But-1-ene (d) But-2-ene
64. On mixing a certain alkane with chlorine and irradiating it Passage Type
with ultraviolet light, it forms only one mono-chloroalkane.
This alkane could be Passage for Questions 73–75: Alkenes on catalytic hydrogenation
(a) propane. (b) pentane. give alkanes. The reactions are exothermic. The heat of hydrogena-
(c) isopentane. (d) neopentane. tion is a measure of stability of alkene. Lesser the heat of hydro-
genation more stable the alkene.
65. 1-Butyne may be converted to butane by reaction with
(a) Zn/HCl (b) Sn/HCl (c) Zn/Hg (d) Pd/H2 73. The relative rate of catalytic hydrogenation of the following
alkenes is
66. Acid catalyzed hydration of alkenes except ethene leads to
the formation of
(a) primary alcohol.
(b) secondary or tertiary alcohol.
(c) mixture of primary and secondary alcohols. I II III IV
(d) mixture of secondary and tertiary alcohols. (a) II > III > IV > I (b) I > IV > III > II
67. Reaction of one molecule of HBr with one molecule of (c) II > IV > I > III (d) III > IV > I > II
1,3-butadiene at 40°C gives predominantly
74. The correct order of heat of hydrogenation is
(a) 3-bromobutene under kinetically controlled conditions.
(b) 1-bromo-2-butene under thermodynamically controlled
conditions.
(c) 3-bromobutene under thermodynamically controlled
conditions. I II III IV
(d) 1-bromo-2-butene under kinetically controlled
conditions. (a) IV > III > I > II (b) II > I > III > IV
(c) IV > III > II > I (d) II > III > I > IV
68 . The following two representations are of
75.
H3C alc. KOH X
C(CH3)3 CH3 (Major)
H and Br
H H Et3N Y
H (Major)
(a) same configuration (b) same molecule Which of the following statement is true?
(c) both (a) and (b) (d) none of these (a) The heat of hydrogenation of X is more than Y.
(b) The heat of hydrogenation of Y is more than X.
Level II (c) Both X and Y has the same heat of hydrogenation.
Multiple Correct Choice Type (d) Both X and Y have same reactivity towards catalytic
hydrogenation.
69. Which of the following on reductive ozonolysis will give only
glyoxal? Passage for Questions 76–77: (A) 1, 2-dibromobutane on reac-
(a) Ethene (b) Benzene (c) Toluene (d) Acetylene tion with n moles of NaNH2 gives (B). (B) reacts with an alkyl iodide
(C) to give (D). (D) on reaction with Na/Liq. NH3 gives (E), (C) under-
70. Which of the following reactions cannot be used for the goes Wurtz reaction to give expected alkane (F) and unexpected
preparation of methane? alkane (G) and an alkene (H). (H) on treatment with CH2I2/Zn gives
(a) Kolbe’s electrolysis cyclo propane.
(b) Soda lime decarboxylation 76. (B) is formed from (A) by production of a gas X, X and n are
(c) Wurtz reaction (a) NH3, 3 (b) NH3, 2 (c) H2, 3 (d) H2, 2
(d) Reduction of CH3Cl with LiAlH4
77. (C) can be converted to (H) by the use of
1. Hg2+ (a) alc. KOH
71. CH CH + 2CH3COOH Products
2. ∆, 300–400°C (b) HI/Red P
Products is/are: (c) KCN followed by KOH
(a) CH3 CH(OCOCH3)2 (b) CH3CHO (d) Mg/ether followed by HCHO
O O O Passage for Questions 78–80: Alkanes may be prepared from
(c) (d)
alkyl halides by the Wurtz method where alkyl halides are reacted
CH3 C OCH3 CH3 C O C CH3
with Na in the presence of ether.

Chapter-13.indd 449 8/8/2016 2:30:16 PM


450 Chapter 13 Aliphatic Hydrocarbons

Na 82. Match the reaction with its name/product obtained.


2R − X Ether
 → R − R + 2NaX
Column I Column II
The mechanism is ionic in nature.
(a) Addition of sulphuric acid to (p) Anti-Markovnikov’s
R–X + 2Na → R–Na+ + NaX propene addition
R– + R–X → R– R + X–
(b) Hydroboration–oxidation of (q) Markovnikov’s
X– + Na+ → NaX propene addition
78. If CH3CH2Cl is reacted with Na in the presence of ether then (c) Hydroboration of propene (r) n-Propyl alcohol
which of the following compounds may not be formed?
(d) Oxymercuration–demercura- (s) Isopropyl alcohol
(a) CH3CH3 (b) CH3CH2CH2CH3
tion of propene
(c) CH4 (d) CH2 CH2

79. When the compound 83. Match the reaction with the type/isomeric products obtained.
Column I Column II
(a) CH2 CH—CH CH2 + H2 (p) One isomeric product
(b) CH2 CH—CH CH2 + CH2 (q) Two isomeric products
CH2
(c) CH2 CH—CH CH2 + HBr (r) Ionic reaction
is reacted with copper at 180°C then which of the following
compounds is formed? (d) C H 2 C H — C H C H 2+ (s) Free radical reaction
Buna-N

Integer Type
(a) (b)
84. An alkyne having molecular mass x × 10 (A) is treated with
Lindlar’s catalyst and H2 to give a compound (B). (B) reacts
with HCl to give a compound (C). When (C) reacts with metal-
lic sodium in presence of ether it gives (D). The molecular
(c) Both (a) and (b) (d)
mass of (D) is 86. What is the value of x?
85.
alc. KOH
80. The reactivity of which hydrogen atom is maximum for halo-
genation by free radical mechanism? Br
CH3 CH CH3 CH2 H The total number of possible products is ______.
(a) (b)
H 86. Ethyl chloride undergoes Wurtz reaction. Then the total num-
H ber of possible products (hydrocarbons) is ______.
87.
(c) CH3 CH CH3 (d) CH CH3
Br2/hv
Products
Matrix Match Type
81. Match the reactants with the reducing agents. The number of monobrominated products possible is ______.
88. The number of p-bonds in the product formed by passing
Column I Column II acetylene through dilute sulphuric acid containing mercuric
(a) CH3 CH CH2 (p) HI, Red P sulphate is ______.
(b) CH3 CH2 Br (q) H2, Pd
(c) Cyclohexane (r) Ni, C2H5OH 89. The number of possible alkynes with molecular formula C5H8
(d) CH3 CH2 I (s) Pd, C2H5OH is ______.

ANSWER KEY
Level I
1. (a) 2. (c) 3. (a) 4. (c) 5. (c) 6. (b)
7. (d) 8. (b) 9. (a) 10. (a) 11. (c) 12. (b)
13. (c) 14. (a) 15. (d) 16. (b) 17. (b) 18. (b)
19. (a) 20. (b) 21. (c) 22. (c) 23. (c) 24. (d)

Chapter-13.indd 450 8/8/2016 2:30:28 PM


Hints and Explanations 451

25. (d) 26. (c) 27. (b) 28. (c) 29. (c) 30. (c)
31. (d) 32. (a) 33. (c) 34. (a) 35. (a) 36. (a)
37. (d) 38. (d) 39. (c) 40. (b) 41. (b) 42. (d)
43. (c) 44. (a) 45. (b) 46. (d) 47. (b) 48. (d)
49. (c) 50. (b) 51. (c) 52. (c) 53. (c) 54. (a)
55. (b) 56. (a) 57. (c) 58. (b) 59. (d) 60. (c)
61. (b) 62. (c) 63. (d) 64. (d) 65. (d) 66. (d)
67. (b) 68. (d)

Level II
69. (b), (d) 70. (a), (c) 71. (b), (d) 72. (a), (b) 73. (a) 74. (b)
75. (b) 76. (a) 77. (a) 78. (d) 79. (b) 80. (d)
81. (a) → q, r; (b) → q; (c) → s; (d) → p 82. (a) → q, s; (b) → p, r; (c) → p; (d) → q, s 83. (a) → p; (b) → p; (c) → q; (d) → s
84. (40) 85. (2) 86. (1) 87. (1) 88. (1) 89. (3)

HINTS AND EXPLANATIONS


Level I 4. (c)
Single Correct Choice O
CH3 C C CH3 + O3 H3C C C CH3
1. (a) (X)
O O
1. NaNH2 2. CH3 CH2 Br O O
− Ozonide
CH3 C CH CH3 C C
2. Zn/H2O
H3C CH2 CH3 H3C C C CH3
1. B2H6/THF
CH3 C C CH2 CH3 C C
(A) H B
3 5. (c) CH4 + H2SO 4 → Noreaction
2. CH3COOH H3C CH2 CH3
C C
H H 6. (b)
(cis alkene)
CH O CH CH2
(B)
CH CH2 CH O
2. (c) O3 CH O CH2 Zn/H2O

Br CH CH2 O CH O
O Ethanedial
Br2/hv Br CH CH O
alc. KOH HBr
Cyclohexa-1,3-diene
(Free radical Peroxide
O CH +
mechanism) (Anti Marko-
3° free radical (X) (Y) vnikov’s rule) (Z)
more stable CHO CH2 CH2 CHO
3. (a) Butanedial
O
18
C6H5 C O O H CH3CH2 CH3CH2 O
Peroxy benzoic acid O3
R CH CH R R CH CH R C CH CH3 C C CH3
O CH3CH2 CH3CH2 O O
18
Mechanism 3-Ethylpent-2-ene
Zn/H2O
R H O R H
18 H 18 H O
+ O O O
C
O C 6 H5 O CH3 CH2 C CH2 + CH3CHO
R H R H C6H5
Pentan-3-one Ethanal
Alkene Transition state
R H HO 7. (d)
18 +
O O O
O3
R H O C6H5
Epoxide 2 HO OH
(Oxirane)

Chapter-13.indd 451 8/8/2016 2:30:34 PM


452 Chapter 13 Aliphatic Hydrocarbons

8. (b) The reaction is as follows: the carbon atom is sp hybridized. As the s character is
maximum in ethyne (50% s and 50% p), it is more close to
Cl− Cl Hg+ the nucleus and hence it’s acidic character is more.
H C C H + Hg2+ H C C H C C
2+ H H 14. (a)
Hg
Acetal-
H+ OH
dehyde
C6H5COOOH H3O+
H Cl Hg+ O
−Hg2+ +
C CH2 C C H OH
Cl H H (X) (Y)
Vinyl chloride PCl5

In case of dil. H2SO4 (H2O is also present), the water mole- Cl


cule is more nucleophilic than the sulphate ion, SO2−4 , which
attacks the complex formed to give vinyl alcohol which being
unstable changes to acetaldehyde. In case of dilute HCl, Cl− Cl
(Z)
ion is more nucleophilic than H2O and it reacts with the com-
plex to form vinyl chloride.
15. (d)
9. (a)
Zn/alk. KOH
1. BH3/THF (a) CH3CBr2CHBr2 CH3C C H
−ZnBr2
CH3 C C H CH3 C C H Alkyne
2. H2O2/OH−
H O H alk. KOH
(Anti Markovnikov (b) CH3 CH2 CH Br2 CH3C C H + 2HBr
addition of water) Alkyne
O
NaOH
CH3 CH2 C H (c) CH3CHBrCH2Br CH3 C CH + 2HBr
(A) Alkyne
Thus, all these give alkynes.
10. (a)
16. (b)
H3O+
Ph C C CH3 Ph C C CH3 H Br
HgSO4
H O H H C C H + HBr H C C H
(Markovnikov
Enol (less stable) rule) 1-Bromoethene
(A)
O H Br
HBr
Ph C CH2 CH3 H C C H
(Markovnikov
A (keto form more stable) rule)
H Br
11. (c) HC C C CH has the shortest C C single bond due (B)
to cylindrical effect, which is generated between the first
and fourth carbon atom. As a result C C bond length H Br
becomes short. alc. KOH
H C C H −HBr
H C C H
12. (b)
H Br H Br
H Br
1, 1-Dibromoethane Vinyl bromide
Zn (B) (C)
ZnBr2 + H2C CH2 H C C H
Ethanol
Ethylene NaNH2
Br H
H C C H
1,2-Dibromoethane
Ethyne
2KOH −HBr (D)
H C C H
Acetylene 17. (b) The reaction is
NaNH2 − +
13. (c) In benzene ring, the carbon atom is sp2 hybridized, in HC CH HC CNa + CH3 Br HC C CH3
−NH3 −NaBr
n-hexane the carbon atom is sp3 hybridized and in ethyne,

Chapter-13.indd 452 8/8/2016 2:30:49 PM


Hints and Explanations 453

18. (b) product is 1,2-addition; and mirror is 1,4-addition. The


HBr + H3C CH2 C C CH3 case is reversed at high temperature.
2-Pentyne H H Br
H3C CH2 CH C(Br) CH3 (1,2) C C H
2-Bromopent-2-ene H C C
H H
+ H3C CH2 C(Br) CH CH3
H H H
3-Bromopent-2-ene C C H HBr
3-Bromobut-1-ene
H C C
Due to the presence of double bond, for each compound cis
and trans isomers will exist. H H Br H H
(1,4)
19. (a) The reaction is Buta-1,3-diene C C H
H C C
H3C CH3 H3C CH3
NaIO4, KMnO4
C C C O+O C H H H
Na2O2
H3C CH3 H3C CH3 1-Bromobut-2-ene

2,3-Dimethyl butane 2 moles of acetone 27. (b)


CH3
H
20. (b) 2-Methyl-3-hexyne is the parent alkyne that gives C Li/NH3
2-methylpropanoic acid and propenoic acid upon C CH3
oxidation. H 3C
H3C
alk. KMnO4
CH3 CH C C CH2 CH3 H
HCl
28. (c) Cyclopropane is the cycloalkane that has highest Baeyer’s
CH3 strain due to small bond angles 65°.
CH3 CH COOH + HOOC CH2 CH3

CH3
29. (c)
21. (c)
R1 O R1
Ph Br R CH C
HBr/R2O2 (Markovnikov R CH C
C CH R2 R2 (Ozonide)
(1 equiv.)
H H addition)
O O
(X) H2O/Zn
Ph
H H H H O
HBr/Dark + HCHO
Ph H (Anti-Markovnikov
addition) C
H Br R1
Br Br Br R2

22. (c) 2 1
O3 30. (c) H2C CH2OH
2CH2(CHO)2
F
1,4-Cyclohexadiene 2-Fluoroethanal

23. (c) The reagent X in the reaction is alcoholic KOH. HO F OH OH


alc. KOH H F H H
BrCH2 CH2Br HC CH
H
24. (d) Acetylene is less reactive than ethylene to an electrophilic H H H H H H
attack. The electronegative group makes the double bond H H F
in ethylene “electron poor” so it is more reactive towards Eclipse Gauche Anti
electrophilic attack.
Stability increases
25. (d) As buta-1,3-diene is most stable, it has the smallest heat of
hydrogenation per mole. Stability depends on repulsion. In anti-conformation, two
electronegative atoms are present on opposite sides; it is
26. (c) When 1,3-butadiene and hydrogen bromide react at most stable. The hydroxy and fluoro groups in the gauche
low temperature in the absence of peroxides, the major conformation is close enough to each other that the vander

Chapter-13.indd 453 8/8/2016 2:31:12 PM


454 Chapter 13 Aliphatic Hydrocarbons

Waals forces between them are repulsive. The eclipsed con- 39. (c) KMnO4 is responsible for OH group formation.
formation has greatest energy.

31. (d) As the alkene units act as monomers which combine to O O O O
C C OH

form polymers, this reaction is more applicable for lower + Mn Mn
Hydrolysis
molecular weight alkenes. C −O C O O
O
32. (a)
− O H
C OH C O C O
H2 H3C CH3 MnO3 +− 2H+ Mn
H3C C C CH3 − −
C C OH O O−
Pd/BaSO4 C C O C
2-Butyne H H O
Syn-1,2-diol
cis-2-Butene
40. (b) The three possible conformation of eclipsed form of
33. (c) But-1-yne is the most acidic because of its sp hybridiza-
butane are:
tion and hence greater electronegativity. It means that
the hydrogen atom is less tightly held by the carbon atom CH3 CH3 H CH3 H CH3
and can be easily removed. Therefore, it is the strongest
acid. In case of but-2-yne there are no hydrogen atoms on
H H H3C
C C, so it is the least acidic. H H H H H H
H CH3 H
34. (a)
1
41. (b) O
3 CH2 Br2 CH2 CH CH CH2
4
H2C 1,4-addition C
2 O3 O
Br Br
O
C
35. (a) The reaction is
cold KMnO4 (alk.)
O
H2C CH2 H2C CH2

OH OH 42. (d)
Ethylene glycol Hg2+ Rearrangement
36. (a) Acetylene will not react with NaOH as it is weakly acidic. HC CH + H2O H2C CHOH CH3CHO
H2SO4
Only strong base like NaNH2 is able to abstract the Acetylene Vinyl alcohol Ethanal
hydrogen.
43. (c)
Zn-Cu
37. (d) CH3I + 2[H] → CH4 + HI OH Cl
Methyl iodide HOCl
CH3 C CH + HOCI H3C C CH H3C C CH
38. (d) Since two alkyl halides can react in three different ways,
Propyne
therefore a mixture of three alkanes instead of the desired OH Cl OH Cl
alkane would be formed. For example, Wurtz reaction Unstable
between two moles of 1-bromopropane gives a mixture −H2O
of three alkanes, that is, hexane, heptane and octane as
O Cl
shown below.
H3C C CH Cl
CH3CH2CH2Br + 2Na + CH3CH2CH2Br →

→ 1,1-Dichloropropan-2-one
1-Bromopropane 1-Bromobutane
CH3CH2CH2CH2CH2CH3 + 2NaBr 44. (a) One staggered conformation and one eclipsed confor-
n-Hexane
mation.

CH3CH2CH2Br + 2Na + CH3CH2CH2Br →



→ H H
CH3 H H H
1-Bromopropane 1-Bromobutane
CH3CH2CH2CH2CH2CH2CH3 + 2NaBr CH3 CH CH3
H
n-Heptane
CH3 CH3 CH3 CH3
2-Methylpropane H
H
CH3CH2CH2Br + 2Na + CH3CH2CH2Br →

→ Staggered Eclipsed
1-Bromopropane 1-Bromobutane
CH3CH2CH2CH2CH2CH2CH2CH3 + 2NaBr 45. (b) Alkynide-forming compounds contain one terminal
n-Octane hydrogen atom that is attached to the carbon atom.

Chapter-13.indd 454 8/8/2016 2:31:25 PM


Hints and Explanations 455

46. (d) Conceptual. Mechanism


47. (b) n-Octane is expected to have highest boiling point
because it is a straight chain isomer. H
Isooctane and 2,2,3,3-tetramethyl butane are branched H3C C C CH3 + O H H 3C C C CH3
chain isomers. Branched chain isomers have a lower boil- −
+
ing point than a straight chain isomer. O H
Hg2+
48. (d) H
O O
O3/H2O
C3H5 C C C3H5 C3H5 C OH + C3H5 C OH Rearrangement
Ozonolysis CH3 CH2 C CH2 H3C C C CH3
(A) (B)
O H
Mg CO2H+
O O H
C3H5 Br C3H5MgBr C3H5 C OH 2-Butanone
Ether
(C) CH3 CH3
Cl2
49. (c) Since the total number of carbon atoms of the two H3C C CH3 H3C C CH2Cl
52. (c) −HCl
products (ethyl methyl ketone and cyclohexanecar-
baldehyde) formed is 11 while the molecular formula CH3 CH3
of the conjugated diene is C13H22; therefore, the ozo- 2,2-Dimethylpropane
nolysis must have produced another carbon product.
Further, since the given compound (C13H22) is an alka-
53. (c) The starting material has 5 carbons and the product has
diene; therefore, these two carbon products must be
9 carbons. Therefore, addition of four carbons atoms is
glyoxal. The ozonolysis reaction can be explained as
required to get the product, which means there is need
follows:
of butyl chloride. The final product has an alkane so only
CH3 H2/Pt will produce the alkane.
C CH CH CH 54. (a) Alkanes have no reaction sites due to presence of only
H3C CH2 C C and C H non-polar bonds where the chemical rea-
1-Cyclohexyl-4-methyl-1,3-diene O3
gents can attack.

CH3 O O 55. (b) The product is a trans double bond. (i) Li in NH3 (ii) NH4Cl
is the only way to turn an alkyne into a trans double
C CH CH CH
bond.
H3CCH2 O O O O
56. (a)
Zn/H2O
H
H3C
C O+O CH + CHO Ozonolysis
OHC CH2 CHO + OHC CH CHO
H3C CH2 Zn
CHO
Ethyl methyl Cyclohexane Glyoxal D
ketone carbaldehyde D

Hence, IUPAC name of the conjugated alkadiene is 57. (c) 2,3-Dimethyl butane contains two type of hydrogen
1-cyclohexyl-4-methylhexa-1,3-diene. atom, that is, 1° and 3°.

CH3 CH3 CH2Cl CH3


50. (b) Cl2
CH3 C C CH3 H3C C C CH3
NaNH2 −NaBr 1° 3°
CH CH CH C−Na+ + CH3Br CH C CH3 H H H H
(A) (B) +
H2O CH3 CH3
51. (c) H3C C C CH3 H3C CH2 C CH3
dil. H2SO4/HgSO4
H3C C C CH3
O
2-Butanone Cl H

Chapter-13.indd 455 8/8/2016 2:31:31 PM


456 Chapter 13 Aliphatic Hydrocarbons

58. (b) Methane (CH4) 1. conc. H2SO4


CH3 CH CH2 CH3CHOH CH3
2. H3O+
59. (d) (2° Alcohol)
CH3 CH3 CH3 CH3 CH3
HBr 1. conc. H2SO4
H3 C CH3 CH3 C CH2 (CH3)3COH
2. H3O+
H3C CH3 (3° Alcohol)
(A) Br H
67. (b) The reaction is
60. (c) Br
+ + HBr
CH2 OH2 CH2 C C C C C C C C
CH2OH +
40° C
H
1,3-Butadiene 1-Bromo-2-butene

68. (d) They are representation of the same molecule with differ-
1, 2-alkyl +
H2O ent conformation.
shift
H
Level II
61. (b) Due to branching, there is decrease in the surface area
Multiple Correct Choice Type
and also decrease in van der Waals forces of attraction.
62. (c) Staggered form is more stable because it has minimum 69. (b), (d)
repulsive forces, minimum energy, least torsional strain O
and thus maximum stability. C6H6 CCl
3
→ 3 OHC CHO
4
Glyoxal
H 1. O3
H H C C H 2. H2O/Zn
OHC CHO
H Glyoxal
H H
H 70. (a), (c) Kolbe’s electrolysis can be used to synthesize higher
Sawhorse
H H
projections alkanes except methane. Wurtz reaction is also used for syn-
H H of ethane thesis for higher alkanes.
H H
71. (b), (d)
Eclipsed Staggered
1. Hg2+
H CH CH + 2CH3COOH CH3CHO + CH3COOCOCH3
HH 2. ∆, 300°− 400°C
H H
Newman’s 72. (a), (b)
H H H projections
H H warm dil. H2SO4
H H of ethane CH3 CH2 C CH CH3 CH2 COCH3
HgSO4
Eclipsed Staggered But-1-yne But-2-one

63. (d) The reaction involved is


warm dil. H2SO4
CH3 C C CH3 CH3 CH2 COCH3
CH3CH2ONa HgSO4
CH3CHCH3 CH CH OH CH3CH CH2 But-2-yne But-2-one
3 2
Propene
Cl
Isopropyl Passage Type
chloride
73. (a) In general, the ease of reduction decreases in the order
64. (d) Neopentane contains one type of hydrogen.
mono substituted > 1,1-disubstituted, 1,2-cis-disubsti-
65. (d) The reaction involved is tuted, 1,2-trans-disubstituted > trisubstituted > tetra-
substituted. Therefore, II > III > IV > I
Pd
CH3 CH2 CH CH2 + H2 CH3 CH2 CH2 CH3
74. (b) Heat of hydrogenation is a measure of the stability of
But-1-ene n-Butane alkenes. Greater the number of alkyl groups attached to
the double bonded carbon, more stable is the alkene. So,
66. (d) Propene gives secondary alcohol whereas 2-methyl the alkene with maximum substitution will have the low-
propene gives tertiary alcohol. est value of heat of hydrogenation.

Chapter-13.indd 456 8/8/2016 2:31:39 PM


Hints and Explanations 457

75. (b) Integer Type


CH3
84. (40)
alc. KOH
Lindlar cataylst
Alkyne (given) Alkene
(X) x × 10 H2 x × 10 + 2
CH3 Saytzeff’s product (More (A) (B)
substituted)
Br (Molar mass = 36.5) HCl
Less heat of
hydrogenation (D)
Na
(C)
Wurtz reaction
Et3N (Mol. mass = 86 given) (x × 10 + 2 + 36.5)

Calculated mass of (D) = 2(10 × x + 2 + 36.5) − 2 × 36.5 + 2


(Y) = 20 x + 6
Hoffman product (Less (After Wurtz reaction, deduct two chlorine atoms and add
substituted) two hydrogen atoms.)
More heat of Therefore,
hydrogenation Mol. mass calculated = Mol. Mass given
20 x + 6 = 86
76. (a)
H3C CH2 CH CH2 + 2NaNH2 86 − 6
x= =4
20
Br Br
(A)
Therefore, the alkyne is propyne with molecular mass 40.
85. (2) The total number of possible products is two.
CH3 CH2 C CH
NaNH2 alc. KOH

Major
CH3 CH2 C C−Na+ + NH3 Br
(X) +

77. (a) Alkyl iodide (C) can be converted to alkene (H) by the use Minor
of alc. KOH through E2 elimination.
86. (1) Only one product is possible.
78. (d) CH2 CH2 cannot be formed because Wurtz method is
used to prepare alkanes.
CH3 CH2 Cl + Na + Cl CH2 CH3
79. (b)
CH3 CH2 CH2 CH3
Cu
2
180° 87. (1) Tertiary carbon selectively brominated over primary or
secondary carbon.

This reaction is called Ullmann condensation reaction. Br


80. (d) Br2/hv
Benzyl hydrogen
H

CH CH3 88. (1)


Benzyl carbon
Hg2+ Rearrangement
H C C H + H 2O H2C CH OH CH3CHO
H2SO4
2p -bond 1p -bond Ethanal
Benzyl hydrogen atom shows maximum reactivity for halo-
genation by free radical mechanism.
89. (3) The possible alkynes are:
Matrix-Match Type
H3C CH2 CH2 C CH H 3C CH2 C C CH3
81. (a) → q, r; (b) → q; (c) → s; (d) → p
H3C C C CH2 CH3
82. (a) → q, s; (b) → p, r; (c) → p; (d) → q, s
83. (a) → p; (b) → p; (c) → q; (d) → s

Chapter-13.indd 457 8/8/2016 2:31:46 PM


458 Chapter 13 Aliphatic Hydrocarbons

SOLVED JEE 2016 QUESTIONS


JEE Main 2016 Mechanism
1. 2-Chloro-2-methylpentane on reaction with sodium methox-
ide in methanol yields: + Cl
δ+ δ−
CH3 + Cl — Cl

C2H5CH2C — OCH3 C2H5CH2C —


— CH2 Intermediate


CH3 CH3
3. The hydrocarbon with seven carbon atoms containing a neo-
I II pentyl and a vinyl group is
(a) 2, 2-dimethyl-4-pentene. (b) 4, 4-dimethylpentene.
— C — CH3
C2H5CH2 — (c) isopropyl-2-butene. (d) 2, 2-dimethyl-3-pentene.

CH3 (Online)
III Solution
(a) I and II (b) All of these (a) The hydrocarbon is 4, 4-dimethylpentene.
(c) I and III (d) III only CH3
H3C
(Offline)
H3C CH
Solution 2

(b)
4. Bromination of cyclohexene under conditions given below
Cl yields:
— —

CH3ONa
CH 3 — C — CH 2 — CH2 — CH3 CH 3 — C —
— CH — CH2 — CH3 (l) Br2/hn
CH3OH

CH 3 CH 3
— C — CH2 — CH2 — CH3
+ CH2 — (ll) Br
Br

CH 3
OCH 3 (a) (b)
— —

+ CH3 — C — CH2 — CH2 — CH3 (lll)


CH 3 Br
Br
In case of products I and II, b-elimination takes place as methoxide Br Br
acts as a strong base in presence of methanol. Product (III) is also
possible because the reaction is carried out between haloalkane (c) (d)
and sodium methoxide (Williamson’s synthesis).
2. The reaction of propene with HOCl (Cl2 + H2O) proceeds
through the intermediate: Br
(a) CH3–CHCl–CH2+ (b) CH3–CH+–CH2–OH (Online)
(c) CH3–CH+–CH2–Cl (d) CH3–CH(OH)–CH2+
(Offline) Solution
(d) In the presence of UV light, allylic C H bond (that is
Solution
carbon adjacent to the double bond) undergoes bromina-
(c) The reaction is tion. This process is called as allylic bromination.
H H H Br
+ Cl
— OH– Allylic
+ HO — Cl + Br2
hn
+ HBr
hydrogens
Propene 1-Chloropropan-2-ylium
3-Bromocyclohexene
OH– H H
(80%)
OH

Cl

1-Chloropropan-2-ol

Chapter-13.indd 458 8/8/2016 2:31:51 PM


14 Aromatic Hydrocarbons

Question Distribution in JEE (Main and Advanced)

3
No. of Questions

JEE (Main)
2
JEE (Adv)

0
2016 2015 2014 2013 2012 2011 2010 2009 2008 2007

Concept Distribution in JEE (Main and Advanced)

Topics Covered
Year
JEE (Main) JEE (Advanced)
2007 Chemical Properties
2008 Chemical Properties Chemical Properties
2012 Structure of Aromatic Compounds
2013 Structure of Aromatic Compounds
2015 Chemical Properties
2016 Chemical Properties

Chapter-14.indd 459 8/8/2016 2:49:26 PM


460 Chapter 14 Aromatic Hydrocarbons

SUMMARY
1. Aromatic compounds are a class of hydrocarbons, so called because many of them possess a fragrant smell. These are carbocylic
compounds containing at least one benzene ring within their structure. Benzene and its derivatives belong to a class of compounds
called arenes. For example, benzene, benzaldehyde or methyl salicylate.
O O

H OCH3
or
OH
Benzene Benzaldehyde Methyl salicylate
(in oil of almonds) (in oil of wintergreen)

2. Other aromatic compounds


(a) Benzenoid aromatic compounds
(i) Representatives of one broad class of benzenoid aromatic compounds, called polycyclic aromatic hydrocarbons (PAH).
(ii) Benzenoid polycyclic aromatic hydrocarbons consist of molecules having two or more benzene rings fused together.
6
5 7
8 1 8 9 1 4
8
7 2 7 2 3

6 3 6 3 2 9
5 4 5 10 4 1 10

Naphthalene Anthracene Phenanthrene


C10H8 C14H10 C14H10

(b) Nonbenzenoid aromatic compounds


(i) The cyclopentadienyl anion, the cycloheptatrienyl cation, trans-15,16-dimethyldihydropyrene, and the aromatic annulenes
(except for [6]annulene) are classified as nonbenzenoid aromatic compounds.
(ii) Another example of a nonbenzenoid aromatic hydrocarbon is the compound azulene.

Azulene
(c) Heterocyclic aromatic compounds
Cyclic compounds that include an element other than carbon are called heterocyclic compounds.
4
5 3 4 3 4 3 4 3

6 2 5 2 5 2 5 2
N N1 O S
1 1 1
H
Pyridine Pyrrole Furan Thiophene

3. Nomenclature of benzene derivatives


(a) Monosubstituted benzenes
(i) Benzene is the parent name and the substituent is simply indicated by a prefix. For example, fluorobenzene, chlorobenzene, etc.
F Cl Br NO2

Fluorobenzene Chlorobenzene Bromobenzene Nitrobenzene

(ii) The substituent and the benzene ring together form a commonly accepted base name. For example, toluene, phenol, ani-
line, anisole.

Chapter-14.indd 460 8/8/2016 2:49:29 PM


Summary 461

H
H H
CH3 O N

Toluene Phenol Aniline

(b) Disubstituted benzenes


(i) Positions are indicated by numbering or the prefixes ortho- (1,2 relation), meta-(1,3 relation), and para- (1,4 relation)
Br Br
Br

Br Br

Br
1,2-Dibromobenzene 1,3-Dibromobenzene 1,4-Dibromobenzene
(o-dibromobenzene) (m-dibromobenzene) (p-dibromobenzene)
ortho meta para

(ii) The dimethyl benzenes are also called xylenes.


CH3 CH3 CH3

CH3 H3C
CH3
1,2-Dimethylbenzene 1,3-Dimethylbenzene 1,4-Dimethylbenzene
(o-xylene) (m-xylene) (p-xylene)

(c) Polysubstituted benzenes


(i) Groups are numbered so as to give the lowest possible numbers to the substituents, with the substituents listed in alpha-
betical order.
Cl Br
1 1
6 6 Br
2 2
5 5 3
3
4 4
Br
1-Chloro-3-ethylbenzene 1,2,3-Tribromobenzene
(not 1,3,4-tribromobenzene)
(ii) When a substituent is one that together with the benzene ring gives a new base name, that substituent is assumed to be in
position 1 and the new parent name is used.
O
2 6
O2N 3 1 1 SO3H
5
OH
4 2
4 6 F F
5 3
NO2
3,5-Dinitrobenzoic acid 2,4-Difluorobenzenesulphonic acid
(iii) When the C6H5-group is named as a substituent, it is called a phenyl (Ph) group.

C6H5

Butylbenzene (Z)-2-Phenyl-2-butene 2-Phenylheptane

Chapter-14.indd 461 8/8/2016 2:49:32 PM


462 Chapter 14 Aromatic Hydrocarbons

(iv) Benzyl is an alternative name for the phenylmethyl group. It is sometimes abbreviated Bn.

CH2 CH2Cl

The benzyl group Benzyl chloride


(the phenylmethyl (phenylmethyl chloride
group) or BnCl)

4. Benzene – structure
(a) According to the resonance theory, benzene molecule is a resonance hybrid of the following two structures.

(not )

I II

These structures are resonance contributors to the structure of benzene. They do not exist as such but are useful to visualize the
actual molecule.
(b) The hybrid structure of benzene is represented by inscribing a circle in the hexagon, which represents the six electrons that are
delocalized about the six carbon atoms of the benzene ring.
(c) Large resonance energy
Due to resonance, the p electron charge in benzene tends to get delocalized over the entire ring resulting in lowering the
energy of the resonance hybrid relative to the individual contributing structures. This decrease in energy is called resonance
energy. The resonance stabilization contributes to the unusual stability of benzene.
5. Structure of aromatic compounds
(a) Aromaticity
Aromatic compounds are those that contain alternate double and single bonds in a cyclic structure. They undergo substitution
reactions rather than addition reactions. The three general requirements for a compound to be aromatic are:
(i) The compound must be cyclic.
(ii) The compound must contain a cyclic cloud of delocalized p  electrons above and below the plane of the molecule. The
delocalization of p electrons over the ring must result in a lowering of the electronic energy.
(iii) The compound must follow Hückel’s Rule (delocalized electrons cloud must contain a total of (4n + 2) p electrons, where
n is an integer equal to 0, 1, 2, 3 … and number of p  electrons are 2, 6, 10, 14...
• Benzene (6 p electrons), naphthalene (10 p  electrons) and anthracene (14 p electrons) are all aromatic compounds.

Benzene Naphthalene Anthracene


6 p electrons 10 p electrons 14 p electrons

• Similarly, cyclopentadienyl anion and cycloheptatrienyl cation (tropylium ion) are also aromatic because these contain
6 p electrons.
H
H H H H

H H H H

+
H H H

Cyclopentadienyl anion Cycloheptatrienyl cation


6 p electrons 6 p electrons
• Heterocyclic compounds such as pyrrole, furan, thiophene and pyridine also behave as aromatic because all have
6 p  electrons.

O S N N
H
Furan Thiophene Pyrrole Pyridine

Chapter-14.indd 462 8/8/2016 2:49:35 PM


Summary 463

(b) Antiaromaticity
(i) When the cyclic molecule (with a conjugated p electron system) has higher energy due to the presence of 4n electrons in
the p electron cloud, the compound is said to exhibit antiaromatic character.
(ii) They are highly unstable, highly reactive and may change shape (i.e. become non-planar) and break some of the p interac-
tions, to avoid instability.
(iii) In other words, systems with 4, 8, 12 …, p electrons are antiaromatic.

+ −

Hypothetical planar Cyclobutadiene 4 p electrons 8 p electrons


cyclooctatetraene 4 p electrons (antiaromatic)
8 p electrons (antiaromatic)

(c) Nonaromaticity
(i) Compounds that meet Hückel’s rule (4n + 2 or 4n p electrons) but do not fulfill one of the other prerequisites for aromaticity
and avoid being antiaromatic by deviating from planarity are called non-aromatic. 1,3,5-Cycloheptatriene is an example of
non-aromatic compound.

1,3,5-Cycloheptatriene
(ii) They are less stable than aromatic compounds.
6. General methods of preparation
(a) Decarboxylation of sodium benzoate
Sodium benzoate on heating with soda-lime undergoes decarboxylation (removal of carbon dioxide) to give benzene.

COONa

Heat
+ NaOH + CaO + Na2CO3

(b) Reduction of phenol

OH

Heat
+ Zn

(c) Cyclic polymerization of ethyne


CH
HC CH Red hot tube
+ +
HC CH
CH

(d) Reduction of benzene diazonium chloride


N2+Cl−

H3PO2
H2O

(e) Hydrolysis of benzene sulphonic acid


SO3H
superheated
stem
+ H2O dil HCl
150°C

Chapter-14.indd 463 8/8/2016 2:49:38 PM


464 Chapter 14 Aromatic Hydrocarbons

(f) Wurtz–Fittig reaction


X R

dry ether
+ Na + RX + NaX

7. Electrophilic aromatic substitution reactions


This is the most characteristic reactions of benzenoid arenes that occurs when they react with electrophilic reagents. The reactions
are called electrophilic aromatic substitution and are represented as
Ar H+ E A Ar E+H A

E
or +E A +H A

The electrophiles are either a positive ion (E+) or some other electron-deficient species with a large partial positive charge.
Mechanism
Step 1: The electrophile takes two electrons of the six-electron p system to form a s bond to one carbon atom of the benzene ring.

δ+ δ− + E E E
E A
H H H
+ A:−
+ +

Arenium ion
(a delocalized cyclohexadienyl cation)

where E is electrophile like Br+, NO2+, SO3 R+, etc.


Step 2: A proton is removed from the carbon atom of the arenium ion that bears the electrophile, restoring aromaticity to the ring.

+ E E
H + H A
A−

(a) Halogenation
Benzene reacts with bromine and chlorine in the presence of Lewis acids to give halogenated product.
Cl
AlCl3
+ Cl2 + HCl
25°C

Chlorobenzene (90%)
Mechanism
Step 1: In the first step, the aromatic ring functions as nucleophile, forming an intermediate sigma complex.
Step 2: In the second step, the sigma complex is deprotonated, restoring aromaticity.
AlCl3 + −
Cl Cl Cl Cl AlCl3

Step 2
Step 1
H Proton transfer Cl
Nucleophilic attack
+ −
Cl

AlCl3
+ AlCl3 + HCl
Cl Cl AlCl3
− H H H
Cl Cl Cl
+ +

Sigma complex

Chapter-14.indd 464 8/8/2016 2:49:42 PM


Summary 465

(b) Nitration
Benzene undergoes nitration on reaction with a mixture of concentrated nitric acid and concentrated sulphuric acid.

NO2
50−55°C −
+ HNO3 + H2SO4 + H3O+ + HSO4

85%

Mechanism
Step 1: The aromatic ring functions as a nucleophile, forming the intermediate sigma complex.
Step 2: The sigma complex is deprotonated, restoring aromaticity.

O H O S O H O O
O −H2O

N −
N
H N+
+
O OH O O+
O
H
Nitronium ion

Step 1 Step 2
H O Nucleophilic attack Proton transfer NO2
N+ H H
O O
+ H H H
NO2 NO2 NO2
+ +

Sigma complex

(c) Sulphonation of benzene


When benzene is treated with fuming sulphuric acid, a sulphonation reaction occurs and benzene sulphonic acid is obtained.

SO3H
Fuming H2SO4

(95%)

Mechanism
Step 1: The aromatic ring functions as a nucleophile forming as nucleophile, forming the intermediate sigma complex.
Step 2: The sigma complex is deprotonated, restoring aromaticity.
Step 3: The resulting anion is protonated.

Step 3
Proton transfer

O O O OH
S S
H O SO3H
H Step 1 O O
O O Nucleophilic attack
S − SO3H
O
O O O
O + H H H Step 2
− − − Proton transfer
S O S O S O
+ +
O O O
Sigma complex

Chapter-14.indd 465 8/8/2016 2:49:45 PM


466 Chapter 14 Aromatic Hydrocarbons

(d) Friedel–Craft’s alkylation and acylation


(i) Alkyation
This reaction involves addition of an alkyl group to benzene using alkyl halides.
R
AlCl3
+ R X + HX

Mechanism
Step 1: Formation of a carbocation

Cl Cl Cl
+
Cl + Al Cl Al− Cl + Cl Al− Cl
Cl Cl
Cl Cl

Step 2: The carbocation then acts as an electrophile and attacks the benzene ring to form an arenium ion.

+ Cl Cl
+
H
+ Cl Al− Cl + HCl + Al
Cl Cl
Cl

(ii) Acylation
The Friedel –Crafts acylation reaction is often carried out by treating the aromatic compound with an acyl halide (often an acyl
chloride).
O
O
AlCl3
+ excess
+ HCl
Cl
benzene,
80°C
Acetyl Acetophenone
chloride (methyl phenyl ketone)
(97%)
The electrophile is the acylium ion formed by the reaction of acyl halide with Lewis acid (AlCl3).
O O
+

R C Cl + AlCl3 R C Cl AlCl3
(iii) Limitations of Friedel–Crafts reactions
• When the carbocation formed from an alkyl halide, alkene, or alcohol can rearrange to one or more carbocations that
are more stable, it usually does so, and the major products obtained from the reaction are usually those from the more
stable carbocations.
• Friedel–Crafts reactions usually give poor yields when powerful electron-withdrawing groups are present on the aro-
matic ring or when the ring bears an NH2, NHR, or NR2 group.
• Aryl and vinylic halides cannot be used as the halide component because they do not form carbocations readily.
• Polyalkylations often occur.
8. Directive influence in arenes
When the monosubstituted product is to be converted into disubstituted one, the existing substituent which is previously present
in the ring influences or directs the incoming group to a particular position. This is referred to as directive influence of the group.
(a) Ortho, para-directing groups
(i) The substituents or groups, which direct the incoming group to ortho and para positions. For example, CH3, C2H5,
Cl, OH, Br, NH2, NHR, NR2, NHCOCH3, OCH3, etc.

NH2 NH2 OH OH
Br Br Br Br
Br2 Br2
H2 O H2O

Br Br
2,4,6-Tribromoaniline 2,4,6-Tribromophenol
(~100%) (~100%)

Chapter-14.indd 466 8/8/2016 2:49:48 PM


Summary 467

(ii) All electron-donating groups are activating groups and are ortho–para directors.
(iii) Halogen substituents are weakly deactivating groups and are ortho–para directors.
(b) Meta-directing groups
(i) The substituents or groups which direct the incoming group to meta position. For example: NO2, CN, CHO, COR,
COOH, COOR, SO3H, etc.
NO2 NO2 NO2 NO2
NO2
HNO3
+ +
H2SO4
NO2
NO2
6% 1% 93%
(ii) With the exception of halogen substituents, all electron-withdrawing groups are deactivating groups and are meta directors.
(c) The effect of substituents on the reactivity of a benzene ring towards electrophilic substitution
Activating substituents Most activating
NH2
NHR
NR2 Strongly activating
OH
OR
O
NHCR
O Moderately activating

OCR Ortho/para-directing
R
Ar Weakly activating
CH CR2

Standard of comparison H
Deactivating substituents F
Cl
Weakly deactivating
Br
I
O
CH
O
CR
O
COR Moderately deactivating
O
Meta-directing
COH
O
CCl
C N
SO3H
+
+
NH2R Strongly deactivating
NH3 +
+ NR3
NHR2 NO2
Most deactivating

13. Orientation in disubstituted benzene


(a) When two substituents are present in a benzene ring, orientation of third incoming electrophile is bit complicated. For example,
CH3 group is o, p-directing and NO2 group is m-directing, thus, in p-nitrotoluene, new electrophile can be in a position meta
w.r.t. NO2 while ortho w.r.t. CH3 group, as indicated by arrows in the following figure.

Chapter-14.indd 467 8/8/2016 2:49:50 PM


468 Chapter 14 Aromatic Hydrocarbons

CH3

O2N

(b) When both the groups are o, p-directing and none of the vacant positions are ortho- and para- w.r.t. each, in such cases:
(i) Strong activating groups generally win over deactivating group or weakly deactivating groups.

O Br O

H Br2, Fe H

OH OH

(ii) If the substituents are meta to each other, substitution of incoming electrophile rarely takes place in between vacant sites.
Cl Cl Cl Cl
Br Br
Br2, Fe + +
Br Br Br Br
1% 37%
Br
62%

9. Addition reactions
(a) With hydrogen
Addition of hydrogen to benzene occurs under pressure using metal catalyst such as nickel.

H2/Ni H2/Ni H2/Ni


+
Slow Fast Fast

Benzene Cyclohexadienes Cyclohexene Cyclohexane

Ni
+ 3H2
200° C

(b) With chlorine


Addition of chlorine occurs in presence of UV light at high temperature
Cl
Cl Cl
UV
+ 3Cl2
500 K
Cl Cl
Cl

10. Oxidation reactions


These yield different products with different oxidizing reagents.
(a) With oxygen
(i) Complete combustion
2C6H6 + 15O2 → 12CO2 + 6H2O
(ii) Catalytic oxidation
9 V2 O5 , 773 K
C6H6 + O2  → HO2CCHCHCO2H + 2CO2 + H2O
2 Maleic acid

Chapter-14.indd 468 8/8/2016 2:49:53 PM


Summary 469

(b) With potassium permanganate


The aliphatic side chain in case of higher homologues of benzene is oxidized with hot KMnO4 to a carboxylic group.

CH2CH2CH3 COOH
Hot KMnO4/OH−
+ 2CO2 + 3H2O

Propylbenzene Benzoic acid

Tip Benzene is not oxidized by cold dilute alkaline potassium permanganate like other hydrocarbons. So the reaction can be used
to distinguish it from alkenes and alkynes which undergo oxidation with this reagent.

(c) With chromyl chloride (Etard reaction)


Toluene reacts with chromyl chloride to form a brown complex which is separated and decomposed with water to give
benzaldehyde.
(d) With ozone
On ozonolysis, benzene and other arenes form a triozonide, which undergoes reductive cleavage with Zn to form
glyoxal.
CH
HC CH H2O CHO
+ 3O3 Benzene triozonide 3 + 3H2O2
HC CH CHO
CH Glyoxal
Benzene

11. Nucleophilic aromatic substitution (SNAr) reactions


The reactions in which benzene ring is attacked by a nucleophile are known as nucleophilic aromatic substitution reactions.
In order for a reaction like this to occur, three criteria must be satisfied:
(a) The ring must contain a powerful electron-withdrawing group (typically a nitro group).
(b) The ring must contain a leaving group (usually a halide).
(c) The leaving group must be either ortho or para to the electron-withdrawing group. If the leaving group is meta to the nitro
group, the reaction is not observed

Br OH

1. NaOH, 70° C
2. H3O+

NO2 NO2

Mechanism
Step 1: In the first step, the aromatic ring is attacked by a nucleophile, forming the intermediate Meisenheimer complex.
Step 2: In the second step, a leaving group is expelled to restore aromaticity.

Cl OH

N − N −
O + O Cl OH Cl OH Cl OH Cl OH O + O
− −
OH − − − Cl

Step 1 Step 2

N − N − N − − N −
O + O O + O O + O O + O
Meisenheimer complex

Chapter-14.indd 469 8/8/2016 2:49:55 PM


470 Chapter 14 Aromatic Hydrocarbons

12. The Birch reduction


Benzene can be reduced to 1,4-cyclohexadiene by treating it with an alkali metal (sodium, lithium, or potassium) in a mixture of
liquid ammonia and an alcohol. This reaction is called the Birch reduction.
Na
NH3, EtOH

Benzene 1,4-Cyclohexadiene

Mechanism
Step 1: A single electron is transferred from the sodium atom to the aromatic ring.
Step 2: Methanol donates a proton to the radical anion, generating a radical intermediate.
Step 3: A single electron is transferred from the sodium atom to the radical intermediate, generating an anion.
Step 4: Methanol donates a proton to the anion, generating an isolated diene.
Step 1 Step 2 Step 3 Step 4
Nucleophilic Proton transfer Nucleophilic Nucleophilic
H attack H H H attack H H attack H H
− O O
H H •
Na
H H H CH3 H H •
Na
H H H CH3 H H

H H H H H H H −
H H H
H H H H H H
A radical A radical An anion
anion
13. Benzylic radicals and cations
(a) Hydrogen abstraction from the methyl group of methylbenzene (toluene) produces a radical called the benzyl radical.

A
C H benzylic H CH2 CH2 C
R
hydrogen
Benzylic −RH
carbon

Methylbenzene Benzyl Benzylic


(toluene) radical radical

(b) Departure of a leaving group (LG) from a benzylic position produces a benzylic cation.
+
C LG C

−LG−

A benzylic cation

(c) Benzylic radicals and benzylic cations are conjugated unsaturated systems and both are unusually stable.

+
C C C C C C C C

+ +

Benzylic radicals are stabilized by resonance Benzylic cations are stabilized by resonance
(d) Halogenation of the side chain: Benzylic radicals
(i) Benzylic halogenation is carried out in the absence of Lewis acids and under conditions that favour the formation of radicals.

Chapter-14.indd 470 8/8/2016 2:49:59 PM


Tools for Problem Solving 471

O O
CH3
light Br
+ N Br CCl4
+ N H

O Benzyl bromide O
(a-bromotoluene)
NBS (64%)
CH2 CH2Cl CHCl2 CCl3

Cl2 Cl2 Cl2


heat heat heat
or light or light or light
Benzyl Dichloromethyl- Trichloromethyl-
chloride benzene benzene
(ii) The greater stability of benzylic radicals accounts for the fact that when ethylbenzene is halogenated, the major product is the
1-halo-1-phenylethane. The benzylic radical is formed much faster than the 1° radical.
X

Fast X2

Benzylic radical 1-Halo-1-phenylethane


(more stable) (major product)

(−HX)

X
Slow X2

1° Radical 1-Halo-2-phenylethane
(less stable) (minor product)

Tip Benzyl and allylic radicals are even more stable than tertiary radical.

TOOLS FOR PROBLEM SOLVING


Electrophilic Aromatic Substitution Reactions

Fridel–C
ation rafts acy
Bromin Frie
d
la tion
el–C
tion raft
rina Su sa lkyla O
Chlo lph
Desulphonation

on tion
ati
on
ion

Br2 Cl2 HNO3 Dilute CH3Cl Cl


rat

Fuming AICl3
Nit

AIBr3 AICl3 H2SO4 H2SO4 AICl3


H2SO4
O
Br Cl NO2 SO3H CH3

Clemmensen
1. KMnO4 reduction
Excess
Fe or Zn. NaOH, heat Zn(Hg). HCl
Reduction NBS
HCl Benzylic 2. H3O+ heat
bromination Oxidation
NH2 CBr3 O

OH

Chapter-14.indd 471 8/8/2016 2:50:01 PM


472 Chapter 14 Aromatic Hydrocarbons

SOLVED EXAMPLES
Nomenclature Solution
(b) According to Hückel’s rule, two conditions should sat-
1. The correct name for the compound shown below is isfy for aromaticity. The planar cyclic compound with
Br (4n + 2) p electrons are aromatic compounds.
I. This carbon ring contains 6 p electrons with all car-
bon atoms in sp2 hybridized state. The angle strain in
Br
this ring is not much so it is also planar and aromatic.
NH2 II. Though nitrogen atoms can avail lone pairs for aro-
(a) 3,4-dibromoaniline (b) 2,4-dibromoaniline maticity, but in this case it is not required. Ring con-
(c) 2,5-dibromoaniline (d) 3,6-dibromoaniline tains 6 p electrons and all atoms in sp2 hybridized
state. So, the compound is aromatic.
Solution III. It contains 8 p electrons, so it is not an aromatic
(c) The correct IUPAC name of the given compound is compound.
2,5-dibromoaniline. IV. Ring contains 6 p electrons with all carbon atoms sp2
4 hybridized, so it is an aromatic compound.
3 Br
5 4. For the compounds given in the options, direction of dipole
2 6 moment is indicated by an arrow. Which of the following
Br 1 implies correct direction of dipole moment?
NH2

2. 4-Amino-2-chlorophenol is (a) (b)


N
Cl O N H
OH
(a) (b)
Cl

NH2 NO2 (c) (d)


OH N
1 H
Cl Cl 2 6
Solution
(c) O (d) 3 5 (b) The nitrogen atom is more electronegative than carbon,
4 so, in general the C N bond polarization is from carbon
H2N NH2 to nitrogen. But in the case of pyrrole, the lone pair of
Solution nitrogen is involved in the aromaticity of the compound
due to which nitrogen becomes positively charged in
(d) The structure of 4-amino-2-chlorophenol is the contributing resonance structures. The resonance
OH hybrid implies that all carbon atoms in it are negatively
1 charged and nitrogen is positively charged, so the direc-
Cl 2 6
tion of dipole moment would be away from the nitrogen
3 5 atom.
4 − −
NH2
− −
N +N +N +N +N
Structure of Aromatic Compounds H H H H H
3. Which of the following molecules would you expect to be Resonance contributing structures of pyrrole
aromatic? d− d−

+ + − + d− d−
N
d +N
H
N Resonance hybrid
+
I II III IV

(a) I, II and III only (b) I, II, IV only The compound is very unstable since it
(c) II, III, IV only (d) All are aromatic contains antiaromatic rings. The p -bond between the

Chapter-14.indd 472 8/8/2016 2:50:07 PM


Solved Examples 473

rings is broken down in such a way so that both the rings CH3 CH2OH
become stable aromatic rings.
−+ (a) (b)

OH
This implies that five membered ring is negatively ionized CH3
CH3
and three-membered ring is positively ionized. Therefore,
the dipole moment’s direction is incorrectly mentioned OH
in the option (c); it should be in opposite direction. (c) (d)

Quick Tip/Alternate solution Though the dipole moment’s OH


direction is always towards more electronegative atom, but (AIEEE 2006)
resonance and aromaticity can change directions, as can be
Solution
seen in the case of pyrrole.
(a) Both the groups, CH3 and OH are ortho, para directing
5. How many p electrons does the smallest aromatic substance in nature therefore Br attaches at 2, 4 and 6 positions in
contain? option (a).
(a) 1 (b) 2 (c) 3 (d) 4 9. Toluene is nitrated and the resulting product is reduced with
Solution tin and hydrochloric acid. The product so obtained is diazo-
(b) Cyclopropenyl cation (has two p electrons). tized and then heated with cuprous bromide. The reaction
mixture so formed contains
6. Which cyclization(s) should occur with a decrease in p electron (a) mixture of o- and p-bromotoluenes.
energy? (b) mixture of o- and p-dibromobenzenes.
+ H2 (c) mixture of o- and p-bromoanilines.
(a)
(d) mixture of o- and m-bromotoluenes.
− (AIEEE 2008)
− + H2
(b) Solution

+ H2 (a)
(c) CH3 CH3 CH3
(d) + H2 NO2
− Nitration
+
Solution
(b) Cyclopentadienyl anion is aromatic in nature, hence Toluene (ortho and
p electron energy decreases. para directing) NO2
Sn/HCl
Sn/HCl
Chemical Properties
CH3 CH3
7. The structure of the major product formed in the following reaction NH2
Cl
NaCN
DMF

I NH2
NaNO2/HCl
is NaNO2/HCl

CN CN CH3 CH3
(a) (b)
NC N2+Cl−

CN I
Cl CN
(c) (d) N2+Cl−
CuBr/HBr
Sandmeyer’s
reaction
CN I
CH3 CH3
(AIEEE 2006)
Br
Solution
(d) It is a nucleophilic substitution reaction. Cl is replaced by CN.
8. The structure of the compound that gives a tribromoderivative
on treatment with bromine water is Br

Chapter-14.indd 473 8/8/2016 2:50:14 PM


474 Chapter 14 Aromatic Hydrocarbons

10. What is the order of rate of electrophilic substitution reaction Solution


for the following compounds? (c) The reaction sequence in option (a), direct alkylation
Cl
CH3 followed by bromination is not a proper method for
(I) (II) adding bromine at meta-position to the ethyl group in
benzene ring because alkyl group is ortho, para direct-
ing. It will require the presence of a group on benzene
ring which is meta directing. Also, direct alkylation will
C(CH3)3 F give polyalkylated benzene ring, so we should avoid it
for monoalkylation.
(III) (IV) In the reaction sequence in option (b), isomers of
1-bromo-3-ethyl benzene would be produced because
bromine shows +R effect due to which it is ortho, para-
(a) I > III > II > IV (b) III > I > II > IV directing group.
(c) I > III > IV > II (d) IV > II > III > I The sequence which will give good yield is given in
option (c).
Solution
COCH3 COCH3
(c) The electrophilic substitution reaction takes place
through two-step mechanism in which the first step is CH3COCl Br2/FeBr3
the rate determining step. The first step is the attack of AlCl3
electrophile on benzene ring and can be accelerated by Br
increasing electron density on benzene ring.
Zn(Hg), HCl
The electron density in benzene ring is increased
or decreased according to the substituent on the ring.
According to electron displacement effects, the groups CH2CH3
attached to the given compounds have the following
electron withdrawing or electron donating tendencies:
– I effect order: F > Cl and + I effect order: tert-butyl
group > methyl group Br
The reaction rate will be more in case of methyl In the reaction sequence in option (d), NO2 group is
group because it is further stabilized by hyperconjuga- deactivating (more than COCH3) when attached with
tion which is not possible in tert-butyl group. benzene ring, so, Friedel–Craft alkylation is not possible
For +R effect: F > Cl since lone pairs of fluorine are in in the presence of such a highly deactivating group.
2p orbitals which results in more effective overlapping
between fluorine and carbon atoms. So, in the case of flu- 13. Which of these is the rate determining step in the nitration of
oride group, the reaction rate will be more, even though it benzene?
is more electron withdrawing than chloride group. (a) Protonation of nitric acid by sulphuric acid.
(b) Protonation of sulphuric acid by nitric acid.
11. Suitable solvent for Friedel–Crafts alkylation of bromoben- (c) Loss of a water molecule by the protonated species to
zene is produce the nitronium ion.
(a) benzene. (b) toluene. (d) Addition of the nitronium to benzene to produce the
(c) nitrobenzene. (d) propylbenzene. arenium ion.
Solution Solution
(c) Because during alkylation of bromobenzene the solvent (d) Step 1
may undergo Freidel–Craft reaction, if benzene is used as O
a solvent. It itself undergo Friedel–Craft reaction but we O
use nitrobenzene as a solvent because it resist to reac- N+ −
N+ slow O
tion to occur. So nitrobenzene & not benzene is a suitable rate determining +
solvent for Friedel–Craft alkylation of bromobenzene. O step

12. The presence of a group on benzene ring affects the fur- (Nitronium ion)
ther reactions of the compound. In attempts to produce 1-
bromo-3-ethylbenzene from benzene, which one of the fol- Step 2
lowing reaction sequence would produce good yield of the O
product? + −
N O H NO2
(a) (i) CH3CH2Cl + AlCl3, (ii) Br2/FeBr3 fast
(b) (i) Br2/FeBr3, (ii) CH3CH2Cl + AlCl3 H O H −H3O+
+
(c) (i) CH3COCl + AlCl3, (ii) Br2/FeBr3, (iii) Zn(Hg) + HCl
(d) (i) HNO3 + H2SO4, (ii) CH3CH2Cl + AlCl3, (iii) Sn/HCl, HO− Step 1 is the rate determining step, where nitronium ion
(iv) NaNO2, HCl/CuBr reacts with benzene to produce the arenium ion.

Chapter-14.indd 474 8/8/2016 2:50:18 PM


Solved Examples 475

14. What would be the product of the following reaction sequence? 16. What is the product of the following reaction?
OCH3
OH O
1. SOCl2
1. H AlCl3
2. C6H6, AlCl3
O 3. Zn(Hg), HCl, heat 2. Zn(Hg), HCl reflux

O
NO2
(a) (b)
OCH3 OCH3 O
OH
(c) (d) (a) (b)

NO2 NO2
Solution
OCH3 OCH3
(b)

OH 1. SOCl2
Cl (c) (d)
+
O O NO2 NO2 O
2. AlCl3
Solution
O (a) This is a Friedel–Crafts acylation followed by a
Clemmensen reduction at the ortho position.
3. Zn(Hg)
OCH3 OCH3
HCl, ∆ O

1. H AlCl3
15. What is the product of the following reaction? 2. Zn(Hg), HCl reflux

HNO3/H2SO4
NO2 NO2
NO2 When a meta directing group is para to the ortho-para
O2N directing group, the incoming group primarily goes to
ortho to the ortho-para directing group.
O2N
(a) (b) 17. For the following reaction what are the reagents A and B,
respectively:
NO2 Cl
NO2 NO2

(c) (d)
A
1.
NO2 B NO2
2.

Solution NO2
2.
B
(d) Nitration of the benzene ring occurs through electro- 1.
A
philic aromatic substitution. Benzene reacts with hot
concentrated nitric acid in the presence of concentrated
sulphuric acid to yield nitrobenzene. Sulphuric acid Cl
increases the rate of the reaction by increasing the con-
(a) Cl2 + AlCl3, conc. HNO3 + H2SO4
centration of the electrophile, the nitronium ion (NO2+ ).
(b) conc. HNO3 + H2SO4, Cl2 + AlCl3
NO2 (c) Cl2 + HNO3, H2SO4 + AlCl3
(d) Cl2, conc. HNO3
HNO3/H2SO4 Solution
(a) In the first reaction, chlorination is followed by nitration.

Chapter-14.indd 475 8/8/2016 2:50:27 PM


476 Chapter 14 Aromatic Hydrocarbons

Cl Cl S S Br S
Br2
+
Cl2 + AlCl3 conc. HNO3 FeBr3
H2SO3
Br

NO2 20. Identify the compound C in the following reaction sequence.


1. Oleum conc. HNO Zn dust
C6H6 2.
 → A  3
→ B 

→C
Chloro group or halogen as a substituent on the ring NaOH H SO 2 4
3. H
+

deactivates the ring towards substitution (electron


withdrawing) but is  ortho,  para director is for the sub- NO2 O2N NO2
stituents. These are two conflicting characteristics
as  ortho,  para orientation is characteristic of electron (a) (b)
releasing groups. This is observed because halogens
withdraw electrons through inductive effect and their NO2 NO2
electron-releasing characteristic is due to resonance
effect. The resonance effect of the halogen group is just OH
as important as its inductive effect in electrophilic aro- O2N NO2 NO2
matic substitution, and this resonance effect makes the
halogen group electron releasing. (c) (d)

NO2 NO2
NO2
conc. HNO3 Cl2 + AlCl3 Solution
H2SO4 (b) In the first step, the electrophilic substitution of benzene
Cl with oleum forms benzenesulphonate which on fusion
NO2 being an electron withdrawing group will be with alkali yields sodium phenoxide. This on acidifica-
meta directing, thus m-chloronitrobenzene is formed. tion forms phenol.
In the second step, the nitration of phenol with concen-
18. Among the following compounds the one that will not trated nitric acid leads to formation of picric acid in poor
undergo Friedel–Crafts alkylation yield. So phenol is first converted to 2,4-disulphonated
OCH3 C2H5 product and then treated with conc. nitric acid.
In the third step, the OH group is removed by passing
(a) (b) over zinc dust to obtain trinitrobenzene.

SO3H OH
NO2
CH3
1. Oleum 1. NaOH
(c) (d) 2. H+

conc. HNO3 H2SO4


Solution
(d) The Friedel–Craft reaction does not take place with OH
nitrobenzene because NO2 is strongly deactivat- O2N NO2 O2N NO2
ing group. Moreover, any co-ordination of AlCl3 with Zn dust
unshared electrons of oxygen of the NO2 group would
further deactivate the ring making the NO2 group
more electrophillic. NO2 NO2
19. If thiophene is an aromatic molecule and reacts similarly as
benzene, how many (neutral) monobromothiophenes could 21. Arrange the following set of compounds in order of their
be obtained in the following reaction? decreasing relative reactivity with an electrophile E+:
(I) Chlorobenzene
S (II) 2,4-dinitrochlorobenzene
Br2 (III) p-nitrochlorobenzene.
FeBr3 (a) I > II > III (b) I > III > II
(c) III > II > III (d) II > I > III
(a) 1 (b) 2 (c) 3 (d) 4
Solution
Solution
(b) Stronger electron-withdrawing groups decrease
(b) There will two possible products. The reaction is the reactivity of benzene rings towards electrophilic

Chapter-14.indd 476 8/8/2016 2:50:33 PM


Solved Examples 477

substitution reactions. Therefore, the order of reactivity is 25. The reaction of toluene with chlorine in presence of ferric
as follows: chloride gives predominantly
Chlorobenzene > p-nitrochlorobenzene > (a) benzoyl chloride. (b) m-chlorotoluene.
2,4-dinitrochlorobenzene. (c) benzyl chloride. (d) o-and p-chlorotoluene.
22. Iodination of benzene carried out in the presence of Solution
(a) H2SO4 (b) HNO3 (d) CH3 is electron releasing and ortho, para directing.
(c) HIO3 (d) Both (b) and (c).
26. What would you expect to be the major product obtained
Solution from the following reaction?
(d) Iodination of benzene is carried out in the presence of O2N
nitric acid or iodic acid because these acids oxidizes O
HI produced during the reaction to I2, and the reaction + AlCl3

thus proceeds in the forward direction. Cl


HNO O2N
C6H6 + I2 
3
→ C6H5 + HI O
(a)
23. Provide the structure of the major product(s) from the follow-
ing reaction.
CH2CH2CH3 O2N
O
Br2
(b)

O2N O
CH2CH2CH3
(c)
CH2CH2CH2Br
(a) (b)
O2N
Br
CH2CHBrCH3 CHBrCH2CH3
(d)
(c) (d)
O
Solution
Solution (a) The major product would result from Friedel–Crafts acyl-
(d) The major product is (1-bromopropyl) benzene as it ation of methyl–substituted benzene ring.
involve the formation of stable benzyl cation as an O2N
intermediate. O
CHBrCH2CH3 + AlCl3
CH2CH2CH3
Cl
Br2 O2N
hν O

24. Arrange the following compounds in order of decreasing


reactivity in electrophilic substitution: Major product
O O Br NO2 27. Which of the following does not undergo oxidation with alka-
line KMnO4 followed by acidification with dilute HCl to give
benzoic acid?
(a) Toluene (b) Ethylbenzene
(c) Isopropylbenzene (d) tert-Butylbenzene
I II III IV V Solution
(a) V > II > I > III > IV (b) II > V > III > I > IV (d) tert-Butylbenzene [C6H5C(CH3)3] does not contain a ben-
(c) IV > I > III > V > II (d) III > II > I > IV > V zylic hydrogen, hence, it does not undergo oxidation.

Solution 28. Provide the product for the following reaction?


(b) II > V > III > I > IV Na2Cr2O7/H2SO4/H2O
Moderately Weakly Weakly Moderately Strongly excess
activating activating deactivating deactivating deactivating

Chapter-14.indd 477 8/8/2016 2:50:43 PM


478 Chapter 14 Aromatic Hydrocarbons

O Br2
No reaction
O CCl4
COH

COH 1-Hexene on reaction with bromine in organic solvent


(a) (b) COH like carbon tetrachloride gives dibromide due to which
bromine loses its original red brown colour to give a col-
O ourless compound.
O O Br2 Br
CCl4
CH CCH3 Br

(c) (d) 31. Which dichlorobenzene might theoretically yield three


CH CCH3 mononitro products?
(a) o-Dichlorobenzene (b) m-Dichlorobenzene
O O
(c) p-Dichlorobenzene (d) All of these
Solution Solution
(b) The reaction is (b) m-Dichlorobenzene would yield three mononitro prod-
O ucts theoretically. The reaction is as follows
COH Cl Cl Cl Cl
Na2Cr2O7/H2SO4/H2O
HNO3
excess Cl Cl + Cl Cl
COH H2SO4

O +
Cl
29. What is the Birch reduction product of the following reaction?
Cl
Na, NH3
EtOH
Cl

32. Predict the product for the following reaction.


(a) (b) O

Na, CH3OH
NH3
(c) (d)
O
O
Solution
(d)
(a) (b)
Na, NH3 − H+
EtOH
O O
Radical anion is
formed at b carbon
(c) (d)
30. Which reagent(s) would serve as the basis for a simple chemical
test that would distinguish between benzene and 1-hexene?
(a) NaOH in H2O (b) Br2 in CCl4
(c) AgNO3 in C2H5OH (d) NaHSO3 in H2O Solution
(a) The reaction is
Solution
O O
(b) The distinction can be made by using the reagent Br2 in
CCl4. Benzene does not react with bromine unless FeCl3
or AlCl3 is added in the solution to generate an electro- Na, CH3OH
phile. Hence, no colour change is observed. NH3

Chapter-14.indd 478 8/8/2016 2:50:54 PM


Advanced Level Problems 479

ADVANCED LEVEL PROBLEMS


1. In the following reaction, (b) Statement 1 is True, Statement 2 is True; Statement 2 is
NOT a correct explanation for Statement 1
O (c) Statement 1 is True, Statement 2 is False
(d) Statement 1 is False, Statement 2 is True
N conc. HNO3
(IIT-JEE 2008)
H X
conc. H2SO4
Solution
the structure of the major product X is
(c) In bromobenzene, it is the mesomeric effect which directs
the incoming electrophile.
O
NO2 Br Br Br
(a) N Br
H FeBr3
+ Br2 +

O2N
O Br
(b) 1,2-dibromo 1,4-dibromo
N benzene benzene
H (major product)

O 3. Which of the following molecules, in pure form, is (are) unsta-


ble at room temperature?
(c) N
H
NO2 (a) (b)

O O
O
(d) O2N N (c) (d)
H
(IIT-JEE 2007)
(IIT-JEE 2012)
Solution
Solution
(b) The lone pair of electrons on the nitrogen atom activates
(b), (c) The compounds in (b) and (c) does not follow Hückel’s
the ring, that is, increases the electron density at ortho and
rule, that is, they do not have (4n + 2)p electrons, so they are
para positions, so the substitution will occur at ortho or
antiaromatic and are unstable at room temperature.
para (preferably) positions.
4. Among P, Q, R and S the aromatic compound(s) is/are
O Cl
N C
AlCl3
H P
conc. HNO3
conc. H2SO4 NaH
Q
O
(NH4)2CO3
O 2N NH C R
100–115°C
O O
2. Statement 1: Bromobenzene upon reaction with Br2/Fe gives O
1,4-dibromobenzene as the major product.
Statement 2: In bromobenzene, the inductive effect of the HCl
S
bromo group is more dominant than the mesomeric effect in
direction the incoming electrophile.
(a) Statement 1 is True, Statement 2 is True; Statement 2 is a (a) P (b) Q (c) R (d) S
correct explanation for Statement 1 (IIT-JEE 2013)

Chapter-14.indd 479 8/8/2016 2:51:00 PM


480 Chapter 14 Aromatic Hydrocarbons

Solution
(a), (b), (c), (d) The reactions involved are
(a) (b)
Cl
+
(a) AlCl3 −
AlCl4
Br
Aromatic (c) (d)
(P)

NaH + Solution
(b) Na + H2
(a) This reaction starts with the formation of electrophile
Aromatic which is required for this reaction, but electrophile is car-
(Q) bocation in the Friedel–Craft alkylation. So, it rearranges
to more stable carbocation by ring expansion. After rear-
(c) (NH4 )2CO3 ∆
→ 2NH3 + CO2 + H2O rangement, electrophilic substitution gives the product
given in option (a).
+ NH3 +
O O− NH2 O NH2 OH 7. What is the major product of the following reaction?
OO
H H
N O
H H H H Br2/FeBr3
+
N N N

−2H2O
HO OH −O OH
Aromatic
H H
(R) O N O
N
O OH
(a) (b)
+ Br
HCl Cl−
(d)
Br
H Br
Aromatic Br N O H
(S) N O

5. In the following sequence of reactions: (c) (d)


KMnO SOCl H /Pd
Toluene 4
→ A 
2
→ B BaSO

2
→C
4

the product C is Solution


(a) C6H5CH3 (b) C6H5CH2OH (a) There are two benzene rings in the reactant ring I and ring II
(c) C6H5CHO (d) C6H5COOH as shown in the figure.
(JEE Advanced 2015) H
N O
Solution
I
(c)
CH3 COOH COCl CHO II

KMnO4 SOCl2 H2/Pd


We can see that ring I contains two substituents NHCOAr
BaSO4 and Ar and ring II contains two groups CONHAr and
Ar. The group NHCOAr is a benzene ring activator, but
Toluene Benzoic acid Benzoyl chloride Benzaldehyde CONHAr is a deactivator, so the reaction will take place
(A) (B) (C) at ring I.
Further, NHCOAr is more activating than Ar group,
6. The major product of the following reaction is
so the electrophile would attack according to NHCOAr
and bromine should add at ortho or para positions w.r.t.
Br
NHCOAr. However, due to stearic hindrance, the ortho
AlBr3 compound is formed in small amount, and the para com-
pound is the major product.

Chapter-14.indd 480 8/8/2016 2:51:06 PM


Advanced Level Problems 481

8. In general, there are three steps to an electrophilic aromatic Fluorobenzene reacts faster because fluorine has more
substitution reaction. These are: positive resonating effect due to good overlap of 2p–2p
(a) formation of an _____________; orbitals of fluorine and carbon. Higher resonance effect
(b) reaction with an aromatic ring to form an ___________; concentrates more electron density at ortho or para
and position.
(c) loss of a _________ to reform the aromatic system.
+X
Solution X X
Electrophile; arenium ion; proton E
E+ E
9. The pKa of cyclopentadiene is found to be 16 while, by contrast, H
X = F, Cl, Br, or I
that of cycloheptatriene is found to be 36. Explain the large dif-
ference found in the two pKa values. Since in nucleophilic aromatic substitution reactions,
the first step is the rate determining so greater inductive
Solution
effect of fluorine helps to remove electron density from
The 20 order of magnitude difference in acidity of cyclo- benzene and eases the attack of nucleophile. Though the
pentadiene compared with cycloheptatriene lies in the C F bond is the strongest, but breaking of the bond is
fact that the conjugate base of cyclopentadiene is the relatively faster.
aromatic cyclopentadienyl anion, a very stable conjugate
base species. However, in the case of cycloheptatriene, X
removal of a proton produces the cycloheptatrienyl anion,
a cyclic, conjugated, planar system with 8 p-electrons, that
is antiaromatic. + Y−

B−
NO2

H H Aromatic Slow
H
anion
H H H X Y X Y X Y X Y X Y
− − −
Fast
B− + X−


Antiaromatic NO2 N N NO2 NO2
−O + −O +
O O−
10. Select the correct statement about the reaction of benzene
and its derivatives. Aniline cannot undergo Friedel–Craft’s reaction because
(a) C6H6 and C6D6 both react with concentrated HNO3 at the the catalyst used in this reaction reacts with nitrogen
same rate. atom and makes it a strong electron withdrawing group,
(b) Chlorobenzene reacts faster than fluorobenzene in elec- which deactivates the benzene ring towards electrophilic
trophilic substitution reactions. substitution reactions.
(c) Chlorobenzene reacts faster than fluorobenzene in + −
nucleophilic substitution reactions. H2N H2N AlCI3
(d) Aniline produces p-methyl aniline on reacting with chlo-
romethane in the presence of BF3. AlCI3

Solution
(a) In electrophilic aromatic substitution reaction, the first
11. Give reasons:
step (attack of the electrophile) is the slowest step, so the
rate of reaction depends on this step only. C D bond is O O O
stronger than C H but this bond breaking takes place N N N
in the second step of reaction which is very fast. Hence, NO2
rates of reaction for both C6H6 and C6D6 are the same. conc. HNO3
+
(a) (i) conc. H2SO4
B
+ H H H
Slow
Y Y Y NO2
+ Y+
+ + NO2 NO2

fast (ii) conc. HNO3


conc. H2SO4
Y NO2
HB+ +

Chapter-14.indd 481 8/8/2016 2:51:11 PM


482 Chapter 14 Aromatic Hydrocarbons

A total of three products are formed during ozonolysis


of 1,2-dimethylbenzene (o-xylene) which means that
o-xylene is a resonance hybrid of the two forms, I and II
because all the three products cannot be obtained from
Pd/ C
(b) any one of the structures.
3 moles of H2
13. Benzene, while unusually unreactive with electrophiles, will
react under certain conditions. However, when it does react,
it does so by ___________ rather than by addition.
Solution
Solution Benzene reacts by substitution rather than by addition
(a) (i) Due to the presence of lone pair on nitrogen atom NO 14. What is the correct order of reagents to achieve the following
group is electron donating and ortho, para directing. synthesis?
(ii) NO2 group is electron withdrawing and meta
directing.
O Br
(b) Due to reduction of central ring, three four membered
antiaromatic rings become stable while on reduction of
terminal ring only one antiaromatic ring can be stabilized.
12. How many products are formed by the ozonolysis of
1,2-dimethylbenzene (o-xylene)?
(a) 2 (b) 3 (c) 4 (d) 5 (a) NBS, light, CCl4
(b) (1) Zn(Hg), HCl reflux (2) NBS, light, CCl4
Solution
(c) CH3MgBr
o-Xylene (1,2-dimethylbenzene) can be written in two forms, (d) HBr
I and II:
Solution
CH3 CH3 (b) The ketone is first reduced with Clemmensen reduc-
CH3 CH3 tion, then the side chain is halogenated through radical
halogenation.
15. Draw all significant resonance structures for the cyclopenta-
I II dienyl anion.
1. O3 1. O3 Solution
2. Zn/H2O 2. Zn/H2O
The resonance structures for the cyclopentadienyl anion are
CH3 CH3 as follows:
2 2 −
C C
1 1 −
O CH O O C CH3

+ O
1,2-Dimethyl glyoxal −
CH O
CH O +
Glyoxal CH O
2
CH O −
Glyoxal

PRACTICE EXERCISE
Level I (c) CH2 CH CH2 + H2

(d) CH2 − −
Single Correct Choice Type CH CH2 + H2
1. Which cyclization(s) should occur with a decrease in p elec- 2. Toluene, when treated with Br2/Fe, gives p-bromotoluene as
tron energy? the major product because the –CH3 group
(a) CH + H2 (a) is meta directing.
2 CH CH CH2
(b) deactivates the ring.
(b) CH2 CH CH2 + + + H2
(c) activates the ring by hyperconjugation.
(d) none of the above.

Chapter-14.indd 482 8/8/2016 2:51:17 PM


Practice Exercise 483

3. Phenyl magnesium bromide reacts with methanol to give O


(a) a mixture of anisole and Mg(OH)Br.
(b) a mixture of benzene and Mg(OMe)Br. (c) C CH2
(c) a mixture of toluene and Mg(OH)Br. Br
(d) a mixture of phenol and Mg(OMe)Br. O
4. The product obtained in the reaction is (d) C CH2 Br
Cl , hv
C6H5CH2 CH3 2

273 K

Br
(a) C6H5CH(Cl)CH3 (b) C6H5CH2CH2Cl
(c) C6H5C(Cl2)CH3 (d) Both (a) and (c) 9. In the Friedel–Crafts acylation, the electrophile is
5. Benzyl chloride (C6H5CH2Cl) can be prepared from toluene by (a) C6H5+ (b) AlCl3−
chlorination with
(a) SO2Cl2 (b) SOCl2 (c) Cl2 (d) NaOCl (c) CH3CO+ (d) C6H5CH2+

6. Which cyclization(s) should occur with an increase in p elec- 10. Arrange the following in order of decreasing boiling point:
tron energy? CH3
(I) (II)
(a) + H2

(b) + + H2 CH3
+
H3C CH3
− CH3
+ + H2 (III) (IV)
(c)
+ CH3
CH3
(d) + H2 CH3
+
(a) I > II > III > IV (b) IV > III > II > I
7. In which of the following polysubstitution takes place? (c) I > III > IV > II (d) II > III > I > IV
COCH3 11. In which case is the indicated unshared pair of electrons NOT
(a) + CH3COCl a contributor to the p aromatic system?
S
CH3 (a) (b)
(b) N
+ CH3Cl

NO2
(c) H2SO4 (c) (d) O+
+ HNO3 −
H
SO3H 12. The conversion can be effected using
(d)
+ H2SO4 CH2CH2CH3 CH2CH2CH3

8. The major product obtained in the reaction Br


O
(a) Br2/CCl4 (b) Br2/H2O
Br2
C CH2 (c) Br2/Fe (d) Br2/benzoyl peroxide
FeBr3
13. A Friedel–Crafts reaction of benzene with chloroform
is expected to be produces Cl
O
(a) C6H5CHCl2 (b) C6H5 C C6H5
(a) Br C CH2
C6H5 H
O (c) C6H5 C C6H5 (d) All of these
(b) C CH2 Br H

Chapter-14.indd 483 8/8/2016 2:51:27 PM


484 Chapter 14 Aromatic Hydrocarbons

14. The number of disubstituted products of benzene is SO3H


(a) 2. (b) 3. (c) 4. (d) 5. (c) + H2O
130-150°C
+ H2SO4
HCl
15. The reaction of toluene with chlorine in the presence of ferric
chloride gives predominantly MgBr O COCH2CH3
(a) benzoyl chloride. (b) m-chlorotoluene. (d) + ClCCH2CH3
(c) benzyl chloride. (d) o- and p-chlorotoluene.
16. Among the following, the compound that can be most read- 22. To be aromatic, a compound must be  ___ ,  ___ , contain alter-
ily sulphonated is nating double and single bonds and have a(n)  ___  of electrons.
(a) benzene. (b) nitrobenzene. (a) noncyclic, coplanar, even number
(c) toluene. (d) chlorobenzene. (b) cyclic, symmetric, odd number
(c) cyclic, planar, Hückel’s (4n + 2)p
17. What is the product of the following reaction? (d) cyclic, aliphatic, Hückel’s (4n + 2)p
Cl2/AlCl3 23. Which reagent(s) would you use to carry out the following
transformation?
Cl Ethylbenzene → 2- and 4-chloro-1-ethylbenzene
(a) Cl2, light, and heat (b) Cl2, FeCl3
Cl (c) SOCl2 (d) C2H5Cl, AlCl3
(a) (b) Cl 24. Which of the following orders regarding the acidity of aro-
Cl matic acids is correct?
(a) p-nitrobenzoic acid < benzoic acid < p-anthranilic acid
Cl Cl
(b) p-nitrobenzoic acid > benzoic acid > p-anthranilic acid
(c) (d) (c) p-nitrobenzoic acid > benzoic acid < p-anthranilic acid
(d) p-nitrobenzoic acid < benzoic acid > p-anthranilic acid
Cl 25. The treatment of benzene with isobutene in the presence of
sulphuric acid gives
18. Which of the following will undergo faster dehydro-
(a) isobutylbenzene. (b) tert-butylbenzene.
bromination?
(c) n-butylbenzene. (d) No reaction.
Br Br
26. Number of bonds in benzene is
(a) (b) (a) 6s and 3p. (b) 12s and 3p.
(c) 3s and 12p. (d) 6s and 6p.

Br 27. Which out of the following set of compounds is least reactive


Br with an electrophile?
(a) Toluene (b) p H3C C6H4 NO2
(c) (d)
(c) p O2N C6H4 NO2 (d) Cannot be predicted
28. Which of the following statements regarding the cyclopenta-
dienyl cation is correct?
19. What is the chief product of the Friedel–Crafts alkylation of
(a) It is aromatic.
benzene with 1-butene and HF?
(b) It is not aromatic.
(a) butylbenzene
(c) It obeys Hückel’s rule.
(b) 2-phenylbutane
(d) It undergoes reactions characteristic of benzene.
(c) 2-methyl-1-phenylpropane
(d) t-butylbenzene 29. Which of the following has the highest melting point?
(a) o-Xylene (b) m-Xylene
20. The direct iodination of benzene is not possible because
(c) p-Xylene (d) Toluene.
(a) iodine is an oxidizing agent.
(b) resulting C6H5I is reduced to C6H6 by HI. 30. Anisole is the name commonly assigned to
(c) HI is unstable. (a) hydroxybenzene. (b) aminobenzene.
(d) the ring gets deactivated. (c) methylbenzene. (d) methoxybenzene.
21. Which of the following is Wurtz–Fittig reaction? 31. n-Octane when heated to 773 K under a pressure of 10–20 atm
Br CH2CH3 and in presence of a mixture of Cr2O3, V2O5 and MO2O3 sup-
(a) ported over Al2O3 as catalyst, gives
+ 2Na + BrCH2CH3 + 2NaBr
(a) o-xylene. (b) m-xylene.
(c) p-xylene. (d) all the three.
CH2CH3
AlCl3
(b) + BrCH2CH3 + HBr 32. What is the bond order of C–C bond in C6H6?
anhyd. (a) 1 (b) 1.5 (c) 2 (d) 2.5

Chapter-14.indd 484 8/8/2016 2:51:34 PM


Practice Exercise 485

33. Which of the following is not insoluble in water? 41. Name the final product obtained when benzene is treated with
(a) Benzene (b) 1,3-cyclohexadiene methyl chloride in the presence of anhydrous aluminum chloride.
(c) 1,4-cyclohexadiene (d) All are insoluble. (a) Chlorobenzene. (b) Toluene.
(c) Ethylbenzene. (d) None of these.
34. Which of the following is not a meta-directing substituent
when present on the benzene ring? 42. Benzyl chloride (C6H5CH2Cl) can be prepared from toluene by
(a) NHCOCH3 (b) NO2 chlorination with
(c) N(CH3)3+ (d) C N (a) SO2Cl2. (b) SOCl2 (c) Cl2 (d) NaOCl
35. A mixture of chlorobenzene (1 mol) and acetanilide (1 mol) 43. We now know that the two Kekule structures for benzene are
is allowed to react with Br2 (0.5 mol) in the presence of trace related in the following way:
amounts of FeBr3. What is the principal product of the com- (a) They are each equally correct as a structure for benzene.
peting reactions? (b) Benzene is sometimes one structure and sometimes the
(a) 1-bromo-4-chlorobenzene other.
(b) 1-bromo-2-chlorobenzene (c) The two structures are in a state of rapid equilibrium.
(c) 1-bromo-3-chlorobenzene (d) Neither of the two structures adequately describes ben-
(d) 4-bromoacetanilide zene; benzene is a resonance hybrid of the two.
36. Why would 1,3-cyclohexadiene undergo dehydrogenation 44. What might be predicted to happen when the following sub-
readily? stance undergoes Friedel–Crafts acylation?
(a) It is easily reduced.
(b) Hydrogen is a small molecule. O2 N CH2
(c) 1,3-cyclohexadiene has no resonance energy.
(d) It would gain considerable stability by becoming benzene. A B
37. The nitration of which of the following compounds will be (a) Substitution occurs in ring B, p- to the methylene group.
most easy? (b) Substitution occurs in ring A, o- to the nitro group.
(a) benzene (b) m-dinitrobenzene (c) Substitution occurs in ring A, o- to the methylene group.
(c) toluene (d) None of these (d) Substitution occurs in ring B, m- to the methylene group.

38. Which of the following is not a condition for a compound to 45. What would you expect to be the major product obtained
be aromatic? from the monobromination of meta-dichlorobenzene?
(a) The molecule must possess planarity. Cl Cl Cl
(b) There should be complete delocalization of the p elec- Br
trons in the ring.
(c) The presence of (4n + 2)p electrons in the ring (Hückel’s
rule) where n is the integer. Cl Cl Br Cl
(d) The molecule must have atleast two double bonds. Br
39. 2-Bromo-4-nitroaniline is I II III
Br (a) I
Br
NH2 (b) II
(c) III
(a) (b)
(d) Equal amounts of I and II
NO2
NO2 Multiple Correct Choice Type
NH2
OCH3 46. Predict which of the following systems would be aromatic
and why?
Br
Br NO2
(c) (d) (a) (b)
H2N
NO2
(c) (d)
40. Which of the following structures will have number of elec-
trons more than 4p electrons?
47. Which of the following compounds are aromatic according to
(a) (b) Hückel’s rule?
(a) (b)
− + N
H

(c) (d) (c) H C (d)


2 CH2

Chapter-14.indd 485 8/8/2016 2:51:41 PM


486 Chapter 14 Aromatic Hydrocarbons

48. Benzoic acid may be prepared by the oxidation of 51. If we take FeCl3 in place of AlCl3 in question number 49, the
CH2CH3 product is
CH2OH (a) Only (a)
(a) (b) (b) Only (b)
(c) (a) and (b) both can be possible
(d) Not given
COCH3 OH
Matrix-Match Type
CH2CH3 CH3
(c) (d) 52. Column I (Reaction) Column II (Electrophile)

Passage Type H2SO4


(a) + HNO3 (p) SO3
Paragraph for questions 49–51: The reaction given below is an
example of Friedel-Craft alkylation reaction.
HNO3
R (b) + HNO2 (q) NO2

Lewis
+R X acid + HX H2SO4
(c) + H2SO4 (r) NO−

In number of cases of Friedel–Crafts alkylation, the final prod-


uct is found to contain a rearranged alkyl group. Generally with AlCl3 +
(d) + n-propylbromide (s) CH3 CH CH3
stronger Lewis acid product is rearranged due to enough polar-
ization of complex while with weak Lewis acid no such effect is
observed. Temperature also favours rearranged product.
49. What is electrophile in given reaction? Integer Type
(a) X+ (b) R+
(c) (Lewis acid X)+ (d) None of the above 53. The number of resonating structures for Dewar’s benzene is
______.
54. The number of fused benzene rings in anthracene is ______.
50. + Me3CCH2Cl
AlCl3
Product 55. The ratio of s and p bonds in benzene is ______:1.

Product is 56. The number of degrees of unsaturation in cyclohexadiene is


______.
Me Me
Me Me 57. The number of meta directors among the following species
Me are
Me
(a) (b) NO2, SO3H, Cl2, OH, NH2, CHO?
58. The number of deactivating groups from the list is ______.
NO2, CH3, COCH3, COOH, Cl, I, NH3

Me
Me
Me Me
Me Me
Me
(c) (d)

ANSWER KEY
1. (b) 2. (c) 3. (b) 4. (d) 5. (c) 6. (a)
7. (b) 8. (b) 9. (c) 10. (b) 11. (b) 12. (c)
13. (d) 14. (b) 15. (d) 16. (c) 17. (d) 18. (d)

Chapter-14.indd 486 8/8/2016 2:51:46 PM


Hints and Explanations 487

19. (b) 20. (b) 21. (a) 22. (c) 23. (b) 24. (b)
25. (b) 26. (b) 27. (c) 28. (b) 29. (c) 30. (d)
31. (d) 32. (b) 33. (a) 34. (a) 35. (d) 36. (d)
37. (c) 38. (d) 39. (c) 40. (d) 41. (a) 42. (c)
43. (d) 44. (a) 45. (b) 46. (a), (c) 47. (b), (d) 48. (a), (b)
49. (b) 50. (a) 51. (a) 52. (a) → q; (b)→ r; (c) → p; (d) → s 53. (3)
54. (3) 55. (4) 56. (3) 57. (3) 58. (5)

HINTS AND EXPLANATIONS


Level I 11. (b) Nitrogen is sp2 hybridized, it has sp2 orbitals and p-orbitals.
The p-orbital is involved in the formation of p-bond. Two
Single Correct Choice Type of the nitrogen sp2 overlap with the sp2 orbitals of adja-
1. (b) Conceptual cent carbon atoms, and nitrogen’s third sp2 orbitals con-
tains is the lone pair.
2. (c) Conceptual
12. (c) The required bromination takes place through electro-
3. (b) philic aromatic substitution reaction.
CH2CH2CH3 CH2CH2CH3
C6H5MgBr + CH3OH → Mg(OCH3 )Br + C6H6 Br2/Fe
Phenyl magnesium Methanol Benzene FeBr2
Br
bromide (strong acid) (weak acid)

4. (d) 13. (d)

Cl Cl +
CHCl3 + AlCl3 CHCl2 + AlCl−4
Cl2, hv
C6H5CH2 CH3 C6H5 C CH3 + C6H5 C CH3 (Electrophile) Cl
273 K
H Cl CHCl2 CH
+
Cl2 + CHCl2
5. (c) C6H5CH3 C6H5CH2Cl + HCl AlCl3

6. (a) Cyclobutadiene is antiaromatic hence it will show an


AlCl3
increase in p electron energy.
7. (b) Friedel–Crafts alkylation reaction forms polysubstitution C6H5
products. Due to activating effect of an alkyl group con-
nected to an aromatic ring, the monoalkylated reaction CH
product is more reactive towards electrophillic substitu- H5C6 C6H5
tion than the original starting material.
14. (b) Benzene yields three isomeric disubstituted products
8. (b) In ring A, CO group as deactivates the benzene ring for
electrophilic substitution. Y Y Y
O
Y
C CH2
Y
A B
Br2 FeBr3 Y
15. (d)
O CH3 CH3 CH3
C CH2 Br Cl
Cl3
9. (c) +
anhyd. FeCl3
O O
+
o-Chlorotoluene Cl
CH3 C Cl + AlCl3 CH3 C + AlCl−4
p-Chlorotoluene
10. (b) With increase in molecular weight, boiling point increases. 16. (c) Toluene is readily sulphonated because it contains methyl
Thus, the correct order is IV > III > II > I. group, which activates the ring.

Chapter-14.indd 487 8/8/2016 2:51:52 PM


488 Chapter 14 Aromatic Hydrocarbons

17. (d) Halogenation of the benzene occurs through electro- Mechanism


philic aromatic substitution.
H+ +
18. (d) H3 C C CH2 H 3C C CH3 +
Br
CH3 CH3
H −HBr
H
CH3 CH3

H3C C CH3 H 3C C CH3


19. (b) Protonation of 1-butene will generate a secondary car- H
bocation that adds to benzene to yield 2-phenylbutane. HSO4− +

20. (b) Conceptual


21. (a) Conceptual 26. (b) 12s and 3p
CH
22. (c) To be aromatic, a ring must
• have one 2p orbital on each of its atoms. CH CH
• be planar or nearly planar, so that there is continuous
overlap or nearly continuous overlap of all 2p orbitals of CH CH
the ring. CH
• have 2, 6,10,14,18, and so forth pi (p) electrons in the
27. (c) Stronger electron-withdrawing groups decrease the reac-
cyclic arrangement of 2p orbitals.
tivity of benzene rings towards electrophilic substitution
23. (b) reactions. Therefore, p-NO2 C6H4 NO2 is least reactive.
28. (b) Conceptual
Cl2
FeCl3
+ 29. (c) p-xylene has highest melting point among the given
Cl Cl compound. This is due the fact it has symmetrical structure,
Ethyl 2-Chloro-1-ethyl 4-Chloro-1-ethyl therefore, its molecules can easily pack and thus greater
benzene benzene benzene energy is required to break its lattice.
30. (d) The IUPAC name of anisole is methoxybenzene.
24. (b) Electron-withdrawing group increases the acidity of CH3
the aromatic acids. Here, nitro group is an electron- O
withdrawing group that increases the acidity. In anthra-
nilic acid NH2 group is electron donor group, which
decreases the acidity.

COOH COOH COOH 31. (d) CH3 − CH2 − CH2 − CH2 − CH2 − CH2 − CH2 − CH3
Cr O , V O and Mo O
Al
2 3 2 5 2 3

O , 773 K, 10 − 20 atm

> >
2 3

CH3 CH3 CH3


CH3
NO2 NH2
+ +
p-Nitrobenzoic acid Benzoic acid p-Anthranilic acid CH3
CH3
25. (b) o-Xylene p-Xylene m-Xylene
CH3
32. (b) The bond order of C C bond in benzene is 1.5.
H3C C CH3 33. (a) Benzene is slightly soluble in water, while both
1,3-cyclohexadiene and 1,4-cyclohexadiene are insolu-
H2SO4
H 3C C CH2 + ble. This is because in the case of benzene, the electrons
are delocalized which are easily polarized resulting in
CH3 greater attraction, whereas in cyclohexadiene, there is no
tert-Butylbenzene
delocalization.

Chapter-14.indd 488 8/8/2016 2:51:56 PM


Hints and Explanations 489

34. (a) NHCOCH3 is ortho, para directing while all other groups Multiple Correct Choice Type
are meta directing.
46. (a), (c) Only those compounds will be aromatic which follow
35. (d) Chloro group in chlorobenzene is a ring deactivating Hückel’s rule and are planar.
group, whereas in acetamido group in acetanilide is a ring (a) There are 10p electrons, that is, n = 2, so it is aromatic.
activator. Thus, the principal product of the competing (b) There are 4p electrons so n is not an integer, it is
reaction will be 4-bromoacetanilide. antiaromatic.
(c) There are two benzene rings with 6p electrons, that is,
36. (d) Conceptual
n = 1, so it is aromatic.
37. (c) In case of toluene, the presence of CH3 group increases (d) The molecule is not planar although it has 6p electrons,
the electron density on the ring and hence nitration so it is not aromatic.
becomes easier. In case of m-dinitro benzene, due to
47. (b), (d)
the presence of electron-withdrawing NO2 groups, the
electron density decreases and the electrophilic substi-
tution becomes difficult. In case of benzene which has
no group attachments will undergo nitration more eas- (a) (b)
N
ily than m-dinitrobenzene but less easily than toluene. H
The order of reactivity will, therefore, be toluene > ben- Non aromatic Aromatic
zene > m-dinitrobenzene.
38. (d) Conceptual H 2C CH2

39. (c) The correct structure of 2-bromo-4-nitroaniline is: (c) (d)

Br NO2 Non aromatic Aromatic

48. (a), (b) The side chain of any type on benzene ring is oxidized
H2N to benzoic acid.
CH2 CH3 COOH
40. (d) Cycloheptadienyl ion has more than 4p electrons and
hence is aromatic. KMnO4
+ 6[O] + CO2 + 2H2O
41. (a) The reaction is Friedel–Craft alkylation reaction.
anhyd. AlCl
C6H6 + CH3Cl 
3
→ C6H5Cl
CH2 OH COOH

42. (c) The reaction is + 2[O]


KMnO4
+ 2H2O
CH3 CH2Cl
Cl2
+ HCl
Passage Type

Toluene 49. (b) First step is the generation of electrophile.

43. (d) Conceptual R X + AlX3 R+ + AlX−4


44. (a) Substitution occurs in ring B, p- to the methylene group as (Lewis (Electrophile)
acid)
ring A contains nitro group which is highly deactivates the 50. (a)
ring towards electrophilic substitution.
Me Me
45. (b) Me
Cl Cl AlCl3
+ Me3CCH2Cl
Br2
FeBr3
Cl Cl 51. (a) Lewis acid generates the electrophile. FeCl3 also acts as
a Lewis acid. Thus, replacement of FeCl3 in place of AlCl3
Br would not affect the product.
meta-Dichlorobenzene Major product

Chapter-14.indd 489 8/8/2016 2:52:01 PM


490 Chapter 14 Aromatic Hydrocarbons

Matrix-Match Type 55. (4) Benzene contains 12s bonds and 3p bonds. Hence, ratio
is 4:1.
52. (a) → q; (b) → r; (c) → p; (d) → s

Integer Type
53. (3) Three possible Dewar structures were considered as
56. (3)
minor resonance contributors in the overall description
of benzene. The major resonance contributors are of
course the two possible Kekulé structures.
Kekule structures Dewar structures
57. (3) Meta directors: NO2, SO3H, CHO.
58. (5) NO2, COCH3, COOH, Cl and I are deactivating out of which
Cl and I are ortho, para directing, while NO2, COCH3,
COOH are meta directing.
(I) (II) (III) (IV) (V)

54. (3)

Chapter-14.indd 490 8/8/2016 2:52:03 PM


Solved JEE 2016 Questions 491

SOLVED JEE 2016 QUESTIONS


JEE Advanced 2016 In the reaction in option (c), the alkene undergoes electrophilic
addition and the carbocation formed undergoes electrophilic
1. Among the following reaction(s) which gives (give) tert-butyl substitution on the benzene ring to form tert-butyl benzene.
benzene as the major product is(are)
Br Cl +
H+
(a) (b) + H2SO4
NaOC2H5 AlCl3

OH
(c) (d) In the reaction in option (d), the carbocation obtained under-
H2SO4 BF3. OEt2
goes 1,2-hydride shift and the resulting cation undergoes elec-
Solution trophilic substitution on the benzene ring to form tert-butyl
benzene.
(b), (c), (d) The reactions of the given compounds are as
follows:
H+ + −
In the reaction in option (a), tert-butyl benzene is not formed, OH + F3B − OEt2 O BF3
instead tert-butyl bromide undergoes elimination reaction.
H
Br NaOC2H5
Elimination
(E2) + 1,2-Hydride
+ + F 3B OH
shift
In the reaction in option (b), tert-butyl benzene is obtained by
Friedel–Crafts reaction with 2-methyl propyl chloride.

+
+ AlCl3 + AlCl4−
Cl

Chapter-14.indd 491 8/8/2016 2:52:07 PM


Chapter-14.indd 492 8/8/2016 2:52:07 PM
15 Environmental
Chemistry*

Question Distribution in JEE Main

3
No. of Questions

JEE (Main)
2

0
2016 2015 2014 2013 2012 2011 2010 2009 2008 2007

Concept Distribution in JEE Main


Topics Covered
Year
JEE (Main)
2008 Atmospheric Pollution
2012 Soil Pollution
2013 Atmospheric Pollution
2014 Atmospheric Pollution
2015 Particulate Pollution, Water Pollution
2016 Water Pollution, Strategies to Control Air Pollution

∗This chapter is part of JEE Main Syllabus only.

Chapter-15.indd 493 7/30/2016 4:40:13 PM


494 Chapter 15 Environmental Chemistry

SUMMARY
1. The study of chemical species in the environment, their origin, reactions, dispersal, effects and fate in air/water/soil is known as envi-
ronmental chemistry.
2. Disruption in the natural environment caused by the release of harmful contaminants (pollutants) into the air, water or soil by natural
or artificial activities is called environmental pollution.
3. There are three types of pollution: air, water and soil. The air is contaminated with exhaust gases, soil polluted with pesticides used in
agriculture and water with toxic effluents from the industry.
4. Pollutants are classified as primary and secondary pollutants depending on the nature of their interaction with the
environment.
(a) Primary pollutants
These are pollutants which when formed and released into the environment remain there as such, causing harmful effects, for
example NO2, SO2 or NO.
(b) Secondary pollutants
These are pollutants that are formed by the chemical reactions between primary pollutants present in atmosphere or hydro-
sphere. For example peroxyacyl nitrates (PAN) formed by the reaction between primary pollutants, hydrocarbons and oxides of
nitrogen in the presence of sunlight.
5. Regions of atmosphere – Effect of air pollution on these regions
(a) The release of undesirable materials into the atmosphere either by natural phenomena or human activity that adversely affect
the quality of air and make it unfit for breathing is known as air pollution.
(b) The atmosphere of the earth is divided into the following layers:
(i) Troposphere is the layer closest to the earth spanning across 10 km from sea level. There is a lot of turbulence and vertical
mixing in this layer. It contains mainly water vapour and clouds.
(ii) Stratosphere
• Lies between troposphere and mesosphere between 10 km and 50 km above sea level.
• There is no turbulence in this layer and it contains gases such as ozone, nitrogen and oxygen and also some amount of
water vapour.
(iii) Tropospheric pollution
• Caused due to undesirable gaseous and solid emissions from vehicles, industries and refineries and natural activities like
forest fires and volcanoes, etc.
• The major air pollutants present in the troposphere are oxides of sulphur, nitrogen and carbon, and hydrocarbons.

Tip The permissible value of a pollutant at which it does not cause any adverse effect to persons exposed to it for eight
working hours a day is known as its threshold limit value. Certain media present in the environment absorb or take up
some amounts of the pollutants and is known as sink for that pollutant

6. Tropospheric pollutants – Gaseous pollutants (Oxides of carbon, nitrogen and sulphur, hydrocarbon); their sources, harmful
effects and prevention

Pollutant Sources Reactions Effects Prevention


Oxides of Volcanic eruptions, SO2 (g) + O3 (g) → SO3 (g) + O2 (g) Environment: Acid rain, smog, Removal of
sulphur (SOx), burning of fossil fuels, effect on marble statues and sulphur from
mainly SO2 and industrial processes, SO2 (g) + H2O2 (l) → H2SO 4 (aq) buildings, corrosion of metals. the fuels using
SO3 vehicular emission Human life: At low concentration chemical scrub-
of 5 ppm (or 1 ppm in elderly), bers like CaCO3,
causes irritation of respiratory using fuels with
tract and eyes, breathlessness and low sulphur
diseases of lungs. content
Plant life: Long exposure slows
down the formation of chlorophyll
in leaves, causing them to turn
yellow (chlorosis).
(Continued)

Chapter-15.indd 494 7/30/2016 4:40:14 PM


Summary 495

(Continued)

Pollutant Sources Reactions Effects Prevention


Oxides of nitro- Burning of fuels, pro- 1483 K Environment: Formation of smog Use of catalytic
N2 (g) + O2 → 2NO(g)
gen (NOx), mainly duction of explosives, and acid rain; corrosive to the converters,
NO, N2O, NO2 fertilizers and nitric 2NO(g) + O2 (g) → 2NO2 (g) metals and cause cotton and absorption of
acid, etc. rayon fabrics to fade. the emitted NOx
2NO(g) + O3 (g) → NO2 (g) + O2 (g) Human life: NO is toxic, causes could be done
2
respiratory diseases, fatal at high using H2SO4 and
concentration (100 ppm); NO is alkaline solu-
less toxic and cannot enter blood tions.
stream.
Plant life: Slow down in photo-
synthesis
Oxides of carbon (COx)
Carbon monoxide Incomplete burning 1 Human life: CO is toxic because it Developing
C + O2 → CO
(CO) of carbon-based 2 lowers the dissolution of oxy- more efficient
fuels; emission from gen in blood by forming carbon internal combus-
C 4H10 (g) + 6O2 (g) →
industrial processes; monoxide–haemoglobin complex tion engines
bacterial decay of CO(g) + 3CO2 (g) + 5H2O(l) which causes drowsiness and
organic matter. lethargy, affects vision and causes
CO2  CO + O
cardiovascular disorders.
CO2 + C  2CO
CO2 hinders respiration.
Carbon dioxide Burning of fossil fuels, C 4H10 (g) + 6.5O2 (g) → Controlled
(CO2) vehicular emission, Environment: Enhanced green- emission from
+ 4CO2 (g) + 5H2O(l)
exhaust from flue house effect resulting in global industry and
gases from industries CaCO3 (s) → CaO(s) + CO2 (g) warming. burning of fossil
and decomposition fuels
of limestone.
Hydrocarbons Incomplete combus- 2HCHO → CH4 (g) + CO2 (g) Environment: Causes photo-
tion of fuels in auto- chemical smog.
mobiles and direct Human life: Carcinogenic at levels
evaporation of fuels. between 500 ppm and 1000 ppm,
Also released by causes lung cancer.
trees, domestic ani-
mals; and anaerobic
decomposition.
7. Greenhouse effect and global warming
(a) A continuous increase in the average temperature of air and water sources (oceans) near the surface of the earth in the past few
decades is referred to as global warming.
(b) The warming of the earth due to re-emission of the sun’s energy absorbed by the earth followed by its absorption by carbon
dioxide and water vapour in the atmosphere and then its incidence back to the earth is called greenhouse effect. The green-
house gases are carbon dioxide, nitrous oxide, methane and ozone.
(c) Causes and effects of increased levels of greenhouse gases and global warming are:

Causes Effects
• Use of petrol, diesel, CNG, etc., by automobiles. • Sea levels will rise due to rising temperatures, as
• Farming practices and land-use changes. Use of chemical the polar caps melt.
fertilizers. • Some species will become endangered and face
• Deforestation extinction due to change in habitat.
• Emission of flue gases from factories. • Diseases will spread fast as most microorganisms
• Release of chlorofluorocarbons from air conditioning and will flourish under the warmer climate conditions.
refrigerating machines.
8. Acid rain is a broad term referring to a mixture of wet and dry acidic depositions from the atmosphere containing higher than normal
amounts of nitric and sulphuric acids.
(a) The deposition of acidic oxides of nitrogen and sulphur released into the atmosphere may occur directly on the solid and liquid
ground surfaces and this is known as dry deposition.

Chapter-15.indd 495 7/30/2016 4:40:16 PM


496 Chapter 15 Environmental Chemistry

(b) The gases are carried by wind from drier regions to wetter regions where they undergo oxidation and then dissolve in rain-
water to form nitric acid, sulphuric acid, sulphurous acid, etc., and fall freely in the form of acid rain. The process is known as
wet deposition. The reactions are:
2SO2 (g) + O2 (g) + H2O(l) → 2H2SO 4 (aq)
4NO2 (g) + O2 (g) + 2H2O(l) → 4HNO3 (aq)

(c) The causes and effects of acid rain are:


Causes Effects
• Volcanic emissions • Rise in the acidity levels in lakes and seas.
• Gases from biological processes taking place on land, • Damage to aquatic life.
oceans, wetlands and marshes. • Adverse effect on plant life on land by causing an imbal-
• Nitrogen oxides formed from free nitrogen in ance in plant nutrients.
atmosphere. • Deterioration of monuments built by man.
• Formation of NO from N2 and O2 due to natural phenom- • Cause of respiratory diseases in human being and
ena like lightning. animals.
• Release of sulphur and nitrogen compounds to the
atmosphere by use of automobiles, factories, electricity
producing plants and combustion of fossil fuels.

9. Particulate pollutants comprise of suspended small particles present in the air, like smoke and dust or liquid droplets, with size
ranging from 0.002 µm to 500 µm. These are of two types:
(a) Viable
These comprise living microorganisms such as bacteria, fungi, algae, etc., which may cause diseases.
(b) Non-viable
(i) These comprise non-living particulate matter.
(ii) These originate from natural sources such as volcanic eruptions, seawater sprays, blowing wind and dust or from factory, etc.

Non-viable
particulate Characteristic Sources Effects
Smoke (soot) Smallest size particles Incomplete combus- • Particulate pollutants larger than 5 μ block the
(0.005 µ) tion of fossil fuels, nasal passage.
May be solid or a garbage, dry leaves, • Particles of around 1 μ in size can get into the
mixture of solid and oil, cigarette. lungs, settle and act as sites for adsorption of
liquid particles carcinogenic materials. For example, Pneumoco-
niosis caused in industrial and mine workers by
Dust Small solid particles Crushing and grinding inhalation of dust particles.
of about 1 µ in size of stone and other • Aerosols such as smoke, fog, dust, mist contain
materials; sawing of oxides of nitrogen and sulphur which can cause
wood; pulverizing acid rain and fluorocarbons that can deplete the
coal; fly ash from fac- ozone layer.
tories and dust storms. • Particulates cause metal corrosion and damage
Mist Spray of liquids (such to buildings and sculpture.
as insecticides) or
condensation of
vapours in the air.
Fumes Condensation of cer-
tain materials that are
present as vapours in
the atmosphere, such
as organic solvents,
metals and metal
oxides.
10. Smog is a combination of two terms, smoke and fog and results from air pollution.
There are two types of smog:
(a) Classical (London) smog
(i) Characterized by the presence of high levels of carbon soot arising from unburnt coal and sulphur dioxide arising from
oxidation of sulphur present in the coal. It is also called as reducing smog.

Chapter-15.indd 496 7/30/2016 4:40:16 PM


Summary 497

(ii) The reactions involved are:

S(s) + O2 (g) → SO2 (g)

SO2 (g) + 21 O2 (g) → SO3 (g)

(b) Photochemical smog


(i) Also called Los Angles smog, is brown in colour and observed in sunny and densely populated cities.
(ii) Oxidizing in nature and consists mainly of the compounds that are produced by reaction of sunlight with chemicals like
nitrogen oxides (NOx) and volatile organic compounds (VOCs) found in polluted air.
Reactions involved in smog formation are:
• Formation of ozone:

NO(g) + O2 (g) → NO2 (g)


UV-B
NO2 (g) 
→ O + NO(g)
O + O2 → O3
NO(g) + O3 (g) → NO2 (g) + O2 (g)

Both NO2 and O3 formed react with unburnt hydrocarbons present in the air to form formaldehyde, acrolein and peroxyacetyl
nitrate (PAN).
• Formation of aldehydes and ketones

3CH4 + 2O3 → 3HCHO + 3H2O


• Formation of peroxyacetyl nitrate
Hydrocarbon + O2 + NO2 + Light → CH3COOONO2

11. Ozone layer depletion


(a) In troposphere, ozone helps to trap heat to keep the earth warm. In stratosphere, it plays an important role by filtering harmful
ultraviolet (UV) rays from the sun, overexposure to which can cause cancer and cataract.
(b) The overall amount of ozone in the stratosphere is maintained by a balance between photochemical production and
recombination. The reactions taking place in ozone cycle are:
UV
O2 (g) → O(g) + O(g)
O(g) + O2 (g) → O3 (g)

O(g) + O3 (g) → 2O2 (g)


(c) The leading cause of ozone depletion is the emission of chlorofluorocarbons (CFCs), also known as freons into the atmosphere
that are used in refrigerators, food containers, insulation materials and plastic foam. The CFCs rise from the earth’s surface to the
stratosphere and reactions involved in ozone depletion are:
UV
F2 C Cl2 (g) F2 C Cl (g) + Cl (g)
CFCs
i
Cl(g) + O3(g) → ClO(g) + O2(g)
Chlorine monoxide
i i
ClO(g) + O(g) → Cl (g) + O2 (g)
The free Cl radical continues to react with ozone and deplete it in the stratosphere.
(d) The decrease in stratospheric ozone over the earth’s polar regions is commonly referred to as the ozone hole. Reactions leading
to ozone hole formation involve free radicals generated by reactions of CFCs with atmospheric ozone are:

ClO(g) + NO(g) → ClONO2 (g)
Chlorine nitrate
i i
Cl + CH4 → CH3 + HCl
Polar stratospheric clouds are formed in winter months and prevent ozone depletion:
ClONO2 + H2O → HOCl + HNO3
ClONO2 + HCl → Cl2 + HNO3

Chapter-15.indd 497 7/30/2016 4:40:19 PM


498 Chapter 15 Environmental Chemistry

In spring, the following reactions lead to ozone depletion:


i i
hu
HOCl → OH + Cl
i
hu
Cl2 → Cl
12. Water pollution
(a) Water pollution is the degradation of water quality. A water pollutant is any biological, physical or chemical substance which,
in an identifiable excess, is known to be harmful to living organisms.
(b) Classification of water pollutants
(i) Pathogens: These are disease causing microorganisms (bacteria and viruses).
(ii) Organic waste: Dead organic matter floating in water bodies is decomposed by microorganisms.
Tip Biological oxygen demand measures the amount of oxygen consumed by microorganisms as they break down
organic matter within small water samples, which are analyzed in a laboratory. The BOD of non-polluted water is below
5 ppm and that of polluted water is above 17 ppm.
When the BOD is high, the dissolved oxygen (DO) content of the water may become too low to support life in water.
The aquatic fauna is affected when DO falls below 6 ppm.

(iii) Chemical pollutants: These include:


• Heavy metals: Arsenic, cadmium, lead, mercury, selenium from agricultural, urban and industrial waste. These enter the food
chain and are accumulated in living tissues.

Tip Biomagnification is the accumulation or increase in the concentration of a substance in living tissue as it moves
through a food web (also known as bioaccumulation).

• Acids [HCl, H2SO4, HNO3 and H3PO4] and alkalis [NaOH, KOH, NH3 and Ca(OH)2] drained with effluents from different
industries.
• Oil spills
• Organic chemicals from use of pesticides and from industrial processes.
• Synthetic detergents

Tip Chemical oxygen demand measures the pollution load in water by measuring the amount of oxygen in parts per
million required for oxidation of total organic matter.

(c) Eutrophication is a process by which a water body develops a high concentration of nutrients, such as nitrogen and phosphorus
(in the form of nitrates and phosphates). These nutrients cause an increase in the growth of aquatic plants and production of blue
green bacteria and algae. The BOD of water goes up when these die and dissolved oxygen decreases affecting aquatic life.
13. Soil pollution
Major soil pollutants are:
(a) Pesticides
These have undergone four stages of development:
(i) First stage used simple inorganic compounds like arsenic that is highly toxic to living beings.
(ii) Second stage used petroleum based sprays and natural plant chemicals. Though safe, these are not very effective.
(iii) Third stage used synthetic organic chemicals, generally called chlorinated pesticides, for example DDT, aldrin, dieldrin, etc.
Most of the insects also become resistant to these pesticides over a period of time.
(iv) The fourth series of pesticides that are introduced are biodegradable and hence less persistent in the environment. These
include organophosphates and carbamates.
(b) Herbicides
These include alachlor, atrazine, cyanazine, metolachlor, sodium chlorate and sodium arsenite. These enter soil in heavy run-offs
and are toxic to living beings above permissible limits.
14. Strategies to control environmental pollution
(a) Integrated waste management
(i) Involves practices of reducing, reusing and recycling waste.
(ii) Control of pollution can be accomplished through chemical, physical or biological treatment and collection (for eventual
disposal), transformation or disposal of pollutants after they have been generated.

Tip Composting is a biochemical process in which organic materials such as kitchen scraps decompose to a rich, soil-like
material. The process involves rapid partial decomposition of moist solid organic waste by aerobic organisms.

Chapter-15.indd 498 7/30/2016 4:40:20 PM


Solved Examples 499

(b) Green chemistry


(i) It is an area in which various processes are carried out in a way that there is no adverse impact on the environment.
(ii) It promotes the use of techniques that avoid use of toxic solvents and reagents and harmful reaction conditions. Some exam-
ples of green chemistry are:
• Use of hydrogen peroxide (H2O2) for bleaching of clothes during laundry instead of aqueous solution of sodium hypochlo-
rite (NaOCl).
• Use of hydrogen peroxide instead of chlorine in bleaching paper.
• Replacing volatile organic solvents with ionic liquids made up of complex ionic components.
Tip Bioremediation is a process by which harmful substances are converted into less toxic or non-toxic compounds
by micro organisms such as yeast, fungi and bacteria.

SOLVED EXAMPLES
Atmospheric Pollution Reaction without polar stratospheric clouds
1. Identify the wrong statements in the following: CFCl3 + UV light → Cli + CFCl2
(a) Chlorofluorocarbons are responsible for ozone layer
depletion. Cli + O3 → ClO + O2
(b) Greenhouse effect is responsible for global warming. ClO + NO2 → ClONO2 (Reservoirs)
(c) Ozone layer does not permit infrared radiation from the With polar stratospheric clouds
sun to reach the earth. O3
ClONO2 + HCl + Clouds Cl2 Cl• ClO•
(d) Acid rains are mostly because of oxides of nitrogen and (Moisture) + +
sulphur. (AIEEE 2008) O3
Cl• ClO•
Solution
(c) Ozone layer does allow infrared radiation from sun to Cl2O2 O2
reach earth, whereas it does not allow to pass UV radiation. The main reason for ozone depletion in polar region is that ice
2. The gas leaked from a storage tank of the Union Carbide plant crystals catalyze ozone depletion reaction.
in Bhopal gas tragedy was 4. Global warming is due to increase of
(a) methylamine. (b) ammonia. (a) methane and nitrous oxide in atmosphere.
(c) phosgene. (d) methyl isocynate. (b) methane and CO2 in atmosphere.
(JEE Main 2013) (c) methane and O3 in atmosphere.
Solution (d) methane and CO in atmosphere.
(JEE Main Online 2014)
(d) The gas leaked from a storage tank of the Union Carbide
Solution
plant in Bhopal gas tragedy was methyl isocyanate
(C2H3NCO). (b) The increasing percentage of CO2 and CH4 in the atmos-
phere is the main cause of global warming. CO2 is most
3. Which of the following statements about the depletion of prevalent greenhouse gas and CH4 is second most preva-
ozone layer is correct? lent greenhouse gas.
(a) The problem of ozone depletion is less serious at poles
because NO2 solidifies and is not available for consuming 5. Two important sinks of CO2 are
ClO• radicals. (a) plants, vehicular exhaust. (b) oceans, plants.
(b) The problem of ozone depletion is more serious at poles (c) oceans, soil. (d) plants, limestone.
because ice crystals in the clouds over poles act as catalyst Solution
for photochemical reactions involving the decomposition (b) Oceans and plants are the two important sinks of CO2.
of ozone by Cl• and ClO• radicals.
(c) Freons, chlorofluorocarbons, are inert chemically, they do 6. Flue gases are generally made free from NOx by
not react with ozone in stratosphere. (a) mixing them with oxygen.
(d) Oxides of nitrogen also do not react with ozone in strato- (b) scrubbing them with H2SO4.
sphere. (JEE Main Online 2014) (c) passing them through heated platinum plates.
(d) passing them through water.
Solution Solution
(b) CFCl3 + UV light → CFCl2 + Cl i (b) The large amount of nitrogen oxides emitted from power
plants and industrial units can be removed by scrubbing
Cl i + O3 → ClO + O2 the flue gases with sulphuric acid. SO2 and NO2 present in
i i the flue gases first react to give NO and H2SO4.
ClO + O → Cl + O2
NO2 + SO2 + H2O → H2SO 4 + NO

Chapter-15.indd 499 7/30/2016 4:40:21 PM


500 Chapter 15 Environmental Chemistry

NO and NO2 then react to form N2O3 that is scrubbed by sul- Solution
phuric acid to form H(NO)SO4: (b) Emission of NOx from vehicular exhaust can be reduced
by using catalytic converters.
NO + NO2 → N2O3 Removal of sulphur from the fuels can be done by
N2O3 + 2H2SO 4 → 2H(NO )SO 4 + H2O combustion, using chemical scrubbers like CaCO3 that
react with SO2 to form less harmful CaSO4.
The flue gases are thus freed of NO2 as well as SO2. The The high amount of CO, CO2 and hydrocarbons in
cleaned flue gases are then released into the atmosphere. The the atmosphere can be minimized by developing more
compound H(NO)SO4 is decomposed in a separate chamber efficient and internal combustion engines that con-
by heating where the following reaction takes place. sume less fuel and emit lesser amount of pollutants.
Compressed natural gas and liquefied natural gas can
2H(NO )SO 4 + 21 O2 + H2O → 2H2SO 4 + NO2 supplement gasoline as fuels. The emission of CO2 can
also be controlled by reducing the amount of harmful
The liberated H2SO4 is recycled into the scrubbing chamber. chemicals used in the industries and minimizing the
burning of fossil fuels.
7. CO is more toxic than CO2 because
(a) it affects the nervous system. 11. Which of the following is not a greenhouse gas?
(b) it damages lungs. (a) CH4 (b) H2O (c) O3 (d) NO2
(c) it reduces the oxygen carrying capacity of haemoglobin. Solution
(d) it forms acid with water.
(d) NO2 is not a greenhouse gas as it does not contribute to
Solution greenhouse effect.
(c) CO is more toxic than CO2 because it reduces the oxygen 12. Which of the following is true about greenhouse effect?
carrying capacity of haemoglobin. (a) The name “Greenhouse” has been given because glass
8. Hydrocarbons turn carcinogenic in concentrations above houses are made of green glass.
(a) 100 ppm (b) 300 ppm (b) Greenhouse effect was observed in houses used to grow
(c) 500 ppm (d) 700 ppm plants and these are made of green glass.
(c) The name “Greenhouse” has been given because the
Solution practice of encasing vegetation in glass chambers to
(c) Hydrocarbons turn carcinogenic in concentrations above protect them from frost particularly in cold countries.
500 ppm. The hydrocarbons in air by themselves alone (d) None of these.
cause no harmful effects. However, they undergo chem- Solution
ical reactions in the presence of sunlight and nitrogen
oxides forming photochemical oxidants. (c) This effect was observed that there was a continued rise
in temperature in such chambers even when the outside
9. Lung diseases are four times more in urban areas than rural temperature remained low. This enabled the warming up
areas. This is due to the presence of of vegetation inside the chamber, resulting in good plant
(a) SO2 (b) CO2 growth.
(c) N2 (d) water-vapour
13. When rain is accompanied by a thunderstorm, the collected
Solution rain water will have a pH value
(a) This is due to the presence of SO2 gas. A concentration of (a) slightly lower than that of rain water without
0.5 ppm of SO2 for full day or 0.2 ppm lasting for 3 days thunderstorm.
has been reported to damage the lungs and increase the (b) slightly higher than that when the thunderstorm is not
rate of mortality. there.
(c) uninfluenced by occurrence of thunderstorm.
10. Match the gas pollutants with their methods of control. (d) which depends on the amount of dust in air.
A NOx (p) Minimizing burning of fossil fuels Solution
B SOx (q) Catalytic converters (a) This is because thunderstorm contains acidic oxides of
(r) Combustion, using chemical scrub- sulphur and nitrogen.
C COx
bers like CaCO3 14. Acid rain is produced in the atmosphere by the oxides of
D CxHy (s) Using CNG, LNG, etc. (a) sulphur (b) nitrogen
(c) carbon (d) both (a) and (b)
(a) A → (p); B → (q); C → (r, s); D →(s)
Solution
(b) A → (q); B → (r); C→ (p, s); D → (s)
(c) A → (r); B → (s); C → (p, q); D → (r) (d) Acid rain is produced in the atmosphere by the oxides of
(d) A → (s); B → (q); C → (r, s); D → (p) sulphur and nitrogen. SO2 and NO2 after oxidation and

Chapter-15.indd 500 7/30/2016 4:40:22 PM


Solved Examples 501

reaction with water produce sulphuric acid and nitric Particulate Pollution
acid which are the major contributors to the acid rain.
17. Photochemical smog consists of excessive amount of X, in
addition to aldehydes, ketones, peroxy acetyl nitrile (PAN),
2SO2 + O2 + 2H2O → 2H2SO 4
and so forth. X is:
4NO2 + O2 + 2H2O → 4HNO3 (a) CH4 (b) CO (c) CO2 (d) O3
(JEE Main Online 2015)
15. Which of the following reactions does not contribute to air
pollution in the troposphere? Solution
(a) C 4H10(g) + 6O2(g) → CO(g) + 2CO2(g) + 5H2O(l) (d) Photochemical smog consists of excessive amount of
ozone (O3) in addition to nitrogen oxides (NOx), per
Lightning
(b) N2(g) + O2(g) → 2NO oxyacetyl nitrate (PAN) and volatile organic compounds
(VOCs) that are produced by reaction of sunlight.
2NO(g) + O2(g) → 2NO2 (g)
18. The smog is essentially caused by the presence of
(c) 2HCHO → CH4 (g) + CO2 (g) (a) O2 and O3.
i
(d) Cl + O3 (g) → ClO + O2 (g) (b) O3 and N2.
(c) oxides of sulphur and nitrogen.
Solution (d) O2 and N2.
(d) The first reaction takes place during incomplete com-
Solution
bustion of fossil fuels and releases oxides of carbon into
the troposphere. The second set of reactions represents (c) Classical smog contains oxides of sulphur, while photo-
formation of oxides of nitrogen in the troposphere. The chemical smog contains oxides of nitrogen.
third reaction represents formation of hydrocarbons due 19. PAN and ozone are
to incomplete combustion of fuels or anaerobic decom- (a) primary pollutants.
position of organic matter. The fourth reaction between (b) secondary pollutants.
chlorine radical and ozone takes place in the stratosphere (c) responsible for respiratory problems.
and leads to depletion of ozone in the stratosphere. (d) particulate pollutants.
Chlorine free radical is obtained from chlorofluorocar-
bons released into the atmosphere. Solution
(b) These are secondary pollutants formed as follows:
hn
F2CCl2(g) F2C Cl(g) + Cl(g) The oxygen free radical reacts with atmospheric oxygen
producing ozone.
16. Which of the following conditions does not show a polluted
O + O2 → O3
air environment?
(a) Air contains particulate matter of size 3 µ. Formation of peroxyacetyl nitrate is as follows:
(b) The rain falling has pH of 4.5. Hydrocarbon + O2 + NO2 + Light → CH3COOONO2
(c) The hydrochlorofluorocarbons emitted by the refrigera-
tion plant. 20. Which of the following is pollution related disorder?
(d) The SO2 and SO3 emitted by the thermal power plant (a) Hypertension (b) Leprosis
have concentration of 7 ppm and 2 ppm, respectively, in (c) Silicosis (d) Ulcer
the air. Solution
Solution (c) Silicosis, also known as potter’s rot, is a form of occupa-
(c) Option (a) refers to a polluted air condition because tional lung disease caused by inhalation of crystalline silica
particulate matter of size greater than 1 µ can get into dust, and is marked by inflammation and scarring in forms
the lungs and act as site for adsorption of carcinogenic of nodular lesions in the upper lobe of the lungs. It is a type
material. of pneumoconiosis. It is normally caused by particulates.
Option (b) refers to a polluted air condition because 21. In a coal fired power plant, electrostatic precipitators are
the rain with pH of about 5 is called acid rain and contains installed to control emission of
nitric and sulphuric acids. It is formed by the presence of (a) SO2
oxides of sulphur and nitrogen in the air that exceed the (b) NOx
permitted values. (c) Suspended particulate matter (SPM)
Option (c) refers to a non-polluted environment (d) CO
because hydrogen-containing chlorofluorocarbons are
destroyed in the troposphere itself, and not taken to strat- Solution
osphere where they could cause destruction of ozone. (c) In the coal fired power plant, electrostatic precipitators
Option (d) refers to a polluted air condition because lev- are installed to control emission of suspended particu-
els of SO2 at even 5 ppm can cause irritation of respiratory late matter. The particulates are usually produced by coal
tract and eyes. SO3 is harmful even at the concentration of fire power plant is soot, dust particulates, metal particles,
1 ppm causing discomfort particularly to the elderly. metal oxides, etc.

Chapter-15.indd 501 7/30/2016 4:40:24 PM


502 Chapter 15 Environmental Chemistry

22. The particle size is smallest in case of (c) decrease in biological oxygen demand.
(a) mist. (b) soot. (c) fumes. (d) dust. (d) all of these.
Solution Solution
(b) Smoke (soot) contain smallest size particles (0.005 µ) that (a) Excessive growth of algae indicates eutrophication in
are released into the atmosphere by incomplete combus- which there is an increase in the amount of nutrients
tion of fossil fuels, garbage, dry leaves, oil, cigarette, etc. present in the water especially nitrogen and phospho-
Smoke particulates may be solid or a mixture of solid and rus. These algae form surface mats and reduce the light
liquid particles. availability to the algae below the surface which further
reduces photosynthesis. Besides, the decomposition of
23. How is photochemical smog produced?
dead algae also increases the biological oxygen demand
(a) Depletion of tropospheric ozone + SOx
and reduces the oxygen content of water.
(b) Incineration of toxic waste
(c) Solar radiation + NOx + organic compounds 27. Which of the following are organic pollutants?
(d) Burning of coal in urban areas + SOx (a) Acids and alkalis from industries
Solution (b) Alkyl benzene sulphonates
(c) Benzene sulphonamides
(c) Photochemical smog consists mainly of the compounds (d) All of these
that are produced by reaction of sunlight with chemicals
like nitrogen oxides and volatile organic compounds found Solution
in polluted air. Burning of coal in urban areas and oxides of (c) Organic pollutants are benzene sulphonamides, adipates
sulphur formed lead to the formation of classical smog. (esters of hexandioic acid), phthalates (esters of phthalic
acid) and DEHP, PCB.
Water Pollution
24. Addition of phosphate fertilizers to water bodies causes 28. Which of the following statements is false?
(a) enhanced growth of algae. (a) The main reason for river water pollution is industrial and
(b) increase in amount of dissolved oxygen in water. domestic sewage discharge.
(c) deposition of calcium phosphate. (b) Surface water contains a lot of organic matter, mineral
(d) increase in fish population. nutrients and radioactive materials.
(JEE Main Online 2015) (c) Oil spill in sea water causes heavy damage to fishery.
(d) Oil slick in sea water increases DO value.
Solution
Solution
(a) Addition of nutrients such as nitrogen and phospho-
rus (in the form of nitrates and phosphates) cause an (d) Oil slick results in reduction of dissolved oxygen (DO).
increase in the growth of aquatic plants and production
29. Which of the following causes water pollution?
of blue green bacteria and algae. This process is known as
(a) Smoke/fly ash
eutrophication.
(b) Automobile exhausts
25. Which of the following is not usually associated with (c) Aeroplanes
eutrophication? (d) PCBs and detergents
(a) Reduced photosynthesis Solution
(b) Increased heavy metal concentrations
(c) Increased nutrient concentrations (d) Polychlorinated biphenyls (PCBs) and detergents cause
(d) Reduced dissolved oxygen water pollution.

Solution 30. The term “biomagnification” refers to


(b) Eutrophication is a process by a body of water that devel- (a) growth of organism due to food consumption.
ops a high concentration of nutrients such as nitrogen (b) increase in population size.
and phosphorus. Their increased concentration leads to (c) increase in the concentration of non-degradable pollut-
growth of aquatic plants in general and in production of ants as they pass through food chain.
blue–green algae. This algal mat reduces the availability (d) decrease in population size.
of light below its surface, and thereby greatly reducing Solution
photosynthesis. The bacteria and algae die and their
(c) Biomagnification is the accumulation or increase in the
decomposition increases the BOD of water, thus the dis-
concentration of a substance in living tissue as it moves
solved oxygen content of water is reduced.
through a food web (also known as bioaccumulation).
Heavy metal concentration is not associated with eutro-
phication. 31. Select the incorrect statement.
26. Eutrophication is caused by (a) Water is considered pure if it has BOD less than 5 ppm.
(a) excess growth of algae. (b) In COD determination, the pollutants resistant to micro-
(b) increase in rate of photosynthesis. bial oxidation are not oxidized by oxidizing agent like
K2Cr2O7.

Chapter-15.indd 502 7/30/2016 4:40:24 PM


Practice Exercise 503

(c) The lower the concentration of DO, the more polluted is Soil Pollution
the water sample.
34. What is DDT among the following?
(d) The tolerable limit of lead in drinking water is 50 ppm.
(a) Greenhouse gas
Solution (b) A fertilizer
(b) In COD determination the pollutants resistant to microbial (c) Biodegradable pollutant
oxidation are oxidized by oxidizing agent like K2Cr2O7. (d) Non-biodegradable pollutant (AIEEE 2012)

32. Which of the following is the best measure for pollution in Solution
water? (d) DDT is a well-known insecticide. It is a non-biodegradable
(a) COD (b) Dissolved oxygen pollutant. DDT is not rapidly metabolized by animals, it
(c) BOD (d) Water transparency affects the reproductive system of animals.
The structure of DDT is
Solution
(c) Dissolved oxygen depends upon various factors, such as Cl
Cl Cl
temperature of the water, the amount of oxygen used
up by organisms, and the amount replenished by pho-
tosynthesizing plants, aeration, etc. COD refers to chem-
ical oxygen demand, and is less specific as it measures
both organic and oxidizable inorganic material that can Cl Cl
be chemically oxidized, rather than just levels of biologi- 1,1,1-Trichloro-2,2-bis
cally active organic matter. Biochemical oxygen demand (4-chlrophenyl) ethane
(BOD) is a more specific measure as it measures the
amount of the dissolved oxygen that would be needed 35. Sodium chlorate is a type of
by the microorganisms to oxidize organic waste present (a) pesticide. (b) herbicide.
in sewage water. (c) fertilizer. (d) insecticides.

33. The industrial waste pipeline that discharges effluent into a Solution
water body is (b) Herbicides are used to control weeds, which are the
(a) a non-point source of pollution. unwanted plants that grow along with the crops and thus
(b) a point source of pollution. compete with them for sunlight, nutrients, etc. Examples
(c) called acid mine drainage. include NaClO3 (sodium chlorate) and Na3AsO3 (sodium
(d) responsible for eutrophication. arsenite).
Solution 36. A new series of pesticides that have been introduced which
(b) Point sources of pollution are distinct and defined; their are biodegradable and hence less persistent in the environ-
origin can be identified and corrective measures for ment include
control can be taken. Therefore, water being discharged (a) phosphates. (b) organophosphates.
through industrial pipeline is a point source that can be (c) nitrates. (d) organosulphates.
treated and the pollutants can be regulated before it is Solution
discharged into the water body.
(b) To overcome the challenge posed by persistence of tox-
Non-point sources of water pollution are diffused and
ins from chlorinated organic pesticides, a new series of
intermittent. They cannot be identified easily, and hence
pesticides have been introduced which are biodegrada-
cannot have specific control measures.
ble and hence less persistent in the environment. These
Acid mine drainage refers to water with high concen-
include organophosphates and carbamates. However,
tration of sulphuric acid that drains from mines, mostly
these have been reported to be nerve toxins and have
coal mines.
resulted in deaths of agricultural workers.
Eutrophication is caused by the presence of excess
nitrogen and phosphorus in run-off waters from agricul-
tural fields.

PRACTICE EXERCISE
Level I 2. Temperature of troposphere decreases with altitude. This is
because of
Single Correct Choice Type (a) high pressure of air. (b) gases present in air.
1. COD refers to (c) lower density of air. (d) all of these.
(a) chemical oxygen demand.
(b) chemistry of diamonds. 3. In BOD test, oxygen plays an important role to
(c) catalyzed oxidation of dissolved salts. (a) destroy inorganic matter.
(d) combined oxygen demand. (b) destroy pollution.

Chapter-15.indd 503 7/30/2016 4:40:24 PM


504 Chapter 15 Environmental Chemistry

(c) destroy waste organic matter. (c) converting fertile land into barren land by dumping ash,
(d) destroy living organism. sludge and garbage.
(d) none of these.
4. Smog is a common pollutant in places having
(a) high temperature. 15. Photochemical smog formed in congested metropolitan
(b) low temperature. cities mainly consists of
(c) excessive SO2 in the air. (a) ozone, peroxyacetyl nitrate and NOx.
(d) excessive ammonia in the air. (b) smoke, peroxyacetyl nitrate and SO2.
5. Drained sewage has BOD (c) hydrocarbons, SO2 and CO2.
(a) more than that of water. (b) less than that of water. (d) hydrocarbons, ozone and SOx.
(c) equal to that of water. (d) none of these. 16. Which of the following type of pollution is caused by invisible
6. The radiations which are concerned with global warming and pollutants?
ozone depletion are, respectively, (a) Thermal pollution. (b) Noise pollution.
(a) UV and IR (b) UV and UV (c) Radioactive pollution. (d) All of these.
(c) IR and IR (d) IR and UV
17. Atmosphere of metropolitan cities is mostly polluted by
7. Water is often treated with chlorine to (a) automobile exhausts. (b) pesticide residue.
(a) increase oxygen content. (c) household waste. (d) radioactive fallout.
(b) kill germs.
(c) remove hardness. 18. The part of the atmosphere where weather occurs is the
(d) remove suspended particles. (a) tropopause. (b) stratopause.
(c) troposphere. (d) stratosphere.
8. Which of the following is not a part of green chemistry?
(a) Photochemistry (b) Sonochemistry 19. About 20 km above the earth there is an ozone layer. Which
(c) Nuclear chemistry (d) Biochemistry one of the following statements about ozone and ozone layer
is true?
9. Which of the following statements about control of particu-
(a) Ozone layer is beneficial to us because ozone cuts out the
late pollution is false?
ultraviolet radiation of the sun.
(a) Gravity settling chamber removes larger particles from
(b) The conversion of ozone to oxygen is an endothermic
the air.
reaction.
(b) Cyclone collector removes fine particles in the diameter
(c) Ozone has a triatomic linear molecule.
range 5–20 µ.
(d) Ozone layer is harmful to us because it blocks radiations
(c) Wet scrubbers are used to wash away all types of
that are useful for photosynthesis.
particulates.
(d) In electrostatic precipitator, the particulates are made to 20. The brown, hazy fumes of photochemical smog are due to
acquire positive charge which are then attracted by the (a) nitrogen dioxide. (b) PAN formation.
negative electrode and removed. (c) aldehydes. (d) SO2.
10. As it passes into the food chain, the concentration of DDT 21. Atmospheric content of CO2 is
(a) remains the same. (b) decreases. (a) 0.0034% (b) 0.034% (c) 0.34% (d) 3.4%
(c) increases. (d) unpredictable.
22. Taj Mahal may be destroyed by
11. Which of the following is the biggest particulate matter? (a) flood in Yamuna.
(a) Soot (b) H2SO4 droplets (b) temperature-mediated spoilage of marble.
(c) Fly ash (d) HNO3 droplets (c) air pollutants from Mathura refinery.
12. Which of the following method is used in green chemistry? (d) all of these.
(a) Use of sunlight and microwaves. 23. What does BOD stand for?
(b) Use of sound waves. (a) Barometer of Decomposition
(c) Use of water as solvent (in complex reactions). (b) Bacterial Oxygen Dependency
(d) All of these. (c) Biodegradable Organic Damage
13. Global warming can be controlled by (d) Biochemical Oxygen Demand
(a) increasing deforestation, slowing down the growth of
human population. 24. Most hazardous metal pollutant of automobile exhaust is
(b) reducing reforestation, increasing the use of fossil fuel. (a) mercury. (b) tin.
(c) reducing deforestation, cutting down use of fossil fuel. (c) cadmium. (d) lead.
(d) increasing deforestation, reducing efficiency of energy 25. During the last two million years or so, the climate of the Earth
usage. has
14. Negative soil pollution is (a) been very nearly constant.
(a) reduction in soil productivity due to erosion and over use. (b) swung sharply both up and down in temperature.
(b) reduction in soil productivity due to addition of pesti- (c) slowly decreased in humidity.
cides and industrial wastes. (d) slowly increased in temperature.

Chapter-15.indd 504 7/30/2016 4:40:24 PM


Practice Exercise 505

26. Which pollutant causes burning sensation of throat and eyes In this analogy, the greenhouse effect would be like
and vomiting sensation? (a) putting the bucket on the stove.
(a) Hydrogen sulphide (b) Sulphur (b) increasing the stream of water pouring into the bucket.
(c) Hydrogen cyanide (d) Arsenic substances (c) decreasing the stream of water pouring into the bucket.
(d) plugging-up some of the holes in the bucket.
27. About 99% of all air in the Earth’s atmosphere is found in
(a) the troposphere. 35. The amount of the nutrients, phosphorus, and nitrogen in
(b) the tropopause. groundwater is usually
(c) the stratosphere. (a) greater in agricultural regions than in natural forests.
(d) the troposphere and the stratosphere. (b) the same in agricultural regions as in natural forests.
(c) an indicator of soil fertility.
28. Water quality determination is based on all of the following (d) an indicator of industrial seepage.
except
(a) effects on public health. 36. When huge amount of sewage is dumped in a river, the BOD will
(b) departure from the norm. (a) increase. (b) remain unchanged.
(c) source of the water. (c) decrease. (d) None of these.
(d) expected end use. 37. Carbon monoxide (CO) is harmful to man because
29. UV radiation from sun causes a reaction that produces (a) it forms carbolic acid.
(a) carbon monoxide. (b) sulphur dioxide. (b) it generates excess CO2.
(c) fluorides. (d) ozone. (c) it is carcinogenic.
(d) it competes with O2 for haemoglobin.
30. Scrubber in the exhaust of a chemical industrial plant removes
(a) Gases like sulphur dioxide. 38. Ozone layer in upper atmosphere (stratosphere) is destroyed
(b) Particulate matter of the size 2.5 micrometer or less. by
(c) Gases like ozone and methane. (a) hydrochloric acid.
(d) Particulate matter of the size 5 micrometer or above. (b) photochemical smog.
(c) chlorofluorocarbon (CFC).
31. Which of the following is responsible for catching most of (d) sulphur dioxide.
Earth’s back-radiation to space?
(a) Water vapour (b) Sulphur dioxide 39. By weight, the most abundant water pollutant is
(c) Particulate matter (d) Carbon dioxide (a) toxic chemicals.
(b) leachate from open dumps.
32. Air pollutants that produce photochemical oxidants include (c) organic waste.
(a) carbon monoxide, carbon dioxide and sulphur dioxide. (d) sediment.
(b) nitrous oxide, nitric oxide and nitric acid.
(c) oxygen, chlorine and nitric acid. 40. Any precipitation (rain, fog, snow or dew) which has pH less
(d) ozone, chlorine and sulphur dioxide. than ___ is called acid rain.
(a) 4.6 (b) 6.6 (c) 5.6 (d) 5.0
33. Foul smell in the water of tanks, ponds, etc. is due to
(a) anaerobiosis. (b) aerobiosis. 41. The best definition of the term water pollution is
(c) biological magnification. (d) psammophytes. (a) release of primary treated sewage into natural water.
(b) water unsuitable for human consumption.
34. The figure below illustrates an analogy of the earth’s ther- (c) overdrafting in coastal areas.
mal balance. Water pouring into the bucket is equivalent to (d) degradation of water quality.
in-coming solar radiation, and the water leaking from holes
in the bucket is like outgoing thermal radiation. The amount 42. Pneumoconiosis is caused by inhalation of
of water in the bucket is equivalent to the amount of heat on (a) coal dust. (b) silica dust.
earth – a higher water level is like higher temperature on earth. (c) cotton fiber dust. (d) asbestos dust.
43. Which of the following is not a greenhouse gas?
(a) CO2 (b) Water vapour
(c) CH4 (d) O2
44. Eutrophication can be observed in
(a) saline soil. (b) desert.
(c) lake. (d) agricultural fields.
45. In which one of the following the biochemical oxygen
demand (BOD) of sewage (S), distillery effluent (DE), paper
mill effluent (PE) and sugar mill effluent (SE) have been
arranged in ascending order:
(a) SE < S < PE < DE (b) SE < PE < S < DE
(c) PE < S < SE < DE (d) S < DE < PE < SE

Chapter-15.indd 505 7/30/2016 4:40:25 PM


506 Chapter 15 Environmental Chemistry

Level II Column I Column II


Multiple Correct Choice Type (a) Phosphate fertilizers in (p) BOD level of water
46. Acid rain contains water increases
(a) peroxyacetyl nitrate. (b) H2CO3 (b) Methane in air (q) Acid rain
(c) HNO3 (d) H2SO4 (c) Synthetic detergents in (r) Global warming
47. Addition of phosphate-containing fertilizers in water bodies water
causes (d) Nitrogen oxides in air (s) Eutrophication
(a) enhanced growth of algae.
(b) decrease in amount of dissolved oxygen in water. 54. Match the substances with their effects.
(c) deposition of calcium phosphate.
Column I Column II
(d) increase in fish population.
(a) Polycyclic aromatic (p) Global warming
48. The gases causing greenhouse effect is (are) hydrocarbons
(a) CH4. (b) NO2. (c) SO2. (d) O3.
(b) Dioxins (q) Photochemical smog
49. The atmosphere contains (c) IR active molecules (r) Carcinogens
(a) mesosphere. (b) hemisphere.
(c) thermosphere. (d) exosphere. (d) Peroxyacetyl nitrate (s) Waste incineration

50. Choose the correct statements 55. Match the effects with the gases responsible for causing
(a) NO is more harmful than NO2. them.
(b) Acid rain contains mainly HNO3.
Column I Column II
(c) CO2 is responsible for greenhouse effect.
(d) CO2 can absorb infrared radiations but does not allow (a) Global warming (p) CO2
them to pass through. (b) Photochemical smog (q) SO2
51. Which of the following conditions do not show the polluted (c) Classical smog (r) NO2
environment?
(d) Acid rain (s) Unsaturated hydrocarbons
(a) Biochemical oxygen demand is 10 ppm.
(b) Eutrophication.
(c) pH of rainwater is 5.6. Integer Type
(d) Amount of carbon dioxide in the atmosphere is 0.03%. 56. Water is considered as impure if it has BOD is greater than ___
52. The ozone layer is not depleted by ppm.
(a) NO. (b) SO2. (c) CxHy. (d) CFCs. 57. Hydrocarbons turn carcinogenic in concentrations above ___
ppm.
Matrix-Match Type
58. In measurement of BODx , x is generally taken as ___.
53. Match the substances/gases with their effects.

ANSWER KEY
Level I
1. (a) 2. (c) 3. (c) 4. (b) 5. (a) 6. (d)
7. (b) 8. (c) 9. (d) 10. (c) 11. (c) 12. (d)
13. (c) 14. (a) 15. (a) 16. (d) 17. (a) 18. (c)
19. (c) 20. (a) 21. (b) 22. (c) 23. (d) 24. (d)
25. (b) 26. (a) 27. (d) 28. (c) 29. (d) 30. (b)
31. (a) 32. (b) 33. (a) 34. (d) 35. (a) 36. (a)
37. (d) 38. (c) 39. (d) 40. (c) 41. (d) 42. (a)
43. (d) 44. (c) 45. (c)

Level II
46. (b), (c), (d) 47. (a), (b) 48. (b), (d) 49. (a), (c), (d) 50. (c), (d) 51. (c), (d)
52. (b), (c) 53. (a) → p, s; (b) → r; (c) → p; (d) → q 54. (a) → r; (b) → s; (c) → p; (d) → q
55. (a) → p; (b) → r, s; (c) → q; (d) → q, r 56. (17) 57. (500) 58. (5)

Chapter-15.indd 506 7/30/2016 4:40:25 PM


Hints and Explanations 507

HINTS AND EXPLANATIONS


Level I 14. (a) Negative soil pollution is reduction in soil productivity
due to erosion and overuse.
Single Correct Choice Type
15. (a) Photochemical smog forms when complex reactions
1. (a) COD refers to chemical oxygen demand. It is the amount take place between nitrogen oxides (NOx) and organic
of oxygen consumed in oxidation of organic and oxidiz- compounds (various hydrocarbons) in the presence of
able inorganic material present in wastewater. COD is sunlight. Under UV radiation, nitrogen dioxide splits into
expressed in mg L−1. nitric oxide and nascent oxygen. Nascent oxygen com-
bines with molecular oxygen to form ozone. The ozone
2. (c) As the gases in this layer decrease with height, the air formed reacts with nitrogen oxide to regenerate nitro-
becomes less dense. So, the temperature in the tropo- gen dioxide. Both nitrogen dioxide and ozone react with
sphere decreases with height. unburnt hydrocarbons present in the air to form formal-
3. (c) In the BOD test, oxygen plays an important role to dehyde, acrolein and peroxyacetyl nitrate (PAN).
destroy waste organic matter. BOD is a measure of the
16. (d) Thermal pollution, noise pollution and radioactive pollu-
dissolved oxygen that would be needed by the micro-
tion are caused by the invisible pollutants. The release of
organisms to oxidize waste organic matter.
radioactive substances in the atmosphere leads to radi-
4. (b) Smog is common pollutant in places having low temper- oactive pollution. Large number vehicles, speakers, and
ature. It occurs generally in cold and humid conditions, horns cause noise pollution.
that is, on early morning hours of winter months. 17. (a) Atmosphere of metropolitan cities is mostly polluted by
5. (a) In drained sewage, a large number of organic and inor- automobile exhausts.
ganic compounds are present. To oxidize these com- 18. (c) Conceptual
pounds, more dissolved oxygen would be needed and
hence BOD of drained sewage water is more than that of 19. (c) O O O. Ozone is a linear molecule.
water. 20. (a) Photochemical smog consists mainly of the compounds
6. (d) IR radiations are concerned with global warming and UV that are produced by reaction of sunlight with chemicals
radiations with depletion of the ozone layer. such as nitrogen oxides (NOx) and volatile organic com-
pounds (VOCs) found in polluted air.
7. (b) Water is treated with chlorine to kill germs.
21. (b) Atmosphere content of CO2 is 0.034%.
8. (c) Green chemistry may be defined as the method of devel-
oping new chemical products and chemical processes 22. (c) Taj Mahal may be destroyed by air pollutants from
or making improvements in the already existing com- Mathura refinery.
pounds and processes so as to make them less harmful
23. (d) Conceptual
to human health and environment. It includes photo-
chemistry (chemical reactions taking place in the pres- 24. (d) Tetraethyl lead is added to petrol to act as an anti-
ence of light), biochemistry (chemical processes in living knocking agent. During combustion on the engine, it
organisms), and sonochemistry (effect of sonic waves is oxidized to PbO that deposits on different parts of
and wave properties on chemical systems). the engine and causes damage. To avoid this damage,
tetraethyl lead is mixed with dichloroethane and dibro-
9. (d) Electrostatic precipitator works on the principle that par-
moethane that convert PbO into PbCl2 and PbBr2. These
ticulates of all sizes acquire negative charge under the
are volatile and released along with the exhaust gases
influence of high electric field. These particles are then
and get introduced into the atmosphere. Thus, engine
attracted towards the positive electrode of the electric
gets protected but air gets polluted.
field where they start accumulating and are then removed.
Pb(C2H5 )4 + O2 + C2H4Be2 + C2H4 Cl2 → CO2 + H2O + PbCl2 + PbBr2
10. (c) Due to biological magnification, the concentration of
DDT increases as it passes into the food chain. 25. (b) Conceptual
11. (c) Fly ash is the biggest particulate matter. It originates 26. (a) Hydrogen sulphide (H2S) causes burning sensation of
from the combustion of high ash fossil fuels. It con- throat and eyes, and vomiting sensation.
tains partially burnt particles of the fuels. This pollutant
is released into the atmosphere from thermal power 27. (d) Conceptual
plants, smelters and mining operations. 28. (c) Conceptual
12. (d) 29. (d) UV radiation from sun causes a reaction that produces
13. (c) Reducing deforestation, cutting down use of fossil fuel ozone
results in reduced production of one of the greenhouse O2 + hn → O + O
gas, that is, CO2. O2 + O → O3

Chapter-15.indd 507 7/30/2016 4:40:26 PM


508 Chapter 15 Environmental Chemistry

30. (b) Scrubbers are air pollution control devices that use Level II
liquid to remove particulate matter or gases from an
industrial exhaust or flue gas stream. Through scrubbers,
Multiple Correct Choice Type
the combustible gases like ammonia, sulphur dioxide 46. (b), (c), (d) Acid rain contains H2CO3, HNO3, and H2SO4.These
and particulate matter released from high temperature are formed from gaseous oxides of carbon, nitrogen and
exhaust fumes are trapped, cooled and release into the sulphur.
atmosphere.
47. (a), (b) Addition of phosphate-containing fertilizers in water
31. (a) Conceptual bodies causes enhanced growth of algae and decrease in
amount of dissolved oxygen in water.
32. (b) Air pollutants that produce photochemical oxidants
include nitrous oxide, nitric oxide, and nitric acid. These 48. (b), (d) The various greenhouse gases are carbon dioxide,
are responsible for the formation of photochemical water vapours, chlorofluorocarbons and oxides of nitrogen.
smog and nitric acid and is responsible for formation of 49. (a), (c), (d) The atmosphere contains mesosphere, thermo-
acid rain. sphere, stratosphere, troposphere, and exosphere.
33. (a) Foul smell in the water of tanks, ponds, etc. is due to 50. (c), (d) NO is less harmful than NO2. Acid rain contains mainly
anaerobiosis. It is the phenomenon in which an organ- H2SO4 and HNO3.
ism does not require oxygen for growth. When large 51. (c), (d) pH of rain water is 5.6. This pH cannot affect to the
amount of organic wastes are released into pond, the normal living things. pH below this can seriously effect liv-
DO content of the pond decreases and ultimately a stage ing things. Amount of CO2 in the atmosphere is 0.03%. This
reaches where there is no DO in pond water. As a result, percentage of CO2 is useful for plants for metabolism and it
only anaerobes can grow in this type of environment cannot affect the environment. If the concentration is above
and release foul odour. 0.03%, then it causes global warming.
34. (d) Conceptual 52. (b), (c) The ozone layer is not depleted by SO2 and CxHy
(hydrocarbons).
35. (a) Conceptual
Matrix-Match Type
36. (a) Biochemical oxygen demand (BOD) is the measure of the
dissolved oxygen that would be needed by the micro- 53. (a) → p, s; (b) → r; (c) → p; (d) → q
organisms to oxidize organic waste present in sewage (a) Phosphate fertilizers in water encourage the formation
water. Hence, when huge amount of sewage is dumped of algae that reduces the dissolved oxygen concentra-
in a river, the BOD will increase. tion of water. This process is known as eutrophication.
37. (d) CO is harmful to man because it has 200 times more The BOD level of water increases due to the presence of
binding capacity to haemoglobin than oxygen. algae that reduces the DO concentration of water.
(b) Methane in air forms carbon dioxide and water that is
38. (c) Chlorofluorocarbons are responsible for destruction of responsible for global warming.
the ozone layer.
CH4 + 2O2 → CO2 + 2H2O
39. (d) Conceptual
(c) Synthetic detergents comprise surfactants. These are
40. (c) Any precipitation (rain, fog, show, or dew) that has pH surface active agents that are organic compounds hav-
less than 5.6 is called acid rain. ing polar or hydrophilic groups such as COOH, SO3H,
NH+4 . These surfactants are soluble in water and increase
41. (d) Conceptual
the BOD level of water.
42. (a) Pneumoconiosis is caused by the inhalation of coal dust. (d) Nitrogen oxides in air are responsible for formation of
43. (d) O2 is not a greenhouse gas. nitric acid rain.

44. (c) Eutrophication can be observed in lake. The addition of 2NO + O2 → 2NO2
phosphorus to water, in the form of the phosphate anion 4NO2 + 2H2O + O2 → 4HNO3
PO3−
4 , encourages the formation of algae, which reduces
NO2 + O3 → NO3 + O2
the dissolved oxygen concentration of water. This pro-
cess is known as eutrophication. NO2 + NO3 → N2O5

45. (c) Distillery effluent releases large amount of organic N2O5 + H2O → 2HNO3
wastes into water, as a result BOD of water is high.
HNO3, formed as above, comes down from the atmos-
Sugar mill effluent releases lesser waste material to the
phere in the form of nitric acid rain.
water and paper mill effluent releases least amount
of organic waste material into water. So, the order of 54. (a) → r; (b) → s; (c) → p; (d) → q
BOD is PE < S < SE < DE. (a) Polycyclic aromatic hydrocarbons (PAHs) produced
by the incomplete combustion of organic matter are a

Chapter-15.indd 508 7/30/2016 4:40:27 PM


Hints and Explanations 509

significant carcinogenic threat found in the environ- (b) Hydrocarbons undergo chemical reactions in the pres-
ment. PAHs cause DNA damage, mutagenesis, etc. ence of sunlight and nitrogen oxides forming photo-
(b) Modern waste incinerators are equipped with pollution chemical oxidants. These photochemical oxidants are
control equipment that reduces dioxin emissions to the responsible for the formation of photochemical
insignificant levels. Incineration of municipal solid waste, smog.
medical waste, sewage sludge, and hazardous waste
together produce less than 3% of all dioxin emissions. (c) Mixture of particulates with gaseous oxides of sulphur
present in atmosphere is called classical smog.
(c) IR active molecules are the cause for the global warming.
When IR radiations come from the sun, they reach the (d) SO2, NO2 present in the atmosphere results in the forma-
earth but due to presence of CO2 on the earth as cover; tion of acid rain.
the IR radiations cannot go out in space. This results in
excessive heating of the earth’s atmosphere. As more
SO2 + H2O → H2SO3
and more infrared radiations are trapped, the atmos- Sulphurous acid
phere becomes hotter and, therefore, temperature rises
and this leads to global warming. SO2 + O3 → SO3 + O2
Ozone
(d) Peroxyacetyl nitrate is formed when peroxyacyl free rad-
icals also combine with NO2 forming PAN that is highly SO3 + H2O → H2SO 4
Sulphuric acid
injurious to eyes.
O O
Integer Type
R C O O +NO2 R C O ONO2
Highly basic (PAN) 56. (17) It is considered that water is impure if it has the BOD
value is greater than 17 ppm.
The presence of excessive O3 together with PAN con-
stitute photochemical smog because these constituent 57. (500) Hydrocarbons turn carcinogenic in concentrations
irritate eyes, throat, nose, and also cause damage to the above 500 ppm. The hydrocarbons in air by themselves alone
plant life. cause no harmful effects. However, they undergo chemical
55. (a) → p; (b) → r, s; (c) → q; (d) → q, r reactions in the presence of sunlight and nitrogen oxides
forming photochemical oxidants.
(a) CO2 gas is responsible for global warming. CO2 cover
absorbs the IR radiations and does not allow the heat 58. (5) In measurement of BODx , x is generally taken as 5 because
radiation of the earth to go out in space. This causes clean water would have a BOD value of less than 5 ppm.
global warming.

Chapter-15.indd 509 7/30/2016 4:40:28 PM


510 Chapter 15 Environmental Chemistry

SOLVED JEE 2016 QUESTIONS


JEE Main 2016 Solution
(c) Biochemical oxygen demand (BOD) is the measure of
1. The concentration of fluoride, lead, nitrate and iron in a water amount of organic material in the water.
sample from an underground lake was found to be 1000 ppb,
40 ppb, 100 ppm and 0.2 ppm, respectively. This water is 3. Which one of the following substances used in dry cleaning is a
unsuitable for drinking due to high concentration of better strategy to control environmental pollution?
(a) iron. (b) fluoride. (c) lead. (d) nitrate. (a) Sulphur dioxide (b) Carbon dioxide
(Offline) (c) Nitrogen dioxide (d) Tetrachloroethylene
(Online)
Solution
Solution
(d) The concentration of fluoride and lead are in ppb (parts
per billion) while the concentration of nitrate and iron are (d) Tetrachloroethylene, also known as perchloroethylene, is
in ppm (parts per million) which is more than ppb. So, the used for dry cleaning and textile processing. It is a chem-
highest concentration is of nitrate i.e. 100 ppm. ical intermediate. It is also used for vapour degreasing in
metal-cleaning operations.
2. BOD stands for:
(a) Biochemical Oxidation Demand
(b) Biological Oxygen Demand
(c) Biochemical Oxygen Demand
(d) Bacterial Oxidation Demand
(Online)

Chapter-15.indd 510 7/30/2016 4:40:28 PM


16 Solid State

Question Distribution in JEE (Main and Advanced)

3
No. of Questions

JEE (Main)
2
JEE (Adv)

0
2016 2015 2014 2013 2012 2011 2010 2009 2008 2007

Concept Distribution in JEE (Main and Advanced)

Topics Covered
Year
JEE (Main) JEE (Advanced)
2007 Unit Cells, Crystal Lattice, Electrical Properties of Solids
2008 Formation of Voids in Close Packing Calculations Involving Unit Cell Dimensions
2009 Calculations Involving Unit Cell Dimensions Point Defects in Crystals, Formation of Voids in Close Packing
Calculations Involving Unit Cell Dimensions, Packing
2010 Packing Efficiency and Packing Fraction
Efficiency and Packing Fraction
2011 Unit Cells Crystal Lattices
2012 Calculations Involving Unit Cell Dimensions Formation of Voids in Close Packing
2013 Formation of Voids in Close Packing Radius Ratio Rule
Unit Cells, Crystal Lattices, Formation of Voids in Close
2014 Packing, Simple Ionic Compounds, Point Defects in
Crystals

2015 Calculations Involving Unit Cell Dimensions Close Packing in Crystals

2016 Close Packing in Crystals

Chapter-16.indd 511 8/5/2016 10:37:23 AM


512 Chapter 16 Solid State

SUMMARY
1. The solid state is characterized by its rigid form and definite shape. The constituent particles (atoms, molecules or ions) in a solid are
held together by strong intermolecular, interatomic or interionic forces. The extent of binding forces between these particles deter-
mines the structure and properties of solids compared to liquid and gases.
2. Solids have high density; are essentially incompressible; undergo little thermal expansion; have a fixed volume, mass and shape; have
strong intermolecular forces and are more rigid.
3. Classification of solids
(i) Based on arrangement of particles
Crystalline • Large number of crystals with well-defined regular shape such as a cube, octahedron or tetrahedron.
solids • Metallic elements like copper, silver, non-metallic elements like sulphur and iodine and most of the ionic
compounds such as NaCl, KNO3 are crystalline solids. Organic compounds such as benzoic acid, naphtha-
lene, etc., are also crystalline.
Amorphous • These do not have a definite geometrical shape.
solids • These are formed by sudden cooling of a liquid and are also sometimes called supercooled liquids or
pseudo solids. For example, coal, coke, glass, plastic, rubber, and quartz glass, etc.
Polycrystal- • Solids that have a structure that falls between the two extremes of amorphous and crystalline solids.
line solids • These are aggregates of large numbers of small crystals or grains within which the structure is regular, but
the crystals or grains are arranged in a random fashion. For example, aluminum and steel.

Tip Amorphous solids soften on heating over a range of temperature and become crystalline at some temperature. For example,
quartz exists in crystalline form and in amorphous form as quartz glass.

Difference between amorphous and crystalline solids


Crystalline solids Amorphous solids
The arrangement of atoms shows long-range three-
They do not show any long-range order.
dimensional order.
They possess sharp melting points. They melt over a wide range of temperature.
They show anisotropy. They show isotropy.
They are cut along specific crystal planes. They are cut along random directions and have irregular surfaces.
They have characteristic heat of fusion. They do not have definite heat of fusion.

Tip Anisotropy is the property by which the magnitude of physical properties such as refractive index, thermal conductivity, etc.,
shows a variation with the direction in which it is measured. Isotropy is the property by which the substances exhibit the same values
of any physical property in all directions.

(ii) Based on nature of bonds


Crystalline solids may be classified into four categories based on the nature of intermolecular forces operating between the con-
stituent particles.
Nature of Nature of interaction Physical and electrical
Type of solid constituent particles between particles properties Examples
Molecular solid Molecules • Dispersion or London • Soft and do not conduct H2, CO2
• Non-polar forces electricity

• Dipole–dipole interac- • Low melting points HCl, SO2


tions
• Hydrogen bonding • Non-conductors of electric-
• Polar ity in both solid and liquid H2O
states
• Hydrogen
bonded
(Continued)

Chapter-16.indd 512 8/5/2016 10:37:23 AM


Summary 513

(Continued)

Nature of Nature of interaction Physical and electrical


Type of solid constituent particles between particles properties Examples
Ionic solid Ions Ionic (Coulombic or elec- • Relatively hard NaCl, MgO, ZnS
trostatic) bonding • Brittle
• High melting points
• Non-conductors of electric-
ity as solids, but conduct
when melted
Metallic solid Positive metal ions in a Metallic bonding • Range from very hard to Fe, Mg, Cu
sea of delocalized “free very soft
electrons”. • Melting points range from
high to low
• Conduct electricity in both
solid and liquid states
• Have characteristic luster
Covalent network Atoms and chemical Covalent bonding • Very hard C (diamond)
solid subunits • Very high melting points SiO2 (quartz)
• Non-conductors of C (graphite)
electricity (graphite is an
exception)

4. Crystal and its characteristic features


(a) A solid material in which the constituent atoms, molecules, or ions are arranged in an ordered pattern extending in all three
spatial dimensions is known as crystal or crystalline solid.
(b) The characteristic features of a crystal are:
(i) Faces: These are generally planar surfaces, arranged on a definite plane depending on internal geometry, by which the crys-
tal is bound. Crystals may have like or unlike faces.
(ii) Edges: These are formed by the intersection of two adjacent faces of a crystal.
(iii) Interfacial angle: It is the angle between the normal to the two intersecting faces.
(iv) Zone and zone axis: The set of faces which meet in parallel edges or would do so if the planes of the faces are extended consti-
tute a zone. The line passing through the centre of the crystal in a direction parallel to the edge of zone is called the zone axis.

Tip The relation between number of faces (F), edges (E) and interfacial angle (C) for a crystal is given by:
F +C =E +2

(c) A crystal is said to be symmetric if it has


(i) Plane of symmetry: A plane passing through the centre of the crystal such that it divides the crystal into two equal portions
that are mirror images of each other.
(ii) Axis of symmetry: A line about which the rotation of the crystal results in the same appearance more than once in one
complete revolution.
(iii) Centre of symmetry: The point in the crystal located such that any line passing through it divides the crystal into two sym-
metric portions.
5. Crystal lattices and unit cells
(a) Crystal lattice
A regular arrangement of the constituent particles of a crystal in a three dimensional space is called crystal lattice or space
lattice. The characteristics of a crystal lattice are:
(i) Each point in a crystal lattice is known as lattice point or lattice site.
(ii) Each point in a crystal lattice represents a constituent particle which can be an atom, ion or a molecule.
(iii) Lattice points when joined by straight lines define the geometry of the lattice.
(b) Unit cell
The unit cell is the smallest three-dimensional portion of a complete space lattice, which when repeated over and again in differ-
ent directions produces the complete space lattice. The unit cell is part of the lattice. The characteristics of unit cell are:
(i) The size of the unit cell is defined using lattice parameters (also called lattice constants or cell parameters).
(ii) These are the relative dimensions of the three edges a, b, c of the unit cell along the three axes (primitives) and the angles
(interfacial angles) between these edges are given by a (angle between b and c); b (angle between a and c) and g (angle
between a and b).

Chapter-16.indd 513 8/5/2016 10:37:23 AM


514 Chapter 16 Solid State

(iii) The position of the atoms in the unit cell is defined by its coordinates (xi, yi, zi) along the crystallographic axes.

Unit cell
Lattice point

6. Crystal systems
(a) Primitive unit cell
(i) The constituent particles (atoms, ions or molecule) are present only at the corners of the unit cell.
(ii) The shapes of unit cells are restricted by symmetry as well as space considerations.
(iii) There are seven unique and basic unit-cell shapes (primitive unit cells) with varying elements of symmetry in a three-
dimensional space as shown below:

Crystal system Edge lengths Angles Examples


Cubic a=b=c a = b = g = 90° NaCl, ZnS, Cu
Tetragonal a=b≠c a = b = g = 90° White Sn, SnO2, TiO2, CaSO4
Orthorhombic a≠ b≠ c a = b = g = 90° Rhombic Sulphur, KNO3, BaSO4
Monoclinic a≠ b≠ c a = g = 90°; b ≠ 90° Monoclinic Sulphur, Na2SO4 ⋅ 10H2O
Triclinic a≠ b≠ c a ≠ b ≠ g ≠ 90° K2Cr2O7, CuSO4 ⋅ 5H2O, H3BO3
Rhombohedral or Trigonal a=b=c a = b = g ≠ 90° Calcite (CaCO3), HgS (cinnabar)
Hexagonal a=b≠c a = b = 90°; g = 120° Graphite, ZnO, CdS
(b) Non-primitive unit cell
In these unit cells, the constituent particles (atoms, ions or molecules) are present at the corners a well as on some other positions
of the unit cell (say faces, edges, etc.). These are further classified as follows:

Body-centred Atoms are present at the corners and one at the


centre of the unit cell. The crystal systems with
this type of unit cell include cubic, tetragonal and
orthorhombic.

Face-centred Atoms are present at the corners and at the centre


of each face of the unit cell. The crystal systems
with this type of unit cell include cubic and orthor-
hombic.

End-centred Atoms are present at the centre of diagonal joining


the nearest neighbours at one set of faces in
addition to the atoms at the corner of the unit cell.
The crystal systems with this type of unit cell are
orthorhombic and monoclinic.

(c) Coordination number


It is the number of particles in a system which surrounds a particular particle as its nearest particle in its neighbour.

(d) Bravais lattices


In 1848, Auguste Bravais demonstrated, based on geometry, that there are, in fact, only 14 possible ways in which similar lat-
tice points can be arranged in regular order in three-dimensional space while maintaining their translational symmetry. For his
efforts, crystal lattices are often referred to as Bravais lattices.

Chapter-16.indd 514 8/5/2016 10:37:25 AM


Summary 515

Crystal classes Bravais lattices


Cubic

Primitive Body-centred Face-centred

Tetragonal

Primitive Body-centred

Orthorhombic

Primitive End-centred Body-centred Face-centred

Monoclinic

Primitive End-centred

Triclinic

Primitive

Rhombohedral or trigonal

Primitive

Hexagonal

Primitive

Chapter-16.indd 515 8/5/2016 10:37:28 AM


516 Chapter 16 Solid State

7. Calculations involving unit cell dimensions


(a) Number of atoms
(i) The number of atoms contained within the unit cell can be calculated by adding the contribution of each constituent par-
ticle to the unit cell.
(ii) An atom at the corner of a unit cell is shared by eight unit cells. Hence each atom contributes 1/8 to the unit cell.
(iii) An atom at the face is a shared by two unit cells, contributing 1/2 to the unit cell.
(iv) An atom within the body of the unit cell is shared by no other unit cell. Hence, each atom contributes 1 to the unit cell.
(v) An atom present on the edge is shared by four unit cells. Hence, each atom on the edge contributes 1/4 to the unit cell.
(b) Mass of the unit cell
The mass of the atoms in a unit cell can be calculated as follows:
Mass of the atoms of unit cell = Number of atoms in a unit cell × Mass of an atom (Matom )
Atomic mass (A)
Mass of one atom =
Avogadro number (NA )

(c) Volume of unit cell


The volume of unit cell for a cubic unit cell is
Vc = a3
(d) Density calculation
(Number of atoms/cell) × (Atomic mass)
Density (r ) =
( Volume of unit cell) × (Avagadro’s number)
nA
or r=
Vc × NA
(e) Volume of space occupied
Number of atoms per unit cell × volume of a sphere (V = 4/3p r3).
8. Cubic lattice – simple, face centred and body centred

Simple cubic Body centred (bcc) Face centred (fcc)


Edge length: In simple cubic packing, the Edge length: The hard spheres touch Edge length: The hard spheres or ion
atoms touch along the edge of the cube. one another along cube diagonal. So, cores touch one another across a face
Hence the cubic cell edge a = 2r, where R the cube edge length, a = 4 r / 3 (since diagonal. Therefore, the cube edge
is the atomic radius. a = 4 r=/ 3 times the length
diagonal of a cube length, a = 2r 2 [since (4r)2 = a2 + a2 ⇒
of one of its sides). The diagonal of (a2 = 8r2)].
the cube covers R + 2R + R = 4R (as the
middle sphere actually touches the two
corner spheres).
One-eighth of an
atom

One atom

Number of atoms per unit cell = (8 × 1/8) Eight atoms at the corners = 8 × 1/8 = 1 One atom at each corner = (8 × 1/8 each)
= 1 atom atom = 1 atom
or rank (z) = 1 Atoms at the body centre = 1 atom Atoms at the six face centres
Number of atoms in a body-centred = (6 × 1/2 each) = 3 atoms
cubic unit cell = 1 + 1 = 2 atoms Number of atoms in a face-centred cubic
or rank (z) = 2 unit cell = 4 atoms
or rank (z) = 4
Number of nearest neighbours = 6 Number of nearest neighbours = 8 Number of nearest neighbours = 12
Atomic mass (MA ) Atomic mass (MA ) Atomic mass (MA )
Mass = Mass = 2 × Mass = 4 ×
Avogadro number (NA ) Avogadro number (NA ) Avogadro number (NA )
(Continued)

Chapter-16.indd 516 8/5/2016 10:37:31 AM


Summary 517

(Continued)
Simple cubic Body centred (bcc) Face centred (fcc)
Volume, Vc = a3 = 8r3 3
 4r   64  Volume, Vc = a3 = (2r 2 )3 = 16r 3 2
Volume, Vc = a3 =  =   3r 3
 3   9

MA MA 2MA /NA 9 MA 4 MA /NA MA 2


Density, r = 1 × 3
= 3
Density, r = 3
= Density, r = =
( 2r ) NA 8N A r (64/9 ) 3r 32 3NA r 3 16r 3 2 8 NA r 3

Volume of space occupied: Volume of space occupied: Volume of space occupied:


4 4 8 4 16
V = p r3 V = 2 × p r3 = p r3 V = 4 × p r3 = p r3
3 3 3 3 3

9. Close-packing in crystals

Coordination
Packing Feature number Figure
Close packing in one dimension
• The closest efficient packing is when the spheres 2
just touch each other.

Close packing in two dimensions


Square close • It is obtained when the second row of spheres is 4
A
packing placed adjacent to the first row. The spheres in the
two rows are thus aligned horizontally as well as A
vertically A
• The arrangement is AAA type.
A
(a)

Hexagonal • It is obtained when the second row of spheres is 6


B
close packing placed in the interstices or depressions of the first-
row spheres A
• The arrangement is known as ABAB type. B
A

(b)

Close packing in three dimensions


From two-
dimensional
hexagonal 12
close packing

(a) Placing The second layer of spheres is placed in the depres-


second layer sion created by the first layer.
over the first
layer

(Continued)

Chapter-16.indd 517 8/5/2016 10:37:36 AM


518 Chapter 16 Solid State

(Continued)
Coordination
Packing Feature number Figure
(b) Placing • The spheres of the third layer can be placed in the
third layer voids of the spheres of second layer such that they
over the are exactly in the same positions as the spheres of
second first layer.
A
layer • Pattern in this type of packing is ABABAB… and is
B
called hexagonal close packing or hcp. A
B
A
B
A

Hexagonal
closest packed
structure
• Atoms in the third layer are placed such that they
are on the positions corresponding to the three
voids in the A layer that were not covered by the
atoms in the B layer, then the third layer is different A
from either A or B and is labeled C. B
• If a fourth layer is placed then it repeats the A layer C
orientation, and succeeding layers repeat the
A
pattern ABCABCA and the pattern is called ccp or
fcc. B
C
A

Cubic
closest packed
structure
Body centred The spheres in the second layer are placed at the top 8
cubic arrange- of the hollow spaces in the first layer and the spheres
ment in the third layer are placed exactly as in the first
layer. a

a
a

10. Packing efficiency and packing fraction


The packing efficiency, f, is defined as the percentage of the total volume of the unit cell actually occupied by particles.
Volume occupied by particles
Packing efficiency = × 100
Total volume of the unit cell

Packing Packing efficiency


Simple cubic (sc) structure
1 × ( 4 / 3)p R 3 p
Packing efficiency = × 100 = × 100 = 52.35%
(2R )3 6
Body-centred cubic (bcc) structure
2 × ( 4/3)p R 3 (8 / 3)p R 3 × 100
Packing efficiency = × 100 = = 68%
( 4/ 3R )3 64 / 3 3R 3
Cubic close packing or face-centred cubic (fcc) Volume of four spheres in the unit celll
arrangement Packing efficiency = × 100
Total volume of the unit cell
16/3p r 3 p
= × 100 = × 100 = 74%
16 2r 3 3× 2
(Continued)

Chapter-16.indd 518 8/5/2016 10:37:39 AM


Summary 519

(Continued)
Packing Packing efficiency
Hexagonal close packing (hcp) Volume of six spheres in the unit celll
Packing efficiency = × 100
Total volume of the unit cell
8p r 3 p
= 3
× 100 = × 100 = 74%
24 2r 3× 2

11. Tetrahedral and octahedral voids


In the close packing of constitutents some interstitial sites are left. These sites in the crystal lattice are called voids.
Voids Features Radius
• The four spheres touch each other in a manner that a tetrahedron is formed
when their centres are joined and the space in the centre is called a tetrahe-
dral site/hole or void.
• The number of tetrahedral voids per unit cell is eight.
Tetrahedral void

Tetrahedral void
The radius of the tetrahedral void
relative to the radius of sphere is
r
0.225. i.e., void = 0.225
rsphere

• An octahedral site/hole or void is formed by six spheres touching each A


other.
B B
• The total number of octahedral void per unit cell is 4.
Octahedral void

A A
B

The radius of the octahedral void


relative to the radius of sphere is
r
0.414. ie., void = 0.414
0.225
rsphere

12. Radius ratio rule


(a) The relationship between the radius, the coordination number and the structural arrangement is called the radius ratio rule.
Radius of cation (r )
Radius ratio =
Radius of anion (R )
The greater is the radius ratio, the greater is its coordination number.
(b) The radius ratio for important crystal systems is shown below:
Radius ratio Anions close packing Coordination number
0.225 to 0.414 Tetrahedral 4
0.414 to 0.732 Octahedral 6
0.732 to 1.00 Body-centred cubic 8
13. Bragg’s law
Bragg’s equation can be used to calculate the distances between repeating planes of the particles in a crystal or the corresponding
wavelengths of the incident X-ray beam.
nl = 2d sinq
where n is an integer (1, 2, 3, 4, …) representing the serial order of diffracted beams, d is the distance between the successive atomic
planes and l is the wavelength.
14. Miller indices for crystallographic planes in cubic unit cells
(a) The Miller indices are conventions used to specify directions and planes in a crystal plane. These are defined as the reciprocal
of the fractional intercepts, which the plane makes with the crystallographic x-, y- and z-axes of the three non-parallel edges of
the cubic unit cell.
(b) For simple cubic crystals, the spacing between adjacent (hkl) lattice planes is given by
a
dhkl =
h + k2 + l2
2

Chapter-16.indd 519 8/5/2016 10:37:41 AM


520 Chapter 16 Solid State

where the lattice vectors are orthogonal and of equal length, denoted by lattice constant a. The notation (hkl) is used to indicate
Miller indices, where h, k and l are Miller indices of cubic crystal plane for x-, y- and z-axes.

15. Simple ionic compounds

Structure Examples
Each of the ions is octahedral- Metal halides (except
Rock salt (or NaCl) type of structure

ly surrounded by six counter Cs), ammonium halides,


Na +
ions, and so this structure is silver halides (except
Cl −
called (6,6)-coordination, that silver iodide) and oxides
is, each Na+ is surrounded by and sulphides of alka-
6 Cl− and each Cl− by 6 Na+. line earth metals (except
Number of Cl− ions in each BeS).
unit cell = 4
Number of Na+ ions in each
unit cell = 4
Total NaCl units in one unit
cell = 4.

Zn2+ ion surrounded The Zn2+ cations are tet- CuCl, CuBr, CuI, AgI, BeS.
tetrahedrally by rahedrally surrounded by
four S2 − ions S2− anions, and each S2−
Sphalerite or zinc blende (ZnS) structure

anion is surrounded by eight


Zn2+ tetrahedral sites, of which half
S2 − are occupied by Zn2+ ions,
and hence the anions are also
four-coordinated. The zinc
blende structure, therefore,
has (4,4)-coordination
Number of S2− ions per unit
cell = 4
Number of Zn2+ ions per unit
cell = 4
S2− ion surrounded Total number of ZnS per unit
tetrahedrally by cell = 4.
four Zn2 + ions

F− Each F− anion is tetrahe- BaF2, BaCl2, SrF2, SrCl2,


Ca2 + drally surrounded by 4 Ca2+ etc.
ions, and each Ca2+ cation is
Fluorite structure (CaF2)

surrounded by 8 F− at tetra-
hedral sites, all of which are
occupied.
Hence, the cations are
eight-coordinated. So the
fluorite structure has (8,4)-
coordination.
Number of CaF2 molecules
per unit cell = 4.

(Continued)

Chapter-16.indd 520 8/5/2016 10:37:42 AM


Summary 521

(Continued)
Structure Examples
Each Cs+ ion is surrounded CsBr, CsI, CsCN, TiCl, TiBr,
Cl − ion surrounded by
Caesium chloride (CsCl) type of structure

by 8 Cl− ions and each Cl− ion TiCN, etc.


Cs+ ions
Cs+ is surrounded by 8 Cs+ ions.
Cl− Therefore, the crystal has
(8,8)-coordination
With 8 Cl− ions at corners
contributing one Cl− and one
Cs+ ion in the body centre
contributing one Cs+, it has
one CsCl per unit cell.

Eight unit cells

The central A ion is surround- NaNbO3, BaTiO3, CaZrO3,


ed by 12 X ions; the B ion by 6 YAlO3, KMgF3, etc.
X ions; and the X ion is linear-
A ly coordinated by 2 B ions.
Perovskite structure

Each O2− is surrounded by 8 Antifluorite structure is


Na+ Na+ ions. Each Na+ by 4 O2− more commonly found
Antifluorite structure (Na2O)

ions. Na2O has (4,8) coordina- in covalent solids. The


O2 − alkali metal oxides M2O
tion. Number of Na2O per unit
cell is 4. (M = Li, Na, K, Rb) crys-
tallize in the antifluorite
structure.

z
x Na

y
O

In normal spinel structure of MgAl2O4, ZnFe2O4, etc.


general formula AB2O4, O2−
Spinel structure

ions are cubic-closed packed,


whereas 1/8th of the tetra-
hedral holes are occupied
by A2+ cations and 1/2 of the
octahedral holes are occupied
by cations B3+.

Chapter-16.indd 521 8/5/2016 10:37:43 AM


522 Chapter 16 Solid State

16. Defects in crystals


(a) Defect refers to a disruption in the periodic order of a crystalline material. These occur in crystals because they lower the energy
of a crystal to make it more thermodynamically stable.

Tip Defects are also known as imperfections in solids.

(b) Defects in solids are further classified as follows:


(i) Point defects: Point defects are imperfect point-like regions in the crystal. Based on their nature, point defects are classified
into three types: stoichiometric defects, impurity defects and non-stoichiometric defects.

Vacancy defects • These result when some of the lattice sites in the crystal are vacant.
Non-ionic solids

• These develop during formation of crystals or when substance is heated.


• Density of the solid decreases as a result of vacancy defect.

Interstitial defects • These arise when some constituent particles (atoms or molecules) occupy
interstitial sites in the crystal.
• The density of the solid increases as a result of interstitial defect.

Frenkel defect (Dislocation • When an ion is displaced


A+ B− A+ B− A+
defect) from its regular position
to an interstitial position, +
A
it creates a vacancy; this
B− B− A+ B−
pair of vacancy–interstitial
is called Frenkel defect.
Stoichiometric defects

• This defect does not result A+ B− A+ B− A+


in any change in density
of the substance.
B− A+ B− A+ B−
Ionic compounds

Examples are ZnS, AgCl, AgBr and AgI.

Schottky defect • When a pair of one cation


and one anion of equal A+ B− A+ B− A+
valence is missing from an
ionic crystal, the condition
of charge neutrality is still B− B− A+ B−
maintained. The pair of
vacant sites, thus formed,
is called Schottky defect. A+ B− A+ A+
• It decreases density of the
solid.
• Shown by highly ionic B− A+ B− A+ B−
compounds, which
have high coordination Examples are NaCl, KCl, KBr, AgBr and CsCl.
number.

Defects in ionic crystals introduced by adding impurities in which


Na+ Cl− Na+ Cl−
the ions are in different valence state than the constituent ions of
the crystal.
Impurity defects

Cl− Ca2+ Cl− Na+

Na+ Cl− Cl−

Cl− Na+ Cl− Na+

(Continued)

Chapter-16.indd 522 8/5/2016 10:37:45 AM


Summary 523

(Continued)
Due to anion- • In these defects, a negative ion may
A+ B− A+ B−
ic vacancies be absent from its lattice site, leaving
a “hole” which is occupied by an
electron, thereby maintaining the A+
B− B− A+
electrical balance.
• Anion sites occupied by electrons in
this way are called F-centres. A+ e− A+ B−
• These F-centres are associated with
F centre
the colour of the compound.
• Solids containing F-centres are para- B− A+ B− A+
Metal excess defect

magnetic, because the electrons


occupying the vacant sites are
Non-stoichiometric defects

(a)
unpaired.
Due to the These defects occur when an extra pos-
A+ B− A+ B−
presence of itive ion occupies an interstitial position
extra cations in the lattice, and electrical neutrality A+
in the inter- is maintained by the presence of an A+
B− B− A+
stitial sites interstitial electron.
e−
A+ B− A+ B−

B− A+ B− A+

(b)
Examples are, ZnO, CdO, Fe2O3 and Cr2O3.
Metal-deficient compounds may be represented by the general formula A1−d Χ. Crystals with metal defi-
deficiency

ciency defects are semiconductors.


Metal

Tip Line defects are groups of atoms in irregular positions. Linear defects are commonly called dislocations or single dimensional
defects.

17. Conducting properties of solids


(a) The conductivity is related to the number of valence electrons and can be explained using the band theory given below.
(i) Atomic orbitals of the metal atoms form molecular orbitals that are so closely spaced that instead of having discrete ener-
gies, the available energy states form bands.
(ii) The essential criteria for any solid to conduct are the presence of electrons in conduction band and the gap between the
valence and conduction band.
(iii) In metals, the valence band overlaps the conduction band, in semiconductors there is a small enough gap between the
valence and conduction bands such that thermal or other excitations can bridge the gap and in insulators the electrons in
the valence band are separated by a large gap from the conduction band.
Energy Energy

Conduction band

Conduction band
Partially
Valence band
filled
Valence band

Conductor Conductor

Energy Energy
Conduction band
Vacant Vacant

Chapter-16.indd 523 8/5/2016 10:37:47 AM


Conduction band

Conduction band
Partially
Valence band
filled
Valence band

524 Chapter 16 Solid State


Conductor Conductor

Energy Energy
Conduction band
Vacant Vacant

Small gap
Large gap

Filled
Valence band
Filled

Insulator Semiconductor

(b) Doping
Pure semiconductor grade silicon needs to be incorporated with impurities for it to develop semiconductor properties. This
process is known as doping.
Impurity Conduction Type
Electron-rich impurities Adding electron-rich impurities Extrinsic; current is carried by n-type semiconduction
like As (or any Group 15 element) excess electrons.
Electron-deficient Group 13 elements (for e.g. Intrinsic semiconductors are not ex- p-type semiconduction
impurities Indium) ternally doped. Current is carried by
the migration of positive centres.

Tip (a) p type semiconduction is characterized by migration of “positive holes” across the crystal.
(b) Superconductors offer no resistance to the flow of current. They are derived from p-block elements as they become super-
conducting at room temperature. At 2−5 K, most of the metals become superconductors.
(c) Intrinsically silicon and germanium conduct a small current. However, the addition of a small amount of dopant atoms
into their regular crystal lattice produces dramatic change in their electrical properties, producing n-type and p-type
semiconductors.

18. Magnetic properties of solids


Solids can be classified into the following five categories based on the magnetic behaviour shown by them.
Type Magnetization Unpaired electrons Net magnetic moment Examples Remarks
Diamag- Weakly magnetized in None 0 NaCl, KCl, TiO2 Weakly repelled
netic a direction opposite when placed near
substance to that of an applied a magnet. Diamag-
magnetic field. netism is independ-
ent of temperature.
Paramag- Magnetized along the 1 or more Present (arises from the Organic free radi-
netic sub- direction of the ap- electron spin angular cals and gaseous
stances plied magnetic field. momentum) nitric oxide
Atoms and ions KMnO4, Na, K, Paramagnetism is
with unfilled inner Cu2+ and Fe3+ shown by these
electron shells ions substances only in
the presence of a
magnetic field.
Ferro- Strongly attracted by Present (due to angular Iron, nickel, At sufficiently high
magnetic applied magnetic field momentum) cobalt CrO2 temperatures, all
substanc- and can be perma- these substances
es nently magnetized; become paramag-
spontaneous magnet- netic.
ization
(Continued)

Chapter-16.indd 524 8/5/2016 10:37:47 AM


Solved Examples 525

(Continued)
Type Magnetization Unpaired electrons Net magnetic moment Examples Remarks
Antiferro- Zero net magnetiza- No net total moment in Metals, alloys and
magnetic tion applied magnetic field salts of transition
substance elements such as
MnO, MnSe, etc

Ferrimag- The atomic dipoles are Fe3O4, and ferri- Such materials are
netic sub- arranged antiparallel tes like MgFe2O4 weakly attracted by
stances to one another and the and ZnFe2O4. magnetic fields.
number of magnetic These lose ferrimag-
moments aligned in netism on heating
one direction is more and become para-
than that aligned in magnetic.
the other direction;
net magnetization is
observed

19. Dielectric properties of solids


(a) Insulators or dielectric are materials used to prevent the loss of electricity through certain parts in an electrical system. Dielectrics
also store electrical charges.
(b) On applying electric field, polarization of charge takes place leading to formation of dipoles. In some cases, permanent dipoles
could be formed which cause the crystal to have net dipole moment by aligning themselves in an ordered manner.
(c) The following properties are shown by polar crystals:
Property Definition Example
Piezoelectricity The electricity produces due to applica- Lead zirconate, quartz, etc.
tion of mechanical stress causing defor-
mation and displacement of ions.
Pyroelectricity Small electric current produced on
heating piezoelectric crystals generates
pyroelectricity.
Ferroelectricity When in piezoelectric crystals having Rochelle’s salt, potassium hydro-
permanent polarized dipoles even in the gen phosphate, barium titanate,
absence of the electric field, on apply- etc.
ing field the direction of polarization
changes.
Anti-ferroelectricity In some crystals, the dipoles point Lead zirconate, etc.
alternately in upward and downward
direction with net dipole moment of the
crystal becoming zero.

Tip When the main function is that of insulation, the materials are called insulators and when the charge storage is the main func-
tion, they are called dielectrics.

SOLVED EXAMPLES
Unit Cells Number of atoms of B (per unit cell) = (1/2) × 5 (because
1 atom is missing from the face) = 5/2
1. In a face-centred cubic lattice, atom A occupies the corner
Formula is A1B5/2 = A2B5
position and atom B occupies the face centre positions. If one
atom of B is missing from one of the face-centred points, the 2. How many unit cells are present in a cube–shaped ideal crystal
formula of the compound is: of NaCl of mass 1g?
(a) AB2 (b) A2B3 (c) A2B5 (d) A2B (a) 2.57 × 1021 unit cells
(AIEEE 2011, JEE Main Online 2014) (b) 5.14 × 1021 unit cells
Solution (c) 1.28 × 1021 unit cells
(c) Number of atoms of A (per unit cell) = 1/8 × 8 = 1 (d) 1.71 × 1021 unit cells

Chapter-16.indd 525 8/5/2016 10:37:48 AM


526 Chapter 16 Solid State

Solution Calculations Involving Unit Cell Dimensions


(a) 1 unit cell has 4 NaCl. So, mass of one unit cell
6. Total volume of atoms present in a face-centred cubic unit cell
58.5
= 4× × 10 −23 g of a metal is (r is atomic radius)
6.023
1 6.02 × 10 23 20 3 24 3 12 3 16 3
Number of unit cells in one g = × = 2.57 × 1021 (a) pr (b) pr (c) pr (d) pr
4 58.5 3 3 3 3
1 6.02 × 1023 (AIEEE 2006)
× = 2.57 × 1021
4 58.5 Solution
3. An ionic compound has a unit cell consisting of A ions at (d) For a face centred cubic unit cell there are 4 atoms per unit
the corners of a cube and B ions on the centres of the faces cell.
4
of the cube. The empirical formula for this compound would For one spherical atom, volume V = p r3
be 3
(a) AB (b) A2B (c) AB3 (d) A3B 4 3 16 3
For 4 atoms, volume V = p r = p r
3 3
Solution 7. Copper crystallizes in fcc with a unit cell length of 361 pm.
1
(c) Number of atoms of A = × 8 (corners) = 1 What is the radius of copper atom?
8
(a) 157 pm (b) 181 pm
1 (c) 108 pm (d) 127 pm
Number of atoms of B = × 6 (faces) = 3
2 (AIEEE 2009)
Hence, formula of compound will be A1B3. Solution
Crystal Lattices (d) In case of fcc, the relationship between edge length
(a) and radius of sphere (r) is
4. In a monoclinic unit cell, the relation of sides and angles are
(2)1 2 a = 4 r
respectively:
(a) a = b ≠ c and a = b = g = 90° 2a 2 × 361
r=
= = 127 pm
(b) a ≠ b ≠ c and a = b = g = 90° 4 4
(c) a ≠ b ≠ c and b = g = 90° ≠ a 8. The edge length of a face-centred cubic cell of an ionic sub-
(d) a ≠ b ≠ c and a ≠ b ≠ g ≠ 90° stance is 508 pm. If the radius of the cation is 110 pm then the
(JEE Main Online 2014) radius of the anion is
Solution (a) 144 pm (b) 288 pm
(c) 398 pm (d) 618 pm
(c) In a monoclinic unit cell
(AIEEE 2010)
a≠b≠c
Solution
b = g = 90° ≠ a
(a) For an fcc arrangement,
Edge length 508
(r + + r − ) = ⇒ (110 + r − ) = ⇒ r − = 144 pm
g = 90° 2 2
9. Lithium forms body-centred cubic structure. The length of the
b = 90° side of its unit cell is 351 pm. Atomic radius of the lithium will
c be
(a) 75 pm (b) 300 pm
a ≠ 90° b (c) 240 pm (d) 152 pm
(AIEEE 2012)
a
Solution
5. For trigonal crystal system, the relationship between parame- (d) For a bcc structure, the relationship between a (length of
ters is the side of the unit cell) and r (atomic radius) is given as
(a) a = b ≠ c ; a = b = 90°; g = 120° 3a = 4 r
where a is given as 351 pm.
(b) a = b ≠ c ; a = b = g = 90°
351
(c) a = b = c ; a = b = g ≠ 90° So, r = 3 × = 152 pm
4
(d) a ≠ b ≠ c ; a = b = g = 90° 10. Sodium metal crystallizes in a body-centred cubic lattice
Solution with a unit cell edge of 4.29 Å. The radius of sodium atom is
approximately
(c) Hexagonal: a = b ≠ c ; a = b = 90°; g = 120° (a) 3.22 Å (b) 5.72 Å
Tetragonal: a = b ≠ c ; a = b = g = 90° (c) 0.93 Å (d) 1.86 Å
Trigonal: a = b = c ; a = b = g ≠ 90° (JEE Main 2015)
Solution
Orthorhombic: a ≠ b ≠ c ; a = b = g = 90°. 3 a 1.732 × 4.29
(d) For bcc arrangement, r = = = 1.86 Å
4 4

Chapter-16.indd 526 8/5/2016 10:37:55 AM


Solved Examples 527

11. The structure of metals can be explained by close packing of (a) 152.5 pm (b) 121.3 pm
metal atoms. In structure of an unknown metal (M), atoms are (c) 148.6 pm (d) 117.1 pm
found to be packed in fcc lattice. Also, the effective radius of
Solution
atoms is 1.414 Å. If its density is found to be 5.8 g cm−3, the
atomic mass of metal is (d) For fcc, the radius is calculated as
(a) 55.8 g mol−1 (b) 111.6 g mol−1 a 4 00
r= = = 1414. pm
(c) 37.2 g mol−1 (d) 30.1 g mol−1 2(2)1/ 2 2(2)1/ 2
Solution The greatest sphere will fit in octahedral void because it is
bigger than tetrahedral void.
(a) Since metal atoms are packed in fcc,
Now, radius of the octahedral void (R) = 0.414 × r = 0.414
4 R 4 × 1.414 × 141.4 = 58.54 pm
2a = 4 R ⇒ a = = =4 Å
2 2 Therefore, diameter = 2R = 2 × 58.54 = 117.08 pm.
where a is the unit cell edge length and R is the radius of Packing Efficiency and Packing Fraction
the sphere.
In fcc lattice, the number of atoms per unit cell (z) = 4, 15. Percentage of free space in cubic close-packed structure and
So, atomic mass of metal is found as in body-centred packed structure are, respectively,
(a) 48% and 26% (b) 30% and 26%
zM r × a 3 × NA (c) 26% and 32% (d) 32% and 48%
r= 3
⇒M=
a NA z (AIEEE 2010)
Solution
5.8 × ( 4 × 10 −8 )3 × 6.022 × 1023
M= = 55.8 g mol−1 (c) This is because packing efficiency of cubic close packing
4
and body-centred packing are 74% and 68%. So, the free
12. The unit cell of a metallic element of atomic mass 108 and space will be 100 − 74 = 26% and 100 − 68 = 32%.
density 10.5 g cm−3 is a cube with edge length of 409 pm. The
structure of the crystal lattice is 16. Experimentally it was found that a metal oxide has formula
(a) fcc. (b) bcc. M0.98O. Metal M, is present as M2+ and M3+ in its oxide.
(c) hcp. (d) simple cubic. Fraction of the metal which exists as M3+ would be
(a) 4.08% (b) 6.05% (c) 5.08% (d) 7.01%
Solution (JEE Main 2013)
(a) The density is given by Solution
z×M (a) Given, metal oxide = M0.98O. If x ions of M are in +3 state,
r=
NA × a 3 then 3 x + (0.98 − x ) × 2 = 2 ⇒ x = 0.04.
So, the percentage of metal in +3 state would be
Here , M = 108 , NA = 6.023 × 1023, a = 409 pm = 4.09 × 0.04
× 100 = 4.08%
10−8 cm, r = 10.5 g cm−2 0.98
Substituting the values, we get 17. Caesium chloride forms a body-centred cubic lattice. If the
Number of atoms per unit cell (z) = 4 radius ratio (r+/r−) is taken as 0.9, what is the packing effi-
So, the structure of the crystal lattice is fcc. ciency of edges to the total available edges?
13. A metallic element exists as cubic lattice. Each edge of the  9 3  3p   1   10 
(a)  (b)  (c)  (d) 
unit cell is 288 Å. The density of the metal is 7.20 g cm−3. How 
 19   8 
  2 3   19 3 
many unit cell will be present in 100 g of the metal?
(a) 6.85 × 102 (b) 5.82 × 1023 Solution
(c) 4.37 × 105 (d) 2.12 × 106 r 
(a) Given that  +  = 0.9 ⇒ r+ = 0.9r−
 r− 
Solution
(b) The volume of the unit cell = (288 Å)3 = 23.9 × 10 −24 cm3 3a
For bcc, 2r+ + 2r− = 3a ⇒ r− =
3.8
m 100
The volume of 100 g of the metal = = = 13.9 cm3 1.8a 3 9a 3
r 7.20 ⇒ 2r+ = =
3.8 19
13.9 cm3 2r
Therefore, packing efficiency of edges = + =
9 3
Number of unit cells in this volume = = 5.82 × 1023
23.9 × 10 −24 a 19
18. Calculate the percentage of volume unoccupied of packing in
13.9 cm3 case of a metal crystal for primitive cubic.
= = 5.82 × 1023
23.9 × 10 −24 (a) 56.5 % (b) 62.8 % (c) 47.6 % (d) 39.8 %
Solution
14. In face-centred cubic (fcc) crystal lattice, edge length is 400 pm.
Find the diameter of the greatest sphere which can be fitted (c) Consider the figure of primitive cubic unit cell as shown
into the interstitial void without distortion of the lattice. in the following figure.

Chapter-16.indd 527 8/5/2016 10:38:00 AM


528 Chapter 16 Solid State

3a
(a) rCs+ + rCl− = 3a (b) rCs+ + rCl− =
2
r r 3a
(c) rCs+ + rCl− = (d) rCs+ + rCl− = 3a
a 2
(JEE Main 2014)
From the figure, we have a = 2r where a is the edge length Solution
and r is the radius of the sphere. Then, number of spheres
present per unit cell = 8 × 1/8 = 1 (c) Body centred cubic lattice of CsCl is shown below.
Volume of sphere = 4/3p r3 Cs+ ion surrounded by
Volume of cube = a3 = (2r)3 = 8r3 8 Cl− ions
Cs+
3 Cl−
4/3p r
Packing efficiency = = 52.4%
8r 3
So, the percentage of unoccupied volume = 100 − 52.4
= 47.6%

Formation of Voids in Close Packing


19. In a compound, atoms of element Y from ccp lattice and those
of element X occupy 2/3rd of tetrahedral voids. The formula of
the compound will be Two Cl− and one Cs+ touch each other at body diagonal
(a) X4Y3 (b) X2Y3 (c) X2Y (d) X3Y4 of cube. Thus,
(AIEEE 2008) + 3a
2rCs + 2rCl− = 3a ⇒ rCs
+
+ rCl− =
Solution 2

(a) In ccp lattice tetrahedral voids are two times of number 23. Rubidium chloride has rock salt structure. If the edge length
of atoms. of unit cell is 658 pm and diameter of chloride ion is 362 pm,
Number of atoms of Y = 4 the radius of Rb+ ion is
2 (a) 232 pm (b) 284.27 pm
Number of atoms of X = × 8 (c) 388.84 pm (d) 148 pm
3
Hence, formula of compound will be X4Y3 Solution

20. The total number of octahedral void(s) per atom present in a (d) In rock salt structure, cations and anions are in contact at
cubic close packed structure is the edges of the unit cell.
(a) 2 (b) 4 (c) 1 (d) 3
(JEE Main Online 2014) Cl− Rb+ Cl−
Solution
Rb+ Cl− Rb
+
(c) In ccp packing total octahedral voids are equal to num-
ber of atom. In ccp 4 atoms and 4 octahedral voids are
Cl− Rb+ Cl−
present. Hence, there is one void per sphere.
21. A solid is formed with three types of atoms A, B and C. A forms a 658 362
2rCl− + 2rRb+ = a ⇒ rRb+ = −r − = − = 148 pm
fcc lattice, while B atoms occupy all tetrahedral voids and 2 Cl 2 2
C atoms occupy half of the octahedral voids. The formula of 24. Consider the ionic and covalent solid crystal structures and
the solid is select incorrect statement?
(a) A2B4C (b) A2B2C (c) AB2C2 (d) AB2C (a) In rock salt structure, the centre to centre distance of cat-
ions and anions is equal to the half of the cell parameter.
Solution
1 1 (b) In fluorite structure, calcium ions form fcc lattice and flu-
(a) The atoms of A per unit cell = 8 × + 6 × = 4 oride ions occupy all tetrahedral voids.
8 2
(c) ZnS has diamond structure in which sulphide ions form fcc
The atoms of B per unit cell = 2 × 4 = 8 lattice and zinc ions occupy half of the tetrahedral voids.
1 (d) In diamond, the centre to centre distance of carbon
The atoms of C per unit cell = ×4=2 atoms is equal to the half of the face diagonal of unit cell.
2
Therefore formula is A4B8C2 A2B4C Solution
(d) In rock salt structure, anion forms fcc lattice and cations
Simple Ionic Compounds occupy all the octahedral voids exist in fcc. Since octa-
hedral void exist at edge and body centre, the centre to
22. CsCl crystallizes in body centred cubic lattice. If a is its edge centre distance of cations and anions is equal to the half
length then which of the following expressions is correct? of the edge length of unit cell.

Chapter-16.indd 528 8/5/2016 10:38:05 AM


Solved Examples 529

In fluorite structure, calcium ions (cations) form fcc lattice Now, aRb+ = 30 + aK + = 30 + 376.181 = 406.181 pm
and fluoride ions (anions) occupy all tetrahedral voids.
ZnS has diamond structure in which sulphide ions form fcc aRb+ 406.181
lattice and zinc ions occupy half of the tetrahedral voids, Therefore, rRb+ = ⇒ rRb+ = = 143.58 pm
2 2 2 2
it implies that sulphide ions are in contact at face diago-
nal but in diamond, carbon cannot touch at face diag- Bragg’s Law
onal. It is so because same size of carbon atom occupy
void of the fcc lattice. Thus carbon atoms touch each 28. The interplanar spacing (in picometers) will be the maximum
other at body diagonal (line at which tetrahedral void when the X-ray beam having wavelength 141 pm is diffracted
exists). at an angle of (Assume that n = 1, sin 20° = 0.34202, sin 27.4°
= 0.4607, sin 35.8° = 0.5826)
25. The unit cell edge in sodium chloride has a length of 564.0 pm.
(a) 20.0° (b) 27.4°
The sodium ion has a radius of 95 pm. What is the diameter of
(c) 35.8° (d) cannot be predicted.
a chloride ion?
(a) 812.3 pm (b) 468.0 pm Solution
(c) 253.0 pm (d) 374.0 pm (a) From Bragg’s equation, nl = 2dsinq, we have
Solution 1× 141 pm
At 20.0°, d = = 206 pm
(d) Edge length = Diameter of cation + Diameter of anion 2 × sin 20.0°
564 = (2 × 95) + Diameter Cl 1× 141 pm
Diameter Cl = 564 − (2 × 95) = 374 pm At 27.4°, d = = 153 pm
2 × sin 27.4°
26. KCl crystallizes in the same type of lattice as does NaCl and 1× 141 pm
CsF in the CsCl type of structure. The molar mass of CsF is At 35.8°, d = = 121 pm
2 × sin 35.8°
twice that of KCl and the a value for KCl is 1.5 times than that
of CsF. Calculate the ratio of the density of CsF to that of KCl. 29. X-rays of wavelength 0.154 nm striking an aluminium crystal,
(a) 2.05 (b) 1.69 (c) 3.2 (b) 4.51 get diffracted at an angle 19.3° showing first-order diffrac-
tion. Calculate the spacing between the plane of aluminium
Solution
atoms in pm.
(b) Since, there are 4 KCl per unit cell, the relation for KCl is (a) 233 pm (b) 145 pm
a13 74.5 (c) 195 pm (d) 180 pm
× NA =
4 r1
Solution
Since there is 1 CsF per unit cell, the relation for CsF is
(a) According to Bragg’s equation, n l = 2d sinq where
a23 2 × 74.5 n = order of diffraction, l = wavelength, q = angle of dif-
× NA =
4 r2 fraction. Given that n = 1, l = 0.154 nm = 154 pm and
a13 1 r2 q = 19.3°. Therefore,
Dividing the above two expressions, we get = ×
4 a23 2 r1 nl 1× 154 154
d= = = = 233 pm
(1.5)3 a23 1 r2 r (1.5) 3 2 sinq 2 sin19.3° 2 × 0.33
Now, a1 = 1.5a2, so we have = × ⇒ 2= × 2 = 1.69
4 × a23 2 r1 r1 4
Point Defects in Crystals
(1.5)3 a23
1 r r (1.5)3
3
= × 2⇒ 2= × 2 = 1.69 30. The appearance of colour in solid alkali metal halides is gener-
4 × a2 2 r1 r1 4 ally due to
27. Each rubidium halide crystallizing in the NaCl type lattice has (a) Schottky defect (b) Frenkel defect
a unit cell length 30 pm greater than for corresponding potas- (c) Interstitial position (d) F-centres
sium salt (rK + = 133 pm) of the same halogen. What is the ionic (JEE Main Online 2014)
radius of Rb+?
Solution
(a) 1265.3 pm (b) 1056.9 pm
(c) 985.6 pm (d) 143.58 pm (d) F-centre is a type crystallographic defect in which anionic
valency in a crystal is filled by one or more electrons.
Solution
31. What type of crystal defect is indicated in the diagram below?
(d) For NaCl type crystal (fcc), r = a/2(2)1/2. Given that
aRb+ = 30 + aK + and rK + = 133 ppm. Na+ Cl− Na+ Cl− Na+ Cl−
Now, we know that KCl also crystallizes in fcc structure, Cl− Cl− Na+ Na+
hence
a + Na+ Cl− Cl− Na+ Cl−
rK + = K ⇒ aK + = rK + × 2 2 = 133 × 2 2 = 376.181 pm
2 2 Cl− Na+ Cl− Na+ Na+

Chapter-16.indd 529 8/5/2016 10:38:09 AM


530 Chapter 16 Solid State

(a) Frenkel defect Solution


(b) Frenkel and Schottky defects (d) Schottky defects arise when one positive ion and one
(c) Interstitial defect negative ion are missing from their respective positions
(d) Schottky defect leaving behind a pair of holes. These are more common
Solution in ionic compounds with high coordination number and
having almost similar size of cations and anions.
(d) Schottky defect. This is because equal number of cations
and anions are missing from their lattice sites. 34. Which of the following statement is not correct about the
defects in crystals?
32. Which of the following statements is correct about the defects
(a) Schottky defects occur in metal as well as ionic crystals
in the crystals?
and decreases the density of crystal.
(a) Schottky defects are observed in the compounds in
(b) Frenkel defect increases density of crystal because it con-
which there is a large difference in the size of cation and
sists of the interstitial atom or ion.
anion, whereas Frenkel is observed when cation and
(c) In crystals of silver halides, high diffusivity of silver ions
anion are approximately of the same size.
cause Frenkel defects.
(b) Schottky defects disturb the ratio of cations and anions,
(d) Schottky defect changes molecular formula of ionic solids.
whereas Frenkel defects maintain the ratio.
(c) Schottky defects lead to lowering in density, whereas Solution
Frenkel defects do not affect the density. (b) In real crystals, a small fraction of the normal atom sites
(d) Both interstitial defects and Frenkel defects cause remain unoccupied. Such vacancies are called Schottky
increase in the density of solid. defects. It causes lower density of solid.
Solution In Frenkel defect, atoms or ions are displaced from their
regular lattice sites to interstitial sites, and the crystal
(c) Schottky defect is observed in ionic solids in which cati-
defect consists of the lattice vacancy plus the interstitial
onic and anionic sizes are approximately the same. It arises
atom or ion. Thus it does not affect the density of the
when equal number of cations and anions are missing
crystal.
from their position. It causes decrease in the density of
The silver halides are examples of crystals in which Frenkel
solid.
disorder is extreme. The crystal structures of these com-
Frenkel defect is observed in ionic solids in which cati-
pounds are established primarily by the anion lattice, and
onic and anionic size difference is large. It arises when
the silver ions occupy highly disordered, almost random,
the smaller ion leaves its original place and moves into
sites. The rate of diffusion of silver ions in these solids is
interstitial spaces. This causes no change in cation–anion
exceptionally high.
ratio, and hence no change in the density.
Interstitial defect arises when a constituting particle is 35. Frenkel defect is noticed in
also placed into interstitial space. This causes an increase (a) AgBr (b) ZnS
in density. (c) AgI (d) All of these.
33. Any deviation from the perfectly ordered arrangement con- Solution
stitutes a defect, and these defects can be classified based on (d) Frenkel defect is shown by ionic solids with large size differ-
imperfection. In a crystal, both the ions are missing from nor- ence between the anion and cation (with the cation usually
mal sites in equal number. This indicates smaller due to an increased effective nuclear charge). ZnS,
(a) F-centre (b) interstitial defect AgCl, AgBr, AgI all display this defect due to the smaller size
(c) Frenkel defect (d) Schottky defect of Zn2+ and Ag+ ions compared to the anions.

ADVANCED LEVEL PROBLEMS


1. The edge length of unit cell of a metal having molecular
weight 75 g mol−1 is 5 Å which crystallizes in cubic lattice. Thus, the metal has a bcc structure for which
If the density is 2 g cm−3 then find the radius of metal atom. 3a 1.732 × 5 3a 1.732 × 5
(NA = 6.023 × 1023). Give the answer in pm. r= = r = = 2.=165 = 216.5=pm Å = 216.5 pm
2.165
4 4 4 4
Solution
z×M 2. A metallic element crystallizes into a lattice containing a
The density is given by r = 3 A sequence of layers ABABAB…. Any packing of spheres leaves
Therefore, a × NA
out voids in the lattice. What percentage by volume of this
N lattice is empty space?
z = r × a3 × A
MA Solution
(6.023 × 1023 mol−1) The sequence ABABAB… indicates hcp unit cell. Now,
= 2 g cm−3 × (5 × 10 −8 cm)3 ×
75 g mol−1 there are 6 atoms per unit cell and volume of the unit cell
=2 is 24 2r 3. Thus, the packing fraction is

Chapter-16.indd 530 8/5/2016 10:38:11 AM


Advanced Level Problems 531

Volume of 6 spheres in 4 Solution


6 × p r3
the unit cell p
Packing efficiency = × 100 = 3
3
× 100 = 12 × 6×(corners)
(b) 100 = 74%+ 2 × (1/2) (at face centres) + 3 (in the
Total volume of the 24 2r 3body
× 2 centre) =6
unit cell
Volume of 6 spheres in 4 5b. The volume of this hcp unit cell is
6 × p r3 64 3
the unit cell 3 p (a) 24 2r 3 (b) 16 2r 3 (c) 12 2r 3 (d) r
cking efficiency = × 100 = × 100 = × 100 = 74%
Total volume of the 24 2r 3
3× 2 3 3
unit cell Solution
Hence, the percentage of empty space = 100 − 74 = 26% (a) Volume of unit cell = base × height
3. Match the crystal system/unit cells mentioned in Column I with 3
Base area of regular hexagon = 6 × × (2r )2 = 6 3 r 2
their characteristic features mentioned in Column II. 4
2
Column I Column II Volume of unit cell = 6 3 r 2 × 4 r = 24 r 3 2
3
(a) Primitive cubic and (p) Have these cell parame-
face-centred cubic ters a = b = c and a = b = g 5c. The empty space in this hcp unit cell is
(a) 74% (b) 47.6% (c) 32% (d) 26%
(b) Cubic and rhombohedral (q) Are two crystal systems
Solution
(c) Cubic and tetragonal (r) Have only two crystallo-
graphic angles of 90° (d) Volume occupied in hcp arrangement = 74%, so the
(d) Hexagonal and mono- (s) Belong to the same crys- empty space is 26%.
clinic tal system 6. The correct statement(s) regarding defects in solids is (are)
(IIT-JEE 2007) (a) Frenkel defect is usually favoured by a very small differ-
Solution ence in the sizes of cation and anion.
(a) ã p, s; (b) ã p, q; (c) ã q; (d) ã q, r. (b) Frenkel defect is a dislocation defect.
(c) Trapping of an electron in the lattice leads to the forma-
4. Statement 1: Band gap in germanium is small. tion of F-centre.
Statement 2: The energy spread of each germanium atomic (d) Schottky defects have no effect on the physical proper-
energy level is infinitesimally small. ties of solids. (IIT-JEE 2009)
(a) Statement 1 is True, Statement 2 is True; Statement 2 is a
Solution
correct explanation for Statement 1
(b) Statement 1 is True, Statement 2 is True; Statement 2 is (b), (c) Frenkel defects occur in compounds in which the
NOT a correct explanation for Statement 1 anions are much larger in size than cations. It is a dislocation
(c) Statement 1 is True, Statement 2 is False effect. The presence of Schottky defects lowers the density
(d) Statement 1 is False, Statement 2 is True of the crystal (means it affects the physical properties of the
(IIT-JEE 2007) crystal). In the metal excess defects, the electron is trapped in
Solution anion vacancies are known as F-centres.
(c) A semiconductor has only small difference in energy 7. The coordination number of Al in the crystalline state of AlCl3
between the filled valence band and empty conduction is ___.
band. The energy gap between valence and conduction (IIT-JEE 2009)
band for germanium is 68 kJ mol−1 at room temperature. Solution
In order to increase its conduction, doping is done.
(6) Coordination number of Al is 6. It exists in ccp lattice with
5. Read the following paragraph and answer the questions that 6 coordinate layer structure.
follow:
8. The packing efficiency of the two-dimensional square unit cell
Paragraph for Questions 5a–5c: In hexagonal systems of crystals, shown in below figure is ___.
a frequently encountered arrangement of atoms is described as a
hexagonal prism. Here, the top and bottom of the cell are regu-
lar hexagons and three atoms are sandwiched in between them.
A space-filling model of this structure called hexagonal closed
L
packed (hcp) is constituted of a sphere on a flat surface surrounded
in the same plane by six identical spheres as closely as possible. (a) 39.27% (b) 68.02% (c) 74.05% (d) 78.54%
Three spheres are then placed over the first layer so that they touch (IIT-JEE 2010)
each other and represent the second layer. Finally, the second layer Solution
is covered with a third layer that is identical to the bottom layer in (d) For the given unit cell, edge length
relative position. Assume radius of every sphere to be r.
a= 2 2r
(IIT-JEE 2008)
Hence the packing efficiency
5a. The number of atoms in this hcp unit cell is ___ 2 × p r 2 2p r 2 p
(a) 4 (b) 6 (c) 12 (d) 17 = = = = 78.5%
[2 2 r ]2 8 r2 4

Chapter-16.indd 531 8/5/2016 10:38:14 AM


532 Chapter 16 Solid State

9. The number of hexagonal faces that are present in a truncated (a) 104 pm (b) 125 pm (c) 183 pm (d) 57 pm
octahedron is ___. (JEE Advanced 2013)
(IIT-JEE 2011) Solution
Solution (a) From the figure, it can be seen that the cation A+ occupies
(8) This can be seen from the figure below. octahedral void formed by the anion X−. The radius ratio
for an octahedral void is rA+ /rX − = 0.414. Now, given that
the radius of anion X− is 250 pm. So the radius of A+ is
rA+ = 0.414 × 250 = 103.50  104 pm
12. If the unit cell of a material has cubic close packed (ccp) array
of oxygen atoms with m fraction of octahedral holes occu-
pied by aluminum ions and n fraction of tetrahedral holes
occupied by magnesium ions, m and n, respectively, are
10. A compound MpXq has cubic close packing (ccp) arrangement 1 1 1 1 1 1 1
(a) , (b) 1, (c) , (d) ,
of X. Its unit cell structure is shown in the following figure. The 2 8 4 2 2 4 8
empirical formula of the compound is (JEE Advanced 2015)
Solution
(a) For ccp lattice, number of atoms (z) = 4
Number of octahedral and tetrahedral holes is 4 and 8
M=
respectively. From the balance of charge on the salt, we
X= have 4 O2− equivalent of −8 charge, which is balanced by
2 Al3+ and 1 Mg2+ or a charge of +8. Therefore, the for-
mula is Al2MgO4.
1
Al3+ is present in of the octahedral holes.
2
1
Mg2+ are present in of the tetrahedral holes.
8
(a) MX (b) MX2 (c) M2X (d) M5X14
13. Sodium metal crystallizes in body-centred cubic lattice with
(IIT-JEE 2012)
the cell edge, a = 4.29 Å. What is the radius of sodium atom?
Solution
Solution
(b) As X is present at the corners of the cube and also at the
In a bcc arrangement, we have
faces of the cube, so the contribution of X will be
1 3a 3 × 4.29
8 (corners) × (contribution of each corner atom) r= = = 1.86 Å
8 4 4
1 14. The figure below shows the location of atoms in three crys-
+ 6 ( faces ) × (contribution of each face ) = 4
2 tallography planes in an fcc lattice. Draw the unit cell for the
corresponding structure and identify these planes in your
As M is present at the body centre and at four edges, so its
diagram.
contribution will be
1
4 (edges) × (contribution of each edge)
4
+ 1(body centre ) = 2
So, the unit cell formula becomes M2X4 and the empirical
Solution
formula will be MX2.
The unit cells are shown in the following figures, and the
11. The arrangement of X− ions around A+ ion in solid AX is given shaded portions represent the planes.
in the figure (not drawn to scale). If the radius of X− is 250 pm,
the radius of A+ is

X−

15. The crystal AB (rock salt structure) has molecular weight


A− 6.023y amu where y is an arbitrary number in amu. If the min-
imum distance between cation and anion is y1/3 nm and the
observed density is 20 kg m−3. Find the (a) density in kg m−3
and (b) type of defect.

Chapter-16.indd 532 8/5/2016 10:38:20 AM


Advanced Level Problems 533

Solution MA × z 58.5 × 4
r= =
(a) The edge length of unit cell, a = 2 × y1/ 3 nm. NA a 3
6.023 × 10 23
× (0.564 × 10 −7 )3
Given that M = 6.023y g mol−1 = 6.023y × 10−3 g mol−1. = 2.16 g cm−3
For rock salt structure, z = 4. Therefore, density is
20. CsBr has bcc structure with edge length 4.3 Å. The shortest
M ( 4 × 6.023 × 10 −3 y kg mol−1) interionic distance in between Cs+ and Br− is
r=z× =
a 3 × NA (2 × y1/ 3 × 10 −9 m)3 × 6.023 × 1023 mol−1 (a) 3.72 Å (b) 1.86 Å (c) 7.44 Å (d) 4.3 Å
= 5 kg m−3 Solution
(b) As the observed density > calculated density, it means  31/ 2   31/ 2 
(a) d =  a=  × 4.3 = 3.7238
that some impurity or extra metal ion is present in inter-  2   2 
stitial sites. Hence, the defect is interstitial defect or metal
excess defect. 21. The coordination number of a metal crystallizing in a hexago-
nal close-packed structure is
16. An element crystallizes in fcc lattice having edge length (a) 12 (b) 4 (c) 8 (d) 6
400 pm. Calculate the maximum diameter of atom which can
be placed in interstitial site without distorting the structure. Solution
Solution (a) Three spheres above, three below, and total six in that,
makes it 12.
In face-centred cubic arrangement (fcc), the interstitial
sites will be octahedral and tetrahedral voids. 22. In a solid AB having the NaCl structure, A atoms occupy the
For octahedral voids, r1/r2 = 0.414 corners of the cubic unit cell. If all the face-centred atoms
(radius ratio for coordination number 6) along one of the axes are removed, then the resultant stoichi-
For tetrahedral voids, r1/r2 = 0.225 ometry of the solid is
(radius ratio for coordination number 4) (a) AB2 (b) A2B (c) A4B3 (d) A3B4
where r1 is the radius of atom in interstitial sites and r2 is Solution
the radius of atom arranged in fcc. Also, 4 r2 = 2a 1 1
For maximum diameter of atom in interstitial site, octa- (d) Number of A atoms = 8 × + 4 × = 3
8 2
hedral sites will be considered (since they are bigger than
1
tetrahedral voids). Then Number of B atoms = 12 × +1 = 4
4
2 × 0.414 × 400 Therefore, the formula is A3B4.
Diameter = 2r1 = 2(0.414 r2 ) = = 117.1 pm
2 2
23. In a compound, the anions (X) form hexagonal close packing
and cations (Y) occupy only one third of octahedral voids. The
17. Which of the following fcc structures contains cations in the
general formula for the compound is
alternate tetrahedral voids?
(a) YX (b) Y2X3 (c) YX3 (d) Y3X
(a) Na2O (b) ZnS (c) CaF2 (d) CaO
Solution
Solution 1 1
(c) The atoms per unit cell = 12 × + 2 × + 3 × 1 = 6.
(b) In ZnS, anions (S2−) are placed in fcc manner and cations 6 2
(Zn2+) are placed in alternate tetrahedral voids. So, the number of anions (X) per unit cell is 6.
Now since it is hexagonal close packing, the number of
18. Which of the following are not true about hexagonal close
octahedral voids in it is just equal to the number of ani-
packing?
ons in it, but only 1/3 of the voids are occupied by cations,
(a) It has a coordination number of 6.
so the number of cations (Y)
(b) It has 26% empty space.
1
(c) It has ABAB ... type of arrangement. = 6× =2
(d) It is as closely packed as body-centred cubic packing. 3
Therefore, formula is Y2X6 YX3
Solution
(a), (d) Hexagonal close packing has coordination number 24. Consider the 2D packing by bigger atoms with radius (R) and
12. Its packing is similar to cubic close packing and not body- small atoms just appropriate to fill the voids. What is the pack-
centred close packing. Its packing efficiency is 74%, so 26% ing efficiency for it?
space is empty and it has ABAB... type of structure.

19. A unit cell of sodium chloride has four formula units. The
edge length of the unit cell is 0.564 nm. What is the density of
sodium chloride?
Solution (a) 78% (b) 92% (c) 52.4% (d) 74%

NaCl has 4 formula units, that is, number of atoms per unit Solution
cell is 4. (b) Considering the unit cell in the figure, we get

Chapter-16.indd 533 8/5/2016 10:38:23 AM


534 Chapter 16 Solid State

where r is radius of small atom and R is the radius of the


bigger atom.
From dimensions of unit cell, 2R + 2r = 2 2R , it implies
r
that = 2 − 1, therefore, r = ( 2 − 1)R and
R
Packing efficiency in 2D packing is the ratio of packed
area to the total available area in the unit cell. Since in a p [R 2 + ( 2 − 1)2 R 2 ]  1 
Packing efficiency = × = p ×  1− 
unit cell, one bigger and one small atom are packed, its 4 R 2  2
p R2 + p r 2 = 0.92 ≈ 92%
Packing efficiency =
(2R )2

PRACTICE EXERCISE
Level I 9. In a crystal, both ions are missing from normal sites in equal
number. This is an example of
Single Correct Choice Type (a) F-centres. (b) interstitial defect.
1. Silver forms face-centred cubic crystals. The atomic radius of a (c) Frenkel defect. (d) Schottky defect.
silver atom is 144 pm. The atoms are in contact along the diag-
10. For a solid with the structure shown in the following figure, the
onal. Calculate the length of an edge of this unit cell.
coordination numbers of the points A and B, respectively, are
(a) 556 pm (b) 325 pm (c) 407 pm (d) 247 pm
2. Total volume of atoms present in a face-centred cubic unit cell
of a metal is (r is atomic radius)
12 3 16 3 20 3 24 3
(a) pr (b) pr (c) pr (d) pr
3 3 3 3
3. Which of the following statements is correct?
(I) The coordination number of cation occupying a tetrahe-
dral hole is 4.
(a) 6, 8 (b) 8, 8 (c) 6, 6 (d) 4, 6
(II) The coordination number of cation occupying a octahe-
dral hole is 6. 11. For tetrahedral coordination, the radius ratio (r+/r−) should be
(III) In Schottky defects, density of the lattice decreases. (a) 0.155 to 0.225 (b) 0.225 to 0.414
(a) I, II (b) II, III (c) I, II, III (d) I, III (c) 0.414 to 0.732 (d) 0.732 to 1.000
4. An element (with atomic mass = 250 u) crystallizes as a simple 12. An alloy of copper, silver, and gold is found to have copper
cubic. If the density of the unit cell is 7.2 g cm−3, what is the forming the simple cubic close-packed lattice. If silver atoms
radius of the element? occupy the face centres and gold atoms are present at the
(a) 1.93 × 10−6 cm (b) 1.93 × 10−8 cm body centre, then the formula of the alloy will be
(c) 1.93 × 10−13 cm (d) 1.93 × 10−12 cm (a) Cu3AgAu (b) CuAg3Au
(c) Cu4Ag2Au (d) CuAgAu
5. The Ca2+ and F− ions are located in CaF2 crystal, respectively, at
face-centred cubic lattice points and in 13. Fraction of total volume occupied by atoms in a simple cube
(a) tetrahedral voids. (b) half of tetrahedral voids. is
(c) octahedral voids. (d) half of octahedral voids. p 3p 2p p
(a) (b) (c) (d)
6. Metallic magnesium has a hexagonal close-packed structure 2 8 6 6
and a density of 1.74 g cm−3. Assuming magnesium atoms to 14. When NaCl is doped with MgCl2, the nature of defect pro-
be spherical, calculate the radius of magnesium atom. (Atomic duced is
mass of Mg = 24.3 u) (a) interstitial. (b) Frenkel.
(a) 1.6 × 10−8 cm (b) 2.5 × 10−8 cm (c) Schottky. (d) impurity defect.
−8
(c) 3.5 × 10 cm (d) 4.5 × 10−8 cm
15. Which of the following fcc structure contains cations in alter-
7. A crystal formula AB3 has A ions at the cube corners and B ions nate tetrahedral voids?
at the edge centres. The coordination numbers of A and B are, (a) NaCl (b) ZnS (c) Na2O (d) CaF2
respectively,
(a) 6 and 6 (b) 2 and 6 (c) 6 and 2 (d) 8 and 8 16. A metal crystallizes in two cubic phases – face-centred cubic
(fcc) and body-centred cubic (bcc) – whose unit cell length
8. Superconductors are derived from the compounds of are 3.5 Å and 3.0 Å, respectively. Calculate the ratio of density
(a) p-block elements. (b) lanthanoids. of fcc and bcc.
(c) actinoids. (d) transition elements. (a) 2.123 (b) 1.259 (c) 5.124 (d) 3.134

Chapter-16.indd 534 8/5/2016 10:38:28 AM


Practice Exercise 535

17. The unit cell that best describe the CsCl crystal lattice is 25. In spinel structure, O2− ions are cubic-closed packed, whereas
(a) bcc unit cell. (b) primitive cubic unit cell. 1/8th of the tetrahedral holes are occupied by A2+ cations and
(c) fcc unit cell. (d) hcp unit cell. 1/2 of the octahedral holes are occupied by cations B3+. The
general formula of this compound is
18. Choose the correct matching sequence from the possibilities
(a) A2BO4 (b) AB2O4 (c) A2B4O (d) A4B2O
given
(1) Crystal defect (p) AB AB AB type crystal 26. Aluminium crystallizes in ccp structure. Metallic radius of alu-
minium is 125 pm. Edge length of the unit cell of aluminium is
(2) hcp (q) Covalent crystal (a) 250 pm. (b) 353.5 pm.
(3) CsCl (r) Frenkel (c) 176 pm. (d) 216.5 pm.
(4) Diamond (s) Face-centred in cube 27. Ice crystallizes in a hexagonal lattice having a volume of the
(5) NaCl (t) Body-centred in cube unit cell as 132 × 10−24 cm3. If density of ice at the given tem-
perature is 0.92 g cm−3, then the number of H2O molecules
(1) (2) (3) (4) (5) per unit cell is
(a) (r) (p) (q) (t) (s) (a) 1 (b) 2 (c) 3 (d) 4
(b) (r) (p) (t) (q) (s) 28. If a be the edge length of the unit cell and r be the radius of an
(c) (r) (t) (p) (q) (s) atom, then for fcc arrangement, the correct relation is
(a) 4 a = 3r (b) 4 r = 3a
(d) (t) (p) (s) (q) (p)
(c) 4 r = 2a (d) 4 r = a / 2
19. In a tetragonal crystal
(a) a = b = g ≠ 90°; a = b = c 29. At a temperature of absolute zero, an intrinsic semiconductor
(b) a = b = g = 90°; a = b ≠ c is
(c) a = b = g = 90°; a ≠ b ≠ c (a) an insulator. (b) a p-type semiconductor.
(d) a = b = 90°, g = 120°; a = b ≠ c (c) a n-type semiconductor. (d) a conductor.

20. Xenon crystallizes in face-centred cubic lattice and edge of 30. The packing fraction for a body-centred cube is
the unit cell is 620 pm, then the radius of xenon atom is (a) 0.42 (b) 0.54 (c) 0.68 (d) 0.74
(a) 219.20 pm (b) 438.5 pm
31. The most unsymmetrical and symmetrical systems are,
(c) 265.5 pm (d) 536.94 pm
respectively
21. The lattice parameters are a = 5.62 Å, b = 7.41 Å, c = 9.48 Å. The (a) tetragonal, cubic. (b) triclinic, cubic.
three coordinates are mutually perpendicular to each other. (c) rhombohedral, hexagonal. (d) orthorhombic, cubic.
The crystal is
32. The interionic distance for caesium chloride crystal will be
(a) tetragonal. (b) orthorhombic.
(c) monoclinic. (d) trigonal. a 3a 2a
(a) a (b) (c) (d)
2 2 3
22. In diamond, each carbon atom is bonded to four other carbon
atoms tetrahedrally. The number of carbon atoms per unit cell 33. In an fcc arrangement of P and Q atoms, P atoms are at the
is corners of the unit cell, Q atoms at the face centres and two
atoms are missing from two corners in each unit cell, then the
(a) 4 (b) 6 (c) 8 (d) 12
formula of the compound is
23. Bragg’s equation is (a) P2Q3 (b) P4Q (c) P4Q5 (d) PQ4
(a) nl = 2q sinq (b) nl = 2d sinq
34. The radius of a divalent cation A2+ is 94 pm and of divalent
(c) 2nl = d sinq (d) l = 2dnsinq anion B2− is 146 pm. The compound AB has
24. In the following structure, the sites S1 and S2 represent (a) rock salt structure.
(b) zinc blende structure.
(c) antifluorite structure.
S2 (d) caesium chloride like structure.
S1 x 35. The ratio of Fe3+ and Fe2+ ions in Fe0.9S1.0 is
(a) 0.28 (b) 0.5 (c) 2 (d) 4
36. An example of body-centred cube is
(a) sodium. (b) magnesium.
(c) zinc. (d) copper.
(a) both octahedral voids. 37. Iron crystallizes in a body-centred cubic structure. The radius
(b) both tetrahedral voids. of Fe atom (if edge length of unit cell is 286 pm) is
(c) S1 – octahedral void, S2 – tetrahedral void. (a) 120.9 pm (b) 123.8 pm
(d) S1 – tetrahedral void, S2 – octahedral void. (c) 23.8 pm (d) 223.8 pm

Chapter-16.indd 535 8/5/2016 10:38:31 AM


536 Chapter 16 Solid State

Level II (c) It has rock-salt type structure.


(d) It can have Frenkel defects.
Multiple Correct Choice Type
47. Which of the following statements are true?
38. Diamond is
(a) An element with bcc structure has two atoms per unit
(a) a covalent solid. (b) non-conductor.
cell.
(c) soft and slippery. (d) sp2 hybridized.
(b) An ionic compound A+B− with bcc structure has one AB
39. Which of the following statements are correct? formula unit per unit cell.
(a) The coordination number of each type of ion in CsCl crys- (c) The shape of the octahedral void is octahedral.
tal is 8. (d) The edge length of the crystal A+B− is equal to the dis-
(b) A metal that crystallizes in bcc structure has coordination tance between A+ and B− ions.
number of 12.
48. If the radius of Na+ ion is 95 pm and that of Cl− ion is 181 pm,
(c) A unit cell of an ionic crystal shares some of its ions with
then
other unit cells.
(a) coordination number of Na+ is 6.
(d) The length of the edge of unit cell of NaCl is 552 pm
(b) structure of NaCl is octahedral.
(rNa+ = 95 pm, rCl− = 181 pm) .
(c) edge of the unit cell is 552 pm.
40. Ferromagnetic substances among the following are (d) edge of the unit cell is 276 pm.
(a) gold. (b) nickel.
(c) silver. (d) gadolinium. Passage Type

41. In which of the following defects, the cations are present in Paragraph for Questions 49 to 51: Density of a unit cell is the same
interstitial sites? as the density of the substance. So, if the density of the substance is
(a) Frenkel defect known, we can calculate the number of atoms or dimensions of the
(b) Schottky defect unit cell. The density of the unit cell is related to its formula mass
(c) Metal deficient non-stoichometric compound. (M), number of atoms per unit cell (z), edge length (a in cm), and
(d) Metal excess non-stoichiometric compound. Avogadro’s constant NA, as
z×M
42. Pick up the correct statements r= 3
a × NA
(a) The ionic crystal of AgBr has Schottky defect.
(b) The unit cell having crystal parameters, a = b ≠ c, 49. An element X crystallizes in a structure having an fcc unit cell
a = b = 90°,g = 120° is hexagonal. of an edge 100 pm. If 24 g of the element contains 24 × 1023
(c) In ionic compounds having Frenkel defect, the ratio atoms, the density is
r+/r− is high. (a) 2.40 g cm−3 (b) 40 g cm−3
(c) 4 g cm −3 (d) 24 g cm−3
(d) The coordination number of Na+ ion in NaCl is 4.
43. Crystal systems in which no two axial lengths are equal are 50. The number of atoms present in 100 g of a bcc crystal (density
(a) triclinic. (b) orthorhombic. = 12.5 g cm−3) having cell edge 200 pm is
(c) monoclinic. (d) tetragonal. (a) 1 × 1025 (b) 1 × 1024 (c) 2 × 1024 (d) 2 × 1026

44. Which of the following are true? 51. A metal A (atomic mass = 60) has a body-centred cubic crystal
(a) In NaCl crystals, Na+ ions are present in all the octahedral structure. The density of the metal is 4.2 g cm−3. The volume
voids. of unit cell is
(b) In ZnS (Zinc blende), Zn2+ ions are present in alternate (a) 8.2 × 10−23 cm3 (b) 4.74 × 10−23 cm3
(c) 3.86 × 10 cm−23 3 (d) 5.86 × 10−23 cm3
tetrahedral voids.
(c) In CaF2, F− ions occupy all the octahedral voids. Paragraph for Questions 52 to 54: In a primitive cubic unit cell, all
(d) In Na2O, O2− ions occupy half of the octahedral voids. the eight corners of the cube are occupied by the same atoms/ions
45. Select the correct statements: and not found anywhere else in the cube. The number of atoms
(a) The conductance through electrons is called p-type within a unit cell is called the rank of a unit cell. For primitive cubic
conduction. unit cell, the rank (z) is 1. In a bcc, the same atoms/ions are present
(b) The conductance through positive holes is called p-type at all the eight corners of a cube and one atom/ion is also present
conduction. at the centre of the cube. These atoms/ions are not present any-
(c) The conductance through electrons holes is called n-type where else in the cube. The rank of a bcc is 2. In an fcc, the same
conduction. atoms/ions are present at all the corners of the cube and are also
(d) The band gap in germanium is small. present at the centre of each square face. These atoms/ions are not
present anywhere else in the unit cell. The rank of an fcc is 4.
46. The density of KBr is 2.75 g cm−3. The length of the unit cell
is 654 pm. Atomic mass of K = 39, Br = 80. Then, which of the 52. How many unit cells are present in 39 g of potassium that
following statements are true? crystallizes in bcc structure (atomic mass of K = 39 u)?
(a) It has 4 K+ and 4 Br− ions per unit cell. (a) 0.5 NA (b) 0.25 NA (c) NA (d) 0.75 NA
(b) It has a body-centred structure. where NA is Avogadro’s constant.

Chapter-16.indd 536 8/5/2016 10:38:32 AM


Answer Key 537

53. Li crystallizes in bcc. The edge length of unit cell is 351 pm. Column I Column II
What would be radius of Li atom? (a) Schottky defects (p) Excess-cations occupy
(a) 151.98 pm (b) 273 pm interstitial sites
(c) 290 pm (d) 76 pm (b) Frenkel defects (q) Equal number of cations
and anions are missing
54. Sodium crystallizes in bcc lattice. If the length of the edge of from their lattice points
the unit cell is 424 pm, density of sodium is (atomic mass of (c) Metal excess defects (r) Act as p-type semicon-
sodium = 23 u) ductors
(a) 10.4 g cm−3 (b) 1.002 g cm−3 (d) Metal deficient defects (s) Are non-stoichiometric
(c) 50.4 g cm−3 (d) 25.68 g cm−3 defects
Paragraph for Questions 55 to 59: X-ray studies show that the 61. Match the types of solid with their examples/properties.
packing of atoms in a crystal of a metal is found to be in layers such Column I Column II
that starting from any layer, every fourth layer is exactly identical. (a) Molecular solid (p) Dry ice
The density of the metal is found to be 19.4 g cm−3 and its atomic (b) Covalent solid (q) Copper
mass is 197 u. (c) Metallic solid (r) Generally behave as
55. The coordination number of metal atom in the crystal is insulators
(a) 4 (b) 6 (c) 8 (d) 12. (d) Ionic solid (s) Generally have low melt-
ing points
56. The fraction occupied by metal atoms in the crystal is
Integer Type
(a) 0.52 (b) 0.68 (c) 0.74 (d) 1.0
62. In hexagonal close packing, the difference in the number of
57. The approximate number of unit cells present in 1 g of metal is tetrahedral and octahedral voids in a unit cell is ___.
(a) 3.06 × 1021 (b) 1.53 × 1021
(c) 3.82 × 10 20 (d) 7.64 × 1020 63. Atoms of element A form hcp arrangement and those of ele-
ment B occupy 2/3rd of tetrahedral voids. The total number of
58. The length of the edge of the unit cell will be A and B per formula unit is ___.
(a) 407 pm. (b) 189 pm. (c) 814 pm. (d) 204 pm.
64. The radius ratio of an ionic solid r+/r− is 0.524. The coordina-
59. Assuming the metal atom to be spherical, its radius will be tion number of this type of structure is ___.
(a) 103.5 pm. (b) 143.9 pm.
(c) 146.5 pm. (d) 267.8 pm. 65. Iron(II) oxide has a cubic structure and each unit cell has side
500 pm. If the density of the oxide is 4 g cm−3, the number of
Matrix-Match Type oxide ions present in each unit cell is ___. (Molar mass of FeO
= 72 g mol−1, NA = 6.02 × 1023 mol−1).
60. Match the imperfection in solids with the characteristic
features.

ANSWER KEY
Level I
1. (c) 2. (b) 3. (c) 4. (b) 5. (a) 6. (a)
7. (c) 8. (a) 9. (d) 10. (c) 11. (b) 12. (b)
13. (d) 14. (c) 15. (b) 16. (b) 17. (a) 18. (b)
19. (b) 20. (a) 21. (b) 22. (c) 23. (b) 24. (d)
25. (b) 26. (b) 27. (d) 28. (c) 29. (a) 30. (c)
31. (b) 32. (c) 33. (d) 34. (a) 35. (a) 36. (a)
37. (b)

Level II
38. (a), (b) 39. (a), (c), (d) 40. (b), (d) 41. (a), (d) 42. (b), (c) 43. (a), (b), (c)
44. (a), (b) 45. (b), (c), (d) 46. (a), (c) 47. (a), (b) 48. (a), (b), (c) 49. (b)
50. (c) 51. (b) 52. (a) 53. (a) 54. (a) 55. (d)
56. (c) 57. (d) 58. (a) 59. (b)
60. (a) → q; (b) → p, q; (c) → p, s; (d) → r, s 61. (a) → p, r, s; (b) → r; (c) → (q); (d) → (r)
62. (6) 63. (7) 64. (6) 65. (4)

Chapter-16.indd 537 8/5/2016 10:38:32 AM


538 Chapter 16 Solid State

HINTS AND EXPLANATIONS


Level I
Single Correct Choice Type B+

1. (c) Edge length for fcc


a = r×2 2 A+
= 144 × 2 × 1414
. = 4 07.23 pm
Coordination
4 no. of A = 6
2. (b) Volume of a sphere = pr3
3 Now coordination number of A is 6 as shown, so of B is 2.
In fcc, number of atoms present per unit cell = 4
8. (a) Superconductors offer no resistance to the flow of cur-
4 16
So, total volume = 4 × p r3 = p r3 rent. They are derived from p-block elements as they
3 3 become superconducting at room temperature.
3. (c) The tetrahedral holes are surrounded by 4 nearest
9. (d) Conceptual.
neighbours. So, the coordination number of cation
occupying tetrahedral hole is 4. Since octahedral hole 10. (c) In fcc arrangement, the coordination number of each
is surrounded by six nearest neighbours. Therefore, CN sphere is 6.
of cation occupying octahedral void is 6. In Schottky a
pair of anion and cation leaves the lattice. Thus, the den- 11. (b) For tetrahedral void, the radius ratio should be 0.225 to
sity of lattice decreases. 0.414 and the coordination number is 4. For example, in
CuCl, CuBr, etc.
Z ×M 1
4. (b) The expression for density is d = 12. (b) Contribution of Cu = 8 × (corners) = 1
a 3 × NA 8
Now, Z = 1 for simple cubic, so 1
1× 250 Contribution of Ag = 6 × (faces) = 3
a3 = ⇒ a = 3.863 × 10 −8 cm 2
7.2 × 6.023 × 1023 Contribution of Au = 1 (body centre) = 1
For simple cubic system, a = 2r. Therefore, Therefore, the formula is CuAg3Au.
a 3.863 × 10 −8 13. (d) In a simple cubic system, a = 2r. Therefore,
r= = = 1.931× 10 −8 cm
2 2
Volume occupied by one atom
Packing fraction =
5. (a) In CaF2 crystal, there are 4 Ca2+ ions and there are 8 tetra- Volume of unit cell
hedral voids all of them are occupied by F− ions. 4 3 4 3
pr pr
p
6. (a) Mass of 1 cm3 of Mg = volume × density = 1 × 1.74 = 1.74 g = 3 3 = 3 3 =
a ( 2r ) 6
6.02 × 1023 × 174
.
Number of atoms in 1.74 g of Mg = 1022 (c)
= 4.31×14. atoms
When small amount of MgCl2 is added to NaCl, come of
24.3
the Mg2+ ions replace Na+ ions and occupy the sites ear-
6.02 × 1023 × 174 . lier occupied by Na+. To maintain electrical neutrality of
= 4.31× 1022 atoms the crystal one more Na+ leaves the lattice, thus generat-
24.3
ing, cation vacancy equal to number of Ca2+ ions added.
Volume occupied by Mg = 74.1% (for ccp) = 0.741 cm 3
Thus Schottky defect is produced.
So, 4.31 × 1022 atoms will have volume = 0.741 cm3 15. (b) In ZnS, the S2− ions form fcc arrangement while Zn2+
0.741 −23 3 ions are surrounded tetrahedrally by four S2− ions,
Volume occupied by 1 atom = = 172
. × 10 cm
4.31× 1022 respectively.
Let r be the radius of Mg atom, then
n × Mm
4 3 3 1 16. (b) r=
p r = 1.72 × 10 −23 ⇒ r 3 = 1.72 × 10 −23 × × NA × a3
3 4 3.14
For face-centred cubic cell n = 4, a = 3.5 Å
⇒ r = ( 4.108 × 10 −24 )1/ 3 = 1.6 × 10 −8 cm
4 × Mm
r fcc = (1)
Coordination number of cation Charge of cation NA × (3.5)3
7. (c) =
Coordination number of anion Charge of anion For bcc lattice, N = 2, a = 3.0 Å
3 2 × Mm
= =3 r bcc = (2)
1 N × (3.0 )3
A

Chapter-16.indd 538 8/5/2016 10:38:38 AM


Hints and Explanations 539

1 3
From Eqs. (1) and (2), we get 33. (d) Contribution of P = 6 × (as two atoms are missing) =
8 4
rfcc 4 33 4 ×3×3×3 1
= × = = 1.259 Contribution of Q = 6 × =3
rbcc 2 (3.5)3 2 × 3.5 × 3.5 × 3.5 2
17. (a) As in CsCl, Cs+ is present at the body centre and the Cl− Therefore the formula is P : Q = (3/4) : 3 = PQ4
ions are present at the corners of the cube.
34. (a) The radius ratio is found as r+/r− = 94/146 = 0.64. This
18. (b) Conceptual means AB has octahedral structural arrangement or rock
salt structure.
19. (b) Tetragonal system has a = b ≠ c and a = b = g = 90°, for
example, in SnO2, TiO2. 35. (a) Let the number of Fe2+ ions present be x. Then Fe3+
= 90 − x
20. (a) For the fcc lattice, 4 r = 2 × a Charge on Fe2+ and Fe3+ should be equal to 100 S2− ions
where a = 620 pm, as face diagonal atoms touch each
So, 2 x + 3(90 − x ) = 2 × 100 ⇒ x = 70
other.
Therefore, number of Fe2+ = 70 and Fe3+ = 20
1 1
r= ×a= × 620 pm = 219.20 pm Their ratio is Fe3+/Fe2+ = 20/70 = 0.28
2 2 2 2
36. (a) In crystals of Na, the Na atom is present at the body cen-
21. (b) For orthorhombic crystal, a ≠ b ≠ c, a = b = g = 90°.
tre as well as the corners of the unit cell.
22. (c) There are 8 carbon atoms present per unit cell in dia- Mg forms ccp arrangement.
mond, each having a coordination number 4. Zn forms ccp arrangement.
Cu also forms ccp arrangement.
23. (b) Bragg’s equation is nl = 2d sinq
37. (b) Edge length, a = 286 pm
24. (d) Tetrahedral void is the empty space left between four 3 3 1.732 × 286
spheres and octahedral is the void or the empty space For bcc, radius of atom, r =a= × 286 = = 123.8 pm
4 4 4
left between six spheres.
3 3 1.732 × 286
25. (b) Let the number of ions be N. r= a= × 286 = = 123.8 pm
4 4 4
Then number of tetrahedral voids is 2N. 3 3 1.732 × 286
The number of octahedral voids is N. r= a= × 286 = = 123.8 pm (body diagonal)
4 4 4
Then A2+ occupies one-eighth of tetrahedral voids which
means 1/8 × 2N = (1/4)N Level II
B3+ occupies half of octahedral voids which means Multiple Correct Choice Type
(1/2) × N = N/2
Therefore, the formula becomes 1/4N : 1/2N : N = AB2O4 38. (a), (b) Diamond is a covalent solid in which the constitu-
ent particles are held together by covalent bonds which
a
26. (b) For ccp or fcc, r = ⇒ a = r 2 2 . Therefore, makes it hard and non-conductor. In diamond, carbon is sp3
2 2 hybridized.
a = 125 × 2 × 1.414 = 353.5 pm
39. (a), (c), (d) CsCl has bcc arrangement with Cs+ at the body
z×M centre and Cl− ions at the corners of the cube and vice versa.
27. (d) Density is given by r =
a 3 × NA So, the coordination number is 8:8.
For fcc edge length (in NaCl) = 2(r+ + r−) = 2(95 + 181) = 552 pm
Substituting given values, we get
z × 18 40. (b), (d) Ni and Gd are ferromagnetic whereas silver and gold
0.92 = ⇒z=4 are not.
132 × 10 −24 × 6.023 × 1023
41. (a), (d) Frenkel defect arises when an ion goes missing from
28. (c) In fcc arrangement
its lattice site and occupies an interstitial site. Here excess cat-
2a = 4 r ions occupy interstitial sites.
Metal excess defect (non-stoichiometric) arises when an extra
29. (a) Because at absolute zero, there will be a gap between
cation occupies an interstitial site and the electrical neutral-
the valence band and the conduction band, so electrons
ity is maintained by an electron present in another interstitial
cannot jump into the conduction band and it will thus be
site.
an insulator.
(8/3)p r3 42. (b), (c) The ionic crystals of AgBr shows both Schottky and
30. (c) For bcc, packing fraction = = 0.68
64r 3 /3(3)1/ 2 Frenkel defects. Generally, Frenkel defect is favoured by crys-
31. (b) Most symmetrical is cubic as all the unit cell parameters tals with large difference between the sizes of positive and
are equal, that is, a = b = c and a = b = g = 90°. negative ions (i.e. r+ /r− is high). The unit cell with given crystal
The most unsymmetrical is triclinic as all the parameters parameter is hexagonal. The coordination number of Na+ in
are unequal a ≠ b = c and a ≠ b ≠ g = 90°. NaCl is 6.

32. (c) As CsCl has bcc structure, the interionic distance 43. (a), (b), (c) For triclinic a ≠ b ≠ c; for orthorhombic a ≠ b ≠ c; for
a monoclinic a ≠ b ≠ c and for tetragonal a = b ≠ c
r= 3× .
2

Chapter-16.indd 539 8/5/2016 10:38:43 AM


540 Chapter 16 Solid State

44. (a), (b) In NaCl crystal, there are 4 Na+ and 4 Cl− ions present z×M
in the crystal. So, there are 4 octahedral voids all of which are 51. (b) The density is given by r = . Therefore,
a 3 × NA
occupied by Na+ ions.
In ZnS, there are 4 S2− ions present in the unit cell so there are 2 × 60
a3 = = 4.74 × 10 −23 cm3
8 tetrahedral voids out of which half of them are occupied by 4.2 × 6.023 × 1023
Zn2+ ions.
In CaF2, there are 4 Ca2+ ions present in the unit cell. So, there 52. (a) 39 g of K = 1 mol, so in a bcc unit cell, number of K atoms
are 8 tetrahedral voids and all of them are occupied by F− ions. =2
In Na2O, each Na+ is surrounded by 4 O2− ions and each O2− Therefore, 1 mol of K is in Na/2 mol unit cells.
ions is surrounded by 8 Na+ ions. The Na+ ions thus occupy
53. (a) Edge length, a = 351 pm in bcc unit cell
half of the tetrahedral voids.
3
45. (b), (c), (d) Conductance through electrons is called n-type 3a = 4 r ; r =a = 151.98 = 152 pm
conduction. 4
z×M 54. (a) Given that a = 424 pm and z = 2, so density is
46. (a), (c) The density is given by r = 3 z×M 2 × 39
a × NA r= =
where M (KBr) = 39 + 80 = 119 u. Substituting values, we get NA × a3 × 10 −30 6.02 × 1023 × ( 424 )3 × 10 −30
z × 119 = 10.4 g cm−3
2.75 = ⇒z=4
(654 ) × 10 −30 × 6.02 × 1023
3
55. (d) As the metal has fcc arrangement, so its coordination
This means that it has fcc arrangement or rock-salt type number is 12.
structure. 56. (c) In fcc arrangement, the packing efficiency is 74%.
47. (a), (b) For bcc, number of atoms present per unit cell is 57. (d) For the metal, 197 g contains 197 × 1023 atoms.
8 × 1/8 (corners) + 1(body centre ) = 2 1 g will contain 6.023 × 1023/197 atoms.
As for fcc arrangement, there are 4 atoms present per
A+B−
with bcc structure, for example, CsCl has bcc arrange- unit cell.
ment with Cs+ at the body centre and Cl− ions at the corners So, the number of unit cells present in 1 g of metal is
of the cube and vice versa which means it has one formula
unit per unit cell. 6.023 × 1023
= 7.64 × 1020
r 95 197 × 4
48. (a), (b), (c) + = = 0.525
r− 181 z×M
58. (a) The density is given by r =
which lies in the range 0.414−0.732. Hence, its coordination a 3 × NA
number is 6 and structure is octahedral.
4 × 197
= 22r
Length of the unit cell rN == 2r (rNa+ + rCl− ) = 2(95 + 181) = 552 pm Therefore, a3 = ⇒ a = 407 pm
6.02 × 1023 × 10 −30 × 19.4

Passage Type a
59. (b) For fcc, r = = 0.3535 × 4 07 = 143.9 pm
49. (b) Given that 24 × 1023 atoms of the element has mass 24 g. 2 2
So, 6.023 × 1023 atoms will have mass
Matrix-Match Type
24 × 6.023 × 1023
M= = 6.02 g 60. (a) ã q; (b) ã p; (c) ã p, s; (d) ã r, s
24 × 1023
The density is given by In Schottky defect, equal number of cations and anions are
missing from their lattice points.
z×M 4 × 6.02 Frenkel defect arises when an ion goes missing from its lat-
r= =
a3 × N A 100 × 100 × 100 × 10 −30 × 6.023 × 1023 tice site and occupies an interstitial site. Here excess cations
= 40 g cm−3 occupy interstitial sites.
Metal excess defect is a type of non-stoichiometric defect as
z×M
50. (c) The density is given by r = . Therefore, the ratio of cation to anion differs from that in normal chemi-
a 3 × NA cal formula. Metal excess defect is caused due to anion vacan-
2×M cies and is similar to Schottky defect, here also conduction is
12.5 = ⇒ M = 30.1 g due to free electrons.
200 × 200 × 200 × 10 − 30 × 6.02 × 1023
Metal deficient defect, is also a type of non-stoichiometric
Now, 30.1 g contains 6.023 × 1023 atoms. So, 100 g will defect which occurs when a cation goes missing from a lattice
site and a similar cation with greater charge occupies adja-
6.023 × 1023 × 100 cent site. Here the conduction is due to positive holes, so it
contain = 2 × 1024 atoms
30.1 acts as a p-type semiconductor.

Chapter-16.indd 540 8/5/2016 10:38:48 AM


Hints and Explanations 541

61. (a) ã p, r, s; (b) ã r; (c) ã (q); (d) ã (r) 63. (7) Let number of ions of element A = N
Dry ice is a molecular solid (non-polar) in which the constitu- Then number of ions of element B = 2 × 2/3N
ent molecules are held together by London dispersion forces The ratio of number of ions of A : B = 1 : 4/3
and is soft with low melting point. Therefore the formula is A3B4 and the total number of ions of
Covalent solids are those in which the constituent parti- A and B per formula unit is 3 + 4 = 7.
cles are held together by covalent bonds; they are hard and
64. (6) The ratio 0.524 lies in the range of 0.414−0.732 which
behave as insulators. For example, diamond.
means the geometry should be octahedral with coordination
Metallic solids are those in which the constituent particles are
held together by metallic bonds; they are good conductors number of 6.
with high melting points. For example, Cu. z×M
65. (4) The density is given by r = 3 . Therefore,
Ionic solids are those in which the constituent ions are held a × NA
together by columbic or electrostatic forces of attraction. z × 72
They are insulators but conduct electricity in molten state due 4= ⇒z=4
to the presence of free electron. 500 × 500 × 500 × 10 −30 × 6.023 × 1023

Integer Type

62. (6) In hcp, the number of atoms present = 6 = number of octa-


hedral voids.
Number of tetrahedral voids = 2 × 6 = 12
So, the difference in both the voids is 12 − 6 = 6

Chapter-16.indd 541 8/5/2016 10:38:49 AM


542 Chapter 16 Solid State

SOLVED JEE 2016 QUESTIONS


Volume of four spheres in the unit cell
JEE Advanced 2016 Packing fraction =
e unit cell
Total volume of the
1. The CORRECT statement(s) for cubic close packed (ccp) three 4 3
pr
p
dimensional structure is (are) =4× 3 3
= = 0.74 = 74%
(a) The number of the nearest neighbours of an atom present 16 2r 3 2
in the topmost layer is 12.
(b) The efficiency of atom packing is 74%. In fcc unit cell, the effective number of atoms is:
(c) The number of octahedral and tetrahedral voids per atom One atom at each corner = 8 corner atoms × (1/8) = 1
are 1 and 2, respectively. Atoms at each of the six face centers: 6 face centered
(d) The unit cell edge length is 2 2 times the radius of the atoms × 1/2 = 3
atom. Total number of atoms = 4
Number of octahedral void = 4
Solution Number of tetrahedral void = 8
(b), (c), (d) Coordination number cannot be 12, for any atom in Therefore, per atom there is one octahedral void and two
the topmost layer, as there is no layer above it. Thus, each atom is tetrahedral voids.
in contact with six atoms in the same layer and three atoms from In fcc (or ccp), the unit edge length is
the layer below it. 4r
For cubic close packing, 2a = 4 r ⇒ a = = 2 2r
2

Chapter-16.indd 542 8/5/2016 10:38:50 AM


17 Solutions

Question Distribution in JEE (Main and Advanced)

3
No. of Questions

JEE (Main)
2
JEE (Adv)

0
2016 2015 2014 2013 2012 2011 2010 2009 2008 2007

Concept Distribution in JEE (Main and Advanced)


Topics Covered
Year
JEE (Main) JEE (Advanced)
2007 Vapour Pressure of Ideal Solutions, Colligative Properties Molecular Weight Calculations
of Dilute Solutions
2008 Vapour Pressure of Ideal Solutions Colligative Properties of Dilute Solutions
2009 Vapour Pressure of Ideal Solutions, Real Solutions Solubility
2010 Vapour Pressure of Ideal Solutions, Colligative Properties
of Dilute Solutions
2011 Colligative Properties of Dilute Solutions, Molecular Colligative Properties of Dilute Solutions
Weight Calculations
2012 Colligative Properties of Dilute Solutions Colligative Properties of Dilute Solutions
2013 Vapour Pressure of Ideal Solutions
2014 Vapour Pressure of Pure Solvents, Vapour Pressure of Molecular Weight Calculations
Ideal Solutions, Colligative Properties of Dilute
Solutions
2015 Vapour Pressure of Ideal Solutions, Colligative Properties Colligative Properties of Dilute Solutions
of Dilute Solutions, Molecular Weight Calculations
2016 Colligative Properties of Dilute Solutions, Solubility Vapour Pressure of Ideal Solutions, Molecular Weight
Molecular Weight Calculations Calculations

Chapter-17.indd 543 8/5/2016 10:44:37 AM


544 Chapter 17 Solutions

SUMMARY
1. A solution is a system in which one or more substances are homogeneously mixed or dissolved in another substance.
A simple solution has two components: a solute and a solvent.
The solute is the component that is dissolved or is the least abundant component in the solution. The solvent is the dissolving agent
or the most abundant component in the solution.
2. Type of solutions
(a) Binary solution
When a solution comprises of two constituents, the mixture is called a binary solution.
(b) Complex solution
When a solution contains more than one solute and/or more than one solvent, the mixture is called a complex solution.
The three states of matter—solid, liquid and gas—give us nine different types of solutions.

Solute + Solvent Solution Example


Solid in solid Solid Alloys, gemstones etc.
Solid in liquid Liquid Sugar in water
Solid in gas Gaseous Camphor in nitrogen gas
Liquid in solid Solid Mercury with sodium/silver
Liquid in liquid Liquid Ethanol in water
Liquid in gas Gaseous Water in air
Gas in solid Solid Hydrogen in palladium
Gas in liquid Liquid O2 dissolved in water
Gas in gas Gaseous Air

(c) True solution


(i) A true solution is one in which the particles of dissolved solute are molecular or ionic in size, generally in the range of
0.1 nm to 1 nm (10−8 cm to 10−7 cm).
(ii) The properties of a true solution are:
• The dissolved solute is molecular or ionic in size.
• The solute remains uniformly distributed throughout the solution and does not settle down with time.
• The solute can generally be separated from the solvent by purely physical means (e.g., by evaporation).
3. Solubility
(a) The term solubility is the amount of one substance (solute) that will dissolve in a specified amount of another substance (solvent)
under stated conditions.
Mass of solute
Solubility = × 100
Mass of solvent
(b) Saturated solution
A saturated solution contains dissolved solute in equilibrium with undissolved solute.

Solute (undissolved)  Solute (dissolved)

Tip At any given temperature, a saturated solution contains the maximum amount of solute dissolved in a given amount of a sol-
vent. It is important to state the temperature of a saturated solution, because a solution that is saturated at one temperature may
not be saturated at another. If the temperature of a saturated solution is changed, the equilibrium is disturbed, and the amount of
dissolved solute will change to re-establish equilibrium.

(c) Factors affecting solubility


(i) Particle size: Since surface-to-volume ratio increases as size decreases, smaller crystals dissolve faster than the larger ones.
(ii) Nature of solute and solvent.
(iii) Concentration of the solution: As the concentration of the solution increases and the solution becomes more nearly satu-
rated with the solute, the rate of dissolving decreases.
(iv) Agitation or stirring.

Chapter-17.indd 544 8/5/2016 10:44:38 AM


Summary 545

(v) Temperature: In most cases, the rate of dissolving of solid increases with temperature. For gases, the solubility decreases with
increase in temperature.
(vi) Pressure: This is generally applicable to solubility of gases and is governed by Henry’s law.
(d) Effect of nature of solute and solvent
(i) Molecular solutions
• When solute and solvent are both polar in nature, they tend to form a solution. For e.g, water and ethanol (C2H5OH).
• When solute and solvent molecules differ in polarity, solutions do not form. For example, water and benzene (C6H6) are
insoluble in each other.
(ii) Ionic solutions
• Polar solvents tend to dissolve polar and ionic compounds. For example, a crystal of NaCl dissolves in water.
• Non-polar solvents tend to dissolve non-polar compounds. For example, non-polar solids like wax are soluble in non-polar
solvents such as benzene.
(e) Effect of temperature on solubility
This effect is assessed using Le Chatelier’s principle. According to this principle, if a system at equilibrium is disturbed, the sys-
tem will change in a direction that counteracts the disturbance and causes the system to return to equilibrium.

Tip The solubility of gases in liquids depends upon the critical temperature specific to a particular gas.

(f) Effect of pressure on solubility of gases


(i) The solubility of a gas in a liquid is directly proportional to the pressure of that gas above the solution.
(ii) Henry’s law
The Henry’s law (also called the Pressure–Solubility law) states that the concentration of a gas dissolved in a given volume
in a liquid at any given temperature is directly proportional to the partial pressure of the gas over the solution.
C gas = K H pgas (K H is constant)

where Cgas is the concentration of the gas and pgas is the partial pressure of the gas above the solution. The proportionality
constant, KH, called the Henry’s law constant, is unique to each gas. The equation is true only at low concentrations and pres-
sures and for gases that do not react with the solvent.
An alternate (and commonly used) form of Henry’s law is
C1 C2
=
p1 p2
where the subscripts 1 and 2 refer to initial and final conditions, respectively.
Henry’s law can also be expressed in terms of mole fraction as “the partial pressure of the gas in vapour phase is proportional
to the mole fraction of the gas in the solution”.
p = KH x
where p is the partial pressure, x is the mole fraction and KH is the Henry’s law constant.

Tip Henry’s law constant KH is different for every gas, temperature and solvent.
(a) For a given gas, it is different at different temperatures.
(b) At a given temperature, it is different for different gases.
(c) At a given pressure, higher the value of KH for the gas, lower will be its solubility in the solution.
(d) The value of KH generally increases with increase in temperature and hence the solubility of gas decreases with increase in
temperature.

4. Vapour pressure of solutions


Vapour pressure is the pressure of the vapour resulting from evapouration of a liquid (or solid) in a closed system.
(a) Vapour pressure of liquid–liquid solutions
When two (or more) components of a liquid solution can evapourate, the vapour contains molecules of each substance. Let ptotal
be the total vapour pressure of the solution due to both the components and let pA and pB be the partial pressures of compo-
nents A and B, respectively.
From Raoult’s law, the partial pressure of a particular component is directly proportional to the component’s mole fraction in the
solution ( p A = x A p oA). Therefore, if xA and xB are the mole fractions of A and B, and then the total pressure can be given by
ptotal = pA + pB

ptotal = x A p oA + xB pBo

ptotal = (1 − xB )p oA + xB pBo = p oA + ( pBo − p oA ) xB

Chapter-17.indd 545 8/5/2016 10:44:40 AM


546 Chapter 17 Solutions

ΙΙΙ From the plot, we can see that:


ptotal = 1. When only component A is present, that is xA = 1 and xB = 0, the
poA + (poB − poA)xB
vapour pressure corresponds to that of pure component A.
As component B is added, the mole fraction of A decreases and
Vapour pressure

ΙΙ so does the vapour pressure, as shown along line I.


2. When only pure component B is present, that is xA = 0 and
o x
B
pB poB xB = 1, the vapour pressure corresponds to that of pure com-
=
pB ponent B. As component A is added, the mole fraction of B de-
creases and so does its vapour pressure, as shown along line II.
pA
poA = po 3. Depending on the vapour pressure of pure components A and
A xA
B, the total vapour pressure over the binary solution varies
Ι between the vapour pressure of A and B as shown along line III.
4. If component A is less volatile than component B, that is
0 0
xA = 1 Mole fraction xB = 1 p oA < pBo, then from the graph we can see that the maximum
xB = 0 xA = 0 value of ptotal is pBo and the minimum value is p oA .

If yA and yB are the mole fractions of A and B, respectively, in the vapour phase, then by Dalton’s law
pA = y A ptotal and pB = yB ptotal

(b) Vapour pressure of solutions of solids in liquids


All liquid solutions of non-volatile solutes (solutes that have no tendency to evapourate) have lower vapour pressures than their
pure solvents. The vapour pressure of such a solution is proportional to how much of the solution actually consists of the sol-
vent. This proportionality is given by Raoult’s law (also known as the vapour pressure–concentration law) which says that the
vapour pressure of the solution, psolution, equals the mole fraction of the solvent, xsolvent, multiplied by its vapour pressure when
pure, that is, p osolvent .

psolution = x solvent p osolvent


Because of the form of this equation, a plot of psolution vs. xsolvent should be linear at all concentrations when the system obeys
Raoult’s law.
Vapor pressure of
pure solvent
psolution

psolution = xsolvent posolvent

0 xsolvent 1

The change in vapour pressure equals the mole fraction of the solute times the solvent’s vapour pressure when pure.
o
( psolvent − psolution )
x solute = o
psolvent
5. Ideal solutions
Solution is said to be ideal if each of its components obeys Raoult’s law for the entire composition range. The enthalpy of mixing
∆Hmix is zero, that is, in preparation of an ideal solution no thermal change is observed.
(i) Vapour pressure of ideal binary liquid solution
1 pAo + ( pBo − pAo ) y A 1  1 1
= = +  o − o  yA
ptotal pBo pAo pBo  pA pB 

where p A( = x A p oA ) and pB( = xB pBo ) are the partial pressures exerted by vapours of the constituents x A , xB = mole fractions

pAo , pBo = vapour pressure of pure constituents. This can be derived as follows:

ptotal = pA + pB = x A ⋅ pAo + xB ⋅ pBo

Chapter-17.indd 546 8/5/2016 10:44:45 AM


Summary 547

ptotal = x A ⋅ pAo + (1 − x A )pBo


ptotal = pBo + ( pAo − pBo ) x A

pA pAo x A
yA = = o
ptotal pB + ( pAo − pBo ) x A

pB xB pBo
yB = = 1 − yA = o
ptotal pA + ( pBo − pAo ) xB

y A pBo
xA =
pAo + ( pBo − pAo ) y A
y A pBo
ptotal = pBo + ( pAo − pBo ) x A = pBo + ( pAo − pBo )
pAo + ( pBo − pAo ) y A

6. Non-ideal solutions
(a) Solutions are non-ideal if they do not obey Raoult’s law over the entire composition range. In preparation of such solutions,
heat is either given out or absorbed (∆Hmix ≠ 0) and the volume of the final solution is not equal to the sum of volume of indi-
vidual components of the solution (∆Vmix ≠ 0). The vapour pressure of the solution is either higher or lower than that predicted
by Raoult’s law.
(b) Depending on type of deviation from ideal behaviour, non-ideal solutions may be classified as showing negative deviation
(lower vapour pressure than predicted) or positive deviation (higher vapour pressure than predicted).

Negative deviation
poB • In such solutions of components A and B, the A B interactions are stronger than
the A A and B B interactions.
• Lesser number of the molecules escapes compared to pure state.
p • Vapour pressure of the solution is lower than that predicted by Raoult’s law.
Vapour pressure

poA • Example of negative deviation is that of solution of phenol with aniline as well as
acetone and water.
B
p

pA

xA = 1 Mole fraction xB = 1
xB = 0 xA = 0

poB Positive deviation


• In such solutions of components A and B, the A B interactions are weaker than the
p A A and B B interactions.
• Since the interactions between solute and solvent are weaker than interactions bet-
ween solute and solute or solvent and solvent, molecules of A and B find it easier to
escape than in pure state.
Vapour pressure

• Vapour pressure of the solution is higher than that predicted by Raoult’s law.
B
p

poA • Examples of mixtures showing positive deviation are: ethanol + hexane, acetone +
carbon disulphide; carbon tetrachloride + benzene; carbon tetrachloride + toluene;
p
A
etc.

xA = 1 Mole fraction xB = 1
xB = 0 xA = 0

Chapter-17.indd 547 8/5/2016 10:44:48 AM


548 Chapter 17 Solutions

7. Azeotropic (constant boiling) mixtures


(a) Liquid solution mixtures which have the same composition in solution as well as in vapour phase are calld azeotropic or con-
stant boiling mixtures and are known as azeotropes.
(b) It is difficult to separate the components of these mixtures by fractional distillation as both the components vapourize at the
same temperature that is the boiling point of the mixture.
(c) The azeotropes have a characteristic boiling point which is either higher (positive deviation) or lower (negative deviation)
than any of its constituents and are called minimum boiling and maximum boiling azeotropes, respectively.
(d) Non-ideal solutions often form azeotropes. For example, ethanol, normally obtained on distillation, is an azeotrope of composi-
tion ethanol (95%) + water (5%). It is a maximum boiling (negative deviation) azeotrope because it boils at 78.2°C; whereas the
boiling point of pure ethanol is 78.4°C and that of water is 100°C.

Tip A separation technique called azeotropic distillation is used for separation of azeotropic mixtures.

8. Colligative properties and determination of molar mass


(a) The colligative properties are properties that depend only on the number of solute particles in a solution, but are independ-
ent of the nature of those particles. The four colligative properties of solutions are vapour pressure lowering, boiling point
elevation, freezing point depression and osmotic pressure.

Tip The basic assumption for describing colligative properties of dilute solutions is that the solute is non-volatile and is not present
in the vapour phase at all.

Colligative
Definition Mathematical relation
property
The presence of a non-
∆pA p oA − pA
volatile solute in a solvent = = xB
Relative lowering of

p oA p oA
vapour pressure

lowers the equilibrium


vapour pressure from that For dilute solutions nA + nB is almost equal to nA
of the pure solvent.
p oA − pA nB wB × MA
= =
p oA nA MB × w A
where wA and wB are the masses of solvent and solute taken and MA and MB are the
molar masses of the solvent and solute.
Addition of a solute raises The difference in the boiling point of a solution containing a non-volatile solute and the
the boiling point of a boiling point of the pure solvent is known as boiling-point elevation (∆Tb)
solution.
Since boiling point ∆Tb = Tb − Tbo
elevation is a colligative
property, its magnitude where ∆Tb is the number of Celsius (or Kelvin) degrees that the boiling point is raised,
is proportional to the Tb is the boiling point of the solution and Tb° is the boiling point of pure solvent.
Elevation of boiling point

relative amount of solvent


and solute. Relation with molality
∆Tb = K b m

where Kb is a constant characteristic of the solvent (for water Kb = 0.512 m is molality). The
constant Kb is called molal boiling point elevation constant or Ebullioscopic constant
Determination of molar mass (using definition of molality)
1000 × wB × K b
MB =
∆Tb × w A

Relation between Kb and enthalpy of vapourization


For dilute solutions, as m → 0, Tb → Tb∗, so

 ∂T  RTb*2 MA
Kb =  b  =
 ∂m  m→0 1000 × ∆Hvap

Chapter-17.indd 548 8/5/2016 10:44:50 AM


Summary 549

Addition of a solute lowers The difference in the freezing point of a solution containing a non-volatile solute and
the freezing point of a the freezing point of the pure solvent is known as freezing point depression (∆Tf )
solution. given by the equation
Since freezing point ∆Tf = Tfo − Tf
depression is a colligative
where ∆Tf is the number of Celsius (or Kelvin) degrees that the freezing point is
property, its magnitude
depressed, Tf is the freezing point of the solution and T of is the freezing point of pure
Depression in freezing point

is proportional to the
solvent.
relative amount of solvent
and solute. Relation with molality
∆Tf = K f m
where molal Kf is constant characteristic of the solvent and m is molality. The constant Kf
is called molal freezing point depression constant or cryoscopic constant.
Determination of molar mass from the definition of molality
1000 × wB × K f
MB =
∆Tf × w A
Relation between Kf and enthalpy of fusion
For dilute solutions, as m → 0, Tf → Tf∗, so
 ∂T  RTf*2 MA
Kf =  f  =
 ∂m  m→0 1000 × ∆Hfusion

The extra pressure The osmotic pressure ∏ is given by


required to establish an
osmotic equilibrium is ∏ = CRT
known as the osmotic where C is the concentration in mol−1.
pressure.
Osmotic pressure

In terms of mole per litre, osmotic pressure is given as


Osmotic pressure is a
colligative property and nB
∏= RT
is dependent only on V
the concentration of where nB is the number of moles of solute and V is the volume of the solution.
the solute particles and
wB
is independent of their ∏V = RT
nature. MB
Mass of an unknown solute in the solution is
wB RT
MB =
∏ V

Tip The values of both Kf and Kb are characteristic of each solvent. The units of constants Kf and Kb are K kg mol−1 or °C (molal)−1.

(b) Types of solutions


(i) Isotonic solutions: When the concentration of ions and other solutes in the blood or plasma is the same as that inside of
a cell, the solution is said to be isotonic. In this case, there is no net movement of water into or out of the cell.
(ii) Hypertonic solutions: If the concentration of solutes in the solutions around the cells is greater than that present in the
cell, water moves out of the cell, shrinkage and dehydration occurs. The solution is called hypertonic.
(iii) Hypotonic solutions: If the concentration of solutes in the solutions around the cells is lesser than that present in the cell,
water moves into the cell, swelling and turgidity occurs and cell may burst. The solution is called hypertonic.
(c) Reverse osmosis and water purification
In reverse osmosis, the direction of normal osmotic flow of water across the membrane is reversed by applying pressure to the com-
partment with high concentration. Then, the solvent flows from the compartment containing a higher concentration of the solute
to that having a lower concentration and this results in increase in concentration of the solute in one and dilution in the other. This
technique is extensively used in desalination of water. It is used for making seawater fit for drinking, particularly on ships.
9. Abnormal molar masses and van’t Hoff factor
(a) In case of a solute that dissociates in the solution, the experimentally determined molar mass is always lower than the actual
value.
(b) When a solute undergoes association on dissolving in a solvent, the number of particles of solute is reduced. The experimentally
determined colligative property is lower than that expected from Raoult’s law and ideal behaviour. The molar mass determined
experimentally is always higher than the actual value.

Chapter-17.indd 549 8/5/2016 10:44:52 AM


550 Chapter 17 Solutions

For example, the experimentally determined values of colligative properties of solution of acetic acid in benzene are lower than
those calculated theoretically. This is because the molecules of acetic acid aggregate in benzene by hydrogen bonding to form
dimers. Similarly, benzoic acid also dimerizes in benzene.
(c) The extent of dissociation or association of a solute in solution is expressed in terms of van’t Hoff factor i. It is defined as the ratio
of experimentally determined value of colligative property to the calculated value of colligative property based on Raoult’s law
and ideal behaviour.
Observed value of colligative property
i=
Calculated value off colligative property
Number of moles of solute after association/dissociation
i=
Number of moles of solute before dissolution
Normal molar mass
i=
Abnormal molar mass

Condition van’t Hoff factor Molar mass


Solute does not associate or Observed and calculated colligative properties are Values of molar mass determined by colligative
dissociate in the solution. the same. properties is the same as the normal mass.
The solute undergoes disso- The observed values of colligative properties are Formula mass is less than that obtained by
ciation in solution. more than the calculated values. i.e., i > 1. measurement of colligative properties.
The solute undergoes asso- The observed values of colligative properties are Formula mass is greater than that obtained by
ciation in solution. less than the calculated values. i.e., i < 1. measurement of colligative properties.

(d) For solutes that associate or dissociate in solution, the modified equations of colligative properties are

Relative lowering of vapour pressure


p oA − pA nB
∆p = =i
p oA nA

Elevation of boiling point ∆Tb = iK b m

Depression of freezing point ∆Tf = iK f m


Osmotic pressure nB
∏=i RT
V

Tips (a) The hypothetical van’t Hoff factor, i, is 2 for NaCl, KCl and MgSO4, which break up into two ions on 100% dissociation. For
K2SO4, the theoretical value of i is 3 because one K2SO4 unit gives three ions.
(b) With decreasing concentration (i.e., at higher dilutions) the experimental van’t Hoff factors agree better with their corre-
sponding hypothetical van’t Hoff factors.
(c) The increase in the percentage dissociation that comes with greater dilution is not the same for all salts.

TOOLS FOR PROBLEM SOLVING


Mass of solute
1. Solubility = × 100
Mass of solvent
2. Relations from Henry’s law: C gas = K H pgas (K H is constant)
C1 C2
=
p1 p2

p = KH x

1 pAo + ( pBo − pAo ) y A 1  1 1


3. Vapour pressure of ideal binary liquid solution = = +  o − o  yA
ptotal pBo pAo pBo  pA pB 

Chapter-17.indd 550 8/5/2016 10:44:55 AM


Solved Examples 551

∆pA p oA − pA
4. Relative lowering of vapour pressure = = xB
p oA p oA
p oA − pA nB wB × MA
For dilute solutions, = =
p oA nA MB × w A

5. Elevation of boiling point (∆Tb)


(a) Boiling point elevation: ∆Tb = Tb − Tbo
(b) Relation with molality: ∆Tb = K b m
1000 × wB × K b
(c) Determination of molar mass: MB =
∆Tb × w A
 ∂T  RTb*2 MA
(d) Relation between Kb and enthalpy of vapourization K b =  b  =
 ∂m  m→0 1000 × ∆Hvap
6. Depression in freezing point
(a) Freezing point depression (∆Tf ): ∆Tf = T of − Tf
(b) Relation with molality: ∆Tf = K f m
1000 × wB × K f
(c) Determination of molar mass: MB =
∆Tf × w A
 ∂T  RTf*2 MA
(d) Relation between Kf and enthalpy of fusion: K f =  f  =
 ∂m  m→0 1000 × ∆Hfusion
7. The osmotic pressure ∏ is given by ∏ = CRT
n w
In terms of mole per litre, osmotic pressure is given as: ∏ = B RT or ∏V = B RT
V MB
Observed value of colligative property
8. van’t Hoff factor i =
Calculated value off colligative property
Number of moles of solute after association/dissocaition
i=
Number of moles of solute before dissolution
Normal molar mass
i=
Abnormal molar mass

SOLVED EXAMPLES
Solubility 2. Henry’s law constant for the molality of methane in benzene at
298 K is 4.27 × 105 mm Hg. Determine the solubility of methane
1. The solubility of pure oxygen in water at 20°C and 1 atm in benzene at 298 K under 760 mm Hg.
pressure is 43 mg O2 per litre of water. When air is in contact (a) 406 × 10 −5 (b) 206 × 10 −3
with water and the air pressure is 585 torr at 20°C, how much (c) 315 × 10 −3
(d) 178 × 10 −5
amount of oxygen is dissolved in 500 mL water? The average
concentration of oxygen in air is 21.1% (V/V). Solution
(a) 7 mg (b) 3.5 mg (c) 16.5 mg (d) 33 mg (d) Given that KH = 4.27 × 105 mm Hg. Now from Henry’s law,
we have p = K H × x where x is the mole fraction. Hence,
Solution
C1 p1 p 760
(b) From Henry’s law: C gas = K H pgas ⇒ = x= = = 178 × 10 −5
C2 p2 K H 4.27 × 105
Given that C1 = 43 mg L−1; p1 = 1 atm = 760 torr; C2 = ?;
Vapour Pressure of Ideal Solutions
21.1
p2 = 585 × = 123.435 torr 3. 18 g of glucose (C6H12O6) is added to 178.2 g of water. The
100
vapour pressure of water for this aqueous solution at 100°C is
Substituting in the above expression, we get (a) 759.00 torr (b) 7.60 torr
123.44 × 43 (c) 76.00 torr (d) 752.40 torr
C2 = mg L−1 = 7 mgL−1 (AIEEE 2006)
760
7 Solution
Therefore, mass of O2 in 500 mL water = × 500 p o − ps n
1000 (d) According to Raoult’s law = x solute = (1)
ps N
= 3.5 mg

Chapter-17.indd 551 8/5/2016 10:45:02 AM


552 Chapter 17 Solutions

Vapour pressure of solvent, Therefore,


po = 760 mm Hg (normal pressure) 17.5 − ps 0.1 17.5 − ps
ps = water for the solution = ⇒ = 0.01
17.5 10 17.5
18 178.2 ⇒ ps = 17.325 mm Hg
n= ; N=
mol.wt. of C6H12O6 mol.wt. of water
On substituting the values in Eq. (1), we get 7. Two liquids X and Y form an ideal solution. At 300 K, the
760 − ps (18 / 180 ) 0.1 vapour pressure of the solution containing 1 mol of X and
= = 3 mol of Y is 550 mm Hg. At the same temperature, if 1 mol
ps (178.2 / 18 ) 9.9
of Y is further added to this solution, vapour pressure of the
1 solution increases by 10 mm Hg. Vapour pressure (in mm Hg)
760 − ps = ps
99 of X and Y in their pure states will be, respectively
760 × 99 − ps × 99 = ps ⇒ ps = 752.4 torr (a) 400 and 600 (b) 500 and 600
(c) 200 and 300 (d) 300 and 400
4. A mixture of ethyl alcohol and propyl alcohol has a vapour (AIEEE 2009)
pressure of 290 mm Hg at 300 K. The vapour pressure of propyl Solution
alcohol is 200 mm Hg. If the mole fraction of ethyl alcohol is 0.6,
(a) Mole fraction of x = 1/4 and of y = 3/4.
its vapour pressure (in mm Hg) at the same temperature will be
(a) 360 (b) 350 (c) 300 (d) 700 Total pressure = vapour pressure of x × Mole fraction of x
(AIEEE 2007) + vapour pressure of y × Mole fraction of y
Solution
o
(b) Let p′ be the vapour pressure of ethyl alcohol in the mix- p ox 3 p y
550 = +
ture and p′′ be the vapour pressure of propyl alcohol. Then, 4 4
total pressure (p) of the mixture will be
550 × 4 = p ox + 3p oy (1)
p = p′ × (Mole fraction)ethyl alcohol + p′′ × (Mole fraction)propyl alcohol
290 − (200 × 0.4 ) When one mole of y is added, then mole fraction of x
290 = ( p ’ × 0.6 ) + (200 × 0.4 ) ⇒ p ’ = = 350 mm Hg becomes 1/(1 + 4) = 1/5 and mole fraction of y becomes
0.6
4/(1 + 4) = 4/5. The total pressure given is 560 mm Hg.
The vapour pressure of ethyl alcohol is 350 mm of Hg. o
p ox 4 p y
5. At 80°C, the vapour pressure of pure liquid A is 520 mm Hg and 560 = +
5 5
that of pure liquid B is 1000 mm Hg. If a mixture of solutions A
and B boils at 80°C and 1 atm pressure, the amount of A in the (2)
560 × 5 = p ox + 4p oy
mixture is (1 atm = 760 mm Hg)
(a) 52 mol% (b) 34 mol% (c) 48 mol% (d) 50 mol% Solving Eqs. (1) and (2), we get p ox = 400 mm Hg and
(AIEEE 2008) p oy = 600 mm Hg
Solution
8. On mixing, heptane and octane form an ideal solution. At
(d) According to Raoult’s law 373 K, the vapour pressures of the two liquid components
ptotal = pAo x A + pBo xB ⇒ 760 = 520 x A + 1000 (1 − x A ) ⇒ x A = 0.5 (heptane and octane) are 105 kPa and 45 kPa, respectively.
Thus, mol% of A = 0.5 × 100 = 50%. Vapour pressure of the solution obtained by mixing of 25 g
heptane and 35 g of octane will be (Given that molar mass of
6. The vapour pressure of water at 20°C is 17.5 mm Hg. If 18 g of heptane = 100 g mol−1 and of octane = 114 g mol−1.)
glucose (C6H12O6) is added to 178.2 g of water at 20 °C, the (a) 144.5 kPa (b) 72.0 kPa (c) 36.1 kPa (d) 96.2 kPa
vapour pressure of the resulting solution will be (AIEEE 2010)
(a) 17.675 mm Hg. (b) 15.750 mm Hg. Solution
(c) 16.500 mm Hg. (d) 17.325 mm Hg.
(b) Number of moles of heptane = 25/100; number of moles
(AIEEE 2008)
of octane = 35/114. Now,
Solution
(d) According to Raoult’s law Number of moles of heptane
Mole fraction of heptane =
p o − ps p o − ps ns Total number of moles
o
= x solute ⇒ = 25/100
p po ns + nw = = 0.45
(25/100 ) + (35/114 )
where po and ps are vapour pressure of pure solvent and
solution respectively and ns and nw are determined as Now, mole fraction of heptane + mole fraction of octane
follows = 1. So, mole fraction of octane = 1 – 0.45 = 0.55.
w s 18 178.2 Total pressure = Mole fraction (heptane) × pC7H16
ns = = = 0.1; nw = = 9.9
Ms 180 18 + Mole fraction (octane) × pC8H18
⇒ ns + nw = 0.1 + 9.9 = 10 = (105 × 0.45 + 45 × 0.55) = 72.0 kPa

Chapter-17.indd 552 8/5/2016 10:45:07 AM


Solved Examples 553

9. For an ideal solution of two components A and B, which of the Applying Dalton’s law for mole fraction in vapour phase,
following is true? we get
(a) ∆Hmixing < 0 (zero).
(b) ∆Hmixing > 0 (zero). pB po x 0.4 × 119
yB = = o B Bo = = 0.682
(c) A B interaction is stronger than A A and B B ptotal pT x T + pB xB 69.8
interactions. y T = 1− 0.682 = 0.318
(d) A A, B B and A B interactions are identical.
(JEE Main Online 2014) 13. Which of the following liquid pairs shows a positive deviation
Solution from Raoult’s law?
(d) For ideal solution all type of interactions A A, B B and (a) Water–hydrochloric acid (b) Acetone–chloroform
A B are identical. (c) Water–nitric acid (d) Benzene–methanol
Solution
10. A solution at 20°C is composed of 1.5 mol of benzene and
3.5 mol of toluene. If the vapour pressure of pure benzene and (d) Benzene–methanol. This is because the A A and B B
pure toluene at this temperature are 74.7 torr and 22.3 torr, interactions are much stronger than A B interactions.
respectively, then the total vapour pressure of the solution So, when methanol combined with benzene the hydro-
and the benzene mole fraction in equilibrium with it will be, gen bonding of methanol breaks and its vapour pressure
respectively increases.
(a) 35.0 torr and 0.480 (b) 38.0 torr and 0.589 14. Pentane and heptane are two hydrocarbon liquids present in
(c) 30.5 torr and 0.389 (d) 35.8 torr and 0.280 gasoline. At 20°C, the vapour pressure of pentane is 420 torr
(JEE Main Online 2015) and that of heptane is 36 torr. What will be the total vapour
Solution pressure of a solution prepared by mixing of equal masses of
(b) The total vapour pressure of the solution is two liquids?
(a) 258.7 torr (b) 278 torr (c) 243.6 torr (d) 456 torr
ptotal = x C6H6 pCo6H6 + x C7H8 pCo7H8
Solution
 1.5   3.5  1.5 3.5
= 74.7 +  22.3 = × 74.7 + × 22.3 (a) The molar masses of pentane (C5H12) and heptane (C7H16)
 1.5 + 3.5   1.5 + 3.5  5 5 are 72 and 100, respectively.
112.05 78.05 The mole fraction in solution which contains equal
= + = 38.02 torrr
5 5 masses of each
Mole fraction of benzene in vapour state is 1/ 72
x C5H12 = = 0.58
pC6H6 / ) + (1100
(172 / )
y C6H6 = (By Dalton′ s law ) 1100
/
ptotal x C7H16 = = 0.42
/ ) + (1/100 )
(172
x C6 H 6 pCo6H6 112.05//5
= = = 0.589 Considering that pentane and heptane form ideal
Ptotal 38.02
solution and using Raoult’s law for solution of volatile
11. If liquids A and B form an ideal solution, components,
(a) the enthalpy of mixing is zero. ps = pCo5H12 × x C5H12 + pCo7H16 × x C7H16
(b) the entropy of mixing is zero.
= 420 × 0.58 + 36 × 0.42 = 258.7 torr
(c) the free energy of mixing is zero.
(d) the free energy as well as the entropy of mixing are each
zero. Alternate Solution The vapour pressure of solution should
lie between the vapour pressures of pure compounds. So,
Solution
option (d) can be neglected.
(a) Since there is no change in the magnitude of the attrac-
tive forces in the two compounds present, the heat
change on mixing, that is, ∆Hmix in such solutions must 15. Benzene and toluene form ideal solution over the entire
be zero. range of composition. The vapour pressure of pure benzene
and toluene at 300 K are 50.71 mm Hg and 32.06 mm Hg,
12. What is the composition of toluene in vapour phase of solution respectively. What will be the mole fraction of benzene in
at 30°C with a benzene–toluene solution which has a mole frac- vapour phase if 80 g of benzene is mixed with 100 g of toluene?
tion of benzene of 0.400? ( pBo = 119 torr and pTo = 37.0 torr) (a) 0.0675 (b) 0.675 (c) 0.35 (d) 0.5
(a) 1.318 (b) 2.318 (c) 3.318 (d) 0.318 Solution
Solution 80
(b) Number of moles of benzene: = 1.02
78
(d) Total pressure of the solution is given by (Raoult’s law)
ptotal = xB pBo + x T pTo = 0.4 × 119 + 0.6 × 37 Number of moles of toluene:
100
= 0.77
= 47.6 + 22.2 130
= 69.8 torr Total number of moles = 1.79. Therefore

Chapter-17.indd 553 8/5/2016 10:45:10 AM


554 Chapter 17 Solutions

1.02 0.77 Solution


xB = and x T =
1.79 1.79 (b) The solutions for which enthalpy of solution is positive,
Total pressure of the solution is given by (Raoult’s law) shows positive deviation from Raoult’s law and forms
non-ideal solution with positive deviation. If these solu-
ptotal = pT + pB = x T pTo + xB pBo tions are made in insulated vessel, it absorbs thermal
energy available in the solution. Insulated vessel stops
Substituting values, we have
outside heat to enter into solution, due to which temper-
1.02 0.77 ature of the solution is decreased.
ptotal = 50.71 × + 32.06 ×
1.79 1.79 For positive deviation, the volume of solution is greater
= 28.89 + 13.79 = 42.68 mm than the sum of volume of both components volume.
Since ∆Hsol = Lattice energy + ∆Hhyd. where lattice energy
Also, for vapour phase, the mole fractions are is always positive and hydration enthalpy is negative, it
p p implies that lattice energy is greater than hydration energy.
y T = T and yB = B
ptotal ptotal
Colligative Properties of Dilute Solutions
28.89
Therefore, yB = = 0.675 19. A 5.25% solution of a substance is isotonic with a 1.5% solu-
42.68 tion of urea (molar mass = 60 g mol−1) in the same solvent. If
Real Solutions the densities of both the solutions are assumed to be equal to
16. A binary liquid solution is prepared by mixing n-heptane and 1.0 g m−3, molar mass of the substance will be
ethanol. Which one of the following statements is correct (a) 210.0 g mol−1 (b) 90.0 g mol−1
regarding the behaviour of the solution? (c) 115.0 g mol−1 (d) 105.0 g mol−1.
(a) The solution formed is an ideal solution. (AIEEE 2007)
(b) The solution is non-ideal, showing positive deviation Solution
from Raoult’s law. (a) Isotonic solutions have same osmotic pressure. Let M be
(c) The solution is non-ideal, showing negative deviation the molar mass of the substance.
from Raoult’s law. ∏1 = C1RT = C2RT = ∏2
(d) n-heptane shows positive deviation while ethanol shows
negative deviation from Raoult’s law. So, C1 = C2. Now, as the densities are same, we have
(AIEEE 2009) 5.25 1.5 60
= ⇒ M = 5.25 × = 210
Solution M 60 1.5
(b) Solution of n-heptane and ethyl alcohol will show posi- 20. If sodium sulphate is considered to be completely dissociated
tive deviation because force of attraction between unlike into cations and anions in aqueous solution, the change in
molecules is weaker than like molecules. freezing point of water (∆Tf ) when 0.01 mol of sodium sul-
17. Which of the following solutions will show positive deviation phate is dissolved in 1 kg of water is (Given Kf = 1.86 K mol−1)
from ideal behaviour? (a) 0.0186 K (b) 0.0372 K (c) 0.0558 K (d) 0.0744 K
(a) C2H5OH + H2O (b) CH3COCH3 + CHCl3 (AIEEE 2010)
(c) H2O + HCl (d) H2O + HNO3 Solution
Solution (c) The expression for ∆Tf is ∆Tf = i × K f × m
(a) Positive deviation is shown by the solutions in which For the reaction Na2SO4 → 2Na+ + SO2−
4 , i = 2 + 1 = 3.
interactions between the molecules of its components Thus,
(solute and solvent) are reduced. It causes increase
in vapour pressure and it is greater than the value cal ∆Tf = 3 × 186
. × (0.011
/ ) = 0.0558 K
culated by using Raoult’s law.
21. Ethylene glycol is used as antifreeze in a cold climate. Mass
In pure water, molecules are held by strong intermolec-
of ethylene glycol which should be added to 4 kg of water
ular force known as hydrogen bonding. But, it is reduced
to prevent it from freezing at −6°C will be (Given that Kf for
when ethanol molecule comes in between two molecules
water = 1.86°C molal−1, and molar mass of ethylene glycol =
of water. So, the solution will show positive deviation.
62 g mol−1.)
18. The value of ∆Hsol for a soluble volatile compound is, say, (a) 204.30 g (b) 400 g
+26 kJ mol−1, and a nearly saturated solution is prepared in (c) 304.60 g (d) 800 g
an insulated container. Which of the following statement is (AIEEE 2011)
correct about this solution? Solution
(a) The solution shows the negative deviation from Raoult’s (d) According to depression in freezing point,
law.
(b) The temperature of solution decreases.  1000 
(c) The hydration energy of compound is more than the lat- ∆Tf = K f × m = K f ×  w2 ×
 w1 × M2 
tice energy.
(d) Volume of solution is exactly equal to the sum of volume where w1 and w2 are the masses of solvent and solute
of compound and solvent. and M2 is the molar mass of solute, respectively.

Chapter-17.indd 554 8/5/2016 10:45:13 AM


Solved Examples 555

 1000  26. Which one of the following statements is false?


∆Tf = 0 − ( −6 ) = 6 = 1.86 ×  w2 × (a) Raoult’s law states that the vapour pressure of a compo-
 4000 × 62 
nent over a solution is proportional to its mole fraction.
On solving, we get w2 = 800 g. (b) Two sucrose solutions of the same molality prepared
22. Kf for water is 1.86 kg mol−1. If your automobile radiator holds in different solvents will have the same freezing point
1.0 kg of water, how many grams of ethylene glycol (C2H6O2) depression.
must you add to get the freezing point of solution lowered to (c) The correct order of osmotic pressure for 0.01 M aqueous
−2.8°C? solution of each compound is BaCl2 > KCl > CH3COOH >
(a) 72 g (b) 93 g (c) 39 g (d) 27 g sucrose.
(AIEEE 2012) (d) The osmotic pressure (Π) = MRT, where M is the molarity
of the solution.
Solution
(b) According to depression in freezing point, we have Solution
w /M (b) Two sucrose solutions of same molality prepared in dif-
∆Tf = K f × m = K f × solute B(solute) × 1000 ferent solvents will not have the same freezing point
w solution
depression. This is because freezing point depression
where wsolution = 1000 g, ΔTf = 2.8°C, MB(solute) = 62 g constant depends upon the nature of solvents used.
mol−1. Substituting, we get
∆Tf = K f × m
1.86 × w solute × 1000
2.8 = ⇒ w solute = 93 g
62 × 1000 27. The van’t Hoff factor for the solute in 0.100 m NiSO4 is 1.19.
23. The vapour pressure of acetone at 20°C is 185 torr. When 1.2 g What would this factor be if the solution behaved as if it were
of a non-volatile substance was dissolved in 100 g of acetone 100% dissociated?
at 20°C, its vapour pressure was 183 torr. The molar mass (a) 2 (b) 4 (c) 3 (d) 7
(g mol−1) of the substance is Solution
(a) 64 (b) 128 (c) 488 (d) 32
(JEE Main 2015) (a) 100% dissociation means a = 1
Solution NiSO4 → Ni2+ + SO2−
4
(a) Using the colligative property (relative lowering of (1−a) a a
vapour pressure) i = (1 − a) + a + a
p0 − ps w2 ⋅ M1 =1 + a
= =2
ps M2 ⋅ w1
185 − 183 1.2 × 58 28. Which one of the following aqueous solutions will exhibit
= ⇒ M2 = 64 g mol−1 highest boiling point?
183 M2 × 100
(a) 0.01 M Na2SO4 (b) 0.015 M glucose
24. Determination of the molar mass of acetic acid in benzene
(c) 0.015 M urea (d) 0.01 M KNO3
using freezing point depression is affected by
(a) dissociation. (b) association. Solution
(c) partial ionization. (d) complex formation. (a) Glucose and urea are non-electrolytes and the van’t
(JEE Main Online 2015) Hoff’s factor for KNO3 is 2 and for Na2SO4 it is 3.
Solution
(b) Acetic acid in benzene dimerizes as KNO3 → K + + NO3− (i = 2)
2CH3COOH  (CH3COOH)2 Na2SO 4 → 2Na+ + SO24− (i = 2 + 1 = 3)
25. In a 0.2 mol aqueous solution of a weak acid HX the degree of
Thus, from the expression ∆Tb = i × Kb × m as ∆Tb ∝ i, 0.01 M
ionization is 0.3. Taking Kf for water as 1.85, the freezing point
Na2SO4 will have the highest boiling point.
of the solution will be nearest to
(a) −0.480°C (b) −0.360°C 29. Equimolar solutions in the same solvent have
(c) −0.260°C (d) +0.480°C (a) same boiling point but different freezing point.
Solution (b) same freezing point but different boiling point.
(c) same boiling and same freezing points.
(a) Consider the reaction,
(d) different boiling and different freezing points.
HX → H+ + X −
1 0.3 0.3 Solution
1− 0.3 0.3 0.3
(c) This is because colligative properties depend upon num-
The total number of moles after dissociation = 1 − 0.3 ber of particles.
+ 0.3 + 0.3 = 1 + 0.3. Hence, the van’t Hoff factor is
i = 1.3/1 = 1.3. 30. A 0.118  m solution of LiCl has a freezing point of −0.415°C.
Using the expression ΔTf = i × Kf × m, we get What is the van’t Hoff factor for this solute at this concentra-
∆Tf = 1.3 × 1.85 × 0.2 = +0.480°C tion? (Given that Kf for LiCl = 1.86°C molal−1).
or Tf = 0°C − 0.480°C = −0.480°C (a) 1.89 (b) 3.26 (c) 5.32 (d) 2.18

Chapter-17.indd 555 8/5/2016 10:45:16 AM


556 Chapter 17 Solutions

Solution But the compound may dissociate in the solution. Here


(a) ∆Tf = i × K f × m each compound dissociates in different way.
∆Tf (I) [Fe(H2O )6 ]Cl3  Fe(H2O )6 ]3+ + 3Cl−
i=
Kf × m (II) [Fe(H2O )5 Cl]Cl2 ⋅ H2O  [Fe(H2O )5 Cl]2 + + 2Cl−
0.415°C (III) [Fe(H2O )3 Cl3 ] ⋅ 3H2O → No dissociation
=
. °C molal−1)(0.118 molal)
(186
(IV) [Fe(H2O )4 Cl2 ]Cl ⋅ 2H2O  [Fe(H2O )4 Cl2 ]2 + + Cl−
= 1.89 In this way, the first compound gives maximum ions
31. A saturated solution is prepared by dissolving 0.2 g of poly- on dissociation, so it causes maximum depression in
peptide in water to give 500 mL of solution. The solution has freezing point and maximum elevation in boiling point.
an osmotic pressure of 3.74 torr at 27°C. What is the approxi- Therefore, solution I has maximum boiling point and
mate molecular mass of the polypeptide? minimum freezing point.
(a) 8559 (b) 2000 (c) 180 (d) 203 In contrast, solution III has minimum boiling point and
maximum freezing point since no ionic bonds exist and
Solution it does not dissociate.
(b) The osmotic pressure is given by
34. The van’t Hoff factor for the solute in 0.1 molal NiSO4 is 1.19.
w/M
∏ = CRT = RT What is the mass of ice separated when 1 L of solution is
V cooled down to −1°C? (Given: Kf(H2O) = 1.86°C molal−1)
wRT 0.2 × 0.082 × 300 (a) 777.7 g (b) 814 g
Therefore, M = = ≈ 2000
∏V 3.74 (c) 630 g (d) Data is insufficient
× 0.5
760
Solution
32. Calculate the osmotic pressure of a solution obtained by mix- (a) ∆Tf = iK f × m ⇒ ∆Tf = 1.19 × 1.86 × 0.1 = 0.221
ing 100 ml of 3.4% solution (w/V) of urea (mol. wt. 60) and 100
So, the freezing point of solution is −0.221°C. On cooling
ml of 1.6% solution (w/V) of cane sugar (mol. wt. 342) at 20°C.
it to −1°C, ice is formed and since salt is more soluble in
(a) 14.73 atm (b) 13.61 atm (c) 7.38 atm (d) 6.82 atm
liquid phase, all solute is left in dissolved form in the liq-
Solution uid phase. Consider no effect on dissociation on cooling
w1 the solution, then at −1°C
(c) For urea, ∏1 V1 = n1RT1 = RT1
M1 1
∆Tf = iK f × m ⇒ m = = 0.45 mol kg−1
w2 1.86 × 1.19
For sugar, ∏2 V2 = n2RT2 = RT2
M2 Now,
Since, 100 mL of urea solution are mixed with 100 mL of w /M
m = B B × 1000
cane-sugar solution, the total volume becomes 200 mL in wA
which 3.4 g urea and 1.6 g sugar is present. 0.1
0.45 = × 1000
200 3.4 wA
∏1 × = × 0.0821 × 293 ⇒ ∏1 = 6.82 atm
1000 60 100
wA = = 222.22 g
(where R = 0.0821 1 atm mol−1 K−1) 0.45
200 1.6 Hence, the mass of ice separated = 1000 − 222.22 = 777.78 g
and ∏2 × = × 0.0821 × 293 ⇒ ∏2 = 0.56 atm
1000 342
So, ∏ total = ∏1 + ∏2 = 7.38 atm Molecular Weight Calculations
33. Consider the one molal aqueous solution of each of the coor- 35. The density (in g mL−1) of a 3.60 M sulphuric acid solution,
dination compounds given below. that is, 29% H2SO4 (molar mass = 98 g mol−1) by mass will be
(I) [Fe(H2O )6 ]Cl3 (II) [Fe(H2O )5 Cl]Cl2 ⋅ H2O (a) 1.45 (b) 1.64 (c) 1.88 (d) 1.22
(III) [Fe(H2O )3 Cl3 ] ⋅ 3H2O (IV) [Fe(H2O )4 Cl2 ]Cl ⋅ 2H2O Solution
The correct statement about them is
(d) Let the density of the solution be r. The molarity of the
(a) I will show maximum freezing point.
solution = 3.60 M (given). This means that 1 L of solution
(b) III will show maximum boiling point.
contains 3.6 mol or (3.6 × 98) g of H2SO4 (98 g mol−1 is
(c) III will show minimum freezing point.
the molecular mass of H2SO4). Since, the solution is 29%
(d) I will show maximum boiling point.
by mass, which means 29 g of H2SO4 is present in 100 g,
Solution therefore 100/r mL solution contains 29 g of H2SO4.
(d) The elevation in boiling point and depression in freezing As volume = mass/density. Now, 1000 mL contains 3.6
point of a solvent depend on the number of particles, or × 98 g of H2SO4. Therefore,
moles dissolved in it. Equal molal concentration indicates 100 29 × 1000 3.6 × 98 × 100
= ⇒ r= = 1.22 gL−1
that equal moles of each compound are dissolved. r 3.6 × 98 29 × 1000

Chapter-17.indd 556 8/5/2016 10:45:21 AM


Advanced Level Problems 557

36. The degree of dissociation (a) of a weak electrolyte, A xB y is For 0.500 M C2H5OH(aq), ∏ = 1 × 0.5 × RT = 0.5RT
related to van’t Hoff factor (i) by the expression: For 0.100 M Mg3(PO4)2(aq), ∏ = 5 × 0.1 × RT = 0.5RT
i −1 x + y −1
(a) a = (b) a = For 0.250 M KBr(aq), ∏ = 2 × 0.25 × RT = 0.5RT
x + y +1 i −1
x + y +1 i −1 For 0.125 M Na3PO4(aq), ∏ = 4 × 0.125 × RT = 0.5RT
(c) a = (d) a =
i −1 x + y −1 So, all have the same osmotic pressure, and so they are
isotonic solutions.
(AIEEE 2011)
Solution 38. The observed osmotic pressure for a 0.10 M solution of
Fe(NH4)2(SO4)2 at 25°C is 10.8 atm. The expected and exper-
(d) The van’t Hoff factor is
imental (observed) values of van’t Hoff factor (i) will be
i −1
i = 1 − a + na = 1 + a (n − 1) ⇒ a = respectively:
n −1 (R = 0.082 L atm K−1 mol−1)
For the reaction A xB y → xA y + + yB x − , n = x + y, we have (a) 5 and 4.42 (b) 4 and 4.00
(c) 5 and 3.42 (d) 3 and 5.42
i −1 i −1
a= = (JEE Main Online 2014)
n −1 x + y −1 Solution
∏ experimental
37. Consider separate solutions of 0.500 M C2H5OH(aq), 0.100 M (a) van’t Hoff factor is given by i =
∏ expected
Mg3(PO4)2(aq), 0.250 M KBr(aq) and 0.125 M Na3PO4(aq) at
25°C. Which statement is true about these solutions, assum- ∏ expected = CRT
ing all salts to be strong electrolytes? = 0.1 × 0.082 × 298
(a) They all have the same osmotic pressure. = 2.4
(b) 0.100 M Mg3(PO4)2(aq) has the highest osmotic
pressure. 10.8
i= = 4.5
(c) 0.125 M Na3PO4(aq) has the highest osmotic pressure. 2.4
(d) 0.500 M C2H5OH(aq) has the highest osmotic pressure. For 100% dissociation the expected value of van’t Hoff
(JEE Main 2014) factor will be 5 as five ionic species will be formed after
Solution dissociation of Fe(NH4)2(SO4)2.
(a) According to van’t Hoff equation, ∏ = iCRT .

ADVANCED LEVEL PROBLEMS


1. 75.2 g of C6H5OH (phenol) is dissolved in a solvent of Kf = 14°C wA 20
m= × 1000 = × 1000
molal−1. If the depression in freezing point is 7 K then find the MA × w B 172 × 50
percentage of phenol that dimerizes.
Therefore,
(IIT-JEE 2006)
Solution 20
∆Tf = i × K f × m ⇒ 2 = i × 1.72 × × 1000 ⇒ i = 0.5
The reaction involved is 172 × 50
2C6H5OH  (C6H5OH)2 3. Read the following paragraph and answer the questions that
Initial moles 1 0 follow:
Moles at equilibrium 1− a a /2 Paragraph for Questions 3a to 3c: Properties, such as boiling
point, freezing point, and vapour pressure of a pure solvent change
Total number of moles at equilibrium = 1 − a + a/2 when solute molecules are added to get homogeneous solution.
⇒ i = 1 − a/2 (association). These are called colligative properties. Applications of colligative
Hence, properties are very useful in day-to-day life. One of its examples is
7 75.2  a 
∆Tf = K f × mobs × i ⇒ = ×  1 −  ⇒ a = 0.75 the use of ethylene glycol and water mixture as antifreezing liquid
14 94  2 in the radiator of automobiles.
So, the percentage of phenol that dimerizes = 75%. A solution M is prepared by mixing ethanol and water. The mole
fraction of ethanol in the mixture is 0.9. Given that
2. When 20 g of naphthoic acid (C11H8O2) is dissolved in 50 g of
Freezing point depression constant of water (K f ) = 1.86 K molal−1
benzene (Kf = 1.72 K kg mol−1), a freezing point depression of
and of ethanol (K f ) = 2.0 K molal−1.
2 K is observed. The van’t Hoff factor (i) is
Standard boiling point of water = 373 K and of ethanol = 351.5 K.
(a) 0.5 (b) 1 (c) 2 (d) 3
Boiling point elevation constant of water (K b ) = 0.52 K molal−1 and
(IIT-JEE 2007)
of ethanol (K b ) = 1.2 K molal−1.
Solution
Vapour pressure of pure water = 32.8 mm Hg and of pure ethanol
(a) Molality can be calculated using given values as = 40 mm Hg.

Chapter-17.indd 557 8/5/2016 10:45:27 AM


558 Chapter 17 Solutions

Standard freezing point of water = 273 K and of ethanol = 155.7 K. We can calculate i = 3 + 1 = 4. Now, from Eq. (1)
Molar mass of water = 18 g mol−1 and of ethanol = 46 g mol−1. wB 1000 0.1 1000
In answering the following questions, consider the solutions to ∆Tf = i × K f × × = 4 × 1.86 × ×
MB WA 329 100
be ideal dilute solutions and solutes to be non-volatile and non-
dissociative. = 0.0226 = 2.26 × 10 −2 = 2.3 × 10 −2 °C
(IIT-JEE 2008)
Freezing point of the solution = –2.3 × 10−2°C (as freezing
3a. The freezing point of the solution M is point of water is 0°C).
(a) 268.7 K (b) 268.5 K (c) 234.2 K (d) 150.9 K
6. For a dilute solution containing 2.5 g of a non-volatile, non-elec-
Solution trolyte solute in 100 g of water, the elevation in boiling point
(d) ∆Tf = Kf × m = 2 × 0.1 × 1000/(0.9 × 46) = 4.83 K at 1 atm pressure is 2°C. Assuming concentration of solute
Freezing point of solution = Freezing point of pure sol- is much lower than the concentration of solvent, the vapour
vent − ∆Tf = (155.7 − 4.83) = 150.9 K. pressure (mm Hg) of the solution is (Take Kb = 0.76 K kg mol−1)
(a) 724 (b) 740 (c) 736 (d) 718
3b. The vapour pressure of the solution M is
(IIT-JEE 2012)
(a) 39.3 mm Hg. (b) 36.0 mm Hg.
Solution
(c) 29.5 mm Hg. (d) 28.8 mm Hg.
(a) When a non-volatile solute is added to pure solvent, there
Solution is decrease in vapour pressure. So, according to relative
(a) Vapour pressure of water = mole fraction of water × po p o − ps
water = 0.1 × 32.8 mm Hg = 3.28 mm Hg lowering in vapour pressure, = x2
po
Vapour pressure of ethanol = mole fraction of ethanol ×
where x2 is the mole fraction of solute, po is the vapour
po ethanol = 0.9 × 40 mm Hg = 36 mm Hg
pressure of the pure solvent and ps is the vapour pressure
Total vapour pressure of solution = 3.28 + 36
of the solution. Now, x2 is also represented as n2/(n1 + n2).
= 39.28 mm Hg.
p o − ps n2
3c. Water is added to the solution M such that the mole fraction Thus, we have o
=
p n1 + n2
of water in the solution becomes 0.9. The boiling point of this Neglecting n2 from the denominator, and substituting
solution is po = 1 atm = 760 mm Hg, we get
(a) 380.4 K (b) 376.2 K (c) 375.5 K (d) 354.7 K
760 − ps 2.5/MB 1000 18
Solution = × = ×m (1)
760 100/18 1000 1000
(b) ∆Tb = Kb × m = 0.52 × 0.1 × 1000/(0.9 × 18) = 3.2 K According to elevation in boiling point, ∆Tb = Kb × m ⇒
Boiling point of solution = ∆Tb + boiling point of pure sol- 2 = 0.76 × m ⇒ m = 2/0.76.
vent = 373 + 3.2 = 376.2 K Putting this value in Eq. (1), we get
4. The Henry’s law constant for the solubility of N2 gas in water
760 − ps 18 2 18 × 2 × 760
at 298 K is 1.0 × 105 atm. The mole fraction of N2 in air is 0.8. = × ⇒ ps = 760 −
The number of moles of N2 from air dissolved in 10 mol of 760 1000 0.76 1000 × 0.76
water at 298 K and 5 atm pressure is = 760 − 36 = 724 mm Hg
(a) 4.0 × 10−4 (b) 4.0 × 10−5
(c) 5.0 × 10 −4 (d) 4.0 × 10−6 7. Benzene and naphthalene form an ideal solution at room tem-
(IIT-JEE 2009) perature. For this process, the true statement(s) is (are) _____.
Solution (a) ∆G is positive. (b) ∆Ssystem is positive
(c) ∆Ssurroundings = 0 (d) ∆H = 0
(a) From Henry’s law, we have
(JEE Advanced 2013)
p = K H xN2 ⇒ 0.8 × 5 = 1 × 105 xN2 Solution
−5
⇒ xN2 = 4 × 10 (b), (c), (d) For an ideal solution formed by benzene and naph-
Therefore, in 10 mol of water, moles of N2= 4 × 10−4 thalene, ∆Hmixing = 0, ∆Ssystem > 0 and ∆Ssurroundings = 0.

5. The freezing point (in°C) of a solution containing 0.1 g of 8. MX2 dissociates into M2+ and X− ions in an aqueous solu-
K3[Fe(CN)6] (molecular weight 329 g mol−1) in 100 g of water tion, with a degree of dissociation (a) of 0.5. The ratio of the
(Kf = 1.86°C molal−1) is observed depression of freezing point of the aqueous solution
(a) –2.3 × 10−2 (b) –5.7 × 10−2 to the value of the depression of freezing point in the absence
(c) –5.7 × 10 −2 (d) –1.2 × 10−2. of ionic dissociation is _____.
(IIT-JEE 2011) (JEE Advanced 2014)
Solution
Solution
(a) The depression in freezing point is given by
(2) The reaction involved is
∆Tf = i K f m (1)
where i can be calculated if we consider the following MX2  M2 + + 2X −
reaction Initial moles 1 0 0
+ 3− Moles at equilibrium 1− a a 2a
K 3 [Fe(CN)6 ] → 3K + [Fe(CN)6 ]

Chapter-17.indd 558 8/5/2016 10:45:30 AM


Advanced Level Problems 559

Therefore, i = 1 − a + a + 2a = 1 + 2a . Given that a = 0.5, so the pressure of solution and po is the normal (initial) vapour
i = 1 + 2 × 0.5 = 2. pressure of the solvent.
75 3
Therefore, pressure of solution = × po = po
9. If the freezing point of a 0.01 molal aqueous solution of a 100 4
cobalt(III) chloride ammonia complex (which behaves as a strong (as vapour pressure decreases by 25%).
electrolyte) is −0.0558°C, the number of chloride(s) in the coordi- Also given that MB = 60 g mol−1, MA = 18 g mol−1, wA = 100 g,
nation sphere of the complex is (Kf of water = 1.86 K kg mol−1) therefore, wB = 111 g. Hence, molality is
(JEE Advanced 2015)
Number of moles of solute
Molality = 8.52 molal−1
× 1000 = 18
Solution Weight of solvent
(a) The depression in freezing point is given by
12. Which of the following 0.1 M aqueous solution will have the
∆Tf = K f × m × i lowest freezing point: potassium sulphate, sodium chloride,
0.0558 = 1.86 × 0.01 × i urea and glucose.
i=3 Solution
Therefore, one mole of complex gives three moles of We know that ∆Tf = i × K f × m. Since i value of K2SO4 is high-
ions in solution. Hence, the complex is [Co(NH3)5Cl]Cl2 est (i = 3), so the depression freezing point is maximum, and
and the number of Cl− ions inside the coordination hence freezing point is minimum.
sphere is 1.
13. The freezing point of equimolal aqueous solution will be
10. The molar volume of liquid benzene (density = 0.877 g mL−1) highest for which solution and why?
increases by a factor of 2750 as it vapourizes at 20°C and that C6H5NH3Cl (aniline hydrochloride), Ca(NO3)3, La(NO3)3,
of liquid toluene (density = 0.867 g mL−1) increases by a factor C6H12O6 (glucose)
of 7720 at 20°C. A solution of benzene and toluene at 20°C
Solution
has a vapour pressure of 46.0 torr. Find the mole-fraction of
benzene in the vapour above the solution. As i is smallest for glucose, hence from the expression
∆Tf = i × K f × m, the freezing point will be highest for C6H12O6
Solution (glucose).
In the vapour phase,
78 × 1 14. 0.2 molal acid HX is 20% ionized in solution. Kf = 1.86 K molal−1
1 mol (or 78 g) benzene has volume at 20°C = × 2750 mL then calculate the freezing point of the solution.
0.877
92 × 1 Solution
1 mol (or 92 g) toluene has volume at 20°C = × 7720 mL
0.867 The reaction involved is
The vapour pressures of pure benzene and toluene are
HX  H+ + X −
pBo 78 × 2750 Initial moles 1 0 0
× = 1 × 0.0821 × 293 pBo = 74.74 mm Hg
760 0.877 × 1000 Moles at equilibrium 1 − 0.2 0.2 0.2
Total number of moles at equilibrium = 0.8 + 0.2 + 0.2 = 1.2 = i
pTo 92 × 7720
× = 1 × 0.0821 × 293 pTo = 22.32 mm Hg Therefore, depression in freezing point is
760 0.867 × 1000
∆Tf = i × K f × m = 1.2 × 1.86 × 0.2 = 0.45°C
Given that pmix = 46 mm Hg. So,
So, the freezing point, Tf = 0 − 0.45 = −0.45°C.
pmix = pBo xB + pTo x T ⇒ 46
= 74.74 xB + 22.32(1 − xB ) 15. The molecular weight of benzoic acid in benzene, as deter-
mined, by depression in freezing point method corresponds
Solving, we get xB = 0.45. to which property of benzoic acid?
11. What weight of the non-volatile solute, urea (NH2 CO NH2) Solution
needs to be dissolved in 100 g of water, in order to decrease
Dimerization of benzoic acid takes place as follows in non-
the vapour pressure of water by 25%? What will be the molal-
polar solvent:
ity of the solution.
Solution 2C6H5COOH  (C6H5COOH)2
The lowering of vapour pressure on addition of solute is given Therefore, Mobs and Mexpt can be calculated and related.
by
po − p wB /MB 16. Consider 100 mg of a protein to be dissolved in just enough
= xB = water to make 10.0 mL of solution. If this solution has an
po w A /MA + wB /MB
osmotic pressure of 13.3 mm Hg at 25 °C, what is the molar
Here wB and MB are weight and molecular weight of solute, wA mass of the protein? (Given that R = 0.0821 L atm mol−1 K−1
and MA are weight and molecular weight of solvent. Also, p is and 760 mm Hg = 1 atm.)

Chapter-17.indd 559 8/5/2016 10:45:34 AM


560 Chapter 17 Solutions

Solution Therefore, nA = wA/MA = 7/68.33. Substituting values in Eq. (1),


nRT we get
We know that osmotic pressure (Π) is given by Π = and
V
molar mass of protein is given by p o − ps 7 × 18 760
Given that: Π = 13.3 mm Hg = 13.3/760 = 0.0175 atm; = = 0.0184 ⇒ ps =
ps 68.33 × 100 1.0184
V = 10 mL = 10/1000 = 0.01 L; R = 0.0821 L atm mol−1K−1; T =
= 746.26 mm Hg
25°C = (273 + 25) K = 298 K; wB = 100 mg = 100/1000 g = 0.1 g.
Substituting all the values in Eq. (1), we get 19. To 500 cm3 of water, 3 × 10−3 kg of acetic acid is added. If 23%
0.1× 0.0821× 298 of acetic acid is dissociated, what will be the depression in
MB = = 13980 g mol−1
0.0175 × 0.01 freezing point? (Kf and density of water are 1.86 K kg mol−1
and 0.997 g cm−3, respectively.)
17. 1.22 g of benzoic acid is dissolved in 100 g of acetone and 100 g
Solution
of benzene separately. Boiling point of the solution in ace-
tone increases by 0.17°C, while that in the benzene increases The reaction involved is
by 0.13°C; Kb for acetone and benzene is 1.7 K kg mol−1 and CH3COOH  CH3COO − + H+
2.6 K kg mol−1. Find molecular weight of benzoic acid in the
Initial moles 1 0 0
two cases and justify your answer.
Moles at equilibrium 1−a a a
Solution
Total number of moles at equilibrium = 1 − a + a + a
Given that 1 kg acetone (Kb = 1.7 K kg mol−1), 1 kg benzene (Kb
⇒ i = 1 + a = 1 + 0.23 = 1.23.
= 2.6 K kg mol−1), wbenzoic = 1.22 g, wacetone = 100 g, wbenzene =
1.23 × 1.86 × 3 × 10 −3 × 103
100 g, ∆Tb(acetone) = 0.17°C and ∆Tb(benzene) = 0.13°C. Hence, ∆Tf = iK f × m = = 0.229°C
60 × 500 × 0.997
1000K b × wB 103
We use the expression ∆Tb =
w A × MB
where wB = wbenzoic = 1.22 g, MB is molecular weight of ben- 20. 0.004 M Na2SO4 is isotonic with 0.01 M glucose. The degree of
zoic acid and wA = wacetone or wbenzene. Hence, substituting dissociation of Na2SO4 is
the values, we get (a) 75% (b) 50% (c) 25% (d) 85%
1000 × 1.7 × 1.22 Solution
For acetone solution: MB = = 122 g mol−1
0.17 × 100 (a) We know that Π(Na2SO 4 ) = iCRT = i (0.004) RT and
1000 × 2.6 × 1.22 Π(glucose ) = CRT = 0.010 RT . As the solutions are iso-
For benzene solution: MB = = 244 g mol−1
0.13 × 100 tonic, which means they have same osmotic pressure, we
have
Molecular weight of C6H5COOH is 122 and thus it is evident i (0.004 )RT = 0.01 RT
that benzoic acid remains as normal molecular species in ace- Solving, we get i = 2.5. Now, consider the reaction
tone but shows 100% dimerization in C6H6, that is, in C6H6 it
exists as (C6H5COOH)2. Na2SO 4  2Na+ + SO24−
18. The degree of dissociation of Ca(NO3)2 in a dilute aqueous Initial moles 1 0 0
solution, containing 7.0 g of the salt per 100 g water at 100°C Moles at equilibrium 1 − a 2a a
is 70%. If the vapour pressure of water at 100°C is 760 mm Hg, Total number of moles at equilibrium = 1 − a + 2a + a
calculate vapour pressure of the solution. ⇒ i = 1 + 2a.
Solution Therefore, a = (i − 1)/2 = (2.5 − 1)/2 = 0.75. So, the degree
The reaction involved is of dissociation is 75%.

Ca(NO3 )2  Ca2 + + 2NO23 − 21. The elevation in boiling point of a solution of 13.44 g of CuCl2
Initial moles 1 0 0 in 1 kg of water using the following information will be (mol.
wt. of CuCl2 = 134.4 g mol−1 and Kb = 0.52 K molal−1.)
Moles at equilibrium 1−aa a 2a (a) 0.16°C (b) 0.05°C (c) 0.1°C (d) 0.2°C
Total number of moles at equilibrium = 1 − a + a + 2a ⇒ i = 1
Solution
+ 2a = 1 + 2(0.7) = 2.4
According to Raoult’s law, (a) The reaction involved is

p o − ps nA CuCl2  Cu2 + + 2Cl−


= xA = (1) Initial moles 1 0 0
po nA + nB
Moles at equilibrium 1 − a a 2a
Given that po = 760 mm Hg, nA = wA/MA = 7/164, nB = wB/MB
= 100/18. However, due to abnormal molar mass, the molar Total number of moles at equilibrium = 1 − a + a + 2a
mass calculated for Ca(NO3)2 is ⇒ i = 1 + 2a = 3 (assuming a = 100%).
Therefore,
Mobs 164
= 1 + 2a ⇒ Mcalc = = 68.33 g mol−1 13.44 / 134.4
Mcalc 2.4 ∆Tb = i × K b × m = 3 × 0.52 × = 0.16°C
1

Chapter-17.indd 560 8/5/2016 10:45:39 AM


Practice Exercise 561

22. 1000 g of 1 molal sucrose solution in water is cooled to Now,


−3.72°C. What weight of ice would be separated out at this Weight of sucrose 1000
temperature? (Given that Kf of H2O = 1.86 K molal−1.) ∆Tf = K f × ×
Gram molecular weight of sucrose Weight of water
(a) 372.5 g (b) 329 g (c) 285.4 g (d) 500 g
254.84 1000
Solution 3.72 = 1.86 × ×
342 W
(a) 1 molal implies that 1 mol or 342 g of sucrose is present in W = 372.57 g
1000 g water or 1342 g solution.
342 Therefore, weight of water protected by sucrose from freezing
Mass of sucrose in1000 g solution = × 1000 = 372.57 g
1342
Therefore, weight of water separated as ice = 745.16 − 372.57
= 254.84 g
= 372.59 g
Therefore, mass of water in the solution = 1000 − 254.84 = 745.16 g
23. Benzene and toluene form nearly ideal solutions. At 300 K,
mass of water in the solution = 1000 − 254.84 = 745.16 g o o
ptoluene = 32 mm Hg and pbenzene = 103 mm Hg. A liquid mix-
Now, ∆Tf = K f × Molality = 1.86 × 1 = 1.86°C ture is composed of 3 mol of toluene and 2 mol of benzene.
On reducing pressure isothermally over the mixture at 300 K,
This implies that solution starts to freeze at −1.86°C, but the mixture vapourizes in a certain range of pressure. The
sucrose will remain in the solution. mole fraction of benzene in the last drop of liquid left is
Consider the molality of sucrose solution at −3.72°C as (a) 0.17 (b) 0.4 (c) 0.6 (d) 0.68
m′. Therefore, Solution
∆T 3.72 (a) On decreasing pressure over the liquid mixture, the first
∆Tf = K f × m′ ⇒ m′ = f = = 2 molal
K f 1.86 vapour occurs at 60.4 mm Hg. Further decrease in pres-
sure increases the vapour content of the mixture. Finally,
Now, since its molality becomes double, the mass of the
when the last drop of liquid is left, its vapour phase con-
solvent left will be half. Hence, the amount of ice sepa-
tains almost the same composition as it was for liquid
rated out is half of the initial mass of water.
phase in the beginning.
Initial water 2
Mass of frozen water or ice = The initial mole fractions for benzene = = 0.4 ; and for
2 toluene = 0.6. 5
745.16 For the vapour phase in equilibrium with the last liquid
= = 372.58 g
2 drop, mole fractions: yB = 0.4 ; y T = 0.6
Alternate Solution From Raoult’s law and Dalton’s law for liquid-vapour equi-
1 molal implies that 1 mol (or) 342 g of sucrose is present libria, the mole fraction of benzene in liquid and vapour
in 1000 g of water or 1342 g of solution. phases are related by
Therefore, mass of sucrose in 1000 g solution
1  pTo  pTo 1
342 = 1 −  + ×
= × 1000 = 254.84 g yB  pBo  pBo xB
1342
Therefore, mass of water in the solution = 10000 − 254.84 1  32  32  1 
⇒ = 1 − + ⇒ xB = 0.17
= 745.16 g 0.4  103  103  xB 

PRACTICE EXERCISE
Level I (a) The solution shows negative deviation.
(b) A B interactions are stronger than A A and B B.
Single Correct Choice Type (c) The solution is an ideal solution.
1. If two substances A and B have pAo : pBo = 1: 2 and have mole (d) The solution shows positive deviation.
fraction in solution 1:2, then the mole fraction of A in vapours is 3. Which of the following aqueous solutions will have the lowest
(a) 0.33 (b) 0.25 (c) 0.52 (d) 0.2 freezing point?
2. The following figure represents boiling point composition dia- (a) 0.10 M KCl (b) 0.10 M Al2(SO4)3
gram of solution of component A and B, what is true among (c) 0.10 M C6H12O6 (d) 0.10 M C12H22O11
the following?
4. What is the composition of the vapour that is in equilibrium at
TAo 30°C with a benzene–toluene solution with a mole fraction of
TBo benzene of 0.400? ( pBo = 119 torr and pTo = 37.0 torr)
T(K) (a) 1.237 (b) 2.237 (c) 3.237 (d) 0.237
5. A 0.0020 molal aqueous solution of an ionic compound
CO(NH3)5(NO2)Cl freezes at −0.00732°C. Number of moles of
ions which 1 mol of ionic compound produces on being dis-
xA = 1 xA = 0
xB = 0 xB = 1 solved in water will be (Kf = −1.86°C molal−1)
(a) 1 (b) 4 (c) 3 (d) 2

Chapter-17.indd 561 8/5/2016 10:45:44 AM


562 Chapter 17 Solutions

6. Phenol associates in benzene to a certain extent to form dimer. (c) number of solute particles present in it.
A solution containing 2.0 × 10−2 kg of phenol in 1.0 kg of (d) number of moles of solvent only.
benzene has its freezing point decreased by 0.69 K. What is
the degree of association of phenol? (Kf for benzene = 5.12 17. Vapour pressure of water at 293 K is 17.535 mm Hg. What will
K molal−1) be the vapour pressure of water at 293 K when 25 g of glucose
is dissolved in 450 g of water?
(a) 0.734 (b) 0.367 (c) 0.437 (d) 0.763
(a) 17.44 mm Hg (b) 174.4 mm Hg
7. The molal freezing point constant of water is 1.86°C molal−1. (c) 34.88 mm Hg (d) 8.72 mm Hg
Therefore, the freezing point of 0.1 M NaCl solution in water is
expected to be 18. The azeotropic mixture of water (boiling point 100°C) and HCl
(a) −1.86°C. (b) −0.186°C. (boiling point 85°C) boils at 108.5°C. When this mixture is dis-
(c) −0.372°C. (d) +0.372°C. tilled, it is possible to obtain
(a) pure HCl.
8. Which of the following pairs of electrolytes will form an iso- (b) pure water.
tonic solution (assume complete dissociation of each salt)? (c) pure water as well as HCl.
(a) 0.02 M K2SO4 ⋅ Al2(SO4)3 and 0.04 M BaCl2 (d) neither HCl nor water in their pure states.
(b) 0.06 M CaCl2 and 0.04 M FeCl3
(c) 0.05 M Al2(SO4)3 and 0.05 M K2SO4 19. Which of the following solutions will have the highest boiling
(d) 0.04 M K4[Fe(CN)6] and 0.05 M K3[Cu(CN)4] point?
(a) 1% glucose (b) 1% sucrose
9. Which of the following statements is correct, if the intermo-
(c) 1% NaCl (d) 1% CaCl2
lecular forces in liquids A, B and C are in the order A < B < C?
(a) B evapourates more readily than A. 20. A solution of a pair of volatile liquids A and B shows negative
(b) B evapourates less readily than C. deviation from Raoult’s law. Which of the following holds true
(c) A and B evapourate at the same rate. for it?
(d) A evapourates more readily than C.
(a) pA < x A pAo and pB < xB pBo .
10. Which will form maximum boiling azeotrope? (b) The intermolecular forces A A, B B < A B.
(a) HNO3 + H2O solution (c) Both ∆Hmix and ∆Vmix are negative.
(b) C2H5OH + H2O solution (d) All of these.
(c) C6H6 + C6H5CH3 solution
(d) C6H6 + H2O solution 21. The vapour pressure of pure benzene at 25°C is 639.7 mm Hg
and the vapour pressure of a solution of a solute in C6H6 at the
11. Which of the following colligative properties can help to deter- same temperature is 631.9 mm Hg. The molality of solution
mine the molar mass of a protein with the greatest precision? will be
(a) Elevation in boiling point. (a) 0.15 (b) 0.258 (c) 0.358 (d) 1.58
(b) Depression in freezing point.
(c) Osmotic pressure. 22. Certain substance trimerizes when dissolved in a solvent A.
(d) Relative lowering of vapour pressure. The van’t Hoff factor is (assume 100% association)
(a) 1 (b) 1/3
12. Equal volumes of ethylene glycol (molar mass = 62 g mol−1)
(c) 3 (d) Cannot be predicted.
and water (molar mass = 18 g mol−1) are mixed. The depres-
sion in freezing point of water is (Given Kf of water = 1.86 K 23. A non-ideal solution was prepared by mixing 30 mL chloro-
molal−1 and specific gravity of ethylene glycol is 1.114.) form and 50 mL acetone. The volume of mixture will be
(a) 0.0033 K (b) 3.33 K (c) 0.333 K (d) 33.3 K (a) > 80 mL. (b) < 80 mL. (c) = 80 mL. (d) − 80 mL.
13. van’t Hoff factors x, y, and z for association, dissociation and 24. 100 g of liquid A (molar mass 140 g mol−1) was dissolved in
no change of solute in the solution, respectively, are in the 1000 g of liquid B (molar mass 180 g mol−1). The vapour pres-
order: sure of pure liquid B was found to be 500 torr. What will be
(a) x < y < z (b) x > z > y (c) x < z < y (d) x > y > z the vapour pressure of pure liquid A and its vapour pressure
14. Liquids A and B form an ideal solution and the former has in the solution, respectively, if the total vapour pressure of the
stronger intermolecular forces. If xA and yA are the mole frac- solution is 475 torr?
tions of A in the solution and vapour phase in equilibrium, (a) 28.7 torr and 32 torr (b) 272.7 torr and 30 torr
then (c) 28.7 torr and 3.2 torr (d) 280.7 torr and 3.2 torr
(a) yA/xA = 1 (b) yA/xA > 1
25. The van’t Hoff factor for BaCl2 at 0.01 M concentration is 1.98.
(c) yA/xA < 1 (d) yA + xA = 1
The percentage dissociation of BaCl2 at this concentration is
15. 0.01 M solution each of urea, common salt, and Na2SO4 are (a) 49% (b) 69% (c) 89% (d) 98%
taken. The ratio of depression of freezing point is
26. Two liquids A and B are mixed at temperature T in a certain
(a) 1:1:1 (b) 1:2:1 (c) 1:2:3 (d) 2:2:3
ratio to form an ideal solution; it was found that the vapour
16. The colligative properties of a solution depend on pressure of A, that is, pA is equal to pB the vapour pressure of B
(a) nature of solute particles present in it. for the liquid mixture. What is the total vapour pressure of the
(b) nature of solvent used. liquid mixture in terms of pAo and pBo ?

Chapter-17.indd 562 8/5/2016 10:45:45 AM


Practice Exercise 563

pAo pBo 2pAo pBo pAo 2pBo (c) p (d)


(a) (b) (c) (d)
pAo + pBo pAo + pBo pAo + pBo pAo + pBo p

27. The vapour pressure of a solvent decreases by 10 mm of mer-


cury when a non-volatile solute was added to the solvent. The
mole fraction of the solute in the solution is 0.2. What should
be the mole fraction of the solvent if the decrease in vapour xA = 1 xA = 0
pressure is to be 20 mm Hg? xA = 1 xA = 0
xB = 0 xB = 1
(a) 0.8 (b) 0.6 (c) 0.4 (d) 0.4 xB = 0 xB = 1

28. The correct relationship between molarity (M) and molality


35. In 100 g of naphthalene, 2.423 g of S was dissolved. Melting
(m) is (r = density of the solution, in kg L−1, M2 = molar mass
point of naphthalene = 80.1°C, ∆Tf = 0.661°C, Kf = 35.7 cal g−1
of the solute in kg mol−1)
of naphthalene. Molecular formula of sulphur added is
mr m
(a) M = (b) M = (a) S2 (b) S4 (c) S6 (d) S8
1 + mM2 1 + mM2 r
1 + mM2 1 + mr 36. van’t Hoff factor for 0.1 M ideal solution is
(c) M = (d) M = (a) 0.1 (b) 1 (c) −0.01 (d) 0.01
mr mM2

29. In a mixture of A and B if the mole fraction of the component 37. A solution is obtained by mixing 300 g of 25% solution and
A in vapour phase is x1 and mole fraction of component A 400 g of 40% solution by mass. What will be the mass percent-
in liquid mixture is x2 ( pAo = vapour pressure of pure A; pBo = age of the solute and solvent of resulting solution?
vapour pressure of pure B), then total vapour pressure of the (a) 23% and 77% (b) 12.5% and 87.5%
liquid mixture is: (c) 33.6% and 66.4% (d) 50% and 50%
po x po x po x po x 38. The elevation in boiling point of a solution of 13.44 g of CuCl2
(a) A 2 (b) A 1 (c) B 1 (d) B 2
x1 x2 x2 x1 in 1 kg of water is (Given Kb = 0.52 K molal−1, molecular weight
of CuCl2 = 134.4 g mol−1.)
30. 0.15 g of a substance dissolved in 15 g of a solvent is boiled
(a) 0.05 K (b) 0.1 K (c) 0.16 K (d) 0.21 K
at a temperature higher by 0.216°C than that of the pure sol-
vent. Find out the molecular weight of the substance. (Kf for 39. Molarity of the liquid HCl if density of the solution is 1.17 g
solvent is 2.16 K kg mol−1.) cm−3 is
(a) 1.01 g mol−1 (b) 10.1 g mol−1 (a) 36.5 M (b) 18.25 M (c) 32.05 M (d) 42.10 M
(c) 100 g mol−1 (d) 1000 g mol−1
40. A 0.004 M solution of Na2SO4 is isotonic with a 0.010 M solu-
31. For a weak monobasic acid, if pKa = 4, then at a concentration tion of glucose at the same temperature. The apparent degree
of 0.01 M of the acid solution, the van’t Hoff factor is of dissociation of Na2SO4 is
(a) 1.01 (b) 1.02 (c) 1.10 (d) 1.20 (a) 25% (b) 50% (c) 75% (d) 85%
32. A 25 mL of certain aqueous solution of KCl required 40 mL of
41. The pH of 1 M solution of a weak monobasic acid (HA) is 2.
0.5 M AgNO3 solution for complete precipitation of Cl− ions. If
Then, the van’t Hoff factor is
KCl ionizes to a complete extent in solution, the freezing point
(a) 1.01 (b) 1.02 (c) 1.10 (d) 1.20
of the solution is (assume molality to be same as molarity and
Kf = 1.86°C molal−1) 42. Boiling point of pure H2O is 373.15 K. If 32.5 g of KCN is dis-
(a) −0.27°C (b) −5°C (c) −2.97°C (d) −5.95°C solved in 100 mL of H2O, what will the boiling point of solu-
33. Benzoic acid undergoes dimerization in benzene solution. tion? (Given Kb for H2O = 0.52 K kg mol−1 and molar mass of
The van’t Hoff factor (i) is related to the degree of association KCN = 65 g mol−1.)
“x” of the acid as (a) 105.20°C (b) 100.52°C (c) 373.67 K (d) 273.67 K
(a) i = (1 − x) (b) i = (1 + x)
(c) i = (1 − x/2) (d) i = (1 + x/2) 43. Depression in freezing point of 0.01 molal aqueous acetic acid
solution is found to be 0.02046°C. One molal urea solution
34. Which of the following graphs represent the positive devia- freezes at −1.86°C. Assuming molarity equal to molality, pH
tion from Raoult’s law? of acetic acid solution is
(a) (b) (a) 2 (b) 3 (c) 3.2 (d) 4.2
p p
44. Vapour pressure of CCl4 at 25°C is 143 mm Hg. If 0.5 g of a
non-volatile solute (molecular weight 65) is dissolved in
100 mL CCl4, find the vapour pressure of the solution. (Given
that density of CCl4 = 1.58 g cm−3).
(a) 141.93 mm Hg (b) 94.39 mm Hg
xA = 1 xA = 0 xA = 1 xA = 0 (c) 199.34 mm Hg (d) 143.99 mm Hg
xB = 0 xB = 1 xB = 0 xB = 1

Chapter-17.indd 563 8/5/2016 10:45:49 AM


564 Chapter 17 Solutions

Level II (c) For association of a solute in a solution, i > 1.


(d) The ratio of van’t Hoff factors for 0.2 M glucose and 0.1 M
Multiple Correct Choice Type
sucrose is 2:1.
45. Which of the following statements are true about osmotic
53. For 0.5 molal aqueous solution of KCl, the important physical
pressure (Π), volume (V), and temperature (T)?
properties at 27°C are Kf (water) = 1.86°C molal−1, Kb(water) =
(a) Π ∝1/V if T is constant.
0.512°C molal−1. Hence, the
(b) Π ∝ T if V is constant
(a) freezing point of solution = −3.72°C
(c) Π ∝ V is T is constant.
(b) boiling point of solution = 100.512°C
(d) ΠV is constant if T is constant.
(c) osmotic pressure = 3.76 atm
46. Which of the following aqueous solutions are isotonic (R = (d) observed molecular mass = 37.25 (approximately assum-
0.082 atm K−1 mol−1)? ing degree of dissociation = 1).
(a) 0.01 M glucose.
54. Non-ideal solutions showing negative deviations are
(b) 0.01 M NaNO3.
(a) acetone + ethyl alcohol.
(c) 500 mL solution containing 0.3 g urea.
(b) acetic acid + pyridine.
(d) 0.04 N HCl.
(c) chloroform + benzene.
47. At constant temperature, the osmotic pressure of a solution is (d) carbon tetrachloride + toluene.
(a) directly proportional to the concentration.
55. The solution showing positive deviation
(b) inversely proportional to the molecular weight of the
(a) have ∆Vmix = positive.
solute.
(b) have ∆Hmix = negative.
(c) directly proportional to the square of the concentration.
(c) form minimum boiling azeotropes.
(d) directly proportional to the square root of the
(d) have lower vapour pressure of each component in the
concentration.
solution than their pure vapour pressure.
48. Which of the following are correct about Henry’s constant, KH?
56. Vapour pressure of a liquid depends upon the
(a) Greater the value of KH, lower is the solubility of the gas at
(a) temperature. (b) surface area.
the same pressure and temperature.
(c) nature of liquid. (d) external pressure.
(b) KH decreases with increase of temperature.
(c) The unit of KH is bar. 57. Which of the following statements are wrong?
(d) All noble gases have the same value of KH at the same (a) The solvent rises from soil to the top of a tall tree due to
temperature. osmosis.
(b) The aqueous solution of NaCl freezes at lower tempera-
49. In an experiment for freezing point depression, which of the
ture than water.
following would not be observed?
(c) The value of colligative property decreases when solute
(a) The vapour pressure of the solution is more than that of
undergoes dissociation.
pure solvent.
(d) For AlCl3, the van’t Hoff factor is 3.
(b) Only solvent molecules solidify at the freezing point.
(c) The vapour pressure of the solution is less than that of 58. An ideal solution is formed, when
pure solvent. (a) its components have the same intermolecular attractions
(d) Only solute molecules solidify at the freezing point. in solution as in pure components.
(b) ∆Hmix = 0.
50. Which of the following statements are correct about the solu- (c) ∆Vmix > 0.
bility of gases in liquids? (d) ∆Smix > 0.
(a) Mole fraction of the gas in the solution is directly propor-
tional to the partial pressure of the gas above the solution. 59. The colligative properties of a solution are
(b) Volume of the gas dissolved measured at the pressure (a) proportional to molality.
used is independent of the pressure of the gas. (b) proportional to number of molecules/ions.
(c) Solubility of gas is always an exothermic process. (c) proportional to each other.
(d) Gibbs energy change of dissolution of a gas may be posi- (d) independent of the nature of the solute, that is, electro-
tive or negative. lyte or non-electrolyte.

51. Which of the following form nearly ideal solution? Passage Type
(a) Chlorobenzene + Bromobenzene
(b) Hexane + Heptane Paragraph for Questions 60 to 63: The properties of solutions
(c) Ethanol + Cyclohexane that depend on the number of solute particles irrespective of
(d) Acetic acid + Pyridine their nature are called colligative properties. These properties
hold good if the solutions are dilute and solute molecules do not
52. Identify the correct statements. dissociate or associate in solution. Abnormal molar masses are
(a) The solution formed by mixing equal volumes of 0.1 M urea obtained by measuring colligative properties if the solute disso-
and 0.1 M glucose will have the same osmotic pressure. ciates or associates in the solution. The extent of dissociation or
(b) 0.1 M K4[Fe(CN)6] and 0.1 M Al2(SO4)3 are isotonic association of a solute in solution is expressed in terms of van’t
solutions. Hoff factor i. It is defined as the ratio of experimentally determined

Chapter-17.indd 564 8/5/2016 10:45:50 AM


Practice Exercise 565

value of colligative property to the calculated value of colligative have positive deviations. On the other hand, if the forces of inter-
property based on the theoretical relations (Raoult’s law and ideal action between A and B molecules are more than those between
behaviour). A A and B B, the non-ideal solutions have negative deviations.
Normal molar mass 64. If two liquids A and B form an ideal solution, then
i=
Observed molar mass (a) the Gibbs energy of mixing is zero.
For solutes that associate or dissociate in solution, the equation (b) the entropy of mixing is zero.
of colligative properties need to be modified to include the van’t (c) the enthalpy of mixing is zero.
Hoff factor. (d) None of these.

∆p 65. Vapour pressure of dilute aqueous solution of glucose is


= i x solute ∆Tf = iK f m 750 mm Hg at 373 K. The mole fraction of the solute is
p
(a) 1/75 (b) 75/76 (c) 1/76 (d) 1/10
∆Tb = i K b m Π = iCRT
66. The vapour pressure of a pure liquid A is 30 mm Hg at 300 K.
The vapour pressure of this liquid in solution with liquid B is
60. A solution contains non-volatile solute of molecular mass M2.
24 mm Hg. The mole fraction of A in solution obeying Raoult’s
Which of the following can be used to calculate the molecular
law is
mass of the solute in terms of osmotic pressure?
(a) 0.4 (b) 0.3 (c) 0.5 (d) 0.8
m  m  RT
(a) M2 = 2 VRT (b) M2 =  2  67. Which of the following statements is correct for non-ideal
Π  V  Π
solutions?
m  m  Π (a) For solutions showing positive deviations, ∆Vmix is nega-
(c) M2 =  2  ΠRT (d) M2 =  2 
 V   V  RT tive but ∆Hmix is positive.
(where V is volume of the solution, m2 is mass of solute and Π (b) For solutions showing negative deviations, the interac-
is osmotic pressure). tions between the components are greater than the pure
components.
61. The molal boiling point constant for water is 0.513°C kg (c) For solutions showing negative deviations, ∆Vmix is posi-
mol−1. When 0.1 mol of sugar is dissolved in 200 g of water, tive but ∆Hmix is negative.
the solution boils at (d) For solutions showing negative deviations, ∆Vmix and
(a) 100.513°C (b) 100.0513°C ∆Hmix are positive.
(c) 100.256°C (d) 101.025°C
68. Vapour pressure of a solution of heptane and octane is given
62. Which of the following equimolar solution is expected to by the equation: p(solution) (mm Hg) = 30 + 70x, where x is the
have the lowest freezing point? mole fraction of heptane. Vapour pressure of pure octane is
(a) 0.5 M H2SO4 (b) 0.5 M Al2(SO4)3 (a) 100 mm Hg (b) 70 mm Hg
(c) 0.5 M HCl (d) 0.5 M acetic acid (c) 30 mm Hg (d) 1.86 mm Hg
63. The elevation in boiling point for 1 M aniline, sucrose, 1 M KCl
Paragraph for Questions 69 to 72: The colligative properties are
and 1 M CaCl2 are in the ratio:
properties that depend only on the number of solute particles in
(a) 1:1:2:3 (b) 3:2:1:1 (c) 1:2:3:4 (d) 2:2:3:4
a solution, but are independent of the nature of those particles.
However, they are applicable only to dilute solutions in which the
Paragraph for Questions 64 to 68: The pressure exerted by the
solutes do not undergo any association or dissociation. The extent
vapour when in equilibrium with a liquid or a solution at a particu-
of dissociation or association of a solute in solution is expressed
lar temperature is known as vapour pressure. When two (or more)
in terms of van’t Hoff factor i. This factor explains the abnormal
components of a liquid solution evaporate, the vapours contain
molecular masses of such solutes in the solution and also helps in
molecules of each substance. All liquid solutions of non-volatile
determining the degree of association or dissociation
solutes (solutes that have no tendency to evaporate) have lower
vapour pressures than their pure solvents. The vapour pressure of 69. A substance exists as a trimer in solution. The van’t Hoff factor
such a solution is proportional to how much of the solution actu- i for the solution is
ally consists of the solvent. This proportionality is given by Raoult’s (a) 1 (b) 1/3
law (also known as the vapour pressure–concentration law) which (c) 3 (d) Cannot be predicted
says that the vapour pressure of the solution equals the mole frac-
tion of the solvent multiplied by its vapour pressure when pure. 70. Benzoic acid exists as a dimer in benzene solution. The rela-
In a binary solution of components A and B, if the interactive tion between the van’t Hoff factor i and the degree of associ-
forces between A A, B B and A B are same the solution is ideal. ation a of the acid is
Practically, no solution is ideal. A non-ideal solution is that solution (a) i = 1 − a (b) i = 1 + a
in which solute and solvent molecules interact with one another (c) i = (1 + a/2) (d) i = (1 + a)/2
with a different force than the forces of interaction between the 71. The van’t Hoff factor of 0.1 M K4[Fe(CN)6] is 3.4 in solution. The
molecules of the pure components. If for the two components value of degree of dissociation is
A and B, the forces of interaction between A and B molecules are (a) 40% (b) 60%
less than those between the A A and B B, the non-ideal solutions (c) 56% (d) 44%

Chapter-17.indd 565 8/5/2016 10:45:52 AM


566 Chapter 17 Solutions

72. For an aqueous solution of a non-electrolyte, the value of Integer Type


van’t Hoff factor is
75. At a certain temperature, the vapour pressure (in mm Hg), of
(a) always equal to 0. (b) less than or equal to one.
CH3OH and C2H5OH solution is represented as: p = 126x + 142
(c) always equal to one. (d) cannot be predicted
where x is the mole fraction of CH3OH. The vapour of solution
Matrix-Match Type containing equal moles of CH3OH and C2H5OH is y + 200 mm
Hg. The value of y is ______.
73. Match the behaviour exhibited with the solutions
76. van’t Hoff factor of an electrolyte A2B3 assuming that it ionizes
Column I Column II 75% in the solution is _____.
(a) Show ideal behaviour (p) Chloroform + benzene 77. The depression in freezing point for 1 M urea, 0.5 M glucose,
(b) Show negative deviation (q) Chloroform + Acetone 1 M NaCl, and 1 M K2SO4 are in the ratio x : 1 : y : z. The value of
from ideal behaviour x + z is _____.
(c) Show positive deviation (r) Benzene + toluene 78. The number of solutions among the following which will boil
from ideal behaviour above 373 K is____.
0.1 M NaCl, 0.1 M glucose, 0.1 M BaCl2 solution, 0.1 M Al2(SO4)3,
(d) Non-ideal solution (s) Carbon tetrachloride
0.1 M KNO3, 0.1 M Na2SO3, 0.1 M K4[Fe(CN)6], 0.1 M KCl
+ chloroform
79. An aqueous solution containing 5% by weight of urea and
10% by weight of glucose. What will be the ∆Tf of solution?
74. Match the van’t Hoff factor with the type of solution.
(Given that Kf for H2O is 1.86°C kg mol−1).
Column I Column II
80. The elevation in boiling point for 0.3 molal Al2(SO4)3 solution
(a) i > 1 (p) Solution of NaCl + H2O as compared to elevation in boiling point of 0.1 molal solu-
(b) i < 1 (q) Colloidal solution tion of Na2SO4 is ____ times.
(c) i = 1 (r) Not possible 81. The van’t Hoff factor i for the species [Fe(H2O)2(CN)5]NO3⋅2H2O.
(d) i = 0 (s) Solution of glucose + H2O is ____.

ANSWER KEY
Level I
1. (d) 2. (d) 3. (b) 4. (d) 5. (d) 6. (a)
7. (c) 8. (d) 9. (d) 10. (a) 11. (c) 12. (d)
13. (c) 14. (c) 15. (c) 16. (c) 17. (a) 18. (d)
19. (d) 20. (d) 21. (d) 22. (b) 23. (b) 24. (b)
25. (a) 26. (b) 27. (b) 28. (a) 29. (a) 30. (c)
31. (c) 32. (c) 33. (c) 34. (d) 35. (d) 36. (b)
37. (c) 38. (a) 39. (c) 40. (c) 41. (a) 42. (a)
43. (b) 44. (a)

Level II
45. (a), (b), (d) 46. (a), (c) 47. (a), (b) 48. (a), (c) 49. (a), (d) 50. (a), (c)
51. (a), (b) 52. (a), (b) 53. (b), (d) 54. (b), (c) 55. (a), (d) 56. (a), (c)
57. (c), (d) 58. (a), (b), (d) 59. (a), (b), (c) 60. (b) 61. (c) 62. (b)
63. (a) 64. (c) 65. (c) 66. (d) 67. (b) 68. (c)
69. (b) 70. (c) 71. (b) 72. (d) 73. (a) → r; (b) → q; (c) → s; (d) → p, q, s
74. (a) → p, q; (b) → p; (c) → s; (d) → r 75. (5) 76. (4) 77. (8) 78. (8)
79. (3) 80. (5) 81. (2)

HINTS AND EXPLANATIONS


Level I we get
Single Correct Choice Type pAo x A pAo x A 1
yA = = =
pA x A + pBo xB
o  p o
x  p o
x
1. (d) Given that pAo : pBo = 1: 2 and xA : xB = 1:2. From the pAo x A  1 + Bo B  1 + Bo × B
 pA x A  p x A
expression A

ptotal = pAo x A + pBo xB 1 1


= = = 0.2
2 2 5
1+ ×
1 1

Chapter-17.indd 566 8/5/2016 10:45:53 AM


Hints and Explanations 567

2. (d) It shows a positive deviation, that is, higher vapour pres- Therefore the freezing point of 0.1 M NaCl in water
sure than predicted. = 0 − 0.372 = − 0.372 °C.
3. (b) Depression in freezing point is a colligative property 8. (d) C × i = constant implies isotonic solutions.
which depends on the number of particles of solute in 0.04 × 5 = 0.2 for K4[Fe(CN)6] and 0.05 × 4 = 0.2 for
solution which is highest in Al2(SO4)3. The reaction for K3[Cu(CN)6]
Al2(SO4)3 is as follows: 9. (d) Due to weakest intermolecular forces; evapouration is
3+ fastest in A.
Al2 (SO 4 )3  2Al + 3SO24−
10. (a) HNO3 + H2O will show negative deviation from Raoult’s
The total number of particles for Al2(SO4)3 is 5. law. In this case, we will observe a maximum boiling point
Proceeding in a similar way, for KCl, number of particles azeotrope in which the boiling point of the azeotrope is
is 2, so the depression of freezing point is less than that higher than pure H2O and that of nitric acid HNO3.
of Al2(SO4)3. 11. (c) Osmotic pressure is used for molar mass determination
Since C6H12O6 and C12H22O11 cannot dissociate into of protein because even a small concentration of protein
ions, the depression in freezing point is the least for (or any solute) produces a relatively large osmotic pres-
them. sure which can be measured precisely. It is given by
4. (d) Total pressure of the solution is given by (Raoult’s law) wRT
Π=
ptotal = xB pBo + x T pTo = 0.4 × 119 + 0.6 × 37 M ×V
= 47.6 + 22.2 = 69.8 torr where w is weight of protein, R is the gas constant, T is
the temperature, M is the molecular mass of protein and
Applying Dalton’s law for mole fraction in vapour phase, V is the volume of the solution.
we get
12. (d) Using the expression ∆Tf = K f × molality. Let us consider
pB po x 0.4 × 119
yB = = o B Bo = = 0.763 1000 mL of ethylene glycol. The number of moles of eth-
ptotal pA x A + pB xB 69.8
ylene glycol is calculated as
y T = 1 − 0.763 = 0.237
1000mL × 1.114 g mL−1 = 1.114 g
5. (d) We know that ∆Tf = i × Kf × m, so 0.00732 = i × 1.86 1114 g
× 0.002. Therefore, i = 1.967 ≅ 2. = 17.9 mol.
62g mol−1
6. (a) Mass of phenol, w2 = 2.0 × 10−2 kg; Kf = 5.12 K Given that Kf = 1.86°C molal−1 and substituting value, we
molal−1; mass of benzene, w1 = 1.0 kg; ∆Tf = 0.69 K. get ∆Tf = 1.86 × 17.9 = 33.33 K .
Therefore, according to the equation
13. (c) For association i < 1, for dissociation i > 1, and in normal
1000 K f w2 case, i = 1 (non-electrolyte).
M2 (observed) =
w1 ∆Tf In case of association, the number of particles in the
1000 g kg−1 × 5.12 K kg mol−1 × 2.0 × 10 −2 kg solution decreases and so the value of i < 1. For example,
= benzoic acid undergoes association to form a dimer.
1.0 kg × 0.69 K
In case of dissociation, there is increase in the number of
10.240 × 10 −2 −2 ions and the value of i > 1. For example, KCl dissociates as
= = 14840.58 × 10
0.69 K+ + Cl− where i = 2.
= 148.4 g mol −1 In case of non-electrolyte, there will be no association or
dissociation and the number of particles will be the same
Also, M2 for C6H5OH = 12 × 6 + 1 × 5 + 16 + 1 = 94 g mol −1 and hence i = 1. So x < z < y
(calculated). Therefore, p po x y po
14. (c) y A = A = A A ⇒ A = A < 1
M (calculated) 94 p p xA p
i= 2 = = 0.633
M2 (observed) 148.4 (Since the liquid A is less volatile, pAo < p)
2C6H5OH  (C6H5OH)2 15. (c) ∆Tf = i K f × m ⇒ ∆Tf ∝ i where i = 1 for urea, i = 2 for NaCl
Initial moles 1 mol and i = 3 for Na2SO4. Therefore, ratio of number of mole-
Moles after dissociation 1−a a /2 cules after ionization = 1:2:3
16. (c) The colligative properties of a solution depend on the
a a
Total moles after dissociation = 1− a + = 1− . number of solute particles irrespective of nature.
2 2
Therefore, 17. (a) Relative lowering of vapour pressure is
25
1− (a / 2) p o − ps nA 17.5 − ps 180
i= → a = 2(1 − i ) = 2 (1− 0.633) = 0.734 = ⇒ =
1 po nA + nB 17.5 25 450
+
180 18
7. (c) From the expression of depression in freezing 25 × 17.5
point, we get ∆Tf = iKf m = 2 × 1.86 × 0.1 = 0.372°C. Solving, we get ps = 17.5 − = 17.4 mm Hg
4525

Chapter-17.indd 567 8/5/2016 10:46:00 AM


568 Chapter 17 Solutions

18. (d) Azeotropes mixture cannot be distilled with change in 25. (a) For, BaCl2, the reaction is
composition. BaCl2  Ba2 + + 2Cl−
19. (d) Elevation of boiling point ∝ i (as CaCl2 , i = 3). Hence, it is Initial moles 1 0 0
highest for CaCl2. Also, the elevation in boiling point is Moles at equilibrium 1 − a a 2a
proportional to molality, which is different in all the four
Total moles after dissociation = 1 − a + a + 2a = 1 + 2a.
cases. Hence, it should also be factored in for finding the
So, i = (1 + 2a)/1 = 1 + 2a.
correct answer.
Given that i = 1.98, so
20. (d) For a solution showing negative deviation from Raoult’s 1 + 2a = 1.98 ⇒ 2a = 0.98 ⇒ a = 0.49
law, the interactions between A B are more than
A A and B B interactions so the escaping tendency of Thus, the degree of dissociation is 49%.
the molecule A B will be less than that of pure liquids.
26. (b) pA = x A pAo and pB = xB pBo = pBo (1 − x A )
The total vapour pressure will be less than that expected
from the Raoults law, that is, pA < x A pAo and pB < xB pBo .
There will be slight decrease in the volume and heat will pBo 2 pAo pBo
If pA = pB , then x A = ; ptotal = 2 pA =
be evolved on mixing two solutions showing negative pAo + pBo pAo + pBo
deviation, that is, ∆Hmix < 0 and ∆Vmix < 0.
p o − ps
21. (d) We know that relative lowering in vapour pressure = 27. (b) Applying Raoult’s law: = x solute, we get
mole fraction of solute. Therefore, po
10 10
p o − ps 639.7 − 631.9 = 0.2 ⇒ p o = = 50 mm Hg
= xB ⇒ = = 0.0122 po 0.2
po 639.7
Now, if po − ps = 20 mm Hg, then
Given that mole fraction of benzene = 1 − 0.0122
= 0.9878. 20
x solute = = 0.4 ⇒ x solvent = 1 − 0.4 = 0.6
The molar mass of benzene = 77.05 g. Therefore, 50
77.05 g contains = 0.0122 moles of solute
28. (a) Molarity (M) = number of moles of solute/volume of
0.0122
1 g contains = solution in liters, molality (m) = number of moles of
77.05 solute/mass of solvent in kg, r is the density of the solu-
0.122 tion = mass of the solution/volume of the solution and
1000 g contains = × 1000 = 1.58 mol M2 = mass of the solvent in kg mol−1
77.05
We can write molality as
22. (b) The reaction involved is
M
3A  A3 m= ⇒ m( r − MM2 ) = M ⇒ M(1 + mM2 ) = mr
r − MM2
Initial moles 1 0 mr
Moles at equilibrium 1−a a /3 ⇒M=
1 + mM2
Total moles after dissociation = (1 − a) + a/3 = 1 − (2/3)a
or i = [1 + (2/3)a]/1 = 1 + (2/3)a 29. (a) According to Raoult’s law, pA = pAo x 2 and Dalton law, pA = ptotal x1
When a = 1, that is, degree of dissociation is 100%, weDalton law, pA = ptotal x1
get the maximum value of i = 1/3.
x
23. (b) Chloroform and acetone form a non-ideal solution, Therefore, ptotal = pAo × 2
x1
Chloroform and acetone have greater attraction when
mixed together than alone, and hence, the solution 30. (c) ∆T = K × m ⇒ m = 0.216 = 0.1
b b
shows negative deviation from Raoult’s law, that is, 2.16
where
∆Vmix < 0 ; ∆Hmix < 0
wA wA
Total volume of solution = < {30 + 50 mL} or < 80 mL m= × 1000 ⇒ MA = × 1000
MA × wB m × wB
100 0.15 × 1000
= = 100 g mol−1
p = p + p = p o
x + p o
x = 140 = 0 .11 0.1 × 15
24. (b) total A B A A B B 1000 100
+ 31. (c) pKa = 4, so Ka = 10−4.
180 140
Therefore, From the expression K a = Ca 2 (Ostwald’s dilution law),
we get
30
475 = pAo × 0.11 + 500(1 − 0.11) ⇒ pAo = = 272.7 torr
0.11 Ka 10 −4
a= = = 10 −1 = 0.10
o
Now, p = p × x = 272.7 × 0.11 = 30 torr C 0 .01
A A A

Chapter-17.indd 568 8/5/2016 10:46:09 AM


Hints and Explanations 569

The reaction involved is 39. (c) Given that r = 1.17 g cm−3. This means 1.17 g of HCl dis-
HA  H+ + A − solved in 1 mL. Therefore,
( m/M ) 1.17 × 1000
Initial moles 1 0 0 Molarity = × 1000 = = 32.05 M
V 36.5 × 1
Moles at equilibrium 1− a a a
40. (c) For isotonic solutions, the osmotic pressure is the same,
Total moles after dissociation = 1 − a + a + a = 1 + a. that is, Π1 = Π2 . Also, for glucose, i = 1 as it is a non-
So, i = 1 + a = 1 + 0.1 = 1.10 electrolyte. Therefore,
32. (c) From the question, 25 mL of KCl ≡ 40 mL of 0.5 M AgNO3 CNa2 SO4 × iNa2 SO4 = C Glucose × iGlucose
20 0.01 × 1
Therefore, 25 × m = 0.5 × 40 ⇒ m = molal ⇒ 0.004 × iNa2 SO4 = 0.01 × 1 ⇒ iNa2 SO4 = = 2.5
25 0.004
Given that Kf = 1.86°C molal−1, therefore,
20 Now, the reaction involved is
∆Tf = i K f m = 2 × 1.86 × = 2.97 °C
25 Na2SO 4  2Na+ + SO24−
Therefore, freezing point of the solution = Tf − ∆Tf = 0 − 2.97 Initial moles 1 0 0
Tf − ∆Tf = 0 − 2.97= − 2.97 °C Moles at equilibrium 1 − a 2a a
= − 2.97 °C Total moles after dissociation = 1 − a +2a + a = 1 + 2a.
So, i = (1 + 2a)/1 = 1 + 2a. Therefore,
33. (c) The reaction is i − 1 2.5 − 1
a= = = 0.75
2C6H5COOH  (C6H5COOH)2 2 2
Initial moles 1 0 Hence, degree of dissociation is 75%.
Moles after dissociation 1− x x/2
x x 41. (a) For HA  H+ + A − , we find that i = 1 + a
Therefore, i = 1 − x + = 1−
2 2 Now, 10 −2 = Ca ⇒ a = 10 −2 as C = 1 M. Therefore, i = 1
+ 0.01 = 1.01.
34. (d) Positive deviation from Raoult’s law means, ptotal is
greater than expected, as A–A and B–B attractive forces 42. (a) KCN → K + + CN−, where i = 2. Hence,
are stronger than A–B attractive force.
32.5 × 1000
35. (d) From the expressions ∆Tb = i × m × K b = 2 × × 0.52 = 5.2°C
65 × 100
RTf2
∆Tf = K f × molality and K f = ×M Therefore,
∆Hfusion1000
Tbs = 373.15 + 5.2 = 378.35 K = 378.35 − 273.15 = 105.20°C
(Given Tf = ∆H = 35.7 ca l g−1) 43. (b) Given that ∆Tf = 1.86°C and m = 1 molal for urea.
M
Therefore, we get Kf = 1.86°C molal−1. Now, for acetic acid
We get, M = 256.12 g mol−1. So, the molecular formula is solution
256.12/32 = 8, that is, S8.
∆Tf = iK f × m ⇒ 0.02046 = i × 1.86 × 0.01
36. (b) For any ideal solution, i = 1.
0.02046
⇒ i= = 1.1
300 × 25 1.86 × 0.01
37. (c) Weight of solute in 300 g solution = = 75g (as 25% of 300 g)
300 × 25 100
= = 75g (as 25% of 300 g) Now, from the reaction CH3COOH  CH3COO − + H+ ,
100
400 × 40 we get i = 1 + a. Hence,
Weight of solute in 400 g solution = = 160 g (as 40% of 400 g)
400 × 40 100 1.1 = 1 + a ⇒ a = 1.1 − 1 = 0.1
= = 160 g ( as 40% of 400 g)
100 Now, the hydrogen ion concentration is
Therefore, the total weight of solution = 300 + 400 = 700 g [H+ ] = Ca = 0.01 × 0.1 = 10 −3 M (as m = C given)
Mass percentage of solute in resulting solution
Therefore, pH = − log[H+ ] = − log(10 −3 ) = 3
235
= × 100 = 33.6%
700 44. (a) According to Raoult’s law
Mass percentage of solvent in resulting solution = 0.5
(100 – 33.6%) = 66.4% p0 − ps 143 − ps
x solute = = 65 ⇒ = 0.00745
38. (a) From the expression ∆Tb = K b × molality, we get p0 0.5 158 143
+
65 154
13.44
∆Tb = 0.52 × = 0.052 K
134.4 × 1 Solving, we get ps = 141.93 mm of Hg.

Chapter-17.indd 569 8/5/2016 10:46:19 AM


570 Chapter 17 Solutions

Level II ∆Tf = i × m × K f = 2 × 0.5 × 1.86 = 1.86 K


Multiple Correct Choice Type ∆Tb = i × m × K b = 2 × 0.5 × 0.512 = 0.512 K
nRT So, the freezing point of solution is = −1.86 K and boiling point
45. (a), (b), (d) We know that Π = CRT = (as C = n/V )
V of solution = 100.512°C. The osmotic pressure is given by
Therefore, Π ∝ T but Π ∝ 1/V.
iCRT = 2 × 0.5 × 0.0821 × 300 = 24.63 atm
46. (a), (c) Both have same molar concentration as well
non-electrolytes. Now, the observed molar mass can be calculated as follows:
m × RT Actual molar mass
47. (a), (b) We know that Π = CRT = . So, Π ∝ C and i=
Π ∝ 1/M. M ×V Observed molar mass
74.5
48. (a), (c) ⇒ Observed molar mass = = 37.25 g mol−1
(b) is wrong because KH increases with increase of 2
temperature. 54. (b), (c) Acetic acid + pyridine, there will be hydrogen bond
(d) is wrong because different noble gases have different formed between hydrogen of CH3COOH and nitrogen of
value for KH at the same temperature. pyridine.
Chloroform + benzene, there will be dipole–dipole interac-
49. (a), (d) Freezing point is the point at which the vapour pres-
tions between the two molecules.
sure of the solid and liquid forms are in equilibrium or have
same vapour pressure, that is why the freezing point of the 55. (a), (d) We know in non-ideal positive deviation, ∆Hmix = pos-
solution is always lower than that of the pure solvent and only itive and ∆Vmix = positive. Also, pA > pAo x A .
solvent molecules will solidify at the freezing point (see graph).
56. (a), (c) With the increase of temperature, vapour pressure also
C
increases. Also with the weak forces of attraction between
E
t the molecules, vapour pressure increases; and if the forces
en of attraction between the molecules are strong then vapour
Solv
pressure decreases.
ion
B lut 57. (c), (d) In osmosis, there will be flow of the solvent molecules
So
Vapour pressure

Frozen to the solution or from a less concentrated solution to a more


solvent concentrated solution.
D The aqueous solution of NaCl freezes at low temperature
because the freezing point of the solution is always lower
A
Freezing point than that of the pure solvent.
∆Tf depression The value of colligative property increases when a solute
undergoes dissociation as the number of particles in dissocia-
tion increases.
Tf Tfo For AlCl3, the reaction is AlCl3  Al3+ + 3Cl− , it is a strong
Temperature (K) electrolyte so a = 1. The value of van’t Hoff factor is 1 + 3 = 4.
58. (a), (b), (d) In an ideal solution
50. (a), (c) Henry’s law states that the mole fraction of the gas in
(i) the solute–solute and solvent–solvent interactions are
the solution is directly proportional to the partial pressure of
identical to those of solute–solvent.
the gas above the solution, that is, pA = K H × x (where KH is
(ii) ∆Hmix = 0
the Henry’s constant).
The solubility of gas does not depend on its volume, but it (iii) ∆Smix > 0
decreases with increase in temperature. The enthalpy change 59. (a), (b), (c) The colligative properties are those which depend
for solubility of gas is negative, so Gibbs energy change is also on the number of solute particles and not on the nature of the
negative (ΔG = ΔH − TΔS). solute.
51. (a), (b) Chlorobenzene + bromobenzene and hexane + hep-
tane as both of them are of same size and hence the intermo- Passage Type
lecular forces are almost same in both, therefore, they show 60. (b) Osmotic pressure Π = CRT. As C = n/V. So, Π = (n/V)RT
ideal behaviour. where n is the number of moles of solute = m2/M2.
52. (a), (b) For both urea and glucose i = 1  m 
For both K4[Fe(CN)6] and Al2(SO4)3, i = 1 + 4α Therefore, Π =  2  RT
 M2V 
For association of a solute in a solution i < 1
 m  RT
Glucose and sucrose undergo neither association nor On rearranging we get, M2 =  2 
dissociation.  V  Π

53. (b), (d) Given that molality of solution = 0.5, van’t Hoff factor 61. (c) According to elevation in boiling point, ΔTb = Kb × m
of KCl = 2. Therefore, where m(molality) = 0.1/200

Chapter-17.indd 570 8/5/2016 10:46:23 AM


Hints and Explanations 571

0.1 Matrix-Match Type


∆Tb = 0.513 × × 1000 = 0.2565 °C
200
73. (a) ã r; (b) ã q; (c) ã s; (d) ã p, q, s
Now, ∆Tb = Tb − Tbo ⇒ 0.2565 = Tb − 100 (100°C for water ) (a) Benzene + toluene shows ideal behaviour as they obey
Raoult’s law, and change in volume and enthalpy will be
∆Tb = Tb − Tbo ⇒ 0.2565 = Tb − 100 (100°C for water )
zero.
Therefore, boiling point of the solution, Tb = 100 + 0.2565 (b) Acetone and chloroform shows negative deviation
= 100.256°C because they form strong hydrogen bonds on mixing.
62. (b) Lower the value of C × i, lower is the depression in freez- (c) Carbon tetrachloride and chloroform shows positive
ing point or in other words, higher is the freezing point. deviation because on mixing them there will be weaker
For H2SO4, the reaction is H2SO 4  2H+ + SO24− , so interactions between them as compared to the individ-
ual molecules.
i = 1 − a + 2a + a = 1 + 2a = 3 (max)
(d) Chloroform + benzene, chloroform + acetone and car-
For Al2(SO4)3, the reaction is Al2 (SO 4 )3  2Al3+ + 3SO24−, bon tetrachloride + chloroform form non-ideal solution
so i = 1 − a + 2a + 3a = 1 + 4a = 5 (max) as they show either positive or negative deviation from
Raoult’s law.
For HCl, the reaction is HCl  H+ + Cl− , so i = 1 − a + a
+ a = 1 + a = 2 (max) 74. (a) ã p, q; (b) ã p; (c) ã s; (d) ã r
For CH3COOH, the reaction is CH3COOH  CH3COO − + H+ , (a) A collodial solution always dissociates so, i > 1.
CH3COOH  CH3COO − + H+ , so i = 1 − a + a + a = 1 + a = 2 (max) (b) Solution of NaCl + H2O will undergo dissociation so, i > 1.
(c) For solution of glucose + water, the value of i =1 because
63. (a) The van’t Hoff factor i for aniline and sucrose = 1; for KCl,
glucose will neither associate nor dissociate.
i = 2; for CaCl2, i = 3. Therefore, the ratio is 1:1:2:3.
(d) i = 0 is not possible.
64. (c) This is because when an ideal solution is formed, no heat
is evolved or absorbed. Integer Type
p° − ps 75. (5) If vapour pressure of solution contains equimolar concen-
65. (c) According to Raoult’s law, = xB
p° tration of both solutes, then mole fraction of each solute, x = 0.5
760 − 750 10 1 Hence, p = 126 x + 142 = 126 × 0.5 + 142 = 205 mm Hg
or = =
760 760 76 Now, 205 = y + 200 ⇒ y = 5.
66. (d) We know that 76. (4) i = 1 + (5 − 1) 0.75 = 4

24 4 77. (8) The depression in freezing point is given by


ps = pAo x A ⇒ 24 = 30 × x A ⇒ x A = = = 0.8
30 5 ΔTf = iKbm
67. (b) For solutions showing negative deviation the interac- For the given solutions, value of i is:
tion between the components are greater than the pure 1 M urea: 1, 1 M glucose = 1; 1 M NaCl = 2; 1 M K2SO4 = 3
components. For solutions showing negative deviation, Ratio of depression in freezing point is 1:1:2:3. Therefore in
ΔVmix < 0 and ΔHmix < 0. In this case, the escaping ten- x:1: y: z; x + z = 1 + 3 = 4
dency of the molecules is reduced and so it shows nega- 78. (8) Whenever a non-volatile solute added in a solvent, its boil-
tive deviation. ing point is always increases. As all the given compounds are
68. (c) Given that vapour pressure of a solution of heptane and non-volatile, all of them will boil above 373 K.
octane is 79. (3) ∆Tf = K f × m . Since solution has 5% by weight urea and
10% by weight glucose, so
pH = 30 + 70 x ⇒ p = 30 + 70 × 0 = 30 mm Hg
Weight of solute
% by weight = × 100
69. (b) For a substance to exist as trimer, i = 1 − a + a/n here a = 1 Weight of solution
and n = 3 so, i = 1/3. If total weight = 100 g, then weight of water = 85 g; weight of
70. (c) The reaction involved is urea = 5 g; weight of glucose = 10 g
Now, ∆Tf = ∆Turea + ∆Tglucose
2C6H5COOH  (C6H5COOH)2
As both are non-electrolytes, i = 1, so
1−a a /2
1000 × 1.86 × 5 1000 × 1.86 × 10
The total number of moles after association = 1 + a /2 ∆T = + = 3.04 °C
60 × 85 180 × 85
71. (b) The value of n for K4[Fe(CN)6 = 5. Substituting in the 80. (5) Elevation in boiling point for Al2(SO4)3 is = 0.3 × 5 × Kb
equation i = 1 + (n − 1)a, we get = 1.5Kb
Elevation in boiling point for Na2SO4 is = 0.1 × 3 × Kb = 0.3Kb
3.4 = 1 + (5 − 1) a ⇒ a = 60%
Five times
72. (d) For non-electrolytes, it will neither undergo dissociation
81. (2) i = 2, as the complex ionizes to give
nor association, so it would be difficult to predict the
van’t Hoff factor. [Fe(H2O )(CN)2 Cl2 ]NO3 ⋅ 2H2O → [Fe(H2O )2 (CN)2 Cl2 ]+ + NO3+ + 2H2O

Chapter-17.indd 571 8/5/2016 10:46:28 AM


572 Chapter 17 Solutions

SOLVED JEE 2016 QUESTIONS


JEE Main 2016 JEE Advanced 2016
1. 18 g glucose (C6H12O6) is added to 178.2 g water. The vapour
pressure of water (in torr) for this aqueous solution is 1. The mole fraction of a solute in a solution is 0.1. At 298 K,
(a) 759.0 (b) 7.6 (c) 76.0 (d) 752.4 molarity of this solution is the same as its molality. Density of
(Offline) this solution at 298 K is 2.0 g cm−3. The ratio of the molecular
 MWsolute 
Solution weights of the solute and solvent,   , is _____.
178.2  MW solvent
(d) No. of moles of H2O (n1) = = 9.9
18 Solution
18 (9) Given that the mole fraction of the solute (xsolute) is 0.1,
No. of moles of glucose (n2) = = 0.1
180 therefore mole fraction of the solvent (xsolvent) is 0.9. So we
have,
ntotal = n1 + n2 = 10
According to relative lowering in vapour pressure nsolute
x solute = = 0.1 (1)
p° − ps n2 (nsolute + nsolvent )
=
p° n1 + n2
760 − ps 0.1 nsolvent
= ⇒ ps = 760 – 7.6 ⇒ ps = 752.4 torr x solvent = = 0.9 (2)
760 10 (nsolute + nsolvent )

2. The solubility of N2 in water at 300 K and 500 torr partial pres- where nsolute and nsolvent are the number of moles of solute
sure is 0.01 g L−1. The solubility (in g L−1) at 750 torr partial pres- and solvent, respectively. Dividing Eq. (1) by Eq. (2) gives
sure is
(a) 0.0075 (b) 0.005 (c) 0.02 (d) 0.015 x solute n 0.1 (3)
= solute =
(Online) x solvent nsolvent 0.9
Solution
Given that the density of solution is 2 g cm−3, we have
(d) Using Henry’s law
Wsolution = density × Vsolution = 2 × Vsolution
p1 = KH S1
p2 = KH S2 Wsolute + Wsolvent = 2 × Vsolution (4)

p1 S1 We know that molality is given by


=
p2 S2
nsolute × 1000
m=
nsolvent × Wsolvent
500 0.01 750 × 0.01
= ⇒ S2 = = 0.015 g L−1
750 S2 500
Substituting from Eq. (3), we have

3. An aqueous solution of a salt MX2 at certain temperature has  0.1 1000


(5)
m= ×
a van’t Hoff factor of 2. The degree of dissociation for this solu-  0.9  Wsolvent
tion of the salt is
(a) 0.50 (b) 0.33 (c) 0.67 (d) 0.80 Molarity is given by
(Online)
nsolute × 1000
M =
Solution nsolvent × Vsolution
(a) Let us assume that degree of dissociation is a.
MX2 → M2+ + 2 X−
Substituting from Eq. (3), we have
(1−a) a 2a
Thus, after dissociation total number of moles formed  0.1 1000 (6)
M= ×
(n) = 3  0.9  Vsolution
Now, we know degree of dissociation
Given that, Molarity (M) = Molality (m), therefore from
i −1 2 −1
a= = = 0.50 Eqs. (5) and (6), we get
n −1 3 −1

Chapter-17.indd 572 8/5/2016 10:46:35 AM


Solved JEE 2016 Questions 573

0.1 × 1000 0.1 × 1000 Solution


=
0.9 × Wsolvent 0.9 × Vsolution (a), (b) Positive deviation from Raoult’s law is observed when the
interaction between the solute and solvent (A−B) is weaker than
Wsolvent = Vsolution the interactions between solute and solute (A−A) and solvent and
solvent (B−B) molecules.
From Eq. (4), we have In the mixture of CCl4 + CH3OH, the addition of carbon tetra-
chloride reduces the intermolecular interactions by breaking the
Wsolute + Wsolvent intermolecular hydrogen bonds in methanol. So A–B interactions
Wsolvent =
2 are weaker than B–B interactions, hence positive deviation from
2Wsolvent = Wsolute + Wsolvent Raoult’s law is observed.
The mixture of CS2 + acetone will show positive deviation from
Wsolvent = Wsolute (7) Raoult’s law because of formation of weaker interactions in the
mixture.
From Eq. (3), we have Benzene and toluene form an ideal solution because the nature
of intermolecular interactions in solute and solvent are the same.
nsolute Wsolute / MWsolute 0.1
= = The mixture of phenol + aniline will show negative deviation from
nsolvent Wsolvent / MWsolvent 0..9 Raoult’s law because of the formation of hydrogen bonds between
the two; hence the solute-solvent interaction is more than
Using Eq. (7) solute-solute and solvent-solvent interactions.
MWsolute
= 9 OH NH2
MWsolvent

2. Mixture(s) showing positive deviation from Raoult’s law at 35°C


is (are)
(a) carbon tetrachloride + methanol.
(b) carbon disulphide + acetone.
(c) benzene + toluene.
(d) phenol + aniline.

Chapter-17.indd 573 8/5/2016 10:46:37 AM


Chapter-17.indd 574 8/5/2016 10:46:37 AM
18 Electrochemistry

Question Distribution in JEE (Main and Advanced)

3
No. of Questions

JEE (Main)
2
JEE (Adv)

0
2016 2015 2014 2013 2012 2011 2010 2009 2008 2007

Concept Distribution in JEE (Main and Advanced)


Topics Covered
Year
JEE (Main) JEE (Advanced)
2007 Nernst Equation, Kohlraush′s Law Electrochemical Series, Law of Electrolysis
2008 Nernst Equation Law of Electrolysis
2009 Electrode Potential, Types of Batteries Electrochemical Series
2010 Electrode Potential Nernst Equation
2011 Nernst Equation Nernst Equation
2012 Electrochemical Series Nernst Equation
2013 Electrochemical Series
Electrode Potential, Electrochemical Series, Law of Electrolysis,
2014 Electrode Potential, Types of Electrochemical Cells
Electrical Conductance, Variation of Molar Conductivity
2015 Electrode Potential, Law of Electrolysis Types of Electrochemical Cells, Electrical Conductance
Applications of
2006 Nernst Equation
Electrolytic Process, Electrochemical Series, Law of Electrolysis

Chapter-18.indd 575 8/8/2016 3:32:07 PM


576 Chapter 18 Electrochemistry

SUMMARY
1. Electrolysis is the process by which electricity is used to enable non-spontaneous redox reactions for producing important
product.
2. Electrochemistry is the branch of chemistry dealing with production of electricity from energy released from spontaneous chemical
reactions which can be used to bring about non-spontaneous chemical transformation.
3. Electrochemical cells
The devices which convert electrical energy into chemical energy or vice versa are called electrochemical cells. These devices are
classified into two main categories:
(a) Electrolytic cells
The devices in which chemical changes occur in the presence of applied electrical energy. For example, a simple electrolytic cell
may be constructed by dipping two copper strips in a solution of copper salt. The impure copper (containing zinc, iron, silver and
gold as impurities) is used as the anode while the cathode is a thin sheet of very pure copper. When the cell is operated at the
correct voltage, the impure copper anode dissolves and the pure copper cathode grows larger.
(b) Galvanic (voltaic cells)
The devices in which electrical energy is generated on account of the chemical reactions occurring in them. These are further
divided into chemical cells and concentration cells.
4. Galvanic cells
(a) A galvanic cell consists of two electrodes immersed in solutions of their respective salts.
(b) The electrode at which reduction (or electron gain) takes place is known as the cathode and the electrode at which oxidation
(loss of electrons) takes place is known as the anode.
(c) Daniell cell is the most popular galvanic cell. It is built by two electrodes, Zn rod dipped in ZnSO4 solution and Cu rod dipped in
CuSO4 solution, connected by a salt bridge.
The Daniel cell is represented as Zn(s) ZnSO 4 (aq) CuSO 4 (aq) Cu(s)
The half-cell reactions are:
Zn(s)  Zn2 + (aq) + 2e −

Cu2 + (aq) + 2e −  Cu(s)

The overall cell reaction is Zn(s) + Cu2+ (aq) → Zn2+ (aq) + Cu(s)
(d) A galvanic cell can also be constructed when a silver electrode dips into a solution of AgNO3 on the left and a copper electrode
dips into a solution of Cu(NO3)2 on the right. The two electrodes are connected by an external electrical circuit and the two solu-
tions are connected by a salt bridge. The half-cell reactions are:
Reduction
Ag+ (aq) + e − → Ag(s)

Oxidation
Cu(s) → Cu2 + (aq) + 2e −

Tip Salt Bridge


For continuous generation of electrical energy, the electrodes in a galvanic cell need to be internally connected. A salt bridge is usu-
ally used to connect the electrodes of a galvanic cell. It is easy to construct and consists of a gel drawn into a clean U-tube. Hence, a
salt bridge is helpful in the following ways:
(a) It brings about internal contact between the electrodes.
(b) It minimizes the liquid junction potential.
(c) It minimizes polarization.

5. Electrode potential
(a) Electrode potential is the potential difference developing between the electrode and electrolyte. It is measured by voltmeter. The
electrode potential which is determined when the concentration of all solutes is unity at standard conditions of temperature and
pressure is known as standard electrode potential.
(b) The tendency of each half-cell to acquire electrons and proceed as a reduction is expressed quantitatively by the half-reaction’s
reduction potential.
(c) Standard reduction potential
When measured under standard conditions, the reduction potential is called the standard reduction potential. By definition,
a potential of 1 volt (V) is produced when 1 joule (J) of energy is used to transport 1 coulomb (C) of electrical charge across the
potential.
1J
1V =
1C

Chapter-18.indd 576 8/8/2016 3:32:08 PM


Summary 577

Tip The standard state for electrochemistry is defined as a system where the temperature is 298 K, all concentrations are 1 M, and
any gases are at 1.00 bar pressure.

(d) Oxidation and reduction potential


(i) If oxidation occurs at the electrode, at equilibrium the potential of the electrode can be termed as oxidation potential. The
electrode in which oxidation occurs at equilibrium is referred to as a negative electrode. For example, zinc electrode with
1 M ZnSO4 at 298 K offers an oxidation potential of +0.76 V.
(ii) If reduction occurs at the electrode, at equilibrium the potential of the electrode is referred to as reduction potential. If
reduction occurs at equilibrium in an electrode, it is termed as positive electrode. The reduction potential of a zinc elec-
trode with 1 M ZnSO4 at 298 K is −0.76 V.
(e) Potential of a cell or EMF
(i) Current flows from one electrode to the other since the two electrodes of the cell are at different potentials. The potential
difference between the two electrodes is known as the cell potential. It is the maximum potential that a given cell can
generate and is represented by Ecell. It depends on the composition of the electrodes, the concentrations of the ions in the
half-cells, and the temperature.
(ii) Standard cell potential: When the system is at standard state, the potential of a galvanic cell is the standard cell potential,
o
symbolized by Ecell . The standard cell potential, which is always taken to be a positive number, represents the difference
between the standard reduction potential of one half-cell and the standard reduction potential of the other.

o  standard reduction potential  standard reduction potential 


Ecell = −
 of the substance reduced   of the substance oxidized 

Tip The term standard cell potential is used when system is at standard state (i.e., 1.00 M, 1.00 bar pressure and 298 K).

(iii) The potential generated by a galvanic cell, when no current is drawn through the cell, is called the electromotive force
(emf). Electromotive force (emf ) is the potential difference between the two terminals of the cell when no current is drawn
from it. It is measured by potentiometer.
(iv) The magnitude of the cell potential is a measure of the driving force behind a reaction.
• The larger the value of the cell potential, the farther is the reaction from equilibrium.
• The sign of the cell potential tells us the direction in which the reaction must shift to reach equilibrium.
• Reactions for which Eo is positive, have equilibrium constants that favour the formation of the products of the reaction.
A reaction with a positive Eo should occur naturally and is referred to as spontaneous.

Tip If charge flows from a cell, some of the cell’s voltage is lost in overcoming its own internal resistance, and the measured voltage
is less than the original Ecell.

6. Reversible and irreversible cells


(a) Reversible cells
For the cell to be termed as a reversible cell, the two conditions required are:
(i) On application of an opposing emf exactly equal to E, the cell reaction stops.
(ii) On application of an emf, infinitesimally more than E, the direction of cell reaction is reversed.
For example, a Daniell cell with emf of 1.09 V is reversible if the following cell reaction stops on application of −1.09 V.

Zn(s) + Cu2+ (aq) → Cu(s) + Zn2+ (aq)

The direction of cell reaction is reversed as follows when the opposing emf is slightly more than −1.09 V.

Cu(s) + Zn2+ (aq) → Zn(s) + Cu2+ (aq)


(b) Irreversible cells
(i) The electrochemical cells that do not satisfy these two conditions are said to be irreversible. For example, consider a cell
consisting of zinc and silver electrodes dipped in a solution of sulphuric acid with the emf of the cell for the following cell
reaction as E.

Zn(s) + 2H+ (aq) → Zn2+ (aq) + H2 (g)

Chapter-18.indd 577 8/8/2016 3:32:09 PM


578 Chapter 18 Electrochemistry

(ii) When an opposing emf is applied, the cell reaction is not reversed but takes place as follows:

Ag(s) + 2H+ (aq) → 2Ag+ (aq) + H2 (g)

7. Electrochemical conventions
S. No Convention Example
The interface across which a potential develops is denoted by either Cu | CuSO 4 (1 M) or Cu; CuSO 4 (1 M)
1.
a single vertical line (|) or a semicolon (;).
Representation of a cell will have the oxidation electrode (anode)
on the LHS and the reduction electrode (cathode) on the RHS. The Cu | CuSO 4 (1 M)|| AgNO3 (1 M)| Ag
2.
salt bridge used to connect the electrodes is represented by double
vertical lines.
3. The emf of a cell is represented as Ecell = Eright − Eleft Salt
Where Eright and Eleft are the standard electrode potentials (reduction bridge
electrode potentials) of RHS and LHS electrodes, respectively.
Anode Cathode

Cu(s) Cu2+(aq) Ag+(aq) Ag(s)


Anode Anode Cathode Cathode
electrode electrolyte electrolyte electrode

Anode half-cell Cathode half-cell

Ecell = Eright − Eleft = EAg+ /Ag − ECu2+ /Cu

8. Measurement of electrode potential using reference electrodes


(a) The potential of an electrode can be measured if the electrode is connected to another electrode of known potential. Such an
electrode is termed as reference electrode. From the measured emf, the potential of the electrode can be obtained by subtract-
ing the potential of the reference electrode.
(b) The criteria for an electrode to act as a reference electrode are:
(i) The potential of such an electrode should be known, under the conditions of utility.
(ii) The potential of the electrode should show minimum variation with temperature.
(c) There are a few more reference electrodes available which are standardized using standard hydrogen electrode (SHE) and then
used as reference electrodes for measuring potential of other electrodes. Such reference electrodes are termed as subsidiary or
secondary reference electrodes. Calomel electrode and silver-silver chloride electrodes are the most popular secondary refer-
ence electrodes.
(d) A standard hydrogen electrode (SHE/NHE) is one in which pressure of hydrogen gas is maintained at 1 atm and the concentra-
tion of hydrogen ions in solutions is 1 M.

Pt , H2 (1 atm)| (1.0 M)HCl

If reduction occurs at the electrode, the reaction taking place will be as follows:

2H+ (aq) + 2e −  H2 (g)

Both oxidation and reduction take place on hydrogen electrode. As the potential cannot be measured directly, it is convention-
ally assumed to be zero at 298 K.
o
(i) The emf of the cell (Ecell ) is determined at a standard condition and since Eo of SHE is assumed to be zero, the potential of the
second electrode can be obtained.
(ii) The potential of an electrode measured at standard condition (298.15 K, 1 M concentration of ions and 1 bar pressure) is
known as standard electrode potential. The electrode potential, as a practice, is reported as reduction potential and is a
measure of reduction potential of an electrode with respect to SHE.

Chapter-18.indd 578 8/8/2016 3:32:11 PM


Summary 579

Half-cell reactions Standard potential of the


Cell set up Overall cell reaction
Anode Cathode cell
H electrode
H2 (g) → 2H+ (aq) + 2e − Cu2+ (aq) + 2e − → Cu(s) E o = ECu
o o
/Cu − EH+ / 21 H2
Cu2 + (aq) + H2 (g) → Cu(s) + 2H+ (aq) cell
2+
connected to
a Cu half-cell The standard reduction
potential of Cu2+ is +0.34 V.
Zn electrode
Zn(s) → Zn2 + (aq) + 2e − 2H+ (aq) + 2e − → H2 (g) 2H+ (aq) + Zn(s) → H2 (g) + Zn2 + (aq) o
Ecell = EHo+ / 1 H − EZn
o
2+
/Zn
and H elec- 2 2

trode The emf of the cell is 0.76 V.


The standard reduction
potential of zinc is negative.

9. Electrochemical series
(a) The standard reduction potentials of many half-reactions are shown in below table. They are arranged in decreasing order – the
half-reactions at the top have the greatest tendency to occur as reduction, while those at the bottom have the least tendency to
occur as reduction.

Stronger oxidizing agent


Half-Reaction E° (V)

F2 (g) + 2e −  2F − (aq) +2.87

S2O28 − (aq) + 2e −  2SO24− (aq) +2.01

PbO2 (s) + HSO 4− (aq) + 3H+ (aq) + 2e −  PbSO4 (s) + 2H2O +1.69

2HOCl (aq) + 2H+ (aq) + 2e −  Cl2 (g) + 2H2O +1.63

MnO 4− (aq) + 8H+ (aq) + 5e −  Mn2 + (aq) + 4H2O +1.51

PbO2 ( s ) + 4H+ (aq) + 2e −  Pb2 + ( aq) + 2H2O +1.46

BrO3− (aq) + 6H+ ( aq) + 6e −  Br − ( aq) + 3H2O +1.44

Au3+ (aq) + 3e −  Au(s) +1.42

Cl2 (g) + 2e −  2Cl− (aq) +1.36

O2 (g) + 4H+ (aq) + 4 e −  2H2O +1.23

Br2 (aq) + 2e −  2Br − (aq) +1.07

NO3− (aq) + 4H+ (aq) + 4 e −  2H2O +0.96

Ag+ (aq) + e −  Ag(s) +0.80

Fe3+ (aq) + e −  Fe2 + (aq) +0.77

I2 (s) + 2e −  2I− (aq) +0.54

NiO2 (s) + 2H2O + 2e −  Ni(OH)2 (s) + 2OH− (aq) +0.49

Cu2 + (aq) + 2e −  Cu(s) +0.34

SO24− (aq) + 4H+ (aq) + 2e −  H2SO3 (aq) + H2O +0.17

AgBr(s) + e −  Ag(s) + Br − (aq) +0.07

2H+ (aq) + 2e −  H2 (g) 0


(Continued)

Chapter-18.indd 579 8/8/2016 3:32:16 PM


580 Chapter 18 Electrochemistry

(Continued)
Stronger oxidizing agent
Half-Reaction E° (V)

Sn2 + (aq) + 2e −  Sn (s) −0.14

Ni2 + (aq) + 2e −  Ni (s) −0.25

Co2 + (aq) + 2e −  Co (s) −0.28

PbSO4 (s) + H+ (aq) + 2e −  Pb (s) + HSO 4− (aq) −0.36

Cd2 + (aq) + 2e −  Cd (s) −0.40

Fe2 + (aq) + 2e −  Fe (s) −0.44

Cr 3+ (aq) + 3e −  Cr ( s ) −0.74

Zn2 + (aq) + 2e −  Zn (s) −0.76

2H2O + 2e −  H2 (g) + 2OH− (aq) −0.83

Al3+ (aq) + 3e −  Al(s) −1.66

Mg2 + (aq) + 2e −  Mg(s) −2.37

Na+ (aq) + e −  Na (s) −2.71

Ca2 + (aq) + 2e −  Ca (s) −2.76

K + (aq) + e −  K (s) −2.92

Li+ (aq) + e −  Li (s) −3.05


Stronger reducing agent

(b) Applications of electrochemical series


(i) To identify the oxidizing and reducing agents.
(ii) To calculate the emf of using the reaction.
(iii) To compare the reactivities of metals as the metal with lower reduction potential can displace the metal with higher reduc-
tion potential.
(iv) To predict if a metal can displace hydrogen to produce H2 gas. Metals with low reduction potential than hydrogen can
displace H2 gas from its acid.
(v) To predict spontaneity of a reaction. Reactions for which Ecell is negative or zero are not feasible. For a spontaneous reac-
tion the cell potential is always positive.
10. Nernst equation
(a) For the reaction Mn + + ne − → M, the Nernst equation is

RT [M]
E = E o − 2.303 log10
nF [M]n +

where R = gas constant, T = Kelvin temperature, Eo = emf of the cell, E = electrode potential of the cell, F = Faraday constant,
n = number of moles of electrons transferred. It defines the relationship of cell potential with ion concentration.

Reactions Corresponding Nernst Equation



ne
aA + bB → cC + dD o RT o RT [C]c [D]d
Ecell = Ecell − ln Q = Ecell − ln
nF nF [A]a [B]b

(Continued)

Chapter-18.indd 580 8/8/2016 3:32:19 PM


Summary 581

(Continued)

Zn(s) + Cu2+ (aq) → Zn2+ (aq) + Cu(s) o RT [Zn2+ ]


Ecell = Ecell − ln
Daniel cell 2F [Cu2+ ]

Zn(s) + 2Ag+ (aq) → Zn2 + (aq) + Ag(s) o RT [Zn2+ ]


Ecell = Ecell − ln
2F [Ag+ ]2

Cu2 + (aq) + H2 (g) →


 Cu(s) + 2H+ (aq) o RT [H+ ]2
Ecell = Ecell − ln
nF [Cu2 + ]pH
2

11. Cell potential and Gibbs energy change


The emf of a galvanic cell is given by a general expression:

o 2.303 RT
Ecell = Ecell − log Q
nF
where Q is the reaction quotient.
n+ n+ [Mn + ][N] [Mn + ]
For a reaction: M(s) + N (aq)  M (aq) + N(s), Q = =
[M][Nn + ] [Nn + ]
n+
o 2.303 RT [M ]
The Nernst equation is: Ecell = Ecell − log n +
nF [N ]
At equilibrium, the reaction quotient Q becomes equal to the equilibrium constant Keq.
2.303 RT
o
Ecell = log K eq
nF
Relation between electrochemical cell and Gibbs energy of reaction
∆rGo = −nFEo and ∆rGo = −RT lnK
12. Types of electrodes
Electrodes of a galvanic cell can consist of different components or constituents. Based on the constituents, they are classified as
follows:
A metal rod is dipped in an Half-cell reaction at the anode:
electrolytic solution of the
Metal–Metal ion half-cell

corresponding metal ion having M(s) → Mn + (aq) + ne −


M(s)
concentration C M. It can be Half-cell reaction at the cathode:
made to act as anode as well as
cathode. Mn + (aq) + ne − → M(s)

Mn+(C M)

Sn2+/Sn, Zn2+/Zn, Ag+/Ag, etc.


A noble metal like platinum The SHE is of this type, having con- Pt|H2 ( p atm)|H+ (C M)
which is simultaneously in con- centration of 1 M and the pressure
tact with a gas and a solution of of H2 gas is 1 atm. Pt|O2 ( p atm)|OH− (C M)
ions of the gas. Half-cell reaction at the anode: H+ (C M|O2 ( p atm)|OH− (C M)
Gas–Gas ion half-cell

H2 ( p atm) → 2H+ (C M) + 2e − H+ (C M)|H2 ( p atm)|Pt


Half-cell reaction at the cathode:
2H+ (C M) + 2e − → H2 ( p atm)
Pt wire

Gas ion (C M)

(Continued)

Chapter-18.indd 581 8/8/2016 3:32:23 PM


582 Chapter 18 Electrochemistry

(Continued)
In this electrode, the electrode For example, the quinhydrone electrode
potential results due to the which can be used to measure the hydro-
presence of ions of the same gen ion concentration (pH) of a solution
substance present in different
oxidation states. OH O
Redox half-cell

+ 2H+ + 2e −

(As anode)
OH O

Mx1+(C1 M)
Mx2+(C2 M)

This half-cell is made by dipping The reactions are as follows:


Metal insoluble–metal salt

a rod of silver coated with a 1. AgCl(s)  Ag+ + Cl−


paste of saturated AgCl at the
bottom, in a solution of ionic 2. Ag+ (aq) + e −  Ag(s)
Ag rod
electrolyte such as NaCl, etc. − −
3. AgCl(s) + e  Ag(s) + Cl
anion half-cell

AgCl
(saturated)

o o RT [Cl− ]
ECl−
/ AgCl/Ag = E Ag+ / Ag − ln
F K sp
An amalgam electrode is a 2.303RT [Zn]
Zn(Hg)| Zn2+ ( x M) o
E = EZn |Zn −
Amalgam
electrode

modification of metal–metal ion 2+ log 2 +


nF [Zn ]
electrode. In place of a metal, its Zn2 + ( x M) + 2e −  Zn(Hg)
solution in mercury is used. where [Zn2+] and [Zn] are the concentrations of Zn2+
ions and Zn metal in the amalgam.

13. Concentration cells


A concentration cell is a galvanic cell which generates electrical energy at the expense of chemical energy. These cells are classified
into two types: electrode concentration cells and electrolyte concentration cells.
(a) Electrode concentration cells
(i) In these cells, the concentration of the electrolyte is the same. The two electrodes contain the same substance but with
different concentration.
(ii) Consider the two hydrogen electrodes having hydrogen gas at different pressures, dipped in the same electrolytic solu-
tion. The cell can be represented as
Pt , H2 ( p1)| HCl( x M)| H2 ( p2 ), Pt
where p1 and p2 are the pressures at which H2 gas is maintained in the cell. Here, p1 is greater than p2. The reactions at the
electrodes are:
Oxidation electrode: H2 ( p1)  2H+ + 2e −

Reduction electrode: 2H+ + 2e −  H2 ( p2 )

The overall cell reaction: H2 ( p1)  H2 ( p2 )


o 2.303RT p
Since Ecell = 0, Nernst equation gives: Ecell = log 1
nF p2

Chapter-18.indd 582 8/8/2016 3:32:25 PM


Summary 583

(b) Electrolyte concentration cells


In these cells, the electrodes contain different concentration of the same electrolyte. The two electrodes are connected directly
through a diffusion membrane. Salt bridge is not required for the completion of the cell.
Pt , H2 (1 atm)| HCl(a1)
HCl(a2 )| H2 (1 atm), Pt
The dashed line indicates the direct contact between the electrodes or through a membrane.
The reactions at the electrodes are:
1
Oxidation electrodes: H2 (g)  H+ (a1) + e −
2
Reduction electrode: H+ (a2 ) + e −  1
H (g)
2 2

The overall cell reaction: H+ (a2 )  H+ (a1)


As there is direct contact between the electrodes, the ions are free to move. H+ ions move from oxidation electrode to reduction
electrode, while Cl− ions move in the opposite direction. If t+ and t− is the transport numbers of H+ and Cl−, respectively, then the
EMF of the cell is
2.303RT a
Ecell = t − log 2
nF a1
14. Conductance of electrolytic solutions
Substances that allow the passage of current through them are called conductors and the phenomenon is called electrical con-
ductance. Conductors are further divided into two categories, namely, metallic and electrolytic conductors.
(a) Metallic/Electronic conductors
In metallic conductors, the conduction is only due to the presence of free mobile electrons and these are also referred to as
electronic conductors. The electronic conductance depends on the nature and structure of metal, the number of valence
electrons per atom and temperature.
(b) Electrolytic conductors
(i) The substances which conduct electricity both in the fused state and in the aqueous solution are called electrolytic con-
ductors or electrolytes. These substances dissociate in molten state or aqueous solution to furnish ions. The conduction
is due to the presence of these mobile ions. Electrolytes are classified into two types:

Those which are almost completely dissociated in their aqueous solu- HCl, HNO3, NaOH, KOH,
Strong electrolytes
tions and have a large value of conductance. NaCl and KCl
Those which do not undergo complete dissociation even in their dilute
aqueous solutions and have low value of conductance. In the solution of CH3COOH, HCOOH and
Weak electrolytes
a weak electrolyte, there exists equilibrium between the undissociated NH4OH.
molecules and their ions.

(ii) Factors affecting electrolytic conductance


• Temperature: For strong electrolytes, increase in temperature increases the speed of the ions resulting in increased con-
ductivity. For weak electrolytes, increase in temperature increases the degree of dissociation. There is increase in both
the number of ions as well as their speed. Hence, the conductivity increases.
• Concentration: Conductance decreases with concentration.
• Nature of electrolyte: In case the solution is dilute, the ions are farther apart, and their interionic interactions can be
neglected, however in a concentrated solution, the interionic interactions are higher, which restricts their free motion,
and reduces the conductivity.
• Ionic size and their extent of solvation: In solution state, the solvent–solute interactions are higher, which restricts the
free motion of ions, thus decreasing the conductivity. On dilution, these interactions decrease, and ionic mobility, and
hence conductance increases.
• Viscosity: In a more viscous solvent, the ionic mobility is low, and thus conductivity of ions is decreased.
15. Electrical resistance
(a) Like any metallic conductor, electrolytic conductor also offers some resistance to the flow of current. The unit used is ohm (Ω)
which in terms of SI base units is equal to (kg m2)/(S3A2). The resistance R offered by an electrolytic conductor is given as
l
R= r⋅
A
where l is length, A is area of cross-section of the conductor, and r is the proportionality constant called resistivity.

Chapter-18.indd 583 8/8/2016 3:32:26 PM


584 Chapter 18 Electrochemistry

(b) Specific resistance or resistivity (ρ)


It is the resistance offered by a conductor of 1 m length and 1 m2 area of cross-section. In SI system, it has unit of Ω m (ohm-meter).
A
r = R⋅
l
16. Electrical conductance
(a) The reciprocal of resistance is known as conductance and is represented by G.
1 A
G= =
R lr

The SI unit of conductance is Siemens, represented by the symbol S and is equal to ohm−1(Ω−1), also called mho.
(b) The reciprocal of specific resistance is called the specific conductance (j ) or conductivity. Therefore,
1 1 l
k = = ×
r R A
Specific conductance is the conductance of the solution placed between two electrodes having the cross-sectional area of
1 m2 and placed 1 m apart. In other words, it is defined as the conductance of 1 cm3 of the solution and has units Siemens per
centimeter (S cm−1) in SI system. Thus, specifc conductance(k) has both units S cm−1 or S m−1 depending upon the units used
in a particular situation.
17. Measurement of the conductivity of ionic solutions
(a) Conductivity cells
The essential components of a conductivity cell include two platinum electrodes of known dimensions, length (l) and area of
cross-section (A), coated with platinum black. The ionic solution for which the resistance is to be measured is confined between
the two electrodes and its resistance is given by
1 l
R= ×
k A
The ratio of length to the cross-sectional area is called the cell constant (G*) and it is a characteristic parameter of conductivity
cell used for the experiment.
l
Cell constant (G *) =
A
(b) Wheatstone bridge
The Wheatstone bridge set up for measurement of resistance consists of two resistances R2 and R3, a variable resistance R1 and
the conductivity cell with unknown resistance Rx.
Conductivity cell
C

R3
AC generator

R1 R2

High impedance meter

When the bridge is balanced, no current passes through the bridge.


Thus, I1R x = I2R1 and I1R3 = I2R2
I2 R x R3 RR
Using this relation we get, = = ⇒ Rx = 1 3
I1 R1 R2 R2
18. Molar conductivity
The conductance of an electrolytic solution due to all the ions obtained from 1 mol of an electrolyte at a given concentration is
called molar conductivity or molar conductance. It is denoted by the symbol Λm.

Chapter-18.indd 584 8/8/2016 3:32:28 PM


Summary 585

k
Λm =
1000 × C

where k is the conductivity of the solution and C is the molar concentration of the solution. If k is expressed in S m−1 and C in
mol m−3, then the units of molar conductance Λm are Sm2 mol−1.

k (S m−1)
Λ m (S m2 mol−1 ) =
1000 (L m ) × Molarity (mol L−1)
−3

k (S cm−1) × 1000 (cm3 L−1)


Λ m (S cm2 mol−1) =
Molarity (mol L−1)
Both the units are related to each other by the equation:

1 S m2 mol−1 = 10 4 S cm2 mol−1 or 1 S cm2 mol−1 = 10 −4 S m2 mol−1

19. Equivalent conductivity


It is the conductance of an electrolytic solution due to all the ions obtained from 1 gram-equivalent (g-equiv.) mass of the electro-
lyte at a given concentration. Therefore,
Λ =kV

Where k is the conductivity of the solution and V is the volume of the solution containing 1 g-equiv. of the substance.
In terms of concentration,
k 0 k (S/cm) × 1000 (cm3 / L )
Λ= or Λ m = × Solubility
1000 × C Molarity (mol / L )

Equivalent conductance has unit of S m2 g-equiv−1.


20. Variation of conductivity with concentration
(a) The electrolytic conductivity increases with increase in concentration for both weak and strong electrolytes. This increase is
because of increase in number of ions per unit volume of the solution of the electrolyte.
(b) The electrolytic conductivity decreases with increase in dilution due to decrease in number of ions per unit volume of the
solution.
(c) The extent of variation of k with concentration is more pronounced for strong electrolytes as compared to weak electrolytes.
(d) We know that the conductivity of a solution at any given concentration is the conductance of a unit volume of solution kept
between two platinum electrodes of unit cross-section area separated by unit length distance. Thus, when A and l are unity in
their appropriate units in m or cm.
kA
G= =k
l
21. Effect of dilution on molar conductance
(a) Molar conductivity increases upon dilution and decreases with increase in concentration. The increase in molar conductivity
on dilution may be due to the increase in the degree of dissociation of the electrolyte or due to decreased ion–ion force of
attraction.
(b) At a particular dilution, the ion becomes totally independent of other ions. At this point, it contributes maximum towards the
conductance of the solution.
0
Λm = Λm
Molor conductivity −vs
k C for acetic acid (CH3COOH) and potassium chloride (KCI).
Molar conductivity, Λ(S−1 cm2 mol−1)

160
140 KCl

100

60

CH3COOH
20

0 20 40 60 80
C× 104

Chapter-18.indd 585 8/8/2016 3:32:29 PM


586 Chapter 18 Electrochemistry

(c) The extent and nature of variation of molar conductivity is however, different for weak and strong electrolytes.
The molar conductivity of a solution at a given concentration is the conductance of volume V of a solution containing one mole
of electrolyte between two electrodes with cross-sectional area A and distance of unit length between them.
kA
Λm = =kV
l
(d) On increasing dilution of the solution, the total volume (V) containing one mole of electrolyte increases and conductivity (k )
decreases. The first factor compensates the effect of the second factor and an overall increase in molar conductivity is observed
with decrease in concentration. As the concentration approaches zero, the molar conductance of the solution reaches the max-
0
imum value. This value is called limiting molar conductivity and is denoted by Λ m .
22. Variation in molar conductivity with concentration
(a) Strong electrolytes
(i) The variation in molar conductivity with concentration for strong electrolytes is given by an empirical relation proposed
by Kohlrausch based on a number of experimental observations. At low concentrations, the relation is represented by the
equation
0
Λm = Λm −k C
where k is a constant that is specific for a given solvent, at a given temperature but varies with the type of electrolyte that
is the number of charges on the ions produced by dissociation of the electrolyte.
(ii) Kohlrausch’s law of independent migration of ions states that limiting molar conductivity of an electrolyte can be rep-
resented as the sum of the individual contributions of the anion and cation of the electrolyte. Mathematically, the law can
be stated as
0
Λm = v + l+0 + v − l−0
0
where Λ m , l 0+ and l 0− are limiting molar conductivities of the electrolyte, cations and anions, respectively and v+ and v−
are the number of cations and anions.

Tip The conductivity of a strong electrolyte increases on dilution slightly. This is explained by the Debye–Hückel–Onsager equation
0 0
Λm = Λm − ( A + BΛ m ) C
0
where A and B are the Debye-Hückel constants. If a solution is at infinite dilution, that is, C is almost zero then Λ m = Λ m .

(b) Weak electrolytes


(i) The variation of molar conductivity of a solution of weak electrolyte is not a linear function of the concentration. The molar
conductivity increases sharply with decrease in concentration and this is attributed to the increase in degree of dissocia-
tion on dilution.
(ii) Weak electrolytes have a lower degree of dissociation and dissociate to a greater extent in dilute solutions. The number of
cations and anions increases thus leading to an increase in molar conductivity.
(iii) In very dilute solutions, the weak electrolytes also dissociate completely and molar conductivity approaches limiting
0
molar conductivity value (Λ m ). However, the values are low and cannot be obtained accurately by extrapolation of curve
0
and Kohlrausch law of independent migration of ions is used to obtain Λ m for weak electrolytes.
Number of moles of electrolyte dissociated x
Degree of dissociation a = =
Initial number of moles of electrolyte n0

x Λ
The degree of dissociation in terms of molar conductivity is a = = m0
n0 Λ m
Λm
At any concentration (C) the degree of dissociation is a = 0
Λm
0
whereΛ m is obtained by measurement of resistance in a conductivity cell and Λ m is evaluated from Kohlrausch law of
independent migration of ions.
(Ca )(Ca ) Ca 2
(iv) For a weak electrolyte, the dissociation constant at concentration C is given by K = =
C (1 − a ) 1− a
2
C Λm C Λ2
K= = 0 0m
0 2 Λ  Λm (Λm − Λm )
(Λm )  1− m0 
 Λm 

Chapter-18.indd 586 8/8/2016 3:32:32 PM


Summary 587

(c) Applications of Kohlrausch law of independent ionic migration


0 0 0 0 0 0 0
(i) To calculate the Λ m of weak electrolytes: Λ m ( CH3 COOH) = l( CH COO − ) + l(H+ ) = Λ m(HCl) + Λ m( CH3 COONa) − Λ m(NaCl)
3

+ l (0Cl− ) + l (0CH COO − ) + l (0Na+ ) −  l (0Na+ ) + l (0Cl− ) 


= l (0H+ )
3

(ii) To calculate the ionic conductance: The ionic conductance ( l +0 ) can be calculated provided the transport number and
molar conductance at infinite dilution are known.
( l 0+ )
` t+ = (t+ is the transport number of the ion)
Λ0
(iii) To calculate the solubility of the sparingly soluble salt: A saturated solution of a sparingly soluble salt is a very dilute solution.
If the specific conductance of the solution is determined, the concentration of the solution can be calculated.
0 k
Λm =
1000 × C
23. Conductometric titrations
Titration Curve Features
Strong acid (HCl) vs. Strong base (NaOH) • Initially with pure HCl, the conductance is very high due to the highly con-
ducting H+ ions.
• Upon adding NaOH, H+ (faster moving) are replaced with Na+ (slow mov-
Conductance (mho)

ing) thereby decreasing the overall conductance till the equivalence point,
where only NaCl is in solution.
• Upon further adding NaOH, the solution starts having an excess of OH−
ions, which increases the conductivity.
Equivalence point

Volume of NaOH (ml)

Weak acid (CH3COOH ) vs. Strong base (NaOH)


• Hydrolysis reaction: CH3COO − + H2O  CH3COOH + OH−
• Due to the common ion effect of CH3COONa during the titration, the disso-
Conductance (mho)

ciation of acetic acid is suppressed.


• Thus, the conductivity increases as the conducting power of sodium ace-
tate is more than that of the feebly ionized acetic acid.

Volume of NaOH

Weak base (NH4OH ) vs. Strong acid (HCl)


• Hydrolysis reaction: H3O + + Cl− + NH4+ + OH− → NH+4 + Cl− + 2H2O
• A fall in conductance is observed up to the end point.
Conductance (mho)

• After the equivalence point, the conductance remains practically constant


because NH4OH has very low conductance compared to HCl or NH4Cl. A
slight curvature is observed near the end point. This is due
NH+4 + 2H2O  NH4 OH + H3O +
which shows the generation of very fast moving H3O+ ions.
Equivalence point
Volume of NH4OH

Mixture of acids (HCl + CH3COOH) vs. Strong base • The conductance of the solution gradually decreases due to the removal of
(NaOH) H+ ions of highly ionized HCl to form unionized water.
• Subsequently, the feebly ionized weak acid is neutralized.
• The conductance increases marginally due to the salts produced. After the
Conductance (mho)

equivalence point, the increase in conductance is appreciable due to the


CH3COOH
NaOH added OH− ions.
vs. NaOH
• The variation in conductance is an indicator of the trend in the neutraliza-
tion reaction.
HCl Equivalence
vs. NaOH Na+ + OH− + H+ + Cl− → Na+ + Cl− + H2O
points
CH3COOH + Na+ + OH− → CH3COO − + Na+ + H2O
Volume of NaOH

Chapter-18.indd 587 8/8/2016 3:32:34 PM


588 Chapter 18 Electrochemistry

24. Faradays laws of electrolysis


(a) First law
The amount of a substance consumed or produced at one of the electrodes in an electrolytic cell is directly proportional to the
amount of electricity that passes through the cell (i.e., w ∝ Q ).

w ∝ Q ⇒ w ∝ It or w = ZIt

where Z is the electrochemical equivalent of that substance.


From Faraday’s first law w = ZQ
or w=Z×I×t (using Q = I × t)
M  molar mass 
or w= × I × t  using Z =
n× F  n × Faraday 
Now, 1 mol of electrons = 1 equivalent of electrons = 96500 C = 1 F
w It
Therefore, = Number of equivalents =
E 96500
M
where E is the number of equivalents =
n-factor
(b) Second law
If the same quantity of electricity is passed in two different solutions, then the amount of substance deposited or dissolved are
proportional to their chemical equivalent weights.
w It w w
Since = , if It = constant, then w/E is constant. So, 1 = 2
E 96500 E1 E2

From Faraday’s second law,


Wt. of M1 deposited Eq. wt. of M1
=
Wt. of M2 deposited Eq. wt. of M2

Tips (a) The charge on one mole of electrons is called a Faraday and represented by symbol F.
(b) To simplify the calculation, an approximation of 1 F = 96500 C mol−1 is commonly used.

25. Products of electrolysis


(a) The products of electrolysis depend on the following:
(i) Nature of material electrolyzed.
(ii) Type of electrode being used.
(iii) Standard electrode potentials of oxidizing and reducing species.

Electrolysis Reactions at cathode Reactions at anode Products


Molten NaCl Na metal, Cl2
Na+ (l) + e − → Na(l) 2Cl− (l) → Cl2 (g) + 2e −
gas
Aqueous solution of NaOH, Cl2 and
H2O (l) + e − → 21 H2 (g) + OH− Cl− (aq) → 1
Cl (g) + e −
2 2
NaCl H2

Overall reaction: NaCl(aq) + H2O(l) → Na+ (aq) + OH− (aq) + 21 H2 (g) + 21 Cl2 (g)

Copper (II) bromide Cu(s) and Br2


Cu2 + (aq) + 2e − → Cu(s) 2Br − (aq) → Br2 (aq) + 2e −

Electrolysis
Overall reaction: Cu2 + (aq) + 2Br − (aq) 
→ Cu(s) + Br2 (aq)

Sulphuric acid 2− − O2 and H2S2O8


2SO2− o
4 (aq) → S2O 8 (aq) + 2e ; E cell = +1.96 V

2H2O(l) → 4H+ (aq) + O2 (g) + 4e − ; Ecell


o
= + 1.23 V

26. Transport (Transference) number


(a) It is the fraction of the total current carried by the cation or anion. It is represented by t+ or t−.

Chapter-18.indd 588 8/8/2016 3:32:37 PM


Summary 589

current carried by the cations q


Transport number of the cation t + = ⇒ t+ = +
total current carried by both the ions Q

current carried by the anions q


Transport number of the anion t − = ⇒ t− = −
oth the ions
total current carried by bo Q
The sum is t + + t − = 1
(b) The current carried by an ion depends on the charge, speed and molal concentration of the ion.
Therefore, t + ∝ Z + m+ v + and t − ∝ Z − m−v −
t + v + l+
(c) The speeds of the ions are proportional to molar conductance, that is, = =
t − v − l−
27. Ionic mobility
Distance travelled by an ion per second under a potential gradient of 1 volt per meter.
Ionic velocity v
Ionic mobility = = = m2 V −1 s −1
Potential gradient dV /dt
28. Batteries
A battery is a combination of cells either in series or parallel or both, in order to obtain the required amount of electrical energy. The
basic components of a battery are follows:
(a) Anode
It contains active materials which spontaneously take part in the oxidation reactions.
(b) Cathode
It contains active materials which spontaneously take part in the reduction reactions.
(c) Electrolyte
It helps in the migration of the ions leading to the generation of electrical energy.
(d)) Separator
It is a thin polymeric membrane which prevents the mixing of products formed at the electrodes. Usually, polypropylene and
polystyrene separators are used.
29. A cell is a battery that is packed with active materials at anode and cathode. As soon as the anode and the cathode are connected
to the load, redox reactions occur spontaneously. This process in which spontaneous redox reaction occurs is called discharging.
(a) Primary and secondary cells
(i) Primary where the reaction occurs only once and cannot be reused once it becomes dead over the course of time. For exam-
ple, dry cell (Leclanche cell).
(ii) Secondary which can be recharged by passing current through it in the opposite direction so that it can be used again.

Type Anode Cathode Electrolyte Electrode reactions


Dry Cell cylindrical graphite rod thick paste
At anode: Zn → Zn2+ + 2e −
(Leclanche zinc con- surrounded of NH4Cl and
cell) tainer by a paste of ZnCl2 At cathode: MnO2 + H2O + 2e − → Mn2O3 + 2OH−
manganese
dioxide Overall cell reaction: Zn + 2MnO2 + H2O → Zn2+ + Mn2O3 + OH−

Alkaline dry cylindrical graphite rod thick paste of


At anode: Zn + 2OH− → ZnO + H2O + 2e −
cell zinc con- surrounded NaOH/KOH
tainer by a paste of and ZnCl2 At cathode: 2MnO2 + 2e − + H2O → Mn2O3 + 2OH−
Primary batteries

MnO2
Overall cell reaction: Zn + 2MnO2 → ZnO + Mn2O3

Mercury cell amalga- paste of mer- A paste of ZnO


At anode: Zn(s) + 2OH− (aq) → ZnO(s) + H2O(aq) + 2e −
(Ruben– mated zinc curic oxide and KOH
Mallory cell) powder with some At cathode: HgO(s) + H2O(aq) + 2e − → Hg(s) + 2OH− (aq)
amount of
Overall cell reaction: Zn(s) + HgO(s) → ZnO(s) + Hg(s)
graphite

Silver button zinc metal silver oxide KOH and NaOH


At anode: Zn(s) + 2OH− (aq) → ZnO(s) + H2O(aq) + 2e −
cell
At cathode: Ag2O(s) + H2O(aq) + 2e − → 2Ag(s) + 2OH− (aq)
Overall cell reaction: Zn(s) + Ag2O(s) → ZnO(s) + 2Ag(s)

(Continued)

Chapter-18.indd 589 8/8/2016 3:32:40 PM


590 Chapter 18 Electrochemistry

(Continued)
Type Anode Cathode Electrolyte Electrode reactions
Lead Storage lead grid a lead grid H2SO4 solution −
At the anode: Pb(s) + SO2−
4 (aq) → PbSO 4 (s) + 2e
Cell (Lead– filled with packed with (20%)
Acid Battery) spongy lead oxide At cathode:
lead PbO2 (s) + 4H+ (aq) + SO24− (aq) + 2e − → PbSO4 (s) + 2H2O(l)
Overall cell reaction:
Secondary cells

Pb(s) + PbO2 (s) + 2H2SO 4 (aq) → 2PbSO4 (s) + 2H2O(l)

Nickel– spongy nickel concentrated


At the anode: Cd + 2OH− → Cd(OH)2 + 2e −
Cadmium cadmi- oxyhydrox- solution of
Batteries um with ide along KOH At the cathode: 2NiO(OH) + 2H2O + 2e − → 2Ni(OH)2 + 2 OH−
(nicad cadmium with small Overall cell reaction:
battery/alka- hydroxide amounts of Cd + 2NiO(OH) + 2H2O → 2Ni(OH)2 + Cd(OH)2
line storage graphite and
battery) other com-
pounds

(b) Fuel cells


(i) Devices used for converting the chemical energy of fuel directly into electrical energy through catalytically activated redox
reactions are fuel cells. The representation of the fuel cell is:
Fuel | Electrode | Electrolyte | Electrode | Oxidant
The chemical reactions involved at the electrodes are:
At the anode: Fuel → Oxidation product + ne −

At the cathode: Oxidant + ne − → Reduction product

Overall cell reaction: Fuel + Oxidant → Oxidation product + Reduction product

(ii) Hydrogen-oxygen fuel cell


This fuel cell combines hydrogen and oxygen to produce electricity, heat and water. As long as hydrogen is supplied, it con-
tinues to produce electricity without being discharged. The chemical reactions involved at the electrodes are:
At the anode: 2H2 → 4H+ + 4e −

At the cathode: O2 + 4H+ + 4e − → 2H2O

The overall cell reaction: 2H2 + O2 → 2H2O


(iii) Advantages of fuel cells:
• They offer high energy conversions (almost 75%).
• These cells have high energy density.
• These cells use inexpensive fuels.
30. Applications of electrolytic process
(a) Corrosion control
(i) Corrosion is the deterioration of materials by chemical interaction with their environment. The most familiar example of
corrosion is rusting of iron exposed to the atmospheric conditions.
2Fe(s) + 23 O2 (g) + xH2O(l) → Fe2O3 ⋅ xH2O(s)
( Chemical composition of rust)

(ii) The important methods for control of corrosion are as follows:


• Protective coatings are used to prevent corrosion at the surfaces of materials. Protective coatings include metallic coat-
ings, chemical conversion coatings, etc.
• Cathodic protection: The principle of cathodic protection involves the elimination of anodic sites and conversion of the
entire metal into cathodic site. It involves placing the iron in contact with a metal that is more easily oxidized.
(b) Electroplating
Electroplating of one metal onto another is done to protect both against corrosion and to improve appearance. For example, a
thin layer of silver can be plated onto jewellery or tableware made from a less expensive metal.

Chapter-18.indd 590 8/8/2016 3:32:43 PM


Tools for Problem Solving 591

(c) Purification of metals


(d) Electrometallurgy
The process of electrolysis is used for extraction of alkali and alkaline earth metals and aluminium from their fused salt solutions.
(e) Preparation of compounds
The process of electrolysis is used in synthesis of a number of organic compounds. For example, hydrocarbons are prepared by
Kolbe’s method which involves electrolysis of salts of carboxylic acids.

TOOLS FOR PROBLEM SOLVING


o  standard reduction potential   standard reduction potential 
1. Ecell =  −  of the substance oxidized 
 of the substance reduced
RT [M]
2. Nernst equation: E = E 0 − 2.303 log10
nF [M]n +
3. Relation between electrochemical cell and Gibbs energy of reaction: ∆rGo = −nFEo and ∆rGo = −RT lnK.
2.303RT a
4. EMF of electrolyte concentration cells: Ecell = t − log 2
nF a1
l
5. The resistance R offered by an electrolytic conductor: R = r ⋅
A
A
6. Specific resistance or resistivity (r ) − r = R ⋅
l
1 A
7. Electrical conductance: G = =
R lr
1 1 l
8. Specific conductance (k ) or conductivity: k = = ×
r R A
1 l
9. Resistance in an ionic solution: R = ×
l k A
10. Cell constant (G *) =
A
11. Measurement of unknown resistance using Wheatstone bridge
I2 R x R3 R1R3
I1R x = I2R1 and I1R3 = I2R2 or = = or Rx =
I1 R1 R2 R2
k k (S/cm) × 1000(cm3 / L)
12. Molar conductance: Λ m = =
1000 × C Molarity (mol / L)
k (S/cm) × 1000(cm3 / L)
13. Equivalent conductance: Λ = × Solubility
Molarity (mol / L)
0
14. The variation in molar conductivity with concentration for strong electrolytes: Λ m = Λ m −k C
0
15. Kohlrausch’s law: Λ m = v + l 0+ + v − l 0−

Number of moles of electrolyte dissociated x


16. The degree of dissociation: a = =
Initial number of moles of electrolyte n0
Λm
0
a=
Λm
17. For a weak electrolyte, the dissociation constant at concentration C is given by
(Ca )(Ca ) Ca 2
K= =
C (1 − a ) 1 − a
2 2
C Λm C Λm
K= =
(Λ )
0
m
2 Λm  Λm
1 − 0 
0
Λm0
(
− Λm )
 Λm 
18. Faraday’s first law: w = ZQ
Wt. of M1 deposited Eq. wt. of M1
19. Faraday’s second law: =
Wt. of M2 deposited Eq. wt. of M2

Chapter-18.indd 591 8/8/2016 3:32:46 PM


592 Chapter 18 Electrochemistry

20.
Transport number of the cation (t+) current carried by the cations q+
t+ = t+ =
total current carried by both the ions Q
Transport number of the anion (t–) current carried by the anions q−
t− = t− =
oth the ions
total current carried by bo Q

t+ v+ l +
21. The speeds of the ions are proportional to molar conductance = =
t− v− l −
Ionic velocity v
22. Ionic mobility = = = m2 V −1 s −1
Potential gradient dV /dt

SOLVED EXAMPLES
Electrode Potential where n is the number of electrons involved in the cell reac-
o o
tion, F is charge on one mole of electrons called faraday
1. Given that EFe 3+
/Fe = − 0.036 V , EFe2+ /Fe = − 0.439 V . The and E o is the standard electrode potential. Substituting
value of standard electrode potential for the change, the values, we have
Fe3+ (aq) + e − → Fe2 + (aq) will be
(a) 0.770 V (b) −0.270 V (c) −0.072 V (d) 0.385 V 966 × 103 J = 4 × 96500 × E o
(AIEEE 2009)
Solution 966000 1 ≈ 2.5 V
Eo = ×
(a) For the reaction, Fe3+ + 3e − → Fe E o = − 0.036 V (1) 96500 4
The Gibbs energy is given by o
3. The standard electrode potentials (EM +
/ M ) of four metals A, B,
∆G1o = − nFE o = −3 × F × ( −0.036 ) = + 0.108 F C and D are −1.2 V , 0.6 V, 0.85 V and − 0.76 V, respectively. The
sequence of deposition of metals on applying potential is
For the reaction Fe2 + + 2e − → Fe E o = − 0.439 V (a) A, C, B, D (b) B, D, C, A (c) C, B, D, A (d) D, A, B, C
(JEE Main Online 2014)
or Fe → Fe2 + + 2e − E o = 0.439 V (2)
Solution
The Gibbs energy is given by
(c) ∆G o = −nFE o
∆G2o = − nFE o = − 2F (0.439 ) = − 0.878 F For a process to be spontaneous, ∆G must be negative,
more negative value of ∆G represents greater spontaneity
For the reaction, Fe3+ + e − → Fe2 + (3) of a process.
o o o
The Gibbs energy is given by ∆G = − nFE = −1FE A B C D
Equation (3) can be obtained by adding Eqs. (1) and (2),
E° −1.2 0.6 0.85 −0.76
therefore
∆G° +1.2 nF −0.6 nF −0.85 nF +0.76 nF
∆G o = ∆G1o + ∆G2o
− FE o = 0.108F + ( − 0.878 )F = − 0.77F Hence sequence of deposition of metal would be
C > B > D > A.
E o = 0.77 V
4. A variable, opposite external potential (Eext) is applied to the
2. The Gibbs energy for the decomposition of Al2O3 at 500°C is as cell Zn|Zn2+ (1 M) || Cu2+ (1 M) | Cu, of potential 1.1 V. When
follows: Eext < 1.1 V and Eext > 1.1 V, respectively, electrons flow from
2
Al O
3 2 3
→ 4
3
Al + O2 , ∆rG = + 966 kJ mol−1 (a) anode to cathode and cathode to anode.
(b) cathode to anode and anode to cathode.
The potential difference needed for electrolytic reduction of
(c) cathode to anode in both cases.
Al2O3 at 500°C is at least
(d) anode to cathode in both cases.
(a) 5.0 V (b) 4.5 V (c) 3.0 V (d) 2.5 V
(JEE Main 2015)
(AIEEE 2010)
Solution Solution
(d) We know that Gibbs energy is related to the potential of (d) If Eext < 1.1 V, then Zn2+ acts as an anode while Cu2+ as
the cell by the relation cathode. Then electrons flow from anode (Zn2+) to cath-
∆G = − nFE o ode (Cu2+).

Chapter-18.indd 592 8/8/2016 3:32:50 PM


Solved Examples 593

The correct statement about the cell is


Eext (a) Standard cell potential is 0.95 V.
− → ↓e−
e (b) During withdrawal of electricity from cell, lead is reduced.
e (c) During withdrawal of electricity from cell, manganese is
od
An oxidized.
(d) During withdrawal of electricity from cell, two electrons
Cathode
are transferred from lead to manganese.
Solution
Cu2+
Zn2+ Zn2+ (d) The reduction potential of MnO2 is more than that of
+ 2e− Cu
PbCl2, so, Pb2+ cannot reduce manganese oxide. For cell
1 M Zn2+ 1M reaction to occur, redox reaction is necessary. Hence, if
If Eext > 1.1 then Cu2+
acts as an anode and Zn2+ as cath- MnO2 is reduced, lead should be oxidized.
ode. Hence, electrons flow from anode (Cu2+) to cathode Reduction half reaction:
(Zn2+). MnO2 + 4H+ + 2e −  Mn2 + + 2H2O ; o
EMnO = 1.23 V
2

Eext
e

e− Oxidation half reaction:
Pb + 2Cl−  PbCl2 + 2e − ; o
EPb = − 0.27 V
Cell reaction:
Cathode Anode MnO2 + Pb + 2Cl− + 4H+  Mn2 + + PbCl2 + 2H2O
o
Cu Ecell = 1.23 + 0.27 = 1.5 V
Zn 2+ + Cu2+
2e− Zn + 2e− 8. Electrolysis is planned for an aqueous solution that contains a
1M Zn2+ 1M mixture of 0.5 M ZnSO4 and 0.5 M NiSO4. On the basis of stand-
ard reduction potential, what products are expected to be
5. At 298 K, the standard reduction potentials are 1.51 V for observed at the electrodes?
MnO 4− | Mn2 + , 1.36 V for Cl2/Cl−, 1.07 V for Br2|Br−, and 0.54 V o o o
[Given: ENi2+
/Ni = − 0.25 V ; E Zn2+ /Zn = − 0.76 V ; E S O2+ /SO2− = 2.01 V ,
for I2|I−. At pH = 3, permanganate is expected to oxidize 2 8 4

Oxidation of water: EHo2 O/O2 = −1.23 V , Reduction of water:


(RT /F = 0.059 V )
EHo2 O/H2 = − 0.83 V]
(a) Cl−, Br− and I−. (b) Cl− and Br−.
(c) Br− and I−. (d) I− only. (a) Anode: Ni, Cathode: S2O2−
8
(JEE Main Online 2015)
Solution (b) Anode: Ni, Cathode: O2

(a) The reduction potential of MnO 4− /| Mn2 + ,is more than that (c) Anode: Zn, Cathode: S2O2−
8
of Cl2/Cl−, Br2/Br− and I2/I−, so permagnate is expected to (d) Anode: H2, Cathode: O2
oxidize Cl−, Br− and I− beacuse Ecell
o > 0.o o
= Ecathode − Eanode = 0.77 −Solution
( −0.14 ) = 0.91 V

6. Consider the following Eo values, EFe o o (b) Since oxidation occurs at anode, the species with the high-
3+
/Fe2+ = 0.77 V and E Sn2+ /Sn = − 0.14 V
est oxidation potential would give a anodic product. As
o o
EFe 3+
/Fe2+ = 0.77 V and ESn 2+
/Sn = − 0.14 V. Under standard conditions the potential for reduction occurs at cathode, the species with the highest
3+ 2+
the reaction Sn(s) + 2Fe (aq) → 2Fe (aq) + Sn (aq) is 2+ reduction potential would give a cathodic product.
The competing reactions at the cathode are reduction
(a) 1.68 V (b) 0.63 V (c) 0.91 V (d) 1.40 V of Ni2+, Zn2+ and reduction of water, out of which Ni2+ has
Solution the maximum reduction potential, so the cathodic prod-
uct is Ni.
(c) At the cathode: Fe3+ + e − → Fe2 + Similarly, competing reactions at the anode are oxida-
2+ − tion of SO2−4 or oxidation of water, out of which oxidation
At the anode: Sn → Sn + 2e
potential is maximum for oxidation of water; so, the anodic
o o o product is O2 gas.
Ecell = Ecathode − Eanode = 0.77 − ( −0.14 ) = 0.91 V
9. At the standard state, a cell uses the half-cell reactions given
7. Consider the half reactions of a galvanic cell given below:
below:
MnO2 + H+  Mn2 + + 2H2O ; o
EMnO = 1.23 V
2 Al3+ (aq) + 3e −  Al(s); EAl
o
3+
/Al = − 1.66 V

PbCl2  Pb + 2Cl− ; EPbCl


o
= − 0.27 V
2
Cu2 + (aq) + 2e −  Cu(s); ECu
o
2+
/Cu = 0.34 V

Chapter-18.indd 593 8/8/2016 3:32:54 PM


594 Chapter 18 Electrochemistry

If the cell uses aluminium and copper metal electrodes, Electrochemical Series
which of the following statements is correct?
(a) Oxidation of Cu occurs at anode. 12. The standard reduction potentials for Zn2+/Zn, Ni2+/Ni, and
(b) Reduction of Al3+ occurs at cathode. Fe2+/Fe are −0.76 V, −0.23 V and −0.44 V, respectively. The
(c) The standard cell potential of the cell is 2.00 V. reaction X + Y2+ → X2+ + Y will be spontaneous when
(d) The mass of copper electrode decreases as electricity is (a) X = Ni, Y = Fe (b) X = Ni, Y = Zn
withdrawn. (c) X = Fe, Y = Zn (d) X = Zn, Y = Ni
(AIEEE 2012)
Solution Solution
(c) Aluminium’s reduction potential is negative but that (d) Looking at the standard reduction potentials, it is clear
of copper is positive, so Cu2+ will be reduced and Al is that Zn is the strongest reducing agent and Ni is the
oxidized in the cell reaction. The Al electrode will act as weakest reducing agent. So, Zn will displace both Ni and
anode and copper electrode will act as cathode. Since Al Fe. Fe will displace only Ni and not Zn as Zn is a stronger
is oxidized at anode, its mass will decrease on withdraw- reducing agent; while Ni will not displace either Zn or Fe
ing electricity from the cell. as it is the weakest reducing agent.
o
Ecell o
= Ecathode o
− Eanode = 0.34 − ( −1.66 ) = 2.00 V Zn + Fe2 + → Zn2 + + Fe
Fe + Ni2 + → Fe2 + + Ni
10. For the reaction
Zn + Ni2 + → Zn2 + + Ni
NiO2 (s) + Cl− (aq) + H+ (aq) → Cl2 (g) + Ni2 + (aq) + H2O , the
standard cell potential is 0.32 V. The value of ∆Go will be So the combination in option (d) is correct.
(a) −61.8 kJ (b) 61.8 J
13. Given below are the half-cell reactions
(c) −30.9 kJ (d) −123.6 kJ
Solution Mn2 + + 2e − → Mn; E o = − 1.18 V
(a) The balanced chemical equation is 2(Mn3+ + e − → Mn2+ ); E o = + 1.51 V

NiO2 (s) + 2Cl− (aq) + 4H+ (aq) → Cl2 (g) + Ni2 + (aq) + 2H2O The Eo for 3Mn2+ → Mn + 2Mn3+ will be
(a) −2.69 V; the reaction will not occur.
We know that ∆G o = − nFE o ; where n implies number of (b) −2.69 V; the reaction will occur.
electrons involved in the reaction, and F is 96500 C mol−1. (c) −0.33 V; the reaction will not occur.
Since NiO2 is converting from Ni4+ to Ni2+, two electrons (d) −0.33 V; the reaction will occur.
are involved per molecule of NiO2. Therefore, (JEE Main 2014)
∆G o = − 2 mol × 96500 Cmol−1 × 0.32 V Solution
6.18 × 10 4 (a) Mn2+ + 2e− → Mn; Eo = −1.18 V (1)
=− = − 61.8 kJ (C × V = J) [Mn3+ + e− → Mn2+] × 2; E° = +1.51 V (2)
1000
[Mn2+ → Mn3+ + e−] × 2; E° = −1.51 V (3)
11. A galvanic cell employs the following half reactions: Adding Eq. (1) and Eq. (3) , we get
Al3+ (aq) + 3e −  Al(s); EAl
o
3+
/Al = − 1.66 V 3Mn2+ → Mn + 2Mn3+

Cu2 + (aq) + 2e −  Cu(s); ECu


o for which E o = − 1.18 − 1.51 = − 2.69 V
2+
/Cu = 0.34 V
Negative electrode potential implies that disproportion
Which of the following statement is correct about it? is not spontaneous for Mn with +2 oxidation state to 0
(a) Copper electrode would work as anode. and +3 oxidation states.
(b) The overall cell reaction is
14. Given:
3Cu(s) + 2Al3+ (aq)  3Cu2 + (aq) + 2Al(s)
(c) The standard cell potential of the cell is 2.00 V. Fe3+ (aq) + e − → Fe2 + (aq); E ° = + 0.77 V
(d) The mass of copper electrode decreases as electricity Al3+ (aq) + 3e − → Al(s); E ° = − 1.66 V
withdrawn from the cell.
Br2 (aq) + 2e − → 2Br − ; E ° = + 1.09 V
Solution
(c) The reduction potential of aluminium is negative, but Considering the electrode potentials, which of the following
that of copper is positive, so Cu2+ will be reduced and Al represents the correct order of reducing power?
will be oxidized in the cell reaction. (a) Fe2+ < Al < Br− (b) Br− < Fe2+ < Al
The Al electrode will work as anode and copper elec- (c) Al < Br− < Fe2+ (d) Al < Fe2+ < Br−
trode will work as cathode. Since Al is oxidized at anode, (JEE Main Online 2014)
its mass will decrease on withdrawing electricity from Solution
the cell. Therefore, (b) The elements with maximum value of standard reduc-
o o o tion potential are most easily reduced and have maxi-
Ecell = Ecathode − Eanode = 0.34 − ( −1.66 ) = 2.00 V
mum oxidizing power and minimum reducing power.

Chapter-18.indd 594 8/8/2016 3:32:57 PM


Solved Examples 595

Br − < Fe2+ < Al For complete discharge, Ecell = 0. So,


 →
o 0.0591 [Zn2 + ]
Most easily oxidized Ecell − log 2 + = 0
n [Cu ]
Maximum reducing power
The reactions are

15. Standard reduction electrode potentials of three metals A, B Zn → Zn2 + + 2e −


and C are respectively + 0.5 V, −3.0 V and −1.2 V. The reducing
Cu2+ + 2e − → Cu
powers of these metals are
(a) B > C > A (b) A > B > C (c) C > B > A (d) A > C > B
o 0.0591 [Zn2 + ]
So, for n = 2, Ecell = log 2 +
Solution 2 [Cu ]
(a) Higher the value of reduction potential, stronger is the o
Using Ecell = 1.10 V and solving we get
oxidizing agent.
o 1.10 × 2 [Zn2 + ] [Zn2 + ] [Zn2 + ]
16. The EM 3+
/ M2+ values for Cr, Mn, Fe and Co are −0.41, +1.57, = log 2 + ⇒ 37.3 = log 2 + ⇒ = (10 )37.3
+ 0.77 and +1.97 V, respectively. For which one of these metals 0.0591 [Cu ] [Cu ] u2 + ]
[Cu
the change in oxidation state form +2 to +3 is the easiest? o o
(a) Cr (b) Co (c) Fe (d) Mn 19. Given ECr3+ /Cr = − 0.72 V, EFe2+ /Fe = − 0.42 V. The potential for
the cell Cr | Cr 3+ (0.1 M)|| Fe2 + (0.01 M)| Fe is
Solution
(a) 0.26 V (b) 0.399 V
(a) Changing oxidation number from +2 to +3 means under-
(c) –0.339 V (d) –0.26 V
going oxidation. As we know from the reactivity series,
(AIEEE 2008)
metals with lower Eo values undergoes oxidation more
easily than those with higher Eo values. Solution
o
(a) Given that ECr 3+ E o 2+ = − 0.42 V. For
/Cr = − 0.72 V and Fe /Fe
Nernst Equation
the reaction 2Cr + 3Fe2 + → 3Fe + 2Cr 3+, according to
17. Given the data at 25°C, Nernst equation
o 0.0591 [Cr 3+ ]2
Ag + I− → AgI + e − ; E o = 0.152 V Ecell = Ecell − log 2 + 3
n [Fe ]
Ag → Ag+ + e − ; E o = − 0.800 V 0.0591 (0.1)2
= ( −0.42 + 0.72) − log
6 (0.01)3
What is the value of log Ksp for AgI?
(a) −8.12 (b) +8.612 (c) −37.83 (d) −16.13 0.0591 (0.1)2
= 0.30 − log
(AIEEE 2006) 6 (0.01)3
Solution
0.0591 10 −2 0.0591
(d) Reversing the first reaction and adding to the second = 0.30 − log −6 = 0.30 − log10 4
6 10 6
AgI + e − → Ag + I− ; E o = − 0.152 V = 0.30 − 0.0394 = 0.2602 V
20. The reduction potential of hydrogen half-cell will be negative
Ag → Ag+ + e − ; E o = − 0.800 V
if:
We get, AgI(s) → Ag+ + I− ; E o = − 0.952 V (a) p(H2) = 1 atm and [H+] = 1.0 M
(b) p(H2) = 2 atm and [H+] = 1.0 M
o 2.303RT (c) p(H2) = 2 atm and [H+] = 2.0 M
Now, Ecell = log K sp . Substituting values, we get
nF (d) p(H2) = 1 atm and [H+] = 2.0 M
(AIEEE 2011)
0.059 0.952 Solution
−0.952 = log K sp ⇒ log K sp = − = − 16.135
1 0.059
(b) The reaction is: 2H+ + 2e − → H2 (g)
o
18. The cell, Zn|Zn2+ (1 M) || Cu2+ (1 M) | Cu (Ecell = 1.10 V), was 0.059  p(H ) 
EH+ /H = EHo+ /H − log  + 22 
allowed to be completely discharged at 298 K. The relative 2 2
2  [H ] 
o 0.0591 [Zn2+ ] At room temperature,
concentration of Zn=2+Eto
Ecell − 2+, thatlog
cellCu is, is
n [Cu2+ ] EHo+ /H = 0
(a) 9.65 × 104 (b) antilog (24.08) 2

(c) 37.3 (d) 1037.3 0.059  p(H ) 


Therefore, EH+ /H = − log  + 22 
(AIEEE 2007) 2
2  [H ] 
Solution
0.0591 [Zn2+ ] Reduction potential of hydrogen half-cell will be nega-
o
(d) Using Nernst equation, Ecell = Ecell − log 2+ tive when p(H2) > [H+]2
n [Cu ]

Chapter-18.indd 595 8/8/2016 3:33:02 PM


596 Chapter 18 Electrochemistry

0.059  2 At standard temperature;


EH+ /H = − log   2+
0.0591 [Sn ] × pH2
2
2  1 o
Ecell = Ecell − ln
0.059 n [H+ ]2
=− × 0.3010 o o o
Ecell = EH+ /H − ESn2+ /Sn = 0 − ( − 0.14 V ) = 0.14 V
2 2

= − 0.008 0.059 1
For pH = 5, Ecell = 0.14 − log −5 2 = − 0.16 V
2 (10 )
21. For the redox reaction:
o
Zn(s) + Cu2 + (0.1 M) → Zn2 + (1 M) + Cu(s) For standard state; Ecell = Ecell = 0.14 V
o
For pH = 2,
taking place in a cell, Ecell is 1.10 V. Ecell for the cell will be 0.059 1
[2.303 (RT/F) = 0.0591] Ecell = 0.14 − log −2 2 = 0.14 − 0.12 = 0.02 V
2 (10 )
(a) 2.14 V (b) 1.80 V (c) 1.07 V (d) 0.82 V
25. For the reaction
Solution
(c) According to Nernst equation, at STP we have I2 (s) + Cr2O72 − (aq) + H+ → IO3− (aq) + Cr 3+ (aq) + H2O; Ecell
o
= 0.135 V.

o 0.0591 [Zn2 + ] 0.0591 1 What is Ecell if [Cr2O72 − ] = 0.01 M , [H+ ] = 0.1 M, [IO3− ] = 0.0001 M
Ecell = Ecell − log 2 + = 1.1 − log = 1.07 V
n [Cu ] 2 0.1
and [Cr 3+ ] = 0.001 M ?
22. For a cell reaction involving a two-electron change, the (a) 0.135 V (b) 0.115 V (c) 0.155 V (d) 0.215 V
standard emf of the cell is found to be 0.295 V at 25°C. The
Solution
equilibrium constant of the reaction at 25°C will be
(a) 1 × 10−10 (b) 29.5 × 10−2 (c) 10 (d) 1 × 1010 (c) The balanced form of cell reaction is

Solution 3I2 (s) + 5Cr2O72 − (aq) + 34H+ → 6IO3− (aq) + 10Cr 3+ (aq) + 17H2O
0.0591 −
 [IO (aq)] [Cr (aq)] 
o 6 3+ 10
(d) We know that Ecell = log K C . o 0.06
n Ecell = Ecell − × log  3 2 − 5 + 34 
n  [Cr2O7 (aq)] [H ] 
Given that n = 2, so we have
 2 × 0.295  The total number of electrons involved in the reaction
K C = antilog  = 1 × 1010
 0.0591  is 30. Substituting for n and concentrations of reactants
and products, we have
−4 6 −3 10
23. The standard emf of a cell, involving one electron change is 0.06  [10 ] [10 ] 
found to be 0.591 V at 25°C. The equilibrium constant of the Ecell = 0.135 − × log  −2 5 −1 34 
30  [10 ] [10 ] 
reaction is (F = 96,500 C mol−1, R = 8.314 J K−1 mol−1)
(a) 1.0 × 101 (b) 1.0 × 1030 (c) 1.0 × 1010 (d) 1.0 × 105 0.06
= 0.135 − log(10 −10 )
30
Solution 0.06
o 0.0591 = 0.135 + × 10 = 0.155 V
(c) We have Ecell = log K C 30
n
1 × 0.591 Law of Electrolysis
So, log K C = = 10 ⇒ K C = 1 × 1010
0.0591
26. A current of 10.0 A flows for 2.00 h through an electrolytic
24. The reaction of tin metal with acid can be written as cell containing a molten salt of metal X. This results in the
decomposition of 0.250 mol of metal X at the cathode. The
Sn(s) + 2H+ (aq)  Sn2 + (aq) + H2 (g); ESn
o
2+
/Sn = − 0.14 V oxidation state of X in the molten salt is: (F = 96,500 C)
(a) +1 (b) +2 (c) +3 (d) +4
Assume that [Sn2+ ] = 1 M and the partial pressure of hydro- (JEE Main Online 2014)
gen gas is 1 atm, then the Solution
(a) cell reaction is spontaneous at pH = 5. (c) From Faraday’s law of electrolysis
(b) cell reaction is non-spontaneous at standard conditions.
EIt
(c) cell reaction is spontaneous at pH = 2. w=
(d) cell reaction is spontaneous for all pH values. 96500
mIt  m
Solution w= , E = 
n factor × 96500  n
(c) Sn(s) + 2H+ (aq)  Sn2 + (aq) + H2 (g) w It
=
Nernst equation for this cell reaction is m n factor × 96500
2+ 10 × 3600 × 2
o RT [Sn ] × pH2 n -factor = =3
Ecell = Ecell − ln 0.25 × 96500
nF [H+ ]2
Thus, oxidation state of metal is +3.

Chapter-18.indd 596 8/8/2016 3:33:08 PM


Solved Examples 597

27. How many electrons would be required to deposit 6.35 g of (d) One ampere current for a day would be required to
copper at the cathode during the electrolysis of an aqueous deposit minimum one mole of each metal.
solution of copper sulphate? (Atomic mass of copper = 63.5 u,
Solution
NA = Avogadro’s constant)
(c) In electrolytic cells, reduction of each metal takes place
NA NA NA NA on respective electrodes.
(a) (b) (c) (d)
20 10 5 2
Fe2 + + 2e − → Fe and Cu2 + + 2e − → Cu
(JEE Main Online 2014)
Solution Since both the cells are connected in series, equal amount
(c) From the reaction of current passes through the cells. From the above reac-
Cu2+(aq) + 2e−  Cu(s) tions, we find that each mole of iron and copper requires
2 F charge is required for 63.5 g two moles of electrons or 2 F current. So, in each cell,
For 6.35 g Cu = 0.2 F charge is required equal moles of metal will deposit at a particular time.
N Now considering inert metal electrodes used for
So, number of electron required = A electrons. the electrolysis, the electricity passed,
5
86400
Q = i × t = 1 × 24 × 60 × 60 = 86400 C = F = 0.9 F
28. Two faraday of electricity is passed through a solution of 96500
CuSO4. The mass of copper deposited at the cathode is Since one mole of metal requires 2 F current, it will not
(atomic mass of Cu = 63.5 u) deposit one mole of any metal.
(a) 63.5 g (b) 2 g (c) 127 g (d) 0 g
(JEE Main 2015) 32. What current would be required to deposit 1.00 m2 of chrome
Solution plate having a thickness of 0.052 mm in 4.5 h from a solution
of H2CrO4? The current efficiency is 74% and density of chro-
(a) According to Faraday’s law of electrolysis and according
mium is 7.19 g cm−3. [Atomic mass of Cr = 52 u.]
to the reaction,
(a) 347 A (b) 185 A (c) 125 A (d) 0.25 A
Cu2 + + 2e − → Cu
Solution
2 F will deposit 63.5 g of Cu.
(a) We know that
29. When during electrolysis of a solution of AgNO3, 9650 C of Mass of Cr deposited = Volume × Density
charge passes through the electroplating bath, the mass of
silver deposited on the cathode will be = (1 × 10 4 cm2 × 0.052 × 10 −1 cm) × 7.19 g cm−3
(a) 1.08 g (b) 10.8 g (c) 21.6 g (d) 108 g = 52 × 7.19 g
Solution Oxidation state of Cr in H2CrO4 = +6 ⇒ Cr 6 + + 6e − → Cr
Therefore, 1 mol or 52 g of Cr is reduced by 6 mol elec-
(b) 1 F of charge is required to deposit 108 g of silver.
trons or 6 F charge. Hence,
96500 C of charge is required to deposit 108 g of silver.
108 6F
9650 C will deposit × 9650 = 10.8 g silver. 52 × 7.19 g would be deposited by × 52 × 7.19 C
96500 52

30. Aluminium oxide may be electrolyzed at 1000°C to furnish (6 F /52) × 52 × 7.19


⇒ Q=I ×t ⇒ i = = 256.9 A ≈ 257 A
aluminium metal (atomic mass = 27 u; 1 F = 96500 C). The 4.5 × 60 × 60
cathode reaction is Al3+ + 3e − → Al. To prepare 5.12 kg of Current efficiency is 74%, so the actual current required is
aluminium metal by this method would require 257
(a) 5.49 × 107 C of electricity. (b) 1.83 × 107 C of electricity. = 347.3 A ≈ 347 A.
0.74
(c) 5.49 × 104 C of electricity. (d) 5.49 × 101 C of electricity.
33. If a direct current deposits 20.5 g of potassium (atomic mass
Solution
39 u) in 1 min, then the amount (in grams) of aluminium
(a) To prepare 27 g of Al, electricity required (atomic weight 27 u) deposited by the same current during
3 F = 3 × 96500 C the same time interval would be
Therefore, 5.12 kg (5120 g) will require (a) 88.8 (b) 14.2 (c) 9.0 (d) 4.7
3 × 96500 × 5120 Solution
= = 5.49 × 107 C of electricity.
27
(d) At the cathode: K+ + e− → K
31. An electric current is passed through two electrolytic cells
Al3+ + 3e− → Al
connected in series. One cell contains Cu2+ and the other con-
Equivalent weights are EK = 39/1 = 39 and EAl = 27/3 = 9
tains Fe2+. Which of the following is the correct statement?
(a) Equal masses of iron and copper are deposited on the From Faraday’s second law of electrolysis,
electrode. WK WAl
(b) More moles of copper are deposited on the electrode. =
EK EAl
(c) Equal moles of iron and copper are deposited on the
(20.5 × 9 )
electrode. So, WAl = = 4.7 g
39

Chapter-18.indd 597 8/8/2016 3:33:11 PM


598 Chapter 18 Electrochemistry

Electrical Conductance Variation of Molar Conductivity


34. Resistance of a conductivity cell filled with a solution of an 37. The equivalent conductance of NaCl at concentration C and
electrolyte of concentration 0.1 M is 100 Ω. The conductivity at infinite dilution is lC and l∞, respectively. The correct rela-
of this solution is 1.29 S m−1. Resistance of the same cell when tionship between lC and l∞ is given as (where the constant
filled with 0.2 M of the same solution is 520 Ω. The molar con- B is positive)
ductivity of 0.02 M solution of the electrolyte will be (a) l C = l ∞ + (B )C (b) l C = l ∞ − (B )C
(a) 124 × 10−4 S m2 mol−1 (b) 1240 × 10−4 S m2 mol−1
(c) l C = l ∞ − (B ) C (d) l C = l ∞ + (B ) C
(c) 1.24 × 10−4 S m2 mol−1 (d) 12.4 × 10−4 S m2 mol−1
(AIEEE 2006) (JEE Main 2014)
Solution Solution
(d) Given that R = 100 Ω. Now, specific conductance is (c) According to Debye–Hückle–Onsager equation

1 l  1 l l −1 lC = l ∞− B C .
k =   ⇒ 1.29 =   ⇒ = 129 m
R  a 100  a  a
When R = 520 Ω, C = 0.2 M we have
Kohlraush’s Law
0 0
38. The molar conductivities ΛNaOAc and ΛHCl at infinite dilution
1 l 1 in water at 25°C are 91.0 and 426.2 S cm2 mol−1, respectively.
k =  = (129 ) Sm−1
R  a  520 0
To calculate ΛHOAc , the additional value required is
0 0 0 0
(a) ΛH2 O (b) ΛKCl (c) ΛNaOH (d) ΛNaCl
Now, molar conductivity (Λm) is Λ m = k × V where V is in
(AIEEE 2006)
cm3. If M is concentration of solution in mol L−1, then
Solution
1000 1 1000 (d) According to Kohlrausch’s law,
Λm = k × = × 129 × × 10 −6
M 520 0.2
Λ 0CH3 COONa = l 0CH COO − + l Na
0
+ (1)
. × 10 −3 = 12.4 × 10 −4 S m2 mol−1
= 124 0
3

ΛHCl = l H0 + + l Cl
0
− (2)
35. Resistance of 0.2 M solution of an electrolyte is 50 Ω. The spe-
0 0 0
cific conductance of the solution is 1.4 S m−1. The resistance ΛNaCl = l Na ++ l
Cl− (3)
of 0.5 M solution of the same electrolyte is 280 Ω. The molar Adding Eqs. (1) and (2) and subtracting Eq. (3) from the
conductivity of 0.5 M solution of the electrolyte in S m2 mol−1 is sum, we get
(a) 5 × 10−4 (b) 5 × 10−3 (c) 5 × 103 (d) 5 × 102
(JEE Main 2014) Λ 0CH3 COOH = l 0CH COO − + l H0 + = Λ 0CH3 COONa + ΛHCl
0 0
− ΛNaCl
3
Solution
39. The equivalent conductances of two strong electrolytes at
(a) Given that specific conductance of 0.2 M solution,
infinite dilution in H2O (where ions move freely through a
k = 1.4 S m−1 = 1.4 × 10 −2 S cm−1 solution) at 25°C are Λ CH3 COONa = 91.0 Scm2equiv −1 and
We know that 0
ΛHCl = 426.2 Scm2 equiv −1. What additional information/
0
1 1 l l quantity is needed to calculate Λ m of an aqueous solution
k = = × ⇒ = 1.4 × 10 −2 × 50 = 0.7 cm−1
r R A A of acetic acid?
0
(a) Λ m of chloroacetic acid (CICH2COOH)
For 0.5 M solution, 0
(b) Λ m of NaCl
1 l 1
k = × = × 0.7 = 2.5 × 10 −3 S cm−1 0
(c) Λ m of CH3COOK
R A 280
(d) The limiting equivalent conductance of H+ ( l H0 + )
and its molar conductivity is
1000 × k 1000 × 2.5 × 10 −3 (AIEEE 2007)
Λm = = = 5 S cm2 mol−1 Solution
M 0.5
(b) According to Kohlrausch’s law equivalent conductance
= 5 × 10 −4 S m2 mol−1
of a weak electrolyte at infinite dilution cannot be deter-
36. The highest electrical conductivity among the following mined experimentally therefore it is calculated by indi-
aqueous solutions is of rect method where we can use equivalent conductance
(a) 0.1 M acetic acid. of three strong electrolytes as given by the following
(b) 0.1 M chloroacetic acid. expression
(c) 0.1 M fluoroacetic acid. Λ 0CH3 COOH = Λ CH
0 0
+ ΛHCl 0
− ΛNaCl
3 COONa
(d) 0.1 M difluoroacetic acid.
Solution 40. The limiting molar conductivities Λ0 for NaCl, KBr and KCl are
(d) This is because the electrical conductivity depends on 126, 152 and 150 S cm2 mol−1, respectively. The Λ0 for NaBr is
the strength of the acid, and difluoroacetic acid is the (a) 128 S cm2 mol−1 (b) 302 S cm2 mol−1
2
(c) 278 S cm mol −1 (d) 176 S cm2 mol−1
strongest among these.

Chapter-18.indd 598 8/8/2016 3:33:17 PM


Advanced Level Problems 599

Solution (c) Sulphuric acid concentration decreases on discharging.


(d) The potential difference observed from one cell of bat-
(a) Λ (0NaBr ) = Λ (0NaCl) + Λ (0KBr ) − Λ (0KCl)
tery is approximately 12 V.
= 126 + 152 − 150 = 128 Scm2 mol−1
Solution
(c) On discharging of lead storage battery, lead oxide con-
Types of Batteries verts into lead sulphate and is deposited on the plates, so,
sulphuric acid withdraws from electrolyte which results
41. In a fuel cell, methanol is used as fuel and oxygen gas is used
into decrease in density of electrolyte.
as an oxidizer. The reaction is
Lead storage battery, in general, can be used to gen-
CH3OH(l) + 23 O2 (g) → CO2 (g) + 2H2O(l) erate potential difference of multiple of 2 V because one
cell produces around 2 V potential difference.
At 298 K, standard Gibbs energies of formation for CH3OH(l), 44. In lead storage battery, during discharging process
H2O(l) and CO2(g) are −166.2 kJ mol−1, −237.2 kJ mol−1 and (a) PbO2 gets oxidized.
−394.4 kJ mol−1, respectively. If the standard enthalpy of com- (b) H2SO4 is produced.
bustion of methanol is −726 kJ mol−1, efficiency of the fuel cell (c) density of H2SO4 solution decreases.
will be (d) density of H2SO4 solution increases.
(a) 90% (b) 97% (c) 80% (d) 87%
(AIEEE 2009) Solution
Solution (c) The following reactions take place in a lead storage bat-
(b) The reaction involved is tery during discharging process:
At the anode: Pb + SO24− → PbSO4 + 2e −
CH3OH(l ) + 23 O2 (g) → CO2 (g) + 2H2O(l); ∆H = − 726 kJ mol−1
At the cathode: PbO2 + 4 H+ + SO24− + 2e − → PbSO4 + 2H2O
Now,
At the overall : Pb + PbO2 + 24H2SO 4 + 2e − → 2 PbSO 4 + 2H2O
∆G = ∆G °(CO2 ) + 2∆G °(H2O) − ∆G °(CH3OH)
In the overall reaction, H2SO4 is consumed and water is
= − 394 − 2 × (237.2) + 166.2 = − 702.6 kJ mol−1 formed (Pb, PbSO4, and PbO2 are solids). Therefore, the
Efficiency of fuel cell is given by density of H2SO4 solution decreases.

∆G 702.6 Applications of Electrolytic Process


× 100 = × 100 = 97%
∆H 726 45. Several blocks of magnesium are fixed to the bottom of a ship
to
42. In hydrogen–oxygen fuel cell, combustion of hydrogen (a) keep away the sharks.
occurs to (b) make the ship lighter.
(a) generate heat. (c) prevent action of water and salt.
(b) remove adsorbed oxygen from electrode surfaces. (d) prevent puncturing by undersea rocks.
(c) produce high purity water.
(d) create potential difference between the two electrodes. Solution
(c) This is done to prevent corrosion caused by water and
Solution
salt.
(d) By creating potential difference between two electrodes,
fuel cell can be used to generate electricity. 46. During the process of electrolytic refining of copper, some
metals present as impurity settle as “anode mud”. These are
43. Consider a typical lead storage battery and select the correct (a) Sn and Ag (b) Pb and Zn
statement mentioned below. (c) Ag and Au (d) Fe and Ni
(a) Lead oxide is dissolved into electrolyte on withdrawing
current. Solution
(b) The density of electrolyte is increased on discharging. (c) The electrolytic method contains valuable metals in the
form of anode mud, that is, silver and gold.

ADVANCED LEVEL PROBLEMS


1. We have taken a saturated solution of AgBr. Ksp of AgBr is l (0Ag+ ) = 6 × 10 −3 Sm2 mol−1, l (0Br − ) = 8 × 10 −3 Sm2 mol−1,
12 × 10−14. If 10−7 mol of AgNO3 are added to 1 L of this solu-
tion, find conductivity (specific conductance) of this solution l (0NO − ) = 7 × 10 −3 Sm2 mol−1.
3

in terms of 10−7 S m−1 units. Given, (IIT-JEE 2006)

Chapter-18.indd 599 8/8/2016 3:33:18 PM


600 Chapter 18 Electrochemistry

Solution o
(a) Eox will increase by a factor of 0.65 from Eox .
Suppose solubility of AgBr in presence of 10−7 M AgNO3 solu- o
tion is S. Then the reactions involved are (b) Eox will decrease by a factor of 0.65 from Eox .
o
(c) Ered will increase by a factor of 0.65 from Eox .
AgBr → Ag+ + Br −
o
S S (d) Ered will decrease by a factor of 0.65 from Eox .
AgNO3 → Ag+ + NO3− Solution
10 −7 M 10 −7 M
(c) On increasing the concentration of NH3, the concentra-
The concentration terms [Ag+] = [S + 10−3] and [Br−] = S. Now, tion of H+ ions decreases. Therefore, Ered increases.
Ksp = [Ag+][Br−], substituting values, we get
2c. Ammonia is always added in this reaction. Which of the fol-
12 × 10 −14 = ( S + 10 −7 )S ⇒ S 2 + 10 −7 S − 12 × 10 −14 = 0 lowing must be incorrect?
which is quadratic in S, solving and ignoring negative root, (a) NH3 will combines with Ag+ to form a complex.
we get (b) [Ag(NH3 )2 ]+ is a stronger oxidizing reagent than Ag+.
(c) In absence of NH3, silver salt of gluconic acid is formed.
−10 −7 ± 10 −14 + 4 × 12 × 10 −14 (d) NH3 affects the standard reduction potential of glucose/
S=
2 gluconic acid electrode.
−10 −7 + 7 × 10 −7
= = 3 × 10 −7 M Solution
2
(d) It is clear from the reaction that NH3 combines with Ag to
In the solution, we have [Ag+] = S + 10−7 = 3 × 10−7 + 10−7 give [Ag(NH3 )2 ]+. Also, the value of Ered
o
of Ag+ is greater
= 4 × 10−7 M; [Br−] = S = 3 × 10−7 M; [NO3− ] = 10 −7 M . +
than [Ag(NH3 )2 ] , which means it is a stronger oxidizing
Now, specific conductance is
k = k Ag+ + k Br − + k NO − agent than Ag+, and in absence of ammonia, silver salt of
3 gluconic acid is formed.
= l (0Ag+ ) × C( Ag+ ) + l (0Br − ) × C(Br − ) + l (0NO − ) × C(NO − )
3 3 3. Read the passage below and answer the questions that
= (6 × 10 −3 × 4 × 10 −7 + 8 × 10 −3 × 3 × 10 −7 + 7 × 10 −3 × 10 −7 ) follow.
Redox reactions play a pivotal role in chemistry and biol-
× 10 −3 Sm−1 ogy. The values of standard redox potential (Eo) of two half-cell
= 55 × 10 −7 Sm−1 reactions decide which way the reaction is expected to pro-
ceed. A simple example is a Daniel cell in which zinc goes into
2. Read the paragraph below and answer the questions that solution and copper gets deposited. Given below are a set of
follow. half-cell reactions (acidic medium) along with their Eo (V) with
Tollens’ reagent is used for the detection of aldehyde. respect to normal hydrogen electrode) values. Using this data
When a solution of AgNO3 is added to glucose with NH4OH obtain the correct explanations to Questions 3a to 3c.
then gluconic acid is formed. I2 + 2e − → 2I− E o = 0.54
Ag+ + e − → Ag; EAg
o Cl2 + 2e − → 2Cl− E o = 1.36
+
/Ag = 0.8 V

C6H12O6 + H2O → C6H12O7 + 2H+ + 2e − ; Ered


o
= −0.05 V Mn3+ + e − → Mn2 + E o = 1.50
Gluconic acid Fe3+ + e − → Fe2 + E o = 0.77
+ − o
[Ag(NH3 )2 ] + e → Ag(s) + 2NH3 ; Ered = 0.337 V
O2 + 4H+ + 4 e − → 2H2O E o = 1.23
 RT F  (IIT-JEE 2007)
Use 2.303 × F = 0.0592 and RT = 38.92 at 398 K 
  3a. Among the following, identify the correct statement.
(IIT-JEE 2006) (a) Chloride ion is oxidized by O2.
(b) Fe2+ is oxidized by iodine.
2a. 2Ag+ + C6H12O6 + H2O → 2Ag(s) + C6H12O7 + 2H+ . (c) Iodide ion is oxidized by chlorine.
(d) Mn2+ is oxidized by chlorine.
Find ln K of this reaction.
(a) 66.13 (b) 58.38 (c) 28.30 (d) 46.29 Solution
Solution (c) Eo values indicate the reduction potential of the ion/
o RT atom. A negative or smaller value of Eo means that the
(b) We know Ecell = ln K . Substituting values, we get
nF electrode is acting as a reducing agent or it is getting oxi-
1 0.0592 2.303 dized as compared to the electrode having higher value
0.8 − 0.05 = × ln K ⇒ ln K = 0.75 × 2 × = 58.35 of Eo. Here, the reduction potential of iodide ion is less
2 2.303 0.0592
than chlorine, so iodide is getting oxidized by chlorine. It
2b. When ammonia is added to the solution, pH is raised to 11. could have been oxidized by Mn or O2 because they too
Which half-cell reaction is affected by pH and by how much? have higher value of reduction potential than iodide.

Chapter-18.indd 600 8/8/2016 3:33:23 PM


Advanced Level Problems 601

3b. While Fe3+ is stable, Mn3+ is not stable in acid solution Solution
because (d) Na−Hg (amalgam) formed at the cathode = 2 mol
(a) O2 oxidizes Mn2 + to Mn3+. = (200 + 23) × 2 = 446 g.
(b) O2 oxidizes both Mn2 + to Mn3+ and Fe 2 + to Fe3+ . 4c. The total charge required for complete electrolysis is
(c) Fe3+ oxidizes H2O to O2. (a) 24125 C (b) 48250 C
(c) 96500 C (d) 193000 C
(d) Mn3+ oxidizes H2O to O2.
Solution
Solution
(d) 2 mol of electrons are required or 2F of charge is required
(d) Reaction of Mn3+ with H2O is spontaneous.
+ − o
for complete electrolysis. Now 1 F = 96500 C. So, 2 F =
At the anode: O2 + 4H + 4e → 2H2O; E = 123
. V 96500 × 2 = 193000 C.
At the cathode: Mn3+ + e − → Mn2 + ; E o = 15
. 0V
5. Electrolysis of dilute aqueous NaCl solution was carried out
o o o by passing 10 milliampere current. The time required to liber-
Ecell = Ecathode − Eanode = 15
. 0 − 123
. = 0.27 V
ate 0.01 mol of H2 gas at the cathode is (1 Faraday constant =
Since Ecell is positive, the reaction is spontaneous because 96500 C mol−1)
Gibbs energy is negative = −nFEcell. (a) 9.65 × 104 s (b) 19.3 × 104 s
3c. Sodium fusion extract, obtained from aniline, on treatment (c) 28.95 × 104 s (d) 38.6 × 104 s
with iron(II) sulphate and H2SO4 in presence of air gives a prus- (IIT-JEE 2008)
sian blue precipitate. The blue colour is due to the formation of Solution
(a) Fe4[Fe(CN)6]3 (b) Fe3[Fe(CN)6]2 (b) We know that
(c) Fe4[Fe(CN)6]2 (d) Fe3[Fe(CN)6]3 (1)
Q = I × t = 10 × 10 −3 × t
Solution
(a) The reactions involved are as follows: In the reaction 2H2O + 2e − → H2 + 2OH− to liberate 0.01
mol of H2, 0.02 F charge is required. The charge Q = 0.02
Na + C + N → NaCN × 96500 C. Substituting in Eq. (1), we get
6NaCN + FeSO 4 → Na4 [Fe(CN)6 ] + Na2SO 4
0.02 × 96500 C = 10 −2 × t ⇒ t = 19.3 × 10 4 s
Na4 [Fe(CN)6 ] + Fe3+ → Fe 4 [Fe(CN)6 ]3
Ferric ferrocyanide 6. For the reduction of NO3− ion in an aqueous solution, Eo is
+0.96 V. Values of Eo for some metal ions are given below:
4. Read the passage below and answer the questions that
follow. V2+(aq) + 2e− → V Eo = −1.19 V
Chemical reactions involve interaction of atoms and mol-
Fe3+(aq) + 3e− → Fe Eo = −0.04 V
ecules. A large number of atoms/molecules (approximately
6.023 × 1023) are present in a few grams of any chemical Au3+(aq) + 3e− → Au Eo = +1.40 V
compound varying with their atomic/molecular masses. To Hg2+(aq) + 2e− → Hg Eo = +0.86 V
handle such large numbers conveniently, the mole concept
was introduced. This concept has implications in diverse areas The pair(s) of metals that is (are) oxidized by NO3− in aqueous
such as analytical chemistry, biochemistry, electrochemistry solution is (are)
and radiochemistry. The following example illustrates a typi- (a) V and Hg (b) Hg and Fe
cal case, involving chemical/electrochemical reaction, which (c) Fe and Au (d) Fe and V
requires a clear understanding of the mole concept. (IIT-JEE 2009)
A 4.0 molar aqueous solution of NaCl is prepared and 500 Solution
mL of this solution is electrolyzed. This leads to the evolution o
(a) , (b), (d) ENO − = 0.96 V and V, Fe, Hg have low value of

of chlorine gas at one of the electrodes (Atomic mass: Na = 23, standard reduction potential with respect to NO3− . So, V, Fe,
3

Hg = 200; 1 Faraday = 96500 C). Hg can be oxidized by NO3− in aqueous solution.


(IIT-JEE 2007)
7. Read the following paragraph and answer the questions that
4a. The total number of moles of chlorine gas evolved is follow.
(a) 0.5 (b) 1.0 (c) 2.0 (d) 3.0 The concentration of potassium ions inside a biological cell
is at least twenty times higher than the outside. The resulting
Solution
potential difference across the cell is important in several pro-
(b) The reaction is NaCl → Na+ + Cl−. The reaction at anode cesses such as transmission of nerve impulses and maintaining
is 2Cl− → Cl2. The number of moles of Cl− = 2 in 500 mL. the ion balance. A simple model for such a concentration cell
Therefore, 1 mol of Cl2 is evolved. involving a metal M is: M(s) | M+ (aq, 0.05 M) || M+ (aq,1 M) | M(s) .
4b. If the cathode is a Hg electrode, the maximum mass of amal- For this electrolytic cell, the magnitude of the cell potential
gam formed from this solution is |Ecell| = 70 mV.
(a) 200 g (b) 225 g (c) 400 g (d) 446 g (IIT-JEE 2010)

Chapter-18.indd 601 8/8/2016 3:33:26 PM


602 Chapter 18 Electrochemistry

7a. For the given cell Solution


(a) Ecell < 0; ∆G > 0 (b) Ecell > 0; ∆G < 0 (d) At the anode: M → M2+ + 2e−
(c) Ecell < 0; ∆Go > 0 (d) Ecell > 0; ∆Go > 0 At the cathode: M2+ + 2e− → M
Solution We know that ΔG = −nFEcell. Given that Ecell = 0.059 V,
1 F = 96500 C and n = number of electrons = 2.
(b) For the given concentration cell
Substituting, we get
2.303 RT C 2.303 RT 1
Ecell = log 2 = log = + ve ∆G = − 2 × 96500 C mol−1 × 0.059 V = − 113873 J mol−1
nF C1 nF 0.05
= − 11.387 kJ mol−1 = − 11.4 kJ mol−1
So, Ecell > 0. For the cell reaction to be spontaneous, ∆G <
0 (as ∆G = − nFEcell).
9b. The solubility product (Ksp; mol3 dm−9) of MX2 at 298 K based
7b. If the 0.05 M solution of M+ is replaced by 0.0025 M solution on the information available for the given concentration cell
of M+, then the magnitude of the cell potential would be is (take 2.303 × R × 298/F = 0.059 V).
(a) 35 mV (b) 70 mV (c) 140 mV (d) 700 mV (a) 1 × 10−15 (b) 4 × 10−15
(c) 1 × 10 −12 (d) 4 × 10−12
Solution
(c) For the first cell Solution
2.303RT 2.303RT (b) Using the Nernst equation at equilibrium
(1.301) = 0.07 ⇒ = 0.0538
F F 0.0591 [M2 + ]a
For the new cell Ecell = − log 2 + c
n [M ]
2.303 RT 1 0.0591 0.001
Ecell = log = 0.0538 log 400 = log 2 + a ⇒ [M2 + ] = 10 −5 = S
F 0.0025 2 [M ]
= 0.0538 × 2.6021 = 0.140 V = 140 mV
From the equation MX2  M2 + + 2X −
8. Consider the following cell reaction: S S 2S

2Fe(s) + O2 (g) + 4H+ (aq) → 2Fe2 + (aq) + 2H2O(l) E o = 167


. V Therefore, Ksp = 4S3 = 4 × (10−5)3 = 4 × 10−15 mol3 dm−9.
At [Fe2+] = 10−3 M, p(O2) = 0.1 atm and pH = 3, the cell poten- 10. An aqueous solution of X is added slowly to an aqueous solu-
tial at 20°C is tion of Y as shown in Column I. The variation in conductivity
(a) 1.47 V (b) 1.77 V of these reactions is given in Column II. Match Column I with
(c) 1.87 V (d) 1.57 V Column II and select the correct answer using the code given
(IIT-JEE 2011) below:
Solution
Column I Column II
(d) For the reaction
P. (C2H5)3N + CH3COOH 1. Conductivity decreases and
2Fe(s) + O2 (g) + 4H+ → 2Fe +2 (aq) + 2H2O(l) E o = 1.67 V then increases
X Y
Using Nernst equation,
Q. KI (0.1 M) + AgNO3 2. Conductivity decreases and
o  0.0591 [Fe+2 ]2 (0.01 M) then does not change much
Ecell = Ecell −  log
 n  [O2 ][H+ ]4 X Y
 0.0591 (10 −3 )2 R. CH3COOH + KOH 3. Conductivity increases and
= 1.67 −   log −1 then does not change much
 4  (10 )(10 −3 )4 X Y
 0.0591 10 −6 S. NaOH + HI 4. Conductivity does not change
= 1.67 −   log −13 much and then increases
 4  10 X Y
 0.0591 Codes:
= 1.67 −  × 7 = 1.57 V
 4  P Q R S
9. Read the paragraph and answer the questions that follow. (a) 3 4 2 1
The electrochemical cell shown below is a concentration cell. (b) 4 3 2 1
M|M2+ (saturated solution of a sparingly soluble salt, (c) 2 3 4 1
MX2||M2+ (0.001 mol dm−3)|M (d) 1 4 3 2
The emf of the cell depends on the difference in concentra- (JEE Advanced 2013)
tions of M2+ ions at the two electrodes. The emf of the cell at Solution
+
298 K is 0.059 V. (a) For P: (C2H5)3N + CH3COOH → (C2H5 )3 NHCH3COO −
(IIT-JEE 2012) X Y
9a. The value of ∆G (kJ mol−1) for the given cell is (take 1 F = First, the conductivity increases due to neutralization of
+
96500 C mol−1) CH3COOH and replacement of H+ by (C2H5 )3 NH, after it
(a) −5.7 (b) 5.7 (c) 11.4 (d) −11.4 becomes practically constant due to buffering, because

Chapter-18.indd 602 8/8/2016 3:33:29 PM


Advanced Level Problems 603

+ Solving, we get
of which [H+] becomes constant and [(C2H5 )3 NH]
o o 0.11
increases. 3EFe 3+
/Fe = − 0.11 ⇒ EFe3+ /Fe = − = − 0.036 V ≈ −0.04 V
3
For Q: KI (0.1 M) + AgNO3 (0.01 M) → AgI + KNO3
For Q: The half reactions and net reactions are as follows:
X Y
Initially, only Ag+ is replaced by K+, so the number of
2H2O → O2 + 4H+ + 4 e − E o = − 1.23 V
ions in the solution remain constant till all of AgNO3 pre-
cipitated as AgI. After this precipitation, conductance 4 e − + O2 + 2H2O → OH− E o = 0.40 V
increases due to increases in number of ions.
4H2O → 4H+ + 4 OH− E o = ?
For R: CH3COOH + KOH → CH3COOK + H2O
X Y For the overall reaction, E o = − 1.23 + 0.40 = − 0.83 V
Initially conductance decreases due to replacement of For R: The half reactions and net reactions are as follows:
OH− by CH3COO− and then slowly increases due to the
increases in number of H+ ions. Cu2 + + 2e − → Cu E o = + 0.34 V
For S: NaOH + HI → NaI + H2O 2Cu → Cu+ + 2e − E o = − 0.52 V
X Y
Initially, conductance decreases due to replacement of Cu2 + + Cu → 2Cu+ E o = ?
H+ ions by Na+ and then increases due to the increase in
For the overall reaction, E o = 0.34 − 0.52 = − 0.18 V
OH− ions.
For S: The half reactions and net reactions are as follows:
11. The standard reduction potential data at 25°C is given
below. Cr 3+ + 3e − → Cr
Eo(Fe3+, Fe2+) = + 0.77 V; Eo(Fe2+, Fe) = − 0.44 V Cr → Cr 2 + + 2e −
Eo(Cu2+, Cu) = + 0.34 V; Eo(Cu+, Cu) = + 0.52 V
Eo[O2(g) + 4H+ + 4e− → 2H2O] = + 1.23 V; Cr 3+ + e − → Cr 2+
Eo[O2(g) + 2H2O + 4e− → 4OH−] = + 0.40 V
Eo(Cr3+, Cr) = – 0.74 V; Eo(Cr2+, Cr) = – 0.91 V For the overall reaction,

Match Eo of the redox pair in Column I with the values given ∆G3o = ∆G1o + ∆G2o
in Column II and select the correct answer using the code o
−1F × ECr 3+
/Cr 2+ = 3 F × 0.74 − 2 F × 0.91
given below the lists.
Column I Column II o
Solving, we get ECr3+ /Cr2+ = − 0.4 V
P. Eo (Fe3+, Fe) 1. −0.18 V
+ −
12. In a galvanic cell, the salt bridge
Q. Eo ( 4H2O  4H + 4 OH ) 2. −0.4 V
(a) does not participate chemically in the cell reaction.
R. Eo(Cu2+ + Cu → 2Cu+) 3. −0.04 V (b) stops the diffusion of ions from one electrode to another.
S. Eo(Cr3+, Cr2+) 4. −0.83 V (c) is necessary for the occurrence of the cell reaction.
(d) ensures mixing of the two electrolytic solutions.
Codes: (JEE Advanced 2014)
Solution
P Q R S
(a), (b) The salt bridge contains neutral electrolyte and hence
(a) 4 1 2 3
does not participate in the cell reaction.
(b) 2 3 4 1
13. Copper is purified by electrolytic refining of blister copper.
(c) 1 2 3 4 The correct statement(s) about this process is (are)
(d) 3 4 1 2 (a) Impure Cu strip is used as cathode.
(JEE Advanced 2014) (b) Acidified aqueous CuSO4 is used as electrolyte.
Solution (c) Pure Cu deposits at cathode.
(d) For P: The half reactions and net reactions are as follows: (d) Impurities settle as anode mud.
Fe3+ + e − → Fe2 + (JEE Advanced 2015)

Fe2 + + 2e − → Fe Solution
(b), (c), (d) In electrolytic refining of blister copper acidified
Fe3+ + 3e − → Fe
aq. CuSO4 act as an electrolyte. At the anode, oxidation of
For the overall reaction, impure copper strip takes place and impurities get settled as
∆G3o = ∆G1o + ∆G2o anode mud.
o
−3 F × EFe 3+
/Fe = − 1 F × 0.77 − 2 F × ( −0.44 )
Cu(s) → Cu2+ (aq) + 2e−

Chapter-18.indd 603 8/8/2016 3:33:31 PM


604 Chapter 18 Electrochemistry

At cathode, made of pure strip, the reduction of copper  0.5   0.5 


2.303 × 8.314 × 300
ions takes place and pure copper gets deposited. 0.03 = log  +  ⇒ 0.03 = 0.03 log  
Cu2+ (aq) + 2e− → Cu(s)
2 × 96500 2
 [Cu ]1   [Cu2 + ]1 
2.303 × 8.314 × 300  0.5   0.5 
0.03 =of a weak acid HX (0.01
14. The molar conductivity of a solution log
M) + ⇒⇒ 0.03 = 0.03 log  
2 × 96500 2
 [Cu ]1   [Cu2 + ]1 
is 10 times smaller than the molar conductivity of a solution
of a weak acid HY (0.10 M). If l 0X − ≈ l 0Y − , the difference in  0.5  0.5
⇒ 1 = log   ⇒ = 10 ⇒ [Cu2 + ]1 = 0.05 M
their pKa values, pKa(HX) – pKa(HY), is .  [Cu2 + ]1  [Cu2 + ]1
(Consider degree of ionization of both acids to be 1.)
(JEE Advanced 2015) 16. Find the equilibrium constant for the reaction
Cu2+ + In2+  Cu+ + In3+ .
Solution
(3) For HX and HY, since l 0X − ≈ l 0Y − , Given that
oo
l C(HX ) a HX 1 EECu Cu++ ==00.15
Cu2+2+//Cu V,EEInoIno2+2+//InIn++ == −−00.4
.15 V, V,EEInoIno3+3+//InIn++ == −−00.42
.4 V, .42 VV .
= =
l C(HY ) a HY 10 Solution
H(+HX ) (Ca )HX (K aC )HX We know that ∆G o = − nFEcell
o
. Thus, we have three reac-
= = (1) tions as
H(+HY ) (Ca )HY (K aC )HY
Cu2+ + e − → Cu+ ∆G1o = − 0.15 F
H(+HX ) 0.01 1
≈ × =1
H(+HY ) 0.1 10 In2+ + e − → In+ ∆G2o = + 0.4 F

Taking negative log of both the sides of Eq. (1), we get In+ → In3+ + 2e − ∆G3o = − 2 × 0.42 = − 0.84 F

1  1  Adding all the equations, we get


pH(HX ) − pH(HY ) = − log(K aC )HX −  − log(K aC )HY 
2  2  Cu2+ + In2+ → Cu+ + In3+
1 1
0 = [pK a (HX ) − 2] − [pK a (HY ) − 1] and ∆G o = − nFE o = ( −0.15 + 0.4 − 0.84 ) F = − 0.59 F .
2 2
As n = 1, we get
0 = pK a (HX ) − pK a (HY ) − 3 ⇒ pK a (HX ) − pK a (HY ) = 3
o o
−0.59 F = − FEcell ⇒ Ecell = 0.59 V
15. Two students make Daniel cells in laboratory. They take
Now, using Nernst equation, we can find the equilibrium
ZnSO4 from common stock with Cu as positive electrode.
constant as
The emf of one cell is 0.03 V more than the other. The con-
centration of CuSO4 in cell of higher emf is 0.5 M. Find the o 0.0591
Ecell = Ecell − log K C
concentration of CuSO4 in second cell. n
[Given that (2.3RT)/F = 0.06 V.] where Ecell = 0 at equilibrium. So, we have
0.0591
Solution 0.59 = log K C ⇒ K C = 1010
1
From the reaction Zn + Cu2+  Zn2+ + Cu , we have:
17. (a) Calculate ∆Gro of the following reaction
2.303 RT [Zn2 + ]
For the first cell: E1 = E1o − log 2 + 1
2F [Cu ] Ag+ (aq) + Cl− (aq) → AgCl(s)
2.303 RT [Zn2 + ] o
Given that ∆G(AgCl) = − 109 kJ mol−1, ∆G(Cl
o −1 o
For the second cell: E2 = E1o − log 2 + 2 −
) = − 129 kJ mol , ∆G(Ag+ ) = 77 kJ m
2F [Cu ]
∆G o = − 109 kJ mol−1, ∆G(Cl o −1 o
) = − 129 kJ mol , ∆G(Ag+ ) = 77 kJ mol
−1
. Represent the above
Subtracting E2 from E1, we get (AgCl)

o
reaction in form of a cell. Calculate Ecell and find
2+ 2+
o 2 .303 RT [ Zn ] o 2 .303 RT [ Zn ] log K of AgCl.
E2 − E1 = E1 − log 2 + − E1 + log 2 + 10 sp
2F [Cu ]2 2F [Cu ]1 (b) 6.539 × 10−2 g of metallic Zn (atomic mass = 65.39) was
2.303RT  [Zn2 + ] [Zn2 + ]  added to 100 mL of saturated solution of AgCl, calculate
= log 2 + − log 2 +  2+
2F  [Cu ]1 [Cu ]2  log10 [Zn ] . Given that
2.303RT  [Zn2 + ] [Cu2 + ]2  2.303RT  [Cu2 + ]2  [Ag+ ]2
= log  ×  = log  
2F  [Cu2 + ]1 [Zn2 + ]  2F  [Cu2 + ]1  Ag+ + e − → Ag o
Ecell = 0.80 V

Given that E2 − E1 = 0.03 V and [Cu2+] in the second cell Zn2 + + 2e − → Zn o


Ecell = − 0.76 V
= 0.5 M. Substituting, we get Also find the number of moles of Ag formed.

Chapter-18.indd 604 8/8/2016 3:33:38 PM


Advanced Level Problems 605

Solution
(a) 100.32/0.0591 (b) 100.32/0.0295
(a) The cell reactions are
(c) 100.26/0.0295 (d) e 0.32 / 0.295
+ 1 −
Ag + Cl → AgCl
2 Solution
Ag → Ag + e − +
(b) Consider the reaction Zn + Fe2 + → Zn2 + + Fe
1 − −
Cl
2 2
+ e → Cl o 0.0591
E = Ecell − log K C
The net reaction is Ag+ (aq) + Cl− (aq) → AgCl(s). n
The cell representation is Ag|Ag+ |AgCl|| Cl− |Cl2 , Pt. o
0.2905 = Ecell
 0.0591
−
 0.1 
log 
 2   0.01
ΔGo = −109 − (−129 + 77) = −109 + 129 − 77 = −57
= −1 × F × Eo o
Solving, we get Ecell = 0.2905 + 0.02905 = 0.32.
o 57000
Therefore, Ecell = = 0.59 V  0.0591  0.0591
96500 o
Now, Ecell = log K C ⇒ 0.32 =  log K C
o
Now, ΔG = −2.303 RT log K. So, we have  n   2 
57 × 1000
logK = = 9.98 ≈ 10 ⇒ K = 1010 Solving, we get K C = 100.32 / 0.0295.
2.303 × 8.314 × 298
20. From the following data
But, Ksp = 1/K, so Ksp = 10−10 ⇒ log Ksp = −10.
The solubility, S is calculated as follows. 2H+ (aq) + 21 O2 (g) + 2e − → H2O(l) Ecell
o
= 1.23 V
Ksp = S2 ⇒ S = (10−10)1/2 = 10−5 mol L−1
Fe2 + (aq) + 2e − → Fe(s) Ecell
o
= − 0.44 V
(b) Adding the following reactions
show that ∆Go for
2Ag+ + 2e − → 2Ag EAg
o
/ Ag = 0.80 V
2Fe(s) + 4H+ (aq) + O2 (g) → 2Fe2 + (aq) + 2H2O(l) is
+

2+ − o
Zn → Zn + 2e EZn = + 0.76 V
/ Zn2+ (a) −322.3 kJ mol−1 (b) −483.5 kJ mol−1
we get (c) −644.6 kJ mol−1 (d) 644.6 kJ mol−1
2Ag+ + Zn2 + → 2Ag + Zn Ecell
o
= 0.80 + 0.76 = 1.56 V Solution
+
2Ag + Zn 2+
→ 2Ag + Zn o
Ecell = 0.80 + 0.76 = 1.56 V (c) The reactions involved are

At equilibrium, (Ecell = 0) 2H+ (aq) + 21 O2 (g) + 2e − → H2O(l) Ecell


o
= 1.23 V (1)

o 0.0591 [Zn2 + ] Fe2 + (aq) + 2e − → Fe(s) Ecell


o
= − 0.44 V (2)
Ecell = log
2 [Ag+ ]2 Multiplying Eqs. (1) and (2) by 2, we get
Therefore,
2 [Zn2 + ] [Zn2 + ] 4H+ (aq) + O2 (g) + 4 e − → 2H2O(l) Ecell
o
= 2.46 V (3)
1.56 × = log +
⇒ log = 52.8
0.0591 [Ag ] 2
[Ag+ ]2
Given that (65.39 × 10−2)/65.39 = 10−3 mol of Zn has been 2Fe2 + (aq) + 4e − → 2Fe(s) Ecell
o
= − 0.88 V (4)
added. Now, 10−6 mol of Ag reacts with 10−3 mol of Zn. Reversing Eq. (4), we get
Therefore, this reaction will move in the forward direction 2Fe(s) → 2Fe2 + (aq) + 4e − Ecell
o
= + 0.88 V (5)
completely. Hence, moles of Ag formed will be 10−6 mol.
Adding Eqs. (3) and (5) , we get
18. Which of the following statements is correct for an electrolytic
2Fe + 4H+ + O2 → 2Fe2 + + 2H2O Ecell
o
= 2.46 + 0.88 = 3.34 V
cell?
(a) Electrons flow from cathode to anode through the+ exter-
2Fe 4H+ + O2 → 2Fe2 + + 2H2O Ecell
o
= 2.46 + 0.88 = 3.34 V
nal battery.
Therefore,
(b) Electrons flow from cathode to anode within the electro-
lytic solution. ∆G o = − nFEcell
o
= − 2 × 96500 × 3.34 = − 644.6 kJ mol−1
(c) Migration of ions along with oxidation reaction at cath- 21. The value of Ksp for AgBr is 5 × 10−13. A galvanic cell is con-
ode and reduction reaction at anode. structed using standard hydrogen electrode as one half-cell
(d) Migration of ions along with reduction reaction at cath- and a silver wire coated with AgBr dipping into 0.1 M HBr
ode and oxidation reaction at anode. solution as the other half-cell. Which of the following state-
Solution ments is correct about the cell?
(d) In an electrolytic cell, electrons do not flow themselves. [Given that AgBr(s) + e −  Ag( s) + Br − ( aq); EAgBr/Ag
o
= 0.07 V]
It is the migration of ions towards oppositely charged
electrodes that indirectly constitute the flow of electrons (a) The oxidation of silver bromide takes place in the cell.
from cathode to anode through internal supply. (b) Reduction of H+(aq) takes place in the cell.
(c) Ecell = 0.129 V .
19. Zn|Zn2 + (a = 0.1 M)||Fe2 + (a = 0.01M)|Fe. The emf of the cell is
0.2905 V. Equilibrium constant for the cell reaction is (d) Ecell = −0.129 V .

Chapter-18.indd 605 8/8/2016 3:33:44 PM


606 Chapter 18 Electrochemistry

Solution AgBr(s)/Ag(s) half-cell is put into 0.1 M HBr solution; so


(c) Given that one half cell is the standard hydrogen elec- even though the concentration of Ag+(aq) is limited but
trode and the other half cell is AgBr(s)/Ag(s). The cell the concentration of Br− (aq) is present.
reaction is Thus,
0.059
At the cathode: 1
H − e −  H+
o
Ecell = Ecell − log[H+ ][Br − ]
2 2 1
At the anode: Ag (s) + e −  Ag(s)
+
= (0.07 − 0 ) −
0.059
log(1)(0.1)
1
Cell reaction: 21 H2 + Ag+ ( s)  H+ + Ag(s)
= 0.07 + 0.059 = 0.129 V
For standard conditions:
The potential is positive, so, the reduction of silver bro-
AgBr(s) + e −  Ag( s) + Br − ( aq); E o = 0.07 V mide takes place in the cell and hydrogen is oxidized.

PRACTICE EXERCISE
Level I 8. How many coulombs of electric current would be needed to
reduce the aluminium in 1.0 mol of Al2(SO4)3 to aluminum
Single Correct Choice Type metal?
1. What mass of Cu could be plated out by electrolyzing aqueous (a) 2.89 × 10 4 C (b) 3.58 × 10 4 C
CuSO4 for 12 hours at 2.0 amperes? (c) 5.79 × 105 C (d) 7.02 × 105 C
(a) 58 g (b) 28 g (c) 120 g (d) 430 g
9. The specific conductance of a saturated AgCl solution is
2. The standard electrode potential of OX−/X− and X−/X2, respec- found to be 1.86 × 10−6 S cm−1 and that for water is 6.0 × 10−8
tively, is 0.94 V and −1.36 V. The Eo value for OX−/X2 will be S cm−1. The solubility of AgCl is (Λ oeq = 137.2 S equivalent−1
(a) −0.42 V (b) +0.42 V (c) 0.21 V (d) −1.04 V cm2)
(a) 1.7 × 10−3 mol L−1 (b) 1.3 × 10−5 mol L−1
−4 −1 (d) 1.3 × 10−6 mol L−1
3. By how much would the oxidizing power of the (MnO 4− | Mn2 + ) (c) 1.3 × 10 mol L
couple change if the H+ ions concentration is decreased 100
10. In passing 3 F of electricity through the three electrolytic
times at 25°C?
cells connected in series containing Ag+ , Ca2+ , and Al+3 ions,
(a) It will increase by 189 mV.
respectively. The molar ratio in which the three metal ions
(b) It will decrease by 189 mV.
are liberated at the electrodes is
(c) It will increase by 19 mV.
(a) 1:2:3 (b) 3:2:1 (c) 6:3:2 (d) 3:4:2
(d) It will decrease by 19 mV.
11. The standard potentials at 25°C for the following half-cell
4. Calculate the electrode potential of copper, if the concentra- reactions are given as
o
tion of CuSO4 is 0.206 M at 23.1°C. Given that ECu2+
/ Cu = +0.34 V.
(a) 0.50 V (b) 0.41 V Zn+2 + 2e − → Zn Ecell
o
= − 0.762 V
(c) 0.32 V (d) 0.28 V Mg2 + + 2e − → Mg Ecell
o
= − 2.37 V
5. Faraday’s laws of electrolysis are related to the When zinc dust is added to a solution of magnesium
(a) atomic number of cation. chloride,
(b) atomic number of anion. (a) no reaction will take place.
(c) equivalent weight of the electrolyte. (b) zinc chloride is formed.
(d) speed of the cation. (c) zinc dissolves in solution.
6. Which of the following statements is correct? (d) magnesium is precipitated.
Galvanic cell converts 12. Passage of three faraday of charge through aqueous solution
(a) chemical energy into electrical energy. of AgNO3, CuSO4, Al(NO3)3 and NaCl will deposit metals at
(b) electrical energy into chemical energy. the cathode in the molar ratio of
(c) metal from its elemental state to the combined state. (a) 1:2:3:1 (b) 6:3:2:6 (c) 6:3:0:0 (d) 3:2:1:0
(d) electrolyte into individual ions.
13. Conductivity (units Siemens, S) is directly proportional to the
7. If the molar conductance values of Ca2+ and Cl− at infinite area of the vessel and the concentration of the solution in it
dilution are, respectively,118.88 × 10−4 and 77.33 × 10−4 S m2 and is inversely proportional to the length of the vessel, then
mol−1, then that of CaCl2 is (Sm2 mol−1) the unit of constant of proportionality is
(a) 118.88 × 10−4 (b) 154.66 × 10−4 (a) S m mol−1 (b) S m2 mol−1
(c) 273 × 10−4 (d) 196.21 × 10−4 (c) S−2 m2 mol (d) S2 m2 mol−2

Chapter-18.indd 606 8/8/2016 3:33:47 PM


Practice Exercise 607

14. A current of 9.65 A is drawn from a Daniell cell for exactly 1 h. 25. In the electrochemical cell
The loss in mass at anode and gain in mass at cathode, respec- H2 (g) 1 atm | H+ (1 M) || Cu2 + (1 M) | Cu(s) ,
tively, are which one of the following statements is true?
(a) 11.43 g, 11.77 g (b) 11.77 g, 11.43 g (a) H2 is anode, Cu is cathode.
(c) 22.86 g, 23.54 g (d) 23.54, 22.86 g (b) Cu is anode, H2 is cathode
15. According to Faraday’s law of electrolysis, the mass deposited (c) Oxidation occurs at Cu electrode.
on an electrode is proportional to (d) Reduction occurs at H2 electrode.
(a) m ∝ Q (b) m ∝ Q2 26. The compound exhibiting maximum value of equivalent con-
(c) m ∝ I 2 (d) Both (b) and (c) ductance in a fused state is
16. If the hydrogen electrodes in two solutions of pH = 3 and pH (a) SrCl2 (b) CaCl2 (c) MgCl2 (d) BeCl2
= 6 are connected by a salt bridge, the emf of the resulting 27. How many seconds would it take a 10.0 A current to produce
cell is enough aluminium from Al3+ to make a 27.0 g aluminium
(a) 0.177 V (b) 0.3 V (c) 0.052 V (d) 0.104 V can?
4 3
(a) 2.89 × 10 (b) 9.65 × 10
17. Standard electrode potential of standard hydrogen electrode
(c) 3.22 × 10 3
(d) 9.65 × 10 4
(SHE) at 298 K is
(a) 0.05 V (b) 0.10 V (c) 0.50 V (d) 0.00 V 28. If 0.3605 g of a metal is deposited on the electrode by pass-
ing 1.2 A current for 15 min through its salt, what will be its
18. Calculate the minimum voltage required to bring about the
valence? (Atomic weight of the metal is 96.)
electrolysis of 1 M copper sulphate solution at 25°C. Given
o o (a) 3 (b) 5 (c) 6 (d) 8
that ECu 2+
/Cu = 0.34 V and EH2 O/H+ = − 1.23 V .
(a) 0.25 V (b) 0.56 V 29. Units of ionic mobility are
(c) 0.89 V (d) 0.42 V (a) m V−1 s−1 (b) m2 V−2 s−1
(c) m2 V−1 s−1 (d) m−2 V s−1
19. Find the value of equilibrium constant for the reaction,
2Fe3+ + I3 −  2Fe2 + + I− . The standard reduction poten- 30. Value of EHo2 O/H2 (1 atm) at 298 K would be
tials in acidic conditions are 0.77 V and 0.54 V, respectively, for (a) −0.207 V (b) +0.207 V (c) −0.414 V (d) +0.414 V
Fe3+ | Fe2 + and I− | I3 − couples
(a) 5.2 × 108 (b) 6.26 × 107 31. Which one of the following metals cannot be obtained on
(c) 3.8 × 109 (d) 4.3 × 1010 electrolysis of aqueous solution of its salts?
(a) Mg (b) Ag (c) Cu (d) Cr
20. Determine the concentration of Cd2+ ions in the following
electrochemical cell: 32. The E° for half cells Fe/Fe2+ and Cu/Cu2+ are –0.44 V and +0.32 V,
respectively. Then
Fe|Fe2+ (0.1M)||Cd2+ (xM)|Cd (a) Cu2+ oxidizes Fe. (b) Cu oxidizes Fe2+.
Given that Ecell = −0.02 V and Eo = 0.04 V at 298 K. (c) Cu reduces Fe2+. (d) Cu2+ oxidizes Fe2+.
(a) 7.4 × 10−4 M (b) 8.2 × 10−4 M 33. The equilibrium constant for the following reaction at 298 K is
−4
(c) 5.6 × 10 M (d) 9.3 × 10−4 M expressed as x × 10y.
21. In an electrolysis of acidulated water, 4.48 L of hydrogen was 2Fe3+ + 2I− → 2Fe2 + + I2 , Ecell
o
= 0.235 V
produced by passing a current of 2.14 A. For how many hours The value of y is ___.
was the current passed? (a) 2 (b) 7 (c) 10 (d) 6
(a) 4 (b) 3 (c) 6 (d) 5
34. Calculate the concentration of NiCl2 in the nickel elec-
22. Specific conductance of 0.01 M KCl solution is x S cm–1. When trode having potential of −0.16942 V at 24.9°C. Given that
o
conductivity cell is filled with 0.01 M KCl the conductance ENi2+
/ Ni = −0.14 V.
observed is y S. When the same cell is filled with 0.01 M H2SO4, (a) 0.152 V (b) 0.1011 M
the observed conductance is z S cm–1. Hence specific con- (c) 0.213 V (d) 0.201 M
ductance of 0.01 M H2SO4 is
(a) xz (b) z/xy (c) xz/y (d) xy/z 35. One coulomb of charge passes through solution of AgNO3
and CuSO4 connected in series and the concentration of the
23. The same amount of electricity was passed through two sepa- two solutions being in the ratio 1:2. The ratio of weight of Ag
rate electrolytic cells containing solutions of nickel nitrate and and Cu deposited on Pt electrode is
chromium nitrate, respectively. If 0.3 g of nickel was depos- (a) 107.9:63.54 (b) 54:31.77
ited in the first cell, the amount of chromium deposited is (c) 107.9:31.77 (d) 54:63.54
(Atomic weight Ni = 59 u, Cr = 52 u)
(a) 0.1 g (b) 0.17 g (c) 0.3 g (d) 0.6 g 36. For HCl solution at 25°C, the equivalent conductivity at infinite
dilution is 425 S cm−1 equiv.−1 The specific conductance of a
24. How many faradays are required to reduce 1 mol of BrO3− to solution of HCl is 3.825 S cm−1. If the apparent degree of dis-
Br−? sociation is 90%, the normality of solution is
(a) 3 (b) 5 (c) 6 (d) 4 (a) 0.9 (b) 10.00 (c) 1.1 (d) 1.2

Chapter-18.indd 607 8/8/2016 3:33:50 PM


608 Chapter 18 Electrochemistry

37. Consider the following electrolytes: (i) AgNO3; (ii) CuSO4; concentration of anodic solution reduced to 0.0005124
(iii) AlCl3; (iv) Bi2(SO4)3. The quantity of electricity needed to equiv. The weight of Cu deposited was 0.03879 g. Calculate
electrolyze separately 1 M solutions of these electrolytes will the transport numbers of Ag+ and NO3− in AgNO3 solution.
be (F is faraday) (a) 0.4214, 0.5786 (b) 0.3256, 0.4568
(a) 1 F, 2 F, 3 F, 6 F (b) 2 F, 3 F, 6 F, 1 F (c) 0.4589, 0.6021 (d) 0.3562, 0.7526
(c) 3 F, 6 F, 2 F, 1 F (d) 6 F, 3 F, 2 F, 1 F
48. The same quantity of electricity that liberates 4.316 g of sil-
38. From the following molar conductivities at infinite dilution, ver from AgNO3 solution was passed through a solution of
Λm0
for Ba(OH)2 = 457 Scm2 mol−1, Λ m 0
for BaCl2 = 240.6 Scm2 mol−1 gold salt. If the atomic weight of gold be 197 and its valency
in the above-mentioned salt be 3. Calculate the weight of
0
Λm for BaCl2 = 240.6 Scm2 mol−1 and Λ m 0
for NH4 Cl = 2129.8 Scm2 mol−1,. calcu-
gold deposited at the cathode and the quantity of electricity
0
late Λ m for NH4 OH. passed.
2 −1 2 −1 (a) 4021.5 C (b) 5023.3 C
(a) 2056 Scm mol (b) 2238 Scm mol
2 −1 2 −1 (c) 6213.5 C (d) 3854.1 C
(c) 2536 Scm mol (c) 2698 Scm mol
49. The standard reduction potential of Cu2+/Cu is +0.34 V.
2+ 3+
39. Emf of the cell Ni|Ni (0.1M)|| Au (1.0 M)| Au will be Calculate the reduction potential at pH = 14 for the
o o above couple in a saturated solution of cupric hydroxide.
2+ = 0.25 V , E
(Given ENi/Ni Au/Au3+ = 1.5 V) Ksp (Cu(OH)2) = 1 × 10−19 M3.
(a) 1.75 V (b) +1.7795 V (a) −1.06 V (b) −0.22 V (c) −1.25 V (d) −0.39 V
(c) +0.7795 V (d) −1.7795 V
50. How many grams of Cl2 are produced when molten NaCl
40. Which of the following is displaced by Fe? undergoes electrolysis at a current of 4.25 A for 35.0 min?
(a) Ag (b) Zn (a) 5.21 g (b) 2.51 g (c) 3.28 g (d) 4.09 g
(c) Na (d) All of these
41. The density of Cu is 8.94 g cm−3. The quantity of electricity Level II
needed to plate an area of 10 cm × 10 cm to a thickness of
Multiple Correct Choice Type
10−2 cm using CuSO4 solution would be
(a) 13586 C (b) 27172 C (c) 40758 C (d) 20348 C 51. Lead storage battery contains
(a) Lead rod as anode.
42. The element which can displace three other halogens from
(b) Lead rod as cathode.
their compounds is
(c) Lead plates coated with PbO2 act as cathode.
(a) F (b) Cl (c) Br (d) I
(d) H2SO4 as the electrolyte.
43. The conductivity of a strong electrolyte
52. Which of the following statements is (are) correct?
(a) increases on dilution slightly.
(a) The reactivity of metals decreases in going down the
(b) decreases on dilution.
electrochemical series.
(c) does not change with dilution.
(b) A metal can displace any other metal placed above it in
(d) depends upon density of electrolytes itself.
the electrochemical series from its salt solution.
44. When current of 5 A is passed through a solution of zinc sul- (c) The oxidizing power of the substances decrease from
phate for 40 min, the amount of zinc deposited at the cath- the top to the bottom in the electrochemical series.
ode is (d) A redox reaction is feasible when the substance having
(a) 4.065 g (b) 8.065 g higher reduction potential gets reduced and the one
(c) 16.065 g (d) 32.065 g having lower reduction potential gets oxidized.
45. An alloy of Pb–Ag weighing 1.08 g was dissolved in dilute 53. Which are true for a standard hydrogen electrode?
HNO3 and the volume made to 100 mL. A silver electrode (a) The hydrogen ion concentration is 1 M.
was dipped in the solution and the emf of the cell setup as (b) Temperature is 25°C.
(c) Pressure of hydrogen is 1 atm.
Pt(s), H2 (g)| H+ (1 M)|| Ag+ (aq)|Ag(s)
(d) It contains a metallic conductor which does not adsorb
o
was 0.62 V. If Ecell is 0.80 V, what is the percentage of Ag in the hydrogen.
alloy? (At 25°C, RT/F = 0.06) 54. Hydrogen gas will reduce
(a) 25 (b) 2.50 (c) 10 (d) 50 (a) heated cupric oxide.
46. Molar conductances of BaCl2, H2SO4, and HCl at infinite dilu- (b) heated silver oxide.
tions are x1, x2, and x3, respectively, equivalent conductance (c) heated zinc oxide.
of BaSO4 at infinite dilution will be (d) heated aluminum oxide.
(a) (x1 + x2 − x3)/2 (b) x1 + x2 − 2x3
(c) (x1 − x2 − x3)/2 (d) (x1 + x2 − 2x3)/2 55. For the cell Tl | Tl+ (0.001 M)|| Cu2 + (0.1 M)| Cu,. given that Ecell is
0.83 V. Then can be increased by
47. A decinormal solution of AgNO3 was electrolyzed between (a) increasing [Cu2+]. (b) increasing [Tl+].
Pt electrodes. After passing a small current for two hours, (c) decreasing [Cu ]. 2+ (d) decreasing [Tl+].

Chapter-18.indd 608 8/8/2016 3:33:52 PM


Practice Exercise 609

56. In which of the following salt bridge is not needed?


AgCl(s) + e − → Ag+ (s) + Cl− (aq); E o = 0.22 V
(a) Pb|PbSO4(s) |H2SO4(aq)|PbO2(s) |Pb
(b) Cd|CdO(s) |KOH(aq)|NiO2(s) |Ni Ag+ (aq) + e − → Ag(s); E o = 0.80 V
(c) Fe(s) |FeO(s) |KOH(aq)|Ni2O3(s) |Ni
(d) Zn|ZnSO4|CuSO4(aq)|Cu The solubility product of AgCl under standard conditions of
temperature (298 K) is given by
57. Select the correct statements if 9.65 A current is passed for 1 h
(a) 1.6 × 10−5 (b) 1.5 × 10−8
through the cellAg | Ag+ (1 M)|| Cu2 + (1 M)| Cu. −10
(c) 3.2 × 10 (d) 1.5 × 10−10
(a) Ag will oxidize to Ag+ and new [Ag+] = 1.36 M.
(b) Ag+ will reduce to Ag and new [Ag+] = 0.64 M. 62. At 25 °C, the standard emf of a cell with reaction involving
(c) Cu2+ will reduce to Cu and new [Cu2+] = 0.82 M. two electron exchange is found to be 0.295 V. The equilibrium
(d) Cu will oxidize to Cu2+ and new [Cu2+] = 0.82 M. constant of the reaction is
(a) 29.5 × 10−2 (b) 10
58. Which of the following statements is(are) incorrect?
(c) 1 × 1010 (d) 29.5 × 1010
(a) Electrolysis of sulphuric acid (dilute or concentrated)
gives H2 at cathode and O2 at anode.
(b) Electrolysis of dilute NaOH solution gives H2 at cathode Paragraph for Questions 63 to 65: Molar conductance of NaCl var-
0
and O2 at anode. ies with concentration according to the equation Λ m = Λ m −b C
0
(c) Oxidation of copper anode occurs in the electroly- where Λ m is molar specific conductance, Λ m is molar specific con-
sis of aqueous copper sulphate solution using copper ductance at infinite dilution, and C is the molar concentration. The
electrodes. variation is shown in the following table:
(d) Electrolysis of aqueous KF solution gives fluorine at the
Molar conductance in
anode. Molar concentration of NaCl
S cm2 mol-1
Passage Type 4 × 104 107

Paragraph for Questions 59 to 62: The devices which convert 9 × 104 97


electrical energy into chemical energy or vice versa are called elec- 16 × 10−4 87
trochemical cells. There are two principal types of electrochemi-
cal cells. A galvanic cell is an electrochemical cell that produces 63. When a certain conductivity cell was filled with 25 × 10−4 M
electricity as a result of spontaneous reaction occurring inside NaCl solution, the resistance of the cell was found to be 1000 Ω.
it. An electrolytic cell is an electrochemical cell in which a non- At infinite dilution, conductance of Cl− and SO2− 4 are 80 S cm
2
spontaneous reaction is driven by an external source of current. −1 2 −1
mol and 160 S cm mol , respectively. What is the molar
In most of the electrochemical cells, indirect redox reactions take conductance of NaCl at infinite dilution?
place and can be expressed in terms of half-cell reactions. Each (a) 147 S cm2 mol–1 (b) 107 S cm2 mol–1
half reaction has a definite value of standard electrode potential. (c) 127 S cm2 mol−1 (d) 157 S cm2 mol–1
The overall reaction is represented by standard electrochemi-
cal conventions. Knowing the standard electrode potentials of 64. What is the cell constant of the conductivity cell?
the half reactions, the standard emf of the cell can be calculated. (a) 0.385 cm−1 (b) 3.85 cm−1
(c) 38.5 cm −1 (d) 0.1925 cm−1
The standard emf further helps in the calculation of free energy
change, equilibrium constant of the cell reaction as well as param- 65. If the cell is filled with 5 × 10−3 (N) Na2SO4, the observed resist-
eters like solubility product of a sparingly soluble salt. A concen- ance was 400 Ω. What is the molar conductance of Na2SO4?
tration cell is a galvanic cell in which both electrodes contain the (a) 19.25 S cm2 mol−1 (b) 96.25 S cm2 mol−1
same electrode material, the same electrolyte but with different (c) 385 S cm mol 2 −1 (d) 192.5 S cm2 mol−1
concentrations.
1 Matrix-Match Type
59. The reaction H2 (g) + AgCl(s) → H+ (aq) + Cl− (aq) + Ag (s)
2 66. Match the quantity with its variation.
occurs in the galvanic cell
Column I Column II
(a) Ag|AgCl(s) ||KCl(aq)|AgNO3(aq)|Ag (a) Molar conductance of a (p) Shows deviations at higher
(b) Pt|H2(g) |HCl(aq)||AgNO3(aq)|Ag strong electrolyte concentration from linear
(c) Pt|H2(g) |HCl(aq)||AgCl(s) |Ag behaviour
(d) Pt|H2(g) |KCl(aq)||AgCl(s) |Ag (b) Molar conductance of a (q) Shows almost no variation
60. The standard electrode potential (Eo) for Ni2+/Ni and Au3+/Au weak electrolyte with dilution at higher
respectively, are −0.25 V and 1.50 V. The emf of the cell will be dilutions
(a) 1.25 V (b) −1.75 V (c) 1.75 V (d) 1.40 V (c) Specific conductance (r) Decreases with dilution
61. The standard reduction potentials for two reactions are given (d) Conductance (s) Increases with increase in
as follows: concentration

Chapter-18.indd 609 8/8/2016 3:33:54 PM


610 Chapter 18 Electrochemistry

67. Match the electrode with its naming convention. Integer Type
Column I Column II 69. Suppose 0.25 mol of propane is subjected to combustion.
If this reaction is used for making a fuel cell, the number of
(a) Electrode on which oxidation occurs (p) Anode
moles of electrons involved in each half cell for this amount
(b) Electrode on which reduction (q) Cathode of propane will be ___.
occurs
70. The number of faraday of electricity required to deposit 81 g
(c) Electrode connected to negative (r) Negative pole of Al from electrolysis of AlCl3 is ___.
pole of the battery
(d) Electrode connected to the positive (s) Positive pole 71. In an electrolysis of acidulated water, 4.48 L of hydrogen was
pole of the battery produced by passing a current of 2.14 A for ____.
72. Three litres of 0.5 M K2Cr2O7 solution have to be completely
68. Match the quantity with the units. reduced in the acidic medium. The number of faraday of
electricity required will be ___.
Column I Column II
73. At equimolar concentration of Fe2+ and Fe3+ what must
(a) Conductance (p) S m2 mol−1
[Ag+] be so that the voltage of the galvanic cell made from
(b) Conductivity (q) S m−1 Ag+/Ag and Fe3+/Fe2+ electrodes equals zero? The reaction
(c) Molar conductance (r) m−1 is Fe2 + + Ag+  Fe3+ + Ag. Determine the equilibrium con-
o
(d) Cell constant (s) S stant at 25°C for the reaction. Given: EAg +
/Ag = 7.99 V and
o
EFe+ /Fe2+ = 0.771V.

ANSWER KEY
Level I
Single Correct Choice Type
1. (b) 2. (a) 3. (b) 4. (c) 5. (c) 6. (a)
7. (c) 8. (c) 9. (b) 10. (c) 11. (a) 12. (c)
13. (b) 14. (b) 15. (a) 16. (a) 17. (d) 18. (c)
19. (b) 20. (d) 21. (d) 22. (c) 23. (b) 24. (c)
25. (a) 26. (a) 27. (a) 28. (a) 29. (c) 30. (c)
31. (a) 32. (a) 33. (b) 34. (b) 35. (c) 36. (b)
37. (a) 38. (b) 39. (b) 40. (a) 41. (b) 42. (a)
43. (a) 44. (a) 45. (c) 46. (b) 47. (a) 48. (d)
49. (b) 50. (c)

Level II
51. (a), (c), (d) 52. (a), (d) 53. (a), (b), (c) 54. (a), (b) 55. (a), (d) 56. (a), (b), (c)
57. (a), (c) 58. (a), (d) 59. (c) 60. (c) 61. (d) 62. (b)
63. (c) 64. (d) 65. (d) 66. (a) → p; (b) → q; (c) → r; (d)→ r, s
67. (a) → p, r; (b) → q, s; (c) → q; (d) → p 68. (a) → s; (b) → q; (c) → p; (d) → r 69. (5) 70. (9)
71. (5) 72. (9) 73. (3)

HINTS AND EXPLANATIONS


Level I ItM
m= where , M(Cu) = 63.5; Z = 2
Single Correct Choice Type FZ
[2 × 12 × 60 × 60 × 63.5]
ItM = = 28.4 g
1. (b) Using Faraday’s law m = 96500 × 2
FZ

Chapter-18.indd 610 8/8/2016 3:33:55 PM


Hints and Explanations 611

2. (a) o
Ecell o
= Ecathode o
− Eanode = 0.96 − 1.36 = − 0.42 V W
10. (c) Faraday’s second law states that = constant
E
3. (b) The reaction involved is On passing equal amount of charge from three electro-
MnO 4− + 5e − + 8H+ → Mn2 + + 4H2O lytic cell
According to Nernst equation, For 1 F charge Ag+ deposited

o 0.059  [Mn2 + ]  Ag+ + e − → Ag


Ered = Ered − log  + 8
5 −
 [MnO 4 ][H ]  M 1
EAg = ⇒ Ag deposited = = 1 mol
1 1
After decrease in concentration, let [H+] = x/100 = x/102,
therefore, Ca2 + + 2e − → Ca

o 0.059  [Mn2 + ] × 1016  M 1


Ered = Ered − log  − + 8  ECa = ⇒ Ca deposited = mol
5  [MnO 4 ][H ]  2 2
−0.059 Al3+ + 3e − → Al
= log1016 = − 0.189 V = − 189 mV
5 M 1
This Ered decreases by 0.189 V. The tendency of the EAl = ⇒ Al deposited = mol
3 3
half cell to get reduced is its oxidizing power. Hence, the 1 1
oxidizing power decreases by 0.189 V. So, mole ratio of Ag+ : Ca2+ : Al3+ =1 : : = 6:3:2
2 3
4. (c) Given that, [CuSO 4 ] = 0.206 ; T = 296.1 K
The reaction taking place is 11. (a) The reaction is Zn + MgCl2 → ZnCl2 + Mg . Therefore,
o o o
Cu2 + + 2e − 


 Cu; n = 2 Ecell = EZn / Zn2+ + EMg2+ / Mg = + 0.762 − 2.37 = −1.608 V

According to Nernst equation, o o o


Since Ecell = Zn/Zn2+ + Eso
is Enegative, Mgno
2+ reaction will take place.
/Mg
o 2.303RT 1 12. (c) According to Faraday’s second law of electrolysis, the
ECu2+ / Cu = ECu2+
/ Cu − log 2 +
nF [Cu ] passage of same charge through different electrolytes
2.303 × 8.314 × 296.1 1 brings in equal equivalents of ions to be oxidized or
= 0.34 − × log
2 × 96 , 500 0.206 reduced at either electrodes.
= 0.32 V
Area × Concentration
13. (b) Conductivity ∝
5. (c) Faraday’s second law states that the number of faradays Length
passed is equal to the number of gram equivalents of elec-
trolytes discharged. Area × Concentration
Conductivity = k ×
Length
6. (a) Galvanic cell converts chemical energy into electrical
energy. These are also known as chemical cells or redox Conductivity × Length S×m
k = = = S m2 mol−1
cells or voltaic cells. Area × Concentration m2 × mol m−3

7. (c) o
Λm 0
(CaCl2 ) = l m (Ca2 + ) + 2l m
0
(Cl− )
9.65 × 1 × 3600
14. (b) Number of farads passed =
= 118.88 × 10 −4 −4
+ 2(77.33 × 10 ) 96500
Let WZn grams of zinc be discharged at anode and WCu
= 273.54 × 10 −4 Sm2 mol−1
grams of copper be discharged at cathode.
8. (c) Al2(SO4)3 → 2Al3+ + 3(SO4)2− 63.5 × 9.65 × 3690
Al3+ + 3e− → Al WCu = = 11.43 g
96500 × 2
1 mol of Al2(SO4)3 will give 2 mol of Al3+. To reduce 1 mol 65.4 × 9.65 × 3600
WZn = = 11.77 g
of Al3+, 3 mol of electrons are required. For 2 mol of Al3+ 96500 × 2
we need 6 mol of electrons or 6 F of electricity, that is,
15. (a) According to Faraday’s law of electrolysis, the mass
6 × 96500 = 5.79 × 105 C.
deposited on an electrode is proportional to the quan-
9. (b) Specific conductance k AgCl = ksolution−kwater tity of electricity passed through the electrolyte. If 1 F
= 1.86 × 10−6 − 6 × 10−8 (96500 C) of electricity is passed through copper solu-
tion, the mass deposited on electrode will be the equiva-
= 180 × 10-8 lent weight of copper ion, that is, 63.5 g.
Therefore, solubility is found as
16. (a) The half cell reactions in the cell are:
1000 −8 1000
SAgCl = k AgCl × = 180 × 10 × At the anode : H2 → 2H+ + 2e −
Λ oeq 137.2
At the cathode : 2H+ + 2e − → H2
. × 10 −5 mol L−1
= 13

Chapter-18.indd 611 8/8/2016 3:34:01 PM


612 Chapter 18 Electrochemistry

The EMF of the cell is given by 2 × 96500 × 4480


Therefore, 4480 cm3 requires = = 38600 C
0.0591 [H+ ]2cathode 22400
Ecell = − log + 2 2 × 96500 × 4480
2 = 38600 C
[H ]anode 22400
Q 38600
0.0591 As Q = I × t ⇒ t = = = 18037.38s = 5h
=− og[H+ ]cathode − 2 log[H+ ]anode )
(2 lo I 2.14
2
0.0591 Specific conductance of KCl xS
=− ( −2 pHcathode + 2 pHanode ) 22. (c) Cell constant = =
2 Conductance of KCl yS
0.0591
=− × 6 = 0.177 V For 0.01 M H2SO4
2 Specific conductance = Cell constant × Conductance
17. (d) A hydrogen electrode is also known as reversible = xz/y
electrode as it can act both as cathode and anode. Its
23. (b) We know that
standard electrode potential at 298 K is taken as 0 V. For
measuring the electrode potentials of other elements, Weight of Ni deposited Equivalent weight of Ni
=
the potential is arbitrarily taken as zero for SHE. Weight of Cr deposited Equivalent weight of Cr
18. (c) Reduction half-cell reaction: 0.3 59 / 2
⇒ = ⇒ x = 0.17 g
x 52 / 3
Cu2 + + 2e − → Cu E o = 0.34 V
Oxidation half-cell reaction: 24. (c) The reaction is BrO3− + 6H+ + 6e − → Br − + 3H2O
+ − o
H2O → 2H + 21 O2 + 2e E = − 123
. V 6 F is required to reduce 1 mol of BrO3− .
Overall cell reaction: 25. (a) In cell representation, cathode is written on the right
Cu2 + + H2O → Cu + 21 O2 + 2H+ side.

E o = 0.34 + ( −1.23) = − 0.89 V 26. (a) Equivalent conductance is the conducting power of all
the ions produced by one gram equivalent of an elec-
So, the minimum voltage required to bring about electro- trolyte in a given solution. In SrCl2, gram equivalent
lysis = 0.89 V. mass is larger as a result of which the compound shows
19. (b) According to the Nernst equation, maximum value of equivalent conductance in a fused
0.059 state.
o
Ecell = Ecell − log K
2 27. (a) For the reaction Al3+ + 3e− → Al
At equilibrium, Ecell = 0. Therefore, Q M
We have from Faraday’s law m = ×
0.059 F Z
o
Ecell = log K where m = mass of the substance discharged at the
2 electrode
0.059
0.23 = log K Q = total electric charge passed through the substance
2 = I (amp) × t (seconds)
0.46 F = 96500 C
⇒ log K = = 7.79 ⇒ K = 6.16 × 107
0.059 M = Molar mass of the substance
20. (d) According to Nernst equation, for the cell reaction Z = number of electrons transferred per ion.
It M
Fe(s) + Cd2 + (aq) → Fe2 + (aq) + Cd(s) So, m= ×
Therefore, F Z
o 0.0591 [Fe2 + ][Cd] m×Z ×F
Ecell = Ecell − log t=
n [Fe][Cd2 + ] I×M
Substituting given values, we get
27 × 3 × 96500
=
0.0591 [0.1] 0.0591 [0.1] 10 × 27
−0.02 = 0.04 − log ⇒ − 0.06 = − log
2 [x] 2 [x]
= 3 × 9650 = 28950 or 2.89 × 10 4 s
[0.1]
2.03 = log ⇒ 2.03 = − 1− log[ x ] ⇒ 3.03 = − log[ x ] 28. (a) We know that q = w/E and q = I × t. So,
[x]
log x = − 3.03 w I ×t 0.3605 1.2 × 15 × 60
= ⇒ =
On taking antilog, we get x = 0.00093 M = 9.3 × 10−4 M. E 96500 96/n 96500

21. (d) The reaction occurring is 2H+ + 2e − → H2 0.3605 × n × 96500 = 1.2 × 15 × 60 × 96

Thus, 1 mol of H2, that is, 22400 cm3 at NTP requires 1.2 × 15 × 60 × 96
Therefore, n= = 2.9803 ≅ 3
2F = 2 × 96500 C 0.3605 × 96500

Chapter-18.indd 612 8/8/2016 3:34:07 PM


Hints and Explanations 613

29. (c) Ionic mobility is defined as the distance travelled by an For example, for Ag = 1 electron, for Cu = 2 electrons,
ion per second under a potential gradient of 1 volt per Al = 3 electrons, etc.
meter.
38. (b) The expressions are
Ionic velocity v (m/s)
Ionic mobility = = = = m2 V −1s −1 0 0 0
Potential gradient dV /dx ( V /m) Λm (Ba(OH)2 ) = l Ba2− + 2l
OH− (1)
30. (c) For water at 298 K, [H+] = 10−7 M 0
Λm 0
(BaCl2 ) = l Ba2+ + 2l
0
(2)
Cl−
Reduction reaction is H+ + e − → 1
H
2 2 0 0 0
Λm (NH4 Cl) = l NH+ + l (3)
RT (pH)1/ 2 (pH)1/ 2 4Cl−
Therefore, Ecell = − ln = − 0.591 log
F +
[H ] [H+ ] Dividing Eqs. (1) and (2) by 2 and subtracting them, and
1 adding this difference to Eq. (3), we get
= − 0.0591 log −7 = − 0.4137  −0.414 V.
10 0 0 0 1 0 1 0 0
Λ m(NH4 OH)
= l NH+ + l
OH− = Λ m[Ba(OH)2 ] − Λ m(BaCl2 ) + Λ m(NH4 Cl)
31. (a) Ag, Cu, Cr all lie below hydrogen in the reactivity series
4
2 2
while Mg lies above hydrogen. A metal with greater oxi- 1 1
= × 457 − 240.6 + 2129.8 = 2238 Scm2 mol−1
dation potential can displace metals with lower oxida- 2 2
tion potential.
39. (b) Cell reaction: 3Ni + 2Au3+ → 3Ni2 + + 2Au
32. (a) Because the Eo value for Cu is more than Fe means it is a
stronger oxidizing agent. 0.0591 [Ni2 + ]3
o
Ecell = Ecell − log
o 0.059  2 × 0.235  6 [Au3+ ]2
33. (b) Ecell = log K C ⇒ log K C =  = 7.96
2  0.059 
0.0591 (0.1)3
⇒ K = x × 107 = (0.25 + 1.5) − log
g 2
6 (1)
0.0591
34. (b) Applying Nernst equation, = 1.75 + × log(1) = 1.75 + 0.0295 = + 1.7795 V
2
o 2.303RT 1
ENi2+ /Ni = ENi2+
/ Ni − log 2 + 40. (a) As Fe is more reactive than Ag but less reactive than Zn
nF [Ni ] and Na.
Here n = 2. So,
2.303 × 8.314 × 297.9 41. (b) The reaction is Cu2+ + 2e − → Cu
−0.16942 = −0.14 + × log x
2 × 96500 Mass
Volume = = Area × Thickness deposited
where x = [Ni2+]. So, Density
−0.16942 = −0.14 + (0.02955)log x = 100 × 10 −2 cm3
−0.02942 = 0.02955 log x
Mass of Cu2+ ion deposited on plate = 1 × 8.94 g
We get x = [NiCl2] = 0.1010 M
EIt EQ
W Now, Mass = =
35. (c) According to Faraday’s second law = constant 96500 96500
E
WAg WCu 63.5 × Q
So , = 8.94 = ⇒ Q = 27172 C
EAg ECu 2 × 96500

M 42. (a) As Eo value for fluorine is the largest among all other hal-
Ag+ + e − → Ag; EAg = ogens which means it is the strongest oxidizing agent.
1
WAg WCu 107.9 107.9 M 43. (a) The conductivity of a strong electrolyte increases on
= = = Cu2 + + 2e − → Cu; ECu = dilution slightly. This is explained by the Debye–Hückel–
EAg ECu 63.54/2 31.77 2
Onsager equation
Therefore,
WAg WCu 107.9 107.9 M
0
Λm = Λm ( 0
− A + BΛ m ) C
= = = Cu2 + + 2e − → Cu; ECu =
EAg ECu 63.54/2 31.77 2 where A and B are the Debye-Hückel constants. If a solu-
Λm Λ tion is at infinite dilution, that is, C is almost zero then
36. (b) a = 0
= m = 0.9 ⇒ Λ m = 382.5 S cm−1 g. equiv−1 Λm = Λm 0
.
Λm 425
44. (a) For the given cell q = I × t = 5 × 40 × 60 = 12000 C
382.5 The reaction is Zn2+ + 2e − → Zn
Therefore, Λ m = k × N ⇒ N = = 10 N
3.825 2F(2 × 96500 C) is required to reduce 1 mol, that is, 65.3 g
37. (a) Quantity of electricity needed is proportional to the of Zn
65.3 × 12000
number of electrons involved in the reduction reaction. 12000 C is required to reduce = 4.065 g of Zn
2 × 96500

Chapter-18.indd 613 8/8/2016 3:34:14 PM


614 Chapter 18 Electrochemistry

45. (c) For the given cell Level II


+
0 0.06 [H ] Multiple Correct Choice Type
Ecell = Ecell − log
n [Ag+ ]
51. (a), (c), (d) Conceptual
0.06 [H+ ] 1
0.62 = 0.8 − log ⇒ −0.18 = 0.06 log 52. (a), (d) Reduction potential means the means the ability
1 [Ag+ ] [Ag+ ] of the species to accept electrons. Metals are good contrib-
⇒ [Ag+ ] = 10 −3 M . utors of electrons. That is why down the group (which is
arranged in order of increasing reduction potential) reactiv-
Weight of Ag = 10−3 × 108 = 0.108 g ity decreases.
0.108
Therefore, weight% of Ag = × 100 = 10% 53. (a), (b), (c) It contains platinum electrode which is inert.
1.08
46. (b) Using Kohlrausch’s law 54. (a), (b) The standard reduction potential of copper and sil-
ver is greater than the standard reduction potential of hydro-
0
Λ mBaCl2
= l 0Ba2 + + 2l 0Cl− = x1 (1) gen. So hydrogen gas reduces cupric oxide and silver oxide.
0
Λ mH2 SO 4
= 2l 0H+ + l 0SO24− = x 2 (2) 55. (a), (d) The reaction involved is 2 Tl + Cu2 + → 2 Tl+ + Cu
0
Λ mHCl = l 0H+ + l 0Cl− = x 3 (3)
o 0.0591 [ Tl2 + ]
Ecell = Ecell − log 2 +
Adding the Eqs. (1) and (2) and subtracting 2 × Eq. (3), 2 [Cu ]
we get So Ecell can be increased by increasing [Cu2+] or decreasing
0
Λ mBaSO 4
= x1 + x 2 − 2x 3 [Tl2+]

47. (a) Given that decrease in concentration = 0.0005124 equiv. 56. (a), (b), (c) Salt bridge is used to eliminate liquid junction
and amount of Cu deposited = 0.03879 g. Therefore, potential that arises due to different speed of ions present
0.03879 in cathodic and anodic compartments. In options (a), (b), (c)
equivalent of Cu deposited = = 0.001216 equiv.
31.9 salt bridge is not required beacuse sulphuric acid and KOH
Then transport number of Ag+ = serves as the electrolyte in both oxidation and reduction cell.
w It 9.65 × 3600 +
Equiv. of Ag+ lost 0.0005124 57. (a), (c) = = = 0.36 equiv. of Ag
t Ag+ = 2+
= = 0.4214 E n × 96500 96500n
Equiv. of Cu deposited 0.001216
= 0.36 mol of Ag+( n = 1) and 0.18 mol of Cu2+(n = 2). So, Ag
Now, t Ag+ + tNO − = 1 ⇒ tNO − = 1 − 0.4214 = 0.5786
3 3
will oxidize to Ag+ and Cu2+ will reduce to Cu.

w Ag EAg 58. (a), (d) During electrolysis of H2SO4, the following two reac-
48. (d) We know that = tions are possible at anode:
w Au
EAu
For dilute: H2SO4, 2H2O → O2 + 4H+ + 4e −
4.37 107.9
Substituting values, we get = ⇒ w Au = 2.62 g
w Au 65.6 For concentrated: H2SO4, 2SO24− → S2O28 − + 2e −
96500 × 2.62 F− ions have much lower oxidation potential than H2O.
Therefore, Q = = 3854.1 C
65.6 Hence, F− ions are not oxidized in the aqueous solution to
give F2.
49. (b) Given that pH = 14, so pOH = 0. Therefore, [OH−] = 1 M. Passage Type
Now, the solubility product is
Ksp = [Cu2+][OH−]2 ⇒ [Cu2+] = 10−19 M.
59. (c) A general representation of galvanic cell by cell nota-
Also given that for the reaction
tion is as follows:
Cu2 + + 2e − → Cu ECu
o
2+
/Cu = 0.34 V.
Anode; anode electrolyte (C1)||Cathode electrolyte (C2);
cathode
Applying Nernst equation, we get
Pt|H2(g)|HCl(aq)||AgCl(s)|Ag
o 0.0591 [Cu] 0.0591 1 o o o
Ecell = Ecell − log 2 + = 0.34 − log −19 60. (c) Ecell = EAu3+
/Au − ENi2+ /Ni = 15
. 0 − ( −0.25) = 175
. V.
n [Cu ] 2 10
= 0.3
34 − 0.56 = − 0.22 V 61. (d) Subtracting the given equations

50. (c) The reaction is 2Cl−(l) → Cl2(g) + 2e−. The number of AgCl(s) → Ag+ (aq) + Cl− (aq) E o = − 0.58 V
Coulombs is
4.25 A × 35.0 min × 60 s min−1 = 8.92 × 103 C Using Nernst equation, we get
The number of grams of Cl2 that will be produced
o 0.0591 [Ag+ ][Cl− ]
− Ecell = Ecell − log
 1 mol e  1 mol Cl2   70.91 g Cl2  1 [AgCl(s)]
g Cl2 = (8.92 × 103 C)    
 96500 C   2 mol e −   1 mol Cl2  0.0591
o
= Ecell − log[Ag+ ][Cl− ] (as [AgCl(s)] = 1)
= 3.28 g Cl2 1

Chapter-18.indd 614 8/8/2016 3:34:20 PM


Hints and Explanations 615

At equilibrium, Ecell = 0. Also, Ksp = [Ag+][Cl−], so Conductance is due to the presence of ions in the solu-
o tion, as the number of ions increases, the conductance also
Ecell = − 0.0591 × log K sp ⇒ − 0.58 = 0.0591 × log K sp
increases.
K sp = antilog( −0.58 / 0.0591) = antilog( −9.813)
67. (a) ã p, r; (b) ã q, s; (c) ã q; (d) ã p
= 1.53 × 10 −10 The anode acts as a negative pole because on oxidation elec-
0.0591  0.0591  trons are released and it is connected to the positive pole of
o
62. (b) Ecell = ⇒ K c = antilog   = 10 the battery. The cathode acts as a positive pole because on
2 log K c  2 × 0.295 
reduction, the positive ions get reduced. It is connected to the
−1 0
63. (c) Given that R = 1000 Ω, Λ m0 2
( Cl− ) = 80 S cm mol ; Λ m(SO 4 ) = 160 S cm
2−
2
mol−pole
negative
1
. of the battery.
0 2 −1 0 2 −1
Λm ( Cl− ) = 80 S cm mol ; Λ m(SO 4 ) = 160 S cm mol .
2−
68. (a) ã s; (b) ã q; (c) ã p; (d) ã r
Now applying equations, 1
o
Λm Conductance = = ohm−1 = Siemen = S
(NaCl) = 107 + b c (1) Resistance
0
Λm (2) 1 1
(NaCl) = 97 + b c Conductivity(k ) = =
Subtracting Eq. (2) from Eq. (1), we get Resistivity r
rl RA 1 l Sm
0 = 10 + b(2 × 10 −2 ) − b(3 × 10 −2 ) Now, R = ⇒ r= ⇒ =k= = = S m−1
A l r RA m2
10 = b(3 × 10 −2 − 2 × 10 −2 )
1000 Sm−1
10 = 10 −2 b or b = 103 Λ m (molar conductance) = k × = = S m2 mol−1
C mol m−3
Therefore,
 1 m
o
Λm (NaCl) = Λ NaCl + b c Cell constant   = 2 = m−1
 A m
= 107 + 103 (2 × 10 −2 )
Integer Type
= 107 + 20 = 127 S cm2 mol−1
64. (d) We have 69. (5) The reaction involved is C3H8 + 5O2 → 3CO2 + 4H2O
0
Λm = Λ m − b C = 127 − 103 25 × 10 −4
70. (9) The reaction is Al3+ + 3e − → Al
3 −2
= 127 − 10 × 5 × 10 = 77 3 F is required to reduce 1 mol, that is, 27 g of Al.
0 1000 27 g is deposited by 3 F
Λm = k × C = 77 = K × V = K × 81
25 × 10 −4 So, 81 g requires 3 × =9F
27
Hence, k = 77 × 25 × 10 −7 = 1925 × 10 −7
Now, 71. (5) The reaction occurring is 2H+ + 2e − → H2
1 l l Thus, 1 mole of H2, i.e., 22400 cc at NTP requires 2F = 2
k = × ⇒ = 1925 × 10 −7 × 1000 = 0.1925 cm−1 × 96500 C
R A A 2 × 96500 × 4480
5 × 10 −3 Therefore, 4480 cc requires = = 38600C
65. (d) We know that N = M × 2 ⇒ M = . As Q = I × t 22400
2
l Q 38600
We found that = 0.1925 cm−1 t= = = 18037.38 s = 5h
A I 2.14
1000 × 2 1 l 1000 × 2
L =k ×V =k × = × × 72. (9) The reaction is Cr2O72− + 14H+ + 6e − → 2Cr 3+ + 7H2O
5 × 10 −3 R A 5 × 10 −3
For 1 mol, 6 F is required.
1 1000 × 2 Given that molarity = 0.5 M, so number of moles
= × 0.1925 ×
400 5 × 10 −3 = molarity × volume (in L) = 0.5 × 3 = 1.5 mol
For 1.5 mol, the quantity of electricity required is 1.5 × 6
= 0.009625 × 10 4 = 9 F.
= 192.5 S cm2 mol−1
o o o
Matrix-Match Type 73. (3) Ecell = EFe 2+
/ Fe3+ + E Ag+ / Ag = − 0.771 + 0.799 = 0.028 V

66. (a) ã p; (b) ã q; (c) ã r; (d) ã r, s At equilibrium, Ecell = 0, so


For a strong electrolyte, at higher concentrations, the greater 0.0591 [Fe3+ ]
o
interionic attractions retard the motion of the ions and hence 0 = Ecell − log 2 +
1 [Fe ][Ag+ ]
the conductance shows deviation from linear behaviour.
For a weak electrolyte, when the concentration decreases, 1
o
Ecell = 0.0591 log ⇒ [Ag+ ] = 0.34 M
the number of ions in the solution increases and hence it [Ag+ ]
shows almost no variation with dilution.
Specific conductance decreases with dilution because the nE o
Now, log K = ⇒ K = 3.0
ions present per cm3 becomes less on dilution. 0.0591

Chapter-18.indd 615 8/8/2016 3:34:30 PM


616 Chapter 18 Electrochemistry

SOLVED JEE 2016 QUESTIONS


JEE Main 2016 Total number of moles of gases
1. Galvanization is applying a coating of = moles of C2H6 + moles of CO2 + moles of H2
(a) Zn (b) Pb (c) Cr (d) Cu 0.2 0.2 0.2
n= + +
(Offline) 2 1 2
Solution = 0.4
(a) Galvanization is a process for cathodic protection of iron to nRT
V=
prevent it from corrosion. It is a process in which coat of Zn is p
applied on the iron or steel article. (0.4 × 0.0821× 273)
= = 8.96 L
2. What will occur if a block of copper metal is dropped into a 1
beaker containing a solution of 1 M ZnSO4? 4. Identify the correct statement:
(a) The copper metal will dissolve with evolution of oxygen (a) Corrosion of iron can be minimized by forming a contact
gas. with another metal with a higher reduction potential.
(b) The copper metal will dissolve with evolution of hydrogen (b) Iron corrodes in oxygen-free water.
gas. (c) Corrosion of iron can be minimized by forming an imper-
(c) No reaction will occur. meable barrier at its surface.
(d) The copper metal will dissolve and zinc metal will be (d) Iron corrodes more rapidly in salt water because its elec-
deposited. trochemical potential is higher.
(Online) (Online)
Solution
Solution
(c) The two conditions necessary for the corrosion of iron to take
(c) In the reaction of copper with zinc ions, zinc is more active
place are presence of moisture and oxygen. Factors that cata-
(easily oxidized) than copper, thus, no reaction will take place.
lyze the process of rusting are the presence of carbon dioxide,
3. Oxidation of succinate ion produces ethylene and carbon diox- acids and impurities. It can be minimized by introducing of a
ide gases. On passing 0.2 faraday electricity through an aque- barrier film between surface of iron and atmosphere. This can
ous solution of potassium succinate, the total volume of gases be done by:
(at both cathode and anode) at STP (1 atm and 273 K) is (i) Painting the surface.
(a) 8.96 L (b) 4.48 L (c) 6.72 L (d) 2.24 L (ii) Coating the surface with oil or grease.
(Online) (iii) Electroplating iron with non-corrosive metals such as
nickel or chromium.
Solution (iv) Covering the surface of iron with layer of more active
(a) The reaction is metal with higher oxidation potential like zinc.
Electrolysis
2CH3COOK + 2H2O CH3 — CH3 + 2CO2 + H2 + 2KOH JEE Advanced 2016

At the anode: (Oxidation) 1. For the following electrochemical cell at 298K,


Pt(s) H2 (g, 1bar ) H+ (aq, 1M) || M4+ (aq), M2+ (aq)|Pt( s)
O O
–2e– [M2 + (aq)]
2CH3 — C — O– 2CH3 — C — O 2CH3 + 2CO2 Ecell = 0.092 V when = 10 x
Methyl [M4 + (aq)]
2CH3 CH3 — CH3 radical o RT
Given: EM 4+
/M2+ = 0.151 V; 2.303 = 0.059
F
At the cathode: (Reduction) The value of x is
(a) −2 (b) −1 (c) 1 (d) 2
+2e – Solution
2H2O 2OH– + 2H
(d) For the given electrochemical cell, the half-cell reactions are
2H H2
At the anode: H2 (g)  2H+ (aq) + 2e −

Chapter-18.indd 616 8/8/2016 3:34:34 PM


Solved JEE 2016 Questions 617

Substituting the given values, we have


At the cathode: M4+ (aq) + 2e −  M2+ (aq)
The overall cell reaction is 0.059 [M2 + ][H+ ]2
Ecell = (E °M4 + / M2+ − E °H+ /H ) − log
2
2 [M4 + ]pH 2
4+ 2+ +
M (aq) + H2 (g) → M (aq) + 2H (aq)
0.059
0.092 = (0.151− 0 ) − log10 x
From Nernst equation, we have 2
2+ + 2
0.092 = 0.151− 0.0245 log10 x
° − 2.303RT log [M ][H ]
Ecell = Ecell
nF [M4 + ]pH 2 −0.059 = −0.0245 x ⇒ x = 2

Chapter-18.indd 617 8/8/2016 3:34:36 PM


Chapter-18.indd 618 8/8/2016 3:34:36 PM
19 Chemical Kinetics

Question Distribution in JEE (Main and Advanced)

3
No. of Questions

JEE (Main)
2
JEE (Adv)

0
2016 2015 2014 2013 2012 2011 2010 2009 2008 2007

Concept Distribution in JEE (Main and Advanced)

Topics Covered
Year
JEE (Main) JEE (Advanced)
2007 Second-Order Reactions, Effect of Temperature on Rate Constant Order and Molecularity of Reactions
2008 Rate of Reactions Zero-Order Reactions, First-Order Reactions
2009 First-Order Reactions Effect of Temperature on Rate Constant
Effect of Temperature on Rate Constant, Order
2010 Rate of Reactions, Zero-Order Reaction
and Molecularity of Reactions
2011 Effect of Temperature on Rate Constant First-Order Reactions
2012 First-Order Reactions First-Order Reactions
2013 Effect of Temperature on Rate Constant First-Order Reactions
Rate of Reactions, Second-Order Reactions, Experimental Determination of
2014 Order and Molecularity of Reactions
Rate Law, Effect of Temperature on Rate Constant
Rate of Reactions, First-Order Reactions, Second-Order Reactions, Effect of
2015
Temperature on Rate Constant

2016 First-Order Reactions, Second-Order Reactions, Elementary Reactions Effect of Temperature on Rate Constant

Chapter-19.indd 619 8/5/2016 11:33:53 AM


620 Chapter 19 Chemical Kinetics

SUMMARY
1. Chemical kinetics deals with the study of the rate of chemical reactions and with the elucidation of the mechanisms by which they
proceed.
2. Rate of a chemical reaction
(a) The rate or velocity of a chemical reaction can be defined as the rate of appearance of one or more of its products, or as the rate
of disappearance of one or more of its reactants in unit time. When chemical reactions occur, the concentrations of reactants
decrease as they are used up, while the concentrations of the products increase as they are formed.

Concentration (moI L−1)

Products

Reactants

Time (s)

With respect to any substance X, the rate of reaction can be given by

( Conc. of X at time t final − Conc. of X at time tinitial ) ∆(Conc. of X )


Rate with respect to X = =
(t final − tinitial ) ∆t

(b) Units of rate of reaction


The unit of rate of reaction is mol L−1 s−1; it is always expressed as a positive value, irrespective of whether the concentration of a
substance increases or decreases.
3. (a) Average rate
The rate of reaction can be measured as rate of disappearance of reactant A or rate of formation of component B. It can be
expressed mathematically as
Decrease in concentration of A ∆A
Rate = =−
Time ∆t
Increase in concentration of B ∆B
or Rate = =+
Time ∆t
The rate obtained from the above equations is called the average rate of reaction and is represented as ravg.
(b) Instantaneous rate
The rate at which a reactant is being consumed at any particular moment is called the instantaneous rate. Mathematically, the
instantaneous rate can be determined by measuring the average rate in the smallest time interval dt (i.e., as ∆t approaches zero).
Thus for the reaction A → B, the instantaneous
− ∆A ∆B
ravg = =
As ∆t → 0, ravg → rinst, so ∆t ∆t

− dA dB
rinst = =
dt dt
4. Dependence of relative rates of reaction on coefficients in the equation
(a) For reactions in which the stoichiometric coefficients of the reactants and products are the same, the rate of disappearance of
any of the reactants is same as the rate of appearance of the products. For example, for the reaction SO2Cl2 (g) → SO2 (g) + Cl2 (g)

∆[SO2Cl2 ] ∆[SO2 ] ∆[Cl2 ]


Rate of reaction = − = =
∆t ∆t ∆t

Chapter-19.indd 620 8/5/2016 11:33:55 AM


Summary 621

(b) In reactions where the stoichiometric coefficients of the reactants and products are not the same, the rate of reaction depends
on the coefficients of the reactants/products in the balanced chemical equation. For example, in the combustion of propane

C3H8 (g) + 5O2 (g) → 3CO2 (g) + 4H2O(g)

∆[C3H8 ] 1 ∆[O2 ] 1 ∆[CO2 ] 1 ∆[H2O]


Rate of reaction = − =− = =
∆t 5 ∆t 3 ∆t 4 ∆t
5. Factors affecting the rate of reactions
(a) Temperature
Most of the chemical reactions are accelerated by increase in temperature. In most cases, the rate of a reaction in a homogenous
reaction mixture is approximately doubled or even tripled by an increase in temperature of only 10°C.
(b) Nature of reactants:
The nature and the strength of the bonds in reactant molecules greatly influence the rate of its transformation into products.
(c) Surface area of reactants
Finely divided reactants, because of their larger surface area, react more rapidly than larger specimens of the same substance.
(d) Catalysts
Presence of small amount of substances called catalysts accelerates the rate of reaction. The action of catalyst is specific to the
reaction and it is chemically unaltered at the end of the reaction.
(e) Concentration
(i) The rate of a chemical reaction is the rate at which the concentrations of interacting substances vary with time. In general, if
dx represents an infinitesimally small change in concentration of any species during the reaction time dt, then rate may be
expressed as
dx
Rate =
dt
(ii) The value of dx/dt may be obtained experimentally by plotting concentration of reactant (or product) against time. The
mathematical expression that relates the rate of reaction to the concentration of either reactants or products is known as the
rate law.
(iii) For any reaction between two components A and B, the rate of reaction is expressed as

d[A]
− = k[A ] x [B] y
dt

Here exponents x and y may or may not be equal to the stoichiometric coefficients of the reactants. This equation is known
as differential rate equation and k is a constant of proportionality, known as the rate constant.
6. Elementary and complex reactions
(a) Elementary reactions
The reactions that proceed in a single step are known as elementary reactions.
(b) Complex reactions
Reactions that involve more than one step, that is more than one elementary reaction, are known as complex reactions. The
sequence of reactions in a complex reaction is representative of the mechanism of reaction. Complex reactions may involve fol-
lowing type of reactions.
(i) Consecutive reactions: These are reactions taking place in a series of steps, represented as

A1 → A2 → A3

(ii) Reverse reactions: These occur in forward and reverse steps and are represented as
A B

(iii) Parallel reactions: In these reactions, some side reactions take place leading to the formation of some byproducts along
with the main product.
A →B

C
7. Order and molecularity of reactions
(a) Order
Order of the reaction is the sum of the powers of the concentration of the reactants in the rate law. Based on this value, reactions
may be classified as first-order, second-order, third-order, etc.

Chapter-19.indd 621 8/5/2016 11:33:57 AM


622 Chapter 19 Chemical Kinetics

(i) Certain reactions in which concentration of the reactant does not affect the reaction rate are called zero-order reactions.
(ii) A reaction whose order is different from the actual due to large excess concentration of one of the reactants is called pseudo-
order reaction.
Some examples of pseudo-order reactions are as follows:
• Hydrolysis of an ester: Ethyl acetate on hydrolysis in aqueous solution using a mineral acid as catalyst forms acetic acid
and ethyl alcohol.
CH3COOC2H5 + H2O → CH3COOH + C2H5OH

Here excess of water is used and the rate law can be written as

Rate = k [CH3COOH][H2O] = k ′[CH3COOH]

This reaction is actually a second-order reaction but, in practice, it is found to be first-order. Hence it is a pseudo-first-order
reaction.
• Hydrolysis of sucrose: Sucrose upon hydrolysis in the presence of a dilute mineral acid gives glucose and fructose.

C12H22O11 + H2O → C6H12O6 + C6H12O6

If large excess of water is present, its concentration is practically constant and the rate may be written as

Rate = k [C12H22O11][H2O] = k ′[C12H22O11]

Though the actual order is 2, its experimental order is 1. Thus it is a pseudo-first-order reaction.
(b) Molecularity
It is defined as the total number of reacting species (molecules, atoms or ions) taking part in an elementary reaction. Molecularity
represents the number of reactant molecules undergoing collisions to bring about the chemical reaction. Based on the value of
molecularity, the reactions are classified as
(i) Unimolecular reactions: These reactions have molecularity 1.
F2O2 (g) → F2 (g) + O2 (g)

(ii) Bimolecular reactions: These reactions have molecularity 2.

CH3COOC2H5 (l) + H2O(l) → CH3COOH(l) + C2H5OH(l)

(iii) Trimolecular or termolecular reactions: These reactions have molecularity 3.

2NO(g) + O2 (g) → 2NO2 (g)

Characteristic features of order and molecularity are tabulated as follows

Molecularity Order of a reaction

It is the number of atoms, ions or molecules that must It is the sum of the concentration terms on which the rate of
collide with one another simultaneously so as to result reaction actually depends or it is the sum of the exponents of
into a chemical reaction. the molar concentrations in the rate law equation.

It is always a whole number. It need not be a whole number.


It can be calculated by simply adding the molecules of It can be determined experimentally only and cannot be
the slowest step. calculated.
Molecularity is applicable only to elementary reac-
tions. The overall molecularity of a complex reaction Order is applicable to elementary as well as complex reactions.
has no significance.

8. Rate law, rate constant and its units


(a) Rate law
The mathematical expression that relates the rate of reaction to the concentration of either reactants or products is known as the
rate law. In general for a reaction, the rate law is given by
dx
= k [Concentration of reactant]n
dt
where dx/dt = rate of reaction, k = proportionality constant, n = nth order of the reaction.

Chapter-19.indd 622 8/5/2016 11:33:59 AM


Summary 623

(b) Rate constant and its units


From the rate law, the expression for rate constant is
dx 1
k= ×
dt [Concentration]n
The expressions for rate constant and their units for reactions of different order are given by:
(i) Zero-order reaction: Here n = 0,

 dx  1
k=  × = mol L−1 time −1
 dt  [Concentration]0
(ii) First-order reaction: Here n = 1,

 dx  1 mol L−1 1
k=  × = × = time −1
 dt  [Concentration]1 time mol L−1

(iii) Second-order reaction: Here n = 2,

 dx  1 mol L−1 1
k= × = × = L mol−1 time −1
 dt  [Concentration]2 time (mol L−1)2

(iv) Third-order reaction: Here n = 3,

 dx  1 mol L−1 1
k=  × = × = L2 mol−2 time −1
 dt  [Concentration]3 time (mol L−1)3

Tip The exponents in the rate law for an elementary process are equal to the coefficients of the reactants in the chemical equation
for that elementary process. For a multistep complex reaction, the rate law for the slowest step in a mechanism should agree with the
experimental rate law.

(c) The characteristics of rate of reaction, rate law and rate constant are summarized as follows:

Rate of reaction Rate law Rate constant


It is the speed with which the The mathematical expression which de-
It is the proportionality constant in the
reactants are converted into notes the experimentally observed rate
rate law and is defined as the rate of
products. It is measured as the of reaction in terms of the concentra-
reaction when the concentration of the
rate of decrease in concentra- tions of reacting species which influence
reactants is unity.
tion of reactants with time. the rate of reaction.

It depends upon the initial It is independent of the initial concentra-


It depends on concentration of reactant.
concentration of the reactants. tion of the reactants.
Its units are always mol It is different for first-, second-, third- and
Its units depend on the order of reaction.
L−1 time−1. zero-order reactions.
9. Differential and integral forms of elementary (simple) reactions
(a) Zero-order reactions
In zero-order reactions, the rate is independent of the concentration of the reactants. The differential form of rate equation is,
therefore,
d[A]
Rate = − = k [A]0 = k
dt
The integrated form of rate law equation for zero order reaction is
[A]0 − [A ]
[A] = − kt + I or k =
t
where I is the constant of integration. At t = 0, the concentration is [A] = [A]0
(i) From the equation, it can be seen that for a zero-order reaction, the rate constant is equal to the rate of reaction at all con-
centrations. The unit of rate constant is (units of concentration)/(units of time) = mol L−1 s−1.
(ii) If the reaction is zero order in A, a plot of the concentration of A vs. time will be a straight line with a slope equal to −k, and
intercept equal to [A]0.

Chapter-19.indd 623 8/5/2016 11:34:02 AM


624 Chapter 19 Chemical Kinetics

Zero-order
reaction
[A]0

Slope = −k

[A]
[A] = −kt + [A]0

t
(b) First-order reactions
These are reactions whose rate is determined by the change of only one concentration term. The differential form of rate equa-
tion is, therefore,
d[A]
Rate = − = k[A ] or d[A ] = − kdt
dt [A ]

where k, which is a proportionality constant, is called the velocity constant, rate constant or specific rate constant.
The integrated form of rate law equation for first-order reaction is

[A ] 1 [A ]
ln = − kt or k = − ln
[A]0 t [A ]0

(i) The integrated rate equation at two time intervals t1 and t2, when the concentrations are [A]1 and [A]2, respectively,
given by

ln[A ]1 = − kt1 + ln[A ]0 and ln[A]2 = − kt2 + ln[A ]0

1 [A ]
The expression for rate constant is k = ln 1
(t2 − t1) [A ]2
(ii) If the reaction is first-order in A, a plot of experimental data of ln A vs. time should fit the integrated form of the first-order
rate law, that is
ln[A ] − ln[A ]0 = − kt
Since the equation has two variables and two constants, it represents equation for a straight line. If the reaction is first-order
in A, a plot of the natural logarithm of the concentration of A vs. time will be a straight line with a slope equal to −k and inter-
cept equal to [A]0.
First-order reaction
ln[A]0

Slope = −k
ln[A]

ln[A] = −kt + ln[A]0


t
(iii) The rate law can also be expressed as

2.303 [A ] [A ] kt
k= log 0 or log 0 =
t [A ] [A ] 2.303

Chapter-19.indd 624 8/5/2016 11:34:06 AM


Summary 625

The slope of plot between log [A]0/[A] vs. time t is k/2.303.

log ([A]0 /[A])


Slope = k /2.303

0 t
(c) For first-order gas phase reaction
When the reactants and products are gaseous, the concentration can be expressed in terms of partial pressure. For example, for
decomposition of N2O5, the rate constant is given by
2N2O5 (g) → 2N2O 4 (g) + O2 (g)

2.303 pi
k= log
t 3 pi − 2 pt

(d) Second-order reactions


(i) When the reaction is 2A → Products: In such types of reactions, two molecules of the same reactant are involved in the
reaction. The rate of reaction is
d[A]
− = k[A ]2
dt
The integrated form of rate law equation is
1 1
= + kt
[A ] [A ] 0

Slope = k
1/[A])

0 t

1 x 
Rate constant of second-order reactions having only one reactant, is given by k =  .
t  a(a − x ) 

The quantity x/(a − x) is dimensionless, so units of k are (concentration)−1 time−1, that is, L mol−1 time−1.
(ii) When the reaction is A + B ã Products: In such reactions, the rate is determined by the change of two concentration terms
of two different reactants.
d[A ]
− = k[A ][B]
dt
where [A] and [B] are the concentration of reactants A and B, respectively, at any given time t, and k is the velocity
constant.

Chapter-19.indd 625 8/5/2016 11:34:08 AM


626 Chapter 19 Chemical Kinetics

The rate constant of second-order reactions having two different reactants is

1  2.303   b(a − x ) 
k=   log 
t  a−b  a(b − x ) 

The units of rate constant are (1/time) × (1/mol L−1) × [(mol L−1)(mol L−1)/( mol L−1)( mol L−1)] = L mol−1 time−1.
(e) Third-order reactions
(i) When the reaction is A + B + C ã Products: If the reactants are all present in different initial concentrations a, b, c and x is
dx
the decrease in the concentration of each reactant at time t, then the third-order rate equation is = k (a − x )(b − x )(c − x )
dt
 1   x (c − a)  c(a − x )  
On integration, we get k =  2   a( a − x )
+ ln  
 t (c − a)    a(c − x )  
(ii) When the reaction is 2A + B ã Products: The concentrations at any time t are (a − 2x) and (b − x). The rate equation
dx
becomes = k(a − 2x )2 (b − x )
dt

1  2 x (2b − a)  b(a − 2 x )  
On integration, we get k=  + ln  
t (2b − a)2  a(a − 2 x )  a(b − x )  

(iii) When the reaction is 3A → Products: In such reactions, a = b = c, for which the rate equation can be written as
dx
= k (a − x )3
dt

1  x (2a − x ) 
On integration, we get k =  
2ta2  (a − x )2 

Tip Integrated rate law equations quantitatively give concentration as a function of time and its form depends on the order of the
reaction. The integrated rate equations can be fitted with kinetic data to determine the order of a reaction.

10. Half-life of reactions


(a) For zero-order reaction
[A]0 − (1/2)[A]0 [A ]
The rate constant and the half-life is given by k = ⇒ t1/2 = 0
t1/2 2k
So the half-life time in a zero-order reaction is directly proportional to the initial concentration of the reactants and inversely
proportional to the rate constant.

t1/2

0 [A]0

(b) For first-order reaction


[A]t [A ]
The rate constant and half-life time is given by ln = − kt or ln 0 = kt
[A]0 [A]t

0.693
Since ln 2 is equal to 0.693, it can also be written as t1/2 =
k
Since k is a constant for a given reaction, the half-life is also a constant for any particular first-order reaction (at any given tem-
perature). It can be easily calculated if the rate constant of the reaction is known.

Chapter-19.indd 626 8/5/2016 11:34:11 AM


Summary 627

t1/2

0 [A]0
(c) Second-order reactions
Half-life period is given by t1/2 = 1/k[A]0. Hence t1/2 of second-order reaction is inversely proportional to the initial concentration
of reactant.

t1/2

0 1/[A]0
(d) Third-order reactions
Half-life period can be deduced as t1/2 = 3/2 k[A]02. It is inversely proportional to [A]02.

t1/2

0 1/[A]02

(e) nth order reactions


1
The half-life period is deduced from the relation t1/2 ∝ . The general expression is
[A ]0n −1

1  2n −1 − 1
t1/ 2 =  .
k (n − 1)  [A ]0n −1 
11. Complex reaction kinetics
Complex reactions Differential equations Integrated rate equations Examples
Consecutive reactions d[A ] 1. The pyrolysis of acetone
= k1[A ] [A] = [A]0 e − k1t
k1 k2
dt 2. Decomposition of ethyl-
A 
→ B 
→C k1
d[B] [B] = [A ]0 (e − k1t − e − k2t ) ene oxide
= k1[A ] − k2 [B] k2 − k1
dt
 k1 
d[C]
= k2 [B] [C] = [A ]0 1− (e − k1t − e − k2t ) − e − k1t 
dt  k2 − k1 
Parallel reactions d[A ] 1. Dehydration of 2-
− = (k1 + k2 )[A] [A ] = [A]0e −( k1 + k2 )t
k1 dt methyl-2-butanol
B k1[A]0 
d[B] [B] = 1− e −( k1 + k2 )t  2. Nitration of phenol
A
k2
C = k1[A] (k1 + k2 ) )  
dt
k2 [A]0 
d[C] [C] = 1 − 1 − e −( k1 + k2 )t 
= k2 [ A ] (k1 + k2 ) ) 
dt

(Continued)

Chapter-19.indd 627 8/5/2016 11:34:16 AM


628 Chapter 19 Chemical Kinetics

(Continued)
Complex reactions Differential equations Integrated rate equations Examples
Reversible or Opposing − d[A ] d[B] [A ] = A eq + x eqexp[ −(k f + kr ) t ] 1. Dissociation of hydro-
reactions = = k f [A ] − kr [B] gen iodide in gas phase
dt dt
kf 2. Reaction between CO

A  
B and NO2 gases
k r

12. Determination of order of simple reaction – experimental methods


(a) Graphical method
Consider a zero-order reaction of the type A → Products. The rate equation is therefore,
dx
= k (a − x )0 = k
dt
For first-order reaction
dx
= k (a − x )
dt
For second-order reaction
dx
= k (a − x )2
dt
Now, if the plot of log (a − x) vs. t is a straight line, the reaction follows first-order. If the plot of 1/(a − x) versus t is a straight line,
the reaction follows second-order.
In general, for a reaction of nth order, a graph of 1/(a − x)n−1 vs. t must be a straight line.
(b) Half-life method
1
A general expression for the half-life is t1/ 2 ∝ n −1
a
Starting with two different initial concentrations a1 and a2 for the same reaction, the half lives are (t1/ 2 )1 and (t1/ 2 )2 respec-
tively, then order of reaction is given by
n −1
(t1/ 2 )1  a2 
=
(t1/ 2 )2  a1 

Taking log on both sides, we get

(t1/ 2 )1 a  (t ) − (t )
log = (n − 1)log  2  ⇒ (n − 1) = log 1/ 2 1 1/ 2 2
(t1/ 2 )2  a1  log a2 − log a1

(c) van’t Hoff differential method


For two different initial concentrations A1 and A2, equations can be written in the form

− dA1 − dA2
= kC1n and = kC2n
dt dt
Taking logarithms and performing subtraction, we get

 − dA1   − dA2 
log10  − log10 
 dt   dt 
n=
(log10 A1 − log10 A2 )

where −dA1/dt and −dA2/dt are determined from concentration vs. time graphs and the value of n can be obtained.
(d) Initial rate method
n1A + n2B + n3C → Products

Suppose the order of the above reaction with respect to A, B and C is x, y and z, respectively.
Then the rate is given by

Rate = k [A ] x [B] y [C]z

Chapter-19.indd 628 8/5/2016 11:34:22 AM


Summary 629

Starting with two different initial concentrations of A, keeping the concentrations of B and C constant, the initial rates of the
reaction are determined, then

(r0 )1 = k [A 0 ]1x [B] y [C]z and (r0 )2 = k [A 0 ]2x [B] y [C]z


Therefore, we have
x
(r0 )1 [A 0 ]1x  [A 0 ]1  (r0 )1 [A ]
= x
=  ⇒ log = x log 0 1
(r0 )2 [A 0 ]2  [A 0 ]2  (r0 )2 [A 0 ]2
log[(r0 )1 (r0 )2 ]
⇒ x=
log{[A 0 ]1 [A 0 ]2 }

This is the order with respect to A. Similarly, order with respect to B and C can be determined. On adding, we get the overall
order of the reaction (= x + y + z).
(e) Ostwald’s isolation method
(i) This method is applicable for reactions in which there are two or more reactants.
(ii) It consists of performing a series of experiments in which all but one of the reactants are taken in excess turn by turn, so
that its concentration does not very sensible during course of the reaction. Then order of reaction is determined with
respect to isolated reactant, i.e, which is taken in small amount.
(iii) The sum of individual order, when each reactant is taken separately in small amount, is the overall order of the reaction.

13. Effect of temperature on rate of reactions


(a) The rate of reaction increases by a factor of about 2 or 3 for each 10°C increase in temperature, although the actual amount of
increase differs from one reaction to the other.
(b) The Q10 temperature coefficient is a measure of the rate of change in chemical system as a consequence of increas-
ing the temperature by 10°C. It is defined as the ratio of specific rates of a reaction at two temperatures differing by
10°C:

R 
Q10 =  (T +10 ) 
 RT 

(c) The relationship between temperature and the rate of a reaction can be explained by assuming that the rate constant depends
on the temperature at which the reaction is run, as suggested by Arrhenius.
(d) According to Arrhenius theory, for a reaction to take place, the reacting molecules must collide to form an intermediate com-
plex which is unstable and breaks up to form the products. This intermediate complex is known as the activated complex and
the energy required to form this complex is called the activation energy.
(e) The relation between temperature and rate constant is given by Arrhenius equation.

k = Ae − Ea /RT

where k is the rate constant for the reaction and A is a proportionality constant that is known as Arrhenius factor or frequency
factor. It is also called pre-exponential factor and is specific for a reaction. R is the ideal gas constant in joules per mole
kelvin (J mol−1 K−1), e is the base of natural logarithms, Ea is the activation energy for the reaction and T is the temperature in
kelvins.
Ea
(f) The logarithmic form of Arrhenius equation is given by ln k = ln A −
RT
We can plot a graph of ln k vs. 1/T. The graph is a straight line whose slope = −Ea/R and intercept = ln A.
(g) Using Arrhenius equation at two different temperatures, the activation energy of a reaction can be calculated by determining
the rate constants of the reaction at these temperatures.

k2 Ea  1 1  k2 Ea  1 1 
log =−  −  or log =  − 
k1 2.303R  T2 T1  k1 2.303R  T1 T2 

(h) A catalyst is a substance that changes the rate of a chemical reaction without itself being used up.
Although the catalyst is not part of the overall reaction, it does participate in the reaction by changing the mechanism of the
reaction. The catalyst provides a path to the products that has a rate-determining step with lower activation energy than that
of the uncatalyzed reaction as shown in the following figure.

Chapter-19.indd 629 8/5/2016 11:34:23 AM


630 Chapter 19 Chemical Kinetics

Number of molecules

Ea catalyzed Ea uncatalyzed
Kinetic energy
A larger proportion of the collisions that occur between reactants now have enough energy to overcome the activation energy
for the reaction, as shown in below figure.

Ea without a catalyst

Ea with a catalyst
Energy

Reactants

∆H °
Products

Reaction coordinate

14. Collision theory of bimolecular gaseous reactions


(a) The basic postulate of collision theory is that the rate of a reaction is proportional to the number of effective collisions per sec-
ond among the reactant molecules.
(b) An effective collision is one in which molecules collide with sufficient kinetic energy and proper orientation in a manner that
the bonds between reacting molecules are broken and new bonds are formed to yield the products.
(c) The number of collisions per second per unit volume of the reaction mixture is known as collision frequency (Z).
(i) According to kinetic theory of gases, for a gas containing only one kind of molecule, the number of colliding molecules per
cm3 per second Z1 is
Z1 = ps 2v (n*)2

where s is the molecular diameter, v the average molecular velocity in cm s−1 and n* the number of molecules per cm3.
(ii) When two different molecules are involved, Z is taken to be Z1,2 and is given by

2
s + s 2  ( M2 + M1)
Z1,2 = n1* n2*  1  8p RT
 2  M1M2

where s1 and s2 are the molecular diameters of the respective molecules, M1 and M2 their molecular masses, n1* and n2*
the numbers of the respective molecules per cm3 at temperature T.
(d) Suppose Z is the number of molecules colliding per cm3 per second in the reacting system containing 1 mol L−1 of reactant and
q is the fraction of these that are activated, then reaction constant k of the reaction is given by
k = Zq
Substituting for q from the following expression

n′ Number of molecules activated


= e Ea /RT =
n* olecules
Total number of mo
we get, k = Ze Ea /RT

Chapter-19.indd 630 8/5/2016 11:34:28 AM


Solved Examples 631

TOOLS FOR PROBLEM SOLVING

1. Rate law of a reaction: Rate = k[A ] x [B] y


2. Integrated zero-order rate law: [A] = − kt + I

[A ] 1 [A ]
3. Integrated first-order rate law: ln = − kt or k = ln
[A]0 t [A ]0
4. Integrated second-order rate law
d[A]
(a) For second-order reaction of the form, 2A → Products; − = k[A]2
dt

1 1
− = kt
[A ]t [A ]0

d[A]
(b) For second-order reaction of the form, A + B → Products; − = k[A][B]
dt
b(a − x )
ln = kt (a − b )
a(b − x )
5. Rate constant and half-life time
Order Expression for rate constant Half-life (at t = t1/2, [A] = [A]0/2)
Zero [A]0 − [A] [A]0 − (1/2)[A]0 [A ]
k= k= ⇒ t1/2 = 0
t t1/2 2k

First [A]0  [A]0  ln 2 0.693


ln = kt ln  = kt1/ 2 ⇒ t1/ 2 = =
[A]t  [A]0 / 2  k k

Second 1 1 1 1 1 1
kt = ln − (for the same reactant) t1/ 2 = ln − ⇒
[A ] [A 0 ] k [A 0 ] / 2 [A 0 ] k [A 0 ]

Third  ([A 0 ] − [A ])(2[A 0 ])  3


1 k[A 0 ]2
kt = 2   (for the same reactant) 2
2[A 0 ]  [A 0 ]2 

Ea
6. Arrhenius equation ln k = ln A −
RT
7. Arrhenius equation at two different temperatures

k2 Ea  1 1  k2 Ea  1 1 
log =−  −  or log =  − 
k1 2.303R  T2 T1  k1 2.303R  T1 T2 

SOLVED EXAMPLES
Rate of Reactions
1 Solution
1. For a reaction A → 2B, rate of disappearance of A is related to
2 1
(b) For the reaction A → 2B , the instantaneous rates are
the rate of appearance of B by the expression 2
d[ A ] 1 d[B] dx − 2d[ A ] d[B]
d[A] 1 d[B] = =+
(a) − = (b) − = dt dt 2dt
dt 2 dt dt 4 dt
d[ A ] d[B] d[A ] d[B] dx − d[ A ] 1 d[B]
(c) − = (d) − =4 = =
dt dt dt dt dt dt 4 dt
(AIEEE 2008)

Chapter-19.indd 631 8/5/2016 11:34:36 AM


632 Chapter 19 Chemical Kinetics

2. Consider the reaction: d[N2O5 ]


− = k1[N2O5 ] (k1 = k )
+ − dt
Cl2 (aq) + H2S(aq) → S(s) + 2H (aq) + 2Cl (aq)
d[NO2 ]
The rate equation for this reaction is rate = k[Cl2][H2S]. Which = 4 k[N2O5 ]
dt
of these mechanisms is/are consistent with this rate equation?
d[NO2 ]
A. Cl2 + H2S → H+ + Cl− + Cl+ + HS − (slow ) = k2 [N2O5 ] ( k2 = 4 k )
dt
Cl+ + HS − → H+ + Cl− + S ( fast ) k1 = 2k
k2 = 4k
B. H2S  H+ + HS − ( fast equilibrium) 2k1 = k2 ⇒ 2k = k′

Cl2 + HS − → 2Cl− + H+ + S (slow)


5. For the reaction,
(a) A only (b) B only 3A + 2B → C + D,
(c) Both A and B (d) Neither A nor B. the differential rate law can be written as:
(AIEEE 2010)
1 d[A] d[C]
Solution (a) = = k[A]n [B]m
3 dt dt
(a) As the slowest step is the rate determining step, so from
mechanism A. d[A] d[C]
(b) − = = k[A]n [B]m
Rate = k [Cl2][H2S] dt dt
3. In the reaction of formation of sulphur trioxide by contact pro- 1 d[A] d[C]
(c) + = = k[A]n [B]m
cess 2SO2 + O2  2SO3 the rate of reaction was measured as 3 dt dt
d [ O2 ]
= 2.5 × 10 −4 molL−1 s −1 . The rate of reaction in terms of 1 d[A] d[C]
dt (d) − = = k[A]n [B]m
3 dt dt
[SO2] in mol L−1 s−1 will be: (JEE Main Online 2015)
(a) −1.25 × 10−4 (b) −2.50 × 10−4 Solution
(c) −3.75 × 10−4 (d) −5.00 × 10−4
(JEE Main Online 2014) (d) 3A + 2B → C + D
Solution
1 d[A] 1 d[B] d[C] d[D]
(d) For the reaction, 2SO2 + O2  2SO3 r=− =− = =
According to differential rate law, 3 dt 2 dt dt dt
1 d[SO2 ] d[O2 ] 1 d[SO3 ] r = k [A]n [B]m (orderisindependent of molecularity )
r=− = =+
2 dt dt 2 dt −1 d[A] d[C]
r= = = k [A]n [B]m
3 dt dt
d[O2 ]
r=− = − 2.5 × 10 −4 molL−1 s −1
dt 6. The rate law for a reaction between the substances A and B is
1 d[SO2 ] given by Rate = k [A]n [B]m. On doubling the concentration of
Rate of reaction in terms of [SO2] = − = 2.5 × 10 −4 A and halving the concentration of B, the ratio of the new rate
2 dt
to the earlier rate of the reaction will be as
d[SO2 ]
= − 5 × 10 −4 molL−1 s −1 (a) 1/ 2( m + n) (b) (m + n)
dt
(c) (n – m) (d) 2(n−m)
4. For the reaction, 2N2O5 → 4NO2 + O2, the rate equation
d[N2O5 ] Solution
can be expressed in two ways − = k[N2O5 ] and (d) The initial and final rates are given by
dt
d[NO2 ]
+ = k′[N2O5 ] k and k’ are related as: m
dt B 
(a) k = k’ (b) 2k = k’ (c) k = 2k’ (d) k = 4k’ Rateinitial = k [A]n [B]m and Rate final = k [2A]n  
2
(JEE Main Online 2014)
Solution The ratio of Ratefinal/Rateinitial is
(b) For the reaction, 2N2O5  4NO2 + O2 Rate final k [2A]n [B/2]m
The rate law is = = 2n − m
Rateinitial k [A]n [B]m
1 d[N2O5 ] 1 d[NO2 ] d[O2 ]
rate = − = = = k[N2O5 ]
2 dt 4 dt dt 7. For the reaction system: 2NO(g) + O2 (g) → 2NO2 (g) volume
− d[N2O5 ] is suddenly reduced to half its value by increasing the pres-
= 2k[N2O5 ]
dt sure on it. If the reaction is of first-order with respect to O2

Chapter-19.indd 632 8/5/2016 11:34:44 AM


Solved Examples 633

and second-order with respect to NO, the rate of reaction


will be 0.05
(a) diminished to one-fourth of its initial value
(b) diminished to one-eighth of its initial value 0.04
(c) increased to eight times of its initial value

[A] moles/liter
(d) increased to four times of its initial value
0.03
Solution
(c) The rate law for the reaction is Rate = k [O2] [NO]2.
0.02
When volume is reduced to half, concentration becomes
double the initial concentration, so the rate law becomes P
Rate = k [2O2] [2NO]2 ⇒ Rate = 8k [O2] [NO]2. 0.01

8. The rate equation for the reaction 2A + B → C is found to be:


rate = k[A][B]. The correct statement in relation to this reaction
is that the
(a) unit of k must be s−1 5 10 15 20 25
Time (min)
(b) value of k is independent of the initial concentration of
A and B. The rate of reaction after 15 min is
(c) rate of formation of C is twice the rate of disappearance (a) 1.2 × 10 −3 M min−1
of A.
(d) t1/2 is a constant. (b) 2.67 × 10 −3 M min−1
(c) 6.67 × 10 −3 M min−1
Solution
(b) Value of k is independent of the initial concentration of (d) 1.33 × 10 −3 M min−1
A and B. Rate constants are independent of concentra- Solution
tion but depend on other factors, such as temperature. (d) Since the graph is not linear, so, average rate and instan-
9. At a certain moment, in the reaction 2N2O5 → 4NO2 + O2 ; taneous rates are different.
Here we need to calculate instantaneous rate after
N2O5 is decomposing at the rate of 108 mg L−1 s−1. The pro-
15 min. For the given reaction,
duction rate of NO2 is
(a) 208 mg L−1 s−1 (b) 108 mg L−1 s−1 d[A ]
Rate = −
(c) 56 mg L−1 s−1 (d) 92 mg L−1 s−1 dt
Solution which is indicated by the slope of the graph at point P.
(d) 2N2O5 → 4NO2 + O2 The slope can be calculated by drawing tangent line at
point P. We need two points on this tangent line to find
From reaction stoichiometry, 2 mol of N2O5 can produce
the slope.
4 mol of NO2, that is, just twice the number. Hence, the The two points are (0, 0.03) and (15, 0.01). Therefore,
rate expression is
0.01− 0.03
1 d[N2O5 ] 1 d[NO2 ] Rate = − = 1.33 × 10 −3 M min−1
− = 15 − 0
2 dt 4 dt
11. The reaction at 1100 K between NO and H2 is represented as
But in the question rate is defined in mass per unit vol-
ume per unit time, so converting this rate into moles per 2H2 (g) + NO(g) → N2 (g) + 2H2O(g)
unit volume per unit time.
Reaction takes place inside a closed vessel and rate of reac-
108 mg L−1 s −1 −1 −1 tion is observed as change in pressure:
Rate of decomposition of N2O5 = = 1m mol L s
108 g mol−1 Initial pressure of Initial pressure of Rate of reac-
H2 (torr) NO (torr) tion (torr s−1)
d[NO2 ]  d[N2O5 ] 
Rate of production of NO2 = = 2 × −  289 400 1.6
dt  dt 
147 400 0.77
= 2 m mol L−1s −1
400 300 1.03
Rate of production of NO2 = 2 m mol L−1 s −1 × Molar mass of NO2 400 152 0.25
= 92 mg L−1 s −1 The rate constant of the reaction is
(a) 163 L2 mol−2 s−1
10. For a reaction A → Product; [A] is plotted against time as −8 2 −2 −1
(b) 3.5 × 10 L mol s
shown in the following figure:

Chapter-19.indd 633 8/5/2016 11:34:49 AM


634 Chapter 19 Chemical Kinetics

The rate expressions for the second reaction is


(c) 9.47 × 10 4 L2 mol−2 s−1
Rate = k [NOBr2 ][NO]. However, [NOBr2] = Kc[NO][Br2].
(d) 1.7 × 106 L2 mol−2 s −1 Therefore,
Solution [NOBr2 ]
Rate = k ⋅ K c [NO][Br2 ][NO], where K c =
(a) In this problem, the rate of reaction is measured in terms [NO][Br2 ]
of change in pressure with time. The rate law can be writ-
ten as or Rate = k′[NO]2 [Br2 ]
d( pN2) 13. A reaction involving two different reactants can never be
= k ( pH2)a( pNO)b (a) unimolecular reaction (b) first-order reaction
dt
(c) second-order reaction (d) bimolecular reaction
The first two data indicate that rate of reaction depends
linearly upon partial pressure of H2, so the order with Solution
respect to H2 is 1. Third and fourth data indicate that (a) This is because the minimum number of reactants is two,
when pressure of NO increases 2 times, rate increases so the molecularity cannot be one (for molecularity = 1,
4 times. So, order with respect to NO is 2. there should be only one reactant).
Now using the first data,
Zero-Order Reactions
2
Rate = k ( pH2)( pNO ) 14. The time for half-life period of a certain reaction A → Products
Rate 1.6 −8 −2 −1 is 1 h. When the initial concentration of the reactant A, is 2.0
k= = = 3.5 × 10 torr s
( pH2)( pNO )2 289 × 4002 mol L−1. How much time does it take for its concentration to
come from 0.50 mol L−1 to 0.25 mol L−1, if it is a zero-order
But we need this rate constant in concentration terms. reaction?
From ideal gas equation, (a) 1 h (b) 4 h (c) 0.5 h (d) 0.25 h
(AIEEE 2010)
n p Solution
C= = ⇒ p = CRT
V RT
(d) For a zero-order reaction, t1/2 = [A]/2k where [A] is the ini-
Putting this value in rate constant, we get
tial concentration. So, k = [A]/2t1/2 = 2/2 × 1 = 2. Also, for
1 1 zero-order reaction,
k = 3.5 × 10 −8 = 3.5 × 10 −8 (RT )2
torr 2 s (1760
/ atm)2 s x (0.50 − 0.25)
−2−2−s1−1(R = 0.082 L atm mol−1 K−1)
k= ⇒t = = 0.25 h
= 163LL2mol
2 mol
s t 1
Alternate solution: 15. Decomposition on NH3 on heated tungsten yields the fol-
lowing data:
Rate = k[H2 ][NO]2
Initial pressure (mm Hg) 65 105 y 185
Using rate from the first data, we have
2 Half-life (s) 290 x 670 820
1.6 289  400  2 −2 −1
=k ×  = 163 L mol s
0RT
760 760RT  760RT  What are the values of x and y in that order?
(a) 420 s, 110 mm Hg (b) 500 s, 160 mm Hg
(c) 520 s, 170 mm Hg (d) 460s, 150 mm Hg
Order and Molecularity of Reactions
Solution
12. The following mechanism has been proposed for the reac-
tion of NO with Br2 to form NOBr: Half life 290 820
(d) = = 4.46 and = 4.43.
Initial pressure 65 185
NO(g) + Br2 (g)  NOBr2 (g)
The values nearly agree. Therefore, the reaction is of
NOBr2 (g) + NO(g)  2NOBr(g) zero-order.
x
If the second step is the rate determining step, the order of ≈ 4.4 ⇒ x ≈ 460 s
the reaction with respect to NO(g) is 105
(a) 1 (b) 0 (c) 3 (d) 2 670
= 4.4 ⇒ y ≈ 150 mm Hg
(AIEEE 2006) y
Solution 16. The unit of the rate of reaction is the same as that of the rate
(d) The reactions involved are constant for a
K
(a) zero-order reaction.

NO(g) + Br2 (g)  c
 NOBr2 (g) (b) first-order reaction.
(c) second-order reaction.
NOBr2 (g) + NO(g) → 2NOBr(g)
(d) half-order reaction.

Chapter-19.indd 634 8/5/2016 11:34:54 AM


Solved Examples 635

Solution (c) 3.47 × 10−5 M min−1


(a) The rate of a reaction is d[conc.]/dt with units as conc. (d) 1.73 × 10−4 M min−1 (AIEEE 2012)
time−1. The rate constant is given by Solution
k = [conc.]1−n time−1 (b) Using the expression of rate constant for first-order reac-
when n = 0, k = [conc.] time−1. tion, we have
17. Time required for 100% completion of a zero-order reaction is 2.303 [A ]
k= log 0
2k a a t [A ]
(a) (b) (c) (d) ak Given that t = 40 min, [A] 0 = 0.1 M and [A] = 0.025 M.
a 2k k
Therefore,
Solution
1 2.303 0.1
(c) For a zero-order reaction, k = [a − x ] . Since it is 100% k= log = 0.03465 min−1
t 40 0.025
complete x = 0. So, k = a/t ⇒ t = a/k.
Now, Rate = k[A] (for first-order reaction) = 0.03465 × 0.01
18. Which is not the correct statement for zero-order reaction? = 3.47 × 10−4 M min−1.
dx
(a) = k [a0 ] (b) at = a0 − kt 21. The reaction
dt
1 1 1 2N2O5(g) → 4NO2(g) + O2(g)
(c) log (a0 − at) = log k + log t (d) = +
a0 − at t k follows first-order kinetics. The pressure of a vessel containing
Solution only N2O5 was found to increase from 50 mm Hg to 87.5 mm
(c) Consider the kinetics of zero order, A → Product. The Hg in 30 min. The pressure exerted by the gases after 60 min
expression for rate law is will be (assume temperature remains constant):
Rate = k[A]0 (a) 106.25 mm Hg (b) 116.25 mm Hg
dx (c) 125 mm Hg (d) 150 mm Hg
= k [A ]0 (JEE Main Online 2015)
dt
Solution
On integrating both sides, we get ∫ dx = ∫ k dt
or [A] = kt + C (a)
At t = 0, we have [A] = [A]0 therefore, C = [A]0. Substituting, 2N2O5 (g) → 4NO2 (g) + O2 (g)
we get
t=0 50 mm 0 0
[A] = − kt + [A]0 ⇒ [A] − [A]0 = − kt ⇒ [A]0 − [A] = kt
t = 30 min 50 – 2x 4x x
Taking log on both sides, we get t = 60 min 50 – 2y 4y y
log([A]0 − [A]t ) = log k + log t
At t = 30 min, net pressure = 87.5, therefore
First-Order Reactions 50 − 2 x + 4 x + x = 87.5
19. The half-life period of a first-order chemical reaction is 6.93 3 x = 37.5 ⇒ x = 12.5
min. The time required for the completion of 99% of the
chemical reaction will be (log 2 = 0.301) The pressure of N2O5 after 30 min is 50 – 2x
(a) 46.06 min (b) 460.6 min ⇒ 50 – 12.5 × 2 = 25
(c) 230.3 min (d) 23.03 min It indicates that after 30 min net pressure of N2O5 reduces
(AIEEE 2009) to half, therefore, for next 30 min the pressure of N2O5
Solution would further reduce to half
(a) Using equation for half-life of first-order reaction, 50 − 2y = 12.5 ⇒ y = 18.75 mm
0.693 0.693 Net pressure at t = 60 min
k= = = 0.1 min−1
t1/2 6.93
ptotal = 50 − 2 y + 4 y + y
2.303 a
t= log = 50 + 3 y
k a− x
where a is the initial concentration and x is the concentra- = 50 + 3 × 18.75
tion at time t. = 106.25 mm

6.93  100 22. In first-order reaction, the concentration of the reactant



t =  2.303 ×  log = 46.06 min decreases from 0.8 M to 0.4 M in 15 min. The time taken for
 0.6932  1
the concentration to change from 0.1 M to 0.025 M is
20. For a first-order reaction, A → Products, the concentration of (a) 30 min (b) 60 min (c) 7.5 min (d) 15 min
A changes from 0.1 M to 0.025 M in 40 min. The rate of the Solution
reaction when the concentration of A is 0.01 M is
(a) Since, the concentration of products becomes half (0.8 to
(a) 1.73 × 10−5 M min−1
0.4 M), means t1/2 is given as 15 min.
(b) 3.47 × 10−4 M min−1

Chapter-19.indd 635 8/5/2016 11:35:00 AM


636 Chapter 19 Chemical Kinetics

For a first-order reaction, 1.1885 x = 1.1885a − a = 0.1885a


0.693 0.693
k= = x 0.1885
t1/ 2 15 = = 0.159 = 15.9 %
a 1.1885
Also, for a first-order reaction,
2.303 [A ] 2.303 0.1 Second-Order Reactions
t= log 0 = log = 30 min
k [A ] (0.693/15) 0.025 26. A reaction was found to be second-order with respect to the
23. The time t1/4 can be taken as the time taken for the concen- concentration of carbon monoxide. If the concentration of
tration of a reactant to drop to 3/4th of its initial value. If the carbon monoxide is doubled, with everything else kept the
rate constant for a first-order reaction is k, then t1/4 can be same, the rate of reaction will
written as (a) remain unchanged (b) triple
(a) 0.10/k (b) 0.29/k (c) 0.69/k (d) 0.75/k (c) increase by a factor of 4 (d) double
(AIEEE 2006)
Solution Solution
(b) For a first-order reaction, (c) For nth order reaction, the rate of reaction ∝ [concentra-
 2.303   1  0.29 tion]n
t1/ 4 = log  =
 k   1 − 1/ 4  k For second-order reaction, the rate of reaction ∝ [con-
centration]2. So, for the given data Rate1 = [CO]2
24. The concentration of a drug in the body is often expressed When concentration of CO is doubled then,
in units of milligrams per kilogram of body weight. The initial Rate2 = [2CO]2 = 4[CO]2 ⇒ Rate2 = 4Rate1.
dose in an animal was 25.0 mg/kg body weight. After 2.00 h,
10.0 mg/kg body weight is disappeared through metabo- 27. Consider the reaction 2A + B → Products. When concentra-
lism. If metabolism is of first-order then the approximate rate tion of B alone was doubled, the half-life did not change.
of metabolism after 2 h is When the concentration of A alone was doubled, the rate
(a) 0.083 mg min−1 (b) 0.076 mg min−1 increased by two times. The unit of rate constant for this
(c) 0.063 mg min−1 (d) 0.042 mg min−1 reaction is
(a) s−1 (b) L mol−1 s−1
Solution (c) no unit (d) mol−1 s−1
(c) For first-order reactions: (AIEEE 2007)
 a  Solution
kt = ln   and Rate = k[Drug]
a − x  (b) In the reaction 2A + B → Products, when concentra-
tion of B is doubled, the half-life did not change, hence
[Drug]0 = a = 25 mg and [Drug]t = a − x = 25 − 10 = 15 mg
the reaction is of first-order with respect to B because
1  a  2.303  25  half-life is independent of concentration for first-order
Therefore, k = ln  = log   = 0.0042 min−1 reaction (t1/2 = 0.693/k). When concentration of A is dou-
t  a − x  120  15 
bled, the reaction rate is doubled, hence the reaction is
Rate = 0.0042 min−1 × 15 mg ≈ 0.063 mg min−1 of first-order with respect to A. So, the overall order is
1 + 1 = 2 and the units of rate constant for a second-
25. The rate constant for a first-order reaction is 3.2 × 10−5 s−1 at order reaction are L mol−1 s−1.
a particular temperature. What percentage of reactants will
react on heating for 1.5 h? 28. The half-life period of a first-order reaction is 15 minutes. The
(a) 16.5% (b) 20.3% amount of substance left after one hour will be:
(c) 19.2% (d) 15.9% 1
(a) of the original amount
4
Solution
1
(d) For a first-order reaction, the integrated rate law is given (b) of the original amount
8
by
1
[A ] [A ] 2.303 [A ] (c) of the original amount
kt = ln 0 = 2.303 log 0 ⇒ k = log 0 16
[A]t [A]t t [A ]t
1
(d) of the original amount
Substituting the given values in the rate law equation, 32
we get
(JEE Main Online 2014)
2.303 a
3.2 × 10 −5 s −1 = log Solution
1.5 × 60 × 60 s a− x
1 hour
−5 −1
(c) Total number of half-lives (n) = =4
a 3.2 × 10 s × 1.5 × 60 × 60 s 15 min
log = = 0.075
a− x 2.303
[A]0
a Substance left after n half-lives =
= 1.1885 ⇒ a = 1.1885 a − 1.1885 x (2)n
a− x

Chapter-19.indd 636 8/5/2016 11:35:06 AM


Solved Examples 637

1 1 (a) 5021 s (b) 3000 s


Substance remaining after 4 half-lives = 4
= (c) 4256 s (d) 2000 s
(2 ) 16
Solution
29. A + 2B → C, the rate equation for this reaction is given as
Rate = k[A][B] (b) For a second-order reaction,
If the concentration of A is kept the same but that of B is dou- 1 x
bled what will happen to the rate itself? k2 = ⋅
t a(a − x )
(a) halved (b) the same
(c) doubled (d) quadrupled When the reaction is 20% complete, x is 20% of the initial
(JEE Main Online 2015) concentration a, that is,
Solution 20a
20% of a = = 0.2a
(c) 100
R = k [A][B]
1 0.2a 1
When concentration of B is doubled, rate So, k2 = × =
500 a(a − 0.2a) 2000a
R′ = k[A] [B] = 2R
Now suppose it takes t1 seconds for the reaction to go for
30. The reaction A → X is a first-order reaction, which takes place
60% completion. Then for 60% completion
in a vessel with initial concentration of A as 10 M. The rate
constant for this reaction is 4.16 h−1. In the presence of a cata- 60a
x= = 0.6a
lyst, the reaction A → Y follows second-order kinetics and the 100
rate constant is 0.2 min−1 M−1. What should be the initial con-
centration of A in the presence of catalyst so that the value of 1 0.6a 1 1 3
t1/2 for both the reactions is the same? Thus, k2 = × ⇒ = ×
t a(a − 0.6a) 2000a t1 2a
(a) 30.0 M (b) 0.5 M (c) 5.0 M (d) 10.0 M
which gives t1 = 3000 s.
Solution
→ X (k1 = 4.16 h−1 = 6.93 × 10 −2 min−1)
k
(b) (i) A 
1
Experimental Determination of Rate Law
33. For the non-stoichiometric reaction 2A + B → C + D, the fol-
→ Y (k2 = 0.2 min−1 M−1)
k
(ii) A 
2

lowing kinetic data were obtained in three separate experi-


0.693 ments, all at 298 K.
Now, for (i) t1/ 2 = = 10 min (for first-order
6.93 × 10 −2 reaction)
Initial concentra- Initial con- Initial rate of forma-
tion (A) centration (B) tion of C (mol L-1 s-1)
It is independent of initial concentration.
1 0.1 M 0.1 M 1.2 × 10−3
Also, for (ii) t1/ 2 = (for a second-order reaction)
ka 0.1 M 0.2 M 1.2 × 10−3
1
or a= −1 −1
= 0.5 M 0.2 M 0.1 M 2.4 × 10−3
(0.2 min M × 10 min)
The rate law for the formation of C is
31. The reaction 2HI(g) → H2(g) + I2(g) has the rate law,
dC dC
Rate = k[HI]2, with k = 0.079 L mol−1 s−1 at 508°C. What is the (a) = k[A][B] (b) = k[A]2 [B]
half-life for this reaction at this temperature when the initial dt dt
HI concentration is 0.10 M? dC dC
(a) 2.3 × 103 s (b) 3.5 × 104 s (c) = k[A][B]2 (d) = k [A ]
dt dt
(c) 1.3 × 102 s (d) 7.6 × 103 s
(JEE Main 2014)
Solution Solution
(c) The initial concentration is 0.10 mol L−1; k
= 0.079 L mol−1 (d) For the reaction 2A + B → C + D, the rate of formation of C
s−1. Substituting these values in the equation below is
1 d[C]
t1/ 2 = = k [A ] x [B] y
k × (initial concentration of reactant ) dt
we get, From the given values in the table, we have
(1)
1 1.2 × 10 −3 = k [0.1] x [0.1] y
t1/ 2 = = 1.3 × 102 s
(0.079 L/mol / s) × (0.10 L/mol) (2)
1.2 × 10 −3 = k [0.1] x [0.2] y
32. A second-order reaction, where a = b, is 20% complete in
(3)
500 s. How long will it take for the reaction to go for 60% 2.4 × 10 −3 = k[0.2] x [0.1] y
completion?

Chapter-19.indd 637 8/5/2016 11:35:12 AM


638 Chapter 19 Chemical Kinetics

Dividing Eq. (1) by Eq. (3), we get (c) the total energy of the reacting molecules at a tempera-
ture, T.
1.2 × 10 −3
x
k[0.1] x [0.1] y 1  1 (d) the fraction of molecules with energy greater than the
= ⇒ =   ⇒ x =1
2.4 × 10 −3
k[0.2] x [0.1] y 2  2 activation energy of the reaction
(AIEEE 2006)
Dividing Eq. (1) by Eq. (2), we get
Solution
1.2 × 10 −3
y
k [0.1] x [0.1] y  1
= ⇒ 1=   ⇒ y = 0 (b) Ea is the activation energy which represents the mini-
1.2 × 10 −3 k [0.1] x [0.2] y  2
mum energy that the reacting molecules must possess
dC before undergoing a reaction. As Ea increases, the rate
= k[A].
Therefore, rate of formation of C is
dt constant k decreases.
34. For the reaction, 2 NO + Cl2 → 2 NOCl at 300 K, following data
36. The energies of activation for forward and reverse reactions
are obtained
for A2 + B2  2AB are 180 kJ mol−1 and 200 kJ mol−1, respec-
Initial concentration tively. The presence of a catalyst lowers the activation energy
Experiment Initial rate
[NO] [Cl2] of both (forward and reverse) reactions by 100 kJ mol−1. The
enthalpy change of the reaction (A2 + B2 → 2AB) in the pres-
1. 0.010 0.010 1.2 × 10 −4 ence of catalyst will be (in kJ mol−1)
(a) 20 (b) 300 (c) 120 (d) 280
2. 0.010 0.020 2.4 × 10 −4 (AIEEE 2007)
Solution
3. 0.020 0.020 9.6 × 10 −4 (a) Consider the following figure,
In absence of catalyst
The specific rate constant will be
(a) 1.2 × 102 mol−2 L2 s −1 (b) 2.5 × 102 mol−2 L2 s −1 In presence of catalyst
Potential energy

(c) 3.2 × 102 mol−2 L2 s −1 (b) 4.3 × 102 mol−2 L2 s −1 D


B 200
Solution
180 80
(a) Let the rate law for the reaction be
Rate = k[NO]x[Cl2]y 100
From Experiment 1, we have
A
1.2 × 10−4 = k[0.010]x[0.010]y (1) C
From Experiment 2, we have
Reaction coordinate
2.4 × l0−4 = k[0.010]x[0.020]y (2)
Dividing Eq. (2) by Eq. (1), we get From the figure, we have
2.4 × 10 −4
(0.020 ) y
∆ r H = E f − Er = AB − CD = 80 − 100 = −20 kJ mol−1
= ⇒ 2 = 2y ⇒ y = 1
1.2 × 10 −4 [ 0.100 ] y 37. The rate of a chemical reaction doubles for every 10°C rise of
From Experiment 3, we have temperature. If the temperature is raised by 50°C, the rate of
9.6 × 10−4 = k[0.020]x[0.020]y (3) the reaction increases by about
Dividing Eq. (3) by Eq. (2), we get (a) 24 times. (b) 32 times.
(c) 64 times. (d) 10 times.
9.6 × 10 −4 [0.020] x
= ⇒ 4 = 2x ⇒ x = 2 (AIEEE 2011)
2.4 × 10 −4 [0.100] y Solution
Hence, the order of the reaction = x + y = 2 + 1 = 3
(b) According to Arrhenius theory,
Rate law for the reaction is Rate = k[NO]2[Cl2]
Temperature coefficient A = 2 (given)
Considering Eq. (1) again, we get
1.2 × 10−4 = k[0.010]2[0.010] k2
( A∆T /10 ) =
k1
1.2 × 10 −4
k= = 1.2 × 102 mol−2 L2 s −1 k2
[0.010]3 250 /10 = 25 = 32 =
k1

Effect of Temperature on Rate Constant Therefore, 32 k1 = k2. This means when the temperature
is raised by 50°C, then the rate increases by 32 times.
35. Rate of a reaction can be expressed by Arrhenius equation as:
38. The rate of a reaction doubles when its temperature changes
k = Ae − Ea / RT
from 300 K to 310 K. Activation energy of such a reaction will
In this equation, Ea represents be (R = 8.314 J K−1 mol−1 and log 2 = 0.301)
(a) the energy above which all the colliding molecules will (a) 48.6 kJ mol−1 (b) 58.5 kJ mol−1
react. (c) 60.5 kJ mol −1 (d) 53.6 kJ mol−1
(b) the energy below which colliding molecules will not (JEE Main 2013)
react.

Chapter-19.indd 638 8/5/2016 11:35:18 AM


Solved Examples 639

Solution
6.0
(d) As per Arrhenius equation:
k2 E  1 1
ln = − a − 

In Keq
k1 R  T2 T1 
Ea  1 1 
2.303 log 2 = −  − 
8.314  310 300 
2.0
which gives Ea = 53.6 kJ mol−1 . 1 −1
1.5 × 10−3 (K ) 2.0 × 10−3
39. The rate coefficient (k) for a particular reactions is 1.3 × 10−3 T
M−1 s−1 at 100°C and 1.3 × 10−3 M−1 s−1 at 150°C. What is the The reaction must be
energy of activation (EA) (in kJ) for this reaction? (R = molar (a) exothermic
gas constant = 8.314 JK−1 mol−1) (b) endothermic
(a) 16 (b) 60 (c) 99 (d) 132 (c) one with negligible enthalpy change
(JEE Main Online 2014) (d) highly spontaneous at ordinary temperature
Solution Solution
k2 Ea  1 1  (a) ∆G ° = ∆H ° − T ∆S °
(b) log = −
k1 2.303R  T1 T2  − RT ln K eq = ∆H ° − T ∆S °
− ∆H ° ∆S °
1.3 × 10 −3 Ea  1 1  ln K eq = +
log =  −  RT R
1.3 × 10 −4 2.303 × 8.314  373 423  −∆H °
Slope = = + ve
Ea R
−1 = ( −0.00031) ⇒ Ea = 60.784 J = 60.784 kJ
19.147 Therefore, ΔH° is negative, hence, the reaction is
40. For the equilibrium, A(g)  B(g), ΔH is −40 kJ mol−1. If the exothermic.
ratio of the activation energies of the forward (Ef ) and reverse
43. The rate constant for the first-order decomposition of ethyl-
(Eb) reaction is (2/3) then:
ene oxide into CH4 and CO is represented as
(a) Ef = 60 kJ mol−1; Eb = 100 kJ mol−1
(b) Ef = 30 kJ mol−1; Eb = 70 kJ mol−1 1.25 × 10 4 K
log10 k (s −1) = 14.34 −
(c) Ef = 80 kJ mol−1; Eb = 120 kJ mol−1 T
(d) Ef = 70 kJ mol−1; Eb = 30 kJ mol−1 Determine Ea of the reaction.
(JEE Main Online 2015) (a) 239.3 kJ mol–1 (b) 103.9 kJ mol–1
Solution (c) 5.42 kJ mol–1 (d) 57.57 kJ mol–1
(c) A(g)  B(g) ∆H = −40 kJ Solution
2 2x 3x  E 
E
Since, f = , therefore, E f = and Eb = (a) Arrhenius equation is k = A × exp  − a 
 RT 
Eb 3 5 5
Taking natural logarithm, we get
Eb − E f = + 40 E Ea
ln(k ) = ln( A) − a ⇒ log(k ) = log( A) −
RT 2.303RT
3x 2x x
− = + 40 ⇒ = 40 ⇒ x = 200 Plot of log(k) vs. 1/T for Arrhenius equation gives a straight
5 5 5
line. The slope of this line is −Ea/2.303R. Comparing this
Therefore,
slope with the given equation’s slope, we get
3x 3 × 200
Eb = = = 120 kJ mol−1 −
Ea
= −1.25 × 10 4
5 5 2.303R
2x 2 × 200 ⇒ Ea = 12.5 × 2.303 × 8.314 = 239.3 kJ mol−1
Ef = = = 80 kJ mol−1
5 5
44. The activation energy of a reaction is 24.0 kcal mol−1 at 27°C
− E a / RT and the presence of catalyst changes its activation energy to
41. In respect of the equation k = Ae in chemical kinetics,
which one of the following statements is correct? one-fourth at the same temperature. The approximate ratio
(a) k is equilibrium constant (b) A is adsorption factor of rate in the presence of catalyst to rate in the absence of
(c) Ea is energy of activation (d) R is Rydberg’s constant catalyst will be (use R = 2 cal mol−1 K−1).
(a) 1.0 × 104 (b) 1.0 × 103
Solution
(c) 2.07 (d) 1 × 1013
(c) Ea is the activation energy, T is the temperature, R is the
gas constant and A is the pre-exponential factor. Solution
(d) From Arrhenius equation, k = Ae − Ea / RT
42. A schematic plot of ln Keq versus inverse of temperature for a Let the rate in the absence of catalyst be k1 = Ae−24/RT and
reaction is shown in the following figure in the presence of catalyst is k2 = Ae−24/4RT

Chapter-19.indd 639 8/5/2016 11:35:29 AM


640 Chapter 19 Chemical Kinetics

Therefore, 46. The activation energy Ea of a reaction can be calculated by


 k  18 × 1000 plotting:
k2
⇒ = e( 24 − 6 )/ RT ⇒ ln  2  = = 30 (a) k against T (b) k against 1/log T
k1  k1  2 × 300 (c) log k against 1/T (d) log k against 1/log T
k  30 Solution
log10  2  = ≈ 13
 k1  2.303 (c) According to Arrhenius equation,
k2 Ea
≈ 1 × 1013 log k = − + log A
k1 2.303RT
r2 k2 Comparing it with straight line equation y = mx + c,
Since, rate ∝ rate constant, so = ≈ 1.0 × 1013
r1 k1 we get slope as −Ea/2.303R on plotting log k vs. 1/T as
45. For a first-order reaction, the units of Arrhenius factor (A) will shown.
be
(a) J K−1 s−1 (b) s−1 −Ea
(c) mol L s −1 −1 (d) J K−1 L−1 s−1 Slope =
2.303 R
Solution log k
(b) Arrhenius’ equation gives the dependence of the rate
constant k of a chemical reaction on the absolute tem-
perature T (in kelvin), where A is the pre-exponential fac-
tor (or simply the prefactor), Ea is the activation energy, 1/T
and R is the universal gas constant
Ea
k = Ae − Ea / RT ⇒ A = ln k +
RT
J mol−1
Therefore units of A = s −1 + −1
= s −1
JK mol−1 × K

ADVANCED LEVEL PROBLEMS


1. Consider a reaction aG + bH → Products. When concentration Solution
of both the reactants G and H is doubled, the rate increases (a) Using half-life period method,
by eight times. However, when concentration of G is doubled For first-order reaction, k1 = 0.693/t1/2 = 0.693/40
keeping the concentration of H fixed, the rate is doubled. The For zero-order reaction k0 = [A]0/2t1/2 = 1.386/(2 × 20)
overall order of the reaction is Therefore,
(a) 0 (b) 1 (c) 2 (d) 3
(IIT-JEE 2007) k1 (0.693/4 0 )
= = 0.5 mol−1
k0 ( 4 0 /1386
. )
Solution
(d) aG + bH → Products. The rate is directly proportional to 3. For a first-order reaction A → P, the temperature (T) depend-
[G]m [H]n ent rate constant (k) was found to follow the equation
When concentration of both the reactant is doubled then, 1
log k = −(2000 ) + 6.0
T
Rate = [2G]m [2H]n = 2m 2n [G][H] The pre-exponential factor A and activation energy Ea, respec-
Then the rate increases eight times which means either tively, are
m = 2 then n = 1 or n = 2 and m = 1. Again, it is given that (a) 1.0 × 106 s −1 and 9.2 kJ mol−1
when the concentration of G is doubled then rate is dou-
bled which means that m = 1 and n = 2. And the overall (b) 6.0 s −1 and 16.6 kJ mol−1
order is m + n = 2 + 1 = 3.
(c) 1.0 × 106 s −1 and 16.6 kJ mol−1
2. Under the same reaction conditions, initial concentration of
(d) 1.0 × 106 s −1 and 38.3 kJ mol−1
1.386 mol dm−3 of a substance becomes half in 40 and 20 s
(IIT-JEE 2009)
through first-order and zero-order kinetics, respectively. Ratio
Solution
(k1/k0) of the rate constants for first-order (k1) and zero order
(k0) of the reactions is (d) Using Arrhenius theory,
(a) 0.5 mol−1 dm3 (b) 1.0 mol dm−3 k = Ae − Ea / RT
(c) 1.5 mol dm−3 (d) 2.0 mol−1 dm3 It is given that
(IIT-JEE 2008) 2000
log k = 6 − (1)
T

Chapter-19.indd 640 8/5/2016 11:35:34 AM


Advanced Level Problems 641

Ea Solution
Also log k = log A − (2)
2.303RT (a), (b), (d) According to first-order reaction,
Comparing both Eqs. (1) and (2) and solving, we get −d[N2O5]/dt = k[N2O5]. The half-life can be found as
 2.303  a
2000 t= log
 k 
Ea
6− ≡ log A − a− x
T 2.303RT
where a = 100 and a − x = 100 − 99.6 = 0.4. The half-life period
log A = 6 ⇒ A = antilog(6 ) = 106 s −1 for first-order reaction is
Ea 2000 ln 2 0.693
= ⇒ Ea = 2000 × 2.303 × 8.314 = 38.3 kJ mol−1 t1/2 = =
2.303RT T k k
If temperature increases, k increases (according to Arrhenius
equation) and since t1/2 is inversely proportional to k, so if k
4. Plots showing the variation of the rate constant (k) with increases then t1/2 decreases.
temperature (T) are given. The plot that follows Arrhenius If the reaction proceeds to 99.6% completion then,
equation is
(a) (b)  2.303  100  2.303  0.693
t 99.6 =  log =  2.4 = 8 × = 8t1/2
 k  0.4  k  k

k k 7. An organic compound undergoes first-order decomposition.


The time taken for its decomposition to 1/8 and 1/10 of its
initial concentration are t1/8 and t1/10, respectively. What is the
[t ]
value of 1/ 8 × 10 ? (Take log 2 = 0.3)
T T [t1/10 ]
(IIT-JEE 2012)
(c) (d)
Solution

k k
(9) Using the expression for first-order reaction
2.303 [A ]
t= log 0
k [A ]
when the compound is decomposed to 1/8th of its initial
T T value then the time taken is
(IIT-JEE 2010)  2.303  1  2.303 
t1/ 8 =  log =  log 8 (1)
Solution  k  (1/ 8 )  k 
(a) According to the Arrhenius equation k = Ae − Ea / RT. As temp- when the compound is decomposed to 1/10th of its ini-
erature increases, rate constant (k) increases exponentially. tial value then the time taken is
5. The concentration of R in the reaction R → P was measured as  2.303  1  2.303 
t1/10 =  log =  log 10 (2)
a function of time and the following data are obtained:  k  (1/ 10 )  k 
[R] (mol L−1) 1.0 0.75 0.40 0.10 Dividing Eq. (1) by Eq. (2), we get
t (min) 0.0 0.05 0.12 0.18
The order of the reaction is ___. t1/ 8 log 8
= = log(23 ) = 3 × 0.3 = 0.9
(IIT-JEE 2010) t1/10 log 10
Solution
[t1/ 8 ]
(0) Considering zero-order kinetics, k1 = x/t = 0.25/0.05 = 5 So, the value of × 10 = 9.
[t1/10 ]
and k2 = x/t = 0.60/0.12 = 5. Since the rate constants are
coming to be constant, so the reaction follows zero-order 8. In the reaction, P + Q → R + S, the time taken for 75% reaction
kinetics. of P is twice the time taken for 50% reaction of P. The concen-
tration of Q varies with reaction time as shown in the figure.
6. For the first-order reaction
The overall order of the reaction is
2N2O5 (g) → 4NO2 (g) + O2 (g)
(a) The concentration of the reactant decreases exponentially [Q]0
with time.
[Q]
(b) The half-life of the reaction decreases with increasing
temperature.
(c) The half-life of the reaction depends on the initial concen-
tration of the reactant.
Time
(d) The reaction proceeds to 99.6% completion in eight half-
life direction. (a) 2 (b) 3 (c) 0 (d) 1
(IIT-JEE 2011) (JEE Advanced 2013)

Chapter-19.indd 641 8/5/2016 11:35:50 AM


642 Chapter 19 Chemical Kinetics

Solution 13. For the reaction 2X(g) → 3Y(g) + 2Z(g)


(d) This is a first-order reaction because t75% = 2 × t50%. The
graph shows that the order with respect to Q is 0, so we Time (min) 0 100 200
can write the rate expression as rate = k[P]1[Q]0. Partial pressure of X (mm Hg) 800 400 200
9. For the elementary reaction M → N, the rate of disappearance Assuming ideal gas condition. Calculate (a) order of reaction,
of M increases by a factor of 8 upon doubling the concentra- (b) rate constant, (c) time taken for 75% completion of reac-
tion of M. The order of the reaction with respect to M is tion and (d) total pressure when pX = 700 mm Hg.
(a) 4 (b) 3 (c) 2 (d) 1
(JEE Advanced 2014) Solution
Solution (i) From the given data, it is observed that t1/2 of the X is con-
stant, that is, 100 min. So, the order of the reaction is 1.
(b) Rate = k[M]n. If n = 3, then on increasing the concentration
0.693 0.693
of M two times we get (ii) Rate constant (k) = = = 6.93 × 10 −3 min−1
t1/ 2 100
Rate = k[2M]3 = 8k[M]3
(iii) Time taken for 75% completion of reaction
t = 2 × t1/ 2 = 2 × 100 = 200 min.
10. If the concentration of a reactant decreases from 800 mol
dm−3 to 50 mol dm−3 in 2 × 105 s in a first-order reaction, the (iv) The reaction can be represented as
rate constant of the reaction would be 2X(g) → 3Y(g) + 2Z(g)
(a) 2 × 10−5 s (b) 1.386 × 10−5 s Initial pressure 800 0 0
−5
(c) 3.45 × 10 s (d) 2 × 105 s Pressure at time t 800 − x (3/2) x x
Solution
Given that 800 − x = 700 mm Hg, so x = 100 mm Hg
(b) For the first-order reaction, we have  3
Hence, the total pressure = 800 − x +   x + x = 700 + 150
2.303 [A ] 2.303 800 + 100 = 950 mm Hg.  2
k= log 0 = 4
log = 1.386 × 10 −5 s
t [A]t 2 × 10 50
14. The rate of reaction is doubled when the temperature is
11. The initial rate of a second-order reaction is 4 × 10 −4 mol L−1 changed from 298 K to 308 K. Calculate the energy of activa-
s−1. If the initial concentration of the reacting substance is tion of the reaction.
0.2 mol L−1, calculate the rate constant. (a) 62.35 kJ mol−1 (b) 56.20 kJ mol−1
(a) 1× 102 L mol−1 s −1 (b) 2 × 102 L mol−1 s −1
(c) 49.86 kJ mol−1 (d) 58.89 kJ mol−1
−2 −1 −1
(c) 1× 10 L mol s (d) 2 × 10 −2 L mol−1 s −1
Solution
Solution (d) The Arrhenius equation can be used to calculate energy
(a) The second-order reaction may be represented as of activation, if rate constant at two different tempera-
2A→Products. The rate law equation is, ture is known
Rate = k[A]2
T1T2 k 
Given that the rate = 4 × 10 −4 mol L−1 s−1; concentration Ea = 2.303R log  2 
T2 − T1  k1 
of reactant A = 0.20 mol L−1. Substituting these values in
the rate law equation, we get Given that T1 = 298 K; T2 = 308 K; k2 = 2k1; R = 8.314 J
K−1mol−1. Substituting these in the equation, we get
4 × 10 −4 = k [ 0.20 × 0.20 ]
 298 × 308   2k 
Ea = 2.303 × 8.314 ×  log  1 
Therefore, the rate constant is calculated as  308 − 298   k1 
4 × 10 −4  91784 
k= = 1× 102 L mol−1 s −1 314 × 
= 2.303 × 8.3 log(2)
0.04  10 
= 2.303 × 8.314 × 9178.4 × 0.301
12. Which of the following statements is incorrect about order of
a reaction? = 52897.78 J mol−1
(a) Order of a reaction can never be equal to zero or frac-
= 58.89 kJ mol−1
tional value.
(b) It is always determined experimentally. 15. The decomposition of N2O5 follows the reaction; 2N2O5(g)
(c) It is equal to the molecularity of an elementary reaction. → 4NO2(g) + O2(g) and it is a first-order reaction. The reac-
(d) It is sum of the powers of concentration terms in the dif- tion was allowed to take place in a closed vessel. After 30
ferential rate law of a reaction. minutes, the total pressure was 284.5 mm Hg, and on com-
Solution plete decomposition the total pressure was 584.5 mm Hg.
Calculate the rate constant of the reaction.
(a) Order of reaction can be zero or have fractional value. (a) 8.7 × 10−3 min−1 (b) 5.2 × 10−3 min−1
(c) 5.2 × 10−3 s−1 (d) 2.5 × 10−3 min−1

Chapter-19.indd 642 8/5/2016 11:35:56 AM


Advanced Level Problems 643

Solution
(b) The rate equation is rate = k[A][B]0 or rate = k [A]. Putting
(b) The reaction can be expressed as rate = 0.1 mol L−1 s−1 and [A] = 0.2 mol L−1, we get

2N2O5 → 4NO2 (g) + O2 (g) 0.1 = k [0.2] ⇒ k = 0.5 s −1


Initial pressure p 0 0
17. A hydrogenation reaction is carried out at 500 K. If the
After 30 min p−x 2x x /2
same reaction is carried out in presence of a catalyst at
On completion 0 2p p /2 the same rate, the temperature required is 400 K. Calculate
the activation energy of the reaction if the catalyst lowers the
After the reaction is over, 1 mol changes to 5/2 mol of
activation energy barrier by 20 kJ mol−1.
product.
Therefore, the initial pressure, p = 2/5 × 584.5 = 233.8 mm Solution
Hg Given that k500 = k400, so we have
x 3x
Total pressure after 30 min = p − x + 2 x + = p + = 284.5 Ea1 Ea2 E T 4
2 2 Ae − Ea1 / RT1 = Ae − Ea 2 / RT2 ⇒ = ⇒ a1 = 1 =
3x RT1 RT2 Ea2 T2 5
or = 284.5 − p = 50.7
2 Given that Ea1 = Ea2 + 20 kJ. Substituting, we get
Therefore, x = 33.8 and p – x = 200 4
Ea1 = Ea1 + 20 ⇒ Ea1 = 100 kJ
5
2.303 p 2.303 233.8
k= log = log = 5.2 × 10 −3 min−1 18. The rate constant for the first-order decomposition of a cer-
t p−x 30 200 tain reaction is given by the equation,

16. For the given reaction A + B → Products , the following data 1.25 × 10 4
were given ln k = 14.34 −
T

Initial conc. of [A] (mol L−1) 0.1 0.2 0.1 Calculate (a) the energy of activation, (b) the rate constant
Initial conc. [B] (mol L−1) 0.1 0.1 0.2 at 500 K and (c) at what temperature will its half-life period
be 256 min?
Initial rate (mol L−1 s−1) 0.05 0.1 0.05
Solution
(a) Write the rate equation. From the Arrhenius equation, we have
(b) Calculate the rate constant. Ea
log k = log A −
Solution 2.303RT
(a) Let the order w.r.t reactants A and B be x and y, The given expression is
respectively. 1.25 × 10 4 14.34 1.25 × 10 4
x y ln k = 14.34 − ⇒ log k = −
Rate = k [A ] [B] T 2.303 2.303T
Putting rate = 0.05 mol L−1 s−1; and [A] = 0.1 mol L−1 and Comparing the two equations, we get
[B] = 0.1 mol L−1, we get Ea
(a) = 1.25 × 10 4 ⇒ Ea = 1.25 × 10 4 × 2.303 × 8.314
2.303R
0.05 = k[0.1] x [0.1] y (1)
= 2.393 × 105 J
Again putting rate = 0.1 mol L−1 s−1; and [A] = 0.2 mol L−1
1.25 × 10 4
and [B] = 0.1 mol L−1, we (b) When T = 500 K, we get ln k = 14.34 − = 14.34 − 25 = −10.66
500
0.1 = k[0.2] x [0.1] y (2) 1.25 × 10 4
ln k = 14.34 − = 14.34 − 25 = −10.66
500
−10.66
Dividing Eq. (1) by Eq. (2), we get Therefore, k = e = 2.35 × 10 −5 s −1
x x (c) When t1/2 = 256 min, rate constant is
 0.05  k[0.1]  1  1
  = x
⇒   =   , therefore x = 1
 2  2 0.693 0.693
0.1 k[0.2] k= = = 4.51× 10 −5 s −1
t1/ 2 256 × 60
Now, 0.05 = k [0.1] x [0.2] y (3) Substituting in the given expression, we get
Dividing Eq. (1) by Eq. (3), we get 1.25 × 10 4 1.25 × 10 4
ln( 4.51× 10 −5 ) = 14.34 − ⇒T = = 513.4 K
x y y T 14.34 + 10.0066
 0.05  k[0.1] [0.1]  1
  = ⇒ 1 =   , therrefore y = 0 1.25 × 10 4 1.25 × 10 4
0.05 k[0.1] [0.2] y
x  2 ln( 4.51× 10 −5 ) = 14.34 − ⇒T = = 513.4 K
T 14.34 + 10.0066

Chapter-19.indd 643 8/5/2016 11:36:04 AM


644 Chapter 19 Chemical Kinetics

PRACTICE EXERCISE
Level I and the initial reaction rate is 4.42 × 10−7 mol L−1s−1. What is
the half-life of this system?
Single Correct Choice Type (a) 1.48 × 103 s (b) 2.63 × 104 s
(c) 1.96 × 10 s2 (d) 3.06 × 103 s
1. The reaction rate increases with rise in temperature due to
(a) increase in activation energy
11. The half-life periods of a reaction at initial concentrations 0.1
(b) increase in effective collisions between molecules
mol L−1 and 0.5 mol L−1 are 200 s and 40 s, respectively. The
(c) decrease in activation energy
order of the reaction is
(d) decrease in minimum kinetic energy (threshold energy)
(a) 1 (b) 1/2 (c) 2 (d) 0
required for the reaction to occur
12. For a first-order reaction,
2. Calculate the rate constant for the following acid–base reac-
(a) t0.75 = 3t0.5 (b) t0.75 = 2t0.5
tion if the half-life for the reaction is 0.0282 s at 25°C and the
(c) t0.75 = 1.5t0.5 (d) t0.75 = 4t0.5
reaction is first-order in the NH4+ ion.
13. A first-order reaction is carried out starting with 10 mol L−1
NH+4 (aq) + H2O(l)  NH3 (aq) + H3O + (aq)
of the reactant. It is 40% complete in 1 h. If the same reac-
(a) 30.4 s−1 (b) 23.9 s−1 (c) 29.6 s−1 (d) 24.6 s−1 tion is carried out with an initial concentration of 5 mol L−1,
k1 the percentage of the reaction that is completed in 1 h

3. For an elementary reaction 2A   B the rate of disappear-
k2 will be
ance of A is equal to (a) 40%. (b) 80%. (c) 20%. (d) 60%.
2k1 2
(a) [A ] (b) −2k1[A ]2 + 2k2 [B] 14. The rate constant, the activation energy and the Arrhenius
k2
parameter of a chemical reaction at 25°C are 3.0 × 10−4 s−1,
(c) 2k1[A ]2 − 2k2 [B] (d) (2k1 − k2 ) [A ] 104.4 kJ mol−1, and 6.0 × 1014 s−1, respectively. The value of
4. For the reaction H2(g) + Br2(g) → 2HBr(g), the experimen- the rate constant as T → ∞ is
tal data suggests Rate = k[H2][Br2]1/2. The molecularity and (a) 2.0 × 1018 s−1 (b) 6.0 × 1014 s−1
order of reaction for the reaction is, respectively, (c) infinity. (d) 3.6 × 1030 s−1
(a) 2 and 2 (b) 2 and 3/2 15. A plot of log t1/2 vs. log C0 is given in the following figure:
(c) 3/2 and 2 (d) 3/2 and 3/2
5. The slope in the log k vs. 1/T curve is 5.42 × 103. The value of
log t1/2

the activation energy is approximately


(a) 104 J mol−1 (b) 208 J mol−1
(c) 104 kJ mol −1 (d) 104 J mol−1 K−1
6. A reaction of the type A(g) + B(g) → Product is taking place log C0
in a vessel. If the volume of the reaction vessel is suddenly The conclusion that can be drawn from this graph is
reduced to one fourth of the initial volume, the ratio of final 1
(a) Order = 1, t1/2 =
rate to initial rate is ka
(a) 4:1 (b) 2:1 (c) 16:1 (d) 1:2 2.303
(b) Order = 1, t1/2 = log 2
7. The plot of log k vs. 1/T is linear with a slope of k
E −Ea Ea −Ea 1
(a) a (b) (c) (d) (c) Order = 0, t1/2 =
R R 2.303R 2.303R 2ka
8. Calculate the order of the reaction in A and B. 1
(d) Order = 2, t1/2 =
[A] (mol L−1) 0.05 0.10 0.05 a
[B] (mol L−1) 0.01 0.05 0.10 16. Half-life period of a first-order reaction is 100 min. After 144.3
min, concentration of reactant is reduced to ___ of the origi-
Rate (mol L−1 s−1) 1.2 × 10−3 2.4 × 10−3 1.2 × 10−3 nal concentration
(a) 1 and 0 (b) 1 and 1 (a) 40% (b) 30% (c) 1/e (d) 1/e2
(c) 0 and 1 (d) 1 and 2
17. The reaction A(g) + 2B(g) → C(g) + D(g) is an elementary pro-
9. The unit of the rate constant depends upon the cess. In an experiment, the initial partial pressure of A and B
(a) temperature of the reaction. are pA = 0.60 and pB = 0.80 atm. When pC = 0.2 atm, the rate of
(b) activation energy of the reaction. the reaction relative to the initial rate is
(c) molecularity of the reaction. (a) 1/48 (b) 1/24 (c) 9/16 (d) 1/6
(d) order of the reaction.
18. A first-order reaction is 50% completed in 20 min at 27°C; the
10. The reaction 2NO2 → 2NO + O2 is second-order with respect to same reaction is 50% complete in 5 min at 77°C. Calculate
NO2. If the initial concentration of NO2(g) is 6.54 × 10−4 mol L−1 the activation energy of the reaction.

Chapter-19.indd 644 8/5/2016 11:36:09 AM


Practice Exercise 645

(a) 35.021 kJ mol−1 (b) 32.568 kJ mol−1 28. A + B → Products


−1 −1
(c) 24.205 kJ mol (d) 29.625 kJ mol [A] [B] Rate
19. Milk turns sour at 40°C three times faster than it does at 0°C. 0.1 0.1 x
This shows that activation energy of souring of milk (in cal) is
0.1 0.2 4x
2.303 × 2 × 273 × 313
(a) log 3 0.4 0.2 16x
40
2.303 × 2 × 273 × 313 1 What is the order of the reaction?
(b) log (a) 1 (b) 2 (c) 3 (d) 4
40 3
2.303 × 2 × 40 29. The rate constant of a reaction is 1.5 × 107 s−1
at 50°C and
(c) log 3
273 × 313 4.5 × 107 s−1 at 100°C. Evaluate the Ea. Given that R = 8.314 J
2.303 × 273 × 313 K −1mol−1.
(d) log 3 (a) 40.21 kJ mol−1 (b) 35.25 kJ mol−1
40
20. The half-life of a first-order reaction is 10 min. If initial amount (c) 22.13 kJ mol−1 (d) 29.06 kJ mol−1
is 0.08 mol L−1 and concentration at some instant is 0.01 mol
L−1, then t is 30. For a reaction A → B, it was found that the rate of reaction
doubles when the concentration of A is increased by 8 times.
(a) 10 min. (b) 30 min. (c) 20 min. (d) 40 min.
The order of reaction is
21. In a second-order reaction, the initial concentration of the (a) 2 (b) 8 (c) 1/2 (d) 1/3
reactant is 0.2 mol L−1. The reaction was 40% complete in
20 min. Calculate the time required for 75% completion of 31. Collision theory is applicable to
reaction. (a) first-order reactions. (b) zero-order reactions.
(a) 90 min (b) 58 min (c) 32 min (d) 74 min (c) bimolecular reactions. (d) intramolecular reactions.

22. The half-life of a substance in a first-order reaction is 15 min. 32. According to the reaction
The rate constant is
(a) 2.46 × 102 min−1 (b) 4.62 × 10−2 min−1 Cr2O72 − + 5H+ + 3HNO2 → 2Cr 3+ + 3NO3− + 4H2O
−2
(c) 6.74 × 10 min −1 (d) 7.18 × 102 min−1
The rate of disappearance of Cr2O72− is found to be 2.4 × 10−4
23. The rate of a reaction increases 8 times when the concentra- mol L−1 s−1. Find the rate of appearance of Cr3+ during given
tion of the reactant increases four times. The order of the reac- time interval.
tion is (a) 4.8 × 10 −4 mol L−1 s −1 (b) 5.9 × 10 −4 mol L−1 s −1
(a) 2.5 (b) 2.0 (c) 1.5 (d) 0.5
(c) 6.2 × 10 −4 mol L−1 s −1 (d) 9.5 × 10 −4 mol L−1 s −1
24. Which represents first-order reaction out of I, II and III in the
following figure? 33. The inversion of cane sugar is represented by
C12H22O11 + H2O → C6H12O6 + C6H12O6. It is a reaction that is
(a) second-order. (b) pseudo unimolecular.
log (a − x)

a−x

(c) unimolecular. (d) third-order.


a

t1/2

34. Identify the order of the reaction.


log

0 Time 0 Time 0 Concentration (a)


t1/2

II II III
(a) I, II and III (b) I and II
(c) II and III (d) I and III
1/Concentration
25. The decomposition of A into product has value of k as (a) 0 (b) 1 (c) 2 (d) 3
4.5 × 103 s−1 at 10°C and energy of activation 60 kJ mol−1. At
what temperature, would k be 1.5 × 104 s−1? 35. If the temperature of a reaction is increased from 25°C to 75°C,
(a) 12°C (b) 24°C (c) 48°C (d) 36°C (a) the reaction rate decreases, but k remains the same.
(b) the reaction rate and k both decrease.
26. The temperature coefficients of two reactions are 2 and 3, (c) the reaction rate increases, but k remains the same.
respectively. Which would be correct for these reactions? (d) the reaction rate and k both increase.
(a) E < E a1 a2
(b) Ea1 > Ea2
36. Which of the following curves represents a first-order reaction?
(c) Ea1 = Ea2 (d) Cannot be predicted. (a) (b)
27. A first-order reaction takes 40 min for 30% decomposition.
t1/2 t1/2
Calculate t1/2.
(a) 77.7 min (b) 27.2 min (c) 55.3 min (d) 67.3 min

a a

Chapter-19.indd 645 8/5/2016 11:36:19 AM


646 Chapter 19 Chemical Kinetics

(c) (d) 0.693


(c) t1/2 (first order) =
k
t1/2 t1/2
[A]0
(d) t1/2 (zero order) =
2k
a a 44. The rate law for the reaction RCl + NaOH → ROH + NaCl is
37. In the presence of a catalyst, the activation energy of a reac- given by Rate = k[RCl]. The rate of this reaction is
tion is lowered by 2 kcal at 27°C. The rate of reaction will (a) doubled by doubling the concentration of NaOH.
increase by (b) halved by reducing the concentration of RCl by one half.
(a) 2 times. (b) 14 times. (c) increased by increasing the temperature of the reaction.
(c) 28 times. (d) 20 times. (d) unaffected by change in temperature.
38. In a particular reaction, the time required to complete half of
45. The half-period T for the decomposition of ammonia on tung-
the reaction was found to increase 16 times when the initial
sten wire was measured for different initial pressures p of
concentration of the reactant was reduced to one-fourth.
ammonia at 25°C. Then
What is the order of the reaction?
(a) 1 (b) 4 (c) 2 (d) 3 p (mm Hg) 11 21 48 73 120
39. Consider the four reactions having activation energies 20 kJ, t (s) 48 92 210 320 525
30 kJ, 40 kJ and 50 kJ, respectively. The percent increase in the (a) it is a zero-order reaction.
rate of the reaction would be maximum when the temperature (b) it is a first-order reaction.
is increased from 300 K for the reaction having activation energy (c) rate constant for reaction is 0.114 s−1.
(a) 20 kJ (b) 30 kJ (c) 40 kJ (d) 50 kJ (d) rate constant for reaction is 1.14 s.
40. For the reaction A + B → C + D, doubling the concentration of 46. At 25°C, the second order rate constant for the reaction
both the reactants increases the reaction rate by eight times
and doubling the concentration of only B simply doubles the I− + ClO − → IO3− + Cl−
reaction rate. The rate law is given as
(a) r = k[A]1/2[B]1/2 (b) r = k[A][B]2 Is 0.0606 M−1 s−1 . If a solution is initially 3.5 × 10−3 M with
(c) r = k[A]2[B] (d) r = k[A][B] respect to each reactants, then choose the correct options.
Level II (a) [A] = 3.29 × 10−3 M after 300 s
(b) [B] = 3.29 × 10−3 M after 300 s
Multiple Correct Choice Type (c) [A] = 0.19 × 10−3 M after 300 s
41. The rate constant of a reaction is given by (d) [B] = 0.19 × 10−3 M after 300 s
k = 2.1 × 1010 exp (−2700/RT) 47. Which of the following statements are not true for a catalyst?
It means that, (a) Catalyst is a part of overall reaction.
(a) log k vs. 1/T will be a straight line with slope = 2700/2.303R. (b) It decreases the activation energy.
(b) log k vs. 1/T will be a straight line with intercept on log k (c) It alters the reaction mechanism.
axis = 2.1 × 1010. (d) Negative catalyst also increases the rate of reaction.
(c) the number of effective collisions is 2.1 × 1010 cm−3 s−1.
48. For a certain reaction A → Products, the t1/2 as a function of
(d) half-life of the reaction increases with increase of
[A]0, is given as follows:
temperature.
[A]0 (M) 0.1 0.025
42. If rate of reaction in terms of disappearance of NH3 is
−d[NH3 ]/dt , for the reaction: t1/2 (min) 100 50

4NH3 (g) + 5SO2 → 4NO(g) + 6H2O(g) Which of the following is true?


(a) The order is 1/2.
then which of the following expressions are correct for rate of (b) t1/2 would be 100 10 min for [A]0 = 1 M.
the reaction? (c) The order is 1.
4 d[O2 ] 3 d[H2O] (d) t1/2 would be 100 min for [A]0 = 1 M.
(a) Rate = − (b) Rate =
5 dt 2 dt
d[NO] 4 d[O2 ] Passage Type
(c) Rate = (d) Rate =
dt 5 dt Paragraph for Questions 49 to 51: The reaction between A and B
is of first-order with respect to A and of zero order with respect to B.
43. Which of the following expressions are correct? The following data were obtained for four experiments:
1
(a) t1/2 (second order) = Experiment [A] (M) [B] (M) Initial rate (M min−1)
2k[A]0
I 0.1 0.1 2.0 × 10−2
2.303 1 II x 0.2 4.0 × 10−2
(b) t1/2 (first order) = log −
k n
(Continued)

Chapter-19.indd 646 8/5/2016 11:36:25 AM


Practice Exercise 647

(Continued) Paragraph for Questions 56 to 58: The progress of the reaction


Experiment [A] (M) [B] (M) Initial rate (M min−1) A  nB with time is represented in the following figure:

III 0.4 0.4 y


IV z 0.2 2.0 × 10−2 B

Concentration (mol L−1)


49. The value of x is
0.5
(a) 0.2 (b) 0.8 (c) 0.1 (d) 0.5
50. The value of y is
(a) 0.2 (b) 0.08 (c) 0.l (d) 0.5
0.3 A
51. The value of z is
(a) 0.2 (b) 0.8 (c) 0.1 (d) 0.5
Paragraph for Questions 52 to 55: Integrated rate law equations
0.1
quantitatively give concentration as a function of time and its
form depends on the order of the reaction The integrated rate
1 3 5 7
equations can be fitted with kinetic data to determine the order Time (h)
of a reaction. The integrated rate equations for zero, first and
second-order reactions are: 56. The value of n is
Zero-order: [A] = –kt + [A]0 (a) 1 (b) 2 (c) 3 (d) 1.5
kt
First-order: log [A] = − + log [A]0 57. The equilibrium constant K will be
2.303
(a) 2 (b) 1.2 (c) 0.5 (d) 6.67
1 1
Second-order: = kt + 58. The initial rate of conversion of A will be
[A] [A]0
(a) 0.1 mol L−1 h−1 (b) 0.2 mol L−1 h−1
These equations can also be used to calculate the half-life periods (c) 0.4 mol L−1 h−1 (d) 0.8 mol L−1 h−1
of different reactions, which give the time during which the con-
centration of a reactant is reduced to half of its initial concentra- Matrix-Match Type
tion, that is, at time t1/2; [A] = [A]0/2.
59. Match the reaction with its order/molecularity.
52. In the formation of sulphur trioxide by Contact process, Column I Column II
2SO2 + O2  2SO3 , the rate of reaction was measured as (a) RCl + H2O → ROH + HCl (p) Molecularity 1
3.0 × 10–4 mol L–1 s–1 in terms of oxygen. The rate of reaction
expressed in terms of sulphur trioxide is (b) COOH (q) Molecularity 3
CO + CO2 + H2O
(a) 3.0 × 10−4 mol L−1 s−1 (b) 6.0 × 10−4 mol L−1 s–1
COOH
(c) 1.5 × 10−4 mol L−1 s−1 (d) 4.5 × 10–4 mol L−1 s−1
(c) 2H2O2 → 2H2O + O2 (r) Molecularity 2
53. For a second-order reaction, the correct plot of t1/2 vs. 1/[A]0 is (d) 2NO + Cl2 → 2NOCl (s) Molecularity 4
(a) (b)
60. Match the order of reaction with their characteristics.
t1/2 t1/2 Column I Column II
(a) Zero order reactions (p) Unit of rate constant:
L mol–1 s–1
(b) First-order reactions (q) Unit of rate constant:
1/[A]0 1/[A]0
(c) (d) mol L–1 s–1
(c) Second-order reactions (r) Number of reactants is at
t1/2
least two
t1/2
(d) Pseudo unimolecular ln 2
reactions (s) t1/ 2 =
k
1/[A]0 1/[A]0 61. Match the quantity with its expression.
54. For a first-order reaction A → Products, the rate of reaction Column I Column II
at [A] = 0.2 mol L−1 is 1.0 × 10−2 mol L−1 min. The half-life of (a) Half-life of first-order reaction (p) Active mass
reaction is:
(a) 832 s (b) 440 s (c) 416 s (d) 14 s (b) Arrhenius equation (q) k = Ae − Ea /RT

55. The plot of [A] vs. t for zero-order reaction gives slope 0.693
(c) Molar concentration (r) t1/ 2 =
k k (d) Half-life period of zero order k
(a) − (b) (c) 2.303k (d) −k a
2.303 2.303 reaction (s)
2k

Chapter-19.indd 647 8/5/2016 11:36:30 AM


648 Chapter 19 Chemical Kinetics

62. Match the terms with their characteristics. Integer Type


Column I Column II
63. The rate of a reaction is expressed as: 3.65 × 10–3 mol L–1 s–1.
(a) Rate of reaction (p) May be fractional
The order of reaction is ___.
(b) Rate constant (q) Whole number
(c) Order of reaction (r) Independent of temp- 64. For a reaction: 2A → A2 the rate of reaction becomes 8 times
erature when the concentration of A changes from a to 3a. The order
(d) Molecularity of reaction (s) Increases with tempera- of reaction is ___.
ture 65. If the half-life of a reaction is inversely proportional to the
cube of initial concentration, the order of reaction is ___.

ANSWER KEY
Level I
1. (b) 2. (d) 3. (c) 4. (b) 5. (c) 6. (c)
7. (d) 8. (a) 9. (d) 10. (a) 11. (c) 12. (b)
13. (a) 14. (b) 15. (b) 16. (c) 17. (d) 18. (c)
19. (a) 20. (b) 21. (a) 22. (b) 23. (c) 24. (a)
25. (b) 26. (a) 27. (a) 28. (c) 29. (c) 30. (d)
31. (c) 32. (a) 33. (b) 34. (c) 35. (d) 36. (b)
37. (c) 38. (d) 39. (d) 40. (c)

Level II
41. (a), (b) 42. (a), (c) 43. (c), (d) 44. (b), (c) 45. (a), (c) 46. (a), (b)
47. (a), (d) 48. (a), (b) 49. (a) 50. (b) 51. (c) 52. (b)
53. (c) 54. (d) 55. (d) 56. (b) 57. (b) 58. (a)
59. (a) → r; (b) → p; (c) → r; (d) → q 60. (a) → q; (b) → s; (c) → p; (d) → r 61. (a) → r; (b) → q; (c) → p; (d) → s
62. (a) → s; (b) → s; (c) → p, r; (d) → q, r 63. (2) 64. (3) 65. (4)

HINTS AND EXPLANATIONS


Level I
Single Correct Choice Type Therefore, Eq. (1) − Eq. (2) gives the net rate of disappear-
ance of A
1. (b) A modest increase in temperature does not affect the
= 2k1[A ]2 − 2k2 [B]
activation energy and the minimum kinetic energy
of the molecules required for the reaction to take 4. (b) Order = 1 + 1/2 = 3/2 and molecularity = 2.
place.  Instead, on increasing the temperature the effec- Ea
tive collisions increase, that is, the fraction of the total 5. (c) Slope = = 5.42 × 103
2.303 R
collisions for which the energy is equal to or more than
activation energy increases. So, Ea = 5.42 × 103 × 2.303 × 8.314 J mol−1
ln 2 0.693 = 104 kJ mol−1
2. (d) For a first-order reaction, half-life(t1/2 ) = = 6. (c) The rate of the gaseous reaction: A(g) + B(g) → Product is
k k
Substituting the given value of k in the equation, we get given by expression
dx
0.693 0.693 −1 − = k[A][B]
0.0282 s = ⇒k = s = 24.574 s −1 ≈ 24.6 s −1 dt
k 0.0282
Concentration = n/V, so
3. (c) From the reactions, we have (Rate)initial = k (n/V) × (n/V)
Rate of disappearance of A is and (Rate)final = k (4n/V) × (4n/V)
d[A ] (Rate )initial 16
− = 2k1[A ]2 (1) (1) So, =
dt (Rate )final 1
Rate of formation of A from the reverse reaction is
7. (d) This is based on Arrhenius equation which is given as
d[B] 1 d[A]
− =+ = k2 [B] Ea
dt 2 dt log k = log A −
d[A] 2.303 RT
⇒ = 2k2 [B] (2)
dt So, plot of log k vs. 1/T has a slope −Ea / 2.303 R.

Chapter-19.indd 648 8/5/2016 11:36:34 AM


Hints and Explanations 649

8. (a) Let Rate = k[A]x[B]y Therefore, the percentage of reaction completed is


1.2 × 10−3 = k[0.05]x × [0.01]y (1) 2
× 100 = 40%
1.2 × 10−3 = k[0.05]x × [0.10]y (2) 5
Dividing Eq. (1) by Eq. (2), we get
14. (b) We know that k = A0e − Ea / RT . When T → ∞, we have k → Ae0
1 = (0.1)y ⇒ y = 0
Now, ⇒ k = A = 6.0 × 1014 s−1.
2.4 × 10−3 = k[0.10]x × [0.05]y (3) 15. (b) log t1/2 = k ⇒ t1/2 ∝ [C0]0. So, order of reaction = 1 and
Dividing Eq. (3) by Eq. (2) and substituting y = 0, we get
2.303
2 = (2)x ⇒ x = 1 t1/ 2 = log 2
k
0.693 0.693
9. (d) Unit of k = [conc]1−n time−1 where n is the order of 16. (c) We find that k = =
t1/ 2 100
reaction. Now,
10. (a) Rate = k[NO2]2  1  1  1
ln   ln   ln  
To find the rate constant solve the rate law with the given  x 0.693  x 0.693  x
k= ⇒ = ⇒ =
data: t 100 100 × 1.443 100 100 × 1
4.42 × 10–7 mol L–1 s–1 = k(6.54 × 10–4 mol L–1)2 0.693
4.42 × 10 −7 mol L−1 s −1  1
Therefore, ln   1 = ⇒
1
= e ⇒ x =
1
k= = 1.03 L mol–1 s–1  x
(6.54 × 10 −4 mol L−1)2 x e

The half-life of the system is found using the half-life 17. (d) R1 = k[A][B]2 = k[0.6][0.80]2. The reaction involved is
equation for a second-order reaction A + 2B → C + D
1
t1/ 2 = 0.6 − 0.2 0.8 − 0.4 0.2 0.2
k × (initial concentration of reactant )
0.4 0.4 0.2 0.2
1
= −1 −1
So, R2 = k(0.4)(0.4)2. Comparing the rates, we get
(1.03 L mol s )(6.54 × 10 −4 mol L−1)
R2 (0.4 )(0.4 )2 1
= =
= 1.48 × 103 s R1 (0.6 )(0.8 )2 6
n −1
t a 
11. (c) We know that 1 =  2  . 18. (c) For first-order reaction, half-life period is given by
t2  a1 
0.693 0.693
Substituting given values, we get t1/ 2 = or k =
k t1/ 2
n −1
200  0.5  Given that at T1 = 27°C = 300 K; t1/2 = 20 min. Therefore,
=  ⇒n=2
40  0.1 0.693
k1 = = 0.0346 min−1
20
12. (b) For a first-order reaction
At T2 = 77°C = 305 K; t1/2 = 5 min. Therefore,
0.693 0.693 0.693
t 0.5 = ⇒k = k2 = = 0.1386 min−1
k t 0.5 5
When the reaction is 75% complete, The activation energy is related to rate constants at two
2.303 [ a] 2.303 [a] different temperatures by the equation
t 0.75 = log = log
k [a − x ] k [a − 0.75a] TT k
Ea = 2.303 R × 1 2 × log 2
2.303 2.303 T2 − T1 k1
t 0.75 = log 4 = × t 0.5 × 0.603 = 2t 0.5
k 0.693 Substituting values in the equation, we get
13. (a) The time taken for completion of the whole reaction is 300 × 350 0.1386
Ea = 2.303 × 8.314 × × log
2.303  a  350 − 300 0.0346
t= log  0  = 19.147 × 2100 × log 4 = 24205.63 J mol−1
k  a − x 0
For 40% completion of the reaction, = 24.205 kJ mol−1
2.303  10 
t1 = log   = 1 h
k  6 19. (a) Using the expression
If the initial concentration, that is, a = 5 mol L−1, then k  Ea  T2 − T1 
lo g  2  =  ,
2.303  5   k1  2.303 R  T1 T2 
t2 = log 
k  5 − x 
Ea  40 − 0 
Given that t1 = t2 = 1 h. So, log(3) =
2.303 R  273 × 313 
 5  5  5 5 2.303 × 2 × 273 × 313
log   = log  or = or x = 2 ⇒ Ea =
 3  5 − x  3 5− x 40
log 3

Chapter-19.indd 649 8/5/2016 11:36:45 AM


650 Chapter 19 Chemical Kinetics

0.693 0.693 kT Ea  1 1
20. (b) k = = = 0.0693 log = −
t1/ 2 10 k283 2.303 R  283 T 
2.303  a  2.303 0.08 1.5 × 10 4 s −1 6000 J mol−1
 T − 283 
So, t = log   = log = 30 min log =

k a− x 0.0693 0.01 4.5 × 10 s 2.303 × 8.3 JK −1 mol−1  283 × T 
3 −1
21. (a) In a second-order reaction, with one reactant, the inte-
On solving, we get T = 297 K = 24°C.
grated rate law equation is given by
1 x  26. (a) More is the temperature coefficient, more is the activa-
k= 
t  a(a − x )  tion energy.
where a = initial concentration of reactant at time = 0; 27. (a) Extent of reaction = 30% and time taken = 40 min
x = number of moles of reactant reacted at time = t; For a first-order reaction,
a − x = concentration of reactant after time t. 2.303 a 2.303 1
t= log = log
Given that a = 0.2 mol L−1, t = 20 min. At 40% comple- k a− x k 1− ( x /a)
tion of reaction, 2.303  1 
40 min = log 
a− x =a−
40
a = 0.6a k  1− 0.3 
100 2.303 1
or x = a − 0.6a = 0.4 a or k = log = 8.92 × 10 −3 min−1
40 min 0.7
The rate constant may be obtained by substituting val- 2.303 log 2
ues in the above equation We know, for a first-order reaction, t1/ 2 =
k
1  0.4 a  1  2  2.303 log 2
k= = = 0.1667 L mol−1min−1 t1/ 2 = = 77.7 min
20  a(0.6a)  20  3 × 0.2  Then,
8.92 × 10 −3 min−1
At 75% completion of reaction,
28. (c) The expression for Rate = k [A]m[B]n
75
a− x =a− a = 0.25a or x = 0.75a Rate = x = k [0.1]m[0.1]n (1)
100 Rate = 4x = k [0.1]m[0.2]n (2)
Substituting values in integrated rate law equation, we Dividing Eq. (2) by Eq. (1), we get n = 2.
get Now, Rate = 16x = k[0. 4]m[0.2]n (3)
1 x  Dividing Eq. (2) by Eq. (3), we get m = 1.
k= 
t  a(a − x )  Therefore, order of the reaction = m + n = 3.

1  0.75a  1 3 3 29. (c) The Arrhenius equation is,


t= = × =6× = 90 min
0.1667  a(0.25a)  0.1667 a 0.2 k = Ae − Ea / RT
0.693 0.693 For the two temperatures, we get
22. (b) k = = = 4.62 × 10 −2 min−1
t1/ 2 15 k2 −Ea  1 1
log = −
k1 2.303 × 8.314  T2 T1 
23. (c) Rate1 = k [A]x
Substituting the given data, we get
Given that Rate2 = 8 × Rate1 = k[4A]x
Therefore, 8 = ( 4 ) x ⇒ x = 1.5. 4.5 × 107 −Ea  1 1 
log =  − 
1.5 × 10 7 2.303 × 8.314  373 323 
24. (a) All follow first-order kinetics. The rate constant is given
by Which gives Ea = 22012.7 J mol−1 or 22.13 kJ mol−1
2.303 a
k= log 30. (d) Rate = k [A]m so 2(Rate) = k[8A]m. Therefore, 2 = [8]m ⇒ m
t a− x = 1/3.
kt
= log a − log(a − x ) 31. (c) Conceptual
2.303
kt 32. (a) The rate of disappearance Cr2O72− is related to the rate
log(a − x ) = log a − (I) of appearance of Cr3+ by the expression
2.303
a d[Cr2O72 − ] 1 d[Cr 3+ ] d[Cr 3+ ]
Again, kt = 2.303 log (II) − =+ ⇒+ = 2 × 2.4 × 10 −4
a− x dt 2 dt dt
The graph III is for a first-order reaction as t1/2 is inde- = 4.8 × 10 −4 mol L−1 s −1
pendent of concentration.
33. (b) The reaction follows pseudo first-order kinetics.
25. (b) Given that k10 = k283 = 4.5 × 103 s −1, k T = 1.5 × 10 4 s −1,
34. (c) We know that t1/ 2 ∝ [A ]n −1 where n is the order of the
T = ? Also
reaction. When n = 2, t1/2 ∝ [A]−1 so a plot of t1/2 versus
Ea = 60 kJ mol−1 = 60000 J mol−1. Using Arrhenius equa- 1/[A] or 1/conc. represents a straight line. Therefore, the
tion, one can write order is 2.

Chapter-19.indd 650 8/5/2016 11:36:55 AM


Hints and Explanations 651

35. (d) On increasing the temperature, the rate of the reaction 1 d[NH3 ] 1 d[O2 ] 1 d[NO] 1 d[H2O]
as well as the rate constant increases because num- − =− = =
4 dt 5 dt 4 dt 6 dt
ber of effective collisions increases and quantitatively,
the effect has been explained by Arrhenius equation d[NH3 ] 4 d[O2 ] d[NH3 ] d[NO]
or − =− or − =
k = Ae − Ea / RT . dt 5 dt dt dt

36. (b) Half-life is independent to the initial concentration of the


reactant. 43. (c), (d)
[A ]0
37. (c) In the absence of catalyst, suppose rate constant is k. Then For zero order reaction t1/2 = ,
2k
Ea
log k = log A − (1) 0.693
2.303 RT For first order reaction t1/ 2 =
k
In the presence of catalyst, suppose rate constant is k′. 1
Now, the activation energy = Ea − 2 (if Ea is in kcal mol−1). For second order reaction, t1/ 2 = .
k[A]0
Therefore,
E −2 44. (b), (c) RCl + NaOH → ROH + NaCl
log k ′ = log A − a The rate expression is Rate = k[RCl]. So, rate of reaction is
2.303 RT
unaffected by doubling concentration of NaOH, because it
Ea 2 kcal mol−1 does not import in rate law equation. The rate of reaction is
= log A − + (2)
2.303 RT 2.303 RT increased by increase in temperature.
Subtracting Eq. (1) from Eq. (2), we get 1 21  48 
n −1
−1 45. (a), (c) t1/ 2 ∝ ⇒ =  ⇒n=0
2 kcal mol p n −1 11  92 
log k ′ − log k =
2.303 RT
p p 11
Therefore, For zero order, t1/ 2 = ⇒k = = = 0.114 s −1
2k 2 t1/ 2 2 × 48
k′ 2 kcal mol−1
log = 46. (a),(b) We have, k = 0.0606 M−1 s−1
k 2.303 (2 × 10 −3 kcal K −1 mol−1)(300 K )
[B0] = [A0] = 3.5 × 10−1 M
= 1.4474 (as R = 2 × 10 −3 kcalK −1 mol−1) 1 1
− = kt
k′ [A ] [A 0 ]
= antilog 1.4474 = 28 ⇒ k ′ = 28 k
k 1 1
− = 0.0606 × 300 = 18.18
1 [A ] [3.5 × 10 −3 ]
38. (d) We know that t1/ 2 ∝ . So,
an −1 or [A] = 3.29 × 10−3 M = [B].
1− n
 1
16 =   ⇒ 4 2 = 4 n −1 ⇒ n = 3 47. (a), (d) Catalyst is not part of the overall reaction, it does par-
 4 ticipate in the reaction by changing the mechanism of the
39. (d) Larger the Ea of reaction greater is the influence of reaction.
change in temperature on rate constant. Negative catalyst or poison decreases the rate of reaction due
40. (c) Given that to increase in activation energy.

r = k[A ] x [B] y (1) 48. (a), (b) Given that 100 ∝ (0.1)1− n and 50 ∝ (0.025)1− n .
Dividing, we get
8r = k[2A] x [2B] y (2) log 2
2 = 41− n ⇒ log 2 = (1− n)log 4 ⇒ 1− n =
x
2r = k[A ] [2B] y
(3) log 4
log 2 log 2 1 1
On dividing Eq. (1) by Eq. (3), we get y = 1; and on divid- ⇒ n = 1− = 1− = 1− =
ing Eq. (2) by Eq. (3), we get x = 2. So, the rate equation is log 4 2 log 2 2 2
r = k [A]2[B].
We have 100 ∝ (0.1)1/ 2 and t1/ 2 ∝ (1)1/ 2 . Dividing these two, we
Level II get
1/ 2
Multiple Correct Choice Type t1/ 2  1 
=  or t1/ 2 = 100 10 min
100  0.1
− E a /RT
41. (a), (b) According to Arrhenius equation, k = Ae .
Therefore, Passage Type
Ea 2700 × 1
log k = log A − ⇒ log k = 2.1× 1010 − 49. (a) Since the reaction is of zero order with respect to B,
2.303RT 2.303RT hence any change in the concentration of B has no effect
42. (a), (c) For the reaction 4NH3 (g) + 5SO2 → 4NO(g) + 6H2O(g), on the rate of reaction.

Chapter-19.indd 651 8/5/2016 11:37:06 AM


652 Chapter 19 Chemical Kinetics

From experiment I, 2.0 × 10−2 M min−1 = k × (0.1)1 Matrix-Match Type


2.0 × 10 −2 M min−1 59. (a) → r; (b) → p; (c) → r; (d) → q
So, k= = 0.2 min−1
0.1M 60. (a) → q; (b) → s; (c) → p; (d) → r
For experiment II, In general, units of rate constant = 1/(time) × 1/(conc.)n−1
4.0 × 10 −2 M min−1 = 0.2 min−1 [A ] When n = 0, units of k = mol L−1 s−1 and when n = 2, units of
k = L mol−1 s−1
4.0 × 10 −2 M min−1 For first-order reaction, t1/2 = 0.693/k or ln(2/k)
[A ] = = 0.2 M
0.2 min−1 For pseudo unimolecular reaction, there has to be atleast
two reactants and the concentration of one of the reactant
50. (b) For experiment III, Rate = 0.2 min−1 × (0.4 M)1 = 0.08 M is taken in excess.
min−1
61. (a) → r; (b) → q; (c) → p; (d) → s
51. (c) For experiment IV, 2.0 × 10 −2 M min−1 = 0.2 min−1× [A ]
62. (a) → s; (b) → s; (c) → p, r; (d) → q, r
2.0 × 10 −2 M min−1 Rate of a reaction increases with increase in temperature as
or [A ] = = 0.1 M
0.2 min−1 the number of effective collisions increases.
52. (b) For the reaction 2SO2 + O2  2SO3 Rate constant also increases with increase in temperature,
according to Arrhenius equation k = Ae − Ea / RT .
1 d[SO2 ] d[O2 ] 1 d[SO3 ]
− =− = Order of a reaction is independent of temperature and can
2 dt dt 2 dt be fractional, for example, for the reaction H2 + Br2 → HBr,
d[SO3 ] d[O2 ] the order is 3/2.
or =2 = 2 × 3 × 10 −4 = 6 × 10 −4
dt dt Molecularity of a reaction has to be a whole number only, it
53. (c) Because t1/2 ∝ [A]1−n which means plot of t1/2 vs. 1/A as cannot be fractional and is independent of temperature.
n = 2 will be a straight line.
Integer Type
54. (d) The rate equation is Rate = k[A]
63. (2) Unit of rate constant is calculated as k[conc]2
1 × 10−2 = k(0.2) ⇒ k = 5 × 10−2 min−1
(for second-order)
So, now t1/2 = 0.693/k = 0.693/(5 × 10−2) = 13.86 or 14 s.
3.65 × 10 −3 molL−1 s −1 = K mol2 L−2
55. (d) For zero order reaction, kt = [A]0 − [A]. Comparing it with 3.65 × 10 −3 molL−1 s −1
a straight line equation y = mx + c gives slope = −k. ⇒ K=
K mol2 L−2
= 3.65 × 10 −3 L mol−1 s −1
Slope = −k
[A] So, it follows second-order reaction.

t
64. (3) The reaction is 2A → A2. The expression for rate of reac-
tion is Rate = k[A].
56. (b) According to the figure, in the given time of 4 h (1 to 5),
concentration of A falls from 0.5 M to 0.3 M, while in Rate = k [X]a (1)
the same time concentration of B increases from 0.2 M
27(Rate) = k [3X]a (2)
to 0.6 M.
Decreases in concentration of A in 4 h = 0.5 − 0.3 = 0.2 M. Dividing Eq. (1) by Eq. (2), we get
Increases in concentration of B in 4 h 0.6 − 0.2 = 0.4 M
a
Thus increase in concentration of B in a given time is 1  1
=
twice the decrease in concentration of A. Thus n = 2. 27  3 
[B]2eq [0.6]2 So, a = 3 or order = 3.
57. (b) K = = = 1.2
[A]eq 0.3
1 1
58. (a) Initial rate of conversion of A = change in concentration 65. (4) In general, t1/ 2 ∝ . Given that t1/ 2 ∝ , this means
of A during 1 h [A]n0 −1 [A]30
0.6 − 0.5 n − 1 = 3 or n = 4.
= = 0.1 mol L−1 h−1
1

Chapter-19.indd 652 8/5/2016 11:37:12 AM


Solved JEE 2016 Questions 653

SOLVED JEE 2016 QUESTIONS


JEE Main 2016 On adding Eq.(1) and Eq. (2), we get the required equation
for overall reaction
1. Decomposition of H2O2 follows a first order reaction. In fifty
minutes the concentration of H2O2 decreases from 0.5 to 0.125 O3 (g) + O• (g) → 2O2 (g)
M in one such decomposition. When the concentration of H2O2
reaches 0.05 M, the rate of formation of O2 will be Therefore, koverall = k1 × k2
(a) 1.34 × 10–2 mol min–1 (b) 6.93 × 10–2 mol min–1 = 5.2 × 109 × 2.6 × 1010
–4
(c) 6.93 × 10 mol min –1 (d) 2.66 L min–1 at STP = 1.4 × 1020 L mol−1s−1
(Offline)
3. The rate law for the reaction below is given by the expression
Solution k[A][B]
1
(c) In 50 min, conc. of H2O2 = of initial conc.
4 A + B → Product
Therefore, 2 × t1/2 = 50 min ⇒t1/2 = 25 min If the concentration of B is increased from 0.1 to 0.3 mol, keep-
ing the value of A at 0.1 mol, the rate constant will be:
0.693
As t1/ 2 = ⇒ k = 0.693 (a) 3k (b) 9k (c) k/3 (d) k
k 25 (Online)
Rate = k[H2O2] (given that it is a first order reaction) Solution
0.693 (d) The value of rate constant k depends on the particular
Rate (H2O2) = × 0.05
25 reaction being studied as well as the temperature at which
= 1.386 × 10−3 mol min−1 reaction occurs. It does not depend on the concentration
2H2O2 → 2H2O + O2 of the reactants.

1 d[H2O2 ] 1 d[H2O] d[O2 ] JEE Advanced 2016


− =+ =
2 dt 2 dt dt
1. According to the Arrhenius equation,
1 d[H2O2 ] d[O2 ]
− = (a) a high activation energy usually implies a fast reaction.
2 dt dt (b) rate constant increases with increase in temperature. This
d[O2 ] 1 is due to a greater number of collisions whose energy
= × 1.386 × 10 −3 = 6.93 × 10−4 mol min−1 exceeds the activation energy.
dt 2
(c) higher the magnitude of activation energy, stronger is the
2. The reaction of ozone with oxygen atoms in the presence of temperature dependence of the rate constant.
chlorine atoms can occur by a two step process shown below: (d) the pre-exponential factor is a measure of the rate at which
collisions occur, irrespective of their energy.
O3(g) + Cl•(g) → O2(g) + ClO•(g); k1 = 5.2 × 109 L mol−1 s−1 (1)
ClO•(g) + O•(g) → O2(g) + Cl•(g); k2 = 2.6 × 1010 L mol−1 s−1 (2) Solution
(b), (c), (d) According to Arrhenius equation
The closest rate constant for the overall reaction O3(g) + O•(g)
→ 2O2(g) is: k = Ae − Ea / RT
(a) 1.4 × 1020 L mol−1 s−1 (b) 3.1 × 1010 L mol−1s−1 In this equation, k is the rate constant for the reaction, A is the
(c) 5.2 × 109 L mol−1 s−1 (d) 2.6 × 1010 L mol−1 s−1 proportionality constant, known as Arrhenius factor or pre-
(Online) exponential factor (it is specific to a reaction), R is the ideal gas
Solution constant in joules per mole kelvin, T is the temperature (K) and
Ea is the activation energy for the reaction.
(a) We have
From the Arrhenius equation, we can see that the statements
O3 (g) + Cl• (g) → O2 (g) + ClO• (g) (1)
• • •
given in options (b), (c), and (d) are correct. However, high acti-
ClO (g) + O (g) → O2 (g) + Cl (g) (2) vation energy (Ea) means slow (and not fast) rate of reaction.

Chapter-19.indd 653 8/5/2016 11:37:15 AM


Chapter-19.indd 654 8/5/2016 11:37:15 AM
20 Surface Chemistry

Questions Distribution in JEE (Main and Advanced)

3
No. of Question

JEE (Main)
2
JEE (Adv)

0
2016 2015 2014 2013 2012 2011 2010 2009 2008 2007

Concept Distribution in JEE (Main and Advanced)

Topics Covered
Year
JEE (Main) JEE (Advanced)
2007 Properties, Classification and Purification of Colloids
2008 Protection of Colloids Properties, Classification and Purification of Colloids
2009 Adsorption Coagulation or Precipitation of Colloids
2011 Adsorption
Properties, Classification and Purification of Colloids,
2012 Adsorption Isotherms
Adsorption Isotherms
2013 Properties, Classification and Purification of Colloids Adsorption
Adsorption, Properties, Classification and Purification of
2015
Colloids

2016 Adsorption Isotherms, Adsorption, Protection of Colloids

Chapter-20.indd 655 8/3/2016 3:08:12 PM


656 Chapter 20 Surface Chemistry

SUMMARY
1. The phenomenon of assimilation of higher concentration of any species at the surface of a solid or liquid as compared to that present
in the bulk of the material is known as adsorption.
2. The solid or liquid on whose surface gas or liquid is adsorbed is called adsorbent or substrate, and the gas or solution which is held
to this surface is called adsorbate.
3. The process of removal of an adsorbed substance from the surface is known as desorption. It occurs when the concentration (or
pressure) of the substance in bulk phase is lowered.
4. The distinguishing features of absorption and adsorption are:

Absorption Adsorption
Absorption is a bulk phenomenon in which the concentration The phenomenon of assimilation of higher concentration of any
of molecular species is uniformly distributed throughout the species at the surface of a solid or liquid as compared to that
body of the solid or liquid. present in the bulk of the material.
Slower process and the attainment of equilibrium takes longer Faster process and attainment of equilibrium takes shorter time.
time.
For example, adsorbents (charcoal or a mixture of products)
For example, ammonia is absorbed by water, whereas it is
used in the gas mask adsorb these gases and prevent their
adsorbed by charcoal.
inhalation.

5. Mechanism of adsorption
(a) The phenomenon of adsorption occurs due to the presence of unbalanced or residual forces at the solid or liquid surface. As a
result of these residual forces, the surface of the solid or liquid is in a state of strain and has a tendency to attract and retain mol-
ecules of another species, thus creating an excessive concentration at the surface.
(b) Adsorption reduces these residual forces, and this result in a decrease in the surface energy which appears in the form of heat.
This is known as heat of adsorption and the process of adsorption is an exothermic process.
6. Adsorption can be classified on the basis of the forces of attraction between the adsorbent and the adsorbate, into two types:
(a) Physisorption, also known as physical adsorption or van der Waals adsorption.
(b) Chemisorption, also called chemical adsorption or activated adsorption.
7. The distinguishing features of two types of adsorption are:

Physisorption Chemisorption
Forces of attraction involved between adsorbent and adsorbate Forces of attraction involved between adsorbent and adsorb-
are weak, long-range van der Waals forces. ate are strong chemical bonds.
No formation of a surface complex. Formation of a surface complex between adsorbate and
adsorbent.
Reversible in nature, as the gas adsorbed can be recovered back Irreversible in nature because it involves formation of chemi-
by lowering pressure or raising the temperature. cal bonds between adsorbent and adsorbate.
Occurs at low temperatures due to low activation energy (∼5 kJ). Occurs at high temperature due to high activation energy.
Magnitude of adsorption decreases with increase in tempera- Magnitude of adsorption increases with increase in tempera-
ture. ture.
Heat (enthalpy) of adsorption is low (20–40 kJ). Heat (enthalpy) of adsorption is high (80–240 kJ mol−1).
Adsorption is generally multilayered. Adsorption is monolayered.

8. Factors affecting adsorption of gases on solids


The extent of physisorption and chemisorption depends on the nature of adsorbent.
(a) At a given temperature and pressure, greater the surface area per unit mass of the adsorbent, greater is the adsorption on its surface.
(b) Easily liquefiable gases, that is, gases with high critical temperatures, are more readily adsorbed because van der Waals forces are
stronger near the critical temperature.
9. Freundlich and Langmuir adsorption isotherms
The curve showing the variation in extent of adsorption with pressure at constant temperature is known as adsorption isotherm.
(a) Freundlich adsorption isotherm
Freundlich gave an empirical expression representing the relationship between extent of adsorption of a gas on a solid and
pressure at a constant temperature.
x
= kp1/n
m

Chapter-20.indd 656 8/3/2016 3:08:12 PM


Summary 657

where x is the mass of the adsorbate and m is the mass of the adsorbent at pressure p; k and n are constants depending upon the
nature of adsorbate and adsorbent. The value of n is usually greater than 1. Taking logarithm of the equation, we have
 x  1
log   = log k +   log p
 m  n
A plot of log (x/m) vs. log p gives a straight line with a slope of (1/n) and intercept of log k as shown below.
log(x/m)

slope = 1/n

log k
log p

This plot can be used to determine the value of k and n graphically.


(b) Langmuir adsorption isotherm
Langmuir gave an expression to determine the extent of adsorption depending on variation in pressure, based on the assump-
tions that:
(i) Adsorption surface is homogeneous and has fixed number of equivalent adsorption sites, capable of adsorbing one gas
molecule each.
(ii) The rate of desorption is proportional to the concentration of adsorbed molecules, that is, to the fraction q of the surface
already covered by gas molecules. At equilibrium,
Rate of adsorption = Rate of desorption
ka p(1− q ) = kdq
where ka and kd are constants of adsorption and desorption and q is the fraction of surface already covered with gas
molecules.
kp
or q= 1
1+ k1p

where k1= ka/kd which is a constant known as distribution constant. Also q is proportional to the amount of gas adsorbed (x)
per unit mass of the adsorbent (m), so
x
= k2q
m
where k2 is another constant. Substituting the value of q from the above equation, we have
x k1k2 p
=
m 1+ k1p
This is known as the Langmuir equation. Taking the reciprocal of Langmuir equation, we have
1 1+ k1p 1 1
= = +
( x /m) k1k2 p k1k2 p k2
p 1 p
= +
( x /m) k1k2 k2
A plot of p/(x/m) versus p gives a straight line with a slope of (1/k2) and intercept of (1/k1k2) from which the values of k1 and
k2 can be calculated.
9. Adsorption from solutions
(a) The relationship between the adsorption of solutes from solution and the concentration of the solution can be expressed in
terms of the Freundlich’s equation as:
x
= kC 1/n
m
where C is the constant depending upon the nature of solute and adsorbent.
Taking the logarithm of the equation, we get
 x  1
log   = log k +   log C
 m  n

Chapter-20.indd 657 8/3/2016 3:08:15 PM


658 Chapter 20 Surface Chemistry

(b) Factors affecting adsorption of solutes from solution by solids are:


(i) The extent of adsorption depends on the nature of the adsorbent as well as adsorbate. This extent is usually greater for
solutes with high molecular mass.
(ii) Adsorption of a solute decreases with rise in temperature due to increase in the kinetic energy of solute particles, thereby
causing these particles to leave the surface.
(iii) Adsorption of solutes from a solution increases with increase in the concentration of solution.
(iv) The extent of adsorption increases with the increase in surface area of the adsorbent.
(c) When the solute is adsorbed by an adsorbent, it is called positive adsorption; whereas when the solvent is taken up by the
adsorbent, it is called negative adsorption.
10. Catalysis
(a) Catalyst is defined as a substance which alters the rate of a chemical reaction without appearing in the end product of the
reaction.
(b) Substances, which when present in small amounts, promote the activity of the catalyst are called promoters. On the other hand
substances that deactivate the catalyst are called anticatalysts or poisons.
(c) There are two types of catalysts based on the activity:
(i) Positive catalyst or accelerator: The catalyst changes the speed of the reaction by making available a path other than
the one which is followed by the reaction in the absence of the catalyst. If path is such that in the absence of the catalyst,
molecules with lower energy can participate in the reaction, the speed of the reaction is increased. Such a catalyst which
accelerates the rate of a reaction is called positive catalyst or accelerator. The example is as follows:
Fe
N2 (g) + 3H2 (g) 
→ 2NH3 (g)

(ii) Negative catalysts or inhibitors: There are certain substances which, when added to the reaction mixture, retard its reac-
tion rate instead of increasing it. These are called negative catalysts or inhibitors. The examples of this type are antifreezes
like glycerol which retards rusting of the machine.
(d) Types of catalytic reactions
The catalysis process may be classified as follows:
(i) Homogeneous catalysis: In this catalytic reaction, the catalyst is in the same phase as the reactants. For example,
NO(g)
2CO(g) + O2 (g) → 2CO2 (g)
(ii) Heterogeneous catalysis: In this catalytic reaction, the catalyst is in a different phase from the reactants. The example is
as follows where the reactants are gases whereas the catalyst is a solid.
Ni
CH2 CH2 + H2 
→ CH3CH3

(e) Theory of heterogeneous catalysis


(i) Intermediate compound theory: According to the theory, the catalyst forms a reactive and unstable intermediate with a
reactant. This intermediate readily reacts with the other reactant to form the product and the catalyst is obtained back in
its original form.
(ii) Adsorption theory: According to this theory, the catalyst forms a reactive and unstable intermediate with a reactant. This
intermediate readily reacts with the other reactant to form the product and the catalyst is obtained back in its original form.
(f) Characteristics of catalysts
(i) Remains unchanged in its composition and amount at the end of the reaction.
(ii) Sufficient in a small amount to promote a chemical reaction except in certain homogeneous reactions.
(iii) Does not affect the thermodynamic parameters of a chemical reaction.
(g) The ability of a catalyst to affect the rate of a reaction is known as its activity.

Tip The metals of Groups 5 to 11 show an increasing order of catalytic activity in hydrogenation reaction.

(h) The ability of a catalyst to affect the rate of certain reactions and prevent the other side reactions is known as selectivity of the
catalyst. This is also known as specificity of the catalyst, which implies that the use of a catalyst is specific to a reaction.

Tip Zeolites are can act as catalysts for reactions in which the size of reactants and/or products selectively fit into the pores and
cavities of the zeolite. This is known as shape-selective catalysis.

11. Enzyme catalysis and its mechanism


(a) The reactions taking place in living cells are mediated by biological catalyst called enzymes.
(b) The enzyme and the substrate combine to form an enzyme–substrate complex. Formation of the complex often induces a
conformational change in the enzyme that allows it to bind the substrate more effectively. This is called an induced fit.
Enzyme+Substrate ↔ Enzyme − Substrate complex ↔ Enzyme+Product

Chapter-20.indd 658 8/3/2016 3:08:15 PM


Summary 659

Products
Substrate

∗ ∗

Enzyme Reaction complex Enzyme

(c) Lock-and-key hypothesis


Enzyme specificity is due to the particular shape of a small part of the enzyme, called active site, which exactly fits a
complementary-shaped part of the substrate. This interaction is analogous to a lock and key; the substrate is the key and the
enzyme is the lock.
12. Colloidal state
(a) A colloid is a broad category of mixtures in which one phase is suspended in the other. The word colloid means a dispersion in
which the dispersed particles are larger than the solute ions.
(b) The size of colloidal particles ranges from a lower limit of about 1 nm (10−7 cm) to an upper limit of about 1000 nm (10−4 cm).
13. Distinction among true solutions, colloids and suspensions
Suspension Colloidal solution True solution
Heterogeneous mixture in which solid Heterogeneous mixture but the colloidal Homogeneous mixture of two or more
particle settle down at the bottom of particles cannot be seen with the naked substrates in a solvent.
the solution. eye.
The particle size of one or more compo- The diameter of the colloidal particle is Solute particles is less than 1 nm (about
nents is greater than 1000 nm. between 1 and 1000 nm (about 500 nm). 0.5 nm).

14. Difference between lyophilic and lyophobic colloids

Lyophilic colloids Lyophobic colloids


Colloidal solutions in which the dispersed particles have Colloidal solutions in which the dispersed phase has a weak inter-
strong affinity for the dispersion medium. action with the dispersion medium are called lyophobic sols.
Colloidal particles pass readily into the dispersion medium The dispersed phase does not like to get dissolved in dispersion
by simple mixing. medium so these require special methods for their preparation.
If the dispersed phase is isolated by removal of dispersion The addition of dispersion medium to the precipitated colloidal
medium, it is possible to reconstitute the colloidal solution particles does not reconstitute the colloidal sol again; hence
simply by adding the dispersion medium again. They are, these are called irreversible sols.
therefore, called reversible sols.
If the dispersion medium is water, the colloidal solution is If the dispersion medium is water, the colloidal solution is called
called hydrophilic sol. hydrophobic sol.

15. Multimolecular, macromolecular and associated colloids (micelles)

Type of colloids Definition Example


Multimolecular The particles (atoms or small molecules) of the dispersed phase Sol of sulphur
aggregate to form particles of colloidal size (diameter < 1 nm).
Macromolecular If the individual particles of the dispersed phase have dimensions sim- Proteins, enzymes, cellulose,
ilar to colloidal size, the dispersion is called macromolecular colloid. rubber
Associated In these colloids, the dispersed phase is made up of particles that Soap or detergent in water
behave as regular electrolytes at lower concentrations but aggregate
at higher concentrations to form particles of colloidal dimension. The
aggregated molecules, thus, formed are called micelles.

Chapter-20.indd 659 8/3/2016 3:08:16 PM


660 Chapter 20 Surface Chemistry

Tip Two important characteristics of micelle formation are Kraft temperature (TK), which is the temperature above which micelle
formation takes place and critical micelle concentration (CMC), which is the concentration above which micelle formation takes
place.

16. Methods of preparation of colloids


(a) Chemical methods
In this method, the colloidal molecules or aggregates are formed by means of chemical reactions such as decomposition,
hydrolysis, oxidation or reduction. Some examples of each type are:
Double decomposition
As2O3 + 3H2S → As2S3 + 3H2O
Yellow sol
Hydrolysis
FeCl3 + 3H2O → Fe(OH)3 + 3HCl
Oxidation
SO2 + 2H2S → 3S(sol) + 3H2O
Reduction
2AuCl3 + 3SnCl2 → 3SnCl4 + 2Au
(b) Electrical disintegration or Bredig’s arc method
The metal that needs to be dispersed is made into two electrodes and dipped in the dispersion medium. An electric arc is struck
between the electrodes and the heat generated by the arc vapourizes the metal. These vapours condense when they come in
contact with dispersion medium cooled in an ice-bath and form colloidal particles that get dispersed to form a sol.
(c) Peptization
This is a process wherein a freshly generated precipitate is converted into a colloidal sol by shaking it in a dispersion medium in
the presence of an electrolyte which is called the peptizing agent. For example, a dark red coloured sol of Fe(OH)3 is obtained
when freshly prepared precipitate is treated with small amount of FeCl3 solution. The precipitate adsorbs the Fe3+ ions from the
electrolyte on its surface and develops positive and negative charges. This causes the precipitate to disintegrate into particles
of colloidal size.
17. Mechanical properties of colloids
(a) Brownian motion
It is the random motion of colloidal particles observed in all colloidal solutions that is independent of the nature of colloidal
particles.
(b) Tyndall effect
A beam of light, becomes clearly visible and sharply outlined when it is passed through a colloidal dispersion. This phenome-
non is known as the Tyndall effect.

Tip The conditions necessary for observation of Tyndall effect are:


(a) the diameter of the particles should be of the order of wavelength of light used and
(b) the refractive indices of the dispersed phase and the dispersion medium should be vastly different.

18. Coagulation and flocculation


(a) If the charge on colloidal particles is removed or neutralized in some manner, the particles approach each other and aggregate
to form a precipitate. This process of setting of colloidal particles is known as coagulation.
(b) Coagulation of lyophobic sols can be carried out by:
(i) Dialysis: By prolonged dialysis all the ions present in the colloidal solution are removed, the colloidal particles settle as a
precipitate.
(ii) Mixing oppositely charged colloidal solutions: When two oppositely charged sols are mixed in equal proportions, their
charges get neutralized and coagulation takes place. This process is known as mutual coagulation.
(iii) Electrophoresis: In this process, the charged particles moving towards oppositely charged electrodes may get discharged
and precipitate out.
(iv) Boiling: On boiling, the number of collisions between the dispersion medium and the charged layer on the colloidal sur-
face increases. This reduces the charge on the particles leading them to coagulate.
(v) Addition of electrolytes
(c) The minimum concentration of electrolyte (in millimoles) required to bring about the coagulation of one litre of colloidal solu-
tion in two hours is called its coagulation or flocculation value. The reciprocal of coagulation value is the coagulating power
of the coagulating or flocculating electrolyte.
(d) Hardy–Schultz rule
It states that greater the valence of the flocculating ion added, greater is its power to cause precipitation.

Tip Order of coagulation for cations: Al3+  Mg2+ > Na+


Order of coagulation for anions: PO34− > SO24− > Cl−

Chapter-20.indd 660 8/3/2016 3:08:17 PM


Solved Examples 661

(e) Protection of colloids


Lyophilic sols can be added to lyophobic sols to protect them from coagulating in presence of added electrolyte. The lyophilic
sol particles form a protective layer around the lyophobic colloidal particles thus protecting them from electrolytes. Such
lyophilic colloids are known as protective colloids.
(f) Gold number
It is defined as the minimum number of milligrams of a protective colloid which just prevents the change in colour of 10 mL of
a red gold sol when 1 mL of a 10% solution of sodium chloride is added to it. The smaller the value of gold number, the greater
is the protecting power of the protective colloid. Therefore, the reciprocal of gold number is a measure of the protective power
of a colloid. For example, Gelatin with lowest gold number (0.005) is considered the best protective colloid.
19. Emulsions and their characteristics
(a) An emulsion is a mixture of two immiscible or partially miscible liquids that are shaken together to form a liquid–liquid
dispersion.
(b) Two types of emulsions: (i) oil or fat dispersed in water (oil-in-water), for example moisturizing cream and (ii) water dispersed in
oil (water-in-oil).
(c) Emulsions, particularly oil-in-water type, are unstable and substances called emulsifying agents or emulsifiers are added
during the preparation of the emulsions to stabilize them.

Tip Milk is an unusual emulsion that is stable because it contains the naturally occurring protein called casein as
emulsifier.

(d) Emulsions also show mechanical properties of colloidal solutions such as Brownian motion and Tyndall effect.
(e) Emulsions can be precipitated by addition of electrolytes, since the particles are negatively charged.

SOLVED EXAMPLES
Adsorption Solution
(c) The phenomenon of adsorption occurs due to the pres-
1. Which of the following statements is incorrect regarding
ence of unbalanced or residual forces at the solid or liq-
physisorption?
uid surface. As a result of these residual forces, the surface
(a) It occurs because of the van der Waals forces.
of the solid or liquid is in a state of strain and has a ten-
(b) More easily liquefiable gases are adsorbed readily.
dency to attract and retain molecules of another species,
(c) Under high pressure, it results into multimolecular layer on
thus creating an excessive concentration at the surface.
adsorbent surface.
Adsorption reduces these residual forces, and this results
(d) Enthalpy of adsorption (∆Hadsorption) is low and positive.
in a decrease in the surface energy which appears in the
(AIEEE 2009)
form of heat.
Solution
(d) 3. Which one of the following characteristics is not correct for
(a) When a gas is held on the surface of solid by van der Waals physical adsorption?
forces without resulting in the formation of any chemical (a) Adsorption on solids is reversible.
bond between the adsorbate and adsorbant, it is called (b) Adsorption increases with increase in temperature.
physisorption. (c) Adsorption is spontaneous.
(b) Easily liquefiable gases, that is, gases having high critical (d) Both enthalpy and entropy of adsorption are negative.
temperature, are adsorbed more strongly because they Solution
have stronger van der Waals forces of attraction.
(c) As the layers of the gas can be adsorbed one over the other (b) Because physical adsorption decreases with increase in
by van der Waals forces, multimolecular layers are formed temperature.
under high pressure. 4. Rate of physisorption increases with
(d) As physisorption involves only van der Waals forces of attrac- (a) decrease in temperature.
tion and no chemical change, the process is exothermic but (b) increase in temperature.
the enthalpy of adsorption is quite low (20–40 kJ mol−1). (c) decrease in pressure.
2. The following statements relate to the adsorption of gases on a (d) decrease in surface area.
solid surface. Identify the incorrect statement among them. Solution
(a) Enthalpy of adsorption is negative.
(b) Entropy of adsorption is negative. (a) According to physisorption,
(c) On adsorption, the residual forces on the surface are Absorbate + Absorbent  Gas/solid + Heat
increased.
(d) On adsorption decrease in surface energy appears as heat. According to Le Chatelier’s principle, decrease in tem-
(JEE Main Online 2015) perature will shift the equilibrium to right or rate of

Chapter-20.indd 661 8/3/2016 3:08:17 PM


662 Chapter 20 Surface Chemistry

physisorption will increase. On increasing the pressure,


the equilibrium will shift in the direction in which there
x
is decrease in number of moles (i.e., in forward direction). = kp0
x m
Adsorption increases with increase in surface area. = kp1/n
m
x
5. Which of the following is true with respect to adsorption? m
(a) ∆G < 0, ∆S > 0, ∆H < 0 (b) ∆G < 0, ∆S < 0, ∆H < 0 T = constant
(c) ∆G > 0, ∆S > 0, ∆H < 0 (d) ∆G < 0, ∆S < 0, ∆H > 0 x
= kp1
m
Solution
(b) For successful adsorption, ∆G = −ve (spontaneous), p
∆S = –ve and ∆H = –ve (exothermic). From the figure, we have
x/m ∝ p1 at low pressure, means straight line at low
6. Adsorption is a pressure
(a) colligative property. x/m ∝ p0 at high pressure means x/m becomes independ-
(b) oxidation process. ent of pressure and
(c) reduction process. 1n
At moderate pressure x/m ∝ p
(d) surface phenomenon.
9. The plot of log (x/m) (along y-axis) vs. log C (along x-axis) in the
Solution Freundlich adsorption isotherm is a horizontal line parallel to
(d) Adsorption is a surface phenomenon, whereas absorp- x-axis when
tion is a bulk phenomenon. (a) n = 0. (b) n = 1.
(c) n = ∞. (d) Such a plot is impossible.
Adsorption Isotherms
Solution
7. In Langmuir’s model of a gas on a solid surface x
(a) the rate of dissociation of adsorbed molecules from the (c) = kC 1/ n so for horizontal line, slope 1/n = 0 or n = ∞.
m
surface does not depend on the surface covered.
(b) the adsorption at a single site on the surface may involve Properties, Classification and Purification of Colloids
multiple molecules at the same time.
(c) the mass of gas striking a given area of surface is propor- 10. The coagulating power of electrolytes having ions Na+, Al3+
tional to the pressure of the gas. and Ba2+ for arsenic sulphide sol increases in the order
(d) the mass of gas striking a given area of surface is inde- (a) Na+ < Ba2 + < Al3+ (b) Ba2 + < Na+ < Al3+
pendent of the pressure of the gas.
(c) Al3+ < Na+ < Ba2 + (d) Al3+ < Ba2 + < Na+
(AIEEE 2006)
(JEE Main 2013)
Solution Solution
(c) Langmuir model shows the dependence of surface cover- (a) As2S3 is a negatively charged sol, so according to Hardy–
age of an adsorbed gas on the pressure of gas above the Schulze rule, greater the valence of the active ion or floc-
surface at a fixed temperature. culating ion, greater will be its coagulating power.

kp 11. Under ambient conditions, which among the following sur-


q= factants will form micelles in aqueous solution at lowest
1+ kp
molar concentration?
where p is the pressure of the gas and q is the fraction of + −
(a) CH3 (CH2 )15 N(CH3 )3 Br (b) CH3 (CH2 )13 OSO3−Na+
adsorbed sites. +
(c) CH3 (CH2 )8 COO −Na+ (d) CH3 (CH2 )11 N(CH3 )3 Br −
8. According to Freundlich adsorption isotherm, which of the fol-
(JEE Main Online 2015 IIT-JEE 2008)
lowing is correct?
Solution
x
(a) ∝ p0 (a) Greater is the size of hydrophobic chain, less is the solu-
m
bility in water and greater is the tendency of surfactant
x
(b) ∝ p1 molecules to be associated to form micelle.
m
x 12. Which of the following electrolytes is least effective in caus-
(c) ∝ p1/ n ing flocculation of ferric hydroxide solution?
m
(a) K3[Fe(CN)6] (b) K2CrO4
(d) All the above are correct for different ranges of pressure
(c) KBr (d) K2SO4
(AIEEE 2012)
Solution
Solution
(c) According to Hardy–Schulze rule, greater the valence of
(d) Consider the Freundlich adsorption isotherm shown in
the active ion or flocculating ion, greater will be its coag-
the following figure:
ulating power.

Chapter-20.indd 662 8/3/2016 3:08:20 PM


Solved Examples 663

13. The volume of a colloidal particle, Vc as compared to the vol- + Electrodes −


ume of a solute particle in a true solution Vs, could be
(a) Vc Vs ≈ 1 (b) Vc Vs ≈ 1023
(c) Vc Vs ≈ 10 −3 (d) Vc Vs ≈ 103
Dispersion
Solution medium
(d) Diameters of the particles in true solution are in the range
Ice-bath
10−8 to 10−7 m, whereas those of colloidal particles lie in
the range of 10−9 to 10−6 m. Hence, by taking particle size
as 10−6 and 10−7, respectively, Vc Vs is given by

Vc ( 4/3)p (10 −6 /2)3 10 −18 In Bredig’s arc method, an electric arc is struck between
= = = 103 the electrodes suspended in a water bath. The heat thus
Vs ( 4/3)p (10 −7 /2)3 10 −21
produced converts the metal into vapours which are
condensed immediately by the water bath and colloidal
14. The disperse phase in colloidal iron (III) hydroxide and col-
size particles are formed.
loidal gold is positively and negatively charged, respectively.
Which of the following statements is NOT correct?
18. Which of the following processes can be used for the purifi-
(a) Magnesium chloride solution coagulates, the gold sol
cation of colloids?
more readily than the iron (III) hydroxide sol.
(a) Coagulation (b) Dialysis
(b) Sodium sulphate solution causes coagulation in both
(c) Flocculation (d) All of these.
the sols.
(c) Mixing the sols has no effect. Solution
(d) Coagulation in both sols can be brought about by
(b) Dialysis is a process used for purification of colloids as it
electrophoresis.
allows separation of small particles from colloids. It can
Solution be used to purify solutions containing protein molecules
(c) On mixing oppositely charged sols, their charge gets and solute.
neutralized. Both the sols can be either partially or com-
pletely precipitated. Protection of Colloids
15. Gold sol is negatively charged. The flocculation values of 19. Gold numbers of protective colloids A, B, C and D are 0.50,
effective ions vary in the order: 0.01, 0.10 and 0.005, respectively. The correct order of their
(a) Sn4+ < Al3+ < Ba2+ < Na+ protective powers is
(b) PO33 − < SO24− < Cl− (a) D < A < C < B (b) C < B < D < A
(c) A < C < B < D (d) B < D < A < C
(c) Na+ > Al3+ > Ba2+ > S4+
(AIEEE 2008)
(d) SO24− < Na+ < PO34− < Ba2 +
Solution
Solution
(c) Gold number is number of milligram of protective col-
(a) According to Hardy–Schulze rule, greater the valence of loid that will prevent precipitation of 10 ml gold solution
the active ion or flocculating ion, greater will be its coag- on addition of 1 ml 10% NaCl solution.
ulating power. Lesser the gold number of a protective colloid, the
16. Which of the following methods is not employed for the greater is its protective power.
purification of colloids?
20. The protective power of lyophilic sol is
(a) Electrodialysis (b) Dialysis
(a) dependent on the size of colloidal particles.
(c) Ultracentrifugation (d) Peptization
(b) expressed in terms of gold number.
Solution (c) expressed by x/m.
(d) Electrodialysis, dialysis and centrifugation are employed (d) directly proportional to the magnitude of charge on it.
for the purification of colloids. Solution
(b) Gold number measures the protective ability of colloid.
17. Metals like silver and copper can be obtained in the colloidal
state by
21. Gold number is minimum in the case of
(a) peptization. (b) Bredig’s arc method.
(a) gelatin. (b) egg albumin.
(c) dialysis. (d) coagulation.
(c) gum Arabic. (d) starch.
Solution
Solution
(b) Metals like silver and copper can be obtained in a colloi-
(a) Gelatin has the minimum gold number. It has the high-
dal state by Bredig’s arc method.
est capacity to prevent coagulation of 10 mL of standard

Chapter-20.indd 663 8/3/2016 3:08:22 PM


664 Chapter 20 Surface Chemistry

gold sol when 1 mL of 10% NaCl solution is added to 24. Which of the following has largest protecting power?
gold sol. (a) Gelatin (gold number = 0.01)
(b) Dextrin (gold number = 15)
22. The coagulation of 10 cm3 of gold sol is completely pre-
(c) Potato starch (gold number ≈ 25)
vented by addition of 0.025 g of starch of it. The gold number
(d) Albumin (gold number ≈ 0.25)
of starch is
(a) 0.025 (b) 0.25 (c) 2.5 (d) 25 Solution
Solution (a) Smaller the gold number, greater is its protective power.
(d) Gold number is the minimum amount of lyophilic col- Emulsions
loid in milligrams which prevents the coagulation of
10 mL gold sol against 1 cm3 of 10% NaCl. 25. The type of emulsion is mentioned against the emulsion in
10 cm3 of gold sol requires 0.025 × 103 mg of starch each of the following. Which is not correctly matched?
So, gold number = 25. (a) Milk: o/w (b) Vanishing cream: w/o
(c) Butter: w/o (d) Cold cream: w/o
23. Gold number of Gum Arabic is 0.15. The amount of Gum
Arabic required to protect 100 mL of red gold sol from coag- Solution
ulation by 10 mL of 10% NaCl solution is (b) Vanishing creams (i.e., stearic acid creams) are o/w
(a) 0.15 m mol (b) 0.15 mg emulsions.
(c) 1.5 m mol (d) 1.5 mg
26. The emulsifying agent present in milk that makes it stable is
Solution (a) lactose. (b) maltose.
(d) Gold number is the minimum amount of lyophilic col- (c) casein. (d) Lactic bacilli.
loid in milligrams which prevents the coagulation of Solution
10 mL gold sol against 1 mL of 10% NaCl. Therefore,
1.5 mg will be used. (c) Casein or milk protein stabilizes milk.

ADVANCED LEVEL PROBLEMS


1. Statement 1: Micelles are formed by surfactant molecules Solution
above the critical micellar concentration (CMC).
Statement 2: The conductivity of a solution having surfactant (c) As Sb2S3 is a negative sol, so, Al2(SO4)3 will be the most
molecules decreases sharply at the CMC. effective coagulant due to higher charge density on Al3+
(a) Statement 1 is True, Statement 2 is True; Statement 2 is a in accordance with Hardy-Schulze rule.
correct explanation for Statement 1. Order of effectiveness of cations:
(b) Statement 1 is True, Statement 2 is True; Statement 2 is
NOT a correct explanation for Statement 1. Al3+ > Ca2+ > Na+ > NH+4
(c) Statement 1 is True, Statement 2 is False.
(d) Statement 1 is False, Statement 2 is True. 3. The correct statement(s) pertaining to the adsorption of a gas
(IIT-JEE 2007) on a solid surface is (are)
Solution (a) Adsorption is always exothermic.
(b) Micelles are the cluster or aggregated particles formed by (b) Physisorption may transform into chemisorption at high
association of colloids in the solution. The formation of temperature.
micelles takes place only above a particular temperature (c) Physisorption increases with increasing temperature but
called kraft temperature (Tk) and above a particular con- chemisorption decreases with increasing temperature.
centration called critical micelle concentration (CMC). The (d) Chemisorption is more exothermic than physisorption;
micelles may contain about 100 molecules or more. The however, it is very slow due to higher energy of activation.
conductivity of the solution decreases sharply atthe CMC. (IIT-JEE 2011)

2. Among the electrolytes Na2SO 4 , CaCl2 , Al2 (SO 4 )3 and NH4 Cl, Solution
the most effective coagulating agent for Sb2S3 sol is (a), (b), (d)
(a) Na2SO4 (a) During adsorption, there is always decrease in surface
(b) CaCl2 energy which appears as heat. In other words, we can say
(c) Al2(SO4)3 that adsorption is an exothermic process.
(d) NH4Cl
(IIT-JEE 2009)

Chapter-20.indd 664 8/3/2016 3:08:23 PM


Advanced Level Problems 665

(b) Physiosorption may transform into chemisorption at high


temperature. For example, dihydrogen is first adsorbed
on Ni by van der Waals forces, molecules of hydrogen
then dissociate to form H atoms, which are held on the

Potential energy
surface by chemisorption.
(c) Low temperature is favourable for physiosorption; it Eads
0
decreases with increase of temperature. High temper- Distance of molecule from the surface
ature is favourable for chemisorption; it increases with ∆Hads = 150 kJ mol−1
increase of temperature.
(d) Energy of adsorption is high in chemisorption which is
about 80–240 kJ mol−1 as compared to physiosorption
where the energy is about 20–40 kJ mol−1. (IV)
4. Choose the correct reason(s) for the stability of the lyophobic (a) I is physisorption and II is chemisorption.
colloidal particles. (b) I is physisorption and III is chemisorption.
(a) Preferential adsorption of ions on their surface from the (c) IV is chemisorption and II is chemisorption.
solution. (d) IV is chemisorption and III is chemisorption.
(b) Preferential adsorption of solvent on their surface from (IIT-JEE 2012)
the solution. Solution
(c) Attraction between different particles having opposite
charges on their surface. (a), (c) In case of physisorption, with the increase of tem-
(d) Potential difference between the fixed layer and the perature and pressure the rate of adsorption decreases
diffused layer of opposite charges around the colloidal because according to Le Chatelier’s principle, increase of
particles temperature and pressure will shift the equilibrium to the
(IIT-JEE 2012) left.
Adsorbate + Adsorbent  Adsorption + Heat
Solution
(a), (d) In case of lyophobic sols, the ionic colloid adsorbs This is shown in Graphs I and III; whereas in case of
ions common to its own lattice during the preparation chemisorption, there is a formation of strong bond
of the sol. So they are stable and also the potential dif- between the adsorbate and the adsorbent and so the
ference which arises between the fixed layer and the dif- rate of adsorption increases with increase in temperature
fused layer makes lyophobic sols more stable. (Graphs II and IV).

5. The given graphs/data I, II, III and IV represent general trends 6. Methylene blue, from its aqueous solution, is adsorbed on
observed for different physisorption and chemisorption pro- activated charcoal at 25°C. For this process, the correct state-
cesses under mild conditions of temperature and pressure. ment is
Which of the following choice(s) about I, II, III and IV is (are) (a) The adsorption requires activation at 25°C.
correct? (b) The adsorption is accompanied by a decrease in enthalpy.
(c) The adsorption increases with increase of temperature.
(d) The adsorption is irreversible.
Amount of gas adsorbed

Amount of gas adsorbed

p constant (JEE Advanced 2013)


p constant Solution
(b) The adsorption of methylene blue on activated charcoal is
physisorption, which causes decrease in enthalpy.
7. In a solution of 100 mL 0.5 M acetic acid, one g of active char-
coal is added, which absorbs acetic acid. It is found that the
T T concentration of acetic acid becomes 0.49 M if surface area of
charcoal is 3.01× 102 , m2 , calculate the area occupied by single
(I) (II) acetic acid molecule on surface of charcoal.
Solution
Amount of gas adsorbed

200 K The number of moles of acetic acid in 100 mL (before add-


250 K ing charcoal) = 0.05 mol
The number of moles of acetic acid in 100 mL (after add-
ing charcoal) = 0.049 mol
The number of moles of acetic acid adsorbed on the sur-
face of charcoal = 0.05 − 0.049 = 0.001 mol
p The number of molecules of acetic acid adsorbed on the
surface of charcoal = 0.001 × 6.023 × 1023 = 6.023 × 1020
(III)

Chapter-20.indd 665 8/3/2016 3:08:24 PM


666 Chapter 20 Surface Chemistry

Given that the surface area of charcoal = 3.01 × 102 m2, so Solution
the area occupied by single acetic acid molecule on the (c) They are fairly stable and cannot be coagulated easily.
2 Some examples are starch, gum, gelatin, etc. dissolved
3 × 10
surface of charcoal is = 5 × 10 −19 m2 in a suitable solvent.
6.023 × 1020
10. Different concentrations of aqueous solutions placed with
8. 20% surface sites have adsorbed N2. On heating N2 gas charcoal which adsorbs a part of solute from solution as
evolved from sites and was collected at 0.001 atm and 298 K reported below at equilibrium.
in a container of volume is 2.46 cm3. Density of surface sites is
6.023 × 1014 cm−2 and surface area is 1000 cm2, find out the Concentration of solution × 102 2.0 4.0
number of surface sites occupied per molecule of N2. x/m 0.185 0.290
Solution Calculate the values of log k and n.
(2) Given that pN2 = 0.001 atm, T = 298 K, V = 2.46 cm3. By Solution
ideal gas equation, we have pV = nRT. So,  x  1
log   = log k +   log C
 m  n
pV 0.001× 2.46 × 10 −3  1
n= = = 1.0 × 10 −7 log 0.185 = log k +   log(2 × 10 −2 )
RT 0.0821× 298  n
Now, molecules of N2 = 6.023 × 1023 × 1 × 10−7
 1
= 6.023 × 1016 log 0.290 = log k +   log( 4 × 10 −2 )
 n
Also, the total sites available = 6.023 × 1014 × 1000
= 6.023 × 1017. Solving, we get x = 0.64, k = 83.5 and log k = 1.92.
Therefore, surface sites used to adsorb N2 = 20% of the 11. Compare the coagulating power of AlCl3 with that of
20 NaCl. Given that their coagulation values are 0.093 and 52,
total sites available = × 6.023 × 1017 = 12.04 × 1016
100 respectively.
Therefore, the sites occupied per molecule of Solution
Surface sites used to adsorb N2 12.04 × 1016
N2 = = =2 Coagulating power is inversely proportional to the coagu-
Molecules of N2 6.02 × 1016 lating value. So,
9. Which of the following statements is correct for lyophilic Coagulating power of AlCl3 Coagulation value of NaCl
=
sols? Coagulating power of NaCl Coagulation value of AlCl3
(a) The coagulation of the sols is irreversible in nature. 0.52
(b) They are formed by inorganic substances. = = 559
0.093
(c) They are self-stabilized.
which means that AlCl3 has 559 times greater coagulation
(d) They are readily coagulated by addition of electrolytes.
power than NaCl.

PRACTICE EXERCISE
Level I 4. A colloidal system has particles of what size?
(a) 10−4 m to 10−9 m (b) 10−5 m to 10−7 m
Single Correct Choice Type −6 −9
(c) 10 m to 10 m (d) 10−6 m to 10−7 m
1. The Brownian movement of colloidal particles is because of 5. Which type of a property is the Brownian movement of colloi-
(a) convection currents in the fluids. dal solution?
(b) unequal bombardments by the molecules of the disper- (a) Electrical (b) Optical
sion medium on colloidal particles. (c) Mechanical (d) Colligative
(c) setting of dispersed phase under gravity. 6. Lyophilic solutions are more stable than lyophobic solutions
(d) thermal gradient in the medium. because
2. When FeCl3 solution is added to NaOH, a negatively charged (a) the colloidal particles have positive charge.
sol is obtained. It is due to the (b) the colloidal particles have negative charge.
(a) presence of basic group. (c) the colloidal particles are solvated.
(b) preferential adsorption of OH− ions. (d) there are strong electrostatic repulsions between the neg-
(c) self-dissociation. atively charged colloidal particles.
(d) electron capture by sol particles. 7. When a colloidal solution is observed under an ultramicro-
3. A catalyst added to a reversible reaction scope, we can see
(a) changes the position of equilibrium. (a) light scattered by colloidal particles.
(b) increases the concentration of products. (b) size of the particle.
(c) increases the equilibrium constant. (c) shape of the particle.
(d) speeds up both forward and backward reactions. (d) relative size of the particles.

Chapter-20.indd 666 8/3/2016 3:08:25 PM


Practice Exercise 667

8. The rate of chemisorption (a) peptization. (b) dialysis.


(a) decreases with increase of pressure. (c) protective action. (d) dissolution.
(b) increases with increase of pressure. 18. The number of phases in colloidal system is
(c) is independent of pressure. (a) one. (b) two. (c) three. (d) four.
(d) is independent of temperature.
19. Which of the following kinds of catalysis can be explained by
9. A catalyst in the finely divided form is most effective because the adsorption theory?
(a) less surface area is available. (a) Heterogeneous catalysis
(b) more active sites are formed. (b) Enzyme catalysis
(c) more energy gets stored in the catalyst. (d) Homogeneous catalysis
(d) none of these. (d) Acid-base catalysis
10. Bleeding is stopped by the application of ferric chloride. This 20. In aerosol, the dispersion medium is
is because (a) solid. (b) liquid.
(a) the blood starts flowing in opposite direction. (c) gas. (d) any of these.
(b) the blood reacts and forms a solid, which seals the blood
vessel. 21. Although nitrogen does not adsorb on surface at room tem-
(c) the blood is coagulated and thus the blood vessel is scaled. perature, it adsorbs on the same surface at 83 K. Which one
(d) the ferric chloride seals the blood vessel. of the following statement is correct?
(a) At 83 K, there is formation of monomolecular layer.
11. A while precipitate of Sn(OH)4 is peptized with dilution HCl.
(b) At 83 K, there is formation of multimolecular layer.
The sol particle will carry
(c) At 83 K, nitrogen molecules are held by chemical bonds.
(a) positive charge.
(d) At 83 K, nitrogen is adsorbed as atoms.
(b) negative charge.
(c) sometimes positive and sometimes negative charge. 22. Adsorption of gases on solid surface is generally exothermic
(d) no charge. because
(a) free energy increases.
12. Which of the following is true about chemisorption?
(b) entropy decreases.
(a) It is reversible in nature.
(c) entropy increases.
(b) It usually occurs at low temperature.
(d) interaction developed between gas and solid particles.
(c) It is highly specific in nature.
(d) The attractive forces between adsorbate and adsorbent 23. Flocculation value is expressed in terms of
are van der Waals forces. (a) millimol L−1 (b) mol L−1
13. Identify the correct statement regarding enzymes. (c) g L−1 (d) mol mL−1
(a) Enzymes are specific biological catalysts that can nor- 24. On addition of 1 mL solution of 10% NaCl to 10 mL gold sol
mally function at very high temperature. in the presence of 0.250 g of starch, the coagulation is just
(b) Enzymes are normally heterogeneous catalysts that are prevented. Starch has the following gold number:
very specific in action. (a) 0.025. (b) 0.25. (c) 2.5. (d) 250.
(c) Enzymes are specific biological catalysts that cannot be
poisoned. 25. Which of the following metal solutions cannot be prepared
(d) Enzymes are biological catalysts that possess well- by Bredig’s arc method?
defined active site. (a) Copper (b) Potassium
(c) Gold (d) Platinum
14. The curve showing the variation of adsorption with pressure
at constant temperature is called 26. The volume of gases adsorbed by 1 g of charcoal at 300 K is in
(a) an isostere. (b) adsorption isotherm. (a) H2 > CO2 > NH3. (b) NH3 > H2 > CO2.
(c) adsorption isobar. (d) All are correct. (c) NH3 > CO2 > H2. (d) CO2 > NH3 > H2.
15. Stability of lyophilic colloids is due to 27. Gelatin protects
(a) the same charge on all the colloidal particles. (a) Au (b) As2S3
(b) the solvation of the colloidal particles. (c) Fe(OH)3 (d) All of these
(c) both (a) and (b).
(d) the fact that they are organic substances. 28. Clouds represent an example of dispersion of
(a) gas in solid. (b) solid in gas.
16. Which of the following statement is correct about Langmuir’s (c) gas in gas. (d) liquid in gas.
adsorption?
(a) It forms a monolayer. 29. Which type of the molecules form micelles?
(b) It has a reversible temperature. (a) Non-polar molecules (b) Polar molecules
(c) It occurs at low temperature. (c) Surfactant molecules (d) All of these.
(d) It is not specific in nature. 30. Which of the following is adsorbed greatly by activated
17. On adding few drops of dilute HCl to freshly precipitated fer- charcoal?
ric hydroxide, a red coloured colloidal solution is obtained. (a) SO2 (b) CO2
This phenomenon is known as (c) CO (d) Water vapour

Chapter-20.indd 667 8/3/2016 3:08:26 PM


668 Chapter 20 Surface Chemistry

31. Which of the following electrolytes will have maximum floc- (c) The adsorbed molecules do not interact with each other.
culation value for Fe(OH)3 sol? (d) The adsorption takes place in unimolecular layer.
(a) NaCl (b) Na2S
41. Which of the following statements are correct?
(c) (NH4)3PO4 (d) K2SO4 (a) Flocculation value is inversely proportional to the coag-
32. Which of the following methods is used for sol destruction? ulating power.
(a) Condensation. (b) Alum is used for cleaning muddy water.
(b) Dialysis. (c) Milk is an emulsion of protein in water.
(c) Diffusion through animal membrane. (d) Gelatin added to ice cream acts as an emulsifier.
(d) Addition of an electrolyte. 42. Which of the following are accompanied by adsorption?
(a) Decrease in entropy of the system
33. Gold number gives the indication of
(b) Decrease in enthalpy of the system
(a) gram-molecules of gold per 1000 mL of colloidal solution.
(c) T∆S for the process is negative
(b) percentage of gold in the suspension.
(d) Increase in enthalpy of the system
(c) charge on the colloids.
(d) protective power of a colloid. 43. Which of the following are lyophilic colloids?
(a) Starch sol (b) Gum sol
34. Which of the following is not applicable to chemisorption?
(c) Gold sol (d) Protein
(a) Effect of pressure is given by Freundlich adsorption
isotherm. 44. Forces of attraction involved between adsorbent and adsorb-
(b) There is formation of a monomolecular layer. ate are
(c) It occurs at high temperature. (a) weak.
(d) It involves the formation of chemical bonds between (b) strong.
adsorbent and adsorbate. (c) long-range van der Waals forces.
(d) strong chemical bonds.
35. Blue colour of water in sea is due to
(a) refraction of blue light by impurities in sea water. Passage Type
(b) refraction of blue sky by water. Paragraph for Questions 45 to 48: The aggregation of Fe(OH)3 sol
(c) scattering of light by water. can be done by adding ionic solution, especially if the solution con-
(d) None of these. tains multiple-charged anions (e.g., phosphate ions). Coagulation
36. In which of the following colloidal systems the dispersion is the process by which the dispersed phase of a colloid undergoes
medium is a solid? aggregation, and thus separates from the continuous phase.
(a) Soap lather (b) Smoke 45. The electrolyte that has the least effect in the coagulation of
(c) Boot polish (d) Clouds Fe(OH)3 is
(a) potassium carbonate. (b) sodium sulphate.
Level II
(c) potassium ferrocyanide. (d) potassium iodide.
Multiple Correct Choice Type 46. Colloidal sulphur particles are negatively charged with thio-
37. Which of the following decrease the activation of a solid sulphate ions, S2O2−3 , and other ions on the surface of the
adsorbent? sulphur. Indicate which of the following would be most
(a) Subdividing the solid adsorbent. effective in coagulating colloidal sulphur.
(b) Polishing the surface of the solid adsorbent. (a) NaCl (b) KCl
(c) Carrying out the adsorption at very high temperature. (c) MgCl2 (d) Al2(SO4)3
(d) Blowing superheated steam through the porous 47. Which of the following solution is negatively charged?
adsorbent. (a) Arsenious sulphide
38. Emulsions also show mechanical properties of colloidal solu- (b) Aluminium hydroxide
tions such as (c) Ferric hydroxide
(a) Brownian motion. (b) Tyndall effect. (d) Silver iodide in silver nitrate solation
(c) Both (a) and (b). (d) none of these. 48. Which of the following anions will have minimum floccula-
tion value for the ferric oxide solution?
39. Crystalloid and colloid differ with respect to
(a) Cl− (b) Br−
(a) Tyndall effect. 2−
(b) particle size. (c) SO 4 (d) [Fe(CN)6]3−
(c) diffusion through a membrane.
Paragraph for Questions 49 to 53: Previously, the term “colloids”
(d) number of particles per unit volume.
was used for a category of substances. However, later, the term
40. The Langmuir adsorption isotherm is deduced based on the colloidal state of matter was preferred. Colloidal dispersions have
assumption: been classified into different types depending upon the physical
(a) The adsorption sites are equivalent in their ability to state of the dispersed phase and the dispersion medium or the
adsorb the particles. nature of interactions between them or the nature of the colloidal
(b) The heat of adsorption varies with coverage. particles. They are prepared in the industry or in the laboratory

Chapter-20.indd 668 8/3/2016 3:08:26 PM


Answer Key 669

by a number of methods and then purified. Their properties have 56. Equilibrium in chemisorption attained
also been studied in detail. Hardy and Schulze made a substantial (a) relatively fast.
contribution in studying the coagulation of the colloids. The pro- (b) not possible.
tective action of lyophilic colloids was studied by Zsigmondy and (c) on changing temperature.
he introduced a term, called “gold number”. (d) relatively slowly.
49. Lyophilic sols are more stable than lyophobic sols because Matrix-Match Type
(a) the colloidal particles have positive charge.
57. Match the colloidal dispersion with the nature of dispersion.
(b) the colloidal particles have negative charge.
(c) the colloidal particles are solvated. Column I Column II
(d) there is strong electrostatic repulsions between the (a) Milk (p) Solid in liquid
particles.
(b) Mist (q) Liquid in gas
50. Which one of the following forms micelles in aqueous solu- (c) Gold sol (r) Solid in solid
tion above certain concentration?
(a) Dodecyl trimethyl ammonium chloride (d) Ruby glass (s) Liquid in liquid
(b) Glucose 58. Match the processes with their purpose.
(c) Urea Column I Column II
(d) Pyridinium chloride
(a) Electrophoresis (p) Purification of colloids
51. Identify the correct statement about the colloids. (b) Electrodialysis (q) Protection of colloids
(a) Colloidal sulphur is a substance. (c) Electrodispersion (r) Preparation of colloids
(b) A colloid can be defined as homogenous system.
(d) Addition of lyophilic sol (s) Coagulation of colloidal
(c) A colloidal state is intermediate between a true solution
particles
and a suspension.
(d) None of these. 59. Match the sol with its type.
52. Gold number is associated with Column I Column II
(a) electrophoresis. (b) amount of pure gold. (a) Gelatin sol (p) Lyophilic
(c) protective colloid. (d) precipitation of colloid. (b) Gold sol (q) Associated
53. The diameter of colloidal particle ranges from (c) Soap sol (r) Multimolecular
(a) 10–9 m to 10–6 m (b) 10–9 m to 10–12 m
(d) Starch sol (s) Macromolecular
(c) 103 m to 10–3 m (d) 10–3 m to 10–6 m

Paragraph for Questions 54 to 56: The adsorption is called phys- Integer Type
ical adsorption or physisorption if the accumulation of gas on the 60. Among the following, the number of substances that will
surface of a solid occurs due to weak van der Waals forces. The adsorb water vapour is ___.
adsorption is called chemical adsorption or chemisorption when Silica, alumina, charcoal, calcium chloride, calcium car-
the gas molecules or atoms are held to the solid surface by chem- bonate, powdered cellulose, kieselguhr, fuller’s earth.
ical bonds that are either ionic or covalent. Chemisorption is often
referred to as activated adsorption because it has a rather high 61. In a linear plot for log(x/m) vs. log p, if the slope is 45° and the
energy of activation. intercept on log(x/m) is 0.3010, then amount of gas adsorbed
in grams at 0.5 atm pressure is ___.
54. Which of the following is not a characteristic of chemisorption?
(a) Adsorption is irreversible. 62. In an experiment, addition of 4.0 mL of 0.005 M BaCl2 to 16.0
(b) ∆H is of the order of 40 kJ. mL of arsenious sulphide sol just causes the complete coagu-
(c) Adsorption is specific. lation in 2 h. The flocculating value of the effective ion is ___.
(d) Adsorption increases with increase of surface area.
63. The coagulation of 100 mL of a colloidal solution of gold is
55. Magnitude of physical adsorption decreases with completely prevented by adding 0.08 g of starch to it before
(a) decrease in temperature. (b) increase in temperature. adding 10 mL of 10% NaCl solution. The gold number of
(c) constant temperature. (d) high pressure. starch is ___.

ANSWER KEY
Level I
1. (b) 2. (b) 3. (d) 4. (c) 5. (c) 6. (c)
7. (a) 8. (b) 9. (b) 10. (d) 11. (a) 12. (c)
13. (b) 14. (b) 15. (c) 16. (a) 17. (a) 18. (b)
19. (a) 20. (c) 21. (b) 22. (d) 23. (a) 24. (d)
25. (b) 26. (c) 27. (d) 28. (d) 29. (c) 30. (d)
31. (a) 32. (d) 33. (d) 34. (a) 35. (c) 36. (c)

Chapter-20.indd 669 8/3/2016 3:08:26 PM


670 Chapter 20 Surface Chemistry

Level II
37. (b), (c) 38. (a), (b), (c) 39. (a), (b), (c) 40. (a), (c), (d) 41. (a), (b), (d) 42. (a), (b), (c)
43. (a), (b), (d) 44. (a), (c) 45. (c) 46. (d) 47. (b) 48. (d)
49. (c) 50. (a) 51. (c) 52. (b) 53. (a) 54. (b)
55. (b) 56. (d) 57. (a) → s; (b) → q; (c) → p; (d) → r 58. (a) → s; (b) → p; (c) → r; (d) → q
59. (a) → p, s; (b) → r; (c) → q; (d) → p, s 60. (5) 61. (1) 62. (1) 63. (8)

HINTS AND EXPLANATIONS


Level I 22. (d) When adsorption takes place, surface energy decreases
which appears in the form of heat.
Single Correct Choice Type
23. (a) Flocculation value is expressed in terms of millimol L−1.
1. (b) Unequal bombardments of solvent particles on colloidal
24. (d) Starch added to 10 mL of gold sol to completely pre-
particles cause zig-zag motion.
vent coagulation by 1 mL of 10% NaCl solution = 0.250 g
2. (b) As an ion adsorbs ions common to its own lattice. = 250 mg.
3. (d) It does not changes the nature and position of the equi- By definition gold number of starch = 250
librium constant, only lowers the activation energy. 25. (b) Highly electropositive metals such as, K, Na, etc. cannot
4. (c) 10−9 to 10−6 m be prepared by Bredig’s arc method.
5. (c) The zigzag random movement of colloidal particles is 26. (c) Easily liquefiable gases are adsorbed more.
called Brownian movement. 27. (d) Protective colloids are used to protect colloids from
6. (c) As they are solvent loving. external reagents. Gelatin is a good protective colloid.
28. (d) Clouds represent an example of dispersion of liquid in
7. (a) This is known as Tyndall effect.
gas.
8. (b) At constant temperature, the extent of adsorption
29. (c) Soaps and detergents are surfactants and form micelles.
increases with increase in pressure. This is in accordance
It helps them in their cleansing action.
with Le Chatelier's principle, according to which the sys-
tem would move to lesser number of molecules with 30. (d) Activated charcoal readily adsorbs water vapour while
increase in pressure. H2 is least adsorbed.

9. (b) This is in accordance with adsorption theory of hetero- 31. (a) The reaction involved is (NH4 )3PO 4 → 3NH+4 + PO34− .
geneous catalysis. Greater is the valency, greater is the coagulating power
and coagulating power is inversely proportional to the
10. (d) Due to coagulation of blood forming a clot. coagulation value or flocculation value. As Fe(OH)3 is a
11. (a) Forms positively charged SnCl4. positively charged sol so, a negative ion will be needed
to coagulate it. Out of the given anions, PO3− 4 has the
12. (c) Chemisorption involves chemical bonds, and hence is
maximum coagulating power and Cl− has the least coag-
specific in nature.
ulating power. In other words, PO3− 4 has the least floccu-
13. (b) Conceptual lation value and Cl− has maximum flocculation value.
14. (b) Isotherms are curves, showing variation of adsorption 32. (d) An electrolyte addition can destroy a sol by coagulating
with pressure at constant temperature (hence the term it.
isotherms).
33. (d) Gold number is used for the protective power of colloid,
15. (c) Solvation of colloidal particles along with same charge smaller the gold number, greater is its protective power.
on colloidal particles favours stability of lyophilic colloids.
34. (a) Physisorption is well dealt by the Freundlich isotherm,
16. (a) Langmuir adsorption isotherm is based on the assump- but not chemisorption.
tion that every adsorption site is equivalent and the abil- 35. (c) Blue colour of water in sea is due to scattering of light by
ity of a particle to bind is independent of whether or not water and this effect is known as Tyndall effect.
nearby sites are occupied.
36. (c) Boot polish contains a solid as a dispersion medium.
17. (a) Peptization takes place forming a colloidal sol of FeCl3.
18. (b) Dispersion medium and dispersed phase are the two Level II
phases of colloids. Multiple Correct Choice Type
19. (a) In this type, the phase of the catalyst and the reactants/ 37. (b), (c) On polishing the surface or making the surface
products are different. smooth, rate of adsorption decreases. The activation of a solid
20. (c) In aerosol, the dispersion medium is a gas while dis- adsorbent decreases with increase in temperature. A quantity
persed phase is a liquid. of heat known as heat of adsorption is evolved on adsorption
21. (b) Multimolecular adsorption takes place at temperature of one mole of gas. By Le Chatelier’s principle, high tempera-
83 K. ture would shift the equilibrium towards desorption.

Chapter-20.indd 670 8/3/2016 3:08:27 PM


Hints and Explanations 671

38. (a), (b), (c) Emulsions also show mechanical properties of 53. (a) The diameter of the colloidal particle ranges from
colloidal solutions such as Brownian motion and Tyndall 1–1000 nm or 10−9 m to 10−6 m.
effect. 54. (b) ∆H for chemisorptions is about 40–400 kJ.
39. (a), (b), (c) Tyndall effect is observed in colloids and not
55. (b) Magnitude of physical adsorption decreases with
crystalloid.
increase in temperature.
The particle size of colloids is 1–1000 nm whereas that of
crystalloid is >1000 nm. 56. (d) Equilibrium in chemisorption is attained relatively
Colloids cannot diffuse through semi-permeable membrane, slowly.
whereas crystalloid can pass through the same membrane.
The number of particles per unit volume is the same for both Matrix-Match Type
crystalloid and colloid. 57. (a) ã s; (b) ã q; (c) ã p; (d) ã r
40. (a), (c), (d) According to is Langmuir adsorption isotherm, Milk is an emulsion of liquid fat in water, that is, liquid in
the whole of the surface is assumed to be unimolecular liquid.
layer of gas molecules. The number of molecules striking per Mist is an aerosol in which dispersed phase is liquid, whereas
unit area is proportional to pressure. There is no interaction the dispersion medium is gas.
between the adsorbed molecules. Gold sol is a sol in which dispersed phase is solid, whereas
41. (a), (b), (d) Flocculation value is inversely proportional to dispersion medium is liquid.
the coagulation value. More is the coagulation value, less is Ruby glass is a solid sol in which dispersed phase as well as
the flocculation value. medium is solid.
The negatively charged colloidal particles get neutralized by 58. (a) ã s; (b) ã p; (c) ã r; (d) ã q
the Al3+ ions (present in alum) and settle down, thus pure Electrophoresis is the movement of colloidal particles under
water can be separated out. the influence of electric field. The colloidal particles reach
Milk is an emulsion of liquid fat in water. oppositely charged electrodes and get neutralized and
Gelatin acts as a stabilizing agent when added to ice cream. coagulated.
42. (a), (b), (c) During adsorption, surface energy decreases and Electrodialysis is the process of purification of colloids from
also the molecules of the adsorbate are held together on the crystalloids by passing them through a semi-permeable mem-
surface of solid adsorbent, so entropy also decreases and brane and electric field is applied to speed up the process.
according to the equation Electrodispersion is a method for the preparation of col-
∆G = ∆H − T∆S loids. Sols of gold, silver are prepared by this method.
∆G is negative only when T∆S will be positive As lyophilic sols are more stable than lyophobic sols so to
prevent the coagulation or precipitation of colloids, lyophilic
43. (a), (b), (d) Starch sol, gum sol and protein are solvent lov-
sols are added.
ing (lyophilic) whereas gold sol is lyophobic.
59. (a) ã p, s; (b) ã r; (c) ã q; (d) ã p, s
44. (a), (c) Forces of attraction involved between adsorbent and
Gelatin sol is an example of lyophilic, macromolecular colloid.
adsorbate are weak, long-range van der Waals forces.
Gold sol is an example of multimolecular colloid.
Passage Type Soap sol is an associated colloid.
Starch sol is a lyophilic, macromolecular colloid
45. (c) Fe(OH)3 is a positively charged colloid and can be coag-
ulated by negatively charged ion. Among the given Integer Type
ions, that is, CO23 − , SO24− , CN− and I− , the least effect
60. (5) Adsorption can be physical or chemical. Silica, alumina,
will be of CN− as the valency is the least.
charcoal, calcium chloride, and powdered cellulose will
46. (d) According to Hardy–Schulze rule, greater the valence adsorb water vapour.
of the active ion or flocculating ion, greater will be its
coagulating power. 61. (1) On plotting log x/m vs. log p we get intercept as log k
which is equal to 0.3010. So, k = 2.
47. (b) Aluminium hydroxide solution is negatively charged.
The slope is 45°, so 1/n = 1 or n = 1.
48. (d) Greater the valence of the active ion or flocculating ion, Therefore, according to Freundlich adsorption isotherm,
greater will be its coagulating power. x/m = kp1/n ⇒ x/m = 2p = 2(0.5) = 1.
49. (c) This is because the lyophobic sols get precipitated by
62. (1) As S4 sol is negatively charged owing to preferential
the addition of electrolytes whereas lyophilic sols do not.
adsorption of S2 ions. Cation would be the effective ion in
50. (a) Dodecyl trimethyl ammonium chloride forms micelles coagulation. Flocculating value = minimum mmol of the
at critical micelle concentration (CMC).
4 × 0.005 × 103
51. (c) A colloidal state is intermediate between a true solution effective ion per litre of sol = = 1.0
4 + 16
and a suspension. For example, sulphur is a substance,
but colloidal sulphur is sulphur dispersed in water, in 63. (8) Amount of starch adsorbed (in 100 mL) = 0.08 × 1000
which sulphur atoms combine to form multimolecules. = 80 mg
52. (b) Gold number is associated with gold sol and is used Gold number = 80 × 10% (NaCl) = 80 × 10/100 = 8.
to compare the protective action of different lyophilic
colloids.

Chapter-20.indd 671 8/3/2016 3:08:27 PM


672 Chapter 20 Surface Chemistry

SOLVED JEE 2016 QUESTIONS


JEE Main 2016 3. Gold numbers of some colloids are: Gelatin: 0.005 – 0.01, Gum
arabic: 0.15 – 0.25; Oleate: 0.04 – 1.0; Starch: 15 – 25. Which
1. For a linear plot of log(x/m) versus log p in a Freundlich adsorp-
among these is a better protective colloid?
tion isotherm, which of the following statements is correct?
(a) Gelatin (b) Starch
(k and n are constants)
(c) Oleate (d) Gum Arabic
(a) log (1/n) appears as the intercept.
(Online)
(b) Both k and 1/n appear in the slope term.
(c) 1/n appears as the intercept. Solution
(d) Only 1/n appears as the slope. (a) Gold number is defined as the minimum amount of pro-
(Offline) tective colloid (in milligrams) that can prevent change in
Solution colour of 10 mL of gold from red to violet by the addition
of 1 mL of 10% of NaCl solution. It is inversely proportional
(d) According to Freundlich adsorption isotherm
to the protective power. Thus, more is the gold number,
x less is the protective power of the lyophilic colloid.
= kp1/ n
m Given that, the gelatin has lowest gold number, thus it
x 1 would be a better protective colloid than the rest of the
log = log p + log k options.
m n
Comparing with straight line equation y = mx + c, m is 4. The most appropriate method of making egg-albumin sol is
slope which is equal to 1/n. (a) break an egg carefully and transfer the transparent part of
the content to 100 mL of 5% w/V saline solution and stir well.
2. A particular adsorption process has the following characteris- (b) keep the egg in boiling water for 10 min. After removing
tics: (i) It arises due to van der Waals forces and (ii) it is revers- the shell, transfer the yellow part of the content to 100 mL
ible. Identify the correct statement that describes the above of 5% w/V saline solution and homogenize with a mechan-
adsorption process: ical shaker.
(a) Adsorption is monolayer. (c) keep the egg in boiling water for 10 min. After removing
(b) Adsorption increases with increase in temperature. the shell, transfer the white part of the content to 100 mL
(c) Enthalpy of adsorption is greater than 100 kJ mol−1 of 5% w/V saline solution and homogenize with a mechan-
(d) Energy of activation is low. ical shaker.
(Online) (d) break an egg carefully and transfer only the yellow part of
Solution the content to 100 mL of 5% w/V saline solution and stir
well.
(d) From the given characteristics, the adsorption is an exam-
(Online)
ple of physical adsorption.
Solution
(i) Physical adsorption is multimolecular layer.
(ii) It decreases with increase in temperature. (a) Only the transparent part of an egg has albumin.
(iii) Enthalpy of adsorption is low 20–40 kJ mol−1 for phys-
ical adsorption.
(iv) Energy of activation is low because in this type of
adsorption no chemical reaction takes place.

Chapter-20.indd 672 8/3/2016 3:08:28 PM


21 Nuclear Chemistry

Question Distribution in JEE (Main and Advanced)

3
No. of Questions

JEE (Main)
2
Jee (Adv)

0
2016 2015 2014 2013 2012 2011 2010 2009 2008 2007

Concept Distribution in JEE (Main and Advanced)


Topics Covered
Year
JEE (Main) JEE (Advanced)
2007 Modes of Radioactive Decay, Radioactive Decay Rate Nuclides
2008 Nucleus Stability
2009 Modes of Radioactive Decay
2010 Modes of Radioactive Decay
2011 Modes of Radioactive Decay
2012 Modes of Radioactive Decay
2013 Nuclear Reactions
2015 Modes of Radioactive Decay
2016 Modes of Radioactive Decay

Chapter-21.indd 673 8/3/2016 3:18:51 PM


674 Chapter 21 Nuclear Chemistry

SUMMARY
1. Radioactivity refers to the particles which are emitted from nuclei as a result of nuclear instability. The most common types of radi-
ation are called alpha, beta, and gamma radiation.

2. Nuclides
(a) A particular combination of protons and neutrons is called a nuclide.
(i) Nuclides with the same number of protons are called isotopes.
(ii) Nuclides with the same mass number are isobars.
(iii) Nuclides with the same number of neutrons are isotones.
(iv) A radioactive nuclide that has excess nuclear energy, making it unstable is called a radionuclide.
39 19 235 231
(b) Isodiaphers are those radioactive nuclides that have the same (N − Z) or (A − 2Z) values. For example, 19 K and 9 F (A − 2 Z = 1); 92 U and 90Th (A − 2 Z
39 19 235 231
19 K and 9 F (A − 2 Z = 1); 92 U and 90Th (A − 2 Z
= 51).
(c) Isosters are those molecules that have the same number of atoms as well as electrons. For example, CO2 and N2O (atoms = 3,
electrons = 22); N2 and CO (atoms = 2, electrons = 14).

3. Stability of nucleus
(a) Forces in the nucleus
(i) In a stable nucleus, the attractive forces are greater than the repulsive forces.
(ii) In an unstable nucleus the repulsive forces exceed the attractive forces and spontaneous fission occurs.
(b) Stability
(i) Even number of protons and neutrons: Elements of even atomic number are more stable and more abundant than neigh-
bouring elements of odd atomic number (Harkin’s rule).
(ii) Certain nuclei are extra-stable due to a filled shell. Nuclei with 2, 8, 20, 28, 50, 82 or 126 neutrons or protons are particularly
stable and have a large number of isotopes. These numbers are termed magic numbers.
(c) The ratio of neutrons to protons
(i) For the most stable nuclei the ratio n/p = 1.
(ii) Elements with higher atomic numbers are more stable if they have a slight excess of neutrons. Thus the ratio n/p increases
progressively up to about 1.6 at Z = 92 (uranium).
(iii) In elements with still higher atomic numbers, the nuclei have become so large they undergo spontaneous fission.

4. Modes of radioactive decay

• Emission of a helium nucleus. 238 234 4


92 U → 90Th + 2He (a -decay)
Alpha
decay

• Usually restricted to the heavier elements in the periodic table.

• Electron is ejected or emitted from the nucleus; the charge on the 0


[negatron ( b − ) emission]
emission

40 40
19 K → 20 Ca + −1e
Electron

nucleus increases by one.


Electron capture (b −)

• Decay by capturing one of the electrons that surround the 40 0 40


19 K + −1 e → 18 Ar + hn (electron capture)
nucleus.
• Electron capture leads to a decrease of one in the charge on the
Beta decay

nucleus.
• The electron captured by the nucleus in this reaction is usually a 1s
electron

• Positron (b +) decay produces a daughter nuclide with one fewer 0


40 40
19 K → 18 Ar + −1e [positron ( b + ) emission]
positive charge on the nucleus than on the parent.
Positron (b +)

• As soon as they come to rest, they combine with an electron


to form two g -ray photons in a matter–antimatter annihilation
emission.

reaction.
0 0
−1 e + −1 e → 2g

(Continued)

Chapter-21.indd 674 8/3/2016 3:18:53 PM


Summary 675

(Continued)
• The daughter nuclides produced by a-decay or b-decay are often 60 60 m 0
26 Fe → 27 Co + −1e
Gamma emission

obtained in an excited state. The excess energy associated with this


excited state is released when the nucleus emits a photon in the The metastable 60mCo nuclide has a half-life
g -ray portion of the electromagnetic spectrum. of 10.5 min. Since electromagnetic radiation
• In some cases, g -emission is delayed, and a short-lived, or meta- carries neither charge nor mass, the product
stable, nuclide is formed, which is identified by a lowercase letter m of g -ray emission by 60mCo is 60Co.
written after the mass number. 60mCo, for example, is produced by 60 m 60
the electron emission of 60Fe. 27 Co → 27 Co +g (g -ray emission)
• Nuclides with atomic numbers of 90 or more undergo a form of 1
Sponta-

252 140 108


98 Cf → 54 Xe + 44 Ru + 4 0 n
fission
neous

radioactive decay known as spontaneous fission in which the


parent nucleus splits into a pair of smaller nuclei.

5. Decay mechanism in neutron-rich and neutron-poor nuclides


A graph of the number of neutrons vs. the number of protons for all stable naturally occurring nuclei is shown below.
120
184
74 W

110 n
=
p 1.49

100

90

80

70
Neutrons

107
60 47 Ag
n
=
p 1.28
50
n
40 p =1
56
Fe
26

30 n = 1.15
p

20

10
20 n
10 Ne p = 1

0 10 20 30 40 50 60 70 80
Protons

(a) The solid line represents a neutron-to-proton ratio of 1:1. There is no stable nuclides with atomic numbers larger than 83.
(b) Nuclei that lie above this line have too many neutrons and are therefore neutron-rich.
(c) Nuclei that lie below this line don’t have enough neutrons and are therefore neutron-poor.
6. Binding energy and nuclear stability
(a) The difference between the mass of an atom and the sum of the masses of its protons, neutrons, and electrons is called the mass
defect.
(b) The mass defect is also known as the binding energy (BE) of the nucleus.
(i) Larger the BE, the more stable is the nucleus.
(ii) The BE of a nuclide can be calculated from its mass defect with Einstein’s equation, E = ∆mc2.
(iii) BE gradually increases with atomic number; they tend to level off near the end of the periodic table.

Tip Calculations of the binding energy can be simplified by using the following conversion factor between the mass defect in atomic
mass units and the binding energy in million electron volts (MeV).
1 u = 931.5016 MeV

Chapter-21.indd 675 8/3/2016 3:18:54 PM


676 Chapter 21 Nuclear Chemistry

(c) Binding energy per nucleon can be obtained by dividing the binding energy for a nuclide by the total number of protons and
neutrons it contains.
7. Kinetics of radioactive decay
(a) Radioactive nuclei decay by first-order kinetics.
(b) From an atom A undergoing radioactive decay to form B, the rate of radioactive decay is
d(N )
Rate = − = k (N )
dt
where N is the initial concentration of the radioactive substance and dN is the amount that disintegrates in time t.

Tips (a) The negative sign shows that the number of isotopes of the atom decrease with increases in time.
(b) The rate of radioactive decay does not depend on the chemical state of the isotope.

(c) The constant k is known as the disintegration or decay constant and is represented by l.
−1 dN
l=
N dt
Disintegration constant can be defined as the fraction of the total number of atoms disintegrating per second.
lnN = lt + A (A is the integration constant)
N N
At t = 0, N = N0, so, ln N = − lt + ln N0 ⇒ ln 0 = lt ⇒ 0 = e lt
N N
or N = N0e − lt ⇒ N = N0 10 − lt / 2.303
(d) The time required for decay of any specific fraction (n) of the isotope is proportional to the initial concentration: t1/ n ∝ [N0 ]
N −N N
(e) The fraction of atoms (a) decayed in time t is given by a = 0 = 1− = 1− e − lt
N0 N0
8. Half-life period
(a) The half-life for the decay of a radioactive nuclide is the length of time it takes for exactly half of the nuclei in the sample to decay.
ln 2 0.693
= t1/ 2 =
l l
(b) The half-life of a nuclide can be used to estimate the amount of a radioactive isotope left after a given number of half-lives. If the
initial amount of radioactive substance is N0, then the amount of radioactive substance decomposed and left after subsequent
half-life times is given below:
Number of half-life times (nt1/2) Amount of radioactive substance (N) Amount of radioactive substance
decomposed (N0)
0 N0 0

( 21 ) N0
1 1
1N = 1− 21 N0 = 21 N0
2 0

( 21 )
2 2
1N = N0 1− 41 N0 = 43 N0
4 0

( 21 ) N0
3 3
1N = 1− 81 N0 = 78 N0
8 0

( 21 )
4 4 1 N = 15 N
1 N = N0 1− 16 0 16 0
16 0

( 21 ) ( 21 )
n n n
N0 1− N0

9. Average life
(a) The average or mean life of an atom is the sum of lives of all atoms divided by the total number of atoms.
Total life time of all atoms
Average life (t avg ) =
Total number of atoms
(b) Let initially the number of radioactive atoms in a sample be N0 and at time t be N. If the number of atoms disintegrating between
time t and (t + dt) is given by dN, then, total life of dN atoms can be given by dN × (t + dt) or dN × t (neglecting dt as it is very small).
Therefore,

Chapter-21.indd 676 8/3/2016 3:18:58 PM


Summary 677

N0 ∞
Total life of all the atoms =

0
tdN =

0
tdN as number of initial number of atoms is very large.

The average life of atom is thus given by


∞ ∞

t avg =
∫ 0
tdN
=
∫ 0
−t lNdt
(as −
dN
= kN )
N0 N0 dt

=
∫ 0
−t lN0e − lt dt
(as N = N0e − lt )
N0
∞ 1
=
∫0
−t le − lt dt =
l
The average life is thus equal to the reciprocal of the decay constant.
(c) The average life in terms of half-life time is
0.693
t1/ 2 = = 0.693 × t avg ⇒ t avg = 1.44t1/ 2
l
If t = 1/l, then N = N0e − lt = N0e −1 = 0.37N0

10. Radioactive equilibrium


(a) For a radioactive process,
A → B → C
Radioactive Daughter Stable
element nuclei element
when the rate of decay of radioactive substance to form element B becomes the same as the rate of formation of C from B, the
system is said to be in radioactive equilibrium.
(b) If l1 and l2 are the disintegration constants for the two reactions and N1 and N2 is the number of atoms present for A and
B at time t, then the rate of formation of B or decay of A is given by l1 N1 and rate of disintegration of B is given by l2 N2.
At equilibrium
l1N1 = l2N2
N1 l2 1/l1 (t avg )1
or = = =
N2 l1 1/l2 (t avg )2

Thus, the number of atoms of A and B are in the ratio of their average life times.
N (t )
Since t1/2 = 0.693/l, substituting in equilibrium relation, we have 1 = 1/ 2 1 .
N2 (t1/ 2 )2

Case 1: When The parent • The number of parent nuclei at any time t is given by:
k1 < k2 and element is longer N1 = N0e −l1t
(t1/2)1 ≈ (t1/2)2 lived than the
daughter element • The amount of daughter element formed at any time t can be calculated using the relation:
but half-lives of N l
N2 = 0 1 [e − l1t − e − l2t ]
both are nearly the l2 − l1
same
• When t is sufficiently large, e −l2t  e −l1t , so the number of daughter nuclei being formed at
any time t is
l1 l1
N2 = ⋅ N0e − l1t = ⋅ N1
l2 − l1 l2 − l1
N1 l2 − l1
Therefore, =
N2 l1
This condition is known as transient equilibrium and is the steady state where ratio of the
parent and daughter element becomes constant.

• The time at which maximum activity of the daughter element is observed can be calculated
using the relation
2.303 l 
tmax = log10  2 
l2 − l1  l1 

(Continued)

Chapter-21.indd 677 8/3/2016 3:19:01 PM


678 Chapter 21 Nuclear Chemistry

(Continued)
Case 2: When When the parent In this case steady state is not achieved.
k1 > k2 and element has shorter
(t1/2)1 < (t1/2)2 half-life time than
the daughter
element.

11. Parallel decay


(a) When a parent element disintegrates through two parallel paths illustrated as
A →B

C
(b) If the rate of disintegration of A to B is given by l1 and that of A to C is given by l2, then total rate of disintegration of A is
l = l1 + l2
(c) Here the decay constants for each process can be expressed as

l1 = [Fractional yield of B] × l
l2 = [Fractional yield of C] × l

12. Activity of radioactive substances, detection and units of radioactivity


(a) The rate at which a radioactive isotope decays is called the activity of the isotope. It is measured as the number of disintegra-
tions taking place per second, that is, −dN / dt = l N
(b) The activity per kilogram (sometimes gram is also used) of a radioactive substance is known as specific activity.
13. Units of radioactivity
(a) Curie (Ci)
It is defined as the amount of radioactive isotope necessary to achieve an activity of 3.700 × 1010 disintegrations per second.
(b) Rutherford (Rd)
It is defined as the amount of radioactive substance that undergoes 106 disintegrations per second.
(c) Becquerel (Bq)
It is an internationally accepted SI unit of radioactivity.

1 becquerel = 1 radioactive decay per second


1 Ci = 3.7 × 1010 disintegrations s−1 = 3.7 × 1010 Bq
(d) Gray (Gy)
It is the SI unit of absorbed dose, and 1 gray corresponds to 1 J of energy absorbed per kilogram of absorbing material.
(e) Rad
It is an older unit of absorbed dose, 1 rad being the absorption of 10−2J kg−1 of tissue.
1 Gy =100 rad.
(f) Sievert (Sv)
It is the SI unit of dose equivalent.
(e) Rem
It is an older unit of dose equivalent. Its value is generally taken to equal 10−2 Sv. Rem stands for roentgen equivalent for man,
where the roentgen is a unit related to X-rays and g -radiation.
14. Radioactive series
(a) The heavy radioactive elements may be grouped into following four decay series.
(i) Thorium (4n) series
(ii) Neptunium (4n + 1) series
(iii) Uranium (4n + 2) series
(iv) Actinium (4n + 3) series
(b) The numbers in brackets indicate that the parent and all the members of a particular series have mass numbers exactly divisible
by four, or divisible by four with a remainder of one, two or three.

Chapter-21.indd 678 8/3/2016 3:19:02 PM


Summary 679

15. Induced radioactivity


(a) The process by which a stable isotope is converted into radioactive element by artificial means (artificial transmutation) is called
artificial or induced radioactivity.
(b) In shorthand notation, the parent (or target) nuclide and the daughter nuclide are separated by parentheses that contain the
symbols for the particle that hits the target and the particle or particles released in this reaction.

27 30
13 Al(a , n)15 P

(c) Artificial radionuclides are also synthesized in nuclear reactors, which are excellent sources of slow-moving or thermal neu-
trons. The absorption of a neutron usually results in a neutron-rich nuclide.
16. Transmutation
(a) Transmutation is the conversion of one chemical element or isotope into another through nuclear reaction.
(b) All of the elements beyond neptunium (atomic number 93 and higher, known as the transuranium elements) are synthetic.
(c) To make the heaviest elements, bombarding particles larger than neutrons are used, such as a-particles or the nuclei of heavier
atoms.
17. Dating by radioactive decay
The 14C dating technique is based on the following assumptions.
(a) 14C is produced in the atmosphere at a more or less constant rate.
(b) Carbon atoms circulate among the atmosphere, the oceans, and living organisms at a rate very much faster than they decay. As
a result, there is a constant concentration of 14C in all living things.
(c) After death, organisms no longer pick up 14C.
(d) Thus, by comparing the activity of a sample with the activity of living tissue, we can estimate how long it has been since the
organism died.
(e) Samples used for 14C dating can include charcoal, wood, cloth, paper, seashells, limestone, flesh, hair, soil, peat, and bone.
18. Nuclear reactions
(a) It is a process in which two nuclei, or a nucleus of an atom and a subatomic particle such as a proton, neutron, high energy
electron or a-particle from outside the atom, react to produce one or more new elementary particles (nuclides).
(b) Some important types of nuclear reactions are:

Particle capture In these reactions the particle used for bombarding is absorbed by the sub- 27 1 28
reaction stance, with or without the emission of g -radiation. 13 Al + 0 n → 13 Al + g

Particle-particle In these reactions, the collision of a nucleus with the bombarding particle 14 1 14 1
reaction leads to formation of a new nucleus and a new elementary particle. Most of 7N + 0 n → 6 C + 0H

the nuclear reactions fall in this category

Spallation reac- In these reactions, high energy projectiles remove a fragment of the heavy 75 2 56 1 1
tion nucleus to form two new nuclei and a new elementary particle 33 As + 1H → 25 Mn + 1H + 12 0 n

Fission reaction In these reactions a heavy nucleus is broken down into two heavy or medi- 235 1 139 94 1
92 U + 0 n → 56 Ba + 36Kr + 30 n
um fragments and is accompanied by release neutrons and large amount of
energy
Fusion reaction In these reactions, light nuclei fuse together to form new nuclei that are com- 7 1 2
paratively heavy and release a large amount of energy 8 Li + 1H → 2 4 He + Energy

19. Nuclear fission


(a) Splitting the nucleus of an atom into smaller lighter parts is called nuclear fission.
(b) Induced fission
Fission reactions can be induced by irradiating samples of heavy nuclides with slow-moving thermal neutrons.
235 1 139 94 1
92 U + 0 n → 56 Ba + 36Kr + 30 n
(c) The amount of fissionable material necessary for the chain reaction to sustain itself is called the critical mass.
(d) Fertile nuclides
Some of the neutrons released in the chain reaction are absorbed by 238U to form 239U, which undergoes decay by the succes-
sive loss of two b − particles to form 239Pu. 238U is an example of a fertile nuclide.
20. Nuclear fusion
(a) Nuclear fusion is a nuclear reaction in which two or more light atomic nuclei come very close to collide at a very high speed
and join to form a new nucleus. During this process, matter is not conserved because some of the matter of the fusing nuclei is
converted to photons (energy).

Chapter-21.indd 679 8/3/2016 3:19:04 PM


680 Chapter 21 Nuclear Chemistry

(b) The reaction that is most likely to fuel the first fusion reactor is the thermonuclear deuterium–tritium, reaction.
2 3 4 1
1 H + 1H → 2He + 0 n

Tip Most of the energy radiated from the surface of the sun is produced by the fusion of protons to form helium atoms within its
core.

(c) Each fusion reaction is characterized by a specific ignition temperature, which must be surpassed before the reaction can occur.

SOLVED EXAMPLES
Modes of Radioactive Decay 23
(c) the nuclide 11 Na.
1. In the transformation of 238 234
92 U to 92 U, if one emission is an
23
(d) the isobar of 11 Na.
a -particle, what should be the other emission(s)?
Solution
(a) Two b − (b) Two b − and one b + 24 23
(c) 12 Mg → 11Na + 11p
(c) One b − and one g (d) One b + and one b +
(AIEEE 2006) 4. Which of the following is the element X?
234 4 0
Solution 92Th → 7 2He + 6 −1 e + X
238 234 4 0 −
(a) 92 U → 92 U + 2 He + 2 −1 e .
Change of 4 units in atomic
(a) Pb (b) Sn (c) Tl (d) Hg
mass is compensated by one a -particle but there should
be emission of two b −-particles also to compensate Solution
0
atomic number. (a) The reaction is 234 4 A
90Th → 72 He + 6 −1 b + Z X.
2. Which of the following nuclear reactions will generate an Now, 234 = 7(4) + 6 (0) + A ⇒ A = 206 and
isotope? 90 = 7(2) + 6 (−1) + Z or Z = 82.
(a) b-particle emission (b) Neutron particle emission Therefore, X is Pb.
(c) Positron emission (d) a-particle emission 5. Calculate the number of neutrons in the 238
92 X atom produced
(AIEEE 2007) after the emission of alpha particle.
Solution (a) 152 (b) 144 (c) 185 (d) 135
(b) In beta (b-particle) emission: A neutron is converted into a Solution
proton. 238 234 4
1 1 0
(b) 92 X → 90Y + 2He
0 n → 1 p + −1e
Number of neutrons = 234 − 90 = 144
14 14 0
6 C → 7 N+ −1e
Atomic number is different but it has the same mass
number. Radioactive Decay Rate
In neutron particle emission: Mass number gets decreased 6. A radioactive element gets spilled over the floor of a room. Its
due to removal of one neutron. half-life period is 30 days. If the initial activity is 10 times the
87 86 1
36 Kr → 36 Kr + 0 n permissible value, after how many days will it be safe to enter
Isotopes have same atomic number but different mass the room?
number. Hence isotope is formed in neutron particle (a) 100 days (b) 1000 days
emission. (c) 300 days (d) 10 days.
In positron emission: Protons are converted into neutron. (AIEEE 2007)
Solution
1 1 0
1 p → 0 n + 1e (a) We know that
19 19 0
10 Ne → 9 F+ 1e  dN 
Activity =  − ∝N
Species with smaller atomic number is obtained.  dt 
In a-particle emission: Mass number is decreased by four
n n
units and atomic number is decreased by two units.  1 N  1
N = N0   ⇒ = 
 2 N0  2 
238 234 4
92 U → 90Th+ 2 He+Energy
n
24 1  1
3. A photon knocks a proton out of 12 Mg nucleus to form Therefore, =   ⇒ 10 = 2n
(a) the isotope of parent nucleus. 10  2 
(b) the isobar of parent nucleus. Taking log on both sides, we get

Chapter-21.indd 680 8/3/2016 3:19:09 PM


Advanced Level Problems 681

log10 = n log 2 24
9. Calculate the half-life of 11 Na if 2.4 × 10−5 g sample disinte-
1 grates at the rate of 7.75 × 1012 atoms s−1.
or n= = 3.32 (as log 2 = 0.3010)
0.301
(a) 0.539 × 105 s (b) 0.439 × 105 s
Therefore, t = n × t1/ 2 = 3.32 × 30 = 99.6 days
(c) 0.269 × 105 s (d) 0.359 × 105 s
7. You have 0.1 gram-atom of a radioactive isotope ZA X (half-life = Solution
5 days). How many number of atoms will decay during the elev- (a) The disintegration constant,
enth day?
Disintegration rate
(a) 2.10 × 1022 (b) 3.05 × 1022 l=
Number of atoms present
(c) 1.31× 1022 (d) 4.32 × 1022
The number of atoms of 24Na present
Solution 1
= 2.4 × 10 −5 × × 6.023 × 1023 = 6.023 × 1017 and given
(c) The number of atoms undergoing decay is N0 = 0.1 × 6.023 24
× 1023 = 6.023 × 1022 atoms. that rate = 7.75 × 1012 atoms s−1.
After 11 days, the number of atoms decayed = On substituting, we get
7.75 × 1012
N0e − lt = 6.023 × 1022 × e −( 0.693 × 11)/ 5
= 6.023 × 1022 × 0.2177 l= = 1.286 × 10 −5 s −1
22 6.023 × 1017
= 1.31× 10
0.693 0.693
8. Calculate the number of disintegrations per minute in Therefore, t1/ 2 = = = 0.5388 × 105 s
l 1.286 × 10 −5
a 1.00 mg sample of 238U, assuming that the half-life is 4.47
× 109 years.
Energy Changes in Nuclear Reactions
(a) 653 (b) 746 (c) 589 (d) 825
10. The observed mass of 56 26 Fe is 55.9375 amu. Using the masses
Solution of proton and neutron as 1.00732 amu and 1.00866 amu,
(b) Rate = lN respectively, calculate the binding energy per nucleon in MeV
and in joules.
0.693 0.693
l= =

You might also like